You are on page 1of 892

C Operative Dentistry 001

C Endodontics 094
Also Ask for...
C Dental Materials 164
~
~( MCQs in
C Oral Anatomy 287 '
~ Pros thodont;ics &

C Oral Histology 343


i Dental Materials
'a
t

C Micro Biology 416 ~ INCLUDES


~ ~ fvll o•plor'lotlons

~
L~ Authentic rfflrences
L_... Handouts for quic;.'lc: """~
C Periodontics 519 ~
t
1,- Quo.sttOns from ,.,;ent 'f"OOll'J,

C Community Dentistry 629


1~
II

General Anatomy 737 l


C
~
~
.Or h••lo Cttnudhn'y
P'Ofril o.mol colleo• & ...~ l n i i ; ~
NI$!......., toho,durf9ottl
Pr At

"°""
nfth lol 10

O."'lall c,;,11~ ~ ll•-O,-tr,,,,19


'°"~..,orh
Biochemistry 853 E- - ~ . N : i l 0 $. ~

~
& - c-"""°"'. t'OC)$..ltGihtoti
C

C Physiology 938

K. Satheesh Kumar Reddy

SWAPNA MEDICAL PUBLISHERS


DENTAL Pulse
MCQ's for Dental PG Entrance Examinations
Arranged Chapter wise - with Explanatory Answers

Volume - I

NINTH EDITION

Author
DR. SATHEESH KUMAR REDDY. K Mos

Co-Author
DR. VENUGOPAL. T MOS

SWAPNA MEDICAL PUBLISHERS


DENTAL PULSE
© By Dr. Satheesh Kumar K.

EIGHTH EDITION - JULY 2014

NINTH EDITION - JUNE 2015

All rights reserved. No part of this publication may be reproduced, stored in a retrieval system, or transmitted,
in any form or by any means, electronic, mechanical, photocopying, recording or otherwise, without the
prior written permission of the publisher.

The author and publisher have taken care in preparation of this book, but make no expressed or implied
warranty of any kind and assume no responsibility for errors or omissions. No Liability is assumed for
incidental or consequential damages with or arising out of the use of the information contained herein. All
Legal matters to be settled under Hyderabad jurisdiction only.

Price: ~1100.00

Published by:
Swapna Medical Publishers
FLAT NO 101, H.No. 3-2-844, Rajeswari Gayatri Sadan,
Kachiguda, Hyderabad - 500 027.
Ph: 92471 88244, 095338 63720
e-mail: smp_hyd@yahoo.co.in, swapnamedicalpublishers@yahoo.com
Webs;te: www.swapnabooks.com

Exclusively distributed by
Sri Venkateshwara Books
Hyderabad.
Mob: 9533863720, 9247188244
e-mail: svbookshyd@gmail.com

Printed at: Hyderabad.


Table of Contents

OPERATIVE DENTISTRY
1 Principles Of Cavity Preparation .............................................................................. 1
2 Dental Cements ..................................................................................................... 11
3 Amalgams ........................................... ................. ............... ........ .............. .... ......... 17
4 Direct Filling Gold ..................... ............................................................................. 29
5 Composites........ .................................................................................................... 33
6 Cast Restorations ................................. ............... ..................................... .... ......... 45
7 Instrumentation ................................... ................. ................. ...... .............. .... ......... 53
8 Miscellaneous ........................................................................................................ 62
9 Synopsis ............................................. ................. ............... ...................... ............. 74

II ENDODONTICS
1 Anatomy Of The Pulp Cavity ................................................................................. 94
2 Diseases Of Pulp And Periradicular Tissues ....................................................... 100
3 Biomechanics ... .................................... ...................................... ............... .......... 108
4 Disinfection And Obturation ...... ........... ................ ..................................... .... ....... 121
5 Bleaching ....... .. ...................................... ...................................... ............... ......... 127
6 Endodontic Surgery ............................... ........................................ .... .................. 131
7 Traumatology .... .................................. ... .... ................................. ............... ... ....... 133
8 Miscellaneous ..................................... .............................. .. ............... ........ ... ....... 141
9 Synopsis .............................................................................................................. 153

Ill DENTAL MATERIALS


1. Physical properties .................................................................................................. 164
2. Impression materials ............................................................................................... 173
3. Gypsum products .................................................................................................... 191
4. Restorative resins ................................................................................................... 198
5. Amalgams ............................................................................................................... 210
6. Metallurgy ............................................................................................................... 220
7. Direct filling gold ................................................................... ................................... 238
8. Dental cements ....................................................................................................... 240
9. Dental ceramics & miscellaneous ........................................................................... 250
10. Synopsis ............ ................................................................................................... 259
IV. ORAL ANATOMY

1. Deciduous dentition ................................................................................................ 287

2. Permanent dentition ................................................................................................ 292

3. Calcification and eruption of teeth .......................................................................... 306

4. Occlusion ................................................................................................................ 312

5. Dento-Osseous Structures and TMJ ............... .. ....... ............................... .. ...... ........ 316

6. Physiological form of tooth and periodontium ......................................................... 320

7. Synopsis ............. ....................................... .............. ............................................... 326

V. ORAL HISTOLOGY

1. Development and growth of teeth ...... ........................ ............................. ....... ......... 343

2. Enamel .................................................................................................................... 349

3. Dentin ................... .............................. ....................................................... ..... ......... 358

4. Pulp ......................................................................................................................... 365

5. Cementum .............................................................................................................. 371

6. Periodontal ligament ............................................................................................... 376

7. Bone, TMJ and Maxillary sinus ............................................................................... 381

8. Salivary glands ........................................................... .. .......................... ......... ........ 388

9. Oral mucous membrane ......................................................................................... 393

10. Tooth eruption, shedding and miscellaneous ........... ............................................... 401

11. Synopsis ................................................................................................................. 405

VI. MICROBIOLOGY

1. General Microbiology ......................... .. ....................... .............................. .. ... ......... 416

2. Bacteriology ......... ....................................................... .. .......................... ....... ....... .. 441

3. Virology ................................................................................................................... 462

4. Mycology and Parasitology ..................................................................................... 476

5. Synopsis ................................................................................................................. 488


VII. PERIODONTICS

1 Gingiva & Tooth Supporting Structures ............................................................... 519

2 Periodontal Microbiology ................ ..... ................ ............... ....................... .......... 528

3 Defense Mechanisms Of Gingiva & Host Response ........................................... 538

4 Calculus & Iatrogenic Diseases ........................................................................... 541

5 Gingivitis ............................................. ................. ............. .. ........ ....... ........ ... ....... 545

6 Gingival Enlargements ........................................................................................ 549

7 Acute Gingival Infections ..................................................................................... 553

8 Periodontal Pocket. .............................................. ............. .. ............... ........... ....... 559

9 Patterns Of Bone Loss And Trauma From Occlusion .......................................... 563

10 Slowly Progressive & Aggressive Periodontitis .................................................... 568

11 Diagnosis, Prognosis And Treatment Plan ........... ............................................... 574

12 Periodontal Instrumentation ................................................................................. 578

13 Plaque Control ..................................................... ............... ...................... .... ....... 585

14 Gingivectomy And Curettage ............................................................................... 593

15 Flap & Mucogingival Surgery............................................................................... 595

16 Treatment Of Furcation Involvement & Osseous Surgeries ........ ... ............ .......... 602

17 Implants and Miscellaneous ................................................................................ 609

18 Synopsis .............................................................................................................. 621

VIII. COMMUNITY DENTISTRY

1 Biostatistics........ ...................... ..... .......... .. .................................. ......................... 629

2 Epidemiology ..... ...................... ........... ....... ........................... ..... ........ ........ .. ........ 647

3 Indices ................................................................................................................. 665

4 Preventive Dentistry ............................................................................................. 680

5 Caries Activity Tests ............................... ... ................................. ................. ......... 703

6 Miscellaneous ...................................................................................................... 706


IX. GENERALANATOMY
1. Osteology ................................................................................................................ 737
2. Salivary & Thyroid glands ........................................ ........................................... .. .. 745
3. Mouth, Pharynx and Larynx ............................................................... ....... .............. 751
4. Tongue ....................................................... ................ .. .............. ........ ............ ...... ... 757
5. Vascular supply of head and neck .......................................................................... 763
6. Nerve supply of head and neck .............................................................................. 776
7. Muscles of head and neck ....................................... .................. ........ ..... .. ..... ...... ... 797
8. Embryology ................................................. ... ........................................... ......... ..... 809
9. Miscellaneous ......................................................................................................... 819
10. Synopsis ................................................................................................................. 829

X. BIOCHEMISTRY
1. Protein metabolism ................................................................................................. 853
2. Carbohydrate metabolism ......................................................................... ............ .. 864
3. Lipid metabolism ..................................................................................................... 875
4. Vitamins ... ...... ..... .............................. ....................... .......................... .. .......... ......... 882
5. Enzymes ............... ... .................................. .................. .............. ........ ............ ...... ... 892
6. Nucleo Proteins & Mineral metabolism ................................................................... 900
7. General Biochemistry ................................................. ................................... ....... .. 907
8. Miscellaneous ......................................................................................................... 913
9. Synopsis ................................................................................................................. 922

XI. PHYSIOLOGY
1. GIT and liver ........................................................................................................... 938
2. Respiratory system ................................................................................................. 945
3. Renal system .......................................................................................................... 954
4. Muscles and Neuromuscular junction ..................................................................... 960
5. Cardiovascular system ............................................................................................ 967
6. Central nervous system ....................... ...... ................. ............. .. ........ .. .......... ......... 974
7. Hematology ........................................................................................ ..................... 986
8. Endocrinology ......................................................................................................... 998
9. Miscellaneous ....................................................................................................... 1012
10. Synopsis .. ............ .............................. ........ ........................................ ..... ......... ..... 1017
FEEDBACK FORM

Students are requested to use this table if they find any mistakes in this book and I wel-
come a feedback so that our juniors will be benefited.
- Dr. Satheesh Kumar K.

Page Q. .No.Nor , n/P01nt


A nswer/Questio , given
· To b e corrected as
N o. P 01nt o.
, OPERATIVE DENTISTRY

OPERATIVE DENTISTRY
I. REFERENCE BOOKS TAKEN:
1. OPERATIVE DENTISTRY BY STURDEVANT - 4th & 5th Editions

2. OPERATIVE DENTISTRY BY STURDEVANT - 5th SOUTH ASIAN Edition (SA)

3. OPERATIVE DENTISTRY BY MARZOUK - 1st Edition

4. OPERATIVE DENTISTRY BY VIMAL SIKRI - 1st, 2nd & 3rd Editions

5. PHILLIPS' (SKINNERS) SCIENCE OF DENTAL MATERIALS - 11th & 12th Editions

1. PRINCIPLES OF CAVITY PREPARATION


1. In class II cavity for inlay, the cavosurface margin of the 7. Enameloplasty is
gingival seat clears the adjacent tooth by a) Filling of enamel fissures with amalgam
a) 0.20 ± 0.05mm b) 0.50 ± 0.20mm b) Elimination of shallow enamel fissures
c) 0.80 ± 0.35mm d) 1.10 ± 0.45mm c) Is same as prophylactic odontomy
(MAN-94) d) All of the above
2. Cavities beginning in the proximal surfaces of bicuspids (MAN-02)
and molars are 8. A prepared cavity is best protected from moisture by
a) Class 1 b) Class 2 a) Cotton rolls b) Saliva ejector
c) Class 4 d) Class 6 c) Rubber dam d) Paper napkin
(MAN-95) (MAN-01)
3. Incipient caries consists of opaque, chalky white areas 9. Retentive grooves in the proximal box of a class II
that appear when the tooth surface is dried, this is amalgam cavity should be
referred as a) Sharp and elongated at the dentinoenamel j unction
a) white spot b) hot spot b) Round and elongated in the dentin
c) translucent zone d) body of lesion c) Round and short in the dentin
(COMED K-07) d) Sharp and elongated in the dentin
4. Pit and fissure caries is seen in ( MAN-01,A PPSC-99)
a) Class I b) Class I compound 10. The term ' Primary Prevention' means all of the following
c) Class II d) Class II compound except
(MAN-97) a) Planning the patient's diet
5. Smear Layer consists of b) Prophylactic odontomy
a) Enamel debris b) Micro organisms c) Topical application of fluoride
c) Dentinal chips d) all of the above d) Extending cavity preparation to prevent Recurrence
(MAN-98) (MAN-99)
6. A butt joint is a 11, Infected dentine shows
a) 90° joint b) 120° joint a) Both organic and inorganic components in reversible form
c) 70° joint d) 180° joint b) Both organic and inorganic components in irreversible form
(MAN-02) c) Oryariic currq.Jun~nls in irr~v~r~il.,l~ rurrr1 aml inurya11ic
components in reversible form

1) B 2) B 3) A 4) A 5) D 6) A 7) B 8) C 9) B 10) D 11) B
Dental ;lut.,e

d) Inorganic component in reversible form and organic 20. The retention for class 5 restoration are placed:
component in irreversible form a) In mesial and distal areas
(MAN-02) b) Occlusal and gingival area
12. Deepening the pulpal floor during cavity preparation c) In all except the axial wall
provides for d) At expense of axial wall
a) Outline form b) Retention form (AIPG-90)
c) convenience form d) Resistance form 21. Extension for prevention is directly related to:
(AIIMS-95) a) Removal of unsupported enamel on proximal surface of
13. In which type of lesion 'eburnation' of the dentine is seen? Class III cavity
a) Acute caries b) Chronic caries b) Depth of the axial wall of a class III cavity preparation
c) Arrested caries d) Root Caries c) Elimination of all carious dentin beyond average depth
(AIIMS-03) of pulpal wall
14. In case of a deep carious lesion incomplete debridement d) The outline form of the cavity preparation
is done to: (AIPG-02)
a) Allow secondary dentin formation first 22. While preparing a Class II cavity on a maxillary first
b) To seal the cavity and create aseptic field molar which of the pulp horns are likely to get exposed:
c) Prevent pulp exposure and allow a thin layer of a dentin a) Mesiolingual and mesiobuccal
to remain b) Distolingual and distobuccal
d) Avoid microleakage c) Mesiolingual and distobuccal
(AIIMS -2K) d) Distolingual and mesiobuccal
15. Beveling of the pulpoaxial line angle of a class II cavity (PGI-2K,AIIMS-92)
is done to: 23. In a class 1 cavity prepared for silver amalgam is
a) Increase the strength of the restoration extended to half the distance between the contact area
b) Improved marginal adaptation and the fissures and the cusp tip, the direction of the
c) To prevent the fracture of enamel buccal and lingual walls is:
d) To prevent the fracture of amalgam a) Diverging b) Converging
(AIIMS, AIPG -06) c) Parallel d) Independent
16. Restoration of a cusp using dental amalgam requires that: (AIPG-99)
a) All enamel be removed to provide bulk 24. The reason for sealing caries during cavity preparation is:
b) Only the enamel be removed to conserve tooth structure a) To eliminate the need for eventual direct pulp capping
c) At least 2 mm of cusp be removed to provide retention form b) Produce an aseptic filed when pulp exposure is inevitable
d) At least 2mm of cusp be removed to provide resistance form c) To allow the formation of secondary dentin before
excavation
17. What is common in class I cavity preparation for d) To produce a hard surface as a foundation for subsequent
amalgam and gold inlay? restoration
a) Buccolingually divergent walls (AIPG-2 K,01)
b) Mesiodistally divergent walls 25. In a cavity preparation in which the remaining thickness
c) Maximum depth is needed in both cases to provide of dentin is more than 1.5mm, the ideal base is:
sufficient thickness a) Ca(OH)z b) Zinc Phosphate
d) All of the above c) Silicophosphate d) Glass ionomer
(AIIMS-91) (AIPG-90)
18. In class 2 inlay preparation, How should the pulpal floor 26. The function of proximal grooves in a class II cavity is:
be placed in comparison with pulpal floor in amalgam a) Resistance form b) Retention form
class 2 preparation? c) Increases strength
a) Deeper d) Resistance and retention form
b) Pulpal floor for amalgam should be placed deeper in (PGI-99)
dentin 27. An incipient carious lesion on an inter proximal surface
c) Same as Amalgam d) None of the above is usually located
(AIIMS-2011) a) At the contact area b) Facial to contact area
19. Class IV cavity: c) Gingival to contact area d) Lingual to contact area
a) Occurs on the proximal surface not involving the incisal (AP-97, PGI-03)
edge of anterior teeth 28. In an ideal restoration, gingival margin should be:
b) Occurs on proximal surface involving the incisal edge of a) Below contact point but 1mm occlusal to gingival crest
anterior teeth b) Below contact point but at level of gingival crest
c) Occurs on proximal surface involving the incisal edge of c) Below gingival crest d) At contact point
posterior teeth (PGI-95)
d) Involves the buccal surface of Anterior and posterior teeth 29. Gingival extension for a restoration should be:
(AIPG-02) a) At the gingival crest

12) D 13) C 14) C 15) D 16) D 17) B 18) B 19) B 20) B 21) D 22) A 23) B 24) C
25) B 26) D 27) C 28) A 29) C
, OPERATIVE DENTISTRY

b) At least 1mm above the alveolar crest 39. To avoid pulpal irritation below a resistant metallic
c) At least 3mm above the alveolar crest restoration, the minimum dentin thickness which
d) Doesn't have any relation with alveolar crest height should remain is:
(AIPG-02) a) 0.5mm b) 1.5mm
30. Resistance form of cavity preparation is: c) 1.0mm d) 2.0mm
a) Flat pulp floor (PGI-99)
b) enough depth of the cavity 40. According to Black's classification caries on lingual pits
c) Restricted size of the external walls of maxillary central incisors are:
d) all of the above a) Class I b) Class II
(AP-01) c) Class III d) Class IV
31. During M.O.D. preparation one of the cusps is under (AIPG-94)
mined, indicated treatment is 41. In a class II cavity in an incisor the retentive points are
a) Reduction of cusp and include in the preparation for on lay placed:
b) Reduction of all the cusps and crown is indicated a) At the DEJ b) In the dentin
c) Strengthened by use of Amalgam core c) In the axial wall d) Never on labial or Lingual walls
d) Cusp is beveled and the patient is advised not to exert (AIPG-91)
force on the cusp 42. Inappropriate margins in restorations are corrected
(AP-98) primarily because
32. Resistance form is that shape of cavity which: a) They retain food debris b) Inhibit proper plaque removal
a) Prevents displacement of restoration c) Disturb occlusion d) Create Sensitivity
b) permits the restoration to withstand occlusal forces (AIPG-97)
c) Allows adequate instrumentation 43. The most common cause of fracture at the isthmus of
d) Allows the restoration to withstand occlusal forces and class II dental amalgam restoration is:
prevent displacement a) Delayed expansion
(PGI-99) b) Inadequate depth at the isthmus area
33. A dental floss is applied to the distal bow of a clamp, its c) Inadequate width at the isthmus area
function is: d) Moisture contamination of the amalgam during placement.
a) To facilitate the removal of the clamp (KAR-97, AP-04)
b) To prevent aspiration of clamp 44. 'Saucering" out of small pits to improve self cleansing
c) To anchor the dam as cervically as possible property is called
d) To stabilize the damp a) Saucerisation b) Enameloplasty
(AIPG-94) c) Odontotomy d) None of the above
34. Which of the following is commonly recommended (AP-01)
procedure for debridement 45. Mesial and distal wall of class V cavity depends on:
a) Citric acid b) Hydrogen peroxide-10% a) Direction of enamel rods
c) Air and water spray d) Hydrochloric acid b) Presence of carious Lesion
(AP-98) c) Contour of gingiva d) Location of contact area
35. The axiopulpal depth of the proximal box in class II (AIPG-2K, AIIMS MAY- 2012)
cavity is 46. The base of class III preparation is
a) 0.2 to 0.8mm b) 0.5 to 1mm a) Axial wall b) Gingival wall
c) 1 to 2mm d) 2 to 3 mm c) Facial wall d) Lingual wall
(AP-99) (KAR-98)
36. Purpose of cavity preparation is 47. While preparing occlusal cavity for amalgam in
a) To receive restorating material mandibular first premolar, the bur
b) To cleanse the caries a) Is kept absolutely vertical
c) To remove bacteria d) None b) Is tilted slightly buccally
(PGl-02) c) Is tilted slightly lingually
37. Intercuspal width ideally for class II amalgam cavity d) Can be kept in any direction
should be: (UPPSC-01)
a) 1/2 b) 1/7 48. Which of the following is not a consideration for
c) 1/3 d) 1/5 obtaining a resistance form during cavity preparation?
(Al-99) a) Stress patterns of teeth
38. When removing a rubber dam. The first step is to: b) Direction of enamel rods
a) Remove the clamp b) Release the holder c) Designing outline form with minimal extension
c) Apply a water soluble Lubricant d) Dovetail preparation of restoration to occlusal loading
d) Cut the i nterseptal rubber with scissors (UPPSC-01)
(AIPG-91, AP-04) 49. Gingivally the depth of a class V cavity is
a) 0.5 - 1 mm b) 0.75 - 1mm

30) D 31) A 32) B 33) B 34) C 35) A 36) A 37) D 38) D 39) D 40) A 41) B 42) B
43) B 44) B 45) A 46) A 47) C 48) D 49) B
Dental ;lut.,e

c) 1-1.25mm d) 2-3mm b) Entirely in dentin


(KAR-98) c) In axial wall
50. For cast restorations the cavity wall should have a taper of d) By placing a groove in t he lingual surfaces
a) 30-45 b) 5-10 (AIPG-07)
c) 2-5 d) none of the above 61. All enamel walls of a cavity preparation must consist of
(KAR-98) full length enamel rods on
51. In modified class III cavity restoration retention is a) Affected enamel b) Affected dentin
obtained mainly by c) Sound enamel d) Sound dentin
a) Acid etching b) Retentive groove/slot (BHU-2012)
c) Retentive point d) Retentive pin 62. In air-abrasive technology the abrasive particles used are
(KAR-98) a) Iron oxide b) Tin oxide
52. During cavity preparation, more incidence of exposure is in c) Aluminium oxide d) Silicon oxide
a) Class V cavity in first premolar (KCET-07)
b) Class II mesio occlusal cavity 63. In an early carious lesion, the first structure to show
c) Class II disto occlusal cavity evidence of destruction is the
d) Class IV cavity a) Enamel prism b) Cuticle
(TNPSC-99) c) Interprismatic substance d) Lamellae
53. Transverse section of Class V through the axial wall is (BHU-07)
a) Convex b) Concave 64. Which of the following principles of the cavity
c) Kidney shaped d) Straight preparation is not considered much in modern methods
(PGI-95) of restorative Dentistry?
54. The cavosurface angle for inlay cavity preparation a) Extension for prevention
a) 90 degree b) 150 degree b) Convenient form
c) Less than 90 degree d) 180 degree c) removal of the undermined enamel
(COMED-06) d) Removal of the carious dentine
55. Caries detection dye can stain the following except: (KCET-08)
a) Granular necrotic tissue b) Dry, leathery dentin 65. The wall absent in class V lesion
c) Reversibly denatured collagen a) Axial wall b) Mesial
d) lrreversibly denatured collagen c) Distal d) Pu lpal wall
(AI-05) (BHU-07)
56. The modified class III preparation uses a dovetail on the 66. It is essential to lubricate dam before applying it. Which
lingual side in of these is NOT a suitable lubricant?
a) Maxillary canine b) Mandibular canine a) Shaving cream b) Liquid soap
c) Mandibular central incisor c) Scrub gel d) Vaseline
d) Mandibular lateral incisor (AIPG-09)
(COMED-06) 67. Which is the WRONG way to apply dam using a wingless clamp?
57. Preparation of class I cavities for dental amalgam, direct a) Put the clamp on the tooth
filling gold or gold in lays have in common b) Place the dam on t he tooth with fingers, and then
a) Occlusally diverging facia l and lingual walls position the clamp over it.
b) Occlusally diverging mesial and distal c) Attach the dam over the clamp and frame outside the
c) Occlusally converging facial and lingual walls mouth, then put the assembly over the tooth using
d) Occlusally converging mesial and distal walls clamp holders OVER THE dam.
(COMED-06) d) Attach the dam over the clamp and frame outside the
58. Which of the following prevents fracture of MO amalgam: mouth, and then put the assembly over the tooth using
a) Beveling of axiopulpal line angle clamp holders UNDER the dam.
b) Occlusal dovetai ls (AIPG-09)
c) Pin retained amalgam 68. When composites are given, ex posed dentin should be
d) Removal of unsupported enamel rods covered with
(AIIMS-2011) a) Varnish b) Calcium hydroxide
59. The non-active carious lesion is distinguished by all except: c) Phosphoric acid d) ZnO eugenol cements
a) Covered by a layer of plaque (AP-09)
b) Soft and leathery in consistensy 69. Dental Pulp under any restoration is best protected by
c) Brown or black discolouration a) Cavity liner b) Cavity base
d) Severe pain during caries removal c) 2 mm thick dentin d) All of the above
(AIPG-07) (UPSC-09)
60. In a class III cavity in an incisor tooth the retentive 70. Which one of the following provides most conservative
points are placed: approach while restoring the tooth?
a) At the expense of facial and lingual walls and not in dentin a) Cast gold b) Dental amalgam

50) C 51) A 52) A 53) A 54) B 55) C 56) A 57) B 58) A 59) D 60) B 61) D 62) C
63) C 64) A 65) D 66) D 67) C 68) B 69) C 70) D
, OPERATIVE DENTISTRY

c) Glass ionomer cement d) Composite resin 81. For gold inlays gingival margin finish line should be
(UPSC-09) a) Shoulder preparation b) Chamfer preparation
71. In cavity preparation, line angle is c) Bevel d) Knife edge
a) Union of three surfaces b) Union of four surfaces (COMEDK-15)
c) The junction of two plane surfaces of different 82. The base of class III preparation is
orientation along a line a) Axial wall b) Lingual wall
d) Internal boundary of a cavity c) Gingival wall d) Facial wall
(KCET-09) (APPG-15)
72. The tip diameter of a 245 bur used for conventional
amalgam preparation is -
a) 2 mm b) 0.8 mm
c) 0.3 mm d) 0.4 mm
(COMEDK-09)
73. The total number of point angles present for Class II
amalgam restorations is -
a) 4 b) 6
c) 8 d) 11
(KCET-09)
74. Whenever the caries cone in enamel is larger or at least
the same size as that in dentin, it is called as
a) Residual caries b) Recurrent caries
c) Forward caries d) Backward caries
(KCET-2011)
75. What is the major difference between a class V cavity
preparation for amalgam and one for composite resin by
the acid-etch technique?
a) Depth b) Convenience form
c) Position of retentive points
d) Angulation of enamel cavosurface margins
(AIPG-2011)
76. Cervical restorations fail mostly due to
a) Inadequate moisture control
b) Marginal gingivitis
c) Cuspal flexure d) All the above
(KAR-2013)
77. Proximal walls in a class I tooth preparation for amalgam
should
a) Converge occlusally b) Diverge occlusally
c) Remain straight d) Rounded
(KAR-2013)
78. The configuration factor ["C" factor] for an occlusal
class I cavity is
a) 2 b) 3
c) 4 d) 5
(MCET-14)
79. Preventive resin restorations are indicated in ?
a) Non-cavitated pit and fissures
b) Deep caries to prevent progress of lesion
c) Cavitated fissures which require restoration
d) Wide cavity to prevent cusp fracture
(AIPG-14)
80. In diagnosing the carious lesions, the tooth surface is
examined visually and tactilely. Which of the following
is NOT considered as visual examination?
a) Cavitations b) Surface roughness
c) Opacification d) Softness of the tooth surface
(COMEDK-15)

71) C 72) B 73) B 74) C 75) D 76) A 77) B 78) D 79) C 80) D 81) C 82) C
Dental ;lut.,e

1. PRINCIPLES OF CAVITY PREPARATION - ANSWERS


1. 'B' [Sturdevant 5th SA ed 435/ 5th ed 303] • go 0 cavosurface angle.
Gingival seat in class II restoration:
• Advocated in amalgam restorations because
• Should extend apical to t he proximal contact or caries amalgam is a brittle material and has low
whichever is great er. edge strength.
Butt
• It should clear the adjacent tooth by 0.5mm. joint • Butt joint produces maximal strength for
• For amalgam restoration the gingival margin is beveled both tooth and amalgam.
at 15 - 20° whereas for cast restorations it is bevelled • It is also advocated in si licate and porcelain
at 30 -40° . restorations.
• If gingival seat is located in root surface, beveling is
cavosurface angle.
• > go
0

contraindicated.
• Advocated in cast and composite restorations.
The facial and lingual walls of the proximal box are extended Lap/Slip • 30 40° marginal metal and 140 150°
such that they clear the adjacent tooth by 0.2 - 0.3mm. joint cavosurface angle.
The facial and lingual walls are placed in their respective • Bevelling enables superior adaptation and
embrasures. burnishability of marginal metal

2. 'B' [Sturdevant 5th SA ed 165/ 5th ed 296] 7. 'B' [Sturdevant 5th SA ed 167/ 5th ed 304]
Black's classification is a therapeutic classification because Enameloplasty is the process of conversion of shallow pit,
it is based on treatment and restoration design. groove or fissures into a rounded or saucered self cleanable
area. Fissures that does not extend more than one-third of
Restorations on: enamel depth are removed by enameloplasty.
• Occlusal surface of molars and premolars.
Class 1 8. 'C' [Sturdevant 5th SA ed 136/ 5th ed 463]
• Facial and lingual surfaces of molars
• Lingual surface of maxillary incisors. Moisture control refers to exclusion of sulcular fluid,
saliva and blood from the operating fie ld. Among rubber
Restorations on the proximal surfaces of
Class 2 dam, suction devices and absorbents, rubber dam is best
posterior teeth
recommended for moisture control.
Restorations on the proximal surfaces of anterior
Class 3
teeth that does not involve incisal angle. 9. 'B' [Sturdevant 5th SA ed 382/ 5th ed 748]
Restorations on the proximal surfaces of anterior Locks for amalgam and the grooves for cast metal
Class 4 restorations provide additional retention for proximal
teeth involving the incisal edge.
portions of tooth preparations. They also increase the
Restorations on the gingival third of the facia l
Class 5 resistance form. These are placed in the axiofacial and
or lingual surfaces of all teeth.
axiolingual line angles 0.2mm inside the DEJ. They are
Restorations on the incisal edge of anterior started at the axiofaciogingival (or axio lingua gingival)
Class 6
teeth or occlusal cusp heights of posterior teeth. point angle and terminated at the axio facio pulpal (or axio
lingua pulpal) point angle.
3. 'A' [Sturdevant 5th SA ed 39/Sturdevant 5th ed 298]
High copper amalgam alloys (that include occlusal dovetail)
4. 'A' [Sturdevant 5th SA ed 165/ 5th ed 295] does not require locks in preparation as they exhibit less creep.

5. 'D' [Vimal Sikri 3"' ed 497/ 1" ed 551] 10. 'D' [Sturdevant 4th ed 277 / 5th ed 290]
Smear layer is microcrystalline debris embedded within Extending the preparation to self cleansing areas and
denatured collagen on the cut surfaces of the tooth. It is including the pits and fissures to prevent recurrence of
1 - 2mm thick and consists of saliva, bacteria, enamel and caries is known as 'Extension for prevention'.
dentin particles. The smear layer debris pushed into the
dentinal tubules forms smear plugs. Preventive measures such as fluorides, oral hygiene, proper
diet and use of enameloplasty, pit and fissure sealants
Smear unit = Smear layer + Smear plugs makes 'constriction for conservation' possible excluding
the need for "extension for prevention".
6. 'A' [Sturdevant 5th SA ed 356,382/ 5th ed 513]
Extension for prevention is least related to silicates as it
contains high amount of fluorides i.e., high anticariogenic
property.
, OPERATIVE DENTISTRY

11. ' B' [Sturdevant 5th SA ed 47/ 5th ed 291] to t he proximal surface is less than 1.6mm to prevent
Infected dentin has bacteria and collagen that is irreversibly undermining of marginal ridge.
denatured. It is not remineralizable and must be removed.
Affected dentin has no bacteria, the collagen is reversibly In cast metal restorations parallel (or) slight divergence of
denatured and remi neralizable. It is dark brown in colour, 2 - 5° per wall provides the primary retention form.
hard in text ure and should not be excavated.
18. ' B' [Sturdevant 5th SA ed 169/ Sturdevant 5th ed 306]
1% acid red solution in 0.2% propylene glycol discloses Amalgam requires a minimum occlusal thickness of 1.5 mm
infected dentin. for adequate strength. So, the pulpal floor depth should not
be less than 1.5 mm. Cast metal requires less thickness to
12. ' D' [Sturdevant 5th SA ed 169/ 5th ed 306] resist fracture. Minimum of 1 mm pulpal depth is required.
Resistance form is obtained by: Ceramics require at least 2 mm pulpal depth.
• Relatively flat pulpal and gingival floors.
19. ' B' [Sturdevant 5th SA ed 165/ 5th ed 296]
• Box shape.
• Inclusion of weakened tooth structure. 20. ' B' [Sturdevant 5th ed 799/ Vimal Sikri 3rd ed 223)
• Rounded internal line angles. Tooth
• Adequate thickness of restorative material. Location of retention grooves
preparation
• Reduction of cusps for capping when indicated. Class II Bucco axial and linguo axial line angles
Class III Axiogingival line angle
Deepening of pulpal floor increases the resist ance form by
increasing the bulk of rest oration . Class V Occluso axial and gingivo axial line angles

13. 'C' [Sturdevant 5th SA ed 40/ 5th ed 290] 21. ' D' [Sturdevant 5th ed 290/ Vimal Sikri 3rd ed 177]
An arrested, dentinal lesion is typically open, hard and dark.
This dentin is termed as sclerotic or eburnated dentin. 22. 'A' (Shobha Tandon 1" ed 289]

14. 'C' (Shobha Tandon 1' 1 ed 335] 23. ' B' [Sturdevant 5th SA ed 363/ 5th ed 715]
Facial and lingual walls should always be made slightly
15. ' D' [Sturdevant 5th SA ed 35 7/ 5th ed 305] occlusally convergent in class I amalgam restoration.
Beveling (or) rounding of pulpoaxial (external) line angle
increases the resistance to fracture of the restoration by If the distance between mesial or distal margin to the
increasing the bulk of the restoration and by preventing proximal surface is more than 1.6mm, then mesial or distal
stress concentration created by the sharp line angle. wall convergence can be made. If the distance is 1.6mm or
Bevelling is done by the gingival marginal trimmer. less, mesial and distal walls should be diverged occlusally.

Ro unding of internal line angles increases the resistance 24. 'C' [Sturdevant 5th SA ed 171/ 5th ed 310]
form of the tooth to fracture.
25. ' B' [Sturdevant 5th SA ed 172/ 5th ed 311]
16. ' D' (Sturdevant 5th ed 301, 869] For effective pulp protection the effective dentinal depth
Rules for cusp capping: (EDD) between pulp and base of restoration should be
2mm. This may include the remaining dentin, liner or base.
• Outline form has extended no more than 1/z the distance
from a primary groove to cusp tip - No cusp capping. If the excavation is deep or very close to pulp - Ca(OHlz
• If the extension is from 1/2 - 2/3'd distance - consider liner is indicated. If the excavation is not deep (or) sufficient
cusp capping. dentin is remained - Zinc phosphate base is indicated, no
• If the extension is more than 2/3'd distance - cusp liner is indicated.
capping is mandatory.
26. ' D' [Sturdevant 5th SA ed 17 2/ 51hed 3 06]
For cusp capping with amalgam the cusp reduction should Often features that enhance retention form of a preparation
be 2mm for functional cusp and 1.5mm for non functional also enhance the resistance form . Retention locks, grooves,
cusp. slots, pins, groove extensions and skirts increases both
retention and resistance forms. (This is according to
For cusp capping with metal the cusp reduction should be Sturdevant and Vimal Sikri)
1.5 mm for functiona l cusp and 1mm for non functiona l cusp.
27. 'C' [Sturdevant 5th SA ed 35/ 5th ed 542]
17. ' B' [Sturdevant 5th SA ed 365,432/ 5th ed 714]
Primary retention form in amalgam cavity preparation is 28. 'A' [Sturdevant 5th SA ed 379/ 5th ed 742]
obtained by occlusal convergence of buccal and lingual Extending the gingival margins into the sulcus should be
walls. The mesial and distal walls should be slightly avoided because subgingival margins are difficult to restore
diverged occlusally if the distance from the marginal ridge and may contribute to periodontal disease.
Dental ;lut.,e

29. 'C' Inadequate marginal seal of the restorations cause


microleakage and plaque accumulation predisposing to
30. ' D' (Sturdevant 5th SA ed 168/ 5th ed 306) secondary caries.
Restoration Thickness to resist fracture
43. 'B' [Sturdevant 4th ed 291/ 5th ed 291)
Amalgam 1.5 mm Adequate depth of t he cavity preparation results in adequate
Porcelain 2 mm thickness of restorative material which prevents its fracture
Cast metal 1 - 2 mm under load. Bevelling of axiopulpal line angle also increases
the bulk of the restoration preventing the fracture of the
31. 'A' [Sturdevant 4th ed 826) restoration at the isthmus area.

32. 'B' [Sturdevant 5th SA ed 169/ 5th ed 304) Increasing the widt h of preparation especially near isthmus
Option 'K is the retention form. makes the tooth more prone for to fracture.

33. 'B' [Sturdevant 5th SA ed 138/ 5th ed 466) 44. 'B' [Sturdevant 5th SA ed 167/ 5th ed 304)
The bow of the retainer should be tied with dental floss of
approximately 12 inches in length. The floss allows retrieval 45. 'A' (Sturdevant 4th ed 756/ 5th ed 798)
of the ret ainer or its broken parts if they are accidentally Class V cavity
swallowed or aspirated by the patient. • Primary determinant of the outline form is the extension
of t he caries.
34. 'C' (Sturdevant 5th SA ed 175/ 5th ed 316) • External shape is related to the contour of the marginal
gingiva.
35. 'A' [Sturdevant 5th SA ed 379/ 5th ed 303) • The axial wall is slightly deeper at the incisal wall, where
The axiopulpal depth of the proximal preparation is 0.2 - there is more enamel (1 - 1.25mm) t han at the gingival
0.8mm into dentin. The greater depth (0.8mm) is placed, wall, where there may be little or no enamel (0. 75 - 1mm).
when the extension is onto the root surface and the lesser
depth (0.2mm) is placed when no retention grooves will be • The direction of mesial and distal walls follow the
direction of the enamel rods.
placed.
• All t he walls are perpendicular to the external surface
The 0.8mm diameter bur makes a ditch cut of 0.5mm into and usually diverge facially. So retention grooves
dentin and 0.3mm into enamel. placed at the axioincisal and axiogingival line angles
or 4 retention coves are placed in each of the four axial
36. 'A' [Sturdevant 5th SA ed 159/ 5th ed 284) point angles to provide the retention forms.
• The axial wall is convex mesiodistally fo llowing the
37. ' D' [Marzouk 1" ed 139) external contour of the tooth.
• Direct filling gold is ideal restorative material for class
38. 'D' (Sturdevant 5th SA ed 150/ 5th ed 479) V cavities.
Steps for placing the rubber dam:
a) Testing and Lubricating the proximal contacts. 46. 'I((Sturdevant 4th ed 746/ 5th ed 788)
b) Punching t he holes. Class III cavity
c) Testing the retainer stability and retention. • Walls present are facia l, lingual, gingival and axial.
d) Positioning the dam over retainer • Access to the proximal surface is usually taken from
e) Attaching the frame. lingual surface.
• Facial margin is extended 0.2 to 0.3mm into the facia l
f) Passing the dam through posterior contact.
embrasure (if necessary) .
g) Passing the septa through the contacts.
• The lingual margin blends with the incisal and gingival
margins with little or no lingual wall.
Steps for removing the rubber dam:
• The facial, lingual and gingival walls should meet the
a) Cutting the interseptal rubber. axial wall at 90° .
b) Removing the retainer. • Retention groove is placed at axiogingival line angle.
c) Removing the dam. • Retention cove is placed at the axiofacioincisal point
angle.
39. ' D' (Sturdevant 5th SA ed 406/ 5th ed 311)
• Lingual dovetail is indicated if additional retention is
needed for larger rest oration .
40. 'A' [Sturdevant 5th SA ed 165/ 5th ed 296)

41. 'B' [Sturdevant 4th ed 706/ 5th ed 790) 47. 'C' [Sturdevant 5th SA ed 364/ 5th ed 721)
The pulpal wall of mandibular first premolar declines
42. 'B' lingually. The facial pulp horn is prominent and located at
, OPERATIVE DENTISTRY

a high Level t han t he Lingual pulp horn. The bur is tilted • Debriding pits and fissures
lingually to prevent exposure of the facial pulp horn. • Roughening of tooth surfaces before bonding

48. 'D' [Sturdevant 5th SA ed 170/ 5th ed 853] Disadvantages


Dovetail preparation provides retention form.
• Difficult t o determine t he cutting progress within the
t oot h preparation
49. ' B' [Sturdevant 4th ed 757/ 5th ed 798]
• Abrasive dust interfered with visibility
50. 'C' [Sturdevant 5th SA ed 432/ 5th ed 307] • Inhalation of abrasive dust
• Does not produce well-defined walls and margins
51. 'A' [Sturdevant 4th ed 753/ 5th ed 538]
Air-abrasion relies on t ransfer of kinetic energy from a
52. 'A' stream of powder particles on the surface of tooth structure
or restoration. It generates finest stream of abrading
53. 'A' [Sturdevant ! th~ 756] particles (Alumina) and causes an effective cutting.

54. ' B' [Sturdevant 5th SA ed 439/ Marzouk 1•t ed 324] 63. 'C' [Shafer 4th ed 438]
For explanation refer t o Q.No. 6 The earliest histopat hological change in the incipient
caries of the enamel is t he loss of interprismatic or inter
55. 'C' [Sturdevant 5th SA ed 47/ Sturdevant 5th ed 277] rod substance of the enamel wit h t he increased prominence
1% acid red solution in 0. 2% propylene glycol is a caries of the rods.
detector dye. It stains infected dentin.
The earliest clinical manifestation of incipient caries of
56. 'A' [Sturdevant 5th ed 538] enamel is the smooth chalky white area of decalcification .
The 3 types of tooth preparation for Class III composite
restorations are: 64. 'A' [Sturdevant 5th ed 290]
• Conventional: for class III preparation on root surface
• Bevelled conventional: indicated for replacing existing 65. 'D' [Sturdevant 5th ed 557]
defective rest oration The walls present in a class V cavity are:
• Modified Class III: Most used type of class III, indicated • Occlusal / incisal wall
for small and moderate lesions • Gingival wall
• Mesial wall
57. ' B' [Sturdevant 4th ed 675]
• Distal wall
58. 'A' [Sturdevant 5th SA ed 382/ Sturdevant 5th ed 305] • Axial wall
Beveling or rounding of external line angles, for example,
axiopulpal line angle prevent s fracture of proximal amalgam. 66. 'D' [Sturdevant 5th SA ed 139,144/ Sturdevant 4th ed
Ro unding of internal lines angles prevents fracture of tooth. 449]
A water soluble lubricant applied in the area of the punched
59. 'D' [Sturdevant 5th SA ed 40/ Sturdevant 5th ed 290] hole facilitat es the passing of t he dam septa through proximal
Pain is a feature of acute caries than of chronic caries. contacts. A rubber dam lubricar,t is commercially available,
but other lubricants such as shaving cream or soap slurry
60. 'B' [Sturdevant 5th ed 533] are also satisfactory. Cocoa butter or petroleum jelly may
In class III and class IV preparations, the external walls be applied at the corners of the patient's mouth to prevent
diverge outwards to provide strong enamel margins and irritation. These two materials however are not satisfactory
therefore Retention Coves (on 3-point angles) or Retention as rubber dam lubricants because both are oil-based and not
grooves (on axio gingival line angle) are prepared in the easily rinsed from the dam after the dam is placed.
dentinal walls to provide t he retention form.
67. 'C' [Sturdevant 5th SA ed 151 fig: 8-21]
61. 'D' [Sturdevant 5th SA ed 164/Sturdevant 5th ed 294] Options A, B and D are the correct methods of application
Strongest enamel margin is formed by fu ll length enamel of rubber dam.
rods whose inner ends are on sound dentin. As enamel
rods are usually perpendicular to the enamel surface, the 68. 'B ' [Sturdevant 5th SA ed 24 7/ Sturdevant 5th ed 587]
strongest enamel margin results in a cavosurface angle
greater than 90 degrees. 69. 'C' [Sturdevant 5th SA ed 406/ Sturdevant 4th ed 295]

62. 'C' [Sturdevant 5th ed 47] 70. 'D' [Sturdevant 5th SA ed 159/ Sturdevant 4th ed 272]
Uses of Air-abrasive technology In the past, tooth preparations were precise resulting in
unifo rm depths, particular wall forms and specific marginal
• Cutting of enamel and dentin
configurations. Such preparations are still required for
• Removal of st ains
Dental ;lut.,e

amalgam and cast metal restorations. But composite 79. '(' [Sturdevant 5th ed 290)
restorations do not require precision tooth preparations. Preventive resin restoration (PRR) is a thin, resin coating
They require only removal of the defect and friable tooth applied to the deep pits and grooves or cavitated fissures
structure for tooth preparation without specific depths, wall of young permanent teeth. Simonsen in 1978 described
designs or marginal form. This is due to physical properties this technique. In addition to restoring the caries, the
of the composite material and the strong bond obtained remaining pits and fissures are protected with sealants. PRR
between the composite and tooth structure. is also called as conservative adhesive resin restoration.

71. 'C' [Sturdevant 5th SA ed 163) Depending on the extent and depth of the carious
lesion, there are 3 types of PRR:
72. 'B' [Sturdevant 5th SA ed 363) • Type A - Suspicious pits and fissures where caries is
A No. 245 bur has head length of 3mm and a tip diameter limited to enamel.
of 0.8mm. This is a inverted cone bur.
• Type B - Incipient lesion extending into dentin that is
small and confined
No.330 is a smaller and pear shaped version of the No.245
bur. Used for the mos.t conservative amalgam preparations. • Type C - Deep caries thereby requiring great er
exploratory preparation in dentin.
73. 'B' [Sturdevant 5th SA ed 163/ Sturdevant 5th ed 297)
80. 'D' [Sturdevant 5th SA ed 79]
74. 'C' [Sturdevant 5th SA ed 39/ Sturdevant 5th ed 289) Carious lesions can be detected both by visual inspection
• Forward caries: The caries cone in enamel is larger or at and tactile exploration. Softness is assessed with tactile
least the same size as that in dentin. exploration, whereas surface ro ughness can be assessed
• Backward caries: when the spread of caries along the subjectively by running an explorer judiciously (tactile) or
by visual observation. So the best answer is option 'D'.
DEJ exceeds the caries in t he contiguous enamel, caries
extends into this enamel from the junction.
81. '(' [Check Explanation of Q. No 4]
75. 'D' [Sturdevant 5th SA ed 299)
82. '(' [Sturdevant 5th SA ed 788]
Class V amalgam preparation has 90 degree cavosurface
angle. For class V composit e preparation, t he margins The outline of Class III cavity is a triangular outline with
the base of the triangle at the gingival floor.
should be beveled resulting in more than 90 degree
cavosurface angle. Additionally refer synopsis for the 3
designs of composite preparation.

76. 'A' [Sturdevant 5th ed 786, 292, 150)


Class V restorations may fail due to
• Inattention to principles of tooth preparation
• Improper manipulation of material
• Inadequate moisture control
• Differences in dentin composition
• Cervical flexure during occlusion
• Adhesion t o different opposite substrates.

The common clinical occurrence of class V restoration


failure is often blamed on inadequate moisture control.

77. 'B' [Sturdevant 5th SA ed 365/ Sturdevant 5th ed 713]


The mesial and distal (proximal) walls should be prepared
with <10° occlusal divergence. This prevents undermining
the marginal ridges of their dentin support. As the facial
and ling ual prepared walls converge, this slight divergence
does not present any retention form concerns.

Note: In very minimal class I cavities, the mesial and distal


walls may be parallel or slightly convergent.

78. 'D' [Sturdevant 5th ed 504]


, OPERATIVE DENTISTRY

2. DENTAL CEMENTS

1. The preferred agent for base formation below restorative 11. Amalgam often tends to discolor the teeth. This can be
resin is: inhibited by using:
a) Calcium hydroxide b) Cavity varnish a) A zinc free alloy b) An alloy containing zinc
c) Zinc oxide-eugenol cement c) Calcium hydroxide on the pulpal floor
d) None of the above d) Cavity varnish on all cut surfaces
(MAN-94, KAR-2K) (KAR-01)
2. Dentin Protection during cavity restoration is most 12. Fluoride rich materials include?
important when the " remaining dentin thickness" is: a) Silicate cement b) Glass ionomer cement
a) 1 - 2mm b) 1.5 mm c) Polycarboxylate cement d) All of the above
c) 0.5 - 1mm d) 0.5 mm (MAN -97)
(MAN-97) 13. Glass ionomer cements are composed of
3. Cavity varnish is indicated under amalgam restorations to: a) Alumino silicate powder and phosphoric acid
a) Prevent galvanic currents trom reaching the pulp b) Alumino silicate powder and polyacrylate liquid
b) Improve the ma, ginal seal of the restorations c) Zinc oxide powder and phosphoric acid
c) Seal the dentinal tubules completely d) Zinc oxide powder and polyacrylate liquid
d) Act as an effective thermal insulator (MAN -95)
(AIIMS-98, MAN-99) 14. Which one of the following dental cement accelerates
4. Calcium hydroxide is preferred to ZOE fillings below a the formation of reparative dentin?
composite resin restoration as: a) Eugenol b) Calcium hydroxide
a) Calcium hydroxide forms a calcific bridge c) Zinc oxide d) Silica
b) Eugenol irritates the inflamed pulp (MAN -97, KAR -2K)
c) Eugenol interferes with the setting of composite resins 15. Minimum thickness for type-I zinc phosphate cement
d) None of the above should be
(MAN-99) a) 15 microns b) 25 microns
5. When using the acid etch technique to restore a Class IV c) 50 microns d) 100 microns
fracture, exposed dentin should first be covered with: (MAN -02, COMEDK -06)
a) Cavity varnish b) Phosphoric acid 16. Increased amount of powder in zinc phosphate cement
c) A calcium hydroxide liner mixture will cause
d) Zinc oxide-eugenol cement a) Decreased strength b) Decreased film thickness
(AIPG-03) c) Decreased solubility d) Increased setting time
6. Which of the following bases are contra indicated under (MAN -2K)
a composite resin restoration: 17. Which of the following is common to both zinc eugenol
a) Calcium hydroxide b) Eugenol base cement and polycarboxylate cement?
c) Zinc phosphate cement d) None of the above a) Polyacrylic acid is liquid
(KAR-2K) b) Chemical bond to tooth structure
7. Cements in dentistry can be used as: c) Chelation
a) Liners b) Bases d) Substitution of eugenol by EBA to increase strength of
c) RC fillers d) All of the above cement
(AP-01) (AIIMS -2K)
8. Cavity varnish is used for: 18. The addition of which of the following can accelerate
a) To reduce thermal conductivity the setting time of zinc oxide cement:
b) To reduce the permeability of acid into dentinal tubules a) Zinc acetate b) Barium sulphate
c) Action as an obtundant d) All of the above c) Zinc sulphate d) Barium chloride
(AIIMS-99) (KAR -2K)
9. Amount of gap required for cement in restoration is: 19. Which of the following show chemical bond with enamel:
a) 10-25 microns b) 2 microns a) Composites b) Direct filling resins
c) 50-100 microns d) No gap required c) Polycarboxylate cements
(PGl-95) d) BIS-GMA resins in pit and fissure sealants
10. (opal varnis h is: (AIIMS -01)
a) ZnO resin b) Ca(OH)z 20. Which of the following cements is most irritant to
c) ZnO glycerin d) All of the above pulpal tissues?
(AIPG-2K) a) Zinc phosphate b) Silicate
c) Glass ionomer d) Polycarboxylate
(KAR -97)

1) A 2) D 3) B 4) C 5) C 6) B 7) D 8) B 9) A 10) A 11) D 12) D 13) B


14) B 15) B 16) C 17) C 18) A 19) C 20) B
Dental ;lut.,e

21. Which of the following do polycarboxylate and GIC have 32. Which of the following cements is most kind to pulp?
in common? a) Glass ionomer b) Polycarboxylate
a) Polysiloxane b) Phosphoric acid c) Silicate d) Resin cement
c) Polyacrylic acid d) Ion leachable glass (AIPG -94)
(KAR -97) 33. Polycarboxylate cement is used for:
22. P" of polycarboxylate liquid is: a) Temporary restoration b) Leuting
a) 5 b) 7 c) Permanent restoration d) Die material
c) 8 d) 1.7 (KAR -01)
34. Silicate cement is indicated in:
23. Which is the cement with adhesive properties? a) Mouth breathers
a) Zinc oxide eugenol b) Zinc polycarboxylate b) Patients with high caries index
c) Silicate d) Zinc phosphate c) In the restoration of posterior tooth
(AIPG -98) d) None of the above
24. Eugenol may be replaced in the zinc oxide eugenol (AIIMS -97)
cement by: 35. Stages of setting of GIC are?
a) Acetic acid b) Alginic acid a) Decomposition, migration, post set hardening,
c) Phosphoric acid d) Ortho-ethoxy acid maturation.
(AIIMS -99) b) Migration, decomposition, maturation, post set
25. The setting time of zinc phosphate may be retarded by? hardening
a) Increase in the ratio of powder to liquid c) Decomposition, post set hardening, migration,
b) Diluting the liquid with water maturation
c) Increase the addition of powder to liquid d) Decomposition, post set hardening, migration,
d) Decrease the addition of powder to liquid maturation
(AIIMS-01, AIPG-02) (AIPG-09)
26. Thermal conductivity of which material is close to dentin: 36. The cement, which is least soluble in oral cavity:
a) Amalgam b) Silicate a) Glass ionomer b) Resin cement
c) Composite resin d) Cavity varnish c) Polycarboxylate cement d) Silico - phosphate cement
(AIIMS-99, AIIMS-95) (KAR -97, PGI -95)
27. The logical explanation for the unique anticariogenic 37. Which of the following restorative material is
property of most silicate cements is: recommended for root surface caries:
a) The reduction in enamel solubility due to fluoride uptake a) GIC b) Composites
by enamel c) Direct filling gold d) Amalgam
b) That beryllium flux is used in silicates (MAN-98)
c) That silicates show very little leakage at the margins of 38. If hypersensitivity develops in a glass ionomer filled
the restoration tooth, indicated treatment:
d) Due to the high silica content a) Extraction
(AIIMS -03) b) Remove the restoration and place a sedative dressing
28. Pulpal medication and thermal protection is given by c) Remove the restoration perform pulpectomy
a) Solution liners (2-5µm) d) No treatment is necessary
b) Suspension liners (25-30µm) (KAR-88)
c) Traditional Liner (0.2-0.3 mm) 39. The role of itaconic acid copolymer in glass ionomer cement:
d) Cement bases a) Reduces the viscosity of the liquid and inhibits gelation
(AIIMS-09) b) Increases the viscosity of the liquid and accelerates
29. Glass ionomer cement is composed of: gelation
a) Zinc oxide powder and poly carboxylic liquid c) Increases cohesion d) To provide radiolucency
b) Zinc oxide powder and phosphoric acid (KAR-99)
c) Aluminia silicate powder and poly carboxylic liquid 40. The advantage of glass ionomer cement reinforced with
d) Aluminia silicate powder and phosphoric acid metal particles is:
(AP -97, AIPG -89) a) Decreased strength but biocompatibility
30. Which of the following cements is most biocompatible b) Increased resistance to abrasion
with pulp? c) Better colour d) Ease of manipulation
a) GIC b) Copper cement (AIIMS-94)
c) Zinc phosphate cement d) Zinc oxide eugenol cement 41. Which is the material of choice to be given in class V cavities:
(AIPG -97) a) Silicate cement b) Polycarboxylate cement
31. The major component of zinc phosphate cement is: c) Glass ionomer cement d) Unfilled resin
a) Zinc oxide b) Phosphoric acid (KAR-2K, PGI-02)
c) Zinc d) None of the above
(AP -03)

21) C 22) D 23) B 24) D 25) D 26) B 27) A 28) C 29) C 30) D 31) A 32) B 33) B
34) B 35) A 36) B 37) A 38) B 39) A 40) B 41) C
, OPERATIVE DENTISTRY

42. Initial best sealing of margins is done best by:


a) GIC b) Acid etched composite
c) Amalgam d) Polycarboxylate
(AIPG-2K)
43. Sandwich technique consists of:
a) GIC and composite b) Amalgam and GIC
c) Amalgam and composite d) None of the above
(PGI-03)
44. Resin modified glass ionomer is designed to:
a) Provide rapid setting to minimize cement acidity
allowing early finishing
b) Reduce the rate of fluoride release
c) Reduce the cost of the product
d) Change the translucency of the products
(MAN-97)
45. After 24 hrs. of setting tensile stength of GIC is:
a) Equal to Znpo 4 b) Greater to Znpo 4
c) Lesser to Znpo 4 d) None of the above
(AIIMS-06)
46. Bond found in GIC is:
a) Covalent bond. b) Ionic bond.
c) Hydrogen bond. d) Metallic bond.
(AIIMS-06)
47. Calcium Hydroxide is used in deep cavities in order to:
a) Form dentin bridge b) To kill microorganism
c) To cause necrosis d) As a base under restoration
(MCET-07)
48. Deep caries (<2 mm of dentine remaining) management
with resin composite restoration would include
a) Calcium hydroxide b) Glass-ionomer cement
c) Dentine bonding agent d) All of the above
(KAR-2013)
49. Vitremer is?
a) Resin modified GIC b) Metal modified GIC
c) Organically modified composite
d) Polyacid modified composite
(PGI JUNE-2012)
50. High Viscosity GIC was discovered in?
a) 1990 b) 1980
c) 1970 d) 2000
(PGI JUNE-2011)
51. Atraumatic Restorative Treatment (ART) technique
utilises
a) Glass ionomer cement b) Composite resin
c) Silver amalgam
d) Reinforced zinc oxide eugenol cement
(MHCET-15)

42) b 43) A 44) A 45) B 46) B 47) A 48) D 49) A 50) A 51) A
Dental ;lut.,e

2. DENTAL CEMENTS - ANSWERS


1. 'A' [Sturdevant 5th SA ed 411,412/ Sturdevant 5th ed 10. 'A' [Sturdevant 5th SA ed 407/ 5th ed 176]
311,312] Varnish is a natural gum such as copal rosin or synthetic
Calcium hydroxide, self cured or light cured GIC and certain resin dissolved in organic solvent such as alcohol, acetone
resin cements are the preferred agents of pulp protection or ether. As it is applied as thin films. Varnish neither
below composites. Varnishes and zinc oxide eugenol possess mechanical strength nor provide thermal insulation.
cements interfere with polymerization of composites.
11. 'D' [Sturdevant 4th ed 297/ 5th ed 312]
Which of the following materials SHOULD NOT be used
12. 'D' [Phillips 11th ed 447]
as liner or base under composite restorations
a) Resin Modified Glass lonomers Anticariogenic property of cements makes the enamel
b) Flowable composite {COMEDK-14) (fluorapatite) resistant to acid mediated decalcification.
c) Zinc Oxide Eugenol d) Compomer Silica-phosphate has the highest fluoride content followed
by silicate and GIC. Polycarboxylate has least fluoride
2. 'D' [Sturdevant 5th SA ed 410,412/ 5th ed 311] content. They release fluoride throughout the Life of
The need for specific liners and bases are linked to the amount restoration but rate of release decreases over time.
of remaining dentin thickness (RDT).
13. 'B' [Sturdevant 5th SA ed 412/ 5th ed 215]
The powder of traditional glass ionomer cement is a calcium
Shallow cavities, • No base, no liner. fluoro alumina silicate glass. The powder is referred to as
with RDT 1.5 - 2mm • Varnish or dentin bonding "ion leachable glass". It contains silica (35 - 40%), alumina
or more agent is indicated. (20 - 30%), aluminium fluoride (1.5 - 2.5%) and calcium
Moderately deep cavities ZOE liner except under and sodium fluorides {20 - 25%).
with RDT (1 - 1.5mm) composite restorations.
Lanthanum, st rontium, barium are added in t races for
Deep cavities with RDT Calcium hydroxide liner radiopacity. The fluorides acts as ceramic fluxes.
(0.5 - 1mm)

3. ' B' [Sturdevant 4th ed 297 / 5th ed 312]


Application of cavity varnish to the prepared walls of Po lyacrylic acid 45%
amalgam: Water 50% (hydrates reaction product)
• Improve the marginal seal of the restoration. Itaconic acid, Maleic
5% (reduces viscosity)
• Prevents micro leakage. acid, Tricarballyic acid
• Prevents discoloration by preventing the diffusion of Traces (increases working time
Tartaric acid
silver ions into the dentin. & decreases setting time)
• Reduces post operative sensitivity by reducing the
infiltration of fluids at the margins. 14. 'B' [Sturdevant 5th SA ed 408/ 5th ed 177]
Calcium hydroxide, because of its high pH value (11) acts as
Varnish applied to the dentinal surfaces of cast gold a pulp irritant and stimu lates re parative dentin formation .
restoration reduces pulpal irritation from Leuting cement. It is also used as a pulp capping agent both in direct and
indirect techniques.
4. 'C' [Sturdevant 5th SA ed 412/ 5th ed 311]
15. 'B' [Phillips 11th ed 458]
5. 'C' [Sturdevant 4th ed 176/ 5th ed 311] Fine grained for luting. Film thickness
Type I Zn PO,
is 25µ.or less.
6. ' B' [Sturdevant 5th SA ed 412/ 5th ed 311] Medium grained for luting and filling.
Type II Zn P0 4
Film thickness is 40~1.
7. 'D' [Phillips 11th ed 445]
16. 'C' [Phillips 111h ed 464]
8. 'B' [Phillips 11th ed 459]
Increased powder in zinc phosphate cement mix decreases
the setting time and solubility,(AP-05) and increases
9. 'A' [Marzouk 1" ed 363] strength and film thickness. But when the mixing slab is
The main objective of die spacing is to create gap for the cooled. the setting time increases and also permits the
final casting - cementing medium. I he commonly used die addition of more amount of powder to liquid without
spacers are insoluble paints applied in a thickness of 25m . developing high viscosity.
This creates space for a cement with thickness of 25m.
, OPERATIVE DENTISTRY

17. 'C' [Phillips 101h~ 568, 180] 28. 'C' [Sturdevant 5th SA ed 408/ Sturdevant 5th ed 175]
Zinc oxide powder reacts with eugenol and polyacrylic Liners are relatively thin layers of material used primarily
acid to form zinc eugenolat e and zinc polycarboxylate to provide a barrier to protect the dentin from residual
respectively. Both are examples of chelation reaction. reactants diffusing out of a restoration.

18. 'A' [Phillips 11th ed 252] Liners:- 2 types


Accelerators of setting reaction of ZOE cement: • Thin film liners
• Zinc acetate • Thick liners
• Alcohol
Thin film Liners (1 - S011m)
• Water Includes varnishes or solution Liners (2 - 5 11m) and
suspension liners (20 - 25 µ m)
• Glacial acetic acid
Thick liners
Retarders of setting reaction of ZOE cement
Also called cement liners (0.2 - 1mm). Used primarily for
• Cooling the glass slab pulpal medications and thermal protection.
• Glycerine
Bases (1- 2mm) provide thermal protection and mechanical
• Olive oil support for the restoration by distributing local stresses
from the restoration across the underlying dentin surface.
• Increasing liquid powder ratio
29. 'C' [Sturdevant 5th SA ed 412/ 5th ed 215]
19. 'C' [Phillips 11th ed 466]
Zinc polycarboxylate is the first cement to show adhesion
30. ' D' [Sturdevant 4th 173/ 5th ed 489]
with tooth structure. Glass ionomer is another such cement
pH of ZOE cement at the time insertion is 7 and it is one
to show chemical bonding with enamel.
of the least irritating of all dental cements and provides
excellent seal against leakage.
20. 'B' (Sturdevant ! th~ 475]
Silicate cement is a severe irritant to pulp tissues. Its pH
31. 'K [Phillips 11th ed 461]
is less than 3 at the time of insertion and it remains below
Working time of zinc polycarboxylate is 2 - 5min. and
7 even after one month. High solubility in oral cavity is
the working time of zinc phosphate cement is 5 min.
another disadvantage of silicate cement.
approximately.
21. 'C' [Phillips 11th ed 466]
32. ' B' (Phillips 11th ed 469]
The most biocompatible cements to the pulp are ZOE and
22. ' D' [Phillips 11th ed 469]
polycarboxylate cements.
pH of liquid of potycarboxylate cement is 1. 7. However,
the liquid is rapidly neutralized by the powder and as the
GIC, ZnPO,, resin cements and silicate cements have an
setting reaction pmceeds, the pH of mix rises rapidly. The
irritating effect on pulp.
larger size of polyacrylic acid molecules limits its diffusion
through dentinal tubules. Despite the initial acidic nature
33. ' B' [Phillips 11th ed 466]
of the polycarboxylate cements, they produce minimal
Zinc polycarboxylate, Zn P0 4, silicophosphate, GI(, ZOE and
irritation to pulp.
resin cements can be used as cementing or leuting agents.
23. ' B' [Phillips 11th ed 466]
34. ' B' [Phillips 10th ed 528]
Silicates have anti cariogenic action due to its high fluoride
24. ' D' [Sturdevant 4th ed 218/ 5th ed 253]
content (15%). So it is indicated in patients having high
Mechanical properties of ZOE cement can be improved by
caries index. If the restorations are allowed to dry it
adding alumina to the powder and orthoethoxy benzoic acid
becomes powdery and opaque aind t hus it is cont raindicated
to the liquid or by reinforcing with polymers. Reinforced
in mouth breathers.
ZOE cements are used for intermediate restorations.
35. 'A' (Sturdevant 4th ed 207]
25. ' D' [Phillips 11th ed 462]
Setting time of zinc phosphate cement can be increased
36. ' B' [Phillips 1Qth ed 576]
by decreasing the rate of addition of powder to liquid. But
ZOE and silicate cements have high solubility and
the best way to control setting time is by regulating the
disintegration rate. GIC and silicophosphate have Low
temperature of the mixing slab.
solubility compared to polycarboxylate and zinc phosphate.
Resin cement is least soluble in oral cavity.
26. ' B'
37. 'K [Sturdevant 4th ed 535/ 5th ed 503]
27. 'A' [Phillips 11th ed 446]
Dental ;lut.,e

38. ' B' Thus GIC is less stiff and more susceptible to elastic
deformation.
39. 'A' [Phillips 11th ed 472)
46. 'B' [Philips 11th ed 467, 475)
40. ' B' [Phillips 11th ed 479)
GIC lacks toughness, wear resistance and cannot withstand
high stress situations. GICs have been modified by the 47. 'A' [Sturdevant 5th SA ed 407/ Sturdevant 5th ed 857)
inclusion of metal filler particles to improve these properties A deep caries excavation close to the pulp which may result
in either an undetected pulpal exposure or a visible pulpal
i) Miracle mix or silver alloy admix: exposure should be covered with a calcium hydroxide liner
that can stimulate formation of dentin bridge (reparative
Ag-Sn powder is added t o GIC powder. None of the
dentin) over the exposure.
properties were improved and it gave a grey or blackish
colour to the cement.
48. 'D' [Sturdevant 5th SA ed 411,412/ Sturdevant 5th ed
311]
ii) Cermet: In this question. it is mentioned ROT is less than 2 mm. So
Glass and metal powders were sintered at high pulp protection should combine calcium hydroxide liners,
temperature and made to react with liquid. It improved resin modified glass ionomer and dentin bonding syst em.
the fracture toughness and wear resistance and at the GIC base protects the Liner from dissolution from the
same time maintained esthetics. etchant used for the composite system.

49. 'A' [Sturdevant 5th SA ed 411 Table)


Silver alloy admix is (APPG-15)
a) Low copper alloy Vitremer is a commercial preparation of resin-modified
b) Modified glass ionomer cement glass ionomer cement.
c) Single composition alloy d) Admix alloy
50. 'A' [Endodontics by Gulabivala 40)
41. The fast setting high viscosity glass ionomer was developed
' C' [Sturdevant ~th~ 535/ 5th ed 503]
Because of the anticariogenic property of GIC, it is the in the early 1990's. These were developed for ART
material of choice for restoring root surface caries in class (Atraumatic restorative) technique.
V locations. GIC is generally not indicated for class I, II or
IV restorations because of its limited strength and wear 51. 'A' [Refer Synopsis]
resistance.

42. ' B'

43. 'A' [Sturdevant 5th SA ed 263/ 5th ed 318)


Bi layered or sand witch restorations use GIC as a liner under
composite rest orations. It increases the retention form, as
GIC bonds both to the tooth and composite and the fluoride
content red uces secondary caries.

44. 'A' [Sturdevant 4th ed 209/ 5th ed 564]


Hybrid or Resin modified glass ionomer:
BISGMA, TEGDMA are added to GIC powder and HEMA to
the liquid. They are usually light cured, less technique
sensitive and may be finished at the time of placement.
The properties are superior to GIC and are recommended for
class V restorations and for class I and II in primary teeth.

COMPOMER (Polyacid modified composite resin):


It is a combination of composit e and GIC. Glass particles
are partially silanated (for bonding with the matrix) and are
added as fillers in the composite resin. There is no water in
the reaction. The properties are inferior to com po sites but
superior to GIC and resin modified GIC.

45. ' B' [Philips 11th ed 451)


Compressive strength and modulus of elasticity (stiffness)
are higher for Zincphosphate cement than GIC. But the
diametral tensile strength is less for zincphosphate than GIC.
, OPERATIVE DENTISTRY

3. AMALGAMS

1. Conventional dental amalgam alloy contains: 11, What appears to be the principle problem with the use
a) Silver, tin, copper and zinc of gallium alloys as an amalgam substitute?
b) Silver, mercury, copper and zinc a) Less compressive strength
c) Silver, tin, palladium and zinc b) More expansion which leads to tooth fracture
d) Silver, copper, iridium and mercury c) Poor biocompatibility
(MAN-94, AP- 05) d) Loss of corrosive products
2. Contamination of zinc-containing amalgam by moisture (MAN-97)
during trituration or condensation: 12. The percentage of unconsumed alloy in low copper amalgams is:
a) Causes a marked expansion a) 20 b) 9.2
b) Increases the setting time c) 8.8 d) 10
c) Results in a sharp decrease in flow (MAN-98)
d) Has no appreciable effect on amalgam 13. Threaded pins are used in large dental amalgam
(MAN-94) restorations to provide:
3. Advantages of pin-retained amalgam restoration are: a) Retention form b) Resistance form
a) Tooth preparation is more conservative c) Occlusal stops for opposing teeth
b) Restoration can be completed in one visit d) Much needed reinforcement of the amalgam
c) Retention form is improved by the use of pins (AIPG-03)
d) All of the above 14. If continuous hypersensitivity develops in recently
(MAN-94) restored tooth:
4. In which class of cavities is amalgam contraindicated: a) No treatment necessary
a) Class 1 b) Class 2 b) Remove the restoration and place ZOE
c) Class 4 d) Class 5 c) Perform pulpectomy
(MAN-95) d) Perform direct pulp capping
5. When a posterior occlusal amalgam makes multiple (AIIMS-03)
contacts with an occluding tooth surface, behaves as 15. The most common cause of fracture at the isthmus of a
though it is in: class II dental amalgam restoration:
a) Flexion b) Compression a) Delayed expansion
c) Shear d) Diametral compression b) Inadequate depth at the isthmus area
(MAN-01) c) Inadequate width at the isthmus area
6. A finished and polished amalgam will: d) Moisture contamination of the amalgam
a) Tarnish with time b) Not tarnish and corrode easily (AIPG-98)
c) Show secondary caries d) Do none of the above 16. The property ofamalgam that makes it undesirable to bevel
(MAN-95) occlusal margins of an amalgam cavity preparation is its:
7. Which of the following restorative materials are likely a) Flow b) Ductility
to be eliminated in near future: c) Brittleness d) Malleability
a) Amalgams b) Glass ionomer (AIIMS-2K)
c) Porcelain d) Composites 17. Occlusal isthmus of a M.O. dental amalgam is resistant
(MAN-98) to fracture if:
8. Average mercury content in well condensed amalgam a) Pulpal floor depth is 1mm
should not exceed: b) Dove tail is present
a) 30% b) 40% c) Axiopulpal line angle is rounded
c) 50% d) 60% d) Unsupported enamel at the gingival cavosurface margin
(MAN-98, 95) is planed
9. All are required for the satisfactory bonding of an (AIIMS-2K)
amalgam restoration in a badly broken molar except: 18. Mercuroscopic expansion in dental amalgam occurs when:
a) Resistance form b) Mechanical retention a) Moisture contamination occurs during condensation of
c) Enamel and dentin conditioning rinccont~~ngama~am
d) Pin retention b) Moisture contamination occurs during trit uration of zinc
containing amalgam
10. The pins in an amalgam restorative serve to: c) Mercury released as a result of electrochemical corrosion
a) Increase strength b) Increase retention re-reacts with Ag-Sn particle
c) Increase resistance of fracture d) Mercury released as a result of electrochemical corrosion
d) All the above re-reacts with Ag-Cu particle
(MAN-97) (KAR-99)

1) A 2) A 3) D 4) C 5) A 6) B 7) A 8) C 9) B 10) B 11) B 12) C 13) A


14) C 15) B 16) C 17) C 18) C
Dental ;lut.,e

19. The maximum amount of mercury vapour allowed in the 28. The tin mercury phase formed during amalgamation is
work place: called:
a) 0.005 mg/m3 b) 0.5 mg/m3 a) Gamma phase b) Gamma-lphase
c) 0.05 mg/m3 d) 0.025 mg/m3 c) Gamma-2phase d) Lathe cut
(KAR-99) (KAR-03)
20. For a class V cavity for amalgam: 29. Floor of class II cavity for amalgam restoration,
a) Mesia-distal walls are parallel and occluso-gingival occlusally should be:
walls converging a) In enamel b) In dentin
b) Mesia-distal walls diverging, occlusogingival walls c) In DEJ
converging d) Just above the roof of pulp chamber
c) MD walls diverging, OG walls diverging (AIPG-93)
d) MD walls and OG walls parallel 30. Cavosurface angle for amalgam is:
(AIPG-02) a) >90° b) < 90°
21. A patient complains of sensitivity in a Mandibular c) 90° d) 0°
premolar. A well condensed class V amalgam restoration ( AP-02,AIPG-9 3)
was placed in that tooth 5 months ago, with no 31. The discoloured corroded superficial layer frequently seen
discomfort for the first 4 months. Since then it has to the surface of amalgam restoration is most likely:
become increasingly sensitive. The problem probably a) Mercury b) A sulfide
relates to: c) Gamma 2 d) Copper oxide
a) Marginal leakage b) Fractured restoration (AIPG-91)
c) Irreversible pulpal damage 32. Direction of placement of retentive pin holes should be:
d) Tooth brush abrasion a) Perpendicular to external tooth surface
(AIPG-02, AIIMS-03) b) Parallel to external tooth surface
22. Improper handling of mercury in the clinic results in c) Parallel to ot her pins
occupational hazard through: d) Never beneath cusps to be restored
a) Absorption of mercury thro ugh skin (AIIMS-92)
b) Mercury along with food 33. Upper limit of urinary mercury attributed to extensive
c) Mercury along with fi lling amalgam restoration
d) Mercury along with water a) 1 µg/g of creatinine b) 3 µg/g of creatinine
(AP-03) c) 2 µg/g of creatinine d) 4 µg/g of creatinine
23. What is the minimal occlusal thickness that will provide (KAR-03)
resistance to fracture of amalgam: 34. Which of the following phase is virtually eliminated in
a) 1.0 mm b) 1.2 mm high copper amalgam:
c) 1.5 mm d) 1.8 mm a) Gamma-I phase b) Gamma-II phase
(KAR-01) c) Beta phase d) None
24. The most frequent complaint of patients shortly after (KAR-97)
amalgam restorations are placed is: 35. Mechanically, the strongest phase in amalgam restoration is
a) Pain on percussion b) An open contact point a) The original gamma phase (Ag3Sn)
c) Cold sensitivity b) The gamma I phase
d) Numbness around the gingiva c) The gamma II phase d) The copper - Tin phase
(PGI-98) (KAR-99)
25. An advantage of using a gold restoration rather than an 36. The most important cause for marginal failure of
amalgam restoration is: amalgam restoration is:
a) The ease of t ooth preparation a) Increased creep b) Decreased strength
b) Better adaptation to the walls of the preparation c) Tarnish and corrosion
c) The greater edge strength of cast gold d) Delayed setting of the material
d) It is less expensive (AIPG-91, TNPSC-99)
(PGI-98) 37. Greatest potential hazard of mercury toxicity occurs due to:
26. Corrosion in Amalgam cracks can be described as: a) Skin contact with mercury
a) Wet corrosion b) Stress corrosion b) Inhalation of mercury vapours
c) Crevice corrosion d) Electrolytic corrosion c) During amalgam restoration
(AP-01) d) Ingestion of amalgam scrap during removal
27. The force of condensation is directly related to: (AIIMS-01, AIPG-95)
a) Surface area of the condenser 38. Pin retained amalgam is not used in:
b) Material to be condensed a) Where margins of restoration rest below the gingiva
c) Physical strength of the operator b) Non vital t eeth
d) Weight of the condenser c) Hypoplastic teeth d) Vital teeth
(AP-98) (AIPG-96)

19) C 20) C 21) C 22) A 23) C 24) C 25) C 26) C 27) A 28) C 29) B 30) C 31) B
32) B 33) D 34) B 35) A 36) A 37) B 38) C
, OPERATIVE DENTISTRY

39. A radiopaque area is observed under a 3-month old d) Short and rounded at t he dentino enamel junction of the
silver amalgam filling. The base used most probably was: buccal and lingual walls
a) Calcium hydroxide b) ZOE (APPSC-99)
c) Formocresol d) Zinc oxyphosphate 49. Polishing agent for amalgam
(PGI-2K, AIPG -06) a) Tin oxide b) Iron oxide
40. Colour dye detector is used for: c) Alumina d) Silica
a) Detecting incipient carious lesions (AP-02)
b) Caries susceptibility 50. The main difference between composite and amalgam as
c) Differentiating between affected and effected dentin restorative material is:
d) All of the above a) Occlusal wear b) Durability
(PGI-95) c) Retention d) Manipulation
41. Bevel in class II amalgam filling is given for: (AIPG-96)
a) Increased retention b) Increased resistance 51. The most retentive type of pin is
c) To prevent microleakage d) Aesthetic reasons a) Cemented pin b) Self threading pin
(AIPG-97) c) Friction locked pin d) amalgam pins
42. Which of the following weakest phase of the set (TN PSC-99. PGI-03)
amalgam: 52. Friction Locked pins are usually retained by
a) Beta-2 b) Gamma a) Threads on pin b) Resiliency of dentine
c) Gamma-1 d) Gamma-2 c) Luting cement d) Channel depth
(KAR-2K, AP- 06) (KAR-98)
43. An overhanging proximo-occlusal amalgam restoration 53. Mercury vapourisation seen during processing while
leads to: using with premeasured capsule during
a) Decreased retention b) Increased retention a) Condensation b) Carving
c) Increased plaque accumulations c) Mulling d) Trituration
d) Fracture of the restoration (AP-04)
(KAR-97, AIPG-02) 54. Angle at gingival bevel in class II inlay cavity
44. After a class V amalgam restoration, patient experiences a) 30° b) 10°
pain, which may be due to? c) 5° d) 50°
a) Dentin (Hyper) sensitivity (AP-02)
b) Irreversible pulpal damage 55. Difference between class II amalgam and cast
c) Injury to adjacent gingiva restoration is
d) Any of the above a) Bevels b) Occlusal resistance form
(AIIMS-92) c) Outline form d) Proximal retention
45. The type of stress to which dental amalgam is most (PGI-03)
resistant: 56. The highest mercury concentration in amalgam filling is
a) Shear's stress b) Tensile stress found
c) Compressive stress d) Impact stress a) At the margin of the restoration
(KAR-97) b) In the centre of the restoration
46. Amalgam contributes to etiology of chronic periodontal c) In the deepest part of the restoration
disease overlong because it acts as: d) None of the above
a) A mechanical irritant b) Plaque retaining area (MAN-94)
c) Nondeficiency tooth surface 57. Admixed high copper alloy powder contains
d) overcontoured tooth surface a) 9-20% Copper b) 13-20% copper
(AIPG-91) c) 9-30% copper d) 13-30% copper
47. Point angles in class II (MAN-01)
a) 4 b) 6 58. Which of the following statements about high copper
c) 5 d) 3 silver alloy compared to conventional alloy in not true?
(AP-02) a) It has increased tensile and compressive strength
48. Retentive grooves of a proximal box in a class II cavity b) It has poor tarnish and corrosion resistance
prepared for amalgam should be: c) Its edge strength is greater
a) Sharp and elongated at the dentino enamel junction of d) Mercury content in the final restoration is less
the buccal and lingual walls (AIIMS-03)
b) Sharp and well defined extending from the gingival to 59. Setting time of amalgam is best controlled by
the occlusal cavosurface angle along the buccal and a) Using spherical particles b) Lathe cut alloy
lingual line angles c) Altering Hg-alloy ratio d) Trituration time
c) Elongated and rounded in dentine at the buccoaxial and (PGI-02)
linguoaxial line angles extending from the gingival wall 60. In spherical alloys as compared to lathe cut:
to the axiopulpal line angle a) Less condensing force is req uired

39) A 40) C 41) B 42) D 43) C 44) D 45) C 46) B 47) B 48) C 49) A 50) A 51) B
52) B 53) A 54) A 55) A 56) A 57) A 58) B 59) D 60) A
Dental ;lut.,e

b) More condensing force is required 72. The lower blood mercury level at which the earliest non
c) Both require same condensing force specific symptom starts appearing at:
d) Manipulation is easy a) 25 ng/ml b) 35 ng/m l
(AIIMS-95) c) 40 ng/ml d) 45 ng/m l
61. Advantage of Zinc containing amalgam is: (AIPG-07)
a) Better handling property b) Dimensional stability 73. Corrosion of amalgam restoration:
c) Resistance to creep d) Toxicity to pulp and dentin a) Can extend upto a depth of 100 - 500 µm
(PGI-95) b) Decreases if tin content of alloy increase
62. Cause of expansion in Zinc containing amalgam is: c) Is promoted by gama phase of alloy particles
a) Zinc+ water b) Water d) Is resisted the most by copper-tin phase in high copper
c) Hydrogen d) Nascent oxygen amalgams
(PGI-95) (AIPG-06)
63. Which of the following amalgam alloys is Least 74. What fraction of inhaled mercury vapour is retained in
susceptible to creep: the body?
a) Lathe cut b) Spherical a) 45 - 50% b) 55 - 65%
c) Microfine d) Dispersion with high copper c) 65 - 85% d) > 85%
(PGI-2K) (AIPG-06)
64. How soon after a moisture contamination does a zinc 75. The ideal ratio of pin in pin restoration between dentin
containing amalgam alloy start expanding: and exposed surface of pin is:
a) 24 hours b) 1-2 days a) 3 : 1 b) 2 : 1
c) 3-5 days d) 7 days c) 1 : 1 d) 1 : 2
(PGI-92) (AIPG-07)
65. Which of the following constituents of amalgam alloy 76. In amalgam cavity preparation the cavosurface beveled at:
decreases expansion a) Less than 90° b) Less than 45°
a) Copper b) Zinc c) More than 10° d) At an obtuse angle
c) Silver d) Tin (AIPG-07)
(KAR-97) 77. The plane of the axial wall of an M.O. cavity prepared for
66. Which of the following silver amalgam alloys have the silver amalgam should be
maximum strength a) A flat straight plane bucolingually
a) Lathe cut b) Spherical b) Parallel to the Long axis of the teeth
c) Admixed d) Single composition c) Concave to allow greater bulk of amalgam
(PGI-97) d) Parallel to the Long axis of tlhe teeth and should follow
67. Which is the recommended force for condensation of the buccolingual contour of the tooth
dental amalgam (AP-08)
a) 35 pounds b) 3 pounds 78. Which of the following is the most preferred luting
c) 15 lb d) 25 Lb agent used for cementing pins in vital teeth?
(COMEDK-04) a) Zinc phosphate
68. A pin retained amalgam: b) Zinc phosphate with cavity Liner
a) Does not reinforce the tooth c) Poly carboxylate d) Glass Ionomer
b) Weakens the tooth (KAR-01)
c) Reinforces the amalgam structure 79. Precarving burnishing is helpful except
d) Improves retention of restoration a) Premalgam removal
(KAR-04) b) To remove more marginal mercury
69. How much fraction of methyl mercury is absorbed from c) To remove old amalgam restoration
the Gut: d) For condensation of marginal mercury
a) 20% b) 50% (AIPG-2012)
c) 80% d) 100% 80. In amalgam filling there is a wide buccolingual space,
(AIPG-06) the wedge technique used should be (OR) If you are
70. Creep rate decreases with: going to restore Lower molar with amalgam which is
a) Increase force of condensation. having cavity with proximal box wide buccolingually
b) Decrease force of condensation. which wedging technique will you use
c) Under or overtrituration. a) Double wedge b) Wedge wedge
d) Dealy in time between trituration and condensation. c) Single wedge d) No wedge
(AIIMS-06) (AIPG-2012, AIIMS NOV-14)
71. The pin used in pin restoration of a root canal treated tooth is: 81. Cavity with normal depth is closest to pulp in
a) Self thread pin b) Cemented pin a) Class I Amalgams b) Class II Amalgam
c) Frictional pin d) Any of the above c) Class V Amalgam d) Cast restortion
(AIIMS-06) (BHU-07)

61) A 62) C 63) D 64) C 65) D 66) D 67) C 68) D 69) D 70) A 71) B 72) B 73) A
74) C 75) B 76) D 77) D 78) A 79) C 80) A 81) C
, OPERATIVE DENTISTRY

82. What is the optimum depth for a pinhole in a tooth with 92. A well packed premeasured capsule without any leak
a vital pulp releases mercury during
a) 4-5 mm b) 2-3 mm a) Condensation b) Trituration
c) 2 mm d) 1-1.5 mm c) Carving d) Mulling
(BHU-07) (AP-10)
83. Axiopulpal line angle is bevelled in amalgam preparation 93. In self threaded pins, the pirn hole is smaller than pin
to facilitate channel by diameter:
a) Bulk for amalgam in this critical zone a) 0.001 inch b) 0.002 inch
b) Easy condensation of amalgam c) 0.003 inch d) 0.004 inch
c) For containment of amalgam within the matrix (AIPG-10)
d) To increase the strength of remaining tooth structure 94. Which is the restoration of choice in a patient having
(COMEDK-08) moderate carious lesion with heavy occlusal forces?
84. Conservative cavity for amalgam restoration is made using: a) gold foil b) amalgam
a) 33 1/2 inverted cone bur c) cast inlay d) ceramic inlay
b) No. 245 non standardized bur (AIPG-10)
c) Pear shaped bur d) Taper fissure bur 95. Discolored areas seen through the enamel in the teeth
(MCET-07) that have amalgam restorations are known as
85. What is the major difference between a class V cavity a) Amalgam tattoos b) Amalgam overhangs
preparation for amalgam and one for composite resin by c) Amalgam blues d) Amalgam ditchings
the acid-etch technique? (KCET-201 1)
a) Depth b) Convenience form 96. One of the following statements regarding pin retained
c) Position of retentive points restoration is NOT true
d) Angulation of enamel cavosurface margins a) Pinholes should never be prepared at different Levels on
(AIPG-09) the tooth
86. When the pins are used in the cavity for amalgam, the b) A minimum of 0.5 mm clearance should be provided
strength of amalgam? around the circumference of the pin for adequate
a) Increased b) Decreased condensation of the amalgam
c) Unchanged d) May increase or decrease c) Pinhole should located on a flat surface that is
(AIPG-09) perpendicular to the proposed direction of the pinhole
87. Most modern precapsulated dental amalgam formulated d) The pinhole should be positioned no closer than 1
with mm to DEJ and no closer than 1.5 mm to the external
a) 50-55% hg by wt. b) 40-45% hg by wt. surface.
c) 25-35% hg by wt. d) 15-25% hg by wt. (KCET-2011)
(AIIMS-2011) 97. In pin retained amalgam restoration, failure is more
88. After amalgam polishing, the outermost surface has likely to occur at the
crystals arranged flattened to tooth surface. This layer a) Pin restoration interface
is known as b) Pin dentin interface
a) Beilby layer b) Beillyl layer c) Both of the above
c) Beibyl layer d) none of the above d) Inside of restoration (Restoration fracture)
(AIIMS-08) (COMEDK-2011)
89. Low creep values of high copper amalgams increase its 98. The color code for the " Regular" size pins in the TMS
a) Solubility b) Corrosion (Thread Mate System) pins system is
c) Brittleness d) Hardness a) Gold b) Silver
(BHU-2012) c) Pink d) Red
90. Indium is added to amalgam? (KCET-2012)
a) It reduces mercury release during mastication 99. In mechanical trituration of amalgam, what is adversely
b) It reduces mercury release during polishing affected?
c) It reduces gamma phase a) Hardness of filling b) Tarnish resistance
d) It reduces gamma 2 phase c) Working time d) Final gloss of the filling
(AIPG-09) (AIMS NOV-13)
91. Burnishing of dental amalgam restoration does NOT 100. The pin that causes minimum stress in dentin is
a) reduce surface microporosity a) Threaded pin b) Cemented pin
b) reduce creep c) Amalga pin d) Friction Lock pin
c) improve the marginal seal (GCET-14)
d) reduce polishing time 101. Amount of fluoride in amalgam is
(AP-09) a) 8% SnF2 b) 10% SnF2
c) 1% NaF d) 1% sNf2
(PGI JUNE-2012)

82) C 83) A 84) C 85) D 86) B 87) B 88) A 89) C 90) B 91) D 92) A 93) D 94) B
95) C 96) A 97) B 98) A 99) C 100) B 101) D
Dental :f~e

102. Least amount of mercury is required in


a) Admixed alloy b) Spherical
c) Hybrid d) Lathe cut alloy
(AIIMS MAY-14)
103. Who was associated with second amalgam war?
a) G V Black b) Charles bell
c) Huggins d) Alfred stock
(AIIMS NOV-14)
104. The clinical condition shown by the arrow in the below
picture depicts

a) Amalgam tattoo b) Late ecchymotic lesion


c) Necrosis of gingiva d) Pigmentation
(APPG-15)

102) B 103) D 104) A


, OPERATIVE DENTISTRY

3. AMALGAMS - ANSWERS

1. 'A' [Sturdevant 5th SA ed 340/ 5th ed 152] 9. ' B' [Sturdevant 4th ed 765/ 5th ed 811]
Conventional or traditional amalgam alloys contain Silver Usually a weakened tooth is best restored with a cast
65%, Tin 30%, Copper 5% and Zinc < 1°lo. restoration which prevents tooth fracture caused by
mastication forces.
2. 'A' [Sturdevant 5th SA ed 341/ Sturdevant 5th ed 155]
Delayed expansion occurs when zinc containing alloys Improved resistance form, pins, slots and amalgam bonding
are contaminated by moisture during trituration or techniques may be required for developing the retention
condensation reaching upto 400 µm/cm. This occurs and resistance forms in adclition to the conventional
3 - 4 days after insertion of amalgam. This is due to formation features.
and accumulation of hydrogen gas in the restoration leading
to pain, sensitivity and protrusion of the restoration. 10. ' B' [Sturdevant 5th SA ed 391/ 5th ed 810]
Refer to Q No. 3
3. 'D' [Sturdevant 5th SA ed 390/ 5th ed 810]
Advantages of pin retained amalgam restorations: 11, ' B' [Vimal Sikri 3n1 ed 234]
• Conservative tooth preparation Amalgam alternatives comprise any materials (composites,
GIC, cast gold alloys) t hat can be used to restore a tooth
• Can be completed in one appointment
instead of using amalgam. Amalgam substitutes contain
• Improved resistance and retention form some components of amalgam (Ag-sn alloy particles) but do
• In expensive compared to indirect restorations not contain mercury. Gallium alloys contain Ag-sn particles
in Ga-Indium liquid.
Disadvantages
• Decreased strength of amalgam Disadvantages of Gallium alloys:
• Microleakage • Surface roughness
• Dentinal microfractures • Expansion leads to tooth fracture
• Marginal discoloration
Note: (KERALA-15) • Difficult manipulation
The relative position of the pin from DEJ in pin-retained
amalgam must be 0. 5-1 mm 12. 'C'

4. 'C' [Sturdevant 5th SA ed 351/ 5th ed 711] 13. 'A' [Sturdevant 4th ed 771]
Indications of amalgam restorations
• Most retentive pin
• Moderate to large class I and class II restorations
• Most frequently used pin
• Class V restorations in nonesthetic areas
Self • The pinhole is 0.0015 to 0.004 inch smaller
• Foundations for badly broken down teeth. than the diameter of the pin
treaded
• Retained by the resiliency of the dentin
5. 'A' [Sturdevant 4th ed 157, 146] pins
• Produces maximum crazing in dentin.
During direct contact by opponent teeth, cusps and/or
amalgam restorations are stretched laterally, producing • The pin should extend 2mm into dentin
tension and perhaps flexion. and 2mm into amalgam
• least retentive
6. ' B' [Vimal Sikri 3 n1 ed 483] • Produces least crazing of dentin
Amalgams without finishing and polishing will have a rough
surface microscopically and results in concentration cell Cemented • The pin should extend 3mm into dentin
and 2mm into amalgam
type of corrosion. pins
• The pinhole should be 0.025 to 0.05mm
larger than the diameter of pin
7. 'A' [Sturdevant 5th SA ed 358/ 5th ed 691]
• Used mostly in nonvital teeth.
8. 'C' [Phillips 11th ed 522] • It is intermediate retentive
The mercury content of finished restoration should be 50wt • Produces crazing and cracks in dentin
°lo approximately. Restorations containing mercury in excess
Friction • The pin should extend 3mm into dentin
of 55% showed an higher incidence of marginal fracture and and 3mm into amalgam
surface deterioration. locked
• The pinhole is 0.025mm smaller than the
pins diameter of the pin
The mercury content of finished restoration with spherical
alloys contain 42 wt% approximately. • The pins are retained by the resiliency of
dentin
Dental ;lut.,e

14. 'C' Types of electrochemical corrosion


15. ' B' [Vimal Sikri 3n1 ed 218,232] Occurs between dissimilar metals in
Galvanic
Enough thickness of restorative material is required to contact. Eg:- Amalgam opposing cast
Corrosion
prevent its fracture under load. Inadequate depth leads to gold rest oratiorn.
inadequate resistance form which leads to fracture of the Metal at the site of maximum stress
Stress
restoration. becomes more reactive than the
corrosion
unstressed metal.
Excess width of isthmus significantly reduces the resistance Crevice or
form of the tooth and restoration. Corrosion occurring in cracks or crevices
concentration
due to accumulation of plaque.
cell corrosion
16. ' C' [Sturdevant 5th SA ed 368/ 5th ed 699)
All amalgams are brittle and have low edge strength. 27. 'A' [Vimal Sikri 3n1 ed 229)
Therefore a 90° cavosurface angle (Buttjoint) is placed on
the cavosurface margins. 1
Condensation force a (D"1amet er of N"b)
1 2
Bevels are contraindicat ed in amalgam rest orations except 28. 'C' [Sturdevant 5th SA ed 342/ 5th ed 157)
the for gingival wall where a 15-20° bevel removes the
unsupported enamel rods. 29. 'B' [Sturdevant 5th SA ed 355/ 5th ed 303)
The pulpal wall of class I and class II should be flat with
17. 'C' [Sturdevant 5th SA ed 382/ 5th ed 701) 1.5-Zmm overall depth or 0.2mm into dentin.

18. ' C' [Sturdevant 5th SA ed 348/ 5th ed 161) 30. 'C' [Sturdevant 5th SA ed 356/ 5th ed 699)
Electrochemical corrosion produces extrusion at margins
(creep), during which mercury from Sn-Hg(yz) rereacts 31. 'B' [Sturdevant 5th SA ed 345)
with Ag-Sn (y) particles and produces further called
Mercuroscopic expansion. 32. 'B' [Sturdevant 5th SA ed 395/ 5th ed 818]
• One pin per missing axial line angle should be used.
19. ' C' [Sturdevant 5th SA ed 350/ 5th ed 165)
The maximum safe level of occupational exposure is 50 • The fewest pins possible should be used to achieve the
micrograms/m 3 of air. The primary risk to dental personnel desired retention for a given restoration.
is from inhalation. • The pins should be non parallel to one another.
• Pinholes should be parallel to the adjacent external
20. 'C' [Sturdevant 4th ed 756/ 5th ed 799] surface of the tooth.
The mesial, distal, gingival and incisal walls of class V
• At least 1mm of sound dentin should be present around
cavity diverge facially. So auxillary retention form is must pin.
for class V. It may be in the form of 2 retention grooves
along axioincisal and axiogingival line angles or 4 retention • The pin should extend 2mm into the dentin.
coves along 4 axial point angles.
Pin for core foundation to what depth in dentin
The grooves and coves should be placed in dentin. restoration (PGI June-2013)
a) 0.5 b) 1 mm
21. ' C' c) 1.5 mm d) 2 mm

22. 'A' [Vimal Sikri 3,d ed 242) 33. 'D'

23. ' C' [Sturdevant 5th SA ed 353/ 5th ed 305) 34. 'B' [Sturdevant 5th SA ed 344/ 5th ed 692]
Restorative material Minimal occlusal thickness
35. 'A' [Sturdevant 4'h ed 15 7/ 5th ed 160]
Amalgam 1.5 mm Increasing order of strength:
Porcelain 2mm • Low copper< admix alloy< single composition
Cast metal 1-Zmm • (y) < < (y)

24. 'C' 36. 'A' [Phillips 11th ed 540]

25. 'C' [Sturdevant 5th SA ed 430/ 5th ed 848) 3 7. 'B' [Sturdevant 4th ed 160)

26. 'C' [Sturdevant 5th SA ed 346/ 5th ed 159) 38. 'C' [Vimal Sikri 3n1 ed 286 / Marzouk 1'' ed 203)
Amalgams undergo two kinds of corrosion, chemical (dry)
and electrochemical corrosion (wet).
, OPERATIVE DENTISTRY

39. 'A' [Sturdevant 4 1h ed 173/ 5th ed 177) 51. ' B' [Sturdevant 5th SA ed 392/ 5th ed 816/ Ref. Q. No.13)
Calcium hydroxide is extremely caustic (pH >11) but when
placed in low concentrations it stimulates the formation of 52. ' B' [Sturdevant 5th SA ed 392/ Sth ed 815/ Ref Q.No.13)
reparative dentin.
53. 'A' [Vimal Sikri 3rd ed 241)
40. 'C' [Vimal Sikri 3"' ed 102/ Sturdevant 5th SA ed 47) As premeasured capsule is triturated and mulled in
an enclosed chamber and i n capsule itself, mercury
41. ' B' [Sturdevant 4th ed 554/ 5th ed 746) vapourization is less evident. hence, mercury vapourzation
will occur during condensation.
42. 'D' [Sturdevant 5th SA ed 343/ 5th ed 15 7)
54. 'A' [Sturdevant 5th SA ed 439/ 5th ed 859)
43. 'C' [Vimal Sikri 1' 1 ed 164)
55. 'A' [Sturdevant 5th SA ed 368,439/ 5th ed 859]
44. ' D'
56. 'A' [Phillips 10!!!_!4 405)
45. 'C' [Phillips 111h ed 511) The concentration of mercury in the margins will be 2-3%
Amalgams have high compressive strength and low tensile higher than the remaining bulk of restorations. These areas are
strength. Compressive strength of amalgam should be critical in terms of corrosion, fracture and secondary caries.
minimum of 310 Mpa and the tensile strength should be in
between 48-70 Mpa. 57. 'A' [Phillips 11'hed 505]
• Low copper alloys - copper is <6% by wt
46. ' B'
• High copper alloys
- Single composition alloys Cu is 13-30% by wt.
47. ' B' [Sturdevant 5th SA ed 163/ Sth ed 297]
- Admixed alloys Cu is 9-20% by wt.
Type of cavity I II III V MOD
Point angles 4 6 3 4 8 58. ' B' [Phillips 10'h ed 3 69)
Line angles 8 11 6 8 14
59. 'D' [Phillips tO'hed 392)
Proximal cavity walls - 4 4 - 8 The setting time is best controlled by altering the trituration
speed or trituration time or both. For a given type of alloy,
48. 'C' [Sturdevant 5th SA ed 382/ 5th ed 748] increased trituration time or speed shortens the working
Retentive grooves or locks provide retention against and setting times.
proximal displacement. They are placed in the axiofacial and
axiolingual line angles in ClassII cavity 0.2 mm into dentin. Spherical alloys require less amalgamation time than the
lathe cut alloys.
49. 'A' [Vimal Sikri 3,d ed 482)
AGENT USES 60. 'A' [Phillips 11th ed 514]
Spherical amalgams (Which contain less mercury) condensed
Smoothening denture, polishing of
Pumice with lighter pressures produce adequate strength.
artificial teeth
Zinc oxide Polishing of amalgam restoration High condensation pressures are required to minimize
Rouge porosity and to expel mercury from lathe cut amalgams.
Polishing of noble metal alloys
(Iron oxide)
61. 'A' [Phillips 10!!!_!4365, 366]
Zirconium silicate Prophylactic paste for polishing
Alloys containing less than 0.01% of Zinc are classified as
natural teeth
non-zinc alloys and the alloys with more than 0.01°lo of zinc
Chromic oxide Stainless steel are classified as zinc containing alloys.
Corundum (white
Grinding of metal alloys Uses of Zinc:
form of Al20)
• Reduces brittleness
50. 'A' [Sturdevant 5th SA ed 234/ 5th ed 508) • Prevents oxidation of silver, copper or tin. Thus it acts
Properties of composites: as deoxidiser or scavenger.
• Linear coefficie111t of thermal expansion is 3 times that • Better handling property.
of a tooth structure.
• Wear resistance is less than amalgam 62. 'C' [Phillips 11th ed 510)
• Smooth surface texture
• Insolubility in oral fluids
• Water absorption
• Low modulus of elasticity
Dental ;lut.,e

63. 'D' [Phillips 11th ed 515] Factors which decrease creep:


Type of alloy % of creep • Low mercury, alloy ratio
Low copper alloys 0.8-8% • High condensation forces
Admixed 0.4-0.5% • Careful attention to the timing of trituration and
Single composition 0.05-0.09% condensation
• Use of High copper alloys
64. ' C' [Phillips 11'h ed 510]
71. 'B' [Vimal Sikri 3,d ed 287]
65. 'D' [Check Explanation Below]
Tin controls the reaction between silver and mercury. 72. 'B'
Without tin, the react ion would be too fast and the setting
expansion would be unacceptable. Tin reduces both the rate 73. 'I([Craig 10th ed 30]
of reaction and the expansion to optimal values. Corrosion in high copper amalgam is resisted most by gama
one phase (Ag 2 Hg 3 ) followed by eta phase (Cu 6Sn 5 )
66. 'D' [Phillips 11th ed 512]
74. 'C'
67. 'C' [Phillips 11th ed 529]
Eventhough forces as great as 15lb (67N) are recommended 75. 'B' [Sturdevant 5th SA ed 391/ Sturdevant 5th ed 816]
for condensation of amalagam, it is doubtful that forces of
that magnitude are generally used. A study of condensation 76. 'D' [Sturdevant 5th SA ed 368/ Sturdevant 5th ed 692,
forces applied by 30 practioners showed that forces in the 699]
range of 13.3 to 17.8N (3- 4 lb) represent the average Among the given options 'D' is the best answer. Strongest
force (AIIMS-06) employed. enamel margin is obtained by:
• Full length enamel rods whose inner ends are on sound
To ensure maximum density and adaptation to the cavity dentin.
walls, the condensation forceshou ld be as great as the alloy
• Cavosurface angle of 90° or more gives strongest
will allow, consistent with patient comfort. It is doubtful enamel margin.
that condenser points greater than 2 mm in diameter will
provide adequate condensation of lathe-cut alloys. • Cavosurface angle should never be <90° as it creates
weak enamel margins.
68. ' D' [Sturdevant 5th SA ed 391/Sturdevant 5th ed 309]
For explanation refer to Q.No. 3 77. 'D' [Sturdevant 5th SA ed 3 77 / Sturdevant 5th ed 740]

69. 'D' [Philips 11th ed 198] 78. 'A' [Marzouk 1st ed 209]
In case of cemented pins, the most retaining cement is zinc
Some important values which are frequently asked:
phosphate followed by polycarboxylate cement, ZOE, etc.
• Methyl mercury is the most toxic form and is absorbed Using varnish with zinc phosphate cement will decrease the
90 - 95% from t he gut. retaining power by almost 40%. For nonvital teeth, copper
• Maximum level of occupational exposure that is phosphate cement is the most retaining cement followed by
considered safe is 50119 of Hg/mm 3• zinc phosphate.
• Hg is volatile at room temperature and has a vapour
pressure of 20mg/m3 which is 400 times the maximum 79. 'C' [Sturdevant 5th SA ed 370/ Vimal Sikri 1st ed 524,
acceptable level. 227]
• Hg is 14 times denser than HF
80. 'A' [Sturdevant 5th SA ed 209/ Sturdevant 5th ed 761]
• About 65-85% of inhaled mercury vapour is retained in
the body. (AIPG-06)
81. 'C'
• Half life of Hg is 55 days {COMEDK-15).
• 15JJg/l of Hg in urine and 4ng/ml of mercury in blood 82. 'C' [Sturdevant 5th SA ed 391/ Sturdevant 5th ed 815]
is the acceptable normal level. The depth of the pinhole varies from 1.3 to 2mm, depending
• The lower blood mercurylevelatwhich earliest non specific upon the diameter of the pin used. A general guide line for
symptoms starts is 35ng/ml and 3011g/ml in urine. pinhole depth is 2mm.

83. 'A' [Sturdevant 5th SA ed 382/ Sturdevant 5th ed 305]


An average half life for the transport of mercury
through the body to the point of excretion of
84. 'C' [Sturdevant 5th SA ed 363/ 5th ed 720, 713]
(COMCDK-15)
Burs used for conservative cavity for amalgam restorations:
a) 50 days b) 55 days
c) 60 days d) 65 days • No. 245 inverted cone bur
• No. 330 bur (pear shaped)
70. 'A' [Philips 11'h ed 516] • No. 169L bur
, OPERATIVE DENTISTRY

The No. 330 bur is a small and pear shaped version of No. Cast inlay
245 bur. This pear-shaped bur is indicated for the most Indicated for large restorations as an alternative to
conservative amalgam preparations. amalgam or composit e when higher strengt h of a casting
alloy is needed or when superior control of contours and
The most conservative tooth preparation for amalgam contacts is desired.
is prepared with (MHCET-15)
a) No.245 bur b) No.330 bur Gold foil restorations
c) No.169L bur d) No.27 1 bur For small carious lesions in pits and fissures of posterior
teeth and the lingual surfaces of anterior teeth. Also for
85. 'D' [Sturdevant 4th ed 529) small proximal surface carious lesions in posterior teeth in
which marginal ridges are not subjected to heavy occlusal
86. 'B' [Sturdevant 5th SA ed 391/ Sturdevant 5th ed 814) forces.

87. ' B' [Sturdevant 5th ed 154) 95. 'C' [Sturdevant 4th ed 409)
Most modern precapsulated amalgams are form ulated wit h Amalgam blues are dark discoloration seen in the t oot h
only 42- 45% by weight. structure adjacent to amalgam restoration. They mimic
secondary caries. It is caused by the gradual diffusion of
88. 'A' [Vimal Sikri ~ Ill ed 476) metallic ions from amalgam into t he dentin. Ot her t han
The polishing agent remove material from the surface esthetics, it does not warrant replacement.
molcule-by-molecule and thus provides a very smooth Amalgam tattoo is discoloration of t he mouth mucus
surface. The microcrystalline layer is referred to as polish membrane resulting from amalgam particles falling into
layer or Beilby layer. small, open wounds created during dental treatment.

89. 'C' [Philips 11th ed 515-16) 96. 'A' [Sturdevant 5th SA ed 395/ Sturdevant 5th ed 819)
High copper amalgams have significant low creep values Whenever 3 or more pinholes are placed, they should be
(<0.4%). This low creep decreases corrosion and solubility. located at different vertical levels on the tooth . This reduces
But however the increased copper content does not have stresses resulting from pin placement in the same horizontal
any effect on brittleness which is inherent with amalgams. plane of the tooth. For other options, refer Q. No 87.

90. ' B' (Journal of Dental Research 1989; 68; 1231 - 33) 97. ' B' [Sturdevant 5th SA ed 398/ Sturdevant 5th ed 828)
Admixed indium in amounts of 4% and greater significantly Failure is more likely to occur at t he pin-dentin interface
decreases the amount of mercury vapour released from than at the pin restoration interface. For explanation, refer
amalgams in 2 ways. point No.36 under pin-retained restorations in synopsis.
i) Indium reduces the amount of liquid mercury required
98. 'A' [Sturdevant 5th SA ed 392/ Sturdevant 5th ed 817)
for the amalgamation reaction. Indium decreases the
surface tension of liquid metals, thus less mercury is
needed for wetting alloy particles.
ii) Indium reduces the release of mercury vapor from the
amalgam, especially during t he setting phase of the Regular
amalgam, especially during t he first 30 mins following Gold
(0.031 inch or 0.78 mm)
trituration.
Minim
Silver
(0.024 inch or 0.61 mm)
91. 'D' [Vimal Sikri 3"' ed 481)
Mini kin
Red
92. 'A' (0.019 inch or 0.48 mm)
Minut a
93. 'D' [Sturdevant 5th SA ed 391/ Sturdevant 5th ed 815) Pink
(0.0.15 inch or 0.38 mm)

94. 'B' [Sturdevant 5th SA ed 351/ Sturdevant s •h ed 694, 99 , 'C' [Phillips 11th ed 525)
924) Mechanical trit uration in an amalgamator uses capsules
Amalgam restorations: that contain preweighed portion of silver powder along
They are indicat ed in clinical situations t hat have heavy with appropriate quantity of mercury.
occlusal function. It may also be more appropriate where a
restoration restores all of the occlusal contact for a tooth. Advantages of mechanical trituration:
• Mixing time can be controlled i.e., time can be set
Amalgam is most considered for:
• Moderate to large class I or class II (including • Attainment of proper consistency of mix
restorations that involve heavy occlusion, that cannot • Achieves maximum strength
be isolated well or that extend onto the root surface)
• Class V restoration that are not esthetically critical.
Dental ;lut.,e

Amalgamator speeds: • In areas in and around the healed extraction sites.


• Low: 3200 to 3400 cycles/min
• Medium: 3700 to 3800 cycles/min
• High: 4000 to 4400 cycles/min

Note:
• For a given alloy/mercury ratio, increased trituration
time and/or speed shortens the working and setting
time.
• Low copper alloys require low amalgamation speed
• High copper alloys require high speed
• Spherical alloys require less amalgamation time than
lathecut alloys.

100. 'B' [Check Explanation of Q. No 13 J


The pinhole is 0.025 to 0.05 mm larger than the diameter
of cemented pin. So cemented pins cause minimum stress
and least crazing in dentin while insertion.

101. 'O' [Fluorides in caries prevention by John. J. Murry 3rd


ed 203]
Fluoride has been incorporated in amalgam for anti caries
effect. Many studies demonstrated that 1°lo stannous fluoride
was effective in reducing the incidence of secondary caries
around the fluoride releasing amalgam restoration. But the
disadvantages are reduction in compressive strength and
corrosion resistance.

102. 'B' [Check Explanation of Q.No.8)

103. 'D' [Essentials of operative dentistry by Anand Sherwood


1st ed 233)
First amalgam war:
In 1843 American society of dental surgeons condemned
the use of all filling material other than gold and declared
them as toxic igniting the war and pledged never to use
amalgam

Second amalgam war:


In Mid 1920s, German dentist Alfred stock started 2nd war.
He claimed that mercury could be absorbed from amalgam
and leads to serious health problems.

Third amalgam war:


Dr. Huggins raised the controversy that mercury released
would affect the CVS and CNS. He has also stated that
patients recovered from diseases like multiple sclerosis and
Alzheimer's diseases after removing their dental amalgam
fillings.

104. 'A' [Burkets 11th ed 124]


The single most common source of solitary or focal
pigmentation in the oral mucosa is the amalgam tattoo
which is a consequence of inadvertent deposition of
amalgam restorative material into the submucosal tissue.
These are found in
• the vicinity of teeth with large amalgam restoration
• Apical region of RCT treated teeth with retrograde
restoration or obt urated with silver points.
, OPERATIVE DENTISTRY

4. DIRECT FILLING GOLD


1. Pure gold fuses at the temperature (in ° C} of 12. The term stepping is associated with
a) 863 b) 1063 a) Root canal preparation b) direct gold condensation
c) 1263 d) 1463 c) Silver amalgam condensation
(MAN-95) d) fracture mandible
2. The carat for gold foil used for restorations is (PGI-97,98)
a) 18 b) 20 13. If the nib diameter is reduced by half, the compaction
c) 22 d) 24 force is
(MAN-95) a) Halved b) double
3. Stress in cold working can be relieved by: c) Increases 4 times d) constant
a) Age hardening b) Annealing (KAR-98)
c) Hydrochloric acid d) Beating 14. The cavosurface margin for direct filling gold are
(KAR-01) prepared with
4. Annealing of gold foil is the process by which a) 90° butt joint b) 45 ° Bevelled
a) Contaminants are removed from the surface c) 30-40° bevel d) 10° bevel
b) Rigidity decreases and ductility increases (KAR-98)
c) both ductility and proportional limit decreases 15. Best marginal integrity is obtained with
d) none of the above a) GIC b) Gold foil
(AP-99) c) Gold Inlay d) Amalgam
5. Which of the following restorative materials requires (AIPG-94)
more skills in manipulative manevours than others 16. Surface hardness of gold foil during condensation
a) Amalgam b) cast gold alloy a) Always increases b) Always decreases
c) pure gold d) composite c) Increased followed by a decrease
(MAN-98,94) d) Not correlated
6. The cohesion of direct gold filling at room temperature (AIPG-93)
is an example of 17. For class V direct filling gold, retention points are
a) wedging b) annealing a) In the enamel completely
c) adaptability d) atomic attraction b) In the occlusal and gingival line angles
(AIIMS-2K) c) In the mesial and distal walls
7. The purpose of heating gold foil before condensing is: d) In the axial wall completely
a) To improve welding quality (AIPG-90,AP-95)
b) To melt the gold! 18. The cohesive gold used as a bulk filler is
c) To remove adsorbed gases on gold surface a) Electra alloy b) Mat gold
d) To increase oxidation c) spherical gold d) gold foil
(AIPG-96,PGI -01) (MAN-01)
8. Alloyed form of direct gold restoration is: 19. Electralloy is an alloy of
a) Goldent b) Mat gold a) Gold and platinum b) Cobalt and chromium
c) Electralloy d) Laminated gold foil c) Gold and calcium d) tin and chromium
(KAR-03) (KAR-97)
9. Tick the odd one out 20. Pitting of direct filling gold usually results from
a) Degassing b) Desorbing a) Using contaminated gold foil
c) annealing d) Metalizing b) Poor finished preparation
(KAR-98) c) Improper stepping of condenser point
10. The amount of force needed to compact direct filling d) using small condenser point
gold properly is influenced mostly by the: (KAR-98)
a) Angle of compaction 21. The malleting force is directed
b) Surface area of the condenser a) perpendicular to the walls
c) Bulk of the surrounding tooth b) parallel to Long axis of crown
d) Temperature at which gold is annealed c) 45° to the wall d) none of the above
(AIPG-92)
11, Gold foil is prepared by 22. Which of the following is true about direct filling gold
a) Condensation b) Beating a) Properly condensed mat gold has no voids
c) Rolling d) annealing b) Whatever may be the type voids are inherent
(AP-01) c) Voids can be avoided with manual condensation

1) B 2) D 3) B 4) A 5) C 6) 0 7) C 8) C 9) 0 10) B 11) B 12) B 13) C


14) C 15) B 16) A 17) B 18) B 19) C 20) C 21) C 22) B
Dental ;lut.,e

d) None of t he above
(AP-96)
23. Gold with highest strength and can be used in stress
bearing area
a) Gold foil b) Mat gold
c) Spherical gold d) Electra lloy
(KAR-04)
24. Ammonia treated gold foil is also known as
a) Cohesive foil b) Non cohesive foil
c) corrugated foi l d) Non corrugated foil
(AIPG-07)
25. The calcium content in alloyed electrolytic precipitate
is -
a) 0.10% b) 0.20%
c) 0.50% d) 0.80%
(COMEDK-09)
26. Powdered gold is mar keted as -
a) William's E-Z gold b) Morgan's E-Z gold
c) Robert's E-Z gold d) Taggart's E-Z gold
(COMEDK-09)
27. The most recent direct filling gold which produces the
hardest surface on condensation is
a) Mat gold b) Encapsulated powered gold
c) Mat gold alloyed with calcium and wrapped in gold foil
d) Platinized gold
(AP-2013)

23) D 24) B 25) A 26) A 27) C


, OPERATIVE DENTISTRY

4. DIRECT FILLING GOLD - ANSWERS


1. ' B' [Phillips 10th ed 431] The smaller the nib area of the condenser the greater the
Properties of pure gold: force delivered. Force delivered is inversely proportional to
• Very soft with Brinnel hardness number - 25. the square of the diameter.
• High tarnish and corrosion resistance.
11. ' B' [Sturdevant 5th SA ed 420/ Vimal Sikri 1" ed 873]
• Most noble of metals. Gold foil is manufactured by beating pure gold into thin
• It is malleable and ductile. sheets. The numbering system reflects the weight and
• It is weldable at room temperature (cold-welded). thickness as well.
• No. 4 thickness - Weight 4 grains - 0.5µm thick.
2. 'D' [Phillips 10th ed 429] • No. 3 thickness - Weight 3 grains- 0.38µ m thick.

3. 'B' [Phillips 10th ed 63] 12. 'B' [Sturdevant 5th SA ed 423/ Phillips 11th ed 554)
The effects of cold working such as Strain hardening, low Stepping refers to the process of pure gold condensation
ductility and distorted grains can be reversed by simply whereby each time condenser overlaps half the previous
heating the metal. This is called Annealing. The more compaction. Compaction proceeds from centre to periphery.
severe the cold working, the more rapidly the effects can Regardless of the technique used, some "Bridging" or void
be reversed by annealing. spaces occurs.

4. 'A' [Phillips 11th ed 551] 13. ' C' [Sturdevant 4th ed 87 6]


Degassing or desorption or Annealing is the process of
application of heat t o gold to remove surface contaminants 14. 'C' [Sturdevant 5th ed 925]
and render gold cohesive for its compaction. Annealing is A very slight bevel of 30° - 40° provides ease in finishing
accomplished by pure ethanol flame or electric annealer the metal margin and removing the remaining rough enamel.

Form of direct gold Method of degassing 15. ' B' [Sturdevant 5th SA ed 419/ 5th ed 919]
Gold foil Bulk annealing on mica tray
16. 'A' [Phillips 11th ed 557]
Goldent Ethanol flame
Electra Uoy Electric annealing 17. 'B' [Sturdevant 4th ed 890/ 5th ed 930]

5. ' C' [Sturdevant 4th ed 873/ 5th ed 919] 18. ' B' [Phillips 11th ed 550]
If exact techniques in tooth preparation design and material Type Use
manipulation are fo llowed, direct filling gold restorations
last for a lifetime. The longevity of direct gold restorations Mat gold Bulk filler
is mainly due to their excellent marginal integrity and Gold foil For external surface veneering
biocom pati bility. Mat foil Veneering

6. ' D' [Phillips' 11th ed 547]


Electralloy Core and finishing
Cohesion of gold occurs by cold welding or metallic
19. 'C' [Phillips 11th ed 415]
bond between increments of gold under the pressure of
compaction.
20. ' C' (Sturdevant ~th~ 876]
7. ' C' [Sturdevant 5th SA ed 421/ 5th ed 920]
21. 'C' [Marzouk 1" ed 396]
Gases such as ammonia, nitrogen, 502 and water vapour
The forces of condensation must be at 45° to cavity walls
are adsorbed onto the surface of pure gold. These make
gold non cohesive. They must be removed to render gold and floors. This allows maximum adaptation of gold against
walls, floors, Line and point angles.
cohesive.

8. 'C' [Phillips 11th ed 550] The forces of condensation must be directed at 90° to
Electralloy is electrolytic precipitate of gold, alloyed with previously condensed gold. This avoids shear components
calcium. The calciuim content of the finished product is that displace the already condensed pieces of gold.
about 0.1%. Its purpose is to produce stronger restorations
by dispersion strengthening. 22. ' B' [Sturdevant 5th SA ed 422/ Phillips 11th ed 556)

23. 'D' [Philips 11th ed 557]


9. ' D' [Sturdevant 5th SA ed 421/ 5th ed 920]
Electralloy on condensation produces the hardest direct
10. ' B' (Sturdevant ~ 11!~ 876] filling gold surface.
Dental ;lut.,e

24. ' B' [Vimal Sikri 3rd ed 316)

25. 'A' [Philips 11th ed 551]

26. 'A' [Sturdevant 5th SA ed 420/ Philips 11th ed 551]


Goldent, a type of powdered (granular) gold, was marketed
originally by Morgan, Hastings Co. but later by William's
Gold Refining Co., Buffalo, New York.
E - Z gold means powdered gold pellets with a wax coating
that must be burned off before compaction.

27. ' C' [Vimal Sikri 3n1 ed 317/ Text Book of Operative
Dentistry by Nisha Garg 1st ed 362]
, OPERATIVE DENTISTRY

5. COMPOSITES

1. Which of the following is an example of a composite material: 12. Esthetic restorative materials are
a) A filled resin b) Colloidal silica a) Glass ionomer cement b) composite
c) gold alloy d) wax c) silicate d) all of the above
(MAN-95) (MAN-97)
2. Radio opacity of composite resins is rendered by 13. An old composite restoration is to be replaced. The best
a) Silica glass b) organic matrix method is
c) barium glass d) fluoride particles a) To remove entire restoration, reetch and refill
(MAN-02) b) To roughen the old restoration, re etch and refill
3. The main advantage of composites over unfilled direct c) composite restorations cannot be replaced or refilled
filling resin is their: d) none of the above
a) Higher solubility in saliva (MAN-98)
b) Lower modulous of elasticity 14. Which is not a property of resin ionomer restorative
c) esthetic excellence material
d) Lower thermal co-efficient of expansion a) It is a dual cure polymerization system
(KAR-97) b) It is radiolucent
4. Composite has c) It releases fluoride d) It adheres to the dentin
a) resin b) filler (MAN -01)
c) resin and filler d) none 15. Restoration of angle fracture of anterior teeth done by
(AP-01) a) Amalgam b) Composite resin
5. Bonding of composite resins to tooth structure is by: c) GIC d) All of the above
a) Covalent bond b) ionic bond (MAN-95)
c) mechanical d) vanderwaal forces 16. Increasing the etching concentration
(KAR-01) a) Increases the micropores formed
6. The chemical used to etch enamel is b) Increases the length of the enamel tags
a) Zinc oxide b) methyl methacrylate c) Causes precipitation of phosphates
c) phosphoric acid d) eugenol d) increases the bond strength
(PGI-98) (MAN-02)
7. The function of the coupling agent in a restorative resin 17. The light from the UV light curing unit can cause:
is to allow: a) Iritis b) Cataract
a) Adhesion of resin particles c) Retinal damage d) Corneal ulceration
b) Bonding between filler crystals (KAR-2K)
c) Bonding between filler and resin 18. The etching of intact enamel wit h phosphoric acid produces:
d) Bonding between tooth and resin a) an increases in surface area
(MAN-94,AIPG-94) b) a decrease in surface area
8. The particle size in microfilled composites is c) An increase in chemical bonding capabilities
a) 0.02-0.04mm b) 0.5-1.0 d) a decrease in chemical bonding capabilities
c) 0.04-0.4 microns d) 0.3-0.4mm (KAR-98)
(AP-03) 19. In class III composite preparation, retention points
9. Dentin bonding agents usually contain should be placed:
a) Only hydrophobic component a) In the axial wall b) Entirely in dentine
b) Only hydrophilic component c) At the dentinoenamel junction
c) Hydrophobic and hydrophilic component d) At the expense of facial and Lingual wall
d) Lyophilic and Lyophobic component (AIIMS-94)
(COMEDK-04) 20. Which of the following should be used to clean the
10. The advantage of using visible light for light cure resins is operating site before giving composite restorations?
a) It is easily processed by any light source a) Pumice slurry
b) Machine is easily available b) Prophy paste containing glycerine
c) It can cure the resin through enamel c) prophy paste containing fluoride
d) The finish is better d) all of the above
(MAN-97) (KAR-01)
11, As compared to UV light, visible light can cure 21. The coupling agent used in composite resin
a) Greater depth of resin b) Lesser depth of resin a) Colloidal silica b) Urethane dimethacrylate
c) Only superficial Layer d) None of the above c) Organo silanes d) Diketones
(MAN-97) (KAR-99)

1) A 2) C 3) D 4) C 5) C 6) C 7) C 8) C 9) C 10) C 11) A 12) D 13) B


14) B 15) B 16) C 17) C 18) A 19) B 20) A 21) C
Dental ;lut.,e

22. Class II composites are inferior to amalgam due to: 32. Advantages of light activated composite resins are:
a) Less abrasion resistance b) Lower compressive strength a) Extended working time
c) Lower tensile strength d) Greater solubility b) Reduced resistance to wear or abrasion
(AIPG-2K) c) Better resistance to wear or abrasion
23. What is the modern approach treatment of a discoloured d) All of the above
crown? (KAR-97)
a) Composite restoration b) Full crown 33. Shade determination of composite resin restoration on
c) Bleaching d) Implant a tooth should be done;
(AIPG-95) a) Before placement of rubber dam
24. Which of the following is not an appropriate material b) On a dry tooth
for composite hand instruments? c) Under bright light d) Using a dry shade guide
a) Stainless steel b) Teflon (AIPG-95)
c) Teflon coated metal d) Nonmetallic 34. The penetration of the fissure sealant into the fissure:
(KAR-01) a) Is not influenced by the wettability of the enamel
25. Improved marginal adaptation in a composite restoration b) Is independent of the chemical composition of the
can be acMeved by: enamel
a) Moisture free environment c) Is the result of a chemical !bond between the sealant
b) Application of pressure and enamel
c) Acid etching d) Continued packing d) Must occur before the polymerization of the material
(AIIMS-99) (AIIMS-01)
26. Acid etching of enamel margins 35. In a deep cavity if a composite restoration is to be
a) Provides chemical retention placed, there is a need for:
b) Provides mechanical retention a) Calcium hydroxide base b) Cavity varnish
c) Is used as resistalilce form c) Resin base d) No need for any of the above
d) Is used for pulp protection (AIIMS-2K)
(KAR-99) 36. The microwave method of processing resin is better than
27. Core built up on a molar tooth should preferably be the conventional in that it provides:
done with a chemically cured composite as: a) Better shine b) Less porosity
a) Light cure resin lacks sufficient strength for a core c) Less shrinkage d) Color stability
b) Core made with a light cure resin would be too opaque (AIIMS-99)
and esthetically unacceptable 3 7. Filled sealants are better than unfilled due to:
c) core made with a light cure resin would be too brittle a) High viscosity b) High strength
and would fracture easily c) Cotton application d) Low abrasion
d) Its lower viscosity allows free flow of resin also around (AIPG-2K)
pins and post 38. Polishing of composite is problematic due to_:
(AIIMS-01) a) Soft matrix and hard filler particles
28. Sensitivity in gingival area of class II light cured b) Hard filler particles
composite restoration after 2 months is due to: c) Hardness of matrix and filler particles
a) Improper cavity preparation at gingival margin d) None of the above
b) Curing from occlu.sal area only (AIIMS-97)
c) Addition of resin in multiple increments 39. Micro abrasion involves the use of:
d) Curing from buccal and lingual aspect a) Sulphuric acid b) Acetic acid
(AIIMS-95) c) Phosphoric acid d) Hydrochloric acid
29. All of the following are true for light cure composite except: (AIPG-04)
a) Held at distance of 1mm from tooth 40. Inorganic phase of the composite aids in
b) Use of orange glass shield for eye protection a) Increasing mechanical strength
c) curing cycle of 40-60 seconds b) Decreasing the coefficient of thermal expansion
d) Adding increments of 1-2 mm c) reducing the polymerization shrinkage
(AIIMS-95) d) all of the above
30. Increased size of filler particle in composite: (AP-98)
a) Increases strength b) Decreases surface roughness 41. The major constituent in composite is
c) Increases exposure time d) decreases viscosity a) BIS-GMA and UDMA b) Glycol and Glycidine
(AIIMS-96) c) GIC and Silicate d) Zinc oxide
31. Cross-linking in resins helps increasing: (AP-03)
a) Strength b) Hardness 42. True about self etched primer is
c) Crazing resistance d) All of the above a) Remove smear layer
(AIIMS-95) b) Does not remove smear Layer but binds it to dentin
c) Increase dentin permeability

22) A 23) C 24) A 25) C 26) B 27) D 28) A 29) C 30) A 31) D 32) A 33) A 34) D
35) A 36) B 37) D 38) A 39) D 40) D 41) A 42) B
, OPERATIVE DENTISTRY

d) Increase sensitivity 54. Photopolymerization stress build-up can be reduced by


(PGI-2011) a) High density curing b) Soft-start polymerization
43. What is the wavelength of visible light used for curing c) Low density curing d) Short exposure time
light cure resin restorations? (AIPG-07)
a) 400-420nm b) 420-475nm 55. All of the following are disadvantages of composite
c) 300-370nm d) 475-500nm restoration except:
(AIPG-95, 05) a) Greater Localized occlusal wear
44. Final finishing of composite resin is usually done b) Recurrent secondary caries
a) At the same time when restoration is place c) Technique sensitive d) More time in replacement
b) 24 hours after restoration is placed (AIIMS-06)
c) 48 hours after restoration is placed 56. All of the following are true about polymerization
d) after 1 week of placement shrinkage of composite resin except:
(KAR-98) a) The more surface area of bonded structure causes
45. Which of the following properties is the most trouble some LESS shrinkage compare to surface area of unbounded
for composite materials used for posterior restorations? structure restoration.
a) Wear resistance b) Water sorption b) Class-V composite restoration causes. Contraction more
c) Coefficient of thermal expansion t han gold restoration .
d) Compressive strength c) Etching and priming reduces shrinkages.
(TNPSC-99) d) One third (1/3rd) of the enamel involves causes more
46. Marginal leakage due to temperature changes commonly shrinkage.
seen in (AIIMS-06)
a) Amalgam alloy b) Unfilled resin 57. Self etching primers
c) filled resin d) Gold alloy a) Simultaneously etch and prime dentin and enamel
(AP-04,05; AIPG-89) b) Etches enamel only
47. Shade selection is done with in how much time to avoid c) Only bonds to enamel d) Only removes smear layer
fatigue of eyes? (KCET-07)
a) 5 seconds b) 10 seconds 58. Retention in a composite restoration is mainly attained by
c) 15 seconds d) 20 seconds a) Chemical adhesion b) Acid etching & Bonding
(AIPG-2012, 2014) c) Inverted Truncated shape d) Undercuts
48. Which lasers can be used to cure composite resins? (KCET-08)
a) Argon b) Nd YAG 59. Discoloration and accelerated wear is seen in composite
c) Er YAG d) CO 2 restoration if a patient is using a tooth paste having
(AP-05) a) Stannous flouride b) Calcium carbonate
49. Microleakage is more in c) Chloroform d) Glycerin
a) Direct gold filling b) Composite (KAR-02)
c) Amalgam d) GIC 60. Whenever tooth structure is cut with bur residual
(AP-06) organic and inorganic components form the
50. About acid etching of dentin all are true, except: a) beilby layer b) hybrid layer
a) It removes smear layer. c) smear layer d) rubbish Layer
b) It expose collagen. (AP-2013)
c) It opens dentinal tubules. 61. What is the approximate stress caused by polymerisation
d) It increase surface energy. shrinkage of composite?
(AIIMS-06) a) 1 Mpa b) 5 Mpa
51. Ormocers are: c) 15 Mpa d) 30 Mpa
a) Organically modified ceramics (AIPG-09)
b) Inorganically modified ceramics 62. A Dentin ' Primer'?
c) Organically modified Glass Inomer Cement a) Etches the dentin
d) Inorganically modified Glass Inomer Cement b) It increases the surface energy & wets the dentin surface
(Al-05) c) Removes the smear layer
52. Nanofillers are in the range of: d) Conditions the dentin
a) 10 - 100 microns b) 0.1 - 1 microns (AIPG-09)
c) 0.01 - 0.1 microns d) 0.001 - 0.01 microns 63. Composites are not recommended for direct posterior
(AI-05) restorations in all the following except:
53. The poly-acid modified composite are known as a) Heavy occlusal stresses
a) Compomers b) Hybrid Ionomers b) Deep subgingival areas
c) Poly Carboxylates d) Poly acrylates c) Interim restorations d) Isolation is not possible
(AIPG-0/, COMEDK-15) (UPSC-09)

43) B 44) A 45) A 46) B 47) A 48) A 49) B 50) D 51) A 52) D 53) A 54) B 55) D
56) A 57) A 58) B 59) C 60) C 61) B 62) B 63) C
Dental ;lut.,e

64. Which of the following is most stable restoration for a d) Rinse away the saliva with water, dry the preparation
G.V. Black class V unstable erosive lesion? with air, then repeats the etching procedure.
a) Acid etch composite b) GIC (AIPG-2009, 11)
c) Resin modified GIC d) Compo mer 7 4. The value of (-factor for class IV composite restoration is
(AIPG-09, AIIMS MAY- 2012) a) 0.25 b) 0.5
65. Which of the following is material of choice in class V c) 1.0 d) 2.0
cavity with abfraction? (KCET-2012)
a) GIC b) Hybrid composite 75. For indirect tooth-colored res to rations the re com mended
c) Microfilled composite d) Compomer gingival-occlusal divergence per wall is in the range of
(AIPG-09) a) Greater than 2-5 degrees
66. When the concentration of the etchant is more than 50%, b) 2-5 degrees
it forms a complex which prevents further dissolution. c) Less than 2-5 degrees d) Less t han 5 degrees
The complex is (KCET-2012)
a) hydroxyapatite 76. The first generation and second generation flowable
b) calcium phosphate composites are intended to be used for
c) monocalcium phosphate monohydrate a) Class V and Class II restorations, respectively
d) none of the above b) Pit-and-fissure sealants and small anterior restorations,
(AP-09) respectively
67. To prevent sensitivity caused by acid etching and to c) Class I and class II restorations, respectively
protect pulp in deep cavities which of the following d) Pit-and-fissure sealants and Class I restorations,
should be used. respectively
a) Ca(OH) 2 liner b) ZOE (KCET-2012)
c) Light cured GIC liner d) Varnish 77. The components of fourth generation dentin bonding
(AIIMS-08) systems are described as
68. Slow or soft start polymerization is seen in a) Self etching adhesive
a) L.E.D. ligh cure units b) Self etching primer/adhesive
b) Laser light cure units c) Total etch, multi bottle
c) Quartz-Tungsten-Halogen light cure units d) Total-etch, single bottle
d) PAC light cure units ( KCET-2012)
(KCET-10) 78. As compared to total etch adhesives, self etching primers are
69. BIS-GMA in composites a) More technique sensitive
a) Replaced st rontium fille r particles b) Less technique sensitive
b) Replaced acid soluble glass particles c) Same in sensitivity d) None of the above
c) Replaced silane coupling agents (BHU-2012)
d) Replaced methacrylate based matrix 79. While halo or line around enamel margin in composite
(KCET-10) restoration is not due to
70. Acid dissolution is most common in which part of rod a) Low intensity light curing
a) Periphery of head b) Head region b) High intensity light curing
c) Rod tails d) equally c) Inadequate etching and bonding
(AIIMS-09) d) Traumatic contouring
71. Percentage ofphosphoric acid where Dicalcium phosphate (BHU-2012)
monohydrate is formed that cannot be rinsed off. 80. The purpose of acid etching prior to composite resin
a) 25% b) 37% restoration is
c) 50% d) 85% a) To soften the enamel
(AIIMS-09) b) To produce surface irregularities
72. Which lasers can be used to cure composite resins? c) To produce a clear enamel surface
a) Argon b) NdYAG d) To remove the smear layer
c) ErYAG d) CO2 (AP-2012)
(COMEDK-2011) 81. While working with dental material the factor C is
73. After the dentist has completed an etching procedure associated with
on a class III composite preparation, the preparation a) Condensation force
gets contaminated with saliva. In response, the dentist b) Tensile strength
should do which of the following? c) Polymerization shrinkage
a) Blow away the saliva with air, and then proceed. d) Carving of amalgam
b) Rinse away the saliva with water, dry the preparation, (AIPG-2012)
then proceed.
c) Wipe away the saliva with cotton pellet, rinse the
preparation with water, dry it with air, and then proceed.

64) C 65) C 66) C 67) A 68) C 69) D 70) B 71) A 72) A 73) D 74) A 75) A 76) D
77) C 78) B 79) A 80) B&D 81) C
, OPERATIVE DENTISTRY

82. A young female patient reports to the dental office with 89. Type I enamel etching pattern
complaint of small gaps between her teeth a) Involves the dissolution of prism peripheries
(COMEDK -2013) b) Involves the dissolution of prism cores
82A. The most feasible and economic treatment option for c) Not related with enamel prism
such a situation would be d) Involves dentin and enamel
a) Porcelain veneers b) Porcelain jacket crown (AP-14)
c) Orthodontic tooth movement 90. The principal resin constituent of polishable composite
d) Diastema closure using composite resin is
a) Poly methyl methacrylate
828. Shade selection for such a treatment should be carried b) Polycarbonates
out under the c) Uret hane dimethacrylate d) Cyanoacrylates
a) Natural day light b) Fluorescent light (AP-14)
c) After placement of rubber dam 91. Glass reinforced composite is used in all of the following
d) When the teeth are dry except?
a) Obturation of young permanent teeth
82C. If a proper placement technique is not followed, it b) Splinting of teeth
would result in c) Space maintainer
a) Tooth sensitivity b) Wear of opposing teeth d) Used in intra-post cementation of endodontically
c) Debonded restoration d) Healthy gingiva. treated tooth
(AIPG-14)
83. After etching you see hand piece oil contamination than 92. (-factor is associated with?
you should do: a) Condensing force for amalgam
a) Re etching for 10 sec & than continue b) Cavity preparation factor for amalgam
b) continue with bonding c) Root curvature
c) dry with air syringe and proceed with bonding agent d) Polymerisation shrinkage in composite
d) Rinse with water and proceed with bonding agent (AIIMS MAY-14)
(NEET-2013) 93. Which of the following contains primer and bonding
84. For photo initiated resin system, the optimum depth of agent in a single bottle and is known as "single
cure is obtained with emitting orifice held component system"
a) within 4 mm of resin surface a) 3rd generation b) 4th generation
b) slightly touching resin surface c) 5th generation d) 6th generation
c) within 1 mm of resin surface (PGI DEC-13)
d) within 3 mm of resin surface 94. Self-etching adhesives?
(AP-2013) a) Incorporates the smear layer into the surface
85. For a bonding agent to be effective wetting angle should b) Removes the smear Layer
be c) Fixes the smear Layer
a) Minimum with dental hard tissue d) Makes impregnation in the smear layer by acidic
b) Minimum with dentin and Maximum with enamel molecules but does not remove it.
c) Maximum with dentin and Minimum with enamel (PGI DEC-11)
d) Maximum with dental hard tissue 95. All of the following help in reducing polymerization
(AIMS NOV-13) shrinkage EXCEPT
86. Diagnodent uses what for detecting caries a) RMGIC liner b) Flowable composites
a) Sound wave b) Visible light c) Anatomic layering technique
c) LASER d) Nanotechnology d) Use of co-polymers like TEGDMA
(AIMS NOV-13) (KERALA-2015)
87. According to scientists recent studies says about pattern 96. What is the thickness of bonding agent when applied
of enamel prism demineralization, true is with brush to condition the dentin which minimises or
a) Head and tail both are resistant to dissolution prevents air incorporation?
b) Head and tail both are not resistant to dissolution a) 50 microns b) 100 microns
c) Head is more resistant to dissolution c) 150 microns d) 200 microns
d) Tail is more resistant to dissolution (PGI DEC-2013)
(AIMS NOV-13) 97. Total-etch concept was put forth by
88. Bonding agents have hydrophilic and hydrophobic a) Nakabayhashi b) Buonocore
groups, hydrophobic group binds which structure c) Bowen d) Fusayama
a) Calcium in enamel (KERALA-2015)
b) Hydroxyapatite group in hard tissue
c) Resin in restoration d) Collagen in dentin
(AIMS NOV-13)

82A) D 828) A 82() C 83) A 84) C 85) A 86) C 87) D 88) C 89) B 90) C 91) A 92) D
93) C 94) D>A 95) D 96) A 97) D
Dental ;lut.,e

5. COMPOSITES - ANSWERS

1. 'A' [Phillips 11th ed 401] 8. 'C' [Phillips 11'" ed 423]


Composite is a combination of two or more materials having Composite Filler Particle Size (11m)
chemically distinct interface between them . Eg . Filled resin.
Conventional composite 8 - 12
2. ' C' [Phillips 11th ed 406] Small particle filled 1- 5
Composite resins contain Microfilled composite 0.04 - 0.4
Resin Matrix • BIS-GMA, urethane dimethacrylate. Hybrid composite 0.6 - 1
• TEGOMA (Viscosity controller)
9. 'C' [Phillips 11th ed 387]
Fillers • Colloidal silica or quartz.
Dentin is hydrophilic and composite resin is hydrophobic.
• Glasses of barium, zirconium give So a dentin bonding agent should contain both hydrophilic
radiopacity. and hydrophobic components.
Coupling • Organosilanes, zirconates and titanates.
agent 10. 'C' (Sturdevant 4th ed 197]
• The function of coupling agent is to
allow bonding between filler and resin. Visible light cure unit:
• Health hazard is virtually eliminated.
3. ' D' [Phillips 11th ed 401] • No warm up time is required.
Advantages of composites over unfilled resins: • Requires a minimum of 20 secs. for adequate curing.
• Lower coefficient of thermal expansion. • The tip should be wit hin 2mm of the composite.
• Low polymerization shrinkage. • It can cure to a depth of 2mm and if Large area is to be
• Low water absorption. cured, it is carried out in increments.
• High abrasive resistance. • Camphoroquinone is the photo initiator, it absorbs
photons of light energy, predominantly at 474nm (blue
Disadvantages of composites: region).
• Less colour stability. • Oiketone is the activator.
• Less smooth finish than unfilled resins. • For darker shades, curing time should be increased.
• Light curing can be accomplished with Quartz-Tungsten-
4. 'C' (Phillips 11th ed 401] Halogen curing units or Light emitting diode (LEO)
curing units.
5. ' C' [Phillips 11'h ed 383]
Bonding of composites to tooth structure occurs 11. 'A' (Sturdevant 4th ed 197]
by micromechanical retention. Acid etching creates UV light activated system:
microporosites into which the resin penetrates resulting in
• Curing occurs due to activation of Benzoin methyl ether
resin tag formation. These tags penetrate to a depth of
5 - 10 µm (PGI June-13) but their lengths are dependent when exposed to UV light.
on the enamel etching time. • Potential health hazard to clinician and patient by UV
light.
6. ' C' (Sturdevant 5th SA ed 182/ Phillips 11thed 385] • Retinal and soft tissue damages.
37% phosphoric acid is used as an etchant. Concentration • Intensity of light source gradually decreases in strength
above 50% results in formation of monocalcium phosphate with use.
mono hydrates which prevents further dissolution. The
etchant is supplied in a gel form to allow control over the area • Require more time to cure - 60 secs.
of placement. Concentration below 30% results in dicalcium • It can cure to a thickness of 1.5mm .
phosphate dehydrate that cannot be easily removed.
12. ' D' [Marzouk 1" ed 169/Sturdevant 5th ed 498 ]
7. ' C' [Phillips 11th ed 406] Tooth coloured restorations are divided into direct and
The function of coupling agent is to provide a bond between indirect restorations.
filler particles and resin matrix. Organosilanes are most
commonly used coupling agents. The methacrylate groups Direct:
of oragnosilanes bonds with the resins while the silanol • Silicate cement
group of organosilanes bonds with filler. • Unfilled resin
Coupling agents improves physical and mechanical • Filled resin
properties and provide hydrolytic stability by preventing • Composite resin
penetration of water .along the filler-resin interface. • Composite resin glass ionomer cement
, OPERATIVE DENTISTRY

Indirect: 21. 'C' [Sturdevant 5th SA ed 226 / 5th ed 197]


• Porcelain
22. 'K [Sturdevant 5th SA ed 236/ 5th ed 508]
• Cast moldable ceramic
Disadvantages of composite restorations:
• Porcelain fused to metal restorations
• Gap formed, especially on root surfaces due to
polymerization shrinkage.
13. 'B' [Phillips 11th ed 437]
• More technique sensitive.
14. 'B' [Sturdevant 5th SA ed 235/ 5th ed 503] • Exhibit greater wear (less abrasion resistance).
Hybrid or resin ionomer is dual cure polymerization system. • High coefficient of thermal expansion resulting in
Polymerization is initiat ed with exposure of light. After microleakage if inadequate bonding technique is used.
that the liquid reacts with glass particles through acid base
• Difficult, time consuming and costly procedure.
reaction. It adheres to dentin and releases fluoride.
23. 'C' [Sturdevant 5th SA ed 311/ s•h ed 641]
15. 'B' [Sturdevant 5th SA ed 241/ 5th ed 529]
Indications of com po sites: Non-Vital bleaching procedures
• Classes I, II, III, IV, V and VI restorations. It is placement of 35% H20 2 liquid into
Thermo-
• Sealants and preventive resin restorations. debrided pulp chamber and the oxidation
catalytic
process is accelerated by placing a heated
• Foundations or core build ups. technique
instrument into the pulp chamber.
• Leuting agent
The reaction between 35% H20 2 and sodium
• Temporary restorations perborate accele rates the oxidation process.
Walking
• Esthetic procedures It is an out of the office technique. The
bleach
- Partial veneers oxidation is continued over a longer period
of time.
- Tooth contour modifications
- Full veneers Vital bleaching procedures
- Diastema closures A solution containing ether, H2 02, HCl is
used.
• Periodontal splinting In-office • Ether - for removal of surface debris.
technique
16. 'C' [Vimal Sikri 3n1 ed 338] • HCl - etches the enamel.
• H20 2- bleaches the enamel.
17. 'C' [Phillips 11th ed 298]
It is dentist prescribed home applied
18. 'A' [Sikri t•ted 351 / Marzouk t •ted 181] Night guard technique. It consists of using a whitening
bleaching paste containing 10 - 15% carbamide
Acid etching affects the enamel in the following ways:
peroxide.
• The surface area of the enamel will increase upto 2000
times that of its original unetched surface. 24. 'A' [Vimal Sikri 3,d ed 382]
• Removes pellicle to expose the inorganic crystalline Usually Teflon coated instruments are used to pick and
component. insert the composite. Stainless steel instrument should not
• Creates a porous layer, the depth of the pores range be used because the composite will stick to the instrument.
from 5 - 10 µm.
25. 'C' [Phillips 111• ed 383]
• Raises the surface area and increases the wettability.
Acid etching provides a reliable and durable method of
• Raises the surface energy. adhesion between resins and tooth structure. Acid etching
is done with 37% phosphoric acid for 15 secs, in permanent
19. 'B' [Sturdevant 4th ed 507/ 5th ed 533] teeth. For enamel only preparations, 30 secs. is considered
The retention grooves or coves in class III composite normal. In fluoridated and primary teeth longer etching time
restorations are placed along the gingivo-axial line angle is required due to the presence of more aprismatic enamel.
and sometimes in the incisoaxial line angle 0.5mm into
dentin. They are never placed on the facio axial and lingua 26. 'B' [Phillips 11th ed 383]
axial line angles.
27. ' D' [Sturdevant 5th ed 598]
20. 'A' [Sturdevant 5th SA ed 236]
Before insertion of composite restorations, the operating 28. 'A'
site is cleaned with pumice slurry to remove plaque, pellicle
and superficial stains. Calculus removal with appropriate 29 • 'C' [Sturdevant 41h ed 201]
instruments is also needed. Prophypastes containing
fluoride, glycerine and flavoring agents should be avoided
to prevent conflict with the acid etch technique.
Dental ;lut.,e

30. 'A' [Vimal Sikri 3rd ed 364] 40. 'D' [Vimal Sikri 3"' ed 364]
Increased filler particle size increases the strength,
viscosity, hardness and surface roughness but decreases the 41. 'A' [Phillips 11th ed 402]
water sorption and polymerization shrinkage.
42. 'B' [Sturdevant 5th SA ed 192/ Sturdevant 5th ed 258]
31. 'D' [Phillips 11th ed 744] Self etch primer does not remove smear layer instead they
Cross linking of resin.s help in increasing the resistance to modify it and get.s it embedded into the hybrid layer.
crazing. It also increases the tensile strength and hardness
to some extent. 43. 'B' [Phillips 10th ed 280/ Sturdevant 4th ed 198]
The photo initiator in light cure systems is camphoroquinone.
3 2. 'A' [Phillips 10th ed 282] It has an absorption range between 400 - 500 nm that
The advantages of light cure over self cure are the extended is in the blue light region of visible light spectrum and
working time and Less sensitivity to oxygen inhibition than predominantly at 474 nm.
self cured system.
Wave length at which camphoroquinone activity is
33. 'A' [Sturdevant 5th SA ed 239/ 5th ed 508] maximum, in visible light cure system is? (PGI -2011)
Guidelines for selection of proper shade: a) 402 nm b) 474 nm
• Use natural Light if possible. c) 484 nm d) 464 nm
• Select shade before applying rubber dam.
44. 'A' [Vimal Sikri 1•t ed 532]
• Do not dry the tooth, as dry teeth become Lighter in
shade as a result of a decrease in translucency. 45. 'A' [Sturdevant 5th SA ed 236/ 5th ed 509]
• If natural light is not available, shade selection is done
under colour corrected operating lights. 46. 'B' [Sturdevant 4thed 255]
• Shade tab should be held beside the area to be restored Unfilled resins have high a, coefficient of thermal expansion
and it should be partially covered by patient's lip. (81 x 10·6 / °C). So they are very sensitive to thermal
changes resulting in micro Leakage.
34. 'D' [Sturdevant 4th ed 188]
47. 'A' [The Journal Of Prosthetic Dentistry August 1987 Volume
35. 'A' [Sturdevant 5th SA ed 247/ 5th ed 516] 58 Number 2 ed 135/Shillingberg 3rd ed 427]
Rapid shade comparisons for no more than 5 seconds to
36. 'B' avoid fatigue of cones in the retina. Prolonged viewing
strains the cones and results in perception of decreased
3 7. 'D' (Check Explanation Below] chroma and value of the sample. First impressions are more
accurate.
Types of Pit and Fissure sealants
1" generation UV Light activated sealant.s. 48. 'A' [Phillips 11th ed 412/ Vimal Sikri 3rded 372]
2"d generation Chemical curing resins. Different curing lights of composite resin:
• Halogen bulb combined with filter
3"' generation Visible Light activated sealants.
• Blue light emitting diode (LED)
4th generation Fluoride containing sealants.
• Laser curing
Filled sealants will have more resistance to wear and Less
flow than unfilled sealants. Argon laser with intensity of 250mW ± 50mW for 10 seconds
per increment is the commonly used Laser.
38. 'A' [Vi mal Si kri 3rd e d 483]
Traditional composites develop a ro ugh surface during 49. 'B' [Sturdevant 5th SA ed 234/ Sturdevant 5th ed 503]
polishing due to wear of the soft resin matrix that leaves Linear coefficient of thermal expansion (LCTE)
the more wear resistant filler particles elevated. Polishing
Enamel 1
is done with aluminium oxide or silicon dioxide.
Silicate 0.996
39. 'O' (Sturdevant 5th SA ed 320/ 5th ed 646] Amalgam 2.29
Microabrasion involves the surface dissolution of the Gold 1.23
enamel by the acid followed by polishing with pumice to
remove superficial stains or defects. 18% HCL along with Unfilled resin 7.11
pumice in the form of paste is commonly used. Porcelain 0.58
Composite 3.16
Macroabrasion is the removal of localized superficial spots,
stains by using a 12-fluted composite finishing bur or a fine As the LCTE of composites is 3 times that of tooth
grit finishing diamond bur. structure there is more chance of opening of margins when
, OPERATIVE DENTISTRY

temperature changes occurs. By bonding composites to the 55. ' D' [Sturdevant 5th SA ed 23 6/Sturdevant 5th ed 507,
etched surface reduces the potential of microleakage. 522]
Advantages of composites:
50. ' D' [Vimal Sikri 3,d ed 341] • Esthetic
Acid etching or conditioning of dentin causes removal of
• Conservative
smear layer, demineralizes superficial dentin of 3 - 7µms,
exposes, microporous collagen Scafford into which the resin • Insulating, low thermal conductivity
will impregnate. • Repairable

37% phosphoric acid, 10% citric acid, malaeic acid are the Disadvantages of composites:
different acid condit ioners. • Gap formation due to polymerization shrinkage, microleakage.
• Recurrent caries
51. 'A' [Vimal Sikri 3,d ed 368]
• Time consuming
Ormocers Organically modified ceramics
• Technique sensitive
Ceromer Ceramic optimized composites
• Exhibit greater occlusal wear
Giomers Pre-reacted glassionomer with composites
• High LCTE, resulting in microleakage
Compo mer Poly acid modified composites
56. 'A' [Sturdevant 5th SA ed 230/ Sturdevant s •h ed 504]
52. 'D' [Sturdevant 5th SA ed 227 / Sturdevant 5th ed 201]
Composites shrink while hardening. This is called as
Macro fillers 10 - lOOµm polymerization shrinkage. This leads to opening of a 'V'
Midifillers 1 - lOµm Shaped gap if the polymerization fo rces are greater than
the bond strength to enamel and dentin.
Minifillers 0.1 - lµm
Microfillers 0.01 - O.lµm . . Bonded surfaces
Configuration factor ((-factor) = f
Unbonded sur aces
Nanofillers 0.001 - O.Olµm
The higher the (-factor, the greater is the potential for
53. 'A' [Sturdevant 5th SA ed 414/ Vimal Sikri 3,d ed 410] bond disruption from polymerization effects.
Poly acid modified composites are called as COMPOMERS. Resin
modified Glassionomers are called as HYBRID IONOMERS. Preparation ( -factor
Class I (High risk) 5/1 = 5
54. ' B' [Sturdevant 5th SA ed 230/ Sturdevant 5th ed 504]
Class II 2/1 = 2
Photopolymerization stress build up can be reduced by the
following types of curing. Class IV (Low risk) 1/4 = 0.25
a) Soft-start technique: 57. 'A' [Sturdevant 5th SA ed 195/ Vimal Sikri 3,d ed 353]
Curing begins with a low intensity and finishes with a high
intensity. This technique allows a slow initial polymerization 58. 'B' [Sturdevant 5th SA ed 169/ Sturdevant 5th ed 512]
and high stress relaxation during early stages and ends at
the maximum intensity. 59. ' ('

b) Ramped-up curing:
Intensity is gradually increased or "ramped up" during the 60. 'C' [Sturdevant 5th SA ed 184/ Sturdevant 5th ed 17 6]
exposure in stepwise mode. Hybrid layer:
It is resin-dentin inter-diffusion zone in process of
c) Delayed curing: adhesion.
Restoration is initially incompletely cured at low intensity
and after sculpting and contouring the resin second Beilby layer:
exposure for the final cure is applied. This delay allows Disorganized molecular surface layer of a highly polished
stress relaxation to take place. metal produced by a series of abrasives of decreasing
d) Incremental built-up coarseness.

61. 'B' [Sturdevant 4th ed 243]


Which among the following will not reduce
polymerisation shrinkage? (APPG-15)
62. 'B' [Sturdevant 4th ed 241 Table 5-1]
a) A stress breaking liner
b) High intensity light curing FUNCTIONS OF:
c) Incremental addition of composite a) Acid:
d) Soft start polymerisation
• Removes smear layer
• Exposes intertubular and peritubular dentin
• Opens tubules. 71. 'A' [Sturdevant 5th ed 248)
• Decreases surface-free energy
72. 'A' [Phillips 11th ed 412)
b) Primer I Adhesion promoter: Argon laser lamps have the highest intensity of all types
• Envelops external surface of exposed collagen of lamps used for curing for composites. They emit light of
wavelength 490nm.
• Includes bifunctional molecules
• Re-establishes surface free energy level 73, 'D' [Phillips 11th ed 385]
Whenever saliva contaminates the already etched enamel and
c) Adhesive/Fluid resin/ Bonding agent:
dentin, it should be rinsed with water, dry it with air (in case
• Includes monomers mostly hydrophobic such as of enamel) or blot dry (in case of dentin), t hen again repeat
BIG-GMA, also contain small amount of hydrophilic the etching procedure. The reason is salivary proteins decrease
monomers such as HEMA the bond strength of composite to enamel and dentin.
• Copolymerises with primer molecules
• Forms hybrid layer by penetrating into interfibrillar 7 4. 'A' [Sturdevant 5th SA ed 230/ Sturdevant 5th ed 504]
spaces In class IV composite,
• Forms resin tags by penetrating into tubules 1 bonded surface 1
(factor= - - - - - - - - = 0.25
4 unbonded surface 4
63. 'C' [Sturdevant 5th SA ed 236/ Sturdevant 5th ed 529)
Contraindications for composite restorations: 75. 'A' [Sturdevant 5th SA ed 284/ Sturdevant 5th ed 612)
• Isolation is not possible. Any preparation (Inlay or onlay) for indirect tooth colored
• If all the occlusal forces will be on restorative material.
(composite, ceramic) should have the fo llowing requirements.
• Restorations that extends on t he root surface. • Adequate thickness of restorative material.
• Heavy occlusal stresses. • A passive insertion pattern.
• A 90-degree cavosurface angle to ensure marginal
64. 'C' [Sturdevant 4th ed 478) strengt h of the restoration.
• All internal and external line and point angles should be
65. 'C' [Sturdevant 5th SA ed 228) rounded.
The low modulus of elasticity allows microfill composite
• The optimal gingival-occlusal divergence should be >2
restorations to flex during tooth flexure, thus better
to 5 degree per wall. (Note: For cast metal it is 2-5
protecting the bonding interface. This makes them the choice
degree) . This is because tooth colored restorations are
for restoring class V cervical lesions where cervical flexure can
adhesively bonded and very little pressure should be
be significant (Eg: bruxism, clenchers, stressful occlusion)
applied during tryin and cementation.
66. 'C' [Sturdevant 5th ed 248) • Isthmus should be at least 2 mm wide to decrease
fracture of restoration.
67. 'A' [Sturdevant 5th SA ed 411/ Sturdevant 5th ed 311)
76. ' D' [Sturdevant 5th SA ed 229)
Medicament / Liner / Sealer First generation: Flowable composite have low filler levels
Shallow Moderate Deep and should be used as pit and fissure sealant and as a liner
excavation excavation excavation under class I and class II composite restoration.
(RDT>2mm) (ROT 0.5-2mm) (RDT<O.Smm)
Amalgam NO/Base/ CH/Base/ Second generation: (Recent version) have higher filler
NO/NO/
Sealer Sealer Sealer levels and improved properties. These can be used to restore
small (minimally invasive) class I and class II restorations.
Composite NO/NO/ DBS NO/Base/ DBS CH/NO/ DBS
Inlay NO/NO/ NO/Base/ CH/Base/ 77. 'C' [Sturdevant 5th SA ed 189/ Sturdevant 5th ed 254)
Cement Cement Cement
a) 4th generation - total etch, multibottle (3 bottles) i.e.
Sealer = Gluma (Glutaraldehyde + HEMA) etch primer+ bonding agent
DBS = Dentin bonding system
Cement = Luting cement (Eg: Resin modified GIC) b) 5th generation - tota etch, single bottle etch+ primer
CH - Ca(OH)2 and bonding agent in one bottle
c) 6th generation -
68. 'C' • Self etch primers i.e. Etcl, and primer in one bottle
+ Adhesive agent in other bottle
69. 'D' [Sturdevant 4th ed 191)
• Self etch adhesives i.e. Etch, primer and adhesive
70. 'B' [Sturdevant 5th e d 20) all in one bottle.

78. 'B' [Sturdevant 5th SA ed 193/ Sturdevant 5th ed 258)


, OPERATIVE DENTISTRY

Advantages of self etch primers over total etch adhesives 83. 'A' [Refer a.No. 73)
• Less technique sensitive
84. 'C' [Sturdevant 5th SA ed 233/ Sturdevant 5th ed 208)
• Less postoperative sensitivity Ideally the tip should be within 1-2 mm of the composite
• User friendly as t here is no etching and rinsing steps. to be effective.

Disadvantage: 85. 'A' [Philips 11th ed 38, 387)


Bonding agent should completely wet the dental hard
• Bond strength to enamel is less.
tissues for producing a good bond. So the contact angle
should be less and approach zero during bonding.
79. 'A' [Sturdevant 5th SA ed 238/ Sturdevant 5th ed 522)
Causes of white halo or Line around the enamel are:
Ideally dentin adhesives should contain hydrophilic groups
• High intensity light curing, resulting in excessive to wet and interact with the moist dentin surface and
polymerization stresses hydrophobic groups to ensure bonding to the restorative
• Inadequate etching and bonding of that area resin.
• Traumatic finishing techniques
86. 'C' [Sturdevant 5th ed 419)
Potential solutions: Diagnodent device is based on principles of caries detection
• Re-etch, prime and bond the area by laser fluorescence. It employs visible red diode laser to
detect a carious Lesion.
• Use slow start polymerization techniques
• Use atraumatic finishing techniques Advantages:
Eg. Light intermittent pressure • Detects very early lesions and permits for practising
minimal invasive and preventive techniques.
Why is a whiteline or halo observed around the enamel • Has high specificity (i.e., No false +ve response)
margin following a composite restoration? (APPG-15)
a) Contamination of t he operating field
• Employs non ionizing radiation
• Can be safely used in pregnant woman.
b) Because of the spaces left between increments during
insertion
c) Overcontouring the restoration 87. ' D' [Sturdevant 5th ed 20]
d) High intensity light curing resulting in polymerisation The basic unit of enamel is enamel rod. In cross section
stress it has a key-hole pattern with head towards crown and
tail towards root. Each rod consists of millions of apatite
80. ' B' & ' D' [Sturdevant 5th ed 246]
crystals. These crystals in rod head are oriented parallel to
Acid etching transforms smooth enamel into an irregular the long axis of rod and in tail they diverge slightly {65
degrees) from long axis.
surface, increases porosity removes smear layer. Acid
etching of enamel increases its surface free energy. On
dentin it decreases the surface energy. Dissolution of the crystals to acid occurs more in the head
region. The tail and the periphery of the head appear to be
81. 'C' [Sturdevant 5th SA ed 230/ Sturdevant 5th ed 504] relatively resistant to acid attack.

82A. 'D' [Sturdevant 5th SA ed 309/ Sturdevant 5th ed 507) 88. 'C' [Phillips 12th ed 265)
Option D is more feasib le because it involves conservative Refer synopsis dentin bonding systems point iii.
tooth structure removal. It is also economical when
compared to other esthetic options. 89. ' B' [Phillips 12th ed 260)
Depending on the concentration, phosphoric acid removes
828. 'A' [Sturdevant 5th SA ed 23 7/ Sturdevant 5th ed 508) the smear layer and about 10 microns of enamel to expose
prisms of enamel rods.
Natural day Light is preferred for shade selection. If natural
light is not possible, color corrected operating lights or
ceiling lights should be used. If the dental operating light Type 1 etching:
is used, it should be moved away to decrease the intensity, Selective dissolution of ename[ rod centres (cores)
allowing the effect of shadows to be seen.
Type II etching:
Dissolution of peri pheral areas of enamel rods.
82C. 'C' [Sturdevant 5th SA ed 239/ Sturdevant 5th ed 523]
Poor bonding technique, intermingling of bonding
materials from different systems, contamination and Irrespective of the type of etching patterns, resin tags are
inadequate tooth preparation results in poor retention and approximately 6 microns in diameter and 10 -20 microns in
length thereby leading to micromechanical interlocking.
debonding of restoration. Tooth sensitivity might not be a
major problem when composite is used as a restoration for
closing gaps between teeth.
Dental ;lut.,e

90. 'C' [Check Question No.2]


UDMA, Bis-GMA and TEGDMA are the widely used resin
matrix ingredients that form highly cross linked polymer
structures in composites and sealants.

91. 'A' [Pediatric dentistry by Muthu 1st ed 315]


Glass reinforced composite means glass fibre reinforced
composite.

Indications for glass FRC:


• Space maintainers
• Splinting of periodontally weakened teeth
• Endodontic posts
• Orthodontic retainers
• Resin bonded bridges
• Denture repairs

92. 'D' [Check explanation of Q.No.56]

93. ' C' [Check explanation of Q.No. 77)

94. 'D > A' [Check Q.No.42/ Sturdevant 5th SA 192]


Self-etch primers include a phosphonated resin molecule
that performs two functions simultaneously
• etching and priming of enamel and dentin
• incorporation of the modified smear layer

95. 'D' [Check Explanation Below]


Bis-GMA/TEGDMA is responsible for polymerization
shrinkage. Techniques to reduce polymerization shrinkage:
• Use of a flowable composite or resin modified GIC as a
liner (low young's modulus of elasticity liner).
• Use of an incremental or layering technique.
• Use of silorane based composites which are based on
cationic ring opening mechanism.

96. 'A' [Phillips 11th ed 393]


After etching and priming the dentin, a bonding agent
should be applied and thinned with a brush rather than with
an airjet to avoid oxygen inhibition. The thickness of the
bonding resin should be attest 50µ m to prevent diffusion of
oxygen from the atmosphere through the coating and there
by prevent oxygen inhibition of the primer and the adjacent
bonding resin during polymerization.

97. 'D' [Phillips 12th ed 261)


Etching both enamel and dentin using 37% phosphoric acid
simultaneously is called total etch technique. This concept
was introduced by Fusayama. Since the total etch technique
usually involves etching with an acid followed by rinsing to
remove the acid, this technique is also known as etch and
rinse technique.
, OPERATIVE DENTISTRY

6. CAST RESTORATIONS

1. Secondary flare provides a marginal metal of d) Can be safely heated above 700°C when present in the
a) 10-20 degrees b) 30-40 degrees form of gypsum
c) 140-150 degrees d) none of the above (AIIMS-92)
(MAN-94,AIIMS-93) 12. During preparation for cast gold restoration, the tooth
2. The size of sprue in casting an average bicuspid M.O.D. tissue which is weakened by cavity preparation and
casting by centrifugal force is subjected to damage by forces of mastication must be:
a) 10 gauge b) 12 gauge a) Reduced and covered by a cast restoration
c) 16 gauge d) 20 gauge b) Cemented firmly on amalgam core
(MAN-98) c) Protected by full coverage of the teeth
3. Sprue should ideally be made of d) Beveled and the patient asked not to exert force
a) Inlay wax b) plastic rod (AIPG-98,01)
c) Hollow plastic d) hollow metal 13. Dental wax pattern should be invested as soon as
(MAN-99,01) possible because of danger of:
4. In a class II mesio occlusal inlay gingival bevel is at a) Distortion due to relaxation of internal st resses
a) 30-40 b) 45-60 b) Drying of the wax pattern
c) 60-80 d) no fixed angulation c) Continued expansion of wax
(MAN-99,2K) d) Reduction in flow of wax
5. Reverse bevel is given for (AIPG-98,AP97)
a) Convenience form b) Resistance from 14. Gypsum bonded investment should not be heated above:
c) Lap sliding fit a) 900 degrees centigrade b) 1000 degrees centigrade
d) To remove the undermined enamel c) 700 degrees centigrade d) 800 degrees centigrade
(MAN-01) (KAR -2K)
6. The final polish of a cast gold alloy to achieve a smooth 15. It is necessary to remove a disto-occlusal inlay from
polished surface is done by maxillary premolar. The procedure of choice is:
a) Pickling b) electropolishing a) Cut through the isthmus and remove the inlay in 2 pieces
c) sand blasting d) rouge b) Use a chisel and mallet
(MAN-2K, AP -05) c) Try to tease the inlay out with help of an elevator
7. In a MO cavity prepared for inlay, mechanical advantage d) Remove the inlay in one piece
to prevent proximal displacement of the restoration is (AIIMS-95)
by establishing: 16. Water quenching a gold alloy casting (within its
a) Adequate pulpal depth b) Parallel walls investment. is done immediately after the dull redness
c) Occlusal dovetai L of the button disappears is done to:
d) Acute Lingual inclination in axiogingival area a) Increase the degree of ductility of the casting
(AIIMS-Ol,AP-04) b) Decrease corrosion
8. The sprue in wax pattern should be placed c) Facilitate the easy removal of the casting from its mold
a) At right angle b) At acute angle d) To achieve a desirable surface on the casting
c) At obtuse angle (AIPG-01)
d) It depends upon type of wax pattern 17. The minimally acceptable restoration for an
(AIPG-04) endodontically treated maxillary first premolar is:
9. The occlusal clearance of an onlay is best detected by: a) An onlay b) An MOD amalgam
a) Depth cuts b) Visual inspection c) A full cast crown d) An occlusal amalgam
c) Wax chew-in d) Articulating paper (PGI-03)
(AIIMS-99,AIPG-2K) 18. Failure of casting to seat on the cast is overcomed by
10. Type III gold alloy does not have following features a) Scraping the cast to seat
except: b) Forcefully position the casting
a) Harder than type IV b) Used for small inlays c) Wash the casting with HCL
c) Easily burnishable than type IV d) Discard if the discrepancy is more than 0.3mm
d) Less gold than type IV (PGI-02)
(AIIMS-95) 19. Which of the following is the functionally generated
11, Binder in investment for dental castings: occlusal registration in an indirect gold casting technique
a) Brings about reduction in expansion a) Centric relation b) Anatomic registration
b) Provides adequate strength to the investment, holds c) Registration of the envelope of occlusion
ingredients together and provides rigidity. d) Static registration of dynamic occlusion
c) Is usually a form of silica (AIPG-92)

1) B 2) B 3) D 4) A 5) C 6) D 7) C 8) B 9) C 10) C 11) B 12) A 13) A


14) C 15) A 16) C 17) A 18) D 19) D
Dental ;lut.,e

20. The asbestos liner is used in a casting ring to c) GV Black d) Bowen


a) Permit expansion of the mold (COMEDK-04)
b) Retard heating of the investment 31. The strength of dental investment for gold alloy is
c) Facilitate venting of the mold dependent on the amount of:
d) Both 1 and 3 a) Silica b) Carbon
(KAR-99) c) Copper d) gypsum
21. The best restoration for posterior tooth with an (KAR-04)
undermined cusp with caries is: 32. Retention of inlay is increased by
a) Cast gold b) Silver amalgam a) Increased axial length of preparation
c) Direct gold filling b) flat pulpal walls
d) Pin retained amalgam restoration c) Rounded internal line angles
(PGI-01) d) all of the above
22. Reverse bevels are given: (KAR-98)
a) To facilitate easy withdrawal of wax pattern 33. Blocking out under cuts in cavity preparation for cad/
b) For purpose of aesthetics cam inlay is carried out by
c) To counteract oblique forces a) Dentist after final cavity preparation stage
d) All of the above b) Laboratory technician before wax pattern Preparation
(AIPG-96,94) c) The system automatically during the optical impression
23. Pain and sensitivity in a tooth with onlay after 4 months d) Dentist in the wax preparation
is due to: (UPSC-01)
a) Supra occlusion b) Gingivitis 34. After an inlay preparation which of the following would
c) Infra occlusion d) None of the above be most appropriate to expose margins for an elastic
(AIIMS-97) impression?
24. In designing a retainer on a noncarious mandibular first a) Surgical intervention
premolar abutment with short clinical crown which of b) Packing with cord impregnated with alum or epinephrine
the following restorations is most appropriate? c) Packing with cord impregnated with Zinc chloride
a) A full crown b) A reverse 3/4 crown d) Packing with Zinc oxide eugenol
c) MOD onlay d) Inlay (APPSC-99)
(AIPG-92) 35. The function of a reservoir in a sprue is to avoid:
25. Contact areas in a gold inlay should be: a) Localised shrinkage porosity
a) Soldered b) Micro porosity
b) Carved out and contoured in the wax pattern c) Generalised porosity d) None of the above
c) A point contact with neighboring teeth (AIPG-99)
d) Rounded and at a distance from neighboring tooth 36. Biocompatibility is best for:
(AIPG-95) a) Beta-titanium b) Stainless steel
26. Pickling is done: c) Nitinol d) Base metal alloys
a) To remove oxide fi lm from casting (AIPG-00)
b) Polish the casting 3 7. Sprue pin is attached to posterior teeth in
c) Improve the strength of casting a) Cusp tip b) contact area
d) Avoid casting defects c) marginal ridge d) proximal surface
(AIIMS-03,KAR-01) (PGI-03)
27. Base metal alloys are useful for 38. Fracture tooth is restored with
a) Cast partial dentures b) Crowns a) Pin retained amalgam b) Cast gold
c) Orthodontic wires d) all of the above c) Both d) Direct gold
(AP-01) (PGI-03)
28. In class II cavity for inlay, the cavosurface margin of the 39. Inlay is
gingival seat clears the adjacent tooth by: a) Cast gold b) Direct gold
a) 0.20± 0.05mm b) 0.50± 0.20mm c) Both d) None of the above
c) 0.80 ± 0.35mm d) 1.10 ± 0.45mm (PGI-03)
(AIIMS-93) 40. The CAD-CAM system uses
29. Short bevel extends a) Cadmium in castings
a) in almost 2/3 of the enamel b) A porcelain furnace with very high temperature
b) all of the enamel and half of the dentine c) uses a video camera to all the prepared tooth
c) all of the enamel only d) 1/3 of the enamel d) None of the above
(AIPG-97 ,COMEDK-04) (KAR-98)
30. Who was the first person to introduce cast inlay 41. Ideal requirement to metal ceramic casting
restoration into dentistry? a) Low fusing ceramics and high fusing alloys
a) William H Taggart b) Micheal Buonocore b) Matching of co-efficients of thermal expansion

20) A 21) A 22) C 23) A 24) A 25) B 26) A 27) D 28) B 29) C 30) A 31) D 32) A
33) A 34) B 35) A 36) A 37) C 38) B 39) A 40) C 41) D
, OPERATIVE DENTISTRY

c) Bonding between ceramic and alloy c) In specially fabricated aluminium vanadium crucible
d) All of the above d) using CAD-CAM technique
(MAN-02) (AIPG-04)
42. Gap between true end of casting ring and wax pattern 53. Passivating alloys are
should be a) Cr, Al, Ti b) Cr, Mo, Ti
a) 1/, inch b) 1/2 inch c) Cr, Fe, Mo d) Cr, Gold, Ti
c) 3/8 inch d) one inch (AIPG-04)
{MAN-02) 54. Crucible indicated for casting base metal alloys is:
43. The most commonly used Titanium alloy for dental and a) Carbon crucible b) Clay crucible
medical purposes is c) Quartz Crucible d) High melting plastic crucible
a) Ti 6Al 4V b) Ti 5Al 4 V (AIPG-04)
c) Ti5Al 5V d) Ti6Al 6 V 55. Grain elongation is due to
(KAR-02) a) Cold working b) Recrystalization
44. Cobalt chromium alloys contain c) Recovery d) Age hardening
a) 30% cobalt and 60% chromium (BH U-07)
b) 60% cobalt and 30% chromium 56. Acute angles in gold gives
c) 1% palladium d) 20% gold a) outline form b) Convenience form
(AIPG-03) c) Esthetics d) Retention form
45. Cleaning of base metal alloy is done by (BHU-07)
a) Warm HCL b) Cold HF 57. Which is the only nearly pure metal used for dental
c) Cold H2 So4 casting purposes?
d) Sand blasting with aluminium oxide a) Gold b) Palladium
(AIIMS-97) c) Silver d) Pure titanium CP Ti
46. Glossy smooth margins of a casting are due to (COMEDK-10)
a) Shrinkage of alloy on cooling 58. The purpose of investment setting expansion is to
b) Investment breakdown compensate for
c) incomplete wax elimination a) Casting Shrinkage b) Casting expansion
d) incomplete gas elimination c) Wax elimination d) Alloy melting
(AIIMS-99) (COM ED K-10)
47. Incomplete casting is the result of 59. The porosity formed due to simultaneous nucleation of
a) More porous investment in the mold solid grains and gas bubbles at the first moment that
b) Narrow sprue diameter the metal freezes at the mold walls is called:
c) Hollow sprue pin d) Large reservoir a) Pinhole porosity b) Gas inclusion porosity
(AIIMS-95) c) Microporosity d) Subsurface porosity
48. Back pressure porosity can be avoided by (KAR-03)
a) Using asbestos Liners 60. The secondary flare is necessary for class II cast gold
b) Placing the sprue at least one quarter inch away from inlay preparation for several reasons but basically the
the end of casting ring direction of the flare results in -
c) Preventing rapicl heating of investment a) Good aesthetics
d) Using a short and wide sprue b) 60° marginal metal which is burnishable
(AP-98) c) 40° marginal metal which is burnishable
49. Localised shrinkage porosity is seen when there is d) 10° marginal metal which is burnishable
a) Thin sprue b) Investment heated strongly (COMEDK-09)
c) Inadequate air discharged) Low casting pressure 61. The Cavo surface gingival bevel for a class II cast gold
(PGI-01) inlay preparation should be
50. The function of a sprue is to a) 0.5 to 1 mm wide and blencl with secondary lingual flare
a) Form an opening for molten metal to enter the mold b) 0.2 to 0.3 mm wide and blend with secondary lingual flare
b) Help polish cast restorations c) 0.2 mm wide and blend with secondary lingual flare
c) Eliminate air bubbles on the wax pattern d) 3 to 4 mm wide and blend with secondary lingual flare
d) Reproduce fine detail (COMED-08)
(AIIMS-98) 62. Which of the following is not required for retention of
51. The inlay wax used for direct technique: cast gold restoration
a) Type I b) Type II a) Cavosurface bevel b) Parallel walls
c) Type III d) Type IV c) beveled axiopulpal line angle
(KAR-2K) d) undermining dentinal walls
52. Titanium casting is done (BH U-07)
a) Under vacuum in argon atmosphere
b) Under air pressure in nitrogen atmosphere

42) A 43) A 44) B 45) D 46) C 47) B 48) B 49) A 50) A 51) A 52) A 53) A 54) C
55) A 56) D 57) D 58) A 59) D 60) C 61) A 62) D
Dental ;lut.,e

63. A patient comes to a dentist with decayed mandibular


1st molar including mesiobuccal and distobuccal cusp,
restoration of choice is
a) Intermediate restoration
b) Composite
c) Cast metal d) GIC
(AIIMS MAY-14)

63) C
, OPERATIVE DENTISTRY

6. CAST RESTORATIONS - ANSWERS

1. ' B' (Sturdevant 5th SA ed 440 / 5th ed 861 Counter or For capping cusps to protect and support
Marzouk 1" ed 316) reverse bevel them.
In intracoronal cavity preparations, bevels are placed on
occlusal and gingival margins while flares are prepared on 6, ' D' [Marzouk 1" ed 388)
facial and lingual proximal walls. These results in Pickling is the process of cleaning the gold casting by 500/o
• 30 - 40° marginal metal which can be easily burnishable warm HCl. Final polishing of cast gold restorations is by the
and adaptable. use of Iron oxide (rouge).
• 140 - 150° strong enamel margin.
Option 'C' sandblasting with aluminium oxide is
Primary flares always have a specific angulation of 45° to recommended for removing the investment material from
the inner dentinal wall paper. Secondary flare is a flat plane base metal alloys. Final finishing of base metal restorations
superimposed peripherally to a primary flare. Secondary is obtained by electrolytic polishing.
flares have different angulations.
7. 'C' (Sturdevant 5th SA ed 434/ 5th ed 852
2. 'B' (Check Explanation Below] Marzouk 1'' ed 323)
Generally sprue former diameter is between 8 - 18 gauge, Class II inlay:
8 gauge is 3mm in diameter and 18 gauge is 0.8mm in • Preparation features that prevents proximal displacement
diameter. The more the diameter, the faster the melt enters of inlay are occlusal dovetai l and proximal grooves.
the mold. • Features that prevent occlusal displacement of inlay are
• Sprue gauge for inlay - 8 - 12 parallel facial and lingual proximal walls.
• Sprue gauge for onlay - 10 - 14
Class II amalgam:
3. ' D' [Marzouk 1" ed 376) • Features that prevent proximal displacement of amalgam
The three types of sprue formers are resin, wax and metal. are occlusal dovetail and proximal locks.
Metal sprue formers have the advantage of rigidity during • Features that prevent occlusal displacement of amalgam
investment. Metal sprue formers may be hollow or solid. are the convergent facial and lingual proximal walls.
Hollow metal sprue formers are preferable as they are more 8, ' B' [Marzouk 1•t ed 377)
retentive to the pattern and supply minimal heat to the The sprue is placed at 45° angle to the wax pattern. If it is
already formulated pattern. directed at 90° to a flat portion of a wax pattern, reverse
flow with turbulence occurs.
4. 'A' [Sturdevant 5th SA ed 441/ 5th ed 859
Marzouk 1•t ed 315) 9. 'C' [Sturdevant 4th ed 824]
• Occlusal bevel in inlay - 30 - 45° Wax interocclusal records are helpful for checking the
• Gingival bevel in inlay - 30° occlusal clearance.
• Gingival bevel in amalgam restoration - 15-20°
Option 'I{ depth cuts serve as guides for the cusp reduction.
Gingival bevel in inlay helps in
10. 'C' [Phillips 11th ed 571/ Marzouk 1'' ed 309]
• Lap / slide fit of the casting Cast gold alloys contain at least 750/o of noble metal
• the 30° metal that is easily adaptable elements. The content of gold decreases from Type I to
• Removal of weak enamel (gingivally declined enamel Type II alloys.
rods)
Type I (Soft) Highest content of gold, used for
Gingival bevel of 15 - 20° (Keral-15) in amalgam restorations small inlays, easily burnishable.
improves the resistance form by removing weak enamel.
Type II (Medium) For inlays, panties.
5. 'C' [Sturdevant 4th ed 822/Marzou k 1" ed 327) Type III (Hard) For inlay.s and on lays subjected to
high stress and small span bridges.
Partial bevel Involves part of enamel. Used for removal
of weakened enamel. Type IV Lower content of gold used for long
Short bevel Involves entire enamel. (Extra hard) span bridges and partial dentures.

Long bevel Involves entire enamel and half of 11. ' B' [Marzouk 1•t ed 379/ Phillips 11th ed 297)
dentin. Most commonly used bevel. Investment mainly consists of binder which provides
Full bevel Involves full enamel and dentin. strength. Silica acts as a refractory and controller of setting
expansion.
Dental ;lut.,e

Investment Binder 22. 'C' [Sturdevant ! Ill~ 822)


Reverse bevels are not indicated in the facial surfaces
Gypsum bonded Alpha hemihydrate of maxillary premolars and first molar as their aesthetics
Phosphate Ammonium magnesium phosphate demand a stub margin. Reverse bevels protect and support
Silicate Silisic acid or colloidal silica the capped cusps.

12. 'A' [Sturdevant 5th SA ed 443/ 5th ed 84 7 23. 'A'


Marzouk 1" ed 308)
24. 'A' [Tylman 8th ed 149)
13. 'A' [Marzouk 1•• ed 374) Short clinical crown requires the use of full crown for
retention.
14. 'C' [Phillips 11th ed 306)
If gypsum bonded investments are heated above 700°C 25. 'B' [Sturdevant 5th SA ed 461/ 5th ed 899
they decompose and release sulfur dioxide gas which Marzouk 1'' ed 375]
contaminate the casting and makes them brittle. The
dimensional changes that gypsum investment undergoes 26. 'A' [Marzouk t " ed 388/ Philips 11th ed 336]
during heating are The casting recovered from investment is dark in colour
due to surface oxides formation. The casting is cleaned by
• Contraction between 200 - 400°C heating it in HCl or sulphuric acid. This is called as pickling.
• Expansion between 400 - 700°C
• Large contraction if heated above 700°C 27. 'D' [Phillips 11th ed 599)
Properties of base metal alloys:
15. 'A' • Low density
• Low percentage elongation
16. 'C' [Marzouk l't ed 384)
Water quenching of investment after casting helps in • High fusing temperature
cracking of investment resulting in easy removal of gold • High hardness
casting. This is particularly useful for gypsum bonded • Tarnish and corrosion resistance
investment used for gold alloys. Recovery of casting from
phosphate and silica ·investments is difficult. 28. 'B' [Sturdevant 5th SA ed 435/ Phillips 11th ed 853)

17. 'A' [Sturdevant 5th SA ed 430/ 5th ed 84 7 29. 'C' [Marzouk 1•t ed 325)
Marzouk 1st ed 308)
Cast restorations are indicated for endodontically treated 30. 'A' [Phillips 11th ed 565)
teeth. An onlay is the minimally acceptable restoration
while full cast crown is ideal for such situation . 31. 'D' [Phillips 11th ed 297)
Investment Type Restorative material casted
18. 'D' [Sturdevant ~!.h~ 217)
The metal margins should be closely adapted to the die by Gypsum bonded Conventional gold alloys
minimum mechanical deformation (burnishing). Marginal Phosphate bonded Metal ceramics
gaps that exceed 0.3 mm should not be burnished. Rat her
Silicate bonded Base metal removable partial dentures
the casting should be remade.
32. 'A' [Marzouk 1" ed 314)
According to ADA, the ideal thickness of cement film is 25
The more the length of preparation wall or axial surfaces,
microns.
the more is the retention. A taper of 2 - 5° per wall
can be made as it does not affect the retention. Shorter
19. ' D' [Sturdevant 4th ed 38)
preparation walls need an approach to exact parallelism as
the retention is less with short walls.
20. 'A' [Marzouk l't ed 3 82)
Use of a casting ring liner (Asbestos, paper, kaolin) or split
33. 'A' [Sturdevant ~ !.h~ 221]
metal ring or rubber ring helps in permitting the expansion
of the mold. i) CAD/ CAM milling:
Ceramics, composites and metal restorations can be
21. 'A' [Sturdevant 5th SA ed 429/ 5th ed 847] machined from CAD (Computer Aided Design) or CAM
Marzouk 1•t ed 326) (Computer Aided Machine) milling. CAD/CAM milling
Even though pin retained amalgam restorations can be used uses digital information about the tooth preparation to
for extensive or complex tooth involvement, restoring with provide a CAD on the video monitor for inspection and
cast restoration is the best choice. modification. Once the 3-D image for the restoration
design is accepted, the computer sets instructions to
guide in cutting (CAM} the restoration from a block of
material.
, OPERATIVE DENTISTRY

ii) Copy milling: 45. ' D' (Phillips 11th ed 3 3 6)


Titanium, composites and ceramic materials can be copy Base metal alloys require sandblasting with aluminium for
milled easily. It uses a replica (wax) of the desired form cleaning the investment after casting. Acid should never be
as a guide for a milling machine. used for cleaning base metal alloys.

34. 'B' (Sturdevant 5th SA ed 454 / 5t• ed 890 46. 'C' [Phillips 11th ed 348)
Marzouk 1" ed 346] Incomplete casting with rounded margins and shiny
The most common impression material used for the indirect appearance occurs due to incomplete wax elimination.
casting technique is "addition silicones". This shiny condition is caused by carbon monoxide left by
residual wax.
If the margins of the preparation are subgingival, a
retraction cord is used to temporarily displace the free 47. ' B' [Phillips 11t• ed 347]
gingiva. To control moisture or haemorrhage the cord may Incomplete casting is due to incomplete filling of mold by
be impregnated with vasoconstrictor or 100% alum or 15% molten alloys. The common causes are
tannic acid. Zinc chloride and silver nitrate cause ulceration • Insufficient venting
and local necrosis if used in high concentrations • High viscosity of metal due to insufficient heating
• Incomplete elimination of wax residues from the mold
Vasoconstrictors are contraindicated in patients with
arrhythmias, hyperthyroidism, diabetes and in those
48. ' B' [Phillips 11th ed 346]
receiving B-blockers, etc.
Back pressure porosity is caused by the entrapped air in
the mold that does not escape through the pores of
35. 'A' [Marzouk 1" ed 377)
the investment (KCET-08). It is prevented by using high
Reservoir is an area in the sprue with dimensions far
casting pressure, proper burnout technique and using long
exceeding that of the thickest portion of the pattern. It is
sprue that makes the thickness of investment material
always indicated when the sprue is long or thin or any other
between end of pattern and casting ring not more than t/4
reason that results in localized shrinkage porosity.
inch (6mm).
36. 'A' [Phillips tO'h.ll, 651)
49. 'A' [Phillips 11th ed 343]
Beta-Titanium is the alloy of titanium, molybdenum and
aluminium. It has an excellent biocompatibility. It is used
50. 'A' [Marzouk t •t ed 376/Phillips 11th ed 319)
as an implant or heart valve material.
51. 'A' [Phillips 11th ed 284]
37. 'C' [Nisha Garg 1st ed 209)
In direct technique of preparation of wax pattern, Type
The sprue should be positioned in the thickest portion
I or medium or Type B wax is used whereas in 'indirect
of the pattern, ideally to the marginal ridges of posterior
technique' Type II or soft or Type C wax is used.
teeth or incisal corners of anterior teeth.
52. 'A' [Sturdevant 4th ed 221]
38. ' B' [Sturdevant 5th SA ed 429/ 5th ed 84 7]
Titanium has high melting point (1668°C) and a special
casting machine with arc melting capability and an argon
39. 'A' [Sturdevant 5th SA ed 429/Marzouk 1" ed 323)
atmosphere is typically used.
40. 'C' [Sturdevant 5th SA ed 282/ 5th ed 610)
53. 'A' [Phillips tQth ed 455]
41. 'D' [Phillips 11•h ed 581]
54. 'C' [Phillips 11th ed 333)
Ceramics have high melting point and low coefficient
of thermal expansion . The alloys used for metal ceramic Crucible Used for
restorations should have Carbon sensitive alloys such as
• High fusing tern perature to resist sag deformation. Quartz palladium and base metal alloys are
• Their coefficient of thermal expansion should match fused
with ceramics. Carbon crucible For high fusing gold alloys
• Should make bolild with ceramic. Clay crucible For fusing high noble alloys

42. 'A' [Marzouk 1't ed 377/ Philips ttt• ed 323) 55. 'A' [Philips 11th ed 576]
There should be a minimum gap of 1/8 - 1/4thinch between
the ends of casting ring and wax pattern. This will provide 56. ' D' [Sturdevant 5th ed 859]
pathways for gas escape during casting.
57. 'D' [Sturdevant 5th SA ed 419]
43. 'A' [Phillips t t •h ed 649)
58. 'A' [Philips 111h ed 306]
44. 'B' [Phillips tQth ed 443)
Dental ;lut.,e

59. 'D' [Phillips 11th ed 345]


Effects of technical factors on porosity
Localized
Type of Sub surface
shrinkage Micro porosity
Porosity porosity
porosity
Increased
sprue Descreased Increased No effect
thickness
Increased
Incareased Descresed No effect
sprue length
Increased
Decreased Increased Decreased
melt temp.
Increased
Decreased Increased Decreased
mold temp.

60. 'C' [Sturdevant 5th SA ed 440/ Sturdevant 5th ed 859]

61. 'A' [Sturdevant 5th SA ed 441/ Sturdevant 5th ed 859)


The gingival bevel should be 0.5 - 1mm wide and should
blend with the lingua,[ secondary flare.

62. 'D' [Sturdevant 5th ed 851, 859]


Undermined dentinal walls should be removed with the
bevelling. Undermined or weak enamel or dentin walls
hinders the path of insertion.

63. 'C' [Sturdeant 5th SA ed 429]


In the present case there is involvement of two cusps by
dental caries. The restoration should provide provision for
cuspal capping. The minimal restoration will be cast metal
on lay.
, OPERATIVE DENTISTRY

7. INSTRUMENTATION
1. If the speed of the aerotor hand piece is increased to d) None of the above
100, 000 rpm then. (AP-03,AP-99)
a) Decreased vibration perception to the patient 12. For the balance of hand cutting instrument:
b) More heat is generated resulting in pulpal damage a) cutting edges of the working end should be centered
c) High efficiency d) all of the above over the long axis of the handle
(MAN-99) b) The cutting edge centered on a line running at right
2. Disadvantages of high-speed instrumentation is angle to the long axis of the handle
a) aerosol spray inhalation c) It should have shorter, straighter shank
b) loss of tactile sensation to the operator d) It should have solid handle
c) heat production d) all the above (KAR-03)
(MAN-99) 13. High speed includes the R.P. M between
3. Ultrahigh-speed rotary cutting tools rotate at a) 20000-50000 b) 50000-100000
a) <60,000 rpm b) 60,000-3,00,000 rpm c) 100000-200000 d) above 200000
c) 3,00,000-5,00,000 rpm d) > 5,00,000 rpm (AP-99,KAR-97)
(MAN-95) 14. The type of chisel with the shank and blade slightly
4. Which of the following instruments will have a four curved is:
number formula a) Straight chisel b) Wedelsaedt chisel
a) ordinary hatchet b) GMT c) Enamel hatchet d) GMT
c) Jeffery hatchet d) all the above (AIPG -99)
(MAN-2K) 15. Instrument used to plane buccoproximal and
5. The most effective method of maintaining dry field in linguoproximal walls in class II cavity is:
dentistry is a) Enamel hatchet b) Bibevelled hatchet
a) Gingival retraction b) Rubber dam c) Chisel d) Single sided chisel
c) Throat swab and cotton d) Suction tips (AIPG-96)
(MAN-02) 16. Instrument which forms a blade angle of more than 125
6. Instrument used for gingival retraction during rubber is likely to be:
dam placement is a) biangle chisel b) Curette
a) retainer no:208 b) retainer no:210 c) chisel d) Angle former
c) retainer no:212 d) retainer no:216 (AIPG-95)
(MAN-2K) 17. The advantage of ultra speed hand piece is:
7. When dental bur is sterilized by autoclaving, which of a) Low frequency and high amplitude make the patient
the following chemical is used for protection of the bur comfortable
a) Sodium nitrate b) Sodium nitrite b) High frequency and low amplitude do not let the patient
c) Sliver nitrate d) Silver nitrite perceive pain
c) Less trauma d) None of the above
8. The first number in the instrument formula indicates (AIIMS-99, AIPG-96)
a) Length of the blade in centimeters 18. On a carbide bur, a great number of cutting blades results in:
b) 1/10th of the width a) Less efficient cutting and a smoother surface
c) angulation of the blade in degrees b) Less efficient cutting and a rougher surface
d) primary cutting edge angle c) More efficient cutting and a smoother surface
(AP -99) d) More efficient cutting and a rougher surface
9. Cutting and grinding procedures are predominantly: (AIPG-03)
a) Two directional b) Unidirectional 19. The hard carious dentin is removed with
c) Three directiona l d) Multidirectional a) Excavator b) Bur at low speed
(KAR-02) c) Bur at high speed d) None of the above
10. Four numbered instrument formula other than G. M.T. is: (PGI-02)
a) Chistle b) Hoe 20. A major factor determining the efficiency of bur is
c) Angle former d) Hatchet a) Taper ang le b) Spiral angle
(KAR-99) c) Head length d) Head diameter
11, Rake angle is negative (TNPSC-99)
a) If rake face is before radial line 21. The angle former is a special type of
b) If rake face is inline with radial line a) Chisel b) Excavator
c) If rake face is behind radial line c) Hatchet d) Rotary instrument
(TNPSC-99)

1) D 2) D 3) B 4) B 5) B 6) C 7) B 8) B 9) B 10) C 11) A 12) A 13) B


14) B 15) A 16) D 17) B 18) A 19) C 20) B 21) B
Dental ;lut.,e

22. The enamel bevel of a gingival floor is done with: c) Class II with wide proximal box
a) Hatchet b) Chisel d) Narrow class II cavity
c) Hoe d) Curette (KAR-03)
(AIPG-99) 34. Honing machine is used for:
23. In dental burs, the angle formed between the back of a) CAD CAM INLAY fabrication
the blade and the tooth surface is called as b) Polishing restorations
a) Clearance angle b) Rake angle c) Mechanical sharpening of Instruments
c) Cavosurface angle d) Point angle d) Caries detection
(UPPSC-01) (KAR-02)
24. Efficiency of carbide bur increases by 35. Ferrier double dow separator works on the principles of
a) Negative (or) Zero rake angle a) Wedge principle b) Traction principle
b) 90° edge angle c) Slow separation d) Delayed tooth movement
c) At high speed d) All of the above (AIPG-07)
(KAR-96) 36. All of the following influence the efficiency of bur, except:
25. Sodium nitrite is used as a) Neck diameter b) Length and diameter of bur
a) Anti plaque agent b) Antirust agent c) Height of taper of bur.
c) Desensitising agent d) Bleaching agent d) Spiral angle and cross-cuts of bur.
(PGI-03) (AIIMS-06)
26. Universal matrix band retainer is also known as 37. In a class V preparation, during application of rubber
a) Ivory 8 retainer b) Tofflemire retainer dam the punch of the tooth to be restored should be
c) Sequiland retainer d) None of the above made:
(MAN-2K) a) According to the template su pplied
27. Matrix band is difficult to place on - - teeth b) According to the existing arch form
a) MO 25 b) DO 37 c) Labial to the position of the tooth to be operated
c) DO 24 d) MO 24 d) Lingual to the position of the tooth to be operated
(AIPG-93) (AIPG-07)
28. Auto matrix is a 38. Most significant angle in bur design
a) Matrix system that automatically gets attached with a) Rake angle b) Clearance angle
retainer and the tooth c) Spiral angle d) Crosscut angle
b) System where a single retainer can be used for all matrix (AP-06)
bands 39. Instrument that has a 4-digit formula:
c) Retainer less matrix system designed for any tooth a) Angle Former b) Hoe
regardless of its circumference c) Hatchet d) Spoon Excavator
d) Retainer less matrix system designed for any tooth with (AIPG-06)
specific circumference 40. Which of the following is not true about hand
(U PSC-01) instruments used in Operative Dentistry?
29. What is the thickness of class II matrix band a) They are generally made of stainless steel
a) 0.002 inches b) 0.002mm b) Nickel-Cobalt-chromium is never used in its fabrication
c) 0.02 inches d) 0.02mm c) Carbon steel is more efficient than stainless steel
(AP-97) d) It can be made with stainless steel with carbide inserts
30. Wedging in the interproximal areas in case of recession (AI-05)
is called 41. A dental bur cuts the tooth structure more effectively
a) Piggy back b) Tie back when rake angle of bur is
c) Tie through d) None of the above a) Positive b) Negative
(AP-00) c) Radial d) None of the above
31. Retainer less matrix that can be used for all teeth (KAR-04)
(Retainer less matrix) 42. One of the following is the best of provent pulp damage
a) Tofflemeir b) IvoryNo.1 during cavity preparation
c) Universal d) Automatrix a) retain smear layer
(AP-99,03) b) Use sharp burs with brush stroke
32. Which tooth separator works on traction principle c) use surface anesthesia d) use adequate water coolant
a) Ferrior b) Wedge (AIPG-07)
c) Elliot d) Ivory 43. As far as the raise of frictional heat, in using various
rotary cutting instruments is concerned which one of
33. In which of the condition "PIGGY BACK" wedging is the following is the most kind to the pulp
indicated? a) Stainless steel bur b) Diamond bur
a) Proximal caries with gingival recession c) carbide bur d) Carbon steel bur
b) Tooth with fluted surface (AIPG-07)

22) B 23) A 24) D 25) B 26) B 27) D 28) C 29) A 30) A 31) D 32) A 33) A 34) C
35) B 36) C 37) C 38) A 39) A 40) B 41) A 42) D 43) C
, OPERATIVE DENTISTRY

44. The main disadvantage of using wing shaped rubber dam b) Rotating rapidly before contacting the tooth
retainer is: c) Placed in contact with tooth before starting
a) It does not provide extra retention of the rubber dam d) Rotating rapidly before entering in to the oral cavity
b) Laceration to the gingival tissue (AIPG-09)
c) Interferes wit h placement of matrix band 55. The retainerless matrix system with four types of bands
d) None of the above fitted around any teeth regardless of its circumference
(AIPG-07) is called
45. The thickness of thin rubber dam is: a) Tofflemire matrix b) Auto matrix
a) 0.002 inch b) 0.004 inch c) Ivory No. 1 matrix d) Ivory No. 8 matrix
c) 0.006 inch d) 0.2 inch (UPSC-09)
(AIPG-07) 56. Carborundum is chemically known as
46. Diamond abrasive having particle size of 125-150 mm is a) chromium carbide b) nickel carbide
classified: c) silicon carbide d) aluminum carbide
a) Fine b) Coarse (AP-09)
c) Medium d) Extrafine 57. For GV Black class V cavities, the retainer of choice is
(AIPG-07) a) No. 212 b) No. 112
47. Enamel hatchet is differentiated from chisel by all c) No. 132 d) No. 221
except: (MCET-10)
a) Curved in one plane only 58. What is the limit of eccentricity of an air rotor bur,
b) Blade is larger beyond which it stops working
c) Blade is perpendicular to the long axis of hancle a) 0.01 b) 0.02
d) Blade is heavier. c) 0.03 d) 0.04
(AIIMS-06) (AIPG-10)
48. Sterilization of hand piece will cause: 59. Distance of the blade from the shank to have
a) Rust of hand piece b) Turbine wear antirotational stability of the instrument?
c) Fibre loss d) Loss of torq ue a) 2 mm b) 3 mm
(AIIMS-06) c) 4 mm d) 5 mm
49. All of the following about use of aerator hand piece are (AIIMS-09)
false except: 60. The disadvantage of winged rubber dam retainer is that
a) The sound frequency greater than 75 decibel (dB) causes it interferes with the placement of
damage to the ear a) Matrix bands b) Dental floss
b) The frequency of rotation of bur is 6000-8000rpm c) Rubber dam frame d) Rubber dam sheet
c) It will harm more in elderly than very young (KCET-2011)
d) Its use in younger teeth is contraindicated. 61. Most often used thickness of the matrix band is
(AIIMS-06) a) 0.0015 cm b) 0.0015 inch
50. The matrix retainer where in anatomy adaptation is c) 0.015 inch d) 0.0028 mm
possible without wedges is: (COM ED-2012)
a) Tofflemire b) Steele's Siqueland 62. The recommended rubber dam retainer for maxillary
c) Ivory no. 1 d) Ivory no. 8 molar anchor teeth is
(KCET-07) a) W7 b) W8
51. The instrument that has four unit formula c) W4 d) W2
a) Gingival margin trimmer (KCET-2012)
b) Enamel hatchet 63. Winged rubber dam retainer in endodontics advantages
c) Hoe d) Chisel are all except
(KCET-07) a) Radiographs are good showing full length of canals
52. Steeglitz pliers is b) Stability
a) Used t o contour matrix bands c) Provide extra bucco-lingual retraction
b) Used to hold the silver point d) Dam, clamp and frame placed in one operation
c) Used to remove roots during hemisection (AIIMS-2011)
d) Used to place rubber dam clamp on the tooth 64. Interdental papilla protruding from the rubber dam,
(KCET-07) most common cause is
53. Hand cutting instruments are composed of a) Inflammation of interdental papillae
a) Handle and blade b) Handle, shank and blade b) Use of light weight rubber dam
c) Shank and blade d) Handle, shank and head c) Punch are placed too far
(KCET-08) d) Punch are placed too close
54. For most effective cutting and long usefulness of a (AIIMS-2011)
tungsten carbide bur, it should be?
a) Rotating slowly before contacting the tooth

44) C 45) C 46) B 47) C 48) C 49) A 50) B 51) A 52) B 53) B 54) B 55) B 56) C
57) A 58) B 59) A 60) A 61) B 62) B 63) A 64) D
Dental ;lut.,e

65. The number 12 in a 3 unit formula 12-6-8 indicates the 76. Use of water spray during restorative procedures has the
blade is following advantages
a) 12 mm in length b) 1.2 mm in length a) Dehydration of oral tissues
c) 12 mm in width d) 1.2 mm in width b) Clean view of cavity can be achieved
(GCET-14) c) Pulp is protected from the heat generated
66. The matrix band for restoring a Class-I with Lingual d) Bacterial cont amination is controlled
extension is (APPG-15)
a) Tofflemire b) Ivory No.8 77. The recommended clamp for class-V tooth preparation is
c) Barton d) T band a) No.213 b) No.212
(GCET-14) c) No.123 d) No.122
67. The cutting efficiency of a bur is improved by (KERALA-15)
a) Applying more pressure
b) More number of blades
c) Large size of bur d) Rake angle
(GCET-14)
68. The design and construction of an ordinary hatchet is
a) Monoangle b) Monoangle, bibevelled
c) Monoangled, bibevelled and cutting edge parallel to
long axis
d) Monoangle, bibevelled and cutting edge perpendicular
to long axis
(GCET-14)
69. The encircled number in the given instrument formula
represents 10-(80)-6-12
a) Width of the blade b) Length of the blade
c) Angulation of the blade to the long axis
d) Angulation of the cutting edge of blade to the long axis
(COMEDK-14)
70. The recommended rubber dam to provide effective
retraction of gingival tissue is
a) Thin b) Extra heavy
c) Heavy d) Medium
(COMEDK-14)
71. Christmas Tree profile variation is attributed to the
following head shape of a diamond abrasive instrument
a) Flame b) Needle
c) Interproximal d) Taper
(AP-14)
7 2. The second number in a four number instrument formula
denotes
a) Blade angle b) Blade length
c) Primary cutting edge angle
d) Blade width
(AP-14)
73. The thickness of matrix band is
a) 0.015- 0.02 inch b) 0.0015 - 0.002 inch
c) 0.0015 - 0.002 mm d) 0.015 - 0.02 mm
(GCET-14)
74. Suffix for rubber dam denotes
a) Winged camp b) Wingless camp
c) Deciduous teeth d) Partially erupting teeth
(PGI JUNE-2012)
75. Percentage of carbon in carbon steel hand instruments
is
a) 1-1.2% b) 10-12%
c) 0.6 to 0.8% d) 6 to 8%
(AIIMS MAY-14)

65) D 66) C 67) D 68) C 69) D 70) C 71) B 72) C 73) B 74) D 75) A 76) C 77) B
, OPERATIVE DENTISTRY

7. INSTRUMENTATION - ANSWERS

1. ' D' [Sturdevant 5th SA ed 120/ 5th ed 343] Eg.: GMT, Angle former.
Adva ntages 1 2 3 4
• Greater tactile sensation to the operator (Width) (Cutting edge) (Length) (Blade angle)
• Used for excavating caries and for angle
finishing and polishing procedures
5. ' B' [Sturdevant 5th SA ed 136/ 5th ed 463]
Disadvantages
Low speed • Tinne consuming 6. 'C' [Marzouk 1'1 ed 459]
• Ineffective cutting No. 212 retainer is called as cervical retainer. It is used
both as a clamp and for gingival retraction during class V
• Patient discomfort due to vibration
tooth preparation.
perception
• Tendency for the bur to roll out The hole should be punched facia l or lingual to arch form
• Carbide burs are easily broken at low depending on surface involved.
speed
Adva ntages 7. ' B' [Sturdevant 4th ed 3 72/ 5th ed 392]
Autoclaving rusts carbon steel instruments and burs. To
• Effective cutting avoid rusting they can be sterilized in hot air oven or
• Patients are less apprehensive ethylene oxide gas sterilizer. They can be autoclaved by
• Less time consumption submerging in a small amount of 2% sodium nitrite solution
• Diamond and carbide burs work well at which is an antirust agent.
high speed
8. ' B' [Sturdevant 5th SA ed 113/ 5th ed 329]
High speed • Used for tooth preparation and removal
of old restorations ' B' [Phillips 11th ed 352]
9.
Disadvantages Cutting and grinding (gross cutting) are predominantly
• High heat generation requiring water unidirectional procedures whereas finishing (fine cutting)
spray and polishing (smoothing) are multidirectional procedures.
• Less tactile sensation to the operator
10. 'C' [Sturdevant 5th SA ed 115/ 5th ed 329]
• Aerosol spray inhalation If the second number (cutting edge angle) is 90 - 100
it is distal gingival trimmer. If it is 75 - 85 it is a mesial
2. ' D' [Sturdevant 5th SA ed 120/ 5th ed 343] gingival trimmer.

3. ' B' [Marzouk 1'' ed 69] 11. 'A' [Sturdevant 5th SA ed 126/ 5th ed 356]
Ultra low speed 300 - 3000 RPM Rake face Surface of bur blade on the leading edge
Low speed 3,000 - 6,000 RPM Clearance face
Medium high speed 20,000 - 45,000 RPM or back face Surface of bur blade on the trailing edge
or flank
high speed 45,000 RPM - 1 lakh RPM
Land Plane surface following the cutting edge
Ultra high speed > 1 lakh RPM
Rake angle Angle between rake face and radial line
4. ' B' [Sturdevant 5th SA ed 116/ 5th ed 329] Zero rake
Rake face coincides with radial line
G. V. Black's instrument formula angle
1'1 number Widtli of blade in tenths of a millimeter • Rake face is leading or ahead or before
Negative rake the radial line
2"d number Length of blade in millimeters
angle • Minimizes fracture and increases tool life
Blade angle relative to the long axis of
3rd number handle in percent of 360°. This is always • Used for hard cutting, brittle materials
less than 50°. • Rake face is trailing or behind the
radial line
~or instruments with their cutting edges at angle other Positive rake
• Increases cutting efficiency
than 90° to long axis of blade has a fou r number formula . angle
• Fracture of bur blade occurs
This is the cutting edge angle (angle between cutting edge
and long axis of handle) in percent of 360° . This is placed • Clogging of chip space occurs
in the second position. This is always greater than 50° .
Dental ;lut.,e

• Angle between clearance face and 19. 'C' [Sturdevant 5th SA ed 120/ 5th ed 343)
tooth
Clearance 20. 'B' [Sturdevant 4th ed 334/ 5th ed 354)
• If clearance face is curved it is called
angle Burs with small spiral angles produces
radial clearance Spiral angle
more efficient cutting at high speeds.
• It eliminates friction
These are notches in the blade edges to
Is the internal angle at the edge formed
increase the cutting efficiency at low
by the two surfaces of blade. Greater Cross cuts
Edge angle speeds. Cross cuts burs used at high
the edge angle less is the chance of the
speeds tend to produce rough surface.
blade to fracture.
Measures the symmet ry of the bur head.
12. 'A' [Sturdevant 5th SA ed 112/ 5th ed 328) It indicates whether one blade is longer
Concentricity
The cutting edge of the blade should be nearer to the long or shorter than the others. It is a static
axis of the handle for proper balance. For instruments measurement not related to function
with longer blades this can be achieved by incorporating Is a dynamic test that determines the
2 or 3 angles in the shank (contra angling). For optimum minimum diameter of hole than can
anti rotational tendency, the blade edge must not be off the be drilled by a given bur. It measures
axis by more than 2mm . Run out
concentricity of head and also the
accuracy with which the center of rotation
13. 'B' [Marzouk l't ed 69) passes through the center of the head.

14. ' B' [Sturdevant 5th SA ed 116/ 5th ed 332) 21. 'B' [Sturdevant 5th SA ed 115/ 5th ed 332)
Chisels are intended for cutting enamel and are Excavators are used for caries removal and refinement of
grouped as: internal parts of preparation like sharpening of internal line
A) i) Straight angles and creating retentive features. They are grouped as
ii) Slightly curved - Wedelstat • Ordinary hatchet
iii) Binangle
• Hoe excavator
B) Enamel hatchet • Angle former
For splitting undermined enamel and for placing
grooves. 22. 'B' [Sturdevant 5th SA ed 116/ 5th ed 33)

C) GMT 23. 'A' [Sturdevant 5th SA ed 126/ 5th ed 356)


For beveling gingival margin and axiopulpal line angle.
24. 'D' [Sturdevant ~!!l.filt 336)
15. 'A' [Sturdevant 5th SA ed 436/ 5th ed 743)
Is an excavator used for preparing 25. 'B' [Sturdevant 4th ed 372/ 5th ed 392)
Ordinary
retentive areas in anterior teeth and
hatchet 26. 'B' [Sturdevant 5th SA ed 204/ 5th ed 7 60)
sharpening internal line angles for direct
(Bibevelled) Universal matrix band retainer is called as Tofflemire
gold fi llings.
retainer. It is ideal when three surfaces (mesial, occlusal,
It is a chisel similar to hatchet except that distal) are prepared and commonly used for the two surface
blade is larger, heavier and unibevelled. class II restoration.
Enamel
It is used for splitting undermined
hatchet
enamel in buccal and lingual walls and in 27. 'D' [Sturdevant 5th SA ed 209/ 5th ed 764)
proximal cavities. Concave or fluted surface present on the mesial side of
maxillary first premolar makes the adaptation of matrix
16. ' D' [Sturdevant 5th SA ed 115/ 5th ed 332) band difficult. 'Wedge wedging' technique is used to hold
Angle former is a specific type of excavator used for the matrix band tightly.
sharpening line angles, creating retentive features in
dentin for gold restorations and also to bevel the enamel 28. 'C' [Sturdevant 5th SA ed 220/ 5th ed 773]
margins. It has primary cutting edge angle other than 90°. Automatrix is a retainer less matrix system with four types of
bands designed to fit all teeth regardless of their circumference.
17. ' B' [Sturdevant 5th SA ed 120/ 5th ed 343) It is indicated for extensive class II preparations.

18. 'A' [Marzouk l't ed 74] The advantage is the auto lock loop without retainer and
The number of blades or teeth in a bur is usually 6 -8. the disadvantages are physiologic contours are difficult to
Increasing the blade number decreases the amount of develop and they are not precont oured.
material engaged by each blade resulting in less cutting,
clogging and smooth finish . Burs used for finishing and
polishing have upto 40 blades.
, OPERATIVE DENTISTRY

29. 'A' [Sturdevant 5th SA ed 204/ 5th ed 761] 39. 'A' [Sturdevant 5th SA ed 115 / Sturdevant 4th ed 311,
The matrices may be 0.002 inc h (0.05mm) or 0.001 inch 312]
(0.025mm) t hick. They act as temporary wall ( or) replace
missing wa ll during introduction of restorative material. 40. 'B' [Sturdevant 4th ed 258/Vimal Sikri 3n1 ed 109]
Various metals used for making hand instruments:
30. 'A' [Sturdevant 5th SA ed 209/ 5th ed 764] • Stainless steel
Is useful i n patient s with gi ngival recession. • Carbon steel
Piggy back A second smaller wedge is placed on the
• Nichrome
first wedge to prevent gingival overhangi ng .
• Stellite: Co-Cr alloys
Is useful in case of wide proximal box.
Double • Monel metal: Ni, Cu and Iron
Wedges are placed from both Lingual and
wedging • Tarno: Chromium a lloy
facial surfaces.
In case of maxillary first premolar, due to
41. 'A' [Vimal Sikri 3 ,d ed 121]
Wedge the presence of mesial concavity, a second
wedging wedge is inserted between the first wedge
42. 'D' [Sturdevant 5th SA ed 129/ Sturdevant 5t• ed 362]
and band.

43. 'C' [Sturdevant 5th SA ed 129/ Sturdevant 5th ed 361]


31. 'D' [Sturdevant 5th SA ed 220/ 5th ed 773]
44. 'C' [Sturdevant 5th SA ed 138/ Sturdevant 5th ed 466]
32. 'A' [Vimal Sikri 3rd ed 153]
Winged retainer has anterior and lateral wings. The Wings
Slow Achieved in 48 hours by the use of copper provide extra retraction of the rubber dam from the
separation wire, r ubber or orthodontic wire. operating field.
Frequently used method. It is achieved
Rapid But the wings often i nterfere with the placement of matrix
following 2 principles viz., Wedge and
separation bands, band retainers and wedges.
traction principle.
Separat ion is brought by insertion of pointed
Wedge 45. 'C' [Sturdevant 5th SA ed 13 7 / Sturdevant 5th ed 464]
wedge shaped device between teeth. Eg:
principle Thick 0.010 inch
Wedges, Elliot separator
Separation is achieved by mechanical Medium 0.008 inch
Traction devices engaging t he teet h by means of Thin 0.006 inch
principle holding arms. Eg: Ferrier double bow, Ivory
adjustable, Non-interfering true separator. 46. 'B' [Sturdevant 5th SA ed 127/ Sturdevant 5 th ed 358]
Diamond particle sizes
33. 'A' [Sturdevant 5th SA ed 209]
Coarse 125 - 150 µm
34. 'C' [Sturdevant 5th ed 336] Medium 88-125µm
Fine 60 - 74 µm
35. 'B' (Vimal Sikri 3 n1 ed 153]
Ve ry fine 88 - 44 µm
36. 'C' [Vimal Sikri 3 rd ed 121,122/ Sturdevant 5th SA ed
125/ Sturdevant 5 th ed 354] 47. 'C' [Sturdevant 5th SA ed 116/ Sturdevant 5th ed 332]
Factors influencing cutting efficiency of bur: Enamel hatchet blade is heavier, larger and beveled on only
one side. It has cutting edges in a plane parallel to the long
i) Rake angle, clearance angle
axis of the handle.
ii) Neck diameter
iii) Spira l angle and Crosscuts 48. 'C' [Sturdevant 4 th ed 378]
iv) Concentricity and run out
49. 'A' [Sturdevant 5th SA ed 130/ Sturdevant 5th ed 363]
v) Influence of load
vi ) Influence of speed so. 'B' [Vimal Sikri 3 n1 ed 159/ Gopikishna manual 376]
vii) Number of teeth or blades Steele's siqueland is a lso known as self adjusting matrix
clamp.
Head length and t aper angle are primarily descriptive. These
factors do not otherwise affect the performance of the bur. 51. 'A' [Vimal Sikri 3 rd ed 116]

3 7. 'C' [Sturdevant 5th SA ed 152] 52. 'B' [Grossman 11th ed 252]

38. 'A' [Vimal Sikri 3 rd ed 120/ Sturdevant 5th SA ed 126] 53. 'B' [Sturdevant 5th SA ed 111/ Sturdevant 5th ed 328]
Dental ;lut.,e

54. 'B' [Sturdevant 4th ed 330, 341] 66. 'C' [Sturdevant 5th ed 735]
For occlusolingual (Class I with palatal extension)
55. 'B' [Sturdevant 5th SA ed 220/ Sturdevant 5th ed 773] preparations, using only tofflemire does not intimately
adapt to the lingual groove area of the tooth. Bartons
56. 'C' [Sturdevant 5th SA ed 128/ Sturdevant 5th ed 359] matrix of tofflemire is a modification. After adapting
tofflemire, a small strip of stainless steel matrix is cut and
57. 'A' [Sturdevant 5th SA ed 152] inserted between the lingual surface of the tooth and the
band already in place. A piece of wedge should be coated
58. 'B' [Marzouk 1" ed 74] with softened green stick compound and inserted between
tofflemire band and the cut piece of matrix. The compound
59. 'A' [Sturdevant 5th SA ed 112/ Sturdevant 5th ed 328] is compressed gingivally, this adapts the steel strip to the
Balance is accomplished by designing the angles of the lingual surface.
shank so that the cutting edge of the blade lies within the
projected diameter of the handle and nearly coincides with 67. 'D' [Sturdevant 5th ed 356]
the projected axis of the handle. Applying more pressure, more number of blades and using
large size bur decreases the cutting efficiency of bur.
For optimal anti-rotational design, the blade edge must not
be off axis by more than 1 to 2mm. All dental instruments Also refer explanations of Question No.11, 18 and 36
and equipment need t o satisfy this principle of balance.
68. 'C' [Sturdevant 5th ed 331]
60. 'A' [Sturdevant 5th SA ed 138/ Sturdevant 5th ed 466] Instruments that have cutting edge parallel to long axis
The wings are designed to provide extra retraction of the are:
rubber dam from the operating field . A disadvantage of • Ordinary hatchet
the winged retainer is that wings often interfere with the
• Enamel hatchet
placement of matrix bands, band retainers and wedges.
Instruments that have cutting edge at an angle (other
61. ' B' [Sturdevant 5th SA ed 204/ Vimal Sikri 3rd ed 154]
than 90°) to the blade:
Matrix bands have thickness of 0.001 inch to 0.002 inch
range. Bands of 0.00 2 inch are widely used because they
1 • Angle former
can be contoured according to the contour of the tooth to • GMT
be restored.
Also check explanation of Q.No.15
62. ' B' [Sturdevant 5th SA ed 138/ Sturdevant 5th ed 466]
Retainer Application 69. 'D' [Sturdevant 5th ed 329]
Check explanation of 0 . No 4 and Q. No 10
W56 Most molar anchor teeth
W7 Mandibular molar anchor teeth 70. 'C' [Sturdevant 5th ed 464]
W8 Maxillary molar anchor teeth Thicker dam is more effective in retracting tissue and is
more resistant to tearing. It is especially recommended
W4 Most premolar anchor teeth for isolating class V lesions in conjunction with a cervical
W2 Small premolar anchor teeth retains. Generally dark, heavy 6 x 6 inch sheets are
Terminal mandibular molar (distal recommended.
W27
surface preparations) anchor teeth
71. 'B'
63. 'A' [Sturdevant 5th SA ed 138/ Sturdevant 5th ed 466]
72. 'C' [Check explanation of Q.No.4]
64. 'D' [Sturdevant 5th SA ed 141/ Sturdevant 5th ed 469]
When the distance between holes is correct, the dam covers 73. 'B' [Check explanation of Q.No.61]
and slightly retracts the interdental papilla. When the
distance between holes is less, the dam stretches resulting 74. 'D' [Shobha Tandon 2nd ed 318]
in space around teeth and this results in interdental papilla If the retainer (clamp) has a suffix of'/!>: after its number,
protruding from the rubber dam. it is a clamp used for partially erupting teeth. 154 A and
8A Ivory clamps are used for partially erupted permanent
65. 'D' [Sturdevant 5th ed 3 29] molar.
Refer explanation of Q.No.4
75. 'A' [Refer Synopsis]
Blade width of this instrument is 12/10 =1.2 mm
76. 'C' [Sturdevant 5th SA ed 120]
, OPERATIVE DENTISTRY

77. 'B' [Check Explanation of Q.No.6/ Sturdevant 3rd ed


784)
No.212 retainer is very much useful for gingival retraction
in Class V cavity. Even though the 'key' was given as 213,
the correct answer according to multiple textbooks is 212.
Dental ;lut.,e

8. MISCELLANEOUS

1. Which of the following should not be used to polish (or) 12. Porosity in porcelain at condensation stage depends on:
finish amalgam a) Shape and size of particle
a) A ball burnisher b) A rubber (Burlew) disk b) Uniform distribution of particle size
c) Finishing bur d) A white stone c) Number of particles d) None of the above
(KAR-02) (AIPG -2K)
2. Final polishing of dental amalgam to get a smooth 13. Condensation shrinkage of porcelain during firing
surface is achieved by depends on:
a) Burnishing b) Use of tin oxide a) Rate of arriving at firing tern perature
c) Carving with finer instruments b) Uniformity of particle size
d) Use of silicon carbide c) Shape and size of particle
(KAR-98) d) Type of investment used
3. Which of the following polishing agent is called (AIIMS -98, AIPG -99)
whitening agent? 14. Phenomenon in which porcelain appears different under
a) Precipitated chalk b) Iron Oxide varying light condition is:
c) Chromium oxide d) Rouge a) Translucency b) Refractive optics
(AIPG-96) c) Metamerism d) 0 pacification
4. Aluminium oxide polishing powder is obtained from (COMEDK -04, AIPG -99)
a) aluminium b) Cryolite 15. Machineable glass ceramic is:
c) Bauxite d) Vulcanite a) Cerestore b) Decor MGC
(AP-03) c) Infusium d) Leucite
s. Which of the following is used for polishing gold restorations (AP -01)
a) Rouge b) Tin Oxide 16. Which is not present in porcelain:
c) Chromium Oxide d) Aluminium Oxide a) Silica b) Feldspar
(AP-98) c) Calcium carbonate d) Amorphous material
6. Hardness of which of the following abrasives is maximum (AIIMS -96)
a) Sand b) Emery 17. Which of the following is the main constituent of porcelain:
c) Boron Carbide d) Silicon carbide a) Feldspar b) Kaolin
c) Quartz d) Clay
7. Polishing of chrome-cobalt castings is done by: (AIPG -96)
a) Electrolytic polisliing b) Electro plating 18. The best tissue tolerated material for crown and bridge:
c) Sand blasting d) Precipitated chalk a) Highly polished porcelain
(AIPG-96) b) Highly polished acrylic
8. Plaque micro-organisms are acidophilic acid and c) Highly glazed porcelain d) Highly polished metal
aciduric. What's the meaning of acidophilic? (KAR -97)
a) Produce acid b) Does not produce acid 19. Porcelain bonded to metal is strongest when it is
c) Can survive in acidic environment a) Air fired b) fired under compression
d) Cannot survive in acidic environment c) Tern pered after firing
(AIPG-2012) d) fired several times before completion
9. Porcelain denture teeth: (AIIMS-2K)
a) Have a higher coefficient of thermal expansion than 20. Glass infiltrated alumina core ceramic is:
acrylic teeth a) Dicor b) lnceram
b) Have a lower abrasion resistance than enamel c) Captek d) !PS-empress
c) Should be used where the inter dental clearance is small
d) Have a higher abrasion resistance than gold 21. The union of metal of porcelain fused restorations is by
(MAN -94) a) Mechanically rough surface
10. The opacity in ceramics is achieved by adding: b) Tin and indium oxides
a) Boric oxide b) Copper oxide c) Nickel and porcelain d) Cobalt and porcelain
c) Silica d) Titanium oxide (AIIMS-01)
(MAN -01) 22. The chemical used for etching porcelain is:
11, To prevent porosity in dental porcelain it should be a) 37% H3 Po4 b) H2 So4
baked: c) Tartaric acid d) HF
a) In presence of air b) In vacuum (KAR-02)
c) For long period d) Under pressure 23. Best cement for cementation of porcelain laminate is
(MAN-97, AIIMS 93) a) Dual cure resin b) GIC

1) B 2) B 3) C 4) C 5) A 6) C 7) A 8) C 9) D 10) D 11) B 12) A 13) C


14) C 15) B 16) D 17) A 18) C 19) B 20) B 21) B 22) D 23) A
, OPERATIVE DENTISTRY

c) Composite d) Polycarboxylate 34. Which of the following is not true about lubricant for
(PGI-94) rubber dam placement?
24. Crack tooth syndrome is diagnosed by: a) Should have water-miscble vehicle
a) Transillumination b) Dyes b) Vaseline is an ideal choice
c) Applying pressure on the tooth c) Should be easy to remove
d) None of the above d) Shoule not interfere with bonding procedure
(KAR -98) (AI-05)
25. When all diagnostic procedures fail to detect proximal 35. For root canal therapy of maxillary canine (Distal
caries, last resort is: caries), the isolation done is:
a) Mechanical separation b) Preparation of the test cavity a) 2nd premolar to opposite lateral incisor
c) Transillumination d) Caries activity tests b) 1st molar to opposite lataeral incisor
(AIIMS-03) c) Adjacent 2 teeth of bot h the sides
26. Proximal caries can be detected best with: d) Not required, only the tooth to be treated is isolated
a) Intra-oral periapical radiographs (AIPG-06)
b) Bitewing radiographs 36. One of the following is not true about treatment of
c) Occlusal radiographs d) Orthopantomographs dentinal hypersens;tivity with dentin bonding agents:
(PGI-99) a) It dentures proyein b) It froms a hybried layer
27. The most common complication of bleaching non vital c) It does not alter dentin permeability
tooth is d) It forms resin tags into dentinal tubule
a) Apical periodontitis b) Cervical resorption (AI-05)
c) Discoloration d) Root resorption 3 7. All of the following are true about dental esthetic
(AIPG-96) correction except:
28. Which of the following drug is commonly used to control a) restoration of all anterior teeth in one appointment will
salivation in operative dentistry enhance the dental esthetics
a) Pilocarpine b) Propanolal b) The indivedual tooth corriction compare to correction of
c) Atropine d) Muscarine all toorh at the same time is more easy
{COMED-06) c) Apparent length size of the tooth can be increased by
29. A diagnosis of small occlusal cavities is most readily made by: change (or alteration) in height of contour of tooth
a) Bite-wing radiographs b) Periapical radiograr d) Apparent length of the can be increade by placing
c) Transillumination groove in the tooth.
d) An explorer and compressed air (AIIMS-06)
(AIPG-06) 38. Affected dentin differs from infected dentin in that, it has
30. Pain in cracked tooth syndrome is a) Been invaded by organisms
a) Continuos in nature b) They both are precisely same
b) Increases on lying down posture c) Not been invaded by microorganisms
c) Elicited main ly when pressure is applied d) Not been demineralized
d) Elicited mainly when applied pressure is relieved (KCET-08)
39. Feldspathic porcelain
31. The distance between 2 holes in a rubber dam sheet is a) Shows reliable chemical bonding with metals
ideally: b) Has high tensile strength
a) 6 mm b) 6.3 mm c) Is no longer used
c) 6.5 mm d) 6.9 mm d) Shows insignificant firing shrinkage
(AIPG-06) (COMEDK-08)
32. Incipient or recurrent caries can be detected before 40. Digital imaging fiber - optic trnns-illumination (DIFOTI)
they are visible on the radiograph by: a) Is used to detect dental caries
a) Visible light b) Ultrasonic light b) Is used to detect enamel fracture
c) Fibreoptic transillumination c) Is used to detect plaque
d) Digital Imaging fibreoptic transillumination d) All of the above
(AIPG-06) (KCET-08)
33. Regarding root caries all of the following are true 41. Cone waxing technique is indicated
EXCEPT: a) For preparing the single pattern in a badly broken down
a) Caused by actinomycosis viscosis. tooth
b) Secondary to gingival recession. b) For multiple patterns
c) Remineralization is difficult. c) For waxing the form of occlusal surfaces for accurate
d) Slow progress articulation of carving casting
(AIIMS-06) d) All of the above
(BHU-07)

24) C 25) A 26) B 27) B 28) C 29) 0 30) 0 31) B 32) 0 33) 0 34) B 35) 0 36) C
37) B 38) C 39) A 40) A 41) D
Dental ;lut.,e

42. Bases provide c) aid in the retention of the restoration


a) No thermal insulation d) give added strength to finished restoration
b) Mechanical support for the restoration (AP-09)
c) Expansion space for cast restoration 53. Most common abrasive used for air abrasion is
d) Exothermic heat for increased pulpal blood circulation a) Carbide particles
(KCET-08) b) Alumina particles of 20 micrometer size
43. All the following are true for non active caries, except: c) Alumina particles of 50 micrometer size
a) Pain on excavation b) Soft & cheesy d) Silicon dioxide particles
c) Dark & hard (AIIMS-09)
d) Presence of dark pigmentation 54. What is a ferrule?
(AIIMS-07) a) Pin retained restorative preparation
44. Gallium containing alloys have: b) Secondary retentive feature for amalgam restoration
a) Silver b) Tin c) Band encircling external dimension of tooth
c) Indium d) Zinc d) Preparation design for composite inlay
(MCET-07) (KCET-10)
45. Ceramic restorations are cemented using: 55. DIFOTI helps in diagnosing
a) GI( cement b) Resin cement a) Bone loss in children
c) Zinc phosphate cement d) Zinc oxide eugenol cement b) Calcium content of enamel
(MCET-07) c) Occlusal caries d) Proximal caries
46. Similarity in the chemical structure of estrogen is seen with (AIIMS-08)
a) HEMA b) Polyether 56. Air Abrasive Restorations are contraindicated in all
c) Bisphenol A d) Gutta percha except?
(COMEDK-08) a) Chronic pulmonary Disease
47. One of the purposes of etching beveled enamel surface b) Recent Extraction
is to c) Subgingival caries d) Deep Caries
a) increase surface energy b) produce macro under cut (PGI-08)
c) to increase convenience d) to prevent caries 57. Skipping effect is seen in
(KCET-08) a) use of liquid etchant b) use of viscous etchant
48. Transformation toughening is found in ceramics c) use of GIC as pit and fissure sealant
containing d) use of composite as pit and fissure sealant
a) Octagonal zirconia b) Tetragonal zirconia (COM EDK-10)
c) Decahedron zirconia d) All of the above 58. Root caries is attributed to action of
(BHU-2012) a) streptococcus mutans b) lactobacillus
49. The most common cause of failure of tooth colored c) actinomyces viscosus J naeslundii
inlays/ onlays is d) (1) and (3)
a) Discoloration b) Debonding (AP-09)
c) Bulk fracture d) Creep 59. Indication for indirect pulp therapy
(BHU-2012) a) No signs or symptoms of pulp degeneration
50. A patient is having very extensive carious lesion, the b) No response to hot and cold
radiograph of lesion shows a probable exposure of the c) Presence of pain on percussion
pulp horn if excavated, what will be the treatment plan? d) Presence of pulp polyp
a) Proceed with endodontic treatment (KCET-10)
b) Excavate the gros.s carious lesion and place appropriate 60. Macroabrasion is
sedative dressing a) The removal of localized superficial white spots and
c) Remove all carious lesion, place sedative dressing and other surface stains or defect s
plan pulpotomy in next session b) Treating deep discolouration in type IV Tetracycline
d) Remove all carious tooth structure and cap the exposure discolouration
(AIPG-09) c) Treating discoloured endodontically treated teeth
51. Affected dentin is to be left in cavity preparation d) Treating any discolouration in anterior teeth
because it - (COM EDK-10)
a) has capacity to remineralize 61. If the rate of loading is decreased, the mechanical
b) Hard, difficult to remove properties of material
c) Impermeable, so protects pulp a) Increases b) Decreases
d) pain on excavation. c) Remains unchanged d) Becomes unpredictable
(MCET-10) (AIIMS-09)
52. Contact areas are always carefully restored in order to 62. The odontoblast which is lost is replaced from primitive
a) protect the gingival tissue mesenchymal cells in:
b) prevent recurrence of decay a) One week b) 15 days

42) B 43) B 44) C 45) B 46) C 47) A 48) B 49) C 50) B 51) A 52) A 53) B 54) C
55) D>C 56) D 57) B 58) D 59) A 60) A 61) B 62) B
, OPERATIVE DENTISTRY

c) 4-6 weeks d) One month d) HIV has been transmitted by blood contaminated fluids
(AIPG-10) that have been heavily spattered or splashed.
63. Which is moisture active adhesive? (KCET-2011)
a) Cyanoacrylate b) Methyl methacrylate 74. Which of the following item in the dental operatory
c) Bis-GMA d) All of the above cannot be classified as a semi-critical item?
(AIPG-10) a) Suction tip b) Hand piece
64. Dental caries is associated with c) Endodontic file d) Water syringe tip
a) S. mitis b) S. rattus (KCET-2011)
c) S. Salivarius d) Staphylococcus 75. "Right rear" operator positio111 refers to
(AIIMS-09) a) 1 O'Clock position b) 5 O'Clock position
65. Sodium hypochlorite in carisolv is used in concentration of c) 7 O'Clock position d) 11 O'Clock position
a) 2% w/v b) 0.5% w/v (KCET-2011)
c) 1.5% w/v d) 5% w/v 76. The gingival cavo surface bevel for a class II preparation
(AIPG-10) to receive a cast gold inlay
66. True about S. mutans are all except a) Results in a 30° metal that is burnishable
a) It is both acidogenic & aciduric b) Results in no benefits whatsoever
b) Requires carbohydrates c) Results in open margin if casting procedures are not
c) Multiply rapidly accurate
d) Requires specific growth conditions d) Results in difficulty in taking wax patterns
(AIPG-10) (COM EDK-10)
67. Which of the following is not used for sterilization of 77. Of all the bevels placed on gold inlay preparation, which
handpiece? is the most important bevel for success of Restoration?
a) ethylene oxide b) chemical vapour a) Occlusal b) Gingival
c) autoclave d) dry heat c) Axiopulpal d) Faciopulpal
(AIPG-10, AIIMS MAY- 2012) (AIPG-09)
68. With slowly advancing enamel caries and rapid 78. Not correct for Nd:Yag lasers:
involvement of dentin, the type of dentin formed is a) Useful for periodontal surgery
a) Secondary b) Tertiary b) Deliver energy in free running pulse
c) Reparative d) Sclerotic c) Absorbed by pigmentation
(AIIMS-09) d) Wavelength is 10600 nm
69. Minimum depth of demineralisation of caries lesion to (AIIMS MAY 2012)
be detected radiographically is 79. A ceramic inlay can be used instead of amalgam in
a) 100 µm b) 200 µm patients with
c) 400 µm d) 500 µm a) High caries rate b) Class I restoration
(AIIMS-09) c) It requires extensive preparation
70. Greatest portion of tooth is formed by d) None of the above
a) Enamel b) Dentin (KAR-2013)
c) Pulp d) Cementum 80. The danger of transmitting infections to the dental
(AIIMS-09) team or to the other patient particularly has a threat of:
71. Zone of carious dentin with demineralization of a) Acquired immune deficiency syndrome (AIDS)
intertubular dentin and formation of fine crystals in b) Hepatitis-B
tubule lumen. c) Tuberculosis d) All of the above
a) Subtransparent dentin (COMEDK -2013)
b) Transparent dentin 81. Abrasive used in air polishing agent is:
c) Affected dentin (diseased) a) NaCl b) 27µ alumina particles
d) Normal dentin c) 50µ alumina particles d) Sodium bicarbonate
(AIIMS-09) (AIIMS MAY 2012)
72. Nd: YAG laser has a wavelength of 82. Microfracure occurring in the cervical area of a tooth
a) 10.60 µm b) 1.06 µm under flexural load is
c) o.60 µm d) 300 µm a) abrasion b) attrition
(AIIMS-09) c) abfraction d) erosion
73. One of the following statements regarding HIV is NOT true. (AP-2013, KERALA-2015)
a) When used properly, all disinfectants except some 83. Caries originating on the root is difficult to manage
quaternary ammonium compounds are said to inactivate because
HIV in Less than 2 minutes. a) It has a comparatively rapid progression
b) HIV is killed by all methods of sterilization. b) It is often asymptomatic
c) Aerosols produced during dental treatments have been c) It is closer to the pulp d) All of the above
found to transmit HIV infection. (KAR-2013)

63) A 64) B 65) B 66) D 67) D 68) D 69) D 70) B 71) A 72) B 73) C 74) C 75) D
76) A 77) B 78) D 79) C 80) D 81) D 82) C 83) D
Dental ;lut.,e

84. To have a better ergonomics in dentistry, most operative 94. Metallic taste after application of stannous fluoride
procedures are completed from, at, or, near, the varnish is due to ?
a) 4 O'clock position b) 12 O'clock position a) Stannous Triflourophosphate
c) 7 O' clock position d) 5 O' clock position b) Stannic Triflourophosphate
(KAR-2013) c) Calcium triflourophosphate
85. Dentinal sensitivity is attributed for d) Tin Hydroxyphosphate
a) Neural stimulation of dentinal tubules (AIPG-14)
b) Craze lines in dentin 95. What is wavelength of Nd: YAG laser ?
c) Dentinoenamel junction a) 2.94 microns b) 1.06 microns
d) Pain transmission through movements of fluid in c) 10.6 microns d) 2.74 microns
dentinal t ubules (AIPG-14)
(COMEDK-14) 96. Maximum demineralization of enamel occurs at a depth
86. Chairside CAD/CAM restoration requires of?
a) Elastomeric impression a) 10-20 µm b) 20-30 µm
b) Dual impression c) 30-50 µm d) 100-200µm
c) Optical impression d) Pindex impression (PGI JUNE-2011)
(AP-14) 97. In detection of dental caries using 'diagnodent', the
87. Dentoalveolar ablations are caused by instrument readings higher than suggest
a) Decalcification by acid beverages the presence of caries
b) Mechanical wear a) 40-45 b) 60-65
c) Forceful frictional actions between oral soft tissues and c) 30-35 d) 20-25
adjacent hard tissues (KERALA-2015)
d) Occlusal wear from functional contacts of opposing 98. The reaction of dentin to moderate intensity attack is
teeth a) Sclerotic dentin b) Reparative dentin
(COMEDK-14) c) Translucent dentin d) Dentin hypersensitivity
88. Incipient caries consists of opaque chalky white areas (KERALA-2015)
that appear when the tooth surface is dried, this is 99. Which of the following laser has wavelength in visible
referred as light spectrum?
a) White spot b) Hot spot a) XeF b) Nd:YAG
c) Translucent zone d) Body of lesion c) Argon d) Co 2
(COMEDK-14) (PGI DEC-2011)
89. Dental porcelain cannot be used for 100. The primary goals of caries prevention are all EXCEPT
a) Making artificial teeth a) Limiting pathogen growth and altering metabolism
b) For fabrication of jacket crowns and inlays b) Increasing resistance oftooth surface to demineralization
c) For filling decayed tooth c) Increasing biofilm pH
d) As a veneer over cast metal crown d) Use of probiotics
(AP-14) (KERALA-2015)
90. If the caries cone in enamel is larger or atleast the same
size as that in dentin then
a) Backward caries b) Forward caries
c) Reversible caries d) Slow caries
(AP-14)
91. The acid used in etching ceramic during repair of ceramic
restorations is
a) Hydrochloric acid b) Hydrofluoric acid
c) Sulphuric acid d) Ortho phosphoric acid
(EMCET-14)
92. Dye used for enamel caries detection
a) Methylene blue b) Procion
c) Acid red d) Basic fuschin
(GCET-14)
93. The dentin desensitizing agents that acts by precipitating
proteins in dentinal tubular fluid is
a) Strontium Chloride b) Potassium oxalate
c) Fluoride d) Hydroxyethyl methacrylate
(MCET-14)

84) B 85) D 86) C 87) C 88) A 89) C 90) B 91) B 92) B 93) A 94) D 95) B 96) A
97) D 98) B 99) C 100) D
, OPERATIVE DENTISTRY

8. MISCELLANEOUS - ANSWERS

1. ' B' [Phillips 11th ed 372] Zirconium oxide, titanium oxide Opaque
Final finishing and polishing of amalgam restorations
should be delayed for at least 24 hrs. Polishing disks and Copper oxide Green
dry polishing powders will raise the temperature of surface Iron oxide Brown
above 60°C which cause irreversible pulp damage. Always a Titanium oxide Yellow
wet abrasive powder in paste form is used.
11. 'B' [Vimal Sikri 3n1 ed 441]
2. 'B' [Vimal Sikri 3 n1 ed 482]
Restoration Polishing agent Different media employed for firing are
Amalgam Tin oxide/ Zinc oxide/ Chalk Inclusion of air results in voids and porosity
Air firing
Composite Aluminium oxide in the fired porcelain
Noble metal alloys Iron oxide (Rouge) Vacuum Vacuum (760 torr) removes air from the
Non precious alloys firing interstitial spaces. Porosity is prevented.
Electrolytic polishing
(Cobalt-Chrome) Gases like helium, hydrogen or steam
Natural teeth Zirconium silicate lS substit uted for the ordinary furnace
Diffusible
atmosphere. When porcelain is self-glazed
Stainless steel Chromium oxide gas
by reheating, it is the best method of
Dentures/ Artificial teeth Pumice reducing voids.

3. 'C' (Check Explanation Below] 12. 'A' [Vimal Sikri 3,d ed 440]
Chromium oxide is used as a polishing agent for stainless The aim of condensation is to pack the particles as close
steel. Iron oxide (or) Rouge is a red abrasive. as possible in order to reduce the amount of porosity and
shrinkage during firing. If only one sized particles are used
4. 'C' [Phillips 11'h ed 370] 45% of void spaces occur. If two sized particles are used
void spaces reduce to 25%. System that uses three sizes of
5. 'A' [Phillips 11th ed 370] powder is known as 'Gap grading system'. Round particles
produce better packing compared with angular particles.
6. 'C' (Check Explanation Below]
Diamond is the ha, dest substance. It is called as super 13. 'C' [Vimal Sikri 3n1 ed 440]
abrasive (7,000 - 10,000 KHN), followed by Boron carbide Firing shrinkage occurs due to loss of water and densification
and silicon carbide (KCET-08}. through sintering. It is about 40% by volume. This can
be reduced by proper condensation and firing technique.
7. 'A' (Vimal Sikri 3,d ed 491] The higher the firing temperature the greater is the firing
The two factors which make polishing difficult for cobalt shrinkage.
chrome alloys are its high hardness and the coarse nature of
the investment powder. The investment is removed from the 14. 'C' [Phillips 11t h ed 712]
casting by sandblasting. Electrolytic polishing is commonly
used as a part of the finishing process for chrome cobalt 15. 'B' [Vimal Sikri 3n1 ed 449]
alloys as it reduces t ime and effort. Electrolytic polishing is
also called as revers e sand blasting. Castable glass ceramic Dieor
8. 'C' (Sturdevant 5th ed 69] Machinable ceramic Dicor MGC
Acidogenic bacteria produce great amounts of acids. Castable apatite ceramic Cera Pearl
Aciduric bacteria are the bacteria that are tolerant of acidic
Inceram (A~0 3),
environments. Glass infiltrated ceramic
Inceram spinal (Mg Al20.)
9. ' D' [Phillips 11th ed 711] Injection molded ceramic IPS Empress, Optec OPC
Porcelain teeth have low co-efficient of thermal expansion Shrink free ceramic Cerastore
and higher abrasion resistance. They should be used only Aluminium porcelain Hi Ceramic
when there is sufficient over jet and overbite.

10. ' D' [Vimal Sikri 3,d ed 43 7]


Pigmented oxides are added to glass to simulate shades of
natural teeth.
Dental ;lut.,e

16. 'D' [Vimal Sikri 3"' ed 434) 26. 'B' [Sturdevant ! th~ 105)
Porcelain is a vitreous ceramic. Its composition is: Bite wing radiographs are most effective means of evaluation
Component Composition Functions of smooth surface caries on posterior teeth. When caries
has invaded proximal surface and has demineralized the
Flux, matrix, dentine, white chalky appearance or shadow under marginal
Feldspar
60 - 80% surface glaze, ridge is evident.
(K20 ALA SiO)
forms leucite
Kaolin 3 - 5% Binder 27. 'B' [Sturdevant 4th ed 608)
Quartz 15 - 25% Filler Cervical resorption is the most common complication of
non vital bleaching and apical periodontitis is the most
Alumina 8 - 20% Glass former common application of bleaching vital tooth.
Boric oxide 2-7% Glass former, flux
Metallic pigments 1% Colouring 28. 'C' [Sturdevant 5th SA ed 158/ Sturdevant 5th ed 491]
Control of salivation in restorative dentistry is mainly
17. 'A' [Vimal Sikri 3rd ed 435) limited to atropine. Atropine is contraindicated in nursing
Feldspar is the primary constituent. Porcelains based on mothers and patients with glaucoma.
Feldspar are called as 'Feldspathic porcelains'. Feldspar
when melted forms a crystalline phase called 'Leucite' and 29. 'D' [Vimal Sikri 3"' ed 94)
a glass phase that will soften and flow. Leucite is the basic
glass former and has high coefficient of thermal expansion. 30. 'D' [Marzouk 1st ed 438)

18. 'C' [Vimal Sikri 3"' ed 438) 31. 'B' [Sturdevant 5th ed 469)
Glazing produces the natural enamel look to porcelain. It is of The distance between holes is approximately 1/41h inch
two types - self and add-on glazing. Prolonged glazing could (6.3mm).
result in "pyroplastic flow' of the material with rounding off
the line angles and loss of surface characteristics. If the distance between 2 holes is excessive, the dam
wrinkles between the teeth. If the distance is less, causes
19. 'B' [Phillips 11th ed 672] dam to stretch.

20. 'B' [Vimal Sikri 3rd ed 446) 32. 'D' [Sturdevant 5th ed 419)

21. 'B' [Phillips tOth ed 606] 33. 'D' [Sturdevant 5th SA ed 38/ Sturdevant 5th ed 289]
Bonding of porcelain to metal occurs by: Root caries is usually more rapid than other forms of caries
and should be detected and treated early.
a) Chemical bonding:- It is the primary bonding
mechanism. Presence of an adherent oxide layer is 34. 'B' [Sturdevant 5th SA ed 139/ Sturdevant 5th ed 467)
essential for good bond formation. Tin oxide and The lubricant should be water-soluble which faci litates
Indium oxide are responsible for chemical bond. In the passing of the dam septa through the proximal contacts.
base metals chromium oxide does this role.
b) Mechanical bonding:- Presence of surface roughness As Vaseline is oil based, it is not satisfactory.
on the metal helps in providing the bond.
35. 'D' [Sturdevant 5th SA ed 141/ Vimal Sikri 3"' ed 167]
22. 'D' [Sturdevant 4th ed 624] Isolate atleast t hree teeth at a time for operative procedures.
Cold HF acid is also useful for cleaning gold palladium alloys.
If root canal treatment has to be done, single tooth
23. 'A' [Phillips 10th ed 283) isolation of that tooth is enough.

24. ' C' [Marzouk 1st ed 438) 36. 'C' [Sturdevant 5th SA ed 336,337)
Cracked tooth syndrome is incomplete fracture of vital
posterior tooth. The fractu res generally result from improper 37. 'B' [Sturdevant 5th SA ed 304)
condensation techniques. This is best diagnosed "Rubber
Wheel Test". Sharp pain on release of biting pressure 38. 'C' [Sturdevant 5th SA ed 47/ Sturdevant 5th ed 291)
(COMEDK-05) helps to identify offended tooth. During Fusayama reported that carious dentin consists of two
release of pressure, rubbing of fractured dentinal surfaces distinct layers an outer and an inner layer.
creates hydrodynamic pressures in the dentinal tubules
and evoke pulpal pain. Radiological findings are absent. The outer layer is referred as infected dentin and inner layer as
Extra coronal cast restoration is the treatment of choice. affected dentin. Infected dentin has bacteria and the collagen
Transillumination detects the cracks in the tooth. is irreversibly denatured. It should be removed during tooth
preparation. Affected dentin has no bacteria, is reversibly
25. 'A' [Vimal Sikri 3rd ed 152] denatured, it is remineralisable and should be preserved.
, OPERATIVE DENTISTRY

These can be distinguished clinically by the discoloration Examples of some crystals which can be used in
and feel of an explorer. 1% propylene glycol stains the transformation toughening of ceramics:
infected dentin. • Alumina (procera all ceram, Inceram alumin
• Leucite (optec HSP, !PS em press, 0 PC)
39. 'A' [Philips 11th ed 660]
• Tet rasilicic floromica (Dicor, Dicor MGC)
40. 'A' [Sturdevant 5th ed 419] • Magnesia alumina spinal (Inceram spinal)

41. ' D' 49. 'C' [Sturdevant 5th SA ed 279/ Sturdevant 5th ed 605]

42. ' B' [Philips 11th ed 459, 460] 50. 'B' (Sturdevant 5th SA ed 412/ Sturdevant 5th ed 128]
Base provides protection against thermal, chemical,
galvanic insults. It provides mechanical support for the 51. 'A' [Sturdevant 5th SA ed 47/ Sturdevant 5th ed 100]
restoration.
52. 'A' [Sturdevant 5th SA ed 203/ Sturdevant 5th ed 63 2]
43. 'B' [Sturdevant 5th SA ed 40/ Sturdevant 5th ed 290)
Non active or arrested caries has the following features: 53. 'B' (Vimal Sikri 3rd ed 428]
The slow rate results from periods when demineralized Air abrasion involves the process of effective removal of
tooth structure is almost remineralized. both hard and soft decay using micro air abrasion dentistry.
Its slow rate allows time for extrinsic pigmentation.
The abrasive powder comprises 95% alumina. Over 50% of
Arrested enamel lesion is brown to black, hard and the abrasive has particle sizes ranging from 15-45µ with
is more caries resistant than contiguous unaffected an average particle size of 275µ, propelled through narrow
enamel. 0.4mm orifices at pressures from 40 - 160 psi.
It is typically open, dark and hard and is termed as
sclerotic or eburnated dentin. 54. 'C' [Ingle 5th ed 926]
Ferrule is a circumferential band of metal that engages
44. 'C' [Vimal Sikri 3"' ed 234] tooth structure and prevents tooth fracture in a post
Gallium alloys contain silver and tin as the solid alloy and endodontic restoration .
liquid contains Gallium-Indium mixture.
55. 'D'>'C' (Sturdevant 5th ed 419]
45. ' B' (Sturdevant 5th ed 616] Digital imaging fibre optic transillumination (Difoti) is
Resin cements are reserved almost inclusively for the use a novel method for early detection of carious surfaces.
with all ceramic restorations. The principle behind transilluminating teeth is that
dimineralized areas of enamel or dentin scatter light more
46. 'C' [Philips 11th ed 199] than sound areas. The advantages of DIFOTI are absence of
Estrogenicity is the ability of a chemical to act as the ionizing radiation, lack of need for film, real time diagnosis
hormone estrogen does in t he body. If these chemicals and higher sensitivity.
are not indigenous to the body, the substance is called a
"Xenoestrogen". Eg: BIS-GMA 56. 'D' [www.daneng.com]

E-Screen Assay is used commonly to assess the 57. 'B' [MC Donald 8th ed 357]
xenoestrogenic activity. The compound in question is Occasionally viscous gel etchant may show a skipping
applied to breast cancer cells that are estrogen sensitive effect, which occurs when the etchant does not completely
and cell growth is measured over a period of 24 - 72 hrs. and uniformly wet the entire enamel surface, and unetched
The compound is deemed estrogenic if the growth rate of areas are evident after washing and drying. if this occurs
cells exceeds that of control without the chemical. etching is necessary

47. 'A' [Marzouk l't ed 181/ Vimal Sikri 3"' ed 338] 58. 'D' [Sturdevant 5th SA ed 29/ Sturdevant 5th ed 76]

48. 'B' [Philips 11th ed 703] 59. 'A' [Ingle 5th ed 866]
When small tough crystals are distributed in glass, the
ceramic is toughened and strengthened because cracks 60. 'A' (Sturdevant 5th SA ed 321/ Sturdevant 5th ed 647]
cannot penetrate crystals as easily as they can penetrate Macro-abrasion
the glass. Technique for the removal of localized, superficial white
spots and other surface stains or defects. It uses a 12-fluted
Dental ceramics based on zirconia crystals undergo composite finishing bur or a fine grit diamond in a high
transformation toughening through transformation of speed hand piece. After the removal of defect, a 30-fluted
zirconia from tetragonal crystal phase to a monoclinic composite finishing bur is used to remove any facets or
phase which makes them extremely strong and tough. striations created by the previous instruments.
Dental ;lut.,e

Micro-abrasion ETOX (ethylene oxide) gas is the greatest method of


Surface dissolution of enamel by the acid and abrasives sterilization used for hand pieces.
to remove stains or defects. Firstly CROLL used 18% HCl +
Pumice, later he modified using 11% HCl + Silicon carbide. Dry heat sterilization of hand pieces is generally not
recommended.
61. 'B' [Sturdevant 5th ed 146]
Mechanical events are both temperature and time 68. 'D' [Sturdevant 5th SA ed 10/ Sturdevant 5th ed Pg 25 -27
dependant. As the temperature increases, the mechanical
property values decrease. The stress strain curve appears 69. 'D' [Ref. Sturdevant 5th ed Pg 104)
to move to the right and downward. The opposite occurs Although dental radiographs may show caries that is not
during cooling. As the rate of loading decreases, the visible clinically, the minimal depth of a detectable lesion
mechanical properties decrease. This is called 'Strain rate on a radiograph is about 5001,1m.
sensitivity' and has important clinical implications. To
momentarily make a material's behaviour stiffer or elastic, Although radiographs tend to underestimate the histologic
strain it quickly. extent of a carious lesion, approximat ely 60% of teeth
with radiographic proximal lesion in the outer half of
Eg.: Remove elastic impressions rapidly so that it will be dentin are likely to be non-cavitated. Thus, many lesions
more elastic and more accurately records the absolute evident radiographically are non cavitated and should be
dimensions. remineralized rather than restored.

62. 'B' [Sturdevant 5th ed 26] 70. 'B' [Sturdevant 5th SA ed 7/ Sturdevant 5th ed 24]
In about 15 days new odontoblasts are differentiated from
mesenchymal cells of the pulp and these odontoblasts lay 71. 'A' (Sturdevant 5th ed 101)
down the reparative dentin.
72. 'B' [Sturdevant 5th ed 344)
63. 'A' [Rosenstiel 3"' ed 433] Type Source Wavelength (nm)
Cyanoacrylates are low viscosity resins that are used as
moisture active adhesives. They are mainly used is as a die Co 2 10600
hardeners (gypsum). Er, Cr: YSGG 2780
Infrared ER:YAG 2940
64. 'B' [Sturdevant 5th e d 69] Nd:YAG 1064
8 streptococcus mutans serotypes have been associated Diode 812, 980
with caries. They have been labeled 'a' to 'h~ He Ne 633
• S. cricetus (serotype a) Visible KTP 532
Argon 514, 488
• S. rattus (serotype b)
• S. ferus (serotype c) Xe F 351
Ultraviolet Xe Cl 308
• S. sobrinus (serotype d,g & h) (Excimer) Kr F 248
Ar F 193
65. 'B'
73. 'C' (Sturdevant 5th ed 3 77)
66. 'D' [Sturdevant 5th ed 69)
Aerosols are invisible particles ranging from 5mm to 50mm
Streptococcus mutans can produce great amount of acids that can remain suspended in air. They carry agents of
(acidogenic), are tolerant of acidic environments (aciduric), respiratory infection borne by the patient. They do not
are vigorously stimulated by sucrose and seem to be the transmit HIV/HBV during dental treatment. Mists are visible
primary organisms associated with caries in humans. droplets of 50mm. Heavy mists tend to settle gradually from
the air after 5 to 15 minutes. Spatter consists of particles
Mutans Streptococci group (MS) are present as a pandemic generally larger than 50mm and are even visible splashes.
infection in humans, that is MS are found in everyone Spatter or splashing of mucosa is considered a potential
regardless of race, ethnic background or geographic origin . route of infection for dental personnel by blood borne
pathogens.
67. 'D' [Sturdevant 5th ed 398]
Autoclave sterilization of hand pieces is one of the most 74. 'C' [Sturdevant 5th ed 385]
rapid methods of sterilization. However, fibre optics dim Critical items:
with repeated heat sterilization in several months to a year,
Includes instruments that contact cut tissues or penetrate
apparently owing to oil residue and debris baked on the tissues. Usually there are disposable or single use items.
ends of the optical fibres. Examples are
Chemical vapour sterilization is recommended for some • Scalpels, blades
types of hand pieces- apparently works well with ceramic - • Endodontic files
bearing hand pieces. • Syringes etc.....
, OPERATIVE DENTISTRY

Semicritical items: 77. 'B' [Sturdevant 4th ed 814, 815]


Items that are handled by gloved hands coated with blood
and saliva or that may touch mucosa. These can be either 78. ' D' [Strudevant's 5th ed 344]
disposable or they can be cleared, sterilized or disinfected. Nd: YAG is an infrared laser used for soft tissue applications
like periodontal surgeries, frenectomy, biopsy and
Examples of semicritical items are: gingivectomy. These lasers operate in both pulsed and
• Air water syringe tip continuous mode. Also see Q.No. 72
• Suction tips
Some important points on lasers:
• Prophy angle
• Pulpal temperature increase of more than 4.5°C to
• Hand pieces 5.5°( are damaging .
• YSGG and Ho: YAG can be used for hard tissue
Non-critical items:
applications. All other lasers are mostly used for soft
These are environmental surfaces such as chairs, benches,
tissue applications.
floors, walls and supporting equipment and are not
ordinarily touched during treatment. These surfaces can be • It is claimed t hat lasers act by hydro kinetic mechanism.
cleaned, disinfected and covered. • At <100°C - there is denaturation proteins, hemolysis,
coagulation and shrinkage
75. ' D' [Sturdevant 5th SA ed 134/ Sturdevant 5th ed 450] >100°C - Water in the tissues boil producing
Positions for a right handed operator: explosive expansion
• 7'0 clock (right front) >400°C - Carbonization of organic materials and
• 9'0 clock (right) onset of some inorganic changes.
• 11'0 clock (right rear) 400-1400°( - inorganic constituents may melt,
recrystallize or vaporize.
• 12'0 clock (direct rear)
79. 'C' [Sturdevant 5th SA ed 27 7/ Sturdevant 5th ed 603]
Right front: Useful for work on:
Indications for indirect (ceramic) inlays over amalgam:
• Mandibular anteriors
• Est hetics
• Maxillary anteriors
• Large defects or previous restorations.
• Mandibular posteriors • Especially those which are wide faciolingually and
require cusp coverage.
Right: useful for work on
• Facial surfaces of maxillary and mandibular right
Contraindication:
posteriors.
• Heavy occlusal forces
• Occlusal surfaces of right mandibular posterior teeth. Eg:- Bruxism, clenching habits
• Inability to maintain a dry field, because these should
Right rear: be cemented with dentin adhesives.
• Position of choice
• Deep subgingival preparations.
• Indirect vision using mouth mirror
• Lingual and incisal surfaces of maxillary teeth 80. ' D' [Sturdevant 5th ed 3 69)
• Direct vision may be used in left mandibular teeth.
81. ' D' [Vimal Sikri 3 n1 ed 428]
Direct rear: Airpolishing should not be confused with air abrasion.
• Lingual surfaces of mandibular anteriors Air abrasion uses greater air pressure and highly abrasive
particles for removing decayed enamel and rough enamel
• Operator is behind the patient.
surfaces, prior to bonding. It uses aluminium oxide particles
with mohs hardness number of 9, which is 4-5 times more
5 O' clock, 3 O' clock and 1 O' clock are positions for a Left-
abrasive than air polishing agents.
handed operator.
Air polishing is accomplished by erosion to remove plaque
Some important points:
and stains. It employs sodium bicarbonate particles of 74
• Always the height of the operating fie ld should be at
µm in size. Also refer Q.No. 53.
the Level of elbow of operator.
• Instruments are exchanged below patients chin and
several inches above the patient's chest.
• The assistant stool should be 4-6 inches higher for
maximal visual access.

76. 'A' [Sturdevant 5th SA ed 441/ Sturdevant 5th ed 859]


Dental ;lut.,e

82. 'C' [Sturdevant 5th SA ed 295/ Sturdevant 5th ed 292] 89. 'C' [Phillips 12th ed 423]
Tooth flexure during abnormal occlusal interaction Applications of dental porcelains:
J, • Crowns and bridges
Lateral or axial bending of the tooth • Artificial porcelain teeth for dentures
J,
• Inlays and onlays
Tensile and compressive stress generated in
the cervical region • Porcelain fused to metal crowns
J, • Endodontic posts
Strain leading to microfractures in cervical • Implant abutments
enamel and tooth loss
J, 90. 'B' [Check Synopsis]
Notch shaped abfraction lesion
91. 'B' [Sturdevant 5th ed 621]
83. 'D' [Sturdevant 5th SA ed 38/ Sturdevant 5th ed 785] Ceramic restorations intra-orally are repaired with
Because of limited visibility, root caries progresses without composites.
the notice of the person. Also because of lack of enamel, it Steps in repairing ceramic restorations:
progresses rapidly and is closer to pulp when compared to • Mechanical roughening of the involved surface with
caries occurring on enamel surfaces. a coarse diamond or airabrasion or grit blasting wit h
aluminium oxide particles.
84. 'B' [Sturdevant 5th SA ed 134/ Sturdevant 5th ed 450]
• 2 min application of 10% HF gel etches the surface and
The most ergonomically comfortable positions in dental creates micro defects to faci litate bonding
practice are 11 o'clock position, 12 o'clock position and
lo' clock positions. • Application of silane coupling agents. Silane mediate
chemical bonding between ceramics and resin
85. 'D' [Sturdevant 5th ed 268] • Application of bonding agent and light curing
The most widely accepted theory for explaining dentinal • Application of Composite
hypersensitivity (DH) is the hydrodynamic therapy proposed
by Branstrom. According to this fluid movement (inward/ 92. ' B' [Operative dentistry by Satheesh Chandra 40]
outward) in dentinal tubules to various stimuli is responsible
for DH. All stimuli except heat cause inward movement of Structure Dye
dentinal fluid thereby stimulating the mechano receprors • Calcein
resulting in DH. Heat cause outward movement of dentinal Enamel • Procion
fluid. • Brilliant blue

86. 'C' [Sturdevant 5th 610] Dentin • Basic fuschin


Chair-side Computer-Aided Design/Computer assisted
• Acid red
manufacturing can fabricate ceramic inlays/onlays from
93. 'A' [Check Explanation Below]
high quality ceramics in minutes. CEREC System is one Oesensitizers act by
such example. Generation of a CEREC restoration begins i) Desensitization of intradental nerves or inhibit
after the dentist prepares the tooth and uses a scanning or prevent transmission of the stimulus itself.
device to collect information about the preparation and its
Eg: Potassium salts like nitrate, chloride and citrate.
relationship wit h the surrounding structures. This step is
termed as optical impression. ii) Occlusion of the dentinal tubules
87. 'C' [Check Explanation Below]
a) Precipitation of proteins:
Dentoalveolar / frictional ablation is a process caused by
juxtaposition of natural and artificial dental surfaces and • Silver nitrate
hyper functional oral soft tissues. It is caused by the action • Zinc chloride
of soft tissues and saliva against the dentition due to • Strontium chloride
vestibular pressures of suction, swallowing, tongue motions
and the intervening forced flow of saliva. b) Plugging of dentinal tubules
• Salts of fluorides and oxalates
88. 'A' [Sturdevant 5th ed 639]
Decalcified or dysmineralization, opaque and chalky white • Bioglass
areas on enamel are called white spots. Poor oral hygiene
after orthodontic t reatment and incipient caries are the c) Dentin adhesive sealers
frequent reasons of white spots. The surface of the enamel • Varnish
is not soft and remineralization is possible. • GIC
• Composites
• Dentin bonding agents
, OPERATIVE DENTISTRY

iii) Lasers:
• Nd-YAG-act by occlusion of tubules
• GaAlA (Gallium Aluminium Arseride laser act by
affecting the neural transmission)

94. ' D' [Sturdevant 5th ed 113)


Stannous fluoride varnish contains tin. Tin helps in
arresting root caries but also because staining and a bitter
metallic taste. Tin hydroxyphosphate gets dissolved in the
oral fluids and gives metallic taste when stannous fluoride
is applied.

95. ' B' [Studevant 5th ed 344]


Wavelength of Nd:YAG Laser is 1064 nm (or) 1.064 microns
(or) 106.4 Angstrom unit.

Note: 1 nanometer = 10 Angstrom

96. 'A' [Mc Donald 8th ed 206]


The acids involved in the initiation of the caries process
are normal metabolic by product of microorganisms and
generated by the metabolism of Carbohydrate. The outer
surface of enamel is more resist ant to demineralization by
acid than in the deeper portion of enamel due to the fluoride
content. So the greatest amount of demineralization occurs
10-15 µm beneath the surface enamel. This process results
in the formation of an incipient subsurface lesion, clinically
called by the name white spot.

97. ' D' [Modern dental assisting by Bird 10th ed 184]


Correlation of diagnodent values with possible
course of action:
• No caries
0-15 • No action is required. Preventive therapy
should be initiated
• Histological enamel caries
15-20 • No action is required but a sealant application
might be beneficial
• Histological dentinal caries.
20-25 • Action is required.
• Preventive and operative care advised
• Histological dentinal caries
25-99
• Mandate preventive and operative care.

98. ' B' [Sturdevant 5th SA ed 45]


Reaction of dentin in a slowly advancing lesion or under an
old restoration is by formation of sclerotic dentin. Reaction
of dentin to moderate intensity irritants is by formation of
reparative dentin.

99. 'C' [Check Explanation of Q.No. 72]

100. ' D' [Sturdevant 6th ed 70]


Dental ;lut.,e

OPERATIVE DENTISTRY - SYNOPSIS

FUNDAMENTALS IN CAVITY PREPARATION ii) Pulpal wall: An internal wall that is both perpendicular
to the long axis of the tooth and occlusal of the pulp.
1. Features in conservative cavity preparation are: iii) Floor or seat: Prepared cavity wall that is flat and
• Minimal extension of cavity walls. perpendicular to the occlusal forces that are directed
occlusogingivally. Eg: Pulpal and gingival walls.
• Rounded internal line angles.
iv) Enamel wall: It is the portion of a prepared external
• Supragingival margins.
wall consisting of enamel.
2. CARIES v) Dentinal wall: is the portion of a prepared external
wall consisting of dentin which may contain retention
i) Pit and fissure caries: This caries occurs in pits and features.
fissures that result from the imperfect coalescence of
the enamel in developmental stage of the tooth. This 5. Simplest classification according to the number of
caries can be represented as cone with base towards surfaces involved:
DEJ and apex towards the enamel surface. • Simple cavity - Only one surface involved.
ii) Smooth surface caries: This caries does not begin in • Compound cavity - Two surfaces involved.
the enamel defect but rather in an area that is habitually
unclean. The base of the cone is towards enamel surface • Complex cavity - Three or more surfaces.
and apex is t owards DEJ.
6. G.V. BLACKS CLASSIFICATION·
iii) Residual caries: Caries that remains in a completed
cavity preparatiorn, whether by operator intention or by Restorations on
accident. • Occlusal surface of molars and premolars.
Class I
• Facial and lingual surfaces of molars.
iv) Forward caries: Where the caries cone in enamel is
• Lingual surface of maxillary incisors.
Larger or atleast the same size as that in dentin.
Backward caries - When the spread of caries along the Restorations on the proximal surfaces of
Class II
posterior teeth.
DEJ exceeds the caries in contiguous enamel and caries
extends into the enamel from DEJ. Restorations on the proximal surfaces of
Root/Senile caries: Occurs on the tooth that has been Class III anterior teeth (KAR- 2013)that does not
exposed to the oral environment. This is usually more involve incisal angle.
rapid than other forms of caries. Restorations on the proximal surfaces of
Class IV
v) Recurrent/secondary caries: This caries occurs at the anterior teeth involving the incisal edge.
borders and underneath the restoration . Restorations on the gingival third of the facial
Class V
vi) Acute/Rampant caries: The carious lesion is light or Lingual surfaces of all teeth.
colored and infectious. It is rapid in damaging the Restorations on the i ncisal edge of anterior
tooth. Class VI
teeth or occlusa l cusp heights of posterior teeth.
vii) Chronic/Slow caries: The lesion is discolored and
fairly hard. The slow rate of destruction allows time for 7. SITE AND SIZE (LATEST CLASSIFICATION):
extrinsic pigmentation. SITE
Lesions originating in the pit and fissure and
3. ANGLES:
smooth surface of enamel of tooth crown. It
i) Line angle - The junction of 2 surfaces along a definite Site 1 includes the buccal pits on mandibular molars
line. and lingual pits on maxillary molars, erosion
ii) Point angle - The junction of 3 surfaces at a point. Lesions on incisal edges and occlusal surfaces.
iii) Internal angle - A line angle whose apex points into All lesions involving contact areas i.e., proximal
the tooth. Site 2
surfaces of anterior and posterior teeth.
iv) External angle - A line angle whose apex points away All gi ngival areas in either enamel or dentin
from the tooth. Site 3
around the circumference of tooth.
v) Cavosurface angle - The angle at the junction of the
walls of the cavity preparation and the external surface SIZE
of the tooth (APPG-15). Size 1 Localised to the point where it is just
(Minimal) beyond remineralisation.
4. WALLS
Larger lesions where there is sufficient
i) Axial wall: An internal wall parallel with the long axis Size 2
sound tooth structure remaining to support
of the teeth. (Moderate)
the restoration.
OPERATIVE DENTISTRY SYNOPSIS

Remaining tooth structure is weak to 10. PRIMARY RESISTANCE FORM:


Size 3 i) It is defined as that shape and placement of the cavity
support the restoration. Cavity design has
(Enlarged) walls that best enable both the restoration and tooth to
to be modified.
Size 4 Tooth structure is very weak. Rest oration withstand masticatory forces along the long axis of the
gives support to the teeth. tooth, wit hout fracture (AIIMS May-14).
(Extreme)
ii) Its features are:
8. STEPS IN CAVITY PREPARATION:
• Relatively flat floors.
a) Initial cavity preparation:
• Box shape.
• Step 1: Outline form and initial depth.
• Step 2: Primary resistance form. • Preservation of cusps and marginal ridges.
• Step 3: Primary retention form . • Rounded internal and external line angles.
• Step 4: Convenience form. • Cusp capping when indicated.
iii) Rounding of internal line angles increases t he resistance
b) Final cavity preparation: to fracture of the tooth structure.
• Step 5: Removal of carious dentin.
Note: For direct gold restorations t he internal angles
• Step 6: Pulp protection. should be sharp and should not be rounded.
• Step 7: Secondary resistance and retention forms.
iv) Rounding of external (Axiopulpal) line angle increases
• Step 8: Procedures for finishing external walls. the fracture resistance of the restoration.
• Step 9: Cleaning, inspecting, conditioning .
v) Minimal thickness of restorative material needed to
resist fracture.
9. OUTLINE FORM AND INITIAL DEPTH:
i) It is establishing the cavity margins in the positions Amalgam 1.5mm
they will occupy in the final preparation and Cast metal 1 - 2mm
preparing the initial depth of 0.2 - 0.8mm pulpally
Porcelain 2mm
into dentin.
ii) For coronal preparations the depth into dentin 11. PRIMARY RETENTION FORM:
should not exceed 0.2 - 0.5mm. It is the shape or fo rm of the prepared cavity that resists
iii) For extensions onto the root, the depth into dentin displacement or removal of the restoration from tipping or
should not exceed 0.8mm. lifting forces.

iv) Connect two close faults (or) restorations if they are RESTORATION PRIMARY RETENTION FORM
less than 0.5mm apart. Amalgam Occlusally converging external walls and
Class I & II occlusal dovetail (Class II)
v) The occlusal surface of pit and fissure cavity is
prepared to a depth of 1.5mm measured at the As the external walls diverge outward,
Amalgam
central fissure. retention grooves or coves are the
Class III & V
primary retention forms.
vi) The gingival margin of the cavity must provide a
minimal clearance of 0. 5mm with the adjacent tooth. Mechanical bond using acid etching and
Composite
dentin bonding agents.
vii) The facial and lingual proximal walls must provide a
Cast metal Parallel longitudinal walls (or) 2-5° of
clearance of 0.2mm with the adjacent tooth.
inlay small angle of divergence.
viii) Enameloplasty is indicated when the depth of fissures
is no more than 1/3'd of the thickness of enamel. Occlusal dovetail and secondary retention grooves aid in
preventing the proximal displacement of restoration by the
ix) In class III preparations it is acceptable to position
occlusal forces. Converging axial walls prevents the occlusal
the incisal margin in the area of contact.
displacement of the restoration
x) The outline form of class V cavities is governed by
the extent of the lesion except pulpally. 12. CONVENIENCE FORM:
xi) Rule for cusp capping: • It is the shape or form of the cavity that allows adequate
observation, accessibility and ease of operation in
If the extension of a cavity from a primary groove
towards the cusp tip is preparing and restoring the cavity.

< 1/z No cusp capping. • For gold foil preparations, it has an additional purpose
of establishing convenience points for starting of gold
1/z to 2/3 Consider capping. foi l condensation.
> 2/3 Cusp capping mandatory.
Dental ;lut.,e

13. PULP PROTECTION: Beveling the cavity margin irn cast gold preparations
i) Atleast 2mm of bulk must be there between restoration produce a cavosurface angle that will result in 30-40°
and pulp. This is called as Effective Dentinal Depth. It marginal metal. This provides a sliding or lap fit joint that
may include dentin, liner or base or combination . improves adaptation.

ii) Bases provide mechanical, chemical and thermal 16 . BEVELLING can serve four purposes in preparations for
protection. Eg. Zinc phosphate, ZOE, Ca(OH) 2, casting:
Polycarboxylate cements.
i) It produces a stronger enamel wall.
iii) Liners are volatile, aqueous suspensions or dispersions
ii) It permits a marginal seal in slightly undersized
of zinc oxide or calcium hydroxide that can be applied
castings.
to cavity surface in a relatively thin fi lm.
iii) It provides marginal metal that is more easily burnished
iv) If the removal of the infected dentin does not extend
and adapted.
deeper than lm m from the initially prepared depth,
then no liner is indicated. iv) It assists in sealing gingival margins of castings that
fail to seat by a very slight amount.
If the excavation is very close to pulp, Ca(OH) 2 liner is
indicated. -
AMALGAMS
If it is in between above two cases, ZOE liner is
indicated. 1. Classification of amalgam alloys:
v) Liners and bases in exposure areas should be applied A. Based on Copper content:
without pressure. Low copper <6%
vi) In case of exposure, atleast 1mm of Ca(OH) 2 must be High copper > 6%
placed which then may be overlaid by a base. • Admixed 9 - 20%
vii) ZOE liner and varnish are contraindicated for use with • Single composition 13 - 30%
composites, as they interfere with the polymerization
B. Based on zinc content:
process.
• Zinc containing - contain more than 0.01°lo of zinc.
viii) Ideally there must be atleast three seats for amalgam • Zinc free - contain less than 0.01 °lo of zinc.
on sound dentin on pulpal floor.
C. Based on alloy particle shape:
ix) Varnish:
• Lathe cut - irregular shape.
• Not used for composites.
• Spherical
• Applied on aill walls for amalgam restorations. • Admixed-contain both lathe cut and spherical alloys.
• Applied on dentinal walls only for cast gold
restorations. 2. Dental amalgam substitutes does not contain mercury.
Eg.: Gallium alloys and mercury-free direct filling alloys.
• Varnish reduces micro leakage, post operative
sensitivity and pulpal irritation from luting cement.
3. Gallium alloys are made up of Ag-Sn particles in Gallium-
Indium liquid. In this case Ga-In substitutes for Hg in
14. SECONDARY RESISTANCE AND RETENSION FORM:
dental amalgams. Gallium melts at 28°C and can be used to
• AMALGAMS: produce liquid alloys at room temperature.
Locks, pins, slots, steps and amalgapins provide the
secondary retention form. Its disadvantages are:
Pins and slots in,crease bot h retention and resistance • Surface roughness
forms. • Marginal discoloration
• CAST RESTORATIONS: • Expansion leads to tooth fracture
• Difficult manipulation
Grooves, groove extensions, bevelled enamel margins
and luting cement.
4. Mercury free direct filling alloys have Ag coated Ag-Sn
Provide the secondary retention features. particles that can be self welded by compaction to create
• Coves are pre pa red for amalgam Class III and V restoration.
restorations and occasionally for facilitating the start
of insertion of certain gold foil restorations. 5. The two types of trituration techniques in use are:
i) Eames or no squeeze cloth or Minimal mercury
15. FINISHING OF THE CAVITY MARGINS: technique: The mercury alloy ratio according to this is
Because of low edge strength or friability of amalgam, 1:1 or 50% mercury.
cavosurface ang le of 90° produce maximal strength for ii) Increased dryness or squeeze cloth technique.
both enamel and the restorative material.
OPERATIVE DENTISTRY SYNOPSIS

6. Most modern precapsulated dental amalgams are formulated 19. Amalgam creep is plastic deformation principally due to
with only 42-45% mercury by weight. very slow metallurgical phase transformations that produces
volume increase or expansion.
7. The mercury content of the finished restoration should be
approximately 50 wt. % or Less while for spherical alloys it 20. Creep is the most common cause of marginal failure.
should be approximately 42 wt.%. Marginal failure is the common cause of failure of low
copper amalgams.
8. According to ADA specification 1, amalgam should not
expand or contract more than 20 µm/cm measured at 37°C, Creep values:
between 5 minutes to 24 hrs. after beginning of trituration. Low copper 0.8 - 8%
High copper
9. Zinc containing alloys lasts 20 - 50% Longer than non zinc
alloys. • Admix 0.4- 0.1%
• Single composition 0.05 - 0.1%
10. Delayed expansion occurs when zrnc containing alloys
are contaminated by moisture during trituration or 21. Electrochemical corrosion of Sn - Hg (y2 ) does not release
condensation reaching upto 400 µm/cm. This occurs due to free mercury int o oral cavity. [nstead mercury reacts with
accumulation of H2 gas in the restoration leading to pain, Ag-Sn (y) particles to produce 'Mercuroscopic expansion'.
sensitivity and protrusion of the restoration. This Leads to ditch formation in the margins due to marginal
fracture.
11. Delayed expansion occurs 3 - 4 days after insertion of
amalgam. 22. MAHLER scale is used to describe the severity of marginal
defects. It has ratings from No. 1 to No. 11.
12. Compressive st rength of high copper amalgam ranges from
380 - 550 Mpa (55,000 - 80,000 Psi) . Scale values of No. 2, 4, 6, 8 & 10 represent the image of
restorations in the series of 5 photographs.
13. y1 crystals (Ag Hg) are small and equiaxed.
Scale values of No. 1, 3, 5, 7, 9 & 11 are used to deal with
14. y2 crystals (Sn Hg) are long and blade Like penetrating the severity of marginal ditching.
throughout the matrix.
23. Mercury toxicity
15. y2 phase is prone to corrosion in clinical restoration. It i) The primary risk to dental personnel is from inhalation
promotes penetrating corrosion (MHCET-15) (from amalgam of mercury vapours. The maximum Level of occupational
surface along y2 crystals and to other crystals at inter exposure considered safe is 50 µg of mercury/ m3 of air.
crystalline contacts) producing porous spongy amalgam.
ii) Mercury vapours may release from Ag-Hg phase which
16. Amalgams undergo 2 types of corrosion. is melted during polishi ng and amalgam removal
procedures.
Chemical corrosion Formation of silver sulphide (Ag 2 S} iii) Rubber dam, high volume evacuation, water cooling
tarnish. unit is used during removal of amalgam.
Electrochemical It is of 3 types - Galvanic corrosion, iv) Mask's can't filter mercury· vapour.
corrosion Crevice corrosion and Stress
v) Mercury has an average half life of 55 days for transport
corrosion.
through the body to the point of excretion.
17. Corrosion products : vi) Mercury toxicity results irn Acrodynia or Pinks disease
Low copper Sn 0, Sn - 0 - Cl in children. The source of the mercury is usually a
(Sn-Hg is involved) teething powder, ammoniated mercury ointment
or calomel lotion. It is characterized by Pinkish
High copper Sn 0, Sn - 0 - CL, Cu 0 discoloration of skin, profuse salivation, ulceration of
(Cu-Sn is involved) the mucosa and premature shedding of teeth.
vii) The three forms of mercury are elemental, inorganic
18. Crevice corrosion occurs in cracks or crevices accumulated
and organic forms.
by plaque and food debris due to drop in Local pH and
decreased oxygen tension. Elemental mercury vapour is inhaled and absorbed
through lungs at 80% efficiency.
Galvanic corrosion occurs between dissimilar metals in
Inorganic mercury exists mainly as sulfide and
contact. The metal with Low EMF undergoes corrosion.
absorbed easily by the gastrointestinal tract.
Stress corrosion occurs in the metal at the site of maximum Mercury can form methyl mercury which is an organic
stress. form . It is main ly carried through food. For most
Dental ;lut.,e

people organically bound mercury in food is the 31. Self threaded pin is t he most retentive and cemented pin is
primary source of mercury exposure. the least retentive of the pins.
viii) Spent capsules and mercury contaminated cotton rolls
32. Pulpal stress is maximal with self threading pin and least
or napkins should be stored in tightly capped pressure
with cemented pins.
container.
33. TMS (Threaded Mate System) is the most widely used self
24. Failures of amalgam restorations: threading pin.
• Bulk fractures of restoration
• Corrosion 34. TMS pins exhibit less microleakage than the friction locked
or cemented pin.
• Excessive marginal fracture
• Sensitivity or pain 35. TMS pins sizes:
• Secondary caries Mini kin 0.48 mm (0.019 inch)
• Fracture of tooth structure Minuta 0.38 mm (0.015 inch)
Minim 0.61 mm (0.024 inch)
25. High copper amalgams have less marginal fracture and
corrosion but they fai l mostly due to bulk fracture due to 36. Failure of pin retained restorations:
fatigue. i) If the failure is within the restoration, the restoration
can fracture.
PIN RETAINED RESTO RATIO NS:
ii) If the failure is at the interface between the pin and
the restorative material, the pin can pull out of the
26. It is defined as any restoration requmng the placement
rest orative material.
of one or more pins in the dentin to provide adequate
retention form and/or resistance form . iii) If the fai lure is within the pin, the pin can fracture
when stressed beyond its ultimate t ensile strength.
27. Advantages of pin retained restorations:
iv) If the failure is at the interface between the pin and the
• Conservation of tooth structure. dentin the pin can pull out of the dentin.
• Completed in one appointment whereas cast restoration
v) Within the dentin, the dentin can fracture.
requires atleast two appointments.
• Increased retention and resistance forms. AMALGAM CLASS-I CAVITY PREPARATION:
• Economical when compared to cast restorations.
1. Ideal outline for class I should not extend the facial/
28. Disadvantages: lingual margins more t han 1/2 way between central groove
• Dentinal microfractures and cusp tip.
• Microleakage
2. If it extends 1/2 - 2/3'd then cusp capping is considered.
• Decreased strength of amalgam
• Resistance form is difficult to develop than cast If it extends > 2/3rd t hen cusp capping is recommended or
restorations. mandatory.
• Perforations.
3. No 245 bur is used for class I cavity preparation.
29. Types of pins are cemented, friction lock and self threading
pins. 4. Proper depth of 1.5 - 2mm shol.lld be established from the
occlusal surface. The 1.5mm depth is established at the
30. central fissure and 2mm is usually the depth of the prepared
external walls.
Friction- Self-
Diameter Cemented
locked threading 5. The minimum distance from external wall near marginal
and depth Pins
pins pins ridge to proximal surface must be 1.6mm (KERALA-2015)
Diameter 0.025-0.05 0.025mm 0.038-0.1 for premolars and 2mm for molars.
of pin hole mm larger smaller mm smaller
than the pin than the pin than the pin 6. If the distance from the external wall near marginal ridge
diameter diameter diameter to proximal surface is 1.6mm or less, the mesial and distal
Pfo depth walls should be diverged.
3mm 3mm 2mm
in dentin
If the distance is greater than 1.6mm the mesial and distal
Pin depth walls converge occlusally.
2mm 3mm 2mm
in amalgam
7. The facial and lingual walls should be converged occlusally.
OPERATIVE DENTISTRY SYNOPSIS

8. Conservative class I cavity outline form consists of gently AMALGAM CLASS II CAVITY PREPARATION:
flowing curves.
1. In class II cavity the punch cut is given in the pit nearest
9. The cavity width should not exceed 1/4th the intercuspal to the involved proximal surface by No. 245 bur.
distance (or) 1-1.5mm of faciolingual width.
2. The isthmus should be no wider than 1/4th intercuspal
Note: Cast inlay is indicated if the cavity width is
distance. Ideally its width should be the width of No.245 bur.
1/3'd the intercuspal distance.
Cast onlay is indicated if the width extends more than 1/3'd 3. Initial procedure in the preparation of the proximal box is
the intercuspal distance. the isolation of the proximal enamel by proximal ditch cut.

10. After excavation there must be atleast 3 flat peripheral 4. During the preparation of proximal ditch cut YJ_'!l of bur
seats on sound dentin. must be in dentin and wn1 in enamel.

11. Shaoes of class I oreoaration· . 5. Ideally the proximal facial/lingual wall of the cavity should
Upper premolar Dumbell clear the adjacent tooth by 0.2 - 0.3mm. Ideally the
gingival wall should clear t he adjacent tooth by 0.5mm.
Lower 1st premolar Butterfly
Lower 2nd premolar Y-shape 6. Ideal dentinal depth of proximal wall in proximal box is 0.5
Upper molar - 0.6 mm in crown and 0. 75 - 0.8 mm in root.
• Mesial Kidney shape
7. Wooden wedge is used to protect the gingiva and dam when
• Distal Heart shape extending gingival wall gingivally.
Lower molar + shape
8. Enamel hatchet or Binangle chisel is used to cleave away
12. Amalgam restorations require 90-100° cavosurface angle the remaining undermined proximal enamel and to establish
thereby providing 80-90° marginal metal. the 90° cavosurface angle.

13. For ideal depth of cavity no liner or base is required. In very 9. Reverse curve in occlusal outline is usually created when
deep cavities Ca (OH) 2 must be used as a base using non mesiofacial enamel wall is parallel to enamel rod direction.
pressure flow technique. Lingually, reverse curve is very slight, often unnecessary.

14. Spherical type of amalgam is recommended in a tooth with 10. Resistance form in class II cavity is obtained by:
direct pulp capping as it requires less condensation forces • Rounding the axiopulpal line angle.
for its compaction. • Bevelling the gingival margin with gingival marginal
Trimmer.
MODIFICATIONS:
Note:
15. Occlusolingual cavit y preparation on maxillary molars is
• If gingival wall is on cementum, bevel is not indicated
indicated when the distal pit, distal oblique ridge and
for amalgam.
lingual fissure are carious or at caries risk. The axial wall
depth is 1.5mm. • Beveling of gingival margin is not done in primary teeth
as the enamel rods direct occlusally in the gingival
16. For facial pit of mandibular molar the penetration cut is region resulting in no unsupported enamel rods.
1.5mm.
11. Proximal locks provide retention against proximal
17. For lingual pit of maxillary incisor the penetration cut is displacement (also occlusal dovetail). They are placed in the
1 - 1.2 mm. axiofacial and axiolingual line angles 0.2mm axial to DEJ.

18. For a cavity preparation on mandibular first premolar: 12.


• The pulpal wall declines towards the lingual cusp. These are boxes prepared in the gingival wall
• The preparation is done more at the expense of the Slots 0.5 - 1mm deep gingivally and 2 - 3mm
facial cusp than of the lingual cusp. length facio lingually.
• The bur must be parallel to the bisector of angle formed Prepared in the gingival wall 0.5 - 1mm deep
Pot holes
by long axis of tooth and the line that is perpendicular gingivally and 0.2mm inside DEJ.
to the plane drawn connecting facia l and lingual cusp
points. 13. With high copper amalgams locks are unnecessary in
preparations that include dovetail as the creep associated
with them is very minimal.
Dental ;lut.,e

14. Apply cavity varnish prior to matrix application t o avoid 7. Secondary retention forms are gi ngival groove, incisal cove
pooling. and lingual dovetail for extensiv,e restorations.

15. In mandibular 1'1 premolar for preparations that does not 8. Gingival retention groove is placed at 0.25mm depth along
cross the transverse ridge prepare the proximal box before the axiogingival line ang le.
the occlusal portion.
9. Incisal retention cove is placed at t he axiofacio-incisal
16. When mesial and distal proximal surface restorations are point angle.
indicated on maxillary 1' 1 molar that has an unaffected
oblique ridge, separate class II cavities are prepared rather AMALGAM CLASS V CAVITY PREPARATION:
than MOD.
1. Because of the difficulty of sealing a cementa[ margin with
17. Modifications are done: composite resin, amalgam is generally considered in class V
• For slot preparation for root caries preparations except when esthetics is of primary concern.
• For rotated teeth, for adjoining restorations
2. Class V amalgam is indicated on the facia l surface of
• For abutment teeth for RPD
mandibular canines, premolars and molars.
• For teeth with unusual att rition form
3. Primary determinant of the outline form is the extension of
18. Amalgam capping requires 1.5mm reduction on non
the caries.
functional cusps and 2mm of reduction on functional cusps.
4. External shape is related to the contour of the marginal
19. Oblique ridge for maxillary molars is included in MOD if:
gingiva.
• Oblique ridge is undermined by caries.
• Crossed by deep fissure. 5. The axial wall is slightly deeper at the incisal wall, where
• If separate outline forms leave less than 0.5mm of there is more enamel (1 - 1.25mm) than at the gingival
tooth structure between them. wall, where there may be little or no enamel (0.75 - 1mm).

20. Teeth with smaller boxes should be restored first. 6. The direction of mesial and distal walls follow the direction
of the enamel rods.
21. If adjacent proximal boxes are similar in size first restore
the most posterior preparation. 7. All the walls are perpendicular to the external surface and
usually diverge facially. So two retention grooves placed at
22. If the adjacent proxi mal boxes are different, prepare the the axioincisal and axiogingival line angles or four retention
larger first so that the preparation for small carious lesion coves placed in each of the four axial point angles provides
may be made conservative by larger preparation but restore the retention form.
the smaller first.
8. The axial wall is convex mesiodistally following the external
AMALGAM CLASS III CAVITY PREPARATION: contour of the tooth.

1. Amalgam class III cavity is usually indicated on dist al 9. Direct filling fold is ideal restorative material for class V
surfaces of maxillary and mandibular canine. cavities.

2. The last step-in application of rubber dam for preparation INSTRUMENTS AND EQUIPMENT
of cavities having proximal caries is wedge placement.
1. Nomenclature and classification of hand instruments is
3. The outline form of the class III cavity for amalgam on given by G.V. black.
distal surface of canine is similar to that for conventional
class III preparation fo r direct tooth colored restoration. 2. Metals used in the manufacturing of hand instruments are
made of two types of steel - Carbon steel and Stainless steel.
4. In class III cavity preparation, axial wall depth is 0.5 -
0.8mm into the DEJ (or) 0.75 - 0.8mm when gingival 3.
margin is on root surface.
Carbon steel Stainless steel
5. Ideally the facial margin is extended 0.2 - 0.3mm into facial Fe - 98% Fe - 81%
embrasure. When lingual approach is used, the lingual wall C - 1- 1.2% Cr - 18%
Composition
can form obtuse angle with the axial wall or it may not Mn - 0.2% C - 0.6-1%
exist and is continuo1Us with the axial wall. Si - 0.2%
Corrosion
6. Conserve the tooth as much as possible at distoincisal Very low High
resistance
corner (canopy).
OPERATIVE DENTISTRY SYNOPSIS

Cutting Low and becomes dull Bibevelled instruments contain beveling on both sides.
High Eg. Ordinary hatchet, osteotome.
efficiency rapidly.
Stiffer, stronger, Hardness is less than 14. To determine whether the instrument has a right or left
Physical
harder but more carbon st eel but it is bevel, the primary cutting edge is held down and pointing
properties
brittle not brittle away, and if the bevel appears on right side, then it is the
Superior at high Superior at low speed right instrument of the pair.
Efficiency speeds and when and when cutting
cutting enamel dentin 15. Mesial bevel - If one observes the inside of the blade
curvature, the primary bevel is visible.
4. Steels are subjected to two types of heat treatments:
Hardening and tempering heat treatment. Distal bevel - If primary bevel is not visible.

5. Hardening heat treatment hardens the alloy, but it makes 16. 4 types of grasps used with hand instruments:
alloy brittle. • Modified pen
• Inverted pen
6. Tempering heart treatment relieves strain and increases
toughness. • Palm and thumb
• Modified palm and thumb
7. Heat treatment for carbon steel is best accomplished in the
absence of oxygen. When carbon steel alloy is heated in Modified pen grasp permits the greatest delicacy of touch.
oxygen the metal oxidizes, discolours and scales.
17. Sharpening stones are of three types - Stationary,
8. Hand instruments are composed of - Handle, shank and mechanical and hand piece sharpening stones.
blade.
18. Stationary stones are often called as oil stones.
The part corresponding to blade in non cutting instruments
is called the 'Nib'. 19. The materials in use for sharoenina stones are·
Arkansas stone For sharpening steel instruments.
9. Instrument 'Balance' is accomplished by designing the
angles of the shank so that the cutting edge of the blade is Industrial abrasives are most common ly
within 2mm of the Long axis of handle. Silicon carbide used for grinding wheels, sand papers
and for sharpening stones.
10. Contra angled shanks - Shanks containing 2 or 3 angles in These are small blocks of metal
the shank to bring the cutting edge near to the projected impregnated with diamond particles
Diamond hones
axis of handle for long bladed instruments. on the surface. Used for sharpening
steel and carbon instruments.
11. Instrument formula:
1 Blade width in tenths of a millimeter. 20. Rx Honing machine is a type of mechanical sharpener.
2 Angle of the cutting edge with the shaft, (cutting edge
21. Sharpness of instrument can be tested by resting the
angle) when other than at right angles to the axis of
cutting edge on a hard plastic surface. If the cutting edge
the blade. This is always greater than 50°
digs over the surface, the instrument is sharp. If it slides,
3 Length of the blade in millimeters. the instrument is dull.
4 Angle of the blade with the shaft (Blade angle) in
22. Spoon excavators are sharpened with hand piece sharpening
centigrades or hundredths of a circle. This is always less
stones
than 50° .
23. Classification of hand instruments:
12. Example 10 - 85 - 8 - 14 indicates:
10 - Blade width in tenths of a millimeter i.e., 1mm. i. Cutting such as excavators, chisels.

85 - Cutting edge angle in centigrades or hundredths of ii. Condensing such as condensers, burnishers.
a circle. iii. Miscellaneous such as mirrors, explorers, probes.
8 - Length of blade in millimeters.
24. Chisels and excavators are the two groups commonly used
14 - Blade angle in centigrades. in oper;iting toot h structures.

13. Single beveled instruments contain bevel on one side


only. Eg. Enamel hatchet and GMT (chisel).
Dental ;lut.,e

25. Speed ranaes· 30. KCP-2000: is an air abrasive equipment used for stain
Ultra low speed 200 - 2000 rpm removal, debriding pit and fissure prior to sealing and
micromechanical ro ughening of t he surfaces to be bonded.
Low speed 2000 - 8000 rpm
Medium high speed 20,000 - 45,000 rpm 31. Parts of a bur are shan k, neck and head.
High speed 45,000 - 1 lakh rpm
32. The common shank designs are:
Ultra high speed Above 1 lakh rpm
• Straight handpiece shank - Used for finishing and
26. The main advantage of increasing the speed of the rotating polishing and for anterior restorations.
instrument is to increase its cutting efficiency. • Latch-type hand piece shank - Short er in lengt h, so
improved access to posterior regions. Used for finishing
27. The first clinically successful air t urbine hand piece (speed procedures at low and medium speed.
3 lakhs rpm) - Bordan airotor hand piece.
• Friction grip shank - Used with high speed hand pieces.
28. Low speed is used for cleaning teeth, caries excavation,
finishing and polishing procedures. High speed is used for 33. Steel burs are mainly used for finishing procedures.
tooth preparation and removal of old restoration.
34. Carbide burs have Tungsten carbide particles in the matrix
29. LASER - Light Amplification by Stimulated Emission of of cobalt or nickel.
Radiation. EXCEMERS are special ultra violet lasers. CO 2 and
NO-YAG are infrared lasers.

35.
INSTRUMENT FEATURE USES
Chisels
• Has straight shank and bevel on one side only. Used • To cleave or split under-mined enamel.
Straight/ curved/ with bot h push and pull motion. • Flatten pulpal floors.
binangle • Curved chisel is called as Wedelstat chisel. It has 4
figure formula.
Cutting edge parallel with the long axis of the handle and • To split under-mined enamel in buccal and lingual
Enamel hatchet beveled on one side proximal walls.
• For placing grooves.
• It is a enamel hatchet with a curved blade. • For beveling gingival margin.
• It has the cutting edge at an angle ot her than 90° to • For beveling axiopulpal line ang le.
Gingival Marginal
t he axis of the blade (4 fi gure formula)
Trimmer (GMT)
• When 2nd figure is 90-100 it is a distal GMT. If it is
85-75 then it is a mesial GMT.
Excavat ors
• Has the cutting edge of t he blade directed in t he same • For preparing retentive areas in anterior teeth
Ordinary hatchet plane as that of the long axis of the handle. • For sharpening internal line angles for direct gold
• It is bibevelled. fillings.
• Cutting edge is perpendicular to the long axis. • To give form to t he internal parts of cavity used on
Hoe enamel of posterior teet h.
• Can be used by pull and push movements.
• Combination of GMT and chisel because it encompass • For sharpening line angles or obtaining retention
Angle former
the advantages of both. form in dentin.
• The cutting edge is either claw like or on a radius. • For removing caries
Spoon excavator • Circular edge is know n as - discoid. • For carving amalgam.
• Claw like edge is known a - cleiod
OTHERS
Known as finishing knives or amalgam or gold knives. For trimming excess filling material on gingival,
Knives
facial and lingual margins of a proximal restoration.
The teet h of the instrument are designed to make the file To trim excess filling material particularly on gingival
Files
either a push or pull instrument. margin
Discoid-cleiod The working ends of this instrument are larger than the • For carving occlusal anatomy.
instrument discoid or cleiod end of an excavator. • To burnish inlay-on lay margins.
OPERATIVE DENTISTRY SYNOPSIS

36. Pressure is directly proportional to the heat generated. 51. Noise Levels in excess of 75db cause hearing damage.
Whenever revolutions per minute is increased pressure has
to be reduced. It should not exceed 4 ounces when using 52. SEPARATORS:
high speeds and not more than 12 ounces with Low speeds. • Separation of the teeth is necessary to restore a proper
contact.
37. Temperature rise during cavity preparation should not
exceed 130° F or 60°C. When using high speeds Low
• The two types of separation are:
pressure and water coolant should be used.
i) Slow separation: The teeth are separated by inserting
38. 'Round bur' is used for: certain materials between them. Eg. Orthodontic
• Initial entry into the tooth. wire, gutta-percha, wood, base plate, etc.
• Extension of preparation. ii) Rapid separation: Most commonly used, quick
method. It can be achieved by wedge principle and
• Preparation of retention features.
traction principle.
• Caries removal.
• Wedge principle:
39. 'Inverted cone'
For providing undercuts in cavity preparation. Eg: Elliot separator, wedges.

40. Pear shaped bur: • Traction principle


• Normal length - For gold foil class I preparations. Eg: Ferrier double bow separator
• Long Length - For amalgam cavity preparation. Ivory adjustable separator
Non interfering t rue separator
41. Tapered fissure:
For inlay and crown preparation. 53. MATRICES:
Matrix system consists of matrix band, retainer and wedge.
42. Cross cuts are needed on fissure burs to obtain adequate
cutting effectiveness at Low speeds. They are not used at Matrix is a device used to contour a restoration to simulate
high speeds as they tend to produce unduly rough surfaces. that of a tooth structure, which it is replacing. The thickness
of a matrix band is 0.002 inches (0.05 mm).
43. Major trends in bur design are - reduced use of crosscut,
extended heads on fissure burs and ro unding of sharp tip Restoration Matrix indicated
angles. Class I with Lingual Barton (Double banded
extension tofflemi re)
44. Number of blades:
• Tofflemire
• The number of blades on a bur is always even. • Ivory 1 & 8 matrices
• For excavating, burs has 6 - 10 blades. • Compound supported matrix
• Finishing and polishing burs has 12- 40 blades. Class II • Copper band matrix
• T-band matrix
• Greater the number of blades smoother will be the
• Automatrix
cutting action at slow speeds.
• Pre contoured sectional matrix
45. Flute is the depressed area between the blades of bur. Class III direct
Transparent matrix strips
tooth coloured
46. Each blade has two faces: Class III silicate • Celluloid strips
Rake face - Surface of bur blade on the leading edge. restoration • Mylar strips
Clearance face - Surface of bur blade on the t railing edge.
Class III resin • Cellophane strips
47. Vibrations above 1300 rpm is practically imperceptible to the restorations • Mylar strips
patient. Thus vibration sensation is lost at high speed range. • L-shaped matrix
48. If air is alone used as a coolant, it dessicates tooth • Transparent crown forms
Class IV direct
structure. Osmotic disturbance result in aspiration of nuclei tooth coloured • Aluminium foi l incisal corner
and odontoblasts into the tubules. This also increases matrix
permeability of dentin and hence result in hypersensitivity. • Anatomic matrix
Distal surface of canine 'S' shaped matrix
49. Types of valves used with hand pieces to prevent cross
contamination are Anti retraction valves. • Plastic matrices
Class V direct • Anatomic matrix
50. One square centimeter cutting of dentin exposes 30,000 - tooth coloured • Aluminium or copper collars for
45,000 number of tubules. non light cured restorations.
Dental ;lut.,e

54. Matrix retainers are gadgets used to retain the matrix bands 57. WEDGES:
in position. • These are the third component of matrix system.
Eg.: Ivory matrix holder No. 1, 8.
• Wedge must be triangular or trapezoidal in cross section.
Tofflemaire universal dental matrix retainer.
Auto matrix. It is useful in patients with gingival recession.
PIGGY
A second smaller wedge is placed on the first
BACK
55. Tofflemire matrix band retainer is ideal to use when three wedge to prevent gingival overhanging .
surfaces (MOD) are prepared and commonly used for the It is useful in case of wide proximal box.
Double
two surface class II restoration. This is of two types - Wedges are placed from both lingual and facial
Wedging
straight and contra angled. The latter is used for lingual surfaces.
positioning. Indicat ed in case of maxillary 1st molar due
Wedge to the presence of mesial concavity, a second
56. AUTOMATRIX is a retainer less matrix system with 4 types wedging wedge is inserted between the first wedge and
of bands designed to fit all teeth regardless of their band.
circumference.

CAST RESTORATIONS

1. The five general designs of cast restorations are:


• Inlays
• Onlays
• Cast restorations with surface extentions
• Pin lays
• Full veneer cast or cast based restorations.

2. DIFFERENCES BETWEEN INLAY AND AMALGAM RESTORATIONS:


CAST INLAY AMALGAM RESTORATION
It is appropriate for moderate to extensive cavity preparations It is appropriate for conservative cavity preparations.
Contraindicated in you ng patients due to the presence of large pulp Indicated in young and old patients because of the simple
chambers and in old patients due to lengthy and multi step procedures. procedure involved.
Costs more to the patient. Costs less to the patient.
No.271 bur is mostly used (KERALA-2015). No. 245 bur is commonly used.
The axial walls should be parallel or slightly divergent occlusally to a Retention form is achieved t hrough axial walls that converge
taper of 2-5° per wall for achieving the primary retention. occlusa lly.
An occlusa l bevel of 30-40° is placed for proper adaptation of the in lay. Bevels are not indicated as amalgam has low tensile strength.
Gingjval bevel of 30 ° is 11laced in the gingival wall to: • Gingival bevel of 15-20° is placed in the gingival wall to
• Remove t he weak enamel t hereby improving the resistance form . remove the unsupported enamel rods t hereby improving
the resistance form .
• To provide a lap, sliding fit of the casting.
• Gingival beveling is not required in deciduous teeth as
• Allow burnishing of the metal margin thereby improving adaptation. there is no unsupported enamel rods in this region.
The proximal portion is box shaped. The proximal box has an inverted trunk shape.
Cavosurface angle is 130-140° (lap joint) Cavosurface angle is 90 - 100° (butt joint)
Marginal metal angle is 30 - 40° Marginal metal angle is 90°.
Superior control of contours and contacts can be achieved. There is less control on the development of proper contours
and contacts.
Flares (Primary + Secondary) prepared on the facial and lingual Flares are not indicated
proximal walls carries the same functions as that of bevels on occlusal
and gingival walls.
The isthmus width should not exceed 1/3rd of intercuspal distance. Isthmus width should not extend 1/4th of the intercuspal
distance.
It is an indirect procedure requiring two appointments. One for tooth It is a direct procedure requiring one appointment.
preparation and the other for delivering the restoration to the patient.
The minimum thickness of restorative material required to resist Requires at least 1.5mm thickness of amalgam to resist
deformation is about 1-2 mm. deformation.
Retention grooves are placed on the axiofacial and axiogingival line Retention locks are prepared on the axiofacial and axiolingual
angles to provide secondary resistance form . line angles for secondary retention.
OPERATIVE DENTISTRY SYNOPSIS

3. 5. If inlay is in supraocclusion, it causes dull pain on pressure


Partial bevel Involves part of enamel used for removal and slight thermal sensitivity after 2 to 3 months.
of weakened enamel.
6. Location of cavosurface margin, occlusal to the free gingival
Short bevel Involves entire enamel. margin is best for periodontal health.
Long bevel Involves entire enamel and half of dentin.
Most commonly used bevel. 7. In restoring the contours on a tooth, the most critical area
Complete Involves entire enamel and dentin. in preventing gingival inflammation is middle third of the
bevel tooth.
Counter or For capping cusps to protect and support 8. Retention and resistance form of class II inlay cavity
reverse bevel them. depends on occlusal dovetail, proximal box form, flat
gingival and pulpal walls.
4. ON LAYS:
i) Onlays are the most indicated and universally 9. Factors that determine whether to prepare an on lay, partial
used cast restorations for individual teeth. It is a veneer crown or a complete crown design for the gold
partly intracoronal and partly extracoronal type of casting are:
restoration. • The demand for esthetics.
ii) It has cuspal protection as the main feature. • The amount of retention available with the remaining
tooth structure.
iii) It is mainly indicated when width of the lesion exceeds
one-third the i ntercuspal distance. • The need to produce contours for supporting prosthetic
appliances.
iv) In the tooth preparation, if the length : width ratio • The importance of keeping essential teeth in the
of the cusp is more than 1:1 but not exceeding 2:1, treatment plan.
cuspal protection is to be considered.
v) If the length : width ratio of a cusp is more than 2:1, 10. William H. Taggart popularized the gold casting in 1907.
cuspal protection is mandatory.
11. Materials used for casting are:
vi) Onlays are the ideal supporting restorations for • Alloys of gold
remaining tooth structure combined with conservative
• Fired porcelain and
tooth involvement.
• Alloys of base metals
vii) The main designing features of onlay are capping
of the functional cusps (buccal cusps of lower and 12. Disadvantages of gold castings are:
lingual cusps of upper teeth) and shoeing of the non • Lack of adaptability to the cavity walls, necessitating a
functional cusps. cementing medium.
viii) The cuspal reduction in onlays is 1.5mm deep. • Non-esthetic look.
ix) The bevel in the intracoronal portion of the wall • Time interval between completion of the cavity
preparation and completiori of the restoration.
constitutes the hollow ground long bevel.
• It is not economical.
The bevel in the extracoronal part of the preparation
is the counter or reverse bevel. 13. The 3 basic methods of gold casting construction:
x) 'Counter bevel' is not placed on the facial cusps of • Indirect technique
maxillary premolars and first molars where the esthetic • Direct method
requirement dictates only a blunting and smoothing of • Indirect-direct
the enamel.
xi) 'Table' is the transitional area between the intracoronal 14. Classification and uses of castina aold allovs·
and extracoronal parts of the preparation. The table Type I (Soft) Small inlays
should be flat and relieved from opposing cusps by Type II (Medium) Inlays, 3/4'h crowns, pontics
atleast 1.5mm.
Type III (Hard) Full crowns, short span bridges
xii) There should be three tables prepared for each cusp,
Long span bridges, clasps and
each with different directions. Table provides the Type IV (Extra hard)
partial denture frame work.
resistance and retention form.
xiii) 'SHOE' serves the purpose of table on the non 15. Base metal alloys have low density, high stiffness (modulus
functional cusp. But it is less relieved than the table. of elasticity), more passivity, low proportional limit and
It also serves as retention and resistance form. less cost when compared to gold alloys.
Dental ;lut.,e

16. Casting shrinkage= Thermal shrinkage + 23. Uses of casting ring liner:
Solidification shrinkage i) Permits unrestricted expansion of the investment.
ii) Acts as a cushion between rapidly cooling metal ring
17. Higher melting alloys tend to exhibit greater shrinkage and
and the more slowly cooling investment during casting
that is why base metal alloys have higher casting shrinkage
and crystallization of gold.
(2.3%) than gold alloys (1.5%) .
24. Objectives of burnout technique:
18. Compensation is made for the casting shrinkage by utilizing
setting expansion, hygroscopic expansion and thermal • Complete removal or volatalization of the wax pattern.
expansion of the investment. • Elimination of excess water of the investment.
• Production of a degree of thermal expansion of the
In high heat casting technique the greater part of the invest ment to compensate the casting shrinkage.
compensation is provided by greater expansion of the
investment. 25. For gypsum investments burnout temperature is 400°C,
maintained for 30 min. Then tern perature is raised to 700°C
19. Composition of waxes: and maintained for 30min.
• Basic constituent
Paraffin • Have tendency to flake For phosphate investments the temperature is from 750 -
900°C maintaining it for 30 min.
• Less smooth and glossy
Gum • Improves smoothness 26. Casting machines commonly in use are centrifugal and air
dam mar • Decreases flaking pressure types.

Carnuaba/ • To decrease flow


27. The modes of fusing alloys are
Candelilla • Imparts glossy surface
• By using blow torch
Ceresin • Replaces part of paraffin
• By electrical induction
20. Type I or Type B wax is used for making wax pattern
28. The fuel used for fusing the alloys is t he combination of
employed in Direct technique.
oxygen and acetylene gas (highest flame temperature).
Type C or Type II wax is used for indirect technique wax
29. The flame has four zones from inner to outer cones:
pattern construction.

21. Inlay waxes should have a maximum flow of 1% at 37°C to Mixing zone Air and gas mix here.
permit carving and removal of wax pattern and a minimal Green in color. Zone of partial
flow of 70% at 45°C, which helps in insertion of wax into Combustion zone
combustion.
the prepared activity.
• Blue in color.
22. Principles for correct spruing of a wax pattern: Reducing zone • Hottest zone.
i) Use of proper gauge of sprue. • Useful for fusing alloys.
Generally the sprue former diameter is in between 8 - • Definitely oxidizing.
18 gauge. Oxidizing zone
• Not useful for fusing.
• For inlay 8-12 gauge
• For onlay 10-14 30. Pickling is the process of cleaning gold casting by 50%
• For MOD 10-12 warm Hcl (NEET- 2013) or 50% H2 SO 4 •

ii) Use of proper length of sprue to position the pattern TOOTH COLOURED RESTORATIONS
properly in relation to the end of the ring.
1. Tooth colored restorations are divided into direct and
The lengt h of the sprue should be such that there should indirect restorations.
not be more than 1/41h inch (6mm) between the ends
of casting ring and wax pattern. Long sprue results in 2. Direct restorations: Restorations that are fabricated in the
localized shrinkage porosity and very short sprue results prepared tooth.
in subsurface porosity. • Silicate cement
• Unfi Lled resin
iii) should be attached to the bulkiest portion of the
pattern, on a surface free of anatomical carvings. • Filled resin
• Composite resin
iv) Attach it to the wax pattern with as little heat as • Glass ionomer cement
possible, to avoid pattern distortion.
OPERATIVE DENTISTRY SYNOPSIS

Indirect restorations: Restorations that are outside of the 14. Disadvantages


prepared tooth. • High polymerization shrinkage.
• Ceramic inlays and onlays • High coefficient of thermal expansion (81xl0·6j°C).
• Cast moldable ceramic • Decomposition of tertiary amine accelerated by natural
• Porcelain fused to metal or U.V. light results in discoloration.
• Laboratory processed composite inlays and onlays.
FILLED RESIN:
3. SILICATE CEMENT:
15. To resolve the deficiencies caused by high polymerization
POWDER LIQUID shrinkage and a high coefficient of thermal expansion,
• Silica - 40% • Phosphoric acid - 40% inert filler particles were added to red uce the resinous
• Alumina - 30% • Aluminium component.
-2%
Phosphate
• Naf } 16. The amount of fi ller content is 25%.
Cryolite - 19% • Magnesium
-6%
CaF2 Phosphate
17. These fi ller particles are not chemically bonded to resin
• Calcium phosphate • Water - 6% matrix.

4. The purpose offluoride salts is to Lower the fusion temperature l8. The incomplete filler resin bond resulted in:
of the glass. Thus they are called as 'CERAMIC FLUXES'.
• Microscopic defects between filler particles and
surrounded resin.
5. It is the first translucent filling material introduced by
Fletcher in 1878 in England. • These defective areas became stained due to fluid
leakage.
6. In the set cement, the phosphate matrix is an irreversible gel • Poor filler retention.
with specific amount of liquid in it. Any loss or reduction in • Poor wear resistance.
liquid will affect its properties especially optical properties.
COMPOSITES:
7. Its hardness and coefficient of thermal expansion is closer
to dentin. 19. It is a 3-dimensional combination of atleast two chemically
different materials with different discrete interphase
8. It is a severe irritant to the pulp. At the time of insertion it separating the two materials.
has a pH of 2.8 and even after one month, it remains below 7.
20. Composition:
9. It has high solubility and disintegrate readily in oral fluids.
• Resin matrix phase or continuous phase (KERALA-2015)
Failure of silicate cement are easy to detect because of
discolouration and Loss of contour when examined with • Reinforced or filler or dispersion phase.
explorer. • Coupling agent.

10. It is contraindicated in mouth breathers, as its surface 21. RESIN MATRIX PHASE CONSISTS OF:
becomes rough and opaque when allowed to dry. • BIS-Phenol Glycedyl methacrylate (BIS-GMA)
• Trietheylene Glycol dimethacrylate (TEGDMA) used as a
UNFILLED RESINS:
viscosity controller
11. Compone nts • Urethane dimethacrylate (UEDMA) .
Resin Methyl methacrylate • Colouring agent.
Initiator Benzoyl peroxide • Initiator.
• Inhibitor.
Activator Tertiary amine
(dimethyl para toluidine) 22. Filler or reinforced phase:
Inhibitor Hydroquinone i) Colloidal silica, quartz ceramic materials, barium,
Cross linking agent Glycol dimethacrylate strontium and zirconium glasses are the filler particles
used in composites.
12. U.V. light transformer used in unfilled resins is Benzoin
ii) Quartz is extremely hard, difficult to polish and causes
Methyl ether. more abrasion of opposing teeth or restorations. So it
is not used commonly.
13. Advantages (compared to silicates)
• Tooth Like appearance. iii) Ba, Zn, Zr and ytrium glasses are currently the most
popular fillers. They produce radiopacity.
• Insolubility in oral fluids.
Dental ;lut.,e

iv) Colloidal silica is a chemical precipitate from a liquid iii) Cavity should be slightly overfilled.
solution as amorphous silica particles.
iv) Shrinkage occurs towards center of the material
v) Pyrogenic silica is a precipitate from a gaseous phase
as amorphous particles. 26. UV light activated composites:
vi) 'Inserts or megafillers' are the glass pieces of size 0.5 i) Curing occurs due to activation of Benzoin methyl ether
- 2mm added to posterior composites at the point of when exposed to UV light.
occlusal contact or high wear. They improve the wear ii) Potential health hazard to clinician and patient by UV light.
resistance.
iii) It causes retinal and soft tissue damages.
vii) If composites contain simply filler and uncured matrix
material it is classified as HOMOGENOUS. iv) Intensity of light source gradually decreases in strength
with use.
If composites contain precured composite fillers or
other unusual filler and uncured matrix material it is 27. Visible Light activated composites:
called as HETEROGENOUS.
i) Consist of a single paste
viii) Filler sizes Initiator - Cam phoroquinone
Mega fill Very large particle Activator - Diethyl-amino-ethyl methacrylate
(amine) or Diketone.
Macro fill • 10 - 100µ
• used in traditional or fine particle ii) Most popular composites today are the visible light
cure composites.
composite
iii) They interact when exposed to light at wavelength
Midi fill 1 - 10µ
range of 400-500nm i.e., blue region of visible light
Minifill 0.1 - 1µ spectrum, predominantly at 474nm.
Microfill • 0.01 - 0.1µ iv) Light curing can be accomplished with Quartz-
• Used in finishing composites Tungsten-Halogen curing units or Light Emitting
Nanofill • 0.005 - 0.01µ Diode (LED) curing units.
• Ultra small fillers. v) Heath hazard is virtually eliminated.
ix) Addition of fillers, increases the strength, hardness, vi) No warm up time is required.
abrasion resistance and decreases the polymerization vii) Requires a minimum of 20 secs. for adequate curing.
shrinkage and water resorption.
viii) The tip should be kept as close as possible to the
restoration and should be cured in increments.
23. COUPLING AGENT:
ix) Each increment should not be greater than 2mm.
i) Organosilanes, zirconates and titanates are the coupling
agents. x) Darker shades require longer exposure time.

ii) They bond filler particles to the resin matrix. xi) Shrinkage occurs towards the light source.
iii) It increases strength, reduces solubility and water 28. The advantages of light cure over self cure are:
absorption.
• Extended working time
24. The classification of composites based on method of curing: • Reduced porosities
• Self cure composites • Better wear and abrasion resistance
• Visible light cure composites
29. Light transmitting wedges promote interproximal curing. Light
• UV light cure composites focusing tips assists for light access into proximal boxes.
• Dual cure composites (self+ light cure)
• Staged curing composites 30. Dual cured composites:
i) It has self curing and visible curing components in the
25. Self-cure composites: same material.
i) Supplied in two pastes ii) Recommended for ceramic in lays that may be too thick
to allow sufficient amount of light to radiate through to
Base plate Contains the initiator Benzoyl peroxide. produce adequate conversion of the monomer.
Accelerator Contains tertiary amine (N-dimethyl-P-
paste toluidine) as activator. 31. MACRO CERAMICS:
This is the reinforcing (filler) phase of 1" generation
ii) It is manipulated by mixing the proper proportion on
composite resins. It consists of silica-silicate based fillers
a mixing pad with an agate spatula (metal spatula like quartz, fused silica, silicate glasses, etc. It has particles
discolors the composite). with diameter between 5 - 75 µm.
OPERATIVE DENTISTRY SYNOPSIS

32. COLLOIDAL AND MICROCERAMICS: • Hybrid type in which resin phase is


This is the reinforcing phase of 2"dgeneration composite resins. reinforced with micro (colloidal) ceramics
6th and agglomerates of sintered micro
Colloidal silica, pyrogenic silica with particles not exceeding generation (colloidal) ceramics.
0.04µm are used as filler particles. Consequently they give
• It has the best physical and mechanical
extremely smooth surface.
properties of all the composites.
33. COMPOSITE RESINS - WEAR THEORIES:
35. CAVITY PREPARATION:
Microfracture It proposes that high modulus filler There are three basic designs of preparations for composites.
They are
theory particles are compressed onto the
adjacent matrix during occlusal Loading, • Conventional
thus creating microfractures in weaker • Beveled conventional
matrix. • Modified
Hydrolysis It proposes that silane bond between
theory the resin matrix and filler particle is 36. CONVENTIONAL PREPARATION DESIGN:
hydrolytically unstable and becomes i) It is similar to the cavity preparations done for
debonded. amalgam restorations.
Chemical It supposes that matrix degradation
ii) It has box Like cavity, slight occlusal convergence, flat
degradation occurs by materials absorbed from saliva floors and undercuts in dentin.
theory and food.
Protection Proposes that the weak matrix is eroded iii) The cavosurface angle is near to 90° (butt joint).
theory between the particles. iv) If an existing amalgam restoration is removed, usually
a conventional preparation is noted.
34. GENERATIONS OF COMPOSITES:
v) This design can be used with posterior composites for
• Consists of macro ceramic reinforcing class I, II and VI.
1st phases in resin matrix.
generation • They have highest surface roughness and vi) When a class II cavity extends onto the root, cavity
mechanical properties. preparation design should be conventional only.
• Consists of colloidal and microceramic vii) Conventional cavity design is done for
reinforcing phases in resin matrix. • Composites
• They exhibit the smooth texture of all the • Amalgam
2nd
composite resins.
generation • Silicate
• Properties of strength and coefficient of
• Porcelain in Lay
thermal expansion are unfavourable due to
Less filler Loading.
37. BEVELED CONVENTIONAL:
• Hybrid composites in which there is a
combination of macro and micro (colloidal) i) Incorporation of an enamel cavosurface bevel in
3rd ceramics as reinforcers in 75:25 ratio. preparations for composite is recommended, as it
generation provides more surface area for bonding and it allows for
• The properties are a compromise between the more preferred end-on etching of the enamel rods.
1st and 2nd generations.
• These are also hybrid types, but instead of ii) These features improve retention, reduce leakage and
macroceramic fillers, these contain heat strengthens the remaining tooth structure.
4th cured irregularly shaped, highly reinforced iii) The bevel is prepared with a flame shaped diamond
generation composite macro particles. instrument, 0.5mm wide and at an angle of 45° to the
• They produce superior restorations but are external enamel surface.
very technique sensitive.
iv) It may be used for class I, II and VI cavities.
• Hybrid system in which resin phase is
reinforced with micro-ceramics and macro, 38. MODIFIED PREPARATION DESIGN:
heat cured spherical, highly reinforced
composite particles. i) It does not routinely extend into dentin and depth of
preparation depends on the pulpalextentofcarious lesion.
5th • Because of the specific shape of the macro-
generation molecules, the workability is improved. ii) Modified cavity preparations are more conservative than
• Surface texture and wearofthese materialsare conventional, since mechanical retention is obtained by
acid etching.
comparable to 2nd generation composites.
• Physical and mechanical properties are
similar to 4th generation composites.
Dental ;lut.,e

iii) Advantages of ca,vosurface margin beveling are: 48. After 24 hours the pH of GIC is 5.3.
• It provides restorative margin to merge with tooth
structure, so no margin discolouration is seen as 49. To increase pH of GIC, ZnO is incorporated in powder. These
that of conventional cavity. cements are called Buffered GI cement or Zynomer.
• Marginal leakage is reduced.
50. Glass ionomers bond best to enamel than dentin and
• Minimal pulpal irritation. cementum.
• Good esthetics.
51. Bonding of GIC is achieved in part by mechanical retention
39. (opal varnish, ZOE are contraindicated as a liner under and in part by chemical chelation.
com po site or any resin because they interfere with
polymerization reaction. 52. GIC shows decreasing levels of fluoride release with time.
Initial rapid fluoride release is fo llowed by slow Long term
40. Quick setting Ca(OH) 2 is recommended as a base under release by diffusion.
composites in deep cavities.
53 . Light cured GI
41. Matrices used for composite resin are Mylar strip and The powder contains initiators for Light curing and liquid
compound supported metal matrix (0.04 mm). component is modified with Hydroxy Ethyl Methacrylate
(HEMA).
42. Finishing of composite resin is carried out with carbide bur,
diamond bur, polishing disc or strip with to and fro motion. The polymerization starts when exposed to light and
subsequently followed by acid base reactions. This is called
43. Glaze is a thin layer of bonding agent that is placed over "DUAL CURE" GIC.
the restoration which helps to create a smooth surface by
filling the surface porosities. 54. PROPERTIES OF GIC:
i) GIC has low fracture toughness and wear resistance.
GLASS IONOMER CEMENT:
ii) Opacity of GIC related to glass is 76%.
44. Other names: iii) Very sensitive to moisture, especially during initial
• Polyalkenoate cement setting reaction. During this period, absorption of
water Leads to weak cement and over drying leads to
• Manmade dentin cracks in the cement. So the surface of cement should
• Dentin substitute be protected by coating with varnish or cocoa butter
• Alumino Silicate Polyacrylic Cement (ASPA) during setting.
iv) Powder liquid ratio is 3:1 by weight. Mixing should be
45. COMPOSITION: done by agate or plastic spatula.
POWDER LIQUID v) 10% polyacrylic acid should be used for conditioning
i) Silica - 35-50% i) Polyacrylic acid - 45% the cavity surface before insertion of the cement.
ii) Alumina - 20-30% ii) Itaconic acid vi) Final finishing is done after 24 hours of insertion.
iii) NaF - 3.6% Malic acid } 5%
iv) Al F3 - 1.5 - 2.5% Tricarballyic acid 55. MODIFICATIONS OF GIC:
v) Aluminium- 4-12% (decreases viscosity) a) Miracle mix or silver cermet:
Phosphate iii) Tartaric acid - Silver-Tin alloy powder is added to GIC powder. None
vi) Traces of barium, increases working time of the properties were improved and it gave a gray
strontium for iv) Water - 50% or blackish colour to the cement. It is also called as
radiopacity 'SILVER ALLOY ADMIX'.

The powder is referred to as 'ion teachable glass'. b) Glass cermet or cermet:


Glass and metal (silver-tin- palladium) powders were
46. It was introduced by Wilson and Kent in 1969. sintered at high tern perature and made to react with
liquid. It improved the fracture toughness and wear
47. CLASSIFICATION: resistance and at the same time maintained esthetics.
Type I Luting cement
Type II Restorative cement c) Resin modified GJC:
Type III Liner BISGMA, TEGDMA are added to powder and HEMA to
Type IV Fissure sealant the Liquid. With exposure of light, polymerization is
Type V Orthodontic cement initiated along the methacrylate groups. After that
Type VI Core build up cement the liquid reacts with glass particles through acid
Type VIII } Posterior packable GIC for base reactions. It improved the wear resistance and
Type IX atraumatic restorations decreased the sensitivity to water attack.
OPERATIVE DENTISTRY SYNOPSIS

d) Bilayered or sand witch restorations: iv) Line and point angles, internal and external angles
In this technique GIC is used as a liner under composite should be well-rounded to avoid stress concentration.
restorations. It increases the retention form as GIC
v) The gingival to occlusal divergence of the cavity
bonds both with the tooth and the composite and the
preparation can be increased from 2 - 5° per wall to
fluoride cont ent reduces secondary caries. 6 - go .

e) Tunneling restorations: vi) Proximal boxes are prepared identically as those for
Joining the occlusal lesion with the proximal lesion by cast metal boxes avoiding bevels and secondary flaring.
means of a prepared tunnel under the involved marginal Cavosurface angle should be 90° (butt joint).
ridge. The marginal ridge remains intact. GIC is used as
vii) The gingival finish line should be a shoulder preparation.
the restorative material in this technique.
6. FABRICATION STEPS:
56.
i) After cavity preparation, an impression is made and a
GI TYPES USES 'master' working cast is poured on diestone.
Traditional GI Liner, base and filling material ii) The die is duplicated, a master die and a refractory dies
Metal modified GI Filling material, base, core build are poured.
up material iii) Dental porcelains are added into the cavity preparation
Light cured GI Liner and base of the refractory die, and fired in an oven. Multiple
Hybrid or Resin Cement, fi lling material and core increments and firings are necessary to compensate for
modified GI build up material sintering shrinkage.
iv) The ceramic restoration is recovered from the refractory
die, cleaned of all investment and then seated on the
INDIRECT TOOTH COLORED INLAYS AND ONLAYS master die and working cast for final adjustments and
finishing.
1. Advantages:
i) They have better physical properties than direct DENTAL ADHESION
restorations.
ii) Increased resistance to abrasion and attrition. 1. ADHESION: is a process of solid and/or liquid int eraction of
iii) Reduced polymerization shrinkage stress. one material (adhesive or adherent) with another (adherend)
at a single interface. Eg.: Pit and fissure sealants.
iv) Ability to strengthen remaining tooth structure.
v) More precise control of contours and contacts. 2. The material or film added to produce the adhesion is known
as the ADHESIVE or ADHERENT and the material to which it
2. Disadvantages: is applied is called the ADHEREND.
i) Increased cost and time.
ii) Technique sensit ive. 3. ADHESIVE JOINT: It is the result of interaction of a layer
of intermediate mat erial (adhesive or adherent) with two
iii) Brittleness of ceramics.
surfaces (adherends) producing two adhesive surfaces.

CERAMIC INLAYS AND ONLAYS 4. Adhesion is classified as -


involves Vanderwaals or electrostatic
3. They have become popular due to the public demand for
Physical interactions that are relatively weak. It
esthetic and durable restorative materials.
bonding occurs when the surfaces are smooth and
chemically dissimilar.
4. The ceramic materials used are Feldspathic porcelain,
castable ceramic {DICOR) and machinable glass ceramic involves bonding between atoms formed
(DICOR MGC). across the interface from the adhesive to
Chemical
the adherent. The extent of this bonding is
bonding
5. CAVITY PREPARATION: limited and the overall contribution to the
bond strength is quite low.
i) The cavity preparations for indirect tooth coloured
inlays and onlays are essentially the same as described is the result o,f interface that involves
for cast metal inlays and on lays except beveling and undercuts and rough surface that produces
Mechanical
secondary flaring which are contraindicated for ceramic interlocking of the materials. Almost every
bonding
restorations. case of dental adhesion is based primarily
on mechanical bonding.
ii) A uniform axial reduction of 1.5 - 2mm is adequate .
iii) All margins should have 90° cavosurface angle for 5. If the mechanical roughness produces microscopically
marginal strength of the restoration. interlocked adhesive and adherent with dimensions of less
than 10µ then it is called as MICRO MECHANICAL BONDING.
Dental ;lut.,e

6. The requirements for adhesion are: • Presence of smear layer on the cut dentin surface.
i) To produce good bonding there must be good wetting. • The potential biologic side effects that different
Complete wetting occurs at 0° contact angle. chemicals can cause within the pulp.
ii) Surfaces being joined should be clean. iii) As most composites are hydrophobic in nature, dentin
bonding agent should be hydrophobic to bond with
7. Bonding Systems composite and hydrophilic to interact with dentin .
These are classified on the basis of the primary adherent.
They are iv) The dentin bonding protocol is:
• Enamel bonding systems • Etching/conditioning - Step 1
• Dentin bonding systems • Application of primer - Step 2
• Amalgam bonding systems • Application of bonding agent - Step 3
v) CONDITIONERS are agents, which aids in removal or
8. Enamel bonding systems: modification of smear layer.
i) They most often consist of an unfilled liquid acrylic Eg. EDTA, 10% phosphoric ;icid, citric ;icid, maleic
BIS-GMA resin monomer mixture placed onto acid-
acid, etc.
etched or conditioned enamel.
vi) PRIMERS are hydrophilic monomers, which are applied
ii) Acid etching is usually done by 37% (30 - 50%)
over the etched/conditioned surfaces for the easy flow
phosphoric acid! which results in the formation of
of bonding agents.
enamel tags into which the resin will fit.
Eg . 2-HEMA and HEMA dissolved in acetone or alcohol.
iii) Concent ration greater than 50% results in the
formation of Monocalci um phosphate monohydrate vii) DENTIN BONDING AGENTS are unfilled resin, which
which prevents further dissolution while below 30% have a role in formation and stabilization of hybrid
results in dicalcium phosphate dihydrate that cannot layer (micro mechanical attachment between resin
be easily removed. and conditioned primed dentin).
iv) Resin tags which form between enamel rod peripheries viii) Even though tags are formed into tubules, bond
are called MACROTAGS and that form across the end of strength is mainly due to micromechanical bonding of
each rod are called as MICROTAGS. interbular dentin.
v) Macro and micro tags are the basis for micro- ix) Clinical longevity of OBA is not as long as EBA due to
mechanical bonding. Most micro tags are only 2-5mm water content.
in length. x) Generations of Dentin bonding agents
vi) Currently the length of application of the etchant Uses glycerol-phosphoric acid
is often 15 seconds. This yields an acceptable bond 1st generation
dimethacrylate.
strength and also conserves time and enamel.
• Uses chloro substituted
vii) Fluoride treated teeth and primary teeth require longer phosphate esters of various
time due to the presence of more a prismatic enamel. monomers.
viii) The etched area is rinsed with water for 20 seconds. 2nd generation • In 1st and 2nd generations
there is no distinction between
ix) After etching, surface area increases upto 2000 times
conditioning, primer and
that of original untreated surface.
bonding agent.
x) Maleic acid, citric acid and oxalic acid are used to etch
3 step procedure i.e., conditioning,
enamel and dentin. 3rd generation
priming and bonding.
xi) If saliva accidentally contaminates the prepared Relies on formation of hybrid
cavity, reapply the acid for 10 seconds. 4th generation layer. Combines step 1 and 2 and
xii) The enamel bonding agent is applied to the etched are called as self etching primers.
surface which enhances the wettability of composites 5th generation Steps 2 and 3 are combined.
to etched enamel.
1 step procedure.
6th generation
9. DENTIN BONDING SYSTEMS: All 3 solutions in one bottle.
i) DBS consists of unfilled, liquid acrylic monomer 7th generation Fluoride releasing bonding agents.
mixture placed onto an acid conditioned and primed
dentin surface. 10. AMALGAM BONDING SYSTEMS:
ii) Dentin bonding is difficult because i) Amalgam bonding systems may be used to bond
amalgam to tooth, amalgam to amalgam or amalgam to
• Dentin is a living heterogenous tissue and contains
other metal substrates.
plenty of fluids.
OPERATIVE DENTISTRY SYNOPSIS

ii) As dental amalgam is strongly hydrophobic and enamel 3. Knoop Hardness Number (KHN) of some of the restorative
is hydrophilic the bonding agent must have the capacity materials and tooth material are
to wet both hydrophobic and hydrophilic surfaces.
Material KHN (Kg/mm 2)
Eg: 4-META (Kerala- 15) is frequently used.
Enamel 300
iii) The primary advantage of amalgam bonding agents is
Dentin 65
sealing of dentin.
Pure gold 52 - 75
11. MULTIPURPOSE BONDING AGENTS: Amalgam 90
Single system useful for enamel bonding, dentin bonding,
Porcelain 412
amalgam bonding and composite-to-metal bonding.
Composites 55
12. Cast restorations bonding system uses luting cements. Acrylic resin 15
Emery 2000
13. BOND STRENGTH:
Silicon carbide 2500
i) Bond strength increases if adhesive joint thickness
decreases. Diamond 7000 - 10000
ii) Amalgam bonding agent - 2-6 Mpa (least)
4. Setting time of some of the dental materials:
Enamel bonding agent - 18-22 Mpa
Material Setting time
Dentin bonding agent - 22-35 Mpa (highest)
Fast setting alginates 1 - 2 min.
iii) When cast restorations are bonded with chemical
Normal setting alginates 2.5 - 4 min .
adhesives (polycarboxylate, GIC) most of the bond
strength results from mechanical adhesion only. Alginate (optimal) 3 - 4 min.
Impression plaster 4 min.
14. Scientists and their discoveries associated with bonding.
Agar 5 min.
• Acid etching Buonocore
GIC Type I 4 - 5 min.
• Hybrid zone con,cept Nakabayashi
GIC Type II 7 min.
• BIS-GMA resin Bowen
Silicate cement 3 - 8 min .
ZOE cement 4 - 10 min.
MISCELLAN EOUS Zinc phosphate cement 5 - 9 min .
Zinc polycarboxylate cement 7 - 9 min.
1. Linear coefficient of thermal expansion for some materials.
Type 1 (hard) ZOE paste 10 min.
Material u (10·6 j°C) • Model plaster
Tooth 9 - 11 • Dental stone or class I stone
Silicate 8 • Die stone or class II stone 12 min.
Unfilled acrylate 81 • Dental stone of high strength
Composites 28 - 45 and high expansion
Amalgam 25 Type 2 (soft) ZOE paste 15 min.
Direct gold 18
Aluminous porcelain 6- 7

2. Linear coefficient of thermal expansion (a) of some of the


dental materials in relation to the tooth enamel are

Porcelain 0.580
Dentin 0.750
Silicates 0.996
Enamel 1.000
Gold 1.230
Amalgam 2.290
Unfilled resins 7.110
Dental ;fc&.,e

ENDODONTICS
I. REFERENCE BOOKS TAKEN:
1. A TEXTBOOK OF ENDODONTIC PRACTICE BY GROSSMAN - 11th & 12th Editions

2. TEXTBOOK OF ENDODONTICS BY INGLE & BARKLAND - 5th, 6th & 9th Editions

3. PATHWAYS OF PULP BY COHEN - 8th, 9th & 10th Editions

1. ANATOMY OF THE PULP CAVITY


1. What% of lower 1st molars show 2 distal canals 7. Accessory canals are most frequently found in:
a) 10% b) 30% a) The cervical one third of the root
c) 60% d) 75% b) The middle one third of the root
(MAN -01) c) The apical one third of the root
2. Two canals are most often seen in the d) With equal frequency in all the above mentioned
a) Maxillary canine b) Mandibular canine (AIPG-96)
c) Maxillary Lateral incisors 8. There are sharp demarcations between pulpal chambers
d) Mandibular first premolar and pulp canals in which of the following teeth?
(MAN -97) a) Mandibular second premolars
3. The mandibular molars generally have b) Maxillary ti rst premolars
a) Two roots & two canals b) Two roots & three canals c) Maxillary Lateral incisors
c) Three toots & two canals d) Mandibular canines
d) None of the above (AIPG -03)
(MAN -97) 9. In the mandibular arch, the greatest lingual inclination
4. The fourth root canal if present in a maxillary 1st molar of the crown from its root is seen in the permanent:
is usually present in: a) Canine b) Third molar
a) Mesiolingual root b) Mesiobuccal c) First premolar d) Central incisor
c) Palatal root d) Distal root (AIPG -98)
(COMEDK-14, AIIMS-95, KAR-98) 10. The mesiolingual root canal of the mandibular 1st molar
5. Cervical cross section of maxillary first premolar has: is found under the
a) A round shape b) Elliptical shape a) Mesio lingual cusp b) Mesio buccal cusp
c) Oval shape d) Square shape c) Central groove d) Mesio Lingual ridge.
(AIPG -94)
6. Of the following permanent teeth, which is least likely 11, The orifice of the fourth canal in a maxillary molar is
to have two roots? usually found:
a) Maxillary canine b) Mandibular canine a) Under the Distofacial cusp
c) Maxillary first premolar b) Lingual to the orifice of the Mesiofacial canal
LI) Marruiliular fi rsl 1.ir~111ular c) Orr a Lin~ runniny fru111 lh~ ublufacial urific~ Lu lh~
(AIPG -98) Mesiofacial orifice

1) B 2) D 3) B 4) B 5) B 6) A 7) C 8) B 9) C 10) C 11) B
ENDODONTICS

d) On a line running from the lingual orifice to the 22. Four canals are seen in:
Distofacial orifice a) Upper 1st molar b) Lower 1st molar
(KAR -01) c) Upper 2nd molar d) Lower 2nd molar
12. A divided pulp canal is most likely to occur in the (PG! -02)
a) Root of a maxillary canine 23. The root canals most likely to share a common apical
b) Root of mandibular canine opening are
c) Root of a maxillary central incisor a) Mesial and distal roots of mandibular premolars
d) Lingual root of a maxillary first molar b) Mesiobuccal and mesiolingual roots of mandibular first
(AIPG -03) molars
13. If the pulp of the single rooted canal is triangular in c) Both A & B d) None of above
cross-section with the base of the triangle located
facially and apex located lingually with the mesial arm 24. Branching of pulpal canals is least likely seen in
longer than the distal, the tooth is most likely: a) Maxillary central incisor b) Upper 1st premolar
a) Max. central incisor b) Max. lateral incisor c) Mand central incisor d) Mand lateral incisor.
c) Mand. second premolar d) Mand. central incisor (AP-04)
(AIIMS-99) 25. The anterior tooth most likely to display two canals is
14. Considering the morphology of root and pulp canals, a a) Maxillary central b) Maxillary lateral
root canal instrument should be placed in what direction c) Mandibular central d) Mandibular lateral
to gain access to the Mesiofacial root of permanent
maxillary first molar: 26. The tooth which usually has the largest pulp chamber in
a) From the mesiobuccal b) From the distobuccal the mouth is the
c) From the mesiolingual d) From the distolingual a) Maxillary central b) Maxillary canine
(AIIMS -91) c) Maxillary 1st molar d) Mandibular 1st molar
15. Mandibular 1st molar has:
a) 2 roots and 2 canals b) 2 roots and 3 canals 27. Incidence of 3 rd root in upper first premolar
c) 3 roots and 3 canals d) 3 roots and 4 canals a) 6% b) 10%
(AIPG -99) c) 12% d) 1%
16. In which anterior tooth are bifurcated roots present: (BHU-07, PGI JUNE-12)
a) Mand. lateral incisor b) Maxillary canine 28. Percentage of distal root with two root canals in
c) Mand. central incisor d) Mandibular premolar mandibular molar
(AIPG-90) a) 10% b) 30%
17. Which of the following has the largest relative c) 60% d) 1%
mesiodistal dimension of the root canal? (BH U-07)
a) Maxillary lateral incisor 29. Access cavity shape in mandibular 1•t molar with 4 canals.
b) Mandibular second premolar a) Trapezoidal b) Round
c) Palatal root of maxillary 1st molar c) Oval d) Triangular
d) Distal root of the mandibular 1st molar (AIIMS-08)
(KAR-99) 30. Apical constriction is otherwise known as -
18. Which root canal is most difficult to prepare in maxillary a) Minor diameter b) Major diameter
molar? c) Radiographic apex d) Tooth apex
a) Mesiobuccal b) Distobuccal (COMEDK-09)
c) Palatal d) Both A and B 31. Bifurcations and trifurcations are most commonly
(PGI-95) observed in
19. The tooth most commonly having bifurcated roots is the: a) Maxillary 1st premolar b) Maxillary 2nd premolar
a) Maxillary central incisor b) Mandibular lateral incisor c) Mandibular 1st premolar
c) Mand. central incisor d) Mandibular premolar d) Mandibular 2nd premolar
(AIIMS-98) (KCET-12)
20. The most easily perforated tooth with a slight mesial 32. In anterior teeth, the starting location for access cavity
or distal angulation of bur after a mandibular central is the center of the anatomic crown on lingual surface at
incisor is: a) Angle to it b) In line to it
a) Maxillary premolar b) Maxillary molar c) Perpendicular to it d) All of the above
c) Mandibular premolar d) Maxillary canine (BHU-2012)
(AIPG-99) 33. Most common chances of pulpal exposure will be there
21. A cross-section of the cervical third of the pulp canal of if pulpal floor is made perpendicular to the long axis of
a maxillary second premolar resembles in shape: tooth:
a) A circle b) A square a) Maxillary 1st premolar
c) A triangle d) An ellipse. b) Maxillary 1st molar
(KAR-97) c) Mandibular 1st premolar

12) B 13) A 14) D 15) B 16) C 17) D 18) A 19) D 20) A 21) D 22) A 23) C 24) A
25) C 26) C 27) A 28) B 29) A 30) A 31) C 32) C 33) C
Dental ;lut.,e

d) Mandibular 2nd premolar


(AIIMS-2011)
34. The access cavity for mandibular first molar typically is
a) Trapezoid b) Triangular
c) Oval d) Round
(AIIMS MAY 2012)
35. Incidence of 2 canals in mandibular incisors is
a) 3 - 12% b) 12 - 20%
c) 20 - 41% d) Less than 3%
(AIIMS MAY-13)
3 6. Which of the following is likely to have bifurcated
roots?
a) Mandibular canine b) Maxillary Canine
c) Mandibular Incisor d) Maxillary Incisor
(COMEDK-14)
3 7. The objective of the access cavity preparation is to gain
direct access to
a) Pulp chamber b) Canal orifice
c) Apical foramen d) Middle third of the canal
(COMEDK-14)
3 8. Which of the following is generally the longest root
canal on the maxillary first molar?
a) Mesiobuccal b) Distobuccal
c) Palatal d) Distolingual
(COMEDK-14)
3 9. C shaped morphological appearance is seen in
a) Maxillary 1st premolar b) Mandibular 1st premolar
c) Maxillary p t molar d) Mandibular znd molar
(PGI JUNE-13)
40. Radiolucency seen along the instrument while BMP is
most probably
a) Lamina dura b) Perforation
c) Extra canal d) Extra root
(PGI JUNE-14)
41. (-shaped canals are found most commonly in
a) Mandibular 1' 1 molar b) Mandibular znd molar
c) Maxillary 1st molar d) Maxillary znd molar
(MHCET-15)
42. In periapical X-ray, the radiodensity of the root is seen
as 'Fast break"
a) Bifurcation of canal b) Calcification of canal
c) Excessive curvature of root canal
d) Meeting of canals
(PGI DEC-13)

34) A 35) C 36) A 3D C 38) C 39) D 40) C 41) B 42) A


ENDODONTICS

1. ANATOMY OF THE PULP CAVITY - ANSWERS

1. ' B' (Grossman 12th ed 211) 10. 'C' [Grossman 12th ed 210)
The access cavity of a mandibular 1st molar is usually It is often incorrectly believed that the mesial ling ual
triangular in shape. If t he 4th canal is present, the shape canal is present under the me·siolingual cusp. Continuous
of occlusal cavity will become trapezoidal. searching in that area will lead to undesirable perforation .

2. ' D' [Grossman 12th ed 205] The mesiobuccal orifice is under the mesiobuccal cusp and
Bifurcations and trifurcations are most common in is usually difficult to find if enough tooth structure is not
mandibular 1st premolar. They present a challenge during removed. This orifice can be explored in a mesiobucco-
cleaning, shaping and obturation. Because of this it is apical inclination into a point angle formed by mesial wall,
known as "Enigma to endodontist'~ buccal wall and pulpal floor of pulp chamber.

3. ' B' (Grossman 12th ed 210) The mesiolingual orifice is present below the central groove.
Mandibular molars contains 2 roots and 3 canals. The mesial It is located in line with the mesiobuccal orifice and mesial
root contains 2 canals i.e., mesio buccal and mesio lingual to distal orifice. It is explored from a distobuccal direction.
and the distal root cont ains distal canal. About 30% of
mandibular molars shows 4th canal in distal root. The distal orifice has an elliptical shape and is usually
present in the centre of tooth buccolingually. It is explored
4. ' B' (Grossman 12th ed 192) from a mesial direction. If the distal orifice is present
The maxillary 1st molar contains 3 roots and 3 canals buccally or lingually, it means that a 4th canal may be
situated mesiobucally, distobuccally and palatally. present.
Mesiobuccal root is narrowest of all canals and it shows
greatest distal curvature because of which it is difficult for 11. ' B' [Grossman 12th ed 192)
instrumen-tation. About 35% of cases show a 4th canal in
mesiobuccal root in palatal direction. 12. ' B' [Grossman 12th ed 204]

5. ' B' (Grossman 12th ed 186) 13. 'A' [Grossman 12th ed 181)
The cervical cross section of maxillary 1st premolar is
elliptical or kidney shaped. 14. ' D' [Check Explanation Below]
In case of MAXILLARY FIRST MOLAR
The access cavity preparation of maxillary 1st premolar • The orifice of mesiobuccal canal is gained access from
differs from G.V. Black's cavity preparation for an occlusal distopalatal direction.
restoration. In Black's preparation the ovoid shape runs
• The distobuccal root canal is gained access from
mesiodistally whereas the endodontic preparation runs
mesiolingual direction.
ovoid in a buccolingual direction.
• The palatal root is gained access from buccal direction.
6. 'A' [Grossman 12th ed 185)
For MANDIBULAR 1st MOLAR:
7. 'C' [Grossman 12th ed 181) • The mesiobuccal orifice is present under mesiobuccal
The periodontal vessels curve around the root apex of a cusp and is explored from mesiobucco apical direction.
developing tooth and entrapped in hertwig's epithelial root • The mesiolingual orifice is present below t he central
sheath. So accessory canals are more common in apical groove and is explored from disto buccal direction.
third of root. Among anterior teeth accessory canals are
• The distal orifice is explored from a mesial direction.
common in mandibular central incisor whereas in posterior
teeth, accessory canals are most common in mandibular 1st
15. ' B' (Grossman 12th ed 210)
molar teeth at furcation area.
16. 'C' [Grossman 12th ed 203 )
8. ' B' (Grossman 12th ed 185)
Among single rooted teeth, bifurcated roots are commonly
The division between root canal and pulp chamber is
seen in mandibular 1st premolar followed by mandibular
indistinct in single rooted teeth whereas in posterior teeth
canines and incisors. Maxillary canines and incisors are
this demarcation is sharp.
least likely to have bifurcated roots.
9. 'C' [Grossman 12th ed 205]
17 . ' D' (Grossman 12th ed 210]
Mandibular 1st premolar contains prominent buccal cusp
and smaller lingual cusp that give the crown a lingual tilt of
18. 'A' [Grossman 12th ed 192)
30°. To compensate for the tilt and to prevent perforations,
Mesio buccal root has greatest distal curvature and is
the enamel is penetrated at the upper 3rd of lingual incline
narrowest of all the three canals.
of facial cusp and directed along long axis of root.
Dental ;lut.,e

19. ' D' [Grossman 12th ed 205] Radiographic apex - is the tip or end of the root determined
radiographically
20. 'A' [Cohen 8th ed 192]
The cerviacal width of maxillary 1st premolar is narrower Root morphology and radiographic distortion may cause the
mesiodistally, the clinician must use cautious to avoid location of the radiographic apex to vary from the anatomic
mesiodistal perforation . There is also a concavity on mesial apex.
side, which makes the area below pulp chamber laterally
thin. So caution should be taken during post build up Apical foramen or Major diameter is the main apical
procedures. This is a less robust (strong) tooth than the opening of the root canal. It is frequently eccentrically
crown size would indicate. located away from the anatomic or radiographic apex.

21. ' D' [Grossman 12th ed 191] Apical constriction or Minor diameter is the apical portion
of the root canal having the narrowest diameter.
22. 'A' [Grossman 12th ed 192, 194]
4th canal is common in mesiobuccal root and it is usually 31. 'C' [Grossman 12th ed 207]
located palatal to main canal.
Two separate MB canals occur in 84% of teeth in which two 32. 'C' [Grossman 12th ed 182]
separate orifices are traced. In anterior teeth, the enamel is penetrated in the centre of
the Lingual surface at an angle perpendicular to it with a
23. ' C' [Grossman 12th ed 208, 210] round bur. After penetration of enamel, the bur is directed
The mesiobuccal and mesiolingual roots of mandibular first along the long axis of tooth (parallel) to reach the pulp
molars are the root canals most likely to share a common chamber.
apical opening. Mandibular bicuspids, which exhibit
additional root formation often, have a single apical 33. 'C' [Cohen 8th ed 206]
opening. The bur is tilted slightly lingually to establish the correct
pulpal wall while preparing cavity in mandibular 1st
24. 'A' [Grossman 12th ed 182] premolar.

25. ' C' [Grossman 12th ed 203] 34. 'A' [Refer Synopsis]

26. ' C' [Grossman 12th ed 191] 35. 'C' [Clinical endodontics by Tronstad 3rd ed 209]
The pulp chamber of maxillary 1st molar is the largest in The incidence of two canals in mandibular incisors accounts
the dental arch. The pulp chamber of the maxillary canine for about 25% and can be as high as 41%. But only 13%
(Option 'B') is the largest of any single rooted teeth. have two separate apical foramena. So 2 canals unite
and exit through one apical foramen i.e, Weine type-2
27. 'A' [Cohen 10th ed 184J canal configuration is the most common after type-1 in
According to Cohen, prevalence of some of the anatomic mandibular incisors.
alterations
3rd root in Maxillary 1st premolar 6% The second canal is most commonly situated towards
lingual and is the most common cause of endodontic failure
Dense indente in maxillary lateral incisor 0.04 - 10% if failed to Locate it.
Second canal in the mesiobucal
root (using microscope) of maxillary 95.2% 36. 'A' [Check Q•• No 16]
1st molar
Two canals in mandibular incisors 41.4% 3 7. 'C' [Cohen 10th ed 150]
Objectives of access cavity preparation are:
Two root canals in distal root of
30% • To remove all caries
Mandibular 1st molar
• To conserve tooth structure
28. ' B' [Grossman 12th ed 211/ Cohen 9th ed 221] • Completely unroof the pulp chamber
• To remove all the vital or necrotic pulp tissue.
29. 'A' [Cohen gth ed 210/ Grossman 12th ed 212] • To locate all roof canal orifices
Approximately 1/3'd of mandibular first molars have four • To achieve straight Line access to apical foramen or to
root canals. If the traditional triangular outline is changed the initial curvature of the canal.
to a more trapezoidal or rectangular form, it permits better • To establish restorative margins that minimizes the
visualization and exploration of fourth canal in t he distal root. marginal Leakage around restoration.

30. 'A' [Ingle 5th ed 511/ Grossman 12th ed 222] 38. 'C' [Cohe n 10th ed 188]
Anatomic apex - is the tip or end of the root determined Palatal root is the Longest, has the largest diameter, and
morphologically. generally offers the easiest access.
ENDODONTICS

39. 'D' [Ingle 6th ed 209]


(-shaped canal is a variation in canal and root morphology
first given by Cooke and Cox in 1979. This results from the
fusion of the mesial and distal roots on either the buccal or
the lingual root surface

Melton classification of (-shaped canals


Continuous (-shape running from pulp
Category 1
chamber to apex without any separation
Semicolon shaped orifice in which dentin
Category 2 separates main (-shaped canal from one
distinct mesial canal
(-shaped orifice with two or more distinct
Category 3
and separate canals.

Due to the higher incidence of root fusion, ( -shaped canals


are more common in mandibular second molars followed
by mandibular 1st molar, mandibular 3rd molar, maxillary
second molar, maxi llary 1st molar, maxillary third molar,
mandibular premolar and maxillary lateral incisor.

40. 'C' [Ingle 5th ed 225]


"Look at the corners of the radiograph and the center will
take care of itself'. This a worthy quote given whenever
suspicion of an additional root canal is there.

The tips how to detect the undetected canals:


1. Follow the image of the test file in the length of the
tooth film, particularly in coronal part of the root. If
an extra dark line is apparent running parallel to the
instrument, is suggestive of additional canal.

2. Fast break: If there is a sudden change in the


radiolucency within a canal, this density change signals
the presence of an additional or bifurcated canal.

41. 'B' (Check Explanation of Q.No.39]

42. 'A' [Check Explanation of Q.No.40]


Dental ;lut.,e

2. DISEASES OF PULP AND PERIRADICULAR TISSUES


1. The treatment of acute Periapical abscess is b) Extirpation of pulp and formocresol dressing
a) Endodontic therapy or extraction c) Pulpotomy d) Pulp capping
b) Incision and drainage only (PGI-97,98, AIPG-92)
c) Pulp capping d) None of the above 12. Acute reversible pulpitis is treated by:
(APPSC-99, MAN-98,99, AIPG-99) a) Sedative filling wait and watch
2. Internal resorption in a tooth is seen as: b) Pulpectomy
a) Ca(OH) 2 pulpotomy b) Replacement resorption c) Pulpotomy d) Pulp capping
c) Non-proliferating lesion d) Desensitizing tooth (PGI-98)
(PGI-2K) 13. Which one of the following is not seen in the acute pulpitis:
3. The initial pulpal response to any insult is: a) Pain disappears on removal of stimulus
a) Necrosis b) Ulceration b) Pain persists on removal of stimulus
c) Calcification d) Inflammation c) Increased threshold for stimulus
(AIIMS-98) d) Severe pain
4. Patient reports with severe throbbing pain in relation (AIPG-98)
to mandibular second molar. The tooth is sensitive to 14. The treatment of acute pulpitis is
both hot and cold food and elicits a painful response on a) Extraction b) Pulpotomy
percussion. What would be your treatment of choice? c) Pulpectomy d) PA curettage
a) Incision and drainage of Periapical area (PGI-97)
b) Endodontic therapy 15. Blood borne seeding of bacteria into an area of
c) Indirect pulp capping d) Partial pulpectomy previously damaged or irritated pulp with a resultant
(AIIMS-92) inflammation is known as
5. The most important aspect of emergency treatment for a) Bacterial endocarditis b) Anchoretic pulpitis
an acute apical abscess is to: c) Chronic hyperplastic pulpitis
a) Produce sedation b) Obturation d) Ulcerative pulpitis
c) Establish drainage d) None of the above {COMEDK-04)
(AIPG -92) 16. Pain due to acute "irreversible" pulpitis is
6. In treating a tooth with a nonvital pulp and a sinus, the a) Spontaneous b) Sharp
sinus should be treated as follows: c) Lasting for short time d) A & B
a) No special treatment b) Cauterisation (PGI-95)
c) Curettage of sinus 17. The Periapical lesion that would most likely contain
d) Use of cautery to eliminate the sinus bacteria within the lesion is
(AIIMS-92) a) Periapical abscess b) Periapical cyst
7. A pulp polyp may arise in connection with: c) Periapical granuloma d) Condensing osteitis
a) Chronic open pulpitis b) Pulp necrosis (COMEDK-04)
c) Acute pulpitis d) Chronic Periapical lesion 18. Type of efferent nerve fibers of pulp is
{AIPG- 06, PGI-02) a) Somatic motor
8. Internal resorption is due to _ _ __ b) Sympathetic postganglionic
a) Pulp necrosis b) Acute inflammation of pulp c) Parasympathetic postganglionic
c) Chronic inflammation of pulp d) Both somatic & autonomic postganglionic fibers
d) None of the above (AIIMS-2011)
(AIPG -97) 19. A patient feels that his molar is extruded from the
9. Most common cause of pulp pathology is: socket and is tender to percussion. This can be due to
a) Microbes b) Trauma a) Periapical cyst b) Periapical granuloma
c) Leakage from filling materials c) Periapical abscess d) Furcation involvement
d) Pressure sensation from condensation of filling materials (AIPG -01)
( PGI-98) 20. Calcification of pulp:
10. Phoenix abscess is a) is in response to ageing
a) Reversible pulpits. b) Acute apical periodontitis b) does not relate to the periodontal condition
c) Acute exacerbation of an existing chronic inflammation c) precedes internal resorption
d) Chronic abscess d) indicates presence of additional canal
(KAR-99) (AIPG-06)
11. Which of the following is the method of treating internal 21. Which one of the following does not commonly survive
resorption: in a periapical lesion?
a) Extirpation of pulp and calcium hydroxide dressing a) Pseudomonas b) Streptococcus

1) A 2) C 3) D 4) B 5) C 6) A 7) A 8) C 9) A 10) C 11) A 12) A 13) A


14) C 15) B 16) D 17) A 18) B 19) C 20) A 21) A
ENDODONTICS

c) Porphyromonas d) Actinomyces 33. Hall mark signs of neuropathic pain are mechanical
(AI-05) hyperalgesia and
22. Cholesterol crystals are occasionally present in which of a) Allodynia b) Alopecia
the following Zone? c) Allocretia d) Allogeusia
a) Zone of contamination b) Zone of irritation (BH U-2012)
c) Zone of necrosis d) Zone of stimulation 34. Internal resorption:
(COMEDK-07) a) Will appear as lesion superimposed over an unaltered
23. The following is not true about phoenix abscess canal in a radiograph
a) Symptoms are similar to acute infection b) Is associated with a systemic disease
b) Associated with non-vital tooth c) Is frequently associated with trauma
c) Radiographically does not show radiolucency in the d) Requires deferment of treatment with "careful watching"
periapical region (NEET-2013)
d) Tooth is t endor on percussion 35. Which of the following may be used to fill the pathologic
void of a non-perforating internal resorption?
24. Pulpal pain is most intense in a) Silver amalgam
a) Acute closed pulpitis b) Acute open pulpitis b) Guttapercha
c) Plup hyperaemia d) Pulp necrosis c) Zinc-oxide eugenol
(COM ED K-08) d) All of the above
25. Which one of the following conditions does not require (NEET-2013, AIIMS 2012)
any treatment unless it becomes symptomatic? 36. Pulpal inflammation is marked by:
a) Subacute pulpitis b) Chronic pulpitis a) Peripheral vasoconstriction
c) Pulp fibrosis d) Apical cyst b) Strangulation of pulp
(UPSC-09) c) Increased vascular permeability
26. Odontogenic pain d) Decreased capillary pressure.
a) Tooth and periodontal ligament pain (NEET-2013)
b) Bone pain 37. A tooth with a chronic alveolar abscess would have all
c) Soft tissue pain d) Jaw pain the following features EXCEPT
(AP-2011) a) Always symptomatic
27. The stimulus, which provokes pain easily? b) Detected only during routine radiographic examination
a) Analgesia b) Allodynia c) Presence of a sinus tract
c) Dysasthesia d) Hyperpathia d) The sinus tract provides continual drainage of the
(AP-2011) periradicular lesion
28. Pulp capillary pressure (KAR- 2013)
a) 14 cm of H20 b) 25 cm of H20 38. Normal intrapulp pressure is
c) 80 cm of H20 d) 120 cm of H20 a) 10mm Hg b) 5mm Hg
(AP-2011) c) 7mm Hg d) 15mm Hg
29. Which of the following fibres are respons ible for (COMEDK- 2013)
conduction of pain impulse? 39. A 45 year old female patient reports to the clinic with a
a) Alpha b) beta history of pain with tooth 37. The tooth appears normal
c) delta d) gamma on clinical and radiographic examination except for the
(COMEDK-2011) fact that pain occurs on biting on a hard object and then
30. Which of the following is the most difficult pulpal or releasing the bite.
periapical pathosis to diagnose? (COMEDK- 2013)
a) Necrotic pulp b) Chronic pulpitis 39A. The most probable cause for the pain may be
c) Internal resorption d) Acute alveolar abscess a) Occult caries b) Periodontitis
(AIPG-2011) c) Cracked tooth d) Maxillary sinusitis
31. Enamel infarctions can be detected by
a) Direct illumination with visible light 398. The various other means of diagnosing such a lesion
b) Ultraviolet light source include:
c) Magnetic resonance imaging a) Methylene blue staining
d) Fibre optic light source b) Tooth slooth
(KCET-2012) c) Transillumination d) All of the above
32. The stabbing nature of pain in trigeminal neuralgia
mimics pain caused by 39C. Immediate treatment of such a tooth involves
a) A cracked tooth b) Acute reversible pulpitis a) Stabilization with orthodontic band
c) Acut e irreversible pulpitis b) Extraction
d) Acute apical periodontitis c) Relief from occlusion
(COMED-2012) d) Full coverage crown

22) B 23) C 24) A 25) C 26) A 27) 0 28) A 29) C 30) B 31) 0 32) A 33) A 34) C
35) D 36) C 37) A 38) A 39A) C 398) D 39C) A
Dental ;lut.,e

40. Odontalgia that is occasionally typical in hypertensive 49. Halo effect surrounding the root of tooth on IOPA X-Ray
patients is a result of is seen with?
a) Headache radiating to the teeth a) Horizontal root fracture
b) Nervous tension and worry b) Root caries
c) Hyperaemia of pulp resulting from increased blood c) Widening of periodontal ligament space
pressure. d) Periapical-periostitis
d) Abnormal stimulation of the sympathetic nerve system (AIIMS MAY-13)
(COMEDK- 2013) 50. Inflammation of the perapical tissue is sustained by
41. A 25 year old male comes with a complaint of throbbing which of the following
pain in the upper part of the cheek or entire side of a) Stagnant tissue fluid b) Necrotic tissue
face, heavy feeling in the face, which exacerbates on c) Microorganisms d) Pus cells
bending down and with associated pyrexia. The most (COMEDK-14)
probable diagnosis is 51. Pulp of involved teeth is vital in the following
a) Acute dento alveolar abscess in relation to upper first a) Apical cyst b) Granuloma
molar. c) Apical periodontitis d) Phoenix abscess
b) Irreversible pulpitis in relation to upper 2nd molar. (GCET-13)
c) Acute maxillary sinusitis 52. Which of the following is most susceptible to local
d) Acute periodontal abscess in relation to upper 1st molar. anaesthetics?
(COMEDK- 2013) a) Aa fibres b) A/3 fibres
42. Pain, temperature, touch from pulp are carried by which c) Ao fibres d) C fibres
one of the following type nerve fibres? (GCET-14)
a) A alpha b) A beta 53. The most important diagnostic test to differentiate
c) A delta d) Fibres between an Acute periapical Abscess and a Phoenix
(AP- 2013) Abscess is
43. According to WHO classification of periradicular a) Pulp testing b) Radiographs
diseases, code number category K04.4 is c) History d) Percussion
a) Acute apical periodontitis (GCET-14)
b) Chronic apical periodontitis 54. Which of the following irrigant is used during opening
c) Periapical abscess. with sinus of a tooth with acute pulpal abscess?
d) Periapical abscess. with sinus to maxillary antrum a) Normal saline b) Hydrogen peroxide
(KAR- 2013) c) Sodium hypochlorite d) EDTA
44. Reason for limitation of indirect pulp capping in deep (AIPG-14)
caries is 55. In root canal treatment, which of the following agent is
a) Internal resorption b) Calcification used to destroy enterococcus faecalis?
c) Chronic pulp inflammation a) NS b) 2% chlorhexidine
d) All of the above c) Ca(OH)2 + 2% chlorhexidine
(KAR- 2013) d) 5.25% hypochlorite
45. Predisposing clinical condition for endodontic flare up? (PGI JUNE-13)
a) Acute periapical abscess 56. The amount of total polyamines and putrescine is higher
b) Acute apical periodontitis in the following conditions
c) Asymptomatic necrotic pulp with periapical lesion a) Necrotic pulps of teeth that are painful to percussion or
d) Pain and swelling since the treatment with spontaneous pain
(AIIMS MAY-13) b) Vital pulps of teeth that are asymptomatic
46. Nerve fibres in pulp are c) Pulps of teeth that have chronic hyperplastic pulpitis
a) Sympathetic efferent post ganglionic d) Pulp associated with an impacted tooth
b) Parasympathetic post ganglionic (COMEDK-15)
c) Motor fibres 5 7. Pain of cracked tooth syndrome is differentiated by
d) Both autonomic parasympathetic and sympathetic fibres a) Pain on biting
(AIIMS MAY-13) b) Pain on release of bite pressme
4 7. Reactive oxygen metabolite in lysosomes is released by c) Continuous pain
a) Peroxidase b) NADPH oxidase d) Decrease threshold for pain
c) Superoxide dismutase d) Catalase (PGI JUNE-14)
(AIIMS NOV-13) 58. In a patient with acute pulpitis, it is difficult for the
48. Short sharp pain of tooth transmitted to brain by patient to locate the pain. This is because?
a) A delta fibers b) C fibers a) No nociceptors are present in the pulp
c) Beta fibers d) Gamma fibers b) Less proprioceptors are present in the pulp
(AIIMS NOV-13) c) Patient cannot speak d) Patient is not cooperative
(AIIMS MAY-14)

40) C 41) C 42) C 43) A 44) None 45) C 46) A 47) B 48) A 49) D 50) C 51) C 52) D
53) B 54) A 55) B&D 56) A 57) B 58) B
ENDODONTICS

2. DISEASES OF PULP AND PERIRADICULAR TISSUES - ANSWERS

1. 'A' [Grossman 12th ed 102) 13. 'A' [Grossman 12th ed 88)


In acute apical abscess, pain is due to build up of pressure In reversible pulpitis, pain produced by thermal stimulus
periapically due to accumulation of pus. Analgesics and disappears after the stimulus is removed whereas in
antibiotics will have little role in these cases. Access cavity irreversible pulpitis, the pain persists even after the
preparation will permit the drainage and eliminate the need removal of stimulus.
for a surgical incision.
14. 'C' [Grossman 12th ed 88)
2. 'C' [Grossman 12th ed 90)
Internal resorption is also known as "Pink tooth of 15. 'B' [Grossman 12th e d 44,76)
mummery" or "odontoclastoma".
16. 'D' [Grossman 12th ed 86,87 )
3. 'D' [Grossman 12th ed 81) In irreversible pulpitis, pain develops following sudden
temperature changes, particularly cold. The pain is
4. 'B' [Grossman 12th ed 87) spontaneous, sharp, piercing or shooting type. The pain is
The condition is acute irreversible pulpitis. The treatment increased when the patient bends over or lies down. This
consists of pulpectomy and placement of suitable is due to change in intrapulpal pressure when the patient
medicament. In posterior teeth, the coronal pulp is changes his position.
removed and formocresol dressing is given as an emergency
procedure. 17. 'A' [Grossman 12th ed 102)

5. 'C' [Grossman 12th ed 102) 18. 'B' [Cohen 8th ed 430)


The innerveration of the pulp includes both afferent neurons
6. 'A' [Grossman 12th ed 112) which conducts sensory impulses (trigeminal nerve) and
The sinus tract prevents exacerbation of the lesion by autonomic (efferent) fibers which provide neurogenic
providing continuous drainage of periradicular lesion. modulation of the microcirculation and perhaps regulate
dentinogenesis. Efferent nerves found in the dental pulp
The origin of sinus tract is traced by inserting a gutta perch are sympathetic post ganglionic fibres.
cone into the sinus tract and taking radiograph. The sinus
tract ultimately heals by granulation tissue after endodontic 19. 'C' [Grossman 12th ed 100)
therapy.
20. 'A' [Cohen 9th ed 504-506]
7. 'A' [Grossman 12th ed 88) The cause of pulpal calcification is largely unknown.
Calcification may occur around a nidus of degenerating cells,
8. 'C' [Grossman 12th ed 90) blood thrombi or collagen fibers. Thus degenerating cells
Internal resorption is due to asymptomatic and chronic serve as a nidus for initiation of calcification. Calcification
irreversible pulpitis. The treatment consists of pulpectomy in mature pulp is assumed to be related to aging process.
and calcium hydroxide dressing which is periodically But sometimes, numerous concentric pulp stones with
renewed until the defect is repaired. no apparent cause are seen in all the teeth of young
individuals. In such cases, the appearance of pulp stones
9. 'A' [Grossman 12th ed 80) may be ascribed to individual biological characteristics.
Anachoresis refers to attraction or fixation of blood borne
bacteria in areas of inflammation. The most common bacteria 21. 'A' [Ingle 5th ed 69, Tab.3-1 / Grossman 12th ed 81)
isolated from root canals include streptococcus viridans The bacteria cultured and identified from the root canals of
followed by staphylococcus albus and staphylococcus aureus. teeth with apical radiolucencies are:
Bacteria Incidence
10. 'C' [Grossman 12th ed 105]
• Fusobacterium
11. 'A' [Grossman 12th ed 92) • Streptococcus 40 - 48%
(predominantly a-hemolytic
12. 'A' [Grossman 12th ed 86) viridans group)
Acute reversible pulpitis can be treated successfully by • Bacteroides
palliative procedures such as placing of ZOE cement. The
• Prevotella,
best treatment is prevention i.e., placing a protective base
under restoration, avoiding marginal leakage, reducing • Peptostreptococcus 31 - 35%
occlusal trauma, etc. When pain is present after proper • Lactobacillus
treatment, the condition is regarded as irreversible, and to • Eubacterium
such cases pulpecto my is the treatment of choice.
Dental ;lut.,e

• Fusobacterium ii) Lymphocytes, plasma cells and macrophages that die


in great number and disintegrate in chronic periapical
• Actinomyces lesions (See zone of irritation).
9 - 25%
• Porphyrononas
• Prevotella iii) The circulating plasma lipids.

22. ' B' [Cohen 9th ed 929) 23. 'C' [Grossman 12th ed 105]
The reaction of periradicular tissues to noxious products Phoenix abscess is an acute inflammatory reaction
of tissue necrosis, bacterial products and antigenic agents superimposed on an existing chronic lesion, such as cyst
from the root canal has been described by Fish. or granuloma. The exacerbation of a chronic lesion is most
commonly associated with the initiation of root canal
The four zones descliibed by Fish are: therapy in a completely asymptomatic tooth. In such a
• Zone of infection tooth, radiographs show well-defined periradicular lesions.
• Zone of contamination
• Zone of irritation 24. 'A'
Pain in acute irreversible pulpitis is more severe described
• Zone of stimulation as gnawing, boring or throbbing under constant pressure.
When no outlet is present, whether because of covering of
i) Zone of infection: decay or a filling or because of food packed into a small
• This is present in the center of the lesion. exposure in dentin pain can be more intense.
• It is characterized by PMNL's.
25. 'C' [Cohen gth ed 447 /Grossman 12th ed 93)
Uncommonly, the cellular elements of the pulp are
ii) Zone of contamination: replaced by fibrous connective tissue. It is assumed that
• It is characterized by round cell infiltration. pulp responds to noxious stimuli by accumulating large
• It demonstrates cellular destruction due to toxins from fiber bundles of collagen. Fibrosis indicates the reparative
the central zone or zone of infection. potential of pulp.
• Because of auto lysis and death of bone cells, the 26. 'A' [Cohen gth ed 35]
lacunae were empty. Odontogenic pain arises from the pulp and for the
• Lymphocytes are prevalent everywhere. periradicular tissues. These structures are fuctionally and
embryologically distinct and pain originating from each of
iii) Zone of irritation: them is perceived differently.
• This zone is characterized by macrophages and 27. 'D' [Farlex medical dictionary]
osteoclasts. Abnormally exaggerated subjective response to painful
• The collagen framework was digested by phagocytic stimuli is called hyperpathia. Allodynia is spontaneous
cells, the macrophages, while osteoclasts attacked the pain. Dysasthesia is a condition in which an unpleasant
bone tissue. This opens a gap in the bone all around the sensation is produced by ordinary stimuli.
center of lesion. That space becomes filled with PMNL'S.
• This zone demonstrates much activity preparatory to 28. 'A' [Grossman 12th ed 31]
repair.
29. 'C' [Grossman 121h ed 32)
A delta and C fibres of the pulp are nociceptors.
iv) Zone of stimulation: (Peripheral zone)
• This zone is characterized by fibroblasts and osteoblasts.
A delta fibers C fibres
• In this zone, the effects of toxins were mild enough to
be stimulant which results in laying down of collagen Myelinated Non myelinated
fibers around the zone of irritation that acts both as Large, fast conducting, low Small, high threshold fiber.
a wall of defense and as a scaffolding on which the threshold fibres
osteoblasts built new bone. Intimate association with Not involved with pulpo
• This new bone was built in irregular fashion. the odontoblastic cell layer dentinal complex. They are
and dentin (pulpo dentinal located centrally in the
Accumulation of cholesterol crystals in apical periodontitis complex) pulp stroma.
lesions with clinical significance in endodontics. These Quick, sharp momentary Dull, poorly localized pain
crystals are believed to be formed from cholesterol released by pain
Drilling, probing Pain is associated with
i) Disintegrating erythrocytes of stagnant blood vessels
hypersensitivity will cause inflammatory tissue
within the lesion. pain damage.
ENDODONTICS

30. ' B' [Cohen 6th ed 415] • MTA is the best mat erial as it provides proper seal and
Chronic pulpitis is difficult to diagnose because the patient strength to the tooth. Using white MTA discoloration of
does not have subjective symptoms; the tooth does not the tooth can also be prevented.
respond to percussion, and radiographs appear normal.
Carious lesions with previous history of episodes of pain 36. 'C' [Grossman 12th ed 310,134]
are the only factor.s by which a provisional diagnosis of Inflammation induces
chronic pulpitis can be made. Histopathology is the only
• Vasodilation
way by which definitive diagnosis can be made.
• Increased capillary pressure and
31. ' D' [Grossman 12th ed 78] • Increased vascular permeability
Enamel crack can be better disclosed by using a dye or by
transilluminating the tooth with a fibre optic. Strangulation means choking of blood vessels at the apical
foramen. Inflammation does not cause strangulation of
32. 'A' [Ingle 6th ed 663] pulp.
Cracked tooth syndrome accounts for many perplexing
diagnostic problems. Patient can experience intermittent 37. 'A' [Grossman 12th ed 109]
episodes of acute pain radiating over the entire side of the Chronic a pical abscess:
face. The pain may mimic pain in trigeminal neuralgia. • Asymptomatic or only slightly symptomatic if the sinus
tract is obstructed.
33. 'A' [Cohen 8th ed 34]
Hall mark sign of neuropathic pain is hyperalgesia. • Sinus tract is the hall mark. This allows continuous
drainage of pus forming in the periapical lesion through
Hyperalgesia has 3 characteristics: the oral mucosa or in rare cases through the skin.
1. Spontaneous pain • Most commonly associated with an apical radiolucency.
2. Allodynia i.e. decreased pain threshold
3. Increased response to painful stimuli. 38. 'K [Grossman 12th ed 31]
Normal hydrostatic pressure in pulp is 10mm of Hg. Irreversible
34. 'C' [Grossman 12th ed 90] changes start in pulp if pressure increases to 35 mm of Hg.
The cause of internal resorption is not known. But often
patients have a history of trauma. 39A. 'C' [Grossman 12th ed 77]
The pain in this case is felt on biting a hard object and then
Clinical features: on releasing the bite. This is characteristic of cracked tooth
or tooth infarction. This unique type of pain is referred to
• Mostly asymptomatic
as rebound or relief pain. This is due to sudden movement
• "Pink Spot" if lesion occurs in crown. of dentinal fluid when the fractured tooth portions move
• Radiographically, there is alteration or enlargement in independently, activating myelinated A-delta fibers in the
the appearance of the pulp chamber or the wall of the pulp and creating a rapid, acute pain response.
root canal due to the elastic activity.
• When desorption progresses into periodontal space, it is 3 98. ' D' [Grossman 12th ed 78]
difficult to differentiate from external resorption. Diagnostic test s for cracked tooth:
• Visual examination aided by use of transillumination.
Note: External resorption, the lesion is superimposed over
an unaltered canal in a radiograph or the outline of the • Also methylene blue dye is used to stain the crack.
pulp space appears through the radiolucent area caused by
• Biting test with the use of rubber wheels, cotton tip
the resorption.
applicators, moist cotton rolls and tooth sloth.
35. ' D' [Nisha Garg 2nd ed 462] • Cold test and EPT will provide information about the
• In internal resorption, obturating the void and status of pulp.
strengthening the root structure is very important.
• Radiographs
• Historically ZOE, Amalgam, GIC, and super EBA have
been used. • Ultrasound (Recent)
• GP has been popularly used to obturate the defect. The
canal apical to the defect is filled with solid GP. While,
3 9(. 'A' [Ingle 6th ed 671]
the resorptive area is usually filled with a material
Treatment of cracked teeth:
that will flow in the irregularities i.e. thermoplastic or
chemically plasticized GP. • Neither enamel nor dentin can be permanently reunited
once a fracture line develops.
• But GP, does not provide strength to the structure and
might also cause discoloration. • All efforts should be aimed at preventing separation of
the fragments and bacteria from colonizing the space.
Dental ;lut.,e

• Stabilize tooth with orthodontic band and wait for 2 K04.31 Residual cyst
weeks.
K04.82 Inflammatory paradental cyst
- If symptoms subside, proceed for fu ll coverage crowns.
- If symptoms persist or aggravated, proceed for RCT 44. 'None' [Essentials of Pediatric oral pathology by Mayur
and then crown . and Sweta Dixit Chaudary Pg 106]
• Extraction, if the tooth has poor prognosis. The question was wrongly framed as all the options are
• Also dentin adhesives, amalgam with retention on both limitations of direct pulp capping but not of indirect pulp
sides of infarctions have been advocated but with less capping; or else the fo urth option should be none of the
success. above instead of all of the above.
• CO2 and Nd:YAG lasers are in experimental stages.
Limitations of direct pulp capping in primary teeth:
• Internal resorption
40. 'C' [Oral medicine by Satish Candra & Chaleen Chandra • Calcification
1st ed 153] • Chronic pulp infection
• Necrosis
41. 'C' (Ingle 6th ed 629) • Interradicular involvement
• In maxillary sinusitis, the patients often feel pressure
in their cheeks and below their eyes. The pain increase 4 5, 'C' (Check Ingle 6th ed 266)
when patients bend over and place their heads below Endodontic flare up is defined as an acute exacerbation of
their knees or while lying down, usually on one side an asymptomatic pulp and / or periapical pathology after
more than the other. the initiation or continuation of RCT.
• The pain is dull aching type that is difficult to localize
or pinpoint. In accordance with the current AAE definition, the most
• Most of the times there is febrile illness. likely predisposing clinical condition for occurrence of
flare up appears to be the asymptomatic necrotic pulp with
42. ' C' [Bergenholtz text book of endodontics 2nd ed 33] periapical lesion.

Option A, B and D (symptomatic) are also predisposing


Conduction conditions for flare ups but not the most common
Fiber Diameter
Function velocity
type (um)
(m/Sec) 46. 'A' [Ingle 6th ed 136]
Motoneuro ns Types of nerve fibers in pulp
A alpha 12-20 70-120
Muscle afferents
Type Function
Mediation of touch A-beta Pressure, touch
A beta 5-12 30-70
and pressure
A-delta Pain
Mediation of pain,
A delta temperature and 2-5 5-30 C Pain
touch Post ganglionic sympathetic:
Mostly mediate (1) Neurogenic modulation of
C 0.4-1.2 0.5-2.5 microcirculation i.e., constriction of
pain Sympathetic
arterioles and decrease blood flow
43. 'A' [Text book of endodontics by Nisha and Amit Garg (2) Inflammatory reaction
2nd ed 27] (3) Regulation of dentinogenesis
WHO classification of periradicular disease Note: All sensory impulses generat ed from pulp tissue result
K04.4 Acute apical periodontitis in the sensation of pain only.
K04.5 Chronic apical periodontitis (apical granuloma)
47. 'B' (Check Cohen 10th ed 355]
K04.6 Periapical abscess with sinus
• Two types of free radicals are produced by neutrophils,
K04.60 Periapical abscess with sinus to maxillary sinus and macrophages that are activated by microbes,
K04.61 Periapical abscess with sinus to nasal cavity immune complexes and inflammatory mediators. They
K04.62 Periapical abscess with sinus to oral cavity are nitric oxide (NO) and Reactive Oxygen Species
(ROS) .
K04.63 Periapical abscess with sinus to skin
• ROS are synthesized via the NADPH Oxidase Pathway
K04.7 Periapical abscess with no sinus and NO is produced by nitric oxide synthase. Within the
K04.8 Radicular cyst (Periapical cyst) lysosomes of neutrophils, macrophages and other cells.
K04.80 Apical and lateral cyst • They destroy the phagocytosed microbes and necrotic
cells.
ENDODONTICS

Functions of ROS & NO. 52. 'D'


Low Levels:
Increases chemokines, cytokines there by potentiating the 53. 'B' (Grossman 12th ed 101)
inflammatory cascade.
54. 'A'
High levels:
- Causes tissue damage like endothelial damage breakdown 55. 'B' & 'D' [Check Explanation Below)
of extracellular matrix and direct injury to erythrocytes, Both NaoCL and Chlorhexidi ne possess and effective
parenchymal cells and tumor cells. antibacterial action against E. faecalis and their level of
- But various antioxidant protective mechanisms present effectiveness depends on :
in the blood and tissues minimise the toxicity of these • Their concentration and form: CHX is available in 0.2,
oxygen free radicals. 1 and 2% in liquid and gel forms. 2% CHX liquid is more
Eg: Catalase effective. 5.25% NaoCl is effective than the dilution
Superoxide dismutase and forms.
Glutathione
• Experimental procedures, model of the micro-organism,
contact duration.
48. 'A' (Check Explanation of Q. No. 29)
• Nerve fibres that respond to electric pulp tester - In many research studies, the results are controversy
A delta with some advocating the superiority of NaoCl while the
• Nerve fibres that are excited by hydrodynamic events remaining advocating ch lorhexidine.
such as air drying and drilling - A delta
• Nerve fibres involved in hypersensitivity - A delta 56. 'A' (Check Explanation Below)
Polyamines are the bacterial end products produced as
• Nerve fibres involved in inflammation - C fibres.
a result of amino acid decarboxylation by decarboxylase
enzymes. Examples of polymines are Putrescine,
49. 'D' [Ingle 6th ed 63 1)
Spermine, Spermicidine and cadaverine. When compared
Occassionally apical periodontitis of maxillary first molar
to asymptomatic teet h, they are detected in greater
will not penetrate the maxillary sinus floor but will
amounts in infected root canals with spontaneous pain,
rather displace the periosteum (periosteal stripping) with
and swelling. It has been hypothesized that intracanal
subsequent formation of a thin layer of new bone on the
polyamines, especially putrescine, might leak out through
periphery of the diseases process. This has been referred
the apical foramen leading to development of pain.
as halo-shadow. This will resolve following endodontic
treatment. Note: Halo-effect is seen with vertical root
5 7. 'B' (Check Explanation of Q.39A)
fract ures.
58. 'B' [Nista Garg 3rd ed 97]
50. 'C' (Cohen 10th ed 560)
The pulp does not contain proprioceptor nerve endings. It
Bacteria play an important role in the etiology of
is difficult for the patient to localize pain unless it involves
inflammation of the periapical tissue. Sundquist classic
the apical periodontal ligament.
study demonstrated that necrotic pulp tissue and stagnant
tissue fluid can not induce and sustain apical periodontitis
in the absence of infection.

51. 'C' (Grossman 12th ed 98)


Apical periodontitis (AP) may occur in a vital or nonvital
tooth.

Causes of AP in a vital tooth:


• Abnormal occlusal contacts
• High points in restoration
• Wedging of foreign object between the teeth
• Traumatic blow t o teeth (Eg: Concussion)

Causes of AP in a nonvital tooth:


• Sequelae of pulpal diseases
• Iatrogenic
- apical extrusion of debris through apical
foramen during RCT.
- Pushing irriga111ts or medicaments orobturating material
through apical foramen.
Dental ;lut.,e

3. BIOMECHANICS
1. The root canal instrument most likely to break during 11, First instrument inserted into the root canal through
use is a the access cavity is
a) File b) Broach a) Barbed broach b) H file
c) Reamers d) Spreaders c) Kfile d) Any of the above
(MAN-97) (KAR -99)
2. While root canal biomechanical preparation apical 12. Difference between physical characteristic of reamers
constriction is enlarged to 50 sized instrument. You and files is:
would like to a) The cross-section of reamers in square and files are
a) Use reverse filling technique triangular in cross-section
b) Reinstrument with larger instrument 1mm short of b) The number of flutes on the blade are more in files than
previous preparation in reamers
c) Fit a 50 mm cone and condense carefully c) The reamers have more flutes in the blade
d) Fill with 40 sized cone d) Files have two superficial grooves to produce flutes in a
(AIIMS-91) double helix design
3. The first instrument to extirpate pulp is (AIPG -95, AIIMS-06, 07)
a) Barbed broach b) K-type file 13. Gates-Glidden drill is:
c) Reamer d) H-type file a) Flexible and used at Low speed
(COMEDK-04, MAN-97) b) Flexible and used at high speed
4. The ideal instrument that is used to widen the canal is c) Inflexible and used at low speed
a) Broach b) File d) Inflexible and used at high speed
c) Fissure bur d) Reamer (KAR-97)
(AIIMS-95, KAR-98) 14. Which of the following is a disinfectant, sterilizing
5. The most effective means of reducing microbes in root agent and an antiseptic?
canal a) Sodium hypochlorite b) Glutaraldehyde
a) Frequent medication of root canal c) Hydrogen peroxide d) Methylated spirit
b) Complete debridement through instrumentation (PGI-95)
c) Systemic antibiotics d) All of the above 15. Power driven instrument is
(PGI-95, AIPG-93) a) Gates glidden drill b) Reamers
6. A giromatic hand piece for endodontic instrumentation c) Giromatic d) Racer-Engine
operates by a: (AIPG -01)
a) Rotating motion b) Oscillating motion 16. Endosonics utilizes which instrument?
c) Zig-Zag motion d) All of the above a) H file b) K file
(KAR-97) c) Endo box d) Reamers
7. File is manufactured from a: (AIPG-94)
a) Triangular blank b) Round blank 17. The best instrument for cutting dentin is
c) Square blank d) Diamond shaped blank a) Reamer b) File
(KAR-97) c) Broach d) Explorer
8. The difference between a reamer and a file is in:
a) The number of flutes 18. The instrument most likely to push debris through the
b) The number of flutes and shape of the shaft apex is
c) The number of flutes and the size of the shaft a) Reamer b) File
d) The shape and size of the shaft c) Broach d) Smooth explorer
(KAR-97)
9. An endodontic reamer is least likely to fracture when 19. D2 on the endodontic file indicates:
inserted in a: a) Diameter at tip of instrument
a) Dry, clean canal b) Wet, clean canal b) Angle of instrument
c) Wet, debris laden canal d) Dry, debris laden canal c) Length of instrument
(KAR-97) d) 1/lOOth mm at end of cutting blade
10. To prevent the fracture of an endodontic instrument (AIPG-94)
the number of turns given inside the canal in a single 20. Working length of root canal is
motion should be: a) 0.5 to 1 mm short of radiographic apex
a) Less than 1/4 of a turn b) 1/4 to 1/z turn b) 1 to 2 mm short of apex
c) 112 of a turn d) More than 3/4 c) Exactly at apex d) 1 to 2 mm apical to apex
(KAR-97) (AIPG-98)

1) B 2} B 3) B 4) B 5) B 6) B 7) C 8) B 9) B 10) B 11) C 12) B 13) C


14) A 15) A 16) B 17) A 18) B 19) D 20) A
ENDODONTICS

21. Gross debridement, lubrication, microbial destruction 30. Root canal is flooded with antiseptic so as to:
and tissue dissolution are done by this irrigating solution a) Prevent the removal of the shavings from the root canal
a) Sodium hypochlorite b) Normal saline b) Where there is an absence of instrumentation
c) RC prep d) Hydrogen peroxide c) Prevent infection during instrumentation
(KAR-02) d) Avoid instrumentation
22. Reamers and files differ mainly in: (PGI-01)
a) Cross sectional form b) Colour coding 31. Purple coloured reamer is numbered:
c) Number of flutes along the blade a) 08 b) 10
d) Instrument formula c) 15 d) 30
(AIPG-96) (AIPG-97)
23. Instrumentation well short of the apical foramen can 32. During root canal instrumentation, the apical
result in: constriction is violated and enlarged to a size no.
a) Forcing necrotic material into the periapical region 50 instrument, which of the following is appropriate
b) Shelfing or ledgi ng the canal treatment for this canal:
c) Traumatizing the periapical tissues a) Obturate with gutta-percha and surgically remove
d) Grossly enlarging the apical foramen extruded excess
(AIPG-90) b) Fit a No.SO gutta-percha cone with sealer and carefully
24. The safest instrument for removing the pulp from the condense
very fine canal is: c) Fit a No.SO gutta-percha cone with sealer but don't
a) Barbed broach b) Small K type file condense so as to keep the G. P. from being pushed into
c) Tempered universal Headstroem file the foramen
d) Smooth broach d) Reinstrument 1 mm short of the fora men to a size No.60
(AIPG-90) or larger and obdurate using an appropriat ely sized
25. Reaming action of the reamer is master cone
a) Pushing regenerating (AIPG-02)
b) Pushing, rotating & regenerating 33. Barbed broach is used for:
c) Pushing & pulling d) None a) Extirpation of pulp b) Enlargement of canal
(PGI-02) c) Obturation of canals d) All of the above
26. Which of the following is correct about root canal (PGIT-98, 97; AIPG-02)
preparation. 34. EDTA, which is an active ingredient in root canal
a) Retention form is provided by 2 mm of near parallel preparation, has its principal action in
walls at the end of root preparation (apical 2/3 rd) to a) Decalcification of dentine
provide tug back of master cone. b) Dissolution of necrotic debris
b) Resistance form is provided by parallel walls of apical c) Lubrication of the canal during instrumentation
seat to preserve the integrity of natural constriction d) All of the above are correct
(apical matrix) (APPSC-99)
c) Extension form - all instrumentation should terminate 35. Hypochlorite does not cause
0.5 mm of short of radiographic apex a) Bleaching b) remove smear layer
d) All of the above are correct. c) Lubricating d) Flush out debris
(PGI-97)
27. is helpful in making radicular access: 36. Angle of Tip of an endodontic instrument
a) Gates glidden drill b) K-File a) 75 + 15 b) 75 - 15
c) K-reamer d) Round bur c) 75 ± 15° d) 80°
(AIIMS-97) (AP-02)
28. Recapitulation: 3 7. Smallest no file is
a) Uses large fi les to flare the root canal a) 4 b) 6
b) Removing debris with a smaller instrument than the c) 8 d) 10
instrument that goes to the apex (AP-02)
c) Use of larger files to flare the root canal but in between 38. Sequence of endodontic instrumentation
use of small instruments to confirm the initial length a) Broach-reamer-file b) Reamer-Broach-file
d) The use of various types of reamers and files to enlarge c) File-Reamer-Broach d) File-broach-reamer
the root canal (PGI-03)
(PGI-01) 39. Most important step in root canal preparation?
29. When pulp cannot be extirpated in narrow canals, which a) Biomechanical preparation
of the following can be used b) Obturation
a) Obtundant b) Astringent c) Access canal opening d) None of the above
c) Haemostatic d) Mummifying agent (PGI-95)
(APPSC-01)

21) A 22) C 23) B 24) B 25) B 26) D 27) D 28) C 29) D 30) C 31) B 32) D 33) A
34) D 35) B 36) C 37) B 38) C 39) A
Dental ;lut.,e

40. Most important criteria in using irrigating solution is b) Can be sterilized by heating
a) Volume of irrigation b) Thickness of syringe used c) With time they become britUe
c) Concentration of irrigant d) It has two forms a and p
d) None of above (AIIMS-05)
(PGI-97) 51. H-file is used to
41. In endodontic practice, periapical radiographs are very a) Locate the canal orifice b) Flare the root canal
useful in assessing the c) For irrigation of root canal
a) Length of the root canal d) All of the above
b) Presence of infection in the pulp (PGl-05, COMED-05)
c) Vitality of the pulp d) All of the above 52. Gutta Percha can effectively be sterilized by?
(MAN-95) a) Hot salt sterilizer b) Chemical solutions
42. The most common error seen during access cavity c) Autoclaving d) Dry heat
preparation of maxillary and mandibular incisors is: (AIIMS-07)
a) Perforation of the lingual or labial walls 53. Instrument No. 40 represents
b) Gouging of pulpal contents a) Diameter at D1 b) Diameter at D2
c) Tooth fracture d) All of the above c) Diameter at D1+D2 d) Diameter at D2-D1
(PGI-99) (PGI-05)
43. RC-prep is a combination of? 54. The Last solution used in root canal treatment to avoid
a) EDTA with carbamide peroxide painful surgical emphysema (AP-08) before dressing is
b) EDTA with hydrogen peroxide a) Normal saline b) EDTA
c) EDTA with sodium hypochlorite c) Hydrogen peroxide d) Sodium hypochlorite
d) EDTA with urea peroxide (PGI- 05)
(APPG-15, COMED-06) 55. Location of root canals is best done by endodontic _ :
44. Resistance form of endodontics is: a) Probe b) Explorer
a) Resists movement of gutta-percha in apical area c) Ultrasonic system d) File
b) To allow use of spreader in lateral condensation (AP-05)
c) Fracture of root while vertical condensation 56. A 15% solution of EDTA has a pH of:
d) None of the above a) 5.5 b) 7.3
(AIIMS-06) c) 7 .9 d) 7
45. Peso reamer is used for: (KAR-04)
a) To remove lingual shoulder of anterior teeth 5 7. According to ISO standardization, root canal files and
b) To enlarge root canal orifices reamers should have a taper of:
c) Post space preparation d) to remove gutta percha a) 0.02 mm per cm b) 0.02 mm per mm
(COMEDK-06) c) 0.03 mm per cm d) 0.03 mm per mm
46. Lentulospirals are used for:
a) Locating canal orifice b) Pulp extirpation 58. Cross section of K-flex file is: (OR) K-flex file is
c) Enlarging coronal third root canal traditionally made form
d) Application of root canal sealer a) Rhomboidal b) Circular
c) Spiral d) Double helix
4 7. Following movement of tip is seen in Giromatic hand piece: (AP-06)
a) Reciprocating 90" arc b) 180" arc 59. All of the following are true regarding removal of dentin
c) Oscillating to and fro moments smear layer during root canal preparation except:
d) Vertical displacement of tip a) Removal of dentin smear layer is done by using 37% EDTA
(AP-06) b) Dentin smear layer contains debris and microoroganism
48. The primary function of access openings is to c) Removal of dentin smear layer removes microorganism
a) Facilitate canal medication d) Removal of dentin smear layer is enhanced by EDTA
b) Provide good access for irrigation (AIPG-07)
c) Aid in locating canal orifices 60. Recently used irrigation solution is:
d) Provide straight line access to the apex a) MTAB b) MTAC
(AIPG-06) c) MTAD d) MTAE
49. The Massermann Kit technique for removing solid objects: (PGI-06)
a) is very short procedure 61. Best method to sterilize absorbent points is:
b) is more successful in posterior teeth than in anterior ones a) Autoclaving b) Chemical sterilizer
c) requires frequent radiographic monitoring c) Glass bead /salt sterilizer
d) is better than the ultrasonic technique d) Passing in ethanol flame
(AP-07)
50. About gutta-percha all are true, except:
a) It contains 60-70% gutta-percha and 20% ZnO

40) A 41) A 42) A 43) D 44) A 45) C 46) D 47) A 48) D 49) C 50) A 51) B 52) B
53) A 54) D 55) B 56) B 57) B 58) A 59) A 60) C 61) C
ENDODONTICS

62. Which of the following best describes the resistance b) Length determination
form in endodontic root canal preparation? c) Obturating with gutta percha in Mcspadden technique
a) The establishment of apical tug back resistance to d) Endodontic implants
prevent the apical root fracture during obturation (KCET-09)
b) The cavity form which is obtained for application of 72. Ni-Ti instruments are flexible because of their-
condenser in vertical compaction a) Plasticity b) Rigidity
c) The cavity form which is designed for application of c) Superelastcity d) Hardness
condenser in lateral compaction (COMEDK-09)
d) Cavity form which is designed to obtain for entry of 73. Instrument having Shank with alternating cutting edges
condenser during obturation a) Real world endo b) Endotech endo. instrument
(AIIMS-06) c) RACE d) K-6
63. The maximum head diameter of ISO Gatesglidden drill# (AIPG-10)
1 is equivalent to: 7 4. All of the following may result in under filling of a root
a) No 50 file at Do b) No 40 fi le at Do canal EXCEPT
c) No 60 file at Do d) No 80 file at Do a) Apical perforation b) Edge formation
(KCET-07) c) Short length estimation
64. In young permanent teeth the best method of sensitivity d) Lateral perforation
testing for traumatized teeth is Carbondioxide snow (AP-09)
which can go up to temperature of 75. Electronic apex locator is used in Endodontics to
a) -20°( b) -30°( determine the
c) - 40°( d) - 78°( a) Root canal morphology b) Working length
(COMEDK-08) c) Pulp stone Location d) Pulp vitality
65. J.S. Quickfill is (COMED-10)
a) The original engine driven Mcspadden's compactor with 76. The most suitable instrument to remove gutta percha
guttapercha already coated from the root canal is
b) Like thermafill another obturator a) barbed broach b) peeso reamer
c) No different than original Mcspadden's compacter c) tapering fissure bur d) round bur
d) Chemically softened gutta-percha (AP-09)
(COMEDK-08) 77. In Ingle's method of Endodontic working length
66. Two pulp canals are usually found in determination, safety allowance is for
a) Mesial root of permanent mandibular first molar a) Possible image distortion or magnification
b) Distal root of permanent mandibular first molar b) To confirm with apical termination
c) Palatal root of permanent maxillary first molar c) Periapical bone resorption
d) Distal root of permanent mandibular second molar d) Recording final working length
(COMEDK-08) (COMEDK-10)
67. Rat tail file is also known as 78. The potentially hazardous irrigant causing an emergency
a) Barbed broach b) R file during treatment is
c) H fi le d) Rasp a) Saliva b) Chlorhexidine
(KAR-03) c) Sodium Hypochlorite d) Metrogyl
68. The root canal instrument most Likely to break during (AIPG-2011)
instrumentation is 79. In an endodontic file, D2 will be
a) H-file b) K-file a) At the tip of the instrument
c) Reamer d) Pathfinder b) At 2mm from the tip of the instrument
(AP-08, KCET-08) c) At 3mm from the tip of the instrument
69. In Endodontics Endotec is used d) At 16mm from the tip of the instrument
a) with Lateral condensation and heated gutta percha (AIPG-2011)
b) with cold lat eral compaction 80. The file which has a non-cutting side is
c) with vertical compaction a) K-files b) Safety K-files
d) with Chloro Percha Technique c) Hedstrom files d) Safety Hedstrom files
(COMEDK-08) (COMED 2012)
70. Step down technique is 81. Most root canals that have not been instrumented are
a) Preparation from apical to middle third too narrow to be reached effectively by
b) Preparation from apical to coronal third a) Microbes b) Disinfectants
c) Preparation from coronal to middle third c) Gutta percha d) None of the above
d) Preparation from coronal to apical third (BHU-2012)
(COMEDK-09) 82. Ultrasonic devices are instruments for the removal of
71. Hedstroem file is a,n instrument used for- dentin from root canal walls have proved to be
a) Canal enlargement a) Effective b) Disappointing

62) A 63) A 64) D 65) A 66) A 67) D 68) A 69) A 70) D 71) A 72) C 73) C 74) A
75) B 76) B 77) A 78) C 79) D 80) D 81) B 82) B
Dental ;lut.,e

c) Lethal d) None of the above a) Gauze soaked in hypochlorite solution


(BHU-2012)) b) 15 seconds in glass bead sterilizer
83. H type endodontic instrument has a spiral edge arranged c) 30 seconds in glass bead sterilizer
to allow cutting during d) Chlorhexidine
a) Pushing stroke b) Pulling stroke (AIIMS MAY 2012)
c) Circular stroke d) Oblong stroke 95. In root canal treatment, the canal orifices are located
(BHU-2012) by:
84. Ultrasonic devices operate at a) Endodontic excavator b) DG-16 Endodontic explorer
a) 25-30 KHz b) 35-40 KHz c) C+ file d) H-file
c) 2-3 KHz d) 5-10 KHz (COMEDK- 2013)
(AP-2012) 96. The number on the handle of a root canal instrument
85. When it is made up of same steel, size 55, which will be denotes
more prone to fracture: a) Length of the instrument in 1/10 of a millimetre
a) Triangular reamer b) Rhomboid cross section file b) Length of the instrument in millimetres
c) Square cross section file d) Headstrom file c) Diameter at the tip in 1/100 of a millimetre
(AIIMS-2011) d) Diameter at the tip in 1/10 of a millimeter
86. Taper present in 20 number GT file is (GCET-14)
a) 0.06, 0.08, 0.10, 0.12 b) 0.006, 0.008, 0.01, 0.012 97. File with R phase technology
c) 0.6, 0.8, 1.0, 1.2 d) 0.03, 0.04, 0.05, 0.06 a) Protaper b) Wave one
(PGI-2011) c) Twisted d) Race
87. In K-file the most common type of motion is (GCET-14)
a) Watch winding b) Linear 98. When root canal instruments (files and reamers) are
c) Rotating through 90° arch sterilized in a glassbead sterilizer at 450 degree F., they
d) Linear & clockwise should be placed in the sterilizer for
(PGI-2011) a) 2 seconds b) 5 seconds
88. File commonly used forflaring of the canal & retreatment: c) 10 seconds d) 20 seconds
a) H-file b) K-file (AP-14)
c) K-flex file d) Lentulospiral 99. The primary principle of treating endodontic periapical
(PGI-2011) lesions is
89. In NiTi rotary electric handpiece, rotation should be a) To eradicate irritants in the root canal system
a) 350-450 b) 250-300 b) To reduce the inflammation in the periapex
c) 150-300 d) 100-150 c) To maintain the health of the periodontium
(PGI-2011) d) To relieve the symptoms
90. In order to complete a cutting circle of canal wall, the (COMEDK-14)
triangular shaft instrument requires: 100. Instrument used for lateral condensation of gutta perch
a) One-fourth turn b) One-third turn along the canal wall
c) One-half turn d) One full turn a) Barbed broach b) Plugger
(NEET-2013) c) Reamer d) Spreader
91. The significance of free eugenol in root canal sealer- (GCET-14)
cements is an increase in: 101. Instruments used for removal of lingual shoulder and
a) Dimensional stability b) Setting time enlarge orifices of a root canal are
c) Cytotoxicity d) Strength a) Gates Glidden drills b) Peeso reamers
(NEET-2013) c) Compactor d) H File
92. The objectives of cleaning and shaping are all the (COMEDK-14)
following EXCEPT 102. All of the following are true about H-files EXCEPT
a) To debride the root canal a) They have good cutting efficiency
b) To shape/contour the root canal walls b) They are fragile and tend to break easily
c) To aid condensation of an inert filling material c) They are used in torquing action
d) Should keep the apical foramen as small as practical d) They have more positive rake angle
(KAR- 2013) (COMEDK-14)
93. The commonly used root canal irrigant, which was used 103. Which of the following is not a function of sodium
for irrigation wounds during world war 1 by Dakin is hypochlorite
a) Saline b) Hydrogen peroxide a) Organic tissue solvent b) Bleaching
c) Buffered sodium hypochlorite c) Antibacterial d) Inorganic tissue solvent
d) Distilled water (GCET-14)
(COMEDK- 2013) 104. Advantage of NiTi over K-files?
94. Cleaning of files in between of endodontic treatment is a) Increased flexibility b) Low resiliency
done by: c) Resistance to breakage

83) B 84) A 85) D 86) A 87) B 88) A 89) C 90) B 91) C 92) C 93) C 94) A 95) B
96) C 97) C 98) B 99) A 100) D 101) A 102) C 103) D 104) A
ENDODONTICS

d) Less expensive
(PGI JUNE-12)
105. Which file is used along with irrigation for ultrasonic
system?
a) 8 b) 10
c) 15 d) 20
(PGI JUNE-11)
106. The superelastic property of the NiTi wire is due to
a) Stress induced phase transition from the austenitic to
the martensitic phase
b) Temperature induced phase transition from the
martensitic to t he aust enitic phase
c) Stress induced phase transition from the martensitic to
the austenitic phase
d) Temperature induced phase transition from the
austenitic to the martensitic phase
(KERALA-2015)
107. In case of retreatment which is ideal for removal of
G.P.?
a) H-file b) K-file
c) GT-file d) Lentulospiral
(PGI JUNE-2011)
108. What is the number of Gate Glidden drill used in mid
canal region?
a) 5-6 b) 3
c) 2 d) 1
(PGI JUN E-2013)
109. Apex locator reading 00?
a) 0.5-1 mm b) 0.75-1 mm
c) 0.25-0.MM d) 0.5-0.75 mm
(PGI JUNE-2013)
110. All the following are the objectives of access cavity
preparation EXCEPT
a) To remove all caries
b) To completely unroof the pulp chamber
c) To achieve straight line access t o apical foramen
d) To have converging axial walls for preservation of toot h
struct ure
(PGI JUN E-2014)
111. The shape of head of a Gates Glidden drill is
a) Flame shaped b) Cone shaped
c) Ball shaped d) Diamond shaped
(COM ED K-15)
112. Anderson and Hillman in 1971 introduced this into
orthodontics?
a) ~ NiTi b) NiTinol
c) Chinese NiTi d) Sentalloy
(PGI JUN E-2013)
113. Schielder principle is
a) Minimal enlarged apical foramen
b) Shape of t he canal can be changed
c) Use of 3-5 times larger than first binding fi le
d) Original canal curvature should be altered to make it
compatible with obturating material
(AIIMS MAY-14)

105) C 106) A 107) A 108) B 109) A 110) D 111) A 112) B 113) A


Dental ;lut.,e

3. BIOMECHANICS - ANSWERS
1. ' B' [Grossman 12th ed 228) 10. 'B' [Grossman 12th ed 230)
Broaches break easily, if they bind in root canal. Because of The instruments should be used with a 1/4 - 1/z turn and
this, they should not be inserted into root canal until the withdrawn with a pull stroke to prevent their fracture.
canal has been enlarged to a size of No. 20 or 25 reamer or file.
11. 'C'[Grossman 12th ed 241)
2. ' B' [Grossman 12th ed 257) Once the canal is penetrated, smooth broach or 10 to 20
This procedure maintains apical constriction and prevents numbered k-files are used for exploration. These instruments
over extension of obturation material. are flexible enough to follow root curvatures and stiff
enough to reach the root apex.
3. ' B' [Grossman 12th ed 241)
Broaches should not be inserted unless the root canal is 12. 'B' [Grossman 12th ed 228)
enlarged to a size of 20 or 25 reamer or file, since they break Less cutting
easily. To avoid this k-type tile is used to extirpate pulp, Conventional
Square blank efficiency than
k-type fi le is flexible enough to follow the root curvatures file
reamer
and stiff enough to reach the root apex.
Rhomboidal/
High flexibility
4. ' B' [Grossman 12th ed 226) diamond shaped
K-Flex file and cutting
File contains more number of flutes and is a four sided with alternating
efficiency
instrument because of which it is highly resistant to torque. high and low flutes
The fi le contains more metal and is least likely to break in Made from round Higher cutting
Headstroem
a root canal. The barbed broach is used for extirpation of blank and contains efficiency and more
or H-Files
pulp and most likely to break in a root canal WHEN TWISTED spiral flutes prone to fracture
MORE than 90° (PGI -95) Made from round Less cutting
blank to produce efficiency, more
Unifi le
REAMER FILE flutes in double resistance to
Made up of triangular blank Made up of square blank helix pattern fracture
Contains less number of Contains more number of 13. 'C' [Grossman 12th ed 232)
flutes for a unit length flut es for a unit length Gates Glidden drill and Peeso reamer are the two types of
High cutting efficiency and Less cutting efficiency and power driven instruments.
more prone to fracture than more resistant to torque
file Gates-Glidden drill is used:
Used with pushing and Used with rasping or • to remove lingual shoulder (PGI - 96) during access
rotating motion pulling motion cavity preparation in anterior teeth.
• to en large root canal orifices..
5. ' B' [Grossman 12th ed 263] • for shaping cervical third of R.C in step-back preparation.

6. ' B' [Grossman 12th ed 230) Peso reamer is used for post space preparation. Both
Giromatic and Racer, both are contra-angle engine driven Gates Glidden drill and peso reamer are aggressive cutting
instruments. Giromatic hand piece uses a barbed broach or instruments and inflexible and should be used at low speed
reamer through 90° reciprocating arc at a speed of 1000 with extreme caution.
cycles/min.
14. 'A' [Grossman 12th ed 226)
The racer hand piece uses a standard file and oscillates the Sodium hypochlorite is most effective among five halogen
file in root canal. These instruments can be used for opening antiseptics. It acts as
root canals but should not be used for canal preparation • lubricant during instrumentation in root canal.
because root canals prepared by hand instruments are • solvent for dissolution of pulp
superior in shape and smoothness.
• antiseptic and disinfectant by combining both
7. ' C' [Grossman 12th ed 228) protoplasm of bacterial cell and destroying it.
• bleaching agent by releasing nascent oxygen.
8. ' B' [Grossman 12th ed 228)
15. 'A' [Grossman 12th ed 23 2)
9. ' B' [Grossman 12th ed 251)
Root canal instruments cut dentin more effectively in a wet 16. 'B' [Grossman 12th ed 231)
environment and the wet debris and dentin are removed Ultrasonic and sonic instruments are used for cleaning and
from the canal by clinging to instrument. shaping root canals.
ENDODONTICS

The ultrasonic instrnment contains a magentostrictive hand The reamers were used with a pushing-rotating motion and
piece which hold a K-fi le or diamond file. It is operated at files were used with a rasping or pulling motion.
20000 - 25000/sec.
26. ' D'
The sonic instruments operate at 1500 - 6500 cycles/min
(25-110/sec.). Ultrasonic system uses sodium hypochlorite 27. ' D' (Grossman 12th ed 182, 188,193,210)
as irrigant, whereas the sonic system uses water as an
irrigant. 28. 'C' (Grossman 12th ed 251)
Recapitulation is returning to a small instrument from time
The irrigating solution used with ultrasonic to time before advancing to larger size. Recapitulation
instruments used for preparation of the root canal is prevents the packing of dentin filling and ensure patency of
(COMEDK-15) root canal through apical foramen .
a) Sodium hypochlorite b) Normal saline
c) Hydrogen peroxide d) Chloramine 29. ' D'

17. 'A' [Check Explanation Below) 30. 'C' [Grossman 12th ed 258)
Reamer contain sharp cutting edges that are spaced further
31. ' B' (Grossman 12th ed 227)
apart than the fi le and therefore it is engaged in dentin
more rapidly and readily facilitates dentin removal. Colour Size
Pink 6
18. ' B' (Check Explanation Below]
The files contain more cutting blades, which are closer Gray 8
together. So there is more chance of debris to be pushed in Purple 10
front of instrument rather than to be caught in the blades White 15
to remove.
Yellow 20
19. ' D' (Grossman 12th ed 226/ Ingle 6th ed 813) Red 25
According to new ISO standards three diameter are specified. Blue 30
Dl at the tip, D2 at the end of the cutting portion and D3
is 3mm from the tip. Green 35
Black 40
20. 'A' (Grossma n 12th ed 247)
All instrumentation procedures should terminate 0.5mm to 32. ' D' (Check Explanation Below]
1mm from radiographic apex t o end in sound dentine. This This results in proper resistance form by maintaining apical
prevents accidental perforation of apex and ensures firm constriction and prevents over extension of obturation
"apical matrix", which is an artificially produced ledge in material.
the apical root cana L.
33. 'A' (Grossman 12th ed 227)
21. 'A' (Grossman 12th ed 226) Classification of endodontic instruments

22. 'C' [Grossman 12th ed 228) Smooth broaches and


Exploring instruments
Files contain more number of flutes than reamers. Though endodontic explores
traditional files are made from a square blank, recent files Extirpating or debriding To remove pulp, debris or
are made in different forms to increase the functional instruments other foreign material
efficiency and fracture resistance. Enlarging instruments or
Reamers and fi les
shaping instruments
Eg: K-flex file - Rhomboidal or diamond shaped cross-
Plu ggers, spreader and
sectional form. Obturating instruments
lentulospirals
23. ' B' [Grossman 12th ed 484]
34. ' D' (Grossma n 12th e d 268]
• Over instrumentation causes forcing of debris into EDTA is used at a concentration of 15% at Ph 7.3. EDTA-C
periapical area (A) contains EDTA and Cetavlon, a quarternary ammonium
• Instrumentation short of apex causes shelfing or [edging compound used for disinfection. A combination of EDTA and
of canal. urea peroxide is known as R-C prep. Disodium salt of EDTA
• Instrumentation with large instruments causes enlarging is used in endodontic therapy for dissolving pulp stones.
of apical foramen.
35. ' B' (Grossman 12th ed 268)
24. ' B' [Grossman 12th ed 241)
36. 'C' (Grossman 12th ed 226]
25. ' B' (Grossman 12th ed 251)
Dental ;lut.,e

37. ' B' [Grossman 12th ed 227) • These nearly parallel walls (retention form) ensure
the form seating of this principle point. These final 2
38. ' C' [Check Explanation Below] to 3mm of cavity are the most crucial for meticulous
Once the canal is penetrated, smooth broach or 10 to 20 care in preparation. This is where the sealing against
numbered K-files are used for exploration . Broaches should future leakage or percolation into the canal takes
not be inserted unless the root canal is enlarged to a size place. This is also the region where accessory or
of 20 or 25 reamer or tile, as they break easily. lateral canals are most apt to be present.

39. 'A' [Grossman 12th ed 263)


v) Resistance form:
Biomechanical preparation should provide smooth, funnel
shaped tapered walls for obturation. It also eliminates • Resistance to overfilling ·is the primary objective of
microorganisms from canal surface. resistance form . This can lbe achieved by terminating
the canal preparation at the denti nocemental
40. 'A' [Grossman 12th ed 266,270) junction which is approximately 0.5mm from the
outer surface of the root.
41. 'A' [Ingle 6th ed 554] • Beyond this point. we are dealing with the tissues
of t he periodontal ligament space, but not the pulp.
42. 'A' [Grossman 12th ed 182, 203) • The tiny remaining portion of the canal (beyond DCJ)
is not properly cleaned and may contain bacteria
43. ' D' [Grossman 12th ed 268) and packed debris. This section of the canal should
be cleaned but not shaped with fine instruments
44. 'A' [Ingle 5th ed 432] such as No.10 or 15 tiles. This action is known as
The principles of root canal preparation are: establishing apical patency.
i) Outline form
ii) Convenience form vi) Extension for prevention:
iii) Toilet of the cavity • The extension of the cavity from the crown to apex is
iv) Retention form necessary to ensure prevention of future problems.
Peripheral en largement of the canal, to remove all
v) Resistance form
of the debris, followed by obturation is the primary
vi) Extension for prevention preventive method.

i) Outline form: 45. 'C' [Grossman 12th ed 232)


• Must be correctly shaped to establish complete
access for instrumentation from cavity margin to 46. 'D' [Cohen 9th ed 244, 245]
apex. Lentulospirals are used for placement of sealer, cement and
calcium hydroxide. It must be operated clockwise in the
• It includes meticulous cleaning of the walls of the
handpiece and started or stopped outside the root canal.
cavity until they feel glassy-smooth, accompanied
If started in the canal, it may cut into the wall of the
by continuous irrigation and thorough debridement.
root canal and break. This instrument effectively drives the
• At the coronal margin of the cavity, the outline form paste into the root canal. Endodontic files, paper points,
must be continually evaluated by monitoring the and syringes also are commonly used to place sealer in the
tension of the endodontic instruments against the root canal system.
margin of the cavity.
47. 'A' [Grossman 12th ed 230/ Refer Q. No. 6]
ii) Convenience form:
• The access should be expanded so that the 48. 'D' [Cohen 9th ed 175, Ingle 6th ed 877]
instruments do not bind, especially when larger and
less flexible instruments are used. 49. 'C' [Grossman 12th ed 487]
Brasseler endo extractor Kit, Masserman Kit, extractor
system and Separated Instrument Retrieval System
iii) Toilet of cavity:
(S.I.R.) are used in retrieval of broken instruments from the
• Success depends to a great extent on whether root canal. Masserman technique uses trephine burs and a
unreachable debris laden with viable bacteria with a specific extraction device. Although effective, this technique
source of substrate (accessory canal or microleakage) may require removal of excessive .amount of radicular dentin,
to survive. Hence the importance of thorough leading to root weakening and the risk of perforation.
douching through irrigation, toilet of the cavity. Therefore the instrument should be used with caution.

iv) Retention form : 50. 'A' [Grossman 12th ed 279]


• It is recommended that the master gutta-percha cone Guttapercha contain 20% of gutta-percha and 66% of ZnO
should tit tightly in the apical 2 to 3mm of the canal. as main components.
ENDODONTICS

51. 'B' [Grossman 12th ed 230) 61. 'C' [Grossman 11th ed 138)
H-file or Hedstroem tile is used to finish the instrumentation Absorbent points, broaches, tiles and other root canal
of the coronal third of the root canal in step back technique. instruments are sterilized immediately before use in a hot-
salt sterilizer.
The popular use of the headstrom file is: (NEET-2013)
62. 'K [Ingle 5th ed 472)
a) To flare the orifices of canal
b) To establish a circular canal in the apical third
c) To prepare the canal for a post restoration 63. 'A'[Cohen 10th ed 292)
d) for rotary cutting of the canal wall According to Cohen, t he first cross-sectional diameter at
the first rake angle of any file is labeled Do. The point
1mm coronal to Do is Dl. Thus a size 1150 inst rument has a
52. 'B' [Grossman 12th ed 281)
Gutta percha cones may be kept sterile in Screw-capped vials diamet er of 0.1mm at Do and a corresponding diameter of
0.82mm at D16 [0.5mm + (16 x 0.02mm)].
containing alcohol. A Guttapercha cone freshly removed from
the manufacturers box should be immersed in 5.2% sodium
hypochlorite for 1 min., (PGI June-12) then rinse t he cone Gates Glidden (GG) instruments are manufactured in a set
with hydrogen peroxide and dry it between 2 layers of sterile and numbered from 1 to 6, with corresponding diameters of
0.5 t o 1.5mm. That means 111 GG
gauze.
instrument will have a maximum diameter of 0.5mm (which
53. 'A' [Refer synopsis/ Grossman 12th ed 226)
is equal to Do of 1150 fi le) andl 116 instruments will have a
54. ' D' [Grossman 11th ed 189) maximum diameter of 1.5mm.
Alternate irrigations with sodium hypoch lorite and hydrogen
64. ' D' [Grossman 12th ed 70)
peroxide have been advocated in root canal preparation.
The advantages of alternating solutions of 3% hydrogen
65. 'A' [Ingle 5th ed 628)
peroxide and 5.2% sodium hypochlorite are -
The various techniques of heat compaction of
• The effervescent reaction, in which it mechanically Guttapercha are:
"bubbles" and pushes debris out of the root canal.
• Vertical compaction of guttapercha that has been heat
• The solvent action of the sodium hypochlorite on the softened in the canal.
organic debris of the pulp tissue.
• Injectable guttapercha techniques
• The disinfecting and bleaching action by both
• Thermo-mechanical compaction
solutions.
• Core carrier techniques
Finally, sodium chlorite should be used before dressing
because hydrogen peroxide can react with pulp debris and Technique Examples
blood to form gas (surgical emphysema) that will cause • Obtura II
Injectable
continuous tooth pain.
(high heat technique) • Inject R-fill
55. ' B' [Cohen 9th ed 17 4, Grossman 12th ed 240) • Thermafill
Core carrier
Endodontic explorers followed by No. 10 to 20 K-files are • Densfill
most commonly used diagnostic and exploring instruments
• MC Spadden compactor
for identification of canal orifices. Thermomechanical
• JS Quickfill
(wit h rotary
56. ' B' [Grossman 11th ed 225) instruments) • Multiphase II Pac Mac
compactors
57. ' B' [Grossman 12th ed 226)
Refer synopsis Using Mcspadden compactor, guttapercha was softened
with rotary action of the instrument in t he canal and is
58. 'A' [Grossman 12th ed 230) moved apically and laterally within the prepared system.
Refer Q.No. 12
In JS Quickfill, the compactors are precoated with
59. 'A' [Grossman 12th ed 268) guttapercha; other features same as Mc Spadden compactor.
Removal of smear layer is done by 15% of solution of EDTA
Multiphase II Pac Mac compactors utilize injectable system
60. 'C' [Cohen 9th ed 244, 246, Grossman 12th ed 270) of coating compactors.
MTAD is a recently introduced irrigation solution. It is a
Mixture of Tetracycline (Doxycycline), an Acid (citric acid) 66. 'K [Grossman 12th ed 210)
and a Detergent. Invitro experiments indicate that MTAD has
potential for removal of smear layer and highly effective in 6 7. ' D'
killing E. fecalis.
68. 'A' [Grossman 12th ed 230)
Dental ;lut.,e

69. 'A' [Ingle 6th ed 1062] working length 0.5-lmm to confirm the length to apical
Cold or lateral compaction is easy and quick to perform while constriction
superior density is gained through vertical compaction of
warm guttapercha. 78. 'C' [Grossman 12th ed 493]
Extrusion of sodium hypochlorine into periradicular tissues
Martin developed a device that appears to achieve the best causes t he following effects due to its tissue dissolving effect.
qualities of both techniques called Endotec II. He claims a) Immediate effects:
that Endotec combines the best of the two most popular
• Swelling
obturation techniques, warm/ vertical and the relatively
simplicity of lateral compaction. It contains battery • Pain
powered, heat controlled spreader/plugger t hat ensures • Ecchymosis
complete thermo-softening of any type of guttapercha.
b) Long term effects:
Endotec II is? (PGI June-12) • Parasthesia
a) Obturation system b) Irrigating device • Scarring
c) Biomechanical preparation • Muscle weakness
d) Antibiotic
• Extrusion of H202 causing tissue emphysema.
70. ' D' (Ingle 5th ed 470, Grossman 12th ed 258]
Treatment includes antibiotics, analgesics, intramuscular
Step-back: Beginning the preparation at the apex and
working back up the canal coronally with larger and larger steriods and surgical intervention with wound debridement.
instruments.
79. 'D' [Grossman 12th ed 226]
Refer point No.6 under endodontic instruments in synopsis.
Step-down: Often called "Crown-down" approach, begins
coronally and the preparation is advanced apically, using
80. 'D' [Grossma n 12th ed 230]
smaller and smaller instruments, finally terminating at the
apical stop. Modifications of H-files:
i) Safety hed stroem: It is H-file with a non cutting side
71. 'A' [Ingle 5th ed 483, Grossman 12th ed 230] in order to prevent ledging in curved canals.

7 2. ' C' [Ingle 5th ed 487, Grossman 12th ed 233] ii) Hyflex file: It has 'S' shape cross section instead of
single helix tear drop cross section of the H-file.
73. ' C' (Cohen 9th ed 312, Grossman 12th ed 235 ] iii) Unifiles: It has double helix cross section . These are
RACE (Reamer with a lternating cutting edges) no longer used.
• Light microscopic imaging of the file shows twisted iv) S-file: It is a variation of unifile
areas (a feature of conventional files) alternating with
straight areas. 81. 'B' [Grossman 12th ed 264]
• Cross sections are triangular or square for # 0.02
instruments with size # 15 and# 20 tips. 82. 'B' [Grossman 12th ed 231)
• The lengths of cutting parts vary from 9 - 16mm. Ultrasonic instrumentation never fulfilled clinical
expectations as a primary method to prepare root canals.
• Alternating cutting edges eliminate the screwing into
They were never predictable, effective of efficient as the
the dentin.
conventional methods. But ultrasonics are promising for:
• Sharp cutting efficiency, combination of triangular
section and alternating cutting edges ensures efficient • Activating irrigants
evacuation of debris and safety tip ensures guidance/ • Removal of smear layer
centring in the canal. • Removal of posts and silver points

74. 'A' [Ingle 6th ed 1123 ] 83. 'B' [Grossman 12th ed 23 0)


H-type file machines the root canal wall when the instrument
75. ' B' [Ingle 5th ed 517 / Grossman 12th ed 249] is pulled and has no cutting effect when pushed.

76. ' B' [Ingle 6th ed 144 / Grossman 12th ed 232] 84. 'A' [Grossman 12th ed 230)
But in key both A and B were given as correct answers.
77. 'A' [Ingle 6th ed 929]
Safety allowance (done before working length x-ray) is the 85. 'D' [Grossma n 12th ed 230]
substraction of 1mm Length from the diagnostic radiograph
before taking the working length. This is for the possible 86. 'I([Grossman 12th ed 234]
image distortion or magnification in the diagnostic film . The profile GT set consists of 4 instruments, having tip size
Safety allowance factor (done after WL x-ray) is adjusting of 20, with taper 0.06, 0.08, 0.10 and 0.12 mm/ mm.
ENDODONTICS

0.12 mm/mm taper is also available in sizes No. 35, No. 50 Sodium hypochlorite:
and No. 70. • It is used in concentrations varying from 0.5 to 7%
• 2.5% and 5% are most commonly used.
87. ' B' (Grossman 12th ed 251)
Files are predominantly used with filling or rasping or push • Very potent antimicrobial agent
or pull action (linear) in which there is little or no rotation • Effectively dissolves pulpal remnants
in the root canals. • 0.5% Naocl which is buffered with a bicarbonate buffer
• Reaming: Motion involving clockwise rotation of is called Dakins solution.
instrument • Unbuffered Naocl has a pH of 11, while buffered form
• Watch winding: Back and forth oscillation right and has pH of 9
left. The angle of rotation is 30-60°C
• Wat ch winding and pull: Disadvantages:
Used with H-files. Watch winding motion can be used • Inability to remove smear layer
with both file and reamer. But most common type of • Unpleasant taste
motion of K-file is push and pull (Linear) motion. • Toxicity

88. 'A' (Cohen 8th ed 528, 888) 94. 'A' [Grossman 11th ed 196)
H-files remove large amount of dentin. The gutta-percha in During instrumentation in the root canal, debris is often
the canal can be heated and H-files can be used to remove collected on the instruments. This debris can clog the flutes
it in case of retreatment. of the instruments and decrease their efficiency. This debris
can be removed by squeezing the blade between the layers
89. 'C' [Grossman 12th ed 234) of gauze and turning the instrument counter clockwise.
The rotational speeds for various Ni-Ti instruments
fall common ly under 150-300 RPM. Light speed rotary 95. ' B' [Cohen 10 ed 156/ Grossman 12th ed 240)
instrumentation system requires 750-2000 RPM. Endodontic explorers have a long tip with a sharp point that
is used to locate the opening of small canals (pathfinder)
90. ' B' [Practical dentis t ry by Balwant Rai Pg 17) during endodontic procedures. They are made of spring
The cross sectional shape of the shaft of the instrument is tempered stainless steel so they retain their shape and
important. The triangular shaft requires a one-third (120 resist breakage. Both tips of DG16 endodontic explorers are
degrees) rotation of the instrument to complete a cutting 16mm in length and set at different angles 45° and 70°.
circle of the root canal wall. Square shaft required a one-
fourth turn (90 degrees) to do the same task. Jw 17 serves the same purpose but its thinner stiffer tip can
be used for identifying calcified canals. A # 17 operative
91. 'C' [Ingle 6th ed 1039/Cohen 10th ed 264] explorer is useful for det ecting any remaining pulp chamber
Free eugenol always remains the mass of zinc oxide eugenol roof, particularly in the area of a pulp horn.
sealers. This contributes to cytotoxicity and irrigation to
the tissues if the free eugenol content increases in the 96. 'C' [Grossman 12th ed 226)
mass.
97. 'C' [Check Synopsis}
92. 'C' [Grossman 12th ed 221, Nisha Garg 2nd ed 232)
Schilder objectives of cleaning a nd shaping of root 98. ' B'
canals:
a) Mechanical objectives: 99. 'A' [Grossman 12th ed 121)
• Continuous tapering canal shape, with the narrowest Check explanation of Q.No.92
cross-sectional diameter apically and the widest
diameter coronally. 100. ' O' [Grossman 12th e d 284]
Spreaders are used for lateral condensation and pluggers
• Walls should taper evenly towards apex
are used for vertical condensation
• To give the prepared root canal the quality of flow.
• Should keep the apical foramen as small as practical. 101. 'K [Grossman 12th ed 232)
GG burs are used to remove the lingual shoulder in anterior
teeth and enlarging the orifices of root canal. They provide
b) Biologic objectives:
straight light access to the initial point of curvature of root
• To debride and disinfect root canal system canal.
• Necrotic debris should not be forced periapically
• Sufficient space for intracanal medicaments and 102. 'C' [Grossman 12th ed 230)
irrigants should be created. H-files cut the dentin in outward pull stroke. They should
never be used in torquing action.
93. 'C' [Ingle 6th ed 998)
103. ' O' [Check Explanation of Q.. No 93)
120 ~~ Dental :f~e

104. 'A' [Nisha Garg 3 rd ed 157] Stress


The shape memory and superelasticity contribute to the Austenite - -- -- -- Martensite
Super elasticity
following advantages of Nili over stainless steel.

Advantages: 107, 'A' (Check Explanation of Q.No.88]


• More flexibility
108. 'B' [Harty's endodontics in clinical practise 4th ed 270]
• Able to conform to canal curvature Gates glidden drills are available in sizes 1 to 6 (with
• Low modulus of elasticity corresponding diameters 0.5 to 1.5 mm). They should
• Good resiliency always be used in the straighter part of the canal. Two
procedural sequences have been proposed.
• Corrosion resistance
• Softer than stainless steel a) Step down - Use of Larger GG first, progressing to
smaller ones.
Disadvantage s: b) Step back - Starts with small drill and progresses to
larger ones.
• Poor cutting efficiency
• Dot no show signs of deformation before they fracture • GG 6 and GG 5 should only be used on the walls of the
• Less fracture resistance access cavity.
• GG 4 no deeper than the canal orifice
105. 'C' [Cohen 10th ed 256] • GG 3 may be used to the mid canal region
After the root canal is shaped, a small file (size 15) is • GG 2 to the beginning of canal curvature
introduced as far apically as possible in the root canal filled
• GG 1 is very fragile, may be used at ultra-low speeds
with irrigant and ultrasonically activated. This No.15 size
provided it is loose in the canal.
file or wire moves freely and the irrigant can penetrate more
easily into the apical part of the canal.
109. 'A' [Ingle 61h ed 852]
0.0 or Apex mark with or without audible signal is the most
Files of sizes 20, 25 may produce less accoustic streaming.
Their use is restricted to very wide canals. accurate apical reference point achieved with an electronic
apex locator. This means it is within 0.5- 1 mm of the apical
106. 'A' [Phillips 1'1 SA ed 435/ Proffit 5th ed 363] constriction. With root ZX apex locator, this accuracy is
reported t o be 95-100%
Austenitic NiTi is the high temperature, low stress form,
while martensitic NiTi is the low temperature high stress
110. 'D' [Cohen 10th ed 150]
form. Transformation between austentic and martensitic
The axial walls should not be convergent. They should
forms of NiTi can be induced by both tern perature and stress.
be divergent and smooth to eliminate any restrictive
The uniqueness of NiTi is that t ransition between the two
interference that could compromise the achievement of
structures is fully reversible and occurs at a remarkably low
straight line access.
tern perature.
111. 'A' [Nisha Garg 3rd ed 155]
This phase transition allows NiTi to exhibit two properties
GG drills are flame (elliptical) shaped burs wit h a Latch
found in no other dental materials-shape memory and super
attachment.
elasticity.
112. 'B' [Check Explanation Below]
Shape Memory (Thermal reaction) refers to the ability of
In 1971, Anderson and Hillman introduced NiTinol in
the material to remember its original shape after being
Orthodontics. In 1988, Walia etal in traduced Nili in
plastically deformed while still in the martensitic form. This
endodontics.
is temperature induced change.
113. 'A' (Check Explanation of Q.No. 92]
Austenitic
Cooling causes After Heating
plastic j I
changed to
deformation "" origina l shape
Martensitic

Super elasticity (Mechanical reaction) refers to the very Large


reversible strains that NiTi wire can withstand due to the
martensitic-austenite phase transition. Materials displaying
super elasticity are austenite alloys that undergo t ransition
to martensite in response to stress. It is a mechanical (stress)
analogue to the thermally induced shape memory effect.
ENDODONTICS

4. DISINFECTION AND OBTURATION


1. Obturation of a root canal should achieve d) Injection molding technique
a) Tug-back b) hermetic seal (AIPG -01)
c) Fluid-free seal d) All of the above 11, Endo sealer containing polyketone is
(MAN-2K) a) Diaket b) Cavit
2. Mc Spadden compactor is of shape: c) Ah 26 d) Hydron
a) Reverse blade of K-file b) Reverse blade of H-file (AP-2K,03)
c) Endosonic instrument d) None of the above 12. The success of a root canal filling is best assessed
(AIIMS-97) a) Clinical observation
3. Activity of all disinfectants is: b) Size of gutta-percha cone used
a) Reduced by organic debris or blood c) Radiographs
b) Increased with heat d) By the diameter of root canal reamer
c) Hastened by adding 5% oxygen (KAR-98)
d) Nullified by ethylene dioxide gas 13. High heat obturation technique refers to:
(KAR -02) a) Thermofill technique b) Ultrafil technique
4. The purpose of a root canal sealer is to: c) Sectional filling d) Obtura II technique
a) Seal the tubules of the dentine (KAR-02)
b) Stimulate healing in periapical region 14. The function of a root canal sealer is:
c) Prevent discoloration a) To fill the gap between the tooth and the fi lling
d) Fill the space between solid core material and the pulp b) As an antiseptic
canal walls c) Aids in healing d) Provide strength
(AIPG-99, PGI-99) (AIPG -99)
5. An apical radiolucent area present in central incisor 15. Gutta-percha is radio-opaque due to presence of:
after 4 months of RCT is due to: a) Zinc oxide eugenol b) Barium sulphate
a) Inadequate obturation & leakage from main canal c) Potassium sulphate d) Barium oxide
b) Leakage from accessory canal (AIPG -99)
c) Leakage from gingival crevice 16. The crown of an RC treated tooth is fractured near the
d) Leakage from access opening gingival margin. The coronal end of the silver cone used
(AIIMS-97) in filling that canal is visible at that level. Treatment is:
6. G.P. is plasticized (softened) by a) To prepare post space alongside the silver cone using
a) Alcohol b) Chloroform burs and Peso reamers
c) Eugenol d) EDTA b) To grind away the coronal part of silver cone using
(AP-01) round burs or end cutting burs
7. Best material for obturating a root canal of a tooth is: c) To remove the silver cone and re-treat the canal using a
a) Thermoplastic GP b) Silver cone gutta-percha; and then to create the needed post space
c) Resorbable paste d) GP with sealer d) To remove the silver cone, notch it, coat it with a freshly
(PGI-2K) mixed sealer replace it in the canal and twist off the
8. The micro organism previously present in the Periapical coronal segment
tissues following o bturation (AIPG-02)
a) Persist and stimulate the formation of a granuloma 17. Which of the following is incorrect regarding the use of
b) Are eliminated by the natural defenses silver points for root canal obturation:
c) Re-enter and re-infect the sterile canal and are removed a) Silver corrodes in tissue fluids
by surgery b) A post cannot be used if a silver point is cemented in
d) Are eliminated by the medicaments used in endodontic root canal
treatment. c) The use of step back method of root canal preparation
(AIIMS-2K) in a circular configuration is difficult to achieve in the
9. The largest component of gutta percha cones is apical segment with silver point
a) Gutta percha b) Zinc oxide d) It will not seal the middle and cervical parts
c) Resins and waxes d) Colouring agents (AIIMS-01, 2K)
(UPSC-01, TNPSC-99) 18. Main cause of failure of endodontic therapy:
10. The vertical fracture of root seen with gutta percha a) Improper biomechanical preparation
obturation is due to: b) Improper access cavity preparation
a) The vertical compression of warm GP points c) Incomplete obturation d) Over extended filing
b) Lateral condensation of GP points (PGI-98. 02; AP-04)
c) Use of spreader

1) D 2) B 3) A 4) D 5) A 6) B 7) D 8) B 9) B 10) A 11) A 12) A 13) D


14) A 15) B 16) C 17) B 18) C
Dental ;lut.,e

19. The main objective of obturation is: 30. The primary gutta-percha cone must fill the canal wall
a) Fill the canal and prevent apical percolation of fluids tightly in the:
b) Fill the canal and prevent discolouration of teeth a) apical third b) middle third
c) Fill the canal to give support to restoration c) cervical third d) entire canal
d) All of the above (COMED-05, AP-05)
(PGI-98) 31. The periapical radiograph of RCT treated tooth shows
20. Best method of condensation wiggly apical seal of master cone is due to:
a) Lateral condensation b) Thermoplasticized a) Apical perforation of master cone
c) Vertical condensation d) None of above b) Excess cement in apical region
(PGI-97) c) Obstruction of master cone at apical third
21. Which of the following can be used as root canal filling d) Short master cone in apical third
materials (AIPG-07)
a) Cements and plastic materials 32. An ideal result of endodontic treatment is deposition of
b) G.P. Points which of the following at the apex:
c) Silver points d) All of the above a) Bone b) Cementum deposition
(PGI-95) c) Dentin d) Periodontal ligment
22. Bismuth compounds are incorporated in root canal (AIPG-07)
filling materials because they are: 33. All are true for root canal sealer except:
a) Radio-opaque b) Germicidal a) Excessive use of sealer tends to extrusion in periapical
c) Radiolucent d) Adhesive areas
(AIPG-89) b) Sealer mediates immune reaction in perapical region
23. Most common cause of RCT failure c) Loss of sealer causes porosity between root filter and
a) Incomplete removal of PA cyst tooth surfaces
b) Non-obturation of accessory canals d) Sealer occupies t he space between root filler and tooth
c) Incomplete debridement with improper obturation surfaces
d) Large size of PA pathology (AIPG-07)
34. Which is not a eugenol sealer?
24. In post core which of following gutta-percha technique a) Grossman sealer b) AH-26
is used c) Tubli seal d) Wach's paste
a) Thermoplasticized b) Vertical (AIPG- 07)
c) Sectional d) Lateral 35. MTA is used for
(PGI-01) a) Apexification only of any dentition
25. Disinfection of root canal obtained with b) Apexification of primary tooth only
a) Irrigation b) Debridement c) Apexification of permanent tooth only
c) Intracanal medicament d) All of above d) Apexogenesis of any dentitio,n
(PGI-99) (BHU-07)
26. Calcium hydroxide is advocated as an inter appointment 36. When is an application of heated injected gutta percha
intracanal medication because of potentially beneficial
a) Its ability to dissolve necrotic tissue a) when there is an open apex
b) Its antimicrobial property b) when there are aberrations o-r irregularities of the canal
c) Its ability to stimulate hard tissue formation c) when the clinician cannot master Lateral condensation
d) Its powerful bleaching effect d) when the canals are curved and small after preparation
(COMEDK-04) (COMED K-08)
27. Rejuvenation of gutta-percha can be changed from a to 3 7. Addition of synthetic resins to zinc oxide eugenol
b form at cement is said to
a) 55 - 59° F b) 55 - 59° C a) Increase the strength b) Reduce solubility
c) 20 - 25° C d) 20 - 25° F c) Reduce film thickness d) None of the above
(AP-02) (AP- 08)
28. The flutes of McSpadden Compactor are similar to: 38. With obtura II system which has a high temperature
a) Lentulo Spiral fi llers b) Engine driven reamers gutta-percha but cools to
c) Flutes of Hedstorem in reverse a) 10°c b) 110°c
d) K-Flex file c) 130°c d) 160°c
(COMEDK-05) (COMEDK-08)
29. Fall in alkalinity of Ca(OH}2 when used as a root canal 39. Thermafil is
dressing starts at a) An endodontic obturater b) An enlarging instrument
a) 1 week b) 2 week c) An instrument to remove pulp
c) 4 week d) 12 week d) Is a cold gutta percha technique
(PGI-2011) (COMEDK-08)

19) A 20) A 21) D 22) A 23) C 24) C 25) D 26) B 27) B 28) C 29) A 30) A 31) D
32) B 33) B 34) B 35) D 36) B 37) A 38) A 39) A
ENDODONTICS

40. High heat obturation technique refers to b) Large canals of ant erior teeth
a) Thermofill b) Ultrafill c) Surgical cases in which root resection is anticipated
c) Sectional filling d) Obtura II d) Incomplete large or irregular canals of a young patient.
(KAR-02) (NEET-2013)
41. Which of the following is urethane dimethacrylate 51. The Lorothidol added in non-eugenol paste has the
(UDMA) resin based endosealer? function of
a) Endorez b) Real seal a) Bacteriost atic b) Fungicide
c) Raeko sealer d) Tubli seal c) Accelerator d) Preservative
(AIIMS-09) (COMEDK- 2013)
42. Most recent form of MTA used as an intracanal irrigant? 52. Advocated apical seal for dowel core is
a) MTA-A b) MTA-C a) 2 mm b) 5 mm
c) MTA-D d) MTA-E c) 8 mm d) 10 mm
(PGI-08) (COMEDK-14)
43. The advantage of silver point as obturating material is 53. The term TUG BACK is used to indicate
that it a) Good fit of accessory gutta-percha point at the apical
a) Can be used in fine tortuous canals third
b) Is easier t o retrieve if retreatment becomes necessary b) Good fit of master gutta-percha point at the apical third
c) Gives a perfect seal at the apical 1/3rd of the root canal c) Good fit of accessory cone in the main body of the canal
d) Provides better adaptation to canal walls compared to d) Good fit of the initial file at the apical third
GuttaPercha (COMED K-14)
(KCET-2011) 54. Extrusion of sealer from root apex will cause
44. Vertical compaction of warm gutta-percha technique a) Irritate tissue and delay healing
was given by? b) Stain periapical tissue
a) Grossman b) Ingle c) Resorb the root d) Lead to closure of apex
c) Schilder d) Cohen (PGI JUNE-13)
(AP-2011) 55. Which of the following needle is used for irrigation?
45. Intracanal medication is a) 23 b) 25
a) Necessary for complete canal clearing c) 27 d) 29
b) Generally contraindicated (PGI JUNE-11)
c) Prevents post-treatment pain 56. Length of spreader short of working length
d) Placed with paper point a) 0.5 mm b) 0.1 mm
(AP-2012) c) 1.5 mm d) 0.25 mm
46. Master cone is (PGI JUNE-13)
a) Fully snug fit with accurate working length 57. EDTA C is?
b) Used by master of staff a) EDTA plus cetavlon b) EDTGA plus cetrimide
c) Inst rument 1st using taking working lengt h c) EDTA plus ciprofloxacin d) EDTA texapon
d) Last file to fit loosely (PGI JUNE-2014)
(AIPG-2012)
47. AH plus & AH 26 are
a) Root canal cements b) Medicated root canal paste
c) Fillers d) Root canal filling material
(PGI-2011)
48. True about vertical compaction technique is:
a) Softened heat & vertical compaction in t hree dimensions
b) One master cone & then filling
c) Cold plugger is used to make space
d) Softened by chemical & condense with hot plugger
(NEET-2013)
49. Ideal quality of root canal filling material is:
a) Make a seal of apically & laterally placed canals
b) Shrink after insertion
c) Should not permit easy removal
d) None of the above
(NEET-2013)
50. Silver cone used as a root canal filling material is
indicated in
a) Narrow curved canals in which enlargement beyond a
No.20/25 instrument is unadvisable.

40) D 41) A 42) C 43) A 44) C 45) A&B 46) A 47) A 48) A 49) A 50) A 51) B 52) B
53) B 54) A 55) C 56) C 57) A
Dental ;lut.,e

4. DISINFECTION AND OBTURATION - ANSWERS


1. ' D' (Grossman 12th ed 283] 18. 'C' [Grossman 12th ed 278)
"Tug back" refer to apical seal fit of master cone. The
obturation material should provide tight apical seal or 19. 'A' [Grossman 12th ed 278)
fluid-free seal to prevent percolation of oral fluids into
periapical area. 20. 'A' [Grossman 12th ed 283-84)
i) Lateral Condensation:
2. ' B' [Grossman 12th ed 291)
• In this technique a primary or master gutta-percha
Uses of H-file
cone is placed in the canal and the remaining space
• Used for finishing coronal third of root canal is filled by accessory or secondary cones wit h the
• Used to flaring the fine and double curved root canals help of spreaders and plu.ggers.
• Used in Mcspadden compaction technique • Most traditionally used. However,in contemporary
endodontics it is not the best technique to achieve
3. 'A' [Ingle 6th ed 995] a three dimensional seal.

4. ' D' (Cohen 10th ed 359) ii) Vertical condensation or warm gutta percha technique.
• Introduced by Schilder
5. 'A' [Cohen 10th ed 3 49) • Used with step back technique of root canal
Inadequate obturation is the most frequent cause of RCT preparation
failure.
• After placing the guttapercha cone vertical
According to study done by Ingle and colleagues,58% of
force is applied with heat ed pluggers so that the
treatment failures were due to incomplete obturation.
guttapercha cones fill the lumen of the canal in all
the three dimensions upto the apical foramen .
6. ' B' [Grossman 12th ed 282)
Chemical solvents like chloroform, xylol and Eucalyptol • Risk of vertical root fracture is main disadvantage of
(plant product) softens and enables the gutta-percha to this technique.
flow into small and tortuous canals.
iii) Sectional method
7. ' D' (Grossman 12th ed 282] • This technique uses sections of gutta-percha cones
to fill a section of the root canal.
8. ' B' [Grossman 12th ed 304) • Void between the sections is the main disadvantage.

9. ' B' [Grossman 12th ed 279) iv) Mcspadden compaction methods:


Composition % Function • A rotating compacting instrument, which is similar
to hedstroem file with the flutes in reverse
Gutta percha 20% Matrix direction, attached to a slow speed contra angle
Zinc oxide 66% Filler hand piece with 8000 - 10000 r.p.m, creates heat.
Heavy metal sulfates 11% Radio pacifier • The compactor forces the softened guttapercha
(Bismuth sulphates) {AIPG -89) apically and laterally.
Waxes or resins 3% Plasticizer v) Chemically plasticized guttapercha
• Used in fine tortuous canals
10. 'A' [Grossman 12th ed 289]
• Guttapercha is softened by chemical solvents like
11, 'A' [Grossman 12th ed 284] chloroform, euclapytol or xylol and is forced into
fine, tortuous canal.
12. 'A'
vi) Thermoplastic technique
13. ' D' [Grossman 12th ed 292] • Uses more heat to increase the plasticity of
gutta percha and enable the operator to fill the root
14. 'A' [Cohen 10th ed 3 59) canal by using less pressure.

15. ' B' [Ingle 6th ed 1020) 21. 'D' [Grossma n 12th e d 283, 285)
Cements like ZOE, Calcium hydroxide are used as obturating
16. 'C' [Grossman 12th ed 440] materials in primary teeth. Silver points are indicated in
fine, tortuous canals.
17. ' B' [Grossman 12th e d 281)
Gutta percha is the most commonly used obturating material.
ENDODONTICS

It is difficult to introduce guttapercha into a narrow root • Root end filling material
canal and it does not seal a canal apically or laterally unless • Repair of perforations
it is combined with a root canal cement or sealer.
• Repair of incomplete vertical root fractures
22. 'A' [Grossman 12th ed 280) • Repair of root resorptions
Heavy metal compounds like bismuth are added as • Obturating material
radio pacifiers.
36. ' B' [Cohen 9th ed 382)
23. 'C' [Cohen 10th ed 349) Injectable guttapercha is especially beneficial when
managing canal irregularities such as fins, webs, cul-de-
24. 'C' [Grossman 11th ed 247) sacs, internal resorption, ( -shaped canals, accessory or
Silver cones are contraindicated in filling a root canal if the lateral canals.
tooth is to be restored with a post and core. Guttapercha is
indicated as obturating material in these cases. Ideally the It is not indicated in case of open apices where there is
length of dowel or post should be 2/3rd of the root canal. potential for extrusion and damaging the periodontium.

25. ' D' [Grossman 12th ed 27 2) 3 7. 'A' [Marzouk 1st ed 261)


Addition of synthetic resins (IRM, Kalzinol) increases the
26. 'B' [Grossman 12th ed 272) strength and decreases solubility.

27. 'B' [Grossman 12th ed 281) 38. 'A' [Grossman 12th ed 292,293)
Guttapercha is heated to 185 - 200°(, a needle is used
28. 'C' [Grossman 12th ed 291) to deliver the softened guttapercha. The guttapercha cools
Refer Q.No. 20 Mc Spadden compaction method. rapidly to 70°( and hardens within 1 min.

29. 'A' [Grossman 12th ed 273) 39. 'A' [Cohen 9th ed 386/ Grossman 12th ed 297)
Thermafill contains a centre carrier (plastic/ stainless steel)
30. 'A' [Ingle 5th ed 601, 602/ Grossman 12th ed 283, 284] which is grooved along 600 of their circumference and has
a coating of guttapercha. This is heated in a specific oven
31. 'D' [Still Searching] (Thermaprep oven) and used for obt uration.

32. 'B' [Ingle 5th ed 653) 40. ' D' [Cohen 8th ed 345/ Grossman 12th ed 292)
Following well-executed treatment, when infection is
terminated, cementoblasts and periodontal ligament slowly 41. 'A' [Cohen 9th ed 269)
resurface the damaged root and even close minor foramina Endorez is a visible - light curable urethane dimethacrylate
that no longer contain neurovascular bundles. resin based sealer. It is hydrophilic and penetrates deep into
dentinal tubules thereby redudng the microbial leakage.
33. 'B' [Ingle 5th ed 3 68, 369)
Root canal sealer container paraformaldehyde in any form is 42. 'C' [Cohen 9th ed 258, 322/ Grossman 12th ed 270)
strictly contraindicated. MTA-D is a recently introduced irrigation solution.
It is a mixture of Doxycycline, an acid (citric acid)
34. ' B' [Ingle 5th ed 581/ Grossman 12th ed 301,303) and a detergent. In-vitro experiments indicates that
EWT (Extended Working Time), Roth's sealer, Tubliseal, MTA-D has potential for removal of smear layer and highly
Wach's cement, MCS (Medicat ed canal sealer) and Nogenol effective in killing E-fecalis.
are different modifications of ZOE cements.
43. 'A' [Grossman 12th ed 281)
CRCS (Calciobiotic root canal sealer), Sealapex, Life, Apexit, As silver cone is stiffer t han gutta-percha, it can be easily
Imbiseal, Vitapex are calcium hydroxide sealers. inserted into a fine tortuous canal.

Diaket and AH-26 are examples of plastic and resin cement. 44. 'C' [Grossman 11th ed 287)
Ketac-endo is GIC cement developed for endodontics. The warm vertical condensation or warm gutta-percha
technique of filling root canals was introduced by Schilder
AH-26 is an epoxy resin containing a non-toxic hardener. Its with the objective of filling lateral and accessory canals
radiopacity is imparted to it by bismuth oxide. It has strong along with the main canal.
adhesive properties and contracts slightly while hardening.
45. 'A' and ' B' [Cohen 8th ed 546, 548)
35. ' D' [Ingle 6th ed 1315)
Applications of Mineral Trioxide Aggregate: 46. 'A' [Cohen 8th ed 319/ Grossman 12th ed 283)
• Vital pulp therapy or Apexogenesis of any dentition.
47. 'A' [Cohen 8th ed 553/ Grossman 12th ed 303]
• Apexification of permanent dentition
Dental ;lut.,e

48. 'A' [Explanation of Q. No.20] into the canal with the master-cone in place, it should
be within 2 mm of the working length. There appears to
49. 'A' [Grossman 12th ed 279] be a correlation between establishing a seal and spreader
Ideal requirements of a filling material are: penetration.
• Easy to introduce and easy to remove from the root
canal 57. 'A' [Check Explanation of Q.No.34)
• Seal canal laterally as well as apically
• Impervious to moisture
• Should not shrink after insertion
• Should be bactericidal
• Should not stain the tooth
• Should not stain irritate periapex
• Should be sterile

50. 'A' [Grossman 12th ed 281]

51. 'B' [Shanthipriya reddy Periodontics 2nd ed 296]


• 2 Types of periodontal pacts are available. They are ZOE
pack and non-eugenol pack.
• Non-eugenol pack is based on the reaction between a
metallic oxide and fatty-acids. It is supplied in 2 tubes.

Composition of non-eugenol pack:


• One tube contains zinc oxide, oil for plasticity, a gum
for cohesiveness and lorothiodol a fungicide.
• The other tube contains liquid coconut fatty acids
thickened with rosin and chlorothymol, a bacteriostatic
agent.

52. 'B' [Grossman 12th ed 447]


The minimum amount of GP that should be left during post
space preparation for adequate apical seal is 4-5 mm.

How much GP should be left to prevent leakage for


post placement? (PGI June-12)
a) 4 mm b) 2 mm
c) 1 mm d) 1.5 mm

53. ' B' [Grossman 12th ed 283]

54. 'A' [Cohen 10th ed 425]

55. 'C' [Cohen 10th ed 246]


• Larger diameter needles (23 to 24 gauge) are less
efficient during irrigation as they are placed at a greater
distance from the apex.
• Smaller diameter needles (24 to 30 guage) can be
placed closer to the apex and so they are efficient.
• 27 guage which has an external diameter of 0.40 mm
are the most commonly used needles in endodontics.
• Special 30 gauge needles like stropko Flexi tip (NiTi),
Max-I-probe, Navi tips are also available.

56. 'C' [Cohen 10th ed 368]


Spreader should be se lected such that it is to be inserted to
within 1-2 mm from the working length. When introduced
ENDODONTICS

5. BLEACHING
1. The most common consequence of bleaching non vital 11, The bleaching material used for night guard vital tooth
teeth is bleaching is
a) Discolouration b) Cervical resorption a) Superoxol
c) Apical periodontitis d) Root resorption b) Anesthetic ether and hydrochloric acid
(MAN-98) c) Sodium bicarbonate d) Carbamide peroxide
2. Su peroxol is (COMEDK-04)
a) 30% H202 12. Discoloration of the teeth results from
b) Combination of H202 + sodium per borate a) Decomposition of the pulp
c) Combination of HCL + H202 b) Filling materials
d) None of the above c) Haemorrhage following pulp extirpation
(MAN -02) d) All of the above
3. Home bleach is also called as
a) Walking bleach b) Thermocatalytic bleach 13. In case of bleaching technique to remove fluorosis stains
c) Power bleach d) All of the above a) 1% ether removes the surface debris
(MAN -2K) b) 36% hydrochloric acid etches the enamel
4. In the walking bleach technique: c) 30% H20 2 bleaches the enamel
a) It uses a heat treatment d) All the above are correct
b) It requires the patient to report in 24 hours
c) Can be done with 35 percent hydrogen peroxide 14. Which of the following is not responsible for endogenous
d) It uses a mixture of sodium perborate and hydrogen staining of teeth during development?
peroxide a) Tetracycline b) Rh incompatibility
(PGI-01) c) Neonatal liver disease d) Vitamin-( deficiency
5. When undertaking bleaching of vital teeth of the (AIIMS-06)
following are done except: 15. Which one of the following is used to bleach a discolored
a) Prophylaxis b) Use of protective eye glasses endodontically treated tooth?
c) Use of local anaesthesia a) Ether b) Chloroform
d) Polishing after treatment c) Superoxol d) Sodium hypochlorite
(AIPG-96) (AIPG- 06)
6. Tooth discoloration is due to: 16. The "in office" non vital bleaching technique is
a) Acute pulpal abscess b) Pulpal hyperemia a) Walking bleach
c) Pulpal death d) None of the above b) Power bleach technique
(AIPG -91) c) Thermo catalytic technique
7. Carbamide solution used for bleaching degrades into d) Night guard technique
a) 0.3% sodium perborate b) 30% hydrogen peroxide (AP-2012)
c) 3% hydrogen peroxide d) 30% sodium perborate 17. Following intracoronal bleaching immediate composite
(AIPG-97) restoration required, what has to be done:
8. Vital bleaching causes a) Treat with catalase
a) Internal resorption b) Cervical resorption b) Wait for 7 days is mandatory
c) External resorption d) Periapical periodontitis c) Treat with H202 for 3 minutes
(AP-2K) d) Not possible
9. A 12-year-old child comes to your office with a history (AIIMS-2011)
of long-term use of tetracycline. The anterior teeth are 18. A twenty year old female patient comes to the clinic
a mild yellowish brown. What method would you use to with discoloration of the upper right central incisor.
remove the stain? This tooth was intact and otherwise asymptomatic.
a) Hydrochloric acid pumice micro-abrasions (COMEDK- 2013)
b) At home bleaching method 18A. The most likely cause for the discoloration in this
c) Superoxol with or without heat patient would be
d) Composite resin veneers a) Pulp necrosis b) Dental resorption
(KAR -02) c) Internal resorption d) Microleakage
10. Night guard bleaching refers to:
a) Laser activated bleaching 188. The treatment most often employed for managing
b) Dentist prescribed home applied technique discoloration in a non-vital teeth is
c) Thermo bleaching d) Photo bleaching a) Microabrasion
(KAR-02)) b) Walking bleach

1) B 2) A 3) A 4) D 5) C 6) C 7) C 8) D 9) B 10) B 11) D 12) D 13) D


14) D 15) C 16) C 17) A 18A) A 186) B
Dental ;lut.,e

c) Over the counter bleaching


d) Veneer

18C. The common sequel following bleaching non vital teeth


are:
a) Ankylosis b) Periodontal pathology
c) External cervical resorption
d) Reinfection

180. Bleaching is indicated in which of the following clinical


situations
a) Dentin discolorations
b) Teeth with discolored composites
c) Tooth discoloration due to caries
d) Teeth with superficial discolorations

19. Glass ionomer cement is used as a barrier over gutta-


percha filing before bleaching an endodontically treated
discolored tooth.
a) To prevent bleaching agent from dissolving the gutta-
percha.
b) To prevent percolation of the bleaching agent into the
apical area
c) To prevent contamination of bleaching agent
d) To prevent discoloration of tooth from obturating
material.

20. Till what age tetracycline should not be given to prevent


discoloration
a) 3 year b) 8 year
c) 12 year d) 18 year
(AII MS MAY-13)
21. Minimum dosage of tetracycline which will show tooth
discoloration is
a) 5 mg/kg body weight b) 20 mg/kg body weight
c) 50 mg/kg body weight d) 80 mg/kg body weight
(AIIMS MAY-13)
22. Dentist prescribed home applied bleaching technique
uses
a) 35% Hydrogen Peroxide
b) 10% Carbamide peroxide
c) 18% Hydrochloric acid
d) Sodium perborate
(AP-14)
23. Microbrasion is a procedure in clinical orthodontics
performed to
a) Clean the bracket base
b) Clean the arch wire
c) Polishing the bracket
d) Removal of white spot lesions.
(COM ED K-15)
24. Discoloration of tooth in tetracycline therapy is because
of formation of?
a) Calcium orthophosphate
b) Dicalcium phosphate dihydrate
c) Calcium Oxide d) Tetracalcium phosphate
( PGI JUN E-12)

18C) C 18D) A 19) B 20) B 21) B 22) B 23) D 24) A


ENDODONTICS

5. BLEACHING - ANSWERS

1. ' B' (Grossman 12th ed 349/ Sturdevant 4th ed 608) 7. 'C' [Grossman 12th ed 347]
Cervical resorption is a common consequence of bleaching
of non-vital t eeth whereas apical periodontitis is a common 8. ' D' [Grossman 12th ed 349]
sequel of bleaching of vital teeth. The possibility of cervical
resorption can be reduced by sealing the root canal orifice 9. ' B' [Grossman 12th ed 353 J
with cavit to prevent percolation of bleaching agent or by Dentist prescribed home bleact,ing methods of vital teeth
keeping a paste of calcium hydroxide and sterile water in or night guard bleaching is treatment of choice.
the pulp chamber.
Ans 'A' is for microabrasion, a method used to remove
2. 'A' [Ingle 6th ed 1389] surface stains or defects. Ans ' C' is for non vital teeth.
Superoxol is a 30% solution of hydrogen peroxide by weight
and 100% by volume in pure distilled water. Superoxol is 10. ' B' [Sturdevant 4th ed 610]
mixed with sodium perborate into a paste and is used in
"walking bleach" procedure. 11, ' D' [Ingle 6th ed 1394J

3. 'A' [Sturdevant 4th ed 609/ Grossma n 12th ed 349) 12. ' D' [Grossman 12th ed 344]
I. NONVITAL BLEACHING TECHNIQUES.
13. ' D' [Grossman 12th ed 358]
• Thermo catalytic or in office technique
Composition of Mc-Inns solution (MAN - 02) used for
• Walking bleach or out of the office or home removal of endemic flourosis stains is
technique
Component Volume Function
1 part anesthetic 0.2 ml Removal of surface
II. VITAL BLEACHING TECHNIQUES.
ether debris
• Power bleach or in office technique.
5 parts of HCL (36%) 1ml Etching of enamel
• Night guard or dentist prescribed home technique.
s parts of H20 2 (30%) 1 ml Bleaching of enamel
Thermocat alytic or in office technique
It involves the placement of 35% of H202 liquid into the 14. ' D' [Grossman 12th ed 344]
debrided pulp chamber and acceleration of t he oxidation
process by placement of a heating instrument into the pulp 15. 'C' [Ingle 6th ed 1389)
chamber.
16. 'C' [Cohen 8th ed 755/ Grossman 12th ed 348 ]
Walking bleach or home technique Option 'B' is in office vital bleach technique. Option 'D' is
In this technique either superoxol i.e., 30% H202 by home vital bleach technique. 0 ption 't,; walking bleach, the
volume or a mixture of sodium perborate and superoxol is bleach material is kept in pulp chamber for up to 1 week
used. and is known as at home nonvital bleach technique.

Power bleach or in office technique 17. 'A' [Grossman 12th ed 352]


In this technique, 35% of H202 is used and the oxidation Composites should be delayed for 2-3 weeks after bleaching,
process is accelerated by applying heat or intense light . to allow for dissipation of residual peroxides that would
interfere with the bonding agents. Catalase treatment at
Night guard bleach or home technique the fina l visit may enhance removal of residual peroxides
In this technique 10-15% of carbamide peroxide is applied but it is still under investigation.
with the help of bleaching trays. Carbamide peroxide
degrades into 3% hydrogen peroxide (active ingredient) Sodium Ascorbate can also be used. It is a buffered form
and 7% urea. of vitamin C that consists of 90% Ascorbic acid bound to
10% sodium, a powerful antioxidant useful in eliminatimg
4. ' D' [Grossman 12th ed 349, Sturdevant 4th ed 609] residual oxygen following bleaching.

5. 'C' [Grossma n 12th ed 353, Sturdevant 4th ed 610] 18A. 'A' [Grossman 12th ed 345]
In this case, the discoloration is present in a single tooth.
6. 'C' [Grossma n 12th ed 345] So, pulp necrosis is the most Likely cause.
Pulpal death is most common cause of tooth discoloration.
There is breakdown of haemoglobin into various pigments 188. ' B' [Q.No.3 and 4]
like hemin, hematin, hemosiderin, hematoporphyrin.
Hydrogen sulfide produced by bacteria combines with these 18C. 'C' [Explanation of Q.No.1]
pigments and darkens the tooth.
Dental ;lut.,e

18D. 'A' [Grossman 12th ed 346]


Bleaching agents like H20 2 can penetrate dentin and release
oxygen that breaks clown the double bond of organic and
inorganic compounds inside the tubule. Options B, C and D
are contraindications of bleaching.

19. 'B' [Grossman 12th ed 349]


• Quality of seal in the apical part as well coronal part
of the root canal system is important to prevent
percolation of fluids.
• In non-vital bleaching, Cavit or GIC or MTA is placed as
a barrier material over the root canal filling material.
• The level of barrier should be 1mm incisal t o CEJ.
• This prevents percolation of bleaching agent into the
root canal fi lling and nearby into the periapex.
• This also confi nes t he bleaching agent to t he crown
thereby preventing cervical root resorption.

20. 'B' [Check KOT 6th ed 714)


Tetracycline when given from mid pregnancy t o 5 months
of extrauterine life, the deciduous teeth are affect ed with
discoloration. When given between 3 months to 6 years of
age, they affect the crowns of permanent anterior dentition.
They should be avoided from 4 months in utero to 8 years.

21. 'B' [Check Journal of Paediatric dentistry 1987;9(3)-


245-46)
Many clinical stuclies revealed that average dose
consumption of 20-35 mg/kg body weight / day during
tooth formation results in discoloration. Doses> 35 mg/kg/
day results in both discoloration and hypoplasia of enamel.

22. 'B' [Check Explanation of Q. No.3]

23. 'D' [Graber 4th ed 621)


After the braces are removed, the first step is treatment
of white spot lesions by allowing natural remineralization.
The second step is external bleaching to help camouflage
white spots. A t hird step for patients with severe white
spot is acid microabrasion to eliminate the external layer of
the lesion, followed by application of CPP-ACP (recaldent)
paste.

When demineralizing capacity of the oral fluids is exhausted


and white spots are established microabrasion is the optimal
way to remove superficial enamel opacities. By this one can
eliminate enamel stains with minimal enamel loss. This can
be achieved with a cl.lstom made gel consisting of 18% HCl,
pumice and glycerine. This is applied for 5 minutes with an
electric too brush and then rinsed. This process is repeated
2 to 3 times a month .

24. 'A' [Text book of oral medicine by Ghom 3rd ed 119)


Tetracycline causes discoloration during formation period.
It reacts with calcium to form calcium orthophosphate
complex. The minimum amount required t o produce
discoloration is 21 mg/kg/body weight.
ENDODONTICS

6. ENDODONTIC SURGERY
1. All of the following incisions are recognized, standard a) Envelope b) Rectangular
approaches for an apicoectomy EXCEPT. c) Semilunar d) Triangular
a) Cervical margin of teeth with buccal extensions (UPSC-01)
b) Figure of eight 10. The prognosis of the broken instrument in root canal is
c) Rectangular d) Semilunar good when
(MAN-94) a) Instrument is less than 20 nos.
2. Which is true about mucogingival flap designs b) Instrument is lodged in the bone beyond the apical area
a) Flaps should be wider at the base c) Instrument obliterates apical third of the canal
b) Flaps should be narrower at the base d) Instrument is in middle
c) Flap margins should not rest on the bone (TNPSC-99)
d) Mucogingival flaps should be avoided 11. Apicoectomy is contraindicated in
(MAN-98, APPSC-99) a) Extensive bone loss b) Over filled root canals
3. Which surgical procedure is indicated after the c) Lateral perforation at apical third
endodontic treatment is completed of a mandibular d) Is not contraindicated
molar with periodontal and carious involvement of the (PGI-97)
bifurcation: 12. Replacement resorption is characterized by:
a) Hemisection b) Fenestration a) Pain b) Apical pathosis
c) Root amputation d) Apical curettage c) Acute inflammation d) Tooth ankylosis
(AIPG-01) (AIIMS-01, AP -95, AIPG -93}
4. Apicoectomy is indicated in which one of the following 13. Apical surgery is Least indicated in
cases? a) Maxillary molars b) Maxillary premolars
a) Persistent positive results of root canal culture for c) Mandibular molars d) Mandibular premolars.
microorganisms
b) Incomplete obturation in apical part of root canal with 14. Most of the failures of replanted tooth are due to
persistent infection a) Replacement resorption b) Surface resorption
c) Fracture of apical one-third area of root in a vital c) External resorption d) Internal resorption
asymptomatic tooth (AP-2K; AIPG-03)
d) Presence of a minor pulp stone in pulp chamber 15. Which of the following directly affects the prognosis of
(AIPG -95) an avulsed tooth:
5. Which of the following are complications of Periapical a) Length of time the tooth was out of the mouth
surgery b) Condition of the dental socket when the tooth was
a) Paresthesia b) Ecchymoses replanted
c) Stitch abscess d) All of the above c) Whether necrotic cementum was removed from the root
(AP-01) d) Whether the pulp was extirpated to prevent Periapical
6. Endodontic surgery should be avoided in: inflammation
a) Short rooted teeth (AIPG-01)
b) Teeth with periodontal disease 16. A 12-years-old child suffers from an avulsed tooth
c) Lingual surface of mandibular molars following trauma. The best medium in which to carry
d) All of above the tooth to the dental clinic is:
(KAR-97) a) Buccal vestibule b) Milk
7. Draining abscess by cutting a window in the bone is c) Water d) Cotton
a) Hiltons method b) Trephination (AIPG-01, AIIMS-97)
c) Marsupialisation d) Odontectomy 17. Success of reimplanted avulsed tooth depends on:
(AP-01) a) Proper antibiotic coverage
8. RCT treated roots with well filled post and core, has a b) Prevention of dehydration of PD ligament of avulsed
periapical radiolucency tooth
a) Apicoectomy and retrograde filling with silver c) Protection of socket d) All of the above
b) Periapical curettage & apicoectomy (PGI-98)
c) Retreat and filling with guttapercha 18. Stabilisation of avulsed tooth requires
d) None of above a) 1-2 Weeks b) 2-3 Weeks
(PGl-99) c) 4-6 Weeks d) More than 6 Weeks
9. In Periapical surgery, which of the following flap designs (AP-99, KAR-98)
limit access to the operative site and often heals with
scar formation

1) B 2) A 3) A 4) B 5) D 6) D 7) B 8) A 9) C 10) C 11) A 12) D 13) D


14) C 15) A 16) B 17) D 18) A
Dental ;lut.,e

6. ENDODONTIC SURGERY - ANSWERS

1. 'B' [Grossman 12th ed 401-02] 13. 'D' [Grossman 12th ed 392]


Apical surgery is done with great caution on mandibular
2. 'A' [Grossman 12th ed 401] bicuspids because of their proximity to the mental foramen .
Cardinal principles of flap design:
• The base of the flap should be wider than the free end 14. 'C' [Grossman 12th ed 442]
to ensure adequate circulation into the flap. External inflammatory resorption is the most frequent failure
to replantation. Three differnet types of resorption have
• The sutured flap margins should rest on solid cortical
been identified: surface, inflammatory, and replacement
bone plate.
(ankylotic resorption) . Replacement resorption refers to
• Incision should be made with a firm, continuous stroke, resorption of the root surface and its substitution by bone,
perpendicular to cortical bone. Short intermittent resulting in ankylosis.
incisions result in tissue tags and ragged margins.
15. 'A' [Grossman 12th ed 380]
3. 'A' [Grossman 12th ed 434] Though 'B', 'C', 'D' also influences the prognosis of avulsed
tooth, extra oral time is the most critical factor affecting
4. 'B' [Grossman 12th ed 391] prognosis. Prolonged extraoral time causes dehydration and
death of root surface cells and impairment of viability of
5. 'D' [Grossman 12th ed 410] periodontal ligament.
Paraesthesia is more common in mandible, especially in the
premolar and molar region (due to accidental penetration l6. 'B' [Grossman 12th ed 380]
of mandibular canal) . Stitch abscess may develop due to Best storage media for carrying avulsed tooth are:
local laceration of tissue or accumulation of debris at
• Tooth socket
site of suturing or tying of knot in the line of incision.
Ecchymosis results in skin discoloration from extravasation • Patient's saliva, where the tooth is held in the buccal
and breakdown of blood adjacent to surgical area vestibule or under the tongue.
• Milk
6. 'D' [Grossman 12th ed 392] • Water
Because of inaccessibility and difficulty to gain access
to surgical site through dense layers of bone, endodontic Balanced Hancks solution is isotonic salt solution used
surgery should be avoided on lingual surfaces of molars or for preserving avulsed teeth. It is available under the t rade
external oblique ridge of mandible name Save-A-Tooth solution.

7. 'B' [Grossman 12th ed 415/lngle 6th ed 1234] 17. 'D' [Grossman 12th ed 380,381)
Trephination is a type of artificial fistulation in which, the
cortical bone is perforated to release the build up pressure 18. 'A' [Cohen 10th ed 649]
and exudate around root apex to release pain. Regardless the type of splint, splinting should be removed
in 7 - 10 days since prolonged splinting may induce
8. 'A' [Grossman 12th ed 391] replacement resorption and subsequent ankylosis.

9. 'C' (Grossman 12th ed 401]


Inaccessibility, excessive hemorrhage, delayed healing, and Period of immobilisation for reimplanted mature
scarring are the disadvantages of semilunar incision. It is tooth (APPG-15)
used when no underlying periodontal problems are present a) 7 to 10 days b) Need not be immobilised
i.e when there is no extensive bone loss. Inspite of its c) 3-4 weeks d) 2-3 weeks
disadvantages, it is most oftenly used in routine apical
surgery because of high healing potential and minimal bone
damage associated with semilunar flap.

10. 'C' (Grossman 12th ed 487]


In such cases, the instrument is removed by apicoectomy
followed by retrograde filling

11. 'A' [Grossman 12th ed 392]

12. 'D' [Grossman 12th ed 387]


ENDODONTICS

7. TRAUMATOLOGY

1. A severe blow to a permanent anterior tooth not d) Non-vital pulp


resulting in fracture, commonly leads to (AIPG - 98)
a) Pulpal death in the absence of treatment 10. The most common age group of fractures of permanent
b) Death of pulp despite treatment teeth
c) Fracture of involved bone a) 8 - 12 years b) 4 - 5 years
d) Mobility of central incisors c) 1 - 2 years d) 3 - 6 years
(MAN -99)
2. The most generally accepted treatment of root fracture 11. Best Prognosis is for
in the apical third of a maxillary central incisor involves a) Fracture of Apical 3rd of root
a) Splinting with orthodontic bands and a sectional arch wire b) Fracture in the Middle 3rd
b) Frequent pulp testing and radiographic examination c) Fracture in the coronal 3rd
c) Extraction of the tooth d) Prognosis is unrelated to the location of fracture
d) Immediate pulp extirpation (AIPG -02)
(MAN -95) 12. Endodontically treated posterior teeth are more
3. The X-ray of a traumatized tooth is important because susceptible to fracture than untreated posterior teeth.
a) To determine root fracture The best explanation for this is:
b) To asses the stage of root development a) Moisture loss b) Loss of root vitality
c) As basis for future treatment c) Plastic deformation of dentin
d) All the above d) Dest ruction of the coronal architecture
(MAN -99) (KAR -2K)
4. In Elli's classification of injured teeth a class IV represents 13. In primary tooth intrusion is common because:
a) Non-vital tooth b) Avulsed tooth a) Alveolar bone is pliable b) Vertical overlap of teeth
c) Fracture involving only the enamel c) Lip protection to max. anterior
d) Fracture involving enamel and dentin but not pulp d) All of the above
(MAN- 02) (PGI -95)
5. A permanent tooth with fracture involving apical one 14. Apical root fracture in an anterior tooth is treated by:
third of root after treatment, would most likely: a) Extraction b) Splinting for 2-3 months
a) Exhibit internal root resorption c) Endodontic treatment d) All of the above
b) Remain vital and functional (AIPG - 91)
c) Be ankylosed 15. A contusion usually result from
d) Exhibit external root resorption and exfoliate a) Cut injury b) Blunt trauma
(AIIMS-92) c) Crush injury d) Penetrating injury
6. A child has mid crown fracture in a permanent (AP -2K)
Maxillary central incisor several months ago, the pulp 16. Enamel fracture without tooth structure loss or
is now necrotic, radiographic examination reveals an incomplete enamel fracture results from
incompletely formed root with an open apex treatment a) Concussion b) Avulsion
of choice is: c) Subluxation d) None of the above
a) formocresol Pulpotomy b) root canal therapy (AP -2K, 03)
c) Calcium hydroxide Pulpotomy 17, In class-IV anterior tooth fracture:
d) Apexification a) No treatment possible
(AIPG -02) b) Treatment to be given but it is problematic
7. Exarticulation of teeth means c) No treatment required d) None of the above
a) Intrusion b) Extrusion (AIPG -93)
c) Subluxation d) Avulsion 18. A severe blow to a permanent anterior tooth, not causing
(PG! - 02) fracture frequently Leads to
8. A traumatized tooth nonvital with or without crown a) Permanent looseness b) Immediate pulpal death
fracture is classified as Elli's. c) Pulpal death, but only if treatment is delayed too long
a) Class-III b) Class-IV d) Pulpal death regardless of treatment
c) Class -V d) Class-VII (PGI -97)
(AIIMS -97) 19. A tooth was re-implanted and after 2 years it gave stony
9. In class -III Ellis fracture: hard on percussion sound what is diagnose
a) Only enamel is involved a) Internal resorption b) External resorption
b) Enamel and dentin are involved c) Replacement resorption d) Inflammatory resorption
c) Enamel, dentin and pulp are involved (AIPG-2012)

1) B 2) B 3) D 4) A 5) B 6) D 7) D 8) B 9) C 10) A 11) A 12) D 13) A


14) D 15) B 16) A 17) B 18) D 19) C
Dental ;lut.,e

20. Apexification is done with d) Displacement in a coronal direction


a) Ca(OH) 2 at the apex b) Ca(OH) 2 direct pu lp capping (KCET-10)
c) Indirect pulp capping d) None 30. What is the treatment in a child having laterally luxated
(PGI - 02) teeth which are not in occlusion
21. Source of ca• 2 fo dentinal bridge is: a) Wait and watch b) Grind the opposite teeth
a) Blood b) Ca(OH) c) Extraction of that t ooth
c) saliva d) ICF d) Immediate repositioning of the tooth
(PGI -95) (AIPG-2012)
22. Apexification is the treatment of choice for a permanent 31. Endodontic therapy is contraindicated for a single
tooth with wide open apex when rooted tooth that has
a) The pulp is necrotic b) The pulp is vital a) discoloration
c) The pulp and root canals are calcified b) an acute apical abscess
d) There is traumatic pulp exposure during cavity c) a vertical fracture of t he root and an associated deep
preparation periodontal defect
(COMEDK - 03) d) a horizontal fractu re of the root and an associated deep
23. 8 year old child had fractured his maxillary central periodontal defect
incisor 10 months ago. The pulp shows no response. (AP-09)
There is no periapical lesion in the radiograph. The 32. An important clinical feature of a concussed tooth is
treatment of choice is a) Tooth is markedly tender to percussion
a) Ca(OH) 2 Pulp capping b) Formocresol pulpotomy b) Tooth is displaced from the socket
c) Conventional root canal treatment c) Evidence of haemorrhage at gingival margin
d) Complete debridement and apexification d) Clinically crown appears shorter
(AIIMS -98,99) (UPSC-09)
24. A 2-hour-old Avulsed tooth is best stored in medium 33. The most common sequelae of a tooth that is stabilized
that contains: after a fracture in the apical third of the root is
a) Tetracycline b) Dexamethasone a) Remains vital and fucntional
c) Ampicillin d) Ibuprofen b) Resorption
(AIPG-07) c) Necrosis d) Ankylosis
25. To store avulsed tooth, which kind of milk is shown to (AP-10)
be suitable? 34. Which of the following is true about thermal and electric
a) Hot Milk b) Cold Milk pulp tests after trauma-
c) Low Fat Milk d) High Fat Milk a) Sensitivity tests evaluate the nerve and circulatory
(COMEDK - 08) condition of the tooth.
26. Crown infarction is b) False positive tests are more likely than false negative
a) Necrosis of pulp following a fracture tests
b) An incomplet e fracture of enamel without loss of tooth c) It may take upto 9 months for normal blood flow to
structure return.
c) Fracture of crown en mass d) It may take a few hours for normal blood flow to return.
d) Vertical fracture of crown (COMEDK-09)
(KCET- 08) 35. An avulsed tooth before replantation should be treated with?
27. There is a real relationship between the incidences of a) Doxycycline b) Dexamethasone
fracture of anterior teeth and the c) Ampicillin d) Ibuprofen
a) caries in those teeth b) hardness of t he enamel (AIPG-10)
c) hardness of dentin d) protrusion of those teeth 36. Splinting of cervical root fracture is done for
(AP-09) a) 40 days b) 4 months
28. An 8 year old child got trauma and got his central incisor c) 4 weeks d) 14 days
avulsed 20 minutes back which got contaminated with (AIIMS-09)
debris. The child rushes to dental clinic with avulsed 37. Which type of trauma is associated with destruction of
tooth, the treatment is protective layer?
a) Clean root surface with saline and reimplant a) Intrusion b) Extrusion
b) Clean and curette root surface and reimplant c) Avulsion d) Lateral luxation
c) Clean root surface, do RCT and reimplant (AIIMS-09)
d) Scrubbing of root surface and reimplant 38. If a tooth has abnormal mobility, but is not displaced
(AIIMS-08, 13) from the socket it is termed as
29. Concussion implies luxation injury a) Concussion b) Subluxation
a) Without displacement and normal mobility c) Extrusive luxation d) Partial avulsion
b) Without displacement and increased mobility (KCET-2011)
c) Displacement in an apical direction

20) A 21) A 22) A 23) D 24) A 25) C 26) B 27) D 28) A 29) A 30) D 31) C 32) A
33) A>C 34) C 35) A 36) C 37) A 38) B
ENDODONTICS

39. Which of the following is true about thermal and 47. For root fracture it is imperative to take x ray at?
electrical pulp tests after trauma? a) 45°, 90° b) 90°
a) Sensitivity tests evaluate the nerve and circulatory c) 90°, 110° d) 45°, 90°, 110°
condition of the tooth (AIIMS MAY-13)
b) False positive t ests are more likely t han false negative 48. Electric nerve tester detects pain by stimulating which
tests nerve fibres
c) It may take up to nine months for normal blood flow to a) A alpha b) A beta
return. c) A delta d) C fibres
d) It may take a few hours for normal blood flow to return. (AIIMS MAY-13)
(COMEDK-2011) 49. The least favourable form of attempted repair of the
40. According to Ellis and Davey classification a fracture of root fracture is
51 involving dentin and associated loss of vitality is a) Healing with calcified tissue
classified as b) Healing with interposition of connective tissue
a) Class 2 b) Class 4 c) Interposition of bone and connective tissue
c) Class 8 d) Class 9 d) Interposition of granulation tissue
(KCET-2012) (COMEDK-14)
41. The most common dental procedure contributing to 50. The incisal edge edge of maxillary central incisor in a 21
vertical root fracture is year old man is chipped off during a fall. He is informed
a) Prosthodontic treatment b) Endodontic treatment that the tooth will erupt a little. This compensatory
c) Periodontal treat ment d) Orthodontic treatment tooth movement is because of increased deposition of
(BHU-2012) a) Dentin b) Pulp
42. The following material with a hemostatic effect can be c) Cementum d) Periodontal ligament
used as a substitute for Formocresol Pulpotomy (MCET-14)
a) Gluteraldehyde b) Ferric sulphate 51. What should be the first treatment for an acutely
c) Mineral Trioxide Aggregate (MT) avulsed tooth?
d) Freeze-dried bone a) Gently replace with finger using light pressure, take
(AP-2012) radiograph and splint
43. Stunned pulp is b) Sterlize the root and reimplant
a) Pulp does not respond for 6-8 weeks after trauma c) Pulp protection
b) Gives stunning appearance on radiograph d) Immediate endodontic t reatment and reimplant
c) Necrosed pulp d) Hyperaemic pulp (AIPG-14)
(AIPG-2012) 52. Best medium to transport/store an avulsed tooth, which
44. Reaction of the tooth to trauma include is not to be reimplanted immediately?
a) Pulpal hyperemia b) Internal hemorrhage a) Viaspan b) Hanks balanced salt solution
c) Internal resorption d) All of the above c) Coconut water d) Milk
(KAR- 2013) (AIPG & AIIMS MAY- 2014)
45. A 10 year old child comes to you with exarticulation of 53. For approximate how much time the 4% buffered
11. glutaraldehyde is applied over the pulp stump during
(COMEDK- 2013) pulpotomy?
45A. This condition is called as a) 10 Sec. b) 60 Sec.
a) Avulsion b) Intrusion c) 2 min d) 4 min
c) Fracture with enemas d) Dilaceration (AIPG-14)
54. Which zone is not seen when repair is done with most
458. Treatment of choice for the above condition is biocompatible material?
a) Extraction b) Replantation a) Zone of necrosis b) Zone of irritation
c) Re-vascularisation d) Composite Restoration c) Zone of calcification d) Zone of proliferation
(AIPG-14)
45C. Choice of transport medium is 55. "An avulsed tooth is replanted, loss of lamina dura
a) Patients Blood b) Hot Water radiographically, high sound on percussion means
c) Milk d) None of the above a) Internal resorpton
b) External resorption
46. In a tooth fracture involving dentin treatment comprises c) Replacement resorption
all except: d) Cervical resorption
a) Composite filling immediately (AIIMS MAY-14)
b) Seal exposed de ntinal tubules 56. What is PULP cap?
c) Periodic recall at 1 week, 1 month, 3 months and 6 a) Capping of the exposed vital pulp with a thin layer of
months inert lining or base material
d) No recalls required

39) C 40) D 41) B 42) B 43) A 44) 0 45A) A 456) B 45C) C 46) 0 47) 0 48) C 49) 0
50) C 51) A 52) A 53) D 54) A 55) C 56) B
Dental ;lut.,e

b) Capping of exposed vital pulp tissue by placing a layer of


mineral trioxide aggregate (MTA) or calcium hydroxide.
c) A method of isolation used during root canal treatment
d) An early stage of tooth development.
(APPG-15)
5 7. The ideal method for restoring a fractured tooth is
a) Open faced stainless steel crown
b) Reattachment of fractured tooth segment
c) Strip crown
d) Composite resin restoration
(MHCET-15)
58. The purpose of pulpotomy in young permanent tooth
a) Prevent tooth fracture b) To induce root formation
c) Formation of hard! base to support restoration
d) To minimize infection
(AIIMS MAY-14)
59. Enamel matrix protein derivatives obtained from
developing porcine teeth approved by FDA is
a) Boplant b) rh BMP-2
c) Bio-Oss d) Emdogain

60. Pain due to thermal change after trauma will occur


during
a) Periapical inflammation b) Periapical necrosis
c) PDL injury d) Root fracture
(PGI JUNE-13)
61. Cvek pulpotomy?
a) Cervical pulpotomy b) Partial pulpotomy
c) Formocresol pulpotomy
d) Glutaradehyde pulpotomy
( PGI JUN E-13)

57) B 58) B 59) D 60) D 61) B


ENDODONTICS

7. TRAUMATOLOGY - ANSWERS

1. ' B' (Grossman 12th ed 361) 9. 'C' [Grossman 12th ed 361)


In injuries that do not fracture the crown or root, the
impact of the blow is transmitted head on to the pulp, 10. 'A' [Grossman 12th ed 361)
which receives the full force of the blow and get seriously Permanent anterior tooth fractures occur mostly in the age
injured where as in injuries that cause fracture, the impact group of 8-11 years while primary anterior tooth fractures
is broken as the crown or root fractures, and the pulp is less occur mostly in the age group of 1 - 2 1/z years
likely to be injured seriously
11, 'A' [Grossman 12th ed 371]
2. ' B' (Grossman 12th ed 3 71 J
Fracture of the apical third of root has the best prognosis 12. ' D' [Grossma n 12th ed 442]
because the pulp in the apical fragment usually remains Endodontically treated posterior teeth are more susceptible
vital and the tooth may remain firm in its socket. Periodic to fractures than untreated posterior teeth. This is due to
vitality testing and radiographic examination with no other loss of coronal structure during access opening. They are
treatment is indicated. If the tooth is mobile it should be more susceptible to vertical root fractures when excessive
splinted with acid etched resin. If the pulp in the coronal enlargement of the root canal is done to receive a post
segment dies, then endodontic treatment is indicated. If crown or when subjected to traumatic occlusion
the tooth fails to recover, the apical root fragment should
be removed surgically. 13. 'A' [Grossman 12th ed 376]

Fracture of middle and coronal third has Less favourable 14. ' D' [Grossman 12th ed 371)
prognosis. RCT or extraction is indicated. Vertical root
fractures have the worst prognosis. 15. ' B' [Shobha Tandon 1st ed 502]
Shallow or deep wound in the mucosa
Laceration
Apical third root fracture in a vital tooth are related caused by a sharp object
by: (COMEDK- 2013) Contusion Bruise produced by impact from blunt object
Ans. Relieve from occlusion and observe
Superficial wound produced by rubbing or
Abrasion
3. ' D' [Grossman 12th ed 363] scrapping
Partial displacement of tooth out of
Extrusion
4. 'A' [Shobha Tandon 1st ed 500] socket
ELLIS AND DEWAY CLASSIFICATION Complete displacement of tooth out of
Exarticulation
socket.
Class-I Fracture involving enamel
Class-II Fracture involving enamel and dentin 16. 'I( [Grossman 12th ed 362]
Class-III Fracture involving dental pulp
Class-IV 17. ' B'
Non vital tooth
Class-V Avulsion 18. ' D' [Grossman 12th ed 361,362)
Class-VI Root fracture with or without crown fracture Vertical fracture of posterior teeth is not as amenable to
Class-VII Displacement of tooth without fracture of conservative endodontic treatment as horizontal fracture. If
crown or root the fracture occurs in a buccoli ngually plane in multi rooted
tooth through furcation, endodontic therapy followed by
Class-VIII Fracture of crown en masse hemisection and full crown coverage of mesial and distal
Class-IX Deciduous tooth fractures segments is indicated.

5. ' B' (Grossman 12th ed 371) If the vertical fracture occurs through the crown furcation
of a maxillary molar in a mesiodistally plane, extraction is
6. ' D' [Grossman 12th ed 331] indicated.
Apexification with calcium hydroxide - CMCP paste is
indicated in nonvital developing tooth with incomplete 19. 'C' [Grossman 12th ed 388]
root formation. This induces the closure of the apex by the
formation osteodentin or osteocementum, which serves as a 20. 'A' [Grossman 12th e d 331]
stop for the guttapercha filling and ensures an adequate seal.
21. 'I( [Ingle 6th ed 1312)
7. ' D' [Grossman 12th ed 378) Experiments with radioactive calcium established that
calcium for the dentin bridge comes from blood stream.
8. ' B' [Grossman 12th ed 362)
Dental ;lut.,e

22. 'A' [Grossman 12th ed 331] AFTER AVULSION, PREPARATION OF ROOT

23. ' D' [Grossman 12th ed 331] A) Extra oral dry time is less than 60 minutes
i) Closed apex
24. 'A' [Grossman 12th ed 382] The root should be rinsed of debris with water or
saline, replanted in as gentle as possible.
25. 'C'
ii) Open apex
26. ' B' [Cohen 9th ed 614/ Grossman 12th ed 363] The tooth is soaked in doxycycline or covered with
Crown infarction is an incomplete fracture of enamel minocycline for 5 minut es, debris is gently rinsed
without loss of tooth st ruct ure. Infarction (crack or craze off and the tooth is replanted.
lines) can be diagnosed by transillumination or indirect
light, fi ber-optic or resin curing light and occasionally by
B) Extraoral dry time more than 60 minutes
clinical examination.
i) Closed apex
27. ' D' [Grossman 11th ed 361) The periodontal ligament is removed by placing it in
3% citric acid for 3 minutes, soaking it in fluoride
28. 'A' [Grossman 12th ed 382) or covering the root with emdogain (A enamel
mat rix derivative that stimulates the regeneration
29. 'A' [Cohen 8th ed 623/Grossman 12th ed 375) of periodontal ligament) and then replanting.
5 Types Of Luxation Injuries Are:
ii) Open apex
a) Concussion:
If replantation is to take place, the open apex
No displacement and no mobility. Pain to percussion is
is treated t he same as the closed apex tooth.
the only feature.
Endodontic treatment may be performed out of the
b) Subluxation: mouth.
Increased mobility and sensitivity to percussion. No
displacement. AFTER AVULSION, PREPARATION OF THE SOCKET
The socket should be left undisturbed before_replantation.
c) Lateral luxation: Emphasis is placed on removal of obstacles within the
Displacement labially, lingually, distally or incisially. socket to facilit ate replacement of t he tooth into the
d) Extrusive luxatio n: socket.
displacement in coronal direction
36. 'C' [Cohen 10th ed 635/ Grossman 12th ed 382)
e) Intrusive luxation: Emergency treatment of root fractures involves repositioning
displacement in an apical direction into the alveolus t he segments into close proximity and splinting in its
orginal position.
30. ' D' [Ingle 5th ed 823/ Grossman 12th ed 376)
Immediate treat ment in a lat erally luxat ed toot h is Traditionally 2-4 months wit h rigid splinting protocol was
immediate repositioning and stabilization. But according recommended (8th ed cohen). But now semirigid splint to
to Pikham Pediatric Dentistry, he recommends extraction of adjacent teeth for 2-4 weeks is recommended.
laterally luxated teeth because of the fear of aspiration.
37. 'A' [Cohen 9th ed 630/Grossman 12th ed 376]
31. ' C' [Ingle 6th ed 108/ Grossman 12th ed 3 75]
38. 'B' (Grossman 12th ed 375)
32. 'A' [Cohen 8th ed 623/ Grossman 12th ed 375) • Luxation - displacement or partial dislocation of a
tooth from its socket.
33. 'A>C' [Cohen 8th ed 623/Grossman 12th ed 371) • Subluxation - injury t o the supporting structure of
In the majority of root fractures the pulp ret ains vitality a tooth resulting in abnormal loosening of the tooth
and pulp necrosis occurs only in 25% of root fractures. without any displacement
Rigid stabilization of the segments (for 2 to 4 months) will
allow healing and reattachment of t he fract ured segments. 39. 'C' (Cohen 10th ed 621)
The pulp may take upto 6 months to return to normal vitality
34. 'C' (Cohen 10th ed 621) after trauma. Option 'P: sensitivity t ests evaluate the nerve
condition and not the circulatory condition. Option 'B' fa lse
35. 'A' [Cohen 9th ed 643/ Grossman 12th ed 382) negative (loss of vitality) are more likely than false positive
tests.

40. 'D' (Shobha Tandon 1st ed 500)


ENDODONTICS

41. 'B' [Ingle 6th ed 676) • Viaspan


Vertical root fracture is common after endodontic treatment • CPP-ACP (Casein Phosphopeptide - amosphous calcium
because it removes most of tooth structure by access phosphate)
opening and shaping the roots. It is also most common in
root canals where post is placed. 46. 'D' [Grossman 12th ed 364/Cohen 10th ed 624)
• Following uncomplicated tooth fracture, immediate
42. 'B' [Cohen 8th ed 811)
treatment should include closing all exposed dentinal
Pharmacologic agents which can be used for pulpotomv
tubules to prevent ingress of bacteria.
in primary teeth are:
• If the fracture fragment is not available, composite or
• Formocresol
temporary restoration should be placed on all exposed
• Glutaraldehyde dentin.
• Calcium hydroxide • If the ROT is less than 0.5mm, then a hard setting
• Collagen calcium hydroxide (Dycal) should be placed in the
• Ferric sulfate deepest part of the dentin followed by composite.
• The tooth should be periodically tested for pulp vitality
Non-pharmacologic agents at 1 week, 1 month, 3 months, 6 months and a year.
• Electrosurgery
• Laser therapy. 47. 'D' [Cohen 10th ed 635)
Radiographs play an important role in the diagnosis of
Ferric sulfat e, electrosurgery and laser therapy have root fractures. These fractures are usually oblique and a
hemostatic effects. single IOPA will not be sufficient to diagnose its presence.
The X-ray beam should pass through t he fracture line to
43. 'A' [Grossman 12th ed 362) make it visible on the radiogra ph. So at least three angled
Immediately after trauma, the vitality test may be negative radiographs in 45, 90 and 110 degree should be taken so
as the pulp is stunned. The nerve bundles which are that in atleast one angulation, the X-ray beam might pass
temporarily paralyzed does not respond to electric pulp through fracture line and image it on the radiograph.
test.
48. 'C' [Ingle 6th ed 53 7]
44. 'D' [Grossman 12th ed 91, 362, 384] Check Synopsis
Pulpal response to dental trauma depends on
49. ' D' [Check Synopsis under important features roof
a) Intensity of trauma
fractures]
b) Effect on neurovascular supply
c) Stage of root development 50. 'C' [Check Explanation Below)
d) Presence or absence of bacteria. The continuous formation of cementum is a compensatory
reaction against shortening of the crown due to wearing
The possible outcomes after trauma are: of enamel. Passive eruption is the apparent lengthening of
the crown due to the loss of attachment or recession of the
• Pulp healing gingiva.
• Pulpitis and necrosis
• Pulp canal obliteration In most adults both active and passive eruption occur to
• Internal resorption some extent and the active eruption is compensated part
by the addition of alveolar bone at the crest of the alveolar
45A. 'A' [Grossman 12th ed 378] bone and at the base of the socket, and by the deposition
of cementum on the root surface.
458. 'B' [Refer explanation of Q.No.24]
51. 'A' [Grossman 12th ed 380)
45C. 'C' [Grossman 12th ed 380) Emergency (first) treatment at the site of avulsion
Transport media for avulsed tooth are: injury:
• HBSS (Hank balanced salt solution) • Wash the tooth in running water
• Coconut water • Avoid touching or scraping
• Propolis • Replace tooth in its socket using gentle steady finger
pressure
• Saliva
• Take the patient to the dentist
• Vestibule of the mouth
• Milk 52. 'A' [Check Explanation Below)
• Saline According to American Association of pediatric dentistry
• Water revised guidelines, the order of preference of transportation
Dental ;lut.,e

medium is via span, HBSS and cold milk. But ot her reviews fragment. This will offer excellent esthetics and functiona l
and studies quoted that extra alveolar time of avulsed results with less chair time.
tooth in milk is upto 6 hours and HBSS is 72-96 hours. The
recommended storage media for avulsed tooth are those 58. 'B' [Cohen 10th ed 629]
having physiologic Ph and osmolarity like milk and HBSS. Indications of full Pulpotomy:
The media that can be recommended (after further invivo • Traumatic exposures of pulp after more than 72 hours
studies) are Viaspan, Propolis and growth factors.
• Carious exposure of a young tooth with partially
developed apex.
53. 'D' [Check Ingle 6th ed 1411]
• Formacresol and glutaraldehyde can be used for Indications of partial (CVEK) pulpotomy:
pulpotomy. But according to Ingle, glutaraldehyde • Exposure of pulp >lmm
is not the ideal material to replace formacreso l for • Exposure for more than 24 hrs.
pulpotomy. • Inflammed pulp tissue at exposure site.
• Unbuffered glutaraldehyde does not penetrate deep
into the tissues. Buffering increases its concentration 59. 'D' [Cohen 10th ed 649]
and fixing ability. Emdogain contains enamel matrix proteins derived trom
• 2%, 4% and 8% buffered glutaraldehyde solutions can developing pig teeth. It is believed to regenerate the tooth
be used. attachment.
• 2% is used for pulpotomies in primary teeth.
60. 'D' [Principles and practice of Pedodontics by Aarthi Rao
• The recommended time for 4% is 4 min and 8% is 2 min. 3rd ed 316]
Pain caused by thermal change signifies pulpal inflammation.
54. 'A' [Cohen 10th ed 626] In case of root fracture, pulp will be injured which will
Zones formed with calcium hydroxide (CH) pulp therapy Lead to inflammation of pulp and any thermal change with
from below CH to normal pulp are: respect to the fractured tooth will alter the blood supply to
i) Zone of Liquefactive necrosis tooth and increase the extravascular fluid which will lead
ii) Zone of coagulative necrosis to increase in intrapulpal pressure and finally patient will
iii) Inflammatory response experience pain due to thermal changes.
iv) Hard tissue barrier with vascular inclusions.
61. 'B' [Cohen 10th ed 627)
The necrosis with pure CH is due to its high PH . But dycal
which is a hard setting CH formulation forms hard tissue
barrier without causing necrosis.

Zones formed with MTA pulp therapy


i) Very narrow or even absent coagulative necrosis
ii) Reparative dentinogenesis zone resulting in dentin
bridge with fewer vascular inclusion (high quality).

MTA is a biocompatible material. The excellent bacteria-


tight seal accounts for the success with MTA.

55. 'C' [Cohen 10th ed 647]


Clinical features of replacement resorption:
• The affected tooth is immobile
• Exhibits a high percussive tone (solid) in contrast
to the low dull, cushioned sound of normal teeth on
percussion .
• Lamina dura and PDL space absent.
• Direct union between alveolar bone and root.
• With time, infra-occlusion of the tooth relative to
adjacent teet h.

56. 'B' [Cohen 10th ed 627]

57. 'B' [Case reports in odontology 2014; 1 (2): 10-17]


Reattachment of the crown fragment to the fractured tooth
is the best method to give the natural shape, contour,
surface texture, occlusal alignment and the color of the
ENDODONTICS

8. MISCELLANEOUS

1. The control teeth used during pulp testing are 11, A sinus associated with a periapical pathology requires:
a) Adjacent and opposing teeth a) Broad spectrum antibiotics
b) Adjacent and contra lateral teeth b) RCT with excision of sinus
c) Suspected tooth only d) Adjacent teeth only c) RCT with Periapical curettage
(MAN-99) d) RCT with no treatment for sinus
2. Perfusion of oxygen supply in a free flap can be correctly (PGl-2K)
estimated by 12. Toothache relieved with cold water indicates:
a) Pulse oximetry b) Laser Doppler flowmetry a) Acute pulpitis b) Arterial hyperemia
c) Prick test d) Fluroscopy c) Irreversible pulpitis d) Venous hyperemia
(AIPG-2012) (PGI-2K)
3. Diagnostic test commonly used to determine the status 13. Initial threshold of the electric test in a traumatic tooth
of vitality of pulp is: is to:
a) Radiography b) Thermal testing a) Establish base line of the physical status ot the pulp
c) Computerized axial tomography scanning b) Is positive in healthy pulp
d) Percussion and palpation c) Is negative in root canal tooth
(AIPG-95) d) Is repeated after 30 days
4. Medical condition contraindicated for endodontic (PGI-01,03)
therapy: 14. False vitality results are common in posterior tooth are
a) Rheumatic heart disease b) Patients with pacemaker most commonly due to
c) Controlled diabetic d) None of the above a) Presence of vital pulp tissue in one of the canals
(KAR-98) b) Thickness of the enamel
5. Electric pulp test for an immature tooth is: c) Presence of pulp stones
a) Positive b) Negative d) Thickness of dentine
c) False-Positive d) False-negative (AP-99)
(PGl-95) 15. Most useful in differentiating an acute Periapical
6. The temperature for testing the vitality by Heat test abscess and periodontal abscess would be
should not exceed a) Anaesthesia test b) Percussion test
a) 65.5 C b) 65.5 F c) Radiographic examination
c) 32.5 F d) 32.5 C d) Pulp vitality test
(AP-01) (APPSC-99, AIIMS-95)
7. Testing with cold is carried out best: 16. Percussion is a dental diagnostic procedure used in
a) To localize pulpal pain determining whether
b) To localize periodontal pain a) The tooth is vital b) The pulp is hyperemic
c) For detecting referred pain c) A periodontitis exists d) All of the above
d) To test pulp necrosis (APPSC-99)
(AIPG-96) 17. Temperature of glass bead sterilizer is
8. Electric pulp test: a) 425°F-475° F for 5 secs b) 475°F-525°F for 5 secs
a) Requires more current for anterior teeth c) 425°F-475°F for 10 secs d) 475°F-525°F for 10 secs
b) Requires more c11Jrrent for posterior teeth (KAR-04)
c) Same response d) Better in inflamed teeth 18. Minocycline is not used in the 3 mix antibiotic paste
(PGl-95) because of only one adverse effect
9. In a tooth, which has been endodontically treated, there a) Discolouration of crown b) Roots become brittle
is pain on percussion and slight mobility due to: c) It reaches out readily d) It has least antimicrobial effect
a) Loose crown b) Tooth fracture (AIPG-2012)
c) Psychosomatic d) Eccentric contact 19. Reversible pulpits is
(PGI-2K) a) Aggravated by heat and relieved by cold
10. A patient comes in dental office for routine dental check b) Aggravated by cold and relieved by heat
up. His upper central incisor which is root canal treated c) No reaction to hot and cold
6 months ago, shows a Periapical radiolucency, but d) Reacts to electric pulp tester
the patient has no previous radiographs of the lesion. (MAN-2K)
Treatment at this time: 20. The tooth is tender on vertical percussion. It may be due to
a) No treatment, observe b) Redo the RCT a) Osteomyelitis b) Periapical inflammation
c) Make a Post and Core d) Do apicoectomy c) Necrotic pulp d) Angular bone loss
(AIPG-02) (MAN-01)

1) B 2) C 3) B 4) D 5) D 6) A 7) A 8) B 9) D 10) A 11) D 12) C 13) A


14) A 15) D 16) C 17) A 18) A 19) A 20) B
Dental ;lut.,e

21. After the stimulation is removed the response of normal 32. When hot nail is placed on vital tooth surface
pulp to hot or cold is: a) There is increase in intrapulpal pressure followed by
a) Brief pain b) Prolonged pain decrease to above normal
c) Brief fee ling of warmth or cold b) There is increase in intrapulpal pressure followed by
d) Prolonged feeling of warmth or cold decrease to normal
(AIPG-01) c) There is increase in intrapulpal pressure followed by
22. It two years after RCT, a patient complains of "Pain decrease to below normal
over the gum" of that tooth and the tooth is tender to d) There is increase in intrapulpal volume without any
percussion, then the treatment of choice is: increase in intrapulpal pressure
a) Extraction (AP - 93)
b) Retreat ment of the root canal 33. The non-invasive method to measure the blood flow is:
c) Apicoectomy and a retro fill a) Electric pulp test b) Percussion
d) An antibiotic course and Re-Evaluation at next visit c) Radiograph d) Laser Doppler flowmetry
(KAR-2K) (AIIMS-06)
23. The sterility of the root canal during treatment is 34. 3M-mix antibiotic paste includes the following as
assessed by intracanal medicament
a) Radiographs b) Clinical observations a) Ciprofloxacin, Metronidazole, Minocycline
c) Bacterial cultures d) Colour of dressings b) Norfloxacin, Metronidazole, Minocycline
{KAR-98) c) Ciprofloxacin, Cefixime, Minocycline
24. Bacteria most commonly isolated from root canals is d) Norfloxacin, Metronidazole, Cefixime
a) Actinomyces b) Streptococci (AIPG-2012)
c) Staphylococci d) Fusobacterium 35. Root canal treated tooth are hard to resorb because of
(AP-2K) a) Lack of vitality of pulp results in loss of neuropeptides
25. Paper points are sterilized by: b) Cementum becomes more calcified
a) Cold sterilization with chemical solution c) Lack of differentiation of odontoclast because of no live
b) Boiling cells
c) Hot salt sterilizer d) All of the above d) PDL become necrosed
(PGI-98, 2K) (PGI-2011)
26. In an endodontically treated tooth: 36. All are true about RVG, except:
a) Resorption is more as compared to a normal tooth a) 80% reduction of patient exposure
b) Resorption is less b) Instant imaging
c) Resorption is same c) Easy to storage and retrieval
d) Resorption is more after 2 years. d) Image is sharper than cause by halogen halide
(PGI-95) (AIIMS-06)
27. In Periapical lesions of average size, time needed for 37. Electric pulp tests may not be performed on patients
osteogenesis is: who have:
a) 1-2 months b) 2-6 months a) Hip implant b) Pace maker
c) 6-12 months d) 2 Weeks c) Dental implant d) Prosthetic eye
(AIPG-91) (AIPG- 06)
28. Which strain of yeast is found in the root canal: 38. Balanced force technique is used for tooth preparation:
a) Candida albicans b) Cryptococcus a) Lateral force is used during preparation
c) Histoplasma d) Actinomyces b) Apical force is used during preparation
(PGI-2K) c) No force is used d) None
29. Elective endodontic treatment is contraindicated in (AP-07)
a) Angina patients b) Recent MI 39. Endodontic implants refers to those:
c) Diabetes d) Hypertension a) Used to obturate the canal
(PGI-97) b) used to stabilize periodontally weakened with poor
30. Ca{OH) 2 is given crown root ratio
a) In weeping canal c) Placed between periosteum and cortex of bone
b) When there is bleeding from root canals d) Which are embedded within the bon
c) As a sterilizing agent (COMEDK- 07)
d) To remove or dissolve Periapical debris 40. Which of the following is used as best root end filling:
(PGI-95) a) MTA b) Amalgam
31. Technique that is best suited for preparing fine and c) ZOE d) Calcium hydroxide
curved canals (AIPG-07)
a) step back b) step down 41. Which of the following is true about master apical file?
c) crown down d) Modified double flare technique a) The last file which is selected for the obturation
(AIPG -96)

21) A 22) B 23) C 24) B 25) C 26) C 27) C 28) A 29) B 30) A 31) D 32) A 33) D
34) A 35) A 36) D 37) B 38) B 39) B 40) A 41) A
ENDODONTICS

b) The initial file which is selected for initial radiographic c) 1 mm below the canal operiing
working length d) 1 mm below the roof of pulpal chamber
c) The smaller file used in between the other file during (AIPG-2009, AIIMS-2012)
biomechanical preparation 52. Single Sitting RCT is indicated in all except?
d) The last file which is used for removal of soft debris from a) An anterior single rooted tooth where esthetics is of
root canal more concern
(AIIMS-06) b) In a patient who require sedation for the management
42. Which among the following Root canal irrigants removes of involved tooth
the smear layer: c) Severe anatomic & procedural difficulty
a) Sodium hypochlorite b) EDTA d) Tooth with pulpal exposure because of trauma
c) Hydrogen peroxide d) Chlorhexidine (PGI-08)
53. Which of the following is a tertiary monoblock system
43. The confirmatory test for pulp vitality is of obturation of the root canal.
a) Thermal test b) Test cavity a) Resilon monoblock obturation
c) Analytical technology electrical pulp tester b) Active Gutta percha
d) Digital pulp videography c) Orthograde obturation with MTA
(KCET-08) d) Gutta Percha and AH plus sealer
44. Theodor Croll 1986 is associated with (AIIMS-09)
a) Use of 18% hydrochloric acid and pumice - microabrasion 54. Which of the following statement regarding internal
b) Use of 12 fluted bur to remove the defects resorption is accurate?
c) Bleaching techniques d) Dakin's solution a) It is rare in deciduous teeth
(COMEDK- 08) b) It is initiated by odontoblasts
45. Which of the following is best suited to diagnose acute c) It is seldom confused with external resorption
apical periodontitis d) It is usually asymptomatic
a) Percussion b) Thermal tests (COMED-10)
c) Electric pulp tests d) Anaesthetic test 55. If a histologic slide of the tooth and the content of its
(AP-08) canal space could be obtained, the most likely finding
46. The stabbing nature of pain in trigeminal neuralgia in the region of radioluceny would be?
mimics pain caused by a) Normal pulp b) Clastic activity
a) A cracked tooth b) Acute reversible pulpitis c) Necrotic pulp d) Marked decrease in cellularity
c) Acute irreversible pulpitis (AIPG-09)
d) Acute apical periodontitis 56. Tevdek sutures used for endodontic microsurgeries are
(COMEDK-08) removed after?
47. Internal resorption may follow all except a) 48-72 hours b) 72-96 hours
a) trauma b) root fracture c) 7 days d) 10 days
c) successful root canal filling (AIPG-09)
d) Enamel fracture 57. In molecular biological methods in endodontics, the
(BHU-07) process of annealing the complementary bases of two
48. Persistant the cultures in a root canal is mostly due to single stranded DNA is known as -
a) Seepage of saliva b) Excessive instrumentation a) DNA-DNA micro processing
c) Less medicament d) Via apical foramina b) DNA-DNA processing
(BHU- 07) c) DNA-DNA hybridization d) DNA-DNA coupling
49. Which of the following is the most difficult pulpal or (KCET-10)
periapical pathosis to diagnose 58. A radiolucent root canal suddenly disappears at the mid
a) Necrotic pulp b) Chronic Pulpitis root portion, the most probable diagnosis is
c) Internal resorption d) Acute alveolar abscess a) Calcifications within the pL1lp
(AIPG-09) b) Bifurcation of the canal
50. A protaper F2 series Gutta Percha when cut 1 mm in c) Sclerosis of the adjacent bone
apical position of canal. The preparation as per in apical d) Ankylosis
region is? (AP-10)
a) 0.29 mm b) 0.30 mm 59. Triple antibiotic paste contairn all except
c) 0.31 mm d) 0.33 mm a) Metronidazole b) Minocycline
(AIPG-09) c) Ciprofloxacin d) Amoxcyllin
51. If a patient has gingival recession, how far should (PGI JUNE-14)
the gutta percha be trimmed back before sealing the 60. Myospasm that is involuntary continuous contraction of
coronal area? a muscle due to overstretching which is already strained
a) 1 mm below the level of the recession because of sustained mouth opening during RCT is-
b) 1 mm below the ACJ a) Charley horse b) Ercess expand

42) B 43) B 44) A 45) A 46) A 47) C 48) A 49) B 50) D 51) A 52) C 53) B 54) D
55) B 56) A 57) C 58) B 59) D 60) A
Dental ;lut.,e

c) Muscle tear d) Hardy stretch c) 7 days d) 10 days


(KCET-09) (AIPG-2011)
61. In preparing an endodontically treated tooth for 73. Laser Doppler flowmetry is a method used to assess
receiving a cast post, the minimal amount of gutta- blood flow in
percha to be maintained for proper apical seal is - a) Microvascular systems b) Microlymphatics
a) 1 mm b) 8 mm c) Micro vesicles d) Micro spaces
c) 4 mm d) 0 mm (BHU-2012)
(KCET-09) 7 4. In endodontic-periodontal lesion pulpal necrosis
62. 1st sign after formocresol pulpotomy failure is a) Follows periodontal changes
a) Internal resorption b) Pain b) Precedes periodontal changes
c) Discoloration of tooth d) Bleeding c) Impedes periodontal changes
(AIPG-10) d) All of the above
63. The approximate time in which ferric sulphate (BHU-2012)
dampened in a cotton pellet is put to the root stump 75. Which is the ideal method of sterilization for snap a ray
while performing pulpotomy is film holder?
a) 10-15 secs b) 60-90 secs a) Hot air oven
c) 2-3 mins d) 4-5 mins b) Cannot be sterilized as it is plastic
(AIPG-10) c) Chemical sterilization d) Autoclave
64. Not true about virulence of endodontic microflora is (KCET-2012)
a) Endotoxin b) Exotoxin 76. The postoperative echymosis and swelling seen
c) Bacterial enzymes d) Microbial interferences periapical region is due to:
(AIPG-10) a) Excessive tension of suture
65. What is the pH of set MTA? b) Oversuture of flap
a) 12.5 b) 14 c) Laceration and tra uma to the surgical flap
c) 10 d) 8.5 d) Retraction of tissue
(AIPG-10)
66. Which of the following protapers have almost similar 77. Possible contra indications for periapical surgery are
size of the tip? a) Controlled diabetis mellitus
a) Sl & Sx b) Sl & S2 b) The danger of damage to adjacent structures
c) S2 & Fl d) S2 & Ll c) A history of rheumatic fever
(AIPG-10) d) Congenital heart disease
67. Fibroblasts in pulp produces collagen (BHU-07)
a) Type I only b) Type II and III 78. Which of the following is false about apexogenesis?
c) Type I and III d) Type II and I a) Done in a non-vital tooth
(AIIMS-09) b) More physiologic process
68. Cytokines associated with periapical lesions c) Pulp tissue with mild inflammation
a) IL-1 b) IL-6 d) Root development continues
c) IL-8 d) TNF-a (KAR- 2013)
(AIIMS-09) 79. Direct pulp capping has a favourable prognosis if the
69. In which of the following is one visit root canal exposure is around
treatment not advocated? a) 1 mm b) 1.5 mm
a) Pulp is necrotic and not symptomatic c) 0.5 mm d) 2 mmm
b) Pulp is necrotic and symptomatic (COMEDK- 2013)
c) Pulp is necrotic and there is a draining sinus t ract 80. Setting time of Mineral Trioxide Aggregate (MTA) is
d) Pulp is vital and symptomatic a) 2-5 minutes b) 5-8 minutes
(COMEDK-2011) c) 1 hour, 45 minutes d) >2 to 3 hours
70. Dry ice is (AP- 2013)
a) Frozen CO2 b) 1, 1,2,2 tetra fluoro ethylene 81. While selecting a post & core system preferably the post
c) Ethyl fluoride d) DichloroDifluoro methane should be
(AP-2011) a) Tapered b) Parallel
71. In chronic hyperplastic pulpitis, the response to vitality c) Threaded d) Non threaded
tests will be (AP- 2013)
a) Normal response b) Hyper response 82. Law of pulse oxymetry is based on
c) Reduced response d) Normal to hyper response a) Doppler's law b) Pascal's Law
(KCET-2011) c) Poille's law d) Beer's law
72. Tevdek sutures used for endodontic microsurgeries are (AIIMS-13)
removed after? 83. The most reliable among vitality tests is
a) 48-72 hours b) 72-96 hours a) heat test b) Cold test

61) C 62) A 63) A 64) B 65) A 66) C 67) C 68) A 69) B>C 70) A 71) C 72) A 73) A
74) B 75) D 76) C 77) B 78) A 79) C 80) D 81) B 82) D 83) D
ENDODONTICS

c) Electric pulp test d) test cavity c) Refer t o physician for the evaluation of maxillary sinus.
(GCET-14) d) Do nothing
84. A patient report to the dentist with trauma to immature (AIIMS NOV-14)
tooth with open apex now dentist plan to do R.C.T. Best 94. Which of the following condition indicate periodontal
way to measure working length in this patient is by ? rather an endodontic lesion?
a) Electronic apex locator a) Acut e pain on vertical percussion with no swelling
b) Radiographic method b) Pain to lateral percussion with wide sulcular pocket
c) Tactile method d) Paper point c) A deep narrow sulcular pocket to the apex with exudates
(AIPG-14) d) Pain to palpation of the buccal mucosa near the tooth
85. 2nd generation apex locators are based on apex
a) Frequency b) Impedance (PGI DEC-2011)
c) Resistance d) Capacitance 95. Pulse oximetry is used in?
(AIPG-14) a) Blood volume b) Blood coefficient
86. Tooth with acute pulpitis will give response to electric c) Laser weavelength for estimation of blood flow
pulp tester? d) Probe with diode is used to measure oxygen saturation
a) High reading than adjacent tooth only (PGI JUNE-2012)
b) High reading than adjacent and contralateral tooth 96. Active GP consists of
c) Lower reading t han adjacent t eeth a) GP with calcium hydroxide
d) Higher reading than opposite tooth b) GP with ZOE
(AIPG-14) c) GP with glass ionomer d) GP with zinc phosphate
87. A neuro peptide molecule that stimulates cell division (PGI JUN E-2014)
in pulpal fibroblasts
a) Epinephrine b) Endorphin
c) Calcitonin-gene-relat ed peptide
d) Vasoactive intestinal polypeptide
(COM ED K-15)
88. All the following are the indicators for single visit
endodontics EXCEPT
a) Acute inflammation of the periapex
b) Nonvital teeth with sinus tract
c) Vital tooth d) Fractured tooth
(PGI JUN E-2014)
89. Which of the following TOLL like receptor is most
specific for bacterial recognition?
a) 1 TLR-1 b) 2 TLR-2
c) 3 TLR-3 d) 4 TLR-4
(PGI DEC-13)
90. Pulp cavity tests are used to check?
a) Pulp blood supply b) Afferent nerve fibers
c) Efferent nerve fibers d) None
(PGI DEC-2011)
91. Hot tooth is?
a) Tooth in line of fracture b) Difficult to anesthetise
c) Vertical line fracture
d) Very sensitive to heat t est
(PGI JUNE-2012)
92. In regenerative endodontic therapy of young permanent
teeth, metronidazole is replaced by
a) Ornidazole b) Doxycycline
c) Chloramphenicol d) Cloxacillin
(MHCET-15)

93. A 12 year old child complains of recurrent pain over


right maxillary pos terior tooth but clinically no caries
can be found. He also reports of cold and fever spells.
What should be advised to the patient?
a) Extract the primary retained tooth i.e., ~4, ~~
b) Extract the tooth 16, 54, 55

84) D 85) B 86) C 87) C 88) A 89) D 90) B 91) B 92) A 93) C 94) B 95) D 96) C
Dental ;lut.,e

8. MISCELLANEOUS - ANSWERS

1. 'B' [Refer synopsis] 12. 'C' [Grossman 12th ed 86)


In thermal and electric pulp testing methods, the diseased
teeth are examined first and t o rule out bias adjacent 13. 'A'
and contralat eral teeth are used as cont rol teeth . (Refer
synopsis) 14. 'A' [Grossman 12th ed 68)
False -ve occurs due to:
2. 'C' [BMC Ear, Nose and Throat Disorders 2006, 6:14] • Pulp stones
Tissue oxygenation and maintenance of microvascular
• High amounts of reparative dentin in the tooth.
blood flow in grafted tissues are crucial for flap viability.
Though several methods can be employed, the best and
False +ve occurs due to
recommended method to assess perfusion of oxygen supply
in a free flap is prick test. No bleeding suggests arterial • Moist gangrenous pulp, which requires maximum current
inflow obstruction while dark blood suggests venous out- to elicit the response.
flow obstruction. • Presence of partially necrotic pulp in one of root canals
in multirooted teeth.
3. 'B' [Refer synopsis]
Thermal testing involves the application of cold and heat 15. 'D' [Grossman 12th ed 101,102)
to a tooth. A response to cold indicates vital pulp and the In case of periapical abscess, the involved tooth is non-vital
patients can quickly point out the painful tooth. A heat while the involved tooth is vital in periodontal abscess.
test is not a test of pulp vitality.
16. 'C' [Grossman 12th ed 60)
An abnormal response to heat indicates pulpal and Percussion test evaluate the status of the periodontium
periapical disorder requiring endodontic treatment. The surrounding teeth. The diseased tooth is examined last in
heat response may be localized or diffused. The preferred percussion test.
temperature for heat test is 65.5°( and for cold test the
preferred temperature is 5°( 17. 'A' [Grossman 12th ed 174)
The hottest part of glass bead sterilizer is along its outer
4. 'D' [Grossman 12th ed 142) rim (AP - 96) and the temperature is lowest in the centre
In all the above cases, when the patient is at risk. RCT can of surface layer of salt . To sterilize an instrument properly,
be done after administration of antibiotic premedication. one should immerse at least a "quarter-inch" below the
• 2gm penicillin V 1 hr before the treatment and lgm 6 salts surface and in peripheral area of the sterilizer. The
hrs after the treatment. main disadvantage of glass bead sterilizer is that only small
instruments can be sterilized. (PGI - 95)
• If the patient is allergic to penicillin, lgm erythromycin
1 hr. before the treatment and 500mg. 6hrs after the
18. 'A' [Cohen 10th ed 255)
treatment.
19. 'A' [Grossman 12th ed 84)
5. 'D' [Grossman 12th ed 68)
False negative res9onse is seen in:
20. 'B' [Grossman 12th ed 60)
a) Recently traumatized teeth
b) Recently erupted teeth with incomplete root formation 21. 'A'
c) Teeth with extensive restoration and pulp protecting
base. 22. 'B'

d) Patients under sedative drugs.


23. 'C' [Grossman 12th ed 49)
Obturation is done after getting successive negative
6. 'A' [Refer synopsis]
cultures from t he canal.
7. 'A' [Grossman 12th ed 69)
24. 'B' [Grossman 12th ed 44]
8. 'B' [Refer synopsis]
25. 'C' [Grossman 12th ed 171-72]
9. 'D'
26. 'C'
10. 'A'
27. 'C' [Grossman 12th ed 305)
11. 'D' [Grossman 12th ed 112]
28. 'A' [Grossman 12th ed 46]
ENDODONTICS

29. 'B' • Pulpless tooth with an unusually short root.


• Internal resorption affecting the integrity and strength
30. 'A' of the root.
• A tooth in which additional root length is desired for
31. 'D' [Grossman 12th ed 259]
improving its alveolar support.
32. 'A' • 40mm extra-long reamers are needed and the root
canal must be enlarged to atleast the size of No. 60
33. ' D' [Cohen 9th ed 20, 236/Grossman 12th ed 72] instrument.

34. 'A' [Cohen 10th ed 255] 40. 'A' [Ingle 5th ed 705, 706/Grossman 12th ed 410]
Ciprofloxacin, Minocycline and Metronidazole comprise
the triple antibiotic paste which can be used as intracanal 41. 'A' [Ingle 6th ed 938]
medicament to disinfect the canal. Minocycline causes The primary G.P. point used to seal the canal is called
discoloration of teeth. That is why Cefaclor has been Master point or primary point.
substituted in place of Minocycline.
42. ' B' [Cohen 9th ed 345/Grossman 12th ed 268]
35. 'A' [Endo-Ortho Relationships IEJ 32, 343-360]
Teeth with previous root canal treatment exhibit less 43. ' B' [Cohen 9th ed 21/Grossman 12th ed 70]
resorption during orthodontic tooth movement. It is Occasionally a tooth might exhibit mixed response to
published that neU1ropeptides will not be released from pulp testing by thermal and electric pulp tests. The most
pulp, as the pulp is removed. This would result in decrease accurate technique to discover a pulp vitality is to begin
of CGRP-IR (Calcitonin Gene Related-Immuno reactive) a preparation in a concealed area of the tooth without
nerve fi bres and a reduction in amount of resorption in anaesthesia. The patient should feel pain if t he pulp is vital
endodontically treated teeth. while the DEJ is passed or as the pulp is approached.

36. 'D' [Cohen 9th ed 119, 120/Grossman 12th ed 505] 44. 'A' [Grossman 12th ed 358]
The digital radiographic resolution is lower than produced Croll Microabrasion technigue
with silver halide emulsions. 18% HCl, pumice and water are mixed
to form a paste to remove the enamel
3 7. ' B' [Ingle 5th ed 215, Last line] discolorations
12-fluted Used for macroabrasion of discolored
38. ' B' [Ingle 5th ed 535/Grossman 12th ed 259]
carbide bur teeth
Balanced force concept uses Flex-R file and is described
as "Positioning and pre-loading an instrument through Nutting Combination walking bleach technigue:
a clockwise rotation and then shaping the canal with 30% H202 + Sodium perborate
a counter clockwise rotation': Many authors discovered Spasser Walking bleach technigue
that greater risk of instrument damage was associated Sodium perborate + water
with clockwise movement': For best results, preparation
MC inn's Mc inn's technigue
is completed in a step-down approach. After mechanical
• 5 parts 30% H202 +
shaping of coronal and middle thirds with Gates-Glidden
• 5 parts 36% HCl +
drill, insertion of the instrument is done with a quarter turn
• 1 part diethylether
clockwise rotation while slight apical pressure is applied.
Cutting is accomplished by making a "Counter clockwise Stewart Thermocatalytic technique
rotation': The counter clockwise rotation and apical Haywood Night guard vital bleaching
pressure act together to enlarge and shape the canal to the and 10% carbamide peroxide in a tray over-
diameter of t he instrument. The clockwise rotation should Heyman the-counter technique
not exceed 90° while the counter clockwise motion must be
Reyto Laser tooth whitening
120° degrees or greater.
45. 'K [Grossman 12th ed 60]
39. 'B' [Ingle 6th ed 1298]
An endodontic implant is a metallic extension of the root
46. 'A' [Cohen 8th ed 25, 61]
with the object of increasing the root-to-crown ratio to give
the tooth better stability. They are useful for treatment of: Condition Characteristics of pain.
• Periodontally involved t eeth requiring st abilization. Trigeminal Severe, sharp shooting pain by
• Transverse root fracture involving loss of apical neuralgia stimulation of a trigger point.
fragment or the presence of two fragments that cannot Cracked tooth Sporadic, sharp sudden jolt of pain
be aligned. syndrome while chewing.
• Pathologic resorption of the root apex incident to
chronic abscess.
Dental ;lut.,e

Acute Quick, hypersensitive response that 51. 'A' [Check Explanation Below]
reversible subsides when the stimulus is removed. If the patient has no gingival recession, the gutta-percha
pulpitis should be trimmed back to 1mm below the Amelocemental
junction. If the patient has gingival recession, the gutta-
Acute Spontaneous, unprovoked, continuous percha should be trimmed back to 1mm below the level of
irreversible pain that lingers after the stimulus is the recession.
pulpitis removed.
Acute apical Spontaneous, throbbing pain that 52. 'C' [Check Explanation Below]
periodontitis persists following removal of stimulus. Advantages of single visit endodontics
1. Immediate familiarity with t he internal anatomy, canal
47. 'C' (Cohen 8th ed 629/Grossman 12th ed 90) shape and contour facilitates obturation.
Internal resorption is caused by chronic inflammatory
2. No risk of bacterial leakage.
tissue in the pulp. So after root canal therapy there are no
chances for internal resorption as the pulp is removed. 3. Reduction of clinic time.
4. Less cost
48. 'A' [Grossman 12th ed 48,49]
Persistent +ve cultures indicate that Leakage between Disadvantages
appointments due to faulty coronal temporary restorations 1. No easy access to the apical canal if there is a flare up.
or undetected perforation. Contamination of canal by any
2. Fatigue to the patient and clinician
sort causes persistent +ve cultures.
3. No opportunity to place an intracanal disinfectant.
49. 'B' [Cohen 6th ed 415)
53. 'B' [Monoblaks in root canal; Journal of endodontics
50. 'D' (Ingle 5th ed 544] 2007 April; 33(4); 391-396)
Protaper instruments: Monoblock means single unit. [)epending on the number
of interfaces present between the bonding substrate and
• Now available as set of 8 instruments
the bulk material core, replacement monoblocks may be
3 - shaping: SX, 51, 52 classified as primary, secondary and tertiary.
5 - finishing: Fl, F2, F3, F4, F5
• Unique design em ploying varying tapers Primary monoblock: has only one interface that extends
circumferentially between the material and the root canal
• Shaping files have tapers that increase coronally
wall.
whereas reverse taper is seen in the finishing files.
• Except SX all files have 14mm Long cutting blades. Eg: i) HEMA containing root filling material (HYDRON)
• Shaping fi les have partially active tips and finishing ii) Orthograde obturation with MTA
files have rounded non cutting tips
• Speed: 250 - 300 rpm. Secondary monoblock : has two circumferential interfaces,
one between cement / sealer and dentin and the other
between the cement and the core material (Fiber post
Tip diameter Taper between or root fi lling material). This is the classical monoblock
File
in mm DO-D3 perceived in the restorative and endodontic Literature.
Sx (No colour) -- --
Eg: i) Cement/sealer+ Guttapercha
51 (Purple) 0.185 --
ii) Panavia cement+ Fiber post
52 (White) 0.20 -- iii) Resilon monoblock obturation system
Fl (Yellow one ring) 0.20 0.07
F2 (Red) 0.25 0.08 Tertiary monoblock: has three circumferential interfaces. A
third interface is introduced between the bonding substrate
F3 (Blue) 0.30 0.09
(cement/sealer) and the core material (post or root filling
F4 ( BLack two rings) 0.40 0.05 material).
F5 (Yellow two rings) 0.50 0.04 Eg: i) Fiber posts that contain an external silicate
coating or unpolymerized resin composite.

• Two usage characteristics are important for protaper. ii) Endorez system, in which gutta-percha cones are
1. Glide path should be created by hand instruments coated with a proprietary resin coating.
before using them. iii) Active GP, gutta-percha cones are surface coated
with glass ionomer filler.
2. Use of Lateral brushing stroke with shaping files to
direct the further used larger files coronally away 54. 'D' [Ingle 5th ed 231/Grossman 12th ed 90)
from danger zones and counteract any screwing /
threading effect. 55. 'B' [Grossman 11th ed 93)
ENDODONTICS

56. 'A' [Practical clinical endodontics by Philip Lumley Pg 64. ' B' [Cohen 8th ed 460, 464 J
103) Any microbe that infects the root canal has the potential to
initiat e a periapical inflammation. However, the virulence
5 7. 'C' [Check Explanation Below] and pathogenicity of individual species vary considerably
DNA hybridization: and can be affected in the presence of other microbes.
DNA strands are allowed to dissociat e and reanneal,
forming hybrid double stranded DNA. This technique Alt hough individual species are usually of Low virulence,
measures the degree of genetic similarity between pools of collectively they are pathogenic due to a combination of
DNA sequences. Commonly used to determine the genetic fact ors. These factors are -
distance between two species. • Interaction wit h ot her organisms in the root canal, so
as t o develop synergistically beneficial partners.
DNA microprocessing
• Release of endotoxins
DNA microprocessor is t he DNA computer. It uses DNA
instead of silicon. The process of using DNA computer to • Synt hesize enzymes t hat damage host tissues.
compute is called DNA microprocessing. • Ability to interfere wit h and evade host defenses
(microbial int erference)
DNA coupling
It is coupling of DNA with glass substrat e or copper or 65. 'A' [Ingle 6th ed 849/Grossman 12th ed 321)
proteins or enzymes., etc. MTA is a biocompatible powder that consists of hydrophilic
particles that set s in the presence of moisture. Hydration of
58. 'B' [Ingle 5th ed 224) the powder res ults in a colloidal gel with a pH of 12.5 t hat
Fast break in canal radiolucency indicates canal bifurcation. solidifies to a hard structure.

59. ' D' [Check Explanation of Q.No.34] MTA prevents microleakage ancl promotes regeneration.

60. 'A' Clinical applications:


• Pulp capping
61. 'C' [Ingle 5th ed 922 / Grossman 12th ed 447) • Apexification
'A' [Cohen 8th ed 815/ 9th ed 844] • Surgical and nonsurgical repair of root perforations and
62.
fractures
The first sign of failure of formocresol pulpotomy is often
internal resorption of t he root adjacent to the area where • Root end filling material
the formocreso l was applied. This may be accompanied by
external resorption, as the failure progresses with more 66. 'C' [Refer Q. No. 50/ Ingle 5th ed 544]
destruction making t he tooth mobile with associated fist ula
development. It is rare for pain to occur wit h t he failure of 67. 'C' [Cohen 8th ed 425)
formocreso l pulpotomy. Fibroblasts are the most numerous cells of the pulp.
These cells synthesize type I and III collagen, as well as
First sign of failure of formocresol pulpotomy is: proteoglycans and glycosaminoglycans. Thus t hey produce
Ans. Internal resorption (AIIMS MAY 2012) and maintain the matrix proteins. As t hey are also able to
phagocytose and digest collagen, fi broblasts are responsible
63. 'A' [Cohen 8th ed 697/ 9th ed 848) for collagen turnover in the pulp.
There are 3 categories of treatment approaches for
pulpotomy in primary teeth. They are 68. 'A' [Cohen 8th ed 469)
The three proinflammatory cytokines that have a central
Devitalization • Formocresol role in periodontal tissue destruction are Int erleukin-1 (Il-
• Ferric sulfate l), IL-6, and tumor necrosis factor (TNF).
• Glutaraldehyde • IL-1P is the predominant form found in human periapical
Preservation • Corticosteroids periodontitis.
• Laser • IL-la is implicated in the pathogenesis of apical
• Electrosurgical periodontitis in rats.
• MTA • IL-6 has been shown to be present in inflamed gingival
Regeneration and adult marginal periodontitis.
• Bone morphogenic proteins
• IL-8 may be active in apical periodontitis in concert
Ferric sulphate is a hemost atic agent. Haemostasis occurs with other chemoattractants.
liy ayylulir1aliun ur liluud µru leins. The ayylulinaled
proteins form plugs that occlude the capillary orifices. A Actions of cytokines:
15.5% solution of ferric sulphate was applied over pulp • Leukocyte adhesion to endothelial cells
stumps for 10-15 seconds and the teeth were restored with • Stimulation of lymphocytes.
stainless steel crowns.
Dental ;lut.,e

• Potentiation of neut rophils 73. 'A' [Cohen 8th ed 16]


• Activation and production of prostaglandins and Laser Doppler Flowmetry is a method to assess blood flow in
proteolytic enzymes microvasculature systems. It is useful for determining pulp
vitality.
• Enhance resorption
7 4. 'B' [Cohen 8th ed 653]
69. ' B>C' [Nishagarg 3rd ed 324]
Pulpal degeneration results in necrotic debris, bacterial
Indications for single visit:
byproducts and other irritants that can move towards
• Fractured anterior teeth where est hetics in concern apical foramen causing periodontal dest ruction with proper
• Vital pulp exposures endodontic t reatment, periodontal disease of pulpal origin
• Symptomatic (acut e) pulpitis should heal on it s own .
• Asymptomatic non-vital pulp with sinus tract. This
75. 'D' [Ingle 5th ed 361]
means no pain and swelling.
All plastic instruments are better sterilized in autoclave.
• Absence of periapical infection
• Teeth used as abutments 76. 'C' [Grossman 12th ed 767]
• Teeth of physically handicapped, geriatric and medically
compromised patients under antibiotic coverage. 77, 'B' [Grossman 12th ed 392)
Contraindications for periradicular surgery:
Contraindications for single visit:
i. Proximity to anatomic structures
• Symptomatic (acute) apical periodontitis with severe
pain on percussion. Eg: Maxillary sinus
• Acute abscess requiring I & D. Nasal fossa
• Asymptomatic or symptomatic non vital teeth with Neurovascular bundles
periapical radiolucency and no sinus tract. ii. Serious systemic health problems
• Teeth wit h anatomic obstacles or procedural faults like
iii. Emotionally distressed patient
broken fi le, perforation
• Over instrumented canals. iv. Inaccessible surgical sites
v. Teeth with poor prognosis
[Even though university has given Option 'C' as correct
Eg: Vertical root fractures
answer, the absolute contrandication of single visit RCT is
symtomatic teeth with necrotic pulp. So we have gone with Non restorable t eeth
option 'B'. Rest is upto you]
78. 'A' [Cohen 10th ed 308)
70. 'A' [Grossman 12th ed 70] Apexification is done in non-vital t oot h. Ref. pedodontics
Dry ice (CO 2 snow) t emperature is-78 degree C. It is able subject, chapter- pediatric endodontics for more details
to penetrate fu ll coverage restorations and elicit a reaction
from the underlying t oot h to cold. 79. '(' [Sturdevants 5th ed 85 6)
Direct pulp capping has a favourable prognosis if t he
71. ' C' [Grossman 12th ed 89] prognosis is less than 1 mm. According to Sturdevant t he
In chronic hyperplastic pulpitis (pulp polyp), tooth respond exposure should be less than 0.5 mm.
feebly or does not respond to thermal test except for
extreme cold application like ethyl chloride spray 80. 'D' [Grossman 12th ed 319]
Composition of MTA:
Pulp polyp is (PGI J une-2012) • Tricalcium silicate
a) Irreversible pulpitis b) Reversible pulpitis • Dicalcium silicate
c) Chronic hyper plastic pulpits • Tricalcium aluminat e
d) All the above • Tetracalcium aluminoferrite
72. 'A' [Practical Clinica l Endodontics By Philip Lumley Pg • Bismut h oxide
103)
Over recent years, suture material choice has moved from Set MTA has a pH of 12.5. It set s in the presence of wat er
black silk to Tevdek braided polyester or monofilament (Hydrophilic). MTA should be mixed with saline and carried
polypropylene sutures such as Prolene. with a messing gun. The hydrated mass slowly solidifies
to a hard structure in about 3 hrs time. So any further
Post-operative Management- Sufficient healing have permanent restorations like obturation restorative fillings
occurred after 48 hours for suture removal; however it should should be delayed upt o 24 hrs.
not be left longer t han 96 hours due t o 'wicking' effect that
may cause post-operative infection of the surgical sit e. For uses of MTA refer Q.No.35 of disinfection and obturation.
ENDODONTICS

Setting time of MTA (PGI June-2014} the moisture or blood on the portion of the paper point that
a) 2 Hour 30 min b) 2 Hour 45 min passes beyond the apex may be an estimation of working
length or the junction between the root apex and the bone.
c) 4 Hours d) 4 Hours 30 mins
85. ' B' (Check Synopsis for apex locator generations]
81. 'B' (Cohen 10th ed 784/ Ingle 6th ed 1439, 1441]
• Types of posts 86. 'C' [Weine endodontic thera py 58]
- Tapered or Parallel
Stat us of pulp Response to EPT
- Threaded or Non-threaded
Hyperemia/reversible Low reading than control teeth
- Metallic or Non-metallic pulpitis
• Tapered posts are the least retentive of all posts. Also Acute pulpitis/ • Low reading than control teeth
they produce high incidence of root fracture when Irreversible pulpitis because of very low threshold
compared to parallel post. of irritability
• Parallel posts have more retention and less incidence • Occasionally normal response
of root fracture when compared to tapered post. Chronic pulpitis High reading
• Parallel posts concentrate stress apically on the post
Necrotic • Negative response
and t apered posts concentrate stress at t he post-core
• False +ve if liquefaction
junction.
necrosis occurs
• Threaded posts produce t he greatest potential for root
fracture and are recommended for post-endodontic Note: Control t eeth for EPT may be neighbouring (adjacent )
restorations. or contralateral teeth.
• Non-threaded posts produce t he least stress and so
have t he least tendency for root fracture. 87. 'C' (Cohen 10th ed 508]
• Parallel post (0 ption-B) is the best preferred post. Proinflammatory neuropeptides are released from pulpal
afferent nerves. They are substance P, CGRP, neurokinin A
(NKA}, NKY, and Vasoactive intestinal polypeptide (VIP).
82. 'D' [Check Pediatric emergency medicine by Jill Ba ren
They cause vascular events such as Vasodilation and
50]
increased vascular permeability.
The principle of pulse oximetry is based on Beer-Lambert
law, which states that the concentration of an absorbing
CGRP demonstrates immunosuppressive activity
substance in solution can be determined from the intensity
characterised by diminution of H202 production by
of light transmitted through that solution.
macrophages, decrease of class-II antigen presentation and
lymphocyte proliferation.
In other words, arterial oxygen saturation is based on
the differential absorption of red and infrared photons by
Substance P and CGRP are mitogenic for pulpal odontoblast
oxyhemoglobin and deoxy-haemoglobin measured by the
like cells and fibroba lsts and propagate the pulpal healing
pulse oximeter.
response.
This law is a combination of 2 laws:
88. 'A' (Check Explanation of Q. No.69]
• Beers law: The i ntensity of t ransmitted light decreases
exponentially as the concentration of the substance 89. ' D' [Text book of immunology by J.K.Sinha 1st ed 23]
increases. Different classes of microbes express different molecular
• Lambert's Law: The intensity of transmitted light patterns that are recognized by different pattern recognition
decreases exponentially as the distance travelled recept ors (PRR) or Toll like receptors (TLR). Since TLRS are
through the substance increases. synthesized before an injection, they are classified as part
of innate immune system. Atleast there are 10 TLRs (TLR
83. ' D' (Clinical endodontics by Tronstad 3rd ed 78] 1-10)
When the sensitivity tests (thermal and elect ric pulp test) • TLR-3 - Is a cell surface receptor for double stranded
give inconclusive results, it will most often be possible to RNA. So it is implicated in viral recognition.
determine whether the pulp is vital or not by the preparation
• TLR-4 - Principal receptor responsible for recognition of
of a test cavity in the crown of the tooth and towards the
bacterial LPS. It is critical t o host defense.
pulp. Test cavity should be a last resort when all other tests
have proved inconclusive.
90. ' B' (Cohen 10th ed 15]
Pulp testing methods assess the responsiveness of pulpal
84. ' D' [Check Ingle 6th ed 929)
sensory (afferent) neurons. The maximum afferent sensory
When RCT is indicated in an immature tooth with wide open
fibers of the pulp are A delta and C fibers.
apex, a relatively reliable means of determining working
length is by gently passing t he blunt end of a paper point
into the canal after profound anesthesia has been achieved,
Dental ;lut.,e

91. 'B' [Cohen 10th ed 692, 673]


Hot tooth is a tooth that is difficult to anesthetize. This
is due to Tetrodotoxin-resistant (TTX-R) sodium channels
which are found primarily on nociceptors. These channels
are relatively resistant to Local anesthetics. This explains
why local anesthetics are less effective when injected into
patients with odontalgia.

92. 'A' [Check Explanation Below]


Regenerative endodontics is a biologically based treatment
for immature permanent teeth. Triple antibiotic paste
contains metronidazole, ciprofloxacin and minocycline.
Some studies replaced metronidazole with ornidazole.

93. 'C' [Neelima Malik 2nd ed 643]

94. 'B' [Cohen 10th ed 658]


• Option A - is due to endodontic lesion
• Option D - May be due to periapical abscess when
there is discharge of pus into the buccal sulcus. Tender
on lateral percussion along with wide pockets are
suggestive of periodontal lesion.
• Option C - This indicates a sinus tract along periodontal
ligament space. This usually the result of a narrow
opening of sinus tract into the gingival sulcus. It can be
traced with a gutta-percha cone to the apex indicative
of endodontic lesion.

95. 'D' [Check Synopsis]

96. 'C' [Check Explanation of Q.No.53]


r ENDODONTICS SYNOPSIS

ENDODONTICS - SYNOPSIS
DIFFERENT DIAGNOSTIC TECHNIQUES: 3 - Mobility more than 1. 5mm with lateral movements and
can be intruded or depressed into the socket.
1. Commonly used methods:
• Visual and t actile inspection 7. Thermal tests: (heat test and cold tests)
• Percussion Pulp sensibility tests elicit neural response from pulp.
• Palpation Eg: Thermal tests

• Mobility and depressibility tests EPT


• Periodont al tests Test cavity
• Thermal tests • For thermal test to give response there should be some
pulp and intact odontoblasts.
• Anesthetic tests
• A delta fibres are stimulated by fluid movement in the
• Test cavity odontoblastic process. Cold causes contraction and heat
• Transillumination causes expansion of tubular fluid.
• Biting NOTE: Where as in EPT, A delta fibres are stimulated
• Staining directly.
• Gutta-percha point tracing for sinus In heat test prolonged heat application will result
• Radiographs in biphasic stimulation of A delta (~ ) fibres initially
followed by C - fibres and may cause lingering pain. So
2. Special methods: heat stimulus is applied for only 5 Sec.
• Xeroradiography • According to Grossman, a response to cold reflects a
vital pulp regardless of whether it is Normal or abnormal.
• Pulse oximetry
A heat test does not confirm vitality.
• Laser Doppler flow metry
• An abnormal response to a heat test however indicates
• Computerized tomography the presence of a pulpal or periapical disorder requiring
• Digital substraction radiography endodontic treatment.
• MRI • Thermal test should be first performed on the teeth to
• RVG be used as controls. (usually contra lateral teeth are
used as controls).
• Computerized expert system
• Location - Place the stimulus on the cervical/ Middle
• Thermographic imaging
3rd because there is less enamel in this region.
• TACT
• While performing these tests, the exposed dentinal
surfaces and restored surfaces should be avoided. The
3. In palpating for enlarged lymphnodes caution should
response of dentin to heat and cold test is usually based
be taken to avoid spread of infection through lymphatic
on the hydrodynamic theory.
system in acute infectious conditions.
• According to Cohen, the preferred temperature for
4. The rationale of mobility test is to evaluate the integrity of performing a heat test is 65.5°C or 150 F.
the attachment apparatus surrounding the tooth. • The temperature of dry ice (CO 2 snow) used in cold test
is -78°C. The disadvantage with CO 2 snow are that it
5. Grades of mobility (according to Grossman and Cohen) may cause infarction lines.
• Dichloro Difluoro methane aerosol, can be used as a
GRADE I Noticeable / barely discernable movement substitute for the CO 2 snow, as it is claimed to be as
of the t eeth within its sockets. effective as CO2 snow and less likely to produce enamel
changes.
GRADE II Lateral/ horizontal mobility within a range
of 1mm or less • Painful response which subsides when the stimulus is
removed from the tooth indicates reversible pulpitis.
GRADE III Movement greater than 1mm or when the
tooth can be depressed into the socket. • Painful response, which lingers after removal of stimulus
indicated irreversible pulpitis.
6. Grades of mobility according to MILLER • Analytical technique pulp tester utilizes a modified
0 - Non mobile (mobility within physiologic limits) technique for thermal tests. It contains a hot Probe tip
and a cold probe tip. The heating of hot probe tip and
1 - Mobility within range of 0-0.5mm
cooling of cold probe tip are controlled separately by
2 - Mobility within range of 0.5 - 1.5mm with lateral the membrane switches on the control panel.
movements.
Dental ;lut.,e

• Heat test - less accurate • The two main varieties of pulp tester available are
Low specific bi-polar and mono-polar. Monopolar pulp testers are
commonly used for vitality tests.

• Cold test - More accurate 12. Xeroradiography:


• It uses a rigid aluminum/ selenium coated photoreceptor
8. Anesthetic test is restricted to patients who are in pain at
plate. The plate is electrically charged, placed in a
the time of test and when the usual tests have failed to
water proof electric cassette positioned in the mouth
identify or localize tne offending tooth.
and exposed to X- rays at lower level of radiation. The
plates can be used repeatedly and the entire process
9. The purpose of performing TRANSILLUMINATION test is to
requires 25 sec for a dry permanent image.
determine the fractured lines, if and when present on the
tooth. • Advantages of xero radiography include:
Produces sharper, clearer and fine details of the
10. Different techniques of staining: image.
• The filling is removed from the suspected tooth and 2% Reduced patient radiation dose.
iodine is placed in the cavity preparation. The iodine Pronounced EDGE ENHANCEMENT EFFECT.
stains the fracture line dark
• A dye is mixed with zinc oxide eugenol and is placed in Pulse oximtery:
the cavity after filling has been removed. The dye will
• It is a non-invasive oxygen saturation monitor.
seep out and stains the facture line.
• Detects pulp vitality by testing the integrity of blood
• The patient is asked to chew a disclosing tablet after
supply to pulp. This method is superior to other vitality
taking out the filling in the suspected fractured tooth.
tests, as it does not rely on lilerve supply to pulp.
The line will be stained.
• Pulse oximtery uses red and infrared wavelengths
11. Electric pulp testing: (EPT) in order to transilluminate a tissue and detects
absorbance peaks due to pulsative circulation and uses
• The electrolyte applied in EPT procedures are Nichollas-
this information to calculate the pulse rate and oxygen
colloidal graphite, Grossman t ooth paste.
saturation.
• The best location for EPT is incisal 2/3rd of labial
surfaces of anteriors and middle third of posteriors. The 13. Laser Doppler flow metry:
reason is that there are pulp horns in these regions and
• A non-invasive method to measure the blood flow.
more A~ fibres are present for stimulation.
• A better and more reliable method for determining the
• During EPT gloves should not be worn since circuit does
pulp vitality then electric pulp testing methods.
not get completed.
• To eliminate a biased decision, EPT, should first be 14. Computerised tomography:
performed on control teeth, favourably, an adjacent
• Blends the concepts of thin laser radiography with the
tooth or contralateral tooth prior to being performed
computed image.
on the effected tooth.
• Gives the 3-d image of roots and root canals.
• EPT cannot be used on patients having cardiac
pacemaker. • Helps to detects the extent of carious lesions extension
of maxillary sinus and its proximity to the root apices.
• EPT is not useful for recently erupted teeth with
immature apex. This is because the relationship • Helps in determining the bucco-lingual and mesio-
between the odontoblasts and the nerve fibers of the distal width of teeth and the presence or absence of
pulp has yet to develop . root canal filling materials and metal posts.
• Reversible pulpitis is characterized by arterial hyperemia Disadvantages:
in which there will be increased amount of blood supply - Expensive
and the tooth responds for lesser amount of current - Time taking
then normal.
- Large skin dose.
• Irreversible pulpits is characterized by pulpal necrosis
and requires more current to elicit a response. 15. Digital substraction radiography:
• Chronic hyperplastic pulpits or pulp polyp, contains • Required areas are enlarged against entire background.
lesser nerve supply then normal pulp. So it also requires
• "IMAGE ENHANCEMENT METHOD ", in which the
more current to elicit response.
area under focus is displayed against a neutral gray
• Electronic pulp testing does not really test the vitality background.
of the pulp as it depends on the nerve supply to the
• Digital substraction radiography is useful in diagnosis of
pulp whereas a pulp vitality depends on blood supply.
incipient carious lesions and to asses the successfulness
of RCT and also periapical lesions.
r ENDODONTICS SYNOPSIS

16. MR I: 3. The mechanical objectives are intended to carve away


• Distinguishes blood vessels and nerves from surrounding restrictive dentin and sculpt a preparation that is thorough ly
soft tissues. cleaned and prepared for obturation three dimensionally.
• M R I should not be used in patients with cardiac
4. Grossman's classification of instruments based on their
pacemakers, metallic rest orations, and ortho-appliance
function:
(Note:- Internal derangement of TMJ can be best
a) Exploring instruments: Smooth broaches, endodontic
diagnosed by MRI)
explorer.
17. RADIOVISIOGRAPHY: b) Debridement (Extirpating instruments)
• Contains 3 components (RADIO, VISIO and GRAPHY Eg: Barbed broaches.
components). c) Shaping (Enlarging) instruments.
• The "RADIO" component consists of a hypersensitive Eg. Reamers and files
intra - oral sensor and a conventional x-ray unit. d) Obturating instruments
• The " VISIO" portion consists of video monitor and Eg: Pluggers, spreaders, lentulospirals
display processing unit.
• The "GRAPHY" component is a high resolution video 5A. ISO grouping of instruments based on method of use:
printer that instantly provides a hard copy of the screen a) Group - I: Hand use only
image.
• Files - K-type and H-type, etc.
• Produces an instantaneous image on a video monitor
• Broaches
thereby reducing radiation exposure by 80%.
• Pluggers
18. Computerised expert system (COMENDEX): • Spreaders
Appropriate diagnostic case facts are entered into the b) Group - II: Same as group I instruments but made to be
computer. The computer checks and gives out the diagnosis. attached to a hand piece. Also included are paste fi llers.

Used for diagnosis of selected pulpal pathosis c) Group - III:


• normal pulp/necrotic pulp i) Engine driven latch type drills or reamers
• Reversible pulpitis / irreversible pulpitis • Gates - Glidden (G-type)
• Infections due to endodontic failures. • Peeso reamers (P-type)
• M-type reamers
19. Thermographic imaging:
• Kurer root facer
• Highly sensitive. non-invasive method of recording the
surface temperature of the body.
ii) Rotary canal instruments
• Provides information on pulpal blood flow and helps in
development of a reliable and objective assessment of • Profile
pulpal vitality. • Light speed
• Quantec
20. Tuned aperture computed tomography:-
• POW-R
• Helps in viewing an object while decreasing the
• HER0-642
superimposition of overlying anatomical structures.
• "TACT image" is composed of a series of 8 digital
d) Group - IV: (Root canal points)
radiographs that are assimilated into one reconstructed
TACT image. • Guttapercha
• TACT images provide better visualization of canals in • Silver points
human molars than conventional films. • Paper points
• An effective diagnostic tool for evaluating primary
recurrent dental caries and simulated osseous defects. 58. Classification of instruments used for cleaning and shaping
(Cohen 10th ed)
ENDODONTIC INSTRUMENTS: a) Group 1: Manual I hand K-type
1. The main stay of endodontics is canal preparation which • K-type
includes cleaning and shaping of root canal. • H-type
• Broach
2. The biological objective of cleaning and shaping procedure
is to remove all the pulp tissue, bacteria and their toxins
from the root canal system. b) Group 2: Low speed latch type
Eg: Gates glidden burs
Paeso reamers
c) Group 3: Engine driven Ni-Ti 12. K-files have 11/2 to 21/2 cutting edges per mm of their
Eg: Profile working end. The angle of flutes to long axis is about 25 -
40 degrees.
Protaper
K3 etc The risk of torsional fracture is less for K-files than for
reamers. These are the first instruments used to extirpate
d) Group 4: Same as group 3 but in addition to adapting pulp. Ultra or endosonics uses K-file.
longitudinally they adopt to the cross-section of the
root canal. 13. Hedstroem files are made from a round blank to produce
spiral flutes.
Eg: Self adjusting fi le (SAF)
They show higher cutting efficiency than K-instruments,
e) Group 5: Engine driven reciprocating but is more prone t o fracture.
Eg: Giromatic
Hedstroem file is the endodontic instrument with maximum
Racer
cutting efficiency.

f) Group 6: Ultrasonic instruments The angle between cutting edge and long axis of the
instrument is about 60 - 65°. Thus they are designed
6. ING LE and LEVINE recommendations: primarily for a linear filling motion.
• Instruments shall be numbered from 10 to 100, the
numbers to advance by 5 units to size 60, then by 10 Due to positive rake angle, they cut in one direction on ly.
points to size 100.
14. K-Flex {D-type fi les)
• Each number shall be representative of the diameter of
• Cross section is rhombus or diamond shaped. The
the instrument in hundredths of a millimeter at the tip.
For Eg: No. 10 is 10/100 or 0.1mm at the tip, No. 25 is rhomboidal blank produces alternating high and low
flutes that are supposed to make the instrument more
25/100 or 0.25mm at the ti p.
efficient in removal of debris.
• Working blade (flutes) begin at the tip designated as
DO (diameter at tip) and shall extend exactly 16 mm Up • They have better cutting abrnity and flexibility than the
the shaft, terminating at 016 (Diameter at 16mm). files wit h square cross section.

According to Cohen 10th ed DO is also termed as cross • The instrument is fabricated of V-4 steel.
section diameter at the 1st rake angle of any file
15. Unifi les:
• The diameter at D16 shall be 32/100 or 0.32mm greater
than that of DO. • Mc-Spadden modified the traditional Hedstroem file,
which were marketed as Burns unifile.
• For example, a No. 20 reamer shall have a diameter
of 0.20 mm at DO and a diameter of 0.20 + 0.32 i.e., • The blades present a S-shape or double-helix design
0.52mm at 016. rather than the single helix teardrop cross-sectional
shape of the hedstroem file.
• This sizing ensures a constant increase in taper of
0.02mm per mm for every instrument regardless of size. • The unifile has two continuous cutting edges, the H-file
has one.
• The tip at the angle of instrument should be 75 ± 15°.
• It is claimed that the unifile can be used for cutting in
7. The traditional reamer is manufactured from a triangular both a filling and a reaming action, hence, referred to
blank, while a file is manufactured from a square blank. as a universal instrument.
• Unifiles are less subject to fracture, but are less efficient.
8. Files are manufactured with tighter flutes, while reamers
are manufactured with looser flutes. • Unifiles are stiff in coronal and middle thirds, but bends
in apical 1/3rd.
9. Triangular blanked instruments cut more effectively but are
less resistant to fracture than square blanked instruments. 16. NICKEL - TITANIUM (Ni - Ti) files:
• Basic composition
10. The reamers are used with a pushing rotating motion (torque) Nickel 54%
whereas files are used with a rasping or pulling motion. Titanium 44%
Cobalt 0.2% - reduces transition temperature
11. Angle of the blade to the long axis of K-reamer is about 10 Boran added to improve surface hardness
- 30°. Hence these instruments are primarily designed to be
used in rotary reaming motion.
• At high temperatures, it exists a body centered cubic
(BCC) lattice, referred to as autestentic phase, which
r ENDODONTICS SYNOPSIS

is stable and stronger. On cooling, this phase t ransfers (iii) M-wire:


to close packed hexagonal, weak phase known as • Thermomechanical process thereby leading to
martensitic phase. increased superelasticity which decreases cyclic
• Ni - Ti alloys also undergo a stress-induced martensitic fatig ue and instrument fracture.
transformation from austenite form . On release of • Eg: Pro ta per Next
stress, the structure reverts back to austenite. Profile vortex
GT series X
• Martensite transformation is induced by low temperature
and high stress.
17. Profile series - 29 (Hand instruments wit h 0.02 taper)
• Ni Ti austenite phase (strong stable phase) ~:
J. E--Stress induced • Profile 29 series hand instruments with 0.02 taper.
Martensitic phase (weak hexagonal phase) • Profile 29 series rotary instruments with 0.04, 0.06
J.E--Stress relieved taper.
Austentitic phase • Profile orifice shaper - 0.05 - 0.08 taper.
• This transformation results in two unique features of • Profile GT rotary instruments.
clinical relevance. • These files have a constant 29% increase in tip diamet er
- Shape memory
- Super elasticity Advantages:
• This super elastic behaviour of Ni-Ti occurs over a • 11 files of ISO 10 - 60 No. instruments are replaced by
limited range with an optimum of 37°C. 8 fi les.
• Ni - Ti fi les suffer no permanent deformation when • More smaller sized instruments and lesser larger sizes.
being used in curved canals, whereas steel instruments • 13, instead of 21 ISO instruments, which have more
undergo permane nt deformation. gradual increase in diameter.
• The main disadvantage of Nitinol is t hat its cutting • The profile orifice shaper have replaced Gates Glidden
efficiency is only 60% than that of matching stain less drill and have been shown to be more efficient in
steel file. preparing the coronal portion compared t o Gates
Glidden. It is also more safer t han Gates Glidden drill
because of its very low speed.
Transformation between the austenitic and martensitic
forms of NiTi can be induced by (KAR- 2013)
a) Temperature b) Stress 18. Golden mediums (malliffer)
c) Both the above d) None of the above • These instruments corresponds in size to half between
standard ISO sizes and are numbered 12, 17, 22, 27, 32, 3 7.
• Advances in Ni-Ti instruments manufactured through • These are series of intermediate sized instruments, and
different thermo-mechanical processes. are a part of flexofile range.
• These flexoreamer and K-flexo fi le instruments in
(i) R-Phase: half sizes made of stain less steel were named Golden
• (R) Rhombo hedral phase is intermediate phase that mediums.
can form during transformation from
t:,, R- Phase 19. MAC-File:
Austenite Martensite • Manufactured from Ni - Ti which gives the inst rument
On cooling "super elasticity".
• Stays relatively loose in the canal.
t, R- Phase
Martensite Austenite • Prevents the canal transportation.
On heating
• Ni-Ti instruments manufact ured in R-phase shows 20. Canal master 'U' instruments:
high flexibi lity and shape memory. • These instruments incorporate 3 major features
• Eg: Twisted fi les A non-cutting pilot ti p (intended to guide the
K3 X F instrument t hrough the canal).
A 1-2mm long cutting head with a minimal cutting
(ii) Controlled memory (CM) files
surface to provide maximum control and limit
• Special thermomechanical process that control
indiscriminate cutting.
memory of Ni-Ti
• Extremely flexib le but wit hout shape memory A smooth round flexible shaft (to allow the
• The instruments does not come to its original shape inst rument t o accurat ely follow curved canals).
in t he canal or on wit hdrawal • The hand instruments are available in sizes 20 - 80 with
• After autoclaving t he shape reverts back to original a number of half sizes between 22.5, 32.5, 42.5, etc.
• Eg: Hyfile X CM • Reduces apical extension of debris.
Dental ;lut.,e

• Creates well centered canal preparation without Ledging Recommended working speed for "Light Speed
and transportation.
Instrument" is (AP- 2013)
• The power assisted canal master instruments are a) 5000-10000 rpm b) 1000-2000 rpm
numbered from 50 - 100. c) 550-650 rpm d) 150-250 rpm
They are Latch type of instruments used in contra-angle
hand piece. They are principally used for coronal flaring 28. Quantec 2000 series:
of the body of canal. • New design Ni-Ti inst ruments designed by Mc Spadden.
• The original Quantec series have 900 tip, which appears
21. Profile series - 29 (Rotary instruments):
to cause problems like zips, elbows and perforations.
• Profile 29 series rotary instruments are available with
• The newer instruments contains non cutting tip or safe
0.04 and 0.06 taper.
cutting tip with 600 angle.
• Profile 0.04 taper is used in terminal part of canal.
• Profile 0.06 taper is used in middle part of the canal. 29. Masserman kit or endoextractor kit is used for removal of
• Profile orifice taper (0.05 - 0.08 taper) is used for broken instruments in root canalls (MAN -98, KAR -03)
preparing coronal section of canal.

22. Flexogates: WORKING LENGTH DETERMINATION


• Derivative of Gates Glidden drill.
1. Working length is the distance from a coronal reference
• Used to enlarge tine apical region of the canal. point to the point where preparation and obturation should
• Less Likely to cause apical transportation end which ideally should be the cementodentinal junction.
• Less # resistant but has breakage point approximately
16mm from the tip making its retrieval easier when 2. Methods of calculating working Length:
broken. A) Radiographic methods:
• Non-cutting guiding tip. • Use of radiographic apex as the termination point.
• A specific distance from the radiographic apex -
23. Giromatic is commonly used reciprocating hand-piece. It lngle's method.
accepts only Latch-type instruments. In this device, the
• According to studies of Kuttler.
quarter turn motion is delivered 300 times per minute.
Various instruments that are used with Giromatic • Others.
• Broach type Rispi file B) Electronic apex locator
• H - style Gire file
C) Paper point evaluation
• K - style Helifile
D) Tactile method
24. Canal finder system delivers a vertical stroke ranging from
0.3 to 1mm. The more free ly the instrument moves in the E) Future trends
canal, the larger the stroke. • Xeroradiography
• Direct digital radiography
25. Peeso reamers are more often used in preparing coronal
portion of the root canal for post and core. Instrument • Surgical operating microscope
fractures always near the shank. • Laser optical disk storage

26. HERO 642: 3. Use of radiographic apex as the termination point :


• High elasticity and rotation with 642 taper of 0.06, In this method, the radiographic apex is taken as t he only
0.04 and 0.02. reproducible site. But filling to the radiographic apex means
filling a "bit too Long".
• Used in a special contra-angled hand piece with a slow
speed of 300 -600 rpm.
4. Specific distance from the radiographic apex:
• The micromega helifiles, which are used in Crown-Down
• The tooth is measured on the preoperative radiography
technique are modified HER0-642 Ni Ti instruments.
and from this length of tooth, atleast 1mm safety
allowance is subtracted to conform the apical
27. Light speed:
termination of root canal at the CEJ.
• 1300 - 2000 rpm.
• In this technique a definite, repeatable plane of
• Very flexib le non cutting shaft. rd~ r~11c~ lo 011 anatomic landma rk 011 Lh~ looUr ar~
• Short effective cutting blade. taken (For intact or well restored teeth, incisal edge of
anterior teeth and cusp height on posterior tooth) .
r ENDODONTICS SYNOPSIS

• Wein modification: • Compares the resistance and


- If radiographically, there is no resorption of root capacitance information to a database
end or bone, shorten the length by 1mm. to determine the apex.
4th
- If periradicular bone resorption is apparent, shorten generation • Eg:
by 1.5mm, and if both root and bone resorption are - Elements diagnostic unit
apparent, shorten by 2mm.
- Bingo 1020 / Ray - X4
5. Grossman's method:
2. Integration of apex locator with the endodontic slow speed
• An instrument is placed in the root canal and a handi piece.
radiograph is taken. Eg: Triauto Zx
• By measuring the length of radiographic images of both X-Smart dual
the tooth and measuring instrument, as well as actual
length of the instrument, the clinician can determine These have the following automatic functions:
the actual length of the tooth by a mathematical (i) Handpiece automatically st ops and reverses once the
formula . torque limit is exceeded.
(ii) The file rotation stops and reverses at the moment the
Actual length of tooth = file tip extends beyond the apical constriciton

Actual length x Radiograph length of CONCEPTS OF CANAL PREPARATION


of instrument tooth
Radiographic length of instrument Apical to coronal approach Coronal to apical approach
• Standard technique • Crowndown pressureless
6. Aoex locators·
• step back • Step down
• Also known as resistance apex locators • Circumferential filing • Double flare techinque
• Work on the principle of Resistance Eg:
• Anticurvature filing • Hybrid
Neosono apex locator
1st • Modified double ft.are
generation • The audiometric method is a variation
on the principle of electrical resistance • Balanced force technique
apex
locators of comparative tissues uses a low
frequency ossillating sound to indicate 1. Step-back method:
when similarity to electric resistance has • It is also called as a flare or telescopic or serial root
occurred by a similar sound response. canal preparation of the root canal.
• works on the principle of impedance • Once the canal has been enlarged in the apical third
• Eg: to atleast No. 25, each consecutive larger root canal
instrument used for shaping the canal is placed short of
- Endocater {AIIMS - 2005)
the apex.
- Sono explorer (earliest) • This results in apical enlargement and marked taper
2nd
- Apex finder Endoanalyzer from apical to coronal.
generation
(combined apex locator and pulp
apex
tester) Advantages:
locators
- Digipex, • Less chance of periapical trauma
- Exact-A-Pex • Facilitates removal of more debris
- Foramatron IV • The development of apical matrix or apical stop prevents
- Pio apex locator overfilling of the root canal
• works on principle of frequency or • Greater condensation pressure can be exerted, which
comparative impedence. often fills lateral canals with the sealer
• Eg:
Disadvantages:
- Endex, Root ZX (AIIMS-2004)
Third • Apical extrusion of the debris through the apex
- Neosono Ultima Ez
generation
apex - Mark V Plus 2. Modified step back:
locators - J USTWO or JUSTY II • The preparation is completed in the apical area and
- Apex finder A.F.A then the step back begins 2- 3mm up the canal.
• Trio Auto ZX is a cordless electric • This gives a short almost parallel retention form to
endodontic hand piece with a built in receive primary gutta-percha cone.
Root ZX apex locator.
Dental ;lut.,e

3. Stepdown technique: • It is more effective in very fine/ very curved canals and
• Gates Glidden drills or large sized files are used in the partially calcified canals.
coronal 2/3rd of the canal and progressively smaller • Uses EDTA as irrigation solution.
files are used from the coronal preparation until desired
length is obtained. 8. LASER Canal preparation:
• Uses ND-YAG (Neodynium Yttrium Aluminium Garnet)
Advantages: laser mounted with fibreoptic to clean and shape the
• Eliminates the extrusion of the debris through the apex root canal.
during instrumentation. • Apical region is prepared with hand instrumentation.
• Achieves complete cleansing of the canal. Helps in • Transformation of dentin into an ionized gas leaving as
achieving a biocompatible seal at the apex. debris on canal walls (Plasma effect ) is observed with
• Prevents post treatment discomfort. this technique.
• Provides coronal escape way that reduces the piston in • Produces remarkable cleanliness of root canals.
cylinder effect responsible for debris extrusion from the
apex. 9. Sonic preparation:
• Operates at 1500 - 6500 cycles / min.
4. Hybrid technique: Proposed by Goenig and Buchanan
• Step down approach was preferred for sonic preparation.
• Uses both step down and step back concepts of
• There is higher incidence of apical zip and elbow
preparation.
formation in this technique.
• Early radicular access (ERA) is obtained with Gates
Glidden drill No. 1 and advancing upto 6 (Step down 10. Ultrasonic preparation:
technique).
• Consists of a peeso electric ceramic unit that generates
• The apical region is enlarged with step back technique. ultrasonic waves which act ivate a magnetostrictive
hand piece.
5. Modified double flare technique:
• The hand piece produces movements of the shaft of the
• One of the better techniques best suited for preparing fi le between 0.001 to 0.004 inches in a frequency of
fine and curved canals (AIPG-96) 20,000 - 25,000 / sec.
• NaOCl is used as irrigant solution.
6. Balanced force concept:
• Increased thermal and mechanical ability of the
• Roane and Sabala developed this technique.
irrigating solution removes debris and tissue effectively
• New K-type fi le design known as flex-Rfile or flex-o fi le and removal of smear layer is also efficient.
or any flexible K-file with a non-cutting tip can be used.
• Coronal flaring should be done with gates glidden drills. 11. Canal Master 'U' preparation:
• Involves 3 steps • This canal master 'U' is not a file or not a reamer.
• It closely resembles Gates Glidden drill in appearance
Step - 1 and action.
Instrument is inserted passively and a 90 degree • This techniques follows a hybrid approach to prepare
clockwise rotation given to engage dentin the canal.

Step - 2
12. Techniques for preparing narrow and curved canals are:
Instrument is held in the canal with sufficient apical
force and then rotated counter clockwise 180-270 • Step down
degrees. The dentinal shavings are removed with a • Modified double flare
clicking sound. • Balanced force technique

Step - 3
Mineral trioxide aggregate (MTA):
The file loaded with dentin debris is removed from the
canal with a clockwise rotation (This loads the debris to • Developed by Mohammad Torabinejad at Lomalinda
the flutes). University.
• Available as grey and white IMTA
• BFT reduces ledging and canal transportation and has • Composition
excellent canal centering ability - Di- and Tricalcium silicate
- Tri Calcium aluminate
7. Canal finder system: - Tetra Calcium aluminoferrite (absent in while MTA)
• Uses canal finder system handpiece with a special H-file Bismuth oxide - Opacifier
utilizing hybrid method of canal preparation.
r ENDODONTICS SYNOPSIS

• Indications: Moisture is req uired for its setting. So blood and


- Pulp capping body fluids have Little impact on the leakage.
- Perforation repair
• Dougan or amalgam carrier can be used to carry the
- Root end filling
mixed MTA.
- Pulpotomy
• Once it was proposed t hat a moist cotton pellet should
- Apexification be placed over the MTA in the root canal until it
• Powder is mixed with distilled water. The hydrated mass sets. But now it is advocated that moisture from the
sets in about 3 hrs and has a pH of 12. 5 periapical fluids is sufficient and moist cotton pellet is
• Advantages: not required.
- Superior den1!:inal bridging • GIC has better bonding than composite to MTA. So a
- Antimicrobial layer of GIC should be laid over MTA before composite is
Biocompatab Le done as a coronal restorative material.

ENDODONTIC ANATOMY
Tooth Root anatomy Floor of pulp chamber Access cavity Other features

MAXILLA One root, one canal Oval mesiodistally Triangular with apex
Central incisor towards cingulam
Maxillary centrals and canines
are least common teeth to show
Lateral One root. one canal Oval Labiolingually than Triangular but smaller bifurcated roots.
incisor Distal curvature of root is central incisor anatomy Palatal abscess formation is
characteristic characteristic of lateral incisor
Canines One root and one canal Ovoid labiolingually Ovoid

First premolar Two roots with two canals Elliptical (or) kidney Ovoid in buccolingual
and two foramina shaped (AIPG -94) direction
-----

Second One root with one canal and Elliptical or ovoid Ovoid in buccolingual
premolar one foramina or one root, direction -----
two canal and two foramina

First molar Three roots , three canals. Triangle with lingual Triangle in mesial half Usually (>80% by microscope)
wall missing of tooth . Base towards 4th canal is found in mesio-
buccal and apex towards buccal root in palatal direction.
palatal side This makes the access cavity
rhomboidal. (COMED 2014)

Secondmolar Three roots and canals Same as above Same as above -----
Third molar May have fused roots, one Same as above Same as above
conical root or four or more One may find c- shaped pulp
independent roots. Root chamber with a c-shaped root
canals may be one to five in canal
number.
MANDIBLE
Central One root, one canal (or)
incisor one root, two canal, with Ovoid labiolingual Triangular shaped or
one apical foramen direction ovoid shaped. Among single rooted teeth,
Lateral incisor Same as above Same as above Same as above bifurcated roots are common ly
seen in mandibular 1st i;iremolar
Canines One root one canal or two Ovoid Ovoid funnel shaped followed by mandibular incisors
roots or two canals and canines.

First premolar One root one canal or one Ovoid Ovoid in buccolingual
root and two root canals direction.
Dental ;lut.,e

First molar 2 roots & 3 canals. Two in Rhomboidal Trapezoidal (AP-2013)


mesial root and one in distal in mesial half of crown. About 30% of mandibular molars
root. It becomes rectangular shows 4th canal in distal root
if a 4 th canal is present

Second molar 2 root & 3 canals. Same as above. Same as above


The mesial root has 1 canal and 1
C-shaped morphology
foramen in 27% cases
most common

Third molar Usually have 2 roots and 2 Same as above Same as above
-----
root canals

ENDODONTIC TRAUMATOLOGY:

Clinical .
Type of Trauma . Treatment Import ant points
presentation
Uncomplicated • Most common • Composite restoration If remaining dentin is less than 0. 5 mm calcium
crown fracture • Enamel and hydroxide liner is given before composite
( Ellies 1 & II) or dentin are
exposed
Complicated • Pulp is Immature teeth: • Pulp capping success rat e is less because
(Ellies Class exposed • Less than 24 hrs: inflamed pulp is not removed and capping
III) • In 24 hrs, only material is placed over the inflamed pulp.
- Pulp capping
2 mm pulp will
- Partial pulpotomy
be inflamed • Partial pulpotomy has high success.

• More than 72 hrs:


• In mature teeth full pulpotomy is contra-
Full pulpotomy (less success)
indicated.

• If Pulp is nonvital:
• In fu ll pulpotomy pulp testing is not possible
- Apexification because there is loss of entire coronal pulp
- Revascularization
Ca (0 H) 2 apexification disadvantages:
Mature teeth:
• Less than 24 hrs: • 3-8 mont hs (Long time) of t reatment time.
- Pulp capping • Weaken the root dentin
- Partial pulptomy • Fracture of tooth
• Irregular swiss cheese dentin bridge.
• If pulp therapy is not possible :
- Pulpectomy (More predictable)
Crown-root Usually oblique • Removal or reattachment of fractured RCT is required if pulp is involved.
fracture fracture fragment.
• Removal of coronal fragment and Surgically or
orthodontically extruding the root fragment
or simple gingivectomy.
r ENDODONTICS SYNOPSIS

Root fracture: • Infrequent Repositioning and semi-rigid splinting for 2-4 In most cases pulp necrosis occurs in coronal
• Cervical • Coronal weeks. fragment only.
• Middle third fragment can Cervical #
be displaced
• Apical If repositio ning is not possible then coronal Healing pattern of the fracture site:
from none
(apical #) segment is extracted and apical fragment is 1. Healing with calcified tissue.
to severe orthodontically extruded. 2. Healing with interproximal connective
(cervical #) . tissues.
Mid root#:
• Three angle 3. Healing with inter proximal bone by
radiography If coronal segment is necrotic: connective tissue
at 45, 90 & Treat the coronal fragmentjustli ke apexificaion. 4 . Interproximal inflammatory tissue without
110 degrees If both segments are necrotic: healing
necessary for
• Surgical removal of apical fragment.
diagnosis.
• Endodontic implants First 3 patterns are considered success.

Apical root #:
• Prognosis is excellent
• Most of t he times pulp is vital in both
fragments
• If coronal pulp dies, then t reat the fragment
just like apexification .
Luxation injuries :
Concussion - Sensitive to - No treatment • There is damage of periodontal ligament and
percussion - repeat Vitality tests cementa[ layer in luxation injuries least with
- No concussion and intrusion the most.
displacement • 2 types of resorption may occur
- No mobility i) External resorption- Damage to cementa[
layer predisposes to external root
resorption
ii) Internal resorpt ion- if pulp tissue becomes
infla med it may predispose to internal
resorption

Types of external root resorption:

Subluxation - Sensitive to - No treatment 1. Surface resorption


percussion - repeat Vitality tests
• Occur in concussion and subluxation.
- increased
• Only localised area of resportion.
mobility
- No • Not visualised radiographically
displacement • Spontaneous repair occurs.
Lat eral luxation Dispalcement 1. Repositioned as soon as possible.
2. Replacement/ Ankylosis resorption:
labially, lingually, 2. Semirigid splinting for 4 weeks is indicated.
distally or • Occurs in intrusive Luxation or avulsion with
3. In mature apex and if apex moved into cortical extended dry time.
incisally.
plate initiate RCT within 2 weeks (7-10 days) • Diffuse damage on >20% of root surface.
Sometimes
after t he injury. • Repair occurs with formation of bone
perforates t he
cortical plate 4. Immature apex: Wait for signs of resulting in abnormal attachment.
revascularization. If pulp fails to recover and
becomes nonvital - apexification is indicated 3. Inflammatory root resorption:
• Necrotic pulp provides stimulus fo r
periodontal ligament space inflammation
in addition to cementa[ damage caused by
trauma.
Intrusion Displacement 1. Orthodontic extrusion or immediate surgical
• This inflammation causes root resorptio n as
in an apical repositioning. well as bone resorption.
direction into the 2. Semirigid splinting for 4 weeks. • This is the most damaging resorption and is
alveolus
3. 3rd and 4 points same as Lateral Luxation. due to not performing RCT when indicated.
Avulsion already discussed in QUESTIONS
Dental !J>u£je

DENTAL MATERIALS
I. REFERENCE BOOKS TAKEN:
1. PHILLIPS' SCIENCE OF DENTAL MATERIALS - 11th, 12th & 1st South Asian (SA} Edition

2. TEXT BOOK OF DENTAL MATERIALS by CRAIG - 12th Edition

1. PHYSICAL PROPERTIES
1. Stress is defined as b) Elasticity of the test spedmen in strain
a) An applied load or force c) Length of the test specimen beneath the force
b) A deformation resulting from an applied load ct) Strain of the test specimen per unit length
c) An external force opposing an applied load (MAN -95)
d) An interna l force opposing an applied load 6. The greatest stress which may be produced in a
(KAR -98, AP -02) material such that stress is directly proportional
2. Strain is defined as to the strain is know as
a) An applied load or force a) The elastic limit b) The proportional limit
b) A deformation resulting from an applied load c) The yield strength d) Modulus of elasticity
c) An external force opposing an applied load (MAN -98, KAR -97)
d) An internal force opposing an applied load 7. The point at which a st ress of a material exhibits a
(AIPG -94, AIIMS -04) specific limited deviation P is called
3. The proportional limit is defined as a) Proportional limit b) Tensile strength
a) The maximum stress in a structure. c) Ultimate strength d) Yield strength
b) The minimum force required to cause a structure (MAN -2K)
to break 8. Hardness number which does not depend on the
c) The maximum stress that can be induced without ductility of metal
permanent deformation a) KHN b) VHN
d) The maximum elongation under tension that can c) RHN d) BHN
be measured before failure. (MAN -2K)
(MAN -94) 9. Ability of an orthodontic wire to spring back to its
4. The modulus of elasticity is defined as original shape is evaluated by
a) The stress at the proportional limit a) Brittleness b) Resilience
b) The strain at the proportional limit c) Tensile strength ct) Toughness
c) The stress/strain ratio within the proportional limit (MAN -01)
d) None of the above 10. Brinnel hardness number of a dental gold alloy is
(MAN -94) directly proportional to its:
5. Compressive stress is computed by dividing the a) Tensile strength b) Elongation
external fo rce by the c) Modulus of elasticity
a) Area of the test specimen upon which the weight d) Modulus of resilience
rests (AIPG -91)

1) D 2) B 3) C 4) C 5) A 6) B 7) D 8) A 9) B 10) A
, DENTAL MATERIALS

11. Ultimate tensile strength refers to: 18. Ductility of a material is a measure of it's:
a) Stress before mpture a) Grain elongation b) Annealing
b) Stress after rupture c) Cold working d) Work hardening
c) Longitudinal compressive strength
d) Horizontal compressive strength 19. The ability of an alloy to withstand mechanical
(AIPG -97) stresses without permanent deformation is
12. When solid gets wet completely contact angle is: reflected by its
a) goo b) oo a) Resilience
c) 0 - 90° d) >90° b) Elastic limit/ Elastic Strain
(AIPG -93) c) Hardness
13. Property of the material, which describes the d) Fatigue resistance
resistance to abrasion, is (KAR -99, KAR-2013)
a) Hardness b) Yield strength 20. The stiffness of a dental gold alloy is determined
c) Modulus of elasticity by its:
d) None of the above a) Proportional limit
(AP -99) b) Modulus of elasticity
14. Modulus of elasticity means c) Ultimate tensile strength
a) Rigidity or stiffness of the material d) Flow
b) Ability to be stretched with permanent deformation (KAR -97)
c) Ductility of a material 21. The ability of the base to resist occlusal forces and
d) Malleability of the metal to support the restoration is affected by
(AIIMS, AIPG -03) a) Compressive strength
15. A material behaves in certain ways above and b) Tensile strength
below the proportional limit on a stress strain c) Young's modulus
curve, which of the following is correct: d) Modulus of elasticity
a) Above the proportional limit a material function (KAR -04)
in a plastic manner, while below the proportional 22. Coefficient of thermal expansion of which of
limit it behaves as an elastic following is most similar to that of tooth?
b) Above the proportional limit a material functions a) Gold in lay b) Acrylic resin
in an elastic manner, while below the proportional c) Silicate cement d) Gold foil
limit it behaves as an plastic (AIIMS -94)
c) Either A or B d) Neither A or B 23. Flow of a material refers to:
(AIPG -02) a) Continued change of the material under a given
16. Sublimation is the conversion of a load
a) Solid directly to gas b) The consistency of a material when mixing
b) Gas to liquid and then to solid c) The homogenecity of gypsum products
c) Gas directly to liquid d) Dimensional change of the material during settings
d) Solid to liquid and then to gas (AIIMS -98)
(KAR -99) 24. The wetting of an adherent surface by an adhesive
17. Which of the following physical property can be is related to:
used to estimate indirectly the proportional limit a) Surface energy of the adtierent
of gold alloys? b) Surface texture of the adherent
a) Vickers hardness number. c) Surface tension of the adherent
b) Knoop hardness number. d) Nature of the adherent that is crystalline or
c) Brinell hardness number. amorphous
d) Rockwell hardness number. (AIPG -01)
(KAR -97)

11) A 12) B 13) A 14) A 15) A 16) A 17) C 18) A 19) B 20) B 21) A 22) C 23) A
24) A
Dental ;lut.,e

2 5. Which of the following properties of dental 34. Space lattice refers to:
materials is time dependant a) Inter atomic movement
a) Creep b) Resilience b) Inter atomic imbalance
c) Elastic limit d) Ultimate strength c) Arrangement of atoms
(PGI -2K) d) Arrangement of molecules
26. Which of the following has the highest modulus of (COMEDK-06)
elasticity? 35. The elastic or plastic deformation to fracture a
a) Dentin b) Enamel material is its:
c) Amalgam d) Composite Resin. a) Toughness b) Brittleness
(KAR -04) c) Young's Modulus d) Proportional limit
27. Which of the following hardness test is a micro (AP- 06)
hardness test? 36. Munsell system is used to:
a) Brinnel b) Knoop a) Define and measure colourr qualitatively
c) Shore-A d) Rockwell b) Define and measure physical properties of gold
(APPSC 99) alloys
28. A restorative material which has a high proportional c) Evaluate brittleness of different alloys
limit, compared with one with a lower proportional d) Measures different colours quantitatively
limit would have (AIPG-07)
a) Greater ductility b) Greater toughness 37. The knoop Hardness Number of Microfilled
c) A higher modulus of elasticity Composite Restorative material ranges from:
d) More resistance to permanent deformation a) 50 - 60 KH N b) 25 - 35 KHN
(APPSC -99) c) 15 - 20 KH N d) 40 - 48 KH N
29. A fluid having constant viscosity that does not (K-CET-07)
depend upon the strain rate is said to be 38. A crystal in metal substructure is:
a) Viscoelastic b) Pseudoplastic a) Grain b) Nucleus
c) Plastic d) Newtonian c) Stroma d) None
(AP-07)
30. Liquids which become more rigid as the rate of 39. Etching of dentin does not include:
deformation increases is termed as a) Removal of smear layer
a) Thixotropic b) Pseudoplastic b) Exposure of collagen fibres
c) Dilatant d) Newton c) Opening of dentina l tubules
d) Increases surface energy
31. Which one of the following is the result of applying (AIIMS- 06)
a load to a wire below its modulus of elasticity on 40. When two metals are completely miscible in
a load deflection diagram? liquid state and they remain completely mixed on
a) Fracture of the wire b) Permanent deformation solidification,the alloy formed is called
c) Spring back d) Increase in stiffness a) Solid solution b) Eutectic mixture
(AIPG-05) c) Peritectic mixture d) Inter metallic compounds
32. The strain that occurs when a material is stressed (KAR-03)
to its proportional limit is: 41. Non metal which conducts electricity
a) Resilience b) Maximum flexibility a) Graphite b) Carbon
c) Toughness d) Elastic limit c) Acrylic d) Porcelain
(AP- 06) (KAR-01)
33. KHN of enamel is close to: 42. For adhesion to be present between solid and liquid
a) Pure gold b) Amalgam a) Liquid should wet solid surface
c) Porcelain d) Composite b) Mechanical interlocking should be present
(PGI-05) c) Should have high contact angle

25) A 26) B 27) B 28) D 29) D 30) C 31) C 32) B 33) C 34) C 35) A 36) D 37) B
38) A 39) D 40) A 41) A 42) A
, DENTAL MATERIALS

d) None of the above 52. Coefficient of thermal expansion of tooth enamel


(AP-08) is closest to
43. The melting point of silver is a) Silicate cement b) Composite
a) 1063° Centigrade b) 960° Centigrade c) GIC d) Polycarboxylate
c) 850° Centigrade d) 1123° Centigrade (AP-10, KAR- 2013)
(KAR-02) 53. In measuring Vickers hardness no. which of the
44. Brazilian test is used to determine the ultimate following is used:
tensile strength of a) Spherical indenter b) Steel ball indentation
a) Ductile materia ls b) Flexible materials c) Rhomboid shaped indenter
c) Brittle materials d) Malleable materials d) 136-degree diamond pyramid
(COMEDK-08) (PGI DEC -2011)
45. KHN value of Amalgam is 54. Deformation that is recovered upon removal of an
a) 90 b) 343 externally applied force or pressure is known as
c) 67 d) 450 a) Young's modulus b) Plastic strain
(BHU-07) c) Elastic strain d) Flexural strain
46. Angles between Adhesive and Adherent is Zero (KAR-2013)
degree, it indicates: 55. A good adhesive should possess all of the following
a) Complete wetting of surfaces EXCEPT
b) Rough surfaces between Adhesive & Adherent a) High adherend wetting
c) Adherent and adhesive molecules are tangent to b) High contact angle
each other c) Low contact angle d) High flow
d) Irregularities present between Adherent surfaces (KAR-2013)
(AIIMS-07) 56. Molecule with permanent dipole
47. Maxwell - Voigt Model is to determine: a) Oxygen b) Helium
a) Elastic Behavior b) Plastic Behavior c) Liquid nitrogen d) Water
c) Viscoelastic Behavior (AIIMS NOV-13)
d) All the above 5 7. Amount of heat that is required to change boiling
(MCET-07) water into vapor is referred to as
48. A material which partially transmits light & a) Latent Heat of vaporization
partially scatters it b) Latent Heat of sublimation
a) Reflector b) Translucent c) Latent Heat of condensation
c) Transparent d) Opaque d) Latent heat of fusion
(MCET-10) (AIIMS NOV -13)
49. Materials which has high compressive strength and 58. Time dependent deformation produced in a
low tensile strength is classified with property of completely set solid subjected to a constant stress
a) Ductility b) Brittleness is termed as
c) Malleability d) Resilient a) Static creep b) Elongation
(AIIMS-08) c) Dynamic creep d) Flexibility
50. The forces that hold atoms together are called (MCET-14)
a) Primary forces b) Secondary forces 59. The type of strain develop when force is applied
c) Cohesive forces d) Ionic forces perpendicular to surface
(KCET-10) a) Compressive strain b) Tensile strain
51. The simplest alloy is a c) Shear stress d) Tensile stress
a) Solid solution b) Eutectic alloy (AIIMS MAY-14)
c) Peritectic alloy d) Ternary alloy
(KCET-10)

43) B 44) C 45) A 46) A 47) C 48) B 49) B 50) C 51) A 52) C 53) D 54) C 55) B
56) D 57) A 58) A 59) A
Dental ;lut.,e

1. PHYSICAL PROPERTIES - ANSWERS


1. 'D' [PHILLIPS' 11th ed 77/ 12th ed 50] The Knoop hardness test is used to obtain the values
Internal resistance to a load that for both exceedingly hard and soft materials. The
Compressive hardness value is independent of the ductility of
tends to compress or shorten a
Stress the material.
body
Internal resistance to a load that 9. 'B' (PHILLIPS' 11th ed 84/ 12th ed 55)
Tensile Stress
tends to stretch or elongate a body Resilience is the amount of energy absorbed by a
Resistance to the sliding of one structure when it is stressed to proportional limit. The
Shear Stress portion of a body over another by elastic area of the stress-strain graph gives resilience;
twisting a torsion load. entire area up to breaking point is a measure of
toughness. The slope of straight line gives Young's
2. 'B' [PHILLIPS' 11th ed 76/ 12th ed 51] modulus.

3. 'C' [PHILLIPS' 11th ed 85/ 12th ed 57] 10. 'A' (PHILLIPS' 11th ed 97/ 12th ed 64]
Stresses above proportional limit cause plastic Brinnel and Rockwell tests are classified as macro
deformation . This st ress is called yield strength or hardness tests and they are not suitable for brittle
proof stress. It is the greatest stress, produced in a materials.
material such that stress is directly proportional to
the strain The Knoop and Vickers tests are classified as micro
hardness tests. Both of these tests employ loads less
4. 'C' [PHILLIPS' 11th ed 80/ 12th ed 55] than 9.8N.
Stress / Strain ratio within the proportional limit is
called Elastic modulus or Young's modulus. It measures The Shore and the Barcol tests are used for measuring
the relative rigidity or stiffness of material. the hardness of rubbers and plastics.

5. 'A' [PHILLIPS' 11th ed 75/ 12th ed 51] The Brinnel test is the one of oldest tests used for
Stress is force per unit area perpendicular to the force determining the hardness of materials and is directly
direction. related to proportional limit and the ultimate
tensile strength of dental gold alloys.
6. 'B' (PHILLIPS' 11th ed 87 / 12th ed 57)
The convenience of the Rockwell test, with direct
7. 'D' [PHILLIPS' 11th ed 87/ 12th ed 57) reading of the depth of the indentation, has lead to
Yield strength is the stress required to produce plastic its wide usage.
deformation.
The Knoop hardness test is used to obtain the values
8. 'A' (PHILLIPS' 11th ed 98/ 12th ed 65) for both exceedingly hard and soft materials. The
Hardness test Material used to measure hardness value is independent of the ductility of
the material.
Knoop Diamond of rhombic shape (The long
hardness test axis of indentation is measured)
The Vickers test is employed for dental casting gold
Vickers Diamond of square base shape (The alloys. It is suitable for determining the hardness of
(Diamond diagonal length of the indentation is brittle materials.
pyramid test) measured)
Diamond of conical shape (The 11. 'A' [PHILLIPS' 11th ed 85/ 12th ed 57)
Rockwell
penetration depth is measured) Ultimate tensile strength (breaking point) 1s the
Steel ball (The diameter of stress at the point of fracture.
Brinells
indentation is measured)
, DENTAL MATERIALS

12. 'B' [PHILLIPS' 11th ed 38/ 12th ed 26]


Linear co-efficient of thermal expansion
The contact angle is the angle formed by the of some materials.
adhesive with the adherend at their interface. The
less the contact angle, the more the wettability and Porcelain 6.6
spreadability. Dentin 8.3
• If the contact angle is < go 0 - the result is rise Silicate 10
of liquid.
Type II GIC 11
• If the contact angle is > goo - the result is
depression of liquid. Enamel 11.4
• The contact angle between saliva and acrylic Amalgam 25
denture base is 75°.
Denture resins 81
• The contact ang le between glass and mercury is
Composites 14-50
135°.

Larger contact angle of the adhesive on a solid • Coefficient of thermal expansion of tooth enamel
surface is (APPG-15) is closest to type II GIC followed by silicates.
a) Complete spreading b) More wetting
c) More adhesion d) No wetting 23. 'A' [PHILLIPS' 11th ed 46/ 12th ed 33]
Flow is the deformation under a small static-load even
13. 'A' [PHILLIPS' 11th ed 43,96/ 12th ed 63] that associated with its own mass. Flow describes the
Hardness is resistance to abrasion and indentation. behaviour of amorphous material such as waxes.
Brinnel and Rockwell are macro hardness tests whereas
Knoop and Vickers are micro hardness tests. 24. 'A' [PHILLIPS' 11th ed 37/ 12th ed 26]
The property of wetting and adhesions are directly
14. 'A' [ PHILLIPS' 11th ed 80/ 12th ed 55] related to surface energy of adhesives. Substances
with low surface energy like Teflon are often used to
15. 'A' [PHILLIPS' 11th ed 80/ 12th ed 56] prevent the adhesion of films to a surface. Metals,
because of their high surface energy, interact
16. 'A' [PHILLIPS' 11th ed 23] vigorously with liquid adhesives.

17. 'C' [PHILLIPS' 12th ed 64] The surface energy of many restorative materials is
higher when compared to that of tooth. So there
18. 'A' [PHILLIPS' 11th ed 97 / 12th ed 63] is greater tendency for the surface and margins of
Ductility is the ability of a metal to be drawn into a restorations to accumulate debris. This property will
wire under tensile load without rupture. Ductility can explain the relatively high incidence of marginal
be measured by percent elongation and cold blend caries seen around dental restorations
test.
25. 'A' [PHILLIPS' 11th ed 46/ 12th ed 33]
Gold is the most ductile and malleable pure metal,
and silver is second. Platinum ranks third in ductility 26. 'B' [PHILLIPS' 11th ed 82/ 12th ed 54]
and copper ranks third in malleability. The elastic modulus of enamel is about three times
greater than that of dentin. But dentin is capable
19. 'B' [PHILLIPS' 11th ed 87/ 12th ed 57] of sustaining significant plastic deformation under
compressive loading before it fractures. So the enamel
20. 'B' (PHILLIPS' 11th ed 81/ 12th ed 53] is stiffer and more brittle than dentin whereas dentin
is more flexible and tougher.
21. 'A' (PHILLIPS' 11th ed 75/ 12th ed 67]
27. 'B' [PHILLIPS' 11th ed 98/ 12th ed 65]
22. 'C' [PHILLIPS' 11th ed 55/ 12th ed 40]
Dental ;lut.,e

28. 'D' [PHILLIPS' 11th ed 87/ 12th ed 57,62) "Toughness" (KAR- 2013) is the ability of a material
to absorb elastic energy and to deform plastically
29. 'D' [PHILLIPS' 11th ed 44/ 12th ed 32) before fracturing. It is measured as the t otal area
Newtonian Ideal liquid. Shear stress is under a plot of tensile stress versus tensile strain.
fluid proportional to the strain state.
Dilatant fluid Viscosity of the liquid increases "Fracture toughness" is the stress factor at the
with increase of shear stress beginning of rapid crack propagation in a solid
containing a crack of known shape and size.
Thixotropic Viscosity of the fluid decreases
fluid under pressure
36. 'D' [Manappallil 2nd ed 23)
Pseudo plastic Viscosity decreases with increase
One of the most commonly used method to define
fluid in shear stress until they reach a
and measure colour quantitatively is the "MUNSELL
constant value.
SYSTEM". In this system the parameters of color are
A characteristic of polymers that represented in 3 dime nsions. (COM ED K-08)
Visco elastic behave as elastic solids (springs)
and as viscous liquids (dashpots) 37. 'B' [PHILLIPS' 11th ed 419/ 12th ed 284)

30. 'C' [PHILLIPS' 11th ed 45/ 12th ed 33) 38. 'A' [PHILLIPS' 11th ed 103/ 12th ed 75)
Grain is a microscopic single crystal in the
31. 'C' [PHILLIPS' 11th ed 622/ 12th ed 54) microstructure of a metallic material. Nucleus is a table
'Spring back' is the amount of elastic strain that a cluster of atoms of a new phase that forms within a
metal can recover when loaded to and unloaded from pa rent phase, such as during the solidification of a
its yield strength or modulus of elasticity metal.

32. 'B' [PHILLIPS' ttth ed 84/ 12th ed 55) 39. 'D' [PHILLIPS' 11th ed 383/ 12th ed 261)
Maximum flexibility is defined as the flexural strain
that occurs when the material is stressed to its 40. 'A' [PHILLIPS' 11th ed 121/ 12th ed 79)
proportional limit. The greatest number of alloys that are useful for
dental restorations are based on solid solutions.
33. '('
Knoop hardness value is independent of the ductility 41. 'A'
of the tested material. Some KHN values are Graphite and Silicone are two non metals that conduct
Enamel 340 electricity.
Amalgam 90
42. 'K [PHILLIPS' 11th ed 37/ 12th ed 26)
Composite 55 To produce adhesion, the liquid must flow easily
Pure gold 75 over the entire surface and adhere to the solid. This
characteristic is known as wetting. If the liquid does
Porcelain 412
not wet the surface of the adherend, adhesion between
the liquid and the adherend will be negligible or non-
34. 'C' [PHILLIPS' 11th ed 30/ 12th ed 20) existent.
A space lattice can be defined as any arrangement
of atoms in space in which every atom is situated 43. 'B' [www.chemicalelements.com/element/
similarly to every other atom. Space lattices may be ag.html)
the result of primary or secondary bonds. The boiling point of silver is 2212 degree centigrade
and the melting point is 961.93 degree centigrade.
There are 14 possible lattice types but many of the
metals used in dentistry belong to the cubic system.

35. 'A' [PHILLIPS' 11th ed 74/ 12th ed 62)


, DENTAL MATERIALS

44. 'C' [Journal of Biomedical Research 2000; 53(6): Peritectic alloy: In addition to the eutectic system,
769-80) limited solid solubility of two metals can result
The two different testing techniques used to determine peritectic transformation.
tensile strength are direct tensile test and diametral Eg: Silver-tin system in amalgam
compression test. Silver-platinum in gold casting alloys

Brazilian cylinder splitting test is type of diametral 52. 'C' [PHILLIPS' 11th ed 55/ 12th ed 40]
compression test and is the widely used test to The coefficient of thermal expansion of tooth enamel
determine the ultimate tensile strength of brittle is 11.4. The coefficient of thermal expansion of GIC
materials. Type II it is 11.

4 5. 'A' [PHILLIPS' 11th ed 362 Tab 13-1) 53. ' D' [Philips 11th ed 98/ 12th ed 64]
For explanation Refer Q. No.8
46. 'A' [PHILLIPS' 11th e d 36/ 12th ed 26)
5 4. ' C' [Phillips 11th ed 87 / 12th ed 53]
4 7. 'C' [PHILLIPS' 11th ed 213/ 12th ed 163] • Elastic strain will disappear after the stress is
Ideally, the impression material should accurately removed, and the material will return to its original
record oral structures and should be easily state.
removable without distortion from the mouth. • In plastic strain, the material will not fully recover
The distortion produced during removal should be
after the removal of stress. It will occur at stresses
minimal. Viscoelasticity describes the dependence above elastic limit.
of an impression material's response to the speed of
• Elastic limit is the greatest stress to which a
removal. The simplest modes that demonstrate t he
material can be subjected such that it returns to
visoelastic behaviour is a Maxwell-Voigt model.
its original dimensions when the force is removed.
48 . 'B' • Viscoelastic means combination of elastic and
Do not allow transmission of light plastic strain, but recovery of only elastic strain
Opaque
occurs as the stress is decreased. The process of
Translucent Scatter the transmitted light
recovery occurs over time.
Transparent Does not scatter the transmitted light
55. 'B' [Phillips 11t h ed 3 7/ 12th ed 26)
49. 'B' [PHILLIPS' 11th ed 94/ 12th ed 6 2] Refer Q. No. 12

50. 'C' [PHILLIPS' 11th e d 23/ 12th ed 18] 56. 'D' [ Phillips 12th e d 20]
The fo rce which binds atoms is the covalent force. Primary bonds:
• Also called as chemical bonds and form strong
51. 'A' [PHILLIPS' 11t h ed 121/ 12th ed 79]
bonds between atoms
The simplest alloy is a solid solution, in which atoms
of two metals are Located in the same crystal structure • They are result of atoms losing, acquiring or
such as Face Centered Cubic (FCC), Body Centered sharing electrons with other atoms to achieve a
Cubic (BCC) and Hexagonal Close Packed (HCP). The stable configuration
structure appears to be entirely homogenous since Eg: Ionic bonds, Covalent bonds & Metallic bonds
on ly one phase is formed during solidification. By far
the most number of alloys used for dental restorations Secondary bonds:
are solid solutions. • These are weaker bonds
• They do not share electrons. Instead, charge
Eutectic alloy is an alloy for which the component variations among atomic groups of the molecule
metals have limited solid solubility. induce dipole forces that attract adjacent
Eg: Silver-Copper system (72% silver and 28% copper) molecules or part of a large molecule.
Eg: Va nder waals forces
Dental ;lut.,e

Vander waals forces:


- They arise from dipole attractions that is +ve is
attracted to the -ve and vice-versa.
- In case of polar molecules, the attraction and
repulsion between molecules are induced by a
permanent dipole movement from asymmetrical
electron distribution within a molecule.
Eg: Water
- In case of non-polar molecules, a temporary
(fluctuating) dipole occurs when the symmetrica l
distribution of electrons in a molecule becomes
asymmetrical temporarily.
Eg: Benzene

Note:
• The electronic structure of an atom is relatively
stable if it has 8 electrons in its outer valence
shell.
Eg: noble gases except helium.

• Helium has only 2 electrons, still it is stable.

57. 'A' [Phillips 12th ed 18)


When water boils, energy is needed
Heat of to transform liquid to vapour. This
vaporization energy is called heat of vaporization

Change from solid to liquid require


Latent heat
additional energy called latent heat
of fusion
of fusion
Solids change directly to vapour.
Sublimation
Eg: dry ice

58. 'A' [Phillips 12th ed 33)


Creep is the time dependant plastic deformation
(strain). When a metal is held at temperature near its
melting point and is subjected to stress, the resulting
strain will increase over time and is called as creep.
When the applied stress is constant it is called as
static creep and iif the applied stress is fluctuating
such as masticatory forces it is called as dynamic
creep.

59. 'A' [Phillips 1st SA ed 53)


The stress that is created when a force is applied
perpendicular to surface is compressive stress. A
compressive stress is associated with a compressive
strain.
, DENTAL MATERIALS

2. IMPRESSION MATERIALS
1. The zinc oxide-eugenol impression pastes harden by d) All the above
a) Chemical reaction b) Cold (MAN -98)
c) Heat d) Pressure 8. The term given to the phenomenon of moisture
(MAN, AIPG -94) absorption by an alginate impression is
2. 4th state of matter is a) Imbibition b) Syneresis
a) Solid b) Liquid c) Hysteresis d) Gelation
c) Gas d) Colloid (MAN -99)
(MAN, AIPG -94) 9. The base paste of the condensation poly silicone
3. All of the following statements about Type II has a low molecular weight dimethyl siloxane with
silkon impression material are true EXCEPT a terminal-group
a) They evolve hydrogen when cast if they are not a) Mercaptan group b) Hydroxyl group
fully cured c) Silane group d) Vinyl group
b) They exhibit a very low setting shrinkage (MAN -2K)
c) They have a lower tear resistance than polysulphide 10. The impression with the least dimensional change
rubbers upon disinfection is
d) They set by condensation polymerisation a) Addition poly silicone
(MAN -94, AIPG -93) b) Agar-agar
4. All of the following statements about an alginate c) Polysulphide d) Polyether
impression are true EXCEPT (MAN -01)
a) It should be rapidly displaced from the mouth 11. Best material for duplicating cast is
b) It may exhibit fluid exudates on the surface as a a) Agar-agar b) Alginate
result of im bibition of water c) Zinc oxide eugenol d) Plaster of paris
c) It will take up water and expand if kept wet (MAN -02)
d) It will shrink as a result of syneresis 12. Palladium is added to polyvinyl siloxane to
(MAN, AIPG -94) a) Act as a scavenger b) Helps as surfactant
5. All of the following can be used to slow down the c) Acts as plasticizer d) Acts as a catalyst
setting of zinc oxide eugenol impression paste (MAN -02)
EXCEPT 13. Syneresis seen in a hydrocolloid gel is
a) Adding a small amount of glycerine a) Seen as water loss b) Water absorption
b) Adding a small amount of water c) Shrinkage d) Gelation
c) Altering the amounts of the two pastes used (MAN -99)
d) Cooling mixing slab 14. Immediate pouring of impressions is most
(MAN -95) critical in
6. Type I and type II zinc oxide impression paste a) Condensation polysilicon
differ with respect to b) Addition Polysilicon
a) Theiruse c) Polyether d) A and B
b) Their hardness after setting (MAN -97)
c) Water content d) Eugenol content 15. Impression techniques are used for recording
(MAN -95) a) Oral mucosa[ tissues b) Dental hard tissues
7. Hysteresis in a hydrocolloid gel is c) Oral mucosa[ tissues and dental hard tissues
a) Moisture absorption d) None of the above
b) Temperature lag between gelation and liquefaction (AP -2K)
tern perature 16. The impression material used to record the
c) Phenomenon of conversion of gel into sol prepared areas on abutment teeth is

1) A 2) D 3) D 4) B 5) B 6) B 7) B 8) A 9) B 10) A 11) A 12) A 13) A


14) A 15) C 16) C
Dental ;lut.,e

a) High viscosity elastomeric impression material b) Physical change


b) Medium viscosity elastomeric impression material c) Evolution d) Chemical change
c) Low viscosity elastomeric impression material (AIPG -91)
d) None of the above 25. The basic constituent of reversible hydrocolloid
(AP -2K) impression material is
17. Vulcanization refers to the setting of: a) Agar b) Alginic acid
a) Reversible hydrocolloid c) Gelatin d) alginate
b) Mercaptan impression material (KAR -99)
c) Zinc phosphate cement 26. Which material undergoes hysteresis?
d) Zinc oxide-eugenol a) Irreversible hydrocolloid
(AIIMS -98) b) Reversible hydrocolloid
18. Alginate impression material is: c) Impression plaster d) Metallic oxide paste
a ) Non- elastic impression material (AIIMS -98)
b) Thermoplastic impression material 27. Before pouring an elastic impression, it is washed
c) More mucostatic than ZOE with slurry of water and stone to:
d) Less mucostatic than ZOE a) Increase gel strength
(AIPG -93) b) Prevent syn eresis
19. Impression plaster is: c) Prevent distortion
a) Less compressive than alginate d) Wash off saliva on impression
b) More compressive than alginate (AIIMS -01, AIPG -93)
c) As compressive as alginate 28. The setting time of irreversible hydro colloids can
d) More compressive than hydrocolloids be decreased by:
(AIPG -93) a) Raising the temperature of water used for mixing
20. Syringe material is b) Using excess water for mixing
a) High viscosity elastomer c) Lowering the temperature of water used for mixing
b) Medium viscosity elastomer d) None of the above
c) Medium & Low viscosity elastomer (AIIMS -01)
d) Low viscosity elastomer 29. Which material is the most difficult to remove
(AP -03) from the patients mouth?
21. Setting time of ZOE is best controlled by a) Metallic oxide paste
a) Adding a drop of eugenol b) Silicone impression material
b) Adding a drop of water c) Reversible hydrocolloid
c) Cooling the glas.s slab d) Impression plaster
d) Altering ratio of two pastes (AIIMS -98)
(PGI -02, AP -97) 30. Rubber base impressions are poured immediately
22. The impression for a diagnostic cast of a partial as they:
edentulous mouth should be taken in: a) Continue to polymerize
a) Impression wax b) Modelling compound b) Become elastic
c) Hydro-colloid d) Hydro cal c) Show imbibition and syneiresis
(AIPG -92) d) Show gelation
23. Sol- gel transformation is seen in (AIIMS -99)
a) Impression plaster b) Hydrocolloids 31. Heavy and light body impression materials are
c) Elastomer d) Impression compound used with:
(AP -99, AP -03) a) Mercaptan b) Silicone
24. Agar impression materials differ from alginate c) Reversible hydrocolloid
impression materials in that the former sets by: d) A and B
a) Mechanical action of saliva (AIIMS -99)

17} B 18} D 19} A 20} D 21} C 22} C 23} B 24} B 25} A 26} B 27} D 28} A 29} A
30} A 31} D
, DENTAL MATERIALS

32. The most mucostatic impression material is 39. Tear strength for impression material is highest for:
a) Thin mix of plaster of paris a) Condensation silicone
b) Zinc oxide eugenol impression paste b) Alginate
c) Free flowing wax d) Reversible hydrocolloids c) Poly sulfide d) Addition silicone
(AIPG -95, AIIMS MAY- 2012) (KAR -04, 02)
33. Impression compound has which of the following 40. Agar is prepared from
characteristic property? a) Bullock heart b) Chemicals
a) Low therma l conductivity c) Sea weed d) Chick cells
b) High flow property (KAR -99)
c) Degradation in presence of moist ure 41. Elastomers are
d) Remain distortion free up to 72 hrs. pouring can a) Alginat e b) Agar
be safely delayed c) Polyethers d) Imp. Compound
(AIIMS -94, PGI -01) (AP -01)
34. The best way to remove a hydrocolloid impression 42. Perforated impression tray are used for:
from the patient's mouth is: a) Alginat e b) Zinc oxide- eugenol
a) Slight rocking of the impression to disengage it c) Agar-agar d) Impression compound
from the undercut (AIPG -2K)
b) Wetting the periphery of the impression with moist 43. The American dental association {ADA)
cotton to break the peripheral seal specification number for non- aqueous elastomeric
c) Sudden jerking of the impression to prevent dental impression material is:
tearing a) 19 b) 20
d) Supporting the impression along with the tray to c) 21 d) 26
prevent disengaging of the tray alone (KAR -02)
(AIPG -01 AIIMS -98) 44. During setting of alginate impression materials
35. Agar syringe material can be used with/as a) Trisodium phosphate reacts with sodium alginate
a) Agar b) Trisodium phosphate reacts with calcium sulphate
b) Alginate impression material c) Colloidal gel changes to sol
c) Both of the above d) Primary impression d) Material in contact with soft tissues sets last
(AP -01) (AIPG -95,93)
36. Palatal secretions affect the setting of all the 45. Which of the following impression material is
impression materials except: elastic:
a) Silicone b) Agar-agar a) Impression compound
c) Impression paste d) Impression plaster b) Zinc oxide eugenol (ZOE) paste
(AIIMS -99) c) Wax d) Polyvinyl siloxane
37. Trisodium phosphate added to alginate contributes (AIPG -03)
towards: 46. Trisodium phosphate in alginate acts as:
a) Increasing the working time of alginate impression a) Retarder b) Reactor
material c) Accelerat or d) Plasticizer
b) Acts as an accelerator (AIIMS -96, PGI -2K)
c) Initiating the setting reaction 4 7. K2S04 in agar-agar is for:
d) Provides gel strength a) Accelerating the setting stage of gypsum
(AIPG -96) b) Increase strength
38. Lenolin is added in ZOE paste to: c) Give dimensional stability
a) Decrease flow b) Increase flow d) None of the above
c) Accelerate reaction (AIPG -2K, AP -98)
d) Decrease irritation due to eugenol 48. Wash or corrective impression is done by:
(AIIMS -97) a) Impression plaster b) Impression paste

32) A 33) A 34) C 35) C 36) B 37) A 38) B 39) C 40) C 41) C 42) A 43) A 44) B
45) D 46) A 47) A 48) B
Dental ;lut.,e

c) Alginate c) Zinc oxide eugenol d) Plaster of paris


d) Rubber base impression material (AP -98 KAR -01)
(AIPG -2K, KAR -94) 5 7. Material causing minimum tissue distortion is:
49. Hypersensitivity reactions like contact dermatitis a) Polysulphide b) Silicone
can be caused by c) Impression plaster d) Zinc oxide eugenol paste
a) polysulfides b) condensation si licones (AIPG -97)
c) addition silicones d) polyether 58. The advantage of ZOE impression paste:
(AP-2012) a) Has dimensional stability
50. The plasticizers used in polyether impression b) Does not adhere to tissues
material is: c) Is easy manipulation
a) Polyether polymer b) Colloidal silica d) Does not require special trays
c) Glycol ether (PG! -99)
d ) Di-vinyl poly (di methyl siloxane) 59. "Brush heap" structure is found in:
(KAR -03) a) Zinc oxide impression material
51. Impression compound base: b) Agar
a) Low thermal conductivity c) Condensation silicone
b) Crystalline structure d) Polyether
c) Formation of cross linkage with heating (PG! 01, KAR -99)
d) High fusion temperature 60. Rinsing of the impression is important to
(AIIMS -98) a) Remove excess impression material
52. In reversible hydrocolloid the property by which b) Remove saliva
the transformation from sol to gel and gel to sol is c) Hydrate the impression
a function of the: d) Accelerate the setting
a) Concentration of fillers and plasticizer (PGI -97, KAR -98)
b) Percentage composition by weight of water 61. Gelation temperature of agar is between
c) Concentration of potassium sulphate a) 20°c to 25°C b) 27°C to 33°C
d) Temperature c) 37°C to so0 c d) 55°C to 60°C
(AIIMS -01) (AP-2012, AIIMS NOV-14)
53. One of the most important advantages of truly 62. Which is best material for RPD impression
elastic impression material would be its capacity a) Impression plaster b) Irreversible hydrocolloid
for: c) Reversible hydrocolloid
a) Close adaptation to soft tissues d) None of the above
b) With drawl without permanent distortion (PG! -03)
c) Reproduction of surface details 63. Alginate impression material is similar to agar-
d) Compatibility with gypsum products agar impression material in the following respect:
(KAR -97) a) Gelation increases in both on increase in
54. Alginates are tern perature
a) Sol b) Gel b) Mixing time is increased to reduce the setting time
c) Hydrocolloid d) Colloid c) Deformation during removal of impression occurs
(AP -2K) due to distortion of gel fibers
55. Inelastic impression material is d) Both can be re-used for fresh impressions
a) Impression compound (AIIMS -2 K)
b) Alginate 64. Which of the following is not affected by saliva?
c) Agar-agar d) Polyether a) Impression plaster b) Impression paste
(AP -2K) c) Silicon impression material
56. Syneresis is associated with d) None of the above
a) Hydrocolloids b) Elastomers (AIPG -96)

49} D 50} C 51} A 52) D 53} B 54) C 55) A 56) A 57) C 58) A 59} B 60} B 61} C
62} B 63} C 64} D
, DENTAL MATERIALS

65. Hardening solutions are used with impression c) Zinc acetate d) All of these
made of (TNPSC -99)
a) Hydrocolloid b) Impression compound 73. Impression compound is €:haracterized by all the
c) Elastomer d) Zinc oxide eugenol following except
(COMEDK -04) a) Warps at room tern perature
66. Retarder in zinc oxide eugenol is b) Is a thermoset material
a) CaCl2 b) Zinc acetate c) Shows increased flow when kneaded with water
c) Alcohol d) Glycerine d) Low coefficient of thermal conductivity
(PGI -03) (AIIMS -94)
67. Elastomers except POLYETHER are 7 4. The low thermal conductivity of impression
a) Hydrophilic b) Hydrophobic compound is overcome by:
c) Water loving impression materials a) Impression is placed in mouth till it gets cold and
d) Potassium alginates sudden removal of impression.
(APPSC -99) b) By melting in boiling water at 50°c for one hour.
68. The cross linking agent of polysulphide rubber c) By heating with ethanol frame and directly placing
base impression material is in patient's mouth.
a) Aromatic sulfonate esters d) Heating in hot water and immediately quenching
b) Stan nous octate in water for 20 minutes.
c) Platinum salt catalyst (PGl-06)
d) Lead dioxide 75. Which of the following is not true about elastomeric
(APPSC -99) impression?
69. Which of the following impression material is a) Sing le mix materials have higher viscosity
rigid? b) Shear thinning is related to viscosity of non phase
a) Zinc oxide-eugenol b) Reversible hydrocolloid impression material
c) Alginate d) Polysulphide rubber c) Improper mixing of material can cause permanent
(TNPSC -99) deformation of impression
70. When making a polysulphide impression for a d) Putty - wash technique of impression reduces
caste crown, custom trays are usually preferred dimensional change of setting
over stock trays for all of the following reasons (AIPG-06)
except 76. Linear contraction of elostomeric impression with
a) Custom trays facilitate uniform contraction of passage of time is highest in:
impression material a) Addition silicone b) Polysulphide
b) Custom trays require less impression materia l c) Polyetner d) Condensation silicone
c) Stock trays may be short in the flange area
d) Impressions made in the custom trays are easier to 77. The process of changing the rubber base product
remove from the mouth or liquid polymer, to a rubber like material is
(UPPSC -01) generally known as:
71. Rapid removal of rubber impressions with a single a) Boiling b) Condensation
firm motion results in c) Vulcanization d) Chain lengthening
a) Reduced dimensional instability
b) Improved adhesion of the impression to the tray 78. Which of the following is true about Agar
c) Minimal permanent deformation hydrocolloid impression material?
d) Reduced contamination by saliva a) Liquefies between 71 - 100°C
(TNPSC -99) b) Solidfies between 50 - 70°C
72. Name the accelerator used in Zinc oxide eugenol c) Facilitates fabrication of metal dyes
paste d) Cannot register fine surface details
a) Olive oil b) Linseed oil (AIPG-06)

65) A 66) D 67) B 68) D 69) A 70) D 71) C 72) C 73) B 74) A 75) A 76) D 77) C
78) A
Dental ;lut.,e

79. Alginates are made dust free by adding: 87. Which of the following is correct regarding
a) Glycol b) Glycerol chemical setting of condensation silicone:
c) Glutamic acid d) alcohol a) Polymerization occurs with repeated elimination
(AP-05) of small molecules
80. Which one of the following increases the b) Polymerization occurs with elimination of single
strength and reduce viscosity of agar hydro colloid byproduct
impression material: c) By condensation of repeated molecules of
a) Borax b) Water monomers
c) Sulfates d) Carbonates d) Condensation does not occur at all
(PGI-05, AP- 08) (AIPG-07)
81. The role of magnesium chloride in zinc oxide 88. Which of the following is correct glass transition
eugenol impression paste: temperature of Impression compound?
a) Retarder b) Modifier a) 43.5°C b) 39°C
c) Plasticiser d) Accelerator c) 65°C d) 100°c
(COMEDK-06) (AIPG, AIIMS-07)
82. The water powder ratio of alginate is: 89. Which of the following is used as surface hardener
a) 100 ml of water to 60 gms of powder in Impression material?
b) 40 ml of water to 40 gms of powder a) 2% potassium sulfate
c) 40 ml of water to 15 gms of powder b) 0.2% potassium sulfate
d) 15 ml of water to 40 gms of powder c) 2% sodium sulfate
(COMEDK-05) d) 4% potassium sulfate
83. To make the vinyl polysiloxane hydrophilic the (AIPG, AIIMS-07)
following is added: 90. A technique of combining reversible and
a) Mineral oil b) Surfactant irreversible hydrocolloid that could bond to
c) Water d) Plasticizer irreversible hydrocolloid is known as -
(COMEDK -05) a) Injecting technique b) Laminate technique
84. Best impression material to be used for securing c) Immersion technique
impressions after crown preparation: d) Tempering technique
a) Alginate b) Agar (COMEDK-09)
c) Elastomer d) Zno paste 91. Which of the following best describes the working
(AP-05) time of impression material?
85. Dimensional stability of hydrocolloid impressions a) After the start of appearing elastic properties of
may be achieved by: impression material
a) Using less water powder ratio b) Just after the start of appearing elastic properties
b) Storing the impression under water of impression material
c) Prolonged manipulation c) The time from start of mixing till just before the
d) Using humidor start of appearing elastic properties of impression
(COMEDK -05) material
86. Dimension stability of hydrocollids may be d) Loss of lusture of impression material
achieved by (AIPG, AIIMS -07)
a) optimizing w:p 92. Minimum flow of Type-I impression compound at
b) using cold water mouth temperature is:
c) prolonged manipulation a) 4% b) 6%
d) using humidor c) 8% d) 10%
(COMED-2012) (AIPG-07)

79} A 80} A 81} D 82} C 83} B 84} C 85) D 86} D 87) A 88} B 89} A 90} B 91} C
92} B
, DENTAL MATERIALS

93. Which of the following is correct arrangement of 100. Putty wash technique:
impression material regarding linear contraction a) Light body and putty used at the same time
in ascending order? b) Putty used first and light body used second
a) Addit ion silicone < Polyether < Polysulfide < c) Light body fi rst and putty later
Condensation silicone d) None of the above
b) Addit ion silicone < Polysulfide < Polyether < (KCET-07)
Condensation silicone 101. Dimensional stability of elastomeric impression
c) Condensation silicone< Polyether < Polysulfide < material can be given in descending order as :
Addition Silicone a) Polysulphide > Polyether> Condensation silicone>
d) Condensation silicone < Polysulfide < Polyether < Addition silicone
Addition Silicone b) Polyether> Condensation silicone> Polysulphide >
(AIPG-07) Addition silicone
94. Polysulfide material is routinely not recommended c) Addition silicone > Polyether > Polysulphide >
because of: Condensation silicone
a) Unpleasant odor and taste d) Addition silicone > Condensation silicone >
b) Low accuracy Polysulphide > Polyether
c) Least permanent deformation (AIIMS-07)
d) Highest curing shrinkage 102. The monophase elastomeric material can be used
(AIPG-07) as both tray and syringe material because it has
95. Disadvantage of polysulphide as impression a) A Long setting time
material: b) It is economical
a) High cost b) Materials leaching out c) Pseudo plastic properties
c) Stretching leads to distortion d) Better recording of tissues
d) Difficulty in cast pouring (COMEDK-14)
(AP-07) 103. Which of the following component acts as an
96. Hydrocolloid material, show all except: accelerator in ZOE impression paste?
a) Hysterisis b) Imbibition a) Zinc Sulphate & Zinc chloride
c) Recrystallisation d) Syneresis b) Zinc chloride & Eugenol
(AP-07) c) Zinc Sulphate & Eugenol
97. Impression material which has a high incidence of d) Glycerin
air porosity is (AIIMS-07)
a) Alginate impression material 104. The relative hardness of elastomers is determined
b) Impression compound using
c) Zinc oxide eugenol impression paste a) Rockwell tester b) Barcol indenter
d) Polyether impression material c) Knoop pyramid d) Shore durometer
(AP-14) (COMEDK-08)
98. Impression material that DO NOT harden by 105. By product of condensation reaction between
chemical reaction: silicone base and alkyl silicate in presence of tin
a) Zinc oxide eugenol b) Impression compound octate
c) Alginate d) Plaster of paris a) Ethyl alcohol b) Glycol
(COM ED K-07) c) Acetate d) Propanolol
99. A laminate impression technique utilizes: (KCET-2012)
a) Syringe agar and chilled t ray alginate 106. Rough surface of elastomeric impression results
b) Syringe agar and t ray agar from:
c) Syringe agar and impression compound a) Inadequate mixing b) Air bubbles
d) Chi lled algi nate and impression compound c) Too rapid polymerization
(COMEDK-07)

93) A 94) A 95) C 96) C 97) A 98) B 99) A 100) B 101) C 102) C 103) A 104) D 105) A
106) D>C
Dental ;lut.,e

d) Incomplete polymerization caused by premature c) Just before the start of appearing elastic properties
removal from mouth of impression material
(AIIMS-07, COMEDK- 2013) d) Loss of luster of impression material
107. Which of the following impression material (AIPG-10)
requires a hardener? 115. Rough and Irregular surface produced on the
a) Agar b) Alginate impression is because of?
c) Elastomers d) Impression compound a) Improper application of pressure during impression
(AIIMS-07) making
108. Most rigid elastomer is b) Air incorporated during mixing
a) Polyether b) Polysulp hide c) Too rapid polymerization
c) Addition silicone d) Condensation silicone d) Presence of moisture in impression area
(BHU-07) (AIPG-10)
109. Most mucostatic among the following 116. Final product in alginate is?
a) Impression compound a) Sodium alginate b) Potassium alginate
b) Alginate c) Trisodium phosphate
c) ZOE paste d) Calcium alginate
d) Elastomer (AP-2011)
(MCET-10) 117. Which of the following impression materials is
110. Most ideally elastic among the following easy to pour and difficult to remove the stone cast
a) Polyether b) Addition silicone from the impressions?
c) Polysulphide d) Condensation silicone a) Addition polysilicone
(MCET-10) b) Condensation polysilicone
111. The ran king of tear strength from the lowest to c) Polyether d) Polysulfide
highest of all impression materials generally is as (KCET-2011)
follows 118. Dustless alginate is produced by
a) Hydrocolloids, silicones, polyether, polysulfide a) Reducing the diatomaceous earth
b) Silicones, hydrocolloids, polyether, polysulfide b) Adding heavy metal salts
c) Polysulfide, hydrncolloids, silicones, polyether c) Coating with dihydric alcohol
d) Hydrocolloids, polyether, silicones, polysulfide d) Altering the matrix
(KCET-10) (AIIMS-09)
112. Which one of the following impression materials is 119. "Two in one stage" impression materials include
elastic, sets by a chemical reaction and is catalyzed a) Zinc-Oxide eugenol impression paste
by chloroplatinic acid- b) Hydrocolloid impression material
a) Condensation silicone c) Elastomeric impression material
b) Polyether d) All of the above
c) Polysulfide d) Poly vinyl siloxane (AP-2013)
(KCET-09) 120. Alginate fillers derived from
113. The impression materials with lowest viscosity is a) Calcium sulphate b) Sodium phosphate
a) Plaster of paris b) Agar-agar c) Potassium alginate d) Diatomaceous earth
c) Zinc oxide eugenol d) Elastomers (AIIMS-13)
(AP-10) 121. In case of addition silicones what should be done
114. Which of the following best describes the working for better cast
time of elastomeric impression material? a) Apply ketone over tray
a) After the start of appearing elastic properties of b) Apply chloroform over tray
impression material c) Add flavouring agent to prevent bad odour
b) Just after the start of appearing elastic properties d) Delay pouring of cast
of impression material (AIIMS NOV-13)

107) A 108) A 109) C 110) B 111) A 112) D 113) A 114) C 115) C 116) D 117) C 118) C 119) C
120) D 121) D
, DENTAL MATERIALS

122. A dentist can best control the setting time of 129. Condensation reaction occurs in
Alginate impression material without altering its a) Agar b) Alginate
properties by c) Polysulfide d) ZOE
a) Using perforated tray (AIIMS MAY-14)
b) Altering water: powder ratio 130. Putty consistency addition silicone is mixed by
c) Reducing the rate of mixing a) Kneading with fingers while wearing gloves
d) Altering the temperature of water b) Kneading with fingers without gloves
(MCET-14) c) On a mixing pad
123. The elastomer having the longest curing time is d) With an auto mixer and dispenser
a) Polysulfide b) Poly ether (COMEDK-15)
c) Addition silicone d) Condensation silicone
(AP-14)
124. An example of rigid reversible impression material
is
a) Agar-agar b) Impression compound
c) Alginate d) Impression paste
(COMEDK-14)
125. Permanent deformation following strain in
compression in increasing order for elastomers
a) Addition silicone, condensation silicone, polyether,
poly sulphide
b) Poly sulphide, poly ether, Addition silicone,
condensation silicone
c) Poly sulphide, poly ether, condensation silicone,
Addition silicone
d) Polyether, poly sulphide, addition silicone,
condensation silicone
(COMEDK-15)
126. Which of the following nonaqueous elastomeric
impression materials has the least shelf life
a) Poly ether b) Addition silicone
c) Condensation silicone
d) Polysulphide
(AIIMS NOV-14)
127. Regarding type I impression compound not true is
a) Incomplete krneading leads to incorporation of
water that increases flow of impression compound
b) Even when needed in small quantity it is not
heated over flame
c) Is softened in water bath
d) Also called as type I compound according to ADA

128. Which of the following component of hydrocolloid


can lead to fixed drug eruptions?
a) CaS04 b) Phenolphthalein
c) Na fluorititanate d) Magnesium oxide
(PGI JUNE-13)

122) D 123) A 124) B 125) A 126) A 127) A 128) B 129) C 130) B


Dental ;lut.,e

2. IMPRESSION MATERIALS - ANSWERS


1. 'A'[PHILLIPS' 11th ed 209/ 12th ed 179] 4. 'B' [PHILLIPS' 11th ed 246/ 12th ed 174]
The chemical reaction consists of hydrolysis of Fluid exuded from the impression seen as droplets
zinc oxide, which reacts with eugenol to form zinc on surface are called syneresis whereas absorption
eugenolate, which is a chelate. Water is needed to of water by the impression when placed in water is
initiate the reaction and it is also a bye-product of the called Imbibition.
reaction. This reaction is also known as autocatalytic
reaction. Zinc acetate, high temperature and humid 5. 'B' [PHILLIPS' 11th ed 253/ 12th ed 179]
environment accelerate the setting reaction. Adding a small amount of water accelerates the setting
reaction of zinc oxide eugenol impression paste and
2. 'D' [PHILLIPS' 11th ed 231/ 12th ed 168) thereby decreases the setting time.
SolL1te which is completely soluble
Solution 6. 'B' [PHILLIPS' 11th ed 253/ 12th ed 178]
and not visible in solvent
Type I ZOE paste (Hard) sets in 10 minutes and
Larger, visible, non-soluble particles type II ZOE paste (soft) sets in 15 minutes.
Suspension (such as sand in water) in solvent. It
exists as 2-phase system 7. 'B' [PHILLIPS' 11th ed 235/ 12th ed 170]
Liquid droplets which are suspended Hysteresis is exhibited by agar, which is a reversible
Emulsion
in water hydrocolloid impression material.
Lies between small molecules in
Colloids 8. 'A' [PHILLIPS' 11th ed 246/ 12th ed 174]
solution and very large particles in
( or) sols
suspension. It also exists in 2 phases.
9. 'B' [PHILLIPS' 11th ed 213/ 12th ed 154]
Condensation silicon polymers terminate in
Colloidal substances may be combinations of any
hydroxyterminated polydimethyl siloxane groups.
states of matter.
Addition silicone (Poly vinyl siloxane) polymers
terminate in vinyl groups.
Liquids or solids in air Aerosols
Gases/ liquids/ solids in liquid Lysosols 10. 'A' [PHILLIPS' 11th ed 222-24 Fig 9-10/ 12th ed
165 Fig 8-14]
Gases in solids Foams
Addition silicones have very little residual
Liquids in solids Solid emulsions polymerization, making them t he most dimensionally
Solids in solid Solid suspension stable of all the dental materials. They are also highly
biocompatible impression materials.
3. 'D' [PHILLIPS' 11th ed 214/ 12th ed 154 ]
11. 'A' [PHILLIPS' 11th ed 24 5]
Addition silicones are the type II silicone impression
Agar and alginate both are useful for duplicating
materials, which undergo addition type of
casts or models. But the most popular is agar because
polymerization reaction terminating in vinyl groups
it can be used many times.
and cross-linked with hydride groups.

12. 'A' [PHILLIPS' 11th ed 215/ 12th ed 154]


The advantages of addition silicone are:
Impurities evolve hydrogen gas from addition silicone,
• Dimensional stability which produces pint point voids in the stone casts, if
• Adequate tear strength (but less than polysulphide) poured immediately after removing from mouth .
• Agreeable odor and color
• Multiple casts can be made from one impression. This can be prevented by adding platinum or
palladium to act as scavenger or by waiting an hour
• No volatile byproduct
before pouring up the impression.
, DENTAL MATERIALS

13. 'A' [PHILLIPS' 11th ed 234/ 12th ed 174) 22. 'C' [PHILLIPS' 11th ed 208/ 12th ed 176)
The gel may loose water by evaporation or by Impression compound and impression plaster are
exudation of fluid onto the surface by syneresis. Both useful for making impressions of edentulous ridges.
of them lead to shrinkage of gel. So casts should be Hydrocolloids are useful for partial edentulous mouth
poured immediately for hydrocolloid materials or else impressions.
wrapped in moist towel to create 100% humidity.
23. ' B' [PHILLIPS' 11th ed 234/ 12th ed 170)
14. 'A' [ PHILLIPS' 11th ed 224/ 12th ed 154]
Ethyl alcohol is the bye product of condensation 24. ' B' [PHILLIPS' 11th ed 208/ 12th ed 153)
setting reaction. Contraction occurs if the bye product Setting reaction of agar ano! impression compound is
evaporates in a set silicone rubber. So casts are poured a function of temperature and is a physical change.
immediately after taking impression from mouth. These are called as thermoplastic material.

In contrast addition silicones should be poured an 25. 'A' [PHILLIPS' 11th ed 235/ 12th ed 170)
hour late to improve the cast surface. The polyether The basic constituent of agar impression material
impression can be poured immediately or after several is agar which is a polysaccharide extracted from
days and the resulting casts will have same accuracy. seaweed. It is a sulfuric ester of galactose. But the
principal ingredient by weight is water (80%).
15. 'C' [PHILLIPS' 11th ed 207/ 12th ed 152)
26. ' B' [PHILLIPS' 11th ed 235/ 12th ed 170)
16. 'C' [PHILLIPS' 12th ed 160)
27. ' D' [PHILLIPS' 11th ed 248/ 12th ed 161)
17. ' B' [PHILLIPS' 11th ed 212/ 12th ed 153) Saliva and blood retards the setting reaction of
Vulcanization is a cross linking process that involves gypsum so these must be removed by washing the
sulphide mercaptan groups. Polysulphide impression impression before pouring it.
materials are also called as vulcanizing materials
or mercaptan materials. The mercaptan groups give 28. 'A' [PHILLIPS' 11th ed 242/ 12th ed 174)
polysulphides its characteristic odor. The higher the temperature of mixing water, the faster
is the gelation. The clinician can safely influence
18. 'D' [PH IL LIPS' 11th ed 208/ 12th ed 153] the gelation time by controlling the temperature of
"ZOE impression paste" and "plaster of paris" are mixing water. But the best way to control the gelation
called mucostatic impression materials because they time is by adding retarders like trisodium phosphate
do not compress the tissue during seating of the during manufacturing.
impression. They are ideal for making impressions of
edentulous structures. 29. 'A'

19. 'A' [PHILLIPS' 11th ed 208/ 12th ed 153) 30. 'A' [PHILLIPS' 11th ed 227 / 12th ed 161]
Impression plaster is rigid impression material. It The rubber base impression materials or polysulphides
records tissues in anatomic form (does not compress), have residual polymerization i.e., they continue
so called as mucostatic impression material. Alginates to polymerize for some time even after setting and
and waxes are other examples of mucostatic impression the bye-products of polymerization may be lost if
materials. impressions are not poured immediately. Both of them
result in shrinkage of the impression.
Impression compound and elastomers are
mucoco mp ressive. 31. 'D' [PHILLIPS' 11th ed 210/ 12th ed 153]

20. ' D' [PHILLIPS' 11th ed 221/ 12th ed 157, 158) 32. 'A' [PHILLIPS' 11th ed 208/ 12th ed 117)

21. 'C' [PHILLIPS' 11th ed 253/ 12th ed 179] 33. 'A' [PHILLIPS' 11th ed 251/ 12th ed 178]
Dental ;lut.,e

34. 'C' [PHILLIPS' 11th ed 237/ 12th ed 174] 42. 'A' [PHILLIPS' 11th ed 243/ 12th ed 173]
Hydrocolloids and e lastomers should be removed with By using perforated trays, the impression adheres
a sudden quick motion because these are Viscoelastic to the tray so that the impression can be withdrawn
materials and a quick force causes less permanent from around the teeth . When rim lock tray is selected,
deformation. Weaving and teasing forces cause a t ray adhesive should be used.
distortion or tearing.
43. 'A' [ PHILLIPS' 11th ed 210/ 12th ed 165]
35. 'C' [PHILLIPS' 11th ed 245/ 12th ed 176]
Laminate technique combines agar-alginate in 44. 'B' [PHILLIPS' 11th ed 241/ 12th ed 174]
which chilled alginate replaces the tray agar. Alginat e
gels by chemical reaction and agar gels by means of 4 5. 'D' [PHILLIPS' 11th ed 208/ 12th ed 153]
contact with cold a lginate.
46. 'A' (PHILLIPS' 11th ed 240/ 12th e d 173]
Agar is also used in triple tray technique i.e, one
impression records the maxillary, mandibular arches 47. 'A' (PHILLIPS' 11th ed 235/ 12th ed 172]
along with their occlusion. Potassium sulphate in agar accelerates the setting
reaction of gypsum and helps to counteract the
36. ' B' [PHILLIPS' 11th ed 237 / 12th ed 170] retarding effect of water and borax of agar on gypsum .
Agar-agar is the true hydrophilic im pression material.
48. 'B' [PHILLIPS' 10th ed 182/ 12th ed 160]
37. 'A' [PHILLIPS' 11th ed 241/ 12th ed 172]
Trisodium phosphate (Retarder) reacts with Ca 49. 'D' [Philips 11th ed 229/ 12th ed 167]
So4 preferably than alginate and thus prolongs the Contact dermatitis and hypersensitivity potential
working time. exists with polyether impression material. Lowest
cell cytotoxicity is with polysulfide and highest with
38. ' B' [PHILLIPS' 11th ed 252/ 12th ed 178] polyether impression material.
Base paste
50. 'C' [PHILLIPS' 11th ed 216/ 12th ed 155]
ZnO Main ingredient (87%)
Base paste of poly,e ther
Decreases the irrit ant Polyether polymer Main ingredient
Vegetable oil
action of eugenol
Colloida l silica Filler
Accelerator paste Glycol ether Plasticizer
Clove oil reduces Accelerator paste
Oil of cloves / Eugenol
burning sensation Alkyl aromatic sultanate Main ingredient
Accelerates and gives Colloida l silica Filler
Rosin
smoother product Glycol ether Plasticizer
Filler Silica type
51. 'A' [PHILLIPS' 11th ed 251/ 12th ed 178]
Lanolin, Resinous balsam increases flow
Zinc acetate or CaCl2 Accelerator 52. 'D' [PHILLIPS' 11th ed 234/ 12th ed 170]

39. 'C' [PHILLIPS' 11th ed 228/ 12th ed 170] 53. 'B' [ PHILLIPS' 12th ed 153]

40. 'C' [PHILLIPS' 11th ed 235/ 12th ed 170] 54. 'C' [PHILLIPS' 11th ed 234/ 12th ed 171]

4 1. 'C' [PHILLIPS' 11th ed 210/ 12th ed 153] 55. 'A' (PHILLIPS' 11th ed 209/ 12th e d 153]
Four types of elast omers are - Polysulphide, addition
silicone, condensation silicone and polyether. 56. 'A' [PHILLIPS' 11th ed 234/ 12th ed 174]
, DENTAL MATERIALS

57. 'C' [PHILLIPS' 11th ed 208/ 12th ed 177] 69. 'A' [PHILLIPS' 11th ed 216/ 12th ed 177]

58. 'A' [PHILLIPS' 11th ed 253/ 12th ed 179] 70. ' D' [PHILLIPS' 11th ed 219/ 12th ed 156]
The main advantage of ZOE paste is its dimensional Custom trays are used with polysulphide impression
stability. ZOE paste is used for making fina l or wash materials. They reduce t he impression material in
impression of edentulous ridges, bite registration, the tray thereby reducing polymerization shrinkage
post surgical pack, temporary reliner, root canal fi ller, and also facilitate uniform contraction (increased
temporary filling material and leuting agent, etc. ZOE accuracy). For silicones, putty-wash technique is
paste is the most commonly used (BH U-07) wash used. So, there is no need of custom tray.
impression material for edentulous jaw.
71. ' C' [PHILLIPS' 11th ed 212 / 12th ed 161]
59. ' B' [PHILLIPS' 11th ed 234/ 12th ed 170]
72. ' C' [PHILLIPS' 11th ed 252 / 12th ed 179)
60. ' B' [PHILLIPS' 11th ed 248/ 12th ed 161) Zinc acetate, calcium chloride, alcohols are the
accelerators used in ZOE paste.
61. ' C' [Philips 11th ed 235/ 12th ed 170)
Agar liquefaction temperature is 70-100°( and 73. ' B' [PHILLIPS' 11th ed 209/ 12th ed 177)
gelation temperature is 37-50°( Impression compound agar and waxes are thermoplastic
materials. The materials, which set by chemical
62. 'B' [PHILLIPS' 11th ed 234/ 12th ed 176) reaction, are called thermoset materials. Heat cure
Alginate is most widely used and most versatile for acrylics are examples of thermoset materials.
RPD impressions
74. 'A' [Manappallil 2"d ed 35,36/ PHILLIPS' 12th ed
63. 'C' [PHILLIPS' 11th ed 239/ 12th ed 174, 176) 178)
Impression compound has low thermal conductivity.
64. 'D' To overcome this the fo Llowing is done:
• During softening of material, the outside will
65. 'A' [PHILLIPS' 11th ed 24 7/ 12th ed 175) soften first and inside last. So to ensure uniform
Gypsum products are not compatible with the softening, the material should be kept immersed
hydro colloid impression materials. The casts will for a long time in the water bath.
become soft. This can be prevented by incorporating
• The layer adjacent to the mouth-tissues will
hardening solutions in the impression material during
remain soft. Therefore, it is important to cool
manufacturing or by immersing the impression in an
the compound throughly before removing the
accelerator solution for the setting of gypsum. The
impression.
hardening solutions are 2% potassium sulphate for
agar and titanium fluoride for alginate materials. 75. 'A' [PHILLIPS' 11th ed 220/ 12th ed 159)
Medium viscosities of elastomers are often used for
66. 'D'
single mix technique.

67. 'B' [PHILLIPS' 12th ed 170] Two separate mixes (heavy and light
Multiple
body) are made. Heavy body is placed
mix
68. ' D' [PHILLIPS' 11th ed 213/ 12th ed 153] in the tray and light body is loaded
technique
Lead dioxide is the cross linking agent in polysulphide in a syringe and impression is made
impression material. This imparts brown color to the Single mix Only one mix is made (medium
material. It is cmss-linked to -SH groups of the technique/ body). Part of it is loaded in tray and
polysulphide polymer. Monophase part of it is loaded into syringe and
technique an impression is made.
Which one of the following is an ingredient in the
composition of polysulphide impression material?
Ans: Lead dioxide (KERALA-2015) 76. ' D' [PHILLIPS' 11th ed 224/ 12th ed 165]
Dental ;lut.,e

77. 'C' At below Tg, the glass structure loses its fluid
characteristics and has significant resistance to shear
78. 'A' [PHILLIPS' 11th ed 234/ 12th ed 170] deformation.

79. 'A' [PHILLIPS' 11th ed 240/ 12th ed 172] Synthetic dental resin is examples of glassy struct ures.
Dustless alginates incorporate polyethylene or
polypropelene glycol on the alginate powder to 89. 'A' [PHILLIPS' 10th ed Pg 134]
agglomerate the particles. Due to this the powders no
longer have a tendency to release fine particles that 90. ' B' [PHILLIPS' 11th ed 245/ 12th ed 176]
cause a health hazard.
91. 'C' [PHILLIPS' 11th ed 223/ 12th 161]
80. 'A' [Manappallil 2"d ed 49]
92. 'B'
81. 'D' [PHILLIPS' 11thed 252/ 12th ed 179]
93. 'A' [PHILLIPS' 11th ed 224/ 12th ed 165]
82. '('
94. 'A' [PHILLIPS' 10th ed 151/ 12th ed 170]
83. 'B' [PHILLIPS' 11th ed 215/ 12th ed 167]
One of the disadvantages of addition silicones is their 95. 'C' [PHILLIPS' 11th ed 227/ 12th ed 161]
inherent hydrophobic nature. A non ionic surfactant is
added to the paste to render it hydrophilic. 96. 'C' [PHILLIPS' 11th ed 234/ 12th ed 174]

Surfactants are added to addition silicones to 9 7• 'A' [ Phillips 12th ed 17 6]


a) Decrease contact angle (COMEDK-15)
b) Increase contact angle 98. 'B' [PHILLIPS' 11th ed 250/ 12th ed 177]
c) Decrease shear stress d) Deere ase setting time
99. 'A' [PHILLIPS' 11th ed 245/ 12th ed 176]
84. 'C' [PHILLIPS' 11th ed 208/ 12th ed 160] Laminate or Agar-Alginate method:
Tray is fi lled with chilled a lginate that bonds with
85. 'D' [PHILLIPS' 11th ed 246/ 12th ed 174] the agar expressed from a syringe. Alginate gels by
2% potassium sulfate or 100% relative humidity are chemical reaction and Agar gels by means of contact
used to reduce dimensional changes of hydrocolloid with the cool alginate.
impressions.
100. 'B' [PHILLIPS' 11th ed 221/ 12th ed 160]
86. 'D' [Philips 11th e d 246/ 12th ed 17 4]
100% relative humidity or 2% potassium sulfate are 101. 'C' [PHILLIPS' 11th ed 224 Fig. 9 - 10/ 12th ed 169
the suggested storage media to reduce the dimensional Tab. 8-6]
change of hydrocolloids particularly agar impressions.
Material Dimensional change
87. 'A' [PHILLIPS' 11th ed 151, 152/ 12th ed 154]
Condensation silicone 0.6

88. ' B' [PHILLIPS' 11th ed 288] Polyether 0.23


Glass transition temperature: (Tg) Polysulphide 0.4
"It is the temperature at which there is an abrupt
Addition silicone 0.17
increase in the thermal expansion coefficient
indicating increased molecular mobility. It is
characteristic of the particular glassy st ructure." It is 102. 'C' [Phillips 12th ed 159]
approximately 35 - 39°( (AIIMS- 07) .
, DENTAL MATERIALS

103. 'A' [PHILLIPS' 10th ed 181/ 12th ed 179] the shore A or shore D scale is the preferred method
Zinc acetate, calcium chloride, primary alcohols, zinc for rubbers or elastomers and is also common ly used
chloride and water are used as accelerators. Inert oils for softer plastics. It is commonly used to assess and
and waxes like olive oil, mineral oil and petrolatum compare the stress-strain behaviour of elastomers.
jelly are the retarders. Shore A is used to test soft elastomers and shore D is
used to test t he hardness of ha rd elastomers. Moh's
104. 'D' [Journal of Rubber Chemistry and Technology; hardness is defined by how well a substance will
2003, Vol.76, pg 419 - 435] resist scratching by other substance. Barcoal hardness
Hardness measurement methods: method is used to determine the hardness of bot h
a) Macrohardness tests (Rockwell and Brinnel) reinforced and non-reinforced rigid plastics.
b) Microhardness tests (Vickers and Knoop)
c) Shore 10 5. 'A' [Philips 11th ed 213/ 12th ed 154]
d) Moh's hardness Ethyl alcohol is the byproduct of condensation
e) Barcoal hardness reaction between silicone base and alkyl si licate in
the presence of tin octoate. The evaporation of ethyl
The shore scleroscope measures hardness in terms of alcohol results in much of the contraction of the set
elasticity of the material. Shore hardness, using either silicone impression.

106. 'D>C' [PHILLIPS' 11th ed 230 Tab 9 - 5]


Common failures that occur with use of non-aqueous elastomeric impression materials
Type of failure Causes
• Incomplete polymerization caused by premature removal from mouth, improper ratio or
Rough or mixing of components or presence of oil or other inorganic material on t eeth.
uneven surface • For addition silicone, agents that contaminate the material and inhibit polymerization
on impression causes rough surface
• Too rapid polymerization from high humidity or temperature
• Excessively high accelerator/ base ratio with condensation silicone.
Bubbles • Air incorporated during mixing
• Too rapid polymerization, preventing flow (In 12th ed Philips' t his point is deleted )
Irregularly
Moisture or debris on surface of teeth
shaped voids
• Inadequate cleaning of impression
Rough or chalky • Excess water left on the surface of impression
stone cast • Excess wetting agent left on impression
• Premature removal of cast
• Improper manipulation of stone
• Failure to delay pour of addition silicone at least 20 mins.
• Premature removal from mouth
• Improper remova l from mouth
• Excess bulk of material
Distortion • Movement of tray during polymerization
• Lack of adhesion of rubber to the tray caused by not applying enough coats of adhesive,
filling tray with material too soon after applying adhesive, or using wrong adhesive
• Lack of mechanical retention for those materials where adhesive is ineffective.
• Development of elastic properties in the material before tray is seated.
• Insufficient relief for the reline material.
• Continued pressure against impression material that has developed elastic properties.

107. 'A' [PHILLIPS' 11th ed 235/ 12th ed 167]


Potassium sulphate is the gypsum hardener in Agar impression material.
Dental ;lut.,e

108. 'A' [Refer Q.No. 58 / PHILLIPS' 11th ed 231, 232 Tab 9 - 6/ 12th ed 164, 169 Tab 8-6]

COMPARATIVE PROPERTIES OF ELASTOMERIC IMPRESSION MATERIALS

Property Polysulfide Condensation silicone Addition silicone polyether

Working time (min) 4-7 2.5-4 2-4 3

Setting time (min) 7-10 6-8 4-6.5 6

Tear strength (N/m) 2500-7000 2300-2600 1500-4300 1800-4800

Percent contraction (at 24hr) 0.40-0.45 0.38-0.60 0.14-0.17 0.19-0.24

Contact angle between set


82 98 98 49
material and water
Hydrogen gas evolution (Y/N) N N y N

Multiple casts N N N N

Stiffness ( 1 indicates greater


3 2 2 1
stiffnes)

Distortion on removal (value 1


1 2 4 3
indicates greater distortion)

109. 'C' [PHILLIPS' 11th ed 208/ 12th ed 153] 114. 'C' [PHILLIPS' 10th ed 142/ 12th ed 161]
"ZOE impression paste" and "plaster of paris" are The measurement of working time begins with the
called mucostatic impression materials because they start of mixing and ends just before the impression
do not compress the tissue during seating of the material has developed elastic properties.
impression. They are ideal for for making impressions Setting time: Time elapsing from the beginning of
of edentulous structures. mixing until the curing has advanced sufficiently that
the impression can be removed from the mouth with
110. 'B' [PHILLIPS' 11th ed 232 Tab 9-7 / 12th ed 169 negligible distortion.
Tab 8-6]
115. 'C' [PHILLIPS' 10th ed 174/ 12th ed 169 Tab 8-5]
111. 'A' [PHILLIPS' 11th ed 232, 244/ 12th ed 169 Tab Causes of rough or uneven surface of elastomeric
8-6] impression material are:
• Incomplete polymerization caused by premature
112. 'D' [MANAPPALLIL 3rd ed 198] removal from mouth.
Impression Material Catalyst • Improper ratio of mixing components.
Condensation Silicone Stan nous octoate • Too rapid polymerization caused by high humidity
& high temperature.
Addition Silicone Chloroplatinic acid
Polysulphide Lead dioxide 116. 'D' [Phillips' 11th ed 241/ 12th ed 172]
Setting reaction:
In condensation silicone, the condensation Potassium alginate + Potassium sulphate+
--+
reaction between - OH groups & tetraethyl Calcium sulphate Calcium alginate
orthosilicate takes place in presence of
(MHCET-15) Option 'C' trisodium phosphate is added as a retarder.
a) Platinum salt b) Stannous octate It reacts with calcium sulphate preferably than
c) Trisodium phosphate d) Ethyl alcohol alginate and thus prolongs the working time.

113. 'A' [PHILLIPS' 11th ed 273 Tab. 10-5]


, DENTAL MATERIALS

117. 'C' [Phillips 11th ed 222/ 12th ed 161] 123. 'A' [Check Synopsis]
The hydrophobicity of the non -aqueous elastomeric
impression materials make it difficult to wet the 124. 'B' [Phillips 12th ed 153]
surface by gypsum products and may result in bubbles. Setting Rigid I
A dilute solution of soap is an effective surfactant. Elastic
mechanism inelastic
Chemical • Plaster • Alginate
Pouring the stone cast in a polyether impression is reaction of paris
much easier when compared to other non -aqueous • Elastomers
elastomers. However, the stiffness of polyether makes • ZOE
it difficult to remove the cast from the impression and Thermally induced Impression Agar
may result in fracture of stone cast during removal. physical reaction compound
(Reversible)
118. 'C' [Craig 11th ed 336]
Alginate powder contains 10-15% of siliceous dust 125. 'A' [Refer Synopsis]
particles. Coating this powder with glycol results in a Increasing order of permanent deformation in
dustless alignate. elastomeric impression materials:
Addition Silicone < Condensation Silicone < Polyether
< Polysulphide
119. 'C' [Mc Cabe dental materials 148]
Elastomers particularly addition silicones (Vinyl
polysiloxane) can be used in two stage or one stage 126. 'A' [Refer Synopsis]
technique.
127. 'A' [Craig 11th ed 372]
• 2-stage impression: The impression with putty
(heavy body) is taken first and then light body Impression compound is classified as:
(wash) impression is taken separately with the Type 1 • Low fusing (green stick compound)
same tray. • Medium fusing (impression compound)
• 1-stage impression: The light body is syringed Type 2 High fusing (Tray compound)
over the tooth preparation and heavy body is
loaded in the impression tray. Then the impression Manipulation of medium and high fusing compound
is taken as normal. This saves time. is by immersing in water bath between temperatures
60-70°(. Then it is kneaded with fingers like a dough
to obtain uniform plasticity. This wet kneading of one
minute improves the flow by double. So option 'A' is
wrong. Manipulation of low fusing (green stick) is by
softening it over the flame.
Potassium alginate Soluble alginate
Cas04 Reactor As the question is about type-I impression compound,
• Zinc oxide Filler particles to option 'N. is false and is the correct answer.
• Diatomaceous earth increase strength and
stiffness of the gel 128. 'B' [Check Explanation Below]
Sodium phosphate Retarder Chromalginate:
These are color changing alginates. It is due to
121. 'D' [Explanation of Q. No. 12] incorporation of pH indicators that change color
during gelation process. Various pH indicators
122. 'D' [Explanation Q.No.28] such as cresol red, phenolphthalein , thymol blue,
The temperature of the mixing water should be naphtholphthalein can be used.
carefully controlled with a standard temperature
usua lly 20 degrees to 23 degrees so that a constant Fixed drug eruptions are the localized sharply
and reliable setting time can be obtained. circumscribed cutaneous drug reaction that recurs
in exactly same location on repeated exposure. The
Dental ;lut.,e

agents which commonly produce them are barbiturates,


paracetamol, phenolphthalein, sulphonamides,
tetracycline and pyrnzonal derivatives.

129. 'C' [Phillips 1st SA ed 159)


Condensation polymerization is the reaction between
two usually dissimilar molecules to form a larger
molecule with the elimination of a smaller molecule
(often water).

In Polysulphide the polymerization reaction yields


water as a by-product. So it is an example of
condensation reaction. The Loss of this small water
molecule has a significant effect on the dimensional
stability.

The other example is condensation silicone, where


ethyl alcohol is a by-product and its evaporation
leads to much of the contraction.

130. 'B' [Phillips 1st SA ed 160)


Sulphur contamination from natural latex gloves
inhibits the setting of addition silicone. The
contamination is so extensive that touching the
tooth with latex gloves before seating the impression
can inhibit the setting of the critical surface next to
the tooth.
, DENTAL MATERIALS

3. GYPSUM PRODUCTS
1. Model plaster (white) used to cast study models 9. Plaster of paris is mixed in
before mixing with water, is largely composed of a) Plastic bowl b) Rubber bowl
a) Cao b) CaC0 3 c) Glass bowl d) Metal bowl
c) (CaSOJ 2 • '2H 2 0
1 d) CaS0 4 -2H 2 0 (AP -01)
(AIIMS -93, MAN -94) 10. Impression plaster containing potato starch is called:
2. The product, which is obtained by calcining a) Plaster of paris b) Soluble plaster
gypsum under steam pressure at 120-130°( or c) Anti - expansion solution
by dehydrating gypsum in the presence of sodium d) Die stones
succinate, is (KAR -02)
a) Alpha - hemihydrates
b) Beta - hemihydrates a) Impression of single tooth
c) Calcium sulphate dihydrate b) Impression of whole teeth
d) Orthorhombic anhydrate c) Replica of single tooth
(MAN -98) d) Replica of whole teeth
3. Water powder ratio of dental stone and plaster is (AP -2K)
respectively 12. Balanced stone is dental stone
a) 0.28 and 0.6 b) 0.6 and 0. 28 a) Which undergoes rapid expansion
c) 0.6 and 3.2 d) 0.28 and 0.98 b) In which accelerators or retarders have been added
(MAN -98) according to need
4. Type III Dental gypsum is c) In which amount of water of hydration is controlled
a) Class II stone b) Densite d) In which the crystals are all of uniform size
c) Class I stone or hydrocal (AIPG -04)
d) Model or lab piaster 13. The main ingredient in dental plaster is:
(MAN -98) a) Calcium sulphate hemihydrate
5. Green strength with reference to plaster means b) Calcium phosphate
a) Dry strength b) Compressive strength c) Calcium anhydrate
c) Strength of dental stone due to green colour d) Calcium sulphate dihydrate
d) The wet strength (AIPG -92)
(MAN -98) 14. The strength of the gypsum specimen when the
6. Beta hemihydrate particles absorb water in excess of that required for the hydrate of
a) more water b) less water the hemihydrate is left in the specimen is called?
c) no water d) none of the above a) Dry strength b) Green strength
(BHU-2012) c) Water strength d) Compressive strength
7. Water of reaction needed to react completely with (KAR -03)
100 g of calcium sulphate hemihydrates to convert 15. Accelerators and retarders are used with gypsum
it to calcium sulphate dihydrate products mainly to control
a) 18.6ml b) 22.2ml a) Setting time b) Setting expansion
c) 30.4ml d) 45.3ml c) Hardness of the set product
(MAN -2K, 01) d) None of the above
8. Powders of dental plaster and dental stone differ (AP -97, 99)
mainly in: 16. Gypsum is used for?
a) Solubility b) Shelf life a) Impressions b) Casts
c) Chemical formula d) Particle porosity c) Die d) Casts and Dies
(AIPG -90; PGI -01) (AP -01)

1) C 2) A 3) A 4) C 5) D 6) A 7) A 8) D 9) B 10) B 11) C 12) B 13) A


14) B 15) A 16) D
Dental ;lut.,e

17. The hygroscopic technique is associated with: c) They are more irregular in shape and porous
a) Investment b) Hydrocolloids d) They are more dense and prismatic in shape
c) Amalgam d) Silicate (KAR -98)
(AIPG -91, AP- 2013) 26. Setting of POP is result of
18. The main difference between dental stone and a) Difference in solubility of CaSo 4 1/2 H20 + CaS0 4 •
dental plaster is: 2H 20
a) Chemical composition b) Reaction between CaS0 4 2H 20 and water
b) Selflife c) Nucleation and growth of CaS0 4 112 H2 0 crystals
c) Shape and size of particles d) Reaction between hemihydrate and dihydrate
d) Solubility in water (APPSC -99)
(AIPG -91, KAR -97) 27. Placing cast under tap water is to be avoided as
19. The most commonly used accelerator in gypsum a) H2 0 interferes with hygroscopic expansion
product is: b) H2 0 In hibits polymerization of dental resin
a) Sodium fluoride b) Potassium fluo ride c) H2 0interferes with crystallization of dihydrate
c) Potassium sulfate d) Aluminium sulfate d) Gypsum is slightly soluble in water and the surface
(KAR -2K, AIPG -97) of the cast will be eroded
20. As per ADA No. 25, minimum amount of setting (APPSC -99)
expansion required for type V gypsum productis 28. Gillmore needle is used for
a) 0.05 b) 0.10 a) Testing the strength of plaster of paris
c) 0.15 d) 0.20 b) Evaluating t he setting tim e of plaster of paris
(KCET-2011) c) Testing the metal hardness
21. Which is a gypsum product? d) Testing the purity of noble metals
a) Stone b) Plaster (TNPSC -99)
c) Investment d) All of the above 29. 2 % solution of borax is used as a surface hardening
(AIIMS -98) agent for
22. Plaster of paris: a) Casting stone b) Dental plaster
a) Is wet calcined hemihydrate c) Dental stone d) Type IV gypsum
b) Has porous and irregular crystals (AIPG -94)
c) Is a- hemihydrate 30. Gypsum product having least expansion:
d) Has a W / P of 0.2 a) Impression plaster b) Model plaster
(PGI -99) c) Stone plaster d} Die stone
23. In plaster of paris the setting time is primarily (AP- 05)
altered by 31. Modifiers are added to gyps um mainly to
a) Altering P / L Ratio a) Modify setting time b) Modify setting expansion
b) Altering temperature of mixing water c) Modify strength d) Decrease the porosity
c) Speed and length of hand spatulation (AP- 05)
d) Addition of accelerators and retarders 32. Most commonly used retarder in gypsum:
(AP -04) a) Na2 So4 b) Nacl
24. Type I gypsum product is also called c) Citrates d) K2 So4
a) Impression plaster b) Class I stone/ Hydrocal (AP-06)
c) Class II stone/Densite 33. ADA specification number for gypsum products are
d) Model plaster described under
(COMEDK -04) a) 1 b) 4
25. The ~ (Beta} hemihydrate of gypsum requires more c) 12 d) 25
water to float its powder particles because (COMEDK-2011)
a) They are more regular in shape and dense
b) They are more regular in shape and highly porous

17} A 18} C 19} C 20} B 21} D 22} B 23} D 24} A 25} C 26} A 27} D 28} B 29} B
30} D 31} A 32} C 33} D
, DENTAL MATERIALS

34. The setting expansion of gypsum products can be a) Increased working time
reduced by: b) Increased setting time
a) Increased spatulation c) Decreased setting time
b) Adding potassium sulfate d) None of the above
c) Less water powder ratio (GCET-14)
d) Allowing setting under water 43. In dental materials setting time is measured by
(COM EDK- 05) which test
35. Finer particle size of silica: a) Rockwell test b) Cold blend test
a) Slower the hygroscopic expansion c) Vickers test d) Vicat needle method
b) Greater the hygroscopic expansion (AIIMS NOV-13)
c) Normal setting expansion 44. Which of the following is false regarding setting of
d) No setting expansion plaster products?
a) Setting expansion increases with increase in
36. A rough/ chalky surface of the cast is due to nucleic density
a) Not waiting for 20 min before pouring b) Most accurate method of controlling setting
b) Excess wetting agent left on impression expansion is by chair side addition of chemicals
c) Air incorporation in the mix c) Potassium sulphate is an accelerator and reduces
d) All of the above the setting expansion
(AP-08) d) Maximum acceleration effect for sodium sulphate
37. The function of 2% potassium sulphate in a is approximately 3.4%
gypsum product is: (PGI JUNE-2014)
a) To regulate the setting expansion 45. Which of the following is incorrect about non-
b) Regulate setting time gypsum die material?
c) Acts as retarder d) None a) All impression materials are compatible with these
(AIIMS-07) b) Polyester produces the most accurate dies
38. Plaster mix: c) Epoxy resin dies are undersized but are used due
a) Rapid spatulation results in decreased setting time to their higher abrasion resistance
b) 2% potassium sulphate is accelerator d) Metal dies are preferable to high strength gypsum
c) Citrates a re retarders dies for captek restoration
d) All of the above (PGI JUNE-2014)
(MCET-07)
39. The strength of gypsum products is generally
expressed in terms of
a) Tensile strength b) Wet strength
c) Green strength d) Compressive strength
(KCET-10)
40. Water: powder ratio for class IV stone is
a) 0.22-0.24 b) 0.24-0.28
c) 0.65-0. 70 d) 0.45-0.50
(AIIMS-14)
41. Within practical Limits, using Less water in mixing
plaster will result in a set product that
a) Contracts b) Is stronger
c) Is more porous d) Is less brittle
(AP-14)
42. Increased spatulation is case of dental stone
results in

34) B 35) B 36) D 37) B 38) D 39) D 40) A 41) B 42) C 43) D 44) B 45) B>A
Dental ;lut.,e

3. GYPSUM PRODUCTS - ANSWERS


1. 'C' [PHILLIPS' 11th ed 259/ 12th ed 183] 5. ' D' [PHILLIPS' 11th ed 271/ 12th ed 188]
Calcium sulfate hemihydrate is the starting materia l Wet strength or green strength is the strength
used for production of casts, models and impressions obtained when excess water than that required for
plasters. It reacts with water to form gypsum or hydration of he mi hydrate is left in the specimen.
calcium sulfate dihydrate. The reaction is exothermic When the specimen is dried of the excess water,
in nature. strength obtained is called dry strength. Dry strength
is two or more times the wet st rength.

6. 'A' [Philips 11th ed 258/ 12th ed 189]


2. 'A'[PHILLIPS' 11th ed 257, 274/ 12th ed 183) ~-hemihydrate particles absorb more water, because
Calcination of gypsum at 110° - 120° C results in the crystals are more irregular in shape and are porous
formation of CaS0 4 • 1/z H2 0, which is a constituent of in character.
dental plasters and stones. Calcination under steam
pressure or by dehydration, in presence of succinate 7. 'A' [Check Explanation Below]
and calcium ch loride produces a -hemihydrate. Dry Water of reaction is same for all gypsum products
calcination produces ~-hemihydrate. i.e., 18.6 ml. of water completely reacts with 100g of
hemihydrate. But gauging water differs with the type
3. 'A' [PHILLIPS' 11th ed 273/ 12th ed 183] of gypsum product. It is 45 ml for type I, 30 ml for type
a -Hemihydrate (1-H emihydrate III and 20 ml for type IV.
(Dental stone) (Plaster of paris)
8. ' D' [PHILLIPS' 11th ed 257/ 12th ed 183]
Method of Calcination is done in Dry calcination Dental plaster (beta-hemi hydrate) and dental stone
ca lei nation closed chamber under is done in open (alpha-hemihydrate) differ in particle size, shape
steam pressure or air and porosity. Crystals of beta-hemihydrate are more
by dehydration irregular in shape and porosity than a-hemihydrate.
W:P ratio 0.28 - 0.30 0.40 - 0.50
9. ' B'
Crystals Dense and prismatic Irregular, spongy
shape and more porous
10. ' B'

Beta-hemihydrate calcination true is (AIIMS-14) 11. 'C' [PHILLIPS' 11th ed 274/ 12th ed 182]
a) In presence of air b) In absence of air Cast is the replica of the entire arch. Die is the replica
c) In presence of Cacl2 (3%) of single tooth. Dies are best prepared with type IV
d) Under steam pressure in autoclave gypsum materials.

4. 'C' [PHILLIPS' 11th ed 274/ 12th ed 190] 12. ' B'


Type I Impression plaster
13. 'A' [PHILLIPS' 11th ed 257/ 12th ed 183]
Type II Model plaster
Type III 14. ' B' [PHILLIPS' 11th ed 271/ 12th ed 188]
Dental stone / Hydrocal
{Class I stone)
High strength stone or 15. 'A' [PHILLIPS' 11th ed 265/ 12th ed 186]
Type IV
improved stone or densite or
(Class II stone) 16. ' D' [PHILLIPS' 11th ed 256/ 12th ed 182]
die stone (KCET-07) .
Dental stone of high strength
Type V
and high expansion.
, DENTAL MATERIALS

17. 'A' [PHILLIPS' 11th ed 270/ 12th ed 187) 24. 'A' [PHILLIPS' 11th ed 273/ 12th ed 190]
If the setting of the plaster or stone is allowed to
occur under water, the setting expansion will be more 25. 'C' [PHILLIPS' 11th ed 258/ 12th ed 189)
than doubled. This is called hygroscopic setting
expansion and this is due to additional crystal growth 26. 'A' [PHILLIPS' 11th ed 259]
permitted in presence of water. The hemihydrate is 4 t imes more soluble than
di hydrate.
The casting shrinkage can be compensated by expansion
of investments. Thus gypsum-bonded investments 27. 'D' [PHILLIPS' 11th ed 278/ 12th ed 192]
contribute to mold expansion by hygroscopic expansion Gypsum is slightly soluble in water. If the cast is kept
and compensate the casting shrinkage. under running water its linear dimension may decrease.
The safe method is soaking in water saturated with
18. 'C' [PHILLIPS' 11th ed 257/ 12th ed 189] Ca50 4 (slurry water).

19. 'C' [PHILLIPS' 11th ed 269/ 12th ed 186) 28. 'B' [PHILLIPS' 11th ed 262/ 12th ed 185)
K2SO 4, NaCl (but at higher concentrations act as Evaluation of initial set of POP:
retarder) accelerate the setting reaction of gypsum • Loss of gloss test: The mix loses its gloss
products. • Initial gillmore needle test - The ti me at which
it no longer leaves an impression .
Powdered gypsum added to hemihydrate also
accelerate the setting reaction as these particles act Evaluation of final set of POP:
as nuclei of crystallization whereas borates, citrates, • Vicat needle test: Time elapsed until the needle
acetates act as retarders of setting reaction. no longer penetrates to the bottom of the mix.
• Final gill more needle test: Time at which
20. 'B' [Phillips 11th ed Pg273/ 12th ed 190]
heavier gillmore needle leaves only a barely (no)
Type Setting expansion (%) perceptible mark on the surface
Minimum Maximum
I 0.00 0.15 29. 'B'
II 0.00 0.30
III 0.00 0.20 30. 'D' [PHILLIPS' 11th ed 273/ 12th ed 190]
Refer to Table-3 in synopsis.
IV 0.00 0.10
V 0.10 0.30
31. 'A' [PHILLIPS' 11th ed 264, 268/ 12th ed 186)
The accelerators and retarders not only regulate
For other details, refer gypsum products in Synopsis. setting time of gypsum products but also they red uce
the setting expansion. The theory of such effects is
21. 'D' [PH IL LIPS' 11th ed 256- 5 7/ 12th ed 182]
not still confirmed.
When plaster is mixed with silica, it is known as an
"investment". Investment containing die stone is 32. 'C' [PHILLIPS' 11th ed 269/ 12th ed 186]
called "divestment".
• K2504

22. 'B' [PHILLIPS' 11th ed 257 / 12th ed 183] Accelerators • Na 2504


Plaster of paris is a ~-hemihydrate obtained by dry • NaCl
calcination of gypsum. The water powder ratio of
plaster of paris ranges from 0.4 to 0.5. • Acetates
Retarders • Borates
23. 'D' [PHILLIPS' 11th ed 265/ 12th ed 186] • Citrates
The most effective way of controlling setting time is
by adding accelerators or retarders. 33. 'D' [Phillips 11th ed 273/ 12th ed 189]
ADA specification number for Gypsum products is 25.
Dental ;lut.,e

34. 'B' [PHILLIPS' 11th ed 268/ 12th ed 187] when the concentration is increased, they act as
Factors causing increase in setting expansion: retarders.
• Less water powder ratio
38. 'D' [PHILLIPS' 11th ed 268, 2 69/ 12th ed 186]
• Allowing setting under water
• Increased rate of spatulation 39. 'D' [PHILLIPS' 11th ed 271/ 12th ed 189, 190]
• a.-hemihydrate
• Fine or smaller silica particles 40. 'A' [Check Synopsis]

Factors causing decrease in setting expansion 41. 'B' [Phillips 12th ed 185,189]
• More W/P ratio The greater the water used for mixing plaster the
more the incidence of porosity.
• Addition of modifiers like K250 4 , Nacl, borax.
Effects of greater W/P ratfo of gypsum products:
• f3-hem ihydrate
• More porosity
• Reduced spatulation
• Less dry (compressive) strength
• Less setting expansion
Prolonged spatulation of gypsum products
increases? (AIIMS May-14)
Effects of less W/P (adequate)
a) Increase hygroscopic expansion
b) Decrease setting time • Less porosity
c) Final hardness d) Compressive strength • Greater compressive strength
• Greater setting expansion
35. 'B'
4 2. 'C' [Phillips 12th ed 185 -89]
36. 'D' [PHILLIPS' 12th ed 169, 176] Increased spatulation within practical limits
causes
37. 'B' [PHILLIPS' 11th ed 26 5/ 12 th ed 186] • Reduced setting time
The operator should attempt to control the setting
• Increased setting expansion
time by adding retarders or accelerators to the
gypsum powder. Retarders generally act by forming • Increased compressive strength
an adsorbed layer on the hemihydrate to reduce its
solubility and on gypsum crystals present to inhibit 43. 'D' [Phillips 12th ed 185]
growth. Organic materials such as glue, gelatin, and
some gums behave in this manner. 44. 'B' [Phillis 1st SA e d 193 ]
The chemical modifiers (accelerators or retarders)
The setting time can be accelerated by adding should be incorporated during the manufacturing of
plaster by the manufacturer and not chair side by the
• Gypsum (by adding of small percentage of slurry
operator.
water in the mixing water).
• 2% potassium sulfate (commonly used) 4 5. 'B>A' [Philips 1 st SA e d 2 22 ]
• Sodium chloride. • Acrylic, polyester and epoxy resins are some of the
non-gypsum die materials.
Retarders used are (ABC)
• These are limited in their compatibility with
• Acetates impression materials. Compatibility is specific and
• Borates applicable only to particular brand rather than to
• Citrates and Ch lo rides chemical types of impression materials.
• Organic materials like glue, gelatin, gums • With acrylic and polyesters resin, the curing
• In small concentrations many inorganic salts contraction is very high and cannot be used when
such as sodium chloride acts as accelerators but accurate die is required.
, DENTAL MATERIALS

• Epoxy dies are undersized in comparison with


the prepared tooth but are used because of their
superior abrasion resista nee.
• In the construction of a metal-ceramic crown that
is made of a metal foil coping (captek), metal
foil burnished on a gypsum die is preferred over
gypsum die.
Dental ;lut.,e

4. RESTORATIVE RESINS
1. All of the following statements about the 8. Hydroquinone is added to methyl methacrylate
differences between self-polymerizing acrylic monomer
resins and heat-cured resins are true EXCEPT a) To prevent polymerization during storage
a) The former have a lower molecular weight b) To initiate release of free radicals
b) The former have higher residual monomer content c) To enable polymerization reaction at room
c) The former are more porous tern perature
d) The former have greater t ransverse strength d) All of t he above
(MAN -94, PGI -99) (MAN -2K, AIPG -96)
2. In heat cure denture base acrylic resins the 9. Radio opacity to composite resins is rendered by
monomer is: a) Silica glass b) Organic matrix
a) Methacrylate b) Ethylmethacrylate c) Barium glass and strontium glasses
c) Methyl ethyl methacrylate d) Fluoride particles
d) Polymethylmeth acrylate (MAN -02, COMEDK -06)
(MAN -99) 10. Cross Linking in denture base resin is contributed by
3. If curing occurs at temperature more than 100° C a) Glycol dimethacrylate
porosity results in which area of denture b) Benzoyl peroxide
a) Hard thick central area c) N-Para toluidine d) Methyl methacrylate
b) Thin Palata l area (MAN -02, COMEDK -06)
c) Thin area of flanges 11. The most important disadvantage of acrylic denture
d) Porosity is uniformly distributed base is
(MAN -97) a) Porosity b) Shrinkage
4. Which of the following is an example of a composite c) Tooth breakage d) Water absorption
material? (MAN -95, AP -97)
a) A tilled resin b) Colloidal silica 12. Porosity present in a acrylic denture is usually the
c) Gold alloy d) Wax result of:
(MAN -95) a) A prolonged curing cycle
5. Use of dimethyl-P-toluidine is indicated for b) Lack of sufficient pressure applied to flask
a) Thermal polymerization of acrylic c) Insufficient acrylic resin monomer
b) Chemical polymerization d) Prolonged bench cooling after curing
c) Retarding the polymerization reaction (AIIMS -92)
d) To inhibit the action of benzoyl peroxide 13. Which of the following constituents is common
(MAN -95) in occlusal sealants, bonding agents, composite
6. Which of the following initiator accelerator system resins?
uses light activation composite? a) BIS GMA
a) Diketone-amine b) Organic acid-metal oxide b) Polymethyl methacrylate
c) Organic acid-peroxide c) Benzoin methyl ether
d) Peroxide-amine d) Silica filter
(MAN -01) (AIIMS -91)
7. Acrylic (cold cure) 14. Acrylic resins are used for:
a) Melts at 100°c b) Softens at 100°c a) Anterior restorations b) Temporary bridges
c) Still requires heat for polymerization c) Denture bases d) All of the above
d) Produces heat during polymerization (PGI -98)
(MAN -99)

1) D 2) A 3) A 4) A 5) B 6) A 7) D 8) A 9) C 10) A 11) B 12) B 13) A


14) D
, DENTAL MATERIALS

15. Marginal leakage related to temperature change b) Benzoyl peroxide free radical
occurs to the greatest extent with: c) Hydroquinone d) Methyl ether
a) Amalgam alloy b) Unfilled resin (AIPG -92, KAR -98)
c) Composite resin d) Direct filling gold 24. Which of the following is present in the powder of
(AIPG 91, 2003) the acrylic resin:
16. To prevent porosity in self-cure acrylic resin, curing a) Methyl methacrylate b) Benzoyl peroxide
should be carried in: c) Hydroquinone d) D-methyl paratolouidine
a) Cold water b) Hot water (PGI -99)
c) Under tap water d) Under vacuum pressure 25. Subsurface porosity is due to
(PGI -98, 97) a) Thermal changes b) Thickness of the resin
17. In self-cure acrylic resin, activator is: c) Lack of temperature
a) Toluidine d) Packing past the dough stage
b) Quaternary ammonium compound (PGI -02)
c) Benzoyl peroxide d) Tertiary amine 26. Light cure system, side effect is:
(PGI -02) a) Iritis b) Cataract
18. The main advantage of composites over unfilled c) Conjunctivitis d) Retinal damage
direct filling resin is their: (AIIMS -97)
a) Higher solubility in saliva 27. If resin is packed in sandy stage what will be seen
b) Lower modulus of elasticity in the processed denture?
c) Esthetic excellence a) Distortion b) Porosity
d) Lower thermal co-efficient of expansion c) Inhomogeneous colour
(KAR -97) d) Increased strength
19. Composite resins in comparison to acrylic have: (AIIMS -94, AIPG -89)
a) Low compressive strength 28. Bonding of composite resins to tooth structure is
b) High abrasive resistance by:
c) High water absorption a) Covalent bond b) Ionic bond
d) High polymerization shrinkage c) Mechanical d) Vanderval forces
(PGI -98) (KAR -01)
20. Composite has 29. Which one of the following is used as filler in
a) Resin b) Filler composite resins?
c) Resin & Filler d) None a) Quartz b) Silica
(AP -01) c) Zinc particles d) Aluminium
21. In processing methyl methacrylate, the resin may (AIPG -94)
show porosity if the flask is placed too soon. 30. The type of spatula used to mix composite is:
Porosity most likely occur a) Plastic b) Stainless steel
a) Throughout the denture c) Iron d) None of the above
b) Near the borders (PGI -98)
c) In the thickest part d) On the denture surface 31. The chemical used to etch enamel is:
(AIIMS -89, -94) a) Zinc oxide b) Methyl methacrylate
22. Boiling point of methyl methacrylate: c) Phosphoric acid d) Eugenol
a) > B.P of water b) < B.P of water (PGI -98)
c) Equal to BP of water 32. Benzoin methyl ether in a polymer indicates that
d) Is at normal room temperature they may be cured in the presence of:
(AP -97, AIPG -93) a) UV light b) Visible light
23. Polymerisation of heat-cured methyl methacrylate c) Infrared light d) Diketone
is initiated by:
a) Tertiary amine

15) B 16) D 17) D 18) D 19) B 20) C 21) C 22) A 23) B 24) B 25) A 26) D 27) B
28) C 29) B>A 30) A 31) C 32) A
Dental ;lut.,e

33. Which of the following procedure is indicated to 39. No trail closure is necessary with which one of the
produce cross-linking in the polymerization of technique during denture fabrication:
linear chains of acrylic: a) Compression moulding technique
a) Addition of grain refining agents b) Injection moulding technique
b) Addition of difu nctional monomers c) Fluid resin technique
c) Increasing the amount of heat required for linear d) Light cured technique
polymerization (KAR -97)
d) Incorporation oif terminating agents 40. The particle size in microfilled composites is
(AIPG -01) a) 0.02 to 0.04mm b) 0.5 to 1.0microns
34. Which of the following may result if an excessive c) 0.01 to 0.1 microns d) 0.3 to 0.4 mm
amount of monomer is incorporated into an acrylic (KCET- 07, PG! JUNE-2012)
resin mixture? 41. A material which is a mixture of two different
a) Excessive expansion materials and its properties are
b) Low impact strength a) Composite b) Complex material
c) Excessive shrinkage c) Combined d) Compound
d) Excessive brittleness (AP -02)
(AIPG -91, AIIMS -99) 42. Acid-etching is done for
35. For photo-curing the composite resin, the light a) 30 Seconds b) 60 seconds
source: c) 90 seconds d) 120 seconds
a) Should be held at a millimeter distance from the (PG! -97)
restoration 43. The polymerization of self-curing resin takes faster
b) Need no protection while in use in
c) Will cure composite upto a depth of 5 millimeter a) Cold water b) Hot water
thickness c) Under pressure d) In vacuum
d) Is fiber optic (PG! -97)
(AIPG -03) 44. Co-efficient of thermal expansion is highest for:
36. Porosity of denture base is due to: a) Amalgam b) Si licate cement
a) Packing at dough stage c) Gold alloy d) Acrylic resin
b) Curing at 160°F for 9 hrs (PG! -02)
c) Inadequate pressure 45. Which of the following provide opacity to
d) All of the above composite?
(AIPG -2K) a) TEGDMA b) BIS-GMA
37. The function of the coupling agent in a restorative c) Ti02 d) None
resin is to allow: (PGI-2011)
a) Adhesion of resin particles 46. The latest method of curing denture base resins:
b) Bonding between filler crystals a) Heat b) Visible light
c) Bonding between filler and resin c) Chemicals d) Lasers
d) Bonding between tooth and resin (AIPG -96)
(AIPG -94) 4 7. Dentin bonding agents usually contain:
38. The imperfection seen on the lingual flange of a a) Only Hydrophobic component
lower denture caused by rapid heating during a b) Only Hydrophilic component
curing cycle is called: c) Hydrophobic and hydrophilic component
a) Localised shrinkage porosity d) Lyophillic and lyophobic component
b) Gas inclusion porosity (COMEDK -04)
c) Micro-porosity d) Surface porosity
(AIPG -01)

33} B 34} C 35} A 36} C 37} C 38} D 39} B 40} C 41} A 42} A 43} B 44} D 45) C
46} B 47} C
, DENTAL MATERIALS

48. One of the claims for the superiority composite 54. For esthetic areas where high luster is required the
resin restoration over silicate cement restorations restoration used usually
is that the resin restorations: a) Glass ionomer restoratives
a) Prevent galvan ic action b) Hybrid resin composites
b) Are practically insoluble in oral fluids c) Microfilled resin composites
c) Adapt to the walls of the cavity better d) Macrofilled resin composites
d) Have a higher coefficient of thermal expansion (COMEDK- 05)
(APPSC -99) 55. Inadequate pressure application during flasking:
49. The advantage of unfilled methyl methacrylate is a) Increases vertical dimension
that: b) Causes teeth to be shrunken in investment
a) It can be finished smoothly c) Causes poor quality of colour
b) It has a Low degree of flow d) Causes fracture of denture
c) It's non-irritation to pulp (AIPG-07)
d) It's wear resistant is high 56. Which of the following absorbs maximum amount
(PGI -93) of water?
50. To cure a heat cure resin, a proper heating cycle is a) Microfilled resin b) Macrofilled resin
necessary: c) Hybrid d) Small particle composites
a) To prevent the porosity (AIPG- 07)
b) To prevent warpage 57. The % of free monomer in a heat - cured acrylic
c) To prevent volume expansion resin is
d) All of the above a) 3% to 5% b) 8% to 10%
(KAR -98) c) 0.2% to 0.5% d) 0.6% to 0.8%
51. About polymerization shrinkage of composite all (KCET-07)
are true, except: 58. The restoration which shows percolation and
a) Polymerization shrinkage is greater if bonded shrinkage along the margin is:
surface area is lesser than unbounded surface area a) Composites b) Unfilled resins
b) Polymerization shrinkage is high if within the c) Amalgam d) Polycarboxylate
enamel margins. (AP-07)
c) Acid etching and priming will decrease 59. What is the disadvantage of using composites as
polymerization shrinkage. restorative material?
d) Microleakage can occur because of polymerization a) Increased thermal conductivity
shrinkage. b) Decreased wear resistance
(AIIMS- 06) c) Decreased thermal conductivity
52. When restoring a tooth with resin material, acid d) none of the above
etching can do all of the following except (AP-07)
a) Increase the surface area 60. The commonly used laser for curing composite
b) Permit chemical bonding between resin and enamel resin is
c) Create surface irregularities in enamel for better a) Nd:YAG b) CO 2
mechanical retention of resin c) ER:YAG d) Argon
d) Condition the tooth surface for better wetting (COMEDK-07)
(COMEDK-06) 61. Insufficient closure of the flasks during packing of
53. Inorganic phase of the composites aid in: denture base resin can result in:
a) Increasing the mechanical strength a) Poor colour stability
b) Decreasing the coefficient of thermal expansion b) Less polymerization shrinkage
c) Reducing the polymerization shrinkage c) Increase in vertical dimension
d) All of the above d) Decrease in vertical dimension
(KAR-04) (AIIMS-07)

48) B 49) A 50) A 51) A 52) B 53) D 54) C 55) A 56) A 57) C 58) B 59) B 60) D
61) C
Dental ;lut.,e

62. What is the polymerization shrinkage if 3:1 70. One bottle system of adhesive resins
powder: liquid ratio is used? a) 4th generation bonding agents
a) 6% b) 8% b) Separate etching step not required
c) 10% d) 21% c) Separate etching step is still required
(MCET-07) d) Contains on ly primer
63. Acrylic resins were first introduced as a denture (KCET-10)
base material in 71. Dentin conditioner has the following function.
a) 1925 b) 1937 a) Removes smear layer
c) 1951 d) 1956 b) Increases surface energy of dentin
(KAR-2K) c) Forms a thin resin Layer over exposed collagen
64. Macromolecules in Dental Resin is attached by: fibrils
a) Covalent bonds b) Sonic bonds d) Helps in bonding with composite
c) Vander Wall's Forces d) Hydrogen Bonds (AIIMS-08, 2012)
(MCET-07) 72. Most advantageous indicatio n of acid etching is
65. Quartz tungsten light cure device, minimum output a) Decrease micro Leakage
of energy should not be less than b) Decrease polymerization shrinkage
a) 300 mw/cm2 b) 350 mw/cm2 c) Decrease coefficient of thermal expansion
c) 400 mw/cm2 d) 450 mw/cm2 d) Decrease porosity in resto rative material
(PGl-2011) (AIIMS-08)
66. Along with polymerisation shrinkage, one of the 73. Residual monomer content of chemically activated
major disadvantages of polymethyl methacrylate resins is -
as denture base resin are its poor a) 0.2% to 0.5% b) 2% to 5%
a) Biological properties c) 0.3% to 0.5% d) 3% to 5%
b) Thermal properties (COMEDK-09, PGI DEC-2013)
c) Mechanical properties 74. VLC resins are also called as -
d) Esthetic properties a) Microwave-activated resins
(UPSC-09) b) Tertiary amine-activated resins
67. Denture acrylics contain cross-linking agents, c) Light activated resins
mainly to improve their: d) Heat activated resins
a) Internal color b) Tissue compatibility (COMEDK-09)
c) Surface hardness d) Craze resistance 75. The accepted polymer to monomer ratio for packing
(IGNOU-10) of polymethyl methacrylate resin is -
68. The most common coupling agents are organic a) 3:1 b) 4:1
silicon compounds called c) 10:1 d) 5: 1
a) parabane b) ligane (COM ED K-09)
c) silane d) chelane 76. Cold cure acrylic expand in water by:
(BHU-2012) a) 1% by volume 0.23% by weight
69. The process wherein two or more chemically b) 10% by volume 0.23% by weight
different monomers, each with some desirable c) 0.1% by volume 0.23% by weight
property, can be combined to yield specific physical d) None of the above
properties of a polymer is known as (NEET-2013)
a) Step-growth polymerization 77. Etching depth after 30 sec for orthodontic bracket
b) Addition polymerization with orthophosphoric acid is:
c) Condensation polymerization a) 10 um - 20 um b) 3 um - 9 um
d) Copolymerizatio n c) 50 um - 110 um d) 200 um - 250 um
(KCET-10) (AIIMS MAY 2012)

62} A 63} B 64} A 65) A 66} B 67) D 68} C 69} D 70) C 71} A 72} A 73) D 74) C
75} A 76} A 77} A
, DENTAL MATERIALS

78. Which of the following is NOT true of polymethyl 85. According to ADA specification number 12, dough
methacrylate? forming time should be less than _ ___ min. from
a) Knoop hardness number 18 to 20 the start of the mixture
b) Tensile strength of 60 MPa a) 90 min. b) 60 min.
c) Density of 1.13 g/cm3 c) 50 min. d) 40 min .
d) Modulus of elasticity of 3.4 GPa (APPG-15)
(KAR-2013) 86. Role of plasticizer in synthetic resins in dentistry
79. During polymerisation of acrylic resin, above what is
temperature benzoyl peroxide form free radicals: a) Increase smoothness
a) 25° c b) 37° c b) To increase bulk
c) 50° C d) 60° C c) To prevent polymerisation shrinkage
(AIIMS-2012) d) To reduce softening and fusion temperature
80. Etchant preferred in gel form than in liquid form (AIIMS MAY-14)
a) Better control over placement 87. Of the light produced by quartz - tungsten -
b) Enhance and concentrate the action of acid halogen, only a small percentage is suitable
c) Helps in visualization while placement for curing composite resin material and most is
d) None of the above converted into heat. How much percentage of this
(AIIMS MAY 2012) light is useful for curing?
81. An acidic agent that dissolves the in organic a) 0.5% b) 5%
structure in dentin that allows infiltration of c) 10% d) 50%
adhesive resin is (COMEDK-15)
a) Dentin bonding agent
b) Dentin conditioner
c) Primer
d) None of the above
(MCET-14)
82. In acrylic restorative materials, the pumping
action of alternately imbibing and excluding fluids
is termed as
a) lmbibition b) Percolation
c) Syneresis d) Evaporation
(MCET-14)
83. What is the wave length of visible light used for
curing light cure restoration?
a) 400 - 420 nm b) 250 - 300 nm
c) 420 - 475 nm d) 300 - 370 nm

84. True about conversion of monomer to polymer is


a) Density changes from 1.19g/cm3 to 0.9 and
volume shrinkage is 21%
b) Density changes from 1.19g/cm3 to 0.9 and
volume shrinkage is 7%
c) Density changes from 0.9g/cm3 to 1.19 and
volume shrinkage is 21%
d) Density changes from 0.9g/cm3 to 1.19 and
volume shrinkage is 7%
(AIIMS NOV-14)

78) D 79) D 80) A 81) B 82) B 83) C 84) C 85) D 86) D 87) A
Dental ;lut.,e

4. RESTORATIVE RESINS - ANSWERS


1. 'D' [PHILLIPS' 11th ed 735/ 12th ed 483] In chemically activated composites benzoyl peroxide
In chemically cured or self cure resins, polymerization is t he initiator and tertiary amine is the activator.
is not complete and contains greater amount of
un reacted monomer. The residual monomer acts as 7. 'D' [PH IL LIPS' 11th ed 7 31/ 12th ed 483]
an irritant and plasticizer and thus decreases the Polymerization of denture base resins is an exothermic
transverse strength of denture resin . reaction.

Also, the colour stability of self cure resins is inferior 8. 'A' (PHILLIPS' 11th ed 723/ 12th ed 475]
to heat cure resins. This property is due to presence Hydroquinone is the inhibitor, which prevents
of tertiary amines, which are susceptible to oxidation polymerization of liquid during storage.
and results in colour changes.
Retarder in monomer to prevent polymerisation
Because of less polymerization, the self cure resins is (AIIMS May-14)
have less shrinkage and greater dimensional accuracy. a) Hydroqui none b) Polyacrylic acid
The self cure resins are more porous than heat cure c) Benzoyl peroxide d) Dibutyl phthalate
resins. To prevent porosity, self cure resins are cured
under vacuum pressure. 9. 'C' [PHILLIPS' 11th ed 406/ 12th ed 280]
Composite resins contains
2. 'A' [PHILLIPS' 11th ed 722/ 12th ed 475]
In heat cure, the monomer consists of unpolymerised Resin matrix BIS-GMA, Urethane dimethacrylate,
met hyl methacrylate, hydroquinone (inhibitor), glycol TEGDMA (Viscosity controller)
dimethacrylate (cross linking agent). Fillers Colloidal silica or quartz, glasses of
barium, zirconium give radiopacity.
The polymer powder consists of prepolymerised
Coupling Organosilanes, Zi rconates,
beads of polymethyl methacrylate, benzoyl peroxide
agent titan ates.
(initiator) .

Self-cure has same composition except that the liquid 10. 'A' (PHILLIPS' 11th ed 723/ 12th ed 475]
consists of dimethyl-p-toluidine (activator) which is If sufficient glycol dimethacrylate is included in the
a tertiary amine. mixture, several interconnections are formed in the
polymer and this provides increased resistance to
3. 'A' [PHILLIPS' 11th ed 732, 740/ 12th ed 488] deformation.
Boiling point of monomer is 100.8°C. If curing
temperature crosses this point, evaporation of 11. 'B' (PHILLIPS' 11th ed 739/ 12th ed 485]
monomer occurs. This causes surface and sub surface The most important disadvantage of acrylic resins is
(interna l) porosity within the thicker portions of polymerization shrinkage. Other disadvantages are
denture. Porosity decreases strength and compromises porosity, water absorption and crazing.
aesthetics and hygiene.
12. 'B' (PHILLIPS' 11th ed 742/ 12th ed 487]
4. 'A' (PHILLIPS' 11th ed 401/ 12th ed 277] COMMON CAUSES OF POROSITY
Curing temperature 5 ubsurface or internal
5. 'B' [PHILLIPS' 11th ed 735/ 12th ed 286]
more than 100°c porosity in thick areas
of denture
6. 'A' (PHILLIPS' 11th ed 408/ 12th ed 287]
In light activated composites, camphoroquinone is Insufficient pressure or Larger voids all over
the initiator and diketone is the accelerator. inadequate material the denture
, DENTAL MATERIALS

Polymer, monomer interaction proceeds in SANDY,


Air inclusion resulting Mostly with fluid resin
STRINGY, DOUGH LIKE, ELASTIC and STIFF stages.
in voids technique
Dough stage is ideal for packing.
If the flask is placed too Porosity in thickest
soon in boiling water part of denture If resin is packed in sandy stage porosity occurs, as
there is no or little interaction of polymer and monomer.
13. 'A' (PHILLIPS' 11th ed 402/ 12th ed 279) Over packing causes excessive thickness and under
packing causes porosity.
14. 'D' (PHILLIPS' 11th ed 747- 48/ 12th ed 474)
Acrylic resins are used as dent ure bases, denture teeth, 28. 'C' [PHILLIPS' 11th ed 383 / 12th ed 259]
reliners, and temporary bridges and as obturators in Bonding of composites to tooth structure occurs by
maxillofacial prosthesis, etc. micro mechanical retention. Acid etching creates
micropores into which the resin penetrates resulting
15. ' B' in resin tag formation. These 'tags' penetrate to a
Unfilled resins have high coefficient of thermal depth of 10 to 20µm, but their lengths are usually
expansion/ contraction of 81x10-6/°C. dependent on the enamel etching time.

16. ' D' 29. 'B>A' [PHILLIPS' 11th ed 406/ 12th ed 280)
Eventhough Option 'X. Quartz was also a filler in early
17. 'D' [PHILLIPS' 11th ed 735/ 12th ed 483] versions of dental composites, because of its hardness,
quartz containing composites are more difficult to
18. 'D' [PHILLIPS' 11th ed 400/ 12th ed 277] polish and will cause more abrasion of opposing
Advantages of composites (filled resins) over unfilled teeth or restorations. So-called amorphous silica has
resins are lower co-efficient of thermal expansion, low the same composition and refractive index as quartz,
polymerization shrinkage, low water absorption and but it is not crystalline and not as hard, thus geately
high abrasive resistance. reducing the abrasiveness of the composite surface
structure.
On the other hand, composites have less colour
stability and do not take the smooth finish as that of 30. 'A'
unfilled resins. Plastic or Teflon coated instruments should be used
with composites.
19. ' B' [PHILLIPS' 11th ed 400/ 12th ed 277)
31. 'C' (PHILLIPS' 11th ed 384/ 12th ed 260]
20. 'C' [PHILLIPS' 11th ed 401/ 12th ed 277] 37% phosphoric acid is used as an etchant.
Concentration above 50% results in formation of
21. 'C' [PHILLIPS' 11th ed 741/ 12th ed 487] monocalcium phosphate monohydrates, which prevents
further dissolution. The etchant is supplied in a gel
22. 'A' [PHILLIPS' 11th ed 732/ 12th ed 481) form to allow control over the area of placement.
Boiling temperature of water is 100°c. Boiling
temperature of methyl methacrylate is 100.8°(. 32. 'A'
Benzoin methyl ether is the activator in UV light
23. 'B' [PHILLIPS' 11th ed 722/ 12th ed 475) curing system.

24. 'B' [PHILLIPS' 11th ed 722/ 12th ed 475) 33. 'B' [PHILLIPS' 11th ed 72 3/ 12th ed 475)
Glycol dimethacrylate (cross linking agent) has two
25. 'A' [PHILLIPS' 11th ed 732/ 12th ed 487) double bonds whereas methyl methacrylate has a single
bond. One molecule of cross-linking agent unites with
26. 'D' [PHILLIPS' 10th ed 298/ 12th ed 290] two molecules of methyl methacrylate. Cross-linking
increases resistance to crazing and deformation.
27. 'B' [PHILLIPS' 11th ed 727/ 12th ed 488)
Dental ;lut.,e

34. 'C' [PHILLIPS' 11th ed 727/ 12th ed 485] 47. 'C' [PHILLIPS' 11th ed 387/ 12th ed 265]
The optimal polymer monomer ratio is 3:1. Excess of Dentin is hydrophilic and composite resin is
monomer leads to increased polymerization shrinkage. hydrophobic. So a dentin-bonding agent should contain
both hydrophilic and hydrophobic components.
35. 'A' [PHILLIPS' 11th ed 411/ 12th ed 290]
The light tip should be held as close as possible to 48. 'B' [ PHILLIPS' 11th ed 400]
the restorative material. Light cures composite up to
a depth of 2mm thickness. 49. 'A' [PHILLIPS' 11th ed 4 20]

36. 'C' [PHILLIPS' 11th ed 742/ 12th ed 487] 50. 'A' [PHILLIPS' 11th ed 732/ 12th ed 487]

37. 'C' [PHILLIPS' 11th ed 406/ 12th ed 286] 51. 'A' [Sturdevant 4th ed 479-4 81/ PHILLIPS' 12th ed
The function of coupling agent is to provide a bond 485]
between filler particles and resin matrix. Organo Composites shrink while hardening. This is called as
silanes are most commonly used coupling agents. The polymerization shrinkage. This leads to opening of a
methacrylate groups of organo silanes bonds with the 'V' Shaped gap if the polymerization forces are greater
resin while the silanol groups of organo silanes bonds than the bond strength to enamel and dentin.
with filler. Coupling agents improves physical and
mechanical properties and provide hydrolytic stability Bonded surfaces
by preventing penetration of water along the filler- Configuration factor (C-factor) = Unbonded surfaces
resin interface.
The higher the C-factor, the greater is the potential
for bond disruption from polymerization effects.
38. ' D' [PHILLIPS' 11th ed 742/ 12th ed 487)

39. ' B' Preparation (-factor


Class I (High risk) 5/1 = 5
40. 'C' [PHILLIPS' 11th ed 423/ 12th ed 281] Class II 2/1 = 2
Composite Filler particle size Class IV (Low risk) 1/4 = 0.25
Conventional comlJ)osite 8 - 12 ~Lm
52. ' B' [PHILLIPS' 11th ed 383/ 12th ed 260]
Small particle filled 1 - 5 µm
Micro filled composite 0.04 - 0.4 µm 53. ' D' [PHILLIPS' 11th ed 403/ 12th ed 280]
Important properties of fillers:
Hybrid composite 0.6 - lµm
• Reinforcement of resin matrix

41. 'A' [PHILLIPS' 11th ed 401/ 12th ed 275] • Reduction in polymerization shrinkage
• Reduction in thermal contraction/expansion
42. 'A' [PHILLIPS' 11th ed 385/ 12th ed 261] • Improved workability
Currently 15 - 30 seconds time is used for acid • Increased radiopacity
etching because it produces sufficient bond strength,
• Reduction in watersorption, staining
conserves enamel and time. Primary teeth and teeth
with high fluoride content require more etching time.
54. 'C' [PH ILLIPS' 11th e d 425/ 1 2th ed 281, 283)
43. ' B'
55. 'K [PHILLIPS' 11th ed 244]
Denture warpage is the deformity or change of shape
44. ' D' [PHILLIPS' 11th ed 55 Ta ble 3.2/ 1 2th ed 40]
of the denture which can affect the fit of the denture.
Warpage can occur during processing (FLASKING) as
45. 'C' [Philips 11th ed 409/ 12th ed 280)
well as at other ti mes.

46. ' B' [PHILLIPS' 11th ed 737 / 12th ed 485]


, DENTAL MATERIALS

56. 'A' [PHILLIPS' 10th ed 285 Table 12.2] 69. ' D' [PHILLIPS' 11th ed 162/ 12th ed 96]
3 types of copolymers are:
57. 'C' [PHILLIPS' 12th ed 483] i) Random copolymer
No sequential order exists among the two
58. ' B'
(or more) monomer units along the polymer chain .
..... ABBABAABABBAAAA
59. ' B' [Sturdevant 5th ed 508/ PHILLIPS' 12th ed 295]
ii) Block copolymer
60. ' D' [Vimal Sikri t5t ed 381/ PHILLIPS' 12th ed 289) Identica l monomer units occur in relatively long
Argon laser is the most suited among various lasers to sequences along the main polymer
cure composite resins. The intensity of laser required ..... AAAAABBBBBBAAAAA
for curing is 250 nnW ±50mW and the time required is
iii) Graft or branched copolymer
10 sec/increment.
Sequences of one type of monomer unit are
61. 'C' [Craig 12th ed 531] attached as a graft onto a backbone of a second
type of monomer unit.
The changes in the vertical dimension of a denture
during processing are mainly caused by variations in AAAAAAA
flask pressure, fla.sk temperature, consistency of the I I
dough and strength of stone mould. The pressure BB
applied during closure of a flask is the most im portant
BB
factor. The vertical opening may be reduced to 0.5mm
rather than reported variations of 2-5mm by taking
BB
proper precautions.
Copolymerization has a strong influence on the physical
62. 'A' [ PHILLIPS' 11th ed 7 27 / 12th ed 4 78] and mechanical properties of t he resulting resin.
When the powder and liquid co mponents are mixed
Eg.: Small amounts of ethyl acrylate may be
in proper positions, a dough like mass is formed. The
accepted polymer-to-monomer ratio is 3:1 by volume. copolymerized with methyl methacrylate to alter the
Using a 3:1 ratio, the volumetric shrinkage may be flexibility and fracture resistance of a denture.
limited t o approximately 7% (0.5% linear shrinkage)
70. 'C' [Sturdevant 5th ed 189/ PHILLIPS' 12th ed
264]
63. ' B' [PHILLIPS' 11th ed 722/ 12th ed 475]
Contemporary bonding agents ca n be classified as
64. 'A' [PHILLIPS' ttth ed 24/ 12th ed 18] total etch and self etch .
Covalent bonding occurs in many organic structures a) TOTAL ETCH:
such as dental resins, in which the compounds link i) 3-step
to form backbone structure of hydrocarbon chains. • Etchant (E) + Primer (P) + Adhesive(A)
A typical characteristic of cova lent bond is their
Eg.: Scotchbond multipurpose Allbond - 2
directional orientation.
ii) 2-step / one bottle (E + PA)
65. 'A' [ Philips 11th ed 412/ 12th ed 2 89]
For Quartz tungsten light cure device, optimal output Eg.: Sing le bond, one step, excite
energy is 400-800 mw/cmZ. The minimum output
energy should not be less than 300 mw/cm2. b) SELF ETCH
i) Self etch primers - 2 step (EP+A)
66. 'B'
Eg.: Clearfill SE bond

67. 'D' [PHILLIPS' 11th ed 723/ 12th ed 490)


ii) Self etch adhesives - 1 step (EPA)
68. 'C' [Philips 11th ed 406/ 12th ed 286] Eg.: Prompt L-POP, Xena III
Dental ;lut.,e

71. 'A' [Sturdevant 5th ed 391/ PHILLIPS' 12th ed Dentin conditioning is more technique sensitive than
261] enamel because of the complexity of dentin structure
like:
72. 'A' [PHILLIPS' 11th ed 382-383/ 12th ed 260] • Living tissue
• 50% inorganic
73. 'D' [PHILLIPS' 11th ed 736/ 12th ed 483]
• 30% organic (type I Collagen)
Chemically activated resins display 3-5% free
monomer whereas heat activated resins exhibit • 20% fluid
0.2-0.5% of free monomer.
Refer exp la nation of Question No.28 also
74. 'C' [PHILLIPS' 11th ed 737/ 12th ed 485]
VLC means visible light cured resins. 82. 'B' [Check Explanation Below]
Acrylics have high co-efficient of thermal expansion
75. 'A' [PHILLIPS' 11th ed 727 / 12th ed 478] and shrinks or expands seven or more times that of
tooth enamel for every degree change in temperature.
76. 'A' [Phillips 11th ed 742/ 12th ed 489] Margins of the filling opens at low temperature
Polymethyl methacrylate exhibits water sorption and oral fluids get into the cavity. Next time when
when placed in an aqueous environment. the temperature rises, the margins close in, due to
expansion and at the same time expel the fluids at
For each 1% increase in weight produced by water the margins of a filling. This is known as percolation
absorption, acrylic resin expands 0.23% linearly. or marginal leakage. This leads to secondary caries,
This linear expansion is approximately equal to the discoloration, pain and failure of the restoration.
thermal shrinkage encountered as a result of the
polymerization process. Hence these processes very 83. 'C' [Phillips 12th ed 287-289)
nearly offset one another. Composites contain camphoroquinone photosensitizer
that absorbs blue Light with wave lengt hs between
77. 'A' [Enamel acid etching- A review, Compendium 400-500 nm. So Light curing units are designed to
Jan 2007; 28(1): 663) emit light within range of 400-500 nm.
Acid etching creates a porous layer of upto 5-50 µ m.
Most resin tags may penetrate 10-20 µ m into the Type of curing unit Light emitted in nm
enamel porosity, but their lengths are dependent on QTH (Conventional)
400-500 nm
etching and rinsing time. (Quartz Tungsten Halogen)
LED
78. 'D' [Phillips 11th ed 166] 440-480 nm
(Light emitting diode)
Modulus of elasticity of polymethyl methacrylate is PAC
2.4 GPa. 400-500 nm
(Plasma arc curing)
Argon laser lamps 490 nm
79. 'D' [Phillips 11th ed 730/ 12th ed 483]
Above 60 degree centigrades, benzoyl peroxide
molecules decompose to yield free radicals that The wavelength of conventional light cure unit
initiate the chain-growth polymerization. is (AIIMS May-14)
a) 400-500 b) 200-300
80. 'A' [Phillips 11th ed 385/ 12th ed 262] c) 600-700 d) 20-50
Gel form of etchant allow precise placement over
a specific area. Gels are made by adding silica or 84. 'C' [Phillips 1st SA ed 112]
polymer beads to the acid. Physical properties of monomer:
• Molecular Weight= 100
81. 'B' [Phillips 12th ed 261) • Boiling Point= 100.8°C
The most widely used etchant/conditioner for bonding • Density = 0.045g/mL at 20°c
to both enamel and dentin is phosphoric acid.
, DENTAL MATERIALS

• Heat of Polymerization = 12.9 KCaljmol


• Volumetric shrinkage during polymerization= 21%

85. 'D' [Phillips 1st SA ed 509)


The t ime required for the resin mixture to reach a
dough like stage is termed as dough forming time.
According to ADA specification 12 for denture base
resins, this consistency should be attained in less
than 40 min. from t he start of the mixing process.
Most products reach that consistency in less than 10
min.

86. 'D' [Phillips 1st SA ed 102]


Plasticizers in dental polymers:
• When added to resins, plasticizers reduce the
softening or melting or fusion temperatures of
resins.
• Their addition makes a resin that is hard and stiff
at room temperature to one that is flexible and
soft. Eg:- PVC water line tubing.

External plasticizer:
Sometimes, the action of plasticizer is analogous
to that of a solvent, with the plasticizing agent
penetrating between the macromolecules and
increasing the intermolecular spacing. This type is
called as external as it is not a part of the polymers
structure.

Internal plasticizer:
• Plasticizing of a resin can also be accomplished by
copolymerization with a suitable comonomer. In
this case, it becomes part of the polymer and acts
as an internal plasticiser.
• Usually reduces strength, hardness and softening
point of resin.

87. 'A' [Check Explanation Below]


The light produced by the 0TH, only 0.5% is useful
for curing, and most is converted at some point into
heat. To minimize any heating that might occur
during light curing, two fi lters (UV and infrared band
pass filters) are ililserted in the path of the light to
eliminate significant amounts of unnecessary light
and convert it into heat within the unit.
Dental ;lut.,e

5. AMALGAMS
1. The term "trituration" means c) 9-30% copper d) 13-30% copper
a) Lysing amalgam alloy (MAN -01)
b) Mixing of amalgam alloy and mercury 9. Compare to conventional amalgams, spherical
c) Removal of excess of mercury amalgams
d) None of the above a) Require more mercury
(MAN -94, AI PG -94) b) Requires heavy compaction forces
2. Dynamic creep is the c) Have Longer setting times
a) Continuing alloying between silver-tin alloy and d) None of the above
mercury during the life of restoration (MAN -98)
b) Deformation of set amalgam during function 10. Finishing and Polishing of Amalgam make the
c) Process whereby alloy is "wetted" by mercury restoration:
d) Spread of amalgam during packing a) Increase in tarnish and corrosion resistance
(MAN -94, AIIMS -93) b) Increase the marginal strength
3. The highest mercury concentration in amalgam c) Decrease the tarnish and corrosion resistance
filling is found d) Increased compressive strength
a) At the margin of the restoration (AIPG -02, AIIMS -01)
b) In the centre of the restoration 11. Which of the following statements about high
c) In the deepest part of the restoration copper silver alloy compared to conventional alloy
d) None of the above is not true?
(MAN -94) a) It has increased tensile and compressive strength
4. The dental amalgam is most resistant to b) It has poor tarnish and corrosion resistance
a) Compressive Stress b) Im pact stress c) Its' edge strength is greater
c) Shear stress d) Tensile stress d) Mercury content in the final restoration is Less
(AIIMS -03)
5. The ADA specification No 1 for composition of 12. Setting time of Amalgam is best controlled by?
amalgam alloy, recommends, a) Using spherical particles
a) 65% silver, 32% tin and 3% copper b) Lathe cut alloy
b) 49% silver, 32% tin and 19% copper c) Altering Hg-Alloy ratio
c) 65% silver, 29% tin and 5-6% copper d) Trituration time
d) None of the above (PGI -02)
(MAN -97) 13. Increase in the residual mercury in silver amalgam
6. According to ADA specification No or number 1, the filling can :
minimum compressive strength for silver amalgam a) Cause fracture of the filling
filling after one hour should be b) Tarnish and corrosion
a) 80 Mpa b) 140 Mpa c) Increase the strength
c) 260 Mpa d) 510 Mpa d) Decrease condensation pressure
(MAN -2K) (AIIMS -98, 95)
7. Which of the following phases of dental amalgam 14. High Cu alloys have all of tlle following except:
has minimum strength a) High tensile strength
a) Gamma 1 b) Gamma 2 b) Low creep
c) Epsilon 1 d) Gamma 3 c) High corrosion d) None of the above
(MAN -01) (AIIMS -95)
8. Admixed high copper alloy powder contains 15. Chronic mercury toxicity results from:
a) 9-20% copper b) 13-20% copper a) Ingestion while removing old amalgam

1) B 2) B 3) A 4) A 5) C 6) A 7) B 8) A 9) D 10) A 11) B 12) D 13) A


14) C 15) D
, DENTAL MATERIALS

b) Skin contact 24. Amalgam restorations give the best clinical service
c) Mercury vapour d) All of the above when the residual mercury content is:
(AIIMS -92) a) 38-42% b) 48-52%
16. In non zinc containing dental amalgam alloy, the c) 58-62% d) 68-72%
percentage of zinc present is (AIIMS -93, MAN -94)
a) Less than 0.02% b) Less than 0.1% 25. Discolouration of silver alloy is due to:
c) Less than 0.01 % d) None of the above a) Tarnish b) Tarnish and corrosion
(KAR -03) c) Wet corrosion d) Dry corrosion
17. In spherical alloys as compared to Lathe cut: (AIPG -2K, KAR -02)
a) Less condensing force is required 26. Percentage of tin in Low copper alloy is
b) More condensing force is required a) 30-35% b) 26-28%
c) Both require same condensing force c) 13-32% d) 22-30%
d) Mani pulation is easy (PGI-2011)
(AIIMS -95) 27. Advantage of zinc containing amalgam is:
18. High copper dental amalgams are superior to other a) Better handling property
amalgams because high copper dental amalgams: b) Dimensional stability
a) Have less marginal breakdown c) Resistance to creep
b) Are workable at lower Hg-alloy ratio d) Toxicity to pulp and dentin
c) Have a higher ratio of tensile to compressive (PGI -95)
strength 28. Cause of expansion in zinc containing amalgam is:
d) Have less resistance to tarnish and corrosion a) Zinc and Water b) Water
(AIIMS -94, AIPG -2K) c) Hydrogen d) Nascent oxygen
19. Well premeasured capsules of amalgam can release (PGI -95)
mercury vapour during: 29. What is the working time of amalgam?
a) Mulling b) Carving a) One to two minutes b) Three to four minutes
c) Condensation d) Trituration c) Five to six minutes d) Seven to eight minutes
(AIPG -2K) (PGI -98)
20. The largest component of amalgam alloy is: 30. Which of the following amalgam alloys is Least
a) Silver b) Tin susceptible to creep:
c) Zinc d) Copper a) Lathe cut b) Spherical
(PGI -98) c) Microfi ne
21. In a high copper amalgam the phase, which is d) Dispersion with high copper
eliminated, is: (PGI -2K)
a) Gamma 1 b) Gamma 2 31. The percentage of copper in high copper alloy is:
c) Gamma 1 and 2 d) No phase is eliminated a) 10-12% b) 0 -6%
(AIPG -99) c) 13-30% d) 20 -30%
22. During amalgamation, trituration is done to: (AIPG -94, AP -03)
a) Dissolve the alloy in mercury 32. How soon after a moisture contamination does a
b) Coat the alloy particles with mercury zinc containing amalgam alloy start expanding:
c) Remove excess mercury from the amalgam a) 24 hours b) 1-2 days
d) Dissolve Hg in alloy c) 3-5 days d) 7 days
(AIIMS -94) (PGI -2 K)
23. Moisture contamination of amalgam can lead to: 33. What is common in amalgam and ceramics?
a) Marginal breakdown b) Shrinkage a) More compressive strength but less tensile strength
c) Delayed expansion d) Increased stresses b) More compressive strength and tensile strength
(PGI -2K, AIIMS -2K) c) Less compressive strength but more tensi le
strength

16) C 17) A 18) A 19) C 20) A 21) B 22) B 23) C 24) B 25) B 26) B 27) A 28) C
29) B 30) D 31) C 32) C 33) A
Dental ;lut.,e

d) Less compressive strength and tensile strength c) Chlorides of tin d) All of the above
(AIPG-2012) (APPSC -99)
34. What is the most frequently used restorative 43. Which is not true about high copper amalgam alloys:
material? a) Low tensile and compressive strength
a) Silicate b) Amalgam b) Low Hg: Alloy ratio
c) Composite d) Gold c) High tensile strength
(AIIMS -97) d) Low creep
35. Which of the following constituents of amalgam (KAR -94)
alloy decreases expansion: 44. The threshold limit value of mercury exposures is:
a) Copper b) Zinc a) 0.01 mg/ m3 b) 0.05 mg/ m3
c) Silver d) Tin c) 0.1 mg /m 3 d) 0.001 mg /m 3
(KAR -97) (COMEDK -14, KAR -95)
36. Which of the following silver amalgam alloys have 45. "Amalgam" means
the maximum strength: a) A metallic powder composed of silver, tin, copper
a) Lathe cut b) Spherical and zinc
c) Admixed d) Single composition b) An alloy of two or more metals, one of which is
(PG! -97) mercury
37. The higher the Hg alloy ratio in dental amalgam: c) An alloy of two or more metals that have been
a) Higher the strength b) Lower the creep value dissolved in each other in the molten state
c) More matrix material formed d) A metallic substance in powder or tablet form that
d) More gamma I phase formed is mixed with mercury
(AIPG -89) (KAR -94)
38. Copper content in low copper amalgams is: 46. Mercury intoxication in dental office mainly results
a) 6% b) 12-30% from:
c) 29% d) 19% a) Direct contact with the mercury
(AP -02) b) Inhalation of mercury vapours
39. Ag-Cu eutectic alloy has a characteristic property c) Ingestion of mercury
of that fusion temp of: d) None of the above
a) The resultant alloy is greater (PG! -95)
b) The resultant alloy is lesser 47. In amalgam alloy which acts as oxygen scavenger:
c) The resultant alloy varies according to the content a) Cu b) Zn
of Ag and Cu c) Pd d) Ag
d) None (AP -04)
(PG! -02) 48. Over-trituration of silver alloy and mercury:
40. Which phase provides maximum strength in a) Reduces contraction
hardened mercury/silver alloy? b) Increases the strength of lathe-cut alloy but
a) Silver/ Mercury phase reduces the strength of spherical alloy amalgam
b) Silver /Tin phase c) Decreases creep
c) Tin/ Mercury phase d) Zinc/ Mercury phase d) Gives al dull and crumbly amalgam
(PG! -93) (AIPG-06)
41. Absorption of mercury in the human body occurs 49. By increasing the percentage of which metal, the
least from strength and hardness of amalgam increases:
a) Lungs b) Gastro-intestinal Tract a) Ag b) Zn
c) Skin d) Kidneys c) Cu d) Hg
(COMEDK-10) (PGI- 05)
42. The tarnished layer of silver amalgam consists of:
a) Sulphides of silver b) Oxides of tin

34} B 35} D 36} D 37} C 38} A 39} B 40} B 41} D 42} A 43} A 44} B 45) B 46} B
47} B 48} B 49} C
, DENTAL MATERIALS

50. Mercury rich condition in a slow setting amalgam c) 3 minutes d) 15 minutes


alloy system in a restoration results in
a) Accelerated corrosion 59. What fraction of inhaled mercury vapors retained
b) Fracture of the restoration in the body?
c) Marginal damage d) All of the above a) 45-55% b) 55-65%
(KAR-04) c) 65-85% d) More than 85%
51. The tensile strength of dental amalgam is (COMEDK-06)
a) 27-55 MPa b) 60-100 MPa 60. High strength amalgam is achieved by
c) 120-140 MPa d) 18-20 MPa a) Maximum matrix and minimum alloy phase
(MHCET-15) b) Minimum matrix and maximum alloy phase
52. A true eutectic alloy has melting point c) Maximum matrix phase
a) Above that of low fusing metal d) Minimum alloy phase
b) Above the melting point of either metal
c) Below that of high fusion metal 61. The effect of trituration on strength in an amalgam
d) Below the melting point of either metal restoration depends on -
(AIPG-2011) a) Amalgam alloy b) Trituration time
53. Creep value of which of the following is the c) Speed of amalgamator
highest? d) All the above
a) Low copper amalgam alloy (KCET-09)
b) Admix alloy 62. Which of the following does not occur in High
c) Single composition alloys Copper Amalgam?
d) Creep value of all the above mentioned alloys is a) Electrochemical Corrosion
same b) Chemical Corrosion
(AIPG- 06) c) Penetrating Corrosion
54. Advantage of minimum mercury technique or d) Corrosion does not occur at all
Eames technique is all except (AIIMS-09)
a) High strength b) Sets quickly 63. Decrease in creep occurs in:
c) Needs no squeezing of excess mercury a) Under trituration or over trituration of amalgam
d) Greater plasticity and adapts well to cavity walls b) Decreases with condensation pressure
(KAR-04) c) Increases with condensation pressure
55. Amalgam achieves 70% of the strength by: d) Cannot be predictable
a) 2 hours b) 4 hours (AIIMS-06)
c) 8 hours d) 16 hours 64. For dental amalgam, the elastic modulus and
(AIPG-05) tensile strength is
56. The solid solution of silver and mercury is called a) 40 Gpa and 60 - 100 MPa
a) Y1 b) Y2 b) 21 Gpa and 27 - 55 MPa
c) ~1 d) y c) 350 Gpa and 10 - 120 MPa
(COM EDK-05) d) 360 Gpa and 125 - 130 MPa
57. Mercury is toxic because it (KCET-07)
a) Complexes with hemoglobin to form methemoglobin 65. Cavo surface angle for amalgam restoration is butt
b) Inhibits hemoglobin synthesis, producing anemia joint as:
c) Inhibits anaerobic glycolysis a) It increass compressive and tensile strengths
d) Binds to sulfhydryl groups b) Decreased compressive, increased strength
(COM EDK-05) c) Decreased compressive, decrease tensile strength
58. Once triturated, the Dental amalgam should be d) Increased compressive, increased tensile strength
condensed with in (MCET-07)
a) 5 minutes b) 6 minutes

50) D 51) A 52) D 53) A 54) D 55) C 56) A 57) D 58) C 59) C 60) B 61) D 62) C
63) C 64) B 65) A
Dental ;lut.,e

66. Adequate mixing of mercury is indicated by: c) Corrosion d) Creep


a) Dry Mix b) Shiny Mix (U PSC-09)
c) Short Mixing Time d) None 74. High copper amalgam can be produced
(MCET-07) a) From smooth atomized particles
67. Which of the following are characteristic feature of b) From irregular and spherical particles
high copper amalgam alloy? c) From lathe cut particles
a) Low compressive strength d) From traditional alloy particles
b) High marginal breakdown (KCET-10)
c) Less marginal# d) High creep 75. Eames technique is otherwise known as
(AIIMS-07) a) No squeeze cloth technique
68. Which is true regarding lathe cut silver alloy: b) Increasing dryness technique
a) Requires least amount of mercury c) Bloting mix
b) Achieves lowest compressive strength at 1 hr. d) Mortar and pestle mix
c) Has tensile strength both at 15 min & 7 days (EMCET-14)
comparable to high copper unicompositional alloy 76. Discolourisation of Ag containing alloy is due to
d) Has low creep a) Wet corrosion b) Dry corrosion
(AIIMS-07) c) Tarnish d) Both Tarnish and Corrosion
69. Co-efficient of Thermal expansion of amalgam is (AP-14)
a) 6.6 a (ppm k-1) b) 11.4 a (ppm k-1)
c) 14.0 a (ppm k-1) d) 25.0 a (ppm k-1)
(KCET-08)
70. Gallium and Indium added to Amalgam replace
a) Silver b) Tin
c) Mercury d) Zinc
(MCET-10)
71. A patient who has had a recent amalgam filling
in the upper teeth has a gold filling in the lower
teeth, the patient complains of pain. The reason
for this can be ma inly
a) improper amalgam filling
b) pulp exposure
c) galvanism d) none of the above
(AP-09)
72. Which one of the following is not an objective of
trituration?
a) Remove oxides from powder particle surface
b) Keep the amount of gamma-1 or gamma-2 matrix
crystals to maximum
c) Pulverize pellets into particles to aid in attack by
mercury
d) Achieve a workable mass of amalgam in minimum
time
(U PSC-09)
73. Outstanding clinical performance of dental
amalgam restoration is related to its
a) Compressive strength
b) Tensile strength

66} B 67) C 68} C 69} D 70} C 71} C 72} B 73} A 74) B 75) A 76) B&D
, DENTAL MATERIALS

5. AMALGAMS - ANSWERS
'B' [PHILLIPS' 11th ed 523/ 12th ed 341) 5. 'C' [PHILLIPS' 11th ed 497 / 12th ed 342)
Trituration is mixing of alloy and mercury. Usually, Ag - 63-70%
the alloy particles are coated with a film oxide, and Low copper alloys
Sn - 26-28% (PGI - 11)
this Layer prevents the penetration of mercury. During (Contains <6% of
Cu - 2-5%
trituration, the oxide layer is removed by abrasion so copper)
Zn - 0-2%
that a clean surface of alloy can come in contact with Ag - 40-60%
the mercury. Single composition Sn - 22-30%
spherical alloys Cu - 13-30%
Spherical alloys usually require less amalgamation Zn - 0-4%
time than the Lathe-cut alloys.
Admixed alloys contains 1
part of high copper spherical
Undertrituration results in weak restoration with
Admixed alloys particles ( silver-copper eutectic
rough surface. The rough surface will increase the
alloy) and 2 parts of low-
susceptibility to tarnish.
copper lathe cut particles

2. 'B' [PHILLIPS' 11th ed 46, 540/ 12th ed 33,353)


Both single composition and admixed alloys are types
Creep is the time dependent plastic deformation,
of high copper alloys. The total copper content in
which occurs when a metal is subjected to a constant
admixed alloys ranges from 9 -20%.
Load near its melting point.
6. 'A' [PHILLIPS' 11th ed 515/ 12th ed 353)
Amalgams will undergo creep as its melting point is
According to IDA, the minimum compressive strength
slightly above the room temperature.
after 1 hr. is 80 mpa. The compressive strength of a
satisfactory amalgam should be at least 310m pa.
Restorations of higher-creep alloys generally
experience greater marginal breakdown than the
Both low-copper and high-copper amalgams have
restorations with Low-creep values.
tensile strengths in the range of 50-70 mpa. Admixed
alloys will have least tensile strength.
Dynamic creep is the deformation of set amalgam
during function. The creep can cause an amalgam
The compressive strength of high-copper sing le
restoration, to extend out of the cavity preparation
composition alloys > high copper admix alloys >
and increases its susceptibility to marginal breakdown.
low-copper alloys.

3. 'A' [PHILLIPS' 11th ed 538/ 12th ed 352) •


7 'B' [PHILLIPS' 11th ed 503, 504/ 12th ed 344)
The concentration of mercury in the margins will be
The physical properties of the hardened amalgam
2-3% higher than the remaining bulk of restorations.
depend on the relative percentages of various micro
These areas are critical in terms of corrosion fracture,
structural phases. The y-phase (unconsumed Ag-Sn
and secondary caries.
particles) is strongest followed by gamma 1 (Ag 2 Hg 3 )
and gamma 2 (Sn 7 Hg8 ). The tin-mercury phase
4. 'A' [PHILLIPS' 11th ed 512/ 12th ed 3 52)
(y2) lacks corrosion resistance and is the weakest
Amalgams have Low tensile strength (48-70 mpa) and
component of dental amalgam.
high compressive strength (minimum of 310 mpa).
Since amalgams are brittle materials with low edge
In high copper alloys, the gamma 2 (Sn 7 Hg 8) is
strength, bevelling at the cavosurface margin is
eliminated and replaced by eta (Cu,?ns} phase, which
contraindicated.
is most resistant to tarnish and corrosion. As a
result, the high copper amalgam alloys will have high
The cavosurface angle in amalgams is 90° and forms
strength, hardness and high corrosion resistance.
a butt end.
Dental ;lut.,e

8. 'A' [PHILLIPS' 11th ed 505/ 12th ed 342] Elimination of y2 phase by eta phase and less amount
Low copper alloys Copper is < 6% by wt. of residual mercury (around 42%) made high copper
High copper sing le alloys more resistant to tarnish and corrosion.
Copper is 13-30%
com position alloys
14. 'C' [PHILLIPS' 11th ed 504/ 12th ed 354]
Admixed alloys Copper is 9-20% by wt.

15. 'D' [PHILLIPS' 11th ed 537/ 12th ed 359]


9. 'D' [PHILLIPS' 11th ed 514/ 12th ed 343]
Though mercury is absorbed through various routes,
Spherical alloys require less mercury than typical
the primary risk to dental personnel is from inhalation.
lathe-cut alloys because they have a small surface
area.
The maximum safe level of occupational exposure is
50 micrograms / m3 of air.
High condensation pressures are required to minimize
porosity and to express mercury from lathe-cut
16, 'C' [PHILLIPS' 11th ed 496]
amalgams. On the other hand, spherical amalgams
Alloys containing less than 0.01% of zinc are
(which contain less mercury) condensed with lighter
classified as non -zinc alloys and the alloys with more
pressures produce adequate strength.
than 0.01% of zinc are classified as zinc containing
alloys.
10. 'A' [PHILLIPS' 11th ed 531/ 12th ed 354]
Amalgams without finishing and polishing will
Uses of Zinc
have a rough surface microscopically and results in
concentration cell-type corrosion. • Reduces brittleness.
• It reacts rapidly with oxygen and their impurities
11. 'B' [PHILLIPS' 11th ed 504/ 12th ed 354] and prevents the oxidation of more important
The y 2 or tin-mercury phase in low copper amalgams elements like silver, copper or tin. Thus it acts as
lacks the corrosion resistance and is the weakest deoxidiser or scavenger.
component of dental amalgam. • Better handling property.

But in high copper amalgams, the y2 phase is replaced On the other hand, if the amalgam is contaminated
by eta phase (Cu 6Sn 5 ), which is most resistant to with moisture during trituration or condensation,
tarnish and corrosion. As a result, the high copper Zn causes delayed expansion. Delayed expansion
alloys will have high strength, hardness and corrosion is due to pressure exerted by hydrogen gas, which
resista nee. is a corrosion product evolved during the reaction
between Zinc and moisture.
12. 'D' [PHILLIPS' 11th ed 525/ 12th ed 347]
The setting time is best controlled by altering the 17. 'A' [PHILLIPS' 11th ed 514/ 12th ed 349]
trituration speed or trituration time or both. For a
given type of alloy and mercury: alloy ratio, increased 18. 'A' [PHILLIPS' 11th ed 504/ 12th ed 357]
trituration time or speed shortens the working and
19. '('
setting times.

Spherical alloys require less amalgamation time than 20. 'A' [PHILLIPS' 11th ed 496/ 12th ed 342]
the lathe-cut alloys.
21. 'B' [PHILLIPS' 11th ed 506/ 12th ed 345]
13. 'A' [PHILLIPS' 11th ed 538]
The mercury content of finished restoration should be 22. 'B' [PHILLIPS' 11th ed 523/ 12th ed 341]
50% approximately. Restorations containing mercury
23. 'C' [PHILLIPS' 11th ed 510/ 12th ed 351]
in excess of 55% showed an higher incidence of
marginal fracture and surface deterioration than the Delayed expansion occurs if the amalgam mix is
restorations that contains mercury in 50% range. contaminated with moisture during trituration or
condensation.
, DENTAL MATERIALS

The expansion begins 4 or 5 days after condensation 35. 'D' [Check Explanation Below]
and results in hyper occlusion and cracks in tooth. Tin controls the reaction between silver and mercury.
The restoration becomes wedged so tightly against Without tin, the reaction would be too fast and the
the cavity walls and the pressure in pulpal direction setting expansion would be unacceptable. Tin reduces
will cause intense pain. both the rate of reaction and t he expansion to optimal
values.
24. 'B' [PHILLIPS' 11th ed 522, 538/ 12th ed 346]
36. 'D' [PHILLIPS' 11th ed 512 Table 17.2/ 12th ed
25. 'B' [PHILLIPS' 11th ed 517, 57/ 12th ed 354] 352 Tab 15-2]
Tarnish is surface discoloration or slight loss of Both compressive and tensile strengths are greater
surface finish or luster of a metal. The discoloration for sing le composition alloys. Compressive strength
of amalgam is due to formation of black silver sulfide. is least for low copper alloys whereas the 24-hour
tensile strength is least for admixed alloys.
Corrosion is the actual deterioration of the metal by
reaction wit h environment. 37. 'C' [PHILLIPS' 11th ed 513/ 12th ed 352]
In cases of higher mercury: alloy ratio, increase in the
Tin chlorides and oxides are the corrosion products final mercury content increases the volume fraction of
of low copper alloys whereas copper oxide is the matrix phases at the expense of alloy particles. As a
corrosion product of high copper alloys. result, amalgams containing higher amounts of final
mercury are weaker.
26. 'B' [Philips 11th ed 497/ 12th ed 342]
38. 'A' [PHILLIPS' 11th ed 502/ 12th ed 342]
27. 'A' [PHILLIPS' 11th ed 500]
39. 'B' (Check Explanation Below]
28. 'C' [PHILLIPS' 11th ed 510/ 12th ed 351] Eutectic alloys are those in which its components
exhibit complete liquid solubility but limited solid
29. 'B' solubility.

30. 'D' [PHILLIPS' 11th ed 512, 516 Table 17.2/ 12th In high copper admixed alloy, the silver-copper phase
ed 352 Tab 15-2] exhibits a eutectic structure at the composition of
High copper single composition alloys will have least silver 71. 9% and copper 28.1%.
creep value followed by admixed and low copper alloys.
Type of alloy % of creep 40. 'B' (PHILLIPS' 11th ed 503/ 12th ed 352]
The unconsumed Ag-Sn particles or y -phase will have
Low copper alloys 0.8 - 8% a strong effect on strength of restoration. The more
Admixed 0.4 -0.5% this phase in the final structure, the stronger is the
amalgam.
Single composition 0.05 - 0.09 %

41. 'D' (PHILLIPS' 11th ed 537/ 12th ed 349]


31. 'C' [PHILLIPS' 11th ed 504/ 12th ed 342]
Although mercury can be absorbed through skin or by
Option 'B' is for low copper alloys.
ingestion, the primary risk is from inhalation.

32. 'C' [PHILLIPS' 11th ed 510/ 12th ed 351]


42. 'A' (PHILLIPS' 11th ed 517/ 12th ed 354]
Options B & C indicate the corrosion products of
33. 'A' [Philips 11th ed 511/ 12th ed 352]
low copper amalgams. Copper oxide is the corrosive
product of high copper a lloys.
34. 'B'

43. 'A' (PHILLIPS' 11th ed 512/ 12th ed 352]

44. 'B' [PHILLIPS' 11th ed 537/ 12th ed 124]


Dental ;lut.,e

45. ' B' [PHILLIPS' 11th ed 496/ 12th ed 340] 59. 'C'

46. ' B' [PHILLIPS' 11th ed 537/ 12th ed 359] 60. 'B' [PHILLIPS' 11th ed 514/ 12th ed 352]

47. ' B' [PHILLIPS' 11th ed 500] 61. ' D' [PH IL LIPS' 11th ed 513/ 12th ed 3 52]
The effect of trituration on strength depends on the
48. ' B' [M anappallil 1st ed 180] type of amalgam alloy, the trituration and speed of the
amalgamatory. Under- or over-trituration decreases
49. 'C' [Manappallil 2n,d ed 177] the strength in both low and high copper amalgams.

50. ' D' [PHILLIPS' 11th ed 519, 540/ 12th ed 352, 357] 62. 'C' [PHILLIPS' 10th ed 382]
• Tarnish and corrosion Gamma-2 phase is implicated in both marginal
failures and active corrosion in traditional alloys;
• Marginal fracture
Oxides and chlorides of tin are the corrosion products
• Increased creep
in traditional alloys. These are present at the
• Decreased strength tooth-amalgam interface and penetrate the bulk of
• High voids restoration. Corrosion products containing copper are
found in high copper amalgams. But the corrosion
51. 'A' [Phillips 1st SA ed 63] process is more limited, because ri-phase is less
Elastic Tensile susceptible to corrosion than y2 phase of traditional
Moduli Strength amalgam.
(GPa) (MPa)
63. 'C' [PHILLIPS' 11th ed 516/ 12th ed 353]
Resin-based composite 17 30-90
Dental porcelain 40 50-100 64. ' B' [PHILLIPS' ttth ed 512/ 12th ed 352]
The tensile strengths of both low and high copper
Polymethyl methacryte 3.5 60
amalgams range between 48 - 70 Mpa.
Amalgam 21 27-55
Alumina Ceramic 350-418 120 65. 'K [Craig 12th ed 242]
Resistance to compression forces is the most
52. 'D' [Phillips 11th ed 131/ 12th ed 8 4]
favourable strength characteristic of amalgam.
Eutectic alloy is an alloy in which component metals Because amalgam is strongest in compression and
have limited solid solubility. Eg.: 72% silver and 28% much weaker in tension and shear, the cavity design
copper. The melting point of eutectic alloy is below should maximize compressive stresses in service and
the melting point of individual metal that is silver or minimize tension or shear stresses
copper. Eutectic literally means, "Low melting".
66. 'B' [Craig 12th ed 250]
53. 'A' [Mana ppallil t •t ed 175/ PHILLIPS' 12th ed 352] Undermixing, normal mixing and overmixing are the
three mixing variables that result from variations in
54. 'D' [Manappallil znd ed 189]
condition of trituration of the alloy and mercury. The
undermixed amalgam appears dull and is crumbly,
55. 'C' [PHILLIPS' 10th ed 379/ Mannapallil 2nd ed 185] the normal mix appears shiny and separates in a
single mass from capsule and the overmixed amalgam
56. 'A' [PHILLIPS' 11th ed 503/ 12th ed 344] appears soupy and tends to stick to the inside of the
capsule.
57. ' D'
67. 'C' [PHILLIPS' 11th ed 504/ 12th ed 357]
58. 'C' [PHILLIPS' 11th ed 52 7/ 12th ed 350 Fig 15-15] Compared t o traditional low-copper amalgam, high
copper amalgams became the materials of choice
because of their high early strength, low creep, good
, DENTAL MATERIALS

corrosion resistance and good resistance to marginal 76. 'B & D' [According to key provided by the university;
fracture. Check Explanation below]
Tarnish is an early indication and precursor of
68. 'C' [PHILLIPS' 11th ed 512, Tab. 17-2/ 12th ed corrosion. In tarnish there will be surface discoloration
352] or loss of Lustre but no disintegration of the metals.
Lathe cut alloys contain irregular particles which Eg: Silver sulphide tarnish.
require more mercury and are important constituents
of lower copper alloys. They have highest percentage Two types of corrosion are:
of creep (2.0%), compared to high copper admix i) Chemical or dry
(0.4%) and high copper sing le composition alloys ii) Electrochemical or wet
(0.13).
Chemical (or) dry corrosion occurs in the absence of
Compressive Tensile moisture. There is direct combination of metallic and
strength Creep strength non -metallic elements to form a chemical compound
(%) at 24 hrs by oxidation, halogenation or sulfurization. Eg:
1hr 7 days discoloration of silver by sulphur. So discoloration of
(MPa)
Lathecut silver containing alloys is both by tarnish and dry
145 343 2 60 corrosion.
Low copper
admix high
137 431 0.4 48 Wet corrosion requires presence of water or electrolyte
copper
as a pathway for the transport of electrons. It is more
Single
important for dental restorations. In oral cavity dry
composition 262 510 0.13 64
and wet corrosion cannot be distinguished separately
high copper
due to the presence of saliva.

69. ' D' [PHILLIPS' 11th ed 55, Tab. 3-2/ 12th ed 40 Tab
3-2]
• Option A is for Aluminious porcelain
• Option 'B' is for enamel
• Option 'C' is for pure gold
• Option 'D' is for amalgam

70. 'C' [Sturdevant 5th ed 153]

71. 'C' [PHILLIPS' 11th ed 61/ 12th ed 43]

72. 'B' [PHILLIPS' 11th ed 523, 524/ 12th ed 341]

73. 'A' [PHILLIPS' 11th ed 512/ 12th ed 352]

74. 'B' [PHILLIPS' 11th ed 505/ 12th ed 345)


Admixed high copper alloy powders usually contain
30-55% of spherical high copper powder. Lathe cut
powders, which are irregular, occupy the remaining.

75. 'A' [Sturdevant 4th ed 150/ Also refer synopsis]


Named as Eames or no squeeze or minimal mercury
technique, Ea mes was the first to promote a low
mercury to alloy mixing ratio.
Dental ;lut.,e

6. METALLURGY
All of the following statements about cast cobalt d) All the type of gold can be heat treated and
alloys are true EXCEPT: annealed
a) They have a modulus of elasticity twice that of (MAN -02)
gold alloys 8. What is the use of boxing an impression:
b) They are more rigid than gold alloys a) Ease of construction of denture
c) They have higher ductility than gold alloys b) Preserving the sulci
d) They have a lower proportional limit than gold c) Ease of pouring an impression
alloys d) All of the above
(MAN; AIIMS -94) (MAN -99)
2. The contraction of gold alloys on solidifying is 9. The strength of a gypsum investment is dependent
approximately: on:
a) 0.4% b) 1.4% a) Carbon content b) Silica content
c) 2.4% d) 3.4% c) Gypsum d) Copper
(MAN -95) (AIPG -07, AIIMS -04)
3. Addition of large amounts of platinum to a casting 10. Resistance to corrosion in a cobalt-chrome casting
gold alloy will is due to presence of:
a) Decrease its strength and ability to be hardened. a) High quality iron b) Chrome
b) Decrease its tarnish resistance c) Cobalt d) Nickel
c) Increase its fusion temperature (AIIMS -2K, AIPG -91)
d) Redden the alloy 11. The main form of iron carbide found in 18-8 steel:
a) Martensite b) Ferrite
4. The pattern for the metallic framework of a c) Austenite d) Pearlite
removable partial denture is fabricated from: (AIPG -92, MAN -94)
a) Inlay wax type-II b) Boxing wax 12. Flux is used during casting to:
c) Modelling wax d) Casting wax a) Remove the oxides formed on the gold alloy
(MAN -2K) b) Lower the melting point
5. Ideal requirement of metal-ceramic casting: c) Replace casting wax d) Speed the burnout process
a) Low fusing ceramic and high fusing alloys (PGI -98, AIIMS -94, NEET- 2013)
b) Matching of coefficients of thermal expansion of 13. NiTi alloys have:
ceramics and alloy a) Shape memory b) Hyper rigidity
c) Bonding between ceramic and alloy c) Corrosion resistance d) Weldable properties
d) All of the above (KAR -98)
(MAN -02) 14. The maximum carbon content of steel used in
6. Gap between true end of casting ring and wax dentistry is:
pattern should be: a) 19% b) 2%
a) 1/4 inch b) t/2 inch c) 1.7% d) 4%
c) 3/8 inch d) one inch (AP -02)
(MAN -02) 15. Pickling:
7. Which of the following is correct: a) Is accomplished by soakirng the casting in baking
a) Type I and Type II gold can be heat treated and soda
annealed b) Causes porosity in gold
b) Type III and Type IV gold can be heat treated and c) Removes surface oxides from gold castings
annealed d) Remove investment from gold casting
c) Type I and IV gold can be heat treated and annealed (PGI -98, AIIMS -96)

1) C 2) B 3) C 4) D 5) D 6) A 7) B 8) C 9) C 10) B 11) C 12) A 13) A


14) C 15) C
, DENTAL MATERIALS

16. Wax pattern is ideally invested immediately to c) Incomplete wax elimination


decrease the dimensional changes caused by: d) Incomplete gas elimination
a) Decrease flow b) Continuous expansion (AIMS -99, AIPG -96)
c) Relaxation of the internal stresses 25. Which of the following zones is used for melting
d) Water loss alloy during casting:
(PGI -01) a) Reducing flame b) Oxidizing flame
17. Stainless steel orthodontic wire can be hardened c) Green flame d) All of the above
by: (OR) Bending orthodontic wire at room (AP -98, AIIMS -94)
temperature is an example of 26. Incomplete casting is the result of:
a) Tempering b) Work/ Strain hardening a) More porous investment in the mould
c) Age hardening d) Precipitation hardening b) Narrow sprue diameter
(PGI -01, AIIMS MAY-14) c) Hollow sprue pin d) Large reservoir
18. The most commonly used Titanium alloy for dental (GCET -14, AIIMS -91)
and medical purposes is: 27. Back pressure porosity can be avoided by:
a) Ti 6Al 4V b) Ti 5Al 4V a) Using asbestos liners
c) Ti 5Al 5V d) Ti 6Al 6V b) Placing the sprue at least one quarter inch away
(KAR -02) from the end of casting ring
19. A thinner mix of a gypsum-bonded investment c) Preventing rapid heating the investment in burnout
will: oven
a) Produce a smoother casting d) Using a short and wide sprue
b) Decrease setting expansion (AP -98)
c) Increase setting expansion 28. Mouth temperature waxes are:
d) Increase thermal expansion a) Iowa wax b) Modeling wax
(AIIMS -01) c) Green stick wax d) None of the above
20. Cobalt-Chromium alloys contains: (PGI -98)
a) 30% cobalt and 60% chromium 29. Setting expansion is advantageous in:
b) 60% cobalt and 30% chromium a) Casts b) Models
c) 1% palladium d) 20% gold c) Investment d) Dies
(AIPG -03, AIIMS -01) (KAR -01)
21. Which one of the following is most elastic? 30. Tempering of steel cause
a) a titanium b) ~ titanium a) increases its toughness
c) Chrome-cobalt-nickel b) increases hardness
d) Nickel titanium c) increases ductility d) decreases its toughness
(AIIMS -94) (PGI-2011)
22. Gypsum bonded investment should not be heated 31. Indium in metal alloys:
over: a) Improve bonding
a) 100°c b) 1,300°C b) Harden the metal ceramic Ag-Pd alloy
c) 1,063°C d) 900°C c) Both A and B d) None of the above
(KAR -98) (AIPG -93)
23. Cleaning of base metal alloy is done by: 32. In a patient with allergy to costume jewelry, which
a) Warm HCI b) Cold HF of the following is contraindicated:
c) Cold H2S04 a) High gold b) Low gold
d) Sand blasting with aluminium oxide c) Palladium - Ag d) Ni - Cr
(AIIMS -97) (AIIMS -96)
24. Glossy smooth margins of a casting are due to: 33. Which of the following will produce the most rigid
a) Shrinkage of aHoy on cooling restoration for framework of same dimension?
b) Investment breakdown a) Wrought alloy b) Palladium alloys

16) C 17) B 18) A 19) B 20) B 21) D 22) A 23) D 24) C 25) A 26) B 27) B 28) A
29) C 30) A 31) C 32) D 33) C
Dental ;lut.,e

c) Cobalt-chrome alloys c) Eliminate air bubbles on the wax pattern


d) Partial denture casting gold in hardened condition d) Reproduce fi ne detail
(AIPG -91) (AIIMS -98)
34. Turbulence of molten gold is due to: 43. Sub surface porosity can be decreased by:
a) Impurities in gold b) Improper venting a) Decreasing the sprue length
c) A and B d) None of the above b) Decreasing the sprue thickness
(AIIMS -92) c) Increasing the melting temperature
35. Apart from composites and ceramics, another d) Increasing the mou ld temperature
material that could be copy milled easily and (KAR -01)
inexpensively is: 44. Softening heat treatment of alloy increases:
a) Molybdenum b) Germanium a) Tensile strength b) Proportional limit
c) Titanium d) Chromium c) Ductility d) Hardness
(KAR -02) (KAR -98)
36. A highly polished surface on a metallic dental 45, The inlay wax most commonly used for direct
restoration aids considerably in the prevention of technique:
the: a) Type 1 b) Type 2
a) Dimensional change b) Thermal conductivity c) Type 3 d) Type 4
c) Warpage d) Tarnish and corrosion (KAR -2 K)
(KAR -97) 46. Reduction in the fusion temperature of dental gold
37. Austenite is an alloy of iron and carbon with the casting alloys is caused by presence of:
iron in: a) Platinum b) Copper
a) Alpha form b) Beta form c) Silver d) Gold
c) Gamma form d) None of t he above (AIIMS -98)
(KAR -03) 47. The purity of gold is expressed in:
38. The yield strength of Austenite (NiTi alloy) is: a) Ca rats b) Percentage
a) 100 Mpa b) 240 Mpa c) Fineness d) Gram
c) 560 Mpa d) 700 Mpa (KAR -98)
(KAR -01) 48. Function of carnauba wax:
39. Which zone is the hottest and efficient burning a) To provide a smooth and glossy surface
zone of the flame used for casting? b) Increase toughness
a) Oxidizing zone b) Reducing zone c) Decrease toughness
c) Combustion zone d) None of the above d) Decrease flow at room temperature
(KAR -2K) (PGI -2 K)
40. Stabilization of stain less steel is achieved by 49. In a metal ceramic crown for a FPO the function
adding: of tin, indium which are readily oxidized minor
a) Carbon b) Chromium constituents is to:
c) Nickel d) Titanium a) Improve bonding b) Decrease porosity
(KAR -99) c) They react with the porcelain opaque layer, which
41. Localised shrinkage porosity is seen when there: masks the metal
a) Thin sprue b) No reservoir d) Improves the thermal expansion between the
c) Investment heated strongly metal and porcelain
d) Inadequate air discharge (AIPG -01)
(AIIMS -99, PGI -01) 50. The porosity formed due to simultaneous
42. The function of a sprue is to: nucleation of solid grains and gas bubbles at the
a) Form an opening for molten metal to enter the first moment that metal freezes at the mold walls
mold is called:
b) Help polish cast restorations a) Pinhole porosity b) Gas inclusion porosity

34} D 35} C 36} D 37} C 38} C 39} B 40} D 41} A 42} A 43} B 44} C 45) A 46} B
47} C 48} D 49} A 50} D
, DENTAL MATERIALS

c) Micro porosity d) Subsurface porosity. d) Silver cyanide


{KAR- 2003, AIIMS 94) {KAR -02)
51. Investment material used for cobalt-chromium 59. Casting shrinkage is compensated by:
alloys is: a) Thermal expansion b) Hygroscopic expansion
a) Gypsum bonded b) Phosphate bonded c) Wax expansion d) All of the above
c) Silica bonded d) None of the above {KAR -98)
{AIPG -98) 60. Waxes are used for:
52. The standard perforated tray for use with alginate a) Primary impression
impression materials may be brought to more b) Corrective impressions
customized contour by: c) Impression of single tooth
a) Utility wax b) Base plate wax d) None of the above
c) Beading wax d) Boxing wax {AP -01)
(KAR -03) 61. The gold alloys used for casting contain at
53. Silica sol: least•..•.. ••• percent of precious metal according
a) Is applied to the walls of the investment to to ADA specification number 5:
produce hygroscopic expansion a) 55 b) 65
b) Increase the strength and the setting expansion of c) 75 d) 85
the phosphate-bonded investments {APPSC -99)
c) Acts as a retarder in the silica bonded investments 62. Passivating alloys are:
d) Acts as a refract ory in the silica bonded investments a) Cr, Al, Ti b) Cr, Mo, Ti
{AP -99) c) Cr, Gold, Ti d) Cr, Fe, Ti
54. The base constituents of dental waxes come from {KAR -04)
three main sources except: 63. Impurities in casting is removed by:
a) Animal b) Mineral a) Annealing b) Degassing
c) Insect d) Vegetable c) Pickling d) Conditioning
{KAR -02) (PGI -03)
55. Boxing wax is also called as: 64. Which of the following refractory is used to
a) Carding wax b) Inlay wax counteract most of the expansion
c) Utility wax d) Sticky wax a) Quartz b) Cristobalite
{AP -99) c) Tridymite d) Corundum
56. Stainless steel is used mostly in dentistry: (PGI -03)
a) To strength amalgam 65. Shape memory is seen in :
b) In orthodontic wires a) Stain less steel wire b) NITINOL wire
c) To construct clasps for partial dentures c) TMA wire d) Aluminium
d) To replace internal gold restorations (KAR -2K)
(PGI -98) 66. Crucible indicated for casting base metal alloys is:
5 7. Shape memory in NiTi is because of stress induced a) Carbon crucible b) Clay crucible
form c) Quartz crucible
a) Austenite to martensite d) High melting plastic crucible
b) Martensite to austenite {KAR -04)
c) Ferrite to austenite d) Austenite to ferrite 67. The best pickling solution for gypsum bonded
(PGI-2011) investment is:
58. A common electrolyte used for plating copper a) Hydroch loric acid b) Nitric acid
in direct dies is: c) Sulphuric acid d) Phosphoric acid
a) Copper cyanide (COMEDK -04; 05)
b) Ionic copper 68. Heat hardening is the process done at
c) Acidic copper sulfate a) Heating 750°C and slowly cooling to zero degrees

51) B 52) A 53) B 54) C 55) A 56) B 57) A 58) C 59) D 60) B 61) C 62) A 63) C
64) B 65) B 66) C 67) A 68) C
Dental ;lut.,e

b) Heating quickly upto 450°C followed by sudden 7 7. Which of the waxes are derived from the lignite:
quenching a) Paraffin wax b) Inlay wax
c) Heating upto 250°C for 15 to 30 min c) Microcrystalline wax d) Montan wax
d) Heating at 750° C and then quenching to room (AP -99)
temperature 78. Inlay wax is used to:
(PG! -02) a) Temporarily cement inlays
69. Corrosion of stainless steel is due to formulation of: b) Make inlay wax patterns
a) Chromium carbide b) Carbon carbide c) Invest inlay patterns
c) Iron carbide d) Nickel carbide d) Box models
(AP -04) (AIIMS -97)
70. Titanium casting is done: 79. Silver plated dyes use an electrolytic bath of:
a) Under vacuum in argon atmosphere a) Silver fluorides b) Silver cyanide
b) Under air pressme in nitrogen atmosphere c) Silver bromide d) Silver iodide
c) In specially fabricated aluminum vanadium (KAR -98)
crucibles 80. 18-8 Stainless steel orthodontic wire has:
d) Using CAD-CAM technique a) 18% Nickel 8% Chromium
(KAR -04) b) 18% Chromium 8% Nickel
71. The rapid decrease in tensile strength and increase c) 18% Chromium 8% Cobalt
in ductility of a wrought metal is seen in which d) 18% carbon 8% Iron
stage of annealing? (PGI -97, KAR -02, AP- 2013)
a) Recovery stage b) Recrystallisation stage 81. Suck back porosity is due to:
c) Grain growth stage d) All of the above a) Attached impurities on undersurface of casting
(APPSC -99) b) Hot spot
72. Gold content of an 18 carat alloy is: c) No rapid cooling
a) 25% b) 50% d) Inadequate melting temperature
c) 75% d) 100% (AIPG -97)
(PG! -95) 82. Disadvantage of having a short sprue is:
73. Which one of the following categories of dental a) Rapid solidification of metal
materials has the highest thermal conductivity? b) No place for reservoirs
a) Unfilled acrylic plastics c) Incomplete evacuation of gases
b) Gold alloys d) Difficulty in removing casting from investment
c) Porcelains d) Zinc phosphate cements (AIPG -94)
(TNPSC -99) 83. Air bubbles formed on the surface of casting is
74. The sprue in wax pattern should be placed: probably due to:
a) At right angle b) At 45° angle a) Improper casting pressure
c) At obtuse angle b) Improper heating of the investment
d) It depend upon type of wax pattern c) Improper angulations of sprue
(AIPG -04) d) Improper wetting of the wax pattern by the
7 5. Most biocompatible material in oral cavity is: investment
a) Platinum b) Palladium (KAR -97)
c) Titanium d) Gold 84. Who was the first person to introduce cast inlay
(PG! -95) restoration into dentistry?
7 6. Quartz in investment material is added for a) William H Taggart b) Buonocore
a) Strength c) G.V. Black d) Bowen
b) Expansion components for casting shrinkage (COMEDK -04)
c) Decrease strength d) Contraction 85. Asbestos liner is used in a casting ring to:
(AIPG -2K, PGI -99) a) Facilitate venting of the mold

69} A 70) A 71} B 72} C 73} B 74) B 75) C 76) B 77) D 78) B 79} B 80} B 81} B
82} C 83} D 84} A 85} C
, DENTAL MATERIALS

b) Retard the heating of the investment d) Calcium sulphate monohydrate


c) Permit expansion of the mold (AP-05)
d) All the above 94. Which of the following is function of fluoride flux
(AIPG -97) in soldering process?
86. The main ingredient of inlay wax is a) It prevents the formation of copper oxide
a) Candellila wax b) paraffin wax b) It prevents the formation of chromium oxide or
c) carnauba wax d) gum dammar dissolves chromic oxide Layer
(AP 2012) c) It prevents the flow of material in undesirable area
87. Which of the following decrease with strain d) Increases the strength of solder
hardening (AIPG, AIIMS -07)
a) surface hardness b) strength 95. Nickel-Titanium alloy has
c) proportion limit d) corrosion resistance a) unreliable spring back
(PGI-2011) b) low spring back
88. Casting flux is used to c) minimal spring back
a) Prevent the oxidation of the metal during melting d) none of the above
b) Increase the melting point of the flux (BHU-2012)
c) Prevent the contamination of the metal and the 96. The deoxidizing agent added to dental alloys is
liner a) Palladium b) Silver
d) All of the above c) Copper d) Zinc
(COMEDK-05) (COMEDK-07)
89. Suck back porosity in the casting can be eliminated 97. Advantage of titanium over other base metal alloys
by a) Low weight b) Low cost
a) Increasing the mould metal temperature difference c) Low strength d) Low melting point
b) Increasing the flow of molten metal (COMEDK-07)
c) Flaring the point of the sprue attachment 98. Function of wetting agents used in casting
d) None of the above procedure is
(KAR-04) a) To felicitate wetting of ring liner
90. Gases dissolved in molten metals are liberated b) To felicitate mixing investment
when cooled giving rise to c) To reduce contact angle of a liquid with wax
a) Suckback porosity b) Gas inclusion porosity surface
c) Localized shrinkage porosity d) For better wax elimination
d) Microporosity (COMEDK-07, COMEDK-15)
(COMEDK-05) 99. The rate of tarnish and corrosion of class II cast
91. The component in very minor quantity in stainless material having VHN 200 is
steel that changes the properties to a maximum a) 0% b) 2 to 3%
level is c) 5 to 10% d) <1%
a) Carbon b) Molybdenum (KAR-04)
c) Chromium d) Nickel 100. Component of inlay wax which makes it flake
(AP-06) resistant is
92. Percentage of gold in high noble alloy is a) Paraffin b) Carnauba
a) < 25% b) 25% c) Gum dammar d) Canadelila
c) >40% d) 0% (COMEDK-08)
(COMEDK-05) 101. Microstructure of cobalt chromium alloy in cast
93. Binder used in Gypsum investment is condition is
a) Silica a) fibrous b) homogeneous
b) Quartz c) inhomogeneous d) amorphous
c) Cristobolite (BHU-2012)

86) B 87) D 88) A 89) C 90) B 91) A 92) C 93) D 94) B 95) D 96) D 97) A 98) C
99) A 100) C 101) C
Dental ;lut.,e

102. Within an hour after cementation of cast gold on lay c) Homogenization heat treatment
patient complains of shooting pain every time the d) Solution heat treatment
teeth comes together. Most probable explanation is? (KCET-09)
a) Supraocclusion of restoration 110. Which of the following increases castability of
b) Retained cement in gingival sulcus base metal alloys -
c) Excess acid in mix a) Tin b) Beryllium
d) Galvanic current caused by gold onlay occluding c) Molybden um d) Carbon
with a large restoration (KCET-09)
(AIPG-09) 111. With indirect spruing, which one of the following
103. Dendrites in cast alloys form by mechanism of porosities can be minimised -
a) constitutional s1Uperconductivity a) Back pressure porosity
b) constitutional s1Upercooling b) localized shrinkage porosity
c) constitutional s1Upergriting c) Pin hole porosity d) Micro porosity
d) constitutional s1Upergraining (KCET-09)
(BHU-2012) 112. During soldering, the solidus temperature of the
104. Which one of the following elements is NOT added filler material should be -
in small amount to be used as grain refiners? a) Greater than the substrate material
a) Iridium b) Ruthinium b) Lesser than the substrate material
c) Indium d) Platinum c) Equal to the substrate material
(COMEDK-10) d) Temperature not relevant
105. Compared with noble metals, base metals show (COMEDK-09)
a) Increased hardness b) Increased ductility 113. The linear solidification shrinkage ofType IV Ni-Cr
c) Better biocompatibility based alloys is -
d) Increased density a) 2.3% b) 1.56%
(COMEDK-10) c) 1.37% d) 3%
106. Binder in Phosphate bonded investment is (COMEDK-09)
a) Magnesium oxide & Phosphate 114. The solvent flux used for base metal soldering is -
b) Magnesium oxide & Phosphoric acid a) Boric acid b) Borax
c) Magnesium oxide & Monoam monium phosphate c) Fluoride d) Rouge
d) Magnesium oxide & Ortho Phosphoric acid (COMEDK-09)
(COMEDK-10) 115. Which of the following cast gold alloys is used for
107. Mechanical properties of Cp Titanium are similar fabricating partial denture frameworks -
to alloys a) Type I - Soft b) Type II - Hard
a) type III and type IV gold c) Type III - Hard d) Type IV - Extra Hard
b) type V and type VI gold (KCET-09)
c) type VII and type VIII gold 116. In base metal alloys, corrosion resistance occurs
d) none of the above due to
(BHU-2012) a) Passivity b) High fusion temperature
108. Which of the following fuel gases used for c) High molecular weight
soldering, has the highest heat content - d) Co-efficient of thermal expansion
a) Hydrogen b) Natural gas (AP-10)
c) Acetylene d) Propane 117. Modern dental noble metal casting alloys generally
(COMEDK-09) have equiaxed fine grain structures because of the
109. The effects associated with cold working can be incorporation of small amounts of
eliminated by a heat treatment called - a) Chromium b) Indium
a) Age hardening heat treatment c) Iridium d) Palladium
b) Annealing heat treatment (KCET 2012)

102) D 103) B 104) D 105) A 106) C 107) A 108) D 109) B 110) B 111) B 112) B 113) A 114) C
115) D 116) A 117) C
, DENTAL MATERIALS

118. The castability of chromium-cobalt base metal 127. Linear coefficient of thermal expansion of metal
alloys when compared to gold based noble metal used for metal ceramic restoration range from
alloys is a) 11.5-12.5x10-6/°C
a) More b) Less b) 10.5-11.5x10-6/°C
c) Equal d) Depends on the alloy type c) 13.5-14.5x10-6/°C
(AP-10) d) 16.5-17.5x10-6/°C
119. Nickel is added to wrought gold alloys as - (COMEDK-2013)
a) Strengthener b) Hardener 128. Annealing is also known as:
c) Whitens the alloy d) Scavenger a) Soft hardening treatment
(AIIMS-09) b) Hard hardening treatment
120. Fins and spines on the casting is seen due to c) Precipitating heat treatment
a) Prolonged heating b) Rapid heating rate d) Stress relieving heat treatment
c) Water film contamination (AIIMS-2012)
d) Too high pressure during casting 129. Quartz is added to gypsum bonded investment to
(AIPG-10) a) Increase the strength
121. Stages of annealing in order b) counter balance contraction of gypsum during
a) Recovery recrystallization, grain growth heating
b) Recrystallization recovery, grain growth c) aid in hygroscopic expansion
c) Grain growth, Recrystallization, recovery d) increase the shelf life of investment
d) Melting point, Grain growth, Recrystallization (AP-2013)
(AIPG-2011) 130. Chromium is used in stainless steel for which
122. Solder and flux used for joining Elgiloy wires are purpose?
a) Gold solder and borax flux a) Corrosion resistance
b) Gold solder and fluoride flux b) Increases the lustre of stainless steel
c) Silver solder and borax flux c) Stabilization of the alloy
d) Silver solder and fluoride flux d) Increases the hardness of steel
(AIPG-2011) (AIIMS-13)
123. Dr. Taggart is associated with 131. The best bio-compatible dental casting alloy is
a) Discovery of Ceramics a) Co-Cr b) Ni-Cr
b) Lost wax tee hnique c) Titanium alloy d) Gold
c) Acid etch technique (COMEDK-14)
d) Bleaching procedures 132. Which of the following characteristics of inlay wax
(COMED-2012) is its major disadvantage?
124. Which of the following fuel gases is the best choice a) Flow b) Rigidity
of heat source to carry out soldering process? c) High thermal expansion
a) Propane b) Acetylene d) Low thermal conductivity
c) Natural gas d) Hydrogen (AP-14)
(COMED-2012, KAR- 2013) 133. Which among these is not an ideal property of
125. The term ordered hardening heat treatment used soldering material?
in which of the following dental alloys? a) Corrosion resistant b) Non-pitting
a) Gold-Copper b) Nickel-Chromium c) High fusing d) Free flowing
c) Nickel-Titanium d) Silver-Palladium (GCET-14)
(KCET-12) 134. Which of the following is not used as antiflux?
126. The best fuel gas when used with oxygen is a) Graphite b) Iron oxide
a) Hydrogen b) Natural gas c) Boric oxide
c) Propane d) Acetylene d) Calcium carbonate dissolved alcohol
(COMED-2012) (APPG-14)

118) B 119) A 120) B 121) A 122) D 123) B 124) A 125) A 126) C 127) C 128) A 129) B 130) A
131) C 132) C 133) C 134) C
Dental ;lut.,e

135. Which of the following statements is not true


about use of silica in the investment materials?
a) It acts as refractory
b) It regulates thermal expansion
c) Quartz and cristobalite are the allotropic form
used in dental investments
d) Beta allotropic form is stable below transition
temperature
(PGI JUNE-2014)
136. Which of the following is a solid solution
a) Amalgam b) Cobalt-chromium
c) Silver-palladium d) Gallium-silver
(AIIMS MAY-14)
137. Laser welding
a) Involves high heat generation
b) Can be done with pure Titanium
c) Is not used in dentistry
d) Can be done with pure gold
(COMEDK-15)
138. Which is false regarding ethyl silicate bonded
investment?
a) Silica gel which reverts to silica on heating
b) Magnesium oxide provides strength
c) It is easy to handle and most commonly used
d) On mixing ammonium chloride silica liquid
converts to gel
(PGI DEC-2013)
139. The component responsible for allergy in dental
casting alloys is
a) Cobalt b) Nickel
c) Copper d) Titanium
(KERALA-2015)
140. Chromium added to steel does not
a) Increase hardness b) Increase tensile strength
c) Increase corrosion resistance
d) Decrease elastic limit
(PGI DEC-2013)
141. The casting shrinkage in gold alloys is maximum in
a) Class II MOD inlays b) Large class V restoration
c) Class II MO Restoration
d) Class 1 inlay
(COMEDK-15)
142. According to OSHA maximum permissible amount
of Ni in dental clinic environment is?
a) 1 mg b) 2 mg
c) 3 mg d) 4 mg
(PGI JUNE-2011)

135) D 136) C 137) B 138) C 139) B 140) D 141) B 142) A


, DENTAL MATERIALS

6. METALLURGY - ANSWERS
1. 'C' [PHILLIPS' 10th ed 451/ 12th ed 383] 6. 'A' [PHILLIPS' 11th ed 323/ 12th ed 215]
Base metal alloys (Ni-Cr, Co-Cr) have high hardness, A minimum gap of 1/4 inch or 6mm between the ends
modulus of elasticity, sag resistance, yield strength, of casting ring and wax pattern will provide pathways
high fusion temperature and casting shrinkage for gas escape during casting.
(2.25%). They have lower proportional limit and
ductility than gold alloys. 7. 'B' [PHILLIPS' 11th ed 575/ 12th ed 374]
Type III and Type IV gold alloys can be hardened by heat
Pure gold is most ductile and most malleable of all the treatment if they contain sufficient amount of copper.
metals. Type I and II cannot be hardened by heat treatment.

2. 'B' [PHILLIPS' 11th ed 5 77 / 12th ed 386 Tab 16-14] 8. 'C' [ PHILLIPS' 11th ed 292/ 12th ed 200]
Solidification sh ri11kage of gold alloys is 1.25-1. 65% Beading of the impression preserves sulcus, whereas
whereas for base metal alloys, it is 2-3%. boxing helps in ease of pouring the impression and
obtaining proper impression dimensions.
3. 'C' [Check Explanation Below] TYPE OF WAX USES
Platinum has high fusion temperature and is one of
Boxing/ Carding To box the impressions before
the eight noble metals. It whitens the alloy and is
wax the plaster or stone is poured.
resistant to tarnish and corrosion. On the other hand,
To attach metal parts during
addition of large amounts of copper decreases fusion Sticky wax
investment soldering
tern peratu re.
Bite or corrective To make impressions of
4. 'D' [Check Explanation Below] wax/ Impression edentulous ridge
Option 'B' Boxing wax is used to build up vertical wax.
walls around the impression to produce desired size Utility wax/ To give contour to impression
and form of base of cast. Option 'K Type II inlay wax Periphery wax trays for use with alginate.
(soft wax) is used in indirect impression technique for To make wax patterns for
Inlay wax
inlays and crowns. in lays.
Carving wax or Used for demonstration
5. 'D' [PHILLIPS' 11th ed 581/ 12th ed 377] demonstration wax purposes.
Ceramics have high melting point and low coefficient
of thermal expansion. 9. 'C' [PHILLIPS' 11th ed 296/ 12th ed 201]
Gypsum serves as a binder and increases the strength
The alloys used for metal ceramic restorations of the investment. The investment contains 25-45%
should have: gypsum.
• High fusing
• Their coefficient of thermal expansion should Silica acts as a refractory during heating and controls
match with ceramics the setting expansion.
• Should make bond with ceramic
Carbon, copper acts as reducing agents while boric
acid, NaCl regulates the setting time and expansion.
All are true for veneering with porcelain except
a) High coefficient of t hermal expansion th an
10. 'B' [PHILLIPS' 11th ed 568/ 12th ed 376]
metal
Aluminum, chromium, titanium have high affinity
b) High sag resistance (PGI June-13)
for oxygen and they form protective oxide film which
c) Solidus temperature of metal should be equal or
prevents further oxidation and corrosion. This is
lower than ceramic
called passivation . The most corrosion resistant
d) Metal should chemically bond with the ceramic
element is titanium.
Dental ;lut.,e

Co-Cr, Ni-Cr, Ni-Co-Cr alloys form chromium oxide on setting expansion is adding modifiers like boric acid
their surfaces which makes them corrosion resistant. and chlorides of Na, K, Li. These substances decrease
They should contain minimum of 12% chromium to gypsum shrinkage during the process of heating.
exhibit passivity.
20. 'B' [PHILLIPS' 11th ed 596]
11. 'C' [PHILLIPS' 11th ed 638/ 12th ed 4 07]
Austenitic stain less steel or 18-8 stain less steel 21. 'D' [PHILLIPS' 11th ed 646/ 12th ed 4 09]
consists of 18% chromium, 8% nickel and 0.15%
carbon. Austenite is the main form of iron carbide 2 2. 'A' [PHILLIPS' 11th ed 297/ 12th ed 204]
and provides resistance to corrosion. Above 700°C, there is decomposition of gypsum
investment with release of sulph ur dioxide. The
12. 'A' [PHILLIPS' 11th ed 564/ 12th ed 222] sulphur dioxide reacts with silver and copper to form
Flux prevents oxidation of mo lten gold alloy and their sulphides and thus contaminates the casting.
also aid in minimising porosity of gold alloys. Borax
powder ground with equal parts of boric acid powder 23. 'D' [PHILLIPS' 11th ed 337/ 12th ed 222]
is commonly used as flux for the casting procedures of Acid should never be used for cleaning base metal
gold alloys. Flouride fluxes are used for casting base alloys.
metal alloys.
24. 'C' [PHILLIPS' 11th ed 348/ 12th ed 227)
13. 'A' [PHILLIPS' 11th ed 648/ 12th ed 4 10] Incomplete casting with rounded margins and shiny
Nickel titanium alloys have shape memory and super appearance occurs due to incomplete wax elimination.
elasticity. The super elastic properties are due to This shiny condition is caused by carbon monoxide
t ransition of austenite to martensite (AP - 2013) left by residual wax.
by stress. These alloys are used in orthodontic wires
and endodontic fi les. 25. 'A' [PHILLIPS' 11th ed 334/ 12th ed 221, 392)
First zone of flame No heat
14. 'C' [PHILLIPS' 11th ed 636/ 12th ed 406]
Steels are iron-based alloys that contain carbon Combustion zone • Definitely oxidizing
content less than 1.2%. (green) • Not used for melting alloys.
Reducing zone • Hottest pa rt.
15. 'C' [PHILLIPS' 11th ed 3 35/ 12th ed 222] (blue) • This zone is used for melting
The surface of the casting appears dark with oxides alloy.
and tarnish. Pickling is the process of removing such
Oxidizing zone Not used
a surface film. 50% Hcl (NEET- 2013) , H2So4 are
generally used as pickling solutions.
26. 'B' [PHILLIPS' 11th ed 347/ 12th ed 227]
Incomplete casting is due to incomplete filling of
50% HCl is the best pickling solution for gypsum
mold by molten alloy. The common causes, which
bonded investments.
inhibit the ingress of the liquefied metal, are
16. 'C' [PHILLIPS' 12th ed 199]
• Insufficient venting - If the air cannot be vented
17. 'B' [PHILLIPS' 11th ed 640/ 12th ed 4 08] quickly, the molten alloy does not fill the mold
Cold working or work hardening hardens stainless before it solidifies.
steel. • High viscosity of metal due to insufficient heating.
• Incomplete elimination of wax residues from the
18. 'A' [PHILLIPS' 11th ed 649/ 12th ed 4 11] mold. If too many products of combustion remain in
the mold, the pores in the investment may become
19. 'B' [PHILLIPS' 11th ed 302, 306/ 12th ed 18 7) fi lled so that the air cannot be vented completely.
Increased water powder ratio or thinner mix decreases • Narrow sprue diameter, etc.
the setting expansion. The best way to increase
, DENTAL MATERIALS

27. 'B' [PHILLIPS' 11th ed 34 6/ 12th ed 227) Ceramics and metals like titanium, which are difficult
Back pressure potrosity is caused by the entrapped air to cast, are fabricated by CAD-CAM process and copy
in the mould that cannot escape through the pores of milling.
the investment (low investment permeability). This
porosity is seen as large voids on t he inner surface 36. 'D'
of casting. It is prevented by using high casting
pressure, proper burnout technique and ensuring that 37. 'C'
the thickness of investment between end of pattern
and casting ring is not more than 1/4 inch (6mm). 38. 'C' [PHILLIPS' 11th ed 646/ 12th ed 40 7]
The austenite phase of Ni-Ti alloys undergoes
28. 'A' [Check Explanation Below] transformation to martensite when exposed to stress
IOWA wax, KORECTA wax NO. 4, H - LPhysiologic Paste, conditions. Super elasticity is exhibited by Ni-Ti wires
and ADAPTOL are examples of mouth temperature or files in martensite phase.
waxes. These waxes are designed to flow at mouth
temperature. These are used in fluid wax technique of 39. 'B' [By Synopsis]
recording posterior palatal seal.
40. 'D' [PHILLIPS' 11th ed 579, 640/ 12th ed 407)
29. 'C' [PHILLIPS' 11th ed 296/ 12th ed 203) Stain less steel is resistant to tarnish and corrosion due
Investment should expand to compensate for the to chromium oxide layer formation (passivation). But
casting shrinkage. when 18-8 stainless st eels are heated to 400-900°(,
chromium combines with carbon and forms chromium
Expansion of gypsum investments : carbide. As a result, the corrosion resistance of steel
• Normal setting expansion - 0.3-0.5% is reduced. This is called sensitization.
• Hygroscopic expansion 1.2-2%
To avoid this, titanium is added to stain less steel as
• Thermal expansion 1-2%
titanium is readily precipitated as titanium carbide in
preference to chromium. This is called stabilization
30. 'A' [PHILLIPS' 11t h ed 598/ 12th ed 408)
of stainless steel.
Tempering is a process of heat treating, which 1s
used to increase the toughness of iron-based alloys. 4 1. 'A' [PHILLIPS' 11th ed 34 2, 320/ 12th ed 226)
Tempering reduces the ductility of the meta ls.
42. 'A' [PHILLIPS' 11th ed 319/ 12th ed 21 3)
31. 'C' [PHILLIPS' 11th ed 586/ 12th ed 382)
Ti n oxide and indi um oxide improves the bonding and 4 3. 'B' [PHILLIPS' 11th ed 346/ 12th ed 226)
hardens Ag-Pd alloys. Sub surface porosity is due to rapid entry of molten
metal into the mold. Sub surface porosity can be
32. 'D' [PHILLIPS' 11th ed 59 7/ 12th ed 375]
reduced by increasing the sprue length and by reducing
the sprue thickness and mould metal temperature.
33. 'C' [PHILLIPS' 11th ed 598/ 12th ed 383]
4 4. 'C' [PHILLIPS' 11th ed 576/ 12th ed 374]
34. 'D' [PHILLIPS' 11th ed 3 21/ 12th ed 214]
Softening heat treatment dlecreases the mechanical
Turbulence of molten gold results when the sprue properties of alloys but increases ductility.
former is attached at right angles. Turbulence can
be prevented by directing t he sprue at 45° to the Hardening heat treatment (Age hardening) increases
proximal area of wax pattern and by attaching the hardness, yield strength, proportional limit but
sprue former at the point of greatest bulk in the decreases ductility.
pattern.
4 5. 'A' [PHILLIPS' 11th ed 284/ 12th ed 195)
35. 'C' [PHILLIPS' 11th ed 567/ 12th ed 386] In 'direct technique' of preparation of wax pattern,
Type I or medium or Type B wax is used whereas in
Dental ;lut.,e

'indirect technique' Type II or soft or Type C wax is The most important plant wax is Carna uba wax. The
used. examples of synthetic waxes are paraffin wax (derived
from petroleu m or coal) and Montan wax (coal and
46. 'B' [PHILLIPS' 10th ed 438] lignite) .
Copper decreases the fusion temperature but increases
the hardness and corrosion resistance. 55. 'A'

47. 'C' [PHILLIPS' 11th ed 574/ 12th ed 368] 56. 'B' [PHILLIPS' 11th ed 636/ 12th ed 407]
Fineness is more practical that karat. Karat is parts of Stainless steel (Fe-Cr-Ni), Elgiloy (Co-Cr-Ni),
pure gold in 24 parts of alloy. Fineness is the number ~-Titanium (TMA) and Nickel titanium alloys can be
of parts per 1000 of gold. For example, pure gold has used for orthodontic wires.
a fineness of 1000, and a 650 fine alloy has a gold
content of 65%. 5 7. 'A' [ Philips 11th ed 6 48/ 12th ed 407]

48. ' D' [PHILLIPS' 11th ed 286/ 12th ed 196] 58. 'C' [PHILLIPS' 11th ed 318/ 12th ed 213]
Carnauba wax decreases flow at mouth temperature
and also contributes to glossiness. 59. 'D' [PHILLIPS' 11th ed 327/ 12th ed 211]

49. 'A' [PHILLIPS' 11th ed 586] 60. 'B' [PHILLIPS' 11th ed 292/ 12th ed 200]
Bite or corrective waxes are t he impression waxes and
50. ' D' [PHILLIPS' 11th ed 346/ 12th ed 226] are limited to edentulous areas.

51. ' B' [PHILLIPS' 11th ed 310/ 12th ed 382] 61. 'C' [PHILLIPS' 12th ed 369]
Base metal alloys, high melting gold alloys, which
are used for metal ceramic restorations, are invested 62. 'A' [PHILLIPS' 11th ed 568/ 12th ed 375]
by phosphate-bonded investments.
63. 'C' [PHILLIPS' 11th ed 335/ 12th ed 222]
Option 'K gypsum-bonded investments are used
for casting gold alloys. These can withstand a 64. 'B' [PHILLIPS' 11th ed 298/ 12th ed 202]
tern perature of 700 °C. Quartz, tridymite and crystobolite are allotropic forms
of silica. When quartz and crystobolite are heated they
Option 'C' silica bonded investments are an invert from low form (alpha) to high fo rm (beta). This
alternative to the phosphate-bonded investments, beta form exhibits rapid expansion . As the inversion
for high temperature casting. They are used in the temperature of crystobolite is less than quartz it is
casting of Base Metal Alloy Partial Dentures. the best choice as a refractory material.

Which investment material is used in the 65. 'B' [PHILLIPS' 11th ed 648/ 12th ed 407]
construction of high fusing base metal partial
denture alloys? (MHCET-15) 66. 'C' [PHILLIPS' 11th ed 33 3]
a) Gypsum bonded b) Phosphate bonded Quartz crucibles are suitable for fusing the alloys
c) Silicate bonded d) Resin bonded which are sensitive to carbon such as palladium and
base metal alloys. Carbon crucibles are for high -fusing
52. 'A' gold alloys and clay crucibles are suitable for high
noble alloys.
53. 'B' [PHILLIPS' 11th ed 311/ 12th ed 210]
67. 'A' (PHILLIPS' 11th ed 335/ 12th e d 221)
54. 'C' [PHILLIPS' 11th ed 285/ 12th ed 196]
Waxes are biosynthesized by many plants and animals. 68. 'C' [PHILLIPS' 11th ed 576/ 12th ed 374]
The most common ly known animal wax is bees wax. Option 'D' is softening heat treatment.
, DENTAL MATERIALS

69. 'A' [PHILLIPS' 11th ed 639/ 12th ed 407) 87. 'D' [Philips 11th ed 88/ 12th ed 404]
By excluding the effects of cold working only ans 'D'
70. 'A' [PHILLIPS' 11th ed 579/ 12th ed 222) is left and it is the correct answer.
Titani um has hi91h melting point (1668°() and a
special casting machine with arc melting capability 88. 'A' [PHILLIPS' 11th ed 335/ 12th ed 222)
and an argon atmosphere is typically used.
89. 'C' [PHILLIPS' 11th ed 344/ 12th ed 226)
71. 'B' [PHILLIPS' 11th ed 634/ 12th ed 405)
90. 'B' [PHILLIPS' 11th ed 344/ 12th ed 227)
72. 'C' [PHILLIPS' 11th ed 574/ 12th ed 368) See synopsis "Porosity"

73. 'B' [PHILLIPS' 12th ed 39) 91. 'A'

74. 'B' [PHILLIPS' 11th ed 321/ 12th ed 214) 92. 'C' [PHILLIPS' 11th ed 570 Box 19-1/ 12th ed 369
The spruce is placed at 45° angle to the wax pattern. Tab 16-1)
High noble
75. 'C' [PHILLIPS' 11th ed 579/ 12th ed 376) Must contain > 40 wt% of gold
alloys

76. 'B' [PHILLIPS' 11th ed 297 / 12th ed 201) Predominantly Contain < 25 wt% of noble metal
Base metal elements
77. 'D' [Kirk-0th mer Food and Feed Technology volume Must contain >25 wt% of noble
2 Pg 627) Noble alloys metal elements
Montan wax, also known as lignite wax or OP wax, is (Au, Pt, Pd, Rh, Rh, Os, Ir)
a hard wax obtained by solvent extraction of certain
types of lignite or brown coa l. 93. 'D' [PHILLIPS' 11th ed 297/ 12th ed 201)

78. 'B' [PHILLIPS' 11th ed 284/ 12th ed 195) 94. 'B' [PHILLIPS' 11th ed 621/ 12th ed 389)

79. 'B' [PHILLIPS' 11th ed 319/ 12th ed 213) 95. 'D' (Philips 11th ed 646/ 12th ed 409)
Ni-Ti alloys have high spring back action.
80. 'B' [PHILLIPS' 11th ed 638/ 12th ed 407)
96. 'D' [PHILLIPS' 10th ed 365, 366/ 12th ed 375)
81. 'B' [PHILLIPS' 11th ed 343/ 12th ed 226)
97. 'A' (PHILLIPS' 11th ed 579/ 12th ed 376)
82. 'C' [PHILLIPS' 11th ed 322/ 12th ed 226) Important properties of Titanium
• Most biocompatible
83. 'D' [PHILLIPS' 11th ed 338/ 12th ed 227)
• Low density (low weight)
84. 'K [PHILLIPS' 11th ed 565/ 12th ed 368) • Highest melting point

85. 'C' [PHILLIPS' 11th ed 323/ 12th ed 215) 98. 'C' [PHILLIPS' 11th ed 339/ 12th ed 216]
Use of split casting ring or flexible rubber ring or ring A wetting agent applied in a thin layer helps in proper
liners like cellulose and aluminum silicate ceramic wetting by the wax to the investment.
liner helps in expansion of the mold and reduces the
effect of confinement. Because of its carcinogenic 99. 'A' [PHILLIPS' 11th ed 342]
potential, asbestos is not used as ring liner.

86. 'B' (Philips 11th ed 285/ 12th ed 196)


Paraffin wax is the main ingredient of inlay waxes,
usua lly in concentration of 40-60% wt%.
Dental ;lut.,e

100. 'C' [PHILLIPS' 11th ed 286/ 12th ed 196) nitrogen. The properties are similar to type III and
type IV gold alloys.
COMPOSITION OF WAXES
• Basic constituent
108. 'D' [PHILLIPS' 11th ed 613/ 12th ed 391)
Paraffin • Have tendency to flake Of the readily available gases, both Propane and
Butane have the good flame temperature and the
• Less smooth and glossy
highest heat content.
• Improves smoothness
Gum dammar
• Decreases flaking 109. 'B' [PHILLIPS' 11th ed 576/ 12th ed 404)
Carnuaba/ • To decrease flow
110. 'B' [PHILLIPS' 11th ed 596/ 12th ed 375)
Candelilla • Imparts glossy surface
Ceresin May replace part of paraffin 111. 'B' (PHILLIPS' 11th ed 343/ 12th ed 225]

101. 'C' [Philips 11th ed 112/ 12th ed 76) 112. 'B' [PHILLIPS' 11th ed 610/ 12th ed 389)
The microstructure of any cast dental alloys consists
of heterogeneous nucleation. 113. 'A' [PHILLIPS' 11th ed 577, Tab. 19-2/ 12th ed 386
Tab 16-14]
102. 'D' [PHILLIPS' 11th ed 61/ 12th ed 43)
Linear solidification shrinkage of casting alloys
103. 'B' [Philips 11th ed 113) Alloy type Casting shrinkage (%)
• Super cooli ng is the cooling of liquid (Molten)
Type I (Au-based) 1.56
metal from its fusion temperature to a point at
which the temperature remains constant. Type II (Au-based) 1.32
• Dendrites in pure metals grow by the mechanism
Type III (Au-based) 1.42
of thermal super cooling, whereas dendrites in
cast alloys form by constitutional super cooling. Type IV (Ni-Cr based) 2.30

104. 'D' [PHILLIPS' 11th ed 116/ 12th ed 79) Type IV (Co-Cr based) 2.30
Grain refinement is the process of reducing grain size
in solid metal by adding an element or compound to 114. 'C' [PHILLIPS' 11th ed 609/ 12th ed 222)
the molten metal and cooling at a prescribed rate. Fluxes for noble metal alloys are based on boric or
borate compounds such as boric acid, boric anhydrate
Eg.: Iridium, ruthenium or rhenium in less than 1 wt% and borax. These are called as Type I fluxes. Copper
for palladium alloys and much smaller for gold alloys. oxide acts as type II flux. Type III is Fluoride flux and
is used for base metal alloys.
105. 'A' (PHILLIPS' 11th ed 599/ 12th ed 375)
Compared with noble metal alloys, base metal alloys 115. 'D' [PHILLIPS' 11th ed 571/ 12th ed 369)
have higher hardness, stiffness, strength and sag
resistance but low density, ductility and castability. 116. 'A' [PHILLIPS' 11th ed 579/ 12th ed 376)

106. 'C' [PHILLIPS' 11th ed 310/ 12th ed 207) 117. 'C' [Philips 11th ed 116/ 12th ed 79)
The term equiaxed means that three dimensions of
107. 'A' [Philips 11th ed 579/ 12th ed 376) each grain are similar.
Commercially pure titanium (CPT) can be used for all
melal and melal ceramic pruslheses as well as for Grains in dendritic structure are elongated. Dendritic
implants and removable partial denture frameworks. micro structures are not desirable for cast dental
There are five unalloyed grades of CPT; (grades 1-4 alloys because the inter dendritic regions can serve as
and grade T) based on concentration of oxygen and sites for crack propagation.
, DENTAL MATERIALS

Addition of about 1 wt% of Iridium, ruthenium or • Temperature varies in one part of the flame to
rhenium as grain refi ners makes modern dental noble another
metal casting alloys as equiaxed microstructures.
12 5. 'A' [Philips 11th ed 5 7 5/ 12th ed 3 7 4]
118. 'B' [PHILLIPS' 11th ed 599/ 12th ed 382] Hardening heat treatment can be done in gold alloys
if they contain sufficient amount of copper.
119. 'A' [PHILLIPS' 10th ed 652]
Gold alloy wires contain Nickel. Nickel provides 12 6. 'C' [Philips 11th ed 613/ 12th ed 391]
additional strengthening, but the amount is limited to
avoid reduction in tarnish resista nee and interference 12 7. 'C' [Phillips 11th ed 55, 581]
with age hardening. The high noble and noble alloys used for metal ceramic
prostheses possess co-efficient of thermal expansion
120. 'B' (PHILLIPS' 10th ed 515/ 12th ed 224) from 13.5-14.8 x10- 6 j°C
Rapid heating rates results in fins or spines on the
casting or a characteristic surface roughness may be 128. 'A' [Phillips 11th ed 576/ 12th ed 404]
evident because of flasking of the investment when Steel is softened by annealing and hardened by cold
the water or stea m pours into mould. working.
The mould should be heated gradually; atleast 60
minutes should e lapse during the heating of the Annealing is the controlled heating and cooling
investment fi lled ring from room temperat ure to 70°C. process designed to produce desired properties in
a metal. This process usually is intended to soften
121. 'A' [Phillips 11th ed 633/ 12th ed 404] metals, to increase their plastic deformation potential,
The effects of cold working (strain hardening) are to stabilize shape and to increase machinability and
decreased ductility and distorted grains. This can be also to relieve stresses. Hot working, Quenching and
reversed simply by healing the metal. This process stress-relief are the other names of annealing .
is called annealing. Annealing takes place in 3
successive stages recovery, recrystallization and grain Note: Annealing in direct filling gold is to eliminate
growth. contaminants from the surface.

122. 'D' [Phillips 11th ed 645/ 12th ed 409] 129. 'B' [Phillips 11th ed 297 / 12th ed 201]
Elgiloy is alloy of cobalt (40%), chromium (20%) and
nickel (15%) used for orthodontic appliances and 130. 'A' [Check Explanation of Q. No.40]
watch springs.The solder and flux for joining elgiloy
are same as st ainless steel i.e. 131. 'C' [Phillips 12th ed 376]
• Solder - silver solder Titanium and titanium alloys are considered the most
biocompatible metals used! for dental restorations
• Flux - fluoride
produced with prostheses.
123. 'B' [Philips 11th ed 565/ 12th ed 368]
The CPTi (commercially pure titanium) is the most
124. 'A' [Philips 11th ed 613/ 12th ed 391]
widely used titanium alloy in dentistry in form of
Ti-6Al-4V, which is an alpha - beta alloy. Vanadium
Propane gas:
has some health concern due to its toxicity. Now
• Best choice fuel gas vanadium replaced with Nobium alloy and is used in
• Highest heat content form of Ti-6Al-7Nb.
• Butane also has the same characteristics
132. 'C' (Philips 12th ed 197)
Acetylene gas: Melting range, viscosity, flow, adaptability, carvability
• Highest flame tern perature and burnout properties are the desirable properties of
inlay wax.
• Chemically unstable gas
Dental ;lut.,e

High co-efficient of thermal expansion and tendency Laser welding in an argon gas atmosphere is a
to warp are the major disadvantages method of choice for joining titanium components
of dental prostheses, because the coherent, high
133. 'C' [Check Synopsis] intensity laser pulse of light can be focused in a small
The fusion tern perature of solder should be atleast region and melt the substrate without much damage
56°C lower than the solidus temperature of the to the surrounding areas. Also the joint will be
substrate (parent) metal. composed of the same titanium, thereby preserving
biocompatibi lity and avoiding galvanic corrosion.
134. 'C' [Phillips 11th ed 564]
Graphite is used commonly as antiflux. It is used 138. 'C' [Phillips 1st SA ed 219]
to limit the spreading of solder and is applied on a Ethyl silicate bonded investment:
casting before the flux application to limit the flow of • Use of this investment is declined because of the
molten metal. more complex and time consuming procedure.
• Binder is the silica gel which reverse to silica
135. 'D' [Check Q. No.9 and 64 ]
(crystobalite) on heating.
Silica exists in four alltropic forms
1) Quartz • Several methods may be used to produce silica or
silicic acid gel binders.
2) Tridymite
3) Crystobalite • Addition of magnesium oxide strengthens the gel.
4) Fused quartz • An aqueous suspension of colloidal silica can
also be converted to a gel by the addition of an
When these allotropic forms are heated, a change in accelerator, such as ammonium chloride.
crystalline form occurs at a transition temperature • Other system for binder formation is based on
characteristic of the particular form of Silica. et hyl silicate.
Inversion temperature of Quartz (alpha) to quartz
(beta) is 575°C; crystobalite (alpha) to crystobalite 139. 'B' [Phillips 1st SA ed 13 2]
(beta) is 200-270°C and tridymite is 117-1 63°(. Nickel is the most allergenic metal known, with an
Beta allotropic form is stable only above transition incidence of allergic reactions between 10-20%.
temperature. Inversion back to lower alpha form
occurs on cooling in each case. 140. 'D' [Check Explanation Below]
Effects of adding chromium to steel:
136. 'C' [Check Q.No.51/ Phillips 1st SA ed 81]
• Increases the ha rdenability of steel
Majority of noble metal casting alloys for dental
restorations are based on solid solutions. Important • Ductility is minimally effected
example is the palladium-silver alloy system. When • Corrosion resistance
this alloy is solidaified, the silver atoms are distributed • Tensile strength increases by 8-100 N/mm2 for
randomly in the face centred cubic structure of every 1% chrome added.
pure palladium, thereby forming a solid solution. • Yield strength increases
No microstructural features can be mechanically
• Elastic limit and elastic ratio increases with no
separated into two or more phases, so only one solid
effect on its toughness.
phase is present.
Carbon added to steel:
137. 'B' [Phillips 1st SA 418]
Commercially pure titanium cannot be soldered with • Strength, hardness increases
a traditional torch-soldering procedure because it • Ductility, machinability reducers
is a highly reactive metal that forms oxides during
soldering procedure and may spontaneously debond Manganese added to steel:
from parent metal surface. • Increases strength but to a lesser degree than
carbon
, DENTAL MATERIALS

• Favourably influences the weldability and


machinability

141. 'B' [Dental materials and their selection by Joseph


0 Brien 3rd ed 240]
In direct method wax shrinks by about 0.4% when
cooled from oral temperature. In indirect method,
wax shrinks about 2%.

Casting shrinkage takes place as the solidified metal


cools to room temperature. The highest shrinkage of
about 2% occurs for large class 5 restoration.

Shrinkage (Wax+alloy) should be compensated by


investment expansion . The more the shrinkage the
more is the requirement of expansion of investment.

Restoration Expansion (%)


Class -1 and Class -2 1.83
Large HOB Three quarters Crown 1.90
Full Crown Small Class 5 2.10
Large Class 5 2.40

142. 'f,{ [Eighth Annual report on carcinogens 148]


OSHA means occupationa l safety and health
administration. OSHA adopted permissible exposure
Limit (PEL) of 1 mg/m 3 as an 8 hour time-weighted
average for nickel metal and 0.007 mg/m3 for nickel
carbonyl.
Dental ;lut.,e

7. DIRECT FILLING GOLD


The cohesive gold used as bulk filler is: 10. Powdered gold is:
a) Electra alloy b) Mat gold a) Made by a combination of chemical precipitation
c) Spherical gold d) Gold foil and atomization
(MAN -01) b) Made by electrolytic precipitation
2. The carat for Au foil used for restorations is: c) Made by beating pure gold at high temperatures
a) 18 b) 20 d) Made by milling pure gold ingots
c) 22 d) 24 (KCET-07)
(AIIMS -97) 11. In type II gold alloys _ _ _ Karat gold is used:
3. Electralloy is an alloy of: a) 24 K b) 22 K
a) Gold and Platinum b) Cobalt and Chromium c) 18 K d) 10 K
c) Gold and Calcium d) Tin and Chromium (AP-06)
(KAR -97, APPG- 15) 12. When Gold fused with porcelain is fired at very
4. Pitting of direct filling gold restoration usually high temperature, greenish discoloration of
results from: cervical margin is due to
a) Using contaminated gold foil a) High firing temperature
b) Poor finished preparation b) Disintegration of opaque layer
c) Improper stepping of condenser point c) Presence of copper d) Formation of excess oxides
d) Using small condenser point (AP-06)
(KAR -98) 13. White gold alloy predominantly is
5. Average particle size of powdered gold is: a) Gold added with palladium
a) 10 um b) 15 um b) Gold added with calcium
c) 30 um d) 0.1 mm c) Gold with much of chromium
(KAR -98, PG! -94) d) Gold with silver
6. The malleting force is directed: (AP-06)
a) Perpendicular to the walls 14. Direct filling gold formed by electrolytic
b) Parallel to long axis of crown precipitation is called by all the terms EXCEPT
c) 45°to the wall d) None of the above a) Crystalline b) Granular gold
(KAR -98) c) Mat gold d) Sponge gold
7. Cohesion of gold foil is an example of: (AIPG-2011)
a) Atomic attraction and welding 15. The pure form of gold used in dentistry is known as
b) Annealing a) Cohesive gold b) Non cohesive gold
c) Wedging d) Adaptation c) Class I casting gold d) Class IV casting gold
(AIPG -94) (KAR-2013)
8. Which of the following is true about direct filling 16. The more appropriate term for annealing of direct
gold: filling of gold is
a) Properly condensed mat gold has no voids a) Bridging b) Cold welding
b) Whatever may be the type, voids are inherent c) Stepping d) Desorbing
c) Voids can be avoided with manual condensation (APPG-15)
d) None of the above
(AP -96)
9. The material used in its pure form in dentistry is:
a) Composite b) Silver
c) Gold d) Amalgam
(AIIMS -97)

1) B 2) D 3) C 4) C 5) B 6) C 7) A 8) B 9) C 10) A 11) C 12) C 13) A


14) B 15) A 16) D
, DENTAL MATERIALS

7. DIRECT FILLING GOLD - ANSWERS


1. ' B' [PHILLIPS' 11th ed 550/ 12th ed / 12th ed to welding of increments together under pressure at
413] mouth temperature rather than by melting.
Mat gold is used to build the internal bulk of
restoration because it can be more easily compacted 8. 'B' [PHILLIPS' 11th ed 557/ 12th ed 415]
within the cavity. Because it is loosely packed, it is
friable and contai ns numerous void spaces between 9. ' C' [PHILLIPS' 11th ed 547 / 12th ed 413]
particles.
10. 'A' [PHILLIPS' 11th ed 551/ 12th ed 413]
2. ' D' [PHILLIPS' 11th ed 546/ 12th ed / 12th ed According to new edition chemically precipitated or
413] powdered gold is now ca lled as "Granulated gold" or
The direct filling gold is 99.99% pure which is similar encapsulated gold powder.
to 24 carat gold. Carat refers to t he parts of pure gold
in 24 parts of an alloy. 24-karat gold is pure gold, 11. 'C' [PHILLIPS' 11thed 575/ 12th ed 368]
whereas 22-karat gold is an alloy containing 22 parts 24 karat gold contains 100% of pure gold. Type II
pure gold and 2 parts of other metals. gold alloys contain 75% of gold. So Type II gold
alloys contains 18k gold.
3. 'C' [PHILLIPS' 11th ed 550]
The electrolytic precipitate of gold is alloyed with 12. 'C' [PHILLIPS' 11th ed 588/ 12th ed 75]
0.1% calcium. Calcium increases the strength by Si lver and copper may cause greenish discoloration at
dispersion strengthening (AP-2013) . the cervica l margins if they are present in the metal
ceramic alloys.
4. 'C' [PHILLIPS' 11th ed 554/ 12th ed 414]
Stepping is the process in which each increment of 13. 'A'
gold is "stepped" by placing the "condenser" point in
successive adjacent positions, as the force is applied. 14. ' B' [Phillips 11th ed 548]
Stepping permits each piece to be compacted over its Refer Direct Filling Gold: poiint No.3 in synopsis.
entire surface so that voids are not bridged.
15. 'A' [Phillips 11th ed 54 7/ 12th ed 414]
Usually small condenser tips are used, as the force of Non-cohesive gold should be disorbed or annealed to
distribution is large without causing much damage to remove the surface contaminants before using it as
oral structures. restorative material.

5. ' B' [PHILLIPS' 11th ed 551/ 12th ed 413] 16. 'D' [Phillips 12th ed 414]
Powdered gold is the atomized or chemically A more appropriate term would be desorption than
precipitated gold. The powders are first mixed with annealing, because the obj ective is to remove the
wax to form pellets and are then wrapped with gold gases adsorbed on to surface of gold.
foi l. So it is also called as encapsulated gold powder.
The average particle size is 15 micrometers.

6. 'C' [PHILLIPS' 11th ed 554/ 12th ed 414]


To ensure dense masses in corners and at the junction
between two walls, the line of force must be directed
to bisect line ang Res and trisect point ang les.

7. 'A' [PHILLIPS' 11th ed 547/ 12th ed 414]


Cohesion of gold foil results from metallic bonding
and cold welding of increments. Cold welding refers
Dental ;lut.,e

8. DENTAL CEMENTS
Fluoride rich materials include? 9. Which of the following show chemical bond with
a) Silicate cement b) Glass ionomer cement enamel (Calcified tissues):
c) Poly carboxylate cement a) Composites b) Direct filling resins
d) All of the above c) Polycarboxylate cements
(MAN -97) d) BIS-GMA resins in pit and fissure sealants
2. Glass ionomer cements are composed of (AIIMS -01)
a) Alumina silicate powder and phosphoric acid 10. Which of the following cements is most irritant to
b) Aluminosilicate powder and polyacrylate pulpal tissues?
c) Zinc oxide powder and phosphoric acid a) Zinc phosphate b) Silicate
d) Zinc oxide powder and polyacrylate Liquid c) Glass ionomer d) Polycarboxylate
(MAN -95) (KAR -97)
3. Which one of the following dental cement 11. Which of the following do polycarboxylate and GIC
accelerates the formation of reparative dentin? have in common?
a) Eugenol b) Calcium hydroxide a) Polysiloxane b) Phosphoric acid
c) Zinc oxide d) Silica c) Polyacrylic acid d) Ion leachable glass
(MAN -97, KAR -2K) (KAR -97)
4. Minimum thickness for type I zinc phosphate 12. pH of polycarboxylate liquid is:
cement should be a) 5 b) 7
a) 15 microns b) 25 microns c) 8 d) 1. 7
c) 50 microns d) 100 microns
(MAN -02) 13. Which is the cement with adhesive properties?
5. Increased amount of powder in zinc phosphate a) Zinc oxide eugenol b) Zinc polycarboxylate
cement mixture will cause c) Silicate d) Glass ionomer
a) Decreased strength b) Decreased film thickness (AIPG -98)
c) Decreased solubility d) Increased setting time 14. When selecting the colour of cement to use for
(MAN -2K) cementing a porcelain jacket crown, the powder
6. The temperature (in ° F) of the mixing slab for being tested is best mixed with:
silicate cements should be a) Glycerin-water solution
a) 55 - 65 b) 65 - 75 b) Cement liquid
c) 75 - 85 d) 85 - 95 c) Alcohol d) Only water
(MAN -95) (AIPG -03, KAR -01)
7. Which of the following is common to both zinc 15. Eugenol may be replaced in the zinc oxide eugenol
eugenol cement and polycarboxylate cement? cement by:
a) Polyacrylic acid is liquid a) Acetic acid b) Alginic acid
b) Chemical bond to tooth structure c) Phosphoric acid d) Ortho-ethoxy acid
c) Chelation (AIIMS -99)
d) Substitution of eugenol by EBA to increase 16. The setting time of zinc phosphate may be retarded
strength of cement by?
(AIIMS -2K) a) Increase in the ratio of powder to liquid
8. The addition of which of the following can b) Diluting the liquid with water
accelerate the setting time of zinc oxide cement: c) Increase the addition of powder to liquid
a) Zinc acetate b) Barium sulphate d) Decrease the addition of powder to liquid
c) Zinc sulphate d) Barium ch loride (AIIMS -01, AIPG -02)
(KAR -2K)

1) D 2) B 3) B 4) B 5) C 6) B 7) C 8) A 9) C 10) B 11) C 12) D 13) B&D


14) A 15) D 16) D
, DENTAL MATERIALS

17. Thermal conductivity of which material is close to a) 1 minutes b) 2 minutes


dentin: c) 3 minutes d) 4 minutes
a) Amalgam b) Silicate (AIIMS -97)
c) Composite resin d) Cavity varnish 26. The major component of zirnc phosphate cement is:
(AIIMS -99, AIIMS -95) a) Zinc oxide b) Phosphoric acid
18. pH of fully set zinc phosphate: c) Zinc d) None of the above
a) 3-4 b) 4-5 (AP -03)
c) 6-7 d) 7-8 27. Which of the following cements is most kind to
pulp?
19. The logical explanation for the unique a) Glass ionomer b) Polycarboxylate
anticariogenic property of most silicate cements c) Silicate d) Resin cement
is: (AIPG -94)
a) The reduction in enamel solubility due to fluoride 28. Polycarboxylate cement is used for:
uptake by enamel a) Temporary restoration
b) That beryllium flux is used in silicates b) Leuting
c) That silicates show very little leakage at the c) Permanent restoration
margins of the restoration d) Die material
d) Due to the high silica content (KAR -01)
(AIIMS -03) 29. Silicate cement is indicated in:
20. Thickness of luting cement is: a) Mout h breathers
a) 20-40µ b) 10-20µ b) Patients with high caries index
c) 60-80µ d) 1-2µ c) In the restoration of posterior tooth
(AIIMS -96, BHU-07) d) None of the above
21. The two cements which hold with the tooth are: (AIIMS -97)
a) GIC and polycairboxylate 30. The dual cure cements should not be used with
b) GIC and ZnP0 4 light transmitting prostheses thicker than
c) ZnP0 4 and silicate d) GIC and silicate a) 1.0 mm b) 1.5 mm
(AIIMS -96) c) 2.0 mm d) 2.5 mm
22. The chief advantage of zinc phosphate cement is (BHU-2012)
its: 31. The cement which is least soluble in oral cavity:
a) Good compressive strength a) Glass ionomer b) Resin cement
b) Film thickness c) Polycarboxylate cement
c) Lack of irritation d) Low solubility d) Silica - phosphate cement
(AIPG -97) (MCET -14, KAR -97, PGI -95)
23. All of the following is used as accelerator in zinc 32. Which of the following is most important in
oxide eugenol cement except strength of ZOE:
a) Acetic acid b) Zinc acetate dehydrate a) Liquid powder ratio
c) Calcium chloride d) Bromoglycerine b) Temperature of mixing stab
(PGI-2011) c) Speed of mixing
24. Which of the following cements is most d) Addition of few drops of water
biocompatible with pulp? (AIPG -2K)
a) GIC b) Copper cement 33. Which cement is irritating to the pulp?
c) Zinc phosphate cement a) Carboxylate cement
d) Zinc oxide eugenol cement b) Zinc oxide-eugenol cement
(AIPG -97) c) Zinc phosphate
25. What is the working time of zinc polyacrylate d) Ethoxybenzoic acid
cement? (AIIMS -2K)

17) B 18) C 19) A 20) A 21) A 22) B 23) D 24) D 25) C 26) A 27) B 28) B 29) B
30) D 31) B 32) A 33) C
Dental ;lut.,e

34. The effect of zinc oxide-eugenol on the pulp? d) It has less occlusal wear
a) Is irritating b) Encourages pulpal fibrosis (TNPSC -99)
c) Is sedating d) Has no effect 44. The setting of zinc phosphate is accelerated by:
(AIIMS -97, PGI -03) a) Adding a drop of water
35. Which component of zinc oxide-eugenol cement b) Increasing mixing time
gives its strength? c) Cooling the slab
a) Rosin b) Zinc oxide d) Adding a drop of oleic acid
c) Zinc acetate d) Oil of cloves (KAR -98)
(AIIMS -97) 45. Which of the following cements have anti
36. Most soluble cement? cariogenic property:
a) ZOE b) Zinc Phosphate a) ZOE
c) SiUcophosphate d) Polycarboxylate b) Calcium Hydroxide
(AIPG-14) c) Zinc silicophosphate
37. The main components in silicophosphate cement: d) Zinc phosphate
a) Zinc phosphate and GIC (KAR -94)
b) Silicate and GIC 46. pH of which cement remains below 7 even after 1
c) ZOE and Silicate month after insertion into the cavity?
d) Zinc phosphate and Silicate a) Glass Ionomer Cement
(AP -02) b) Zinc phosphate
3 8. Secondary caries is least likely seen with: c) Resin cement d) Silicate cement
a) Silicate cement b) Zinc phosphate (COMEDK -04)
c) Polycarboxylate d) GIC 47. Which of the following characterizes the silicate
(AP -04) cement:
39. Percentage of zinc in ZOE cement is: a) It is pulpal irritant
a) 60% b) 70% b) It is high ly cariogenic
c) 80% d) 90% c) Chemically attaches to the tooth structure
(AP -02) d) It produces thinnest film surface
40. Most preferable cement for temporary restoration: (KAR -99)
a) GIC b) ZOE 48. The most commonly used, most biocompatible and
c) Ca (OH) 2 adhesive cement is:
d) Zinc polycarboxylate cement a) Zinc phosphate b) Zinc oxide eugenol
(AP -04) c) Calcium hydroxide d) Glass ionomer
41. Which of the following cements is contraindicated (APPSC -99)
in mouth breathers: 49. Which of the following cements bonds to tooth
a) Silicate b) ASPA structure, which has an anticariogenic effect, has a
c) Polycarboxylate d) Zinc phosphate degree of translucency and does not irritate the pulp?
(KAR -98) a) Polycarboxylate cement
42. Why GIC is best restorative material in children: b) Resin cement
a) High strength b) Good marginal integrity c) Silicate cement
c) It can bond to both enamel & dentin d) Glass ionomer cement
d) It has less occlusal wear (AIPG- 06)
(PGI -95) 50. Torsional force is:
43. The principle application of zinc phosphate cement a) Compression b) Tensile force
is: c) Shear d) Transverse bending force
a) In final cementation (AIPG- 05)
b) As temporary cementation
c) As a temporary fi lling material

34} C 35} B 36} D 37} D 38} A 39} B 40} B 41} A 42} C 43} A 44} A 45) C 46} D
47} A 48} D 49} D 50} C
, DENTAL MATERIALS

51. All of the following are true for traditional solution 57. Which cement base has the highest modulus of
liner, except: elasticity?
a) It is not necessary in moderately deep cavities a) Zinc polycarboxylate
under glass ionomer restoration b) Polymer reinforced ZOE cement
b) Have Film thickness of 1 -50 microns c) Zinc phosphate d) Glass inomer cement
c) Do not provide thermal or electric insulation (COMEDK-06)
d) Protect the pulp from reaction products leaching 58. Frozen slab technique is applicable to:
out of restoration a) Zinc phosphate cement
(AIPG- 05) b) Glass Ionomer cement
52. All of the following is used as accelerator in zinc c) Zinc oxide Eugenol cement
oxide eugenol cement except: d) Resin cement
a) Acetic acid b) Zinc acetate dehydrate (COMEDK-05)
c) Calcium chloride d) Bromoglycerine 59. Ethoxy benzoic acid increases the strength and
(PGI- 06) solubility of the cement:
53. The average life of silicate cement restoration is: a) Zinc oxide eugenol b) Polycarboxylate cement
a) 2 years b) 4 years c) Zinc phosphate cement
c) 6 years d) 8 years d) Silicate cement
(KAR-04)
54. The principal constituent of set silicate cement 60. Which of the following acts as accelerator in Zinc
matrix is: oxide eugenol:
a) Hydrated Aluminium phosphate a) ZnCl2 + Eugenol b) ZnCl2 + ZnSO 4
b) Hydrated Aluminium phosphate with calcium c) Eugenol + ZnO d) Zn(OH)z + ZnO
fluoride (AIPG-07)
c) Hydrated Aluminosilicate gel 61. In non-eugenol pastes the constituents are mixed
d) Hydrated Aluminium phosphate crystals, Ca F2 in
aluminosilicate gel. a) Phosphoric Acid b) Ethyl alcohol
c) Benzoic acid d) None of the above
55. The cavity varnish applied reduces post operative (AP-08)
sensitivity in amalgam restoration by: 62. Strength of zinc phosphate is effected by
a) Decreasing conduction of heat to pulp a) Temp of glass slab b) P:L ratio
b) Minimize marginal leakage around restoration c) Consistency of mix d) Alteration of mixing time
c) By altering the chemical composition of restoration (B HU-07)
materials 63. Mechanism of adhesion of GIC restoration with
d) By preventing penetration of corrosion products tooth surface is by means of?
into the dentinal tubules a) Carboxyl group b) Chelates with Metal Ions
c) C=C double bond d) Polymer Chains
56. Age hardening is a process done at: (AIPG-09, AIIMS MAY- 2012)
a) Heating up to 200 - 450°( for 15 - 30 min and 64. Frozen slab technique of mixing zinc phosphate is
quenching in water used for:
b) Heating at 700°( for 10 - 15 min and quenching a) cementing crowns b) temporary dressing
in water c) base
c) Heating at 700°( for 1 hour and quenching in d) cementing orthodontic bands
water (IGNOU-10)
d) Heating above 700°( for 30 min and quenching in 65. The setting time of ZnP04 cement can be retarded
water by
(PGI-06) a) mixing more powder to the liquid
b) mixing powder to liquid, checked by water

51) B 52) D 53) B 54) D 55) B 56) A 57) C 58) A 59) A 60) B 61) C 62) B 63) A
64) A 65) C
Dental ;lut.,e

c) slower addition of powder to liquid c) The mix appears thick at the start of mixing but
d) faster addition of powder to liquid after 30 seconds of additional spatulation it
(AP-09) becomes more fluid
66. About zinc oxide eugenol all are true except - d) water accelerates but heat retards the setting of
a) ZnO is converted to Zn(OH)2 zinc oxide eugenol cements
b) Reaction is autocatalytic (COMED-2012)
c) Water is the byproduct of reaction 72. Cavity varnishes have
d) Dehydrated ZnO reacts with dehydrated eugenol a) average bonding to tooth
(AIIMS-09) b) low bonding to tooth
67. Mode of failure dUJring dislodgement of prosthesis c) excellent bonding to tooth
cemented with zinc poly carboxylate cement is d) no bonding to tooth
usually seen at the (BHU-2012)
a) Cement tooth interface 73. Advantage of Zinc phosphate over GIC is:
b) Cement prosthesis interface a) High compressive strength
c) Cleavage through the cement layer b) Tensile strength
d) Fracture of tooth or prosthesis c) High modulus of elasticity
(AIPG-2011) d) Diametral strength
68. Which glass ion omer cement develops early (NEET-2013)
resistance to water intake on setting? 74. Restoration commonly recommended for cervical
a) Reinforced auto care glass ionomer cement abrasion is
b) Compomer a) Sillicate b) Poly carboxylate
c) Resin modified glass ionomer cement c) Glass ionomer d) Composites
d) Ceramic reinforced glass ionomer cement (AP-14)
(AIPG-2011) 75. The selection of a pulp protecting agent does not
69. Which one of the following restorative materials depend on
is having coefficient of thermal expansion close to a) Remaining dentin thickness
that of tooth enamel and dentin? b) Choice of restorative material
a) Microfilled composite resins c) Symptoms of the patient
b) Pure gold d) Stage of tooth development
c) Silver amalgam (GCET-14)
d) Type II Glass ionomer cement 76. ZOE is not used for permanent cementation
(AIPG-2011) because of?
70. Which one of the following dental cements does a) Less strength b) High solubility
not contain water as part of the composition of the c) Poor marginal seal d) Poor pulpal protection
liquid component? (PGI DEC-2011)
a) Zinc phosphate cement 77. Which of the following is false about glass ionomer
b) Zinc oxide eugenol cement cement?
c) Zinc silicophosphate cement a) Acid treatment of the tooth surface is required
d) Zinc polycarboxylate cement before placement
(KCET-2012) b) Poor marginal seal
71. All of the following statements are true for zinc c) Low fracture resistance
oxide eugenol cements except d) Has ability to renew broken ionic bonds
a) Equal lengths of base paste and accelerator paste (MHCET-15)
are mixed together until the mix has a uniform
colour
b) Increase in temperature and humidity shorten the
setting time

66} D 67) B 68} C 69} D 70} B 71} C&D 72} D 73} C 74) C 75) D 76) A 77) B
, DENTAL MATERIALS

8. DENTAL CEMENTS - ANSWERS


1. 'D' [PHILLIPS' 11th ed 446/ 12th ed 323] 5. 'C' [PHILLIPS' 11th ed 465/ 12th ed 317]
Anticariogenic prnperty of cements makes the Increased powder liquid ratio decreases the setting
enamel (Fluorapatite) resistant to acid mediated time and solubility, and increases strength and film
deca lcification. Silicophosphate has the highest thickness. But when the mixing slab is cooled, the
fluoride content fo llowed by silicate and GIC whereas setting time increases and also permits the addition of
Polycarboxylate cement has least fluoride content. more amount of powder to Liquid without developing
high viscosity.
They release fluoride throughout life of restoration
but rate of release decreases over time. 6. 'B' [Year book of dentistry by Darlington 19]

2. 'B' [PHILLIPS' 11th ed 472/ 12th ed 320] 7. 'C' [PHILLIPS' 11th ed 490/ 12th ed 331]
The powder of t raditional glass ionomer cement is a Zinc oxide powder reacts with eugenol and polyacrylic
calcium-flouro-alumino-silicate glass. The powder is acid to form zinc eugenolate and zinc polycarboxylate
referred to as "ion leachable glass". It contains silica respectively. Both are examples of chelation reaction.
(35-40%), alumina (20-30%) and aluminum fluoride
(1.5-2.5%), calcium and sodium fluorides (20-25%). 8. 'A' [PHILLIPS' 11th ed 572]
Accelerators of setting reaction of ZOE cement
Lanthanum, strontium, barium are added in traces for • Zinc acetate
radioopacity. The fluorides act as ceramic flux.
• Alcohol
• Water
Liquid contains
• Glacial acetic acid
Polyacrylic acid 45%
50%
Water Retarders of setting reaction of ZOE cement
(hydrates reaction product)
• Cooling the glass slab
• Itaconic acid
• Glycerine
• Maleic acid 5% (reduces viscosity)
• Olive oil
• Tricarballyic acid
• Increasing liquid powder ratio
Traces
Tartaric acid (Increases working time &
decrease setting time) All of the following are added as an accelerator
in ZOE cement except? (PGI Dec-2011)
3. 'B' a) Acetic acid b) Zinc acetate dehydrate
Calcium hydroxide, because of its high pH value, acts c) CaCl2 d) Bromoglycerine
as a pulp irritant and stimulates reparative dentine
formation . It is a lso used as a pulp-capping agent, 9. 'C' [PHILLIPS' 11th ed 467 / 12th ed 318]
both in direct and indirect pulp capping. Zinc polycarboxylate is the first cement to show
adhesion with tooth structure. Glass ionomer is
4. 'B' [PHILLIPS' 11th ed 451/ 12th ed 314] another such cement to show chemical bonding with
Zn PO 4 is oldest of all luting cements and is called as enamel.
universal luting ce ment
10. 'B' [PHILLIPS' 11th ed 446]
Silicate cement is a severe irritant to pulp tissues.
Type I Fine grained for luting. Film thickness is
It's pH is less than 3 at the time of insertion and
Zn PO, 25 micromete rs or less.
it remains below 7 even after one month. High
Type II Medium grained for luting and fi lling.
solubility in oral cavity is another disadvantage of
Film thickness is 40 micrometers.
silicate cement.
Dental ;lut.,e

The cement filling mat erial which has t he most Underlying dentin should be protected against the
severe injurious effect on pulp is (COMEDK-2013) infi ltration of acid via the dentina l tubules to prevent
Ans. Silicate cement pulpal injury.

11. 'C' [PHILLIPS' 11th ed 471/ 12th ed 318] 19. 'A' [PHILLIPS' 11th ed 446]

12. 'D' [PHILLIPS' 11th ed 469/ 12th ed 319] 20. 'A' [PHILLIPS' 11th ed 451/ 12th ed 314]
pH of liquid ofpolycarboxylate cement is 1.7. However,
the liquid is rapidly neutralized by the powder and 21. 'A' (PHILLIPS' 11th ed 467, 471/ 12th ed 318]
as the setting reaction proceeds, the pH of mix rises
rapidly. 22. 'B'

In addition, the larger size of polyacrylic acid molecules 23. 'D' [Manappalil 2nd ed 40)

limits its diffusion through dentinal tubules. Despite


the initial acidic nature of the polycarboxylate 24. 'D' [PHILLIPS' 11th ed 489/ 12th ed 331]

cements, their products produce minimal irritation to pH of ZOE cement at the time of insertion is 7 and it
pulp. is one of the least irritating of all dental cements and
provides excellent seal against leakage.
13. ' B & D' [PH IL LIPS' 11th ed 466/ 12th ed 318]
25. 'C' [PHILLIPS' 11th ed 468/ 12th ed 320]
14. 'A' (PHILLIPS' 12th ed 330] Working time of zinc polycarboxylate is 2.5 min. and
the working time of zinc phosphate cement is 5 min.
15. ' D' [PHILLIPS' 11th ed 253/ 12th ed 331]
approximately.
Mechanical properties of ZOE cement can be improved
by adding alumina to the powder and orthoethoxy 26. 'A' [PHILLIPS' 11th ed 461/ 12th ed 316]
benzoic acid to the liquid or by reinforcing with Composition of Zinc phosphate cement
polymers. • ZnO - 90%
Powder
• MgO - 10°/o
Reinforced ZOE cements are used for intermediate
restorations. • Phosphoric acid
• Aluminum phosphate
16. ' D' [PHILLIPS' 11th ed 463/ 12th ed 317] Liquid
• Water
Setting time of zinc phosphate cement can be increased
• Zinc phosphate
by decreasing the rate of addition of powder to liquid.
But the best way to control setting time is by regulating
the temperature of the mixing slab. 27. 'B' (PHILLIPS' 11th ed 469/ 12th ed 320)
The most biocompatible cements to pulp in decreasing
Introduction of powder into the liquid for the first order are ZOE and polycarboxylate, GIC, ZnP0 4 • Resin
few increments increases working and setting times cements and silicate cements have an irritating effect
by reducing the amount of heat generated. on pulp.

17. 'B' 28. 'B' (PHILLIPS' 11th ed 466/ 12th ed 319)


Zinc polycarboxylate, Zn PO 4, silicophosph ate, ZOE,
18. 'C' [PHILLIPS' 11th ed 464/ 12th ed 318] GIC and resin cements can be used as cementing or
The pH of the mix after mixing is approximately 2, leuti ng agents.
but increases rapidly to a pH of 6 in 24-48 hours. So
damage to pulp from acid occurs during the first few ZOE and resins are used as temporary restorations.
hours after insertion.
, DENTAL MATERIALS

29. ' B' [PHILLIPS' 11th ed 446] GIC < ZnPo 4 <ZOE< Polycarlboxylate.
Silicates have anticariogenic action due to its high
fluoride content (15%). So it is indicated in patients 37. 'D' [PHILLIPS' 10th ed 568]
having high caries index. Zinc silicophosphate cement consists of Zno,
silicate powder and phosphoric acid liquid. It has
If the restoration is allowed to dry, it becomes anticariogenic action.
powdery and opaque and so it is contraindicated in
mouth breathers. 38. 'A' [PHILLIPS' 11th ed 446]

30. ' D' [Philips Pg 487 / 12th ed 330] 39. ' B'
In dual cure cements, chemical activation is slow and
provides extended working time until the cement is 40. ' B' [PHILLIPS' 11th ed 490/ 12th ed 331]
exposed to the curing light. They should not be used
with light transmitting prostheses thicker than 2.5 41. 'A' [PHILLIPS' 10th ed 528]
mm. Anything thicker than this should be bonded
with chemically curable cement. 42. 'C' [PHILLIPS' 11th ed 475/ 12th ed 320]

31. ' B' [PHILLIPS' 11th ed 486] 43. 'A'


ZOE and silicate cements have high solubility and
disintegration rate. GIC and silicophosphate have 44. 'A' [PHILLIPS' 11th ed 462]
low solubility compared to polycarboxylate and zinc Increased mixing time (spatulation time) retards
phosphate. Resin cement is least soluble in ora l cavity. the setting reaction as the matrix is destroyed. But
whereas in case of dental stones increased spatulation
32. 'A' [PHILLIPS' 11th ed 490/ 12th ed 331) within limits accelerates the setting t ime.

33. 'C' [PHILLIPS' 11th ed 464/ 12th ed 318] 45. 'C' [PHILLIPS' 11th ed 446)

34. 'C' [PHILLIPS' 11th ed 489, 490] 46. 'D' [PHILLIPS' 10th ed 528)
ZOE cement has an obtundant or sedative action on
pulp. 47. 'A' [PHILLIPS' 11th ed 458]

35. ' B' [PHILLIPS' 10th ed 180/ 12th ed 331) 48. ' D' [PHILLIPS' 11th ed 475/ 12th ed 320)
Though ZOE cement is most biocompatible, it does
36. ' D' [Check Explanation Below] not adhere to tooth structure.
A better indication of solubility and erosion is
achieved by testing the materials in acidic media. 49. ' D' [PHILLIPS' 11thed 446, 467, 469/ 12th ed 320]
The maximum values of material loss allowed in the
standard ISO are: 50. 'C' [PHILLIPS' 10th ed 52/ 12th ed 53)

Requirement of cements Acid solubility/ erosion 51. ' B' [Sturdevant 4th ed 170]
specified in ISO luting Maximum mm/hour
and base 52. 'D' [Manappallil 2nd ed 40)
Zinc phosphate 1
53. 'B'
Polyca rboxylate 2
• Silicophosphate
0.05 54. ' D'
• GIC
55. 'B' (PHILLIPS' 11th ed 459/ 12th ed 314)
According to clinical studies solubility of cements in Varnish reduces the infiltration of irritating fluids
the mouth from least to most soluble are: through marginal areas.
Dental ;lut.,e

56. 'A' [Manappallil 2"d ed 324] 59. 'A' [PHILLIPS' 10th ed 575/ 12th ed 331]

57. 'C' [PHILLIPS' 11th ed 475/ 12th ed 311] 60. 'B' [PHILLIPS' 10th ed 180, 181/ 12th ed 179]
Zinc phosphate has high modulus of elasticity of
13.7GPa. It is quite stiff whereas GIC is Less stiff. 61. 'C' [PHILLIPS' 11th ed 253/ 12th ed 331]
Eugenol causes burning or stinging sensation. EBA
58. 'A' [PHILLIPS' 11th ed 463/ 12th ed 317] (Orthoethoxy benzoic acid) is a valuable substitute
Cooling the slab markedly reduces the chemical for eugenol in this regard.
reaction between the powder and the Liquid, thereby
regarding the formation of the matrix.

62. ' B' [Craig 12th ed 491]


Effect of manipulative variables on selected properties of zinc phosphate cement

Compressive
Film thickness Solubility Acidity Setting time
Strength
l P/L ratio l l i i Slower
Rapid addition of l i i l Faster
powder to liquid
i Mixing temp. l i i i Faster
Water contamination l T T l Faster

63. 'A' [PHILLIPS' 11th ed 475/ 12th ed 321] 67. 'B' [Phillips 11th ed 4 57/ 12th ed 312]
The carboxylic acid side groups are capable of chelating Two modes of failure are associated with cements
surface ions on the glass particles or calcium ions i) Fracture of the cement
from the tooth structure. ii) Failure of the cement prosthesis interface. This is
the most common failure.
64. 'A' [PHILLIPS' 11th ed 463/ 12th ed 317]
Cement prepared on a cool slab permits cementation Zinc polycarboxylate bonds chemically with tooth
of cast restorations as the mix will be less viscous. and if failure occurs, it occurs at cement prosthesis
interface.
65. 'C' [PHILLIPS' 11th ed 462/ 12th ed 317]
68. 'C' [Phillips 11th ed 482/ 12th ed 3 26]
66. ' D' [PHILLIPS' 11th ed 490/ 12th ed 331] Moisture sensitivity and low early strength are the
disadvantages of conventional GIC. RMGIC has been
• The setting mechanism for ZoE materials consists
introduced to overcome these two inherent drawbacks.
of Zinc oxide hydrolysis and a subsequent reaction
between zinc hydroxide and eugenol to form a
69. 'D' [Phillips 11th ed 55/ 12th ed 40]
chelate.
Material Co-efficient of
• Water is needed to initiate the reaction, and it is
thermal expansion
also a byproduct of the reaction.
Dentin 0.75
• Accelerators for setting reaction Enamel 1.00
- humid environment Silicates 0 .996
- acetic acid GIC Type II 0.96
- high temperature Porcelain 0.58
Unfilled resins 7.11
- zinc acetate dehydrate
Inlay wax 35.1
, DENTAL MATERIALS

70. 'B' [Philips 11th ed 490/ 12th ed 331]

71. 'C' & 'D' [Philips 11th ed 253/ 12th ed 179]


Both water and heat accelerates the setting reaction
and thereby reduces setting time.

72. 'D' [Philips 11 ed 459/ 12th ed 314]

73. 'C' [Phillips 11th ed 475/ 12th ed 311]


The modulus of elasticity of GIC is only about one-
half that of zinc phosphate cement. Thus GIC is less
stiff and more susceptible to elastic deformation. In
this regard, it is 11ot as desirable as zinc phosphate
cement.

74. 'C' [Atlas of GIC by G.J. Mount Pg 170]


Both GIC and composite can be used for restoring
cervical abrasion lesions. But according to the key
as well as the above reference the most common ly
recommended is GIC.

Note: Cervical abrasion most commonly involve root


surfaces.

GIC will adhere very effectively to the burnished


sclerotic surface of the root through ion-exchange
mechanism. Also these non -carious cervical lesions
have smooth surface and GIC is the best for any form
of chemical adhesion.

75. 'D' [Phillips 12th ed 315]


The selection of a pulp protecting agent i.e., varnish,
liner or a base depends on
• remaining dentin thickness
• type of protection required i.e., chemical (or)
thermal (or) electrical protection
• type of restorative material chosen. Eg: If
composite restoration is chosen, then Ca(OH)2
and GIC liner are selected.
• whether any caries prevention benefits are required

Selection of pulp protection agents does not depend


on stage of tooth development.

76. 'A'[Check O.No.15]

77. 'B' [Nisha Garg 2nd ed 489]


GIC exhibits good marginal seal.
Dental ;lut.,e

9. DENTAL CERAMI CS & MISCELLANEOUS


1. Porcelain denture teeth: b) Uniformity of particle size
a) Have a higher coefficient of thermal expansion c) Shape and size of particle
than acrylic teeth d) Type of investment used
b) Have a lower abrasion resistance than enamel (AIIMS -98, AIPG -99)
c) Should be used where the inter dental clearance is 9. Phenomenon in which porcelain appears different
small under varying light condition is:
d) Have a higher abrasion resistance than gold a) Translucency b) Refractive optics
(MAN -94) c) Metamerism d) Opacification
2. Chemical tempering in porcelains is done to (COMEDK -04, AIPG -99)
foterrupt crack propagation by: 10. Porcelain is best fired :
a) Transformation toughening a) In open air b) Under air compression
b) Dispersion of crystalline phase c) When several layers are fired simultaneously
c) Inducing residual compressive strength d) Under minimum pressure
d) All of the above (AIIMS -95)
(MAN -02) 11. Machineable glass ceramic is:
3. The opacity in ceramics is achieved by adding: a) Cerestore b) Decor MGC
a) Boric oxide b) Copper oxide c) Infusium d) Leucite
c) Silica d) Titanium oxide (AP -01)
(MAN -01) 12. Porosity in porcelain can be prevented by:
4. To prevent porosity in dental porcelain it should be a) Thoroughness of condensation
baked: b) Rapidity of firing
a) In presence of air b) In vacuum c) High firing temperature
c) For long period d) Under pressure d) All of the above
(MAN97, AIIMS 93) (PGI -98)
5. Porosity in porcelain at condensation stage 13. Which is not present in porcelain:
depends on: a) Silica b) Feldspar
a) Shape and size of particle c) Calcium carbonate d) Amorphous material
b) Uniform distribution of particle size (AIIMS -96)
c) Number of particles d) None of the above 14. Which of the following is the main constituent of
(AIPG -2K) porcelain:
6. Pyroplastic flow of porcelain is due to: a) Feldspar b) Kaolin
a) High temperature during glazing c) Quartz d) Clay
b) Low temperature during glazing (AIPG -96)
c) Improper condensation 15. Flux used in dental ceramics is:
d) High firing temperature a) Alumina b) Silica
(AIPG -2K, 97) c) Kaolin d) Boric oxide
7. In porcelain metal elements are embedded in the (PGI -95)
matrix of 16. The best tissue tolerated material for crown and
a) Rubber matrix b) Glass bridge:
c) Plastic d) Resin matrix a) Highly polished porcelain
(AIPG -02) b) Highly polished acrylic
8. Condensation shrinkage of porcelain during firing c) Highly glazed porcelain
depends on: d) Highly polished metal
a) Rate of arriving at firing temperature (KAR -97)

1) D 2) C 3) D 4) B 5) A 6) A 7) B 8) C 9) C 10) B 11) B 12) A 13) D


14) A 15) D 16) C
, DENTAL MATERIALS

17. Opaque porcelain: c) Medium bisque d) High bisque


a) is Feldspathic glass
b) contains opacifier like zirconium and titanium 25. Quartz in Dental porcelain is
oxide a) Strengthener b) Binder
c) Not used to mask the colour of the cement used in c) Pigment d) Crack minimizer
placing of restoration (COMEDK-06)
d) Used to produce the characteristic translucence, 26. The maximum shrinkage during firing process in
which is an innerent aspect of the incisal edge in ceramic occurs in:
natural teeth a) High Bisque stage b) Low Bisque stage
(PGI-2001, KAR -98) c) Medium Bisque stage
18. Dental porcelains are crystalline materials of: d) Fusion stage
a) Rubber b) Plastic (AIPG, AIIMS -07)
c) Organic substances d) Glass 27. Which of the following bur is used in the
(MAN -94) preparation of cavity restored with porcelain fused
19. Dicor restoration is: to metal?
a) Two coloured restorations a) Carbide bur b) Diamond bur
b) Heat pressed cera mies c) Stainless steel bur
c) Castable ceramic d) None of the above d) Diamond bur for porcelain and carbide bur for
(AIIMS -94) metal
20. The main purpose of adding metallic oxides in (AIPG-07)
ceramic is: 28. Super abrasive is
a) Colour matching b) Transparency a) Sand b) Silicon carbide
c) Opacity d) None c) Diamond d) Aluminum oxide
(AIPG -14, PGI -95) (KCET-08)
21. Porcelain binds to metal in PFM crown by: 29. The first porcelain tooth material was introduced
a) Metallic bind b) Mechanical bond by
c) Chemical bond d) B and C a) DeChemant b) Ash
(PGI -01) c) Plateau d) Mcclean
22. Coefficient of thermal expansion of metal-ceramic (COMEDK-08)
alloys is: 30. The preferred material for repair of fractured
a) Same as porcelain b) More than porcelain denture base is
c) Less than porcelain a) Chemically cure resin
d) More than or equal to porcelain but not less b) Heat cure resin
(PGI June-13) c) Light cure resin d) Vinyl resin
23. In method of firing: (COMEDK-10)
a) A shorter firing period at a higher temperature is 31. Chemical method of strengthening porcelain
considered better involves
b) A large time period at a lower temperature is a) exchange of aluminium and sodium ions
preferred. b) exchange aluminium and potassium ions
c) A long firing period at a very high temperature is c) exchange of sodium and potassium ions
preferred d) none of the above
d) A short firing period at a very low temperature is (AP-09)
preferred 32. Dispersion of crystalline phase to strengthen
ceramics results in
24. Stage recognized in the firing of dental porcelain a) dispersed porcelain b) glazed porcelain
given below. Which of state is not correct? c) aluminium porcelain d) all of the above
a) Cold stage bisque b) Low bisque (AP-09)

17) B 18) D 19) C 20) A 21) D 22) D 23) B 24) A 25) A 26) C 27) D 28) C 29) A
30) A 31) C 32) C
Dental ;lut.,e

33. Compared with dental alloys, CERAMICS show d) Mercury sulphide


a) High ductility b) High hardness (COMEDK-09)
c) Low esthetic value 41. Which of the following releases single wavelength
d) Susceptibility to brittle fracture of energy?
(COMEDK-10, KERALA-2015) a) QTH b) PAC
34. For porcelain fused to metal crown, the porcelain c) LED d) Argon lasers
should have? (AIPG-10)
a) High fusion expansion 42. Ceramics fired to metals are processed by
b) High fusion temperature a) Heat pressing b) Machining
c) Linear coefficient of thermal expansion less than c) Sintering d) Slip casting
that of metal (AIPG-2011, KERALA-2015)
d) Linear coefficient of thermal expansion more than 43. Clogging of an abrasive wheel with debris causing
that of metal reduction of abrasive action is called
(PGl-08) a) Blinding b) Buffing
35. Greening occurring when porcelains are fired on c) dressing d) truing
silver free alloys may be attributed to (KCET-2012)
a) Vaporization of silver from the walls of 44. When porcelain is baked against metal, it should
contaminated furnaces possess a
b) Surface diffusion of silver from the marginal metal a) high fusion expansion
c) More rapid silver diffusion in sodium containing b) high fusion temperature
glass c) linear coefficient of thermal expansion less than,
d) Silver ionization by porcelains with high oxygen but close to that of metal
potential d) linear coefficient of thermal expansion greater
(KCET-10) than, but close to that of the metal
36. The ideal temperature of water bath for softening (AP-2012)
fluid wax is about 45. Major drawback of porcelain is
a) 51 to 54 degree F b) 51 to 54 degree C a) High compressive strength
c) 70 to 75 degree F d) 70 to 75 degree C b) Brittleness
(KCET-10) c) Translucency d) Hardness
37. Electrolyte used for plating copper dies is: (COMEDK-14)
a) Ionic copper b) Copper cyanide 46. Porcelain bonded to metal is strong when it is
c) Silver cyanide d) Acidic copper sulphate a) Air fired b) Tempered after firing
(IGNOU-10) c) Fired under compression
38. Crocus cloth is - d) Fired several times before completion
a) Emery b) Garnet (GCET-14)
c) Pumice d) Rouge 4 7. Lithium disilicate containing crystals in ceramic
(KCET-09) crown are?
39. The melting point of titanium is _ _ _ _ __ a) IPS Empress b) Captek
degree celsius - c) Dicor d) IPS Em press 2
a) 1056 b) 1560 (AIPG-14)
c) 1668 d) 1886 48. Shrink free ceramic is known as?
(COMEDK-09) a) Cerestore b) Captek
40. The most toxic form of mercury is - c) IPS Empress d) Dicor
a) Methyl and ethyl mercury (AIPG-14)
b) Mercury vapour 49. According to US FDA, the machine used for taking
c) Inorganic mercury forms intraoral radiographs is classified as
a) Class-I b) Class-II

33} D 34} C 35} A 36} B 37} D 38} D 39} C 40} A 41} D 42} C 43} A 44} C 45) B
46} B 47} D 48} A 49} B
, DENTAL MATERIALS

c) Class-III d) Class-IV 5 7. For air blasting of ceramic veneer the particle size
(PGI JUNE-2014) of alumina is used
50. Regarding dispersion strengthening a) 25 micrometer b) 50 micrometer
a) Increasing the strength of ceramic by dispersing c) 75 micrometer d) 100 micrometer
their matrix with the crystalline phase of the same (PGI DEC-13)
material 58. Incorrect regarding boric c:,xide is incorporated in
b) The crystalline phase works by hindering the crack ceramics?
propagation a) Forms glass phase
c) The strengthening 1s due to polymorphic b) Forms a separate lattice interspersed with the
transformation silica
d) CAD CAM procedures in these materials further c) Increases viscosity of metal
decrease their surface flaws d) It strengthen the surrounding structures
(PGI JUNE-2014) (PGI DEC-13)
51. Animal test to check the biocompatibility of dental
material are all except
a) Buehler test b) Implantation test
c) Ame's test d) Sensitization test
(AIIMS NOV-14)
52. Which is used for etching porcelain?
a) 9.6% HF b) 1.23% HF
c) 37% HF d) 4% HF
(PGI JUNE-13)
53. What is true for fixed prosthesis?
a) It is difficult to remove prosthesis when cemented
with cement having maximum compressive
strength and low tensile strength
b) It is difficult to remove prosthesis when cemented
with cement having maximum compressive
strength and low shear strength
c) Cement is weal'<est phase in fixed prosthesis
d) Increased thickness of cement will cause less flaw
compared to small thickness
(PGI DEC-13)
54. Metal oxides are added in porcelain provide all
except
a) Strength b) Improve bonding
c) Cosmetic d) Impart colour
(AIIMS MAY-14)
55. Which of the following is not a natural abrasive
a) Arkansas stone b) Pumice
c) Tripoli d) Rouge
(MHCET-15)
56. Enamel is permeable to?
a) Bacteria b) Bacterial product
c) Retroviruses d) Peroxidase
(PGI DEC-13)

50) B 51) C 52) A 53) C 54) A 55) D 56) D 57) B 58) C


Dental ;lut.,e

9. DENTAL CERAMICS - ANSWERS


1. 'D' [PHILLIPS' 11th ed 698, 699/ 12th ed 463]
Zirconium oxide, Titanium Oxide Opaque
Porcelain teeth have low co-efficient of thermal
expansion and higher abrasion resistance. They Copper oxide Green
should be used only when there is sufficient overjet Iron oxide Brown
and overbite.
Titanium Oxide Yellow
2. 'C' [PHILLIPS' 11th ed 701/ 12th ed 441]
Strengthening of porcelains (brittle) occurs
4. 'B' (PHILLIPS' 11th ed 672/ 12th ed 433]
through :
Firing in vacuum (or) reduced pressure prevents
porosity. Firing for a long time causes changes in
the Leucite content and produce stresses sufficient
I. Development of residual compressive stresses
to cause crack in the porcelain size of the powder
A) Ion exchange ( or) chemical tempering particles influence the porosity and firing shrinkage.
Involves exchange of larger potassium ions for
the sma ller sodium ions. 5. 'K (PHILLIPS' 11th ed 671/ 12th ed 433]

B) Thermal tempering 6. 'K


Involves quenching (cooling) the surface while
7. 'B' (PHILLIPS' 11th ed 669/ 12th ed 429]
it is hot and in the molten state.

8. 'C' [PHILLIPS' 11th ed 671/ 12th ed 433]


C) Thermal compatibility
The thermal contraction co-efficient of meta l 9. 'C' [PHILLIPS' 11th ed 712/ 12th ed 38]
is slightly larger than porcelain. This slight
mismatch increases strength of porcelain. 10. 'B' (PHILLIPS' 11th ed 672/ 12th ed 433]

II. Disruption of crack propagation 11. 'B' (PHILLIPS' 11th ed 681/ 12th ed 445]
A) Dispersion of a crystalline phase - Dispersing Castable or
Alumina. machinable glass Dicor, Dicor MGC.
ceramic
B) Transformation toughening - With partially
Glass infi ltrated • Incera m ( Al20 3 )
stabilized zirconia (PSZ)
ceramic • Inceram spinel (MgAl20J

Which among the following is NOT a method to Injection Molded • IPS Empress
strengthen ceramic restoration (PGI June-14) ceramic • IPS Empress 2
a) Three layer laminate technique Shrink free ceramic Ce restore
b) Stiffer supporting materials
c) Higher therma l coefficient of expansion of
12. 'A' [ PHILLIPS' 11th ed 6 71/ 12th ed 433]
ceramic as compared to metal substructure
Thorough condensation helps in producing dense
d) Minimize number of porcelain firing cycles
packing. Dense packing of porcelain powder decreases
porosity and firing shrinkage.
3. 'D' [PHILLIPS' 11th ed 670/ 12th ed 429]
Pigmented oxides are added to glass to simulate the 13. 'D' [PHILLIPS' 11th ed 667/ 12th ed 430]
shades of natural teeth.
Porcelain is a vitreous ceramic consisting of silica,
soda, or potash, feldspar, pigments, opacifiers, and
glass modifiers. Glass modifiers decrease sintering
, DENTAL MATERIALS

temperature and increase the coefficient of thermal Vacuum firing or diffusible gas like helium, hydrogen
expansion and solubility of porcelain. firing reduces the porosity.

14. 'A' [PHILLIPS' 11th ed 668/ 12th ed 430) 24. 'A' [Vimal Sikri pt ed 447)
Feldspar when melted forms a crystalline phase called
leucite and a glass phase that will soften and flow 25. 'A' [Vim al Sikri 1st ed 440)
leucite has high coefficient of thermal expansion and
is the basic glass former. 26. '(' [Hussain pt ed 211)
Refer to synopsis
15. ' D' [PHILLIPS' 11th ed 670/ 12th ed 389)
27. ' D' [Sturdevant 4th ed 341, 342)
16. 'C' [PHILLIPS' 12th ed 438)
28. 'C' [PHILLIPS' 11th ed 368/ 12th ed 244]
17. ' B' [PHILLIPS' 11th ed 670/ 12th ed 432)
Feldspar is colourless. Oxides of tin, zirconium and 29. 'A' [PHILLIPS' 11th ed 660/ 12th ed 424)
titanium are added to produce opaque porcelains to The first porcelain tooth material was patented in
mask the colour of the metal. 1789 by Dechemant in collaboration with a French
pharmacist Duchtaeau. In 1808 Fonzi, an Italian
18. ' D' [PH IL LIPS' 11th ed 66 7/ 12th ed 429) dentist invented 'terrometallic' porcelain tooth . In
1817, Planteau, a French dent ist introduced porcelain
19. 'C' [PHILLIPS' 11th ed 681/ 12th ed 445) teeth to the United States. In 1817, Peale developed
a baking process in Philadelphia. Later Ash improved
20. 'A' [PHILLIPS' 11th ed 670/ 12th ed 432) version of the porcelain tooth in 1837 in England.
The main purpose of adding metallic oxides to porcelain In Germany, Pfaff developed a technique to make
is for shade matching to simulate natural tooth impressions of mouth using plaster of Paris in 1756.
In 1903 Dr. Charles Land introduced one of the first
21. 'D' [PHILLIPS' 11th ed 678/ 12th ed 437) ceramic crowns to dentistry. The first commercial
Porcelain binds to metal in metal ceramic restorations porcelain was developed by Vita Zahn-Fabrik in 1963.
by mechanical interlocking and chemical bonding.
Mechanical interlocking is provided by proper wetting 30. 'A' [PHILLIPS' 11th ed 748/ 12th ed 492)
of the metal surface by porcelain. The advantage of using chemically activated resins
for repairing is that they can be polymerized at room
The adherent metallic oxides, which are formed during temperature.
the degassing cycle form a good chemical bond with
porcelain. 31. 'C' [PHILLIPS' 11th ed 701/ 12th ed 441)

Tin oxide and Indium oxide are formed from precious 32. 'C' [PHILLIPS' 11th ed 702 / 12th ed 441)
metal alloys while chromium oxide is formed from
base metal alloys. 33. 'D' [PHILLIPS' 11th ed 699/ 12th ed 439)

22. ' D' [PHILLIPS' 11th ed 674/ 12th ed 435) 34. 'C' [PHILLIPS' 10th ed 602 , 603/ 12th ed 435)
Co-efficient of thermal expansion of alloy should be
close or nearly same as that of porcelain. The metal 35. 'A' [PHILLIPS' 11th ed 583, 584)
should have slightly higher value, to avoid residual The disadvantage of silver co ntaining alloys is the
tensile stresses. potential for porcelain discoloration when silver
vapour is released and deposited on the porcelain
23. 'B' [PHILLIPS' 11th ed 671, 672/ 12th ed 433) surface.
Firing at a lower temperature for a long time reduces
the porosity and gives a life like appearance. Also Silver free alloys contain no silver and do not
increase in number of firings increases the porosity. contribute to porcelain discoloration. So the greening
Dental ;lut.,e

is due to the silver vapours of previously fabricated


Processing
restorations. Example of ceramic material
method

36. 'B' [Journal of Prosthetic dentistry, 101(4), 279- Hot pressing IPS impress 2
282] Casting Dicor
Inceram spinal, Inceram Alumina,
37. ' D' [www.i-chem.com] Slip casting
Inceram Zirconia
Copper sulfate acidified with sulfuric acid is preferred
electrolyte for electroplating of copper dies. CAD/ CAM Cerecvitablocs, ceron lava
Copy milling A variety of ceramic products
38. ' D' [Check Explanation Below]
Crocus cloth is a very fine (1200 grade) jeweller rouge Sintering PFM
(i ron oxide) i mpregnated clock for polishing of nicks,
rough spots and burrs. It is very gent le but effective 43. 'A' [ Philips 11th ed 3 64]
for polishing hard metal parts. Abrasive binding:
Clogging of the abrasive instrument with debris.
39. 'C' [PHILLIPS' 11th ed 108/ 12th ed 376]
Abrasive dressing:
40. 'A' (PHILLIPS' 11th ed 198/ 12th ed 124] • It reduces the instrument to its correct working
Mercury occurs in 3 forms; metallic, inorganic and size and shape.
organic forms. Methyl and ethyl mercury are organic • It is used to remove clogged debris from t he
forms. Methyl mercury is the most toxic form of mercury abrasive instrument.
and is efficiently absorbed from the gut (90 - 95%).
Truing:
41. ' D' [PHILLIPS' 11th ed 412/ 12th ed 289] Procedure through which the abrasive instrument
Argon Laser Lamps have the highest intensity and is run against a harder abrasive block until the
emit a single wavelength (490 nm) abrasive instrument rotates in the hand piece without
eccentricity when placed on the substrate.
LED Lamps emit radiation only in the blue part of the
visible spectrum between 440 and 480 nm. 44. 'C' [Philips 11th ed 591]
• Linear coefficient of thermal expansion (ex): If
QTH lamps - They have quartz bulb with a tungsten
exp > exm then this temperature difference will
filament that irradiates both UV and white light that
create a large tensile hoop stresses causing crack
must be filtered to remove heat and wavelengths
propagation in the porcelain veneer.
except those in the violet-blue range (400-500 nm)
• exp > exm compressive hoop stresses develop in
PAC lamps use xenon gas that is ionized to produce porcelain which are better tolerated by porcelain
plasma. The high intensity white light is filtered to as it got a very high compressive strength.
remove heat and to allow blue light (400-500 nm) to
be emitted. 45. 'B' [Phillips 12th ed 439]
Porcelains are brittle in nature and have low tensile
42. 'C' [Phillips 11th ed 671/ 12th ed 433] strength. They have no mechanism for yielding to
Sintering is the process of firing the ceramic so that stress without fracture unlike metals.
the particles of powder sinter together properly to
form the prosthesis. ceramic fused to metal (PFM) is Also Refer explanation of Que No.2 for strengthening
processed by sintering. Other methods of processing of porcelains.
ceramics are:
46. 'B' (Check Explanation Below]
Air firing and multiple firing induces more porosity.
Vacuum firing, firing in diffusible gas atmosphere
, DENTAL MATERIALS

and cooling the fused porcelain (tempering) under exist in an organism between immune, inflammatory
pressure minimizes the porosity in porcelain. and circulatory system.
Eg;-Ames mutagenicity test, Cytotoxicity tests.
47. ' D' [Phillips 12th ed 447]
IPS Empress and IPS Empress 2 are Hot-Isostatically 2. Animal tests:
pressed (HIP) glass ceramics. They are indicated for Involves the placement of test material in an
anterior veneers. organism. These tests have the ability to permit an
intact biological system to respond or interact with a
IPS empress: test material. They are high cost, requires lengthy time
It is leucite based ceramic and contains 35% by and their relevance in simulating human responses is
volume of leucite. It is translucent and is indicated as challenged.
anterior veneers. Eg:- Implantation tests, Buelers closed patch test,
Sensitization tests.
IPS e mpress II:
It is lithia disilicate based ceramic and contains 65- 3. Usage tests:
70% by volume of lithia disilicate. This is used as core These are the most clinically relevant tests performed
ceramic for crowns and bridges. It is opaque and has either in animals or humans (clinical trial). No
high flexural and fracture toughness than I PS Empress histology can be performed, no evaluation of the real
damage can be done. Also they are more complex and
48. 'A' [ Explanation of Q. No.11] extremely costly. Also legal Land confidentiality issues
occur.
49. ' B' [Phillips 1st SA ed 120]
US FDA classified devices into three possible risk 52. 'A' [Check Explanation Below]
based classes. 9.6% HF is used for etching porcelain veneers, inlays
General controls that exempt from and fractured porcelain surfaces prior to bonding.
Class-I 510(K).
Low risk Eg:- dental burs, face bows and 53. 'C' [Phillips 1st SA ed 332]
preformed crowns. The cement layer is the weakest link of a prosthetic/
tooth assembly, therefore cements with higher
Special controls that require 5lO(K) bond strengths are preferred. Cements having high
Class-II
Eg:- Cavity varnish denture teeth compressive, shear and flexural strengths are preferred
Moderate
impression materials, device for Also to maximize retention, the cement layer should
risk
taking dental radiograph be thin to minimize pores.
Premarket approval of devices
that must demonstrate safety and 54. 'A' [Check Explanation of Q. No.20,21]
Class-III
effectiveness without relying on a
High risk
predicate device. 55. ' D' [Phillips 1st SA ed 256]
Eg:- Dental implants. Natural abrasives include Arkansas stone, chalk,
corundum, emery, diamond, garnet, pumice, quartz,
50. ' B' [Check Explanation of Q.No.2] sand, Tripoli and zirconium silicate. Cuttle and
Kieselguhr are derived from remnants of living
51. 'C' [Phillips 1st SA ed 143] organisms.
Three types of tests are used to analyse the
biocom patibility of dental materials. Synthetic/manufactured abrasives are silicon carbide,
aluminium oxide, rouge, tin oxide and synthetic
1. Invit ro t ests: diamond.
The material or an extract of the materia l is placed in
direct or indirect contact with some biological system 56. 'D' [Phillips 11th ed 181]
outside of an organism. They are fast, inexpensive but Although the enamel is permeable to some substances
lacks the ability to simulate complex interactions that such as the peroxides in bleaching agents, it is
Dental ;lut.,e

generally not permeable to material components,


bacteria or bacterial products.

57. 'B' [Check Explanation Below]


Steps in bonding of ceramic veneer:
• First the glaze is removed wit h aluminium oxide
sand blasting (50 um size particles) for 2-4 sec.
• The ceramic surface is etched with 9.6% HP for 2
min.
• Application of primer
• Bonding of veneer.

58. 'C' [Phillips 1st SA ed 459]


Boric oxide acts as a glass modifier to decrease the
viscocity, to lower the softening temperature and to
fo rm its own glass network. Because boric oxide forms
a separate lattice interspersed with the silica lattice,
it still interrupts the more rigid silica network and
lowers the softening point of glass.
=====================================================fr DENTAL MAT ER IALS SYNOPSIS J-~ 39 i-=J===""""~

DENTAL MATERIALS - SYNOPSIS


to accumulate debris with resultant high incidence of
PHYSICAL PROPERTIES
marginal caries.
1. Heat Of Vaporization: It is the amount of energy
9. Hue: It is the specific colour produced by a specific
needed for transformation of a boiling liquid into
wavelength of light. It describes the dominant colo ur
vapor. For example, 540 cal. of heat is required to
of an object, for example, red, green or blue.
vaporize lgm . of water at 100°C and 1 atmosphere
pressure.
10. Value (B rilliance) is the lig htness or darkness of an
object. Lighter shaded objects are with highest value
2. Latent Heat Of Fusion: It is the amount of energy
and dark shaded objects are with lower value.
released (in form of heat) when a liquid freezes. When
1 gm. of water freezes, 80 cal. of heat is released.
11. Saturation (Ch roma) is the a mount of co lour per unit
area of an object. Chroma represents the degree of
3. Melting Temperature: It is the amount of energy
saturation of a particular hue. For example, some
required for transformation of solids (such as metals)
teeth appear more yellow th an others.
into liquid state.

12. Translucency is the property of an object that permits


4. Sublimation: It is the process in which some solids
the passage of light through it but does not give any
transform directly to a gas phase.
distinguishab le image.
5. Ionic bond: Eg.: Gypsum, phosphate cements.
13. Metamerism: Objects that appear to be colour
Covalent bond: Eg.: Dental resins. matched under one type of light may appear different
Metallic bond: Eg.: Pure gold. under another light source. This phenomenon is called
Hydrogen bond: Eg.: H20. meta merism.

Vanderwalls fo rces are characterized by physical forces. 14. Fluorescence: The energy that a tooth absorbs
is converted into light with longer wavelengths,
6. Glass Transition Te mperature and becomes a light source. This phenomenon is
The temperature at which there is an abrupt increase called fluore scence. Fluorescence makes a definite
in the thermal expansion coefficient, indicating contribution to the bright ness and vital appearance
increased molecular mobility is called the glass of a human tooth.
transition temperature and it is characteristic of the
particular glassy structure. 16. Benzold-brucke Effect: (PGI J une-2011)
Change in colour/ hue due to change in brightness or
7. The force of attraction between like molecules is light intensity.
called cohesion. The force of attraction between
17. The study of matter flow characteristics is called
unlike molecu les is adhesion (APPG-15).
rheology. The resistance to motion is called viscosity
(APPG-15).
The material or film added to produce the adhesion is
known as the adhesive, and the material to which it 18. Newtonian: It is idea l fluid with constant viscosity
is applied is called the adherend. and demonstrates a shear stress that is proportional
to the strain rate. Newtonian exhibits a constant
8. The capacity for the adhesion is directly related to slope of shear stress plotted against strain rate i.e.,
the surface energy of the material. Because of the the plot is a straight Line.
higher surface energy of many restorative materials
compared with that of the tooth, there is a greater 19. Pseudoplastic: The vi scosity decreas es with
tendency for the surface and margins of the restoration increasing shear rate.
Dental ;lut.,e

20. Dilat ant: These liquids become more rigid with 28. Elastic/ Young's Modulus/ Modulus Of Elasticity:
increasing shear rate i.e., as the rate of deformation It describes the relative stiffness or rigidity of a
increases, these liquids become more viscous. material. It is the ratio of stresses to strain within the
proportional limit.
21. Thixotro pic: A liquid, which becomes less viscous and
more fluid under pressure is referred to as thixotropic. Modulus of elasticity
Dental prophylaxis pastes, plaster, resin cements and Tensile stress Compressive stress
(OR)
some impression materials are thixotropic. Tensile strain Compressive strain

22 . The hydrophobic materials (polysulphide, addition and UNITS= Giga Pascal or Gig a Newton/ m2
condensation silicones) have a water contact angle of
95 degrees whereas hydrophilic materials (Alginate, 29. The shear modulus can be calculated from the formula
Agar, and Polyether) will have a water contact angle G = 0.38 E.
of 30-35 degrees
Where, G = Shear modulus; E = Elastic modulus
23. Stress:
Stress is the force per unit area. It is the internal Thus, the shear modulus is always 38% of elastic
force exerted through out a unit of area that resists modulus.
an external force applied for the area. (AP-2013)
• Tensile stress tends to stretch or elongate a body. 30. Yield Strength:
• Compressive stress tends to compress a body. It is the strength at which a material begins to
• Shear stress resists the sliding of one portion of a function in a plastic manner. It is defined as the stress
body over another. at which a material exhibits a limiting deviation from
proportionality of stress to stress.
24. Strain:
It is expressed as change in length per unit length of 31. Resilience: (Energy)
body when a stress is applied. Resilience is basically an express of energy. It is defined
as the amount of energy absorbed by a structure when
25. Poisson's Ratio: it is stressed not to exceed its proportional limit.
When a object is subjected to a tensile or compressive
stress, there is simultaneous axial and lateral strain. 32. Toughness:
Within the elastic range, the ratio of lateral to the It is the amount of energy required to fracture
axial strain is called poisson's ratio. For an ideal a material and it is a measure of the resistance to
isotropic material of constant volume, the ratio is 0.5. fracture. Toughness is directly related to strength and
ductility of the material.
26. Elastic Limit:
It is the maximum stress that a material can withstand 33. Brittleness is opposite to that of toughness. It is
without permanent deformation. defined as the inability of a material to sustain plastic
deformation before the fracture occurs.
27. Proportional Limit :
It is the greatest stress that may be produced in a 34. Ductility: (APPG-15)
material such that the stress is directly proportional It is the ability of the material to withstand permanent
to strain. deformation under a tensile load without rupture.

For a material to satisfy HOOKE's law the elastic stress 35. Malleability:
must be proportional to the elastic strain. Hooke's It is the ability of the material to withstand permanent
law is applicable to all orthodontic wires except deformation under a compressive load without rupture.
super elastic A-NiTi wires (PGI June-12).
36. Flexibility is defined as the strain that occurs when
the material is stressed to its proportional limit.
=====================================================fr DENTAL MAT ER IALS SYNOPSIS J-~ J1 ===""""~
tc=J

GYPSUM PRODUCTS
TYPE - I TYPE - II TYPE - III TYPE - IV TYPE - V
Dental stone Class II dental Dental stone with
or hydro cal stone or imp roved high strength and
or class I dental stone or high expansion or
Other name Impression plaster Model plaster
stone den site or high extra hard, improved
strength dental dental stone.
stone
W / P ratio 0.5 - 0.75 0.45 - 0.50 0.28 - 0.30 0.22 - 0.24 0.18 to 0.22
Setting time 4 to 5 min. 12-15 min. 12-15 min. 12-15 min. 12-15 min.
- -
Setting
0.15% 0.30% 0.20% 0.10% 0.30%
expansion
Compressive
strength 800 1300 3000 5000 7000
(PSI Units)
• Final wash To fill the flask Construction Die material Improve die materia l
impressions in denture of casts
Uses
• Used in mucostatic construction
impression
technique

IMPRESSION MATERIALS
• Also known as
First dental elastomers
Vulcanizing impression materials
Mercaptan impression materials
Thiokol
• Contains Lead dioxide polymer that gives polysulphide its characteristic brown colour.
• The polymerization reaction is exothermic in nature.
• It has Long setting time of 8 to 12 minutes. It has unpleasant odor and colour. Both of
these properties cause patients discomfort. Heat and moisture accelerate the setting
time.
• It is messy and stains clothes.
Po Lysulp hides • Shows highest permanent deformation. If maximal accuracy to be maintained, the
cast should be poured within the first 30 minutes.
• Shows high polymerization shrinkage due to loss of the by-product (water).
• Polymerisation shrinkage and permanent deformation can be minimized by taking
minimum quantity of impression material and by using custom acrylic tray.
• Shows highest tear strength.
• Hydrophobic in nature. So the impression area should be dried before making an
impression.
• Shows high flexibility and so the material can be released from undercut areas with a
minimum stress.
• Causes Lowest cell death count and so the biocompatibility is best (AIIMS Nov-14).
• Polysulphides are the only impression materials that can be electroplated (PGl-2011)
Dental ;lut.,e

• Also known as Room Temperature (RTV) silicones.


• Highest curing shrinkage due to evaporation of ethyl alcohol byproduct. To avoid this,
cast should be poured within t he first 30 minutes.
• Less permanent deformation than polysulphides.
• Pleasant colour and odor.
Condensation
• Low tear strength.
silicon
• Hydrophobic in nature.
• Flexibility is high when compared with polysulphides. So, the impression can be
removed from undercuts without distortion.
• Has a limited shelf Life because of oxidation of the tin component.
• Handling with Latex gloves is contraindicated.

• Also known as polyvinyl siloxane or vinyl poly siloxane impression materials.


• No volatile BY-PRODUCT.
• Lowest curing shrinkage and lowest permanent deformation with best dimensional
stability. So casts can be poured after several hours.
• Extremely hydrophobic.
Addition silicones • Shows least setting time of all the elastomeric impression materials.
(pseudo plastic
impression • Handling with latex gloves is not indicated because, the sulfur present in the natural
latex gloves inhibits the setting of the addition silicone impression materials.
materials)
• Shows good shelf Life than condensation silicones.
• Hydrophobic in nature.
• Vinyl impression materials can be disinfected by immersing in 10% hypoch lorite or 2%
glutaraldehyde solutions.
• Sometimes hydrogen may be evolved and may cause pinpoint voids in stone casts.

• First elastomer to be developed primarily to function as an impression material. All the


other materials were adapted from other uses.
• Stiffest of all elastomeric impression materials. So extremely difficult to remove from
undercut areas because of the high modulus of elasticity.
• High tear strength next to polysulphides.
• Dimensionally more stable. The casts can be poured immediately or after several hours,
or after several days. Pouring up the impression and removing the cast several times
Polyether does not alter the dimensional stability of the polyether.
• Shows highest cell toxicity and so the biocom patibility is not good.
• Hydrophilic in nature (Note: Agar and alginate are also hydrophilic)

Which of t he fo llowing impression materials is NOT hydrophobic? (KAR-2013)


a) Polyethers b) Reversible hydroco lloid
c) Polysulfide polymer d) Condensation silicone
=====================================================fr DENTAL MAT ER IALS SYNOPSIS J-~ 33 i-=J===""""~

5. Amalgamation reaction:
DENTAL AMALGAM
i) For low copper a lloys
1. An amalgam is an alloy that contains mercury as one Ag 3Sn+Hg___.Ag2Hg 3+ Sn 8 Hg + Ag 3Sn
of its constituents (y) (yl) (y2) (y)
unreacted
2.
ii) For high copper admixed alloys
Composition ,of Conventional (low copper)
amalgams y+Ag-Cu+Hg ~ yl + Cu 6Sn 5 + (y+Ag-cu)
Silver Increases expansion, increases strength (ri) Unreacted
(65%) and whitens the alloy
Decreases expansion, decreases iii) For high copper single composition alloys
Tin (29%) strength, hardness and reduces tarnish
and corrosion resistance. Ag-Sn-Cu+Hg -----.y1 + 11 + Unreacted alloy
Particles (y)
Copper Increases hardness, strength and
6.
(<6%) expansion.
Zinc Scave11ger / deoxidiser, increases the Phase Importance
(< 1%) longevity of the restoration. Strongest phase, the more this phase
Ag 3sn (y) in the set amalgam, more will be the
3. CLASSIFICATION OF AMALGAM ALLOYS: strength.
A) Based on copper content:
• Base centered cubic lattice.
(i) Low copper -<6%
(ii) High copper - >6% Ag2Hg3 (Y1) • Increases the tarnish and corrosion
• Admixed - 9 - 20% resistance
• Sing le composition - 13 - 30% • Weakest phase

B) Based on zinc content: Sn 8 Hg (y2) • Decreases the tarnish and


corrosion resistance and increases
• Zinc containing - Contain more than 0.01%
of zinc. the creep
• Zinc free - Contain less than 0.01 % of zinc. Replaces y2 phase in high copper
Cu 6Sn 5 (11)
amalgams.
C) Based on alloy particle shape:
This phase occurs in some high
• Lathe cut - Irregular shape Cu 3Sn (i:)
copper single composition amalgams.
• Spherical
• Admixed - Contain both lathe cut and
7. The high copper alloys are superior to low copper
spherical alloys.
alloys because of their resista nee to tarnish, corrosion,
creep and high strength. This increase in physical and
4. Generations of amalaam:
mechanical properties is due to the elimination of
I Ag+ Sn (binary)
SnHg (y 2) phase.
II Ag + Sn + Zn + Cu
Ag-Sn + Ag-Cu (eutectic alloy). Ag- 8. Mercury alloy ratio according to EAMES or Minimum
III Cu particles act as strong fillers and mercury technique is 1:1 or 50% mercury. The
increase the strength of amalgam mercury content of the finished restoration should be
IV (High approximately 50 Wt.% while for spherical alloys it
Alloy+ 29% copper.
copper alloys) should be approximately 42 Wt.%.
Alloy+ Indium.
V
Indium acts as a scavenger. Increased dryness technique is another technique
VI Alloy+ Palladium / gold / platinum for proportioning mercury and alloy.
Dental ;lut.,e

9.
• Produced by milling or lathe cutting a cast ingot of amalgam alloy. The particles are irregular in
shape.
Lathe cut • Requires more amalgamation time.
alloys • Requires more mercury, hence have inferior properties.
• Less plastic and gains strength with greater condensation forces .
• The recommended mercury: alloy ratio for most modern lathe-cut alloys is 1:1 or 50% mercury.
• Produced by atomizing the liquid alloy. The particles are spherical shaped .
• Because of their smaller surface area, spherical alloys require less mercury and hence they have
better properties.
• Requires less amalgamation time.
Spherical • More plastic and gains good strength with lighter condensation forces.
alloys • Because spherical alloys are extremely plastic, a contoured and wedged matrix band is essential
to prevent overhanging margins, improper contacts and flat proximal contours.
• Larger condensers should be used for condensing spherical amalgams, as their resistance to con-
densation forces is less.
• The recommended mercury content for spherical alloys is 42%.

10. During setting amalgam undergoes 3 distinct 15. Any pain that occurs immediately after insertion may
dimensional changes: be due to high points of the restoration, fracture of
• initial contraction due to dissolution of alloys the tooth, etc.
particles in mercury and growth of y1 phase.
• expansion due to impingement of y1 crystals. 16. Manipulations that result in reduced residual mercury
• contraction due to absorption of unreacted mercury. favours contraction. They are
• Low mercury alloy ratio.
11. Modern amalgams exhibit a net contraction. • Higher condensation pressure
• Longer trituration ti me and use of small particle
12. According to ADA specification 1, amalgam should not alloy size.
expand or contract more than 20µm/cm measured at
37°C, between 5 minutes to 24 hrs after beginning of
17. Compressive strength of ama lgam should be minimum
trituration.
of 310 Mpa and the tensile strength should be in
between 48-70 Mpa .
13. Delayed expansion occurs when zinc-containing alloys
are contaminated by moisture during trituration /
18. Amalgams have high compressive strength and low
condensation reaching upto 400µm/cm. Delayed
tensile strength. It is for this reason cavosurface
expansion occurs 3-4days after insertion of amalgam.
angle should be 90° (butt type) to prevent fracture
This is due to formation and accumulation of H2 gas
of the marginal material. Bevels are not indicated for
in the restoration leading to pain, sensitivity and
amalgams.
protrusion of the restoration.

19. Increasing order of strength .


Delayed expansion occurs 3 - 4 days after insertion of
Low copper < admix alloy < si ngle composition
amalgams.
(Yz) < (Y1) < (y).
14. Mercuroscopic expansion occurs when mercury from Which of the following silver amalgam alloys have
Sn-Hg ( y2 ) reacts with Ag-Sn (y) particles. the maximum strength? (GCET-14)
a) Lathe cut b) Spherical
c) Admixed d) Single composition
=====================================================fr DENTAL MAT ER IALS SYNOPSIS J-~ Js i-=J===""""~

20. Creep is defined as the time dependant plastic Release of free mercury occurs if burnishing increases
deformation. The higher the creep, the greater is the the temperature above 60°C (140°F).
marginal deterioration.
• Low copper 0.8 - 8% 30. Finishing and polishing should be done 24 hrs. after
• High copper 0.4-0.1% insertion.

21. Tarnish is surface discoloration on a metal whereas 31. Use of dry polishing powders and disks is
corrosion is the actual deterioration of the material. contraindicated as they can raise the surface
temperature above 60°C (140° F) causing irreversible
Tarnish product Ag 2S is main tarnish pulpal damage. Wet abrasives in the form of pastes
product of amalgams should be used.

Corrosion products 32. Tin oxide, zinc oxide, pumice flour and precipitated
Low copper SnO, Sn-0-Cl chalk are commonly used for finishing and polishing
(Sn-Hg is involved) of amalgams.

High copper SnO, Sn-0-Cl, CuO 33. Polishing should be done with the rubber cup tipped
(Cu-Sn is involved) in such a way that the edge rotates from amalgam to
tooth.
22. Both low and high copper amalgams undergo chemical
and electro chemical corrosion. 34. The maximum level of occupational exposure
considered safe is 50 µgm of mercury/ m3 of air.
23. Electro chemical (galvanic, stress or crevice) corrosion
is not a mechanism of mercury liberation from set 35. The primary risk to dental personnel is from inhalation
dental amalgams. of mercury vapors.

24. The main objective of trituration is to wet the alloy 36. Old amalgam restorations should be removed with
particles with mercury. Increased trituration time or slow speed using water-cooling and high volume
speed shortens the working and setting times. evacuation.

25. Condensation is usually started at the center and 37. Contact dermatitis or Type IV hypersensitivity
condenser point is stepped by little towards the reactions can occur due to mercury allergy.
cavity walls.
38. Marginal defects are most frequently occurring defects
26. Condensation force is inversely proportiona l to square in amalgams.
of the diameter of the nib. Normally condensation
force is 3-4 lb (2 - 3 kg.) per increment. Condensers 39. MAHLER SCALE is used to describe the severity of
have serrated nibs. marginal defects. It has ratings from No. 1 to No. 11.

27. Larger condensers should be used for condensing Scale values of No. 2, 4, 6, 8, 10 represent the image
spherical amalgams, as their resista nee to condensation of restorations in the series of 5 photographs.
forces is less.
Scale va lues of No.1, 3, 5, 7, 9,11 are used to deal
28. The purpose of carving is to simulate the anatomy of with the severity of marginal ditching.
tooth structure. Carving should proceed in a direction
parallel to or slightly towards the margin of the
prepared tooth.

29. Burnishing of slow setting alloys is contra indicated


as it can damage the margins of the restoration.
Dental ;lut.,e

8. Gold alloys are heat hardened in order to improve


METALLURGY physical properties.
1. Application of 'Lost wax technique' for the fabrication
of cast inlay was first reported by Taggart (BHU-07). 9. Softening heat treatment (solution heat t reatment) is
indicated for structures that are to be ground, shaped
2. Noble metal alloys and base metal alloys are the or cold worked. The casting is heated for 10 min. at
common dental casting alloys. 700°C and immediately quenched in water. Ductility is
increased but tensile strength, proportional limit and
3. Noble metal alloys (Gold, palladium, platinum) hardness are decreased.
contain at least 75% of noble metal content while
predominantly base metal alloys (Ni, Ti, Co, Cr. .. ) 10. Hardening heat treatment (Age hardening) is
contain less than 25% of noble metal content. accomplished by aging the casting for 15-30 minutes
at temperature between 200-450°(. proportional
4. Pure gold is soft and ductile. So it is commonly alloyed limit, hardness and yield strength are increased but
with copper, silver palladium, platinum, zinc, indium ductility is reduced.
and nickel, etc to improve its properties.
11. Cobalt chrome alloys can be cold worked but cannot
5. be heat hardened, whi le gold can be heat hardened
but can not be cold worked.
Metal Properties

• Provides tarnish and corrosion 12. Base metal alloys have low density, high stiffness
Gold resistance (modulus of elasticity), more passivity, low proportional
• Increases ductility and malleability limit and less cost when compared to gold alloys.

• Increases the fusion temperature,


13. Casting shrinkage= Thermal+ Solidification shrinkage
Platinum • Decreases casting shrinkage,
• Decreases tarnish and corrosion 14. Higher melting alloys tend to exhibit greater shrinkage
• Similar to platinum but is less and that is why base metal alloys have higher casting
Palladium shrinkage (2.3%) than gold alloys (1.5%).
expensive than platinum
• Decreases fusion temperature,
15. Alloys used for metal ceramic restoration should have
Copper
• Principal hardener and gives the
• High fusing temperature to resist sag deformation.
alloy a red colour
• Coefficientofthermalexpa rnsion closer to Porcelain.
Zinc • Scavenger for oxygen
• Potential to bond with porcelain
• Its Oxides help in bonding of
Indium porcelain 16.
• Acts as grain refiner and scavenger. Greenish discoloration of porcelain due
Greening to silver vapour escaping from alloy
6. Classification and uses of casting gold alloys surface into porcelain during firing
Type I (soft) Small inlays Also called wet strength of gypsum
Green
Type II (medium) Inlays, 3/4th crowns, panties product. The wet strength is two or
strength
more times Less than the dry strength.
Type III (hard) Full crowns, short span bridges
Greening Observed in silica bonded investments
Type IV Long span bridges, clasps and
shrinkage due to drying of colloidal silica gel
( Extra ha rd) partial denture frame work
17. The formation of protective oxide film by a reactive
7. Type III and IV alloys can be heat hardened if they substa nee, which prevents corrosion and further
contain sufficient amount of copper. oxidation, is called passivation.
=====================================================fr DENTAL MAT ER IALS SYNOPSIS J-~ 3, ~J===""""~

Among aluminum, chromium and titanium, titanium 3. The 3 forms of Iron carbide are ferrite, martensite and
is the most corrosion resistant. For passivation at austenite:
least 12% of chromium should be present. Ferrite Marten site Austenite

18. Ni-Ti alloys exhibit shape memory and super elasticity It is pure It is formed It is the stable
iron at room when austenite form of iron at
Alloy Constituents
temperature is cooled rapidly temperature
Elgiloy Cobalt - Chromium - Nickel (quenched) between 912-
~-titanium Titanium - Molybdenum - 1394 °C
(TMA) Aluminum
Least Highest strength Moderate
Nitinol Nickel - Titanium strength strength
Titanium Titanium - Aluminum - Vanadium
Good Least corrosion Highest
alloys (Ti-6Al-4V) corrosion resislance corrosion
resistance resistance
19. "Tempering" treatment of alloys reduces hardness
but increases toughness Least used • Used for • 18-8 (18%
in dentistry making Cr, 8% Ni)
surgical Steels are
20. "Cold working"/ Strain hardening/ Work hardening
and cutting used for
increases hardness, strength but decreases ductility.
instruments making
(Age hardening is hardening heat treatment employed orthodontic
• Shape memory
for gold alloys) wires
is due to
change from • Greater ease
21. The effects of cold working can be reversed by simply austenite to of welding
heating the metal. It is called as "Annealing". martensite

STEELS
WAXES
1. Steels are iron based alloys that usually contain less 1.
than 1.2% carbon. The maximum carbon content of
Classification of waxes
chrome cobalt alloy is 0.5%.
• Inlay wax
2.
Pattern waxes • Casting wax

Carbon Steel Stain less Steel


• Base plate wax

Fe - 98% Fe - 81 %
Processing waxes • Boxing wax

C - 1-1.2% Cr - 18%
• Utility wax
Composition
Mn - 0.2% C - 0.61%
• Sticky wax

Si - 0.2
Impression waxes • Corrective wax

Corrosion
• Bite registration wax
Very low High
resistance
2. Comgosition of waxes
Cutting Low and becomes
effi ciency
High
dull rapidly
• Basic constituent

Stiffer, stronger, Hardness is less


Paraffin • Have tendency to flake
Physica l • Less smooth and glossy
harder but more than carbon steel
properties Gum dammar • Improves smoothness
brittle but is not brittle
Superior at high Superior at
(COMEDK-08) • Decreases flaking

Efficie ncy
speeds and low speed and Carnuaba / • To decrease flow
used for cutting used for cutting Candelilla • Imparts glossy surface
enamel dentin Ceresin • May replace part of paraffin.
Dental ;lut.,e

3. Inlay waxes should have a maximum flow of 1% at


CASTING PROCEDURE
37°C to permit carving and removal of wax pattern
and a minimal flow of 70% at 45°(, which helps in 1. Sprue former
insertion of wax into the prepared cavity. • Provides channel through which molten alloy can
reach the mold after the wax has been eliminated.
4. Waxes have low thermal conductivity and high • Its diameter should be the same as the thickest area
coefficient of thermal expansion (350 x 10-6/°C) of the wax pattern.
(BHU-07, APPG-15]. • It should be attached to the thickest portion of the
wax pattern to prevent turbulence.
5. Glass transition te mperature of in lay wax is 35°C. • Its length should be such that there is 6mm of distance
from the casting ring to t he end of mold cavity.
6. Direct technique Indirect technique • It should be sprued at 45° angles to the proximal area.
Employ Type I (or) Type B Employ Type II (or)
2. Die materials
waxes Type C waxes
• Greatest accuracy with very slight
Medium wax
expansion during setting
(Paraffin should have a high Gypsum
Soft wax • Have least resistance to abrasion
melting point than that (Die stone)
used for Type II waxes) • Die material of choice with hydro
Pattern is subjected to Pattern is not colloid impressions
a change from mouth to subjected to change • They shrink on setting
room temperature so more from mouth to room Epoxy • High resist ance to abrasion
distortion occurs. temperature resins • Most common ly used die material in
Diffic ulty in carving and Easier for carving and labs
removal of pattern removal of pattern
Electro
• Has a lethal potential due to the
usage of silver cyanide
7. Distortion is the most serious problem, which is due plated
to release of inherent stresses. silver • Polysulphide impressions can be
easily electro plated
8. Main components of base plate wax are paraffin 75% Amalgams ---------------
parts, bee's wax12% and added colors. Ceramics ---------------
Silico
9. phosphate ---------------
Type Uses
To build up vertical walls around cements
Boxing/
impression to produce desired size
Carding 3. Ring liners (Asbestos, cellulose or ceramic liner) are
and form of base cast.
placed inside the casting ring, which will allow for
To establish the initial arch form,
mold expansion.
vertical dimension and plane of
Base plate
occlusion by using as a tray. 4. Burn out is the process of elimination of wax from
To prepare a desirable contour mould cavity and achieving thermal expansion.
Utility I
to a perforated tray for use with
Periphery 5.
hyd rocolloids.
To prepare pattern for metallic Investment Burnout temperature
Casting
fra mework of R.P.D. 400°C in 20 min. and maintain it
Gypsum
For joining meta l parts before for 30 min ., raise the temperature
ivestments
soldering and for joining fragments to 700°C and maintain it for 30 min .
Sticky
of broken dentures before repair.
Phosphate Tern perature range from 750-900°C
To record non-undercut edentulous investments and maintaining it for 30 min
Impression
portion of the mouth.
=====================================================fr DENTAL MAT ER IALS SYNOPSIS J-~ 39 ===""""~
i-=J

CASTING DEFECTS

i) DIMENSIONAL ERRORS IN CASTING


Problem Cause Precaution

Casting too large Excessive expansion Use correct tern perature and correct investment
Casting too small Too little mould expansion Heat the mould sufficiently
Distorted casting Distorted wax pattern Correct handling of wax

ii) ROUGH SURFACE ONCASTING


Problem Cause Precaution

a) Investment Avoid overheating of mould and alloy


breakdown
b) Air bubbles on Wax • Correct use of wetting agent
Rough surface • Correct vacuum investing
c) Weak surface of • Avoid too high investment w/p ratio .
investment • Avoid dilution of investment from too much wetting
agent
Fins on casting Cracking of investment • Avoid too rapid heating of investment

iii) POROSITY
Type Cause Prevention

• Shrinkage of molten alloy on cooling. • Use thick sprue.


Suck back
• Due to premature termination of • Placing reservoir.
(Localized
the flow of molten metal during • Flaring the point of attachment
shrinkage, shrink
solidification (KCET-08) • Attach sprue at thickest portion of wax
spot porosity)
• Irregular voids in casting pattern
• Escape of air is prevented due to • Adjusting the sprue such that there
bulk of investment is only 6mm (1/4 inch) distance from
Back pressure
• Rounded margin in casting casting ring to nearest part of pattern.
• Sufficient casting pressure.
• Proper burnout.
Pin hole, • Avoid overheating and prolonged heating
Spherical voids in casting. Due to rapid
subsurface, • Use long and thin sprue
entry of alloy in to the mold
gaseous inclusions
Micro porosity Rapid solidification shrinkage Increase melt temperature

iv) INCOMPLETE CASTING


Problem Cause Precaution

Incomplete casting • Incomplete wax elimination • Proper venting


• Insufficient venting • Sufficient casting pressure
• Highly viscous molten alloy • Proper burnout
• Proper heating of alloy
Dental ;fc&.,e

DENTAL INVESTMENTS
INVESTMENT PROPERTIES
• Alpha- Hemihydrate is the binder
• Useful for casting gold alloys
• When heated above 700°( decomposition occurs and 50 2 is emitted which contaminates the casting
Gypsum • Type-I - Casting shrinkage compensated by thermal expansion
bonded • Type-II - Casting shrinkage compensated by hygroscopic expansion
• Type-III - For partial dent ure construction with gold alloys
• Normal setting expansion is 0.5%
• Hygroscopic expansion is 1.2-2%
• Thermal expansion is 1-2%
• Magnesium oxide and mono ammonium phosphate is the binder
• Used for high melting alloys like cobalt/chromium and metal ceramic restorations
• Colloidal silica suspensions are used in place of water which helps in greater expansion of the
Phosphate
investment
bonded
• Phosphate investment with carbon is used if casting alloy is gold
• Phosphate investment without carbon is used for carbon sensitive alloys such as base metal,
silver palladium alloys
• Silica is the binder which may be derived from ethyl silicate or silica gel
• Used for high fusing base metal partial denture alloys
Silica bonded
• In addition to settin g shrinkage "green shrinkage" occurs due to loss of alcohol and water from
the gel.
• The investment can be heated upto 1200°(

CEMENTS

Zinc oxide

Phosphoric Polyacrylic
acid acid

Silica

ZPC - Zinc phosphate cement


PCC - Polycarboxylate cement
GIC Glass Ionomer cement
=====================================================fr DENTAL MAT ER IALS SYNOPSIS J-~ J1 ===""""~
tc=J

2. Most dental cements set by acid-base reaction except (A) SILICATE CEMENT
calcium hydroxide and resin cement. Resins set by
polymerization reaction. 1. Composition:
Powder Liquid
3. Restorative cements can be classified as temporary
or short-term (for days to weeks), intermediate term i) Silica - 40% i) Phosphoric acid - 52%
(for weeks to months) and permanent or long term ii) Alumina - 30% ii) Aluminum phosphate
(for years). -2%
iii) NaF }
Cryolite 19% iii) Magnesium phosphate
4. Fluoride releasing cements are silicate, GIC,
CaF2 -6%
Silicophosphate arnd polycarboxylate (least).
iv) Calcium phosphate iv) Water - 6%
Fluoride contributes to the anticariogenic property
by inhibiting demineralization and enhancing 2. The purpose of fluoride salts is to lower the fusion
remineralisation of carious enamel. temperature of the glass. Thus they are called as
'ceramic fluxes'.
Among the fluoride releasing cements the amount
of fluoride release and longevity of release are more 3. Siliacate cement was the first transclucent filling
in GIC. material by fletcher in england (AP-2013) in 1878.

5. Among the cements ZOE cement is the most soluble 4. Properties:


and resin cement is the least soluble in oral fluids.
• Silicate is the strongest of all dental cements. It
6. Silicate cement has most irritant effect on pu lp while has a compressive strength of 180 Mpa.
ZOE cement has least irritant effect on pulp.
• Setting time is 3 - 8 min.
7. Among all the cements, silicate cement has highest • Its hardness (70 KHN) and coefficient of thermal
compressive strength while resin cement has the expansion is closer to dentin.
highest tensile strength.
• It is a severe irritant to the pulp. At the time of
8. GIC and polycarboxylate are the cements that produce insertion it has a pH of 2.8 and even after one
a truly adhesive (chemical) bond to tooth structure. month, it remains below 7.

• Silicate has high solubility and disintegrates


9. The bonding of resin cements to tooth occurs by readily in oral fluids. They become stained over a
micro-mechanical retention. period of ti me.

10. Factors increasing the setting time: • It has anticariogenic property due to release of
• Cooling the mixing slab fluoride and so the incidence of secondary caries
and contact caries is less.
• Decreased powder liquid ratio
• Prolonged mixing time • Its refractive index is similar to enamel and dentin.
• Loss of water from the liquid It is used as an anterior esthetic restorative
material.

• It is contraindicated in mouth breathers, as its


surface becomes rough and opaque when allowed
to dry.

5. Finishing and polishing should be delayed for several


days. The surface should be protected by coating with
varnish.
(B) GLASS IONOMER CEMENT 5. Light polymerization:
The powder contains initiators for light curing and
1. Other names: liquid component is modified with hydroxy ethyl
• Polyalkenoate cement methacrylate (HEMA).
• Man made dentin
The polymerization starts when exposed to light and
• Dentin substitute
subsequently followed by acid base reactions. This is
• Alumina silicate poly acrylic cement (ASPA) called "DUAL CURE" GIC.

2. Composition: 6. Properties:
Powder • Silica - 35-50% i) GIC has low fracture toughness and wear resista nee.
• Alumina - 20-30%
ii) Very sensitive to moisture, especially during initial
• NaF - 3-6%
setting reaction. During this period, absorption
• AlF3 - 1.5-2.5% of water leads to weak cement and over drying
• Aluminum phosphate - 4-12% will lead to cracks in the cement. Therefore, the
• Traces of barium, Strontium for surface of cement should be protected by coating
radiop acity with varnish or cocoa butter during setting.
Liquid • Polyac rylic acid - 45%
iii) Bonds chemically to tooth structure - The bond of
• Itaconic acid } enamel is always higher than that of dentin.
• Ma leic acid - 5% (Decreases
• Tricarballylic acid viscosity) iv) Relatively biocom patible - The pulpal reaction
• Tartaric acid - increases working time is greater than ZOE but less than zinc phosphate
cement.
• Water - 50%
v) Antica riogenic property is due to fluoride release.
The powder is referred to as 'ion leachable glass'
7. Powder liquid ratio is 3:1 by weight. Mixing should be
3. Classification:
done by agate or plastic spatu la.
Type I Luting cement

Type II Restorative cement 8. 10% polyacrylic acid should be used for conditioning
the cavity surface before insertion of the cement.
Type III Liner
9. Final finishing is done after 24 hours of insertion.
Type IV Fissure sealant

Type V Orthodontic cement 10. Modifications of GIC:


a) Miracle Mix (or) silver cermet:
Type VI Core build up cement Silver-Tin alloy powder is added to GIC powder.
Type VIII } Posterior packable GIC for None of the properties were improved and it gave
Type IX atraumatic restorations a gray or blackish colour to the cement. It is also
called as silver alloy admix.
Among the first three types i.e, type I, type II and
type III (conventional and light cure) types the b) Glass cermet or cermet:
highest cumulative release of fluoride after 30 days is Glass and metal (Silver-tin-titanium) powders were
from glass ionomer liner (light cure) [ KCET-08) sintered at high temperature and made to react
with liquid. It improved the fracture toughness and
4. GIC was introduced as a potential replacement for wear resistance and at the same time maintained
si licate cement. It has been evolved as a hybrid from the esthetics.
the silicate and polycarboxylate cement.
=====================================================fr DENTAL MAT ER IALS SYNOPSIS J-~ 33 ===""""~
i-=J

c) Resin modified GIC: (C) ZINC PHOSPHATE CEMENT


BisGMA, TEGDMA are added to powder and HEMA to
the liquid. With exposure of light polymerization is 1. Other names:
initiated along the methacrylate groups. After that • Crown and bridge cement
the liquid reacts wit h glass particles through acid • Zinc oxyphosphate cement
base reaction. It improved the wear resistance and
decreased the sensitivity to water attack. 2. It is the oldest of the luting cements and serves as
a standard fo r comparison with new cements.
d) Compomer (Polyacid modified composite resins):
It is a combination of composite and GIC. Glass
3. Composition:
particles are partially silanated (for bonding
with the matrix) and are added as fillers in the Powder
composite resin. There is no water in the reaction. Zinc oxide (90%) Basic ingredient
The properties were inferior to composites but
Magnesium oxide(8.2%) Aids in sintering
superior to GIC and resin modified GIC.
Other oxides (0.2%) Improves smoothness of
e) Bilayered or Sandwich Restorations: (Bismuth, calcium) mix
In this technique GIC is used as a liner under
Silica (1.4%) Filler
composites restorations. It increases the retention
form as GIC bonds both the tooth and composite Liquid
and the fluoride content reduces secondary caries.
Phosphoric acid (38.2%) Reacts with zinc oxide
f) Tunneling restorations: Water (36%) Controls rate of reaction
Joining the occlusal lesion with the proximal lesion
Aluminum or Retards reaction
by means of a prepared tunnel under the involved
Zinc phosphate (16.2%)
marginal ridge. The margina l ridge remains intact. GIC
is used as the restorative material in this technique. Aluminum (2.5%)
---------
Zinc (7.1%)
g) Atraumatic Restoration (ART)
Involves removal of affected tooth structure with
4. Setting reaction:
hand instruments, followed by restoring with GIC
Phosphoric acid reacts with zinc oxide forming zinc
material (GC Fuji VIII).
phosphate. Aluminum of the liquid is essential for
cement formation. It forms complexes with phosphoric
h) High viscosity GIC:
acid to form zinc alumina phosphate gel.
Used for Atrau ma tic restorative treatment. They
contain small practice sizes and a high P/L ratio,
5. Adding small quantity of powder into liquid in
yielding greater compressive strength and excellent
increments increases the sett ing and mixing time and
packa bility. Also used for core buildups, primary
permits more powder to be incorporated into the mix.
tooth fillings and intermediate restorations.
6. Cooling the mixing slab markedly increases the setting
i) Calcium aluminate GIC:
time. It is the most effective method of controlling
It is a hybrid product of ca lcium aluminate and
the setting ti me.
GIC. The GIC components are responsible for
early properties (i.e., setting time, viscosity and
7. Prolonged spatulation ret ards the setting reaction as
strength). The calcium aluminate contributes
the formed matrix is effectively destroyed and extra
to basic pH, biocompatibility and reduction in
time is needed to rebui ld the bulk of the matrix. (In
microleakage. Also ca lled as hydraulic cement.
dental stones, prolonged spatulation accelerates the
setting reaction)
Dental ;lut.,e

Increased spatulation of ZnP04 cement (GCET-14) (E) ZINC OXIDE - EUGENOL CEMENT
a) Decreased working time
b) Increased setting time 1. Classification:
c) No effect on the setting time • Type I For temporary cementation
d) No effect on the working time
• Type II For permanent cementation

8. The acidity of the cements is quite high as the time • Type III - Temporary filling material and base
they are inserted in the tooth. Fresh mix has a pH • Type IV - Cavity liners
of 2 and then increases rapidly in 24-48 hours. Any
damage to pulp occurs during the first few hours after 2. Composition:
insertion.
POWDER
(D} ZINC POLY CARBOXYLATE CEMENT Zinc oxide - 69% Principal ingredient
Magnesium oxide -29 .3% To reduce brittleness
1. First cement developed with a potential for adhesion
to tooth structure. Zinc stearate - 1% Accelerator, plasticizer
Zinc acetate - 0.7% Accelerator
2. Composition:
Magnesium oxide Reacts with eugenol
Powder Like ZnO
Zinc oxide Basic ingredient LIQUID
Magnesium oxide Modifier Eugenol - 85% Reacts with zinc oxide
Bismuth and Improves Olive oil- 15% Plasticizer
Aluminum oxide smoothness of mix
Stannous fluoride Anticariogenic property 3. Addition of drop of water, alcohol and acetic acid
accelerates the setting reaction while addition of a
Liquid
drop of glycerin retards the reaction.
Polyacrylic acid
Itaconic acid
Maleic acid
Tricarbalyllic acid
} decreases viscosity
4. Prope rties:
• Solubility is highest among dental cements.
• Least irritating among all dental cements, so called
3. Properties: as pulp friendly cement.
• Because of the larger size of the polyacrylic acid • It has obtundant (or) soothing effect on the pulp
molecules, it is less irritant to pulp than zinc in deep cavities.
phosphate cement.
5. Modified ZOE cements:
• The pH of liquid is 1. 7 and that of freshly mixed
cement is 3-4. After 24 hours, pH of the cement is i) EBA - Alumina modified cement
5-6. Powder Liquid

• Cement bonds chemically with the tooth structure Zinc oxide - 70% EBA - 62.5%
due to chelation of the carboxyl group with calcium Alumina - 30% EL1genol - 37.5%
in the tooth structure.
ii) Polymer reinforced ZOE cement.
• Unlike ZnPo 4 cement, adhesion is better to smooth
surface than rough surface. iii)Cements containing vanillate esters.
• These cements are good thermal insulators and so
can be used as base material.
=====================================================fr DENTAL MAT ER IALS SYNOPSIS J-~ Js i-=J===""""~

(F) CALCIUM HYDROXIDE CEMENT 4. Properties:


They neither possess mechanical strength nor provide
1. Ca (OH)2 is a relatively weak cement. thermal insulation because of its thin fi lm . They are
insoluble in oral fluids.
2. Applications:
• For direct and indirect pulp capping. 5. Contraindications:
Varnish interferes with
• In apexification technique. Composite resins
polymerization.
• In pulpotomy (apexogenesis) Varnish interferes with the
GIC & Polycarboxylate
• As a root cana l sealer potential for adhesion
• As a root cana l disinfectant
6. Dentin bonding agents have replaced the role of
• As a cavity liner and base
varnish in sealing the restorations.

3. Properties:
(H) CAVITY LINERS
• The solubility in water is high. So it should not be
exposed at the margins when used as a liner.
1. A cavity liner is used like a cavity varnish to provide a
• Solubility is higher when exposed to phosphoric barrier against the passage of irritants from cements
acid and ether, so care should be taken during acid or other restorative materials and to reduce the
etching and application of varnish . sensitivity of freshly cut dentin .
• The cement is alkaline in nature. It has a pH of 11.
The high alka linity and its consequent antibacterial 2. The liners are volatile or aqueous suspensions of zinc
and protein-lysing effect helps in format ion of oxide or calcium hydroxide that can be applied to a
"Reparative dentin". cavity surface in a relatively thin film.

• It is an effective antibacterial agent. 3. The suspension liners neither possess strength nor
provide thermal insulation.
(G) CAVITY VARNISH
4. Hard setting Ca(OH) 2, ZOE (Type IV) and GIC liners
1. Cavity varnish is a solution of one or more resins placed in thickness of 0.5mm, have adequate strength
which when applied onto the cavity walls, evaporates to withstand the condensation forces.
leaving a thin resin film, that serves as a barrier
between the restoration and t he dentinal tubules. 5. If removal of infected dentin does not extend deeper
than 1mm. from initially prepared pulpal or axial wall
2. Composition: - No liner indicated.
Natural gum such as copal, resin or synthetic resin
dissolved in organic solvent such as alcohol, acetone Very deep excavations and known pulpal exposures -
or ether. Ca(OH) 2 liner. If excavation is in between above two
- ZOE liner
3. Applications:
• In amalgam restorations it decreases the micro 6. Calcium hydroxide (or) GIC liner shou ld be used under
leakage due to margin sealing. composite restorations. ZOE liner is contraindicated
as the eugenol in the cement may int erfere with
• It also prevents discoloration by preventing the
polymerization of composite.
penetration of silver ions into the dentin.
• It may be used as a surface coating over GIC, 7. Calcium hydroxide stimulates reparative dentin
silicate restorations to protect from dehydration. formation while ZOE has an obtundant effect on pulp.
• It prevents galvanic shock if applied over metallic
restorations.
Dental ;lut.,e

(I) CEMENT BASES with acid-containing side chains such as phosphoric


acid. Upon contact with tooth surface, the acidic
1. A base is a thicker layer of cement placed under groups bonds to calcium ions in the tooth structure
permanent restoration to encourage recovery of the making the cement so ca lled Self adhesive resin
inj ured pulp and to protect the pulp from thermal, cement.
chemical and galvanic insults.
4. Polymerization occurs by chemical, light or dual cure
2. The base serve as substitute of dentin that has been mechanisms.
destroyed by caries or cavity preparation.
5. Dual cure are most commonly used than light cure
3. Low strength bases - ZOE, Calcium hydroxide because with light cure there might be incomplete
polymerization of the cement under a prosthesis.
High strength bases - Zn PO 4 , GIC, polycarboxylate
6. Dual cure should not be used wit h prosthesis thicker
4. Properties: than 2.5 mm. In this situation chemica l cure cement
is best.
i) for effective therma l protection the base should
have a minimal thickness of 0.75 mm
7. Procedure of treatment of prosthesis:
ii) The base must have sufficient strength to withstand
fo rces of condensation and masticatory stresses. NOTE: Procedure for preparing tooth surfaces remains
iii) same for each system.

Restoration Base
a) Metallic prosthesis:
Amalgam Ca(OH) 2 or ZOE The bondable surface of one prosthesis should be
Direct filling gold ZnP0 4 , GIC or Polycarboxylate roughened.

Composite Ca(OH)z or GIC • For base metal - Grit blasting with alumina
(or) Electrochemical etching
5. Base does not prevent micro lea kage and acid • For noble alloys - 0.5 µm layer of tin oxide
penetration when used with amalgam or gold foil. should be deposited
A varnish or dentin -bonding agent is used to assist electrochemically
sealing of the restoration.
After roughening the surface should be silanized.
6. With ZnP0 4 as a base, the sealant should be applied
first followed by base. b) Polymeric prosthesis:
• Surface should be grit blasted
7. If the base is Ca(OH) 2, ZOE, GIC or polycarboxylate, the
• Some systems have a special adhesive based on
base should be placed first followed by the sealant.
the same monomer that is used to fabricate the
prosthesis. Atleast 30 minutes must be allowed
(J) RESIN CEMENTS:
for adequate penetration and diffusion of the
adhesive in to the prosthesis to occur before
1. These are low viscosity versions of composites and
are insoluble in oral fluids. curing.

c) Ceramic prosthesis:
2. A dentin bonding agent is needed to promote the
adhesion of resin cements to dentin. • Resin cements are luting agents of choice for
all ceramic prosthesis.
3. But recent developments eliminated the steps of • For feldspathic (silica basaed) porcelain tissue
etching, priming and bonding. These new resin surface should be etched with HF and a silane
cements rely on modified dimethacrylate monomers coating is applied.
=====================================================fr DENTAL MAT ER IALS SYNOPSIS J-~ ":31 ~J===""""~

• For alumina and zirconia based ceramics which • ELECTRALLOY


do not contain silica, etching and Silanization - Electrolytic precipitate alloyed with 0.1 %
are not effective. calcium.
- Calcium increases the strength and hardness.
d) Orthodontic brackets:
Tooth facing side, requires mechanical retention in C) Powdered Gold
the form of: • It is chemically precipitated gold powder with
an average particle size of 15µm.
• Metal mesh for metal bracket
• Powdered gold wrapped into cohesive gold foil,
• Dimples or ridges for ceramic/ polymer bracket
creating balls is called goldent.

NOTE : Orthodontic bands are cemented with GIC -


4. Cohesion of gold foil is an example of atomic attraction
because their removal is facilitated by using air to
and welding.
dessicate and weaken the cement
5. Non-cohesive gold is made cohesive by heating it
DIRECT FILLING GOLD before insertion into the cavity. This process is called
Degassing/ Annealing/ Desorption.
1. Direct gold are those gold restorative materials that
Form of direct gold Method of degassing
are manufactured for compaction directly into the
prepared cavity. Gold foil Bulk annealing on mica tray
Goldent Ethanol flame
2. 100% gold or 24 karat gold is used for this purpose.
Electra lloy Electric annealing
3. Forms of DFG:
6. The cavity preparation for direct gold restorations
A) Gold Foil: should be with sharp internal line angles and point
• It is manufactured by beating pure gold into angles which allows a convenient starting.
thin sheets.
7. Condensation is a process of cold working, hence it
• Gold foil size is 4x4 inches. increases the hardness, strength and result in fibrous
• The sheets of Gold foil which weigh 4 gms. are grain structure.
termed as No. 4 foils
8. Force of condensation is i nversely proportional to
• Surface impurities and foil treated with ammonia the square of the diameter of the nib. The forces of
(MHCET-15) makes the gold foil non cohesive. condensation must be 450 to ,cavity walls and floors and
• It is used generally for the external surface must be directed 900 to previously condensed increment.
veneering of the restoration
9. Stepping is the process of condensation whereby each
time condenser overlaps the previous compaction .
B) Electrolytic Precipitate
• MAT GOLD: 10. Bridging is the creation of void spaces due to failure
- It is used as bulk filler to completely compact each increment of gold.
- It is available as strips or cakes
11. DFG is considered to be ideal for Class V cavities and
• MAT FOIL the most desirable outline form for a Class V cavity is
- Mat gold wrapped in gold foil trapezoidal form .
- It is also used for veneering the external
12. A bevel, which is not greater than 0.2mm in width,
surface of the restoration like gold foil.
can be placed in Class I cavities.

13. Percentage elongation is the property of the gold that


allows it to be burnished.
Dental ;lut.,e

14. Preparation of direct filling gold is highly technique 3. Chemically activated composites:
sensitive procedure and is contra indicated in chi ldren i) Supplied in two pastes
and elder adults who cannot sustain the lengthy Contains the initiator Benzoyl
procedures and high condensation forces. Basic paste
peroxide.
Contains tertiary amine as
COMPOSITES Accelerator
activator
paste
(N-dimethyl-P- toluidine)
1. Composite is a compound of two or more different
constituents wit h properties that are superior to
ii) It is manipulated by mixing the proper proportions
those of the individual constituent.
on a mixing pad with an agate spatula (metal
spatula discolors the material).
2. Composition:
a) Resin matrix: The matrix consists of Bis-Phenol iii) Cavity should be slightly overfilled.
Glycedylmethacrylate (BisGMA) and Triethylene
Glycol dimethacrylate (TEGDMA). TEGDMA is iv) Shrinkage occurs towards center of the material.
used as a viscosity controller.
4. Light activated composites:
b) Fillers - Silica
• Consists of a sing le paste
Addition of fillers increases the strength,
hardness, abrasion resistance and decreases the Initiator - camphoroquinone
polymerization shrinkage and water sorption. Activator - diethyl-amino-ethyl-methacrylate
(amine) (or) Diketone
c) Coupling agent - Organosilane, zirconates (or)
titanates. They bond the fi ller particles to the • They interact when exposed to light at wavelength
resin matrix. of 400-500 nm i.e., blue region of the visible light
spectrum.
d) Inhibitor - hydroquinone • Ultra violet Light curing is not indicated because
Prevents premature polymerization of its limited depth of penetration through tooth
structure. It also poses a potential health hazard.
e) Opacifiers and coloring pigments
• The source of light is a t ungsten ha logen bulb.
Titanium dioxide ( Ti02) and aluminum oxide
are the opacifiers. • The tip should be kept as close as possible to the
restoration and should be cured in increments.
The coupling agent between fi ller particle and • Each in crement should not be greater than 2mm
resin is (GCET-14) and the exposure time should be 40-60 secs.
a) Benzoyl peroxide
• Darker shades require Longer exposure time.
b) Benzoin methyl ether
c) Vinyl silane d) Hydroquinone • Light emitted may cause reti nal damage and can be
prevented by using protective eyeglasses.
To increase radiopacity of composite
a) Aluminium oxide is added • Shrinkage occurs towards the light source.
b) Ferric ch loride is added
c) Sodium ch loride is added 5. Cavity preparation for composites is beveled except
d) Iron oxide (AIIMS MAY-14) when margins are in cementum
Which of the following is added in composites
to provide opacity? (PGI Dec-2011)
a) BIS-GMA b) TEG-DMA
c) Ti02 d) None
=====================================================fr DENTAL MAT ER IALS SYNOPSIS J-~ 39 ===""""~
i-=J

6. CLASSFICATION OF COMPOSITES AND THEIR PROPERTIES

• Contain filler particle size of 8-12 micrometers.


• It is the composite with largest filler size.
Conventional
composite
• High strength and hardness.
• Less water sorption and co-efficient of thermal expansion
• Polishing is difficult and results in rough surface that tends to retain stains.
• Contain smallest filler particle size 0.04-0.4 micrometers.
• Colloidal silica is used as a filler

Micro filled
• It has lowest filler content of 50% Wt
composite • Lowest strength and hardness
• Highest thermal expansion Co-efficient and water Sorption
• Excellent esthetics due to the increased smoothness

• Contain Filler particles of size 1-Sµm


• It has the good surface smoothness like microfilled com po sites and improved physical
Small particle properties like conventional composite
• It has the highest compressive strength of all the composites
Hybrid • It has filler particle size of 0.6-lmicro meters
composite • They have smooth finish and better esthetics than small particle but yet have similar physica l
properties.

7. Properties: c) Toughness:
a) Degree of conversion ( DC) • The interfacial bond of strong filler with weak
• Measure of °lo of C-C double bonds that have resin matrix contributes to toughness. More
been converted to single bonds to form a energy is needed for the crack to propagate.
polymeric resin. This makes composite tougher.
• High DC, better the physical and mechanical • If coupling agent is absent, the filler particles
properties. cannot absorb stresses in the matrix and act as
if they were voids, thereby weakening instead
• D.C. depends on factors such as resin
of strengthening the matrix.
composition, light transmission and
concentrations of initiator, accelerator and • If the interfacial bond is weaker than the
inhibitor. matrix, then a process called 'crack blunting'
occurs.
• DD.C of 50-70% can be achieved and there is
difference in chemical and light cure.
d) Curing shri nkage & wear:
Explained in Subject - Operative Dentist ry
b) Mat rix const raint :
• Filler partials bonded to matrix reduces thermal
8. Recent advances:
expansion and contraction of composite.
I) In Monomers:
• When temperature rises, matrix tries to expand
but due to the presence of filler that has lower a) Poly carbonate dimethacrylate:
or zero CTE, prevents or reduces the expansion. • Uses carbonate links instead of urethane links
• During cooling, matrix contracts but the to connect methacrylate ends to the control
contraction is prevented or reduced by the section of monomer.
presence of fillers. • Advantages:
- Packable like amalgam
Dental ;lut.,e

- Bulk curing b) Polyhedral oligomeric silsesquioxane (POSS).


- Less curing shrinkage • Is a hybrid like ormocer but also contain dispersed
Eg. ALERT fillers inaddition to inorganic siloxane network.
• Advantages:- High ly polishable
Wear resistance
b) High Molecular weight urethane with a rigid centra l
Eg: Artiste nano-hybrid composite
section and flexible end groups. Eg. KALORE
• Advantage - Less curing shrinkage
III. In photo curing:
MARC DEVICE (Managing accurate resin curing)
c) Dimethacrylate with a bulky, space filling control
group • Clinicians have inadequate means to determine
• Eg: Venus Diamond whether sufficient energy has. been delivered to reach
• Uses TCDDMA (Tricyclodecane dimethacryloxy an adequate cure level and depth of cure.
methylene), a bulky space fi lling monomer. • This. system measures both irradiance and energy
received by simulated preparations in a mannequin
• Advantage:
head.
- Lowest curing-stress resins.

BONDING
d) High molecular weight phase-sepa rating dicarbonate
with hydrophobic side chains. 1. Bonding of composites to the tooth structure is by
• It has also a bulky central group, but is more Micro-mechanical bonding
hydrophobic. Eg: 'N' Durance.
• Advantages: 2. Enamel bonding:
- Reduced water absorption Acid etching is usually done by 37% (30-50%)
phosphoric acid, which results in the formation of
- Reduced solubility
enamel tags into which the resin will fit.
- Reduced curing stress
3. Concentration greater than 50% results in the
e) Silorane-Ring opening tetra fu nctional epoxy formation of monocalcium phosphate monohydrate,
s.iloxane. Eg: Filtek LS. which prevents further dissolution whi le below 30%,
• Different chemistry of cationic ring opening results in dicalcium phosphate dihydrate that cannot
mechanism be easily removed.

• Combination of silorane and oxirane


4. Length of application of an etchent is 60 secs
• Advantages: (currently 15 secs.). Fluoride treated teeth and
- reduced curing shrinkage primary teeth require longer time due to the presence
of more aprismatic enamel .
II. In filler phases:
5. Maleic acid, citric acid and oxalic acid are used to
a) Organically modified ceramic oligomers (ORMOCER)
etch both enamel and dentin simultaneously.
o Hybrid structures containing organic reactive
monomers bound to inorganic siloxane network. 6. Enamel bond agents are usually low viscous unfilled
o Advantages: BisGMA resins and they enhances the wettability of
- Biocompatibility etched enamel.
- Excellent esthetics
7. Dentin bonding protocol is:
- Reduced shrinkage
• Etching/conditioning - Step 1
Eg. Ceram 'X' • Application of primer - Step 2
• Application of bonding agent - Step 3
=====================================================fr DENTAL MAT ER IALS SYNOPSIS J-~ J1 tc=J===""""~

8. Dentin bonding is difficult because dentin is a living


heterogenous tissue, and contains plenty of fluids.
DENTURE BASE RESINS

1. Thermoset resins - Become permanently hard when


9. CONDITIONERS are agents, which aids in removal or
heated above a critical temperature and they do not
modification of smear layer.
soften again on heating.
Eg.: EDTA, 10% phosphoric acid, citric acid, Maleic
Eg.: Vulcanite.
acid, etc.

2. Thermoplastic - Softens under heat, becomes


10. PRIMERS are monomers, which are applied over the
sufficiently plastic to mold into any shape and on
etched/conditioned surfaces for the easy flow of
cooling, will harden in to that particular shape. They
bonding agents.
soften again when reheated.
Eg.: HEMA and 2- HEMA dissolved in acetone or
Eg.: Impression compound and acrylic resins.
alcohol.

3. Condensation polymerization - Involves combining


11. DENTIN BONDING AGENTS (OBA) are unfilled resins,
two dissimilar molecules into a third entirely different
which have a role in formation and stabilization of
product.
hybrid layer (micro-mechanical attachment between
resin and conditioned primed dentin).
Eg.: Phenol and formaldehyde to form Bakelite.
12. As most composites are hydrophobic in nature, OBA
4. Addition polymerization - When similar molecules
should be both hydrophobic to bond with composite
are combined the reaction is termed addition reaction.
and hydrophilic to interact with dentin.
In this type, the monomer is repeated many times to
form polymer and no bye products are formed.
13. Generations of Dentin bonding agents
Uses glycerol-phosphoric acid Eg.: Polymethylmethacrylate.
1st generation
dimethacrylate
• Uses chloro substituted 5. Copolymerization - Resin formed by two or
phosphate esters of various more chemically different monomers is called a
monomers copolymer and the process of formation is called as
2nd generation
• In 1st and 2nd generations copolymerization.
there is no distinction between
conditioning, primer and 6. Cross-Linking - Formation of 3-dimensional molecules
bonding agent by interaction between individual linear chains. It
3 step procedure i.e., reduces crazing and water absorption.
3rd generation conditioning, priming and
bonding 7. Stages of polymerization are initiation, propagation,
termination and chain transfer.
Relies on formation of hybrid
4th generation layer. Com bin es Steps 1 and 2 and
are called as self etching primers
5th generation Steps 2 and 3 are combined
1 step procedure. All 3 solutions
6th generation
in one bottle.
7th generation Fluoride releasing bonding agents
Dental ;lut.,e

8. Composition of heat activated resins: 15. External porosity occurs due to inadequate pressure,
Powder Liquid application and improper mixing of monomer and
polymer.
• Polymethyl- • Methylmethacrylate
methacrylate 16. Tissue conditioners a re highly plasticized acrylic
resins used as temporary soft liners to condition the
• Benzoyl Peroxide • Glycol dimethacrylate
inflammed and distorted t issues caused by ill-fitting
(Initiator) (Cross linking agent)
dentures.

• Dibutyl Phthalate • Hydroqui none


It consists of polymer powder and a liq uid containing
( Plasticizer) (Inhibitor)
aromatic ester in ethanol.

9. Composition of self-cure resins is similar except an


CERAMICS
ACTIVATOR - tertiary amine (dimethyl-p-toluidine) is
added to the liquid.
1. Ceramic is a compound of metallic and non-metallic
compounds. Porcelains are glass ceramics.
10. Theinitiatorin light activated resin is camphoroqui none
and the activator is visible light. These resins are
All porcelains are ceramics but not all ceramics are
described as composite having a matrix of urethane
porcelains.
dimethacrylate, microfine silica and resin monomers.
2. Dental ceramics are most suitable tooth colored
11. Most common ly used fabrication technique
restorative materials used to make denture teeth,
Compression moulding .
single unit crowns, fixed partial dent ures and labial
veneers.
12. No trial closures are required for injection moulding
technique (AP-14).
3. Classification:

13. Technique resulting in less porosity - Microwave • According to firing temperature


heating. i) High fusing - 1290-1370°C
Property Heat Cure Self cure ii) Medium fusing - 1095-1260°C
iii) Low fusing - 870-1065°C
Heat Necessary Not necessary
Tertiary amine • According to method of firing
Activator Heat (dimethyl
i) Air fired i.e., at atmospheric pressure
P-toluidine)
ii) Vacuum fired i.e., at reduced pressure
Strength More Less
Porosity Less More • According to method of application

Unreacted i) Core porcelain


Less (0.5%) More (4-5%}
monomer ii) Dentine or body porcelain

Polymerisation iii) Enamel porcelain


More Less
shrin kage
4. Feldspar (K 20. Al 20 3• 6Si0 2 ) acts a basic glass
Colour stability Greater Poor former, flux, matrix and surface glaze. It undergoes
Denture repair Not preferred Preferred incongruent melting at 1150-1530°C and forms
"leucite" crystals, which has a large, co-efficient of
14. Porosity (interna l) occurs in the central thick areas of expansion than feldspar.
the denture if resin is heated above the boiling point
of monomer (100.8°C). 5. Metallic oxides are added to obtain various shades
simulating natural teeth.
=====================================================fr DENTAL MAT ER IALS SYNOPSIS J-~ 33 i-=J===""""~

14. Ceramics have high compressive strength and low tensile


Metallic oxide Colour
strength.
Iron oxide, Nickel oxide Brown
Copper oxide Green 15. It is insoluble and impermeable to oral fluids. However,
t opical fluorides such as APF gel can ca use staining.
Cobalt oxide Blue
Titanium, zirconium Opacity 16. To overcome the low tensile strength, porcelain is
Manganese oxide Lavender bonded to an alloy substructure called metal fused
porcelains.
6. Aluminous porcelain contain upto 40-50% of
17. The alloy used for bond should have
alumina added to core porcelain. Alumina interrupts
the crack propagation by the process of dispersion • Coefficient of therma l expansion - Similar to
strengthening. porcelain
• Fusion temperature - Higher than porcelain to
7. Porcelain powders are mixed with liquid to form a resist sag deformation.
plastic mass and is condensed to form the porcelain • Capable of forming good bond with porcelain.
restoration. Then it is fired in the furnace for sintering. • Should not contain copper or silver as they discolor
porcelain.
8. Proper condensation gives dense packing. Dense
packing reduces the porosity and shrin kage in the 18. Bonding of porcelain to metal occurs by both chemical
fired porcelain. and mechanical bonding.
9.
Stages in firing Changes in porcelain 19. Phosphate bonded investments are used for casting
metal ceramics
Very porous, little shrinkage
Low bisque occurs as the particles lack 20. Opaque porcelains help to mask the colour of the
cohesion underlying metal.
Shrinkage is more, complete 21.
Medium bisque cohesion of particles occur and
Castable glass ceramic Dicor
still porous
Shrink free ceramic Cerestore
Shrinkage is complete, slight
High bisque porous and the mass exhibits Injection moulded
IPS empress
smooth surface cramic

Glass infi ltrated dental


• In-ceram (Al20 3)
10. Glazing is a process, which gives a smooth and glossy • In-ceram spinet
surface, enhance esthetics and help in hygiene. ceramic
(MgAl20)

11. Glazed porcelain is stronger than ung lazed. Self- 22. Glass ceramics are composite materia ls of a glassy
glazing can be produced by heating to its fusion matrix phase and a crystal phase.
temperature and maintaining for 5 min. The glass
grains flow at the surface to form a vitreous glazy 23. After the glass article is formed, nucleation and
layer. crystal growth are created during a controlled heat
treated called "ceramming process (AP-2013}".
12. Self glaze is more permanent than applied glaze.
The process of microscopic plate like crystal nu-
13. Pyroplastic flow of material occurs due to high glazing cleation and growth of crystalline material with-
temperature, which causes rounding of sharp line in the glass matrix is (APPG-15)
ang les and edges of restoration. a) Amalgamation b) Heat treatment
c) Crystallisation d) Ceramming
Dental ;lut.,e

24. CAD-CAM (Computer Aided Design and Computer Aided Corrosion


Machining) is used to prepare metal and ceramic 5. It is the actual deterioration of a metal by a reaction
inlays and crowns without need for impression or with its environment.
casting procedure.
6. Corrosion is of 2 types:
Composite, ceramics and titanium can be copy milled . A. Dry / Chemical Corrosion: in which the meta l
Copy milling is another method of preparing all metal reacts to form oxides, chlorides and sulphides in
crowns or copings for metal ceramic crowns without the absence of electrolytes.
using casting technique.
Eg.: Formation of Ag-Sin dental alloys containing
TARNISH AND CORROSION silver, Oxidation of alloy particles in denta l
amalgam.
Tarnish
B. Wet/ Electro Chemical Corrosion: It requires the
1. It is the surface discoloration or alteration of the
presence of water or other fluid as electrolyte. It
surface finish or luster.
is of 3 types - crevice conosion, stress corrosion
and galvanic corrosion.
2. It generally occurs due to formation calculus, plaque
on the surface of the metal.
7. Noble metal alloys resist corrosion because they have
positive EMF.
3. It also occurs due to formation of oxides, sulfides or
chlorides. (AIIMS May-14)
8. Chromium, aluminum and titanium form strong
adherent oxide films on their surface to protect from
4. Tarnish is the forerunner of corrosion.
corrosion. This is called "passivation"

9. Types of corrosion
• Occurs between dissimilar metals in contact.
• The metal with low EMF acts as anode and metal with high EMF acts as cathode.
Galvanic • Eg: Amalgam opposing cast gold restoration.
corrosion Anode = Amalgam
Cathode = Gold restoration
• Amalgam undergoes corrosion and liberated mercury weakens the gold restoration.
• Metal at the site of maximum stress becomes more reactive than the unstressed metal.
• Anode = stressed metal
Stress
Cathode = unstressed metal
corrosion
• The stressed metal undergoes corrosion.
Eg.: Orthodontic wire, Amalgam
• Accumulation of food debris in crevices produces one type of electrolyte and normal saliva
produces another type of electrolyte
Crevice or • Anode = metal covered with food debris
concentration Cathode = metal with no food debris
cell corrosion • The oxygen tension in the bottom of the pit is less than the periphery of the pit.
• Anode = low oxygen tension pit area
Cathode = peripheral pit area
=====================================================fr DENTAL MAT ER IALS SYNOPSIS J-~ Js ===""""~
i-=J

SOLDERING 9. "Flux" is used to remove oxide coating on the parent


metal and to protect the metal surface from oxidation
during soldering procedures.
1. Brazing:
Joining metal parts together by melting a filler metal
Borax and boric acid fluxes a re used with noble
between them at a temperature below the solidus
metal alloys. Fluoride fluxes are used with base
temperature of the metal being joined. Here the
metal alloys, as they help to dissolve the more stable
fusion temperature of filler material is greater than
450°(. chromium, nickel oxides.

2. Soldering:
The composition of flux in cludes Borax glass (55%)
Boric acid (35%) and Silica 10%.
Joining metal parts together by melting a filler metal
between them at a temperature below the solidus
Fluoride fluxes contain boric acid and potassium
temperature of the metal being joined. Here the fusion
fluoride in 1: 1 ratio
temperature of fi ller material is less than 450°(.

10. "Anti flux" is used to limit the follow of molten solder


3. Welding:
over metals being joined.
Joining two or more metal pieces by applying pressure
(cold welding) or heat (hot welding) or both.
Eg.: Graphite, lead, rouge (Irnn oxide), etc.
Spot welding is used to join orthodontic components.
11. The most commonly used heat source for melting
solder is gas air or gas oxygen torch.
4. Joining of metals before porcelain veneering is
called pre-soldering and join metals after porcelain
12. The various "fuels" used are hydrogen, natural gas,
veneering is ca lled post soldering.
acetylene and propane.
5. Dental solders are alloys that are used as intermediary
Propane is the best choice because of its highest
or filler material to join two or more metallic parts.
flame temperature and fla me heat.
Gold and silver solders are common ly used in dentistry.
Acetylene has highest flame temperature but it is
Silver solders are Low fusing solders and are used for unstable and the temperature varies from one part of
the flame to the other.
soldering stainless steel or other base meta l alloys.

13. Zones of flame from inner to outer space.


6. Composition Of Solders:
• Gold
1
Cold mixing
Unburned gas Not used
• Silver zone
Gold solder • Copper Combustion
• Tin 2 Oxidizing Not used
zone (Green)
• Zinc
Used for
• Silver
3
Reducing
Hottest zone soldering and
Silver solder • Copper zone (Blue) melting alloys
• Zinc
Oxidizing
4 Burned gas Not used
7. The fusion (flow) temperature of the solder should be zone
at least 56°C lower than the solidus temperature of
the substrate (parent) metal. 14. There should be optimum gap between metal parts to
be joined. If the gap is too narrow, strength is limited
8. For a solder to wet and flow properly, the parent metal and if the gap is too wide, the joint strength will be
must be free of oxides. controlled by the strength of the solder.
Dental ;lut.,e

15. Pure titanium can be laser welded in an argon • Remineralizing bonding agents like inclusion of
atmosphere. MTA in bondind agents.

16. Cast joining is a process of joining two components 3. Bone grafting materials:
of fixed partia l denture by means of casting molten • New bioactive glasses that contain more boron and
metal into t he interlocking region between the silica for osteogenesis.
invested components. • Extraction socket grafting materials.
• Im plants with nanotechnology coatings.
17. Orthodontic Spot Welding:
Heat and pressure are the two basic principles invo lved 4. Stimulus responsive "Smart" materials:
in orthodontic spot welding. • New Zirconia ceramics that prevent propagation of
cracks.
It is very important that the passage of current at • Composites that change color when irradiated.
the weld spot may Joe of very short duration i.e., not • Smart GIC.
more than 1;10th of a second. If the current is passed • Varnish that release xylitol for anticaries effect.
for longer duration, it will result in weld decay due to • Resins capable of releasing Chlorhexidine.
precipitation of carbides from metal
5. Self-assembling materials:
POLIS HING AGENTS Materials that automatically construct pre-specified
assemblies for t ooth regeneration based on molelcular
biology.
Agent Uses
Smoothening dentures, polishing 6. Self healing materials:
Pumice
of artificial teeth • An automatically initiated response to damage or
Zinc oxide Polishing of amalgam restoration failure.
• The self healing system developed for epoxy
Rouge Polishing of noble meta l alloys composite. If a crack occurs in epoxy composite,
Zirconium Prophylactic paste for polishing some microcapsules rupture near the crack and
silicate natural teeth release dicyclopentadiene which subsequently fills
and repairs the cracks.
Chromic oxide Stainless steel
Tin oxide Metallic restorations 7. Bioactive materials:
Corundum • MTA type cements
(white stone • Ti02 nanoparticals containing dental resins and
Grinding of metal alloys implants.
or mineral
form of Al2 0 3 )
GREEN DENTISTRY
EMERGING TECHNOLOGIES IN COMPOSITES
1. ECO dentistry.
1. Antimicrobial materials:
• Fluoride containing composites. 2. Insta lling amalgam separators, using digital X-ray
• MDPB (Methacryloxy dodecyl pyridinium bromide imaging in place of photographic images.
antimicrobia l monomers.
• MMP (matrix metalloproteinase) inhibiting 3. Using washable coats.
components incorporated in the composite.
4. Disinfectants for surfaces, and dental water lines that
2. Remineralizing agents: are safe and fast.
• Nanoparticals of hydroxy apatite for a nticaries
repair effect of composite restorations. 5. Use of detergent that both disinfects the tubing and
also chelates mercury to enhance amalgam separation.
, ORAL ANATOMY

ORAL ANATOMY
I. REFERENCE BOOKS TAKEN:
1 . WHEELER'S DENTAL ANATOMY AND OCCLUSION - 8th & 9th EDITION

2. WHEELER'S DENTAL ANATOMY AND OCCLUSION - 6th EDITION

1. DECIDUOUS DENTITION
1. Which of the followi ng deciduous molars bears the c) Three d) Four
greatest resemblance to a premolar (AIPG-97)
a) Maxillary first b) Maxillary second 7. The primary teeth that present the most outstanding
c) Mandibular first d) Mandibular second morphologic deviations from permanent teeth are:
(MAN-95) a) Central incisors b) Mandibular first molar
2. The largest mesiodistal diameter in primary c) Mandibular second molar
dentition is: d) Canines
a) Mandibular l't molar b) Mandibular znd molar (AIPG-02)
c) Maxillary 2"d molar d) None of the above 8. Roots of primary and secondary teeth are different
(MAN-97) in that the primary roots are more divergent and
3. Primary dentition period is: flaring. The other difference:
a) From birth to 11 years a) Primary roots show less accessory and lateral canals
b) From 6 months to 11 years b) Primary roots resorb more easily
c) From 6 months to 6 years c) Primary roots are shorter
d) From 6 years to 11 years d) None of the above
(MAN-97) (AIIMS- 99)
4. How many cusps does the crown of the maxillary 9. The primary lateral incisors begin to calcify about
first prima ry mo lar have? the:
a) 2 b) 3 a) 4th month of life b) 4 th month in utero
c) 4 d) 5 c) 4 1 month in utero d)
1 2
5 th month in utero
(MAN-95) (KAR- 2K)
5. Which of t he fo llowing has t he maximum 10. When viewed occlusally, a primary mandibular
faciolingual diameter of the crown: second molar closely resembles which of following
a) Deciduous maxillary znd molar mandibular teeth?
b) Deciduous maxillary 1 st molar a) Permanent second molar
c) Deciduous mandibular 2"d molar b) Permanent first molar
d) Deciduous mandibular 1st molar c) Second premolar d) Primary first molar
(MAN-99) (AIPG-98)
6. How many roots does a primary maxillary first molar 11. In t he majority of sit uat ions, the first deciduous
have: tooth to erupt is the:
a) One b) Two a) Mandibular central incisor

1) A 2) B 3) C 4) C 5) A 6) C 7) B 8) A 9) C 10) B 11) A
Dental ;lut.,e

b) Mandibular first molar c) Mesiobuccal d) Di stobuccal


c) Maxillary central incisor (APPSC-99 ,AIIMS-07)
d) Maxillary second molar 19. Cusp of carabelli can be considered a normal finding
(AIIMS-93,AP-05) on:
12. Pulp chambers and root canals in deciduous teeth: a) Deciduous mandibular 1st molar
a) Wide and deep b) Shallow and narrow b) Deciduous maxillary 1'1 molar
c) Wide and narrow d) Shallow and wide c) Deciduous maxillary z nd molar
(AP-01) d) Deciduous mandibular 2"d molar
13. Cervical bulge. True is
a) Enamel rods at cervical region directed occlusally 20. Mesiodistal width is more than cervico-incisal
b) Results in constriction at cervical region length of crown in:
c) Is characteristic of deciduous teeth a) Permanent central incisors
d) All of the above b) Deciduous maxillary central incisors
(AP-01) c) Permanent maxillary canines
14. Primary teeth differ from permanent teeth in d) Permanent mandibular lateral incisors
fo Llowing aspects (KAR- 98)
a) Thickness of enamel and dentin 21. With respect to their permanent successors the
b) Pulp chambers are longer sum of the mesiodistal diameters of the first and
c) Roots are flaring second deciduous molars is generally.
d) All of the above a) Less b) Greater
(PGI- 95) c) The same d) Not related
15. Which statement is correct about deciduous
dentition? 22. Among the differences between teeth in the
a) Calcification of maxillary primary first premolar is deciduous and the permanent dentition is that:
completed at birth a) Buccal surfaces of the permanent molars are flatter
b) Mandibular first molar erupts at one and half years b) Cervical ridges are more pronounced in permanent
of age molars
c) The crowns of primary dentition are more yellowish c) Root trunks of deciduous teeth are larger compared
than permanent with their overall root lengths
d) The primary dentition is initiated at 101h month in d) Crowns of deciduous incisors are wider mesiodistally
uterus than their cervico incisal lengths
(AIPG- 94)
16. Transverse ridge is prominent on occlusal surface of 23. The direction of enamel rods in deciduous teeth is:
which of the following deciduous teeth? a) Same as permanent teeth
a) Mandibular first molar b) Inclined in an apica l direction in the cervical third
b) Maxillary first molar of crown
c) Mandibular second molar c) Inclined in an occlusal direction in the cervical
d) Maxillary second molar third of crown
(AP-04, PGI JUNE-13) d) None of t he above
17. How many pulpal horns are there in the mandibular
second primary molar? 24. During the development of a tooth, sound
a) 3 b) 6 coalescence of the lobes results in
c) 4 d) 5 a) Grooves b) Fissures
(COMEDK-04) c) Cusps d) All of the above
18. The highest (longest) and sharpest cusp on the (KAR-2013)
deciduous mandibular first molar is:
a) Mesiolingua l b) Distolingual

12} D 13} C 14} D 15} B 16} A 17} D 18} A 19} C 20} B 21} B 22} D 23} C 24} A
, ORAL ANATOMY

25. What is the thickness of the layer of prismless


enamel found in primary teeth?
a) 25 µm b) 50 µm
c) 75 µm d) 100 µm
(AIPG-06)
26. The sharpest cusp of primary first molar tooth is:
a) Mesiobuccal b) Distobuccal
c) Mesiolingual d) Distolingual
(AIPG- 07)
27. The pulp of first primary molar contains:
a) Four pulp horn and three root canal
b) Three pulp horn and three root canal
c) Two pulp horn and two root canal
d) Two pulp horn and three root canal

28. Which among the following is NOT a common


finding of the deciduous dentition?
a) Tight contacts, absence of spacing
b) Shallow overjet and overbite
c) Flush terminal plane d) Ovoid arch form
(KCET-2010)
29. The crowns and roots of primary molar at their
cervical portions are more
a) Slender mesiodistally
b) Slender anteroposteriorly
c) Bulkier mesiodistally
d) Bulkier anteroposteriorly
(BHU-2012)
30. Resorption of root of primary incisors starts at
a) 4 yrs b) 6 yrs
c) 8 yrs d) 10 yrs

31. At what age does calcification of the primary


dentition begin?
a) 6-10 weeks in utero
b) 13-16 weeks in utero
c) 22-26 weeks in utero
d) 30-34 weeks in utero
(COMED-14)

25) A 26) C 27) A 28) A 29) A 30) A 31) B


Dental ;lut.,e

1. DECIDUOUS DENTITION - ANSWERS


1. 'A' [Wheeler's 8th ed 67/ 9th ed 46, 60] 9. 'C' [Wheeler's 8th ed 53/ 9th ed 23]
• Primary maxillary 1•t molar resembles permanent
premolars. 10. 'B' [Wheeler's 8th ed 94, 97 / 9th ed 64, 65]
• Primary maxillary znd molar resembles permanent
11. 'A' [Wheeler's 8th ed 58/ 9th ed 41]
maxillary 1st molar.
• Primary mandibular 2nd molar resembles permanent
12. 'D' [Wheeler's 8thed 71/ 9th ed 48, 49 Fig 3.6]
mandibular 1't molar.
• The pulp chambers of deciduous teeth are larger
• Primary mandibular 1't molar does not resemble any
and pulp horns are placed in a higher position.
of the permanent tooth.
• The enamel pulp and dentin thickness is limited
making the pulp chambers shallow.
The deciduous tooth which does not have
• The pulp canals are wider and the apical portion
resemblance to any other teeth (deciduous or
of the canal is much less constricted than that of
permanent) is (KERALA-2015)
permanent teeth.
a) Maxillary first molar b) Mandibula r first molar
c) Maxillary 2nd molar d) Mandibular canine
13. 'C' [Wheeler's 8th ed 68, 70/ 9th ed 47, 49 Fig 3.6]
When viewing the mesial and distal aspects, the
2. 'B' [Wheeler's 8th ed 75/ 9th ed 52, Tab 3-1]
curvature (or) bulge of the crown at cervical third above
• Largest mesiodistal diameter in primary dentition
the cervical line is called cervical ridge (or) cervical
- Mandibular 2nd molar. bulge (or) cervico enamel ridge. More pronounced
• Largest buccolingual diameter in primary dentition cervical ridges are characteristic of deciduous teeth.
- Maxillary 2nd molar.
14. 'D' [Wheeler's 8th ed 71/ 9th ed 48, 50]
3. 'C' [Wheeler's 8th ed 33/ 9th ed 28]
• From 6 months to 6 years - Primary dentition 15. 'B' [Wheeler's 8th ed 53/ 9th ed 23, 47]
First primary tooth bud is initiated at 61h week of
• From 6 years to 11 years - Mixed dentition
intrauterine life.
• From 12 years onwards - Permanent dentition
16. 'A' [Wheeler's 9th ed 64]
4. 'C' [Wheeler's 8th ed 86/ 9th ed 59, 60]
• Primary 1st molars - 4 cusps. 17. 'D' [Wheeler's 9th ed 50, Fig 3-78]
• Primary 2nd molars - 5 cusps.
18. 'A' [Wheeler's 8th ed 93/ 9th ed 64]
5. 'A' [Wheeler's 8th ed 75/ 9th ed 52] The long and sharp mesiolingual cusp is an outstanding
feature in the deciduous first mandibu lar molar.
6. 'C' [Wheeler's 8th ed 83/ 9th ed 58]
Primary maxi llary first molars have 2 buccal 19. 'C' [Wheeler's 8th ed 88/ 9th ed 60]
and 1 palatal roots.
20. 'B' [Wheeler's 8th ed 68/ 9th ed 4 7]
7. 'B' [Wheeler's 81 h ed 91/ 9th ed 46, 62] The crowns of deciduous anterior teeth are wider
mesiodistally in comparison with their crown length
8. 'A' [Wheeler's 81 h ed 70, 71/ 9th ed 4 7, 48] than are the permanent teeth.
I he roots of deciduous teeth are Longer, slender, more
divergent and flaring with thinning out rapidly as the 21. 'B' [Wheeler's 8th ed 448/ 9th ed 281]
apices are approach. The flare allows more room to The sum of the mesiodistal widths of primary first and
accommodate the developing permanent tooth buds. second molars is greater than mesiodistal width of
, ORAL ANATOMY

premolars. This width difference is called the Leeway 31. 'B' [Wheeler's 9th ed 37 Table 2-4]
space of Nance. It is 0.9mm per quadrant in the upper Maxillary and mandibular central incisors are first
arch and 1.7mm per quadrant in the lower arch. primary teeth that begin to calcify at 13-16 weeks in
utero.
22. 'D' [Wheeler's 8th ed 68/ 9th ed 47]

23. 'C' [Wheeler's Sth ed 71/ 9th ed 49]


The enamel rods in primary teeth slope occlusally in
primary teeth and gingivally in permanent teeth at
cervix.

24. 'A' [Sturdevant 5th ed 23/ Wheelers 9th ed 8]


Grooves or fissures are formed at the junction of the
developmental lobes of the enamel. Sound coalescence
of the lobes results in grooves; faulty coalescence
results in fissures.

25. 'A' [Orban's 13th ed 58/ 13th ed 58]


A relatively structureless layer approximately 30 micro
meters thick called prismless enamel. It is found in
70% of permanent teeth and all deciduous teeth. This
prismless enamel is absent over cusptips and present
towards cervical areas of enamel surfaces.

26. 'C' [Wheelers 81h ed 84, 93/ 9th ed 58, 64]


The mesiolingual cusp is the most prominent cusp
in the primary first molar. It is also the longest and
sharpest cusp. The distolingual cusp is the poorly
defined cusp.

27. 'A' [Wheelers 6th ed 59/ 9th ed 50, Fig 3-7]

28. 'A' [Wheelers 8th ed 65/ 9th ed 45]


The development of adequate spacing in primary
teeth is a very important factor in the development of
normal occlusal relations in the permanent dentition.
Tight contacts and absence of spacing is not a common
finding of the deciduous dentition.

29. 'A' [Wheeler 9th ed 58, 62]

30. 'A' [Dental Anatomy by Weiss and Scheid 169/


Wheelers 9th ed 23, 45/0rban's 13th ed 333 fig
13-1]
Only about 3 years after root completion, primary
tooth roots begin to resorb, usually at the apex or
on one side near the apex. Resorption of deciduous
incisor begins at 4.5 yrs and at 6 yrs it is shed.
Dental ;lut.,e

2. PERMANENT DENTITION
1. The shape of the occlusal surface of the permanent d) Maxillary and Mandibular second molar
maxillary first molar is: (MAN- 98)
a) Oval b) Trapezoidal 9. Which of the following teeth have more than one
c) Triangular d) Rhomboidal occlusal form:
(MAN-94,AIIMS-93) a) Maxillary 1st premolar
2. Which of the following is the largest root: b) Maxillary 2nd premolar
a) The lingual root of a maxillary first molar c) Maxillary 1st molar d) Mandibular ist molar
b) The lingual root of a maxillary second molar (MAN-99)
c) The root of a mandibular canine 10. The most common curvature of the palatal root of
d) The root of a maxillary canine maxillary first molar is:
(MAN-95) a) Facial b) Lingual
3. Palatogingival groove is found in: c) Distal d) Mesial
a) Maxillary Lateral incisor (MAN-97 ,AIIMS-92)
b) Maxillary first premolar 11. Most common morphological variation is seen in:
c) Maxillary first molar d) All of the above a) Maxillary lateral incisor
(MAN-2K) b) Mandibular lateral incisor
4. Greater crown bulk distal to the faciolingual c) Maxillary canine d) Mandibular canine
bisecting plane of a tooth is most typical of the (AIIMS-96,AIPG-98)
mandibular 12. Which of the premolars is usually the smallest:
a) Central incisor b) Lateral incisor a) Maxillary first b) Maxillary second
c) Canine d) Second premolar c) Mandibular first d) Mandibular second
(AIPG-97,MAN-95) (AIPG-03)
5. Largest cusp in permanent mandibular first molar 13. A pronounced developmental groove is usually
is: present on the mesial marginal ridge of a permanent:
a) Mesiobuccal b) Di stobuccal a) Mandibular third molar
c) Mesia lingual d) Distal b) Mandibular second premolar
(MAN- 98) c) Maxillary second molar
6. In which of the tooth is the mesiodistal d) Maxillary first premolar
measurements is greater lingually than facially: (AIPG-03)
a) Maxillary l't premolar 14. A distinguishing characteristic of a maxillary first
b) Mandibular 2"d premolar premolar that identifies it as right or left is the:
c) Mandibular ist premolar a) Height of the lingual cusp
d) Maxillary ist molar b) Mesial inclination of the lingual cusp
(MAN- 99) c) Flattened area beneath the contact on the distal
7. If a Mandibular canine has bifurcated roots, they surface
are most commonly placed: d) Greater length of the mesial cusp ridge on the
a) Mesial and distal b) Facial and lingual lingual cusp
c) Mesiobuccal and distobuccal (AIPG-98)
d) Mesiolingual and distolingual 15. Which of the following premolars has a mesial
(MAN-99) marginal ridge more cervical than the distal
8. Five cusps are present in: marginal ridge:
a) Maxillary first molar only a) Maxillary 1st premolar
b) Mandibular first molar only b) Maxillary 2nd premolar
c) Maxillary and mandibular first molar c) Mandibular 1st premolar

1) D 2) D 3) A 4) C 5) A 6) D 7) B 8) C 9) C 10) A 11) A 12) C 13) D


14) B 15) C
, ORAL ANATOMY

d) Mandibular znd premolar 24. Number of roots in maxillary 2"d molar is


(AIIMS-2K) a) 2 b) 3
16. The shortest root on a maxillary first molar is: c) 4 d) 5
a) Mesio-buccal b) Dist o-buccal (AP-03)
c) Lingual d) All are of equal lengt h 25. In comparison to maxillary central incisor, maxillary
(KAR-99) canine has a height of contour that is:
17. Using occlusal morphology as guide, the Mandibular a) More
third molar is most similar to: b) Less
a) Maxillary permanent first molar c) Same
b) Mandibular permanent first molar d) Different on different t eeth
c) Mandibular permanent second molar (AIIMS- 2K)
d) Mandibular deciduous second molar 26. A maxillary right canine may be distinguished from
(KAR-99) a maxillary left canine because:
18. The total number of cingula in each dentition is: a) The root curves to mesial in the apical 1/3'd
a) 6 b) 12 b) The distal half of canine sliows more convexity than
c) 24 d) 32 the mesial half
(AIPG- 98) c) Facially the cusp tip is placed distal to Bisecting
19. The mesial surface of the crown is almost parallel to line
long axis and the root of a: d) Lingually the cervical line slopes mesially
a) Maxillary 1st premolar (AIPG-02)
b) Mandibular znd Premolar 27. Single central pit is formed on occlusal surface of:
c) Maxillary canine d) Mandibular canine a) Mandibular first premolar
(AIPG- 02) b) Mandibular second premolar
20. The root of maxillary lateral incisors if curved is c) Maxillary first premolar
usually in which direction? d) Maxillary second premolar
a) Distal b) Mesi al (AIPG- 94)
c) Facial d) Palatal 28. Which of the following features of a permanent
(PGI-98,AIPG-96) maxillary lateral incisor aids in distinguishing it
21. Which of the following is a smallest posterior tooth: from a mandibular lateral incisor?
a) Maxillary first premolar a) Flattened root
b) Mandibular first premolar b) More pronounced lingual fossa
c) Maxillary second premolar c) Thinner incisal ridge
d) Mandibular second premolar d) Sharper proximal incisal ang les
(AP-98) (AIPG-98)
22. The permanent anterior tooth, which is most 29. On which of the following permanent teeth is it
commonly atypical, is: most difficult to distinguish between mesial and
a) Maxillary central incisor distal aspects?
b) Maxillary latera l incisor a) Maxillary central incisor
c) Mandibular central incisor b) Mandibular central incisor
d) Mandibular 1st premolar c) Mandibular lateral incisor
(PGI- 99) d) Mandibular second premolar
23. Tooth with smallest root is: (AIPG-98)
a) Maxillary central incisor 30. Mesiolingual grooves may be found on which of the
b) Mandibular lateral incisor following permanent teeth?
c) Maxillary lateral incisor a) Mandibular second molars
d) Mandibular central incisor b) Mandibular first premolars
(AIPG-97) c) Maxillary first molars

16) B 17) C 18) B 19) D 20) A 21) B 22) B 23) D 24) B 25) A 26) B 27) B 28) B
29) B 30) B
Dental ;lut.,e

d) Maxillary canines 40. Oblique ridge in maxillary first molar joins which of
(AIPG-98) the following cusps:
31. Cusp of carabelli is present on _ _ surface of the a) DL and DB b) ML and MB
permanent maxillary molar: c) ML and DB d) DL and MB
a) Mesial surface b) Palatal surface
c) Buccal surface 41. Mandibular second premolar 3-cusp type is
d) Mesial half of the palatal surface developed from:
(AP-2K) a) 3 b) 4
3 2. Difference between mandibular and maxillary c) 5 d) 6
premolars is that in the mandibular premolars: (MHCET-15)
a) The buccal and lingual cusps are more prominent 42. Corner stones of dental arches are:
b) The lingual cusp is sharper and longer a) Canines b) Firrst premolars
c) The lingual surface rounded and less developed c) First molars d) Second molars
d) The buccal surface is rounded and less developed
(AIIMS-94) 43. Mesial slope of buccal cusp is longer than distal
33. Upper 4 resembles 5 from which aspect? slope in:
a) Mesially b) Lingually a) Maxillary z nd premolar
c) Occlusal d) Distally b) Maxillary ist premolar
(AIPG-93) c) Mandibular canine
34. The Largest permanent tooth in the mouth usually d) Maxillary canine
is:
a) Maxillary canine b) Mandibular canine 44. Mesial concavity making the prognosis poor for a
c) Mandibular first molar furcation involvement is present in:
d) Maxillary first molar a) Maxillary canine b) Maxillary 1'1 premolar
(KAR-98) c) Mandibular 1st premolar
35. Largest tooth in the mandibular arch is: d) Maxillary z nd premolar
a) Canine b) Deciduous first molar
c) Permanent first molar 45. The occlusal surface shape of a 3 cusped mandibular
d) Third molar second premolar:
(KAR- 02) a) Oval b) Elliptical
3 6. Longest tooth of the maxillary arch is: c) Square d) Tetrahedral
a) Permanent incisor b) First premolar
c) Permanent canine d) Deciduous incisor 46. Tooth with most variable occlusal anatomy:
(KAR-02) a) Maxillary third molar
37. No. of cusps in mandibular t st molar: b) Mandibular third molar
a) 4 b) 3 c) Maxillary second premolar
c) 5 d) 6 d) Mandibular second premolar
(AP-01) (APPSC-99)
38. Which of the following has a bifid root? 4 7. Tritubercular cusps are:
a) 32, 42 b) 31, 41 a) ML b) MB
c) 34, 44 d) 13, 23 c) DB d) All of the above
(AIIMS- 2K)
39. Rounded protruberances on the incisal edges of 48. Acute angled cusps in permanent maxillary first
newly erupted permanent incisors are: molar are:
a) Mamelons b) Tubercles a) DB and ML b) MB and DL
c) Enamel pits d) Cingulum c) MB and DB d) ML and DL

31} D 32} C 33} B 34} D 35} C 36} C 37) C 38} C 39} A 40} C 41} C 42} C 43} B
44} B 45} C 46} A 47} D 48} B
, ORAL ANATOMY

49. Which cusp is poorly developed in a permanent 58. The maxillary molar crown of an early fossil primate
maxillary second molar: featured a trigone made of three main cusps; these
a) DB b) DL cusps were:
c) MB d) ML a) Distobuccal, distolingual and Mesiolingual
b) Distolingual, Mesiobuccal, and linguo bucca l
50. Cervical cross section of maxillary first premolar is:
c) Mesiobuccal, Distobuccal and Mesiolingual
a) Dumbell b) Kidney shape
d) Mesioling ual, Mesiobuccal and distolingual
c) Elliptical d) Oval
(MAN- 95)
59. The two major fossa of permanent maxillary first
51. Which bifurcation is closet to the cervical line
molar:
(or) which surface has short root trunk length in a
a) Central fossa and mesial fossa
maxillary permanent first molar:
b) Central and distal fossa
a) Buccal b) Mesial
c) Mesial and distal triangular fossa
c) Distal d) None of the above
d) Distal fossa and distal triangular fossa

52. Permanent Mandibular central incisor develops from:


60. The lingual cusp of lower first premolar is compared
a) 1 lobe b) 3 lobes
to the development of:
c) 4 lobes d) 5 lobes
a) Cusp of carabelli
b) Distolingual cusp of molars
53. In the triangle formed by the projection of the
c) Lingual cusp of second premolar
orifices of the canals of a maxillary molar, the:
d) Cingulam of canine
a) Line connecting mesial with distal is longest
b) Line connecting mesial with lingual is longest
61. Permanent mandibular first molar is developed
c) Line connecting distal with lingual is longest
from ___ lobes:
d) Angle at the mesio facia l canal is obtuse
a) 3 b) 4
c) 5 d) 6
54. What is the functional form of anterior teeth from
the mesial or the distal aspect?
62. The geometric outline of the buccal surface of
a) Rhomboidal b) Trapezoidal
posterior teeth are:
c) Elliptical d) Wedge-shaped
a) Trapezoidal b) Rhomboidal
(APPSC-99)
c) Quadrilateral d) Square
55. The premolar with the occlusal groove pattern that
may simulate the letter "Y" is:
63. Which of the following is true for permanent teeth:
a) Maxillary first b) Maxillary second
a) Maxillary roots are inclined distally whe re as
c) Mandibular second d) Mandibular first
Mandibular roots are incli ned mesially
(APPSC-99)
b) Maxillary molars are buccally inclined and
56. Which line angle area is most acute when a
Mandibular molars are inclined lingually
permanent maxillary second molar is viewed
c) Buccal cusps of lower and lingual cusps of upper
occlusally?
are t he fu nctional cusp
a) Mesiofacial b) Distofacial
d) All of the above
c) Mesiolingual d) Dist olingual
(AIPG-98)
64. Fourth canal is present in:
5 7. In which of the following a nonfunctional cusp is
a) Mesiobuccal root of maxillary first molar
present:
b) Distal root of mandibular first molar
a) Maxillary pt molar b) Maxillary 2 nd molar
c) Both of t he above
c) Mandibular 1st molar d) Mandibular 2 nd molar
d) None of t he above
(MAN-99)

49) B 50) B 51) B 52) C 53) B 54) D 55) C 56) A 57) A 58) C 59) B 60) D 61) C
62) A 63) D 64) C
Dental ;lut.,e

65. Which of the following is true of permanent first 71. Which of the following permanent teeth is usually
molar's occlusal surl'ace: bilaterally symmetric when viewed labially and
a) Maxillary molar has 2 major fossa, 2 minor fossa, 4 incisally:
triangular ridges, 1 oblique ridge and 5 grooves a) Mandibular lateral incisor
b) Mandibular molar has 1 major fossa, 2 minor fossa, b) Mandibular canine
5 triangular ridges and 4 grooves c) Mandibular central incisor
c) Rhomboidal shape in maxillary molars and d) Maxillary central incisor
hexagonal shape in Mandibular molars
d) All of the above 72. Which of the following has a nonfunctional cusp?
a) Mandibular canine
66. The permanent mandibular second molar differs b) Maxillary second premolar
from the permanent mandibular first molar in c) Mandibular first premolar
number of: d) None of the above
a) Cusps b) Roots (KAR- 04)
c) Lingual grooves d) Marginal ridges 73. Average of length of maxillary cuspid is:
a) 26 mm b) 33 mm
67. The maxillary tooth exhibiting the greatest c) 21 mm d) 18 mm
variation in root alignment is: (KAR- 04)
a) Central incisor b) First premolar 7 4. Maxillary first molar has how many roots?
c) Third molar d) Second molar a) 1 facia l and 2 lingual
b) 1 lingual and 2 facial
68. If the pulp of a single rooted tooth canal were c) 1 mesial and 2 buccal
triangular in cross section with the base of the d) 1 mesial and 1 distal
triangle located facially and the apex lingually with (AP- 05)
a longer mesial side than distal side; the tooth 75. The term diphydont means:
most likely is : a) Second dentition b) Two same sets of dentition
a) Maxillary central incisor c) Two separate sets of dentition
b) Maxillary second premolar d) More than two sets of dentition
c) Mandibular lateral incisor (COMEDK-07)
d) Mandibular central incisor 7 6. The eruption age of maxillary permanent lateral
incisor is:
69. The root trunk of permanent mandibular first molar a) 8 - 9 years b) 6 - 7 years
buccally is: c) 10-11 years d) 8-9 months
a) Equal is length to the mesial bifurcation of the (COMEDK-05)
maxillary first molar 77. The number of point angles in a permanent
b) Equal is length to the buccal bifurcation of the mandibular first molar is:
maxillary first molar a) 6 b) 8
c) Equal in length to the crown c) 4 d) 10
d) None of the above (COMEDK-05)
78. Steepest cusp is seen in:
70. The most constant and valuable trait to differentiate a) Maxillary 1' 1 premolar
among maxillary first, second and third molars is: b) Maxillary 1'1 molar
a) Number of roots b) Depth of central fossae c) Mandibular II"d molar
c) Comparative size of the cusp of carabelli d) Mandibular l't molar
d) Relative position of the distolingual grooves (AIIMS-06)

65) D 66} A 67) C 68} A 69} A 70} D 71} C 72} C 73) A 74) B 75) C 76) A 77) C
78} A
===========================================~~~==f, ORAL ANATOMY

79. Which of the following tooth shows largest Mesio- c) Lateral incisors d) Canine
buccal ridge: (AP-2010)
a) Maxillary p t premolar 87. Ratio of mesiodistal width to height of crown of
b) Maxillary 2nd premolar maxillary canine is
c) Mandibular 1st molar d) Mandibular 2nd premolar a) 6:10 b) 8:10
(AP-07) c) 6:8 d) 7:10
80. Which teeth the proximal aspect shows rhomboidal (PGI- 2011)
appearance: 88. In maxillary 1st permanent molar, the 2 obtuse
a) Maxillary posteriors b) Mandibular posteriors angles are
c) Maxillary ant eriors d) Mandibular anteriors a) Mesiolingual and mesiofacial
(AP- 07) b) Mesiofacial and distolingual
81. According to Sch our & Massler the crown completion c) Distofacial and mesiolingual
of both permanent maxillary and mandibular first d) Distofacial and mesiofacial
molars is: (AP-2010)
a) Ph - 2 years b) 21/2 - 3 years 89. Dentin islands are frequently found in the root
c) 3 - 4 years d) 4 - 5 years canals of which of the following permanent teeth?
(COMEDK- 07, 15) a) Maxillary lateral incisor & mandibular canine
82. The maxillary lateral incisor is smaller than the b) Maxillary 2nd premolar & maxillary 1st molar
maxillary central incisor on all aspects EXCEPT c) Maxillary 2nd premolar & mandibular canine
which one of the following? d) Maxillary 1st molar & mandibular 2nd premolar
a) Crown length b) Mesiodistal crown width (AIPG- 2010)
c) Faciolingual crown width 90. The number of line angles in a permanent maxillary
d) Root length central incisor is
(COM EDK- 07) a) Two b) Four
83. Crown formation of all permenent teeth except c) Six d) Eig ht
third molars is completed between (COMEDK- 2009)
a) Birth to 8 years b) Birth to 12 years 91. Additional period required for enamel calcification
c) Birth to 6 years d) 6 years to 12 years after eruption into oral cavity
(BHU-07) a) 5 yrs b) 2 yrs
84. Following eruption, the root of the maxillary central c) 8 yrs d) 1 yr
incisor completes development at what approximate (AIIMS MAY-13)
age? 92. In tooth numbering system 55 indicates
a) 7 years b) 8 years a) Permanent right Maxillary 1st Premolar
c) 9 years d) 10 years b) Permanent left Maxillary 1st Premolar
(COM EDK-08) c) Deciduous right Maxillary 1st Molar
85. An 8-yr old child comes to your clinic with large d) Deciduous right Maxillary 2nd Molar
front teeth having jagged margins, what is the (GCET-14)
treatment plan for this patient? 93. Cusp of carabelli is present on
a) Smoothen the jagged margins and apply fluoride a) Permanent Maxillary 1st Molar
varnish b) Permanent Mandibular 1st Molar
b) Build up other teeth to large size c) Permanent Maxillary 2nd Molar
c) Extraction of big teeth d) Permanent Mandibular 2nd Molar
d) Assure him and send him back (GCET-14)
(AIPG-2009) 94. According to FDI tooth numbering system the
86. In the mandibular arch, in which tooth maximum number 45 refers to
lingual inclination is present a) deciduous mandibular right second molar
a) 1st premolar b) 3rd molar b) mandibular right second premolar

79) A 80) B 81) B 82) D 83) A 84) D 85) D 86) A 87) B 88) C 89) C 90) C 91) B
92) D 93) A 94) B
Dental ;lut.,e

c) maxillary right second premolar 102. Linear depression on a tooth surface


d) deciduous maxillary right second molar a) Pit b) Groove
(GCET-14) c) Fossa d) Sulc us
95. Number of developmental lobes in human (COMEDK-15)
permanent anterior teeth 103. A development disturbance in the enamel of
a) 1 b) 2 mandibular first molar crown usually occurs
c) 3 d) 4 between which of the following ages
a) 8 months prenatal to one year postnatal
96. The mesial marginal developmental groove is b) Birth to 3 years postnatal
present on c) 3 years postnatal to 5 years postnatal
a) Maxillary 1st Premolar d) 6 years postnatal to 8 years postnatal
b) Maxillary 2nd Premolar (COMEDK-2015)
c) Mandi bular 1st Premolar
d) Mandibular 2nd Premolar
(GCET-14)
97. Which of the following grooves separates cusp
ridges from marginal ridges?
a) Supplemental
b) Developmental
c) Mesio marginal developmental
d) Marginal ridge developmental
(GCET-14)
98. Minimum number of lobes required for tooth
formation?
a) 5 Lobe b) 2 Lobes
c) 3 Lobes d) 4 Lobes
(AIPG-14)
99. The occlusal outline of the permanent mandibular
first molar is usually similar to Trapezoid
a) Circle b) Square
c) Trapezoid d) Parallelogram
(AP-14)
100. The ridge that descends from the cuspal tip towards
the central part of the occlusal surface in a maxillary
molar is
a) Triangular ridge b) Marginal ridge
c) Transverse ridge d) Oblique ridge
(KERALA-2015)
101. In 1947 a committee at ADA recommended
symbolically, which of the following tooth
numbering system
a) Zsigmondy/Palmer system
b) ADA system
c) Two digit system
d) Zsigmondy/Palmer system & ADA system
(COMEDK-15)

95} D 96} A 97} B 98} D 99} C 100} A 101} A 102) B 103) B


, ORAL ANATOMY

2. PERMANENT DENTITION - ANSWERS


1. 'D' [Wheeler's g th ed 276/ 9th ed 178]
Shapes of the occlusal surfaces of some teeth
Permanent maxillary 1't premolar Hexagonal
Permanent mandibular 1't premolar Diamond
Permanent mandibular znd premolar Square
• Permanent maxillary 1't molar
Rhomboidal
• Primary maxillary znd molar
Permanent maxillary 2nd molar Rhomboidal with more obtuse angles
Permanent maxillary 3,d molar Heart shape
Permanent mandibular 1't molar Hexagonal/Trapezoidal
Primary maxillary 1't molar Rectangular

2. ' D' [Wheeler's g th ed 192/ 9th ed 127]

• Maxillary canine (10+17mm)


Longest tooth in oral cavity
• Mandibular canine (11+1 6mm)
Longest crown in maxillary arch Central incisor (10.5mm)
Longest crown in mandibu lar arch or longest crown in oral cavity Mandibular canine (11mm)
Maximum root length in maxillary arch/longest root in oral cavity Maxillary canine (17mm)
Maximum root length in mandibular arch Mandibular canine (16mm)
Shortest tooth of the dentition or shortest tooth in maxillary arch Maxillary second molar (6.5+11mm)
Shortest tooth in mandibular arch Second molar (7+13)
Shortest crown in the dentition or shortest crown in maxillary arch Maxillary second molar (6.5mm)
Shortest crown in mandibular arch Second molar (7mm)
Shortest root in maxillary arch or shortest root in oral cavity Maxillary second molar (11mm)
Shortest root in mandibular arch Central incisor (12.5mm)
Largest mesiodistal diameter of crown in anteriors Maxillary central incisor
Largest mesiodistal diameter of crown in oral cavity Mandibular l't molar
Largest labiolingual diameter or crown in anteriors Maxillary canine
Largest buccolingual diameter of crown in oral cavity Maxillary 1st molar
Posterior tooth with longest root Palatal root of maxillary 1st molar
Largest mesiodistal diameter in primary dentition Mandibular z nd molar
Largest buccolingual diameter in primary dentition Maxillary 2"d molar (AP- 2013)

3. 'A' [Wheeler's g •h ed 161/ 9th ed 106]


• Maxillary lateral incisor - Palatogingival groove
• Mandibular 1st premolar - Mesiolingual development groove
• Maxillary 1st premolar - Mesial marginal developmental groove
Dental ;lut.,e

4. 'C' [Wheeler's 8th ed 203/ 9th ed 132] 11. 'A' [Wheeler's 8th ed 161/ 9th ed 106]
Greater crown bulk distal to the faciolingua l bisecting Maxillary lateral incisors vary in form more than any
plane of a tooth is typical of both maxillary and other tooth in the mouth next to the third molar. Peg
mandibular canines. shape, pointed tubercle as part of cingulam, deep
development grooves which extend down on the root
5. 'A' [Wheeler's 8th ed 312/ 9th ed 197] lingually with a deep fold in the cingulam, twisted
The largest cusp in Permanent Mandibular 1st Molar is roots and distorted crowns are some of the morphologic
Mesiobuccal and the smallest cusp is the Distal Cusp variations seen in maxillary lateral incisor.
{AP-08).
12. 'C' [Wheeler's 8th ed 239/ 9th ed 157]
The largest cusp in permanent maxillary 1s1 molar is
mesiolingual and the smallest cusp is distolingual. 13. ' D' [Wheeler's 8th ed 227, 229/ 9th ed 149-150]
Cusp of carabellis is present only in some families. The mesial marginal developmental groove present in
the maxillary first premolar crosses the mesial marginal
6. 'D' [Wheeler's 8th ed 276/ 9th ed 178, 180] ridge and ends on the mesial surface of the crown.
This is due to presence of carabelli and larger
mesiolingual cusp. It is also wider mesially than 14. ' B' [Wheeler 81h ed 228 Fig 9.14/9th ed 150 Fig 9.15]
distally. Maxillary first premolar can be identified as right or
left by observing its characteristic features.
7. 'B' [Wheeler's 8th ed 358/ 9th ed 225, 138] i) Mesial slope of bucca l cusp is longer than distal
Bifurcated roots and root canals are sometimes seen slope
in mandibular canines. If they are present they are
ii) Mesial concavity (or) depression below the mesial
situated one labially and one lingually. Among the
contact area.
anterior teeth mandibular canines have bifurcated
iii) Mesial marginal developmental groove.
roots commonly. Among the posteriors mandibular first
premolars have bifid roots most commonly.
The lingual cusp is smaller, spheroidal with a pointed
cusp tip. The mesial and distal slopes are equal and
8. 'C' [Wheeler's 8th ed 264, 297/ 9th ed 172, 189]
meet at 90°. The lingual cusp tip is mesial to the
Five cusps are present in permanent maxillary and
buccal cusp.
mandibular first molars and primary maxillary and
mandibular second molars.
15. 'C' [Wheeler's 8th ed 249/ 9th ed 164]
Usually the distal marginal ridge is at a lower level
The fifth cusp present in permanent maxillary first
than mesial marginal ridge except for the mandibular
molar and primary maxillary second molar is called the
first premolar where the distal marginal ridge is at a
Cusp of carabelli ( or) tubercle of carabelli. This cusp
higher level than mesial marginal ridge.
is attached to mesiolingual surface of the mesiolingual
cusp and is approximately 2mm cervical to the cusp
16. ' B' [Wheeler's 8 1h ed 265/ 9th ed 17 3]
ridge of the mesiolingual cusp.

The fifth cusp present in permanent mandibular fi rst 17 • 'C' [Wheeler's 8th ed 331/ 9th ed 207]
molar is distal cusp and in primary mandibular second
molar is buccal cusp. 18. 'B' [Wheeler's 9th ed 7]
Cingulam is present in the lingual surface of incisors
and canines (6+6).
9. 'C' [Wheeler's 81h ed 260/ 9th ed 178]

10. 'A' [Wheeler 8th ed 273, Fig 11 ,8/ 9 th ed 178, Fig 19. ' D' [Wheeler's 8th ed 211/ 9th ed 134]
11.8]
Palatal root is the longest and distobuccal root is the 20• 'A' [Wheeler's 8th ed 167I 9th ed 109]
smallest of the three roots of maxillary first molar.
21. ' B' [Wheeler's 8 1h ed 239/ 9th ed 157]
, ORAL ANATOMY

22. 'B' [Wheeler's 8th ed 161/ 9th ed 106] 34. 'D' [Wheeler's 8th ed 264/ 9th ed 171]
Maxillary first molar is the largest tooth in the
23. 'D' [Wheeler's Sth ed 18, Tab 1-1/9th ed 13, Tab 1.1] permanent dentition. In mandibular arch permanent
first molar is the largest tooth.
24. 'B' [Wheeler's 8th ed 285, 287 / 9th ed 182-183]
Maxillary 2nd molars have four well-formed cusps, and 35. 'C' [Wheeler's 8th ed 297 / 9th ed 189]
three roots.
36. 'C' [Wheeler's Sth ed 191/ 9th ed 125]
25. 'A' [Wheeler's Sth ed 130-132/ 9th ed 86-88]
37. 'C' [Wheeler's 8th ed 297 / 9th ed 189]
26. 'B' [Wheeler's 8th ed 193 Fig. 8.2/ 9th ed 126-28 Mandibular first molar has 5 well-developed cusps - two
Fig 8.2] buccal, two lingual and a distal cusp. It has two well-
developed roots - one mesial and one distal and three
27. 'B' [Wheeler's Sth ed 260/ 9th ed 169] root canals - Mesiobuccal, Mesiolingual and distal.
Occasionally a fourth canal is present in distal root.
28. 'B' [Wheeler's 8th ed 169/ 9th ed 109]
In the maxillary lateral incisor, the lingual fossa is 38. 'C' [Wheeler's 8th ed 359/ 9th ed 163]
more pronounced with developmental grooves. Its
mesioincisal angle is sharp whereas disto incisa l angle 39. 'K [Wheeler's Sth ed 105/ 9th ed 9]
is rounded. Permanent incisors develop from 4 lobes, three labial
and one lingual. The lingual lobe is represented by
The lingual surface of mandibular lateral incisor is the cingulam and the 3 labial lobes terminate incisally
marked by the absence of the developmental grooves in rounded eminences called Mamelons, which are
with little pronounced cing ulam and ling ual fossa. prominent in newly erupted incisors.
Both the mesia l and distal incisal line angles are sharp.
40. 'C' [Wheeler's 8th ed 277 / 9th ed 180]
29. 'B' [Wheeler's 8th ed 179/ 9th ed 119] Oblique ridge is formed by the union of triangular ridge
The mandibular central incisor is bilaterally symmetrical of distobuccal cusp and distal ridge of mesiolingual
when viewed labially and incisally. The mesial and distal cusp. The oblique ridge is reduced the level of marginal
incisal angles are very sharp and the mesial and distal ridges at t he center of occlusal surface.
outlines of the crown make a straight drop downward.
41. 'C' [Wheeler's 8th ed 239/ 9th ed 157]
The incisal ridge is straight and is at right ang les to the Tooth Lobes
long axis of the tooth. Because of all these features it
is difficult to distinguish the mesial and distal aspects • Permanent incisors
of mandibular central incisor and so the right and left • Permanent canines 4
central incisors are difficult to identify. • Permanent premolars
• 3-cusp type mandibular 2 nd premolar 5
30. 'B' [Wheeler's 8th ed 247/ 9th ed 163]
• Permanent maxillary l't molar
5
31. 'D' [Wheeler's sth ed 271/ 9th ed 175] • Permanent mandibular l51 molar
• Primary maxillary 2nd molar
5
32. 'C' [Wheeler's 8th ed 239/ 9th ed 157] • Primary mandibular 2nd molar

33. 'B' [Wheeler's 8th ed 251/ 9th ed 235, 236]


Other permanent molars 4
From the lingual aspect, little variation may be seen Primary incisors 1
between 4 and except that the linual cusp of of second
premolar is longer, which makes the crown longer on The minim um number of developmental lobes in
the lingul sde. permanent teeth is four (AHMS MAY 2012) .
Dental ;lut.,e

42. 'C' [Wheeler's 81h ed 191/ 9th ed 125] • Buccal bifurcation is 4mm above the cervical line.
First permanent molar is located at the center of the • Distal bifurcation is 5mm above the cervical line.
fully developed adult jaw anterioposteriorly. They are
called "Corner stones" of dental arches. 52. 'C' [Wheeler's s th ed 105/ 9th ed 71]

Canines are called the "Corner teeth" that help to 53. ' B' [Wheeler's Sth ed 349/ 9th ed 219, 220]
form a foundation for normal facial expression at the
corners of the mouth. 54. ' D' [Wheeler's Sth ed 112/ 9th ed 74]

43. 'B' [Wheeler's 8th ed 222/ 9th ed 145] 55. 'C' [Wheeler's 8th ed 260/ 9th ed 169]
Mesial slope of the buccal cusp shorter than distal slope
placing the buccal cusp mesial to a line bisecting the 56. 'A' [Wheeler's Sth ed 276, 287/ 9th ed 184]
buccal surface of crown is seen in maxillary canines,
mandibular canines and maxillary second premolar. The 57. 'A' [Wheeler's 8th ed 271/ 9th ed 172]
mesial slope is longer than distal slope in maxillary The cusp ridge of 5th cusp or cusp of carabelli is
first premolar. approximately 2mm cervical to the cusp ridge of the
mesiolingual cusp.
44. 'B' [Wheeler's Sth ed 226/ 9th ed 149]
58. 'C' [Wheeler's 8th ed 276/ 9th ed 7]
45. 'C' [Wheeler's 8th ed 260/ 9th ed 169]
59. ' B' [Wheeler's 8th ed 277 / 9th ed 180]
46. 'A' [Wheeler's 8th ed 287 / 9th ed 184] Permanent MAXILLARY first molar contains:
i) 2 major fossa - centra l and distal fossa
4 7. 'D' [Wheeler's Sth e d 27 6/ 9th ed 180]
ii) 2 minor fossa - distal and mesial t riangular
The tritubercular cusps are mesiobuccal, distobucca l
iii) 5 developmental grooves
and mesiolingual cusps. These are the primary cusps.
iv) 1 oblique ridge
Distolingual cusp is a secondary cusp.
v) 4 cusp ridges
48. 'B' [Wheeler's Sth ed 276/ 9th ed 180]
In t he permanent maxillary 1st molar, mesiobuccal and Permanent MANDIBULAR 1!! molar contains:
distolingual cusps are acute angled while mesiolingual i) 1 major fossa - centra l fossa
and distobuccal cusps are obtuse angled. ii) 2 minor fossa - Mesial and distal triangular
iii) 4 developmental grooves
49. 'B' [Wheeler's Sth ed 285/ 9th ed 182] iv) 5 cusp ridges
Someti mes in maxillary second molar, the distolingual
cusp is poorly developed making it to resemble a third 60. ' D' [Wheeler's 8th ed 239/ 9th ed 163]
molar (heart shape).
61. 'C' [Wheeler's 8th ed 106/ 9th ed 71-72, Fig 4.12]
50. 'B' [Wheeler's 8th Ed 345/ 9th ed 216]
Cervical cross section Outline form 62. 'A' [Wheeler's Sth ed 112/ 9th ed 73-74, Fig 4.16]

Maxillary 1st premo lar Kidney shape


63. ' D' [Wheeler's 8th ed 116, 460/ 9th ed 288]
Maxillary z nd premolar Ovoid
Maxillary 1st molar Rhomboidal 64. 'C' [Wheeler's Sth ed 347, 362/ 9th ed 220, 227]

Mandibular 1st molar Quadrilateral 65. ' D' [Wheelers 8 1h ed 27 6, 277, 312/ 9th ed 178, 180)

51. 'B' [Wheeler's Sth ed 273/ 9th ed 176] 66. 'A' [Wheeler's 8th ed 315/ 9th ed 201]
• Mesial bifurcation is 3mm above the cervical line. Permanent mandibular second molar contains 4 cusps.
The distal cusp is usually absent.
, ORAL ANATOMY

67. 'C' [Wheeler's 8th ed 287, 291/ 9th ed 184-185) The point angles of the posterior teeth ( 4)
The roots of maxillary third molars are usually fused Mesiobucco occlusal Mesioling uo occlusal
and have a more distal relation to the roots of maxillary Distobucco occlusal Distili nguo occlusal
second molar.
78. 'K [Wheelers 8th ed 215/ 9th ed 141)
68. 'A' [Wheeler's 8th ed 341/ 9th ed 213) The buccal cusp of maxillary first premolar long and
sharp resembles canine. The second premolars both
69. 'A' [Wheeler's Sth ed 269, 308/ 9th ed 176, 194) upper and lower are less sharp resembling molars.
The root trunk length buccally in the mandibular first
molar is 3mm which is equal t o the mesial bifurcation 79. 'A' [Wheelers 8th ed 222/ 9th ed 145)
length (3mm) of the maxillary first molar. Longer distal slope of buccal
Maxillary canine
cusp than mesial slope
The root trunk length Lingually of the mandibular first
molar is 4mm, which is equal to the buccal bifurcation Maxillary 1'1 Longer mesial slope of the
premolar buccal cusp than distal slope
length (4mm) of maxillary first molar.
Mandibular Longer distal slope of the bucca l
70. 'D' [Wheeler's 8th ed 2 70, 28 7, 291/ 9th ed 17 4, canine cusp
183, 185)
The disto-lingual cusp becomes progressively smaller 80. 'B' [Wheelers 9th ed 73-7 4, Fig 4.16)
in second and third maxillary molars. For explanation refer to synopsis miscellaneous point
No. 19.
71. 'C' [Wheeler's 8th ed 172, 179/ 9th ed 113, 119)
81. 'B' [Wheelers 8th ed 45, 53/ 9th ed 36, Tab 2.3)
72. 'C' [Wheelers Sth ed 239/ 9th ed 157)
The mandibular fi rst premolar has a large buccal cusp 82. 'D' [Wheelers s th ed 161/ 9th ed 106)
which is long and well formed with a sma ll non-
functioning lingual cusp no longer than a cingulam 83. 'K [Wheeler's s th ed 53 Tab 2-3/ 9th ed 36, Tab 2.3)
found on maxillary canines.
84. 'D' [Wheeler's 8 1h ed 53, Tab 2.3/ 9th ed 36, Tab 2.3]
73. 'A' [Wheelers 8th ed 18/ 9th ed 13, Tab 1.1)
Maxillary canine 85. 'D' [Wheeler's 8th ed 105/ 9th ed 71)
Length of crown - 10mm At age of 8 years incisors erupt and newly erupted
Length of root - 17mm incisors have mamelons, so the child does not require
any treatment.
Mandibular canine
Length of crown - 11mm
Length of root - 16mm 86. 'K [Wheeler's 8th ed 244/ 9th ed 157)
Ref. Synopsis
74. 'B' [Wheelers 8th ed 267 / 9th ed 171)
87. 'B' [Wheeler's 9th ed 131, Tab 8.1)
75. 'C' [Wheelers 81h ed 1/ Fuller's Concise Dental Maxillary Canine
Anatomy 4th ed 3) • M-D diameter of crown - 7.5
• Length of crown - 10
76. 'A' [Wheelers 81 h ed 53/ 9th ed 36, Tab 2.3]
Mandibular Canine
77. 'C' [Wheelers 8th ed 15/ 9th ed 10, Fig 1.15) • M-D diameter of crown - 7.0
Point angles of the anterior teeth ( 4)
• Length of crown - 10
Mesiolabio incisal Mesiolinguo incisal
Distolabio incisal Distolinguo incisal
88. 'C' [Wheeler's 8th ed 276/ 9th ed 180)
Ref. Q. No. 48 for explanation
Dental ;lut.,e

89. 'C' [Wheeler's 9th ed 217, 225] 91. ' B' [Ten Cate's 8th ed 141]
Amelogenesis is a slow developmental process that
90. 'C' [Wheeler's 8th ed 14/ 9th ed 9] can take as long as 5 years to complete on the crowns
Ref. Synopsis Point 19 in introduction of some teeth and maturation stage occupies two-
thirds of it.

92. 'D' [Wheeler's 9th ed 4]


A two digit system proposed by Federation Dentaire Internationale (FOi) for both primary and permanent dentitions
has been adopted by World Health Organisation (WHO) and accepted by other organizations such as International
Associations for Dental Research.

Permanent dentition: Deciduous dentition:


First digit Quadrant First digit Quadrant
1 Maxillary right quadrant 5 Maxillary right quadrant
2 Maxillary left quadrant 6 Maxillary left quadrant
3 Mandibular left quadrant 7 Mandibular left quadrant
4 Mandibular right quadrant 8 Mandibular right quadrant

PRIMARY DENTITION

Upper right Upper left

55 54 53 52 51 61 62 63 64 65
85 84 83 82 81 71 72 73 74 75

Lower right Lower left

Upper right Upper left


18 17 16 15 14 13 12 11 21 22 23 24 25 26 27 28
48 47 46 45 44 43 42 41 31 32 33 34 35 36 37 38
Lower right Lower left

93. 'A' [Wheeler's 9th ed 180] • In premolars, they are called as mesial, buccal,
Developmentally, only three major cusps can be distal and lingual lobes.
considered as primary, the mesiolingual cusp, • In mandibular second premolar, which often gas
mesiobuccal cusp and distobuccal cusp. The distoli ng ual two lingual cusps, they are mesial, buccal, distal,
cusp is common to all of the maxillary molars; any mesiolingual and distolingual lobes.
other additional one, such as cusp of Carabelli on first
• In permanent molars, they are named the same as
molars, must be regarded as secondary.
cusps

94. 'B' [Wheeler's 9th ed 4] 96. 'A' [Wheeler's 9th ed 150]


Mesial marginal developmental groove is seen in
9 5. 'D' [Wheeler's 9th ed 71)
Maxillary 1st Premolar. This groove crosses the mesial
Each lobe represents primary centre of formation. In
marginal ridge and ends on the mesial surface of the
permanent anterior teeth, the four lobes are called as
crown.
mesia l, labial, distal and lingual lobes.
, ORAL ANATOMY

97. 'B' [Wheeler's 9th ed 8]


Ridge crossing obliquely the occlusal
A developmental groove is a shallow groove or line
surfaces of maxillary molars formed
between the primary parts of the crown or root. Oblique
by the union of triangular ridge of
Supplemental groove is less distinct and is a shallow ridge
the DB cusp and the distal cusp ridge
linear depression on the surface of the tooth, but it is
of the ML cusp
supplemental to the developmental groove and does
not mark the junction of primary parts.
101. 'I( [Wheelers 9th ed 2]
In 1947, a committee of the ADA recommended the
98. 'D' [Wheeler's 9th ed 71]
'symbolic' Zsigmondy/palmer system. Because of the
See Q. No. 95
difficulties with keyboard notation, the ADA officially
recommended 'universal' numbering system. This
99. 'C' [Textbook of Dental Anatomy and Oral Physiology
system also had limitations and lack of widespread use
by Manjunatha BS 1st ed 116]
internationally. Finally the WHO adopted FDI notation.
Even-though answers is trapezoidal according to
the above reference and university Key, many books
102. 'B' [Wheelers 9th ed 8)
say that the occlusal outline of mandibular is either
Hexagonal (not in options) according to Wheeler's 9th Long depression or valley in
ed 197] or pentagonal (not in options) according to the surface of a tooth between
Woelfel's Dental Anatomy By Rickne C. Scheid 8th ed Sulcus ridges and cusps. The sulcus has
148]. a developmental groove at the
junction of its inclines.
According to Woelfel's Dental Anatomy By Rickne C. Shallow groove or line between
Developmental
Scheid 8th ed 148, the various occlusal outlines are the primary parts of the crown or
groove
root.
Trapezoid Less distinct shallow linear
Mandibular second molar depression which is supplemental
(Tapered rectangle) Supplemental
• Maxillary second molar to a developmental groove and
Paralleologram groove
• Maxillary second molar does not mark the junction of
(Rhomboid) primary parts.
(4-cusp type)
Hear shaped or Maxillary second molar (3-
103. 'B' [Check Synopsis Point No.69]
Triangular cusp type)
In maxillary and mandibular first molars, the
Pentagon Mandibular first molar
calcification starts at birth and the crown formation
is completed by 21/2-3 years. Any developmental
100. 'A' [Wheelers 9th ed 8)
disturbance (hypoplasia, hypocalcification) during
Ridge that descend from the tips of this period will manifest in their crowns.
the cusps of molars and premolars
toward the central part of the
Triangular
occlusal surfaces. Named after the
ridge
cusps to which they belong.
Eg: Triangular ridge of the buccal
cusp of the maxillary first premolar
When a buccal and a lingual
Transverse
triangular ridge join, they fo rm a
ridge
transverse ridge.
Dental ;lut.,e

3. CALCIFICATION AND ERUPTION OF TEETH


1. The last primary tooth to be replaced by a permanent 7. The first permanent tooth to erupt in the oral cavity
tooth is usually the: is the:
a) Maxillary second molar a) Mandibular central incisor
b) Mandibular second molar b) Mandibular first molar
c) Maxillary canine c) Maxillary central incisor
d) Mandibular canine d) Maxillary fi rst molar
(MAN,AIPG-97) (MAN-95;KAR-99)
2. The most common sequence of eruption of the 8. First evidence of initiation of primary dentition is
permanent maxillary teeth in children is: usually around:
a) 1-6-2-3-1,-7-5-8 b) 2-3-1-6-7-5-li-8 a) Ii months in utero b) Bi rth
c) 6-1-2-4-3-5-7-8 d) 6-1-2-3-4-5-7-8 c) 4 months after birth d) One year after birth
(MAN-94,97;AP-02) (MAN-97)
3. Calcification of permanent first molar usually 9. Eruption date of deciduous maxillary 2nd molar is:
begins in the: a) 20 months b) 18 months
a) Third month of intrauterine life c) 24 months d) 28 months
b) In t he sixth mont h of intrauterine life (MAN-2K)
c) At birth 10. While charting the dentition of a child, the
d) In the third month of extra uterine life pedodontist finds that the following teeth are
(MAN-95,98) present in all the four quadrants; permanent
4. A radiograph of 4-year-old child reveals no evidence central incisor, permanent lateral incisor, deciduous
of calcification of mandibular second pre molar. cuspid, deciduous first molar, deciduous 2nd molar
This indicates and permanent 1•1 molar. The eruptive pattern may
a) It is too early to make fina l prediction be considered most nearly normal for a child aged:
b) The teeth may develop later a) 3-6 years b) 7- 10 years
c) Primary second molar extracted to allow mesial c) 11-14 years d) 15-18 years
drifting of permanent first molar (MAN-94)
d) The child may never develop premolars 11. Which of the following is dental formula of a 9 year
(MAN- 98) old child:
5. The usual order of appearance of the primary teeth a) 6EDC21/6EDC21 b) 6EDCBA/6EDC21
in the mouth is: c) EDC21/EDC21 d) 76EDC21/76EDC21
a) Central incisors, first molars, lateral incisors, (MAN-99)
second molars, canines 12. Dentition of humans is:
b) Central incisors, lateral incisors, canines, second a) Herbivorous b) Omnivorous
molars, first molars c) Carnivorous d) None of the above
c) Central incisors, lateral incisors, first molars (AIIMS-95)
canines, second molars 13. Number of teeth seen on X-ray at birth:
d) Central incisors, lateral incisors, first molars second a) 20 b) 22
molars, canines c) 24 d) 30
(MAN- 95) (AIIMS- 95, TN PSC- 99)
6. Generally in majority of children, the sequence of 14. A child has 12 permanent and 12 deciduous teeth,
eruption of permanent teeth in mandible is what is his age:
a) 1-6-2-3-5-4-7-8 b) 6-1-2-4-5-3-7-8 a) 8 1/2 years b) 11 1/z years
c) 6-1-2-3-4-5-7-8 d) 6-1-2-4-3-5-7-8 c) 4 l/2 years d) 13 1'2 years
(MAN- 97) (AIPG- 03;PGI- 02)

1) C 2) C 3) C 4) B 5) C 6) C 7) B 8) A 9) D 10) B 11) A 12) B 13) C


14) A
, ORAL ANATOMY

15. At birth the following teeth are calcifying: 23. Which of the following gives the best description of
a) Primary incisors & first molar erupted permanent teeth typically seen in a 9-year
b) Primary canines & second molars old
c) Permanent canines a) On ly molars
d) Permanent 3,d molar b) All teeth except third molars
(PGI-02) c) 1,2,4,5,6,7 in all quadrants
16. What is the amount of time required by erupting d) 1,2,6 in all quadrants
premolar to move through 1 mm of bone as measured (KAR- 98)
on a bite-wing radiograph? 24. Calcification of all teeth except last molars are
a) 3-4 months b) 4-5 months completed by:
c) 5-6 months d) 6-7 months a) 2 years b) 4 years
(COMED, PGI-2012) c) 6 years d) 8 years
17. There are two normal children of the same age and (AIPG-98,96)
sex. In one, all the permanent teeth except the third 25. Growth of all permanent teeth except last molars is
molars have erupted. In the other, no permanent completed by:
canines or premolars have erupted. How old are a) 6 years b) 8 years
these children? c) 12 years d) 17 years
a) 7 years b) 9 years (AIPG- 95)
c) 11 years d) 13 years 26. Once the enamel formation is complete, the tooth
(KAR,AIPG-01) will erupt after approximately:
18. Kalu, a 6-year-old child, received tetracycline. a) One year b) Two years
Noticeable discoloration will be seen in : c) Three years d) Four years
a) Premolars, incisors and 1'1 molars (KAR-01)
b) Canine and 2nd molars 27. The root completion of permanent teeth is achieved
c) Canin es, premolars and 2nd molars after__ years of eruption:
d) Incisors and 1st molars a) l - l1h b) Z - 3
(AIPG- 01) c) 3 - 4 d) 4 - 5
19. As compared to permanent tooth, root formation in (KAR-9 9;AP-98;AIPG-93)
primary tooth: 28. The correct dentition in a 9-year-old child is:
a) Take less time a) 12CDE6 b) 12C456
b) Take more time c) 123DE6 d) 123456
c) Same time (AIPG-01)
d) Is complete when tooth erupts in oral cavity 29. Before eruption the position of permanent
(PGI- 95) mandibular incisor buds relative to primary incisors
20. Calcification of t'1ird molar begins at: is:
a) 8 months b) 18 months a) Superior and facial b) Inferior and facial
c) 8 years d) 16 years c) Superior and lingual d) Inferior and lingual
(AIIMS-95,94) (AIPG-91)
21. A six-year molar begins to calcify at: 30. The deciduous canine emerges:
a) Six months IU b) At birth a) Before lateral incisor b) Before first molar
c) 1 year after birth d) 3 years age c) After first molar d) After second molar
(PGI-01,02) (PGI-2K)
22. The time taken for primary tooth root completion 31. Which of these teeth are highly sturdy and usually
after eruption: the last ones to be lost?
a) 3 years b) 1 year a) First molar b) Central incisors
c) 2 years d) 1 month c) Canines d) Premolars
(KAR-03) (AP-2012)

15) A 16) B 17) C 18) C 19) A 20) C 21) B 22) B 23) D 24) D 25) D 26) C 27) B
28) A 29) D 30) C 31) C
Dental ;lut.,e

32. Calcification of roots of deciduous teeth is a) 6 months in utero b) 6 months after birth
completed by: c) 9 months in utero d) 9 months after birth
a) 2 years b) 4 years (UPSC-2001)
c) 6 years d) 8 years 41. Beginning of eruption of permanent tooth starts:
{AIIMS-94) a) After completion of root
33. In the majority of situations, the first deciduous b) When apical third root is still to be formed
tooth to erupt is the: c) When half of the root is still to be formed
a) Mandibular central incisor d) When root formation starts
b) Mandibular first molar (TNPSC-99)
c) Maxillary central incisor 42. The tooth bud for the third molar is initiated at
d) Maxillary second molar about the age of:
{KAR- 03, 98;AP- 05) a) 61hmonth LU b) 1•t yearoflife
34. The eruption are of maxillary permanent lateral c) 2"d year of life d) 8'h year of life
incisor is: (TN PSC-99)
a) 8-9 years b) 6-7 years 43. In how many years, the mandibular central incisors
c) 10-11 years d) 8-9 months erupt as permanent teeth?
(COMEDK- 04) a) 8-9 years b) 10-12 years
35. Primary canine erupts: c) 6-7 years d) 17-25 years
a) After D b) Before B (TNPSC-99)
c) After B d) After E 44. The first macroscopic indication of morphologic
{PGI-99) development of primary incisors approximately at:
3 6. In anxious parent calls you that her 4-month-old a) 11 weeks in utero b) 14 weeks in utero
child has still no teeth in his mouth then you c) 16 weeks in utero d) 6 weeks in utero
should: {COMEDK-06)
a) Call and do radiographic survey 45. The teeth that are retained for the longest period
b) Tell her that it is normal at this age of time are:
c) Child has retarded growth a) Maxillary incisors b) Maxillary canines
d) Systemic problem c) Mandibular incisors d) Mandibular canines
(PGI-97) (COMEDK-06)
3 7. First evidence of calcification of maxillary 46. A new tooth is erupting in the mouth of a one and
permanent central incisors occurs at: a half year old child. This tooth is most likely to be
a) 3-4 months b) 10-12 months a deciduous:
c) 2-3 years d) 3-4 years a) Maxillary central incisor
(KAR-01) b) Mandibular lateral incisor
38. First permanent molars begin to calcify at: c) Maxillary lateral incisor
a) 6 months of intra uterine life d) Maxillary canine
b) At birth {COMEDK- 05)
c) Before deciduous incisors 47. The process of exfoliation of primary teeth is
d) About 1 year after birth between
(COM ED K-04,AIIMS-91) a) Seventh and eight years
39. Root completion of the mandibular first molar b) Seventh and ninth years
takes place by the age: c) Seventh and tenth years
a) 7-8 years b) 8-9 years d) Seventh and twelfth years
c) 9-10 years d) 10-11 years (BHU-2012)
{APPSC-99) 48. The first evidence of calcification of primary
40. Maxillary primary cuspid enamel completion takes anterior teeth begins approximately between:
place at: a) 14 to 17 weeks in utero

32) B 33) A 34) A 35) A 36) B 37) A 38) B 39) C 40) D 41) C 42) D 43) C 44) B
45) B&D 46) D 47) D 48) A
, ORAL ANATOMY

b) 9 to 12 weeks in utero
c) 6 to 9 weeks in utero
d) 17 to 20 weeks in utero
(COMEDK-07, KERALA- 15)
49. Which of the following is true about calcification of
teeth?
a) Calcification of primary teeth is almost complete at
time of birth
b) Calcification of all primary teeth and few permanent
teeth complete at birth
c) Calcification of all permanent teeth complete at
birth
d) Calcification of primary teeth starts around birth
(AIPG-2009)

49) A
Dental ;lut.,e

3. CALCIFICATION AND ERUPTION OF TEETH - ANSWERS


1. '(' [Wheeler's 8th ed 45, 46/ 9th ed 30-31] 8. 'A' [Wheeler's 8th ed 53/ 9th ed 23]
Permanent maxillary canines are t he last teeth to
erupt into oral cavity (after 3rd molars). They replace 9. 'D' [Wheeler's 81 h ed 53/ 9th ed 23)
the deciduous canine and erupt into oral cavity at the
age of 11-12 years. 10. 'B' [Wheeler's 8 1h ed 53/ 9th ed 31)
As the permanent lateral incisors in all the fo ur
2. 'C' [Wheeler's 8th ed 449/ 9th ed 282] quadrants have erupted, the age should be above 8
Dentition Eruption sequence years and as the deciduous canines, molars are present
the age should be below 10 years.
Permanent maxillary 6-1-2-4-3-5-7-8 (or)
dentition 6-1-2-4-5-3-7-8 11. 'A' [Wheeler's 8th ed 34, 53/ 9th ed 25, 31)
Permanent mandibular
6-1-2-3-4-5-7-8
dentition 12. 'B' [Wheeler's 8 1h ed 113/ 9th ed 75]

Primary dentition
13. 'C' [Wheeler's 8 1h ed 53/ 9th ed 23, 31)
Maxillary /Mandibular arch A-B-D-C-E At birth all the deciduous teeth (20) and permanent
first molars (4) can be seen in an x-ray.
3. 'C' [Wheeler's 81h ed 53/ 9th ed 31)
First evidence of cakification of permanent teeth has 14. 'A' [Wheeler's 8 1h ed 34, 53/ 9th ed 25, 31)
the range of t ime fro m birth (first molars) to 8 years 12 permanent teeth (8 incisors and 4 first molars) and
(Third molars) . By 8 years calcification of permanent 12 deciduous teet h (4 canines and 8 molars) could be
teeth is completed except for third molars. present at 8 1/2 years.

Calcification of deciduous teeth begins about the 4th 15. 'A' [Wheeler's 8 1h ed 53/ 9th ed 23, 31)
month of prenatal life (centra l incisors) and by 6th
month all of the deciduous teeth will begin to develop. 16. 'B' [Principles and Practice of Pedodontics by Arathi
Rao 2nd ed 128]
4. 'B' [Wheeler's 8 1h ed 53/ 9th ed 31) Erupting premolars usua lly requires 4-5 months to
move through 1 mm of bone as measured on a bite-
5. 'C' [Wheeler's 3th ed 58/ 9th ed 41] wing radiograph.

6. 'C' [Wheeler's 8th ed 449/ 9th ed 282] 17. 'C' [Wheeler's 8 1h ed 53/ 9th ed 31]

7. 'B' [Wheeler's 8 1h ed 53/ 9th ed 3 0-31) 18. 'C' [Wheeler's 81h ed 53/ 9th ed 31 J
First permanent tooth to calcify is maxillary pt molar Tetracycline causes staining of teeth, which are in the
and first permanent tooth to erupt is mandibular 1st process of calcification. At 6 years of age, first molars
molar. The general rule is mandibular teeth tend to and incisors are calcified completely so staining does
precede maxillary teeth in the process of eruption not occur in them. Canines, premolars and 2"d molars
both in deciduous and permanent dentition. are in the process of calcification, so they are affected
by tetracycline. Calcification of permanent teeth is
completed by 8 years with the exception of 3rd molars.
Which of the following permanent tooth is first
to calcify? (PGI Dec-2011) 19. 'A' [Wheeler's 8 1h ed 53/ 9th ed 23, 31]
a) Maxillary permane nt 1st molar
Usually in permanent teeth, root completion occurs
b) Mandibular permanent 1st molar
2112 - 3 years after eruption. In deciduous teeth it
c) Maxillary central incisor
takes 1 - Ph year for root completion after eruption.
d) Mandibular central incisor
, ORAL ANATOMY

20. 'C' [Wheeler's 8th ed 53/ 9th ed 31] 36. ' B' [Wheeler's 8th ed 53, 58/ 9th ed 41]

21. 'B' [Wheeler's 8th ed 53/ 9th ed 31] 3 7. 'A' [Wheeler's 8th ed 53/ 9th ed 31]
Permanent first molars are called six-years molars
as they erupt into the oral cavity at 6 years of age. 38. 'B' [Wheeler's 8th ed 53/ 9th ed 31]
Permanent second molars are called as twelve-year
molars. 39. 'C' [Wheeler's Sth ed 53/ 9th ed 31]
Permanent mandibular first molar erupts at 6 years of
22. 'B' [Wheeler's Sth ed 53/ 9th ed 23] age. For root completion it takes 21/2 - 3 years after
eruption . So the root completion of first molars occurs
23. ' D' [Wheeler's 8th ed 53/ 9th ed 31] by 9-10 years.

24. 'D' [Wheeler's s•h ed 53/ 9th ed 31) 40. 'D' [Wheeler's s•h ed 53/ 9th ed 23)

25. ' D' [Wheeler's 8th ed 53/ 9th ed 31] 41. 'C' [Wheeler's §_!.h_ed 38]

26. 'C' [Wheeler's 8th ed 53/ 9th ed 31] 42. 'D' [Wheeler's 8th ed 53/ 9th ed 31]

27. 'B' [Wheeler's 8th ed 53/ 9th ed 31] 43. 'C' [Wheeler's 9th ed 31, 119]

28. 'A' [Wheeler's 8th ed 53/ 9th ed 31] 44. 'B' [Wheelers 8th ed 1/ 9th ed 1, 23]
The first set of teeth to be seen in the mouth is the
29. 'D' [Orban's 13th ed 333 Fig 13-1] primary dentition which begins to form prenatally
The permanent mandibular incisors are positioned about 14 weeks in utero.
apically and lingually to the deciduous mandibular
incisors. 45. 'B & D' According to the key provided (or) 'B>D'/
[Wheeler's 9th ed 25, 30 Fig 2-4]
30. 'C' [Wheeler's 8th ed 58/ 9th ed 41] Primary maxillary canine erupt at the age of 19 months
and mandibular canines erupt at age of 20 months.
31. 'C' [Wheeler's 9th ed 125] Primary mandibular canines a re exfoliated and replaced
earlier (10 yrs+/- 9 months) as compared to primary
32. 'B' [Wheeler's 8th ed 53/ 9th ed 23] maxillary canines(ll yrs +/- 9 months). So primary
When the child is 2 - 21/2 years of age all the primary maxillary canines are retained for the longest period
teeth are erupted into oral cavity. Root calcification of time.
(completion) of primary teeth is completed by 3-4
years of age. 46. 'D' [Wheelers 8th ed 3 2/ 9th ed 23]

By 12-13 years all the permanent teeth except third 47. 'D' [Wheeler's 9th ed 29]
molars are expected to erupt and root formation is • By 2 to 3 years of age primary dentition eruption is
completed by about 14-16 years of age. completed. They remain in service from 6 months
to 11.5 or 12 years of age.
33. 'A' [Wheeler's 8th ed 58/ 9th ed 23]
• Exfoliation starts from 7 to 12 years.
The first deciduous tooth to erupt in the mouth
is mandibular central incisor, which erupts at
48. 'K [Wheelers 8th ed 32, 35/ 9th ed 23, 25]
approximately 6 months of age.
49. 'A' [Wheeler's 8th ed 51/ 9th ed 37, Tab 2-4]
34. 'K [Wheeler's 8th ed 53/ 9th ed 31]

35. 'A' [Wheeler's 8th ed 58/ 9th ed 41]


Dental ;lut.,e

4. OCCLUSION
1. The maxillary teeth which have single antagonist b) Buccal slopes of lingual cusps of maxillary posterior
are: teeth
a) Lateral incisors b) Permanent canines c) Lingual slopes of lingual cusps of mandibular posterior
c) Permanent central incisors teeth
d) Third molars d) All slopes of lingual cusps of mandibular posterior
(MAN-94,95) teeth
2. In occlusion, the teeth have (AIIMS-03)
a) Cusp-to-cusp contact 9. In the intercuspal position, the lingual cusp of
b) Edge-to-edge contact maxillary 2"d premolar contacts the:
c) Marginal contact a) Distal fossa of mandibular 2"d premolar
d) Surface-to-surface contact b) Mesial fossa of mandibular 1st molar
(MAN-94;AIIMS-93) c) Distal marginal ridge of mandibular znd molar
3. In normal occlusion, with which grooves/ surface d) Distal marginal ridge of mandibular 1st premolar
of the permanent mandibular first molar does the (AIPG-02;AIIMS-94)
mesiobuccal cusp of the permanent maxillary first 10. In ideal intercuspation, maxillary canine articulates
molar occlude: with:
a) Distobuccal groove b) Mesial surface a) Mandibular canine and 1st pm
c) Mesiobuccal surface d) Transverse groove b) Mandibular canine and lateral incisor
(MAN-95) c) Mandibular 1st premolar and 2"d premolar
4. Group function occlusion is common in: d) Mandibular canine only
a) 10-14 years b) 15-25 years (AIPG-02)
c) Above 30 years d) Edentulous patients 11. In an ideal occlusion buccal cusps of maxillary
(MAN- 02) teeth, occlude with:
5. When a molar has no opposing tooth, it can become: a) Fossa and ridges b) Grooves and embrasures
a) Extruded b) Intruded c) Lingual cusps of mandibular teeth
c) Protruded d) Retruded d) Occlusal pits and fissures of mandibular teeth
(MAN- 95) (AP-97,AIPG-94)
6. Over jet is defined as: 12. The key to occlusion is:
a) Horizontal overlap b) Vertical overlap a) Maxillary permanent first molar
c) Transverse plane discrepancy b) Maxillary permanent second molar
d) All the above c) Maxillary primary first molar
(MAN-2K) d) Maxillary primary second molar
7. In centric occlusion, the cusp tip of the maxillary (AIPG-91)
canine is in alignment with which mandibular tooth: 13. If a permanent first molar is lost, the permanent
a) Facial embrasure of canine and premolar second molar drifts to the:
b) lncisal embrasure of canine and premolar a) Buccal side b) Distal side
c) Distal ridge of tne cusp of mandibular canine c) Mesial side d) Lingual side
d) Mesial ridge on the facial cusp of mandibular first (AIIMS-93)
premolar 14. The incisal edges and the incisal thirds of facial
(AIPG-01) surfaces of mandibular incisors and canines
8. Normal dentition in centric occlusion, opposing generally oppose lingual surfaces of maxillary
contact may be expected at: incisor and canines:
a) Buccal slopes of buccal cusps of maxillary posterior a) Within the incisal thirds
teeth b) At the junction of middle and cervical thirds

1) D 2) D 3) C 4) C 5) A 6) A 7) A 8) B 9) A 10) A 11) B 12) A 13) C


14) A
, ORAL ANATOMY

c) At the linguo cervical ridges a) First premolar b) Second premolar


d) Immediately incisa l to cingulam c) Deciduous 2nd molar
(AIPG-98) d) None
15. Distobuccal cusp of 27 falls into: (MCET-14)
a) Embrasure of 37 & 38 24. Centric holding cusp is?
b) Embrasure of 36 & 37 a) Mesiobuccal cusp of maxillary 1st molar
c) Mesial pit of 37 d) None of the above b) Mesiolingual cusp of mandibular first molar
(AIPG-93) c) Distolingual cusp of mandibular 1st molar
16. Maximum contact between occlusal surfaces, of d) Mesiopalatal cusp of maxillary 1st molar
maxillary and mandibular teeth occur during (AIIMS MAY -14)
a) Centric occlusion b) Rest position
c) Protrusive contacts d) Laterotrusive contacts

17. Maxillary facial and mandibular lingual cusps


require sufficient occlusal length and horizontal
overlap for:
a) Esthetics b) Centric stability
c) Chewing efficiency d) Soft tissue protection

18. In occlusion the teeth have:


a) Cusp to cusp contact b) Edge to edge contact
c) Marginal contact d) Surface to surface contact
(KAR- 04)
19. Desmodont is another name for:
a) The tooth with one wall pocket
b) Tooth with three walled pocket
c) Periodontal ligament
d) Dehiscence
(AIPG-05)
20. Curve passing through the buccal and lingual cusp
tips of the mandibular buccal teeth is:
a) Wilson curve b) Mansoon curve
c) Curve of Spee d) Catenary curve
(AIIMS-06)
21. Facial occlusal tine in the maxillary arch is formed
by
a) Stamp cusps b) Centric holding cusps
c) Supporting cusps d) Non-supporting cusps
(KCET-2012)
22. The position of maximum intercuspation of upper
and lower teeth is referred to as centric
a) Occlusion b) Relation
c) Position d) Bite
(MCET-14)
23. At the age of nine years, the mandibular permanent
first molar has its distal contact with which of the
following teeth?

15) A 16) A 17) D 18) D 19) C 20) A 21) D 22) A 23) D 24) D
Dental ;lut.,e

4. OCCLUSION - ANSWERS
1. 'D' [Wheeler's 8 1h ed 109, Fig 4-13/ 9th ed 73, Fig 4. 'C' [Wheeler's 6 th ed 415]
4.15] Canine guided occlusion is usually seen in young
In centric occlusion, each tooth has two antagonist individuals with unworn dentition. During lateral
teeth in the opposite arch with the exception of mandibular movement, the opposing upper and lower
mandibular central incisors and maxillary third molars, canines of the working side contact thereby causing
which have only one antagonist in the opposing jaw. disocclusion of all posterior teeth on the working and
balancing sides.
2. 'D' [Wheeler's 6 th ed 401]
In occlusion the teeth have: Group function occlusion is common in middle age
i) Surface contact. group and if the canines are weak. In addition to
canines, certain other posterior teeth contact on the
Eg.: Incisal portions of mandibular anteriors
working side during lateral movement of mandible.
contacts with the lingual surfaces of maxillary
anteriors.
5. 'A' [Wheeler's 8 1h ed 459/ 9th ed 288]
ii) Cusp-fossa contact
Eg.: Mesioling ual cusp of maxillary 1'1 molar 6. 'A' [Wheeler's 81h ed 453, Fig 16-16/ 9th ed 283,
occludes with t he centra l fossa of mandibular 1'1 Fig 16-16]
molar. Overjet is the horizontal overlap of the incisal ridges
iii) Cusp - Embrasure contact. or buccal cusp ridges of the maxillary teeth to the
iv) Ridge - Sulcus contact incisal ridges or buccal cusp ridges of the mandibular
teeth in cent ric occlusion. The presence of overjet in
Eg.: Triangular ridge of the distolingual cusp of
the molars prevents cheek biting~
the mandibular first molars fit into the ling ual
groove su lcus of the maxillary first molar.
Overbite is the vertical overlap of the teeth in which
the incisal ridges of maxillary anterior teeth extend
3. 'C' [Wheeler's 6 th ed 400]
below the incisal ridges of mandibular anterior teeth
In normal occlusion when teeth are placed in centric occlusion relation.
1) Mesiobuccal cusp of Occludes in the
upper 1' 1 molar mesiobuccal groove of 7. 'A' [Wheeler's 8 th ed 462, Fig 16-28/ 9th ed 289,
lower molars Fig, 16-28]
In centric occlusion, maxillary teeth contact with the
2) Distobuccal cusp of Occludes with
the
distobuccal groove of namesake teeth and teeth distal to it in opposing arch
upper molars
lower molars while mandibular teeth contact with the namesake
teeth and teeth mesial to it in opposing arch.
3) Mesiolingual cusp of Occludes with the
upper molars central fossa of lower The mesia l cusp ridge of maxillary canine opposes
molars distal cusp ridge of mandibular canine. Distal cusp
4) Distolingual cusp of Occludes in the distal ridge of maxillary canine opposes the mesial slope
upper molars triangular fossa and of mandibular 1st premolar. Whereas the cusp tip of
distal marginal ridge of maxillary canine aligns with the facia l embrasure of
same namesake molars mandibular canine and 1st premolar.
and mesial marginal
ridge of the molars 8. 'B' [Wheeler's 8 1h ed 460, 462/ 9th ed 288-289]
distal to their namesake. Lingual cusps of maxillary posteriors and buccal cusps
of mandibular posteriors are called supporting cusps
The molar relationship (1, 2) is often referred as
( or) functional cusps (or) holding cusps (or) stamp
KEY OF OCCLUSION
cusps.
, ORAL ANATOMY

Buccal cusps of maxillary posteriors and lingual cusps 18. 'D' [Wheelers gth ed 401]
of mandibular posteriors are called nonsupporting
cusps (or) guiding cusps. 19. 'C' [Orbans 10th ed 199/ 13th ed 173]
The ot her names for Periodontal Ligament are
In centric occlusion, opposing contact is expected at Desmodont, Gomphosis, Pericementum, Dental
the buccal slopes of lingual cusps of maxillary posterior periosteum, alveolodental ligament, periodontal
teeth and lingual slopes of buccal cusps of mandibular membrane.
posterior teeth.
20. 'A' [Wheelers Sth ed 116/ 9th ed 78, Fig 4-22)
9. 'A' [Wheeler's 6th ed 407] For explanation refer to synopsis Miscellaneous Point
Cusp Fossa relation No. 43 .

Lingual cusps of Triangular fossa of 21. 'D' [Wheeler' s 9th ed 288-90, 230 Fig 16-29]
premolars mandibular premolars. Lingual occlusal line is formed by supporting cusps.
Buccal cusp tip of Occludes partly with the
lower 1st premolar opposing 1st premolar and 22. 'fl [Wheeler's 9th ed 263]
canine. • Centic occlusion or intercuspal position is defined
Buccal cusp tip of Mesial occlusal fossa of as maximum intercuspation of teeth.
lower 2"d premolar opposing second premolar
• Centric relation is the position of the mandible (or
10. 'A' [Wheeler's 8th ed 462/ 9th ed 289, Fig 16-28] path of opening and closing without translation
of condyles) in which the condyles are in their
11. 'B' [Wheeler's 6th ed 407] uppermost position in t he mandibular fossae and
related anteriorly to the distal slope of the articu lar
12. 'A' [Wheeler's 6th ed 406] eminence.

13. 'C' [Wheeler's gth ed 459 Fig. 16-25/ 9th ed 288, 23. 'D' [Wheeler's 9th ed 2 5 Fig 2-4, 31 Table 2-2]
Fig 16-25] The mandibular permanent second molar forms the
If a permanent 1st molar is lost: distal contact of mandibular permanent fi rst molar and
• There is mesial tilting of second molars. erupts at the age of 11-13 yrs.
• Distal tilting of second premolars.
24. 'D' [Check Explanation of Q.No.8]
• Extrusion of opposing molar.

14. 'A' [Wheeler's 6th ed 398]

15. 'A' [Wheeler's Sthed 459, Fig. 16-24/ 9th ed 288,


Fig 16-24]

16. 'A' [Wheeler's gth ed 469/ 9th ed 294]

17. 'D' [Wheeler's 8 1h ed 453/ 9th ed 284]


The presence of horizontal overlap in the molars
prevents cheek biting.

Excessive vertical overlap of the anterior teeth


may result in tissue impingement and is known as
impinging overbite.
Dental ;lut.,e

5. DENTO-OSSEOUS STRUCTURES AND TMJ


1. Basically TMJ is a: c) Lingual to the maxillary canines
a) Ball and socket joint d) Lingual to the mandibular first molar
b) Hinge and axis joint (AIIMS-02)
c) Synovia l joint d) Diarthrodal joint 9. Which of the following movement(s) is /are
(MAN-98,AIIMS-91) involved in the opening of the mouth?
2. Temporo-mandibular ligament is attached to: a) Only hinge (rotation)
a) Lateral aspect of TMJ b) Only translation
b) Posterior aspect of TMJ c) Hinge fo llowed by translation
c) Mandibular condyle d) Coronoid process d) Translation followed by hirnge
(AIIMS-95) (AIIMS-93,MAN-94)
3. Which of the following muscles moves the condyle 10. Articular disc of TMJ is made up of:
and articular disc anterior and downwards on a) Elastic cartilage b) Fibrocartilage
glenoid fossa: c) Bony processes
a) Lateral pterygoid b) Medial pterygoid d) Fibrous ligament and hyaline cartilage only
c) Temporalis-anterior fi bers (AIPG-95,KAR-02)
d) Temporalis - posterior fibers 11. All of the following muscles are elevators of the
(AIPG-2K) mandible except:
4. Forward movement of mandible is done by: a) Digastric b) Massetor
a) Lateral pterygoid b) Medial pterygoid c) Medial pterygoid d) Temporalis
c) Temporalis - anterior fibers (AIIMS-93)
d) Temporalis - posterior fibers 12. Articular disc of TMJ is:
(AIPG-2K) a) Thickest posteriorly b) Narrow in the centre
5. Position and movement of articular disc of TMJ is c) Strengthened by lateral pterygoid
controlled by fibers of: d) All of the above
a) Lower head of lateral pterygoid muscle (AIIMS-90)
b) Sphenomandibular ligament 13. Which of the following muscle helps in depressing
c) Superficial head of medial pterygoid muscle the mandible:
d) Upper head of lateral pterygoid muscle a) Temporalis b) Massetor
(AIIMS-92,89) c) Lateral pterygoid d) Medial pterygoid
6. Which muscle is responsible for translation of (AIIMS-99)
condyle? 14. The zygomatic bone does not articulate with:
a) Medial pterygoid b) Masseter a) Frontal bone b) Maxillary bone
c) Superior constrictor of pharynx c) Nasal bone d) Temporal bone
d) Lateral pterygoid (AIIMS-91)
(AIIMS-91) 15. The side towards which mandible moves is called
7. Thinnest labial plate is found in the region of: the:
a) Upper central incisors a) Tooth contact side b) Balancing side
b) Lower central incisors c) Working side d) Non - working side
c) Upper canines d) Lower canines (TNPSC-99)
(AP-96,98) 16. Which of the following is not a structural element
8. In which of the following areas is the alveolar of the temporomandibular joint?
process the thinnest: a) Sigmoid notch b) Articular disc
a) Lingual to the maxillary central incisors c) Capsular ligament d) Joint cavit ies
b) Buccal to the mandibular centra l incisors (APPSC- 99)

1) D 2) A 3) A 4) A 5) D 6) D 7) B 8) B 9) C 10) B 11) A 12) D 13) C


14) C 15) C 16) A
, ORAL ANATOMY

17. Articular disc of TMJ receives insertion from which 25. Which of the portions of the intra articular disc is
muscle: the thinnest?
a) Medial pterygoid b) Lateral pterygoid a) Anterior b) Intermediate
c) Massetor d) Tempora lis c) Posterior d) Retrodiscal
(PGI-97) (KAR -2013)
18. On opening the mouth, if the jaw deviates to the
right side, it indicates the paralysis of:
a) Right lateral pterygoid
b) Left lateral pterygoid
c) Right medial pterygoid
d) Left medial pterygoid
(TNPSC- 98)
19. Which of the following movements are performed
by a non - working condyle?
a) Straig htward
b) Down wards forwards and lateral
c) Down wards forwards and medial
d) Down wards Back wards and medial
(AIIMS-01)
20. Depression of mandible is achieved by:
a) Digastric b) Lateral pterygoid
c) Geniohyoid d) All of the above
(AIIMS-90)
21. Retrusion of mandible is achieved by:
a) Lateral pterygoid b) Temporalis
c) Masseter d) None of the above

22. Lateral movement of condyle is caused by:


a) Contralateral Latera l pterygoid muscle
b) Both lateral and medial pterygoid muscle
c) Epsilatera l Lateal pterygoid muscle
d) Bilateral contraction of lateral pterygoid muscle
(AIIMS- 06)
23. The premolar alveoli are what shape in cross
section:
a) Rectangular b) Triangular
c) Kidney shaped d) Oval

24. Which is the predominant factor in the formation


of the alveolar process:
a) Eruption of teeth b) Normal process of growth
c) Lengthening of the condyle
d) Overall growth of the bodies of the maxilla and the
mandible
(AIPG-05)

17) B 18) A 19) C 20) D 21) B 22) B 23) C 24) A 25) B


Dental ;lut.,e

5. DENTO-OSSEOUS STRUCTURES AND TMJ - ANSWERS


1. 'D' [Wheeler's 81h ed 411/ 9th ed 259) 4. 'A' [Wheeler's 5 th ed 429 / 9th ed 270)
TMJ is an example of diarthrosis, and its movements are Mandibular
in a combination of gliding (translation) movements Masseter, temporalis, medial
closing
and hinge (Rotation) movement. pterygoid
(elevators)

2. 'A' [Wheeler's 81h ed 415, 416, Fig. 15-6/ 9th ed Mandibular Latera l pterygoid, suprahyoid
261-262, Fig 15-6) opening muscles (dig astric, mylohyoid and
(depressors) geniohyoid).
THE LIGAMENTS OF TMJ ARE
Retrusion Posterior fibers of temporalis
Capsular
Completely surrounds the condyle Protrusion Lateral and medial pterygoid
liga ment
This is the external (lateral) portion Combined action of elevators and
Lateral
of capsular ligament, which is retruders on working side and
Temporo- movements
attached to the zygomatic process protruders on non-working side.
mandibular
of temporal bone above and lateral
ligament
posterior margin of the neck of 5. 'D' [Wheeler's 81h ed 421/ 9th ed 266)
condyle below.
6. ' D' [Wheeler's 81h ed 421/ 9th ed 266)
Spheno- It has attached above to the
mandibular sphenoid bone and below to lingula
7. 'B' [Wheeler's 8 1h ed 396 Fig. 14-18/ 9th ed 248,
liga ment of the mandible on medial side.
Fig 14-18)
Stylo Extends from the styloid process to
mandibular the posterior border of the ramus of 8. 'B' [Wheeler's 8 1h ed 396 Fig. 14-18/ 9th ed 248,
ligament the mandible just above the angle. Fig 14-18]

Temporomandibular ligament acts as a suspensory 9. 'C' [Wheeler's§_!!! ed 358, 366/ 9th ed 263]
ligament during moderate mouth opening movements The opening movement of the jaws is a combination
called hinge movements, where forward movement of of a hinge movement and a sliding (translation)
the condyle is very slight. movement. In the first phase of opening there is
hinge movement (rotatory) around a transverse axis
Sphenomandibular ligaments act as a suspensory passing through the center of condyles. In the second
ligament during wider opening movements, where the phase the condyle moves downward and forward along
condyle moves forward rapidly. the posterior slope of the articular tubercle (there is
gliding movement and hinge action combined, the disc
3. 'A' [Wheeler's 8 1h ed 421/ 9th ed 266) moves along with the condyles).
Lateral pterygoid originates on the outer surface of
lateral pterygoid plate and greater wing of sphenoid. 10. 'B' [Wheeler's 8 1h ed 411/ 9th ed 262)
It inserts on the anterior surface of neck of condyle Articular disc (or) meniscus is composed of dense
and articular disc. Lateral pterygoid is anatomically fibro us connective tissue and is located between
suited for protraction, depression and contralateral the condylar head and mandibular fossa. The disc
abduction i.e., for example, contraction of right lateral is biconcave in shape and divides the joint cavity
pterygoid causes movement of the mandible towards into upper and lower joint spaces. The central area
left. of the disc is relatively avascular, devoid of nerves
and has limited reparative ability. The disc is thick
Lateral pterygoid assists in translation of condyle anterioposteriorly and thin centrally.
downward, forward and contralaterally during jaw
opening. 11. 'A' [Wheeler's 8 1h ed 430/ 9th ed 270-271]
, ORAL ANATOMY

12. 'D' [Wheeler's 8th ed 417/ 9th ed 262-263] 25. 'B' [Oral anatomy, histology and embryology by
Berkovitz Pg 254/ Wheeler's 9th ed 262, 264, Fig
13. 'C' [Wheeler's Sth ed 430/ 9th ed 270] 15-10]
The inter articular disk or meniscus is a fibrous tissue
14. 'C' [Wheeler's 8th ed 387/ 9th ed 244] and is divided into 3 parts i.e.,
• Thicker anterior section called anterior band
15. 'C' [Wheeler's Sth ed 468/ 9th ed 293]
• Middle thinnest intermediate zone
During lateral movements, the side to which the
• Thickest posterior section called posterior band.
mandible moves is called the working (or) functional
side and the side opposite to which the mandible
moves is called the balancing side.

16. 'A' [Wheeler's S'h ed 393/ 9th ed 245-246, Fig 14-


14]
Sigmoid notch (or) mandibular notch forms the sharp
upper border of the ramus between the condyle and
coronoid process.

17. 'B' [Wheeler's 81h ed 417, 421/ 9th ed 263]


Articular disc of TMJ receives insertion from the upper
head of lateral pterygoid muscle.

18. 'A' [Wheeler's 81 h ed 421/ 9th ed 2 66]

19. 'C' [Wheeler's 9th ed 293]

20. 'D' [Wheeler's 8th ed 429/ 9th ed 270]

21. 'B' [Wheeler's 81 h ed 430/ 9th ed 271]

22. 'B' [Wheeler's 81h ed 431/ 9th ed 271]

23. 'C' [Wheeler's S'h ed 386/ 9th ed 243-244]


Teeth Shape of Alveolus
Incisors Triangular
Canines Ova l
Premolars Kidney shape
Molars Three distinct alveoli

24. 'A' [Wheeler's 8 1h ed 384/ 9th ed 243]


Dental ;lut.,e

6. PHYSIOLOGICAL FORM OF TOOTH AND PERIODONTIUM


1. The mesial contact area of the permanent canine is 7. The widest incisal embras ure is normally found
at the: between which of the following permanent
a) Middle third maxillary teeth:
b) Junction of incisal and medial thirds a) Central incisors
c) Junction of the middle & cervical thirds b) Central and lateral incisors
d) None of the above c) Lateral incisor and canine
(MAN-94) d) First and z nd premolar
2. Largest embrasure lies between: (AIPG-02)
a) Maxillary central incisors 8. Distal contact of upper first premolar is:
b) Maxillary central and Latera l incisor a) Lower than mesial contact area
c) Maxillary canine and first premolar b) Little cervical to junction of occlusal and middle
d) Maxillary lateral incisor and canine third of crown
(AIPG-94) c) Higher than mesial contact area
3. The largest embrasure in posterior teeth is the: d) A and B
a) Buccal b) Distal (AIIMS-02)
c) Li ngual d) Occlusal 9. An imaginary occlusal curve that contacts the
(MAN-95, TNPSC-99) buccal and lingual cusp tips of mandibular buccal
4. The tooth that is commonly seen in an abn ormal teeth is called the
relation and contact with its adjacent teeth of the a) Catenary curve b) Curve of spee
same arch is: c) Monson curve d) Wilson curve
a) Mandibular first premolar (AIIMS-01)
b) Mandibular lateral incisor 10. Mesial contact area of the permanent canine is at the:
c) Maxillary central incisor a) Middle third
d) Maxillary lateral incisor b) J unction of the incisa l and middle thirds
(MAN-94) c) J unction of the middle anal cervical thirds
5. Divergence from contact area in proximal surfaces d) None of the above
causes embrasures: (AIIMS- 92)
a) Facially b) Lingually 11. Generally, contact areas between posterior teeth
c) Cervically are located in which directions from the center of
d) Facially, lingually, cervically & occlusally the proximal surface?
(AIIMS-97) a) Facially and occlusally
6. The contact area on the distal surface of a maxillary b) Facially and gingivally
first premolar should be placed in: c) Lingually and gingivally
a) Middle 3 rd of proximal surface with lingual embrasure d) Lingually and occlusally
greater than the facial embrasure (AIPG-98)
b) Occlusal 3rd of proximal surface with the lingual 12. Which of the following do not contribute to arch
embrasure greater than facia l embrasure stability?
c) Occlusal 3rd of proximal surface with facial a) Cusps b) Rootforms
embrasure greater than lingual embrasure c) Embrasures d) Periodontal fibers
d) Middle 3 rd with facial embrasure greater than the (APPSC-99)
lingual embrasure 13. The contact area on the distal side of a maxillary
(AIPG- 01,02) first premolar is located:
a) Middle of the middle third with buccal embrasure
wider than lingual

1) B 2) C 3) C 4) D 5) D 6) A 7) C 8) D 9) D 10) B 11) A 12) C 13) C


, ORAL ANATOMY

b) Middle of the middle third with lingual embrasure c) Mandibular first molar
wider than buccal d) Maxillary second premolar
c) Litter cervical to the junction of occlusal and middle
third of the crown with wide occlusal em bra sure 21. Posterior embrasures are generally larger on the
d) Cervical third with buccal embrasure wider than Lingual than on the facial, with the contact area
lingual embrasure within the facial moiety, except between maxillary:
(AIPG- 99) a) Premolars b) 1st and 2 nd molar
14. Contact areas of anterior teeth which are placed in c) l't premolar and canine
centre of middle third: d) First molar and 2 nd premolar
a) Mesial contacts of maxillary laterals and canine
b) Distal contacts of maxillary centrals and laterals 22. Protective crests of contour on buccal and lingual
c) Distal contacts of maxillary laterals and canine surfaces are found:
d) Mesia l contacts of maxillary fi rst and second a) In the middle third of t he lingual
premolar b) Nearest the occlusal third of the lingual
c) In the cervical third of the buccal
15. Which anterior teeth have both mesial and distal d) Both A and C
contact areas at the incisal third:
a) Mandibular centrals b) Mandibular laterals 23. The interdental papilla is located in the:
c) Both A and B d) None of the above a) Facial embrasure b) Lingual embrasure
c) Cervical embrasure d) Occlusal embrasure
16. Which four mandibular teeth are so aligned that
when viewed from the occlusal, a straight line may 24. All are true about calciotraumatic line, except:
be drawn bisecting the contact areas: a) Due to caries
a) 5,6,7,8 b) 4,5,6,7 b) Due to death of odontoblastic layer
c) 3,4,5,6, d) 2,3,4,5 c) It remains after caries is rremoved
d) Because of odontoblastic migration from cell rich
17. Which teeth show less curvature on the crown above zone
the cervical line than any other teeth: (AIPG-06)
a) Maxillary anterior b) Mandibular canine 25. The primary centre of formation of each Lobe is
c) Mandibular anteriors d) Maxillary canine present in:
a) Apex of the root b) Tip of the cusp
18. Which of the following are the functions of contact c) Centre of the crown d) Centre of the tooth
areas?
a) Distribution of occlusal stresses 26. The faciolingual angu latio n of the upper central
b) Protection of periodontium incisor is approximately:
c) Stabilization of dental arches a) 20° b) 28°
d) All of the above c) 33° d) 38°

19. The distance from the CEJ to the alveolar crest 27. In primates; a tooth with single conical cusp and
normally is: single root is seen in:
a) 1 - 1.5 mm b) 2 mm a) Haplodont b) Heterodont
c) 2 - 2.5 mm d) 3 mm c) Acrodont d) Thecodont
(AIPG- 07)
20. Buccal surface of posterior teeth are wider than 28. The biting forces at molar region during mastication
lingual surface mesiodistally except in: is:
a) Maxillary first molar a) 20 to 55 pounds b) 30 to 75 pounds
b) Mandibular second premolar

14) C 15) C 16) A 17) C 18) D 19) A 20) A 21) D 22) D 23) C 24) B 25) B 26) B
27) A 28) D
c) 50 to 75 pounds d) 90 to 200 pounds b) Contact point and the area above and below the
(KCET-07) contact point between the adjacent teeth
29. The curvature of the cervical line of most teeth will c) Contact point between the adjacent posterior teeth
be approximately d) Embrasures incisal and gingival to the contact
a) 2 mm less distally than mesially (KERALA-2015)
b) 1 mm less distally than mesially
c) 2 mm less lingua lly than buccally
d) 1 mm less lingually than buccally
(BHU-2012)
30. All embrasure spaces are reflections of the
a) Form of the teeth involved
b) Size of the teeth involved
c) Eruption sequence involved
d) None of the above
(BHU-2012)
31. The term "contact point" which is often used to
designate the contact of teeth i the same arch is a
a) Synonym b) Antonym
c) Misnomer d) Laevonomer
(BHU-2012)
32. The embrasure areas in the following areas are
continuous
a) Labial and Lingual b) Lingual and Occlusal
c) Incisal and Occlusal d) Labial and Occlusal
(COMEDK -2013)
3 3. In ideal occlusion, facial cusps of maxillary
posterior teeth oppose
a) Grooves and embrasures
b) Grooves only
c) Marginal ridges and embrasures
d) Marginal ridges only
(AP-14)
34. Cusp fossa and cusp embrasure relationship is given
by/ Cusp fossa and cusp embrasure relationship is
popularised by
a) Thomas b) Huffman and Regenos
c) Stallard d) Stuart
(PGI JUNE-2014)
3 5. Which of the following factor does not contribute
for maintaining occlusal stability?
a) Embrasures b) Tooth structure
c) Loss of teeth d) Periodontal disease
(MHCET-15)
36. The term 'connectors' or the 'interdental contact
area' refers to the
a) Contact point between the adjacent anterior teeth

29} B 30} A 31} C 32} A 33} A 34} B 35} A 36} B


, ORAL ANATOMY

6. PHYSIOLOGIC FORM OF TOOTH AND PERIODONTIUM - ANSWERS


1. 'B' [Wheeler's 81h ed 132/ 9th ed 88] Also their lingual surfaces are converged resulting in
Contact area of upper teeth: wide lingual embrasures.

~en:ral
incisor
< M- lncisal third
D - Junction of incisal and
middle third
But in case of maxillary 1st molar, the lingual surface
is wider than facial surface, resulting in a small lingual
embrasure.
M - Junction of incisal and
Lateral
incisor < middle third
D - Middle third
4. 'D' [Wheeler's 5th ed 161/ 9th ed 106]
Maxillary lateral incisors vary in form most commonly.
This makes the existence of an abnormal relation and

Canines < M - Junction of incisal and


middle third
D - Middle third
5.
contact with its adjacent teeth.

'D' [Wheeler's 8th ed 130/ 9th ed 86]


The curved proximal surfaces of the contacting teeth
roll away from the contact area creating embrasures
M - Cervical to the junction of
First &
second
premolars
< occlusal and middle third
D - Cervical to junction of
occlusal and middle third
6.
occlusally, cervically, lingually and labially (or)
buccally.

'K [Wheeler's 81h ed 132, Fig.5-15/ 9th ed 87, 88]

First molars < M - Cervical to junction of


occlusal and middle third
D - Middle thirds
7.

8.
'C' [Wheeler's 8th ed 132, Fig. 5-15/ 9th ed 87, 88]

' D' [Wheeler's 8th ed 132/ 9th ed 88]

Second &
third molars
< M - Middle third
D - Middle thirds
9. ' D' [Wheeler's §_th_ed 80, 81/ 9th ed 78]
Curve of Spee - It refers to the antero-posterior
curvature of the occlusal surfaces beginning at the
2. 'C' [Wheeler's 8th ed 132/ 9th ed 88]
tip of lower cuspid and following the cusp tips of the
When two teeth contact in the same arch, their
bicuspids and molars continuing as an arc through the
curvatures adjacent to contact areas form spillway condyle.
spaces called " Embrasures".
Curve of Wilson - This is a curve that contacts the
As the distal slope of canine is long and also the
buccal and lingual cusp tips of the mandibular buccal
first premolar has a long mesial slope, this makes the
teeth . It results for inward inclination of the lower
contact at a high level on the crown, opening a wide
posterior teeth. As the teeth are aligned parallel to
occlusal embrasure.
the direction of medial pterygoid there is optimum
resista nee to masticatory forces.
Smallest embrasure lies between centra l incisors.
Widest incisal embrasure lies between lateral incisor
Linear curve, which runs in antero-posterior
and canine.
direction (AIIMS-2012)
a) Curve ofvon spee b) Curve of monsoon
3. 'C' [Wheeler's 8th ed 135/ 9th ed 90]
c) Curve of Wilson d) Bonwill
From the occlusal aspect, anterior teeth will have their
contacts centered labiolingua lly.
10. ' B' [Wheeler's 8th ed 132/ 9th ed 88]
In case of posterior teeth, the contact areas are
11. 'A' [Wheeler's 81h ed 135/ 9th ed 89-90]
present slightly buccal to the center buccolingualy.
Dental ;lut.,e

12. 'C' [Wheeler's 8th ed 130/ 9th ed 86] by expansion. This produces compression, distortion
Embrasures serve as and disruption of the abjacent dentina l tubules.

• Spillway for the escape of food during mastication, Teeth with deep penetrating carious lesions can be
a physiologic form which reduces forces. treated only by partial removal of the deep carious
• It prevents food from being forced through the lesion and insertion of calcium hydroxide or mineral
contact area. trioxide aggregate containing dressing materials for
a period of a few weeks or months. During this period,
13. 'C' [Wheeler's 8th ed 132, Fig.5-15/ 9th ed 87, 88] the odontoblasts form new reparative dentin along
the pulpal surface underlying the carious lesion. The
14. 'C' [Wheeler's 8th ed 132/ 9th ed 88] cavity can be reopened and t he remaining carious
lesion removed without endangering the pulp. This
15. 'C' [Wheeler's 8'h ed 132, 133/ 9th ed 88] treatment is known as indirect pulp capping.

16. 'A' (Wheeler's Sth ed 134, Fig.5-17/ 9th ed 89, 91] On removal of the caries, the mesenchymal cells of
the cell rich zone differentiate into odontoblasts to
17. 'C' [Wheeler's 8th ed 140 Fig.5-23/ 9th ed 94] replace those that have necrosed. These newly formed
odontoblasts can produce well-organized dentin or an
18. 'D' [Wheeler's 8th ed 121, 123/ 9th ed 82] amorphous, poorly calcified, permeable dentin. The
demarcation zone between secondary and reparative
19. 'A' [Wheeler's Sth ed 127/ 9th ed 85] dentin is called the 'calciotraumatic line'.

20. 'A' (Wheeler's 8th ed 135/ 9th ed 178] 25. 'B' (Wheelers 8 1h ed 105/ 9th ed 71]

21. 'D' [Wheeler's Sth ed 135, 136/ 9th ed 91] 26. ' B' [Wheelers 8th ed 456/ 9th ed 286, Box 16-1]

22. 'D' [Wheeler's 61h ed 95, 111/ 9th ed 91-93 Fig 27. 'A' (Wheelers 8 1h ed 10/ 9th ed 69]
5-18]
Crests of contour (or) curvature of the labial and 28. 'D' [Phillips Science of Dental Materials 11th ed 93]
buccal surfaces are at the cervical thirds and lingually Biting force:
the Crests of contom will be at the middle thirds of • on molar region is 90 to 200 pounds (400-890 N)
the crown.
• on the premolars is 50 to 100 pounds (222-445 N)
23. 'C' [Wheelers 9th ed 76] • on the cuspids is 30 to 75 pounds(133-374 N) and
The interdental pailla occupies the lingual embrasure • on the incisors is 20 to 55 pounds (89 to 111 N).
just below the contact point and above the crest of
alveolar bone. 29. 'B' [Wheeler's 9th ed 9 5]

24. 'B' [Text book of Oral Histology by Satish Chandra 30. 'A' (Wheeler's 9th ed 88]
and Shaleen Chandra 1st ed 94/ Shafer's 7th ed 521]
During adva need stages of caries, walls of the individual 31. 'C' [Wheeler's 9th ed 83]
tubules undergo further decalcification leading to Contact area is a better term than contact point.
their confluence. Along with this, there is an increase
in the diameter of the dentinal tubules. This occurs 32. 'A' (Wheelers 9th ed 86]
due to packing of the tubules with microorganisms. The Spillway spaces (or) embrasures that widen
Small 'liquefaction foci' are formed by the breakdown out from the area of contact labially or buccally and
of a few dentinal tubules. This 'focus' is an area of lingually are ca lled labial (or buccal) and lingual
ovoid destruction, parallel to the course of the tubules embrasures. These embrasures are contin uous with the
and filled with necrotic debris that tends to increase interproximal spaces between the teeth. The incisal
, ORAL ANATOMY

or occlusal embrasures and the labial (or buccal) and


lingual embrasures are continuous.

33. 'A' [Wheeler's 9th ed 279, 291 Fig 16.31A]


In Class I molar relationship, the mesiobuccal cusp of
the permanent maxillary first molar is located within
the mesiobuccal developmental groove of permanent
mandibular first molar. The triangular ridges of maxillary
buccal cusps of premolars and molars occlude with the
buccal embrasure of mandibular premolars and molars.

34. 'B' [Check Explanation Below]


Sheppard Enter bolus exit balance concept
Devan Neutrocentric concept
Stallard and
organic concept of occlusion
stuard
Huffman and Cusp-fossa and cusp-embrasure
Regenos relationship

35. 'A' [Wheelers 9th ed 299]


The stability of the occlusion and the maintenance of
tooth position are dependent on all of the forces that
act on the teeth. Occlusal forces, eruptive forces, lip
and cheek pressure, periodontal support and tongue
pressure are all involved. As long as all these forces
are balanced, the teeth and the occlusion will remain
stable. The loss of teeth, tooth structure or occlusal
supporting cusps or a decrease in their support from
periodontal disease or trauma is a factor in maintaining
occlusal stability.

36. 'B' [Wheelers 9th ed 83]


Dental ;lut.,e

ORAL ANATOMY - SYNOPSIS


INTRODUCTION

1. Types of dentitions
Monophyodont Presence of only one set of dentition for entire life.
Diphyodont • Presence of two sets of dentition
• Human beings are diphyodonts (COMEDK-08)
Polyphyodont Presence of more than t wo sets of dentition.
Homodont All the teeth have same shape without distinction such as incisors, canines, premolars and
molars.
Heterodont Presence of different groups of teeth.
Bunodont • Primitive type of teeth seen in primates like cats, dogs, etc.
• Contains simple conical cusps.
Haplodent • Seen in reptiles like crocodiles.
• Simplest cone fo rm of teeth with single root. (AIIMS-07)
• On ly si mple hinge movements of jaws are seen .
Triconodont • Seen in early mamma ls.
• Three cusps are arranged in line with the largest cusp in the center.
Tritubercula r stage • Three cusps are arranged in triangle form .
Quadritubercular • cusp is formed and an occlusal contact relationship between the upper and lower
4 th

stage jaws is established.

2. Except molars, all the permanent teeth (incisors, canines • Thecodont: The tooth is attached by periodontal
and premolars) are known as succedaneous teeth because membrane and it does not undergone significant
they take the place of their primary predecessors. changes.

3. Permanent molars are not succedaneous t eeth, as 5. The first primary tooth to erupt into oral cavity is
they develop from the distal extension of the dental mandibular central incisors. The first permanent tooth
lamina. Successiona l lamina, which is present on the to erupt into oral cavity is mandibular 1'1 molar.
lingual side of primary dental lamina, is absent for
permanent molars. 6. Sequence of eruption of primary teeth is ABDCE
A 4 - 6 months
4. Tooth attachment can be of two types B 6 - 8 months
a) Ankylosis or direct attachment of tooth to bone. D 12 - 16 months
C 16 - 20 months
• Pleurodont: Tooth attached to inner margins of
E 20 - 30 months
bone.
• Acrodont: Tooth is attached to crest of bone. 7. The sequence of eruption of permanent teeth is
• In mandible - 6-1-2-3-4-5-7-8
b) Attachment of tooth to bone socket.
• In maxilla - 6-1-2-4-3-5-7-8 (or)
• Gomphosis: The tooth is attached to bony
6-1-2-4-5-3-7-8.
socket through periodontal membrane, which
allows on ly limited movement.
, ORAL ANATOMY SYNOPSIS

8. The first succedaneous tooth to erupt into oral cavity 16. The universal system of notation
is mandibular central incisor. a) For primary dentition is:

9. The fist non - succedaneous tooth to erupt into oral (Right) ABCDE IFGHIJ (Left)
cavity is mandibular 1 st molar. TSRQP ONMLK

10 The last succedaneous tooth to erupt into oral cavity


b) For permanent dentition is:
is maxillary canine.
1 2 3 4 5 6 7 8 9 10 11 12 13 14 15 16
11. Mamelons represent the number of lobes from which
32 31 30 29 28 27 26 25 24 23 22 21 20 19 18 17
a particular tooth is developed. Usually mamelons
abrade due to contact with opposing teeth, except in
17. The Zsigmondy/palmar notation
malocclusion conditions (mostly in class II division 1),
where they escape incisal wear. a) for primary dentition is

E DC B A A B C D E
No. of E DC B A A B C D E
Teeth
lobes
Primary incisor 1 Thus for a single tooth such as maxillary right central
incisor the designation is AJ. For the mandibular Left
Primary 2"d molars 5
central incisor, the notation is given as!A.
• 3 - cusped lower second premolar
• Maxillary & Mandibular permanent 5
b) for the permanent dentition is
1st molars

• Permanent incisors, canines, 8 7 6 5 4 3 2 1 1 2 3 4 5 6 7 8


premolars 4 8 7 6 5 4 3 2 1 1 2 3 4 5 6 7 8
• ALL other molars
For example, the right maxillary p t molar is designated
12. Initial calcification of primary teeth starts 4 months as 16, and the Left mandibular central incisor is 1J
in utero for primary central incisors. The last primary
tooth to show initial calcification is second molar (6 18. The Federation Dentaire Internationale (FDI) system
months in utero). was accepted by W.H.O.

13. Initial calcification of permanent teeth starts at birth The FDI system of tooth notation for primary teeth is
for p t molars. The Last permanent tooth to show initial as follows:
calcification is 3 rd molar, at the age of 8 years.
55 54 53 52 51 61 62 63 64 65
14. By the age of 2 - 2 1/2 years of age, all the primary 85 84 83 82 81 71 72 73 74 75
teeth are erupted into oral cavity. Root formation of
primary teeth is completed by 3-4 years of age i.e., • Numeral 5 indicates maxillary right.
1 - 1 112 years after their eruption. • Numeral 6 indicates maxillary Left.
• Numeral 7 indicates mandibular Left.
15. By the age of 12-13 years all the permanent teeth
are erupted into the oral cavity. Root formation of • Numeral 8 indicates mandibular right.
permanent teeth is completed by 14-16 years of age
i.e., 2-3 years after their eruption. For example, the number 51 refers to the maxillary
right primary central incisor.
Dental ;lut.,e

The FOi system of tooth notation for permanent 3 6


teeth is as follows:
6 11
4 8
18 17 16 15 14 13 12 11 21 22 23 24 25 26 27 28
48 47 46 45 44 43 42 41 31 32 33 34 35 36 37 38 Vary
8 14
Thus in FOi system the first digit indicates the
quadrant for permanent dentition (1-4) and for the
primary dentition (5 to 8). DECIDUOUS TEETH

The second digit indicates the tooth within the 1. First primary toot bud is initiated at 6 th week of intra
quadrant: 1 to 8 for the permanent teeth and 1 to 5 uterine life.
for the primary teeth.
2. Calcification of primary t eet h starts at 4th month and
19. A line angle is formed by the junction of two surfaces completed by 6th month of intraut erine life.
and derives its name from the combination of the two
surfaces that join. 3. The first tooth to erupt is mandibular central incisor at
the age of 6 mont hs.
The line angles of anterior teeth (6) are
Mesio labial Oisto labial 4. The order of eruption in both maxillary and mandibular
Mesio lingual Oisto lingual arches is A B O C E.
Mesio incisal Oisto incisal
5. By around 2 112 years all the deciduous teeth erupts
Note: The term mesiodistal line ang le does not exist. into the oral cavity.

The line angles of posterior teeth (8) are 6. Root completion occurs 1 - 1 112 years after eruption
Mesiolabial Oistola bial of the crowns.
Mesioli ng ual Oistoli ngual
Mesio occlusal Oisto occlusal 7. The cusp of deciduous canine is much longer and
sharper than permanent canine. The mesial slope is
Labio occlusal Linguo occlusal
longer than distal slope whereas the opposite is true
for permanent canine.
20. A point angle is formed by junction of three surfaces.
8. The crowns of deciduous arnterior teeth are wider
The point angles of anterior teeth ( 4) are
mesiodistally in comparison with their crown length
Mesiolabioi ncisal Mesiolinguoi ncisa l than are the permanent teeth.
Oist olabioincisal Oistoli nguoincisal
9. Their roots are longer, slender, more divergent and
The point angles of posterior teeth ( 4) are flared to accommodate the developing permanent
Mesiobuccoocclusal Mesioli nguoocclusal tooth crowns.
Oistobuccoocclusal Oistoli nguoocclusal
10. The pulp chambers are larger and pulp horns are highly
placed.

11. The enamel and dentin thickness is limited making the


Point Line
pulp chambers shallow.
angles angles
4 8 12. The pulp canals are wider and 11:he apical portion of the
6 11 canal is much less constricted than that of permanent
teeth.
, ORAL ANATOMY SYNOPSIS

13. Cervica l ridge is well developed and is called as Molar • The presence of development grooves in
tubercle of Zuckercandl in upper first molar. maxillary laterals and mandibular centrals creates
accessibility and worserns the prognosis of the
14. The smallest toot lh is lower central incisor and the tooth during periodontal treatment.
smallest molar is upper first molar. • Dense in dente is most common in upper laterals.
• Latera l incisors are the teeth, which COMMONLY
15. Primary maxillary 1' 1 molar resembles permanent have abnormal relation to adjacent teeth.
premolars. Primary maxillary 2 nd molar rese mbles
• The apical third of root exhibits curvature distally
permanent maxillary p t molar. Primary mandibular
and palatally. So infection from lat erals spreads
2 nd molar resembles permanent mandibular 1st molar.
palatally result ing in PALATAL ABSCESS.
Primary mandibulal' 1st molar does not resemble any of
the permanent teeth.
C) Maxillary Canines:
• Also known as cuspids or CORNER TEETH OR EYE
16. The long and sharp rnesiolingual cusp is an
TEETH (as t hey develop near orbital floor) .
outstanding feature of deciduous first mandibular
molar. It is the largest of all cusps in lower 1'1 molar. • Most common ly impacted teeth next to third
molars.
17. The fifth cusp present in deciduous upper second molar • The root is longest and strongest of all teeth.
is called the Cusp of Carabelli. • Maxillary and mandibular canines are the longest
teeth in oral activity
The fifth cusp in lower second molar is the buccal cusp (Maxillary canine - Crown 10mm, root 17mm
(distal cusp is the 5th cusp in permanent lower first Mandibular canine - Crown 11, root 16 mm)
molar).
• These teeth exhibit longest path of eruption.
18. The enamel rods of deciduous teeth slope occlusally • These teeth have maximum mesiodist al angulation
at the cervical region, whereas they scope gingivally (17°). Mesial half resembles incisors and distal
(cervically) in the permanent teeth. half resembles premolars.
• In canines and premolars (except maxillary first
19. Primary molar in which anatomy of pulp cont raindicates premolar) mesial slope is shorter than distal slope.
an MOD preparation is the mandibular first molar. • Least common tooth to sl,ow bifurcated roots.
• It is the tooth, which is blocked out commonly due
PERMANENT DENTITION to lack of space.

A) Maxillary Central Incisor: D) Mandibular central incisors:


• Mesio incisal angle is sharp while the disto incisal • Smallest of all the teeth.
angle is rounded. • Both the mesioincisal and distoincisal line angles
• Mesial curvature (3.5mm) is the most pronounced a re sharp.
compared to any other tooth. • The incisal ridge is straight and is approximately at
right angles to the long axis of the tooth.
• The incisal edges of newly erupted permanent
incisors show mamelons. • These features make it difficult to distinguish
between right and left teeth after extraction.
B) Maxillary Lateral Incisors: • The lingual surface is smooth with least prominence
of lingual fossa.
• Lingual fossa is deepest with lingual pits.
• Exhibits few developmental lines and grooves in
• MOST OFTEN shows variation in size, shape and
the entire dentition next to mandibular lat eral
form next to th ird molars (AIPG-05) .
incisors.
• Most common missing tooth next to third molars
• The incisal curvature of the cervical line is marked
(In deciduous teeth maxillary lateral incisor is the
extending approximately one-third the length of the
most common missing tooth).
crown.
Dental ;lut.,e

• The labiolingual diameter of the crown is • The crown of mandibular 1st premolar has lingual
always greater than mesiodistal diameter. This tilt of 30° to the long axis of root. To com pensate
reinforcement makes them to bear the greatest for the tilt, the enamel is penetrated at the lingual
masticatory stresses occurring in labio lingual incline of facial cusp and directed along the long
direction. axis of the root.
• This tooth usually has one root canal, but two • Lateral perforations are most often seen in
canals may be found quite frequently. mandibular premolars.
• Bifurcations and trifurcations are most common.
E) Mandibular lateral Incisor: They present a challenge to endodontist during
• Smallest tooth next to mandibular incisors biomechanical preparation. So known as "enjgma
• The incisal edge is not at right angles to a line to endodonost'~
bisecting the crown and root labio lingually. • The surface of the crown shows a characteristic
• The incisal edge follows the curvature of the dental mesfolingual developmental groove.
arch, giving the crown the appearance of being • Lower premolars shows maximum incidence of dens
twisted slightly on its root base. evaginatus.

F) Mandibular Canine: J) Mandibular 2M Premolar:


• Longest crown. • Shows greatest variation in eruption timing.
• Among anterior teeth bifurcated roots and root • 2-cusp type appears round and 3-cusp type (buccal
canals are most common in mandibular canine. and 2 lingual) appears square with 'Y' shaped
occlusal pattern.
G) Maxillary First Premolar: • In 3-cusp type the buccal cusp is largest and
• It has two cusps and two roots. Lingual cusp is distoli ngual cusp is the smallest.
shorter and sharper than buccal cusp and its cusp • It is the tooth that shows greatest variation in
tip is mesial to buccal cusp. occlusal form next to maxillary third molar.
• Largest of all the premolars.
• Shows a characteristic well defined K) Maxillary t !! Molar:
Mesia[ Marginal Developmental Groove in the • Largest permanent tooth.
enamel of mesial marginal ridge. • Also known as corner stones of dentition
• Least common to show furcation involvement. The • First permanent tooth to calcify
prognosis will be poor if at all furcation involvement
• Shows greatest buccolingual diameter of all the
occurs.
teeth.
• Root concavities are more marked in maxi llary 1'1
• Four cusps - Mesiobuccal, distobuccal, mesio lingual
premolars, mesiobuccal root of maxillary 1st molar
and disto lingual. Mesiolingual cusp is the largest
and both roots of mandibular 1'1 molars and the followed by mesiobuccal, distolingua l, distobuccal
mandibular incisors. and fifth cusps.

H) Maxillary 2M premolar:
• 60% of population may have a 5 th cusp called cusp
of carabelli on mesial half of mesiolingual cusp.
• It has 2 cusps and single root. All the angles will
Cusp of carabelli is also seen in maxillary deciduous
be less acute and hence ovoid appea ranee. second molar.
• The cusps of 2nd premolar are shorter than 1'1
• The mesiolingual cusp is the largest cusp in the
premolar, with the bucca l and lingual cusps more
dentition and is also the most primitive cusp.
nearly the same length.
• Shows 3 roots - Mesiobuccal, distobuccal and
• No mesial marginal developmental groove is seen. palatal.
• Palatal root is the longest root among the posterior
I) Mandibular t n Premolar:
teeth.
• Smallest of all tile premolars.
, ORAL ANATOMY SYNOPSIS

• Palatal root is the largest root, to show facial and N) Mandibular First Molar:
lingual concavities. • First tooth to erupt into oral cavity.
• Distobuccal is the smallest root and is COMMON LY • It shows greatest mesiodistal diameter of all the
RESECTED IN FLJIRCATION INVO LVEM ENT. teeth.
• The roots are in close proximity to maxillary sinus. • It is the most common tooth lost due to caries.
• The distal bifurcation is located 5mm above the • It is the on ly permanent that stays with deciduous
cervical line. dentition for longer periods.
• The bifurcation on the buccal side is located 4mm • It shows 5 cusps - mesiob uccal, distobuccal, distal,
above the cervical line. mesiolingual and distolingual.
• The bifurcation on the mesial side is located 3mm • Mesiobuccal is the largest cusp followed by
above the cervical line. mesiolingual, distolingual, distobuccal and distal
• In an ideal occlusion upper 1st molar shows buccal cusps.
inclination whi le the second and third molars are • The mesiolingual cusp joins distobuccal cusp
inclined disto buccally. along the floor of the central fossa. This is called
• Occlusal view dryopithecus pattern.
- Outline is rhomboidal. • Transverse ridge is seen formed by union of buccal
- Mesiobuccal and distolingual angles are acute. and lingual triangular ridge.
- Mesiolingual and distobuccal angles are obtuse. • It is the commonly involved tooth with furcation
involvement.
- Oblique ridge runs between mesiolingual cusp
to distobuccal cusp. • It is the most amenable tooth for hemisection or
bicuspidasation.
- The four major cusps are separated by H-shape
occlusal groove. • The bifurcation is located 3mm above the cervical
line.
- Early trigon or primary cusps are represented by
mesiobuccal, mesiolingual and distobuccal • Perforations are most common in the lingual wall
- The distolingual cusp becomes progressively during access cavity preparation.
smaller in second and third molars. • Highest incidence of taurodontism and condensing
osteitis.
L) Maxillary Second Molar:
0) Mandibular Second Molar:
• Normally shows 3 cuspal occlusal anatomy (trigone
cusps) • Posterior tooth with maximum facio lingua l
inclination (20°) .
• Smaller than first molar
• Posterior tooth with maximum mesiodistal
• Cusp of carabelli is absent.
inclination.
• Rhomboidal form of occlusal surface is more
• It has 4 cusps - mesiobuccal, distobuccal,
pronounced.
mesiolingual and distolingual. Distal cusp is
Maxillary 3!!! Molar: absent.
M)
• Most commonly impacted tooth.
P) Mandibular Third Molar:
• It is the tooth that shows greatest variation in
• It greatly resembles mandibular 2"d molar.
occlusal form (heart shaped occlusal form)
• It most often appears as a developmental anomaly • If the 3,d molar is congenitally absent from one
varying considerably in size, contour and relative side of mandible or maxilla, it will most likely be
position to the other teeth. absent from the other side.
• The roots are shorter and commonly fused. • The distal inclination of roots is greater.
• Maxillary and mandibular 3rd molars are the teeth • Their partia l eruption may result in periodontal
that commonly show largest variations in root defects and resorption of distal root surfaces.
morphology.
Dental ;lut.,e

MISCELLANEOUS

FEATURES MAXILLARY 1ST MOLAR MANDIBULAR 1ST MOLAR


SIZE OF CUSPS ML (Largest) - MB - DB - DL - Cusp of carabelli MB (Largest) - ML - DL - DB - Distal
3 roots, Mesiobuccal, distobuccal and palatal. 2 roots, Mesial and distal.
NO. OF ROOTS Distobuccal is the smallest and pa latal is the
largest of the three roots.
3 root canals. 3 root canals.
• Mesiobuccal, distobuccal, palatal. • Mesiobuccal, Mesiolingual and distal.
• Mesiobuccal is the narrowest of all canals and • About 30% of mandibular molars
ROOT CANALS it shows greatest distal curvature because of contain a 4 th canal in the distal root.
which it is difficult for instrumentation.
• About 35% of cases shows a 4th cana l in the
mesiobuccal root in a palatal direction.
• The orifice of mesiobuccal canal is gained • The mesiobuccal orifice is present
access from disto palatal direction. under the mesiobuccal cusp and it
• The distobuccal root canal is gained access can be located in mesiobucco-apical
from mesiolingual direction. inclination into a point angle formed
by mesial wall, buccal wall and pulpal
• The palatal root is gained access from buccal
floor of pulp chamber.
direction.
• The mesiolingual orifice 1s present
below the central groove. It is located
ACCESS FOR
in line with the mesiobuccal orifice
ROOT CANALS
and mesial to the dista l orifice. It is
explored from a distobuccal direction.
• The distal orifice has an elliptical shape
and is usually present in the center
of tooth buccolingualy. It is explored
from a mesial direction. If the distal
orifice is present buccally or lingually,
it indicates the presence of a 4 th canal.
Usually trapezoidal with round corners,
SHAPE OF
Triangular but may change into rectangular if a
ACCESS CAVITY
second distal canal is present.
• 4 pulp horns, MB, ML, DB, DL. There is no pulp • 5 pulp horns - MB, ML, DB, DL, distal.
horn for 5 th cusp or cusp of carabelli. • The pulp horn heights from highest to
• Mesiofacial pulp horn is most commonly lowest are MB, ML, DB, DL, distal.
PULP HORNS
involved in Class II cavity preparation. • The pulp horn, which is commonly
exposed during class II cavity
preparation, is mesiolingual horn.
, ORAL ANATOMY SYNOPSIS

• Mesial bifurcation is 3mm above the cervical • Buccal bifurcation is 3mm above the
line. cervical line.

FURCATIONS
• Buccal bifurcation is 4mm above the cervical • Lingual bifurcation lS 4mm above the
line. cervical line.
• Distal bifurcation is 5mm above the cervical
line.
• 2 major fossa - central and distal. • 1 major fossa - central fossa.
• 2 minor fossa - mesial and distal triangular • 2 minor fossa - mesial and distal
OCCLUSAL fossa . triangular fossa.
ANATOMY • 5 developmental grooves. • 4 developmenta l grooves
• 4 cusp ridges. • 5 cusp ridges.
• 1 oblique ridge.

LINE ANGLES
• MB and DL line angles are acute in nature.
• ML and DB line angles are obtuse in nature.
• It is formed by mesiolingual and distobuccal
OBLIQUE cusps.
RIDGE • Oblique ridge is reduced to the level of marginal
ridges at the center of occlusal surface.

3. The premolar, which exhibits a three-cusp pattern, is In comparison with the permanent mandibular
mandibular 2"d premolar. Maxillary 2nd and 3rd molars canine, the maxillary canine in the same mouth is
also exhibit three-cusp pattern. Ans. is wider mesiodistally (AP-2013)

4. The deciduous molar, which contains a prominent 7. RIGHT and LEFT MAXILLARY CANINES are distinguished
transverse ridge 0 11 its occlusal table, is mandibular by -
first molar.
i) Presence of more convexity on distal half than on
mesial half.
The permanent teeth, which shows prominent
transverse ridge, are maxillary 1st premolar and ii) The distal cusp ridge is longer than mesial cusp
maxillary 1st molar. ridge.

5. PRIMARY maxillary canine differ from PERMANENT 8. MANDIBULAR RIGHT anot LEFT CANINES are
maxillary canine in that - distinguished by
i) It has sharp pointed cusp tip. • Shorter mesial cusp ridge.
ii) The mesial cusp ridge is longer than distal cusp • The mesial surface of mandibular canine is straight
ridge. and is parallel to the long axis of crown.

6. Permanent MAXILLARY canine differs from permanent 9. Teeth, in which the MESIAL CUSP RIDGE IS LONGER
MANDIBULAR canine in that - than distal cusp ridge are
i) Maxillary canine has more mesiodistal diameter. • Primary maxillary canine.
ii) Contains well-developed cingulum as well as lingual • Permanent maxillary first premolar - facial cusp.
fossa.

iii) The crown appears to be short with a long root.


iv) The cusp tip is labial to bisecting line.
Dental ;lut.,e

10. Teeth in which the DISTAL CUSP RIDGE IS LONGER than 19. SUMMARY OF GEOMETRIC LINES.
mesial cusp ridge are I) Triangular or wedge shaped
• Primary and permanent mandibular canine. A. Mesial and distal aspects of six anterior teeth.
• Permanent maxillary canine.
• Permanent maxillary 2nd premolar. II) Trapezoids with longest uneven surface
towards occlusal or incisal surface.
11. Primary cusps of TRIGONE PATTER N are mesiobuccal, A. Labial and lingual aspects of six anterior
distobuccal and mesiolingual. Supplementary cusp or teeth.
secondary cusp in trigone pattern is distoli ngual.
B. Bucca l and lingual aspect of posterior teeth.

12. Mandibular incisor is bilaterally symmetrical when


III) Trapezoids with shortest uneven surface
viewed both labia lly and incisally. The cuspal ridge
is perpendicular to a line bisecting tooth. Because towards occlusal surface.
of these reasons, it is difficult to distinguish between A. Proximal aspects of all maxillary posterior
the mesial and dista l sides, and so the tooth cannot be teeth.
differentiated into right or left.
IV) Rhomboids
13. Maxillary lateral incisor can be identified from mandibular A. Proximal aspects of all mandibular posterior
lateral incisor by presence of deep lingual fossa. teeth .

14. Mandibular lateral incisor can be identified from 20. Mesiodistal and labiolingual diameters of primary
mandibular central incisor in that and permanent teeth:
• The crown is bilaterally asymmetrical as the dista l i) In case of maxillary teeth, the buccolingual
half of the crown appears to be stretched to make diameter is greater than mesiodistal diameter
contact with mandibular canine. except for maxillary central and lateral incisors.
• When viewed incisally, the distal portion of crown
appears to be twisted lingually. ii) In case of mandibular anteriors and premolars, the
• The cervical line is more apically placed on the buccolingual diameter is greater than mesiodistal
lingual side than on labial side. diameter.

15. The crown of mandibular canine appears to bend iii) In case of mandibular molars, the mesiodistal
distally on its root base. This is an important feature diameter of crown is greater than labia lingual
to differentiate maxillary and mandibular canine. diameter of crown.

16. The cusp tip of maxillary canine is labial to the center iv) Among permanent anterior teeth, maxillary central
of crown labia lingually (cusp t ip is present labial to the incisor has a greatest M-D diameter of (8 .5mm)
bisecting line) and mesial to the center mesiodistally. while permanent lower central incisor has t he least
The cusp tip of mandibular canine is present lingual to diameter of 5mm. The labia lingual diameter of
the bisecting line. crown is greater for maxillary canine (9mm) while
the lower central incisor has the least diameter
17. The oblique ridge is present in permanent maxillary 1'1 of 6mm. Thus the lower central incisor is smallest
molar, deciduous maxillary 2nd molar and in some cases tooth in all dimensions.
of deciduous maxillary 1st molar.
v) Among posteriors, mandibular 1'1 molar has the
18. The buccal surface of deciduous 2nd mandibular molar greatest mesiodistal diameter of 11mm, while all
shows three cusps of equal size. They are mesiobuccal, the premolars has the least diameter of 7mm.
buccal and distobuccal. The buccal surface of
permanent mandibular 1st molar shows 3 cusps with vi) The buccolingual diameter is greatest for maxillary
uneven distribution, two buccal and one distal cusp. 1st molar while the mandibular 1st premolar has
, ORAL ANATOMY SYNOPSIS

the least diameter of 7mm. Thus, the mandibular 26. Three cusped pattern is seen in mandibular 2nd
1st premolar is the smallest posterior teeth in all premolar. The order of cusps from largest to smallest
dimensions. is facial, mesiolingual and distolingual.
i) 3 cusp type - square type - occlusal grooves form
21. The curvature of cervical line is greatest on the mesial the letter Y.
surface (3.5mm) of permanent maxillary central incisor
ii) 2 cusp type - round type - occlusal grooves forms
and is least (0 mm) on distal surface of maxillary and
the letter H.
mandibular posteriors.
27. Normally in posterior teeth, more of the occlusal surface
22. Usually, in case of posterior teeth, the M-D diameter
is seen from distal aspect. This is due to presence of
on the buccal surface is greater than the lingual
mesial marginal ridge at high level compared to distal
surface. The exception where M-D diameter is greater
marginal ridge. The exception is mandibular 1st
on lingual surface is maxillary 1" molar. This is due
premolar.
to presence of large mesiolingual cusp and cusp of
carabelli on the lingual surface.
28. The roots of anterior teeth are curved in a distal
direction. In case of posterior teeth, the mesial roots
23. The primary molar, which has prominent transverse
shows distal curvature and the distal roots shows
ridge, is mandibular molar.
mesial curvature.

24. The permanent teeth with transverse ridges are


But the roots of mandibular premolars do not show any
maxillary 1st premolar and mandibular 1st molar.
curvatures.
25. Important developmental grooves
29. Prominent buccal cervical ridge is one of the
Palato gingival groove that characteristic features of mandibular 1st molar.
Maxillary lateral
extends from enamel to
incisor
cementum of root is present in 30. Staining of teeth:
Mesioli ngual developmental i) Metabolic disturbances and systemic disorders in
Mandibular 1st
groove is present in prenatal period mainly affect primary teeth.
premolar
(COMEDK-2013)
ii) Prenatal tetracycline therapy mainly affects
1
Mesial marginal developmental Maxilla ry 1' primary teeth.
groove premolar iii) Staining due to erythroblastosis fetalis affects
Maximum number of the primary teeth only.
Mandibular
developmental grooves are
central incisor
present on cingulum of iv) In porphyria, both deciduous and permanent may
• Mandibular show a red or brownish discoloration. Under UV
central incisors light the teeth always exhibit red fluorescence.
Developmental depressions on
• Upper canines
both mesial and distal sides of v) Ingestion of fluoridated water and tetracycline
roots are seen in • Mesial root of
postnatally (upto 7 yrs. life) will cause staining
mandibular 1st
of permanent teeth.
molar
Deep concavities on mesial Maxillary 1st vi) Critical period for tetracycline induced
surface are present in premolar discoloration:
Deep concavities on distal Maxilla ry 1'1 • Primary incisors: 4 months in utero to 3
surface are present in molar months postpartum.
Largest root with facial and Palatal root of • Primary Canines: 5 months in utero to 9
lingual concavities upper 1st molar months postpartum.
Dental ;lut.,e

• Permanent anteriors: 4 months post partum ii) Mandibular teeth contact with name sake teeth
to about 7 years of age. and the teeth mesial to it in the opposing arch.
Eg.: Mandibular lateral incisor contacts with
vii) Teeth affected by tetracycline will have a maxillary lateral incisor and maxillary central
yellowish or brownish gray discoloration and the incisor.
teeth fluoresce a bright yellow under U.V. light.
Dentin is more heavily stained than the enamel. iii) The teeth with only one antagonist are mandibular
central incisor and maxillary third molar.

viii) Compared to tetracycline, doxycycline or


Mandibular central incisor contacts with maxillary
oxytetracycline will cause less tooth discoloration.
central incisor only while the maxillary third
molar contacts with mandibular 3,d molar only.
31. Among single rooted teeth, bifurcated roots are
most common in mandibular l't premolar followed by
iv) Maxillary canines has antagonists both in anterior
mandibular canines and mandibular incisors.
(Mandibular canine) and posterior segment
(mandibular 1 st premolar).
Maxillary canines and Incisors are least likely to have
bifurcated roots.
Mandibular 1st premolar also has antagonists
both in anterior (maxi llary canine) and posterior
32. Among anterior teeth accessory canals are most
segment (maxillary l't premolar).
common in mandibular central incisor, whereas in
posterior teeth, they are most common in mandibular
v) The anterior teeth will not have any contacts in
1st molar.
centric occlusion and retrusive movements.

33. The bifurcations and trifurcations in mandibular l't


vi) During protrusive contacts maxillary teeth contact
premolar present challenge during cleaning, shaping
with name sake teeth and distal teeth, while the
and obturation. Because of this, it is known as Enigma
mandibular teeth contact with name seek teeth
to endodontist.
and the teeth mesial to it.
Eg.: During protrusive movements, the maxillary
34. The premolar with single central pit is mandibular
central incisor contacts with mandibular right
second premolar.
central and lateral.

35. In maxillary 1st premolar, the facial cusp is well


vii) During lateral working movements, maxillary
developed compared to lingual cusp. In maxillary 2"d
teeth contact with name sake teeth and mesial
premolar both cusps are equally developed.
teeth, while the mandibular teeth contact with
name sake teeth and teeth distal to it.
36. The premolars with maximum buccolingual diameter are
maxillary premolars. Mesiodistal dimension is same for all 39. Baume's classification of primary molars.
premolars.
The distal surface of upper and lower
Flush 2nd deciduous molars will erupt in
37. The posterior teeth with greatest overall length are
Maxillary and Mandibular premolars (8.5+14=22.5 mm). terminal one vertical plane. The permanent
plane molars will erupt in a flush or end on
38. Contact relations between the teeth . relationship
i) Usually maxillary teeth contact with name sake • The distal surface of lower second
teeth and the teeth distal to it in the opposing deciduous molar is more mesial to
arch. Mesi al
distal surface of 2"d deciduous molar.
Eg.: Maxillary central incisor contacts with step
• The permanent molars will erupt in
mandibular central incisor and mandibular lateral Angles Class I occlusion.
incisors.
, ORAL ANATOMY SYNOPSIS

8 inch in diameter with its center in the region


The distal surface of lower second
of glabella.
Distal deciduous molar is distal to upper
step second deciduous molar. The permanent • The curve usually does not exceed 5 to 10
molars will erupt in Class II occlusion. degrees from the horizontal plane of orientation
when viewed in frontal plane.
40. The difference between the total width of deciduous • In involves molar teeth and has its concavity
canines and molars to the total width of permanent facing upwards.
canines and premo lars is known as Leeway space.

The amount of Leeway space is about 1.8mm (0.9mm iii) Antimonsoon curve: (Lateral curve)
on each side) in maxillary arch and about 3.4mm • A curve of occlusion, which involves the teeth
(1.7mm on each side) in mandibular arch. anterior to second premolars.
• It has its convexity facing upwards.
41. Early mesial shift of permanent molars from end-on
to Class I relation occurs by utilizing primate spaces • The 2nd premolars are not involved in any curve
(present mesial to maxillary canines and distal to as they lie on a horizontal plane
mandibular canines).
iv) Pleasure curve:
Late mesial shift of permanent molars from end-on to
• The combination of monson and antimonson
Class I relation occurs by utilizing Leeway spaces.
curves in posterior occlusion is often referred
to as pleasure curve.
42. In Canine guided or mutually protected occlusion,
only there is contact between posterior teeth in
centric occlusion with no contacts of anterior teeth. v) Curve of Wilson:
• It is a curve that contacts the buccal and lingual
In eccentric movements, the posteriors a re relieved cusp tips of the mandibular buccal teeth.
from occlusion and there is contact between the • It is medic-lateral on each side of the arch and
canines only. it is due to ling ual inclination of mandibu lar
posteriors.
Group function refers to multiple contacts in lateral
• The curve of Wilson is concave for mandibu lar
or eccentric mandibular movements.
teeth and convex for maxillary teeth.

Incisal guidance refers to contact of anterior teeth • It is a cross-arch, cross-tooth curve and
during protrusive movements of mandible. indicates the difference between supporting
and non-supporting cusps in occlusion.
43. CURVATURES OF OCCLUSAL PLANES:
44. The lingual cusps of maxillary posterior teeth and
i) Curve of spee: (Anterio-posteriorcurve) [AIIMS-12]
buccal cusps of mandibular posterior teeth are known
• It is an imaginary line touching the buccal as supporting cusps.
cusps of all the lower teeth from lower canine
backwards and approximates to the arc of a Centric stops are areas of contact that a supporting
circle of radius 4" inches or 10 cm. cusp makes with the opposing teeth.
• A continuation of this curve backwards in natural
dentition will nearly pass th rough the head of Maxillary arch contains 6 centric cusps and 7 centric
condyle. stops while the mandible contains 6 centric stops and 6
centric cusps.

ii) Curve of monson: (Lateral curve)


45. The total number of occlusal contact points in dentition
• The curve of occlusion in which each cusp and
is 138.
incisal edge conforms to a segment of sphere of
Dental ;lut.,e

46. Types Of Contacts In Occlusion: 50. Ridge and sulcus apposition:


• Surface contact i) The triangular ridges of buccal cusps of maxillary
molars are accommodates into buccal groove sulci
• Cusp to fossa contact
of mandibular molars.
• Ridge embrasure contact
• Ridge and groove contact ii) The distolingual cusp of mandibular 1'1 molar
fits into the lingual groove sulcus of maxillary
47. Surface contact: 1'1 molar.
During functional movements, the incisal portions
of mandibular anteriors come into contact with the iii) The oblique ridge of maxillary 1' 1 molar fits
lingual surfaces of maxillary anteriors. into the sulcus formed on occlusal surface
of mandibular 1st molar, marked by the junction
48. Cusp and embrasure or ridge-embrasure contad: of distobuccal, central and lingual developmental
In centric occlusion, the cusp tip of maxillary canine grooves
opposes the facial embrasure between lower canine
and premolar i.e., the cusp tip does not make a contact
with any teeth.

49. Cusp to fossa contact:


i) Mesiolingual cusps of maxillary ist molars fits into
central fossa of mandibular first molar.

ii) The distolingual cusps of maxillary 1 st molars are


in apposition to the distal triangular fossae of
mandibular 1st molars and often to the mesial
marginal ridge of mandibular 2"d molar.

iii) The mesiobuccal cusps of mandibular molars


are in apposition to the distal fossa or distal
marginal ridge of the tooth above and mesial to
its namesake (Eg.: 1 st molar mandibular to the
distal of second maxillary premolar).
iv) The distobuccal cusps of mandibular molars
are accommodated by the central fossae of
their namesakes in the opposite jaw (Eg.: The
distobuccal cusp of mandibular ist into centra l
fossae of maxillary 1st molar).

v) The buccal cusp tip of first mandibular premolar


occludes partly with the maxillary 1 st premolar and
partly with maxillary canine while the buccal cusp
tip of second mandibular premolar approaches
the mesial occlusal fossa of the opposing second
premolar.

vi) The lingual cusps of maxillary premolars are in


apposition with the triangular fossa of mandibular
premolars.
, ORAL ANATOMY SYNOPSIS

51. Faciolingual and mesiodistal angulations of different teeth


Angulation / inclination of teeth
Mesiodistal Buccoling ual
7 6 5 4 3 2 1 1 2 3 4 5 6 7
30 100 50 90 17° 70 zo 28° 26° 16° 50 50 30 10°
7 6 5 4 3 2 1 1 2 3 4 5 6 7
14° 100 90 60 60 oo zo 22° 23° 12° 90 90 20° zo

i) Teeth with maximum mesiodistal inclination Maxillary canines (17°)


ii) Teeth with minimum mesiodistal inclination Mandibular lateral incisor (0°)
iii) Posterior teeU1 with maximum mesiodista l inclination Mandibular second molars ((14°)
iv) Posterior teetln with minim um mesiodistal inclination Maxillary z nd premolar (5°)
v) Teeth with maximum faciolingual inclination Maxillary central incisor (28°)
vi) Anterior teeth with least facio lingual inclination Mandibular canines (12°)
vii) Teeth with least facio lingua l inclination Maxillary P 1 premolar (5°)
viii) Posterior teetln with maximum facio lingual inclination Mandibular z nd molar (20°)

52. When two teeth in same arch are in contact, their curvatures adjacent to the contact areas form spillway spaces
called embrasures. The embrasures serves two purposes.
i) Forms spillway for escape of food during mastication.
ii) Prevents food from being forced through the contact area.

53. Contact Areas In Maxillary Teeth:

C.I. ~----= M - Incisal third


D - Junction of incisal and middle third

L.L------= M- Junction of incisal and middle third


D - Middle third
M - Junction of incisal and middle third
Canine---
- - - - 0 - Middle third

(Note: Maxillary canine has long distal slope)

Premolars - Both mesial and distal contact areas are present cervical to the junction of occlusal and middle Third.

p t molars -=
----
= M - Cervical to Junction of occlusal and middle third
D - Middle third

2•d and 3rd molars - Both mesia l and distal contact areas are present in the middle 3rd of crown.

It canbe emphasized that the contact levels on maxillary molars (and even on premolars to some extent) tend to be
centered in the middle 3rd of anatomical crown.
Dental ;lut.,e

54. Contact Areas In Mandible:


Central and lateral incisors - Both M& D contact areas are present at the incisal 3'd.
- - - - M - Incisal 3rd
Canine - - - - D - Cervical to the junction of incisal and middle 3rds

Premolars - Both M & D contact areas are present at the junction of occlusal and middle 3rd.
First, second and third molars - Both contact areas are present at the center of middle thirds of crowns.

55. Contact Areas From Occlusal Aspect:


Maxillary central incisors, lateral incisors, } The contact areas are centered buccolingually.
Premolars, molars and Mandibular Teeth

M - Contact areas are centered buccolingualy.


Maxillary
canine -=:::::::::= D - Contact areas present buccal to the center on the mesial surface of 1st
premolar.

56. Functions of contact areas:


• The main function is to prevent impaction of food in interproximal areas.
• Forming of embrasures.
• Stabilisation of dental arches and protection of periodontium.

57. The curvature of cervical line of most teeth will be approximat ely 1mm less distally than mesially.

58. Number of teeth in oral cavity at different ages


AGE (yrs) NO. OF TEETH TEETH PRESENT
1 yr 12 primary Cl, LI, & 1sr Molar
2 1/2 20 primary All primary teeth
6 yrs 24 teeth 20 primary teeth + 4 permanent molars
7 yrs 24 teeth 16 primary + 8 permanent
(B, C, D, E) + (1, 6 )
OR
14 primary + 10 permanent
(C,D,E, upper B) + (1,6,lower 2)
8-9 24 teeth 12 primary+ 12 permanent
(C, D, E) + (6, 1, 2)
9- 10 24 Teeth 12 primary+ 12 permanent
(C, D, E) + (6, 1, 2)
OR
10 primary + 12 permanent
(D, E, upper C) + ( 6, 1, 2,lower 3)
10-11 24 Teeth 4 primary+ 20 permanent
( E) + (6,1,2,3,4,5)
OR
6 primary + 18 permanent
(E, upper C) + (6, 1,2,4,5,Lower 3)
OR
24 permanent teeth
12 28 Teeth 28 Permanent teeth
, ORAL ANATOMY SYNOPSIS

DENTO OSSEOUS STRUCTURES AND TMJ


The head of
one side glides
59. Each maxilla consists of 4 processes - The zygomatic, forward, but the
nasal (or frontal), palatine and alveolar processes. Chewing movements
head of opposite
side rotates in a
60. TMJ is condylar or Diarthrodal or ging lyo-arthroidal vertical axis.
joint as it has both hinge and sliding (gliding)
movements.

61. Three basic movement patterns of the joint during


mastication are hinge movement, gliding movement
and translation of condyle.

62. The osseous portions of the joint are squamous part of


the temporal bone (glenoid fossa) , articular eminence
of temporal bone and the condylar process of mandible.

63. The posterior slope of the eminence and the anterior


aspects of condyle are functional articular surfaces,
not the mandibular fossa.

64. Interposed between the condyle and temporal bone is


the articular disc or meniscus. Meniscus is not seen
on radiographs.

65. The disc divides the joint into superior and inferior
compartments.

66. Average vertical dimensions of chewing cycle are


between 16 - 20 mm. Average lateral dimensions of
chewing cycle are between 3 -5 mm.

67. The upper compartment permits gliding movements


and the lower compartment permits both gliding and
rotatory movements.

68. DIFFERENT MOVEMENTS OF TMJ


Action Type of Movement
Protrusion Gliding movement
Slight opening of mouth Hinge movements.
Hinge followed by
Wide opening of mouth
gliding
During opening of mandible Hinge followed by
from retruded contact position gliding
Movement fro m retruded
Only hinge
contact position unto terminal
movements.
hinge axis
Dental ;lut.,e

69. CHRONOLOGY OF HUMAN DENTITION.


FIRST EVIDENCE OF CROWN ROOT
TOOTH ERUPTION
CALCIFICATION COMPLETION COMPLETION
PRIMARY DENTITION
UPPER JAW
Central incisor 3-4 mos. in utero 4 mos. 71'2 mos. Ph - 2 yrs

Lateral incisor 4 1h mos. in utero 5 mos. 8mos. Ph - 2 yrs

Canine 5 1/4 mos. in utero 9 mos. 16 -20 mos. 21/2 - 3 yrs


First molar 5 mos. in utero 6 mos. 12 - 16 mos. 2 - 21/2 yrs
Second molar 6 mos. in utero 10 - 12 mos. 20 - 30 mos. 3 yrs
LOWER JAW
Centra l incisor 4112 mos. in ute ro 4 mos. 61'2 mos. Ph - 2 yrs

Lateral incisor 41/2 mos. in ute ro 41/2 mos. 7 mos. Ph - 2 yrs

Canine 5 mos. in utero 9 mos. 16 -20 mos. 21/2 - 3 yrs


First molar 5 mos. in utero 6 mos. 12 - 16 mos. 2 - 21/2 yrs
Second molar 6 mos. in utero 10 - 12 mos. 20 - 30 mos. 3 yrs
PERMANENT DENTION
UPPER JAW

Centra l incisor 3 - 4 mos. 4 - 5 yrs. 7 - 8 yrs 10 yrs


Lateral incisor 10 mos. 4 - 5 yrs. 8 - 9 yrs. 11 yrs
Canine 4 - 5 mos. 6 - 7 yrs. 11 - 12 yrs 13 -15yrs
First premolar 11/2 - 13/4 yrs. 5 - 6 yrs 10 - 11 yrs. 12 - 13 yrs
Second premolar 2 - 21/4 yrs 6 - 7 yrs. 10 - 12 yrs. 12 - 14 yrs.
First molar At birth 21/2 - 3 yrs 6 - 7 yrs. 9 - 10 yrs
Second molar 21/2 - 3 yrs. 7 - 8 yrs. 12 - 13 yrs. 14 - 16 yrs
Third molar 7 - 9 yrs 12 - 16 yrs. 17 - 21 yrs. 18 - 25 yrs.
LOWER JAW
Central incisor 3 - 4 mos. 4 - 5 yrs. 6 - 7 yrs 9 yrs
Lateral incisor 3 - 4 mos. 4 - 5 yrs. 7 - 8 yrs. 10 yrs
Canine 4 - 5 mos. 6 - 7 yrs. 9 - 10 yrs 12 - 14 yrs
First premolar 13/4 - 2 yrs. 5 - 6 yrs 10 - 12 yrs. 12-13yrs
Second premolar 21/4 - 21'2 yrs 6 - 7 yrs. 11-12 yrs. 13 - 14 yrs.
First molar At birth 21/2 - 3 yrs 6 - 7 yrs. 9 - 10 yrs
Second molar 21/2 - 3 yrs. 7 - 8 yrs. 11 - 13 yrs. 15 - 16 yrs
Third molar 8 - 10 yrs 12 - 16 yrs. 17 - 21 yrs. 18 - 25 yrs.
r ORAL HISTOLOGY

ORAL HISTOLOGY
I. REFERENCE BOOKS TAKEN :
1. ORBAN'S ORAL HISTOLOGY AND EMBROLOGY - 11th, 12th & 13th EDITIONS

2. TEXTBOOK OF ORAL HISTOLOGY by TENCATE - 5th, 7th & 8th EDITION

1. DEVELOPMENT AND GROWTH OF TEETH


1. The bud, cap and bell stage of the developing tooth c) Bell stage begins
are based on growth of the d) Dental lami na begins forming
a) Stellate reticulum b) Dental papilla (AIPG -98)
c) Enamel organ d) Dental sac 7. First appearance of toot h formation occurs in:
(MAN-2K) a) 61h weekIU b) 2"dweekIU
2. Dental Lamina is active up to c) 9th week IU d) 4 th month IU
a) lyear b) 2years (PGI -98)
c) 4 years d) 5 years 8. Hertwig's epithelial root sheath is made up of
(MAN -02, 2K) a) Outer and inner enamel epithelium
3. Dental papilla gives rise to b) Stratum intermedium
a) Pulp only b) Pulp & Dentin c) Stellate reticulum
c) Pulp & periodontal ligament d) Hyaline layer of Hopewell-smith
d) All of the above (KAR -02)
(MAN -97) 9. In multirooted teeth, formati on of multiple roots is
4. Ectodermal cells in tooth bud are responsible for due to presence of:
a) Determination of shape of crown and root a) Epithelial diaphragm b) Cell rests of Malassez
b) Formation of periodontal ligament c) Toot bud division or bell stage
c) Formation of cementum d) None of the above
d) Formation of pulp (AIIMS -89)
(MAN -98) 10. Fo llowing is derivative of dental lamina:
5. Disturbance during the morphodifferentiation a) Epithelial rests of Ma lassez
stage of tooth development will result in b) Enamel pearls
a) Change in number of teeth c) Rests of serrae d) All of the above
b) Ameloblastoma (PGI -95, AP-08, MCET-07)
c) Change in form and shape of the teeth 11. The formation of the dental Lamina is initiated by
d) Hypoplasia a) Forebrain b) Rathke's pouch
(MAN -99) c) Neural crest cells d) Odontoblasts
6. In considering the process of eruption, t he pre- (AIIMS -99)
functional eruptive stage of a tooth begins when 12. Accessory canals result from
the: a) Defects in cementogenesis
a) Bud stage begins b) Cap stage begins b) Dividing epithelial bridges

1) C 2) D 3) B 4) A 5) C 6) D 7) A 8) A 9} A 10) C 11) C 12) C


Dental ;lut.,e

c) Break in Hertwig's root sheath 21. High levels of albumin is present in


d) Adherent epithelial rests a) Inner enamel epithelium
(KAR -99) b) Outer enamel epithelium
13. The h ertwig's epithelial root sheath is essential to c) Stellate reticulum d) Stratum intermedium
development of the root because it
a) Gives rise to cementoblast s that produce cementum 22. Nutrition for ameloblasts after 1st layer of enamel is
of the root laid is
b) Moulds the shape of roots and stimulates a) Dental sac b) Odontoblastic process
differentiation of odontoblasts c) REE d) Dental papilla
c) Given rise to odontoblasts that lay down dentin of (PGI -2K)
the root 23. Membrana performativa is seen in
d) Remains as an essential constituent of the a) Bell stage b) Cap stage
periodontal liga ment c) Bud stage d) Advanced bell stage
(AP -98)
14. A supernumerary tooth results from a deviation 24. Enamel pearls are found in
during a) Occlusal surface of premolars
a) Initiation b) Differentiation b) Furcation areas of permanent molars
c) Apposition d) Calcification c) On the incisal edges of incisors
(KAR -01) d) None of the above
15. Which of the following anomalies occur during the
initiation and proliferation of tooth development? 25. Which of the statement is correct
a) Amelogenisis imperfecta a) Enamel cannot form in absence of dentin
b) Dentinogenesis i mperfecta b) The formation of enamel and dentin are independent
c) Enamel hypoplasia d) Oligodontia of each other
(KAR -97) c) Enamel fo rmation can occur in absence of dentin
16. Blood supply of am eloblast for most of its life cycle d) None of the above
comes from
a) Dental papilla b) Reduced enamel organ 26. Atypical or osteodentin is formed due to disturbance
c) Dental Sac d) Dental Pulp during
(COMEDK-09) a) Morphodifferentiation
17. The successors of deciduous teeth develops from b) Histodifferentiation
a) Successional lamina b) Dental lamina c) Apposition d) Initiation
c) Stellate reticulum d) Neutral ectodermal cells
27. Any insult to cells in apposition stage results in
18. Lip furrow band is a) Hutchinson's incisor b) Osteodentin formation
a) Vestibular lamina b) Successional lamina c) Enamel hypoplasia d) Anodontia
c) Dental lamina d) Basal lamina
28. Order of development of tooth germ is
19. Outer and inner enamel epithelium first seen in a) Bell-bud-cap b) Cap-bell-bud
a) Bud stage b) Adva need bell stage c) Bud-cap-bell d) Bell-cap-bud
c) Cap stage d) Bell stage
29. Cementum formation is seen
20. Temporary structures in enamel organ are (before a) After dentin formation
enamel formation) b) Before dentin formation
a) Enamel knot b) Enamel cord c) Both are formed at same ti me
c) Both A and B d) Enamel cuticle d) None of the above
(KAR -03) (AP -02)

13} B 14} A 15} D 16} C 17} A 18} A 19} C 20} C 21} C 22} A 23} A 24} B 25} A
26} B 27} C 28} C 29} A
r ORAL HISTOLOGY

30. Stellate reticulum is seen in d) Cells of stratum intermedium


a) Dental lamina b) Enamel organ (COMEDK -2013)
c) Hertwig's root sheath 38. The cap stage in tooth development is characterized
d) All of the above by
(AP -02) a) morphodifferentiation
31. Morphological stages of tooth development are b) reversal of functiona l polarity
explained based on: c) formation of hammock ligament
a) Shape of enamel organ d) Increased mitotic activity
b) Shape of tooth germ (AP-2013)
c) Functions of enamel organ 39. The transitory structures that may be seen during
d) Shape of dental papilla early tooth development are
(COM EDK- 05) a) Enamel Knot b) Enamel Navel
32. The disturbances occurred during "Calcification" c) Enamel Cord d ) ALL of the above
stage of tooth development is seen in : (GCET-14)
a) Peg laterals 40. During cap stage signalling in odontogenic
b) Microdontia epithelium is done by?
c) Supernumerary tooth a) Dental follicle b) Enamel knot
d) Interglobular dentin c) Dental papilla d) Inner enamel epithelium
(AIIMS-06) (GCET-14)
33. Tooth develops from: 41. During which stage of tooth formation the size of
a) Ectoderm, mesoderm the tooth is determined?
b) Ectoderm a) Apposition b) Histodifferentiation
c) Mesoderm c) Morpho differentiation
d) Ectoderm and e ndoderm d) Calcification
(PGI-06) (AIIMS NOV-14)
34. Which of the following is not an ecto-mesenchymal 42. Under light microscopy, an active synthetic cell
derivative? demonstrates
a) Dentin b) Cementum a) Flattened nucleus b) Open faced nucleus
c) Pulp d) Enamel c) Close faced nucleus d) Vesicular nucleus
(AP-07) (COMEDK-2015)
35. The relationship between the pulpaland periodontal 43. The following structure acts as a buffer against
tissue complex begins during dental development physical forces in the enamel organ
at a) Stratum intermedium b) Outer enamel epithelium
a) Adolescent stage b) 5-10 years c) Cervica l loop d) Stellate reticulum
c) Embryonic stage d) None of the above (KERALA-2015)
(BHU-2012)
36. In cap stage, regulating factor for cusp shape
formation is present in
a) Dental papilla b) Dental follicle
c) In ner enamel epithelium
d) Enamel knot
(AIPG-2012)
3 7. The cells of the enamel organ which differentiate
into future amelo blasts are
a) Cells of inner enamel epithelium
b) Cells of outer e11amel epithelium
c) Cells of stellate reticulum

30) B 31) A 32) D 33) A 34) D 35) C 36) D 37) A 38) D 39) D 40) B 41) C 42) B
43) A
Dental ;lut.,e

1. DEVELOPMENT AND GROWTH OF TEETH - ANSWERS


1. '(' [Orban's 11th ed 31/ 12th ed 23/ 13th ed 27] epithelium entrapped within gingiva. Option 'A& B' are
The bud, cap and bell stage are named after t he shape remnants of Hertwig epithelial root sheath.
of enamel organ(epithelia l part of tooth germ).
11. 'C' [Orban's lith ed 28/ 12th ed 22/ 13th ed 24, 25]
2. 'D' [Orban's 11th ed 31/ 12th ed 23/ 13th ed 25]
Even after 5 years it may be active in 3rd molar area 12. 'C' [Orban's 11th ed 44/ 12th ed 31/ 13th ed 35, 37]
after it has disappeared elsewhere. Remnants of dental Accessory canals may occur either due to loss of root
lamina persist as epithelial pearls or islands within the sheath cells or due to inclusion of blood vessels during
jaws and gingiva. root development.

3. ' B' [Orban's 11th ed 34/ 12th ed 25/ 13th ed 26] 13. ' B' [Orban's 11th ed 41/ 12th ed 30/ 13th ed 35]
The cells of dental papilla will form pulp and dentin
and the cells of dental sac will form cementum and 14. 'A' [Orban's 11th ed 45/ 12th ed 33/ 13th ed 46]
periodontal ligament.
15. 'D' [Orban's 11th ed 45/ 12th ed 33/ 13th ed 46]
4. 'A' [Orban's 11th ed 31/ 12th ed 23/ 13th ed 26-27] Lack of initiation may result in oligodontia or anodontia.
Abnormal initiation may result in supern umerary teeth .
5. 'C' [Orban's 11th ed 47 / 12th ed 35/ 13th ed 46]
Disturbances in morphodifferentiation results in 16. 'C' [Orban's 12th ed 65/ 13th ed 74]
supernumerary roots or cusps, twinning, suppression As long as the inner enamel epithelium is in contact
of parts like cusp or root. Hutchinson's incisor is due with the dental papilla, it recieves its nut rient supply
to disturbance in morphodifferentiation. from the blood vessels of dental papilla. When dentin
forms, it cuts off the ameloblasts from the source of
6. 'D' [Orban's 11th ed 28/ 12th ed 22/ 13th ed 25] nourishment. From then, they are supplied by the
The dental lamina serves as the primordium of capillaries of dental sac.
ectodermal portion of deciduous teeth. The permanent
molars arise directly from a distal extension of dental 17. 'A' [Orban's 11th ed 31/ 12th ed 23/ 13th ed 25]
lamina. Successional la mina is the lingua l extension of dental
lamina. Permanent anteriors and premolars develop
7. 'A ' [Orban's 11th ed 28/ 12th ed 22/ 13th ed 25] from successional lamina whereas the permanent
molars arise directly from distal extension of dental
8. 'A' [Orban's 11th ed 41/ 12th ed 30/ 13th ed 35] lamina.
Hertwig's root sheath includes the outer and inner
enamel epithelium only. It does not include the stratum 18. 'A' [Orban's 11th ed 31/ 12th ed 23/ 13th ed 25]
intermedium and stellate reticulum. Lip furrow band or vestibular lamina is present buccal
to the dental lamina. It forms the oral vestibule
9. 'A' [Orban's 11th ed 42/ 12th ed 31/ 13th ed 35] between the alveolar portion of jaws and the lips and
Prior to beginning of root formation, the HERS forms cheeks.
epithelial diaphragm. The outer and the inner enamel
epithelia bend at the future cementoenamel junction 19. 'C' [Orban's 11th ed 34/ 12th ed 25/ 13th ed 28]
into a horizontal plane, narrowing the wide cervical
opening of the tooth germ. Differential growth of the 20. 'C' [Orban's 11th ed 36/ 12th ed 26/ 13th ed 29)
epithelial diphragm in the multi rooted teeth causes Both enamel knot and cord are temporary structures
the root trunk into two or th ree roots. and disappear before enamel formation begins. Their
function is to act as reservoir of dividing cells for the
10. 'C' [Orban's 13th ed 26] growing enamel organ. Also enamel niche, enamel
Options 'C' rests of serres are remnants of dental laminal navel are the tern porary or transitory structures
r ORAL HISTOLOGY

21. 'C' [Orban's 11th ed 36/ 12th ed 26/ 13th ed 29) Consequently their organmng influence on
The spaces in stellate reticulum are fi lled with albumin mesenchyma l cells of dental papilla is disturbed and
rich mucoid fluid that gives the stellate reticulum a as a result atypical dentin or osteodentin is formed
cushion like consist ency and th us protects the delicat e
enamel forming cells. 27. 'C' [Orban's 11th ed 48/ 12th ed 35/ 13th ed 46)
Both hypoplasia and hypo calcification occurs as a
22. 'A' [Orban's 11th ed 83/ 12th ed 65/ 13th ed 74) result of an insult to cells in appositional stage.

23. 'A' [Orban's 11th ed 39/ 12th ed 29/ 13th ed 33) 28. 'C' [Orban's 11th ed 32/ 12th ed 25/ 13th ed 26)
Membrane preformativa separates the enamel organ
and dental papilla before the dentin formation; and it 29. 'K [Orban's 11th ed 43/ 12th ed 23/ 13th ed 35)
determines the morphology of the crown.
30. 'B' [Orban's 11' h ed 35/ 12'h ed 25/ 13th ed 29)
24. 'B' [Orban's 11th ed 44/ 12th ed 31/ 13th ed 35)
31. 'K [Orban's 11th ed 32/ 12th ed 25/ 13th ed 27)
25. 'A' [Orban's 11th ed 47 / 12th ed 35/ 13th ed 38) Morphological stages of tooth development, i.e. bud,
Dentin is necessary for the differentiation of inner cap and bell stage are named after the shape of enamel
enamel epithelium into enamel forming ameloblasts. organ.
So enamel does not form in the absence of dentin.
32. ' D' [Orban's 11th ed 112/ 12th ed 87/ 13th ed 101)
26. 'B' [Orban's 11th ed 47 / 12th ed 35/ 13th ed 46)
Normally the ameloblasts causes the differentiation 33. 'A' [Orbans 11th ed 5,28/ 121 hed 7, 18/ 13th ed 7, 26)
of mesenchymal cells of dental papilla into dentin Most of skeletal and connective tissues with exception
forming odontoblasts. In vitamin A deficiency, the of enamel are derived from ectomesenchymal tissue.
ameloblasts fail to differentiate properly. Enamel is derived from ect oderm on ly.

34. 'D' [Orban's 11thed 5/ 12thed 7/ 13th ed 7, 26]


Enamel Dentin Cementum Bone
Embryology Ectoderm Ectomesenchymal Ectomesenchymal Mesoderm

Tissue Epithelial Connective Connective Connective


Formative cells Ameloblasts Odontoblasts Cementoblasts Osteoblasts

Degradative cells Odontoclasts Odontoclasts Odontoclasts Osteoclasts


Mineral content 96% 70% 45 to 50% 50 to 60%

Yes; Only pain No.


Sensitive No Yes
appreciation (Due to absence of nerve supply)
None in dentin. Tissue
Blood and Avascular. Cells supplied by Vessels in
Avascular fluid circulates in
nutritive supply diffusion from POL bone.
tubules

35. 'C' [Orban's 13th ed 26)

36. ' D' [Orban's 13th ed 29, 45)


• Enamel Knot: The cells in the center of the enamel organ are densely packed to form enamel knot.
• Enamel cord: It is the vertical extension of the enamel knot.
Dental ;lut.,e

• Enamel Septum: The enamel cord extends to meet the outer enamel epithelium and form enamel septum.
• Enamel Navel: It is the small depression at the point of meeting of enamel cord and outer enamel epithelium.
• Enamel niche is the apparent structure created by the plane of histological sections (COMEDK-2015) cutting
through a curved dental lamina so that mesenchyme is surrounded by the dental epithelium. It appears as a
double attachment of dental lami na to the overlying epithelium.

Recent studies shows that enamel knot acts as a signaling center expressing growth factors which play an important
role in determining the shape of the tooth

37. 'A' (Orbans 13th ed 32)


Fate of various cells in enamel organ:
• Inner enamel epithelium - differentiate into ameloblasts
• Stellate reticulum - collapses and does not exist
• Outer enamel epithelium - cells flattens and thrown into folds.

38. 'D' (Orbans 13th ed 37, 38, Fig 3-2 on Pg 26)


Mitosis or proliferation of cells occurs mainly in bud, cap and early bell stage. For morphodifferentiation the important
stage is advanced bell stage. Reversal of the functional polarity occurs in the organizing stage of life cycle of
ameloblasts (also refer 16-point synopsis). Cushion hammock ligament is hypothesized as an important structure for
tooth eruption.

39. 'D' (Check Explanation of Q. No. 36)

40. 'B' (Check Explanation of Q. No. 36)

41. 'C' (Orbans 13th ed 38)


Morphodifferentiation establishes the morphologic pattern, or basic form and relative size of future tooth.

42. 'B' (Check Explanation Below]


An 'active' synthetic cell demonstrates large nucleolus, open faced nucleus, columnar shaped cells and large amount
of rough endoplasmic reticulum and more golgi vesicles.

43. 'A' (Refer Explanation of Q.No.21)


r ORAL HISTOLOGY

2. ENAMEL
1. Perikymata are the external surface manifestations 7. Ratio of diameter of enamel rods in the DEJ to the
of enamel surface is
a) Enamel rods a) 1:4 b) 1:1
b) Incremental lines of retzi us c) 1:2 d) 2:1
c) Nasmyth's membrane (MAN -02)
d) Pellicle 8. Blunt cell processes seen on the developing
(MAN -94) ameloblasts are called as :
2. The percentage of inorganic matter in fully a) Tomes fibres b) Tomes layer
developed enamel is about c) Tomes processes d) Odontoblastic processes
a) 66% b) 76% (MAN -2K)
c) 86% d) 96% 9. lnterwining of the enamel rods at the cusp tips and
(MAN -94) the incisal edges of a toot1'1 is called
3. The enamel of the posterior teeth is thickest at a) Enamel spindles b) Gnarled enamel
a) Cervical third of buccal surface c) Incremental lines of woven
b) Middle third of buccal surface d) Incremental lines of retzius
c) Middle third of lingual surface (MAN -2K)
d) Occlusal surface 10. Incremental Lines of retzius are seen in
(MAN -98) a) Enamel b) Dentin
4. Hunter Schreger band are c) Cementum d) Pulp
a) Dark and light bands of enamel seen in longitudinal (AP -99, 03)
ground section 11. All are true regarding the enamel spindles except
b) Dark and light bands of enamel seen in horizontal a) Produced by a me lo blasts
ground section b) Found in the region of cusps
c) Dark and light bands of dentin seen in longitudinal c) Surrounded by inter prismatic enamel
section d) Terminate as rounded process
d) Dark and light bands of dentin seen in horizontal (KAR -99)
section 12. Ionic exchange between enamel surface and
(MAN -98) environment:
5. Enamel tufts are a) Does not take place once enamel matures
a) Thin tuft like structure extending from enamel b) Stops after 2 years of eruption
surface to dentin-enamel j unction c) Continues till adult life
b) Tuft like structure arising from dentin-enamel d) Continues through out life
junction towards enamel surface (AIIMS -90)
c) Odontoblastic processes extending into enamel 13. Approximately how many enamel rods will be
d) None of the above present in a maxillary molar?
(MAN -98) a) 5 million b) 7 million
6. Following is not an external manifestation of c) 9 million d) 12 million
enamel (KAR -01)
a) Enamel tufts b) Enamel lamella 14. All are true about the striae of retzius except
c) Incremental lines of enamel a) Constitute the rest lines with in the enamel rods
d) Enamel cuticle b) Have high inorganic content
(MAN -02) c) They are areas of increased porosity
d) Allow the movement of water and small ions
(KAR -01)

1) B 2) D 3) D 4) A 5) B 6) A 7) C 8) C 9) B 10) A 11) A 12) D 13) D


14) B
Dental ;lut.,e

15. Neonatal lines are found in all of the following 23. Specific gravity of enamel is :
except a) 2.8 b) 3.8
a) Enamel of primary incisors c) 4.8 d) 2.3
b) Enamel of permanent canines
c) Enamel and dentin of permanent first molars 24. Inorganic content of enamel is about
d) Dentin of permanent mandibular incisors a) 35% b) 50%
(AP -98; AIPG -96) c) 4% d) 96%
16. On microscopic examination enamel rods have:
a) Key hole appearance in cross-section 25. Length of enamel rods is
b) Paddle appearance in cross-section a) Greater than the thickness of enamel
c) Lanullate appearance in cross-section b) Less than the thickness of enamel
d) None of the above c) Equal to the thickness of enamel
(AIPG -96) d) None of the above
17. Which of the following structures is not of
ectodermal origin: 26. The diameter of the rod is
a) Hunter-schreger bands a) 4 µm b) 9 µm
b) Enamel spindles c) 7 µm d) 2 µm
c) Enamel tufts d) Enamel lamellae
(AIIMS -2K) 27. Which of the following statement is correct
18. The formative cells of which of the following dental a) In cross sections enamel rods appear hexagonal
tissues disappear once tissue is formed and resembles fish scales
a) Enamel b) Dentin b) The bodies of rods are near occlusal and incisal
c) Periodontal ligament d) Cementum surfaces, where as the tails point cervically
(AP -98) c) The appatite crystals are parallel to bodies and
19. Which of the following tissues have no reparative deviate 65 ° from the tails.
capacity: d) All of the above
a) Enamel b) Dentin
c) Cementum d) Periodontal ligament 28. True about direction of enamel rods
(PG! -98, 97) a) They originate at right angle to dentin surface
20. The enamel has no capacity of self -repair because b) They are almost vertical near the incisa l edge (or)
a) It has only a small percent of organic content cusp tip areas
b) Its formative cells are lost once it is completely c) In the cervical third of deciduous crowns they are
formed almost horizontal in direction
c) It is essentially a keratin tissue and has no blood d) In cervical region of permanent teeth, they deviate
vessels in apical direction
d) It has no direct connection with the active cells of e) All of the above
the dental pulp
(AIIMS -2K) 29. Thin leaf like structures that extend from enamel
21. The type of fibrous element in enamel is : surface in to DEJ are
a) Tono filament b) Keratin like fib re a) Enamel spindles b) Ernamel lamella
c) Collagen fibre d) Vimentin fi lament c) Enamel tufts d) Perikymata
(KAR -01)
2 2. Calcification in enamel starts at : 30. Dentino enamel junction is
a) Enamel matrix b) Root a) Non-scalloped
c) CEJ d) DEJ b) Straight
(AIIMS -95) c) Scalloped and the convexities are directed towards
dentin

15} B 16} A 17} B 18} A 19} A 20} B 21} B 22} D 23} A 24} D 25} A 26} A 27} D
28} E 29} B 30} C
r ORAL HISTOLOGY

d) Scalloped and the convexities are directed toward c) Only on odontoblasts


enamel d) Only by ameloblasts
(TNPSC -99)
31. Hertwig's root sheath is formed from 39. Ameloblast is
a) Stratum inter r111edium a) A cell from which tooth enamel is formed
b) Stellate reticulum b) A tissue from which tooth erupts
c) Outer enamel epithelium c) A tumour of the jaw
d) Cervical loop d) Pertaining to dentin
(TNPSC -99)
32. Chronological hypoplasia is 40. Synthesis of enamel matrix proteins occurs in the
a) Hypoplasia of local origin a) Outer enamel epithelium
b) Hypoplasia of systemic origin b) Ameloblast
c) I lypoplasia of hereditary origin c) Stratum intermedium
d) None of the above d) Stratum granulosum
(UPSC 01)
33. Moth eaten appearance of enamel is seen in 41. The small ridges, perikymata, seen on facial
a) Odontodysplasia b) Amelogenisis imperfecta surfaces of canines are the result of
c) Dentin dysplasia d) Mottled enamel a) Hyperplasia b) Hypocalcification
c) Normal development
34. Formation of dentin by odontob lasts begins in d) Interstitial development
a) Morphogenic stage b) Organizing stage (APPSC -99)
c) Desmolytic stage d) Formative stage 42. Among the following, the structure that is most
calcified is
35. Which of the following is correct a) Enamel lamella b) Enamel rod
a) Each rod is formed by 4 ameloblasts c) Enamel spindle d) Enamel tuft
b) Each ameloblast contributes to 4 different rods (UPSC -01)
c) Head of the rod is formed by one ameloblasts, and 43. Enamel spindles are formed by
tail is formed by three other a meloblasts a) Cracks b) Ameloblasts
d) All of the above c) Odontoblasts d) Hypocalcified rods
(APPSC -99)
3 6. The striking difference between enamel and other 44. Neonatal line is also referred to as
mineralized tissues is a) Retzius line b) Line of cross striations
a) Presence of high percentage of organic matrix c) Incremental lines d) Line of enamel tufts
b) Presence of incremental lines (TNPSC -99)
c) Enamel has high reparative capacity 45. Incremental lines of Retzius appear
d) Most of organic component is lost during a) As lines in the enamel running at right angles to
mineralization the enamel surface
b) As lines in the dentin running at right angles to
37. Shape of enamel rod is the dentine-enamel junction
a) Key hole or paddle shaped c) In enamel and follow the apposition pattern
b) Square d) In dentin and follow the appositional pattern
c) Round (APPSC -99)
d) None of the above 46. Nutritional supply of the ameloblasts during most
of their life cycle is from the:
38. Enamel is laid down a) Dental pulp b) Odontoblastic processes
a) Mainly by odontoblasts c) Dental sac d) Reduced dental organ
b) Mainly by ameloblasts (PGI -01)

31) D 32) B 33) A 34) B 35) D 36) D 37) A 38) D 39) A 40) B 41) C 42) B 43) C
44) C 45) C 46) C
Dental ;lut.,e

4 7. Enamel rods at the cervical area and at the occlusal 56. The average width of enamel rods is
edge and incisal tip in permanent teeth are a) 10 microns b) 5 microns
a) Straight b) Shallow c) 2 microns d) 0.5 microns
c) Bent d) Curved (COMEDK-09)
(AP -99) 5 7. Maximum amount of phosphate is seen in
48. Metalloprotienases are used in which stage of the a) Bone b) Ernamel
amelogenesis: c) Gingiva d) Dentine
a) Organising b) Formative/ Secretory (AIIMS-09)
c) Protective d) Desmolytic 58. Dark bands that are present in enamel rods at
(PGI-06) intervals of 4 µms are
49. The thickness of prismless enamel in deciduous a) Neonatal bands b) Cross striations
teeth is: c) Striae of Retzi us d) Hunter-Schregar bands
a) 25 µm b) 50 µm (KCET-2011)
c) 75 µm d) lOOµm 59. Which of the following is correct about enamel
(AIPG-07) crystals?
50. The class of low molecular weight calcium binding a) Same as dentin crystals
proteins seen predominantly in developing enamel b) Contains organic portion in the center
are: c) Roughly Hexagonal in shape
a) Enamelin b) Amelogenin d) Their diameter is about 5 microns in size
c) Tuftelin d) Ameloplakin (AP-2011)
(COMEDK- 07) 60. Enamel can act in a sense like a
51. Gnarled enamel is seen in: a) Permeable membrane
a) Pro xi ma l surface of tooth b) Impermeable membrane
b) Cervical region of tooth c) Semipermeable membrane
c) Cusp tips of tooth d) All of the above d) Infrapermeable membrane
(COMEDK- 06) (BHU-2012)
52. The 'Key hole' pattern appearance in the cross 61. Non acidic, nonionic protein which helps in enamel
section is feature of mineralization
a) Aprismatic enamel b) Prismatic enamel a) Tuftelin b) Sheathlin
c) Intertubular dentin d) Intratubular dentin c) Enamelin d) Amelogenin
(COMEDK-08) (AIPG-2012)
53. Neonatal lines are seen in 62. Enamel lamellae
a) Enamel of deciduous incisors a) Elevation on outer surface of enamel
b) Enamel of permanent incisors b) Dentinal tubule in enamel
c) Enamel of premolars c) Uniform arrangement of enamel rods
d) Dentin of all teeth, as if is formed first d) Enamel projection in dentin
(AP-08) (AIIMS-13)
54. The shallow furrows on the enamel surface, where 63. Water content in enamel by volume
striae of Retzius end are known as? a) 2-3% b) 6-7%
a) Cracks b) Pellicle c) 20-25% d) 10-12%
c) Perikymata d) Enamel lamellae (AIIMS-13)
(COMEDK-08) 64. A novel acidic non amelogenin enamel protein
55. Which of these DOES NOT contain collagen which helps in mineralization?
a) Enamel b) Dentin a) Fillagrin b) Tuftelin
c) Bone d) Cementum c) Sheathlin d) Amelogenin
(COMEDK-09) (AIIMS MAY-2014)

47) C 48} B 49} A 50} B 51} C 52} B 53} A 54) C 55) A 56) B 57) B 58) B 59} C
60} C 61} D 62} D 63} B 64} B
r ORAL HISTOLOGY

65. The defect in matrix formation during formation of


enamel leads to
a) Gnarled enamel b) Enamel tufts
c) Enamel hypoplasia d) Mamelons
(COMEDK-2015)
66. Amelogenesisimperfecta is due to?
a) ENAM b) MSX-1
c) DSPP d) PAX-9
(PGI DEC - 2013)
67. Following is NOT a derivative of neural crest cells
a) Enamel b) Dentin
c) Periodontal ligament d) Alveolar bone
(COMEDK-2015)
68. Most nature enamel is found at?
a) Apical region b) Incisal edge and cusp tip
c) Middle region d) Cervical region
(PGI JUNE-2012)

65) C 66) A 67) A 68) B


Dental ;lut.,e

2. ENAMEL - ANSWERS
1. 'B' [Orban's 11th ed 62/ 12th ed 51/ 13th ed 58] The number of enamel rods ranges from 5 million in
The number of perikymata gradually decreases form lateral incisor and 12 million in upper 1st molar.
cervical region to occlusal or incisal region because
of undisturbed development of enamel prior to birth. 14. 'B' [Orban's 11th ed 60/ 12th ed 51/ 13th ed 57]
Perikymata are usually absent in prenatally developed
occlusal parts of deciduous teeth, whereas t hey are 15. 'B' [Orban's 11th ed 63, 117/ 12th ed 54/ 13th ed 60]
present in postnatal cervical parts. In deciduous teeth and in permanent molars and Lower
incisors, in which the enamel and dentin are formed
2. 'D' [Orban's 11th ed 50/ 12th ed 46/ 13th ed 51] partly before and partly after birth, the prenatal and
Enamel contains 96% of inorganic material and Ii% postnatal enamel and dentin, are separated by the
of organic material. The inorganic material is mainly neonatal line or ring.
com posed of hydroxyappatite crystals.
Note:
3. 'D' [Orban's 11th ed 49/ 12th ed 45/ 13th ed 50] According to some authors, neonatal line is not seen
in permanent lower incisors.
4. 'A' [Orban's 11th ed 59/ 12th ed 50/ 13th ed 57]
Hunter-schreger bands are due to change in direction 16. 'A' [Orban's ttth ed 52/ 12th ed 47/ 13th ed 53, 54]
of rods; and are regarded as a functional adaptation to Enamel rods appear in cross section as keyholes (or)
minimize the risk of cleavage due to occlusal forces. fish scales (or) staggered arches (or) stacked arches.

5. 'B' [Orban's 11th ed 70/ 12th ed 56/ 13th ed 62] 17. ' B' [Orban's 11th ed 72/ 12th ed 58/ 13th ed 65]
Enamel spindles are odontoblastic process, which
6. 'A' [Orban's 1tth ed 70/ 12th ed 56/ 13th ed 62] extend into enamel. Enamel spindles are mesenchymal
Perikymata are transverse, wave like grooves, believed in origin .
to be the external manifestations of Incremental lin es
of Retzius. Enamel cuticle is a delicate membrane that 18. 'A' [Orban's ttth ed 86/ 12th ed 68/ 13th ed 75, 86]
covers the entire ena mel of the crown of a newly erupted When t he enamel has fully developed, the ameloblasts
tooth but is probably soon removed after mastication. cease to be arranged in a well-defined layer and is
Enamel lamellae are thin leaf like structures that transformed to reduced enamel epit helium. The
extend from the enamel surface towards DEJ. function of reduced enamel epithelium is protecting
the mature enamel by separating it from the connective
7. 'C ' [Orban's 11th ed 52/ 12th ed 47/ 13th ed 53] tissue until the tooth erupts. Premature degeneration
of the reduced enamel epithelium may prevent the
8. 'C' [Orban's 1tth ed 89/ 12th ed 69/ 13th ed 77] eruption of a tooth .

9. ' B' [Orban's 11th ed 59/ 12th ed 50/ 13th ed 57] 19. 'A' [Orban's ttth ed 86/ 12th ed 68/ 13th ed 75, 86]
As the ameloblasts disappear after forming enamel,
10. 'A' [Orban's 11th ed 60/ 12th ed 51/ 13th ed 57] there is no capacity of self-repair in enamel.

11. 'A' [Orban's t t th ed 72/ 12th ed 58/ 13th ed 65] 20. ' B' [Orban's 11th ed 86/ 12th ed 68/ 13th ed 7 5, 86]
Enamel spindles are formed by extensions ofodontoblast
processes into enamel before calcification. 21. 'B' [Orban's 11th ed 51/ 12th ed 46/ 13th ed 51]

The principal mineral content of dental hard


12. 'D' [Orban's 1tth ed 49/ 12th ed 45/ 13th ed 51]
tissues is (GCET -14)
a) Ca6 (P04) 10 (0H) 2 b) Ca 10 (P04) 10 (0H) 2
13. 'D' [Orban's 11th ed 51/ 12th ed 46/ 13th ed 53]
c) Ca 10 (P04) 6 (OH) 2 d) Ca 6 (P04 \(OH) 2
r ORAL HISTOLOGY

22. ' D' [Orban's 11th ed 95/ 12th ed 73/ 13th ed 85) 36. ' D' [Orban's 11th ed 93/ 12th ed 70/ 13th ed 86)
Calcification of enamel proceeds from cusps or incisal Nearly 90% of initially secreted protein as well as
edges towards cervical line and from DEJ it proceeds water are lost in mineralizat ion. This is the striking
towards surface. difference between enamel and other mineralized
tissues.
23. 'A' [Orban's 11th ed 49/ 12th ed 46/ 13th ed 51)
37. 'K [Orban's 11th ed 92/ 12th ed 69/ 13th ed 53)
24. 'D' [Orban's 11th ed 50/ 12th ed 47/ 13th ed 51)
Organic Inorganic Water 38. 'D' [Orban's 11th ed 86/ 12th ed 68/ 13th ed 71)
content content
39. 'A' [Orban' s 11th ed 86/ 12th ed 68/ 13th ed 71)
Enamel 4% 96% 6
Dentin, Bone 35% 65% 22 40. 'B' (Orban's 11' h ed 83/ 12•~ ed 66/ 13th ed 76]
Cementum 55% 45% -
41. 'C' [Orban's 11th ed 62/ 12th ed 51/ 13th ed 58)

25. 'K [Orban's 11th ed 51/ 12th ed 46/ 13th ed 53)


The enamel rod has a length of 9µm, breadth of 5~Lm 42 • 'B' [Orban's 12th ed 73 / 13th ed 53, 61, 63, 65)
and diameter of 4µm. The enamel rod length is greater
than the thickness of enamel because of the oblique 43 · 'C' [Orban's 11 th ed 72/ 12 th ed 5 8/ 13th ed 65)
direction and the wavy course of the rods.
44. 'C' [Orban's 11th ed 63/ 12th ed 54/ 13th ed 60)

26. 'A' [Orban's 11th ed 52/ 12th ed 47 / 13th ed 53)


45. 'C' [Orban's 11th ed 60/ 12th ed 61/ 13th ed 57)

27. 'D' [Orban's 12th ed 47-48/ 13th ed 53, 54)


46. 'C' [Orban's 11th ed 83/ 12th ed 66/ 13th ed 7 4)
Before the appearance of 1st layer of dentin, ameloblast
28. ' E' [Orban's 11th ed 57/ 12th ed 50/ 13th ed 54)
derive nutrition from dental papilla. After dentin layer
formation they receive nutrition from the capillaries of
29. ' B' [Orban's 11th ed 68/ 12th ed 54/ 13th ed 61)
dental sac that surround the outer ename l epithelium.

30. 'C' [Orban's 11th ed 70/ 12th ed 56/ 13th ed 64]


47. 'C' [Orban's 11th ed 57/ 12th ed 50/ 13th ed 54)
The rods are rarely straight and usually follow a wavy
31. 'D' [Orban's 11th ed 80/ 12th ed 65/ 13th ed 72)
course throughout. The rods are vertical in the region
of edge or tip of the cusp and changes to a oblique
32. ' B' [Orban's 11th ed 100/ 12th ed 78/ 13th ed 86)
direction in the incisal and cuspal region. The rods are
Hypoplasia and hypocalcification may be caused by
horizontal at the cervical and centra l parts of crown
systemic, local or hereditary factors. Hypoplasia of
in deciduous teeth. In permanent teeth the rods bent
systemic origin is termed as chronologic hypoplasia,
cervically in the cervical region.
because the lesion is found in the areas of those teeth
where the enamel was formed during the systemic
48. 'B' [Tencate's 7th ed 159, 175)
disturbance.
Enamelysin is a member of the matrix metalloproteinase
family, and its mRNA has been cloned from pig,
33. 'A' [Orban's 11th ed 101/ 12th ed 79/ 13th ed 87)
human, cow, and mouse. Enamelysin is secreted into
the enamel matrix during the secretory stage through
34. ' B' [Orban's 111h ed 83/ 12th ed 66/ 13th ed 74)
transition stage of enamel development
Formation of dentin starts in organizing stage and
enamel is formed in formative stage.
49. 'A' [Orban's 11th ed 62/ 12th ed 51/ 13th ed 58]
A relatively structureless or prismless layer of enamel,
35. ' D' [Orban's 11th ed 92/ 12th ed 69/ 13th ed 79 , 81)
approximately 30µm thick is present in 70% of
permanent teeth and all deciduous teeth.
Dental ;lut.,e

50. 'B' [Orban's 11th ed 451/ 12th ed 333/ 13th ed 51] 58. ' B' [Orbans 11th ed 57/ 13th ed 54]
Amelogenins and enamelins are two major classes of Each enamel rod is built up of segments separated by
calcium binding proteins. Amelogenins are relatively dark lines that give it a striated appearance, which
low-molecular-weight proteins secreted by ameloblasts becomes more visible in grournd sections. The rods are
and are predominant proteins in the developing enamel segmented because the enamel matrix is formed in a
matrix. They will disappear in enamel maturation rhythmic manner. The length of each segment is about
period. Enamelins have very high affinity for binding 4µm.
apatite crystals and remain until late stages of
enamel maturation of the enamel proteins, 90% are 59. 'C' [Orbans 12th ed 47/ 13th ed 51]
heterogenous group of gene-specific, low molecular The diameter of enamel crystal or rod is about 4µm .
weight proteins known as amelogenins. Rods does not contain any organic portion in the
center.
The remaining 10% of enamel protein consists of
enamelin, tufelin arnd amelin . Enamelin is also known 60. 'C' [Orban's 13th ed 51)
as "sheathelin". Enamel acts as a semipermeable membrane permitting
complete or partial passage of certain molecules like
51. 'C' [Orban's 11th ed 59/ 12th ed 50/ 13th ed 57] 14C-labeled urea, Iodine etc. and dyes.

52. 'B' [Orbans 11th ed 53/ 12th ed 47 / 13th ed 53] 61. 'D' [Orban's 13th ed 51]
Non amelogenins like Enamelin, t ufte lin and
53. 'A' [Orbans 11th ed 63/ 12th ed 54/ 13th ed 60] ameloblastin are acidic proteins.
Because of undisturbed and even development of the
enamel prior to birth, perikymata are absent in the 62. 'D' [Orbans 13th ed 61)
occlusal enamel of deciduous teeth, whereas they are Enamel lamellae are thin leaf like structures that
present in the postnatal cervica l parts. extend from the enamel surface towards the DEJ. They
may sometimes penetrate into dentin . They consist of
54. 'C' [Orbans 11th ed 62/ 12th ed 51/ 13th ed 58) organic materia l with very little mineral content.
Perikymata are believed to be the external
manifestations of the striae of retzius. Also check Synopsis Point 14 in chapter ENAMEL.

55. 'A' [Orban's 12th ed 46/ 13th ed 51] 63. ' B' [ Check Explanation of Q. No 24]
The organic material of enamel contains proteins
and lipids. Proteins are amelogenins (90%) and 64. 'B' [Orbans 13th ed 51)
non amelogenins (10% like enamelin, ameloblastin,
and tuftelin) . Enamel proteins do not contribute to Constitute 90% of enamel
structuring of enamel. This is in contrast to collagen, proteins. Low molecular weight,
which is the principal protein of bone and dentin. hydrophobic and are rich in
Amelogenins
praline, histidine, glutamine
56. 'B' [Orban's 12th ed 46/ 13th ed 53) and leucine.
Eg. Amelogenin
57. 'B' [Orban's 12th ed 50, 107, 180/ 13th ed 5, 194, Constitute 10% of enamel
206) prot eins. High molecular
Amount of inorganic content i.e. hydroxyapatite is weight proteins and are rich in
highest in enamel, therefore phosphate content is glycine,as.partic acid and serine
greater in enamel. Non-amelogenins
Eg. Ena melin (acidic)
• Highest fluoride content - cementum Tuftelin (acidic)
• Highest carbonate content - bone Ameloblastin (also called as
• Highest magnesium content - dentin amelin and sheathilin)
r ORAL HISTOLOGY

65. 'C' [Orbans 13th ed 86]

66. 'A' [Tencates 7th ed 187, 188]


Amelogenesis imperfecta can have different
inheritance patterns depending on the gone that is
altered.
• Mutations in the ENAM gene are the most frequent
known cause and are most commonly inherited in
an autosomal dominant pattern. This means one
copy of the altered gene in each cell is sufficient
to cause the disorders.
• The disorder can also occur in autosomal recessive
pattern from the mutations in the ENAM, MMP 20,
KLK 4, FAM 20A or SLC24A4 genes. This means two
copies of the gene in each cell is altered.
• About 5% cases are caused by mutations in the
AMELX gene and are inherited in an X-linked
pattern. In most cases, males with an X-linked
form of this condition experience more severe
dental abnorma lities than affected fema les.

67. 'A' [Orbans 13th ed 7]


Neural crest cells are unique population of cells that
develop from the ectoderm along the lateral margins
of the neural plate. These crest cells that migrate in
the trunk region form mostly neural, endocrine and
pigment cells. Those cells that migrate in the head and
neck contribute to skeletal and connective tissues like
cartilage, bone, dentin etc. The only one that is not
formed by crest cells is tooth enamel. Enamel forming
cells (ameloblasts) are derived from ectoderm lining
the oral cavity.

68. 'B' [Orbans 13th ed 85]


Refer Explanation of Q.No.22
Dental ;lut.,e

3. DENTIN
Dentinal tubules are c) Water irrigation d) None of the above
a) Concave b) Straight (AIIMS -97)
c) S-shaped d) None of the above 10. Tome's fibres are:
(MAN -94, KAR -98) a) With out but not with in tine dentin
2. The cells that form secondary dentin are b) Odontoblastic processes in dental tubules
a) Cementoblasts b) Fibroblasts c) Type of sharpey's fibres
c) Odontoblasts d) Osteoblasts d) Originate from tome's granular layer
(MAN -95) (AIPG -95)
3. Dead tracts in ground section appears as 11. The dentin receptors are unique because
a) Dentinal tubules a) They elicit pain to hot and cold
b) Coarse fibril bundles arranged at right angle to the b) They elicit pain to touch and pressure
dentinal surface c) They elicit pain to chemicals
c) Black in transmitted light and white in reflected d) Elicit only pain as a response
light (KAR -01)
d) White in transmitted light and dark in reflected 12. The main bulk of dentin is formed by
light a) Peritubular dentin b) Mantle dentin
(MAN -98) c) Inter tubular dentin d) Predentin
4. The organic lining of the calcified dentinal tubu Les (AP -03)
is known as 13. The zone of dentin most recently formed and
a) Lamina propria b) Lamina lucida un calcified is known as:
c) Lamina limitans d) La min a densa a) Mantle dentin b) Circumpulpal dentin
(MAN -2K) c) Predentin d) Secondary dentin
5. Which of the following dentinal layers is more (KAR -2 K)
mineralized 14. Inter globular dentin results due to:
a) Peritubular dentin b) Inter tubular dentin a) Failure of coalescence of calcospherites
c) Dentinal tubules d) Circum pulpal dentin b) Fracture of the dentin
(MAN -02, MHCET-2015) c) Artifact in light microscopy
6. The type of tertiary dentin which contains cellular d) Disturbance in dentinal tubules
inclusions is found in (KAR -2 K)
a) Sclerotic dentin b) Mantle dentin 15. Dentinogenesis by odontoblasts first begins at
c) Predentin d) Osteodentin a) Pulpal end b) Cusp tip
(MAN -01) c) Tooth bud stage d) Cervical area
7. The dentin adjacent to the dentinoenamel junction (AP -99, MAN -98)
is called the 16. Sclerotic dentin has following features:
a) Mantle dentin b) Circumpulpal dentin a) Caries susceptible b) Insensitive
c) Predentin d) Secondary dentin c) Hypersensitive d) Resistant to caries
(MAN -2K, COMEDK-15) (AIIMS -95)
8. Korffs fibres are seen in: 17. The microhardness of dentin is about:
a) Mantle dentin b) Secondary dentin a) 1/2 that of enamel b) 1/7 that of enamel
c) Reactionary dentin d) Indeed dermal tubules c) 1/4 that of enamel d) 1/5 that of enamel
(AIIMS -90) (KAR -2K)
9. Migration of odontoblasts into dentinal tubules 18. Shape of crown and the size of the roots is
occurs during _ __ determined by
a) Cavity preparation b) Dehydration a) Pulp b) Dentin

1) C 2) C 3) C 4) C 5) A 6) D 7) A 8) A 9) A 10) B 11) D 12) C 13) C


14) A 15) B 16) D 17) D 18) B
r ORAL HISTOLOGY

c) Enamel d) Cementum 26. 1, t formed dental tissue


a) Enamel b) Dentin
19. Physically and chemically, the dentin is closely c) Cementum d) Bone
related to (PGI -99)
a) Bone b) Acellular cementum 27. Dentinal tubules are calcified in
c) Enamel d) None of t he above a) Sclerotic dentin b) Primary dentin
c) Secondary dentin d) Interglobular dentin
20. Organic portion can be separated from the mineral (PGI -97)
by 28. Excess deposition of calcium in the dentinal tubules
a) Incineration/organic chelation is referred to as
b) Decalcification a) Sclerotic dentin b) Mantle dentin
c) Devitrification d) Combustion c) Interglobular dentin d) Intertubular dentin
(APPSC 99)
21. Formula of hydroxyapatite crystal is 29. Dentin formed through out the life is
a) Ca3 (Po4 ) 2 • CaCo3 b) 3Ca(PoJ 2 • Ca(OH) 2 a) Sclerotic dentin b) Secondary dentin
c) CaPo4 • Ca(OH) 2 d) None of the above c) Reparative dentin d) All of the above
(APPSC -99) (KAR -98)
22. The type of dentin that is formed prior to root 30. Regarding teeth which statement is wrong?
completion a) Enamel is harder than dentine
a) Inter tubular dentin b) Enamel has no cells
b) Peritubular dentin c) Ameloblast secrete enamel and dentine
c) Circum pulpal dentin d) Odontoblast produce dentine
d) Secondary dentin (KAR-04)
31. 'S' shape of dentinal tubule is due to:
23. Accentuated incremental lines in dentin due a) Incremental deposition
to disturbances in matrix and mineralization b) Peritubular dentin
process is c) Crowding of odontobLasts
a) Incremental lines of retzius d) Due to excess amount of organic component
b) Incremental lines of vonebner (AP-06)
c) Contour lines of owen 32. The formation of which of the following does not
d) None of the above represent normal physiological process of dentin
formation:
24. The pain transmission through dentin is a) Primary and secondary dentin
a) Direct neura l stimulation b) Secondary dentin and circumpulpal dentin
b) Fluid or hydrodynamic theory c) Tertiary dentin and sclerotic denti n
c) Transduction theory d) All of the above
d) All of the above (KAR- 04)
33. All of the following is true about dentinal tubules
25. Which of the following is correct except:
a) Reparative dentin is characterized by having fewer a) Inner pulpal layer cont ains more dentinal tubules
and more twisted tubules than normal dentin and than outer dentin layer
may include odontoblasts b) The diameter of dentin tubules ranges from 5 to 7µm
b) Dead tracts appear black in transmitted and white c) The diameter of dentinal tubule is more at pulpal
in reflected light layer than outer layer
c) Sclerotic/transparent dentin appears transparent or d) The bonding is more difficult on the inner layer of
light in transmitted light and dark in reflected light dentin compared to out er surface
d) All of the above (AIIMS-06)

19) A 20) A 21) B 22) C 23) C 24) D 25) D 26) B 27) A 28) A 29) B 30) C 31) C
32) C 33) B
Dental ;lut.,e

34. Fluid flow, in or out of the dentinal tubules is 41. During the formation of primary teeth
proportional to the tuble diameter by a) Enamel forms more cervically than dentin
a) Third power of the diameter b) Enamel forms faster than dentin
b) Sixt h power of the diameter c) Dentin forms faster t han enamel
c) Square of the diameter d) Processes of t heir cells remain in cont ent wit h each
d) Fourth power of the diameter other
(KCET- 09) (AP-10)
3 5. The average diameter of coronal dentinal tubules 42. Matrix vesicles
near the pulp is: a) Are found in mantle dentin
a) 0.2-0.5 microns b) 2-3 microns b) Contain collagen
c) 0.2-0.3 microns d) 7 microns c) Produce alkaline phosphatase
(AIPG- 05) d) Inhibit mineralization
36. In adult teeth, the type of dentin present underlying (COM ED K- 09)
dentino-enamel junction is 43. Transmission of stimulus across dentin is
a) Predentin b) Mantle dentin a) Direct activation of nerve endings
c) Interglobular dentin d) Tertiary dentin b) Correlated with the innervation density of the
(COMEDK-08) dentin
3 7. All factors governing dentine permeability EXCEPT c) Stimulus induced fluid flow
a) Smear layer d) There is no transmission of stimulus across dentin
b) Fluid Convection (AP-2012)
c) Length of dentin al tubules 44. During hydrodynamic theory movement of fluids in
d) Diffusion coefficient dentinal tubules cause:
(AIIMS-07) a) Movement of odontoblasts
3 8. The most accepted theory for dentin sensitivity is b) Touch the nerves of pulp to cause pain
or c) Minimum effect on odontoblast
Which of the following is the most accepted thoery d) Stimulation of odontoblastic process
of transmission of dental sensation (AIIMS -2012) (NEET-2013)
a) Transduction theory 45. Which of the areas of the tooth is most sensitive
b) Direct neura l sti mulation during cavity preparation?
c) Hydrodynamic theory a) Enamel (anterior teeth)
d) Hydrostatic theory b) Enamel (Posterior teeth)
(KCET-08) c) Dentin d) Dentinoenamel junction
39. Compared to the inter tubular dentin, peritubular (KAR-2013)
dentin is characterized by which of the following? 46. Matrix vesicles play important role in the
a) Greater stainability mineralization of
b) Lesser content of inorganic salts a) Dentin b) Ernamel
c) Greater content of inorganic salts c) Pulp d) Periodontal ligament
d) Greater content of large collagen fibers. (AP-14)
(AP- 09) 4 7. False statement regarding interglobular dentin is
40. One of the following statement is unique about a) The term interglobular dentin refers to organic
circumpulpal dentine matrix that remains unmineralised because the
a) Collagen fibers are oriented perpendicular to DEJ mineralizing globules fail to coalesce
b) Vonkorff fibers are present b) This occurs most often in the circumpulpal dentin
c) Contain phosphophoryn just below the mantle dentin
d) Requires matrix vesicle for mineralization c) Where the pattern of mineralization is likely to be
(COMEDK- 10) oppositional than globular

34} D 35} B 36} B 37} B 38} C 39} C 40} C 41} C 42} A 43} C 44} B 45) D 46} A
47} C
r ORAL HISTOLOGY

d) Large areas of interglobular dentin is a characteristic


feature of childhood hypophosphatasia
(AP-14)
48. Localized formation of dentin on the pulp-dentin
border formed in reaction to caries or restorative
procedures is known as
a) Tertiary dentin b) Secondary dentin
c) Primary dentin d) Interglobular dentin
(KERA LA-2015)

48) A
Dental ;lut.,e

3. DENTIN - ANSWERS
1. '(' [Orban's 11th ed 107/ 12th ed 85/ 13th ed 94] 5. 'A' [Orban's 11th ed 110/ 12th ed 86/ 13th ed 95]
Dentinal tubules resembles 'S' in shape. They The dentin that immediately
Peritubular
start at right angles from the pulpal surface and surrounds the dentina l tubules. It is
end perpendicular to the dentinoenamel and dentin highly mineralized.
dentinocemental junctions. The tubules are almost This is the main body of dentin.
Intertubular
st raight near root tip and along the incisal edges and It is located in between dentinal
dentin
cusps. tubules.
It is the recently formed
2. 'C' [Ten cate's 8th ed 174] unminera lized dentin and is located
Primary dentin represents the dentin formed prior Predentin adjacent to the pulp t issue. As the
to root completion. Secondary dentin represents calcification continues predentin
dentin formed after root completion and contains becomes dentin.
fewer tubules than primary dentin. Both are produced
by odontoblasts. Reparative (or) tertiary dentin 6. ' D' [Orban's 11th ed 125/ 12th ed 98/ 13th ed 46,
is produced in response to pathological effects of 108)
attrition, abrasion, and dental caries. Reparative dentin is characterized by few irregular
dentinal tubules. Sometimes it contains cellular
3. 'C' [Orban's 11th ed 125/ 121h ed 98/ 13th ed 108] inclusions with in the matrix and is known as
Dead tracts: Degeneration of odontoblastic process osteodentin . Osteodentin is seen in vitamin A
near narrow pulpal horns occurs and the tubule is deficiency and in the conditions where ameloblasts fail
filled wit h air. These appear dark in transmitted light to differentiate properly.
and white in reflected light (KCET-11). The term dead
tract was coined by Sir Wilfred Fish (KCET-08) . 7. 'A '[Orban's 11th ed 116/ 12th ed 90/ 13th ed 98,
100]
Sclerotic dentin: The tubule lumen is obliterated with Mantle dentin is the first formed dentin in the crown
mineral. The sclerotic (or) transparent dentin appears underlying the DEJ. It is thus the outer or more
transparent in transmitted light and dark in reflected periphera l part of dentin.
light. Sclerotic dentin is resistant to caries.
8. 'A' [Orban's 11th ed 130/ 12th ed 102/ 13th ed 100)
Dead tracts are t he initial step in the formation of
sclerotic dentin. 9. 'A' [Orban's 11th ed 134/ 12th ed 105/ 13th ed 115)
Air driven cutting instruments causes aspiration of
4. 'C' [Orban's 11th ed 110/ 12th ed 86/ 13th ed 9 6] odontoblasts into the dentina l tubules.

Organic lining of the dentinal


Lamina limitans 10. ' B' [Orban's 11th ed 113/ 12th ed 88/ 13th ed 98]
tubules
Projections of ameloblasts into
The connective tissue that Tomes process
Lamina propria the enamel matrix.
supports the epithelium
Odontoblastic processes within
Zones of basa l lamina between Tomes fibres
Lamina lucida & dentinal tubules.
the epithelial cells and
Lamina densa Granular layer in root caused by
connective tissue
the coalescing and looping of
Tomes layer
terminal portions of the dentinal
tubules adjacent to the cementum

11. ' D' [Orban's 11th ed 164/ 12th ed 126/ Ten cate 8th
ed 198, 200]
r ORAL HISTOLOGY

12. 'C' [Orban's 11th ed 110/ 12th ed 87/ 13th ed 96] 26. 'B' [Orban's 11th ed 106/ 12th ed 85/ 13th ed 93]

13. 'C' [Orban's 1tth ed 110/ 12th ed 87/ 13th ed 96] 27. 'A' [Orban's 1tth ed 125/ 12th ed 98/ 13th ed 108]

14. 'A' [Orban's 11th ed 117/ 12th ed 90/ 13th ed 101] 28. 'K [Orban's 11th ed 125/ 12th ed 98/ 13th ed 108]
Inter globular dentin are hypomineralised zones
caused by the failure of fusion of calcospherites. It is 29. 'B' [Orban's 1tth ed 116/ 12th ed 90/ 13th ed 100]
a defect of mineralization.
30. 'C' [Orban's 1tth ed 86/ 12th ed 68/ 13th ed 2, 35]
15. 'B' [Orban's 11th ed 126/ 12th ed 98/ 13th ed 110]
31. 'C' [Tencate 5th ed 145, 152/ 8th ed 177]
16. 'D' [Orban's 11th ed 126/ 12th ed 98/ 13th ed 108] The dentin al tubules follow an S-sha ped path from the
outer surface of the dentin to the perimeter of the
17. 'D' [Sturdevant's 4th ed 26/ 5th ed 27] pulp in coronal dentin. These primary curvatures result
The over-all average hardness value for enamel from the crowding of odontoblasts. The secondary
and dentin for all teeth was 343 ±23 and 68 ±3, curvatures are smaller oscillations within the primary
respectively. The hardness of dentin averages one fifth curvatures, but how they are produced has not
that of enamel, and its hardness near the DEJ is about established.
three times greater than near the pulp.
32. 'C' [Orban's 12th ed 90, 98/ 13th ed 100, 108]
18. 'B' [Orban's 11th ed 106/ 12th ed 85/ 13th ed 93] Tertiary and sclerotic dentins are reparative, response
or reactive in nature. These are formed in reaction to
19. 'A' [Orban's 11th ed 106/ 12th ed 85/ 13th ed 93] trauma such as caries, attrition, abrasion, erosion or
cavity preparation. Formation of tertiary and sclerotic
20. 'A' [Orban's 1tth ed 107/ 12th ed 85/ 13th ed 94] dentin is protective in nature.
Inorganic constituents can be removed by decalcification
and organic constituents are removed from the mineral 33. 'B' [Orban's 11th ed 108/ 121 h ed 85/ 13th ed 94-95]
by "incineration" or "organic chelation". The diameter of dentinal is more at pulpal layer (3 to
4µ m) and smaller attheir outer ends (1 µ m). An increase
21. 'B' [Orban's 11th ed 107/ 12th ed 85/ 13th ed 94] in number of tubules with depth and, consequently,
The other formula of Hydroxylapatite, also called increase in dentin wetness, make bonding to deeper
hydroxyapatite (HA), is Ca5(P04)3(0H), but is usually dentin more difficult than to superficial dentin
written Ca10(P04)6(0H)2 to denote that the crystal
unit cell comprises two entities. 34. 'D' [Sturdevant 4th ed 17 2/ 5th ed 17 6]
Fluid flow a. d' x 2n x t.P
22. 'C' [Orban's 11th ed 116/ 12th ed 90/ 13th ed 100] nl
d diameter of tubule
23. 'C' [Orban's 11 1h ed 117/ 12th ed 90/ 13th ed 101]
DP pulpal pressure difference
Incremental lines in dentin are called "Imbrication
n viscosity
lines" or "incremental lines of Von Ebner". They
L tubule length
reflect the daily rhythmic deposition of dentin matrix.

35. 'B' [Orban's 11th ed 108/ 121 h ed 85/ 13th ed 94-95]


Accentuated incremental lines because of disturbances
in matrix and min eralization are known as "contour
36. 'B' [Orban's 11th ed 116/ 12th ed 90/ 13th ed 96]
lines of Owen".

37. ' B' [Sturdevant 4th ed 240-243/ 5th ed 248-252]


24. 'D' [Orban's 11th ed 123/ 12th ed 97 / 13th ed 104]
Factors affecting dentin permeability are
25. 'D' [Orban's 11 1h ed 124-25/ 12th ed 97-98/ 13th ed • Vasoconstrictor which reduces fluid flow in tubules
107-108] • Radius and length of tubules
Dental ;lut.,e

• Viscosity of dentin fluid 44. ' B' [Orban's 13th ed 104]


• Pressure gradient
• Molecular size of substance dissolved in tubular 45• 'D' [Journal of Indian dental Association Jan, Feb
1967 Pg 27/ Ten cate's 8th ed 200)
fluid
Teet h are high ly sensitive at DEJ, followed by the deep
• Rate of removal of substance by the blood vessels
layers of dentin near the pulp, on the cementum and
in pulp affect permeability
most sensitive of all, of course in the pulp.

38. 'C' [Orban's 1tth ed 123/ 121h ed 96/ 13th ed 104-06) 46. 'A' [Orbans 13th ed 112,217,219)
Matrix vesicles are small membrane bound structures
39. 'C' [Orban's 12th ed 85/ 13th ed 95) which are 25-250nm in diameter, lying free in the
Peritubular dentin is about 9% more mineralized than matrix, where calcification is going on. These are
inter tubular dentin outgrowths of cell membrane that bud from osteoblasts,
odontoblasts and chondrocytes. The vesicles are rich
40. 'C' [Orban's 12th ed 89, 90/ 13th ed 98, 100] in phospholipids that have high affinity for calcium
Dentine contains a unique phosphoprotein, called ions. They also contain Annexins which incorporates
phosphophoryn, is of particular interest. During Ca+2 ions.
dentin formation, phosphophoryn (PP) initiates
mineral deposition in localized regions by matrix- The function of matrix vesicles are
mediated mineralization mechanism . It is a highly a) To accumulate calcium ions
phosphorylated protein unique to circumpulpal dentin, b) Their membranes furn ish binding sites for the
being absent from predentin and mantle dentin and nucleation of hydroxyapatite crystals
bone. c) Involved in minera lization of mantle dentin and
woven bone.
Option A, Band Dare features of mantle dentin.
4 7. 'C' [ Check Synopsis]
41. 'C' [Orban's 12th ed 102/ 13th ed 57, 112]
48. 'A' [Refer Explanation of Q.No.2]
42. 'A' [Orban's 12th ed 90/ 13th ed 100)

43. 'C' [Orban's 13th ed 104]


Various theories of pain transmission of pain in
dentin are:
• Direct neural stimulation: Direct nerves in dentin
get stimulated. This theory is not accepted
• Transduction theory: Odontoblast process is
excited by stimulus and the impulse is transmitted
to the nerve endings in the inner dentin.
• Hydrodynamic theory: Various stimuli affect
fluid movement in the dentinal tubules thereby
stimulating pain by mechanical disturbance of
the nerves closely associated with odontoblast.
The most acceptable theory of pain transmission
through dentin is hydrodynamic theory.

NOTE:
• Cold stimuli cause inward movement of fluid.
• Dry or dessication causes outward movement of
fluid.
r ORAL HISTOLOGY

4. PULP
1. Which of the following structures is not found in a c) Cell regenerated zone
living pulp? d) Cell free zone
a) Collagen fibres b) Haversian fibres (AP -03, 99)
c) Non-myelinated nerve fibres 10. The cells of the dental pulrp which are responsible
d) Reticulum for the deposition of reparative dentin are:
(MAN -94, AIIMS -93) a) Osteoblasts b) Odo ntoblasts
2. The dental pulp contains nerve endings/ receptors c) Ameloblasts d) Osteocytes
for (AIPG -99)
a) Pain b) Pressure 11. Histologically the dental pulp most closely
c) Proprioception d) Temperatu re resembles :
(MAN -94) a) Nerve tissue b) Vascular tissue
3. The dental pulp is derived form c) Granulation tissue d) Loose connective tissue
a) Dental papilla b) Dental sac (PGI -2K, KAR -98)
c) Odontoblasts d) Stellate reticulum 12. Pericytes are found:
(MAN -94) a) In thymus
4. Pulp responds to all stimuli by pain because it has b) Around capillaries of pulp
only c) In gallbladder
a) Free nerve endings b) Krause end bulbs d) Along with heparin on surface of mast cells
c) Pacinian corpuscle d) Ruffini corpuscle (AIIMS -94)
(COMEDK- 09) 13. The odontoblasts killed during cavity preparation
5. All of the following are true about functions of are derived from:
dental pulp except: a) Unaffected odontoblasts from the pulp
a) Formation of dentin b) Nerve supply to dentin b) Undifferentiated cells
c) Sends impulses to CNS c) Histocytes d) Osteoblasts
d) Nerve supply to enamel through fibres (AIIMS -99)
(AP -04, AIIMS -95) 14. Proprioceptors are seen in all except:
6. Odontoblasts are derived from a) Skeletal muscle b) TMJ
a) Undifferentiated mesenchymal cells c) Pulp d) None of the above
b) Histocytes (AIIMS -95)
c) Macrophages d) Lymphocytes 15. Cells occurring in greatest number in pulp are
(AIIMS -97, 91) a) Cementoblasts b) Fibroblasts
7. The primary fun ction of the dental pulp is: c) Osteoblasts d) Ameloblasts
a) Nutritive b) Production of dentin (COMEDK-08,COM ED-14)
c) Production of enamel 16. Pulp tissue contains all except:
d) Vascular supply to the teeth a) Fibroblast b) Capillary
(KAR -98, AIIMS - 2001) c) Collagen d) Cell rests of malassez
8. The radicular pulp is continuous with tissues of the (AIPG -93)
periapical area via the: 17. The cell free zone in pulp lies:
a) Pulp canal b) Pulp chamber a) Above odontoblast b) Below odontoblast
c) Mental foramen d) Apical foramen c) In radicular pulp d) In pulp horn
(AIPG -98, 03) (KAR -01)
9. Weil's zone of pulp is 18. Aging process affects pulp tissue by
a) Cell degenerated zone a) Decreasing the size and shape of pulp tissue and
b) Cell rich zone cellular component

1) B 2) A 3) A 4) A 5) D 6) A 7) B 8) D 9) D 10) B 11) D 12) B 13) B


14) C 15) B 16) D 17) B 18) A
Dental ;lut.,e

b) Increases cellular component at the expenses of 27. Which of the following is not correct
fibre component a) Free denticles are entirely surrounded by pulp
c) Increases vascularity of the pulp tissue(free of dentin)
d) Decreases the ca lcific component of the pulp b) Embedded denticles are entirely surrounded by
(AP -98, PGI -97) dentin(embedded in dentin)
19. The number of pulp organs in a person is c) Attached denticles are partly fused with dentin
a) 32 b) 20 d) False denticles contains many dentinal tubules
c) 52 d) 62
28. The development of pulp begins
20. The total volumes of all permanent pulp organ a) 6th week of IU b) 3th week of IU
a) 0.38 cc b) 0.58 cc c) 10th week of IU d) 181h week of IU
c) 0.02 cc d) 3.8 cc
29. Size of pulp chamber
21. The size of apical foramen of maxillary teeth in a) Increases with age b) Decreases with age
adult is c) Increased in some cases and decreases in some
a) 0.4 mm b) 0.3 mm cases
c) 0.2 mm d) 0.7 mm d) None of the above

22. The 2"d most prominent cells in pulp are 30. Protein secreted by odontoblast
a) Fibroblasts b) Defense cells a) Chitin b) Keratin
c) Odontoblasts c) Collagen d) Elastin
d) Histocytes or resting wandering cell (AP -04)
31. The primary response of the pulp to tissue
23. The fibroblasts associated with capillaries are destruction is
a) Plasma cell b) Histocytes a) Necrosis
c) Lymphocytes d) Pericytes b) Ulceration
c) Calcific degeneration
24. Which of the following is true d) Inflammation
a) Pulpal pressure is highest among body tissues (APPSC -99)
b) Majority of nerve that enter the pulp are non 32. Unmyelinated nerve fibres of pulp are
myelinated and are sympathetic in nature a) Sensory in nature b) Respond to hot and cold
c) Plexus of Rashkow or parietal layers of nerves is c) Carry pain sensation
formed by network of peripheral axons located d) Associated with blood vessels and a re sympathetic
adjacent to cell rich zone in nature
d) All of the above
33. Vitality of pulp depends on
2 5. The average life time of primary pulp in oral cavity a) Blood supply b) Nerve supply
is c) Lymphatic supply d) All of the above
a) 8.3 years b) 5 years
c) 7 years d) 2 112 years 34. Dystrophic calcification is seen most commonly in
which of the following oral tissues
26. Regressive changes in pulp include a) Enamel b) Pl.llp
a) Increased fibrotic component c) Dentin d) Periodontal ligament
b) Decreased cellular component (AIIMS -99)
c) Calcification in blood vessels near apical foramen 35. The plexus of Rashkov is present in which of the
d) All the above following zones in pulp?
a) Cell free zone b) Cell rich zone

19} C 20} A 21} A 22} C 23} D 24) D 25} A 26} D 27) D 28} B 29} B 30} C 31} D
32) D 33} A 34} B 35} A
r ORAL HISTOLOGY

c) Odontoblastic zone d) Pulp core


(COM EDK-05)
36. The diameter of the largest arterial vessels in the
pulp range from:
a) 50-100 µm b) 101-150 µm
c) 151-200 µm d) 201-250 µm
(COM EDK- 07)
37. Pain originate in the pulp due to
a) free nerve ending in cell rich zone
b) free nerve ending at sub odontoblastic layer
c) Krause bulb at the center of the pulp
d) Meisseners corpuscles at cell rich zone
(B I IU-07)
38. The pulp stem cells are
a) Pluripotent in nature
b) Totipotent in nature
c) Multipotent in nature
d) None of the above
(MCET-14)
39. The connective tissue cells of pulp are derived from
a) The pool of undifferentiated mesenchymal cells
b) The pool of fibroblasts
c) The pool of undifferentiated ectodermal cells
d) The pool of osteoblasts
(COMEDK-2015)
40. The normal immunocompetent residents of dental
pulp are
a) Fibroblasts b) Odontoblasts
c) Lymphocytes d) Fibrocytes
(COMEOK-2015)

36) A 37) B 38) A 39) A 40) C


Dental ;lut.,e

4. PU LP - ANSWERS
1. 'B' [Orban's 11th ed 145/ 12th ed 112-13/ 13th ed The network of nerves located adjacent to the cell
124, 135, 209] rich zone is called the parietal layer (or) t he plexus of
Rashkow.
2. 'A' [Orban's 11th ed 164/ 12th ed 126/ 13th ed 137-
138] 10. ' B' [Orban's 11th ed 148/ 12th ed 116/ 13th ed 127,
Pulp lacks the receptors of heat, touch, pressure or 138/ Ten cate's 8th ed 175, 190]
chemicals and proprioception. Any stimulus will elicit
response in the form of pain on ly. 11. 'D' [Orban's 12th ed 118/ 13th ed 121]

3. 'A' [Orban's 11th ed 36/ 12th ed 26/ 13th ed 30] 12. 'B' [Orban's 11th ed 157/ 12th ed 120/ 13th ed 133]
Dentin, pulp are derived from dental papilla and
cementum, periodontal ligament are derived from 13. ' B' [Orban's 11th ed 148/ 12th ed 116/ 13th ed 125,
dental sac. 127]

4. 'A' [Orban's 12th ed 124/ 13th ed 135] 14. 'C' [Orban's 13th ed 137-138]
Proprioceptors are the receptors, which detect changes
5. 'D' [Orban's 12th ed 126-27/ 13th ed 139] in the tone or contraction of a skeletal muscle.
Functions Of Pulp Are
• Inductive:- I nduce oral epithelial differentiation 15. 'B' [Orbans 11th ed 145/ 12th ed 113/ 13th ed 124]
into dental and enamel organ formation. Fibroblasts are the most numerous cell type in the
pulp. Odontoblasts are second most prominent cells in
• Formative:- pulp organ produces dentin
the pulp.
• Nutritive:- t hrough its blood supply nourishes
dentin through odontoblastic process.
16. ' D' [Orban's 11th ed 145/ 12th ed 113/ 13th ed 124,
• Protective:- sensory nerves respond with pain to 130]
all stimuli
• Defensive and reparative:- through its defense cells 17. 'B' [Orban's ttth ed 145/ 12th ed 113/ 13th ed 122]
and by producing reparative and sclerosis of dentin.
18. 'A' [Orban's 11th ed 167/ 12th, ed 129/ 13th ed 140]
6. 'A' [Orban's 11th ed 148/ 12th ed 116/ 13th ed 125] Aging process affects the pulp tissue by decreasing
Undifferentiated mesenchymal cells are called the cellular component and increasing the fibrous
totipotent cells. They transform into fibroblasts, component.
odontoblasts or macrophages when the need arises.
These are the primary cells in the very young pulp. 19. 'C' [Orban's 11th ed 139/ 12th ed 109/ 13th ed 120]
Normally there are 52 pulp organs, 32 in the permanent
7. 'B '[Orban's 11th ed 164/ 12th ed 126/ 13th ed 139] and 20 in primary teeth.

8. 'D' [Orban's ttth ed 141/ 12th ed 110/ 13th ed 121] 20. 'A' [Orban's 11th ed 139/ 12th, ed 109/ 13th ed 120]
The total volume of the permanent teeth pulp organs
9. 'D' [Orban's 11th ed 145/ 12th ed 113/ 13th ed 122] is 0.38 cc, and the mean volume of single adult human
The central region of both the coronal and the radicular pulp is 0.02 cc.
pulp contains large nerve trunks and blood vessels.
21. 'A' [Orban's 11th ed 141/ 12th ed 110/ 13th ed 121]
Peripherally it is composed of odontoblasts, cell free The size of apical foramen in maxillary teeth is 0.4mm.
zone (WEIL'S zone) and the cell rich zone. The size of apical foramen in mandibular teeth is
slightly sma ller, being 0.3 mm in diameter.
r ORAL HISTOLOGY

22. 'C' [Orban's 11th ed 148/ 12th ed 116/ 13th ed 125] 32. 'D' [Orban's 11th ed 161/ 12th ed 124/ 13th ed 135]
Large myelinated fibres mediate the sensation of pain.
23. 'D' [Orban's ttth ed 157/ 12th ed 120/ 13th ed 133] Unmyelinated fibres are associated with blood vessels
of pulp and are sympathetic in nature. They are the
24. 'D' [Orban's 12th ed 119-20/ 13th ed 131, 135] majority of nerve fibres ent ering the pulp.

25. 'A' [Orban's ttth ed 166/ 12th ed 128/ 13th ed 139] 33. 'I{[Orban's 1tth ed 177/ 13th ed 148]
Vitality of the pulp depends on its blood supply. Electric
26. 'D' [Orban's t2th ed 128/ 13th ed 140-41] and thermal pulp testing provide information about
the status of the nerves ano! so check t he sensitivity
27. 'D' [Orban's 11th ed 170/ 12th ed 128/ 13th ed 141] of pulp and not its vitality.
Pul~ stones (or) denticles are classified as:
• Free, attached and embedded. 34 • 'B' (Orban's ll'h ed 171/ 12'h ed 123/ 13th ed 142)
• True, false and diffuse calcifications.
35. [Tencates 5th ed 185 - 188/ 8th ed 197]
'/I{
True Contains dentinal tubules. Developed Sub odontoblastic plexus of Raschkow is present in
denticles from remnants of root sheath. cell free zone of weil.
Does not contain dentinal tubules.
False
Appears as concentric layers of 36. '/!{ [Orbans t11 h ed 156/ 12'h ed 118/ 13th ed 131)
denticles
calcified tissue
I rreg ula r calcific deposits following 37. 'B' [Tencate's 5th ed 185 - 188/ 8th ed 197)
fibre bundles (or) blood vessels. The dental pulp is richly innervated (the mean number
Diffuse
Diffuse calcifications are found in of fibers entering premolars is over 900). Nerves enter
calcifications
the root canal and denticles are seen the pulp through the apical foramen and each nerve
more frequently in the coronal pulp. fiber is estimated to produce atleast eight terminal
branches, which ultimately contribute to an extensive
28. 'B' [Orban's 11th ed 173/ 12th ed 131/ 13th ed 142) plexus of nerves in the cell free zone just below the
cell bodies of the odontobla.sts. This plexus of nerves
29. 'B' [Orban's ttth ed 175/ 12th ed 132/ 13th ed 140] are called the sub odontoblastic plexus of Raschkow
and it occupies the cell free zone of weil. Most of the
30. 'C' [Orban's tt' h ed 151/ 12th ed 115/ 13th ed 126] nerve bundles terminate in the Raschkow plexus as
Option 'B' keratin is the protein that is present in free, unmyelinated nerve endings, a small number of
enamel (BHU-07). axons pass between the odontoblast cell bodies to
enter the dentinal tubules.
31. 'D' [Orban's 12th ed 134/ 13th ed 143)

38. 'A' [Check Explanation Below]


The Dental Pulp Stem Cells (DPSC) are pluripotent in nature.
• Totipotent stem cells can differentiate into embryonic and extraembryonic cell types. Such cells
can construct a complete, viable organism.
Toti potent
• These cells are produced from the fusion of an egg and sperm cell. Cells produced by the first
few divisions of the fertilized egg are also totipotent.
• These cells are like totipotent stem cells in that they can give rise to all tissue types. Unlike
totipotent stem cells, however, they cannot give rise to an entire organism.
Pluripotent • Pluripotent cell has the potential to differentiate into any of the three germ layers
Eg :- Dental pulp stem cells
Embyronic stem cells
Dental ;lut.,e

• Progenitor cells have the potential to differentiate into multiple, but limited cell types .
• For example, a hematopoietic cell can give rise to several types of blood cells (i.e. red blood cells,
Multi potent white blood cells, platelets, etc.) but not into brain cells or other types of cells.
Eg: Hemopoietic cell,
Mesenchyma l stromal cells,
Bone marrow cells
• Oligopotency is the ability of progenitor cells to differentiate into a few cell types. It is a degree
of potency.
• Examples of oligopotent stem cells are the lymphoid or myeloid stem cells.
Oligopotent • A lymphoid cell specifically, can give rise to various blood cells such as B and T cells, however,
not to a different blood cell type like a red blood cell
Eg :- Lymphoid stem cells,
Myeloid stem cells
• In cell biology, a uni potent cell is the concept that one stem cell has the capacity to differentiate
into only one cell type.
E.g:- spermatogenic stem cells.
Uni potent
• Hepatoblasts, which differentiate into hepatocytes (which constitute most of the liver) or
cholangiocytes (epithelial cells of the bile duct), are bipotent. A close synonym for uni potent cell
is precursor cell.

39. 'A' [Orbans 13th ed 125]


Refer explanation of Q.No.6

40. 'C' [Orbans 13th ed 128]


The immunocompetent cells of pulp are macrophages, dendritic cells, lymphocytes, most cells and plasma cells.
r ORAL HISTOLOGY

5. CEMENTUM
1. Cementum 9. Acellular cementum is typically found in:
a) Is derived from the sheat h of hertwig a) Coronal half of root b) Apical half of root
b) Is acellular in the apical third of the tooth c) Apex of root d) Entire root
c) Often overlaps the enamel (PGI -98, AIPG -97)
d) Does not contain collagen fibres 10. Cementum that forms in conjunction with root end
(MAN -94) formation and eruption
2. Cellular cementum and compact bone contain a) Cellular cementum b) Acellular cementum
a) Sharpey's fibres and elastic fibres c) Both of the above d) None of the above
b) Collagen fibres and blood vessels (AP -99)
c) Canaliculi and incremental lines 11. Which of the following is true
d) Lacunae and elastic fibres a) The uncalcified matrix of cementum is called
(MAN -97) cementoid
3. Butt joint in cementoenamel junctions seen is b) Cementum contains type I collagen predominantly
% of cases c) Cementodentinal junction is sometimes scalloped
a) 10% b) 30% in deciduous teeth and smooth in permanent teeth
c) 60% d) 90% d) All of the above
(MAN -02)
4. Secondary cementum is generally confined to: 12. Intermediate cemental layer
a) Cervical third of the root a) Does not exhibit features of either dentin or
b) Middle third of the root cementum
c) Apical third of the root b) Predominantly seen in apical two thirds of root s
d) No secondary cementum forms of molars and premolars and is rare in incisors or
(KAR -03) deciduous teeth
5. Inorganic content of cementum is: c) Represents area of entrapment of hertwig's sheath
a) Less than bone b) More than bone in dentin or cementum
c) Equal to that of bone d) All of the above are correct
d) More than that of enamel
(KAR -2K) 13. If cementum is not formed
6. Maximum fluoride content is seen in a) Ankylosis may occur b) Exfoliation of teeth occur
a) Dental cementum b) Dental pulp c) Delayed eruption d) No change occurs
c) Enamel d) Dentin
(KAR -99) 14. If overgrowth of cementum does not improve the
7. At the CEJ , which of the following occurs functional qualities of cementum, it is termed as
a) Cementum meets enamel a) Hyperplasia b) Hypoplasia
b) Cementum overlaps enamel c) Hypertrophy d) Excementoses
c) Cementum does not meet enamel
d) All of the above 15. Chronic periapical inflammation is usually
(KAR -01) associated with
8. Which of the following is a rare occurrence a) Ankylosis b) Resorption of cementum
a) Enamel meet cementum at a point c) Hypertrophy of cementum
b) Enamel overlaps cementum d) Hyperplasia of cementum
c) Cementum overlaps enamel
d) Both never meet 16. Cementum is
(PGI - 02) a) Avascular b) Vascular

1) C 2) C 3) B 4) C 5) A 6) A 7) D 8) B 9) A 10) A 11) D 12) D 13) B


14) A 15) D 16) A
Dental ;lut.,e

c) Has blood supply initially only 26. Which dental tissue doesn't contain nerves?
d) None of the above a) Bone b) Dentin
c) Cementum d) PU!lp
17. Which of the following is correct (AP-05)
a) Cementum is more resistant to resorption than bone 27. Inorganic content of cementum is
b) Bone is more resistant to resorption than cementum a) 45 to 50% b) 50 to 55%
c) Both are equally resorbable c) 55 to 60% d) 60 to 65%
d) None of the above (COM ED K-08)
28. Cellular cementum is
18. At the CEJ cementum overlaps enamel about a) First formed cementum
a) 55 to 60% b) 60 to 65% b) Less calcified than acellular type
c) 65 to 70% d) 70 to 75% c) More calcified than acellular type
(UPSC -01) d) Less irregular
19. Intermediate cementum is (COMEDK- 10)
a) Between enamel and cementum 29. Intermediate cementum is
b) Found at dentinocemental junction a) Highly defined cellular zone near cementa- dentinal
c) Apical third d) Coronal third junction (CDJ)
(KAR -97) b) Highly defined fibrillar zone near CDJ
20. Trauma or excessive occlusal forces causes_ __ c) Poorly defined zone near cemento-dentinaljunction
of cementum d) both a and b
a) Hyperplasia b) Hypertrophy (KCET-10)
c) Resorption d) None of t he above 30. Bundle bone is part of
a) Periodontal ligament b) Alveolar bone
21. Reestablishing the former outline of root surface is c) Cementum d) Longbones
a) Anatomic repair b) Functional repair (COMEDK-09)
c) Physiologic repair d) Hyperplasia 31. The cementoenamel junction is approximately at
the level at which the root canal becomes
22. Root lengthenfog is contributed by a) Twisted b) Curved
a) Cellular cementum b) Acellular cementum c) Pulp chamber d) Lateral canal
c) Intermediate cementum (BHU-2012)
d) All of the above 32. Hardness of fully mineralized cementum is
a) Less than dentin b) More than dentin
23. Which of the following statement is true c) Equal dentin d) None of the above
a)
Cementum is thinnest at CEJ (20-50µm) (COMEDK -2013)
b)
Cementum is thickest towards the apex (150-200µm) 33. Which of the following is true of cellular cementum?
The apical foramen is surrounded by cementum
c) a) Forms during root formation
All of the above
d) b) Forms after the eruption of the tooth
c) Seen at the coronal portion of the tooth
24. Which of the following is least mineralized? d) Formation is a slow process
a) Cementoid b) Incremental line (KAR-2013)
c) Cellular cementum d) Acellular cementum 34. Type of Cementum located in the furcation areas of
(AIPG-07) multi-rooted teeth is
25. The region in which cementum formation is most a) Cellular Extrinsic fiber cementum
rapid is: b) Cellular Intrinsic fiber cementum
a) Middle b) Coronal c) Cellular Mixed Stratified cementum
c) Apical d) Interradicular d) Acellular Intrinsic fiber cementum
(KAR-04) (COMED-14)

17) A 18) B 19) B 20) C 21) A 22) A 23) D 24) A 25) C 26) C 27) A 28) B 29) C
30) B 31) C 32) A 33) B 34) C
r ORAL HISTOLOGY

35. The premature loss of Reduced Enamel Epithelium


(REE) permits connective tissue to come in direct
contact with enamel thus depositing a thin layer of
cementum on the cervical enamel. This cementum
is of which type?
a) Cellular mixed fiber cementum
b) Cellular mixed stratified cementum
c) A cellular extrinsic fibercementium
d) Acellularafibrillar cementum
(MHCET-2015)
36. Which of the following is Cemento-dentinal
junction?
a) Most important anatomical landmark for
determining the location of pulp chambers and
root canal orifices
b) Point where pulp tissue ends and periodontal tissue
begins
c) Space between major and minor foramina
d) Rounded edge that differentiates the termination
of the cementa l canal from the exterior surface of
root.
(APPG-2015)

35) D 36) B
Dental ;lut.,e

5. CEMENTUM - ANSWERS
1. '(' [Orban's ttth ed 193/ 12th ed 147/ 13th ed 163] 14. 'A' [Orban's 11th ed 195/ 12th ed 149/ 13th ed 164]
In about 60% teeth, cementum overlaps the cervical If the cementa[ overgrowt h improves the functional
end of enamel. In 30% teeth, cementu m meets enamel quality of cementum it is termed as cementa[
in a sharp line and in about 10% teeth, enamel and hypertrophy.
cementum do not meet.
15. 'D' [Orban's 11th ed 196/ 12th ed 149/ 13th ed 164]
2. 'C' [Orban's 11th ed 187/ 12th ed 144/ 13th ed 153, Extensive hyperplasia of cementum surrounding
158] the root like a cuff is seen in chronic periapical
Cementum is avascUJlar. Cells in cellular cementum are inflammation.
cementocytes, which are similar to osteocytes. They
lie in the spaces designated as lacunae. Cementum 16. 'A' [Orban's 11th ed 198/ 12th ed 150/ 13th ed 166]
contains only collagen fibers. Cementum contains only
collagen fibres. 17. 'A' [Orban's ttth ed 197/ 12th ed 150/ 13th ed 166]
Cementum is more resistant to resorption than bone,
3. 'B' [Orban's 11th ed 192/ 12th ed 147/ 13th ed 162- and it is for this reason that orthodontic tooth
163] movement is possible. Cementum is avascular while
bone is richly vascularised. Thus degenerative processes
4. 'C' [Orban's 11th ed 184/ 12th ed 141/ 13th ed 157] are much more easily affected by interference with
Acellular or primary cementum covers the cervical or blood circulation in bone.
half of the root while cellular cementum or secondary
cementum is present in the apical third of root and 18. ' B' [Orban's 11th ed 193/ 12th ed 14 7/ 13th ed 163]
represents cementum formed after the tooth reaches
the occlusal plane. Cellular cementum contributes to 19. 'B' [Orban's 11th ed 192/ 12th ed 14 7/ 13th ed 162]
lengthening of the root.
20. ' C' [Orban's 11th ed 198/ 12th ed 149/ 13th ed 166]
5. 'A' [Tencate 8th ed 95, 205]
21. 'A' [Orban's 11th ed 198/ 12th ed 149/ 13th ed 166/
6. 'A' [Orban's ttth ed 180/ 12th ed 137/ 13th ed 152] Tencate 8th ed 218]
In most cases, repair of cementa[ resorption occurs by
7. 'D' [Orban's 11th ed 192 -93/ 12th ed 147/ 13th ed formation of acellular or cellular cementum and the
162-163] former outline of the root surface is reestablished. It
is called anatomic repair.
8. 'B' [Orban's 11th ed 192/ 12th ed 147/ 13th ed 163]
Enamel overlapping cementum is a very rare occurance In functional repair, the cementum is deposited
in a thin layer and a bay like recess remains. The
9. 'A' [Orban's 11th ed 184/ 12th ed 141/ 13th ed 160] periodontal space is restored to its norma l width by
formation of bony projection.
10. 'A' [Orban's 11th ed 188/ 12th ed 145/ 13th ed 160]
22. 'A' [Orban's 11th ed 188/ 12th ed 145/ 13th ed 160]
11. 'D' [Orban's 12th ed 138/ 13th ed 152, 155, 160]
23. ' D' [Orban's 11th ed 184/ 12th ed 141/ Tencate 8th
12. 'D' [Orban's 11th ed 192/ 12th ed 147/ 13th ed 162] ed 218]

13. 'B' [Orban's 11th ed 194/ 12th ed 148/ 13th ed 163] 24. 'A' [Orban's 12th ed 138-39/ 13th ed 155]
Hypophosphatasia 1s a rare hereditary disease
characterized by the absence of cementum, resulting
in loosening and premature loss of deciduous teeth.
r ORAL HISTOLOGY

25. 'C' [Carranza clinical periodontology 11th ed 35]


Cementum formation is most rapid in the apical
regions, where it compensates for tooth eruption,
which itself compensates for attrition .

26. 'C' [Tencate 51hed 287, Tab. 13-3/ Orban's 13th ed 2]

27. 'A' [Orban's 111h ed 180/ 12th ed 137/ 13th ed 151]

28. 'B' [Orban's 121h ed 142/ 13th ed 153]

29. 'C' [Orban's 12th ed 145/ 13th ed 162]


Sometimes dentin is separated from cementum by a
zone called 'Intermediate cementum layer', which
doesnot exhibit features of either dentin or cementum.
It is dentinal in origin. It contains no tubules.
Sometimes it is continuous and sometimes found in
isolated areas.

30. 'B' [Orban's 11th ed 250/ 13th ed 225]


Ref. Q.No. 5 for explanation

31. 'C' [Orban's 13th ed 163/ 13th ed 163]

32. 'P.: [Orban's 13th ed 151/ 13th ed 151]

33. 'B' [Orban's 13th ed 153/ 13th ed 153]


Cellular cementum forms after the eruption of tooth
and in response to functional demand. It is seen at
the apical portion of the tooth. Acellular cementum
forms during root formation and is seen at the
coronal portion of the root. Compared to the cellullar
cementum, the incremental lines of salter are closer in
acellular cementum.

34. 'C' [Refer Synopsis]

35. 'D' [Carranza 10th ed 75]

36. 'B' [Ingle 6th ed 1517]


Cementodentinal junction is a histologic structure
where cementum and dentin meets near the root apex.
It is also called as minor apical diameter which ends
0.5-1 mm short of the apical fora men .
Dental ;lut.,e

6. PERIODONTAL LIGAMENT
Which of the following are the predominant 9. The attachment apparatus of tooth is composed of:
connective tissue cells of the periodontal ligament a) Gingiva, cementum and alveolar bone
a) Cementoblasts b) Fibroblasts b) Gingiva and cementum
c) Osteoblasts d) Cell rests of malassez c) Cementum and periodontal ligament
(MAN -94, PGI -95) d) Periodontal ligament, the cementum and the alveolar
2. The periodontal ligament bone
a) Derives its blood supply primarily from the branches (KAR -97)
of vessels entering the pulp 10. The vascular supply of the periodontal ligament is:
b) Has a slow rate of turn over a) Greatest in the middle-third of a single rooted tooth
c) Contains epithelial cells b) Greatest in the middle-third of a multirooted tooth
d) Comprises primarily of type II collagen c) A net-like plexus that runs closer to the cementum
(MAN -94) than to the bone
3. Which of the following is not the principle fibre d) A net-like plexus that runs closer to the bone than
group of the periodontal ligament to the cementum
a) Horizontal b) Ob~que (AIIMS -01)
c) Alveolar crest d) Transseptal 11. Fibres of periodontal ligament embedded in the
(MAN -01) bone are
4. Anchoring fibrils are composed of a) Sharpey's fibres b) Tomes fibres
a) Type V and VII collagen c) Elastic fib res d) Ray's fibres
b) Type I and III collagen (AIIMS -97)
c) Type IV collagen d) Type IV and III collagen 12. Group of fibres, which resist the masticatory forces
(MAN -01) a) Dentogingival b) Transeptal
5. Collagen molecule exhibits all of the following c) Oblique d) Horizontal
features except (PGI -02)
a) Triple pleated sheath 13. Intermediate plexus is seen in the:
b) Mostly contains glycine residues a) Cementum b) POL
c) Exhibit cross striations at 64 µm c) Pulp d) Dentin
d) Intracellular in nature (AIPG -98)
(MAN -02) 14. In mammals independent and tough suspension for
6. Remnants of the Hertwig's root sheath are found in teeth is provided by
a) Gingiva b) Alveolar bone a) Alveolar sockets b) Cementum
c) Periodontal liga ment d) Cementum c) Gubernacular cords d) Periodontal membrane
(MAN -01) (KAR -99)
7. Age changes in periodontal ligament include which 15. Bone adjacent to periodontal ligament that contains
of the following? a great number of sharpey's fibres is known as
a) Increased fibroplasia b) Increased vascularity a) Lamina dura b) Bundle bone
c) Increased thickness c) Lamina propria d) Lamina densa
d) Decreased in number of cementicles (KAR -02)
(AIPG -95) 16. The most abundant principal fibre group in
8. All of the following cells of periodontal ligament periodontal ligament is:
are of mesenchymal origin except a) Horizontal b) Transeptal
a) Fibroblasts b) Multi totipotent cells c) Apical d) Oblique
c) Cementoblasts d) Cell rests of malassez (KAR -02)
(AP -98)

1) B 2) C 3) D 4) A 5) D 6) C 7) A 8) D 9) D 10) D 11) A 12) C 13) B


14) D 15) B 16) D
r ORAL HISTOLOGY

17. Collagen biosynthesis occurs inside the: c) Reversal lines d) Canaliculi


a) Chondroblasts b) Odontoblasts (COMEDK- 09)
c) Osteoblasts d) Fibroblasts 27. True about osteoclasts
(KAR -97) a) Monocytes are precursor cells of osteoclasts
18. Which of the following groups of fibres are not b) The plasma membrane of osteoclasts adjacent to
attached to alveolar bone? bone that is being resorbed is known as striated or
a) Transseptal b) Horizontal ruffled border
c) Oblique d) Apical c) Contain lysosomes wit h acid phosphates
(AIIMS -89) d) All of t he above are true
19. Principal fibres of periodontal ligament are
attached to 28. Periodontal ligament is made up of:
a) Alveolar bone proper b) Bundle bone a) Type I collagen
c) Lamellar bone d) Cortical bone b) Type I and type III collagen
(AP -99) c) Type I and type II collagen
20. Periodontal ligament has predominantly: d) Type I and type IV collagen
a) Type II collagen fibres
b) Oxytalan fibres 29. Which of the following is correct
c) Elastic fibres d) Type I collagen fibres a) Fibronectin helps in attachment of fibroblasts to
(AIPG -96) collagen
21. The calcified mass found in the PDL are: b) Cementicles are found in periodontal ligament of
a) Cementicles b) Osteoblasts order individuals
c) Cementoclasts d) Osteoclasts c) The large nerve fibres are myelinated and are
(PGI -99, AIPG -98) concerned with discernment of touch
22. Sharpey's fibres are derived from : d) The small nerve fibres may or may not be myelinated
a) Hertwig's root sheath and are concerned with discernment of pain
b) Epithelial rests of malassez e) All of the above
c) Alveolar bone d) Dental follicle
(AIPG -95) 30. Cementicles are found in the
23. Intermediate plexus is absent in which of the a) Gingiva b) Periodontal ligament
following principal fibres: c) Alveolar bone d) Cementum
a) Gingival b) Oblique (COMEDK -04)
c) Horizontal d) Transseptal 31. The most abundant group of fibres in the PDL are
(AIPG -98) a) Trans-septa l b) Horizonta l
24. Periodontal fibres joining cemental surface of one c) Oblique d) Apical
tooth, to cemental surface of adjacent tooth are (PG! -97)
called: 32. Width of the periodontal ligament is least at
a) Gingivodental b) Cementa[ fibres a) Cervical third b) Apical third
c) Horizontal fibres d) Transseptal fibres c) Fulcrum of rotation d) It has uniform width
(AIPG -95) (KAR -99)
25. Sharpey's fibres are present in: 33. The rests of malassez in the periodontal ligament
a) Bone b) Periosteum are derived from
c) Periodontal ligament a) Denta l pulp b) Lamina propria
d) All of the above c) Odontogenic epithelium
(KAR -98) d) Vestibular lamina
26. Interdependency of osteoblasts and osteoclasts in (APPSC -99)
bone remodeling is called as -
a) Lacunae b) Coupling

17) D 18) A 19) B 20) D 21) A 22) D 23) A 24) D 25) A 26) B 27) D 28) B 29) E
30) B 31) C 32) C 33) C
Dental ;lut.,e

34. Development of supporting tissues of the tooth is 42. In a dried skull, holes on the lingual aspects of the
initiated by deciduous teeth are called -
a) Cells of dental follicle a) Volkmann canals
b) Cells of odontogenic layer b) Canals of Hirschfeld and zuckerland
c) Cementoblasts d) Cells of the gingiva c) Haversian canals d) Gubernacular canals
(TNPSC -99) (COMEDK- 09, 11)
35. The main function of the horizontal fibres of P.D 43. A special feature of the periodontal ligament
ligament is fibroblasts is
a) Prevent extrusion a) The number of organelles
b) Prevent rotation b) Presence of actin fibres and shape change
c) Maintains the mesiodistal width c) Size of the cells
d) All of the above d) Collagen fibre formation
(APPSC -99) (AP-2012)
36. The periodontal ligament is approximately 44. Epithelial rests of Malassez are found in
a) 0.25 mm in thickness a) Pulp b) Gi ngiva
b) 0.5 mm in thickness c) Periodontal ligament d) Alveolar mucosa
c) 0.75 mm in thickness (MHCET-2015)
d) 1.0 mm in thickness
(KAR -98)
3 7. Narrowing of periodontal ligament in labially
placed mandibular canine with age is due to:
a) Due to deposition of cementum and bone
b) Down growths of gingival epithelium
c) Gingival cysts formed from cell rests
d) Reversal of function of horizontal and oblique
fibres
(AP-06)
38. Width of PDL is
a) 0.10 mm b) 0.50 mm
c) 0.25 mm d) 0.75 mm
(BHU-07)
39. Centre of the disc in TMJ is
a) Avascular
b) Devoid of nervous tissue
c) Avascular & deviod of nervous tissue
d) None of the above
(MCET- 10)
40. The type of bone present in the labial area of
anterior teeth is
a) Cortical b) Ca ncellous
c) Osteophytic d) Exophytic
(AIPG- 09, 11)
41. The component of future T.M.J shows development
at?
a) 6 weeks b) 10 weeks
c) 18 weeks d) 16 weeks
(AIPG- 09)

34) A 35) C 36) A 37) A 38) C 39) C 40) A 41) B 42) D 43) B 44) C
= = = = = = ==='r ORAL HISTOLOGY

6. PERIODONTAL LIGAMENT - ANSWERS


1. 'B' [Orban's 11th ed 209/ 12th ed 161/ 13th ed 180)
Extend obliquely from the cementum
Alveolar beneath the junctional epithelium
2. 'C' [Orban's 11th ed 207 / 12th ed 160/ 13th ed 186) crest to the alveolar crest. They prevent
Periodontal ligament consists of type I collagen group extrusion and resist lateral tooth
predominantly and has a rapid t urnover rate. movements.
Extend at right angles to the long axis
3. 'D' [Orban's 11th ed 222/ 12th ed 170/ 13th ed 190, Horizontal
of the tooth from the cementum to
191) group
the alveolar bone.
Periodontal ligament consists of collagen and oxytalan
Extend from the cementum in a
fibres. Collagen fibers gathered into bundles are termed
coronal direction to the bone. These
principal fibers. They are alveolar crest, horizontal, Oblique
are numerous, constitute main
oblique, apica l and interradicular group of fibers. group
attachment and resist t he vertica l
masticatory forces.
4. 'A' [Rubin's Pathology 6th ed 1115/ Periodontics
Apical They are not found in incompletely
revised 1st ed 9/ Tencate 8th ed 60)
group formed roots.
The anchoring fibrrns are com posed of type VII collagen .
They extend from basal lamina of epithelia l cells and
13. 'B' [Orban's 11th ed 224/ 12th ed 173/ 13th ed 191)
attach to the lamina reticularis by wrapping aroung the
Fibers arising from cementum and bone are joined
reticular fiber bundles. Anchoring fibrils are essential
in the mid region of the periodontal space, giving
to the functional integrity of the dermoepidermal
rise to a zone called intermediate plexus where rapid
junciton.
remodeling of fibers occurs.
5. 'D' [Orban's 13th ed 189)
14. 'D' [Orban's 11th ed 204/ 12th ed 156/ 13th ed 173)
Collagen is extra cellular substance.
In humans the teeth are attached to the socket of bone
through a fibrous connective tissue that is noticeably
6. 'C' [Orban's 11th ed 215 / 12th ed 167/ 13th ed 186)
cellular and vascular called periodonta l ligament. This
type of attachment is called Gomphosis.
7. 'A' [Orban's 13th ed 199)

8. ' D' [Orban's 11th ed 216/ 12th ed 168/ 13th ed 186) 15. 'B' [Orban's 11th ed 247 / 12th ed 201/ 13th ed 226)
Rests of malassez are epithelial in origin . They are
remnants of Hertwig's epithelial root sheath and are 16. ' D' [Orban's 11th ed 222/ 12th ed 170/ 13th ed 190)
found close to cementum.
17. 'D' [Orban's 11th ed 213/ 12th ed 166/ 13th ed 180)
9. ' D' [Orban's 11th ed 203/ 12th ed 155/ 13th ed 172)
18. 'K [Orban's 12th ed 253/ Tencate 8th ed 227)
10. 'D' [Orban's 11th ed 229/ 12th ed 175/ 13th ed 194) Transseptal fibres extend! interproximally from
cementum to cementum of adjacent teeth over the
11. 'A' [Orban's 11th ed 222/ 12th ed 170/ 13th ed 190) alveolar crest. They are reconstru cted even after the
destruction of the alveolar bone due to periodontal
Collagen fibers in periodontal ligament are embedded
diseases.
into cementum om one side and into alveolar bone
on the other side. The embedded fibers are termed
sharpey's fibers. 19. ' B' [Orban's 11th ed 247 / 12th ed 201/ 13th ed 226)

20. 'D' [Orban's 11th ed 222/ 12th ed 170/ 13th ed 189)


12. 'C' [Orban's 11th ed 222/ 12th ed 170/ 13th ed 190)
Periodontal ligament main ly contains type I and type
III collagen with type I appearing predominantly.
Dental ;lut.,e

21. 'A' [Orban's 11th ed 230/ 12th ed 177/ 13th ed 196] 38. 'C' [Orbans 11th ed 235/ 12th ed 173/ 13th ed 173]
Cementicles are calcified bodies fo und in the periodonta l The normal range of width of periodontal ligament is
ligament. They are seen in older individuals. They 0.15 to 0.38mm.
remain free in the connective t issue. When they are
adherent to the cementum, they form excementoses. 39. 'C' [Orban's 12th ed 304/ 13th ed 359]
The fibro articular disk functions as a shock absorber. It
22. 'D' [Orban's 11th ed 207 / 12th ed 160/ 13th ed 190- is oval, fibrous, avascular and non-innervated. The disk
191] is biconcave in sagittal section, with a thick anterior
band, a thick posterior band and a thin intermediate
23. 'A' [Orban's 11th ed 224/ 12th ed 173/ 13th ed 191] zone.
Option 'A' Gingival fibres are not principal fibres.
40. 'A' [Orban's 12th ed 201/ 13th ed 228]
24. 'D' [Orban's 11'h ed 293/ 12'h ed 253/ 13th ed 262, Cortical plates consist of co mp act bone and forms
264] outer and inner plates of alveolar processes. They are
thinner in maxilla than mandible. They are thickest
25. 'A' [Orban's 11th ed 222/ 12th ed 170/ 13th ed 190- in the premolar and molar region of the lower jaw on
191] buccal side. In the region of anterior teeth of both
jaws, the supporting bone usually is very thin.
26. 'B' [Molecular cancer research April 2010/ 13th ed
222] 41. ' B' [Orban's 12th ed 305/ 13th ed 361]
At approximately 10 weeks, the component of future
27. 'D' (Orban's 11th ed 212/ 12th ed 163/ 13th ed 182] joint show the first indication in mesenchyme between
the condylar cartilage of mandible and the developing
28. 'B' [Orban's 11th ed 222/ 12th ed 170/ 13th ed 189] tern poral bone.

29. 'E' [Orban's 11th ed 230/ 12th ed 177/ 13th ed 181, Two slit like joint cavities and an intervening disc
196] make their appearance in this region at 12 weeks.

30. 'B' [Orban's 11th ed 230/ 12th ed 177/ 13th ed 196] 42. 'D' [Orban's 12th ed 285/ 13th ed 336-338, Fig
13.6]
31. 'C' (Orban's 11th ed 222/ 12th ed 170/ 13th ed 190]
43. 'B' [Orban's 13th ed 180-81]
32. 'C' (Orban's 11th ed 235/ 12th ed 173/ 13th ed 200]
Width of periodontal ligament range fro m 0.15- Periodontal Gingival
0.38mm. It is thin in the middle region of root at
fibroblast fibroblast
the fulcrum of rotation and in functionless, embedded
Origin Ectomesenchymal Mesodermal
teeth. It is wide in teeth under excessive occlusal
st resses. Mo re due t o more
Proliferative
expression of ALP and Less
capacity
33. 'C' (Orban's 11th ed 205/ 12th ed 156/ 13th ed 186] cyclic AMP
Presence of actin
34. 'A' [Orban's 11th ed 205/ 12th ed 156/ 13th ed 225] micro filaments and
Eruptive fibronecti n forms
No
3 5. 'C' [Orban's 13th ed 189] capacity contractile apparatus
generating forces for
36. 'A' [Orban's 11th ed 235/ 12th ed 173/ 13th ed 173] tooth eruption

37. 'A' (Orban's 13th ed 163] 44. 'C' [Check Explanation of Q.No.8]
-==============Ir ORAL HISTOLOGY

7. BONE, TMJ AND MAXILLARY SINUS


1. Alveolar process helps in providing d) None of the above
a) Support and retention to t he complete denture (AIPG -96)
b) Boundaries fo r the tongue during speech 9. Mature bone is otherwise called as:
c) Attachment to muscles of facial expression a) Woven bone b) Irregular bone
d) Support the natural teeth c) Lamellar bone d) Resorbing bone
(MAN -2K) (KAR -03)
2. The maxillary sinus: 10. The marrow found in the mandible is:
a) Opens in to the inferior meatus of nose a) All fatty marrow b) All red marrow
b) Has the 1'1 molar projecting into its floor c) All fibro us marrow
c) Is fully developed by the age of 8 years d) A combination of fatty and fibrous marrow
d) Has good drainage in upright position (KAR -98)
(KAR -97) 11. Basal bone is :
3. If a child's teeth do not form, this would primarily a) That part of the bone embracing apices of
effect the growth of the: root of teeth
a) Basal bone b) Whole face b) Lower border of the mandible
c) Mandible d) Alveolar bone c) Ramus of the mandible
(KAR -2K, MAN -95) d) That part of the bone containing teeth
4. Organ of chievity is seen near the: (KAR -01)
a) Medial surface of the mandible 12. Which of the following is true
b) Naso palatine fora men a) All the bones of upper face develop by int ra
c) Mental foramen d) Foramen caecum membranous ossification
(KAR -2K) b) Incus and malleus develops from meckels cartilage
5. Calcium content of lamina dura is: c) The third auditory ossicle or stapes develops from
a) More than the other parts of alveolar bone 2nd branchial arch
b) Less than the ot her parts of alveolar bone d) All of the above
c) Same as in other parts of alveolar bone
d) Less than the cementum 13. Which of the following is not true
(KAR -98) a) Alveolar bone proper surrounds the root and gives
6. Reversal lines which may be seen on the cribriform attachment to fibres of periodontal ligament
plate (alveolar bone proper) of the alveolar process b) Supporting alveolar bone surrounds the alveolar
indicate the cessation of: bone proper and supports the socket
a) Osteoblastic activity b) Osteoclastic activity c) The cortical plates are t hinner in maxilla than in
c) Myeloid activity d) Healing activity mandible
(AIPG -97} d) They a re thickest in anterior portion of jaw
7. Bone at forming stage has
a) Osteoclasts, osteoblasts, osteoid 14. In a healthy mouth, the distance between the
b) Odontoblasts & megakaryocyte cementoenamel junction and the border of alveolar
c) Megakaryocytes, osteoblasts, osteoclasts bone proper is
d) None a) Constant b) Changes with age
(PGI -02) c) Depends on inclination of teeth
8. Cibriform plate is also known as: d) None of the above
a) Lamina dura
b) Alveolar bone proper
c) Supporting alveolar bone

1) D 2) B 3) D 4) A 5) A 6) B 7) A 8) B 9) C 10) D 11) A 12) D 13) D


14) A
Dental ;lut.,e

15. Which of the following is wrong c) 58 mm d) 28 mm


a) The architecture of type I spongiosa is seen in
mandible 22. Mandible develops by
b) Type II spongiosa is seen in maxilla a) Endochondral ossification only
c) Condylar process & angle in mandible, and b) Membranous ossification only
tuberosity in maxilla contain hemopoietic cellular c) Both of the above d) None of the above
marrow
d) Bundle bone contains a lesser amounts of calcium 23. Which of the following is true
salts than other types of bony tissue a) Maxillary sinus comm un icates with environment
through middle meatus and nasal vestibule
16. Which of the following is true b) Maxillary sinus is a four- sided pyramid
a) Cutting cone or resorption tunnel is the area of c) In most of the cases, the main ostium is present in
resorption seen in alveolar bone posterior third of hiatus semilunar
b) CAMP or cyclic adenosine monophosphate levels are d) All of the above
increased on pressure side of bone and this results
in bone resorption 24. The type of epithelium lining maxillary sinus is
c) Immature bone / embryonic bone seen during a) Simple columnar
fract ure hea ling contains less calcified material and b) Pseudo stratified columnar and ciliated
greater number of large osteocytes c) Stratified squamous d) Simple squamous
d) All of the above
25. The tooth present close to maxillary sinus is
17. Role of meckel's cartilage in development of a) 1st molar b) 2 nd premolar
mandible c) pt premolar d) 2 nd molar
a) It has a major role b) It has a minor role
c) It does not have any role 26. Functional importance of maxillary sinus is
d) None of the above a) Lightening of skull weight and production of
bactericidal lysozyme to the nasal cavity
18. The components of future TMJ shows development b) Protects the brain against exposure to cold air by
at arresting air in sinus temporarily
a) 6 weeks b) 18 weeks c) Resonance of voice and enhancementof faciocranial
c) 10 weeks d) 16 weeks resistance to mechanical shock
(AIIMS MAY-13) d) All of the above
19. The condyle of mandible is composed of
a) Compact bone b) Cancellous bone 27. Which of the following is not correct
c) Cancellous bone covered by thin layer of compact a) In case of pituitary gigantism, all sinuses assume a
bone larger volume than normal
d) Compact bone covered by cancellous bone b) In chronic infections, the pain may mimic neuralgia
of dental origin
20. Which of the following is correct c) Malignant lesions of maxillary sinus produce t heir
a) Roof of mandible contains thin, compact bone manifestations in maxillary teeth
b) Articular tubercle is composed of spongy bone d) None of the above
covered with thin compact bone
c) Condyle contains red bone marrow 28. The articular cartilage is charracterized by all of the
d) All of the above following features except:
a) It is devoid of perichondrium
21. Normal interincisal distance is b) It has a rich nerve supply
a) 48 mm b) 38 mm c) It is avascular

15} D 16} D 17} C 18} C 19} C 20} D 21} A 22} C 23} D 24} B 25} A 26} D 27} D
28} B
r ORAL HISTOLOGY

d) It lacks the capacity to regenerate 37. Haversian systems ( osteons) are found primarily in
(KAR -04) the
29. The type of bone present in the labial area of a) Spongy bone b) Periosteum
anterior teeth is c) Alveolar bone proper d) Endosteum
a) Cortical b) Cancellous (COMEDK-2011)
c) Osteophytic d) Exophytic 38. The intimal B cells of synovial membrane of
(AIPG-2011) temporomandibular joint are
30. The muscle which is commonly involved in myofacial a) Fibroblast like b) Macrophage like
pain dysfunction syndrome c) Lmphocyte like d) Plasma cell like
a) Lateral pterygoid b) Massetor (COM ED K-2011)
c) Temporalis d) Medial pterygoid 39. The process of recesses formation in maxillary sinus
is most frequently seen with
31. Second arch derivatives are all except a) Zygomatic process b) Frontal process
a) Stapes b) Malleus and incus c) Alveolar process d) Palatine process
c) Styloid process (COMEDK -2013)
d) Superior part of hyoid bone 40. Maxillary sin us is described as a four sided pyramid
containing base, apex, roof and floor. The apex of
32. Fibres which continue from bone to ligament are maxillary sinus extends
a) Sharpey's fibres b) Tome's fibres a) Laterally into zygomatic process of maxilla
c) Ray's fibres d) All of the above b) Medially into zygomatic process of maxilla
c) Laterally into lateral nasal wall
33. Basic metabolic unit of bone is: d) Superiorly into floor of orbit
a) Osteon b) Osteoblast (COMEDK -2013)
c) Osteocyte d) Osteoclast 41. Immature bone is termed as
(COMEDK-06) a) Cancellous bone b) Lamella r bone
34. Which of the following enzymes are involved in c) Woven bone d) Compact bone
bone formation? (GCET- 14)
a) Alkaline phosphatase and urease 42. Maxillary antrum dips in the region of?
b) Catalase and phosphorylase a) Between 2nd premolar and 1 molar
c) Alkaline phosphatase and phosphorylase b) Between buccal roots of furcation of 1st molar
d) Acid phosphatase and catalase c) Between 1 & 2 premolar
(AP- 06) d) Between 1 & 2 molars
35. Alveolar bone proper is also known as: (PG! DEC- 2011)
(or) 43. The audit maxillary sinus measures about
Alveolar bone proper is seen as lamina dura in an a) 33mm in height, 23mm in width
IOPA radiograph b) 33mm in width, 23mm in height
(AP- 2013) c) 30mm in height, 8 mm in width
a) Lamina lucida b) Lamina densa d) 8mm in height, 30mm in width
c) Lamina propria d) Lamina dura (COM ED K-2015)
(KAR-04)
36. When osteoblast lay new bone from outside to
inside in a osteon, the active formation area is
called:
a) Haversion cone b) Filling cone
c) Cutting cone d) Howships cone
(COM EDK- 07)

29) A 30) A 31) B 32) A 33) A 34) C 35) D 36) B 37) C 38) A 39) C 40) A 41) C
42) A 43) A
Dental ;lut.,e

7. BONE, TMJ AND MAXILLARY SINUS - ANSWERS


1. 'D' [Orban's ttth ed 241/ 12th ed 200/ 13th ed 2 24] 7. 'A '[Orban's 12th ed 201/ 13th ed 209, 216]
Alveolar process is t hat part of the maxi lla and mandible Osteoblasts secrete organic matrix of bone, which is
that forms and supports the teeth. It consists of known as osteoid. Osteoid at first is devoid of mineral
Surrounds the root of the teeth. salts. Some of osteoblasts become embedded in the
Alveolar mat rix and form osteocytes.
It consists of lamellar bone and
bone proper
bundle bone.
Mineralisation always lags behind formation of bone.
Surrounds the alveolar bone proper
and gives support to the socket. Resorption is carried out by osteoclasts. The processes
Supporting involved in resorption are deca lcification, degradation
It consists of outer, inner cortical
alveolar bone of matrix and transport of soluble products to blood.
plates and spongy bone filling the
area between the plates.
8. ' B' [Orban's 11th ed 249/ 12 1h ed 200/ 13th ed 227]
2. 'B' [Orban's 12th ed 314/ 13th ed 3 70, 3 77] The alveolar bone proper is perforated by many
Maxillary sinus is a four-sided pyramid and opens openings t hat carry branches of interalveolar nerves
into the middle meatus of nose through ostium. It is and blood vessels into the periodontal ligament and is
closer to the root s of maxillary first molar, fo llowed by therefore ca lled as cribriform plate.
second premolar and second molar in decreasing order
of frequency. 9. 'C' [Orban's 13th ed 217]

3. 'D' [Orban's ttth ed 241/ 12th ed 200/ 13th ed 225] 10. ' D' [Orban's 11th ed 243/ 12th ed 201/ Carranza
Alveolar process develops only during the eruption of 11th ed 40]
teeth. It is diminished in the loss of teeth and does
not develop if the teeth do not form. 11. 'A' [Carranza 11th ed 38]

4. 'A' [Shafer's 5th ed 149/ 6th ed 106/ 7th ed 111] 12. ' D' [Orban's 11 1h ed 239/ 12th ed 200/ Tencate 8th
ed 31, 112, Tab 3-1]
5. 'A' [Orban's 13th ed 227]
Bundle bone is that bone in which the principal fibers 13. ' D' [Orban's 111h ed 243/ 12 1h ed 201/ 13th ed 228]
of the periodontal ligament are anchored. It contains The cortical plates are thickest in the premolar-molar
more ca lcium salts per unit are than other types of regions of mandible, especially on the buccal side. In
bone tissue. the anterior region the cortical plates are thin.

Radiographically, it is also referred to as the lamina 14. 'A' [Orban's ttth ed 244/ 12th ed 202/ 13th ed 229]
dura, because, of increased radiopacity, which is due The distance between CEJ and alveolar crest is constant
to the presence of thick bone without trabeculations, in healthy mouth and is about 1.5mm.
that X-rays must penetrate and not to any increased
mineral content. 15. ' D' [Orban's 12th ed 201/ 13th ed 227-228]
Classification of spongy bone
6. 'B' [Orban's 11th ed 253/ 12th ed 204/ 13th ed 229-
The trabeculae are regular and horizontal
30]
in a ladder like pattern . Present often
Resting lines in bone correspond to the rest periods of Type I
in mandible and has a distinct trajectory
continuous bone apposition. Reversal lines indicate pattern.
the cessation of resorption fo llowed by apposition
Trabeculae are numerous, delicate
of new bone on the old. The scalloped outline of
and irregularly arranged. Found more
the Howship's lacunae is visible as a darkly stained Type II
commonly in maxilla, lacks the distinct
cementing line called reversal line. trajectory pattern
r ORAL HISTOLOGY

16. ' D' [Orban's 12th ed 205-06/ 13th ed 231-232/ Cortical Plates - consists of compact bone and forms
Tencate 8th ed 118] the outer and inner plates of the alveolar processes.
The cortical plates, continuous with the compact layers
17. 'C' [Orban's 11th ed 239/ 12th ed 200/ 13th ed 225, of maxillary and mandibular body, are generally much
Fig 9.8] t hinner in the maxilla, than in the mandible. They
Meckel's carti lage has close appositiona l relation to are thickest in the premolar and molar region of the
developing mandible but makes no contribution to it. lower jaw especially on the buccal side. In the maxilla,
Mandible primarily develops as an intramembranous the outer cortical plate is perforated by many small
bone, which develops from mesenchymal condensation openings through which blood and lymph vessels pass.
lat eral to meckel's cartilage. The condylar cartilage In the lower jaw, the cortical bone of alveolar process
ossifies by endochondral ossification. is dense. In t he region of anterior teeth of both jaws,
the supporting bone usually is very thin. No spongy
18. 'C' [Orban's 11' h ed 409/ 12'h ed 305/ 13th ed 361) bone is found here, and cortical plate is fused with the
alveolar proper.
19. 'C' [Orban's 11th ed 409/ 12th ed 305/ 13th ed 362]
The condyle and articular tubercle is composed of 30. 'f,{ [Orban's 1tth ed 416/ 12th ed 311/ 13th ed 366]
ca ncellous bone covered by com pact bone. The roof
of mandibular fossa is made of thin layer of compact 31. 'B' [Orban's 11th ed 239/ 12th ed 200/ Tencate 8th
bone. ed 31, Tab 3-1]
Malleus and incus are derivatives of meckels cartilage
20. ' D' [Orban's 12th ed 305, 308/ 13th ed 362] (first arch). Stapes, styloid process, stylohyoid
ligament, superior cornua of hyoid bone are derivatives
21. 'A' [Orban's 1tth ed 416/ 12th ed 311/ 13th ed 366] of second arch (hyoid arch)

22. 'C' [Tencate 8th ed 109, 112] 32. 'A' [Orban's 11th ed 253/ 12th ed 204/ 13th ed 226]
Muscle, tendon or ligament is attached to surface of
23. 'D' [Orban's 11th ed 421/ 12th ed 313/ 13th ed 369- bone by sharpey's fibres, which can be seen penetrating
71] the basic lamellae.

24. 'B' [Orban's 11th ed 422/ 12th ed 314/ 13th ed 372] 33. 'A' [Tencate 5th ed 104/ Orban's 13th ed 209]

25. 'A' [Orban's 11th ed 422/ 12th ed 314/ 13th ed 377] 34. 'C' [Orbans 11th ed 452/ 12th ed 330/ 13th ed 219]

26. 'D' [Orban's t tth ed 432/ 12th ed 320/ 13th ed 375] 35. 'D' [Tencate 5th ed 274, 275/ Orban's 13th ed 227]
Alveolar process consists of two parts i.e., alveolar
27. 'D' [Orban's 11th ed 432/ 12th ed 320/ 13th ed 377, bone proper and supporting alveolar bone. Alveolar
378] bone proper is a thin lamella of bone that surrounds
Malignant Lesions of maxillary sinus will produce their the root of tooth and gives attachment to principal
primary manifestations in the maxillary teeth, which fibres of periodontal ligament. Alveolar bone proper
include pain, loosening, supraeruption and bleeding is also referred as bundle bone because it provides
from gingival sulcus. attachment for the POL fiber bundles. It is also called
as lamina dura because of an increased radiopacity
28. 'B' [Orban's 11th ed 413/ 12th ed 309/ 13th ed 359] due to presence of thick bone without trabecluation.
Articular disc is co mposed of thick fibrous tissue. The
fibrous tissue covering the condyle, articular eminence, The supporting alveolar bone supports the alveolar
as well as the large central area of the disc, are devoid bone proper and gives support to the socket.
of blood vessels and nerves and have limited reparative
capacity. 36. 'B' [Tencate 5th ed 122-24/ Orban's 13th ed 135]

29. 'f,{ (Orbans 12th Ed 201/ 13th ed 228]


Dental ;lut.,e

A cell:
Replacement of old bone by new is
Second cell type. It is rich in golgi complex and
Bone called Bone turnover. Bone turnove r
lysosomes. It is macrophage like cell.
turnover rates of 30% to 100% per year are
common in rapidly growing children.
Third type:
As osteoclasts move through bone, the Is has a cellular morphology between cell types A and B.
leading edge of resorption is termed as
Cutting "Cutting cone". It is characterized in 39. 'C' [Orbans 13th ed 371)
cone cross section by a scalloped array of In the course of development, the maxillary sinus
Howship's lacunae, each housing an often pneumatizes the maxilla beyond the boundaries
osteoc last. of the maxillary body. Some of these processes of
Behind t he cutting cone there is maxilla consequently become invaded by the air space.
migration of uninucleated cells, which These expansions are termed as recesses.
Cement differentiat e into osteoblasts, and
( or) produce a coating called as Cement or Process of maxilla Frequency of recesses
reversal line. The cement is a thin layer Alveolar 50%
Reversal
of several phosphoproteins that act as Zygomatic 41.5%
line
a mineralized adhesive and bind the old
Frontal 40.5%
bone to new bone to be secreted.
Palatine 1.75%
On the top of cement line, the
osteob lasts begin to lay down new Note:
Filling bone matrix from the outside to inside.
• The occurrence of zygomatic recess brings the
cone The entire area of the osteon where
superior alveolar nerve bundles into proximity with
active formation occurs is termed the
space of the sinus.
filling cone.
• Frontal recess invades the contents of the intra-
orbital canal.
37. 'C' [Orbans 11th ed 250/ 13th ed 225)
• Alveolopalatine recesses reduce the amount of
Alveolar process consists of two parts
the bone between the dental apices and the sinus
1. Alveolar bone proper
space.
2. Supporting alveolar bone - cortical bone and
spongy bone 40. 'A' [Orbans 13th ed 370]
Maxillary sinus is described as a four-sided pyramid.
Alveolar bone proper consists of bundle bone and The four sides are related to maxilla in the following
lamellated bone. The lamellae form the haversian manner
systems.
• Anterior - to the facial surface of the body of
maxilla
Supporting bone consists of compact bone, which form
• Inferior - to the alveolar and zygomatic processes
the outer and inner plates of the alveolar process,
and spongy bone, which fills the areas between these • Superior - to the orbital surface
plates and the alveolar bone proper. • Posterior - to the infratemporal surface

38. 'A' [Orbans 11th ed 414/ 13th ed 365] Base of sinus faces medially towards the nasal cavity.
The intimal cells of synovial membrane of TMJ are of 3 Apex is pointed laterally towards the body of zygomatic
types. bone.

B cell:
This is rich in rough endoplasmic reticulum and is
called the fibrob last like cell.
r ORAL HISTOLOGY

41. 'C' [Orbans 13th ed 217)


• Immature bone characterized by
interwined collagen fibers oriented in
many directions
• Deposition and mineralization is faster
than lamellar
Woven • Enriched in Bone sialoprotein and
bone Bone Acidic Glycoprotein-75
• Mineral density is lower; water content
is higher
• Removed by osteoclasts entirely
• Mat rix vesicles participate in initiation
of mineralization.
• Distinctive orderly arrangement of
collagen fibres are seen. The direction
of ti brils in any la mella lies at right
angles to that of me fibrils in the
Lamellar adjacent lamella
bone • Enriched in osteocalcin
• Only a portion of lamellar matrix of a
given bone is resorbed at one time.
• Calcification of lamellar bone is by
collagen mediated mechanism

42. 'A' [Endodontics by K. Gulabivala 1st ed 112)


In majority of persons the floor of the sinus dips down
into the immediate neighbourhood of the second
premolar and first molar.

43. 'A' [Orbans 13th ed 369)


Dimensions of maxillary sinus (length X height X
width)
• Age of 1 15 x 6 x 5.5 mm
• Age of 15 31.5 x 19 x 19.5 mm
• Adults 34 x 33 x 23 mm
Dental ;lut.,e

8. SALIVARY GLANDS
1. The normal pH of the saliva is about 10. Bartholins duct is the name of:
a) 5.5 b) 9.5 a) Parotid duct b) Submandibular gland
c) 7.5 d) 8.5 c) Sublingual duct d) Lacrimal duct
(MAN -94) (AIIMS -01)
2. Of the total amount of saliva secreted by all the 11. Which of the following is correct about serous
salivary glands, about 60% is secreted by the glands
a) Parotid glands b) Sublingual glands a) They are specialized for the synthesis, storage and
c) Submandibular glands secretion of proteins
d) None of the above b) They contain secretory granules in the apical
(MAN -91,) cytoplasm and the secretion of granule content
3. The neutralization of saliva is due to occurs by "exocytosis"
a) Mucin b) Ammonia c) Serous cells are pyramidal in shape
c) Amino acids d) Bicarbonates d) All of the above
(AIIMS -2K; AP -04,05)
4. Wharton's duct drains the: 12. True about myoepithelial cell
a) Pancreas b) Parotid a) Also known as basket cell
c) Lacrimal gland d) Submandibular gland b) Structure is similar to smooth muscle and contains
(KAR -97) actin & myosin
5. Among various glands, salivary gland secretion is c) The cell expel secretions by contraction
unique in that, its secretions are controlled by d) All of the above
a) Hormones b) Nerves
c) Chemicals d) All of the above 13. Main function of salivary duct is
(AP -01) a) To convey the saliva secreted by the terminal
6. The content of saliva include all except: secretory units to oral cavity
a) Amylase b) Urea b) Antibacterial action by secreting lysoenzymes and
c) Lysoenzyme d) Lipase lactoferrin
(KAR -97) c) Synthesis of secretory glycoproteins by kallikrein,
7. The duct(s) of gland(s) opening into the floor of an enzyme in striated duct cells
the mouth is (are): d) None of the above
a) Submandibular and sublingual
b) Sub mandibular and parotid 14. Which of the following is correct
c) Sublingual and parotid a) Parotid - stenson's duct - pure serous
d) Von Ebner b) Submandibular - wharton's duct - mixed and
(KAR -01) predominantly serous
8. Sublingual gland is a: c) Sublingual gland - bartholin's duct - mixed and
a) Mixed gland b) Serous gland predominantly mucous
c) Mucous gland d) Present in root of tongue d) All of the above
(KAR -98)
9. Saliva is secreted by 15. Serous demilunes are seen in
a) Major salivary glands a) Salivary glands b) Pancreas
b) Minor salivary glands c) Liver d) Gastric glands
c) Major & minor salivary glands
d) Parotid gland 16. Vonebners glands are
(AP -03) a) Glands at apex of tongue

1) C 2) C 3) D 4) D 5) B 6) D 7) A 8) A 9) C 10) C 11) D 12) D 13) A


14) D 15) A 16) C
r ORAL HISTOLOGY

b) Posterior ling ual mucous glands c) Provide signals to the acinar secretary cells for
c) Posterior lingual serous glands structural organization
d) Glosso palatine glands d) All of the above
(KERA LA-2015) (KCET- 10)
17. Total volume of saliva secreted by submandibular 26. Myoepithelial cells are present in -
gland daily is a) Striated duct
a) 750 ml b) 450 ml b) Intercalated & terminal duct
c) 1500 ml d) 1000 ml c) Serous cells d) Mucous cells
(MCET- 10)
18. Pure mucous glands are 27. Carmalt's glands are
a) Labial and buccal glands a) Major salivary gland
b) Glosso palatine & palatine glands b) Minor salivary glands (Retromolar)
c) Vonebners glands c) Minor salivary gland (Lingual)
d) Lingual glands d) Taste Buds
(KCET-2011)
19. Acid neutralizing substance in saliva 28. Saliva which is formed in salivary glands, when
a) NH 3 b) Carbonate passes from salivary gland to duct orifices, it
c) Bicarbonate d) Chlorides undergoes numerous ion exchanges and as a result
(AP -04, AIIMS -2K) saliva become as compared to plasma:
20. The glands of Blandin and Nuhn are a) Hypertonic sometimes
a) The posterior lingual glands b) Hypotonic
b) The anterior lingual glands located near the apex of c) Isotonic d) Hypertonic always
tongue (AIIMS NOV 13)
c) The glossopalatine glands 29. Tuft cells are receptors seen on
d) The palatine glands a) Cell rich zone b) Lining of maxillary sinus
(APPSC -99; COMEDK- 05) c) Salivary Duct d) TMJ Capsule
21. A purine which is found in saliva is: (AIIMS NOV 13)
a) Uric acid b) Thymine 30. Saliva has the following functions, EXCEPT
c) Adenosine d) Hypolithinium a) Defence function b) Digestive function
(PGI- 06) c) Absorptive function d) Lubrication function
22. Which of the following is not present in saliva: (MHCET-2015)
a) Lysozyme b) Phosphate 31. Salt sensation is produced by
c) Bicarbonate d) Trypsin a) Alkaloids b) Organic substances
(AP-07) c) Ionizing substances d) Acidic substances
23. An enzyme present in saliva which causes cell wall (COMEDK-2015)
lysis:
a) Lysozhyme b) Peroxidase
c) Lactoferin d) Hyaluronidase
(KCET-07)
24. Mucin in mucous cells usually demonstrated by
a) PAP stain b) H & Estain
c) Mucicarmine stain d) Mallory stain
(KCET-08)
25. The functions of myoepithelial cells may be
a) Initiating contraction
b) Support for the end piece during active secretion of
saliva

17) B 18) B 19) C 20) B 21) C 22) D 23) A 24) C 25) D 26) B 27) B 28) B 29) C
30) C 31) C
Dental ;lut.,e

8. SALIVARY GLANDS - ANSWERS


1. '(' [Orban's ttth ed 365/ 12th ed 274/ 13th ed 309]
Gustin Taste
PH of saliva varies from 6. 7 - 7.4 whereas parotid saliva
may have a greater ra nge, fro m 6.0 - 7.8. The primary • lg A
buffering system of saliva is fo rmed by bicarbonate. • Lactoferrin
Antibacterial action.
• Lysozyme
2. 'C' [Orban's 1tth ed 365/ 12th ed 274/ 13th ed 309]
The whole amount of saliva secreted daily by humans
• Peroxidase

is 750 ml of which 60% is produced by submandibular


7. 'A '[Orban's ttth ed 357/ 12th ed 269/ 13th ed 304]
glands, 30% by parotids, 5% from sublingual and 7%
fro m t he minor salivary glands.
8. 'A' [Orban's ttth ed 357/ 12 1h ed 269/ 13th ed 309]
3. 'D' [Orban's 13th ed 309] Parotid Purely serous
Mixed, with serous units
4. 'D' [Orban's 11th ed 3 5 7/ 12th ed 2 69/ 13th ed 304] Su bmandi bular
predominating
Opens near maxillary Mixed, with mucous units
Stensons Su blingual
Parotid z nd molar in the buccal
duct predominating
mucosa
Opens at the side of 9. 'C' [Orban's 11th ed 365/ 12th ed 274/ 13th ed 303]
Subma ndi- Wharton's
lingual frenum
bular duct
(caruncula sublingualis) 10. 'C' [Orban's 11th ed 357/ 12th ed 269/ 13th ed 304]
a) Bartholins Opens near
duct submandibular duct . 11. ' D' [Orban's ttth ed 340/ 12th ed 259/ 13th ed 293]
Sub lingual b) Several Open along the
12. 'D' [Orban's 12th ed 263/ 13th ed 296]
mi nor sublingual fold
ducts indepe ndently.
13. 'A' [Orban's 11th ed 351/ 12th ed 266/ 13th ed 297]

5. ' B' [Orban's 11th ed 354/ 12th ed 26 7/ 13th ed 307]


14. 'D' [Orban's 11th ed 357/ 12th ed 269/ 13th ed 303,
The parasympathetic (secretomotor) supply to
304]
Submandibular and sublingual salivary glands is
throug h chordatympanic nerve and that of parotid 15. 'A' [Orban's ttth ed 349/ 12th ed 265/ 13th ed 304]
gland is through glossopharyngeal nerve. Demilune is a combination of serous and mucous unit
in which mucous cells are capped by several serous
Parasympathetic sti mulat ion produces copious, watery cells.
saliva while the sympathet ic stimulation produces
thicker, and less quantity of saliva. 16. 'C' [Orban's 11th ed 360/ 12th ed 271/ 13th ed 306]
Lingual minor salivary glands are divided into anterior
6. 'D' [Orban's 1tth ed 3 65/ 12th ed 2 74/ 13th ed 310]
and posterior region glands.
Water accounts for a bout 99% of saliva.
Anterior lingual glands (G lands of Blandin and Nuhn)
Constituent Function
are chiefly mucous. Posterior lingual serous glands are
Glycoprotein Lub rication pellicle formation called Von Ebner's glands. Vonebners glands are purely
Bicarbonate Buffer (neutralization of acid serous, and are present in between muscle fibres of
maintains pH) tongue below vallat e papillae.

Amylase Digestive enzyme


17. ' B' [Orban's 11th ed 365/ 12th ed 274/ 13th ed 309]
r ORAL HISTOLOGY

18. ' B' [Orban's 11th ed 359/ 12th ed 271/ 13th ed 305] • Molar or retromolar (Carmalt's glands)
Labial and buccal minor salivary • Inferior apical/ anterior lingual glands - Blandin-
Mixed
gland Nuhn glands
Glossopalatine and palatine glands Purely mucous • Posterior lingual glands - Von-Ebner glands.

Vonebners glands Purely serous


28. ' B' [Ten Cate's 8th ed Page 270]
The lumi nal and basolateral membranes of the end pieces
19. 'C' [Orban's 11th ed 365/ 12 1h ed 274/ 13th ed 309] and intercalated ducts have abundant transporters
that function to produce a net reabsorption of Na+
20. ' B' [Orban's ttth ed 360/ 12th ed 271/ 13th ed 306] and Cl- resulting in formation of hypotonic saliva.
The ducts also secrete K+ arnd HC03- but little if any
21. 'C' [Orbans 11th ed 365/ 12th ed 274/ 13th ed 308] secretion and reabsorption of water occurs in striated
Adenosine is present in saliva in form of cyclic AMP. or excretory ducts.

22. 'D' [Orbans ttth ed 365/ 12th ed 274/ 13th ed 309] 29. 'C' [Ten Cate's 8th ed Page 269]
Tuft cells (caveolated or brush cells) are seen in the
23. 'A' [Orbans 11th ed 366/ 12th ed 275/ 13th ed 310] epithelium of the main excretory duct of salivary
glands. They have long, stiff microvilli and apical
24. 'C' [Orbans 11th ed 459/ 12th ed 338/ 13th ed 398] vesicles and are thought to be receptors of some type.
Muciramine and Mucihematen are the two dyes that The shape of tuft cells varies from pear-shaped, to
are used for non -specific staining of mucins. PAS barrel-shaped and goblet-shaped. Their proposed
technique is used to identify neutral mucins. Alcian function is absorption and crnemoreception.
blue, toluidine blue, coloidal iron and aldehyde fuschin
methods are used to localize the acid mucins. 30. 'C' [Orbans 13th ed 310]
Functions of saliva
25. 'D' [Orban's 12th ed 263/ 13th ed 296] 1. Oral Cavity:
• Prevents desiccation
26. 'B' [Orban's 121h ed 263/ 13th ed 296]
Myoepithelial cells: • Washing action to flush away debris and bacteria
• Related to secretory and intercalated duct cells • Lubrication
• Contains cytokeratin intermediate filament and • Protects chemical and thermal insult
contractile acti n filaments • Buffering capacity due to bicarbonates,
phosphates and proteins
Functions • Proline rich proteins, glycoproteins, statherin
• Contractile process promotes remineralisation
• Accelerate outflow of saliva from acini • Antibacterial action from peroxidases,
• Shorten and widen intercalated ducts Thiocyanate, which catalyses the formation
of hypothiocyanate ion. Lysozyme,
• Supports parenchyma
immunoglobulins, lactoferrin also contribute
• Contribute to secretory pressure in the acini or this action.
duct.
2. Digestion
27. 'B' [Thoma's Oral pathology 3rd ed 962/ Ongole,
Praveen: Clinical Manual for Oral Medicine & 3. Mastication and deglutition
Radiology 1st ed 228] 4. Taste perception
Various minor salivary glands
5. Speech
• LabialfBuccal
6. Tissue repair
• Palatine
• Tonsillar (Weber s glands) 7. Excretion
Dental ;lut.,e

31. 'C' [Check Explanation Below]


Salty taste is elicited by ionized salts, mainly by
the sodium ion concentration. Sour taste is caused
by acidic substances that is by the hydrogen ion
concentration. The more acidic the food, the stronger
the sour sensation. Sweet taste is caused by organic
chemicals such as sugars, glycols, alcohols, ketones
etc.Bitter taste is caused by organic substances that
contain nitrogen, alkaloids like quinine, caffeine,
strychnine and nicotine.
r ORAL HISTOLOGY

9. ORAL MUCOUS MEMBRANE


1. In which of the fo Llowing papillae of the tongue are 9. Stratum granulosum is not present in:
the taste buds predominantly Located? a) Hyper orthokeratosis b) Hyper parakeratosis
a) Circumvallate b) Filiform c) Non keratinized epithelium
c) Foliate d) Fungiform d) Sulcular epithelium
(MAN -94) (AIPG -02)
2. Which of the following has immune function in the 10. Keratohyaline granules are found in:
oral mucous membrane a) St. granulosum b) St. spinosum
a) Melanocyte b) Merkel cell c) St. basale d) Prickle cell layer
c) Langerhan cell d) Kera ti nocyte (AIPG -02; COMEDK- 07)
(MAN -01) 11. Non keratinized epithelium is found over:
3. Lamina densa of the gingival basement membrane is a) Attached gingiva b) Free gingiva
rich in c) Interdental papilla d) Gingival sulcus
a) Type I and III collagen (AIIMS -99)
b) Type III collagen 12. The position of mucogingival line
c) Type IV collagen d) Type I collagen a) Shifts apically with age
(MAN -01) b) Shifts coronally with age
4. Cells of the spinous Layer are generally c) Constant throughout the life
a) Larger than the basal cells d) None of the above
b) Smaller than the basal cells (AP -2K)
c) Same size as that of basal cells 13. Merkel cells are found in which of the following
d) None of the above tissue?
(MAN -01) a) Epidermis b) Papillary layer of dermis
5. Epithelial cells which ultimately keratinize are c) Reticular layer of dermis
known as: d) Hypodermis
a) Keratinizing cells (KCET-2011)
b) Melanocytes 14. Epithelium of oral mucous membrane is
c) Non -keratinizing cells a) Keratinized b) Non keratinized
d) Clearcells c) Ortho, para and non keratinized
(MAN -2K) d) Only para keratinized
6. The major cells seen in gingiva are (AP -01)
a) Fibroblast b) Odontoblasts 15. Masticatory mucosa in the oral cavity covers the
c) Cementoblasts d) Merkel cells a) Floor of the mouth and soft palate
(MAN -02) b) Alveolar mucosa and vestibular fornix
7. Biomechanism that unites the epithelium to the c) Lip and cheek
tooth surface is d) Gingiva and hard palate
a) Internal basal lamina (KAR -01)
b) Junctional epithelium 16. Color of the normal gingiva is
c) Reduced enamel epithelium a) Pink b) Red
d) Epithelial attachment c) Coral pink d) None of the above
(MAN -01) (AP -99)
8. In the tongue, bitter taste is more perceived at: 17. The red zones of Lips have:
a) Tip b) Sides a) 30% sebaceous glands
c) Back d) Front b) 75% sebaceous glands
(KAR -00) c) very small number of sebaceous glands, if any

1) A 2) C 3) C 4) A 5) A 6) A 7) A 8) C 9) C 10) A 11) D 12) C 13) A


14) C 15) D 16) C 17) C
Dental ;lut.,e

d) absolutely no sebaceous glands c) Circumvallate d) Foliate


(AIIMS -90) (COMEDK -04)
18. The connective tissue of the gingiva is known as 27. Supporting cells of taste buds are called as
the: a) Sustenticular cells b) Taste cells
a) Lamina dura b) Dental cuticle c) Von ebner cells d) Acini
c) Lamina propria d) Fibroblasts (COMEDK -04, 05)
(KAR -97, KAR -98) 28. Which of the following is non-keratinocyte?
19. The thinnest epithelium of oral cavity is found in a) Langhan's cell b) RBC
the: c) Langerhans cell d) Grey cell
a) Buccal gingiva b) Lingual gingiva (COMEDK -04)
c) Oral surface of lip 29. Stratum germinatum is
d) Sublingual mucosa (floor of mouth) a) Stratum basale b) Stratum granulosum
(KAR - 2K, AI PG- 05) c) Stratum basale and parabasal spinous cells
20. Elongated rete pegs are seen in: d) Stratum spinosum
a) Alveolar mucosa b) Floor of the mouth
c) Attached gingiva d) Buccal mucosa 30. Which of the following is correct
(AIPG -96) a) Non-keratinized epithelium is characterized by
21. Difference between the skin and mucosa of cheek absence of stratum granulosum and stratum
is: corneum. The surface cells are nucleated
a) Thin lamina and non keratinized mucosa of cheek b) Parakeratinised epithelium is characterized by
b) Keratinized mucosa of cheek superficial cells with pyknotic nuclei and absence
c) Rete pegs of stratum granulosum
d) Thick lamina propria on skin of cheek c) In ortho keratinisation the superficial cells lose
(AIIMS -89) their nuclei, but stratum granulosum is present
22. Epithelium of the inner surface of the gingival d) All of the above
sulcus:
a) Has no rete pegs b) Has prominent rete pegs 31. Jacobson's organ is
c) Is keratinized d) Is para-keratinized a) Auxillary olfactory sense organ
(PGI -01) b) Ellipsoid/Cigar shaped
23. Minor salivary glands are present in the submucosa c) Undergoes degeneration after 4 months of
through out the oral mucosa except for the intrauterine life
a) Gingiva and anterior part of hard palate d) All of the above
b) Tongue and soft palate
c) Retromolar area and hard palate 32. Stippling is
d) Lip and posterior third of the hard palate a) Due to alternate elevations and depressions in
(UPSC -01) epithelium
24. Keratohyaline granules are more evident in b) Functional adaptation to mechanical impact
a) Keratinised b) Non keratinized c) Disappeared in progressing gingivitis due to edema
c) Parakeratinised d) Orthokeratinized and this change is reversible
(COMEDK -04) d) All the above
2 5. Bu Lk of Lamina propria of the gingiva is made of
collagen type 33. Langerhans cell is
a) I b) II a) Restricted to zones of orthokeratinisation
c) III d) IV b) Involved in immune response
(COM EDK -04) c) Functions is similar to macrophages by picking up
26. Partially keratfoized papilla is antigen and presenting it to lymphocytes
a) Fungiform b) Filiform d) All of the above

18} C 19} D 20} C 21} A 22} A 23} A 24} D 25) A 26} B 27) A 28} C 29} C 30} D
31} D 32} D 33} D
r ORAL HISTOLOGY

34. The function of merkel cells is 43. Vonebners glands open into which papilla:
a) Sensory function b) Secretory function a) Foliate b) Filiform
c) Neuro sensory d) Nutritive function c) Circumvallate d) Fungiform
(AP-06)
35. The vermilion border requires frequent moistening 44. Buccal mucosa is -
because a) Keratinized b) Non-keratinized
a) It contains more No. of sweat glands c) Parakeratinisedd) Orthokerati nized
b) It contains more No. of sebaceous glands (COMEDK- 09)
c) It contains less number of sebaceous gland 45. The high level clear cell present in the oral
d) It contains less number of sweat glands epithelium is:
a) Melanocyte b) Lymphocyte
36. The mucous membrane of cheeks and lips c) Merkel cell d) Langerhans cells
a) Is attached to buccinator muscle in cheeks (KCET- 09)
b) Is attached to orbicularis oris in lips 46. The oral epithelium is attached to the enamel via
c) A & B are correct d) None of the above a) Reticular fibers b) Collagen fibers
c) Hemidesmosomes d) Desmosomes
37. After the eruption of crown, the reduced enamel is (AP- 09)
known as 47. Basement membrane
a) Primary attachment epithelium a) Contains lamina lucida & Lamina dura
b) Secondary attachment epithelium b) Consists luratihyaline granules
c) Primary enamel cuticle c) Consists of lamina densa and lamina lucida
d) Reduced enamel epithelium d) Consists of desmosomes and hemi-desmosomes
(COM EDK- 10)
38. "Epithelial attachment" is 48. 'Stratum Germinativum' of the oral epithelium is
a) Attachment of ameloblasts to the tooth the term given to
b) Microscopically, it resembles basal lamina a) Basal cell layer b) Granular cell layer
c) Derived from secondary attachment epithelium c) Spinous cells and granular cells
d) All of the above d) Basal cells and parabasal spinous cells
(UPSC- 09)
39. Dendritic cell located in the stratified squamous 49. Membrane coating granules may
keratinizing epithelium of the oral cavity include a) Appear as circular with an amorphous core in
a) Keratinocytes b) Mast cells keratinized epithelium
c) Melanocytes d) Myoepithelial cells b) Originate from golgi system
(UPSC -01) c) Packed with tonofilaments
40. The anatomic crown is shorter than the clinical d) Helps in adhesion of fully lkeratinized epithelial cells
crown ofa tooth in which of the following instances: (COMED -10)
a) Impaction b) Gingivitis 50. Which papillae are completely keratinized-
c) Occlusal wear d) Gingival recession a) Fungiform b) Filiform
(AIPG -03) c) Circumvallate d) Foliate
41. Circumvallate papillae are supplied by (COMEDK- 09)
a) Lingual nerve b) Glossopharyngeal nerve 51. All of the following is lined by Stratified Squamous
c) Hypoglossal nerve d) Vagus Epithelium, except?
(COMEDK, PG! - 05) a) Lips b) Tongue
42. Masticatory mucosa is: c) Roof of the soft palate
a) Para keratinised b) Ortho keratinised d) Oropharynx
c) Non keratinised d) Sub keratinised (PG!- 08)
(AP-05)

34) C 35) C 36) C 37) A 38) D 39) C 40) D 41) B 42) B 43) C 44) B 45) D 46) C
47) D 48) D 49) B 50) B 51) C
Dental ;lut.,e

52. Protein making up the bulk of keratohyaline


granules in stratum gran ulosum of keratinized
epithelium is
a) Involucrin b) Vinculin
c) Filagg ri n d) Nectin
(COMEDK-2013)
53. Long connective tissue papillae and keratinized
epithelia are a feature of these parts of oral mucosa
a) gingiva and alveolar mucosa
b) hard palate and gingiva
c) buccal and alveolar mucosa
d) hard and soft palate
(AP-2013)
54. The range of level of fluoride secreted by the glands
into the mouth is
a) 0.004-0.005 ppm b) 0.006-0.007 ppm
c) 0.007-0.05 ppm d) 0.007-0.08 ppm
(COMEDK -2013)
55. Least pain sensitivity in oral cavity is in which
mucosa
a) Gingiva b) Buccal mucosa
c) Labial mucosa d) Dorsal mucosa of tongue
(AIIMS NOV- 13)
56. False statement about melanocytes is
a) Melanocytes arise embryologically from the neural
crest epithelium
b) Melanocytes lack desmosomes and tonofilaments
c) Melanocytes lack dendritic processes
d) Melanin is synthesized within the melanocytes
(AP-14)
5 7. The intermediate filaments of epithelium are
a) Glial filaments b) Desmin
c) Vimentin d) Cytokeratin
(APPG-2015)

52) C 53) B 54) C 55) B 56) C 57) D


r ORAL HISTOLOGY

9. ORAL MUCOUS MEMBRANE - ANSWERS


1. 'A' [Orban's 11th ed 307/ 12th ed 239/ 13th ed 270) 4. 'K [Orban's 11th ed 272/ 12th ed 219/ 13th ed 248-
Circumvallate papillae are the largest papillae of the 249)
tongue. Taste buds are numerous in lateral surface of Keratinised epithelium contains stratum basale,
vallate papillae and are absent in filiform papillae. stratum spinosum, stratum granulosum and stratum
corneum. The cells become larger and flatter in size
2. 'C' [Orban's 1tth ed 297 / 12th ed 234/ 13th ed 254) from basale to corneum. Layers in non-keratinised
Dendritic cells (or) epithelium are stratum basale, stratum intermedium
Synthesize melanin and stratum superficiale. The epithelial cells retain
melanocytes
their nuclei in non-keratinised epithelium, nuclei
Langerhans cell Immune response
become pyknotic in parakeratinised epithelium and
Neural pressure sensitive the nuclei are disappeared in keratinised epithelium.
Merkel cell
receptor cell.
5. 'K [Orban's 11th ed 281/ 12th ed 221/ 13th ed 253)
Melanocytes, langerhan's cells and merkel cells are
called non-Keratinocytes or clear cells. 6. 'A' [Carranza 11th ed 22)

3. 'C' [Orban's 1tth ed 272/ 12th ed 219/ 13th ed 342) 7. 'A ' [Orban's 11th ed 310/ 12th ed 241/ 13th ed 242)
The basal lamina is made up of a clear zone (lamina The internal basal lamina of J.E unites the epithelium
lucida) present just below the epithelial cells, a to tooth whereas the external basal lamina of J.E
dark zone (lamina densa) beyond lamina lucida and unites the epithelium to connective tissue of gingiva.
connective tissue. Lamina densa contains type IV
collagen and lamina lucida contains laminin. 8. 'C' [Orban's 11th ed 308/ 12th ed 240/ 13th ed 270)
• Bone Taste Perceived Papilla Nerve
Type
• Gingivone sensations at associated mediated
one
• Principal fibers of periodontal ligament Tip of
Sweet Fungiform Chordatympani
Type tongue
Cartwolage
two Lateral
Salt border of Fungiform Chordatym pani
• Reticular ti bres
tongue
• Granulation tissue
(Note:- Among 10 types of Ehler dan los Lateral
Glosso
Type syndrome type IV (ecchymotic) will have Sour border of Foliate
pharyngeal
three defective type III collegen. The patients tongue
may have rupture of large arteries as well Posterior
Circum Glosso
as intestine resulting in life threatining Bitter part of
val late pharyngeal
condition tongue
• " Floor on the four" Basement membrane
or floor is supplied by type four. 9. 'C' [Orban's 11th ed 281/ 121h ed 221/ 13th ed 254)
Collagen is arranged in chicken-wire
configuration. 10. 'K [Orban's t t th ed 279/ 12th ed 220/ 13th ed 249)
Type
(Note:- The syndrome associated with a
four
defect in type IV collagen in basement 11. 'D' [Orban's 11th ed 309/ 12th ed 240/ 13th ed 271)
membrane of renal glomeruli is Alport Sulcular epithelium, junctional epithelium and
Syndrome. The patient exhibits interdental col are non-keratinised areas in gingiva.
hematuria and renal diseases)
Sulcular epithelium lacks retepegs.
Type Present in anchoring fibrils of lamina densa
seven
Dental ;lut.,e

12. 'C' [Carranza 11th ed 13] The papillae of the gingiva are long, slender, numerous
The attached gingiva is separated from the alveolar with few elastic fibres. The papillae of alveolar mucosa
mucosa by mucogingival line. The free gingiva is are short with many elastic fibres.
separated from the attached gingiva by free gingival
groove. 21. 'A' [Orban's 11th ed 299/ 12th ed 236/ 13th ed 265]
The mucosa of the cheek is non-keratinised. Minor
13. 'A' [Orban's 11th ed 297 / 13th ed 254] salivary glands are found between bundles of
In addition to keratinocytes, oral epithelium also buccinator muscle. The cheek, lateral to the corner of
contains melanocytes (derived from neural crest), the mouth, contain isolated sebaceous glands called
langerhans cell (derived from bone marrow and merkel Fordyce's spots
cell (derived from neural crest). Merkel cell are neural
pressure sensitive receptor cells .. 22. 'A' [Orban's 11th ed 309/ 12th ed 240/ 13th ed 271]

14. 'C' [Orban's 11th ed 269/ 12th ed 216/ 13th ed 244] 23. 'A' [Orban's 1ith ed 284/ 12th ed 227 / 13th ed 256,
262]
15. 'D' [Orban's 11th ed 283/ 12th ed 225/ 13th ed 239] Antero lateral part orfatty zone of hard palate contains
Oral mucosa is divided into: adipose tissue in the submucosa. The posterolateral or
i) Keratinised areas glandular zone contains salivary glands.
• Masticatory mucosa (gingiva, hard palate)
24. ' D' [Orban's 11th ed 279/ 12th ed 220/ 13th ed 249]
• Vermilion border of lip.

25. 'A' [Carranza 11th ed 21]


ii) Non keratinised areas
• Lining mucosa (lips, cheeks, alveolar mucosa, 26. 'B' [ Orban's 11th ed 304/ 12th ed 237/ Tencate 8th
vestibula r fornix, ventral surface of tongue and ed 304]
floor of the mouth).
Present on the latera l border of
• Specialized mucosa (Dorsum of tongue) Foliate papillae the posterior parts of tongue, and
contains taste buds.
16. 'C' [Orban's 11th ed 293/ 12th ed 232/ 13th ed 260]
The vascular supply, thickness and degree of Keratinised, thread like and do not
Filiform papillae
contain taste buds.
keratinisation and tlhe presence of pigment containing
cells produce the cora l pink color of gingiva. Fungiform Mushroom shaped, round, reddish
papillae with few taste buds.
17. 'C' [Orban's 11th ed 298/ 12th ed 235/ 13th ed 264] Present infront of the 'V' shaped
The red zone (or) vermillion border is the transitional Sulcus terminalis. 8-10 in
zone between the skin of the lip and mucous membrane Circumvallate number, largest and contain many
of the lip. The papi llae of the laminapropria are long papillae taste buds. Vonebners salivary
reaching deep in to the epithelium and carry capillary gland ducts open into the trough
loops close to the surface, giving rise to red color of of circum vallate papilla.
the lips. This zone contains only occasional sebaceous
glands, subjecting it to frequent drying. 27. 'A' [DiFiore's atlas of Histology with functional
correlations 12th ed 296]
18. 'C' [Orban's 11th ed 265/ 12th ed 213/ 13th ed 242]
28. 'C' (Orban's 11th ed 297/ 12th ed 234/ 13th ed 254]
19. 'D' (Orban's 11th ed 301/ 12th ed 236/ 13th ed 266]
The mucosa of floor of the mouth (sublingua/ mucosa) 29. 'C' (Orban's 11th ed 269/ 12th ed 216/ 13th ed 246]
and inferior surface of tongue are thin . The basal cells and suprabasal spinous cells undergo
mitosis, synthesize DNA, therefore called as "Stratum
20. 'C' [Orban's 11th ed 292/ 12th ed 233/ 13th ed 262] germinatum". Stratum spinosum is active in protein
synthesis.
r ORAL HISTOLOGY

30. ' D' [Orban's 11th ed 282/ 12th ed 226/ 13th ed 254] 45. 'D' [Orban's 12th ed 221/ 13th ed 254]

31. 'D' [Orban's 11th ed 288/ 12th ed 230/ 13th ed 258] 46. 'C' [Orban's 12th ed 247I 13th ed 271]

32. 'D' [Orban's 12th ed 230-31/ 13th ed 260] 47. 'D' [Orban's 12th ed 216/ 13th ed 242]
Basal cells of oral epithelium are heavily packed with
33. 'D' [Orban's 111h ed 297 / 12 1h ed 234/ 13th ed 254] tonofilaments, which are adaptations for attachment.
They contain hemidesmosomes and desmosomes.
34. 'C' [Orban's 11th ed 297 / 12th ed 234/ 13th ed 254]
attach epithelium to the
35. 'C' [Orban's 11th ed 299/ 12th ed 235/ 13th ed 264] Hemidesmosomes
connective tissue
Desmosomes connect adjacent basal cells
36. 'C' [Orban's 11'h ed 301/ 12'h ed 236/ 13th ed 265)
48. 'D' [Orban's 12th ed 216/ 13th ed 246]
37. 'A' [Orban's 11th ed 310/ 12th ed 241/ 13th ed 273] Basal and parabasal spinous cells are referred to as
After the ameloblasts finish formation of the enamel 'Stratum germinativum', but only the basal cells can
matrix, they leave a thin membrane on the surface of divide.
the enamel ca lled primary enamel cuticle. After this
the epithelial enamel organ is reduced to a few layers 49. 'B' [Journal of ultrastructure research, 60(2) Pg
of cells called reduced enamel epithelium; which 212/ Tencate 8th ed 287]
cover the entire enamel. Once the tip of the crown has
emerged, the reduced enamel epithelium is termed as 50. 'B' [Orban's 12th ed 23 7/ 13th ed 26 7]
primary attachment epithelium.
51. 'C' [Orban's 12th ed 236/ 13th ed 267]
38. 'D' [Orban's 12th ed 247-48/ 13th ed 273]
52. 'C' [Human Oral mucosa: development, structure
39. 'C' [Orban's 11th ed 297 / 12th ed 234/ 13th ed 253] and function Pg 39/ Carranza 11th ed 16]
The protein ma king the bulk of keratohyalin granules
40. ' D' [Orban's 12th ed 240-41/ 13th ed 277] is fi llagrin. Also sulfur-rich protein loricin is present.
The part of the t ooth covered by enamel is called Fillagrin and loricin are useful markers of epithelial
anatomic crown and the part of the tooth exposed differentiation. Involucrin contributes to the
in the oral cavity is called clinical crown. In gingival 'Cornified cell envelope' in the cell membrane of
recession the root is exposed making anatomic crown keratohyalin cells.
shorter than clinical crown.
53. 'B' [Orban's 13th ed 256, 262]
41. 'B' [Orban's 11th ed 308/ 12th ed 240/ 13th ed 270] Long connective tissue papillae are characteristic of
gingiva . This helps in histologic differentiation of
42. 'B' [Tencate 5th ed 378/ 8th ed 301, 302, Tab 12-6] gingiva from alveolar mucosa. In alveolar mucosa , the
Epithelium of masticatory mucosa is freq uently papillae are quite low and non-keratinized. Among
orthokeratinized, although quite normally there are the options, hard palate and gingiva i.e. option B is
parakeratinized areas of the gingiva and occasionally appropriate.
of the palate.
54. 'C' [Fluoride in preventive dentistry by James R.
(Note:- Gingiva is parakeratiinized in 75%, keratinized Mellberg, Louis W. Ripa]
in 15%, and nonkeratinized in 10% of the population) The level of fluoride secreted by the glands into the
mouth depends upon many factors, but under normal
43. 'C' [Orban's 11th ed 360/ 12th ed 271/ 13th ed 269] conditions, it is very low andl in the range of 0.007 to
0.05 ppm.
44. 'B' [Orban's 121 h ed 235/ 13th ed 265]
Dental ;lut.,e

55. 'B' [James Avery 3rd ed 282]


Sensitivity of lips and oral mucosa
Sensation Greatest Moderate Least
Pain Lips, Pharynx, base of the tongue, teeth Anterior Tongue, Gingiva Buccal mucosa
Heat Lips Anterior teeth Ventral tongue, palate
Cold Lips, posterior palate Base and ventral tongue Dorsum tongue, Buccal mucosa
Touch Lips, t ip of the tongue, anterior palate Gingiva Base of the tongue, Buccal mucosa

56. 'C' [Ten Cate's 7th ed 310]


Melanocytes appear as clear cells in hematoxylin and eosin stained sections. Silver stains reveal a spider like
(dendritic) appearance.

5 7. 'D' [Orbans 13th ed 244]


Cytokeratins form cytoskeleton of all epithelia l cells, along with microfilaments and microfibrils. They are termed as
intermediate fi laments as their diameter (7-11 nm) is intermediate between larger micro tubules (25n m) and smaller
microfi laments (4-6 nm).

Functions:
1. Provide mechanical linkages and distribute forces over a wide area.
2. Function as stress bearing structures and are important in maintaining cell shape.
3. There are 20 types classified into Type-I (basic; 1-8) and type II (acidic; 9-20)
r ORAL HISTOLOGY

10. TOOTH ERUPTION, SHEDDING AND MISCELLANEOUS


1. In resorption of roots of the primary teeth, the c) Shedding of deciduous teeth is delayed in the
dental pulp absence of their successor teeth
a) Plays a passive role d) None of the above
b) Becomes a fibrotic non-vital mass
c) Initiates resorption from the inner surface of roots 9. Tooth buds of permanent mandibular incisors are
d) Aids in formation of secondary dentin slowing situated:
down resorption a) Lingual to primary teeth
(AIIMS -2K) b) Inferior to primary teeth
2. Movement of tooth to close the intermaxillary gap c) Labial to primary teeth
during eruption is due to d) A and B
a) Physiological tooth migration (AIPG -02, AIIMS -95)
b) Active eruption 10. Hammock ligament is present:
c) Passive eruption a) Between temporal and sphenoid bone
d) None of the above b) Between hamular notch and mandible
(AP -01) c) In apical area of a tooth
3. Tooth eruption is due to d) As a part of deep cervical fascia
a) Osteoclastic activity (AIIMS -94)
b) Proliferation of cells at crypt 11. Epithelial attachment is derived from
c) Exfoliation of primary tooth a) Reduced enamel epithelium
d) Eruptive forces b) Denta l papilla
(AIIMS -97) c) Inner enamel epithelium
4. Teeth with out antagonist if erupt is called d) Dental sac
a) Stable b) Over erupted (PGI -02)
c) Supra erupted d) Submerged 12. Which enzyme is associated with the mineralization
(AP -03) a) Acid phosphatase
5. Resorption of dental tissue is caused by b) Alkaline phosphatase
a) Osteoclasts b) Osteoblasts c) Adenosine triphosphate
c) Odontoclasts d) Odontoblasts d) Amino peptidase
(AP -2K) (MAN -01)
6. The most accepted theory of tooth eruption is 13. A suitable fixative for a routine biopsy specimen is
a) Hydrodynamic theory a) 10% formalin b) 40% formalin
b) Clone theory c) 20% formalin d) 10% alcohol
c) Periodontal ligament traction theory (KAR -2K; AIIMS -94)
d) Vascular theory 14. Oral tissue for microscopic examination is prepared
(COMEDK -04) by
7. Gubernacular canal guides the eruption of a) Embedded in paraffin and sectioned
a) Primary teeth b) Permanent teeth b) Embedded in paradion and sectioned
c) Both A and B d) None of the above c) Frozen and sectioned
d) Specimens ground into thin sections
8. Which of the following statement is false e) All of the above
a) Resorption of roots of deciduous incisors and
canines begins on their Lingual surfaces 15. Dehydration of s pecimen is carried out by
b) Resorption of roots of deciduous molars begins on a) By using 10% formalin
their inner surfaces b) By using liquid or solid Co 2

1) A 2) B 3) D 4) C 5) C 6) C 7) B 8) D 9) D 10) C 11) A 12) B 13) A


14) E 15) C
Dental ;lut.,e

c) By using increasing percentages of alcohol 24. Osteoblasts appear basic gram staining due to:
d) By using 5% nitric acid a) Increased ribonucleic acid
b) Acidic ground substance
16. Extracted teeth should be preserved in c) Increased mitochondria
a) Saline b) Xylene d) Presence of basic ground substance
c) 10% Formalin d) 10% alcohol (AP-06)
25. Which type of collagen fibers demonstrates
17. Frozen sections can be used for the demonstration "Chicken wire" configuration?
of a) Type I b) Typell
a) Fat b) Mucin c) Type IV d) Type IV
c) Cartilage d) Iron (COMEDK- 07)
(KAR - 99,03) 26. Which of the following factors are primarily
18. Specific stain for fat cells is contributory to the shedding of deciduous tooth?
a) Eosin & haematoxylin a) Hydrostatic b) Vascular pressure
b) Carbo[ fuschin c) Oncotic pressure d) Masticatory pressure
c) Sudan III d) Cresyl violet (COMEDK- 07)
(TNPSC -99) 27. The deciduous teeth can be differentiated from
19. The cells, which are responsible for removal of bone permanent teeth by:
matrix in bone resorption, are a) Enamel prismatic arrangement
a) Osteoblast b) Fibrocytes b) Dentinal t ubule arrangement
c) Polymorphs d) Osteoclasts c) Primary and secondary dentin
(KAR - 01) d) Mineral content
20. The apical foramen of a fully developed permanent (AIIMS-06)
tooth is usually lined by 28. Calcified tissue is absent in:
a) Dentin b) Cementum a) Enamel b) Dentin
c) Epithelial rest cells d) The Epithelial diaphragm c) Cementum d) Pulp
(APPSC -99) (AP- 07)
21. Gubernacular canal and gubernacular cord are seen 29. The first sign of future deciduous teeth as a
in relation to thickening of oral ectoderm appears at about
a) Permanent teeth b) Deciduous teeth a) 25th day of Intra uterine life
c) Succedaneous teeth d) none of the above b) 34th day of Intra uterine life
c) 56th day of Intra uterine life
22. Which of the following is incorrect? d) 20 wks of Intra uterine life
a) During shedding resorption of deciduous anteriors (KAR-03)
occurs on lingual surface of roots. 30. The maximum rate of eruption, as the tooth
b) Odontoclasts are characterized by high level of cusps reach the surface epithelium is around
enzyme "acid phosphatase". _ _ _ _ IJm/day.
c) Upper lateral incisors are most commonly retained a) 100 b) 75
deciduous teeth. c) 50 d) 25
d) Submerged/ ankylosed teeth are permanent teeth. (COMEDK-2011)
31. In mineralization which one of the following is
23. Accentuated incremental lines in dentin are known as Epitaxy?
a) Incremental lines of retzius a) Booster Mechanism b) Seeding Theory
b) Incremental lines of von Ebner c) Alkaline Phosphatase Theory
c) Contour lines of owen d) Cartier/s Adenosine Triphosphate Theory
d) None of above. (AP-2013)
(MAN-02)

16) C 17) A 18) C 19) D 20) B 21) C 22) D 23) C 24) A 25) D 26) D 27) D 28) D
29) C 30) B 31) B
r ORAL HISTOLOGY

10. TOOTH ERUPTION, SHEDDING AND MISCELLANEOUS - ANSWERS


1. 'A' [Orban's 11th ed 398/ 12th ed 299/ 13th ed 351)
• Consists of single layer of
Resorption of roots of primary teeth (shedding) is
Inner enamel colum narr cells called ameloblasts
carried out by highly specialized cells identical to
epithelium • Present on concave surface in
osteoclasts called odont oclasts. It has high levels
of acid phosphatase enzyme. These are found most cap and bell stages
commonly on surfaces of the roots in relation to the The peripheral cell layer on the
Outer enamel
advancing permanent tooth . convex surface in cap and bell
epithelium
stages.
Osteoclasts/odontoclasts exhibit an intense A thin membrane on the surface of
activity of the following enzyme (KERALA-2015) Primary
enamel. It is formed by ameloblasts
a) Alkaline phosphatase enamel
after enamel matrix is completely
b) Adenosine triphosphatase cuticle
formed.
c) Esterase d ) Acid ph osphatase
Nasmyth's It is remnant of primary enamel
2. ' B' [Orban's 11th ed 314/ 12th ed 245/ 13th ed 274) membrane cuticle after eruption.
Reduced Modified epithelial organ in which
Active Actual movement of the teeth towards enamel the cells are reduced to few layers
Eruption the occlusal plane. epithelium of flat cuboid cells.

Passive Exposure of the crown due to apical Primary After the crown is emerged into
Eruption shift of the gingiva. attachment oral activity the reduced enamel
epithelium epithelium is termed as primary
3. 'D' [Orban's 13th ed 339-340) (Junctional attachment epithelium
epithelium)
4. 'C' [Orban's 13th ed 344) During passive eruption, the
primary attachment epithelium
5. 'C' [Orban's 11th ed 392/ 12th ed 295/ 13th ed 351) Secondary is separated from the enamel. At
attachment this stage, the primary attachment
6. 'C' [Orban's ttth ed 381/ 12th ed 289/ 13th ed 340) epithelium epithelium is replaced by secondary
attachment epithelium, which is a
7. ' B ' [Orban's 11th ed 377/ 12th ed 285/ 13th ed 336) derivative of gingival epithelium.
Gubernacular canal contains connective tissue mass
called gubernacular cord, which have a function in 12. 'B' [Orban's 11th ed 452/ 12th ed 339/ 13th ed 393)
guiding the permanent tooth as it erupts.
13. 'K [Orban's 11th ed 474/ 12th ed 352/ 13th ed 411)
8. 'D' [Orban's 1tth ed 387/ 12th ed 295/ 13th ed 348)
14. ' E' [Orban's 12th ed 349-50/ 13th ed 410)
9. 'D' [Orban's 11th ed 373/ 12th ed 282/ Tencate 8th Option 't,:. is most commonly used method of preparing
ed 233, 235) soft tissues for study.

10. 'C' [Orban's 1 tth ed 380/ 12th ed 289/ 13th ed 339) Option 'B' and Option 'D' are used for calcified tissues
like bone and teeth.
11. 'A' [Orban's 11th ed 310/ 12th ed 244/ 13th ed 273)
If the paraffin and paradion would destroy the tissue,
then Option 'C' i.e., frozen section method is used.
Dental ;lut.,e

15. 'C' (Orban's 11th ed 471/ 12th ed 350/ 13th ed 411] 28. 'D' [Tencate 5th ed 69/ Orban's 13th ed 2]
The specimen is dehydrated by passing through
increasing percentages of alcohol i.e., 40%, 60%, 29. 'C' (Orban's 12th ed 22/ 13th ed 25]
80%, 95% and abso lute alcohol (100%).
30. ' B' [Tencate 7th ed 271/ 8th ed 236]
16. 'C' (Orban's 11th ed 477/ 12th ed 354/ 13th ed 415] The rate of eruption depends on the phase of movement
The teeth used for ground sections should not be of tooth . During the intraosseous phase, the rate
allowed to dry out after extraction, because drying averages 1 to 10 microns per day; it increases to about
makes the hard tissues brittle and the enamel may 75 microns per day once the tooth escapes from its
chip off in the process of grinding. To avoid this, the bony cell. This rate persists until the tooth reaches the
extracted teeth are preserved in 10% formalin. occlusal plane, indicating that soft connective tissue
provides little resistance tooth movement.
17. 'A' [Orban's 1t•h ed 448/ 12'h ed 332/ 13th ed 415)
31. 'B' (Orban's 13th ed 219]
18. 'C' (Orban's 11th ed 448/ 12th ed 332/ 13th ed 392] Theories of calcification
a) Nucleation theory
19. 'D' (Orban's 11th ed 252/ 12th ed 204/ 13th ed 215] b) Role of matrix vesicles
c) Role of alkaline phosphatase
20. 'B' (Orban's 11th ed 141/ 12th ed 110/ 13th ed 121-
122] Nucleation theory is based on the concept of seeding
or epitaxy. Epitaxy means the orientated overgrowth of
21. 'C' (Orban's 11th ed 377/ 12th ed 285/ 13th ed 336] bone mineral on certain well crystallized areas of the
The gubernacular canal and cord help in guiding the organic matrix. This theory differs from other theories
permanent teeth during the process of eruption. of mineralization which assign no active role to the
organic matrix in the mineralization process.
22. 'D' (Orban's 11th ed 405/ 12th ed 302/ 13th ed 357]
Submerged teeth are deciduous teeth.

23. 'C' (Orban's 11th ed 117/ 12th ed 90/ 13th ed 101]


Option 'P{ is seen in enamel. Option 'B' indicates the
normal incremental Lines or imbrication lines in dentin.

24. 'A' (Orban's 13th ed 211]


Osteoblasts exhibit abundant and well developed
protein synthetic organelles. The intense cytoplasmic
basophilia is due to abundance of rough endoplasmic
reticulum (endoplasmic reticulum studded with
ribosomes).

25. 'D' [Tencate 5th ed 366, See table/ 8th ed 60]

26. 'D' [Tencate 5th ed 302, 305/ Orban's 13th ed 355]

27. 'D' [Check Explanation Below]


The main crystallite orientation and prism arrangement
was fo und to be identical in deciduous and permanent
enamel. Deciduous enamel has a lower mineral content
and therefore a higher amount of spaces in it and is
more susceptible to dental caries and dissolves in acid
more rapidly.
ORAL HISTOLOGY SYNOPSIS

ORAL HISTOLOGY - SYNOPSIS

DEVELOPMENT AND GROWTH OF TEETH

1. The outer enamel epithelium is separated from the dental sac, and the inner enamel epithelium from the dental
papilla, by a delicate basement membrane.

2. Both enamel knot and enamel cord of stellate reticulum are temporary structures and act as a reservoir of dividing
cells for the growing enamel organ.

3. All the 4 layers (IEE, OEE, Stratum intermedium and stellate reticulum) are seen in bell stage of tooth development.

4. The cervical portion of enamel organ gives rise to the epithelial root sheath of Hertwig. Hert.wig's root sheath consists
of outer and inner enamel epithelia only, and it does not include stratum intermedium and stellate reticulum.

5.
Stages Results of disturbance
• Lack of initiation may result in absence of single tooth or multiple teeth (anodontia ). The upper
lateral incisors followed by third molars and lower second premolars are common ly involved.
• Abnormal initiation may result in development of supernumerary tooth . Mesiodens is the most
Initiation
common supernumerary teeth. Maxillary 4 th molar is the second most common supernumerary
tooth.
• Sometimes fused or geminated teeth are formed.
• This phase reaches its highest development in the bell stage of enamel organ.
Histo
differentiation • Dentinogenesis imperfecta or formation of atypical dentin is example of disturbance of
histodifferentiation stage.
• The advanced bell stage is important stage of morphodifferentiation. This stage establishes
the morphologic pattern, or basic form and relative size of future tooth.
Morpho
differentiation • Supernumerary cusps (talon cusp ) or roots, twinning, loss of cusps or roots, or malformed or
peg shaped tooth (Hutchinson's incisor), dens in dente and macrodontia are examples of
ab normal morp hodifferentiation.
• Both enamel hypoplasia and hypo calcification can occur as a result of an insult to the cells
Apposition responsible for the apposition stage.
• Intrinsic staining or concrescence is due to abnormal apposition.

DENTIN

1. Types of dentin
a) Based on time of formation
• Primary dentin (includes mantle and circumpulpal dentin)
• Secondary dentin

b) Based on structure
• Peritubular
• Intertubular
• Predentin
Dental ;lut.,e

c) Dentin associated with age changes


• Dead tracts
• Sclerotic dentin
• Reparative dentin

d) TYPES OF DENTIN

Predentin • 1•1 formed dentin and is not mineralized.


• Located adjacent to the pulp tissue and is 2 to 6 µm wide.
• It is the dentin that surrounds the dentinal tubules.

Peritubular • Highly mineralized than inter tubular dentin .


• It constricts the dentinal tubules to a diameter of lµm near DEJ .
dentin
• It is twice as thick in outer dentin than in inner dentin, but in decalcified sections, the tubule
diameter appears similar in inner and outer dentin because of loss of peritubular dentin.

Intertubular
• Forms the main body of dentin .

dentin • It is located between dentinal tubules.


• After decalcifi cation, inter tubular dentin is retained, whereas peritubular dentin is not .

• It is the 1•1 formed dentin fo the crown underlying the DEJ. It is the peripheral or outer most
Mantle dentin part of primary dentin.
• It is thus the area of initial dentin matrix formation .

Circum pulpal
• Represents the dentin formed prior to root completion .
dentin • It forms the bulk of primary dentin .
• The circumpulpal dentin contains slightly more mineral than mantle dentin .
Secondary dentin • Represents the dentin formed after root completion and contains fewer tubules than primary
(adventitious dentin .
dentin) • There is usually bend in the tubules where primary and secondary dentin interface.
• It is the dentin that is formed in response to extensive abrasion, caries, bacterial toxic
Reparative dentin products and chemicals from restorative materials. All these stimulate pulpal response leading
(tertiary or to reparative dentin formation.
response dentin) • Reparative dentin is characterized by having fewer and more twisted dentinal tubules than
normal dentin.
• In some cases, as a result of caries, attrition, abrasion, cavity preparation or erosion, the
odontoblastic processes may disintegrate and the empty tubules may become filled with air.
• They appear black in transmitted and white in reflected light.
Dead tracts
• The degenerative changes mainly appear in the area of narrow pulp horns, because of crowding
of odontoblasts.
• Dead tracts are considered as the initial step in formation of sclerotic dentin .
• Formation of sclerotic dentin is a protective mechanism .
Sclerotic or • In case of caries, attrition, abrasion or erosion, sufficient stimuli are generated to cause
transparent collagen fibres, calcium salts and apatite crystals to begin appearing in dentin tubules. This
dentin condition is prevalent in older individuals, especially in the roots.
• Sclerotic dentin appears light in transmitted light and dark in reflected light.

Interglobula r • Formed by small globular hypomineralised areas, that fails to fuse into homogenous mass.
dentin • Interglobular dentin forms in the crowns of teeth in the circumpulpal dentin just below the
mantle dentin.
Osteodentin • Seen in Vit. A deficiency during development.
ORAL HISTOLOGY SYNOPSIS

2. Dentino enamel junction: of outer surface ( near the pulpal cavity), the tubules
are larger in diameter and are loosely packed.
• The dentine-enamel junction is usually scalloped
and the convexities of the scallop are directed
8. The ratio of dentinal tubules per unit area on the pulpal
towards the dentin.
and outer surfaces of dentin is about 4:1. Near the pulpal
• In dentinogenesis imperfecta, the usual surface of the dentin, the number of dentinal tubules per
scalloping is absent, and this result in early loss of square millimeter varies between 50,000 to 90,000.
enamel. The dentin undergoes rapid attrition and
the occlusal surfaces of deciduous and permanent 9. There are more tubules per unit area in the crown than
molars are severely flattened. in the root.
• Lack of normal scalloping of the DEJ, passage of
many dentinal tubules into the enamel, formation 10. The dentinal tubules have lateral branches through out
of irregular dentin, increased tendency to form pulp dentin, which are termed canaliculi or microtubules.
stones, and hypermobility of the TMJ, resulting
in repeated dislocations of the jaw are features 11. The total volume of all permanent teeth pulp organs is
of "Ehlers-Danlos" syndrome. 0.38 cc, and the mean volume of a sing le adult human
pulp is 0.02 cc.
3. Cemento dentinal junction:
12. Molar pulps are 3-4 times larger than incisor cusps.
• Thecementodentinaljunction is smooth in permanent Canine has the longest pulp.
teeth. It is sometimes scalloped in deciduous teeth.
• Sometimes, a intermediate cementum layer, which 13. The average length of time of a primary pulp functions
does not exhibit the characteristic features of in the oral cavity is only about 8.3 yrs.
either dentin or cementum is seen in apical 2/3rds
of roots of molars and premolars and is rarely ENAMEL
observed in incisors and deciduous teeth. This layer
represents areas where cells of Hertwig's epithelial 1. On the cusps of human molars and premolars the
sheath become trapped in rapidly deposited dentin enamel attains a maximum thickness of about 2 to 2.5
or cementum matrix. mm, thinning down to almost a knife-edge at the neck
of the tooth.
4. Korff's fibres are described as the initial dentin
deposition along cusp tips. 2. The specific gravity of enamel is 2.8.

5. Dentin is elastic and subject to slight deformation. It is 3. The number of enamel rods has been estimated as
harder than bone and softer than enamel. Because of its ranging from 5 million in the lower lateral incisors to
low mineral salts, it is more radiolucent than enamel. 12 million in the upper l't molars.

6. De ntinal tubules: 4. The length of most rods is greater than the thickness
• The course of d!entinal tubules is straight near the of the enamel because of the oblique direction and
root tip and along the incisal edges and cusps. wavy course of the rods. The rods located in the cusps
But in the crown, they follow a S-shaped gentle (thickest part of enamel) are longer than those at the
curve. The first convexity of the S-shaped curve cervical areas.
is directed towards the apex of the tooth.
5. The rods measure about 5µm in breadth, 9µm in length
• The tubules start at right angles from the pulpal
and 4µm in diameter.
surface and end perpendicular to the dentinoenamel
and dentinocemental junctions.
6. The diameter of enamel rods increases from the dentino
enamel junction towards the surface of the enamel at
7. The ratio between the outer and inner surfaces of
a ratio of about 1:2.
dentin is about 5:1. Because of the larger surface area
Dental ;lut.,e

7. The bodies of enamel rods are nearer to occlusal and 12. Surface structures of enamel are structure less layer
incisal surfaces, whereas the tails point cervically. {30µm ), perikymata, rod ends, and cracks
• Structure less layer (hyper mineralized) of
8. The apatite crystals are parallel to long axes of the
enamel is 30µm in thickness and is found in 70%
rods in their bodies or "heads" and deviate about 65°
of permanent teeth and all deciduous teeth. It is
from this axis as they fan out into the tails of the
found mostly in the cervical areas of enamel and
prisms.
are least common over the cusp tips.

9. The enamel rods in the cervical region of deciduous • Perikymata are transverse, waves like grooves
teeth deviates in occlusal direction while the rods in and are believed to be external manifestations
the cervical region of permanent teeth deviates in of the striae of retzius. Because of undisturbed
gingival direction. development of the enamel prior to birth,
perikymata are absent in the occlusal parts of the
10. In the region of cusps/incisal ridges, the bundles of deciduous teeth, but are present in the postnatal
rods seem to interwine more irregularly. This optical cervical parts.
appearance of enamel is called gnarled enamel. • Enamel rod ends are sha llowest in the cervical
regions and are deepest near the incisal or occlusal
11. Hunter Schreger bands are alternating dark and edges.
light bands that can be seen in longitudinal ground
section. They represent regular change in the direction • Cracks are narrow, fissure like structures and are
of rods, which is regarded as a functional adaptation, believed to be outer edges of lamellae. If the cracks
minimizing the risk of cleavage of cleavage in the axial develop in the unerupted teeth, it becomes filled
direction under the influence of occlusal masticatory with developing cells and if the cracks develop after
eruption, it becomes filled with organic substances
forces.
of oral cavity.

13.
• Seen in enamel and are attributed to periodic bending of the enamel rods,
or variations in the basic organic structures.
Incremental lines of Retzius
• Perikymata are external manifestations of striae of retzius.
• Neonatal ring or neonatal line is an accentuated incremental line of retzius.

Incremental Lines of Von • Seen in dentin


Ebner or Imbrication lines • The distance between the lines vary from 4 to 8µm.
• Accentuated incremental line of Von Ebner that occurs in dentin as a result
of disturbances in the matrix and mineralization process.
Contour lines of Owen
• The neonatal line present in the dentin indicates an accentuated contour
line.
• Present both in dentin and enamel.
• In dentin: Concentric rings which appear in transverse section parallel to
enamelodentinal junction and they mark the coincidence of the primary
Schreger's lines
curvatures of dentinal tubules
• In enamel: A series of lines or bands in the enamel, visible in longitudinal
section of a human tooth by reflected light.
Incremental lines of Pickerill • Horizontal imbrication lines on the surface of tooth enamel.
Incremental Lines of Salter • Seen in cementum.
ORAL HISTOLOGY SYNOPSIS

14.
Enamel • Thin leaf like structures that extend from enamel towards DEJ
lamella. • Usually develops in planes of tension.
• 3 types, type'/( type B, type C. Type A are restricted to enamel while the Type 'B' and Type 'C' may
extend into dentin.
• Type 'I\ lamella is composed of poorly calcified rod segments.
• Type 'B' consists of degenerated epithelial cells. (Unerupted teeth)
• Type 'C' is present in erupted teeth and are filled with organic matter.
• Enamel lamella are considered as site of weakness in a tooth and may form a road of entry for
bacteria that initiate caries.

Enamel • Enamel tufts arise at DEJ and reach into the enamel (reverse to that of enamel lamella) to about
tufts 1/5th to 1/3'd of its thickness
• They are usually seen in horizontal sections (Hunter Schreger bands aire seen in longitudina l
sections).

Enamel • These arise from odontoblastic process and extend into enamel epithelium before hard substances
spindles are formed.
• The direction of enamel spindles in the enamel corresponds to the original direction of the
ameloblasts i.e., at right angles to the surface of dentin.
• Since the enamel rods are formed at an angle to the axis of ameloblasts, the direction of spindles
and rods is divergent.

15. Stratum intermediu.im is believed to play a role in production of the enamel itself, either through control of fluid
diffusion into and out of ameloblasts or by actual contribution of necessary formative elements or enzymes.

16. LIFE CYCLE OF AMELOBLASTS:


Morphogenic • Formation of 'terminal bars', which represent points of close contact between the cells.
stage

• Dentin formation begins in terminal phase of this stage.


Organizing stage • Reversal of functional polarity takes place by migration of the centrioles and Golgi bodies
from proximal to distal ends.

• Ameloblasts enter their formative stage after the first layer of dentin has been formed
• It is the stage of enamel matrix formation.
Formative stage
• The presence of dentin seems to be necessary for the beginning of enamel matrix
formation.

Maturative stage • Enamel maturation or full minera lisation occurs in this stage.

• After complete formation of enamel, the ameloblastic layer transform into reduced
Protective stage enamel epithelium, and this layer protects mature enamel (Anomalies will develop if
connective tissue comes in contact with the enamel).

• The reduced enamel epithelium induces atrophy of the connective tissue and helps in
Desmolytic stage eruption of tooth.
• Premature degeneration of REE may prevent eruption of a tooth.
Dental ;lut.,e

17.
Tomes processes • Tomes process is the projection of ameloblasts into enamel matrix.
• They show "Picket fence" arrangement
(Note: - "Picket fence" or "tomb stone" arrangement of basal layer of cells is characteristic
of odontogenic keratocyst).
• Formation of tomes process and terminal bars is the 1't step in enamel rod formation .
Tomes fibres
• These are cytoplasmic extensions of the odontoblasts.
(odontoblastic process)
Tomes granular layer. • Seen in dentin of root and is formed by coalescing and looping of the terminal
portions of the dentinal tubules.
• This zone increases from CEJ to the root apex.

18. In odontodysplasia (ghost teeth), both the apposition in the cell-free or weil's zone and is formed by
and maturation of enamel are disturbed. The teeth periphera l axons of myelinated nerve fibres.
shows "moth eaten", poorly calcified enamel
6. The most important function of pulp analogue is to
(Note: Moth eaten appearance of bone is seen in induce oral epithelial differentiation into dental lamina
chronic osteomyelitis and carcinoma of alveolus.) and enamel organ formation. The pulp analogue also
induces the developing enamel organ to become a
19. Enamel rods are keyhole or paddle shaped. The bulk particular type of tooth. The pulp organ cells produce
of the head of each rod is formed by one ameloblast, the dentin that surrounds and protects the pulp.
whereas the tail is formed by three other ameloblasts.
Thus each enamel rod is formed by 4 ameloblasts and 7. Based on t he structure, pulp stones are classified
each ameloblast contributes to four different rods into true denticles, false denticles and diffuse
calcifications.
PULP
True denticles are rare and are located close to the
1. The average size of apical fora men in the maxillary apical foramen. They contain dentinal tubules and are
teeth in the adult is 0.4mm. In the mandibular teeth, caused by inclusion of the epithelial root sheath into
it is slightly smaller, being 0.3mm in diameter. pulp.

2. Peripherally, the pulp is circumscribed by False denticles do not exhibit dentinal tubules and
specialized odontogenic region composed of appear as concentric layers of calcified tissue.
• Odontoblasts (dentin forming cells)
Diffuse calcifications appear as irregular calcific
• The cell-free zone (Weil's Zone) deposits in the pulp tissue, usually following
• The cell-rich zone. collagenous fibre bundles or blood vessels.

3. Fibroblasts are the most numerous cell types in the Denticles are often found in the coronal pulp, while
pulp tissue. Odontoblasts a re 2"d most commonest diffuse ca lcifications are usually found in the root
cell type. canal and less often in the coronal area.
4. The majority of the nerves that enter the pulp are non
8. Based on the location, pulp stones can be classified
myelinated. They are found in close association with
into free, attached and embedded denticles.
the blood vessels of the pulp and are sympathetic in
nature.
Free denticles are entirely surrounded by pulp tissue,
5. Parieta l layer or plexus of Rashkow is composed of attached denticles are partly fused with dentin and
myelinated nerve fibres. Plexus of Rashkow is present embedded denticles are entirely surrounded by dentin.
ORAL HISTOLOGY SYNOPSIS

iii) Cellular mixed stratified cementum


CEMENTUM
• Contains cells and extrinsic (Sharpey's fibres)
1. Cementum has the HIGHEST FLUORIDE content of all and intrinsic fibres and contains cells.
mineralized tissues and is WITHO UT NERVE SUPPLY. • It is a co-product of fibroblasts and
cementoblasts.
2. The uncalcified matrix of cementum is called
Cementoid. • It appears in the apical third of roots and in
furcation areas.
3. Human cementum is AVASCULAR.
iv) Cellular intrinsic fibre cementum:
4. The primary function of cementum is to furnish a
• Contains cells but no collagen (Sharpey's
medium for the attachment of collagen fibres.
fibres).
5. Cementum is thinnest at CEJ and thickest toward the • Formed by cementoblasts and in humans it fills
apex. resorption lacunae.
• First formed cementum
7. Cemento enamel junction:
• Do not contain cementocytes.
• It is the cementum that is formed • In about 60% to 65% of cases cementum overlaps
before the tooth reaches the the enamel.
occlusal plane. It covers the • In about 30% there is an edge-to-edge butt joint.
Acellular cervical third or half of root.
cementum • Sharpey's fibres make up most of the • In 5-10% the cementum and enamel fail to meet.
structure of acellular cementum,
which has a principal role in 8. In cases of cementum resorption, if the repair process
supporting tooth. re-establish the former outlirne of the root surface, it is
• More calcified than cellular called anatomic repair, and if the root outline is not
cementum reconstructed it is called functional repair.
• Formed after the tooth reaches the
occlusal plane. PERIODONTAL LIGAMENT
• It is more irregular and contains
cementocytes in lacunae. 1. The average thickness of periodontal ligament 0. 15 to
Cellular 0.38 mm.
cementum • Cellular cementum is more frequent
on the apical half.
2. Insufficient stimulation causes thinning of periodontal
• If is always thickest around the apex
a11d, by its growth, contributes to ligament, atrophy offibres, osteoporosis of the alveolar
the length of root. bone, and reduction in bone height. Insufficient
stimulation result from an open-bite relationship,
and absence of functiona l antagonists or unilateral
6. 'Schroeder' classified cementum into -
chewing habits that neg lect one side of mouth.
i) Acellular afibrillar cementum (present below CEJ)
• Contains no cells, no fibres. 3. The fibres arising from cementum and bone are joined
• Produced by cementoblasts. in the mid region of the periodontal space giving
• Found in coronal cementum. rise to a zone of distinct appearance, the so called
fotermedjate plexus. The intermediate plexus provides
a site where rapid modeling of fibres occurs.
ii) Acellular extrinsic fibre cementum
• Contains Sharpey's fibres and lacks cells.
• Produced by fibroblasts and cementoblasts.
• Found in the cervical third of roots.
Dental ;lut.,e

4. PRINCIPAL FIB RES OF PERIODO NTAL LIGAMENT

Alveolar crest group • Extend obliquely from the cementum to alveolar crest just beneath the junctional
epithelium.
• They prevent extrusion of tooth and resist lateral tooth movements.
• Their incision does not significantly increase the tooth mobility.
Horizontal group • Extend at rig ht ang les to the long axis of tooth from the cementum to the alveolar
bone.
• Maintains the mesiodistal width of the tooth .
Oblique group • Most numerous fibres of periodontal ligament.
(Note:- Dentogingival fibres are most numerous fibres of gingiva)
• Extend from the cementum in a coronal direction obliquely to the bone.
• They bear the brunt of vertical masticatory stresses and transform into tension on
the alveolar bone.
Apical group • The apical fibres radiate from the cementum to the bone at the fundus of socket.
• They do not occur in incompletely formed roots.
Inter radicular fibres • Extend from the crest of interradicular septum to the furcation of multirooted teeth.
Transeptal fibres • Extend interproximally over the alveolar crest and are embedded in the cementum of
(Also considered as adjacent tooth.
fibres of gingiva) • They are remarkably constant finding and are reconstructed even after destruction of
the alveolar bone.

Therefore the basal and the parabasal spinous cells are


ORAL MUCOUS MEMBRANE
referred as the stratum germfoatum.
1. Based on function, the oral mucosa can be classified
4. Stratum granulosum is na med for the basophilic
into -
keratohyaline granules.
• Mastkatory mucosa
Eg.: Gingiva and hard palate 5. The uppermost cells of stratum spinosum contain
numerous dense granules known as keratinosomes or
odland bodies, which are, modified lysosomes. They
• Lining or reflecting mucosa contain a large amount of acid phosphatase, an enzyme
Eg.: Lip, cheek, vestibular fornix, alveolar mucosa, involved in destruction of organelle membranes.
floor of mouth and soft palate.
6. The stratum corneum mainly contains keratinocytes.
• Specialized mucosa Keratinocytes Non-keratinocytes

Eg.: Dorsum of tongue and taste buds. Keratinocytes of oral • La ngerha n's cell
epithelium • Merkel cells
2. Keratinising oral epithelium has 4 cell layers. • Melanocytes
• Stratum basale
• Stratum spinosum 7. Orthokeratinized layer contains well-defined stratum
• Stratum gran ulosum granulosum, with no nuclei in the stratum corneum.
• Stratum corneum
In parakeratinised epithelium the stratum corneum
contains pyknotic nuclei, and the keratohyaline
3. The basal cells and parabasal spinous cells synthesize
granules are dispersed, not giving rise to stratum
DNA and undergo mitosis to provide new cells.
granulosum. Non-keratinised epithelium has neither
ORAL HISTOLOGY SYNOPSIS

granulosum nor st ratum cornea. The superficial cells 15. In most mammals the nasopalatine ducts are patent
have viable nuclei. The gingiva is 75% para keratinised, and together with Jacobson's organs, are considered
15% keratinised and 10% non-keratinised. as auxillary olfactory sense organs.

8. Melanocytes are Located in the basal and spinous Jacobson's organ is a small ellipsoid (dgar shaped)
layers. They synthesize melanin in organelles called structure lined with olfactory epithelium that extends
melanosomes. Melanin granules are phagocytosed from the nose to the oral cavity.
by other cells of epithelium, which are known as
melanophages or melanophores. 16. In three dimensional view, the interdental papilla of
posterior teeth is tent or col shaped, whereas it is
9. Langerhan's cells are modified monocytes derived from pyramidal between the anterior teeth.
the bone marrow. They are antigen-presenting cells
for lymphocytes. They contain specific granules called When the interdental papilla is tent or col shaped,
'Birbeck's gran ules' and shows marked ATPase activity. the oral and the vestibular corners are high, and the
central part is like a valley. The central concave arc is
Langerhans cells are absent in the junctional called the "COL". The COL is non-keratinised and is
epithelium of gingiva. more vulnerable to periodontal disease

10. Merkel cells harbor nerve endings, and contain tactile 17. The gingiva can be divided into "free gingwa ,
receptors. "attached gingiva" and " interdental papilla".

11. The dividing line between the free gingiva and the
attached gingiva is called free gingival groove. The
• Masticatory mucosa
facial aspect of attached gingiva is demarcated from
Keratinised areas • Vermilion border of lip
the alveolar mucosa by mucogingival junction.
• Oral or outer epithelium

• Col Under absolutely normal or ideal conditions, the


depth of gingival sulcus is O mm. In normal clinical
• Junctional epithelium
Non keratinised conditions, the probing depth of normal gingival
area
• Sulcular epithelium
sulcus in humans is 2 to 3 mm.
• Lining mucosa
• Specialized mucosa 18. The width of attached gingiva is the distance between
the base of gingival sulcus and the mucogingival
12. Keratinisation of oral mucosa varies in different areas junction. The width of attached gingiva increases with
in the following order: Palate (most keratinized), age. Increased width of attached gingiva is also seen
gingiva, tongue and cheek (least keratinized). in supraerupted teeth.

13. The degree of keratinisation diminishes with age and 19. The gingiva is characterized by a surface that appears
the onset of menopause. stippled. StippUng is shown by attached gingiva and
central core of interdental papillae.
14. The following zones can be observed in hard palate
• Gingival region, adjacent to the teeth. Stippling varies with age. It is absent in infancy,
appears in some children at about 5 years of age,
• Palatine raphae, also known as median area,
increases until adulthood, and may disappear in old
extending fro m the incisive papillae or palatine
age.
papillae posteriorly.
• Anterolateral area or fatty zone between the Microscopically stippling is produced by alternate
raphae and gingiva. rounded protuberances and depressions in the gingival
• Posterolateral area or glandular zone between the surface. The papillary layer of connective tissue
raphae and gingiva. projects into the elevations.
Dental ;lut.,e

20.
• They are formed in lamina propria, in the region of incisive papillae.
Epithelial
pearls
• They are remnants of epithelium in the line of fusion between the pa latine processes.
• They consist of concentrically arranged epithelial cells that are frequently keratinised .

Epstein pearls • These are palatal cyst s of neonate that are found along the median raphae of the hard palate
and appear to be derived from entrapped epithelial remnants along the line of fusion.
• It appears as a tiny globule of enamel firmly adherent to the tooth, which arises from a small
Enamel pearls group of misplaced ameloblasts.
(Ena meloma) • It appears as small, focal excess mass of enamel mostly in the bifurcation or trifurcation areas
of maxillary mola rs.
• One of the histologic features of carcinoma .
Keratin pearls • Presence of individual cell keratinisation and formation of numerous keratin or epithelial
pearls is characteristic feature of a well-differentiated epidermoid carcinoma.

than other types of bone tissue. Such areas are seen in


MISCELLANEOUS
roentgenograms as dense radio opacities.
1. The alveolar process can be distinguished into two
6. Serous, mucous and myoepithelial cells forms the
parts, the alveolar bone proper and the supporting
terminal secretory units of salivary glands. The
alveolar bone.
secretions of these units are collected into intercalated
ducts, which empty into the striated ducts.
The alveolar bone proper surrounds the root of the
tooth and gives attachment to principal fibres of the
7. The secretion of the secretory proteins or secretory
periodontal ligament.
granules occurs by exocytosis.
The supporting alveolar bone surrounds the alveolar
8. Myoepithelial cells are stellate shaped and are also
bone proper and gives support to the socket.
known as "basket cell". The structure of myoepithelium
is similar to th at of smooth muscle and contains actin,
2. The interdental and interradicular septa contain
myosin and related proteins.
the perforating canals of zuckerkandl and hirschfeld
(nutrient canals), which house the interdental and
9. Myoepithelial cells have contractile function. They
interradicular arteries, veins and nerves.
help in expelling of secretions from the lumina of
secretory units.
3. Hematopoietic cellular marrow (red bone marrow) is
found in the condylar process, in the ang le of the
10. Within a lobule, the sma llest ducts are the intercalated
mandible, in the maxillary tuberosity.
ducts. The lumen of intercalated duct contains Na•, CL·
and K· and HC0 3 · ions.
4. The alveolar bone proper, forms the inner wall of the
socket, and is perforated by many openings that carry
The concentration of Na• and CL· ions is similar to
branches of the internlveolar nerves and blood vessels,
that of plasma. The K· ions concentration is greater
and is the refore called the cibriform plate.
than plasma and the concentration of HC0 3· is variable
depending on specific gland.
5. The alveolar bone consists partly of lamellated and
partly of bundle bone.
11. Glands of Blandin and Nuhn are anterior lingual
glands located near apex of tongue. They are chiefly
Bundle bone is th at bone in which the principal
mucous in nature. The ducts open on the ventral
fibres of the periodontal ligament are anchored. Since
surface of the tongue near the lingual frenum.
bundle bone contains more calcium sa lts per unit area
ORAL HISTOLOGY SYNOPSIS

The posterior lingual glands are mixed in nature. The Lingual lipase is produced by lingual serous glands. It
posterior serous glands are called von Ebner's glands initiates the digestion of dietary lipids and hydrolyzes
and are located between muscle fibres of the tongue, triglycerides to monoglycerides diglycerides, and fatty
below the vallate papillae. acids.

Their ducts open into the trough of the vallate papillae 17. The shedding of deciduous teeth is the result of
and at the rudimentary foliate papilla on the sides of progressive resorption of roots of teeth and their
the tongue. supporting periodontal ligament.

12. Glossopalatine and palatine glands are purely mucous 18. The cells responsible for the removal of dental ha rd
in nature. The labial and buccal glands are mixed in tissue are odontoclasts and they characteristically
nature contains high amount of enzyme acid phosphatase.

19. Remnants of deciduous teeth are most commonly seen


• Glosso palatine
associated with lower second premolars. The reason
Purely mucous • Palatine
is that the roots of the lower second deciduous molar
• Anterior lingual glands are strongly curved or divergent.
Purely serous • Parotid .
20. Most commonly retained deciduous teeth are the upper
• Submandibular (mainly serous)
lateral incisors.
Mixed
• Sublingua l (Mainly mucous)
• Labial and buccal glands 21. If the permanent tooth is ankylosed or impacted, its
• Posterior lingual glands. deciduous predecessor may also become retained.
This is most frequently seen with the deciduous and
13. The total volume of saliva secreted daily by humans is permanent canine teeth.
approximately 750ml, of which 60% is produced by the
submandibular glands, 30% by the parotid, 5% from 22. The floor of sinus is closest to the roots of first molar,
the sublingual and 7% from the minor salivary gland. followed by roots of the second premolar, first premolar
and second molar in that order of frequency.
14. Water accounts for 99% or more of the saliva and the
inorganic ions, secretory proteins, and glycoproteins
make the remaining 1%.

The major inorganic ions of the saliva are Na+, K+ Cl·


and HC0 3- . The HC0 3- forms the primary buffering
system of saliva.

15. The pH of saliva varies from 6.7 to about 7.4. Parotid


saliva may have a greater pH of 6.0 to 7.8.

16. Saliva participates. in digestion by providing a fluid


environment for solubilization of food and through the
action of amylase and lingual lipase enzymes.

Amylase enzyme acts on carbohydrates to produce


glucose and maltose. Its action continues for up to
30 minutes in the stomach before the amylase is
inactivated by the acid PH and proteolysis.
Dental ;fc&.,e

MICROBIOLOGY
I. REFERENCE BOOKS TAKEN:
1. TEXT BOOK OF MICROBIOLOGY BY ANANTHANARAYAN - s th & 9th EDITIONS

2. TEXTBOOK OF MEDICAL PARASITOLOGY BY CK JAYARAM PANIKER - 5 th & 7 t h EDITIONS

1. GENERAL MICROBIOLOGY
1. The predominant immunoglobulin in saliva is 8. In patients with asthma due to an allergic cause the
a) IgA b) IgD serum levels of which immunoglobulin increases
c) IgE d) IgG a) IgD b) IgM
(MAN-1994) c) IgA d) IgE
2. Immunoglobulin crossing placenta is (MAN-1996)
a) lg M b) lg A 9. The best sterilization method for a hand-piece is
c) lg G d) lg D a) Ethylene oxide gas b) dry heat
(MAN-2002) c) Chemical vapour d) all the above
3. Steam under pressure is used in _ _ sterilization (MAN-1999)
a) Pasteurization b) Autoclave 10. Type of receptor present on T-cells are -
c) Kochs sterilizer d) hot air oven a) lg A b) lg G
(MAN-2002) c) Prostaglandins d) CD4
4. The term 'animalcules' to oral microorganisms is (MAN-1996)
given by 11. The minimum time required for the sterilization of
a) Wd miller b) Leuwenhoek surgical instruments by moist heat at 134° ( is:
c) Robertson d) Socranci a) 3 minutes b) 15 minutes
(MAN- 2000) c) 30 min utes d) 60 minutes
5. The chemotactic factor is (MAN-1995)
a) C2a b) C3b 12. Hot air oven cannot be used for sterilising:
c) C4a d) C5a a) Clothes b) Instruments
(MAN-2001) c) Culture media d) Needles
6. Which of the following immunoglobulins has the (MAN-1995)
highest mean serum concentration in humans 13. Dark ground microscopy is useful to identify:
a) IgA b) IgD a) Mycoplasma b) Chlamydiae
c) IgG d) IgM c) Rickettsiae d) Spirochaetes
(MAN-1995, BHU-07) (KAR-1998)
7. Example of a immune complex hyper-sensitivity 14. Zero growth rate is observed during one of the
reaction is phases of the bacterial growth curve:
a) Atopic allergy b) Serum sickness a) Lag b) Exponentia l growth
c) Transfusion reaction d) Contact dermatitis c) Stationary phase d) Decline
(MAN-2000) (KAR-1997)

1) A 2) C 3) B 4) B 5) D 6) C 7) B 8) D 9) A 10) D 11) A 12) C 13) D


14) C
, MICROBIOLOGY

15. The interaction of antigens with antibodies on 23. Compared with autoclave, use of a dry heat oven
the surface of a mast cell leads to degranulation for sterilization has the following disadvantage:
and anaphylaxis. The mast cell granules produce a) Time-consuming b) Less effective
anaphylaxis because they contain: c) Causes instruments to rust
a) Lysosomes b) Proteolytic enzymes d) Causes sharp instruments to dull
c) Lymphotoxins d) Vasoactive mediators (AIIMS-1994)
(KAR- 2001) 24. The component that sensitizes bacteria and virus
16. Exotoxins are: to UV irradiation
a) Lipid-polysaccharide complex a) Lipids b) Carbohydrates
b) Protein compo und c) Inorganic salts d) Nucleic acids
c) Lipoprotein d) None (AIIMS- 2000)
(PGI- 2002) 25. Sterilisation is
17. The lg found in exocrine secretions is: a) Pathogens killed but spores are not affected
a) IgG b) IgD b) Only pathogenic micro-organisms killed
c) IgM d) IgA c) Non-pathogenic killed
(AIPG-1998) d) All pathogenic and non -pathogenic microorganisms
18. By using phase contrast microscope: killed
a) Internal structmes are differentiated in living cells (AIPG- 2012)
b) Internal structures are differentiated in dead cells 26. Disinfection of the rooms can be done by using
c) Internal metabolic activities can be observed a) 20% phenol
d) External capsule formation can be observed b) High efficiency particle arrestors
(KAR-2002) c) 40% formaldehyde vapour
19. Bacterial spores are destroyed by d) Chlorine gas
a) Boiling at 100°c (KAR-1999)
b) Disinfection by hypochlorite solution 27. Rideal Walker test is used to determine the efficiency
c) Autoclave d) Pasteurisation of the
(KAR- 1999) a) Disinfectant b) Moist heat sterilisation
20. Blood agar is c) Antibiotics d) Dry heat sterilization
a) Simple media b) Transport media (KAR-1999)
c) Enriched media d) Differential media 28. Who first introduced solid media:
(KAR-1999) a) Louis Pasteur b) Robert Koch
21. Which of the following is not a feature of c) Hensen d) Ogston
anaphylaxis? (KAR-1999)
a) Delayed reaction b) Immediate reaction 29. All are not true for cultivation of anaerobic
c) Passive transfer by serum organisms except
d) None of the above a) Incubator b) anaerobic jar
(AIIMS- 1995) c) Refrigerator d) centrifuge
22. The difference between sterilization and (KAR-1999)
disinfection is tllat: 30. Sterilization of liquid paraffin is done by:
a) Disinfection kills all spores but sterilization does a) Autoclaving b) Hot air oven
not c) UV-Radiation d) Inspissation
b) Sterilization kills all spores but disinfection does (KAR-2003)
not 31. Multiple drug resistance of bacteria depends on
c) Sterilization involves chemicals and disinfection which of the following:
involves physical methods a) Resistance transfer factor (RTF)
d) Sterilization is easier to achieve b) Colicigenic factor (Col)
(PGI-1999) c) Fertility factor (F +)

15) D 16) B 17) D 18) A 19) C 20) C 21) A 22) B 23) A 24) D 25) D 26) C 27) A
28) B 29) B 30) B 31) A
Dental ;lut.,e

d) All of the above d) 121 °C x 15 mts. x 15 lbs pr.


(KAR-2003) (KAR-1998, 99)
3 2. Glutaraldehyde is preferred more than formaldehyde 40. Sunlight acts as sterilizing agent because
because: a) Long wavelength b) Short wave length
a) It has got more bactericidal activity c) Ozone d) Heat
b) It has got more bacteriostatic activity (AIPG-2012)
c) It has got no deleterious effect on the cement and 41. Immunoglobulin implicated in atopy and
lenses of instruments anaphylaxis is:
d) Its application is easier a) IgE b) IgM
(KAR-2002) c) IgG d) lgA
3 3. The major antiseptic value of soap lies in its ability to: (PGl-1998, 97)
a) Kill microbes b) Remove microbes 42. Which of the following is not true of boiling water
c) I nhibit microbia l growth a) It can kill hepatitis B vi rus
d) Dehydrate microbial cells b) Used for sterilizing surgical instruments
(AP-1996) c) Can kill mycobacterium tuberculosis
34. Best method of sterilizing disposable syringes: d) Can kill vegetative bacterial cells
a) Hot air oven b) U.V. rays (AIIMS-2K, 2002)
c) Gamma rays d) Boiling 43. Synthesis of antibodies in plasma cells occurs in:
(KAR-1998) a) Golgi body b) Endoplasmic reticulum
35. Which of the following antibacterial substance is c) Nucleus d) Mit ochondria
present in eggs and saliva? (PGl- 1998)
a) Lysozyme b) Secretozyme 44. In penicillin allergy, penicillin acts as a:
c) Albumin d) Isozyme a) Hapten b) Carrier
(AIPG-1994) c) Super antigen d) Toxin
36. A type of immediately occurring reaction in which (AIIMS-2001)
antigens combine with antibodies already attached 45. Which of the following is an enrichment medium
to the surface of mast cells and basophils is called: a) Blood agar b) Selenite F Broth
a) Type I hypersensitivity c) Mac Conkey agar d) Nutrient Broth
b) Type II hypersensitivity (KAR-2003)
c) Type III hypersensitivity 46. Which of the following is not a Koch's postulate?
d) Type IV hypersensitivity a) The bacteria should be able to be isolated from a
(AIIMS- 1999) diseased tissue
37. Most antibodies are produced in the: b) When inoculated in experimental animals, bacteria
a) Heart and the liver may or may not cause disease
b) Brain and the meninges c) Pure cultures can be obtained
c) Thymus and the appendix d) Bacteria should be able tro be isolated from the
d) Spleen and the lymph nodes diseased tissue in the experimental animals
(AIPG-1998) (KAR-2001)
3 8. Tyn dallisation is carried out for: 4 7. An example of naturally acquired passive
a) One day b) Two successive days immunity is
c) 60 minutes d) Three successive days a) Hepatitis vaccination
(KAR- 2002) b) Gamma globulin injection
39. The optimal time and temperature required for c) Immune blood transfusion
sterilization in an autoclave is d) Placenta l t ransfer of antibodies
a) 140°C x 2 hrs. x 10 lbs pr. (MAN -1999, AIPG-2001)
b) 121°F x 15 mts. x 15 lbs pr. 48. The anaphylaxis should be treated first with
c) 121 °C x 15 sec. x 15 lbs pr. a) Epinephrine b) Steroids

32} C 33} B 34} C 35} A 36} A 37} D 38} D 39} D 40} B 41} A 42} B 43} B 44} A
45} B 46} B 47} D 48} A
, MICROBIOLOGY

c) Anti histamines d) Antibiotics d) None of the above


(AP-2003) (COM EDK-04)
49. Disinfection frees the surface from 59. The binding site for complement on the lgG
a) Vegetative forms b) Spore forms molecule is in the
c) Bot h of the above d) None of t he above a) VL domain b) CL domain
(AP-1999) c) CHl domain d) CH2 domain
50. Dry heat destroys microorganisms by (COMEDK- 04)
a) Lysis b) Oxidation 60. The immediate type of hypersensitivity in which
c) Hydrolysis of DNA d) Coagulation of proteins histamine doses not play a major role is
(AIIMS-2K, 2K1; AP-05) a) Urticaria b) Asthma
51. A darkfield microscope would be useful in c) Anaphylaxis d) Arthus reaction
examining blood for: (APPSC- 99)
a) Treponema b) Actino myces 61. IgG antibodies have a half-life of approximately
c) Streptococcus d) Mycobacterium a) 1 hour b) 1 day
(AIPG-2003) c) 1 Week d) 1 Month
52. Robertsons cooked meat medium is best suited for (APPSC- 99)
cultivation of: 62. Toxoids are
a) Clostridia b) S. epidermidis a) antigenic and toxic b) antigenic and non-toxic
c) Cholera d) Mycobacterium c) non-antigenic and toxic
(KAR-2001) d) non-antigenic and non-toxic
53. Immunoglobulins are secreted by: (APPSC- 99)
a) Macrophages b) Plasma cells 63. Detergents used to disinfect dental instruments
c) T-cells d) Neutrophils in the office kill bacteria by interfering with
(KAR-2001) functions of the cell
54. The medium of choice for growing most fungi is: a) wall b) nucleus
a) Blood agar b) Tissue culture c) membrane d) capsule
c) Sabouraud's agar d) Thioglycollate medium (APPSC- 99)
(AIPG-2003) 64. lgG has subclasses
55. Serum sickness syndrome is an example of: a) 1 b) 2
a) Anaphylactic shock c) 3 d) 4
b) Angioneurotic edema (TNPSC-99)
c) Cell-mediated immunity 65. Endotoxins differ from exotoxins in that endotoxins
d) Systemic arth us reaction a) are proteins b) are heat labile
(AIPG-98) c) are high ly antigenic
56. The distinguishing characteristic of a positive d) activate complement by the alternate pathway
delayed-type hypersensitivity skin test is (TNPSC- 99)
a) Erythema b) Necrosis 66. Adjuvant given along with antigens are going to
c) Induration d) Vasculitis a) increase toxigenicityb) increase antigenicity
(COM EDK-04) c) reduce the antigenicity
57. Secretory lg is: d) reduce the toxigenicity
a) lgA b) lgE (TNPSC-99)
c) IgM d) IgD 67. The efficacy of the moist heat sterilization
(AIPG- 96) technique commonly used to sterilize laboratory
58. Human immunoglobulins are divided based on ware and culture media is tested by using (OR)
a) Functional differences Sterilisation control for moist heat is
b) Structural differences (COMEDK-2013)
c) Complement fixation a) using Bacillus stearothermophilus

49) A 50) B 51) A 52) A 53) B 54) C 55) D 56) C 57) A 58) B 59) D 60) D 61) D
62) B 63) C 64) D 65) D 66) B 67) A
Dental ;lut.,e

b) using non-pathogenic strain of clostridium tetani c) 5% d) 15%


c) using chemical indicator
d) using methylene blue 77. One of the following staining methods is an
example of negative staining
68. Following reactions are produced by heterophil a) Grams' staining b) Fontana's staining
antigens except c) India ink preparation
a) Forssman antigen antibody reaction d) Zheil-Neilson's staining
b) Weil-Felix reaction
c) Paul bunnel reaction 78. The best skin disinfectant is
d) Widal reaction a) Chlorhexidine b) Alcohol
(TNPSC- 99) c) Savlon d) None of above
69. B.C.G., Untrue is
a ) Live vaccine b) T.B. vaccine 79. Noguchi's medium is used for
c) Orally administered d) All of the above a) Bordetella b) Brucella
(KAR-99) c) Borrelia d) None of above
70. Characters of exotoxin include all, except
a) Protein polysaccharide complex 80. Culture medium for corynebacterium diphtheria
b) Heat labile a) Loefflers serum slope
c) High ly potent d) Has specific tissue affinity b) Mc Conkey
(TNPSC-99) c) Saboraud agar
71. First released immunoglobulin after primary d) Lowenstein Jensen medium
immunization
a) IgD b) IgE 81. Smith noguchi's media is used for
c) IgG d) IgM a) Salmonella b) Klebsiella
(AP-2004) c) Spirochetes d) Bacillus
7 2. Anaerobic bacteria can be cultured in
a) L-J medium 82. Dorset's egg medium is used for cultivation of
b) Robertson cooked meat medium a) Staphylococcus b) Streptococcus
c) Loefflers medium d) Sabourauds agar c) Gonococcus d) Mycobacterium
(KAR-98)
73. Exaltation is 83. Graft versus host reaction is caused by
a) Decreased Virulence b) Increased Virulence a) B-Lymphocytes b) T-lymphocytes
c) No change d) None c) Monocytes d) Leukocytes
(BHU-07)
74. Louis Pasteur is not associated with 84. The mechanism of genetic transfer where a phage
a) Introduction of complex media serves as a vehicle is:
b) Discovery of Rabies Vaccine a) Transformation b) Translation
c) Discovery of M. tuberculosis c) Conjugation d) Lysogeny
d) Disproved spontaneous regression theory
85. Sporulation occurs in
75. Bacteria with tuft of flagella at one end are called a) Lag phase b) Log phase
a) Monotrichate b) Peritrichate c) Stationary phase d) Decline phase
c) Bipolar d) Lophotrichate
86. Mesophilic organisms are those that grow best at
76. The usual concentration of agar used in agar temperature of
medium is a) -20°C to - l°C b) -l°C to+ 20°c
a) 10% b) 2% c) 25°C to 40°C d) 55°Cto80°C

68} D 69} C 70} A 71} D 72} B 73} B 74) C 75) D 76) B 77) C 78) A 79} C 80} A
81} C 82} D 83} B 84} D 85} C 86} C
, MICROBIOLOGY

87. Which of the following organisms are arranged in c) Virus d) Gram -ve bacteria
cubical packets of eight cocci
a) Staphylococcus b) Micrococcus 97. Phagocytosis enhanced by coating the surface of
c) Sarcina d) Aerococcus antigen is called
a) Opsonisation b) Chemotaxis
88. Flash pasteurization of milk is done at what c) De coding d) CFT
temperature
a) 125 °C for new seconds 98. Opsonisation is by
b) 60°C for 15 minutes a) IgA b) IgE
c) 72°C for 5 minutes d) 72° for 15 - 20 Sec. c) IgG d) IgM

89. Hospital dressing is best disposed by 99. The exact part of the antigen that reacts with the
a) Incineration b) Dumping immune system is called as
c) Autoclaving d) Burying a) Clone b) Epitope
c) Idiotope d) Paratope
90. Spores are disinfected by
a) Glutaraldehyde b) Betapropiolactone 100. Antigen combining site of the antibody is
c) Formaldehyde d) Hexachlorophen a) Idiotope b) Paratope
c) Epitope d) Hapten
91. Operation theatres are sterilized by
a) Carbolic acid spraying 101. Immunoglobulin consists of
b) Washing with soap and water a) 2 light, 2 heavy chains
c) Formaldehyde fumigation. b) 1 heavy, 2 light chain
d) ETO gas c) 1 light , 1 heavy chain
d) 3 light, 1 heavy chain
92. Most of the drug resistance occurs due to
a) Transduction b) Transformation 102. Function of lg A is
c) Mutation d) Conjugation a) Acts as a mucosal barrier for infection
b) Circulating antibody
93. Plasmid c) Kills virus infected cells
a) Involved in multidrug resistance transfer d) Activates macrophages
b) Involved in conjugation
c) Imparts capsule formation 103. Secretory piece of IgA is synthesized in
d) Imparts pili formation a) T-cells b) B-cells
c) Lymph nodes d) Mucosal epithelium
94. The antigen used in Weil Felix test is obtained
from 104. Complement binding immunoglobin via the
a) Pseudomonas b) E. coli classical pathway is -
c) Proteus d) Staphylococcus a) IgG & IgM b) IgG & IgA
c) IgG & IgD d) IgD & IgE
95. Immunoglobin, which is produced first by the fetus (GCET-14)
in response to infection 105 VDRL is a -
a) IgG b) IgA a) Slide flocculation test
c) IgM d) IgD b) Tube floccu lation test
c) Tube agg lutination test
96. Endotoxins are produced by: d) Latex agg lutination test
a) Fungi b) Gram +ve bacteria

87) C 88) D 89) A 90) A&C 91) C 92) D 93) A&B 94) C 95) C 96) D 97) A 98) C&D 99) B
100) B 101) A 102) A 103) D 104) A 105) A
Dental ;lut.,e

106. Most sensitive test for antigen detection is - 115. The serum concentration of which of the following
a) Radioimmuno Assay human IgG subclass is maximum?
b) ELISA a) IgGl b) lgG2
c) Immunoflourescence c) IgG3 d) IgG4
d) Passive hemag lutinat ion (AIPG-05)
116. Immunity that is conferred to the foetus by
107. Delayed tuberculin test response is due to transfer of IgG and lgA antibodies is called as
a) B lymphocytes b) T lymphocytes a) Active acquired immunity
c) Monocytes d) Histiocytes b) Passive acquired immunity
c) Natural active immunity
108 Prozone phenomenon is due to - d) Passive natural immunity
a) Disproportionate antigen -antibody levels (PGI- 05)
b) Excess antigen 117. Which of the following provides best guarantee for
c) Excess antibody sterilization in the heat sterilizer?
d) Hyper immune reaction a) Using chemical indicator strip or pouch
b) Recording a temperature/pressure readings from
109. Haptane is - sterilizer gauge
a) Same as epitopes c) Using a bacterial spore test
b) Carrier required for specific antibody production d) Determining the ability of a sterilizer to kill the
c) High molecular rprotei n hepatitis B virus
d) None of the above (KAR- 04)
118. Active artificial immunization is induced by the
110. Virus infected cell is killed by - administration of all of the Following EXCEPT
a) Interferons b) Macrophages a) Bacterial products b) Toxoids
c) Neutrophils d) Autolysis c) Vaccines d) Antitoxins
(COMEDK- 05)
111. Function of T-lymph ocyte is/are - 119. Selective media for MYCOBACTERIUM
a) Production of interferon a) Sabourauds medium
b) Lymphokine production b) Agar - agar
c) Rosette formation c) Lowenstein Jensen medium
d) All of the above d) Loefflers serum slope
(AP-05)
112. IL-1 and TNF - a during inflammation are secreted 120. Type I hypersensitivity is mediated by which of the
by following immunoglobulins?
a) Plasma cells b) Activated macrophages a) IgA b) IgG
c) Lymphocytes d) Platelet c) lgM d) IgE
(PGI- 2011) (AIPG- 05, NEET- 2013)
113. Glasswares are sterilised by 121. Which of the following are correctly matched?
a) Autoclaving b) Hot air oven a) Transfer of antibody from mother to child is
c) Incineration d) Formaldehyde through colostrum and acquired passive immunity
(COMEDK-06) naturally
114. The protective effects of breast milk are known to b) Injection of antibodies (hepatitis) is artificially
be associated witll: acquired passive immunity
a) IgM antibodies b) Lysozyme c) Antigenic stimulus given by vaccine (polio) is
c) Mast cells d) IgA antibodies artificial active imm unity
(AIPG-05) d) All of the above
(KAR-04)

106) A 107) B 108) C 109) B 110) A 111) D 112) B 113) B 114) D 115) A 116) D 117) C 118) D
119) C 120) D 121) D
, MICROBIOLOGY

122. A woman with infertility receives an ovary 131. IL-1 is produced by:
tran splant from Iler sister who is an identical Twin. a) Helper T cells b) Helper B cells
What type of graft it is? c) Monocytes d) Macrophages
a) Xenograft b) Autograft
c) Allograft d) Isograft 132. Same or closely related antigens present in
(AIPG-05) different biological species are known as
123. Teichoic acid is present in a) Sequestrated antigens
a) Cell wall of Gram positive organisms b) Isoa ntigens
b) Cell wall of Gram negative organisms c) Haptens d) Heterophile antigens
c) Cytoplasm of Gram positive organisms (KCET-2012)
d) Cytoplasm of Gram negative organisms 133. CD4 cells recognize the antigens in association
(KAR-04, KERALA-2015) with
124. Cold sterilization refers to the process of a) MHC I b) MHC II
sterilization by wse of c) MHC III d) B-cell receptor
a) ultra violet rays b) Ultrasonic vibrations (KCET-2012)
c) Infrared rays d) Gamma rays 134. T cells multiplication is stimulated by:
(COM EDK-05) a) Macrolin b) Heat
125. Which of the following is a live vaccine: c) Bovine serum d) Phytohaemagglutinin
a) 17 D b) Salk
c) Hepatitis d) HDCV 135. T cell matures in :
a) Payers patch b) Lymph node
126. The prototype of Type II hypersensitivity reaction c) Thymus d) Bursa of Fabricius
is:
a) Arthus reaction b) SLE 136. Killer and Helper cells are part of:
c) Auto immune hemolytic anemia a) B cells b) T cells
d) Contact dermatitis c) Monocytes d) Macrophage

127. Type IV hypersensitivity reaction is mediated by 13 7. Which component of the cell wall is responsible
which of following? for endotoxic activity:
a) B-Cells b) Active T cells a) Region I b) Region II
c) NK cells d) Plasma cells c) Region III d) Boivin antigen
(KCET-07)
128. The Killer cells are associated with 138. Endoscopes are sterilized by using:
immunologic response. a) Autoclaving b) Boiling
a) Type I b) Type II c) Cidex d) Gamma radiation
c) Type III d) Type IV
139. Moist heat kills all of the following EXCEPT:
129. Grave's disease is an example of type _ _ _ __ a) Brucella b) Mycobacterium
immunologic response: c) Salmonella d) Coxiella burnetti
a) Type I b) Type II
c) Type III d) Type IV 140. Hot air oven is used for sterilization of all EXCEPT:
130. Rh incompatibility is which type of hyper- a) Glassware b) Rubbertubes
sensitivity reaction? c) Sharp instrument d) Liquid paraffin
a) Type I b) Type II
c) Type III d) Type IV 141. One of the following statements about sterilization
is not true:
a) Liquid paraffin ~ Dry heat

122) D 123) A 124) D 125) A 126) C 127) B 128) B 129) B 130) B 131) D 132) D 133) B 134) D
135) C 136) B 137) C 138) C 139) D 140) B 141) D
Dental ;lut.,e

b) Glass syringes~ Boiling at 160°C for 60 min c) Compliment activation


c) Flash pasteurization of milk ~ 72° C for 15 d) Reagin activity
seconds, followed quickly by rapid cooling to 13 °C
for lower 151. The reaginic antibody is:
d) Vaccines~ Ethylene oxide a) IgG b) IgA
c) IgD d) lgE
142. Oil and Grease are sterilized by:
a) Hot air oven b) Autoclaving 152. The type of immunoglobulin most commonly
c) Irradiation d) Ethylene dioxide increased in multiple myeloma is
a) IgG b) IgA
143. The best method of sterilization of dressing powder c) IgM d) IgD
is: (COMEDK-07)
a ) Autoclaving b) Hot air oven 153. The following immunoglobulin is not known to fix
c) I nspissation d) Tyndallization complement
a) lgE b) IgM
144. Which of the following is true about pasteurization? c) IgA d) IgG
a) It kills bacteria and spores (COMEDK-07)
b) It kills all bacteria except thermoduric bacteria 154. Cell wall deficient bacteria are
c) It kills 95% of microorganisms a) Escherichia Coli b) Staphylococci
d) Tubercle bacilli are destroyed c) Spirochetes d) Mycoplasma
(KCET-08)
145. Physical sterilization was discovered by: 155. The earliest identifiable cells of T-Cell lineage
a) Robert Koch b) Louis Pasteur during maturation are
c) John Hunter d) Alexander Flemings a) CD7+ Pro-T-Cells b) CD4+ Pre-T-Cells
c) CD8+ Pro-T-Cells d) CD8+ Pre-T-Cells
146. Portion of immunoglobulin molecule with (KCET-08)
molecular weight of 50000: 156. Purpose of sterilization is to eliminate
a) Secretory piece b) H chain a) Bacteria b) Viruses
c) L chain d) J piece c) Spore formers d) Fungus
(AIIMS-09)
147. Cleavage of lgG molecule by Papin gives rise to: 157. A person working in an abattoir presented with a
a) One Fe fragment and two Fab fragments papule on hand, which turned in to an ulcer. Which
b) One Fe fragment and four Fab fragments will best help in diagnosis
c) Two Fe Fragments and two Fab fragments a) Polychrome Methylene blue
d) Two Fe Fragments and one Fab fragment b) Carbal Fuschin
c) Acid Fast Stain d) Calcoflour White
148. Secondary response is mediated by: (AIIMS-09)
a) IgG b) IgA 158. A child presents with infective skin lesions of the
c) IgM d) IgE leg. Culture was done which showed gram positive
(COMED-14) cocci in chains and haemolytic colonies. Which of
149. Pentameric structure: the following tests will best identify the organism?
a) IgM b) IgG a) Bile solubility b) Optochin sensitivity
c) IgA d) IgD c) Bacitracin sensitivity
d) Catalase positive
150. The function of IgD is: (AIIMS-09)
a) Antigen recognition by B cells 159. Which is type II hypersensitivity?
b) Localization protection in external secretion a) Anaphylaxis

142) A 143) B 144) B 145) B 146) B 147) A 148) A 149) A 150) A 151) D 152) A 153) A 154) D
155) A 156) C 157) A 158) C 159) D
, MICROBIOLOGY

b) Delayed hypersensitivity c) ELISA d) DNA probe


c) Immune complex mediated (COMEDK-09)
d) Cytotoxic / cytolytic type 168. The term epitope refers to
(IGNOU-10) a) Complete antigen molecule
160. The manufacturing unit of surgical needle & b) Hapten
sutures are sterilized by? c) Immunogen
a) Garn ma-Rays b) X-Rays d) Smallest antigenic determinant
c) Autoclaving d) Boiling (COMEDK-09)
(PGI-08) 169. Chemotactic cytokine is
161. When horse serum is injected intravenoulsy in to a a) IL-1 b) IL-6
rabbit and again in to the skin two or three weeks c) IL-8 d) TNF
later, what is the necrotizing reaction that occurs (AIIMS-09)
at the site of the second injection 170. Cold sterilization refers to the process of
a) Atopy sterilization by use of-
b) Anaphylaxis a) Ultra violet rays b) Ultrasonic vibrations
c) Arthus phenol1ilenon c) Infrared rays d) Gamma rays
d) Serum sickness. (COMEDK-09)
(AIIMS-08) 171. Components of innate immunity that are active
162. Bacterial cell does not possess: against viral cells includes
a) dna b) Ribosome a) NK Cells b) Cytotoxic T cells
c) Mitochondria d) Cell wall c) B cells d) Memory B cells
(IGNOU-10) (AIPG-10 )
163. Negative phase is seen in _ _ _ _ _ immunity 172. Phase contrast microscopy is based on principle of
a) Active b) Passive a) Different refractive indices of object
c) Herd d) Local b) Different reflective indices of object
(KCET-10) c) Light scattering d) Light attenuation
164. Post streptococcal acute glomerulo nephritis is an (AIIMS-09)
example for 173. Which of the following is not used as disinfectant
a) Type I hypersensitivity a) 1-2% cetrimide b) 100% alcohol
b) Type II hypersentitivity c) 2% Lysol d) 5% chloroxylene
c) Type III hypersentitivity (AIIMS-09)
d) Type IV hypersensitivity 174. Which of the following organisms does NOT react
(KCET-09) with gram's stain?
165. The cell wall deficient bacteria a) Actinomyces israelii b) Candida albicans
a) Rickettsiae b) Mycoplasma c) Mycobacterium tuberculosis
c) Chlamydiae d) Ehrlichiae d) Streptococcus mutan
(KCET-10) (KCET-2011)
166. The process of attenuation can be achieved by all 175. Anaphylaxis refers to the
except a) Severe reaction followings the injection of protein
a) Growing bacteria in adverse environment solutions in a sensitized individual
b) Growing bacteria in unconventional host b) Severe reaction followi ng primary injection of
c) Serial passage in an experimental host protein solutions
d) Repeated cultures in artificial media c) State of immunity developed by repeated injections
(KCET-09) of any foreign substance
16 7. Antibiotic sensitivity and resistance of micro d) Severe reaction resulting from sensitivity to
organisms is determined by common allergens
a) Direct microscopy b) Culture (COMEDK-2011, 2014)

160) A 161) C 162) C 163) A 164) C 165) B 166) C 167) B 168) D 169) C 170) D 171) A 172) A
173) B 174) C 175) A
Dental ;lut.,e

176. Soluble components of complement system 184. True about conjugation completed in bacteria?
account for about a) Sex pilus is used
a) 1-2 mg/ml of total serum protein b) Cell to cell contact does not occur for horizontal
b) 3-4 mg/ml of total serum protein gene transfer
c) 5-7 mg/ml of total serum protein c) Both strand of DNA is t rarnsferred
d) 7-8 mg/ml of total serum protein d) Chromosomal DNA is not transferred
(COMEDK-2011)
177. Out of the following hypersensitivity reactions, in 185. A boy has shortness of breath presents in OPD
which type a single dose of the antigen can act as injected with penicillin has never history of
both the sensitizing and shocking dose. allergy; on examination after 48 hours his
a) Anaphylaxis b) Arthus reaction RBC's are getting destroyed. He has diagnosed
c) Serum sickness d) Contact dermatitis with AIHA (immuohemolytic anemia). Type of
(KCET-2011) hypersensitivity is?
178. Complement factor through which common a) Type I b) Type II
pathway begins: c) Type III d) Type IV
a) C3 b) C5 (AIPG-14)
c) C2 d) Protein B 186. Electron microscopy was introduced by
(NEET-2013) a) Leeuwenhoek b) Robert Koch
179. Stain used to make the wetted area of filter paper c) Ruska d) Paul Ehrlich
strip more visible is (KERALA-2015)
a) Eosin stain b) Haemotoxin stain 187. Bacteria are simple genetic units with all of the
c) Ninhydrin stain d) Blue stain following properties EXCEPT
(COMEDK -2013) a) They are diploid
180. The key cell types involved in the acquired b) Their genetic material is organized into a single
immunity include all EXCEPT chromosome
a) B lymphocytes b) Erythrocytes c) Their DNA has intervening sequences (introns) in
c) T lymphocytes almost all genes
d) Antigern presenting cells (APC) d) They use same genetic code as all the eukaryotes
(AP -2013) (COMEDK-15)
181. Which of the following is NOT a live attenuated 188. An example of high level disinfectant is
vaccine? a) Phenol b) Glutaraldehyde
a) Tuberculosis (BCG) b) Typhoid c) Alcohol d) Povidone Iodine
c) Varicella Zoster virus d) Cholera (COMEDK-15)
(KAR -2013)
182. Sterilization is a process in which:
a) Kill all pathogenic micro-organisms
b) Kill all non-pathogenic micro-organisms in the
environment
c) Deactivate all pathogens not spores
d) Deactivate all spores and kill pat hogenic organism
(NEET-2013)
183. The development of cells of immune system in the
individual is called as
a) Ontogeny b) Phylogeny
c) Allogeny d) Heterogeny
(GCET-14)

176) B 177) C 178) A 179) C 180) B 181) D 182) D 183) A 184) A 185) B 186) C 187) C
188) B
, MICROBIOLOGY

1. GENERAL MICROBIOLOGY - ANSWERS


1. A' [Ananthanarayan 8th ed 98 / 9th ed 97) 12. 'C' [Ananthanarayan 8th ed 31 / 9th ed 30)
IgA is 2nd most abundant class of imm unoglobulins. Hot air oven is most widely used method of
It exists in two forms i.e. serum IgA and secretory IgA . sterilization by dry heat. A holding period of 160° C
Secretory IgA is synthesized by plasma cells situated for one hour is used to sterilize glassware, swabs and
near mucosal or glandular epithelium and protects pharmaceutical products like Liquid paraffin, dusting
IgA from denaturation by bacterial proteases. powder etc..

2. 'C' [Ananthanarayan 8th ed 97 / 9th ed 96) Culture media are sterilized by filtration or autoclave.
IgG is major serum immunoglobulin and constitutes
80% of the total. It has a half life of 23 days. 13. 'D'[Ananthanarayan 8th ed 13 / 9th ed 10)
TYPES OF MICROSCOPY:
3. ' B' [Ananthanarayan 8th ed 33 / 9th ed 31) i) In optical or Light microscope, examination of
wet films or hanging drops indicates the shape,
4. ' B' [Ananthanarayan 8th ed 3 / 9th ed 3) arrangement, motility and approximate size of
cells.
5. ' D' [Ananthanarayan 8th ed 119 / 9th ed 122)
Both C3a and C5a are anaphylatoxic and chemotactic
ii) In phase contrast microscopy the structures
in nature.
within the bacterial cell and the surrounding
medium, which differ in thickness or refractive
6. 'C' [Ananthanarayan 8th ed 97, 98 / 9th ed 96)
index are clearly visible. In this microscope, the
IgG is a major serum immunoglobulin, consist ing
"phase differences" are converted into differences
about 80% of the total. Fore subclasses of lgG have
in intensity of light, producing light and dark
been recognized (IgGl, lgG2, IgG3, IgG4 ). The four
contrast in the image.
IgG subclasses are distributed in human serum in
the approximate proportions of 65%, 23%, 8%, 4%,
respectively. iii) In Dark field microscope, reflected light is used
instead of the transmitted light used in ordinary
7. ' B' [Ananthanarayan 8th ed 167 / 9th ed 166) microscope. With visible light, the limit of
Both arthus reaction and serum sickness are examples of resolution is about 300n m.
imm une complex diseases or type III hypersensitivity
reactions. ARTHUS reaction is a local manifestation of
iv) The interference microscope not on ly reveals
generalized hyper-sensitivity while SER UM SICKNESS
cells organells but also quantitative measurements
is a generalized reaction.
of the chemical constit uents of cells such as lipids,
proteins and nucleic acids.
8. ' D' [Ananthanarayan 8th ed 100 / 9th ed 164)
IgE is responsible for anaphylactic type of
hypersensitivity. Elevated levels of lgE are seen in v) The polarization microscope enables the study
allergic conditions like hay fever, eczema, and asthma. of intracellular structures using differences in
lgE is believed to have a special role in parasitic birefringence.
infections.
vi) In electronic microscope, a beam of electrons is
9. 'A' [Ananthanarayan 8th ed 37 / 9th ed 35) employed instead of beam of light. The resolving
power is 0.1nm.
10. ' D' [Ananthanarayan 8th ed 127 / 9th ed 113)

11. 'A' [Ananthanarayan 8th ed 32 / 9th ed 30)


Dental ;lut.,e

14. 'C' [Ananthanarayan 8th ed 24 / 9th ed 22] 21. 'A' [Ananthanarayan 8th ed 163 / 9th ed 163]
Phases of bacterial growth curve:
A} Lag phase: 22. 'B' [Ananthanarayan 8th ed 30 J 9th ed 29]
• Adaptation of bacteria to new environment
23. 'A' [Ananthanarayan 8th ed 32 / 9th ed 30]
• Bacteria have maximum cell size towards the
In hot air oven, a holding jDeriod of 160°( for one
end of Lag phase.
hour is used to sterilize instruments. The optimal time
B} Log or Exponential phase: and temperature required for sterilization in an auto
• The cells start dividing and regular growth of clave is 121 °( for 15 minutes only.
bacteria occurs. The cells stain uniformly.
• The bactericidal drugs are more active in this 24. 'D' [Ananthanarayan 7th ed 29 J 9th ed 33]
phase.
25. 'D' [Ananthnarayan 8th ed / 9th ed 28]
C) S_tationary phase:
Sterilization kills all living microorganisms either in
• Zero growth rate is observed since birth rate
vegetative or spore state.
equals death rate. S_porulation occurs at this
stage.
26. 'C' [Ananthanarayan 8th ed 36 / 9th ed 35]
• The cells show irregular staining.
D) Phase of decline: 27. 'A' [Ananthanarayan 8th ed 38 / 9th ed 36]
• Decline in number of organisms. "Rideal walker test" is used to test the efficiency of a
disinfectant. It is also known as 'phenol coefficient"
• Involution forms are common in this phase.
test. Salmonella typhosa is the test organism.
15. 'D' [Ananthanarayan 8th ed 162 / 9th ed 163]
28. 'B' [Ananthanarayan 8th ed 39 / 9th ed 3 9]
16. 'B' [Ananthanarayan 8th ed 78 / 9th ed 75]
29. 'B' [Ananthanarayan 8th ed 46 / 9th ed 46]
Exotoxins Endotoxins The most reliable and widely used anaerobic method
Proteins Li popolysacch arides is mcintosh - fildes anaerobic jar.

Heat labile Heat stable


30. 'B' [Ananthanarayan 8th ed 31 / 9th ed 31]
Highly antigenic Weakly antigenic
Generally formed Endotoxins form an integral part 31. 'A' [Ananthanarayan 8th ed 66 / 9th ed 61]
by G+ve bacteria of cell wall of G -ve bacteria Most of the drug resistance occurs due to plasmid
mediated conjugation. Plasmids (episomes) are
extrranuclear genetic elements and are known as
17. 'D' [Ananthanarayan 8th ed 98 / 9th ed 9 7]
cytoplasmic carriers of genetic information.
18. 'A' [Ananthanarayan 8th ed 13 / 9th ed 11]
32. 'C' [Ananthanarayan 8th ed 36 / 9th ed 34]
2% Glutaraldehyde is known as "CIDEX"
19. 'C' [Ananthanarayan 8th ed 2 2 J 9th ed 31]
Method of sterilization Test organism 33. 'B' [Ananthanarayan 8th ed 37 / 9th ed 36]
Spores of a non-toxigenic
Dry heat
strain of clostridium tetani 34. 'C' [Ananthanarayan 8th ed 35 / 9th ed 33]
U.V.rays are used for disinfecting enclosed areas such
Spores of Bacillus stean-
Sterilization heat as operation theatres and laboratories
thermophilus

Rideal-Walker . test or Sa lmone lla typ h"1 bac1"ll"1 35. 'K [Ananthanarayan 8th ed 85 / 9th ed 80]
Phenol coeffi c1ent test Lysozyme is present in tears, egg, saliva and nearly in
all secretions except in CSF, sweat and urine.
20. 'C' [Ananthanarayan 8th ed 40 / 9th ed 40]
, MICROBIOLOGY

36. 'A' [Ananthanarayan 8th ed 163 / 9th ed 161] • Inoculation of such pure cultures into suitable
laboratory animals should reproduce the lesions of
37. 'D' [Ananthanarayan 8th ed 123 J 9th ed 132] the disease.
Plasma cell is the antibody secreting cell and it is • It should be possible to reisolate the bacterium
formed from the B- lymphocytes. While plasma cell in pure culture from the lesions produced in the
is main antibody producing cell, lymphocytes also experimental animals.
synthesizes antibodies to some extent.
47. 'D' [Ananthanarayan 8th ed 88 / 9th ed 81]
38. 'D' [Ananthanarayan 8th ed 33 / 9th ed 31] A) Naturally acquired passive immunity:
For media containing sugars or gelatin an exposure of Placental transfer of antibodies fro m mother to
100°( for 20 minutes on three successive days is used. child.
This is known as "Tyndallisation" or "Intermittent
sterilization".
B) Naturally acquired active immunity:
• Results from either a clinical or an inapparent
The principle is that the first exposure kills the
infection by a parasite.
vegetative bacteria, and the spores, which are in a
favorable medium, will germinate and are killed in the • Most of the adults possess natural active
subsequent occasions. immunity to poliomyelitis due to repeated
inapparent infections with the polioviruses
fro m childhood.
39. 'D' [Ananthanarayan 8th ed 34 J 9th ed 30]

40. 'B' [Ananthnarayan 8th ed 31 / 9th ed 29] C) Artificially acquired passive immunity:
The germicidal effect of sunrays is due to its Injection of antibodies in form of vaccination .
ultraviolet rays which are of high frequency and short
wavelengths in electromagnetic spectrum. D) Artificially acquired active immunity:
Antigenic stimulus is given in form of vaccination.
41. 'A' [Ananthanarayan 8th ed 164 J 9th ed 163]
Anaphylaxis is acute, fata l and systemic form of 48. 'A' [Ananthanarayan 8th ed 164 J 9th ed 163]
type I reaction whereas atopy is chronic, non fatal
and localized form of type I reaction. 49. 'A' [Ananthanarayan 8th ed 30 / 9th ed 28]

42. 'B' [Ananthanarayan 8th ed 32 / 9th ed 31] 50. 'B' [Ananthanarayan 8th ed 31 / 9th ed 29]
The killing effect of dry heat is due to protein
43. 'B' [Ananthanarayan 7th ed 125 J 9th ed 136] denaturation, oxidative damage and the toxic effect of
elevated levels of electrolytes. The lethal effect of moist
44. 'A' [Ananthanarayan 9th ed 87] heat is due to denaturation & coagulation of protein.
Hapten is a partial antigen and is unable to stimulate
antibody formation unless it attaches to carrier Based on the explanation you may get confused
molecule. Both haptens and antigens can induce between options B & D. Read below for more
hypersensitivity reactions. clarification.

45. 'B' (Ananthanarayan 8th ed 40 / 9th ed 42] Dry heat dehydrates the cells, and the resulting lack of
water reduces the probability of protein coagulation.
46. 'B' [Ananthanarayan 8th ed 5 J 9th ed 4) Dry heat acts mostly by oxidizing or burning the cell
Koch's postulates: constituents.
• The bacterium should be constantly associated
with the lesions of the disease. Also Microbiology Principles and Explorations by
• It should be possible to isolate the bacterium in Jacquelyn G. Black 8th edition Pg 3 says that dry
pure culture from the lesions heat probably does most of its damage by oxidizing
molecules.
Dental ;lut.,e

Ana Techniqs in Biotechnology By Goutam Each Imm unoglobulin molecule consists of two
Bhowmik Pg 97 and An Introduction to Practical fractions.
Biotechnology by S. Harisha Pg 133 Says that dry a) Fe - Insoluble fraction
heat kills microorganisms through a process of protein b) Fab - Soluble fractio rn.
oxidation rather than protein coagulation.
(This is antigen binding site)
51. 'A' [Ananthanarayan 8th ed 372 / 9th ed 373]
The site of 'C' binding is located on Fcpiece of the lg
Morphology and motility of treponema can be seen
molecule. Eg.: CH2 domain on IgG.
under dark ground/ dark field/ phase contrast
microscope.
IgD, or IgE classes of immunoglobulins do not fix
complement (C).
52. 'A' [Ananthanarayan 8th ed 256 / 9th ed 43]
Robertson cooked media is most widely used fluid
60. 'D' [Ananthanarayan 8th ed 166-67 / 9th ed 166]
medium for culture of anaerobes
Arthus reaction is due to antigen antibody immune
complexes that cause tissue damage and inflammatory
53. 'B' [Ananthanarayan 8th ed 96 / 9th ed 136]
infiltration.
54. 'C' [Ananthanarayan 8th ed 601 / 9th ed 43]
61. 'D' [Ananthanarayan 8th ed 98 / 9th ed 96]
Sabouraud's agar, cornmeal agar and Czapek-Dox are
the commonest culture media used in mycology.
62. 'B' [Ananthanarayan 8th ed 79 / 9th ed 74]
Bacterial exotoxins are very potent immunogenic and
55. 'D' [Ananthanarayan 8th ed 167 / 9th ed 166]
are toxic in nature. They are inactivated by chemical
(formaldehyde or forma lin) or thermal (heat) methods.
56. 'C' [Ananthanarayan 8th ed 356 / 9th ed 167]
Such treatment alters toxin into toxoids, which are
Delayed type hypersensitivity (DTH) skin test is a
antigenic and non-toxic.
standard tool to assess in vivo cell-mediated immunity.
Mantoux method using 4-5 common recalled antigens
Toxoids are used in prevention of Diptheria, Salmonella
is recommended. Average induration size of > or= 5
and Tetanus.
mm defined as a positive test. Erythema is not taken
into account.
63. 'C' [Ananthanarayan 8th ed 35 / 9th ed 36]
Detergents acts on the phosphate groups of cell
57. 'A' [Ananthanarayan 8th ed 98 / 9th ed 96]
membrane. As a result, the membrane looses its semi
permeability and the cell proteins are denatured.
58. 'B' [Ananthanarayan 8th ed 96 / 9th ed 94]
Each immunoglobulin consists of 2 light (L) and
64. 'D' [Ananthanarayan 8th ed 98 / 9th ed 97]
2 heavy chains (H), which are held together by
disulphide bonds. The H chains are structurally and
65. ' D' [Ananthanarayan 8th ed 78 / 9th ed 123]
antigenicaly distinct for each class. Based on this, five
classes of immunoglobulins have been recognized.
66. 'B' [Ananthanarayan 8th ed 88 / 9th ed 150]

Immunoglobulin class H chain


Gram-ve bacilli show an adjuvant effect due to their
lipopolysaccharide fraction. Bordetella pertusis, acts
Ig G Gamma as a good adjuvant for diphtheria and tetanus toxoids
Ig A Alpha in triple vaccine (DPT).
lg M Mu
Aluminum hydroxide or phosphates are other
lg D Delta commonly used adjuvants used in human vaccines.
lg E Epsilon
6 7. 'A' [Ananthanarayan 8th ed 34 / 9th ed 3 2]

59. 'D' [Ananthanarayan 8th ed 97 / 9th ed 94]


, MICROBIOLOGY

68. 'D' [Ananthanarayan 8th ed 93 / 9th ed 89] 79. 'C' [Ananthanarayan 7th ed 42 / 9th ed 378]
Weil - Felix reaction is used in serodiagnosis of typhus
fevers. The antigen used in the Weil - Felix test is 80. 'A' [Ananthanarayan 8th ed 233 J 9th ed 237]
obtained from proteus bacilli.
Culture Media Organisms

Paul Bunnel test is used in diagnosis of Infectious Loeffler's serum slope Corynebacteri um diphtheria
mononucleosis.
Robertson cooked For anaerobic culture
69. 'C' [Ananthanarayan 8th ed 356] media Eg.: Clostridium (COMED-13)
BCG vaccine is given by intradermal injection. Smith noguchi's For anaerobic organisms
medium Eg.: Spirochetes
70. 'A' [Ananthanarayan 8th ed 78 J 9th ed 75]
Exotoxins are heat labile proteins whereas endotoxins Dorset's egg medium Mycobacterium.
are heat stable lipo polysaccharides.
Lowenstein - Selective media for
Jensens medium Mycobacterium tuberculosis
71. 'D' [Ananthanarayan 8th ed 99 / 9th ed 98]
Sabouraud's agar Most of fungi
72. 'B' [Ananthanarayan 8th ed 43, 47 / 9th ed 43,47]
81. 'C' [Ananthanarayan 7th ed 42 / 9th ed 378)
73. 'B' [Ananthanarayan 8th ed 78 J 9th ed 7 4]

82. 'D' [Ananthanarayan 8th ed 291,348 / 9th ed 346)


74. 'C' [Ananthanarayan 8th ed 4 / 9th ed 3) Salmonella & Mycobacterium can be cultured on
Tuberculus bacilli and vibrio cholera are discovered by Dorset's egg media.
Robert Koch.
83. 'B' [Ananthanarayan 8th ed 138, 145 J 9th ed 186)
75. 'D' [Ananthanarayan 8th ed 20 / 9th ed 19]
Flagella are organs of locomotion. 84. 'D' [Ananthanarayan 8th ed 64 / 9th ed 59)
Flagella arranged around the cell In lysogenic or phage conversion, the phage "DNA"
Peritrichous Eg: Typhoid bacilli, Clostridium, confers genetic information to a bacterium. This may
Salmonella influence the antigenic characteristics and change
the bacteria into toxigenic.
Monotrichous Single flagella
Eg: Cholera vibro
Eg.: Diphtheria bacilli.
Lophotrichate Flagella at one end
Eg: Spi rilla 85. 'C' [Ananthanarayan 8th ed 24 / 9th ed 23)

Bipolar (or)
Flagella at both poles 86. 'C' [Ananthanarayan 8th ed 26 / 9th ed 25)
amphi trichous
• "Mesophilic bacteria" grow best at temperatures
of 25-40°C.
76. 'B' [Ananthanarayan 8th ed 39 J 9th ed 39)
• "Psychrophilic bacteria" grow best at tern perature

77. 'C' [Ananthanarayan 8th ed 14 / 9th ed 12] below 20°(. These organisms may spoil the
Indian ink or Nigrosin is useful in demonstration of refrigerated food.
bacterial capsules which do not take simple stains. • "Thermophilic organisms" grow best at
temperatures of 55-80°C. These organisms spoil
78. 'A' [Ananthanarayan 8th ed 36 / 9th ed 35) the underprocessed canned food
Betadine, Tincture of iodine and chlorhexidine are
commonly used skin disinfectants. Chlorhexidine is 87. 'C' [Ananthanarayan 8th ed 16 J 9th ed 14)
most active against gram +ve organisms and fairly
effective against gram-ve organisms.
Dental ;lut.,e

98. 'C' & 'D' [Ananthanarayan 7th ed 89 / 9th ed 96)


ARRANGEMENTS OF BACTERIA
Streptococci Chains. 99. 'B' [Ananthanarayan 8th ed 92 / 9th ed 88)
Staphylococci Grape like irregular clusters Option "D' paratope is the antigen combining site of
the antibody.
Meningo cocci &
arranged in pairs
pnemococci
100. 'B' [Ananthanarayan 8th ed 92 / 9th ed 88)
Corynebacterium cuneiform or chinese pattern
101. 'A' (Ananthanarayan 8th ed 96 / 9th ed 94)
88. 'D' [Ananthanarayan 8th ed 32 / 9th ed 30]
Pasteurisation of milk 102. 'A' [Ananthanarayan 8th ed 99 / 9th ed 9 7)
• Holder method - 63°( for 30 minutes.
103. 'D' [Ananthanarayan 8th ed 98 / 9th ed 97]
• Flash process - 72 °( for 15 - 20 seconds.

104. 'A' [Ananthanarayan 8th ed 118 / 9th ed 121)


89. 'A' [Ananthanarayan 8th ed 31 / 9th ed 29]
Classical complement pathway, which is activated
at (1, is stimulated by lgG and IgM. Alternate
90. 'A & C' [Ananthanarayan 8th ed 36 / 9th ed 34]
complement pathway, which is activated at C3a, is
stimulated by IgA.
91. 'C' [Ananthanarayan 8th ed 36 / 9th ed 36]

105. 'A' [Ananthanarayan 8th ed 105 / 9th ed 106)


92. 'D' [Ananthanarayan 8th ed 66 / 9th ed 62]
Slide flocculation VDRL Test
93. 'A & B' [Ananthanarayan 8th ed 66 / 9th ed 61] Tube floccu lation Kahn test
Tube agglutination Widal test
94. 'C' [Ananthanarayan 8th ed 108 / 9th ed 109)
Weil Felix reaction and Paul bunnel reactions are due Latex For detection of HBsAg,
to heterophil antigens. agglutination and (-reactive protein.
Slide agglutination Diagnosis of Salmonella,
95. 'C' [Ananthanarayan 8th ed 99 / 9th ed 98] Shigella

96. 'D' [Ananthanarayan 8th ed 78 / 9th ed 7 4]


106. 'A' [Ananthanarayan 8th ed 113 / 9th ed 112)

97. 'A' [Ananthanarayan 7th ed 104 / 9th ed 112]


107. ' B' [Ananthanarayan 8th ed 168 / 9th ed 167)
Opsonization is the process of increasing ability
for phagocytosis of foreign body. In opsonisation,
108. 'C' [Ananthanarayan 8th ed 108 / 9th ed 105)
opsonin antibodies (C3b, IgM, IgG) attach to the
bacteria and facilitate the phagocytosis of that
109. ' B' [Ananthanarayan 8th ed 91 / 9th ed 87)
bacterium.
Hapten is a partial antigen and is unable to stimulate
antibody formation unless it attaches to large carrier
IgG Protects the body fluids molecule.
IgA Protects the body surfaces
110. 'A' [Ananthanarayan 8th ed 148, 447 / 9th ed 115)
IgM Protects the blood stream Interferons and natural killer cells kill virus-infected
Mediates reaginic (allergic) hyper cells. Interferons increases p53 activity, which kills
IgE
sensitivity virus-infected cells by promoting apoptosis.
Recognisation molecule on the surface
IgD 111. ' D' [Ananthanarayan 8th ed 127,149 / 9th ed 113)
of B lymphocytes

112. ' B' [Ananthnarayan 8th ed 148 / 9th ed 154)


, MICROBIOLOGY

113. ' B' [Ananthnarayan 8th ed 31 / 9th ed 30] 123. 'A' [Ananthnarayan 8th ed 17 / 9th ed 16]

114. ' D' [Ananthnarayan 8th ed 89 / 9th ed 97] 124. 'D' [Ananthnarayan 8th ed 35 / 9th ed 33]

115. 'A' [Ananthnarayan 8th ed 97 / 9th ed 96] 125. 'A' [Ananthnarayan 8th ed 451 / 9th ed 489]
IgG has four subclasses (IgGl, IgG2, IgG3, IgG4) Yellow fever vaccine is a vaccine used against yellow
each possessing a distinct type of gamma chain, fever. The vaccine consists of a live, but attenuated,
identifiable with specific antisera. The four IgG strain of the yellow fever virus called 170.
subclasses are distributed in approximate proportions
of 65%, 25%, 8% and 4% respectively. The human diploid cell rabies vaccine (H.D.C.V.)
is a cell cultured inactivated rabies vaccine for pre-
116. ' D' [Ananthnarayan 8th ed 88 / 9th ed 82] and post-exposure prophylaxis in humans. The virus
Innate or Native immunity is the resistance to is propagated on human diploid cells. The virus is
infections which an individual posses by virtue of his inactivated with B propiolactone.
genetic and constitutional make-up. It is not affected
by prior contact with microorganisms or immunization. 126. 'C' [Ananthnarayan 8th ed 166 / 9th ed 166]
It may be considered at the level of the species, race
or individual. The overall im munity in a community 127. 'B' [Ananthnarayan 8th ed 168 / 9th ed 167]
and is relevant in the control of epidemic diseases is Type IV or delayed hypersensitivity reactions
called Herd immunity. are typically provoked by intracellular microbial
infections or haptens like simple chemicals applied on
A community having resbtance to particular skin. The reaction is induced by sensitized 'T' cells,
infection is showing which kind of immunity? which on contact with the specific antigen, release
(PGI June-2012) cytokines that cause biological effects on leucocytes,
a) Herd immunity b) Active immunity macrophages and tissue cells.
c) Passive immunity d) Humeral immunity
128. 'B' [Ananthnarayan 8th ed 130-31 / 9th ed 166]
117. 'C' [Ananthnarayan 8th ed 32, 34 / 9th ed 32] When circulating lymphocytes are classified by their
surface markers into T and B cells, about 5 - 10% of
• The spores of a nontoxigenic strain of clostridium
the cells lack features of either type. They are called
tetani are used as a microbiological test of dry
null cells or large granular lymphocytes (LGL). The
heat efficiency.
most important member of this group is natural killer
• The spores of "bacillus stearoth ermophilus (NK) cells. Others are - antigen dependent cytotoxic
(COMED-13)" are used for testing the effi ciency cells (ADCC) and the lym phokine activated killer
of moist heat.
(LAK) cells. The term NK cell is used as a common
name for all null cells. Natura l killer cell possess
118. ' D' [Ananthnarayan 8th ed 88 / 9th ed 82] spontaneous cytotoxic (remember that type II
Active artificial immunization is induced main ly by
reactions are cytotoxic) towards various target cells
vaccines. Vaccines are preparations of live or killed mainly malignant and virus infected cells
microorganisms or their products used for immunity.
Option 'D' is example of artificial passive imm unity.
129. 'B' (Ananthnarayan 8th ed 166 / 9th ed 166)
In some type II reactions, the antibody combines with
119. 'C' [Ananthnarayan 8th ed 348 / 9th ed 346] cell surface receptors and dirupts norma l function ,
either by uncontrolled activation ( agonist effect as
120. 'D' [Ananthnarayan 8th ed 166 / 9th ed 162] caused by antibody "Long- acting thyroid stimulator
in Grave's disease or thyrotoxixosis)
121. 'D' [Ananthnarayan 8th ed 88 / 9th ed 81]
130. 'B' (Ananthnarayan 8th ed 166 / 9th ed 166)
122. ' D' [Ananthnarayan 8th ed 178 / 9th ed 182] Lysis of RBC due to inco mpatibility, autoimmune
Dental ;lut.,e

anemias, hemolytic disease of new born, hemolytic


anemia, agranulocytosis and thrombocytopenic Eg.: Forssman antigen is widely present in animals,
purpura caused by drugs and myasthenia gravis are birds, plants and bacteria except rabbits. So anti-
examples of Type II reactions. Forssman antibody can be prepared in these
aimals.
Cytokines are biologically active substances released
by T-lymphocytes (lymphokines) or by monocytes • Other heterophile antigens are responsible for some
or macrophages (monokines). The term interleukin diagnostic serological reactions in which antigens
was introduced for those products of leucocytes unrelated to etiological agents are employed.
which exert a regulatory influence on other cells. Eg.: The Weil-Felix reaction is typh us fever, Paul
Interferons, growth factors and others were fo und to Bunnel test in infectious mononucleosis cold,
have similar effects. Therefore, all of them have been agglutinin test in primary atypical pneumonia.
grouped under the term cytoki nes.
133. ' B' [Anantharayan 8th ed 130 and Refer Synopsis
They are always peptide mediators or intercellular for MHC complex/ 9th ed 139]
messengers. They differ from endocrine hormones in In humans
being produced not by specialized glands but by widely • MHC antigens= HLA antigens
distributed cells and acting not systemically but
• MHC complex= HLA complex
locally on the producing cells (Paracrine effect) or
directly on the producing cells themselves (autocrine
There are two types HLA antigens
effect).
1) HLA or MHC class I antigens (antigen A, Band C)
are found on surface of all nucleated cells.
131. 'D' [Ananthnarayan 8th ed 147, Tab 16.2 / 9th ed
154] Functions:
Inter leukin Secreted by • Principal antigens involved in graft rejection .
IL-1, IL-8 Macrophages and Monocytes • Function as components of hormone receptors.
• Associated with CDS cells in MHC Class I
IL-2, IL-3 T cells
rest riction.
IL-4, IL-5 TH cells
IL-6 Tw Macrophages, fibroblasts 2) HLA or MHC class II antigens
IL-7 Spleen, Bone marrow stromal cells Restricted to immune system cells like
IL-9, 12, 13 T-cells macrophages, dendritic cells, activated T cells and
B cells.
IL-10 T, B Cells macrophages
Functions:
IL-11 Bone marrow stromal cells
• Responsible for graft versus host response and
mixed leukocyte reaction.
IL-I causes proliferation and differentiation of T,
B and other cells (mainly T-cells). By stimulating • Associated with CD4 helper cells in MHC class
T-cells, it produces IL-2 and other lymphokines. II restriction.

MHC restriction:
Interleukin 2 is secreted by (KERALA-2015)
T cells respond to processed antigens on the
a) B Lymphocyt es b) T Lymphocytes
macrophages and other cells only when they are
c) Macrophages d) Monocytes
presented along with the self MHC antigen. This is
the importance of MHC antigens in immune system.
132. ' D' [Ananthnarayan 8th ed 93 / 9th ed 109]
This phenomenon is called MHC restriction.
• Heterophile antigens are the same or closely
related antigens in different biological species,
134. 'D' [Ananthnarayan 8th ed 128 / 9th ed 128]
classes and kingdoms.
, MICROBIOLOGY

146. 'B' [Ananthnarayan 8th ed 96 / 9th ed 96]


135. 'C' [Ananthnarayan 8th ed 128 / 9th ed 129] Each immunoglobulin molecule consists of two pairs
T-cell precursors from the yolk sac, fetal liver and bone of polypeptide chains of different sizes. The smaller 'L'
marrow migrate to the thymus during the embryonic chain has a molecular weight of approximately 25,000
and postnatal stages. and the heavy "H" chain of 50,000. The L-chain is
attached to the H-chain by a disulphide bond. The
136. 'B' [Ananthnarayan 8th ed 130 / 9th ed 133] two 'H' chains are joined together by 1-5-S-S bonds.
Based on their surface markers, target cells and
fun ctions, T cells are classified as: 14 7. 'A' [Ananthnarayan 8th ed 9 6 / 9th ed 94]
• Helper/inducer cells, with CD4 surface marker
148. 'A' [Ananthnarayan 8th ed 139 / 9th ed 96]
• Suppressor T cells with CD8 surface marker
The antibody response to initial antigenic stimulus
• Cytotoxic T cell with CD8 surface marker and differs qualitatively and quantitatively from the
• Memory cells both CD4 and CD8 markers response to subsequent stimuli with the same antigen.
The former is called the primary response (slow,
137. 'C' [Ananthnarayan 8th ed 18 / 9th ed 16] sluggish and short-lived with a long lag phase) and
The lipopolysaccharides (LPS) present on the cell the latter is the secondary response (prompt, powerful
walls of gram -ve bacteria account for their endotoxic and prolonged with a short Lag phase). The antibody
activity and 'O' antigen specificity. They were formerly formed in the primary response is predominantly
known as the boivin antigen . The LPS consists of IgM {COMEDK-08) and in the secondary response is
three regions. Region I is the polysaccharide portion IgG.
determining the 0-antigen specificity. Region II is the
core polysaccharide. Region III is a glycolipid portion A single injection of an antigen helps more sensitizing
(lipid A) and is responsible for the endotoxic activities. or priming the immunocompetent cells producing the
particular antibody than in the actual elaboration of
138. 'C' [Ananthnarayan 8th ed 36 / 9th ed 34] high levels of antibody. Effective levels of antibody
Cidex (2% Glutaraldehyde) is used for sterilizing are usually induced only by subsequent injections
instruments such as cytoscopes, endoscopes, and of antigen. For this reason that non living vaccines
bronchoscopes. It can be safely used to treat are given in multiple doses for active immunization.
corrugated rubber anesthetic tubes, face masks, The first injection is known as the priming dose and
plastic endotracheal tubes, metal instruments and subsequent injections are known as booster dose.
polythene tubing. With live vaccines, a single dose is sufficient as
multiplication of the organism in the body provides
139. 'D' [Ananthnarayan 8th ed 32 / 9th ed 30] a continuing antigenic stimulus that acts as both
priming and booster dose.
140. 'B' [Ananthnarayan 8th ed 31 / 9th ed 30]
When an antigen is injected into an animal already
141. 'D' [Ananthnarayan 8th ed 31-3 2 / 9th ed 32] carrying the specific antibody in circulation, a
Vaccines are steri lized by moist heat below 100°(. temporary fall in the level of circulating antibody
Vaccines of non-sporing bacteria are heat inactivated occurs due to combination of the antigen with the
in special vaccine baths at 60°( for one hour. antibody. This is ca lled negative phase. It is followed
by an increase in titre of antibody exceeding the
142. 'A' [Ananthnarayan 8th ed 31 / 9th ed 30] initial level.

143. 'B' [Ananthnarayan 8th ed 31 / 9th ed 30] For induction of hypersensitivity reactions, the host
should have had contact with the antigen. The initial
144. 'B' [Ananthnarayan 8th ed 32 / 9th ed 30] contact sensitizes the immune sytem, leading to
priming of the appropriate 'B' or 'T' lymphocytes. This
145. 'B' [Ananthnarayan 8th ed 4 / 9th ed 4] is known as sensitizing or priming dose. Subsequent
contact with the allergin causes manifestations
Dental ;lut.,e

of hypersensitivity. This is known as the


'shocking dose'. Serum sickness differs from other Presentation of cutaneous anthrax (Malignant
types of hypersensitivity reaction in that a single pustule)
injection can serve both as sensiting dose and the • Papular stage:
shocking dose.
Begins as papule following introduction of spores
through an opening in the skin and resembles an
149. 'A' [Ananthnarayan 8th ed 99 / 9th ed 98]
insect bite.

150. 'A' [Ana nth narayan 8th ed 100 / 9th ed 98]


• Vesicular stage:
151. 'D' [Ananthnarayan 8th ed 100 / 9th ed 162] Papule evolves into a painless vesicle.

152. 'A' [Ananthnarayan 8th ed 98 / 9th ed 99] • Ulcerative stage:


Vesicles change into necrotic ulcer wit h a
153. 'A' [Ananthnarayan 8th ed 118, 119 / 9th ed 118]
charact eristic central black eschar, marked edema,
IgG and IgM are involved in classical pathway while
lymphangitis, fever, malaise and headache and
IgA and IgD are involved in alternate complement
other systemic symptoms.
pathway.

• Mc Fadyean Reaction:
Among subclasses of IgG, IgG7 is not involved in
complement pathway. When blood film containing anthrax bacilli are
stained with polychrome methylene blue for a few
154. 'D' [Ananthnarayan 8th ed 387 / 9th ed 16] seconds and examined under the microscope, an
amorphous purplish material is noticed around the
155. 'A' [Ananthnarayan 8th ed 128 / 9th ed 134] bacilli. This represents the capsular material.

156. 'C' [Ananthanarayan 8th ed 30 / 9th ed 28] 158. 'C' [Ananthanarayan 8th ed 206 / 9th ed 215]
Sterilization: Gram positive cocci in chains are streptococci. Majority
Defined as the process by which an article, surface or of hemolytic streptococci that cause human infection
medium is freed of all living microorganisms, either in belong to Group A i.e., streptococcus pyogenes.
the vegetative or spore state.
Streptococcus pyogenes (Group A) can be
Disinfection: differentiated from other hemolytic gram positive
Destruction or removal of all pathogenic organisms or cocci in chains by Bacitracin sensitivity.
organisms capable of giving rise to infection.
Streptococci from staphylococci Catalase test
Antiseptics:
Group ASt reptococci from other
Chemical disinfectants which can be safely applied Bacitracin test
~-hemolytic streptococci
to skin or mucous membrane and are used to prevent
infection by inhibit ing the growth of bacteria. Pneumococci from other
Optochin test
a-hemolytic streptococci
157. 'A' [Ananthanarayan 8th ed 246 / 9th ed 247] Staphylococcus aureus from
The occupation of patient and the clinical presentation Coag ulase test
other staphylococci
is suggestive of cutaneous anthrax. Polychrome
methylene blue strain produces the Mc Fadyean 159. 'D' [Ananthanarayan 8th ed 162 / 9th ed 166]
reaction that is characteristic of anthrax bacillus. Ref. Synopsis
Butchers, farmers and workers in industries such
as meat packing or wool factories are commonly 160. 'A' [Ananthanarayan 8th ed 35 / 9th ed 33]
affected, abattoir stands for slaughter house and Ionising radiation:
people working in these are at special risk.
, MICROBIOLOGY

X-rays, gamma rays and cosmic rays are lethal to reactions comprises of localized reaction called Arthus
DNA and other vital constituents. They have very reaction and generalized (systemic) form called Serum
high penetrative power. Since there is no appreciable sickness.
increase in temperature in this method, it is referred
to as cold sterilization . Commercial plants use gamma Immune complexes occur in many diseases, including
radiation for sterilizing items like plastics, syringes, bacterial, viral and parasitic infections (Eg.: Post
swabs, catheters, animal feeds, card boards, oils, streptococcal glomerulonephtritis, hepatitis 8, malaria),
greases fabrics and metal foils. disseminated malignancies and autoimmune conditions.

161. 'C' [Ananthanarayan 8th ed 166 / 9th ed 166] 165. 'B' [Ananthanarayan 8th ed 16 / 9th ed 387)
Arthus observed that when rabbits were repeatedly
injected subcutaneously with normal horse serum, 166. 'C' [Ananthanarayan 8th ed 78 / 9th ed 74]
there occurred intense loca l reaction consisting of Pat hogenicity is the ability of a microbial species to
edema, induration and hemorrhage necrosis. This is produce disease while the term Virulence is applied
known as Arthus reaction. The tissue damage is due to the same property in a single organism.
to the formation of antigen-antibody precipitates
causing complement activation and release of Enhancement of virulence is known as Exaltation and
inflammation molecules. can be demonstrated experimentally by serial passage
in susceptible hosts.
162. 'C' [Ananthanarayan 8th ed 16 / 9th ed 16]
Contents of Bacterial cell: Reduction of virulence is known as atten uation and
• Cell wall can be achieved by passage through unfavourable
• Plasma membrane hosts, repeated cultures in artificial media, growth in
• Cytoplasm high temperature or in presence of weak antiseptics,
• Ribosomes dessication or prolonged storage in culture.
• Mesosomes
167. 'B' [Ananthanarayan 8th ed 50 / 9th ed 635)
• Granules
• Vacuoles 168. 'D' [Ananthanarayan 8th ed 91 / 9th ed 88]
• Nuclear body (DNA)
• Capsule
• Flagella
• Fimbriae

Note: IL-1 (a ,~) • Proliferation and differentiation of


The bacteria without cell wall are Mycoplasma. T, B and other cells
• Induce acute phase proteins
163. 'A' [Ananthanarayan 8th ed 86 / 9th ed 146) IL-2 • Promote growth and
Active immunity is the resistance developed by an differentiation of T and B cells
individual as a result of antigenic stimulus. During • Promote cytotoxicity of T & NK
development of active immunity, there is often a cells
negative phase during which the level of measurable • Secretion of other lymphokines
immunity may actually be lower than it was before IL-3 • Colony stimulating factor
the antigenic stimulus. This is because the antigen
IL-4 • Proliferation of B and cytotoxic T
combines with any pre-existing antibody and lowers
cells lg E production
its level in circulation. In passive immunity there is
• Enhance MHC class II and lg E
no negative phase.
receptors
164. 'C' [Ananthanarayan 8th ed 168 / 9th ed 166] IL-5 • Proliferation of eosinophils
Immune complex diseases or Type III hypersensitive • Stimulate lgA and lgM production
Dental ;lut.,e

• To be effective they must be used at a concentration


IL-6 • Promote B cell differentiation, IgG
production of 60 - 90% in water.
IL-7 • B and T cell growth factor
Lysol:
IL-8 • Neutrophil chemotactic factor
• Active against a wide range of organisms.
TUMOR NECROSIS FACTORS • They are not readily inactivated by the presence
TNF-a Tumour cytotoxicity, lipolysis, wasting, of organic matter and are thus good general
phagocytic cell activation, antiviral disinfectants.
and anti parasitic effects, endot oxic
shock Ch loroxylenol:
TNF-P Induce other cytokines' • It is a phenolic compound. The commercial 4.8%
solution is known as dettol and is used for surgical
COLONY STIMULATING FACTORS
antisepsis and disinfection.
GM-CSF T-Cell and macrophage growth 174. 'C' [Ananthanarayan 7th ed 351 / 9th ed 13)
stim ulation Mycobacteria do not stain readily with gram stain.
G-CSF Granulocyte growth stimulation After staining with basic dyes they resist discoloration
M-CSF Macrophage growth stimulation by alcohol. So Gram staining is not useful. They can
INTERFERONS be stained with carbol fuchsin by the ziehl-Neelsen
IFN-p Antiviral activity method or fluorescent dyes (auramine 0, rhodamine) .
IFN-a Antiviral activity, macrophage
175. 'A' [Ananthanarayana 7th ed 161 / 9th ed 163)
activation, MHC Class I & II expression
on cells
17 6. 'B' [Ananthanarayana 9th ed 121)

170. 'D' [Ananthanarayan 9th ed 33)


177. 'C' [Ananthnarayan 7th ed 165 / 9th ed 167)
Serum sickness, which is a type III hypersensitivity
171. 'A' [Ananthanarayan 8th ed 86 / 9th ed 137)
reaction (Immune complex disease), differs from
Refer to explanation of Q. No. 128
other types of hypersensitivity reaction in that a
single injection can serve both as sensitising dose
172. 'A' [Ananthanarayan 8th ed 12 / 9th ed 10)
and shocking dose. This syndrome is currently more
commonly seen following injection of penicillin.
173. 'B' [Ananthanarayan 8th ed 36 / 9th ed 163)
Cetrimide:
178. 'A' [Ananthanarayan 8th ed 118 / 9th ed 121)
• Soapy powder with a faint fishy odour used as
• Complement is a complex of 9 factors Cl to C9.
1- 3% solution. It has good cleansing action,
There are 2 pathways for complement activation
efficiently removes dirt, grease, tar and congea led
blood from road side accident wounds. - Classical pathway
• In combination with chlorhexidine, it is one of the - Alternate pathway (Properdin pat hway)
most popular hospitals antiseptic and disinfectant • These pathways differ only in initial steps. Once
for surgical instruments, utensils, etc. C3 activation occurs, the subsequent steps
are common in both pathways. So with C3, the
Alcohol: common pathway starts.
• Ethanol (ethyl alcohol) and isopropyl alcohol are
most frequently used. Also refer Q. No. 104.
• These are mainly used as antiseptics and act by
179. 'C' [Check Explanation Below]
denaturing bacterial prot eins.
Ninhydrin staining is used to assess the area of filter
• They have no action on spores. paper wetted by gingival crevicular fluid.
, MICROBIOLOGY

180. 'B' [Anathanarayana 8th ed 123 / 9th ed 84) 182. 'D' [Ananthnarayana 6th ed 23 / 9th ed 28)
Both T and B lymphocytes, plasma cells and antigen
presenting cells are primarily concerned with the 183. 'A' [Immunology by Klaus, & Elgert 2nd ed 14)
immune response. Phylogeny:
The developmental history of the immune system
during evolution
181. 'D' [Elements of immunology by Fahim Halim Khan
Ed 345 / Ananthanarayana 9th ed 310]
Ontogeny:
TYPES OF VACCINES: The developmental history of the immune system in
1. Natural live vaccines: an individual within a species. The first antibody to
Preparations include natural non-pathogenic appear in phylogeny is also the first to appear during
organisms but which still produce specific immunologic development in an individual. Thus,
immunity. ontogeny recapitulates phylogeny' and vice versa.
Eg:- Sma ll pox and cow pox vaccines.
2. Live attenuated vaccines: 184. 'A' [Ananthanarayan 9th ed 60)
Bacteria or virus which are attenuated or Plasmids (DNA) are extra-chromosomal elements that
weakened, such that they do not cause any are frequently transferred in conjunction. Cell to cell
disease are included in the vaccines. contact occurs for gene transfer.
Eg:- BCG, MMR, 17D, Varicella - zoster, Sabin
polio and typhoid vaccine. 185. 'B' [Check Synopsis]

3. Inactivated or killed vaccines: 186. 'C' [Ananthanarayan 9th ed 8/ Check Synopsis]


Eg:- Salk polio, Rabies, Pertusis vaccines. Ru ska Electron microscope
Study of antitoxin and toxin
4. Toxoids: Paul Ehrlich
neutralization
These are non -toxic yet immunogenic derivatives
Joseph Lister Antisepsis
of exotoxins. Eg:- Diphtheria and tetanus vaccines
18 7. 'C' [Text book of medical biochemistry by Vasudevan
5. Recombinant vaccines:
8thed 251/ Lippincott biochemistry 5th ed 396)
Employs vaccines with recombinant DNA
One of distinctive characteristics of bacteria is that
technology
it possesses haploid (single set of chromosomes).
Eg:- HBs Ag (hepatitis B) vaccine
But, in cases of a bacteria cell that received a new
piece of DNA via some type of genetic process (Eg:
6. DNA vaccines:
Conjugation), it is possible to have two distinctly
Eg:- Rabies, influenza, HIV vaccines
different sets of genetic information is the same
cytoplasm. Such cell is effectively diploid for that
7. Viral vector vaccines
information; however, mos.t DNA transferprocesses
Live Viruses are used Vaccines.
move only a fraction of the total genome, and the
Eg:- Vaccinia virus, adeno virus and Canary Pox
resulting cell is on ly a merodiploid, or partial
virus
diploid.

8. Bacterial vector vaccines:


Note: But, according to some authors, bacteria have
Attenuated bacteria are used in vaccines
their DNA in a single circular chromosome (not a set)
Eg:- Cholera, Typhoid
and reproduce via asexual process like binary fission
(not by mitosis or meiosis),the concept of haploid or
9. Polysaccharide vaccines:
diploid is not actually applicable to bacteria.
Eg:- Hemophilus influenza type B vaccine,
Vaccine for pneumococcal pneumonia, Vaccine for
meningococcal meningitis, Typhoid (Newer)
Dental ;lut.,e

Common features of Prokaryotic and eukaryotic cell:


• Genetic information encoded in DNA using
identical genetic code.
• Similar mechanism of transcription, translation
including similar ribosomes.
• Shared metabolic pathways (Eg: Glycolysis and
TCA Cycle)
• Chemical energy in the form of ATP.
• Presence of RNA Synthesizing enzymes.
• Genes are often interrupted by sequences that do
not appear in the final RNA. These intervening
sequences that are removed are called introns.

188. 'B' [Disinfection, Sterilization and Preservation by


Seymour S. Block 5th ed 895)
Semi-critical items like endoscopes, laryngoscopes,
and anesthesia breathing circuits that contact but
do not penetrate mucous membranes, require high
level disinfection, that means, these items must be
free of microorganisms other than bacterial spores.

Examples of high level disinfectants:


• Glutaraldehyde
• Ortho-phthaladehyde (OPA)
• Stabilized hydrogen peroxide
• Chlorine and its compounds

Non-critical items come into contact with intact


skinonly. As skin is an effective barrier, these
instruments required a low level disinfectants like:
• Alcohols
• Hypochlorite
• Phenolic solutions
• Iodophor
• Quaternary ammonium solutions
, MICROBIOLOGY

2. BACTERIOLOGY
1. The causative organism of syphilis is c) Flagella d) None of the above
a) Borrelia burgolorferi (KAR-97; MAN -2001)
b) Chalamydia trachomatis 9. The specific test for Treponema pallidum is
c) Leptospira interrogans a) VDRL b) Kahn's test
d) Treponema pallidum c) TPHA d) Widal test
(AIPG -94) (MAN-2002)
2. On a stained slide, clostridium tetani has the 10. Electron microscopic examination of the bacterial
appearance of a flora of necrotising ulcerative gingivitis indicates
a) Bunch of grapes b) chain of beads the presence of microorganisms within noo-
c) Drum stick d) safety pin n ecrotic tissues in advance of other bacteria. The
(MAN-95) organisms involved are
3. An oxidase-positive, gram-negative rod which a) Cocci b) Spirochetes
produces a bluish-green pigment has been c) Bacteriophages d) Filamentous rods
grown in culture from a swab obtained from an (MAN-97)
infected burn wound. This organism is susceptible 11. The difference between gram positive and negative
to gentamicin, ticarcillin and tobramicin, but bacteria lies in their
resistant to all the other antibiotics. Which of the a) Cell wall b) Cell membrane
following organisms is it likely to be? c) Nuclear material d) Mesosomes
a) Escherichia coli b) Klebsiella pnemoniae (KAR-97)
c) Proteus mirabilis d) Pseudomonas aeruginosa 12. The following are applicable to bacterial genome
(MAN-95) except:
4. In initial stages of typhoid, salmonella can be a) It is composed of DNA
detected in b) It does not contain histones
a) Feacus b) Urine c) It is circular
c) Blood d) All of the above d) Its DNA has both intrans and extrons
(MAN-98) (KAR-97)
5. Bacteria reproduce mainly by 13. The commonest method of detection of diphtheria
a) budding b) binary fission carriers is:
c) sporing a) Schick test b) Dick test
d) bacterial components produced by host cells c) Casoni's test d) Charles' test
(MAN-99) (AIPG-99)
6. Signs of typhoid fever is/ are 14. Each of the following can cause food poisoning
a) Haemorrhage b) Perforation except:
c) Osteomyelitis d) All of the above a) Cl. difficile b) Staphylococcus
(MAN-97) c) Cl. welchii d) Cl. botulinum
7. All the following are true of streptococcus except (AIPG-99)
a) It's a group of lactic acid producing bacteria 15. Toxic shock syndrome is due to the following
b) Shows adherence to dental enamel virulence factor:
c) Most common in humans a) M protein b) Pyrogenic exotoxin
d) Most common in animals c) Streptolysin 0 d) Carbohydrate cell wall
(MAN-2001) (AIIMS-2003)
8. The structure involved in bacterial attachment to 16. The bacterial cell wall has all of the following
cell surfaces is properties except:
a) Capsule b) Fimbria a) It consists of a mixed polymer called peptidoglycon

1) D 2) C 3) D 4) C 5) B 6) D 7) A 8) B 9) C 10) B 11) A 12) D 13) A


14) A 15) B 16) C
Dental ;lut.,e

b) It is the structure principally responsible for the 25. The genetic ability of a bacterium to grow in the
reaction of gram staining presence of several antibiotics is passed in vivo
c) It is a unique flexible plastic structure from one bacterium to another by:
d) It contains D-isomers of amino acids a) Conjugation b) Rarefaction
(KAR-98) c) Transduction d) Transformation
17. Endotoxins have all but one of the properties given (AIPG-2003)
below: 26. Bacteria acquire characteristics by all of the
a) Produced by gram negative bacteria following except:
b) Produced by gram positive bacteria a) Through plasmids
c) Not secreted outside the cell b) Incorporating part of host DNA
d) Cannot be toxoided c) Through bacteriophages
(KAR-9 7) d) Through conjugation
18. Anaerobic bacteria grow (AIPG-2002)
a) in the presence of oxygen 27. The virulence factors of Neisseria gonorrhea include
b) in the presence of nitrogen all of the fo [lowing except:
c) in the absence of oxygen a) Outer membrane proteins
d) differential media b) lgA Protease
(KAR-99) c) M-Proteins d) Pili
19. Organ of locomotion in bacteria is (AIIMS- 2003)
a) Fimbria b) Flagella 28. Which of the following enzyme is required to
c) Capsule d) Cell wall induce the formation of a plasma clot pathogenic
(KAR-99) staphylococci:
20. Phage conversion is required for: a) Lysozyme b) Fibrinolysin
a) Tularemia b) Diphtheria c) Coagulase d) S protein
c) Gonorrhoea d) All of the above (AIPG-2003, AIIMS-2002)
(AIPG-93) 29. In a patient with typhoid, diagnosis after 15 days
21. For uniform staining reaction, morphology and of onset of fever is best done by:
biochemical activity, it is advisable to study a) Blood culture b) Widal test
bacterial cultures during the: c) Stool culture d) Urine culture
a) Lag phase b) Death phase (AIPG-2002, AIIMS-99)
c) Stationary phase d) Logarithmic phase 30. "Grape bunch" shaped colonies are seen in:
(AIPG-2003) a) Streptococcus b) Staphylococcus
22. Wasserman test is diagnostic of c) E.coli d) Gonococci
a) Syphilis b) Gonorrhea (AIIMS-2K)
c) TB d) Typhoid 31. Staphylococci typically occur in:
(AP-2001) a) Pairs b) Chains
23. One of the following is a treponemal test for c) Tetrads d) Irregular clusters
diagnosis of syphilis (AIPG-2003)
a) FTAB b) Widal test 32. Which of the following is anaerobic:
c) Wasserman test d) Kahn test a) E.coli b) Bacteroides
(KAR-2K) c) Pseudomonas d) Klebsiella
24. Bacteria with potent exotoxin is (AIPG-2001)
a) Clostridium tetani b) Pseudomonas 33. One of the following is an agglutination test in the
c) Shigella d) Klebsiella diagnosis of typhoid:
(AP-97) a) Widal test b) Eleks test
c) Mantoux test d) Wasserman test
(KAR-2K)

17) B 18) C 19) B 20) B 21) D 22) A 23) A 24) A 25) A 26) B 27) C 28) C 29) B
30) B 31) D 32) B 33) A
, MICROBIOLOGY

34. Inactivated microorganisms are used in the b) Staphylococci


manufacture of which of the following: c) Beta hemolytic streptococci
a) Salk vaccine b) Tetanus toxoid d) Pneumococci
c) Sabin's oral vaccine d) ALLof the above (AP-2001)
(AIPG-98) 43. Which of the following is an acid-fast bacillus
35. With regards to temperature requirement, most a) Mycobacterium bacilli
pathogenic bacteria are: b) Treponema pallidum
a) Psychrophiles b) Mesophiles c) Neisseria gonorrhea d) All of the above
c) Cryophiles d) Thermophiles (AP-2001)
(KAR-97) 44. One of the following is a zoonotic disease
36. The major intestinal pathogens which are non- a) Anthrax b) Typhoid
lactose fermenters are: c) Bacillary dysentery d) Cholera
a) Salmonella b) Klebsiella (KAR-2001)
c) Escherichia d) Paracolons 45. Diphtheria is pathogenic only when infected with
(AIPG-98) a) Beta phage b) Alfa phage
3 7. A protoplast is best characterized as a bacterial c) Lambda k12 d) Delta phage
cell: (KAR-2001)
a) With a cell wall but free of a capsule 46. The facultative anaerobic bacteria are those who
b) Containing a cell wall and a capsule are:
c) Free of a cell wall and a capsule a) Unable to grow in t he presence of oxygen .
d) Uniquely sensitive to penicillin b) Unable to grow in t he absence of oxygen.
(AIPG-2003) c) Able to grow in the presence of carbon dioxide.
38. Malignant pustule is referred to: d) Able to grow in the presence or absence of oxygen.
a) Facio-cervical actinomycosis (KAR-97)
b) Cutaneous antlnrax 47. The most common site, where organisms are
c) Infected squamous cell carcinoma present in a typhoid carrier is:
d) None of the above a) Spleen b) Gall Bladder
(KAR-2003, AIIMS-2K) c) Liver d) Salivary gland
39. Causative agents of "Vincent's angina" (KAR-2001)
a) Borrelia vincenti and fusobacterium fusiformis 48. Actinomycosis is caused by:
b) Borrelia recurrents and bacteroides a) Fungus
c) Borrelia burgoloferi and lactobacillus b) Acid fast, non-motile bacillus
d) Leptospira and Treponema pallidum c) Anaerobic, gram positive, non-acid fast bacteria.
(KAR-98, 2001) d) Retrovirus
40. Resistance of Staphylococcus aureus is due to: (KAR-2002)
a) Pleomorphism 49. Gram -ve bacteria stain during gram staining:
b) Elaboration of an enzyme that destroy penicillin a) Blue color b) Red color
c) Penicillin analogue production c) Green d) Colorless
d) Lack of nucleic acid in the cell wall (AP-2000)
(AIIMS-2001) 50. The special stain used for treponema pallidum is
41. Group A ~-hemolytic streptococcal infection of the a) Giemsa staining b) Gram staining
pharynx may be followed in some by: c) Fontanna silver d) Ziel Neelson
a) Diphtheria b) Typhoid fever (AP-2000, 2003)
c) Rheumatic fever d) Syphilis 51. Silver impregnation method of staining is useful
(KAR-2002) to demonstrate
42. Common inhabitant of throat and tonsils is a) Bacteria l flagella b) Spirochetes
a) Alpha hemolytic streptococci (S. viridans)

34) A 35) B 36) A 37) C 38) B 39) A 40) B 41) C 42) A 43) A 44) A 45) A 46) D
47) B 48) C 49) B 50) C 51) C
Dental ;lut.,e

c) Both of the above d) None of the above 60. Encapsulation in bacteria helps in
(KAR-98) a) Spore formation b) Decreased virulence
52. Staphylococcal bacteria are c) Prevent their phagocytosis
a) Gram negative b) Tend to form chains d) Oxygen effect
c) Found in grape like clusters (AIPG-2012)
d) Associated with dental caries 61. Which of the following have the capability to
(PGl-2000) produce extra cellular polysaccharides
53. The temperature required for cultivating a) Strep. mutans b) Strep. pyogenes
mycobacterium c) Peptostreptococcus d) Enterococcus
a) - 2 degrees b) 0 degrees (PGI-1997)
c) 27 degrees d) 37 degrees 62. Neurotoxin of tetanus acts on
(AP-98) a) Synapse b) Muscle
54. Nagglers reaction is used in the diagnosis of c) Brain d) Spinal Cord
a) Malaria b) Whooping cough (PGI-98)
c) Gas gangrene d) Wool sorters disease 63. Tuberculin test screens for
(KAR-2001) a) Humoral immunity
55. Salmonella typhi can be isolated at different times b) Cell mediated immunity
from c) Complement function
a) Feaces b) Blood d) Phagocyte dysfunction
c) Urine d) All the above (COMEDK-04)
(KAR-97) 64. Organism most frequently associated with urinary
56. Loss of capsule in bacteria is generally associated tract infections is
with a) Neisseria gonorrhea b) Escherichia coli
a) Decrease in virulence c) T-strain mycoplasma
b) Loss of i nfectivi ty d) Streptococcus fecalis
c) Inability to spread through tissue (APPSC - 1999)
d) Increase in invasiveness 65. Granules in the Corynebacterium diphtheria are
(KAR-2001) called by following names, except
5 7. Which of the following characteristics of bacteria a) Metachromatic granules
is not true? b) Refractile granules
a) Unicellular b) Free living c) Babes Ernst granules
c) Having either DNA or RNA d) Volutin granules
d) Cell wall containing muramic acid (TNPSC-1999)
(TNPSC-99) 66. Growth factor needed for Salmonella
58. Which of the following structures, found external a) Tryptophan b) Niacin
to the bacterial cell wall, are involved in bacterial c) B-12 d) Citrate
attachment to cell surfaces? (KAR-1998)
a) Capsule b) Flagella 67. Widal test is specific for:
c) Pili d) Mesosomes a) Plague b) Leprosy
( COMEDK-04) c) Pemphigoid d) Typhoid
59. The single most important laboratory test for (KAR-1997)
determining the virulence of staphylococci is 68. Subacute bacterial endocarditis is caused by
a) Mannitol fermentation a) Streptococcus viridans
b) Hemolysis of sheep erythrocytes b) Haemolytic streptococci
c) Detection of coagulase c) Staphylococcus aureus
d) The catalase test d) Nisseria
( COMEDK-04) (AP-97)

52) C 53) D 54) C 55) D 56) A 57) C 58) C 59) C 60) C 61) A 62) D 63) B 64) B
65) B 66) A 67) D 68) A
, MICROBIOLOGY

69. Thumb print appearance in culture film smear is seen c) Scarlet fever d) Pneumonia
a) Bacillus anthracis b) Brucella species
c) Bordetella pertussis d) Clostridium welchii 78. The diagnostic investigation for syphilis is
a) TPI test b) VDRL test
70. The diagnosis of gonorrhea is established by- c) TPHA test d) FTAB test
a) Compliment fixation tests
b) Pili agglutination tests 79. True about VDRL test-
c) Haemagglutination tests a) Non - Specific b) Slide flocculation test
d) All of the above tests c) Best fo llowed for drug therapy
d) All
71. Culture medium for Corynebacterium diphtheria
a) Loefflers serum slope 80. False +ve for VDRL is seen in -
b) Mc Conkey a) Yaws b) Lepromatous leprosy
c) Saborauds agar c) Malaria d) All
d) Lowenstein Jensen Medium
81. Actin omycetes is-
72. Which of the following are non-spore-forming a) Gram positive bacteria
anaerobic bacteria that assume major significance b) Gram negative bacteria
in disease processes- c) Fung us
a) Bacteroides fragilis d) A yeast like form
b) Fusobacterium nucleatum
c) Peptostreptococcus anaerobius 82. Rickettsiae
d) Actinomyces Israeli e) All of the above a) Multiply within living Cell
b) Transmitted by arthropod vectors
73. Virulence of Gonococcus is due to c) Respond to tetracycline t herapy
a) Pili b) Cell Membrane d) All of the above
c) Its intracellular location
d) Cytolytic enzymes 83. Q fever is caused by -
a) Rickettsia tsutsugam ushi
74. What is the transport medium for cholera- b) R. prowozekii
a) Tellurite Medium b) Cary Blair Medium c) R. quintana d) C. burnetti
c) Venkatraman - Ramakrishnan Medium
d) B&C 84. Causative organism for gas gangrene
a) Clostridium t etani b) Clostridium welchii
75. Vibrio cholera was discovered by c) E. Coli d) Strep. faeca lis
a) Koch b) Meki ntoff
c) John snow d) Virchow 85. Drumstick appearance is seen in -
a) Cl. tetani b) Cl. tetanomorphum
76. Which organism causes prosthetic valve c) Cl. sphenoids d) All
endocarditis within 60 days of surgery-
a) St rept ococcus viridans 86. Opacity around colonies of Clostridium perfringens
b) Staphylococcus epidermis is due to -
c) Staphylococcus aureus a) Theta toxin b) Lecithinase
d) Fungus c) Desmolase d) Cytokinin

77. Staphylococcus does not causes- 87. Nagglers reaction detects-


a) Osteomyelitis b) Abscess a) Coagulase b) Hya luronidase

69) C 70) D 71) A 72) E 73) A 74) D 75) A 76) B 77) C 78) C&D 79) D 80) D 81) A
82) D 83) D 84) B 85) A 86) B 87) C
Dental ;lut.,e

c) Lecithinase d) None of the above a) Type of hemolysis b) Gram staining


c) Growth characteristics
88. Commonest mycobacterial infection in tropical d) Bile solubility
countries-
a) M. leprae b) M. avium intracellulare 97. Chlamydia causes all of the following except
c) M. tuberculosis d) Kansasi a) Trachoma b) Non-gonococcal urethritis
c) Pneumonia d) Parotitis
89. In T.B. cell mediated immunity is by -
a) CD 4+ b) CD 3+ 98. A veterinary doctor had pyrexia of unknown origin.
c) B - lymphocytes d) None of above His blood culture in special laboratory media was
positive for gram negative short bacilli which was
90. Rapid diagnosis of Tuberculosis is possible with oxidase positive. Which one of the following is the
a) Ziel-Nelson b) Kin Young stain likely organism grown in culture?
c) Auramine-Rhodamine stain a) Pasturella spp. b) Francisella spp.
d) Giemsa stain c) Bartonella spp. d) Brucella spp.
(AIPG-06)
91. Mycobacterium tuberculosis is differentiated from 99. All of the following organisms are known to survive
other atypical mycobacteria by intracellularly except:
a) Niacin test b) AFB staining a) Neisseria meningitides
c) PAS staining d) None b) Salmonella typhi
c) Streptococcus pyogenes
92. Leprosy bacilli can be grown in d) Legionella pneumophila
a) Foot paid on nine branded armadilo (AIPG-05)
b) Tail of guinea pig 100. All of the following lab tests can be used to
c) Testes of albino rats distinguish Streptococcus mutans from other oral
d) Testes of guinea pig streptococci, except
a) Gram staining
93. Leprosy bacilli are transmitted from person to b) Fermentation of mannitol and sorbital
person by all except c) Production of intracellular and extracellular
a) Discharge from nasal mucosa adherent polysaccharide
b) From open sore d) Colony morphology on saliva agar
c) Through intact skin (KAR-04)
d) Through breast milk 101. Bacteria acquire characteristics by all the following
except
94. Streptococcus is classified based on a) Through Plasmids
a) M Protein b) Cultural characteristics b) Incorporating part of host DNA
c) Bile solubility d) Cell wall carbohydrate c) Through bacteriophages
d) Through conjugation
95. Lancefields classification of B hemolytic (COMEDK-06)
streptococcus is based on 102. Borelia vincenti is a
a) M Protein a) Mycoplasma b) Mycobacteria
b) Polysaccharide C antigen c) Spirochaete d) Chlamydia
c) Type of lipid in cell membrane (KAR-04)
d) Physiological properties 103. Borrelia vincenti is always associated with :
a) Spirochaetes b) Fusiform bacilli
96. Pneumococcus can be differentiated from c) Treponema vincenti d) None of the above
streptococcus by

88) C 89) A 90) C 91) A 92) A 93) D 94) D 95) B 96) D 97) D 98) D 99) C 100) A
101) B 102) C 103) B
, MICROBIOLOGY

104. Pathogenic treponemas can be maintained in: testing. All the following antibiotics will be
a) Tissue culture b) Chick embryos appropriate EXCEPT:
c) Artificial media d) Rabbit testis only a) Vancomycin b) Imipenem
c) Teichoplanin d) Linezolid
105. Polypeptide capsule is seen in
a) Corynebacteriu m diphtheriae 114. cAMP reaction is shown by which streptococci?
b) Clostridium welchii a) Group A b) Group B
c) Staphylococcus aureus c) Group C d) Group D
d) Bacillus anthracis
(COMEDK-06) 115. Antigen used in Weil-Felix reaction :
106. Negative staining is used to demonstrate a) E.coli b) Haemophilus
a) Bacterial spore b) Bacterial flagella c) Proteus d) Staphylococcus
c) Bacterial capsule d) Bacterial fimbriae
(COMEDK-06) 116. The mechanism of genetic transfer where a phage
107. Teichoic acid is present in cell walls of serves as a vehicle is:
a) Gram positive bacteria a) Transduction b) Translation
b) Gram negative bacteria c) Lysogeny d) Conjugation
c) Yeast d) Protozoa
(AP-06) 117. The most primitive mode of gene transfer occurs
108. The most common pathogens responsible for by:
nosocomial pneumonias in the ICU are: a) Transduction b) Translation
a) Gram positive organisms c) Cell fusion d) Conjugation
b) Gram negative organisms
c) Mycoplasma d) Virus infections 118. Fimbriae are demonstrated by?
(AIPG-05) a) Culture b) Gram stain
109. Pasteur developed the vaccine for: c) Biochemical reaction
a) Anthrax b) Rabies d) Haemagglutination test
c) Cholera d) All
119. A 6 yr old boy comes with abdominal pain, vomiting
110. Test for differentiating virulent from non-virulent after consuming milk products within 6 hrs. The
tubercular bacilli: most likely organism responsible is:
a) Aryl sulphatase test b) Niacin test a) Staph. aureus b) Salmonella
c) Neutral red test d) Catalase peroxidase test c) B.cereus d) Clostridium botulinum

111. Culture medium used for Bordatella pertussis is: 120. Staphylococcal toxic shock syndrome (SSS
a) Wilson Blair medium syndrome) is due to
b) Bile broth a) Enterotoxin A b) Enterotoxin B
c) Bordet Gengou medium c) Enterotoxin D d) Enterotoxin F
d) Robertson cooked meat medium
121. Staphylococcal food poisoning includes the
112. Cough plate method is used to identify: following features:
a) Y. pestis b) Pertussis a) Incubation period of 24 hrs
c) Mycoplasma d) M-Tuberculosis b) Requires immediate antibiotics
c) Common with dairy products
113. A diabetic patient developed cellulitis due to d) Heat labile enterotoxin
Staphylococcus aureus, which was found to be
Methicillin resistant on the antibiotic sensitivity

104) D 105) D 106) C 107) A 108) B 109) D 110) C 111) C 112) B 113) B 114) B 115) C 116) C
117) D 118) D 119) A 120) D 121) C
Dental ;lut.,e

122. Phospholipid used to investigate syphilis by the c) Congo red d) Auramine and Rhodamine
reagin test is: (KCET-08)
a) Cardiolipin b) Plasminogen 131. An elderly male patient presented with fever,
c) Palmityl lecithin d) Serine chest pain, and dry coughp; sputum culture
showed growth on Charcoal Yeast Extract Medium,
123. Which of the following is obligate aerobe? the organism is
a) Pseudomonas aerogenosa a) H. influenza b) Moraxella catarrhalis
b) E. coli c) Legionella d) Burkholderia cepacia
c) Proteus d) C. diphtheria (AIIMS-09)
132. ASO (Antistreptolysin 0) test is used for the
124. In small intestine, cholera toxin acts by: diagnosis of
a) ADP - ribosylation of the G regulatory protein a) Rheumatoid arthritis
b) Inhibition of adenyl cyclase b) Typhoid fever
c) Activation of GTPase c) Rheumatie fever d) Rickettsial fever
d) Active absorption of NaCl (U PSC-09)
133. Bacterial species which is protective or beneficial
125. Erysipelas is caused by to the host is
a) Group B Staphylococci a) Streptococcus sanguis
b) Group A steptococci b) Porphyromonas gingivalis
c) Gonococci d) Pneumococci c) Treponema denticola
(COMEDK-07) d) Spirochetes
126. The most frequently encountered clostridial (COMEDK-10)
species in gas gangrene is 134. Following are the characteristics of Vincent's
a) Clostridium botulinum angina EXCEPT
b) Clostridium welchii a) Ulcerative gingivostomatitis
c) Clostridium tetani b) Caused due to malnutrition
d) Clostridium diffi cite c) A symbiotic infection
(COMEDK-07) d) Caused by Leptospira interrogans
127. The causative organism for cat-scratch disease is (KCET-10)
a) Chlamydia trachomatis 135. The spirochete which is associated with
b) Donovania granulomatis fusospirochetosis is-
c) Bartonella henselae a) Treponema pallidum b) Treponema pertenue
d) Hemophilia ducreya c) Borrelia burgdorferi d) Borrelia vincenti
(KCET-02) (KCET-09)
128. Group A carbohydrate of Str. pyogenes cross reacts 136. Pathogenesis of diphtheria is attributed to-
with human a) Invasion b) Endotoxin
a) Synovial fluid b) Myocardium c) Exotoxin d) Capsule
c) Cardiac valves d) Vascular intima (COMEDK-10)
(KCET-08) 137. Mycobacterium tuberculosis is best demonstrated
129. Which one of the following enteric organisms is by-
anaerogenic and non motile? a) Gram's stain b) H & Estain
a) Shigella sonnei b) Salmonella typhi c) Ziehl-Neelsen stain d) PAS stain
c) Proteus mirabilis d) Klebsiella pneumonia (KCET-09)
(COMED-2012) 138. Leptospirosis, all are true except
130. The commonly used fluorescent dye in the a) Faeces & urine of rodents is the cause
detection of tubercle bacilli b) onset of IgM in 2 days
a) Acridine orange b) Thioflavin c) Incubation period is 10 days

122) A 123) A 124) A 125) B 126) B 127) C 128) C 129) A 130) D 131) C 132) C 133) A 134) D
135) D 136) C 137) C 138) B
, MICROBIOLOGY

d) It is sensitive to penicillin b) Lactobacillus acidophilus


(AIPG-10) c) Streptococcus mutans
139. Dental Plaque adheres to tooth surface by- d) Veilonella parvula
a) Dextran b) Epithelial cells (KCET-2011)
c) Bacteria d) Sucrose 14 7. Pseudomonas aeruginosa
(AIPG-10) a) Produces heat stable enterotoxin
140. All are true about widal test except b) Causes Shanghai fever
a) Base lines differ depending on the endemicity of c) Cannot be destroyed at 55 degree at 1 hr.
the disease. d) Does not produce any pigment
b) 0 antibodies last longer and hence is not indicative (COMEDK-2011)
of recent infection 148. A full course of immunization against, Tetanus with
c) H antigen cannot differentiate between subtypes 3 doses to Tetanus toxoid, confers immunity for how
d) High titre value in a sing le widal test is not many years?
confirmative a) 5 b) 10
(AIIMS-09) c) 15 d) 20
141. About N. gon orrlloea, all are true except- (COMEDK-2011)
a) Most common cause of urethritis in male 149. Which of the following is true about tetanus
b) All stains are highly susceptible to penicillin acquired through traumatic wound?
c) Wide spread axoregnic stains cause disseminated a) Clostridium tetani travels via the nerves to the
gonococcal infection anterior horn dells in spinal cord.
d) Found exclusively in human beings. b) The tetanospasmin component of the exotoxin
(AIIMS-09) act on the post-synaptic neurones and prevents
142. Lepromin test is useful for impulse transmission.
a) Diagnosis b) Prag nos is c) The patient should be given tetanus toxoid
c) Treatment plan d) Epidemiology intravenously
(AIIMS-09) d) Causes positive Nagler reaction.
143. About Bacteroids. True are all except (AIPG-2011)
a) Susceptible to many antibiotics 150. All are true about bordetella pertussis except
b) Present main ly in mixed infections a) Most common cause is B. petrussis
c) Most common species is B fragilis b) It may occur in children and young adult after
d) Bile resistant anerobes vaccination
(AIIMS-09) c) Bacteria culture from nasopharyngeal swab is the
144. About psedomonas. All are true except- definitive diagnosis
a) Strict aerobes d) Presence of antibodies in serum is diagnostic of
b) Can grow in disinfectants in hospital the disease
c) Infection is mostly due to endogenous source (AIPG-2011)
d) Most common organism in burn patient 151. Actinomycosis is a
(AIIMS-09) a) Aerobic bacterial disease
145. Which is not true about vibrio cholera b) Anaerobic bacterial disease
a) It is non-halophilic c) Aerobic fungal disease
b) Grows on simple media d) Anaerobic fungal disease
c) Man is the only natural host (KCET- 2012)
d) Cannot survive in extracellular environment 152. Black pigmented anaerobes include all of the
(AIIMS-09) following except
146. Salivary peroxidise system is known to be effective a) Tan nerella b) Porphyromonas
against c) Buccae d) Prevotella
a) Actinomyces viscosus (KCET- 2012)

139) A 140) B 141) B 142) B 143) A 144) C 145) D 146) B 147) B 148) B 149) A 150) D 151) B
152) C
Dental ;lut.,e

153. Each of the following viruses possesses an outer 161. Cat scratch disease is caused by
Li po protein envelope except a) Moraxella b) Bartonella
a) Varicella Zoster Virus c) Pseudomonas d) Kochalimala
b) Papilloma Virus (GCET-14)
c) Influenza Virus 162. The most frequent non-sporing anaerobic bacilli
d) Human immunodeficiency virus isolated from clinical specimens is
(KCET- 2012) a) Bacteroides fragilis b) Clostridium tetani
154. Among the infectious diseases transmitted by c) Prevotella melaninogenica
food, one is d) Fusobacterium nucleatum
a) Viral stomatit is b) Diptheria (COMEDK-14)
c) Tuberculosis d) Typhoid fever 163. Microscopic examination of a pus sample shows a
(AP- 2012) filamentous organism which is acid fast with 1 %
155. All are true about legionella pneumophilia except sulphuric acid. Which of the following organisms
a) Aquatic bodies are main habitat could it be?
b) It can replicate in amoeba a) Actinomyces israeli b) Nocardia brasiliensis
c) Urine test is a reliable method c) Streptomyces somaliensis
d) Person to person transmission d) Blastomyces dermatitidis
(AIIMS MAY 2012) (COMEDK-14)
156. Which of the following fulfil all the criterias of 164. Actinomyces viscous that initially colonises the
Koch's postulates? POL environment most likely attaches to the
a) Treponema pallidum b) Mycobacterium leprae pellicle or saliva coated surface through?
c) N. gonorrhoeae d) None of the above a) Membrane protein b) Heat sensitive protein
(KAR -2013) c) Fimbriae d) Outer membrane protein
157. The urine sample of a patient has been sent to the (AIPG-14)
laboratory to look for Leptospira. The specimen is 165. Characteristic infection of nocardia asteroides is?
to be screened by use of the a) Colonic diverticula b) Brain abscess
a) Scanning microscope c) Secondary dissemination to liver
b) Inverted microscope d) Diarrhoea
c) Dark ground microscope (AIPG-14)
d) Electron microscope 166. DPT Vaccine is not contraindicated in?
(COMEDK -2013) a) Progressive neurological disorder
158. Endotoxin of gram -ve bacteria that do not cause b) Severely immunocompromised patient
pathogenesis of natural disease c) High allergic response to 1st dose
a) Mycobacterium b) Klebsiella d) Severe convulsions in a patient
c) Vibrio d) E.coli (AIIMS NOV-14)
(AIIMS NOV-13) 167. Botulinum toxin causes?
159. Clostridium tetani are following type of bacteria a) Tachycardia
a) Spore bearing aerobes b) Bilateral cranial nerve involvement
b) Non sporing aerobes c) Fever d) Hypertension
c) Spore bearing aITTaerobes (PGI Dec-2011)
d) Non sporting anaerobes 168. Not true about vibrio cholera is
(MCET-14) a) Increase water and sodium loss by debilitating
160. Borrelia vincentii is a/an diarrhoea
a) Gram negative spirochete b) Toxic protein is cholera toxin
b) Gram positive c) Activate adenylatecyclase
c) Acid Fast bacilli d) None of the above d) Gram positive rod
(GCET-14) (AIIMS NOV-14)

153) B 154) D 155) D 156) D 157) C 158) C 159) C 160) C 161) B 162) A 163) B 164) C 165) B
166) B 167) B 168) D
, MICROBIOLOGY

2. BACTERIOLOGY - ANSWERS
1. ' D' [Ananthanarayan 8th ed 3 72 / 9th ed 416] • VDRL test (slide flocc ulation)
• Kahn test (Tube flocc ulation)
2. 'C' [Ananthanarayan 8th ed 249 / 9th ed 381]
Bacteria Shape & Arrangement Specific tests using pathogenic treponemes are:
• TPI test (Complement fixation test)
• Meningococci
• Gonococci In pairs • FTA-ABS test (Fluorescent treponemal antibody
• Pneumococci absorption test)

Clostridium tetani Drumstick appearance • TPHA (Treponema pallidum hemagglutination


assay)
Cuneiform or Chinese letter
Diphtheria bacilli
pattern Option 'C' Widal test (Tube agglutination) is diagnostic
Anthrax Bamboo stick appearance test for typhoid fever.
Grape bunch like irreg ular
Staphylococci 10. ' B' [Ananthanarayan 8th ed 377 / 9th ed 379)
appearance
Streptococci Chains 11. 'A' [Ananthanarayan 8th ed 15 / 9th ed 16)
Leprosy bacilli Cigar bundle appearance The gram reaction is mainly due to the difference in
the permeability of bacterial cell wall. The cell wall
Actinomyces Branched fi lamentous form
of gram-negative organisms permits the outflow of
iodine dye complex during decolorisation. Gram-
3. ' D' [Ananthanarayan 8th ed 315-16 / 9th ed 315] positive bacteria resist decolorisation and retain the
primary stain.
4. 'C' [Ananthanarayan 8th ed 295 / 9th ed 296]
True about staining procedures is? (AIPG-14)
5. ' B' [Ananthanarayan 8th ed 23 / 9th ed 22)
a) Groccot GMS stains green
b) Gram negative stains Red
6. ' D' [Ananthanarayan 8th ed 293-94 / 9th ed 295)
c) Gram +ve stains black
The most important complications of typhoid fever
d) Calcoflor stains white
are intestinal perforation, hemorrhage and circulatory
collapse. Osteomyelitis is a rare sequel.
12. 'D' [Ananthanarayan 8th ed 59 / 9th ed 18)
Both intrans and exons are seen in higher forms of life.
7. 'A' [Ananthanarayan 8th ed 216 / 9th ed 213]
13. 'A' [Ananthanarayan 8th ed 236 / 9th ed 241)
8. ' B' [Ananthanarayan 8th ed 21 / 9th ed 20)
Option 'C' Flagella are organs of locomotion. Identification tests for different bacteria
Test Bacteria
Option'/\ Capsule is sharply defined structure around
Naggler's reaction Clostridia
the cell surface. When it is loose and undemarcated,
it is called slime layer. Capsule serves as a source of Satellitism H. influenza
nutrition. It contains virulence factors and makes the Dick test, Scheultz-
bacten'a resistant to phagocytosis. Streptococci
charlton reaction

9. 'C' [Ananthanarayan 8th ed 376 / 9th ed 375] Elek's test, Schick test Dipt heria
Non-specific serological tests for diagnosis of Milk ring test Brucellosis
syphilis using cardiolipin antigen are
Koch's phenomena Tuberculosis
• Wasserman test (Complement fixation test),
Dental ;lut.,e

String test Vibrio cholera Whooping cough


Dermal necrosis
(Bordetella pertusis)
Montoux test Tuberculosis
Plague toxin Necrosis, hemorrhage
Option 'C' i.e., Casoni's test is diagnostic test
for hydatid cyst. It is a skin test based on type-I 25. 'K (Ananthanarayan 8th ed 65 / 9th ed 60)
hypersensitivity reaction.
26. 'B' (Ananthanarayan 8th ed 63-64 / 9th ed 59)
14. 'A' (Ananthanarayan 8th ed 251 / 9th ed 265)
"Clostridium difficile" is associated with antibiotic 27. 'C' (Ananthanarayan 8th ed 227 / 9th ed 231)
associated pseudomembranous colitis. Pili are special type of fimbria, and they act as
virulence factors by promoting attachment to host
15. 'B' (Ananthanarayan 8th ed 199 / 9th ed 202) cells and inhibiting phagocytosis. They are usually
present on gram-negative male bacteria and help
16. 'C' [Ananthanarayan 8th ed 17 / 9th ed 16) in attachment to female bacteria in conjugation.
The bacterial cell wall is rigid and accounts for the The on ly gram-positive organism having pili is
shape of bacteria. Corynebacterium diphtheria.

17. 'B' (Ananthanarayan 8th ed 78 / 9th ed 74) 28. 'C' [Ananthanarayan 8th ed 198 / 9th ed 204)
Endotoxins are neat stable, weakly antigenic Clumping factor, is the bound coagulase which is
lipopolysaccharides and form an integral part of cell responsible for coagulase test.
wa ll of gram negative bacteria.
29. 'B' (Ananthanarayan 8th ed 296 / 9th ed 298)
18. 'C' [Ananthanarayan 8th ed 45 / 9th ed 24) Dreyer's tube is used for H agglutination and Felix
tube is used for O agglutination. Equal volumes of
19. 'B' [Ananthanarayan 8th ed 19 / 9th ed 19) sera are mixed in both the tubes and incubated at
Fimbria functions as organs of adhesion. 37°C overnight.

20. 'B' [Ananthanarayan 8th ed 235 / 9th ed 238) H agglutination leads to formation of loose, cotton
wooly clumps, while O agglutination is seen as disc
21. 'D' [Ananthanarayan 8th ed 24 / 9th ed 22) like pattern at the bottom of the tube.

22. 'A' (Ananthanarayan 8th ed 375 / 9th ed 374) 30. 'B' (Ananthanarayan 8th ed 195 / 9th ed 199)

23. 'A' [Ananthanarayan 8th ed 376 / 9th ed 375) 31. 'D' [Ananthanarayan 8th ed 195 / 9th ed 199)

24. 'A' [Ananthanarayan 8th ed 257 / 9th ed 261) 32. 'B' (Ananthanarayan 8th ed 265 / 9th ed 268)
Exotoxins of bacteria Action • Grows only in presence of
Obligate oxygen.
Neurotoxin, muscle aero bes
Clostridium botulinum Eg.: Cholera vibrio, Pseudomonas
paralysis.
Neurotoxin, muscle • Can grow under both aerobic and
Clostridium tetani
contraction anaerobic conditions
Inhibition of protein Facultative • Most bacteria of medical
Diptheria toxin anaerobes importance are facultative
synthesis
anaerobes.
Enterotoxins of Loss of fluid from small
Eg.: Enterobacteriaceae fami ly.
Staphylococci intestine
Erythrogenic toxin of Obligate • Grows only in absence of oxygen.
Scarlet fever
Streoptococci anaerobes Eg.: Clostridium & Bacteroides.
, MICROBIOLOGY

• Can grow in presence of low Penicillin resistance is due to production of beta


Micro- lactamase or Pencillinase, which inactivates penicillin
oxygen tension.
aerophilic by splitting the beta lactam ring. So beta lactamase
Eg.: Actinomyces. inhibitors like clavulanic acid and sulbactum should
be given .
33. 'A' (Ananthanarayan 8th ed 296 / 9th ed 298)
Methicillin and cloxacillin are examples of
34. 'A' (Ananthanarayan 8th ed 88, 486 / 9th ed 488) pencillinase resistance pencillins and these are
Salk vaccine is killed (injectable) polio vaccine. Sabin effective in treatment of infections caused by
vaccine is live (Oral) Polio vaccine. pencillinase producing staphylococci.

Route of 41. 'C' [Ananthanarayan 8th ed 210 / 9th ed 214)


Disease Condition of viru s Acute rheumatic fever and acute glomerulo- nephritis
administration
are two important non -suppurative sequels of
a) Live attenuated Oral
Polio streptococcal pyogenes. The lesions are believed to
b) Killed Subcutaneous be the result of hypersensitivity to some streptococcal
MMR component.
(Measles,
Live attenuated Subcutaneous 42. 'A' (Ananthanarayan 8th ed 205, 216 / 9th ed 218)
Mumps,
Rubella)
43. 'A' [Ananthanarayan 8th ed 347 / 9th ed 13)
Hepatitis Sub unit Subcutaneous Acid fastness of tuberculus bacilli is due to presence
Intradermal of mycolic acid, in cell wall.
Small Pox Live multiple Bacteria Staining
puncture
Pnemococci Indian ink
Rabies Killed Subcutaneous
Methylene blue, Alberts,
C. diptheriae
Neisser's staining
3 5. ' B' (Ananthanarayan 8th ed 26 / 9th ed 2 5]
The optimum tern perature at which growth of most Spirochaetes Giemsa stains, Levaditis
bacteria occurs is 3 7°C. Nocordia, TB, Leprosy Acid fast (Zieh l Neelsen)

36. 'A' (Ananthanarayan 8th ed 270 / 9th ed 299] For rapid diagnosis Auramine and Rhodamine
ofT.B. (l<CET-08)
Salmonella, Shigella Non lactose fermenters
Escherichia, Klebsiella Lactose fermenters 44. 'A' [Ananthanarayan 8th ed 245 / 9th ed 246)
Para colons Late lactose fermenters Anthrax, Brucella, Leptospira, Rickettisia, Borrelia,
and Coxiella burnetti are common examples of
37. 'C' [Ananthanarayan 8th ed 18 / 9th ed 16] zoonotic diseases.
When lysozyme acts on gram -positive bacteria, a
protoplast, which is devoid of cell wall is formed. 45. 'A' [Ananthanarayan 8th ed 234 / 9th ed 238)
Option 'C 'is for E. coli
With gram-negative bacteria, the result is a spheroplast,
46. 'D' (Ananthanarayan 8th ed 25 / 9th ed 24)
which retains some residual component of cell wall.

38. ' B' [Ananthanarayan 8th ed 245 / 9th ed 247) 47. ' B' [Ananthanarayan 8th ed 297 / 9th ed 296)

39. 'A' [Ananthanarayan 8th ed 380 / 9th ed 379) 48. 'C' (Ananthanarayan 8th ed 391 / 9th ed 391)

40. ' B' [Ananthanarayan 8th ed 197 / 9th ed 201) 49. ' B' [Ananthanarayan 8th ed 15 / 9th ed 16)
Dental ;lut.,e

Gram staining consists of four steps:


DraL1ghtsman (Concentric
• Primary staining with gentian violet, which Pnemococcus
ring) appearance
combines chemically with cytoplasm of the
bacterial cell and renders all organisms violet.
55. 'D' [Ananthanarayan 8th ed 295-96 / 9th ed 297]
• Application of dilute iodine which fixes the basic
dye to smear and thus act as mordant.
56. 'A' (Ananthanarayan 8th ed 19 / 9th ed 19]
• Decolorisation with alchohol. G+ve bacteria,
as they contain more acidic protoplasm, results
5 7. 'C' (Ananthanarayan 8th ed 19 / 9th ed 18]
decolourisation and retain the primary colour,
Bacteria contains double stranded circular DNA
violet while gram negative organisms get
decolorized.
58. 'C' (Ananthanarayan 8th ed 20-21 / 9th ed 20]
• Counterstaining with a dye of contrasting colour Pili are special type of fim bria. They are usually found
i.e. carbol-fuschin. As the gram-negative
on male bacteria and help in attachment to female
organisms are in decolorized state, they take up
bacteria during conjugation.
the counter stain and turn to red.
• The Gram reaction is related to the permeability of 59. 'C' (Ananthanarayan 8th ed 201 / 9th ed 202)
bacterial cell wall and cytoplasmic membrane to
the dye-iodine complex. 60. 'C' [Ananthnarayan 8th ed 376 / 9th ed 18]
• The G+ve bacteria becomes G-ve when the cell wall
is damaged. 61. 'K (Ananthanarayan 8th ed 216 / 9th ed 218)
Streptococcus mutans breaks down the dietary sucrose
50. 'C' [Ananthanarayan 8th ed 372 / 9th ed 374] and produces acid and dextrans. The acid damages
Spirochetes can be stained by silver impregnation dentine and the dextrans help in formation of plaque,
methods. Fontana's method is useful for staining films which finally lead to carious destruction of tooth.
and levaditi's method for staining tissue sections.
62. 'D' (Ananthanarayan 8th ed 258 / 9th ed 260)
51. 'C' [Ananthanarayan 8th ed 14 / 9th ed 3 74]
63. 'B' (Ananthanarayan 8th ed 168 / 9th ed 166)
52. 'C' [Ananthanarayan 8th ed 195 / 9th ed 199]
64. 'B' [Ananthanarayan 8th ed 274 / 9th ed 670)
53. 'D' [Ananthanarayan 8th ed 348 / 9th ed 346]
65. 'B' (Ananthanarayan 8th ed 18 / 9th ed 237]
54. 'C' [Ananthanarayan 8th ed 252 / 9th ed 258]
Nagg ler's reaction is given by alpha toxin of 66. 'A' [Ananthanarayan 8th ed 289 / 9th ed 291)
"Clostridium perfringens".
BACTERIAL GROWTH FACTORS
Tryptophan Salmonella typhi
Bacteria Cultural Character
Glutathione Gonococci
Staphylococcus Oil paint appearance
To differentiate human tubercle
Bordertella pertusis Thumb print appearance Niacin test
bacilli from atypical mycobacteria.
Corynebacterium Daisyhead/Frog's egg/
diptheria Poached egg colony 6 7. 'D' [Ananthanarayan 8th ed 296 / 9th ed 298)
Mycoplasma Fried egg appearance
68. 'A' [Ananthanarayan 8th ed 216 / 9th ed 218)
Clostridi um • SABE - Strep. sanguis (Strep. viridans group)
Naggler reaction
perfringens • Acute endocarditis- Staph. aureus or pyogenes
Satellitism Haemophilus • Post operative endocarditis - Staph. albus or
epidermis
Niesseria Lenticular shaped
, MICROBIOLOGY

69. 'C' [Ananthanarayan 8th ed 335 / 9th ed 333] 86. 'B' [Ananthanarayan 8th ed 252 / 9th ed 257]

70. 'D' [Ananthanarayan 8th ed 229 J 9th ed 233] 87. 'C' [Ananthanarayan 8th ed 252 J 9th ed 254]

71. 'A' [Ananthanarayan 8th ed 233 / 9th ed 237] 88. 'C' [Ananthanarayan 8th ed 575 / 9th ed 345]
Option 'B' Mycobacterium avium intracellulare is now
72. 'E' [Ananthanarayan 8th ed 265-66 / 9th ed 268] considered as the most common infection in AIDS
patients.
73. 'A' [Ananthanarayan 8th ed 227 / 9th ed 231)
89. 'A' [Ananthanarayan 8th ed 351 / 9th ed 350)
74. 'D' [Ananthanarayan 8th ed 303 J 9th ed 309]
90. 'C' [Ananthanarayan 8th ed 353 / 9th ed 353)
75. 'A' [Ananthanarayan 8th ed 302 / 9th ed 303)
91. 'A' [Ananthanarayan 8th ed 349 / 9th ed 347)
76. 'B' [Ananthanarayan 8th ed 202 / 9th ed 206)
92. 'A' [Ananthanarayan 8th ed 365 / 9th ed 368)
77. 'C' [Ananthanarayan 8th ed 199 / 9th ed 203]
Scarlet fever is due to pyrogenic exotoxin (Dick or 93. 'D' [Ananthanarayan 8th ed 366 / 9th ed 366)
scarlatina[ toxin) of streptococci.
94. 'D' [Ananthanarayan 8th ed 205 J 9th ed 209)
78. 'C & D' [Ananthanarayan 8th ed 376 J 9th ed 375)
Though TPI was the most specific test for diagnosis 95. 'B' [Ananthanarayan 8th ed 205 / 9th ed 209)
of syphilis, because of extreme complexity, it has
been replaced by FTA-ABS and TPHA tests, which are 96. 'D' [Ananthanarayan 8th ed 207 / 9th ed 213)
simpler and specific for diagnosis.
97. 'D' [Ananthanarayan 8th ed 418-419 / 9th ed 416)
79. 'D' [Ananthanarayan 8th ed 375 / 9th ed 374)
98. 'D' [Ananthnarayan 8th ed 341 / 9th ed 340)
80. 'D' [Ananthanarayan 8th ed 376-77 / 9th ed 374)
99. 'C' [Ananthnarayan 9th ed 209/
81. 'A' [Ananthanarayan 8th ed 391 / 9th ed 391) Sthed 224 for option A
287 for option B
82. 'D' [Ananthanarayan 8th ed 405-06 / 9th ed 405) 205 for option C
653 for option DJ
83. 'D' [Ananthanarayan 8th ed 410 / 9th ed 411]
100. 'A' [Ananthnarayan 9th ed 210)
84. 'B' [Ananthanarayan 8th ed 254 / 9th ed 257) Refer synopsis
Cl. tetani Tetanus or lock jaw
Cl. perfringens Gas gangrene 101. 'B' (Ananthnarayan 8th ed 63 / 9th ed 59]

Cl. difficile Acute pseudomembranous colitis


102. 'C' [Ananthnarayan 8th ed 380 / 9th ed 371)
Cl. botulinum Botulism or food poisoning

103. 'B' (Ananthnarayan 8th ed 380 / 9th ed 379)


85. 'A' (Ananthanarayan 8th ed 256 / 9th ed 259)
Shape of the spores Organism 104. 'D' [Ananthnarayan 8th ed 372 / 9th ed 372)
Spindle shaped Cl. bifermentans
Club shaped Cl. Perfringens 105. 'D' [Ananthnarayan 8th ed 242 / 9th ed 256)
Tennis racket Cl. tertium
Drum stick Cl. tetani 106. 'C' [Ananthnarayan 8th ed 14 / 9th ed 18)
Dental ;lut.,e

107. 'A' [Ananthnarayan 8th ed 15 / 9th ed 16] The enterotoxin responsible for food poisoning is heat
stable. Meat and fish or milk products cooked and
108. 'B' [Ananthnarayan 8th ed 626 / 9th ed 645] left at room temperature after contamination with
Aspiration in unconscious patients and pulmonary staphycocci are common items responsible.
ventilation or instrumentation may lead to nosocomial
pneumonia. Gram -ve bacilli and multidrug resistant 122. 'A' [Ananthnarayan 8th ed 375 / 9th ed 374]
Staph. aureus are the common pathogens.
123. 'A' [Ananthnarayan 8th ed 315 / 9th ed 314]
109. 'D' [Ananthnarayan 8th ed 4 / 9th ed 3]
124. 'A' [Check Explanation Below/ 9th ed 308]
110. 'C' [Ananthnarayan 8th ed 349 / 9th ed 348] Once established in the human small bowel, the
organism produces cholera toxin, which consists of
111. 'C' [Ananthnarayan 8th ed 335 / 9th ed 333] a monomeric enzymatic moiety (the A subunit) and
a pentameric binding moiety (the B subunit). The
112. 'B' [Ananthnarayan 8th ed 338 / 9th ed 336] B pentamer binds to GMl ganglioside, a glycolipid
on the surface of epithelial cells that serves as the
113. 'B' [Ananthnarayan 8th ed 201 / 9th ed 205) toxin receptor and makes possible the delivery of
Benzyl penicillin is the most effective antibiotic, if the the A subunit to its cytosolic target. The activated
strain is sensitive. Methicillin was the first compound A subunit (Al) irreversibly transfers ADP-ribose from
developed to combat resistance due to penicillinase nicotinamide adenine dinucleotide to its specific
production by straphylococci. For life threatening target protein, the GTP-bindirng regulatory component
methicillin resistant staphycococcal aurues (MRSA) of adenylate cyclase.
infections, vancomycin is the drug of choice. Linezolid
and teicoplanin are the other drugs. The ADP-ribosylated G protein upregulates the activity
of adenylate cyclase; the result is the intracellular
114. 'B' [Ananthnarayan 8th ed 206 / 9th ed 217) accumulation of high levels of cyclic AMP. In intestinal
epithelial cells, cyclic AMP inhibits the absorptive
115. 'C' [Ananthnarayan 8th ed 280 / 9th ed 89) sodium transport system in villus cells and activates
the secretory chloride transport system in crypt
116. 'C' [Ananthnarayan 8th ed 63 / 9th ed 59) cells, and these events lead to the accumulation of
sodium chloride in the intestinal lumen. Since water
117. 'D' [Ananthnarayan 8th ed 64-65 / 9th ed 60) moves passively to maintain osmolality, isotonic fluid
accumulates in the lumen. When the volume of that
118. 'D' [Ananthnarayan 8th ed 20-21 / 9th ed 20) fluid exceeds the capacity of the rest of the gut to
Fimbriae are antigenic. Hemagglutination provides resorb it, watery diarrhea results.
a simple method for detecting the presence of such
fimbria. 125. 'B' [Ananthnarayan 8th ed 209 / 9th ed 214)
The two typical streptococcal infections of the skin
119. 'A' (Ananthnarayan 8th ed 198 / 9th ed 202) are erysipelas and impetigo. These diseases are caused
Enterotoxin of Staph. aureus is responsible for the by Strep. pyogens which is group A streptococci.
manifestations of staphylococcal food poisoning
like nausea, vomiting and diarrhea 2 - 6 hrs after 126. 'B' [Ananthnarayan 8th ed 254 / 9th ed 257)
consuming contaminated food. Tetanus • Cl. tetani

120. 'D' [Ananthnarayan 8th ed 198 / 9th ed 202] • Cl. perfringens


Food poisoning
Enterotoxin type F or pyrogenic exotoxin C is most • Cl. botulinum
often responsible for toxic shock syndrome. Acute colitis • Cl. difficile

121. 'C' [Ananthnarayan 8th ed 198 / 9th ed 202]


, MICROBIOLOGY

• Neurological symptoms
• Cl. perfringens
- Confusion
• Cl. septicum
- Headache
• Cl. novyi
Gas gangrene - Lethargy
• Cl. histolyticum
group - Encephalopathy
• Cl. fallax
• Shortness of breath
• Cl. bifermentans
• Bradycardia
• Cl. sporogenes

132. 'C' [Ananthanarayan 8th ed 212 / 9th ed 214]


127. 'C' [Ananthnarayan 8th ed 412 / 9th ed 413)
In rheumatic fever and glomerulonephritis, a
retrospective diagnosis of streptococca l infection
128. 'C' [Ananthnarayan 8th ed 210 / 9th ed 214)
may be established by demonstrating high levels of
Acute rheumatic fever and acute glumerulonephritis
antibody to streptococca l toxins.The usual test done is
are two important non-suppurative sequelae of
'antistreptolysin O' titration. ASO titres higher t han
Streptococcus pyogenes infections. The essential
200 are indicative of prior streptococcal infection.
lesion in rheumatic fever is carditis, including
High levels are usually fo und in acute rheumatic fever
connective tissue degeneration of the heart valves
but in glomerulonephritis, titres are often low.
and inflammatory myocardial lesions characterized by
Aschoff nodules.
133. 'A' [Ananthanarayan 8th ed 216 / 9th ed 218]

129. 'A' [Ananthnarayan 8th ed 283 / 9th ed 287]


134. 'D' [Ananthanarayan 8th ed 380 / 9th ed 379]
Mobile, aerobic and Vincents angina or ulcerative oropharyngit is is caused
Salmonella typhi
facultative anaerobes by Borrelia Vincenti or Treponema vincenti and
Klebsiella pneumonia Nonmobile and aerogenic fusiform bacilli (Fusobacteri um fusiform).
Shigella sonnei Non motile and anaerogenic
Proteus mirabilis Mobile 135. 'D' [Ananthanarayan 8th ed 380 / 9th ed 379]

130. 'D' [Ananthnarayan 8th ed 348 / 9th ed 347) 136. 'C' [Ananthanarayan 8th ed 233 / 9th ed 237]
The pathogenic effects of diphtheria bacilli (Gram
131. 'C' [Ananthanarayan 8th ed 409 / 9th ed 401] Positive bacilli) are due to its very powerful
Legionnaire's disease is caused by Legionella. exotoxin. Toxin causes local necrotic changes and
Legionella have fastidious growth requirements and the disintegrating epithelial cells, leucocytes,
grow on complex media such as buffered charcoal erythrocytes and bacteria together constitute
yeast extract. the pseudomembrane, which is characteristic of
diphtheria.
Signs and symptoms of Legionnaire's disease:
• High grade fever 137. 'C' [Ananthanarayan 8th ed 352 / 9th ed 13]

• Mild dry cough


138. 'B' [Ananthanarayan 8th ed 383 / 9th ed 380)
• Chest pain Leptospirosis:
• Gastrointestinal symptoms • It is a zoonosis and humans are unnatural or 'end'
- Nausea hosts and do not transmit infection further. Humans
- Vomiting are infected when the leptospires in contaminated
- Diarrhoea water (by the urine of carrier animals like dogs,
cattle, rodents and pigs) enters the body through
- Abdominal pain
cuts or abrasions on the skin or through intact
mucosa of mouth, nose or conjunctiva.
• Incubation period is usually 10 days.
Dental ;lut.,e

• Antibodies appear in the blood of untreated • If untreated, infections at these sit es can lead to
patients after 5 - 7 days of illness. local complications such as
• Benzyl penicillin should be administered I.V. for - In females
upto 7 days in a daily dose of 6 - 8 mega units. Endometritis, salpingitis, tuboovarian abscess,
bart holinitis, peritonitis and perihepatitis.
139. 'A' [Ananthanarayan 8th ed 216 / 9th ed 216] - In males
Peri urethritis and epio!idymitis
140. 'B' [Ananthanarayan 8th ed 296 / 9th ed 298]
- In newborn
Widal test is a tube agglutination test. Salmonella
Ophthalmia neonatorum
typhi exhibits 3 well studied antigens.

• Hor Flagellar antigen is unique to S. typhi. • Disseminated gonococemia is uncommon and


• 0 or somatic antigen is polysaccharide in nature manifested as skin lesions, tenosynovitis arthritis
and is widely distributed in sa lmonella species. and endocarditis or meningitis.
• Acute urethritis is the most common clinical
• Vi or Virulence antigen is capsular in nature and is
manifestation in males.
found only in recently isolated stains.
• Gonococci have become resistant to antibiotics
2 types of tubes are generally used for the test like penicillin, ampicillin and tetracycline and no
longer reliable agents for it:he treatment.
• Narrow tube with a conical bottom (Dreyer's
agglutination tube) for H agglutination. • Ceftriaxone or ciprofloxacin plus Doxycycline for 7
days works well against gonococci.
• Short round bottomed tube (Felix tube) for
0 agglutination .
142. 'B' [Ananthanarayan 8th ed 368 / 9th ed 367]
Uses of Lepromin test:
Interpretation of Widal test
i) To classify the lesions of leprosy patient.
• The antibody level of individuals in a population
of a given area gives the baseline titre. ii) To assess the prognosis and response to
• Single test is usually of not much value. treatment. A positive reaction indicates good
prognosis and a negative reaction indicates bad
• Patients treated with antibiotics in the early stage prognosis.
may not give positive results.
• H antibody persists longer than O antibody. iii) To assess the resistance of individua ls to leprosy.
• Vaccines against Salmonella may give fa lse positive
Lepromin test is not used to diagnose leprosy nor does
reactions.
it indicate prior contact with the lepra bacillus.
Note:
0 antibodies rise first but declines quickly after the 143. 'A' [Ananthanarayan 8th ed 267 / 9th ed 269]
illness, perhaps because of their non protein nature. • Bacterioids are the most common anaerobes
Though less specifi c than H antibody they serve as a isolated from clinical specimens. They are non-
good index. sporing, non-motile, strict anaerobes, slender
rods, branching forms or coccobacilli, seen singly
141. 'B' [Ananthanarayan 8th ed 228 / 9th ed 233] or in pair or in short chains.
• Gonorrhoea is a venereal disease and commonly • B. fragilis is the most frequent of the non-sporing
manifests as cervicitis, urethritis, proctitis and anaerobes isolated from clinical specimens.
conjunctivitis. • Bacteroides are essentially resistant to penicillin
and first gene ration cephalosporins.
• They are susceptible to metronidazole, clindamycin
and chloramphenicol.
, MICROBIOLOGY

144. 'C' [Ananthanarayan 8th ed 316 / 9th ed 315] The bacillus is not particularly heat resistant, being
• Pseudomonas is a large group of aerobic, non- killed at 55°C in one hour. Ps. aeruginosa produces a
sporing, gram negative bacilli, motile by polar number of pigments, the best known being pyocyanin
flagella. and fluorescein. Pyocyanin is produced only by
Ps. aeruginosa but fluorescen may be produced by
• Pseudomonas aeruginosa are the most troublesome
many other bacteria.
agents causing nosocomial (Hospital) infections.
It is the most common and most serious cause of
Pathogenicity of pse udomo nas ae ruoginosa.
infection in burns.
• Nosocomial infections ( hospital acquire d
• Hospital equipment, articles such as bed pans,
infect ions)
Lotions, ointments and eye drops, plants, flowers,
stocks of distilled water may be frequently - Bedsores
contaminated. - Infections of wounds
• It is resistant to common antibiotics and antiseptics. - Eye infections
- Ear infections
145. 'D' [Ananthanarayan 8th ed 302 / 9th ed 307]
• Most common cause of infection in burns
• Vibrios are classified into halophilic and non- • Shanghai fever: it is self limited febrile illness
phalophilic vibrios based upon salt tolerance resembling typhoid fever.
• Non halophilic vibrios
- V. cholerae 148. 'B' [Ananthnarayan 6th ed 241 / 9th ed 262)
- V. mimicus Tetanus immunisation consists of 3 doses of tetanus
toxoid given intramuscularly with an interval of 4-6
• Halophilic vibrios
weeks between the first two injections and the third
- V. Vulnificus dose 6 months later. A fu ll course of immunisation
- V. Alginolyticus confers immunity for a period of at least 10 years. A
- V. Parahemolyticus booster dose of toxoid is recommended after 10 years.
• Extracellular environment such as saline waters of Too frequent injection of toxoid should be avoided as
coastal areas and surface waters from their natural hypersensitivity reactions may occur.
habitat.
149. 'A' [Ananthnarayan 8th ed 259 / 9th ed 260)
• Man is the only natural host for cholera infection.
Cl. tetani produces two distinct toxins - a
• They grow well on ordinary media. hemolysin (tetanolysin) and a powerful neurotoxin
(tetanospasmin). Tetanos!]asmin is the toxin
146. 'B' [Shafer 6th ed 428 J 9th ed 428] responsible for tetanus. The toxin produced from
The salivary glands secrete salivary peroxidise and the germinated spores is absorbed by the motor
thiocyanate (SCN-), which act on hydrogen peroxide nerve endings and transported to the central nervous
generated by certain bacetria. This oxido-reductase system intraxonally. Tetanospasmin resembles
system catalyzes the oxidation of the thiocyanate ion strychnine in its effects, except that tetanospasmin
to hypothiocyanate ion. The product reacts readily
acts presynaptically, unlike strychnine, which acts
with sulfhydryl compounds of low molecular weight postsynaptically. Nagler reaction is positive for
and thereby inactivates many bacterial enzymes of clostridium perfringes not for clostridium tetani.
glycolytic pathway. This antibacterial system is known
to be inhibitory towards Lactobacillus acidophilus 150. 'D' [Ananthnarayan 8th ed 335 J 9th ed 334)
and Streptococcus cremoris, by preventing cells from Bordetella pertussis causes pertussis or whooping
accumulating lysine and glutamic acid, which are cough. The duration of immunity after whole-cell
essential for their growth. pertussis vaccination is short-lived, with little
protection remaining after 10 to 12 years. "Culture of
147. 'B' [Ananthnarayan 8th ed 315-17 / 9th ed 315)
nasopharyngeal secretion remains the gold standard of
The growth of Ps. aeruginosa occurs at a wide range of diagnosis". Bordetella pertusis bacilli are demonstrated
temperatures, 6-42°C, the optimum being 37 degreesC.
Dental ;lut.,e

by microscopy or more reliably by culture in the early • They multiply and survive inside free living amoeba
stage of disease. Antibodies develop late and help and protozoa.
on ly in retrospective diagnosis on ly. • Man to man transmission does not occur.
• Causes legionnaires disease.
151. 'B' [Ananthnarayan 8th ed 391 / 9th ed 391]
Actinomyces is anaerobic, non-acid fast ray like • Human infection occurs by inhalation of aerosols
bacteria. produced by cooling towers, air conditioners and
shower heads. They survive for long and carried
over long distances.
152. 'C' [Cohen 8th ed 297 / 9th ed 269]
Recent taxanomic changes for previous bacteroides
species. Bacteroides now been placed in the geneses 156. 'D' [Ananthanarayan 8th ed 5 / 9th ed 4]
porphyromnonas (asaccharolytic) and prevotella Refer Q. No.46 of general microbiology for Koch's
(sacc harolytic). postulates. No bacteria can satisfy all the postulates
1) Porphyromonas - black pigmented in every case.
• P. asaccharolyticus
157. 'C' [Ananthanarayan 8th ed 372 / 9th ed 382)
• P. gingivalis
• P. endodontallis • Spirochetes like Treponema, Borrelia and Leptospira
can not be seen under the light microscope.
2) Prevotella - black pigmented • Their morphology and morphology and mobility
• P. melaninogenica can be seen under dark ground or phase contrast
• P. denticola mic rosco pe.
• P. loescheii • They cannot be stained with ordinary bacterial
• P. intermedia stains. They require negative staining.
• P. nigrescens • They can be stained by silver impregnation
• P. corpori s methods like Fontana's method for staining films
• P. tannerae and Levaditi's method for tissue sections.

3) Prevoltella - non pigmented 158. 'C' [Ananthanarayan 9th ed 308)


• P. buccae Vibrios produce enterotoxin called cholera toxin (CT)
• P. bivia which is implicated in pathogenesis of cholera. They
• P. oralis also produce endotoxin, like gram negative intestinal
• P. oris bacilli. This endotoxin apparently has no role in the
• P. ruminicola pathogenesis of cholera but is responsible fo r the
immunity induced by killed vaccines.
Tannerella forsythensis, is black-pigmented, gram-
negative anaerobic rods, have been strongly 159. 'C' [Ananthanarayan 9th ed 251)
implicated as major pathogen in the etiology of
periodontal diseases. 160. 'A' (Ananthanarayan 9th ed 379]
Borrela vincenti is a motile gram-ve spirochaete. It is
153. 'B' [Ananthnarayan 8th ed 549 / 9th ed 440) not acid-fast.
Papilloma is nonenveloped. VZV, Influenza and HIV
are enveloped. 161. 'B' (Ananthanarayan 9th ed 413)
Cat scratch disease is an infectious disease caused
154. 'D' [Ananthnarayan 8th ed 293 / 9th ed 295) by fastidious intracellular gram negative bacterium
called Bartonella henselae or Bartonella quintana. It
155. 'D' [Ananthanarayan 8th ed 400 / 9th ed 401) is common ly found in children following a scratch or
• The natural habitats for legionella pneumophilia bite from a cat within about 1-2 weeks. Presented as
are aquatic bodies including lakes and streams. tender, swollen Lymph nodes near the site of the bite
or scratch or on the neck usually limited to one side.
, MICROBIOLOGY

Fever, malaise, arthritis, join pain and decreased Brain abscess can be caused by a broad range of
appetite may occur. The disease resolves micro-organisms including, bacteria, fungi and
spontaneously. B. henselae can be demonstrated in parasites. However opportunistic pathogens such
lymph node biopsy smear and sections by warthin- as N.asteroides, Aspergillus and Toxoplasma gondii
starry staining. are identified in patients who have an underlying
immunodeficiency that results from corticosteroid
162. 'A' [Ananthanarayan 9th ed 269] treatment, cancer chemotherapy or HIV infection.
B. fragilis is a non-sporing, non-motile, strict
anaerobe, it is the most frequent of the non-sporing 166. 'B' [Park 18th ed 135]
anaerobes isolated from clinical specimens like blood, Contraindications for DPT vaccine:
pleural and peritoneal fluids, brain abscesses, wounds • Severe allergic reaction occurred after a previous
and urogenital infections. dose of DPT
• Neurological disorders
• Convulsions

Note: In case of allergy to DPT, subsequent


immunization should be done with DT vaccine only.
Acti nomyces anaerobe Yes No Pertusis component should be removed.
Partially
Nocardia Aero be Rare with 1% 16 7. 'B' [Ananthanarayan 9th ed 2 64]
H2504 Botulinum toxin acts by blocking the production
Streptomycis Aero be Yes No or release of acetylcholine at the synapses and
neuromuscular junctions. Onset is marked by
164. 'C' [Check Explanation Below] diplopia, dysphagia and dysarthria (speech difficulty)
Fimbriae have been detected to be major adherence due to cranial nerve involvement. The face appears
mechanism of the following oral species: expression less as a consequence of bilateral facial
• P. gingivalis nerve dysfunction. A symmetric descending paralysis
is the characteristic pattern, ending in death by
• A. viscosus
respiratory paralysis. Involvement of lower cranial
• A. actinomycetem comitans nerves causes dysphonia, dyarthria and dysphagia.
• S. salivarious
• S. parasanguinis 168. 'D' [Ananthanarayan 9th ed 303, 307]
Vibrios are gram-ve, curved bacilli that are actively
165. 'B' [Ananthanarayan 9th ed 393] motile. It causes cholera. It produces enterotoxin
Transmission of N. asteroides occurs by contaminated called Cholera Toxin (CT), Toxin Co-regulated
soil. Pilus (TCP, which is a colonization factor) and LPS
endotoxin. CT is the more potent toxin. Cholera toxin
Clinical forms: activates the adenylate cyclase enzyme in cells of
1. Cutaneous infection the intestinal mucosa leading to increased levels of
• Local abscesses intracellular cAMP, and the secretion of H20, Na+, K+,
• Cellulitis Cl·, and HC0· 3 into the lumen of the small intestine.
• Mycetoma like actinomyces The lost H20 and electrolytes in mucosal cells are
replaced from the blood. Thus, the toxin-damaged
2. Systemic infection: cells become pumps for water and electrolytes causing
• Mainly caused by N. asteroides the diarrhea, loss of electrolytes, and dehydration
• More often in immunodeficient persons that are characteristic of cholera.
• Lung abscess, pneumonia
• Brain abscess
Dental ;lut.,e

3. VIROLOGY
1. HIV is transmitted by all of the following routes d) Contain DNA but no RNA
except (KAR-2001)
a) Saliva b) Needle prick injury 9. Antibody to HSV will begin to appear
c) Blood transmission d) Sexual intercourse a) in a week and reach a peak in 3 weeks
(AP-14, MAN-2002) b) only after one year
2. A rise in the anti-HBc immunoglobulin in a patient c) no antibodies are present in primary HSV
indicates d) antibodies are present irn recurrent and chronic
a) Acute infection b) carrier state apthous stomatitis
c) prodroma l phase d) convalescence (KAR-99)
(MAN-2000) 10. Most common opportunistic infection in AIDS is:
3. All of the following infections may be transmitted a) Cryptococcosis b) Tuberculosis
by dental instruments except c) Candidiasis d) Aspergillosis
a) HIV b) Hepatitis C (KAR-2003)
c) Hepatitis B d) Hepatitis E 11. Australia antigen is associated with:
(MAN-2000) a) Hepatitis B surface antigen in acute hepatitis
4. HIV infection is known to be transmitted during b) AIDS
all of the following EXCEPT c) Chronic leukemia d) Basal cell carcinoma
a) breast feeding (AIPG-96)
b) child birth 12. Viruses can be cultured in all except:
c) heterosexual intercourse a) Chick embryo b) Blood agar
d) kissing c) Guinea pigs d) Cell culture
(MAN-95) (AIPG-93)
5. The culture media used for cultivation of HSV is 13. Confirmatory test for HIV infection is a:
a) Chocolate agar a) ELISA b) lmmunodot
b) Robertson's cooked-meat broth c) RIPA d) Western Blot
c) Chorio allantoic membrane (KAR-2002)
d) Sabourad's agar 14. Following hepatitis B infection through blood
(MAN-99) transfusion, disease manifests in:
6. Cytopathic effect is shown by: a) 1 week b) 6 weeks
a) Virus b) Bacteria c) 3 months d) 6 months
c) Spirochete d) Anaerobe (AIIMS-2001)
(AIIMS-96) 15. Bacteriophage was discovered by:
7. Half-life of free HIV in plasma is: a) Robert Koch b) Twort and d'herelle
a) 24 hours b) 6 hours c) Menkin d) Metchinkoff
c) 12 hours d) 3 months (KAR-98)
(KAR-2002) 16. The "window period" in HIV infection means:
8. Which of the following would you consider to be a) The time lapse between the infection and detection
general properties of viruses of viral antibodies
a) New virus particle arises directly (by division) b) The time lapse between the infection and
from preexisting viruses development of AIDS
b) Fall in to the general size range of 200-3000 c) The time lapse between obtaining the sample and
angstrom unit detection of virus in the lab
c) Contain equal proportions of protein, lipo d) None of the above
polysaccharide and nucleic acids (PGl-2001)

1) A 2) A 3) D 4) D 5) C 6) A 7) B 8) B 9) A 10) C 11) A 12) B 13) D


14) A 15) B 16) A
, MICROBIOLOGY

17. ELISA is: 25. Rabies virus


a) A name of a scientist a) Cytotropic b) Dermatotropic
b) A radiologic procedure c) Chromophilic d) Neurotropic
c) A pioneer in dentistry (AP-2002)
d) An enzymatic i mm une reaction 26. In which of the following immunization is given
(KAR-98) after infection is started
18. Human immunodeficiency virus (HIV) affects: a) Rabies b) Poliomyelitis
a) Red blood cells b) Fibroblasts c) Influenza d) Herpes
c) Helper T lymphocytes (CD 4 ) (AP-2004)
d) Mast cells 27. The time gap between appearance of koplik's spot
(KAR-1998) and cutaneous rash in measles is:
19. The virus HIV is produced and propagated in: a) 24 Hrs b) 3-4 days
a) Dendritic cells of the lymphnode germinal c) 2 weeks d) 10 days
epithelium (KAR- 2004)
b) Circulating T4 Lymphocytes 28. Incubation period of herpes zoster is
c) B- lymphocytes a) 7-14 days b) 1 month
d) Epithelial cells of the sex organs c) 1-2 years d) 3-6 months
(AIIMS-99) (AIPG- 2012)
20. The classic opportunistic infection in acquired 29. Baby born to mother suffering from AIDS, all are
immune deficiency syndrome is: true except
a) Apthous stomatitis a) Failure to thrive b) Have infections
b) Tuberculosis c) 50% chances to have AIDS
c) Pneumocystis carinii pneumonia d) All are t rue
d) Herpetic gingivostomatitis (KAR-99)
(KAR-2000) 30. True about prions:
21. The following hepatitis virus is transmitted by a) Composed largely proteins without any nucleic
faeco-oral route: acid
a) Hepatitis B Virus b) Hepatitis C Virus b) Phase in which virus cannot be demonstrated in
c) Hepatitis D Virus d) Hepatitis E Virus host cell
(KAR-98) c) Viruses which are genetically deficient
22. Influenza is caused by d) Viral components may be synthesized but
a) Orthomyxo virus, which is a DNA virus maturation & assembling is defective.
b) Paramyxo virus, which is a RNA Virus (TNPSC-99)
c) Paramyxo virus, which is a DNA Virus 31. The Dane particle is the
d) Orthomyxo virus, which is a RNA virus a) HBV b) IgG anti HAV
(AP-98) c) Delta Virus d) HBC Ag
23. Virus are sharply differentiated from bacteria by: (TNPSC-99)
a) Presence of either DNA or RNA 32. Pigs are important source of:
b) Simple structure a) Japanese encepha litis
c) Complex multiplication b) Kuru
d) All c) Yellow fever d) Rabies
(KAR-2000) (KAR-98)
24. Both intranuclear and intracytoplasmic inclusions 33. The best way of detecting the presence of viruses
are present in which of the following viruses in the infected cell- culture is
a) Pox Virus b) Measles virus a) Cytopathic changes in culture cells
c) Hepatitis B virus d) HIV b) Presence of viral protein in the infected monolayer
(KAR-2003) c) Haemagglutination test

17) D 18) C 19) B 20) C 21) D 22) D 23) D 24) B 25) D 26) A 27) B 28) A 29) C
30) A 31) A 32) A 33) A
Dental ;lut.,e

d) Haemagglutination inhibition test. a) Gastrointestinal tract


(TNPSC-99) b) Nasal mucosa
34. Herpes virus c) Lung d) Skin
a) Acquires its envelope from nuclear membrane
b) Acquires its envelope from nucleolar membrane 43. EBV causes all except -
c) Acquires its envelope from cytoplasmic membrane a) Carcinoma of nasopharynx
d) None of the above. b) Pa pilloma
(APPSC-99) c) Infectious mononucleosis
35. True about immune response of hepatitis Bis d) Burkitt's lymphoma
a) Antibody of HBs Ag is associated with resistance
to infection 44. Human immune deficiency virus is ____ virus
b) Antibody to HBC is not protective a) Rhea b) Retro
c) Highest titres of anti HBC are found in persistent c) Rhabdo d) Flavi
carriers of HBs Ag
d) CMI disappears soon after recover. 45. True about HIV are all except -
(TNPSC-99) a) DNA virus b) Attacks CD4 cells
36. The presence of which of the following factors in c) Macrophages are the reservoir
viruses makes protective vaccines a possibility? d) Decrease CD4 count in late stages
a) Enzymes b) Protein coat
c) Polysaccharide d) Lipids 46. Presence of HBe Ag in patients with hepatitis
(APPSC-99) indicates -
3 7. Enterically transmitted NAN B hepatitis B. a) Simple carriers b) Late Convalescence
a) Hepatitis C b) Hepatitis D c) High infectivity d) Carrier status
c) Hepatitis E d) Hepatitis F
(TNPSC-99) 47. All are oncogenic except -
38. Certain viruses have been isolated in crystalline a) EB virus b) Papilloma virus
form and have been found to be c) Herpes simplex virus
a) Nucleotides b) Phospholipids d) Varicella zoster virus
c) Scleroproteins d) Nucleoproteins
(APPSC-99) 48. Dengue fever vector is a -
39. Which of the following methods is adopted for a) Anopheles b) Aedes
screening HIV infection? c) Culex d) Mansoni
a) Virus isolation (AIPG-2001)
b) Western blot followed by ELISA 49. Which does not have a viral etiology -
c) ELISA followed by western blot technique a) Carcinoma Cervix b) Hepatoma
d) Polymers chain reaction c) Nasopharyngeal carcinoma
(TNPSC-99) d) Lymphoma
40. The HIV virus can be destroyed invitro by which of
the following 50. DNA covering material in a virus is called as-
a) Boiling b) Ethanol a) Capsomere b) Capsid
c) Cidex d) All the above c) Nucleocapsid d) Ernvelope

41. Australian antigen is: 51. Viron is defined as -


a) HB Ag b) HBs AG a) Extracellular infectious virus particle
c) HBV d) None of the above b) Smallest virus
(KAR-97) c) A Smallest particle similar to virus
42. Portal of entry of poliovirus in mainly- d) None of the above

34) A 35) A 36) B 37) C 38) D 39) C 40) D 41) B 42) A 43) B 44) B 45) A 46) C
47) D 48) B 49) B 50) B 51) A
, MICROBIOLOGY

52. " Plaque test" is used routinely in virology lab's for a) Give inmunoglobulins for passive immunity
a) Identification of viral disease b) Give ARV
b) To separate specific clone of virus c) Immediately stitch wound under antibiotic
c) To maintain certain viral culture coverage
d) To prepare vaccines d) Immediately wash wound with soap and water
(AIPG-05)
53. One virus particle prevents multiplication of 2nd 62. The overall effect of HIV is to gradually impair the
virus. This phenomena is - immune system by interference with
a) Viral interference b) Mutation a) Helper T lymphocytes
c) Supervision d) Permutation b) Natural killer cells
c) Plasma cells d) Macrophages
54. During the ' Window period' of patient with AIDS (KAR-04)
a) ELISA IS - ve b) Western Blot is - ve 63. Which of the following is a RNA virus?
c) Both are - ve d) PCR is - ve a) Measles virus b) Herpes virus
c) Papavo virus d) Adeno virus
55. The only virus, which has double stranded RNA, is - (AP-05)
a) Bunya virus b) Reo virus 64. Most common tumor associated with AIDS is
c) Calci virus d) Rhabdo virus a) Carcinoma b) Kaposi's sarcoma
c) Melanoma d) Ewing's sarcoma
56. Which of the following correctly indicates the (KAR-04)
infectivity of hepatitis virus in human? 65. Filters used to purify air in biological safety cabinets?
a) HBCAg b) HBeAg a) HEPA (High efficiency particulate air)
c) Anti-HBC d) Anti - HBS b) Sietz filters
( PGI-06, AI IMS NOV-14) c) Berkfield fi lters d) Millipore/Porcelain filters
5 7. Viruses can be isolated from clinical samples by (AIPG-14)
cultivation in the following except: 66. EBV is responsible for all except
a) Tissue culture b) Embryonated eggs a) Nasopharyngeal carcinoma
c) Animals d) Chemically defined media b) Burkitt's lymphoma
(AIPG-05) c) Hepatoma d) Infectious mononucleosis
58. Hepatitis B infection spreads through all of the (KAR-04)
following routes except 67. Type of human papilloma virus associated with
a) Blood transfusion b) Sexual contact carcinoma cervix:
c) Faeco oral route d) Perinatal transmission a) Type 6,12,18 b) 16,18,31
(KAR-04) c) 6,8, 11 d) 3,10,19
59. Which of the following represents the serologic
evidence of recent Hepatitis B virus infection 68. Patients with organ transplants are most frequently
during " window" period? infected with:
a) HBs Ag b) IgM anti - HBc a) Hepatitis A b) Hepatitis B
c) Anti HBs d) None of the above c) CMV d) EBV
(KAR-04)
60. Hepatitis C virus belongs to which one of the 69. Viral action is differentiated from bacterial action
following virus groups? by:
a) Picorna viruses b) Herpes viruses a) Interferon production
c) Hepadna viruses d) Flavi viruses b) Toxin production
(COMEDK-05) c) Lymphocytes production
61. For the treatment of case of class III dog bite, all d) Neutrophils production
of the following are correct except: (AIPG-07)

52) A 53) A 54) C 55) B 56) B 57) D 58) C 59) B 60) D 61) C 62) A 63) A 64) B
65) A 66) C 67) B 68) C 69) A
Dental ;lut.,e

70. The small nonparticulate protein leading to a) Vertical transmission


enhanced replication of HBV as well as HIV is b) IV drug abuse
a) HBc Ag b) HBs Ag c) Heterosexual promiscuity
c) Hbe Ag d) HBx Ag d) Homosexual promiscuity
(KCET-07)
71. What is the sequence, which a retro virus follows 79. Anti HBsAb indicates:
on entering a host cell? a) Resistance to hepatitis B
a) RNA- DNA- RNA b) RNA- DNA b) Acute infection
c) DNA- RNA d) DNA- RNA-DNA c) Good prognosis d) Hepatocellular carcinoma

72. In HIV, gp 120, envelope glyco proteins bind 80. Serological study for prevalence of Hepatitis B is
specifically to: best done by:
a) CD 8 T-cells b) CD 4 T-cells a) Surface antigen b) Surface antibody
c) B - cells d) NK cells c) Core antigen d) Core antibody

73. Which of the following viruses appears to be 81. Presence of HBe Ag in patients with hepatitis
involved in the pathogenesis of Kaposi's sarcoma indicates:
a) Human Herpes Virus 3 a) Simple carriers b) Late convalescence
b) Human Herpes Virus 1 c) High infectivity d) Carrier status
c) Human Herpes Virus 8
d) Human Herpes Virus 4 82. Which of the following is not a pox virus?
(KCET-07) a) Cowpox b) Molluscum contagiosum
74. The virus which causes Aplastic anemia in chronic c) Small pox d) Chicken pox
hemolytic disease is:
a) Adeno b) Hepatitis 83. Which of the following does not go into chronic
c) EB virus d) Parvo virus hepatitis stage?
a) HBV b) HCV
75. Seroconversion in HIV infection takes place in : c) HOV d) HEV
a) 2 weeks b) 4 weeks
c) 9 weeks d) 12 weeks 84. Hepatitis A virus is best diagnosed by:
a) IgM antibodies in serum
76. ELISA test when compared to western blot b) Isolation from stool
technique is: c) Culture from blood d) Isolation from bile
a) Less sensitive, less specific
b) More sensitive, more specific 85. Which of the following belongs to cell fraction
c) Less sensitive, more specific derived vaccine:
d) More sensitive, less specific a) Measles b) Mumps
c) Rubella d) Hepatitis B
77. Babu, a 28 yr old male comes with compaint
of exposure 3 wks back, having cervical 86. The only group A arbovirus causing epidemic
Lymphadenopathy, hepatosplenomegaly. Diagnosis disease in India:
of HIV is done by: a) Chandipura b) Chikungunya
a) ELISA b) Western blot c) Dengue d) Kyasanur forest disease
c) P24 antigen d) Lymph node biopsy
87. Dengue virus belongs to:
78. Commonest mode of AIDS transmission in India is a) Flavi virus b) Togaviridae
by: c) Reoviridae d) Rhabdoviridae

70) D 71) A 72) B 73) C 74) D 75) B 76) D 77) C 78) C 79) C 80) A 81) C 82) D
83) D 84) A 85) D 86) B 87) A
, MICROBIOLOGY

88. The word german measles is applied for 96. A dentist suffred from Hepatitis B infection 3
a) Rubella b) Rubeo la months back. His laboratory tests are normal
c) herpes simplex d) herpetic gingivostomatitis but he is not allowed by the medical board to do
(AP-08) surgical practice. He is
89. Which of the following statement about P24 a) Inactive carrier b) Healthy carrier
antigen of HIV is NOT true? c) Convalscent carrier d) Paradoxical carrier
a) It can be detected during the window period (AIIMS-09)
b) Free P24 antigen disappears after the appearance 97. Characteristic feature of retrovirus is
of lgM response to it a) Ribonuclease
c) Virus load parallels P24 titre b) Reverse tra nscri ptase
d) It remains during asymptomatic phase c) DNA polymerase
(KCET-08) d) Restriction endonuclease
90. Mark true in following ? (AIPG-10)
a) Hanta virus pulmonary syndrome is caused by 98. Salivary protein, which prevents transmission of
inhalation of rodent urine and faces human immunodeficiency virus via saliva, is
b) Kyanasur forest disease is caused by bite of wild a) Sialoperoxidase b) Secretory lg A
animal. c) Salivary leukocyte proteinase inhibitor
c) Lyssa virus is transmitted by ticks d) Histidine rich proteins
d) None (COMEDK-2011)
(AIPG-09) 99. Dengue Hemorrhagic fever is caused by
91. Which of the following viruses are most likely to a) Virus b) Bacteria
cross placenta and cause foetal defect? c) Parasite
a) Herpes Simplex b) Mumps d) Bacteria superadded on virus
c) Rubella d) Papilloma (AIPG-2011)
(IGNOU-10) 100. A young pregnant woman presents with fulminant
92. Kaposi's sarcoma (KS) usually occurs when (04 hepatic failure. The most likely aetiological agent
Lymphocyte counts are- is
a) Above 600 b) Between 1000-2000 a) Hepatitis B virus b) Hepatitis C virus
c) Below 200 d) Above 2000 c) Hepatitis E virus d) Hepatitis A virus
(COMEDK-09) (AIPG-2011)
93. Which of the following viruses produces both 101. Interferon interferes with replication of virus at
intranuclear and intracytoplasmic inclusion the time of
bodies- a) Protein synthesis b) DNA/RNA replication
a) Chicken pox b) Rabies c) When virus enters the cell
c) Small pox d) Measles d) Uncoating of the virus proteins capsule
(AIPG-09) (AIPG-2011)
94. Virus is cultured in 102. Prions true is:
a) Saubourd's agar b) Rogosa medi um a) Readily inactivated by autoclave at 121 C
c) Nonembryonated egg b) Evokes strong immunogenic reaction
d) Embryonated egg c) Have long incubation periods
(AIIMS NOV-13, AIPG-2012) d) Contains DNA/ RNA
95. Which of the following exposure carries the (AIIMS MAY 2012)
maximum risk of transmission of HIV 103. Which of the following microbial cell is most
a) Blood transfusion b) Sexual intercourse resistant to antiseptics & disinfectants?
c) Needle prick d) Transplacental a) Mycobacteria b) Spore
(AIPG-10) c) Prions d) Coccidia
(NEET-2013)

88) A 89) D 90) A 91) C 92) C 93) D 94) D 95) A 96) C 97) B 98) C 99) A 100) C
101) A 102) C 103) C
Dental ;lut.,e

104. Which of the following lesions is NOT an infection d) Taenia solium


from human papilloma virus? (MCET-14)
a) Verruca vulgaris 110. Rate of transmission of HIV from infected Mother
b) Focal epithelial hyperplasia to infant is
c) Condyloma acuminatum a) 100% b) 66%
d) Keratoacanthoma c) 30% d) 5-10%
(KAR-2013) (GCET-14)
105. Which of the following viruses does not infect 111. Which of the following is a DNA virus
salivary glands? a) Hepatitis A virus b) Hepatitis B virus
a) Echo virus b) Hepatitis C c) Hepatitis C virus d) Hepatitis D virus
c) HIV d) Orthomyxovirus (COMEDK-14)
(KAR-2013) 112. Rabies vaccine not to be given in case of?
106. 8 Year old boy priesented with swelling on both a) Rat bite b) Cat bite
sides of the face, below the ears of 4 days duration. c) Dog bite d) Monkey bite
It first started on the left side and then 3 days (AIIMS NOV-14)
later on the right side 113. A pathogen that causes a haemorrhagic fever that
(COMED-14) is NOT endemic in India is
106A. Possible diagnosis will be a) Plasmodium falciparum
a) Bacterial sialadenitis b) Ebola virus
b) Sialadenosis c) Leptospira d) Dngue
c) Epidemic parotitis d) Sialadenitis (MHCET-15)
(COMEDK-14) 114. Percutaneous transmission of HIV
106B. Cause for the disease will be a) 3% b) <1%
a) Herpes simplex virus c) 3-5% d) >5%
b) Cytomegalo virus (PGI JUNE-14)
c) Paramyxo virus d) Hepatitis c virus 115. Which of the following is not true about H.
(COMEDK-14) influenza?
106C. Immunization to this disease can be achieved by a) Rarely presents as meningitis in children less than
the following vaccination 2 months of age
a) MMR b) BCG b) Capsular polypeptide protein is responsible for
c) Hepatitis B d) Polio virulence
(COMEDK-14) c) Requires factor V and X for growth
107. Indicator of viral replication in Hepatitis B d) Most common invasive disease of H. influenza is
infection is meningitis
a) HbsAg b) HbeAg (AIIMS NOV-14)
c) IgM anti HBc d) Anti HBe 116. All dentists must take a prophylaxis for hepatitis B
(COMEDK-14) by vaccinating themselves at
108. Causative organism for Molluscum contagiosum a) 0, 1 and 6 months intervals
belongs to which virus family? b) 0 and 6 months intervals
a) Adenovirus b) Pox virus c) 0, 1 and 2 months intervals
c) Picorna virus d) Rhabdovirus d) 0, 1,2 and 12 months intervals
(AP-14) (APPG-15)
109. Common cause of diarrhoea in AIDS cases is due to
a) Plasmodium falciparum
b) Cryptosporidium
c) Ascaris lumbricoides

104) D 105) D 106A) C 106B) C 106C) A 107) B 108) B 109) B 110) C 111) B 112) A 113) B 114) B
115) B 116) D
, MICROBIOLOGY

3. VIROLOGY - ANSWERS
1. 'A' [Ananthanarayan 8th ed 581 / 9th ed 582]
HCV
HIV transmission occurs by sexual contact with Percutaneous Insidious Present
(Hepaci virus)
infected persons, through blood and blood products
and from infected mother to babies (during, before or HOV Percutaneous Insidious Nil
after birth). Though it is present in saliva, it is not HEV Fecal-oral Acute Nil
transmitted through saliva, because of presence of
anti-HIV salivary protein called secretory leucocyte
4. ' D' [Ananthanarayan 8th ed 581 / 9th ed 582]
protease inhibitor.
5. 'C' [Ananthanarayan 8th ed 432 / 9th ed 470]
2. 'A' [Ananthanarayan 8th ed 543 / 9th ed 548] Viruses are intracellular obligate parasites, they
• First marker to appear in cannot be grown on any inanimate culture medium.
blood after infection. Its They can be cultivated by inoculation into animals,
detection is essential for the embryonated eggs or tissue cultures.
diagnosis of HBV infection
HbsAg
• Vaccines contain genetically 6. 'A' [Ananthanarayan 8th ed 435 / 9th ed 436]
engineered HBS Ag particles. Many viruses cause morphological changes in cultured
Also known as Australian cells and these changes are known as cytopathic
antigen effects. They are useful in identification of viruses.
It is the core antigen. It is not Virus Cytopathic effect
HBcAg present in blood as it is enclosed Entero virus • Crenation of cells
within HBsAg coat.
• Degeneration of cell sheet
It is an indicator of active
Herpes virus Discrete focal degeneration
HBeAg intra hepatic viral replication
reflecting high infectivity. Adena virus Granular clumps resembling
grapes.
Protective antibody appears
Anti HBs after the disappearance of Measles virus Syncitium formation
HBs Ag.
Earliest antibody marker and 7. 'B'
Anti HBc-lgM denotes recent or acute HBV
infection 8. ' B' [Ananthanarayan 8th ed 426 / 9th ed 428]
Viruses do not have cellular organisation. They may
Denotes remote or chronic have either DNA or RNA but never both. They multiply
Anti HBc-lgG
infection by a complex process and not by binary fission and are
Anti HBe Denotes low infectivity unaffected by antibacterial antibiotics.

3. ' D' [Ananthanarayan 8th ed 546 / 9th ed 540] 9. 'A' [Ananthanarayan 8th ed 471 / 9th ed 470]

Mode of Carrier
Virus Onset 10. 'C' [Ananthanarayan 8th ed 575 / 9th ed 576]
infection state
The common opportunistic infections in AIDS are
HAV Fecal- oral oral candidiasis, herpes zoster, hairy cell leukoplakia,
(hepato virus) (ingestion) Acute Nil salmonellosis, tuberculosis and Pneumocystis carinii.
Percutaneous,
HBV The malignancies associated with AIDS are Kaposi's
sexual,
(Hepadna sarcoma and lymphomas.
vertical Insidious Common
virus)
(during birth)
Dental ;lut.,e

11. 'A' [Ananthanarayan 8th ed 539 / 9th ed 544] 23. 'D' [Ananthanarayan 8th ed 425 / 9th ed 427]
Australian antigen is the surface component of HBV
associated hepatitis. It is also known as hepatitis B 24. 'B' [Ananthanarayan 8th ed 444 / 9th ed 444]
surface antigen (H BsAg). Inclusion bodies are demonstrated in virus-infected
cells histologically. They may be int racytoplasmic
12. 'B' [Ananthanarayan 8th ed 432-33 / 9th ed 434] (Rhabdo virus, pox viruses), intranuclear (Herpes
viruses) or both (Measles virus). The intracytoplasmic
13. 'D' [Ananthanarayan 8th ed 578 / 9th ed 579] inclusion bodies in rabies (Rhabdo virus) are known
Serological tests for anti-HIV antibodies are of two as negri bodies.
types screening and confirmatory tests. The most
commonly used screening test is ELISA. Western blot 25. 'D' [Ananthanarayan 9th ed 532]
test is the confirmatory test.
26. 'A' [Ananthanarayan 8th ed 531 / 9th ed 537]
14. 'A'
27. 'B' [Ananthanarayan 8th ed 509 / 9th ed 512]
15. 'B' [Ananthanarayan 8th ed 455 / 9th ed 456] Koplick's spots are pathognomonic intraoral spots
Bacteriophages are viruses that infect bacteria. They that appear 2-3 days before the onset of cutaneous
play an important role in the transmission of genetic rash in measles (Rubeola virus) . They are bluish white
information between bacteria by the process of ulcerations on the buccal mucosa.
transduction.
28. 'A' [Jawetz Medical Microbiology 24th ed 403 / 9th
16. 'A' [Ananthanarayan 8th ed 578 / 9th ed 579] ed 472]
It takes 2-6 months for the appearance of HIV • The incubation period of varicella or chicken pox
antibodies after infection and the individual is highly is 2-3 weeks with an average of 14 or 15 days.
infectious during this period. This seronegative
• Primary infection caused by (VZV) varicella-zoster-
infective stage is known as "window period". During vrius is varicella also called as chicken pox. It is
this period both ELISA and WESTERN BLOT tests are common in early childhood. Herpes zoster is the
negative.
counterpart that occurs in adults.

17. 'D' [Ananthanarayan 8th ed 113 / 9th ed 114]


29. 'C' [Ananthanarayan 8th ed 581 / 9th ed 583]

18. 'C' [Ananthanarayan 8th ed 572 / 9th ed 574]


30. 'A' [Ananthanarayan 8th ed 441 / 9th ed 442]
The primary pathogenic mechanism in HIV infection
Prions are virus like agents. They are proteinaceous
is the damage caused to CD 4+ T lymphocytes. The T4
infectious particles without any detectable nucleic
cells decreases in nu mber (below 200 mm 3 ) and the
acid.
T4 ' T8 ratio is reversed. The function of monocyte,
macrophages NK cells and glial cells, which contain
Viriods are subviral agents cliaracterized by absence
CD4 antigen, is also affected.
of extracellular dormant phase (virion) and by a
genome much smaller than those of viruses.
19. 'B' [Ananthanarayan 8th ed 5 7 2 / 9th ed 5 7 4]

31. 'A' [Ananthanarayan 8th ed 539 / 9th ed 543]


20. 'C' [Ananthanarayan 9th ed 576]

32. 'A' [Ananthanarayan 8th ed 517 / 9th ed 521]


21. 'D' [Ananthanarayan 8th ed 54 7 / 9th ed 551]

33. 'A' [Ananthanarayan 8th ed 433 / 9th ed 436]


22. 'D' [Ananthanarayan 8th ed 494 / 9th ed 497]
Orthomyxoviridae includes influenza viruses and
34. 'A' [Ananthanarayan 8th ed 467 / 9th ed 466]
para myxoviridae includes mumps, measles and para
influenza viruses. Bot h are RNA viruses.
35. 'A' [Ananthanarayan 8th ed 543 / 9th ed 548]
, MICROBIOLOGY

36. 'B' [Ananthanarayan 8th ed 426 / 9th ed 429] 44. 'B' [Ananthanarayan 8th ed 570 / 9th ed 571]

3 7. 'C' [Ananthanarayan 8th ed 546 / 9th ed 540] 45. 'A' [Ananthanarayan 8th ed 570 / 9th ed 571]

38. 'D' [Ananthanarayan 8th ed 426 / 9th ed 428] 46. 'C' [Ananthanarayan 8th ed 543 / 9th ed 548]
Virus is essentially a nucleic acid surrounded by a
protein coat called capsid. The capsid protects the 47. 'D' [Ananthanarayan 8th ed 563 / 9th ed 565]
nucleic acid, introduces viral genome into host cells
by adsorbing readily to cell surfaces, and determines 48. 'B' (Ananthanarayan 8th ed 519 / 9th ed 523]
the antigenic specificity of the virus Dengue and yellow fever are transmitted by Aedes
aegypti mosquito.
39. 'C' [Ananthanarayan 8th ed 5 78 / 9th ed 5 79]
49. 'B' [Ananthanarayan 8th ed 562 / 9th ed 565]
40. 'D' [Ananthanarayan 8th ed 5 72 / 9th ed 5 73)
50. 'B' [Ananthanarayan 8th ed 426 / 9th ed 428]
41. 'B' [Ananthanarayan 8th ed 539 / 9th ed 544]
51. 'A' [Ananthanarayan 8th ed 425 / 9th ed 427)
42. 'A' (Ananthanarayan 8th ed 485 / 9th ed 490)
52. 'A' (Ananthanarayan 8th ed 436 / 9th ed 437)
43. 'B' [Ananthanarayan 8th ed 475 / 9th ed 475)
Epstein-Barr virus is a herpes virus. It's infection may 53. 'A' [Ananthanarayan 8th ed 438 / 9th ed 436]
lead to infectious mononucleosis, burkitts lymphoma
and nasopharyngeal carcinoma. 54. 'C' [Ananthanarayan 8th ed 578 / 9th ed 579)

55. 'B' [Ananthanarayan 8th ed 440 / 9th ed 440)


DNA Viruses RNA Viruses
1. POX VIRIDAE (largest) 1. PICORNAVIRIDAE (Smallest)
Eg.: Small pox, Molluscum contagiosum Eg.: Polio, Coxsackie, Rhinovirus (common cold)
Hepatitis A
2. HERPES VIRIDAE
Eg.: Herpes simplex, Varicella-Zoster, EB Virus 2. ORTHOMYXOVIRIDAE
Cytomegalo virus Eg.: Influenza

3. ADENOVIRIDAE (COMEDK-08) 3. PARAMYXOVIRIDAE


Adenoviruses are major cause of non-bacterial Eg.: Measles (Rubeola), Mumps
pharyngitis and tonsillitis presenting as febrile common
cold 4. REOVIRIDAE (double standard RNA)

4. PAPOVAVIRIDAE (Single stranded DNA) 5. RETRO VIRUS (HIV)


Eg.: Human papilloma virus
6. TOGA VIRIDAE (Rubella)
5. HEPADNAVIRIDAE (Hepatitis B virus)
7. RHABDOVIRIDAE (Rabies virus)
6. PARVOVIRIDAE (Parvovirus)
8. FLAVIVIRIDAE (Hepatit is C and G)
Which one is given below is a DNA virus (COMED-15)
a) Adenovirus b) Parvovirus 9. CALCIVIRIDAE (Hepatitis E)
Dental ;lut.,e

56. 'B' [Ananthnarayan 8th ed 540 / 9th ed 547) 65. 'A' [Environmental Microbiology by Ian L. Pepper,
Charles P. Gerba, Terry J. Gentry 3rd ed 106)
5 7. 'D' [Ananthnarayan 8th ed 426 / 9th ed 434 J Biosafety cabinets are designed to prevent biological
exposure to personal and the environment. They protect
58. 'C' [Ananthnarayan 8th ed 542 / 9th ed 546) experimental material from being contaminated when
HBV is a blood borne virus and the infection is appropriate practices and prrocedures are followed.
transmitted by parenteral, sexual and perinatal routes. They use high efficiency particulate air (HEPA) filters
The virus is maintained in the large pool of carriers in their exhaust and supply systems to protect against
whose blood contains circulating virus for long exposure to particulates including biological agents
period. Blood of carriers and less often of patients used in the cabinet.
is the most important source of infection. The carrier
state is more common among males. 66. 'C' [Ananthnarayan 8th ed 475 / 9th ed 565)

59. 'B' [Ananthnarayan 8th ed 540 / 9th ed 548) 6 7. 'B' [Ananthnarayan 8th ed 563 / 9th ed 553 J
Anti-HBC is t he earliest antibody marker to be seen
in blood. As anti-HBC remains lifelong, it serves as a 68. 'C' [Ananthnarayan 8th ed 474 / 9th ed 545)
useful indicator of prior infection with HBV, even after Cytomegalovirus infection can occur in transplant
all the other viral markers, becomes undetectable. recipients, cancer patients on che motherapy and HIV
Initially, anti-HBC is predominantly IgM (indicates infected patients
recent infection), but after 6 months, it is main ly IgG
(indicates remote infection). 69. 'A' [Ananthnarayan 8th ed 447 / 9th ed 448)
Interferons are host coded proteins produced by
60. 'D' [Ananthnarayan 8th ed 545 / 9th ed 440) cells on induction by viral or non-viral inducers.
Refer Q. No 55 Interferon by itself has no direct action on viruses
but it acts on other cells of same species rendering
61. 'C' [Ananthnarayan gth ed 532-533 / 9th ed 536) them refractory to viral infection. On exposure to
interferon, cells produce a protein, which selectively
62. 'A' [Ananthnarayan 8th ed 572 / 9th ed 574) inhibits translation of viral mRNA.
The receptor for the HIV virus is the CD4 antigen
and therefore the virus may infect any cell bearing 70. 'D' [Ananthnarayan 8th ed 540 / 9th ed 545)
CD4 antigen (CD4 helper T cells, B lymphocytes, The genome of hepatitis B virus has a compact
monocytes and macrophages, glial cells and microglia structure with four overlappirng genes i.e., S, C, P and
in CNS). (04+ helper/inducer T-lymphocytes are X. The S_ gene codes for the Surface antigen. The t
principal cells to possess CD4 antigen. The primary gene codes for Core antigen. The f_ gene is the largest
pathogenic mechan ism in HIV infection is the damage and coded for DNA Polymerase enzyme.
caused to the CD4+ T-lymphocytes. The T4 cells
are decrease in number and the T4:T8 cell ratio is The 'X' gene codes for a small non particulate protein
reversed. Infected T4 cells do not appear to release (HBxAg), which has transactivating effects on both
normal amounts of interleukin-2, gamma interferon viral and some cellular genes. This leads to enhanced
and other lymphokines. This has a marked damping replication of HBV, as well as of some other viruses,
effect on the cell mediated response. such as HIV virus. HBxAg and its antibody are
present in patients with severe chronic hepatitis and
63. 'A' [Ananthnarayan 8th ed 439 / 9th ed 440) hepatocellular carcinoma.
Refer Q. No. 55
71. 'I( [Ananthnarayan 8th ed 573 / 9th ed 574)
64. 'B' [Ananthnarayan 8th ed 575 / 9th ed 576)
72. 'B' [Ananthnarayan 8th ed 570 / 9th ed 574)

7 3. 'C' [Ananthnarayan 8th ed 4 77 / 9th ed 4 77)


, MICROBIOLOGY

74. 'D' [Ananthnarayan 8th ed 550 / 9th ed 554) 89. 'D' [Ananthnarayan 8th ed 576 / 9th ed 572)
Parvovirus B19 induces aplastic crisis in children with The genome of HIV contains three structural genes i.e,
chronic hemolytic anemias. Gag, Pol and Env. The gag gene determines the core
and shell of the virus. It is expressed as a precursor
75. 'B' [Ananthnarayan 8th ed 574 / 9th ed 575) protein P55 which can be cleaved into three proteins,
Within 3-6 weeks of infection with HIV, about 50% of P15, P18 and P24, which make up the viral core and
persons experience low grade fever, malaise, headache, shell. P24 antigen is the major core antigen and can
lymphadenopathy. Spontaneous resolution occurs be detected in serum during the early stages of HIV
within weeks. Tests for HIV antibodies are usually infection before the appearance of antibodies (IgM
negative at the onset of the illness but become positive antibodies appear in about 4-6 wks and are followed
during it course. Hence this syndrome has been called by IgG antibodies) . If the in fecting viral load is small
'Seroconversion illness'. HIV antigenemia (P24 antigen) (needle-stick injury), the process may considerably
can be demonstrated at the beginning of this phase. delayed.

76. 'D' [Ananthnarayan 8th ed 578 / 9th ed 579) 90. 'A' [Ananthanarayan 8th ed 522 / 9th ed 527)
• Hanta virus belongs to Arboviruses.
77. 'C' [Ananthnarayan 8th ed 576 / 9th ed 579)
• Arboviruses are transmitted biologically by
arthropod vectors but Hanta virus is not
78. 'C' [Ananthnarayan 8th ed 580 / 9th ed 582)
transmitted by arthropods but is maintained in
In USA, HIV was transmitted predominantly among
nature within rodent reservoirs that may transmit
male homosexuals. The commonest cause of viral
infection directly to humans.
hepatitis in India is enterically transmitted NANB.
• Hanta virus causes hemorrhagic fever with renal
syndrome.
79. 'C' [Ananthnarayan 8th ed 540 / 9th ed 548)
• Hanta viruses are natura l pathogens of rodents,
80. 'A' [Ananthnarayan 8th ed 540 / 9th ed 543) viremia is present in irnfected rodents and the
virus is shed in urine, feces, saliva in high titres.
81. 'C' [Ananthnarayan 8th ed 543 / 9th ed 548) Transmission from rodent to rodent and rodent to
human is primarily respiratory by inhalation of
82. 'D' [Ananthnarayan 8th ed 461 / 9th ed 471) virus contained in dried excreta.
• Kyanasur forest disease is transmitted by ticks.
83. 'D' [Ananthnarayan 8th ed 546 I 9th ed 550) • The genus lyssavirus includes rabies virus. All
warm blooded animals including humans can be
84. 'A' [Ananthnarayan 8th ed 538 / 9th ed 582) infected. They are never transmitted by the bite of
Diagnosis of Hepatitis A is usua lly by detection of insects.
IgM anti-HAV antibody that appears during the late
incubation period. 91. 'C' [Ananthanarayan 8th ed 550 / 9th ed 555)

85. 'D' [Ananthnarayan 8th ed 544 / 9th ed 548) 92. '(' [Ananthanarayan 8th ed 575 / 9th ed 575]

86. 'B' [Ananthnarayan 8th ed 515 / 9th ed 518] 93. 'D' [Ananthanarayan 8th ed 509 / 9th ed 444)
Chikungunya presents with sudden onset of fever,
crippling joint pains, lymphadenopathy and 94. ' D' [Ananthnarayan 8th ed 432 / 9th ed 434)
conjunctivitis. Viruses ca n be cultivated in t he following meth ods:
• In noculation into animals
The vector is Aedes aegypti. No vaccine is available.
• Embryonated eggs
87. 'A' [Ananthnarayan 8th ed 513 / 9th ed 440) • Tissue cultures: Organ culture, explant culture &
cell culture
88. 'A' [Ananthnarayan 8th ed 550 / 9th ed 555)
Dental ;lut.,e

. . . . 98. 'C' [Anantnarayana 6th ed 553 / 9th ed 583]


Salivary protein called secretary leucocyte protease
Approximate chance
inhibitor has anti-HIV activity.
Types of exposure of infection per
exposure
99. 'A' [Anantnarayana 6th ed 485 / 9th ed 523)
Sexual intercourse 0.1 - 1% Hemorrhagic fever Dengue virus
II Blood & blood products > 90% Chikungunya Alpha virus
Dengue Flavivirus
Tissue or organ
III 50 - 90% Yellow fever Flavivirus
donation

IV Injections & injuries 0.5 - 1% 100. 'C' [Anantnarayana 8th ed 518,519 / 9th ed 550]
Type Ehepatitis or non-A non-B hepatitis is responsible
V Mother to baby 30% for the majority of epidemic and sporadic hepatitis in
adults. A unique feature is the clinical severity and
96. 'C' [Ananthanarayan 8th ed 76 / 9th ed 545] high case fatality rate of 20-40% in pregnant women,
Carrier especially in the last trimester of pregnancy.
is a person who harbours the pathogenic microorganism
without suffering from any ill effect because of it. 101. 'A' [Ananthnarayan 8th ed 454 / 9th ed 448]
Interferon produces a protein TIP (translation in hi biting
Healthy carrier protein) which selectively inhibits translation of
Is the one who harbours the pathogen but has never viral mRNA into viral proteins. Therefore, during viral
suffered from the disease caused by the pathogen. transcription (protein synthesis) interferon interferes
with replication of virus.
Convalescent carrier
Is the one who has recovered fro m the disease and 102. 'A' [Ref. Ananthanarayan 8th ed 553 / 9th ed 442]
continues to harbour the pathogen in his body Prions are resistant to physical and chemical agents
such as heat, irradiation, formalin and UV rays.
Temporary carrier Theycan be deactivated by using autoclave.
This state lasts less than 6 months.
103. 'C' [Burton's Microbiology for the health Sciences
Chronic carrier 380 / 9th ed 442]
This state lasts for several years sometimes even for Prions possess the highest innate resistance to
the rest of one's life. chemical germicides fo llowed by spores, coccidia
(Cryptosporidiu m), Mycobacteria, small viruses (polio
Contact carrier virus and coxsackie virus), fung i, vegetative bacteria
One who acquires the pathogen from a patient. (staphylococcus, pseudomonas) and medium size
viruses (Herpes and HIV).
Paradoxical carrier
One who acquires the pathogen from another carrier. 104. 'D' [Shafer 6th ed 343 / 9th ed 553]
Option A, B and C are caused by HPV virus.
97. 'B' [Ananthanarayan 8th ed 570 / 9th ed 571] Keratoacanthoma is a rapidly growing well
Structure of retrovirus differentiated neoplasm of squamous epithelium. Risk
All retroviruses have an outer envelope consisting of factors include sun exposure and immunosuppression
lipid and viral proteins. The envelope encloses the core, (HIV).
made of viral proteins, within which lie two molecules
of viral RNA and enzyme reverse transcriptase (an RNA 105. ' D' [Burketts 11th ed 205 / 9th ed 500]
dependent DNA polymerase which can transcribe RNA The viruses responsible for the majority of virally
into DNA). induced salivary gland infections are:
• Paramyxo virus
, MICROBIOLOGY

• Cytomega lo virus (CMV) Both ebola and dengue causes haemorrhagic fever.
• HIV Dengue has been referred as endemic in India. Ebola
• Hepatitis C virus (HCV) is endemic in West Africa, not in India.
• Echo
114, 'B' [Check Explanation of Q. No. 9 5)
• EBV
Percutaneous means needle stick rnJunes. The
• Para influenza
average risk of HIV transmission following accidental
percutaneous injury (needle-stick) involving an HIV-
106A. 'C' [Ananthanarayan 9th ed 507) infected source patient is approximately 0.3%
Mumps is an acute infectious disease commonly
affecting children and characterised by non- 115. 'B' [Ananthanarayan 9th ed 329)
suppurative en largement of the parotid glands. The Factor 5 and 10 are required for the growth of H.
swelling is unilateral to start with but may become influenza. It causes most serious meningitis and is
bilateral. more common in chi ldren between two months and
three years of age. This age incidence has been
1068. 'C' [Ananthanarayan 9th ed 407) correlated with the absence of bactericidal anti PRP
Check explanation of Q. No 55 (Poly ribosyl orbital phosphate) antibodies. The
virulence is due to capsular polysaccharide which is
106C. 'A' [Check explanation of a.no 34 in chapter 2) endotoxin and not an exotoxin (protein).

107. 'B' [Check Explanation Q. No 2) 116. 'D' [Check Explanation Below]


There are data to support 2 schedules of dosing.
108. 'B' [Check Explanation Q. No 55) The 3 dose schedule (standard regimen) is 0, 1
and 6 months. For more rapid protection (High risk
109. 'B' [Travellers' Diarrhea by Ericsson, Dupont, individuals like dentists, nursing professionals etc) a
Steffen 2nd ed 115) 4 dose schedule (0, 1, 2 and 12 months) results in
Protozoa[ infections of the GIT are the most common the development of protective anti-HBs titres by 3
cause of diarrhoea in patients with AIDS and remains months. The fourth dose (at 12 months) is required to
one of the most challenging AIDS related conditions maintain prolonged protective anti-HBs titres.
to treat. Cryptosporidia and microsporidia being the
most commonly identified protozoa. For hemodialysis patients, a 4 dose schedule at 0, 1,
2 and 6 months is recommended.
Cryptosporidiosis occurs in 3-11 % of patients with
AIDS in all risk groups, but most frequent in male
homosexuals.

110. 'C' [Refer Q. No 95]

111. 'B' [Refer a. No 55)


Hepatitis A, C, E and G are RNA viruses.

112. 'A' [Text book of Paediatric infectious diseases by


Parthasarathy 1st ed 425)
Rabies vaccine should be considered after an exposure
to animal bite like dog, cat or money. Rabies vaccine
is usually not given after a bite by rat or rabbit or
squirrel.

113. 'B' [Text book of paediatrics by Parthasarathy 3rd


ed 1136)
Dental ;lut.,e

4. MYCOLOGY AND PARASITOLOGY


1. Fungal infection of human beings is called as c) Nails, hair and skin
a) Mucorsis b) Mycosis d) Superficia l skin and deep tissue
c) fungosis d) Micromia
(KAR-2000) 11. Candida is most often implicated in causation -
2. The mechanism by which most fungi cause disease a) Conjunctivitis b) Tenea capitis
is c) Desert rheumatism d) Thrush
a) exotoxin product ion b) lecithinase production
c) hypersensitivity d) coagulase production 12. Asexual spores of fu ngf are fo llowfng except -
(APPSC-99) a) Arthospores b) Chlamydospores
3. Germ tubes are formed mainly by c) Blastospores d) Ascospores
a) Candida albicans b) Candida stellatoidea
c) Candida tropicalis d) Candida pseudotropicalis 13. Candida albicans causes all of the following except-
(TNPSC-99) a) Endocarditis b) Mycetoma
4. Fungus capable of forming fungal ball is c) Meningitis d) Ora l thrush
a) Penicillium b) Aspergillus
c) Mucor d) Rhizopus 14. Transmission of all of the following intestinal
(TNPSC-99) parasites takes place by the feco-oral route except:
5. Aflatoxins are produced by a) Giardia lambia b) Ascaris lumbriocoides
a) Aspergillus niger b) Aspergillus fumigatus c) Strongyloides d) Entamoeba
c) Aspergillus flavus d) All of the above (AIPG-2001)
(KAR-03) 15. Hydatid cyst is:
6. The culture media for fungus is - a) Parasitic in nature b) Fungal
a) Tellurite medium b) NNN medium c) Congenital d) Viral
c) Chocolate agar medium (AIIMS, AIPG-96)
d) Sabourauds medium 16. Investigation of choice for invasive amoebiasis is:
a) Indirect hemagglutination
7. A sporangium contains - b) ELISA
a) Spherules b) Sporangiospores c) Counter immune electroporesis
c) Chla mydospores d) Oidia d) Microscopy
(AIPG-2002)
8. The medium of choice for culturing yeast form of 17. L. D. bodies are seen in:
dimorphic fungi is - a) Kalahazar b) Toxoplasmosis
a) Brain - heart infusion c) Malaria d) Sleeping sickness
b) Sabouraud's (KAR-2000)
c) Sabouraud's plus antibiot ics 18, Black water fever is caused by:
d) Any medium incubated at 35-37°( a) Plasmodium vivax
b) Plasmodium falciparum
9. Pseudohyphae are seen in - c) Leishmania donovani
a) Alternaria b) Asp erg illus d) Microfilaria
c) Oosporium d) Candida albicans (KAR-2001)
19. Megaloblastic anaemia is caused by:
10. Dematophytes are fungi infecting - a) Dog tapeworm b) Hookworm
a) Subcutaneous tissue c) Fish tapeworm d) Threadworm
b) Systemic organs (KAR-97)

1) B 2) C 3) A 4) B 5) C 6) D 7) B 8) C 9) D 10) C 11) D 12) D 13) B


14) C 15) A 16) B 17) A 18) B 19) C
, MICROBIOLOGY

20. Causative organism of Kalahazar is: 30. True about Toxaplasma gondi is, it is carried by -
a) Plasmodium ovale a) Cats b) Dogs
b) Leishman donovani c) Rats d) Cow
c) Entamoeba histolytica
d) Toxoplasma 31. Cysticercosis cellulosae is caused by -
(KAR-97) a) T. Solium b) Echinococcus granulosus
21. Parasitic inflammation would show predominantly c) T. Saginata d) H. nana
a) Lymphocytes b) Neutrophils
c) Eosinophils d) Basophils 32. Which worm is longest -
(TNPSC-99) a) T. Solium b) T. Saginata
22. All the following amoeba live in the large intestine c) Hook worm d) A. Lumbricoides
except -
a) E. coli b) E. nana 33. Kolmer test is a screening test done for
c) E. gingivalis d) E. histolytica a) Syphilis b) Tuberculosis
c) Gonorrhea d) Lymphoma
23. Amoebic liver abscess can be diagnosed by (KCET- 2012)
demonstrating - 34. Thread worm is:
a) Cysts in the sterile pus a) Enterobius b) Ancylostoma
b) Trophozoites in the pus c) Ascaris d) Necator
c) Cysts in the intestine
d) Trophozoites in the feces 35. Which is false about Wucheraria bancrofti
a) Causes filariasis b) Body is slender and long
24. All are true about Entamoeba histolytica except: c) Terminal nuclei absent
a) Cyst are 8 nucleated d) Man and anapheles mosquito are hosts
b) Cyst are 4 nucleated
c) Trophozoites colonise in the colon 36. Highest incidence of anemia in the tropics is due
d) The chromatid bodies are stained by iodides to:
a) Hook worm b) Thread worm
25. Commonest site of extra intestinal amoebiasis is: c) Ascaris d) Guinea worm
a) Brain b) Liver
c) Spleen d) Lungs 37. Leishmania is cultured in media -
a) Chocolate agar b) NNN
26. All are seen in a cyst of E. histolytica except - c) Tellurite d) Sabourauds
a) Glycogen mass b) Chromatid bars
c) Eccentric nucleus d) Refractile nucleus 38. Medusa lock appearance in X-ray is seen in:
a) Ascariasis b) Tapeworm
27. In transmission of malaria, mosquito bite transfers: c) Hook worm d) Ascariasis and tapeworm
a) Sporozoite b) Merozoite
c) Hypnozoite d) Ga metocyte 39. Rapid evaluation of fungal hyphae/spores can be
achieved with
28. Sporozoites of plasmodium falciparum are shaped: a) Grocott Gomori Methenamine silver
a) Dot b) Comma b) Hematoxylin and Eosin
c) Banana d) Sickle c) KOH wet mount
d) Peri-iodic Acid Schiff
29. Malaria pigment is: (KCET-2012)
a) Bilurubin b) Haemoglobin 40. Culture media of candida is:
c) Iron d) Haematin-globin pigment a) Methylene blue dextrose agar

20) B 21) C 22) C 23) B 24) A 25) B 26) D 27) A 28) D 29) D 30) A 31) A 32) B
33) A 34) A 35) D 36) A 37) B 38) A 39) C 40) B
Dental ;lut.,e

b) Saboraud's medium 49. Madura mycosis produces:


c) Pingolevin d) All of the above a) Brown to black granules
(AIIMS-06) b) White to yellow granules
41. Common name for Trichuris trichura c) Red granules
a) Round worm b) Whip worm d) No granules
c) Tape worm d) Seat worm
(AP-07) 50. Candidal hyphae can be stained using
42. The following is not true of Candida albicans: a) Van Gieson's stain b) Periodic acid Schiff stain
a) Yeast like fungu.s b) Forms chlamydospores c) Masson Trichrome stain
c) Blastomeres seen in isolates d) Toluidine blue stain
d) Seen in immunocompromised and causes (KCET-07)
meningitis in them 51. Delhi boil refer to:
a) Solar Keratosis b) Malignant pustule
43. The following statement is true regarding fungal c) L. Tropica sore d) Venereal ulcer
infection : (KCET-07)
a) Dermatophyte infections are exclusively man to 52. Following are the characteristics of Cryptococcus
animal neoformans EXCEPT
b) Rhinosporidium causes deep infection in man a) Noncapsulated fungus infecting man
c) C. albicans is not pathogenic to lab animals b) Causes meningitis
d) Candida infection is usually endogenous c) Melanin production related to virulence
d) Basidiomycetes yeast
44. Opportunistic infection is caused by: (KCET-10)
a) Penicillium b) Mucor 53. A 30 year old patient developed high fever of
c) Aspergillus d) All sudden onset. Peripheral blood smear showed
cresent shaped gametocytes. Malaria pigment
45. Methods for the diagnosis of superficial fungal was dark brown in colour. Which of the following
infections: malarial parasites is the causative agent?
a) Microscopic examination of skin scrapings a) Plasmodium vivax b) Plasmodium malarie
b) KOH staining c) Plasmodium falciparum
c) Wood light examination d) Plasmodium ovale
d) All (KCET-10)
54. Normal ratio of CD4 (T4) : CDS (TS) is-
46. The inflammatory type of tinea capitis with a) 2:1 b) 1:2
superadded secondary bacterial infection leading c) 3:1 d) 1:3
to a painful, circuimscribed, boggy, and indurated (MCET-10)
lesion, is called: 55. Man is the only reservoir fo r
a) Kerion b) Alopecia areata a) Salmonella b) Campylobacter jejuni
c) Pilomatrixoma d) Tenia incognito c) E. histolytica d) Y. enterocolitica
(AIPG-10)
4 7. Sclerotic bodies measuring 3-5m in size, multi- 56. Lesions caused by Entameoba histolytica
septate, chestnut, brown color is characteristic of: predominantly involve
a) Histoplasmosis b) Rhinosporodiosis a) Caecum and large intestine
c) Phaeohypomycosis d) Chromoblastomycosis b) Duodenum
c) Stomach d) Jejunum and Ileum
48. Reynolds Braude phenomenon is shown by: (COMEDK-10)
a) Candida albicans b) Mucor 5 7. Schuffner's dots are seen in infection due to
c) Cryptococcus d) Aspergillus a) Plasmodium falciparum

41) B 42) C 43) D 44) D 45) D 46) A 47) D 48) A 49) A 50) B 51) C 52) A 53) C
54) A 55) C 56) A 57) B
, MICROBIOLOGY

b) Plasmodium vivax d) Characterized by painful genital ulcers


c) Plasmodium malariae (AIPG-2011)
d) Plasmodium ovale 67. A mother donated a kidney to her daughter
(COMEDK-10) having chronic renal failure, which kind of graft it
58. Which of the following parasite causes represents:
autoinfection- a) Allo b) ISO
a) Giardia lambella b) lsospora belli c) Xeno d) Auto
c) Balantidium coli d) Taenia solium (AIPG-2011)
(COMEDK-09) 68. CD4 count in normal healthy adult is
59. Which stage of plasmodium vivax is infective to a) 500 b) 200
mosquito c) 1000 d) 300
a) Sporozoite b) Gametocyte (AIPG-2011)
c) Merozoite d) Zygote 69. Most fungi of medical importance belongs to
(AIIMS-09) a) Zygomycetes b) Ascomycetes
60. Probiotic organism used c) Basidiomycetes d) Deuteromycetes
a) E coli b) Bifido bacteria (COMED-2012)
c) Staphylococcus d) Salmonella 70. A Cerebrospinal fluid of a 2 years old child has
(AIPG-10) been sent to the laboratory to detect the presence
61. Which of the following is not transmitted by lice? of capsulated yeast. The staining technique most
a) Trench fever b) Relapsing fever commonly employed for the purpose is
c) Q fever d) Epidemic typhus a) India ink preparation
(AIPG-10) b) Methanamine silver stain
62. Sleeping sickness is caused by c) Ziehl-Neelsen stain
a) Housefly b) Sandfly d) Phyte-ferraco stain
c) Tick d) Reduvid bug (COMEDK -2013)
(AIPG-10) 71. Which of the following is true about amoebic liver
63. In falciparum malaria, causes of anemia are due to disease?
all except a) Left lobe of liver is commonly involved
a) Hemolysis b) Malabsorption b) Portal system is the main filtration system
c) Spleen sequestration c) Trophozoites can be visualized in the pus
d) Bone marrow depression d) Non-suppurative form is the serious form of extra
(AIPG-10) intestinal amoebiasis
64. Injection abscesses due to use of contaminated (NEET-2013)
vaccines occurs in infections caused by 72. True about endemic typhus
a) M. kansasii b) M. ulcerans a) Man is the only reservoir of infection
c) M. chelonae d) M. smegmatis b) Flea is a vector of disease
(COMEDK-2011) c) A rash developing into an eschar is characteristics
65. Which of the following is most potent antigen of the disease
for sHmulating both humoral and cell mediated d) Culture is diagnostic
immunity? (AIPG-14)
a) Adjuvant b) Proteins 73. The commonly used stain in the study of fungal
c) Polysaccharides d) Lipids infections are
(AIPG-2011, PGI DEC- 2011) a) Gomori methanamine silver
66. Donovanosis, true is b) PAS
a) Caused by Calymmatobacterium granulomatis c) H & E d) All the above
b) Lymphadenopathy is remarkable & diagnostic (GCET-14)
c) Penicillin is dr1Ug of choice

58) D 59) B 60) B 61) C 62) D 63) B 64) C 65) B 66) A 67) A 68) C 69) D 70) A
71) C 72) B 73) D
Dental ;lut.,e

74. The reservoir of protozone infection caused by


toxoplasma gondii in children is usually
a) cats b) Sheeps
c) Dogs d) Rats
(COMEDK14)
75. The following clinical presentation qualify for the
term "Severe Falciparum Malaria" EXCEPT
a) Severe anemia b) Acidosis
c) Pulmonary oedema
d) High grade feve r with rigors
(COMEDK14)
76. One of the following is NOT a protozoal infection
a) Malaria b) Amoebiasis
c) Trichomoniasis d) Trichinellosis
(AP-14)
7 7. A young male present with loose motions and
intermittent abdominal pain over the past 1 year.
Wet mount stool specimen showed the presence
of multiple ova which are more than 100u in
diameter. The causative organisms for the disease
shall not include?
a) Fasciola gigantic b) Gastrodiscoides hominis
c) Echinostoma ilocanum
d) Opisthorchis viverrini
(AIIMS NOV-14)

74) A 75) D 76) D 77) D


, MICROBIOLOGY

4. MYCOLOGY AND PARASITOLOGY - ANSWERS


1. 'B' [Ananthanarayan 8th ed 601 / 9th ed 595] 13. 'B' [Ananthanarayan 8th ed 607 / 9th ed 611]
Oral thrush, vaginitis, cutaneous candidiasis
2. 'C' [Ananthanarayan 8th ed 605 / 9th ed 592] are common lesions of candidiasis. Intestinal
candidiasis is a sequel to oral antibiotic therapy and
3. 'A' [Ananthanarayan 8th ed 607-08 / 9th ed 612] bronchopulmonary candidiasis is rare complication of
Candida albicans can be identifi ed from other candida pre-existing pulmonary disease.
species by growth characterist ics, sugar assimilation
and fermentation tests. It produces germ tubes within Septicemia, endocarditis and meningitis occur as
two hours when incubated in human serum at 37°C. terminal complications in immunosuppressive and
leukemic patients.
4. 'B' [Ananthanarayan 8th ed 613 / 9th ed 609]
Pulmonary aspergillosis, colonizing aspergillosis 14. 'C' [Jayaram Paniker 6th ed 171]
(Fungal ball) and disseminated aspergillosis are Transmission of strongyloides occurs by the
various systemic manifestations of Aspergillosis penetration of infective fi lariform larvae when a
person walks barefoot on contaminated soil or by
5. 'C' [Ananthanarayan 8th ed 614 / 9th ed 615] autoinfection.
Aflatoxicosis is produced by Aspergillus flavus while
ergotoxicosis is produced by the fungus Claviceps 15. 'A' [Jayaram Paniker 6th ed 152]
purpurea. Hydatid cyst contains hydatid fluid. This fluid is used
as antigen for casoni's intradermal test, which is a
6. 'D' [Ananthanarayan 8th ed 601 / 9th ed 592] dignostic test for Echinococcus. Casoni's test is an
Sabourauds glucose agar and cornmeal agar are the example of type I hypersensitivity reaction
commonest culture media used in mycology.
16. 'B' [Jayaram Paniker 6th ed 26-27]
7. 'B' [Ananthanarayan 8th ed 601 / 9th ed 590)
17. 'A' [Jayaram Paniker 6th ed 57]
8. 'C' [Ananthanarayan 8th ed 601 / 9th ed 607] Kala-azar or Lei sh maniasis or Dum Dum fever or tropical
spleenomegaly is caused by Leishmania donovani.
9. 'D' [Ananthanarayan 8th ed 607 / 9th ed 612]
Candida albicans is a pathogenic yeast like fungus, 18. 'B' [Jayaram Paniker 6th ed 86]
grow partly as yeast and partly as elongated cells
resembling hyphae which form pseudomycelium. 19. 'C' [Jayaram Paniker 6th ed 86, 142]

10. 'C' [Ananthanarayan 8th ed 604 / 9th ed 596] 20. 'B' [Jayaram Paniker 6th ed 56]
Dermatophytes are fungi t hat infect only superficial
keratinised tissues - the skin, hair and nails. They are 21. 'C' [Jayaram Paniker 6th ed 9]
classified into 3 genera - Trichophyton, microsporum
and epidermophyton. 22. 'C' [Jayaram Paniker 6th ed 30]
E. gingivalis has only trophozoite form. It lives in
11. 'D' [Ananthanarayan 8th ed 607 / 9th ed 611] the gingival tissues and is abundant in unhygienic
mouths.
12. 'D' [Ananthanarayan 8th ed 601 / 9th ed 590]
Sporangiospores, blastospores, arthrospores and 23. 'B' [Jayaram Paniker 6th ed 27]
ch lamydospores are asexual spores. Zygospores, Hepatic involvement is the most common extraintestinal
ascospores and basidiospores are sexual spores. complication of amoebiasis. It causes amoebic
hepatitis and liver abscess. In liver abscess, diagnostic
aspiration of pus demonstrates trophozoites.
Dental ;lut.,e

24. 'A' [Jayaram Paniker 6th ed 16-17] 34. 'A' [Jayaram Paniker 6th ed 183]
E. histolytica occurs in three forms - the trophozoite, Pin worm, thread worm, seat worm Enterobius
precystic and cystic stages. Trophozoite has single
nucleus and exhibits motility by pseudopodium. Hook worm Ancylostoma
Mature cysts are quadrinucleated wit h eccentrically Whip worm Trichuris trichura
placed nuclei and contains chro matid bodies and
Round worm Ascaris
glycogen masses. Tetranucleate stage is infective
stage of E. histolytica to man.
35. 'D' [Jayaram Paniker 6th ed 199)
Elephantiasis or filariasis is caused by Wucheraria
25. 'B' [Jayaram Paniker 6th ed 22, 69]
bancrofti. Man is the definitive host and female culex
mosquito is the intermediate host.
26. 'D' [Jayaram Paniker 6th ed 17]

36. 'A' [Jayaram Paniker 6th ed 180)


27. 'A' [Jayaram Paniker 6th ed 69]
An adult ancylostome can suck 0.2ml of blood a day
Human infection comes through t he bite of infective
and bleeding from the site continues due to anti
female anapheles mosquito. Sporozoites are infective
coagulant activity by its secretions. This causes iron
stages to man where as gametocytes are the infective
deficiency anemia.
stages to mosquito.

3 7. 'B' [Jayaram Paniker 6th ed 59]


So gametogony occurs in man and sporogony occurs
in the mosquito.
38. 'A'

28. 'D' [Jayaram Paniker 6th ed 76]


39. 'C' [Ananthnarayan 8th ed 605 / 9th ed 609]
Sporozoites are sickle-shaped and merozoites are
The routine method of diagnosis of fila mentous fung i
banana shaped
(dermatophytes) is by examination of KO H mounts.
Scrapings are taken from the edges of ringworm
29. 'D' [Jayaram Paniker 6th ed 71]
lesions and mixed with a drop of 10% KOH on a
slide and mounted. Microscope examination reveals
30. 'A' [Jayaram Paniker 6th ed 97]
branched septate hyphae.
T. gondii is recognized as the most common protozoan
parasite globally. Domestic cat is its definitive host
40. 'B' [Ananthnarayan 8th ed 607 / 9th ed 611]
and all other species are merely intermediate hosts.
41. 'B' [Jayaram Panikar 6th ed 165]
31. 'A' [Jayaram Paniker 6th ed 148]
Cysticercus bovis is caused by Taenia saginata
42. 'C' [Ananthnarayan 8th ed 607 / 9th ed 611]
32. 'B' [Jayaram Paniker 6th ed 144]
43. 'D' [Ananthnarayan 8th ed 607 / 9th ed 611]
Taenia solium Pork tapeworm Candidosis is an opportunistic endogenous infection.
Taenia saginata Beef tapeworm
Diphyllobothri um Latum Fish tapeworm 44. 'D' [Ananthnarayan 8th ed 613 / 9th ed 615]
Echinococcus granulosus Dog tapeworm
Hymenolepsis nana Dwarf tapeworm 45. 'D' [Ananthnarayan 8th ed 605 / 9th ed 606]
The routine methods of diagnosis of fungal
infections are:
33. 'A' [Syphilis Werewolf of Medicine Ed 7]
The Wassermann test is a antibody test for syphilis • KOH mounts
based on complement fixation. The Wassermann test • Wood's lamp
has been refined as Kahn test and Kolmer test. These • Culture
tests were rarely used today. These are replaced with
• Microscopic examination of skin scrapings
VDR Ltest.
, MICROBIOLOGY

Selection of infected hair for examination is facilitated or kala-azar or Oum Oum fever or Burdwan fever of
by exposure to UV light (Wood's lamp) . tropica l splenomegaly.
• Leish mania tropica causes cutaneous lei sh maniasis
Species identification is possible only by culture or oriental sore or Delhi boil or Aleppo boil or
examination. Specimens are inoculated on Sabouraud's Bagdad or biskra button.
medium and incubated at room temperature.
52. 'A' [Ananthanarayan 8th ed 610 / 9th ed 613]
46. 'A' [Ananthnarayan 8th ed 605 / 9th ed 611] Cryptococcus neoformans:
• It is a capsulated fungus. Infection is acquired by
47. 'D' [Ananthnarayan 8th ed 609/ 9th ed 609]
inhalation. Most infections are asymptomatic.

48. 'A' [Ananthnarayan 8th ed 608 / 9th ed 612]


• Pulmonary cryptococcosis may lead to mild
Reynolds - Braude phenomenon:
pneumonitis.
It is a rapid method of identifying C. albicans. It is • Dissemination of infection leads to visceral,
based on its ability to form germ tubes within 2 hours cutaneous and meningeal infection.
when incubated in human serum at 37°C. • Meningeal infection is the most serious type of
infection.
49. 'A' [Ananthnarayan 8th ed 608 / 9th ed 610] • Cryptococcosis is also called as 'European
Blasto mycosi s'.
50. 'B' [Ananthnarayan 8th ed 601 / 9th ed 612]
• Two perfect stages of the fungus have
been discovered. They belong to the class
51. 'C' [Jayaram Pani ker 6th ed 62]
Basidiomycetes. They are termed as Filobasidiella
• Leishmania donovani causes visceral leishmaniasis neoformans and F. basilispora.

53. 'C' [Jayaram Panikar 6th ed 80]


COMPARISON OF THE CHARACTERISTICS OF PLASMODIA CAUSING MALARIA
P. vivax P. falci parum P. malaria P. ovale
Hypnozoites Yes No No Yes
Erythrocyte Young erythrocytes but
Reticulocytes Old erythrocytes Reticulocytes
preference can infect all stages
Rings, trophozoites,
Stages found in Only rings and
schizonts, Sa me as vivax Same as vivax
peripheral blood gametocytes
gametocytes
Delicate, small 1.5µ m
Large, 2.5µ m, usually double chromatin &
Similar to vivax Similar to vivax
Ring stage single, prominent multiple rings are
but thicker more compact
chromatin common, Accore forms
found
Large irregular,
Small, compact, seldom Characteristic Compact coarse
Late trophozoite actively amoeboid,
seen in blood smear band fo rm pigment
prominent vacuole
Small, compact, seldom
Schizont Large filling red cell Medium size Medium size
seen in blood smear
6 - 12 in daisy
12 - 24 in irregular 8 - 24 in grape like 6 - 12 irregularly
Number of merozoites head or rosette
grape like cluster cluster arranged
pattern
Dental ;lut.,e

Sausage or banana
Spherical, compact,
shaped pale blue or
Microgametocyte pale blue cytoplasm, Same as vivax Same as vivax
pink cytoplasm, large
diffuse nucleus
diffuse nucleus
Large, spherical, Crescentic, deep blue
Macrogametocyte deep blue cytoplasm, cytoplasm, compact As in vivax As in vivax
compact nucleus nucleus
Normal, Enlarged, oval,
Normal size, Maurer's
Enlarged, pale with occasionally fim briated
Infected erythrocyte Clefts, sometimes
Schuffner's dots Zieman n's prominent
basophilic stippling
stippling Schuffner's dots
Dilation of
2 2 3 2
schizogo ny (days)
Average incuba-tion
14 12 30 14
period (days)
Appearance of
gametocyte after
4-5 10 - 12 11 - 14 5- 6
parasite patency
(days)
Duration of sporogony
9 - 10 10 - 12 25 - 28 14 - 16
in mosquito (days)

54. 'A' (Check Explanation Below] 56. 'K (Jayaram Panikar 6th ed 20)
There are two main types of T-cells. T4 cells, also Entamoeba histolytica primarily causes lesions in
called CD4+, are "helper" cells. They lead the attack crypts of lieberkuhn in the colon (large intestine).
against infections. TS cells (CD8+), are "suppressor" Flask shaped ulcers are characteristic feature of
cells that end the immune response. CD8+ cells can entamoeba.
also be "killer" cells that kill cancer cells and cells
Other sites affected in amoebiasis
infected with a virus.
Liver Amoebic hepatitis, amoebic abscesses

The ratio of CD4 cells to CD8 cells is often reported. Lung Amoebic em pyema, very rare involvement
This is calculated by dividing the CD4 value by t he • Brain
CD8 value. In healthy people, this ratio is between • Adrenals Abscesses
0.9 and 1.9, meaning that there are about 1-2 CD4 • Kidney
cells for every CD8 cell. Genitals Necrosis and sloughing
Skin Extensive necrosis and sloughing
CD4/CD8 ratio is used to help evaluate and track the
progression of HIV infection and disease. CD4 cells 57. 'B' [Jayaram Panikar 6th ed 76)
are the main target of HIV, and the number of CD4
cells will decrease as HIV progresses. 58. 'D' [Check Explanation Below]
In humans, auto infection of T. solium eggs can occur
55. 'C' [Jayaram Panikar 6th ed 17) by reverse peristalsis of the intestine.
Entamoeba histolytica:
Trophozoites reside in mucosa and sub mucosa of large Auto infection is seen in:
intestines of man. E. histolytica passes its life cycle in • Strongyloides stercoralis
only one host i.e., man. Man acquires in the infection
• Enterobius vermicularis
by ingestion of water and food contaminated with
mature quadrinucleate cysts. • Hymenolopsis nana
• Taenia solium
, MICROBIOLOGY

59. 'B' [Jayaram Panikar 6th ed 64]


• Tick typhus
• Viral hemorrrhagic fever
60. 'B' [Krause's Food, Nutrition & Diet Therapy, 11th Hard tick
• Tularemia
ed 303, 309]
• Human babesiosis
Probiotics are live microorganisms thought to be
healthy for the host when administered in adequate • Q fever
Soft tick
amounts by improving the intestinal microbial • Relapsing fever
balance. Itch mite • Scabies
• Guinea worm disease
Eg.: Bifidobacteri a Cyclops
• Fish tape worm
Lactobacillus organisms
Eubacterium organisms • Anopheles - Malaria
Certain yeasts • Culex - Japanese encephalitis
west nile fever, & filariasis
Mosquito
Probiotics are commonly consumed as part of • Aedes - Yellow fever, Dengue,
fermented foods with specially added live active Chikungunya fever, Rift valley
cultures, such as in Yogurt or as dietary supplements. fever

Uses: 62. 'D' [Jayaram Panikar 6th ed 45]


• In chronic intestinal inflammatory diseases Sleeping sickness is also known as trypanosomiasis
• Prevention of pathogen induced diarrhea or chagas disease. It is caused by T. cruzi and is
transmitted by reduvid bug.
• Urinogenital infections
• Atopic diseases Reduvid bug is also called as 'triatomid bug'.

61. 'C' [Ananthanarayan 8th ed 411/ 9th ed 413 J 63. 'B' [Jayaram Panikar 6th ed 86]
a fever: Causes of anaemia in falciparum malaria I
Q fever is caused by coxiella burnetii. Bandicoot may malignant tertian malaria:
be the primary reservoir. It is transmitted by ixodid
• Mechanical destruction of parasitized red cells
ticks in animals. Ticks do not seem to be important in
human infection. • Reduced erythropoiesis in bone marrow
• Lysis and phagocytosis of uninfected red cells
Clinical features: • Autoimmune destruction of red cells
• Interstitial pneumonia • Spleen sequestration
• Hepatitis, meningoencephalitis or endocarditis
64. 'C' [Ananthnarayan 8th ed 360/ 9th ed 362]
Injection of contaminated vaccines results in injection
Arthopod Arthopod borne disease
abscesses. Two species of atypica l mycobacteria,
• Kala-azar Mycobacterium chelonae and Mycobacterium
• Oriental sore fortuitum are causative agent of injection abscess.
Sand fly
• Oraya fever
• Rift valley fever 65. 'B' [Ananthnarayan 8th ed 81,91/ 9th ed 81,92]
Tse-Tse fly • Trypanosomiasis / sleeping sickness Most naturally, occurring antigens are proteins and
polysaccharides. A certain degree of structural
• Epidemic typhus diversity is required for antigenicity. So proteins
Louse • Relapsing fever which are composed of about 20 different amino-
• Trench fever
acids are better antigens than polysaccharides which
Reduvid • Chagar disease / South American have only four or five monosaccharide units. Gelatine,
bug Trypanosomiasis which is a protein, is not antigenic because of its
structural unstability
Dental ;lut.,e

66. 'A' [Harrison 17th ed 991 - 992, 832, 1040) Diagnosis of cryptococcosis is established by
Donovanosis (granuloma inguinale) is a chronic, demonstration of capsulated, budding yeast cells
progressive, granulomatous disease involving skin in the lesions and by culture. India in k preparation
and subcutaneous t issue of genital, inguinal and ana l stains t he capsules. In cryptococcal meningitis,
region, caused by Calymmatobacterium gran ulomatis. capsulated yeast cells are present in CS F. The fungus
This bacterium is now reclassified as Klebsiella can be cultured in sabouraud's agar.
granulomatis. The single or multiple subcutaneous
nodules erodes the skin to produce clear, sharply 71. 'C' [Medical parasitology by Rajesh Bhatia & RL
defined painless ulcer. "Pseudobuboes" may form Ichhpujani 3rd ed 30-40)
that break down to form inguinal ulcers. In contrast Two types of amoebic liver diseases can occur
to lymphogranuloma venerum, lymphadenopathy is i.e., acute non-suppurative hepatomegaly and
absent. Azithromycin (1st Choice) and Doxycycline suppurative amoebic hepatitis. Amoebic liver disease
(2nd choice) are drugs of choice. Penicillin is not begins when E. histolytica trophozoites penetrate
effective for t reating donovanosis. through the colonic mucosa, travel through the portal
circulation, and reach the liver.
67. 'A' [Ananthnarayan 7th ed 177/ 9th ed 178)
NAME DONOR SYNONYM Right lobe of liver is commonly involved. Non -
suppurative type usually do mot demonstrate amoeba
Autogenous, autogenic and is assumed to be pre-suppurative stage of amoebic
Autograft Self
graft liver abscess. Suppurative (Amoebic abscess) is the
• Ident ical twin/ most common and serious form of extraintestinal
member of same amoebiasis.
Isologous,
inbred strain.
Isograft syngenic,
• Different individual,
syngraft
72. 'B' [Ananthanarayan 9th ed 507)
genetically identical Rickettsia infections
with recipient (i) Endemic typhus is caused by R. typhi. The
Genetically unrelated Homograft, reservoir of infection are rats and the vector
Allograft member of same allogenic is (transmitted by) flea. Humans acquire the
species graft disease through flea bite, contaminated air and
water. Human is the dead end.
Heterograft,
Xenograft Different species (ii) Epidemic typhus is caused by R. prowazekii.
xenogenice.
Reservoirs of infection are squirrels. Vector is
louse and flea. Human body is the host, can
68. 'C' [Robbins basic pathology 7th ed 197)
be transmitted to other person by Louse. A
The normal CD4 count ranges fro m 800-1200/mm3.
characteristic rash appears starting on trunk
CD4 count less than 500/mm 3 may be seen in HIV
spreading over limbs but sparing face palms and
patients. In a patient with full blown AIDS, the CD4
soles.
count is less than 200/ mm 3•
(iii) Recrudescent typhus (Brill-Zinsser disease) is a
69. 'D' [Ananthnarayan 8th ed 601/ 9th ed 595)
form of epidemic typhus, where t he rickettsiae
Classification of fungi
may remain intact in lymphoid tissues for years
and at times may be reactivated leading to
• Phycomycetes
recrudescent typh us.
• Ascomycetes - includes yeasts and fi lamentous
fungi 73. 'D' [Ananthanarayan 9th ed 592)
• Basidiomycetes Stains used for fungi:
• Fungi imperfect (Deuteromycetes) - Fungi of • Calcoflour white (CFW) staining
medical im porta nee belong to this group. • Gram stain
• India ink
70. 'A' [Ananthanarayan 8th ed 610/ 9th ed 614) • PAS and methenamine si lver stain
, MICROBIOLOGY

74. 'A' [Jayaram Paniker 7th ed 89]


Toxoplasmosis is one of the more common parasitic
zoonoses world-wide caused by T. gondii. It can
infect all warm blooded anima ls and humans. The
definitive hosts are the domestic cats. Other animals
and humans are intermediate hosts.

75. 'D' [Jayaram Paniker 7th ed 77]


Clinical features of severe malaria:
• Impaired level of consciousness
• Respiratory distress, pulmonary edema
• Anaemia, ischemia
• Jaundice
• Repeated convulsions
• Shock
• Renal failure

Laboratory features of severe malaria:


• Hypoglycemia
• Acidosis (usuaHy lactic acidosis)

Therapy:
• IV quinidine gluconate 10mg/kg in normal saline
over 1-2 hours followed by 0.02mg/Kg/min
continuous infusion.
• ICV monitoring is necessary.

Uncomplicated malaria has non-specific symptoms


like headache, fever, fatigue, abdomen pain etc ...

76. 'D' [Jayaram Panikar 7th ed 167]


Trichinellosis is caused by round worm (nematode)
called T. spiralis. It is one of the smallest nematodes
infecting humans.

77. 'D' [Check Explanation Below]


Organism Ova size
Fasciola gigantic 130-150 microns
Gastrodiscoides hominis 127-166 microns
Echinostoma ilocanum 100 microns
Opisthorchis viverrini <100 microns
Dental ;lut.,e

MICROBIOLOGY - SYNOPSIS
10. Loeffler isolated the DIPTHERIAE BACCILUS
INTRODUCTION
11. Schauddin and Hoffman discovered the spirochaete
1. Karl Landsteiner
of syphilis.
• Gave the theory of "blood grouping" in 1900.
• Discovered the rhesus factor 12. Twort and d'Herelle discovered lytic phenomenon in
bacterial cultures.
2. Alexander Fleming
• Discovered the 1st antibiotic "penicillin" in 1928 13. The term virus was coined by Beijerinck.

3. Ronald Ross 14. The first human disease proved to have a viral etiology
• Discovered the transmission of malarial parasite by was yellow fever.
female anopheles mosquito.
15. The possibility that virus infection could lead to
4. Louis Pasteur malignancy was fi rst put forth by Ellerman and Bang.
• Introduced different techniques of sterilization and
developed steam sterilizers. 16. A specific humeral factor or "antibody" was described
• Invented pasteurization of milk and urine. by Von Behring and Kitasato.
• Coined the term vaccine.
17. Metchin Koff discovered the phenomenon of
• Discovered anthrax vaccine in 1881, and developed
phagocytosis.
anti-rabies vaccine.
18. Bacteria are unicellular and do not show true
5. Robert Koch
branching, except in the so-called "higher bacteria"
• First introduced solid media {Acti nomycetales).
• Int roduced staining technique and methods of
obtaining bacteria in pure cultures using solid 19. Bacteria, blue green algae Prokryotes
media
• First person to do hanging drop operation. Fungi, other algae,
} Eukaryotes
Slime moulds, protozoa
• Discovered TUBERCLE BACILLUS and VIBRIO
CHOLERA
• Proposed "KOCH POSTU LATES" by which an organism
20. Tvoes of stainina· .
can be accepted as the causative agent of that Supra vital The cell is killed during staining.
particular disease. Vital The cell is live after staining.
• He is considered as "FATHER OF MICROBIO LOGY" Dyes such as methylene blue
Simple or basic fuschin provide colour
6. Leewenhock gave the tern "ANIMALCULES" to staining contrast but impart the same
microorganisms colour to all bacteria.
The background is stained against
7. Hansen described !LEPROSY BACILLUS, so leprosy is
which t he unstained bacteria
also known as Hansen's disease. Negative
stand-out in contrast . Useful in
staining
staining of bacterial capsules,
8. Niesser described U1e GONOCCUS
slender bacterial like spirochetes,
9. Ogston discovered the STAPHYLOCOCCUS
MICROBIOLOGY SYNOPSIS

26. Slime layer and capsule:


Cells and structures th at are too
Most of the bacteria secrete a viscid material around
Impregnation thin are thickened by impregnation
the cell surface. When this is organized into sharply
methods of silver on the surface.
defined structure, it is known as the capsule.
Eg.: Bacterial flagella, Spirochete
Differential Includes Gram stain and Acid fast Capsules too thin to be seen under the light microscope
stains stain are called microcapsules.
Eg.: Pneumococcus
21. Acid-fast staining: (Ziehl and Neelson staining)
When the secreted material is a loose undemarcated
i) Discovered by Ehrlich and modified by Ziehl and secretion, it is called slime layer.
Neelson. Eg.: Leuconostoc.

ii) The smear is stained by strong solution of 27. The slime may be polysaccharide (pnemococcus) or
carbol fuschin and then decolorized with 20% polypeptide (anthrax bacillus) in nature.
sulphuric acid and then counter-stained with a
contrasting dye such as methylene blue. 28. Some bacteria may have both a capsule and a slime
layer.
iii) Acid fast bacteria - Retains red (fuschin) colour Eg.: Streptococcus salivarius
Other bacteria - takes counter stain (Blue)
29. Capsular material is antigenic and when mixed with
iv) Spores can be stained by acid- fast staining. specific anticapsular serum, it becomes swollen.
This capsule swelling or quelling reaction was fi rst
22. The lipopolysaccharides present on cell walls of G-ve described by Neufeld in pne mococci.
bacteria account for their endotoxic activity and 0
antigen specificity. They were formerly known as the 30. Some gram negative bacilli carry hair-like surface
biovin antigen . appendages called fimbriae or pili. These can be seen
only under the electron microscope. Fimbriae function
23. Mesosomes (chondroids) are more prominent in G+ve as organs of adhesion.
bacteria. They are principal sites of respiratory enzymes
in bacteria and are analogous to the mitochondria of Eg.: Escherichia, Klebsiella
eukaryotes.
31. Sex pili are special type of fimbriae. They are found
24. Voluntin granules (metachromatic or Babes-Ernst on "male" bacteria and help in attachment to fema le
granules) are highly refracture basophilic bodies bacteria, forming hollow conjugation tubes through
consisting of polymetaphosphate . These bodies can which genetic material is transferred.
be stained by Albert's or Neisser's staining techniques.
32. Sporulation in bacteria is not a method of reproduction.
Voluntin granules - are characteristically present in Spores formed in bacteria are endospores (eg: Bacillus,
diphtheria bacilli. They represent a reserve of energy clostridrum) whereas in fungi they are exospores.
and phosphate for cell metabolism but they are most
frequent in cells grown under conditions of nutrititonal 33. Sporulation is iniated by the appearance of clear
deficiency. area, usually near one end of the cell, which gradually
becomes more opaque to form the "fore spore". Though
25. Plasmids or episom es are extra nuclear genetic elements some spores may resist boiling for prolonged periods,
consisting of D.N.A. They are seen in some bacteria spores of all medically important species are destroyed
and are considered as cytoplasmic carriers of genetic by autoclaving at 120°C for 15 mins.
information. They may confer certain properties like
toxigenicity and drug resistance. Spores can be stained by Ziehl-Neelson technique.
Dental ;lut.,e

34. Streptobacillus non iliformis, is a good example of 43. Osmotic Effect on bacteria:
L-form, which is a morphologically abnormality that Exposure to hypertonic solutions
develops either spontaneously or in presence of Plasmolysis may cause osmosia withdrawal of
penicillin. L-forms resemble mycoplasma. water and shrinkage of protoplasm

35. The internal of time between two cell divisions or Sudden transfer of a bacteria from a
the time required for a bacterium to give rise to two concentrated solution to a distilled
Plasmoptysis
daughter cells under optimum conditions, is known solution causes imbibition leading
as the generation time or population doubling time. to swelling and rupture of cell
For many medica lly important bacteria, the generation
time is about 20 minutes. 44. H-ion concentration and bacteria:
Some acidiphilic bacteria such as lacto bacilli grow
36. "Total count" is the total number of cells in a sample, under acidic conditions. Cholera vibrio is very sensitive
irrespective of whether they are living or not. to acid, but tolerate high degrees of alkalinity.

37. "Viable count" is the total number of cells that are Strong solutions of acid or alkali readily kill most
capable of multiplication. This method is used widely in bacteria, with exception of mycobacteria.
water bacteriology for estimation of the presumptive
coliform count in drinking water. 45. The process by which the prophage DNA confers
genetic information to a bacterium is called lysogenic
38. Bacterial nutrition: or phase conversion.
Phototrophs Derives energy from sunlight
Of great medical importance is lysogenic conversion
Obtains energy from chemical of diphtheria bacilli, which acquire toxigenicity by
Chemotrophs
reaction lysogenisation with beta phage.
Can synthesize all their organic
Autotrophs 46. In conjugation process, male or donor bacterium,
com pounds themselves
makes physical contact with female bacteria through
Unable to synthesize their own conjugation tubes (modified sex pili).
Heterotrophs
metabolites.
The F-factor is a transfer factor that contains genetic
39. In presence of oxygen, H20 2 and other toxic peroxides information necessary for synthesis of sex pilus. The
are formed in obligate anaerobes. The enzyme catalase process of transfer of host genes through the F-factor
which splits H20 2 is present in most aerobic bacteria resembles transduction and tias therefore called as
but is absent in the anaerobes. "sexduction".

40. Some bacteria like Brucella abortus require much 47. RTF or Resistance transfer factor is a plasmid of great
higher levels of carbon dioxide for growth (capnophi lie) medical importance as it is responsible for the spread
of mulH drug resistance among bacteria.
41. The lowest temperature that kills a bacterium under
standard conditions in a given time is known as the Transfer of multiple drug resistance was demonstrated
"Thermal Death Point". Under moist conditions most between E. Coli and Shigella strains and in Vibrio,
vegetative, mesophilic bacteria have a thermal point Pseudodomonas and Pasturella.
between 50-65°C and most spores between 100 and
120°(. 48. Mutational resistance may be step-wise as seen
with penicillin or one-step, mutations as seen with
42. Water is a essential ingredient of bacterial protoplasm streptomycin.
and hence drying is lethal to cells. Some delicate bacteria
like Treponema pallidum are highly sensitive; while In clinical practice, mutational resistance is of great
others like staphylococci with stand drying for months. importance in tuberculosis. If a patient is treat with
MICROBIOLOGY SYNOPSIS

streptomycin alone, initially the bacilli die in large 55. The smallest unit of antigenicity is known as the
number but soon resistant mutants appear. antigenic determinant or epitope. The combining
area on the antibody molecule, corresponding to the
49. Restriction endonucleases cleave double stranded DNA epitope, is called the parato pe.
(foreign DNA) that enter the bacterial cell.
56. "Ring test" is simplest type of precipitation test.
50. Southern blotting is a technique for identifying DNA Examples are grouping of streptococci by the Lancefield
fragments and an analogous procedure for analysis of RNA technique and Ascolis thermoprecipitin test.
is called Northern blotting. Immunoblotting or western
blotting is used for identification of proteins (antigens) . 57. The classical example of CFT is the Wassermann
reaction, formerly the routine method used for the
51. Types of infections.: serodiagnosis of syphilis.
It indicates a condition where
due to infection or sepsis 58. The term 'complement' refers to a system of factors
Focal infections which occur in normal serum and are activated
at localized sites such as
appendix, becomes generalized characteristically by antigen-antibody interaction.

Nosocomial Types of cross infection


59. Pfi effer in 1894 that cholera vibrios were lysed when
infections occurring in hospitals. injected intraperitoneally int o specifica lly immunized
Iatrogenic guinea pigs. This is called Pfieffer's phenomenon.
Physician induced infections
infections
60. Complement is a complex of nine different fractions
In apparent or
Clinical effects are apparent i.e., from C1 to C9 • The fraction C1 occurs in serum as a
sub-clinical
calcium ion dependent complex, which on chelation
Infection without any with EDTAyields three proteins C, 9, V, and S.
Atypical infection characteristic clinical
manifestation 61. The 'C' cascade is triggered -off by two parallel but
independent mechanisms. They differ only in the initial
52. Sources of infection: steps and once C3 activation occurs, the next steps are
A person who harbor the pathogen common in both pathways. Those two pathways are -
Healthy
but has never suffered from the i) Classical 'C' pathway
carrier
disease
ii) Alternative or properdin pathway
A person who has recovered from
Convalescent
the disease and continued to 62. The classical 'C' pathway is the first pathway identified,
carriers
harbor the pathogen in his body but it is a more recently evolved mechanism. The
Contact A person who acquires the pathogen first step in classical pat hway is binding of C1 to the
carrier from a patient antigen-antibody complex.

Paradoxical A carrier who acquired the pathogen


63. The alternative or properdin pathway is a more primitive
carriers from another carrier
system for non-specific innate imm unity. In this
pathway activation of C3 without prior participation
53. Enhancement of virulence is known as exaltation and of C142 occurs. The first step in alternative pathway is
reduction of virulence is known as attenuation.
the binding of C3b to an activator.

54. Exotoxins are generally formed by G+ve bacteria and 64. Plasma cells (which are derived from 8-lymphocytes)
some by G-ve organisms such and Shiger's dysentery are antibody secreting cells.
bacillus, cholera vibrio and E.Coli.
Dental ;lut.,e

65. When circulating lymphocytes are classified into T and B cells, about 5 - 10 % of the cells lack the feat ures of either
type. They are called as Null cells or large granular lymphocytes (LGL) .

66. Both dendritic cells and macrophages act as antigen presenting cells.

67. Antibodies that are produced by a single cone and directed against a single antigenic determinant are called
monoclonal antibodies. Monoclonal antibodies are very useful tools for diagnostic and research techniques.

68. Cytokines are high ly potent harmone like substances that regulate immunological, inflammatory and reparative host
responses. Eg.: Lymphokines, monokines, interleukins, interferons.

They differ from endocrine harmones in being produced not by specialized glands but by widely distributed cells. They
act locally near the producing cells (paracrine effect) or directly on the producing cells themselves (autocrine effect).
They have multiple effects on t he growth and differentiation of various cell types.

69.

It is the transfer of genetic information through the agency of free DNA


Transformation
Eg.: Pneumococci, Bacillus, Hemophilus

• The transfer of a portion of the DNA from one bacterium to another by a bacteriophage.

Transduction • Transduction appears to be more wide spread mechanism of gene transfer among prokaryotes.
The plasmids determining penicillin resistance in staphylococci are transferred from cell to cell
by transduction .

Tra nsversion Substitution of a purine for a pyramidine and vice versa in base pairing.

Transcription Replication of messenger RNA from the DNA and is mediat ed by the enzyme "RNA polymerase II"

Translation Transfer of information from MRNA to proteins resulting in polypeptide fo rmation

Reverse Formation of DNA from RNA and is mediated by 'reverse transcriptase". Reverse transcription is
transcription seen in oncogenic viruses.

Certain structurally and genetically discrete segments of DNA that have the ability to move
Transposition around between chromosomal and extra-chromosomal DNA molecules are called "Transposons
or jumping genes" and this mode of genetic transfer, is called transposition.

Transfection The infection of bacterium by the naked phage nucleic acid.

STERILISATION

1. "Sterilization" is the process by which an article, surface or medium is freed of all microorganisms either in the
vegetative or spore state.

"Disinfection" is the destruction or removal of all pathogenic microorganisms or organisms giving rise to infection
but not necessarily their spore forms.

"Antisepsis" indicate the prevention of infection by inhibiting the growth of bacteria in wounds or tissues.
MICROBIOLOGY SYNOPSIS

2. DRY HEAT STERILIZATION OR HOT AIR OVEN


i) The killing effect of dry heat is due to protein denaturation, oxidative damage and toxic effect of elevated levels
of electrolytes.
ii) Usually 160°( for two hours or 120°C for six hours is widely employed for sterilization of cutting instruments.

3. MOIST HEAT STER[LIZATION OR AUTO CLAVE

i) The lethal effect of moist heat is due to denaturation and coagulation of protein.

ii) Usually at temperature of 121 °C at 15 pounds of pressure is rec om mended for 15 minutes or 134 °C at 30 lb of
pressure is recommended for 3 minutes.

Method of sterilization Test organism

Dry heat Spores of a non-toxigenic strain of clostridium tetani


Moist heat Spores of Bacillus stean-thermophilus
Rideal Walker test or Phenol Coefficient test Salmonella typhi bacilli

4. Methods Of Sterilization Of Important Materials


MATERIAL METHOD
All suture materia ls except catgut Autoclave
Catgut Ionizing radiation
Rubber gloves, surgical instruments
Autoclave
and most of culture media
Forceps, sca lpel, scissors, needles Hot air oven
Disposable syringes Ethylene oxide
Suture materials Glutaraldehyde
Hospital wastes Incinerator
Vaccines/serum/ antibiotic solutions Filtration
Operation theatres Fumigation with formaldehyde
Preparation of the skin for surgery Iodine, Spirit, Savlon, Chlorhexidine, etc.
Absorbent points, broaches, files and
Hot-salt sterilizer or Glass bead sterilizer
other root canal instruments
Cotton pliers, cement spatulas By passing the working blade quickly through a flame several times
Swabbing the surface with tincture of themerosal followed by double
Dappen dishes, Glass slab
swabbing with alcohol
Immersing in 5.2% sodium hypochlorite for 1 min. followed by rinsing
Gutttapercha cone
with H20 2 and drying
Slowly passing them back and forth through a Bunsen flame for 3 or 4
Silver cones
times (or) by immersion in the hot salt sterilizer for 5 sec.
Loeffler's serum lope
Inspissator
Lowenstein Jensen's medium
Sugar media Tyndallisation or intermittent sterilization
Dental ;lut.,e

CULTURE MEDIA

I. Simple Media • Eg.: NUTRIENT AGAR


• Simplest and most common medium in routine diagnostic
laboratories.
• Used to study antibiotic sensitivity.

II. Complex media • Eg.: Mac Conkey agar


a) Differential • Used to differentiate lactose and non lactose fermenters .
(Contains substances to distinguish • Salmonella, Shigella - Non lactose fermenters
different types of types of bacteria)
• Escherichia, Klebsiella - Lactose fermenters
• Paracolons - Late lactose fermenters

b) Enriched • Bloodagar - To study hemolysis


(Substances such as blood, serum or egg • Loeffler's serum slope - For Corynebacterium diptheriae
are added to basal medium)

c) Enrichment • Selenite F broth - Salmonella, shigella


(Substances which have stimulating • Tetrathionate broth - Salmonella typhi and para typhi
effect on the bacteria to be grown or an
inhibitory effect on those to be suppressed
are incorporated into medium)

d) Selective media i) Lowenstan Jenson medium - For Mycobacterium


tuberculosis
ii) Wilson and Blair medium - For Salmonella typhi and
para typhi
iii) Thio sulphate citrate bile - For Vibrio cholera
salt sucrose medium (TCBS)

e) Indicator media • Wilson and Blair medium - Salmonella typhi


(Contains an indicator which changes Salmonella typhi reduces sulphite to sulphide in presence of
colour when bacterium grows in them) glucose and the colonies of S. typhi have a black metallic
sheen.
• McLeod's medium - Oiptheria bacillus

III. Transport media • Stuarts medium - Gonococci


• Cary Blair and Venkatraman
Ramakrishnan medium - Cholera vibrio
• Pike's medium - Streptococci

IV. Anaerobic media • Robertson's cooked meat media - For Clostridium


• Thioglycolate medium - For growth of Actinomyces
MICROBIOLOGY SYNOPSIS

IMMOUNOGLOBULINS
Class H Chain Half Life % of lg Important features
• Major serum immunoglobulin.
• Normal serum concentration is 8-16mg per ml.
• Only maternal immunoglobulin that cross the placenta and provides
natural passive immunity in newborn.
lgG Gamma 23 days 80
• Classical complement fixation is positive.
• Smallest imm unoglobulin.
• Also known as WARM immunoglobulin.
• Predominant immunoglobulin in the gingiva l fluid (GCF).
• Earliest immunoglobulin synthesized by foetus in response to infection.
• Oldest immunog lobulin class.
• The presence of IgM in the Fetus or new born indicates intrauterine
infection and is useful in diagnosis of congenital infections such as
syphilis, rubella, HIV infection.
• IgM antibodies are short lived and disappear earlier. Hence their
IgM Mu 5 days 5-8
demonstration in serum indicates recent infection.
• Both IgG and IgM class of immunoglobulins activates the classical
pathway of complement fixation.
• Also known as COLD immunoglobulin.
• Both IgG an IgM are involved in Type II or cytotoxic type of
hypersensitivity reactions.
• znd most abundant class.

• Occurs in two forms i.e., Serum IgA and Secretory IgA. Secretory IgA
is synthesized fro m mucosa l or glandular epithelium and protects lgA
from proteolysis.
lg A Alpha 6 days 10 - 13 • Predominant immunoglobulin in saliva.
• Immunoglobulin present in exocrine secretions.
• Caries vaccine is prepared based on action of lg A in the oral cavity.
• It activates the alternate pathway of complement fixation.
• Both IgA and IgG are secreted in milk.
• Has unique property of heat labiality i.e., inactivated at 56° C in one
hour.
• Has affinity for mast cells of same species (Homocytotropism)
• Mediates Prausunitz-Kustner reaction, a method to detect atopic
2- 8 antibody.
lgE Epsilon 1
days
• Responsible for anaphylactic type of hypersensitivity reactions.
Eg.: Contact dermatitis
• Chiefly produced in the linings of respiratory and intestinal tracts.
• IgE is believed to have a special role in parasitic infections.
• Structurally resemble IgG.
IgD Delta 1-5 1 • IgD and IgM serve as recognisation receptors for antigens.
• Activates the B-lymphocytes for antibody production or suppression.
Dental ;lut.,e

HYPERSENSITIVITY REACTIONS
Time
Clinical
Type required for Mediators Im~ortant exam~les
Syndrome
manifestation
IgE histamine • Reactions due to administration of penicillin .
and other
1. Anaphylaxis Minutes
vasoactive
Type I amines
(lgE type) • Bronchial asthma

2. Atopy Minutes - do -
• Allergic dermatitis
• Angioneurotic edema
• Hay fever (seasonal allergic rhinitis)
• Reactions in dental pulp and periapical
infections.
Antibody • Autoimmune hemolytic anemia .
Type II mediated Hour to days lg G, lg M, C • Idiopathic thrombocytic purpura.
damage • Blood transfusion reactions.
• Erythroblastosis fetalis.
• Myasthenia gravis.

1. Arthus
• Intrapulmonary arth us like reaction to
inhaled antigen.
reaction lg G, lg M, C,
(local
Hours to days
leucocytes • Farmer's lung in which there is allergic
alveolitis in response to bacterial antigen
manifestation)
Type III from mould hay.
2. Serum • Steven Johnson syndrome.
sickness
- do - - do -
• SLE, Polyarteritis nodosa, Sjogren's syndrome.
(Generalized • Skin disease.
manifestation)
• Various forms of glomerulo nephiritis.

1. Tuberculin or
T Cells, • Ma ntoux Test
Hour to days lymphokines
Infection type
Type IV macrophages
2. Contact - • Contact dermatitis due to allergic response
dermatitis - to a number of chemicals ( Eg.: acrylic)

EXAMPLES OF GRAM + VE AND GRAM -VE ORGANISMS

Gram +ve Coed Gram -ve Coed Gram +ve Badllf Gram -ve Badllf
• Staphylococci • Gonococci • Diptheria • Salmonella
• Streptococci • Meningococci • Anthrax • Proteus
• Pnemococci • Tetanus • Pseudomonas
AERO BE • Tuberculosis (obligate aerobe)
• Actinomyces • E. Coli
• Cholera
• Klebsiella

ANAEROBE • Peptococci • Veillon ella • Clostridium • Bacteroides


• Peptostreptococci • Fusobacterium
MICROBIOLOGY SYNOPSIS

IDENTIFICATION OF BACTERIA
GRAM +VE LOGARITHAM
Gram Stain

t
Gram +ve (purple/ blue)

Cocci Bacilli
Eg.: Clostridium(anaerobe)
Corynebacterium
1
Cata lase +ve ( clusters) Catalase -ve (chains)
Staphylococcus Streptococcus

l
Coagulase +ve Coagulase -ve
Eg.: S. aureus Eg.: S. epidermis
(S.pyogenes) (S. albus)

Green ( a) he mo lysis Clear ( ~) hemolysis No hemolysis

I Eg.: Enterococcus

Capsu Le/Quelling +ve


optochin sensitive
No capsule
optochin resistant
j
Eg.: S. pnemoniae Eg.: S. viridans Bacitracin Bacitracin
sensitive resistant
Eg.: Group A S.pyogenes Eg.: Grollp B S.agalactiae

GRAM -VE LOGARITHAM


Gram Stain

+
Gram -ve (Pink)

Cocco bacilli Ba Hli


Eg.: H. influenza, Bartonella henselae

t
Maltose
t
Maltose Lactose fermenter Lactose non-fermenter
fermenter non-fermenter Eg.: Klebsiella
~
Eg.: N.meningitides Eg.: N. gonorrhea Escherichia coli Oxidase +ve Oxidase -ve
Eg.: Pseudomonas Eg.: Shigella, Salmonella
Dental ;lut.,e

8. Important toxins of staphylococci:


BACTERIOLOGY
• Enterotoxin, which is responsible for the
STAPHYLOCOCCI: manifestations of staphylococcal food poisoning.
• Toxic shock syndrome toxin (TSST).
1. The name staphylococci is due to its typical occurrence • Exfoliative (epidermolytic) toxin which is
of the cocci in grape-like clusters (Bunch of grapes) responsible for SSSS or staphylococcal scalded
skin syndrome. The severe form of SSSS is known
2. as Ritter's disease in the new born and toxic
Staph aureus • Coagulase test is+ ve epidermal neurolysis in older patients.
(Staph. pyogenes) • Causes acute endocarditis
9. During laboratory diagnosis, of staphylococci,
Staph. epidermidis • Coag ulase test is -ve coagulase test is done by two methods - tube and
(Staph. albus) • Causes post-operative slide coag ulase tests. The tube coagulase test detects
endocarditis free coagulase. The slide test detects bound coagulase.

3. On nutrient agar slope, the confluent growth presents STREPTOCOCCI


a characteristic "Oil-paint" appearance.
1. Classification of streptococd:
4. The useful screening procedure for differentiating a) a -hemolytic streptococci:
Staph. aureus from Staph. epidermidis in mixed cultures
• Produce a greenish discolouration with partial
is, phosphatase reaction, as the former gives prompt
hemolysis around colonies. The zone of lysis is
phosphatase reaction (colonies assumes a bright pink
small.
colour), while t he latter is usually -ve or weakly +ve.
• Known as viridans streptococci or st reptococcus
5. Pencillinase resistance of staphylococci is of three viridans, which is a group of miscellany of
types. streptococci.
• Production of beta lactamase (penicillinase that Eg.: Str. mitis, Str. mutans, Str. salivarius, Str.
inactivates penicillin resistance by splitting bet a sanguis (most often responsible for bacterial
lactum ring. endocarditis)
• Changes in bacterial surface receptors, reducing • The a -streptococci are normal commesals in the
binding of beta-lactam antibiotics to cells. throat and may cause opportunistic infections
• Development of tolerance to penicillin by which the rarely.
bacterium is on ly inhibited but not killed.
b) B-hemolytic streptococci:
6. The plasmids determining penicillin resistance in • Produce a sharply defined clear, colourless zone
staphylococci are transferred fro m cell to cell by of hemolysis. More pathogenic streptococci
transduction. Acquisition of resistance by transduction belong to this group.
is common in staphylococci. • p-hemolytic streptococci were classified by
Lancefield into twenty lancefield groups based
Staphylococci also exhibit plasmid-borne resistance to on nature of a carbohydrate 'C' antigen on the
erythromycins, tetracyclines and almost all clinically cell wall. The groups are named from A - V
useful antibiotics except vancomycin. For life (without I and T). Most of streptococci that
threatening staphylococcal infections, vancomycin is produce human infections belong to group A
the drug of choice. and are known as Str. pyogenes.

7. Techoic acid, an antigenic component of cell walls of


c) y-hemolytic /non-hemolytic/indifferent streptococci
staphylococci, facilitates adhesion of the cocci to the
• Known as enterococus group
host cell surface and protects them from complement-
Eg.: Str. fecalis
mediated opsonisation (Also seen in streptococci).
MICROBIOLOGY SYNOPSIS

2. Hydrolysis of pyrrolidonyl napthylglamide (PYR • Vancomycin is the drug of choice in life


test) and fai lure to ferment ribose are useful in threatening infections.
differentiating Str. pyogenes from other streptococci. • Acute rheumatic fever and acute glomerulo
nephritis are two important non-suppurative
Also sensitivity to bacitracin is em ployed as a sequel of Str. pyogenes.
convenient method for differentiating Str. pyogenes
• The essential lesion in rhematic fever is carditis,
from other hemolyt ic streptococci.
including connective tissue, degeneration of
heart va lves and inflammatory myocardial lesions
3. TOXINS OF STREPTOCOCCI:
characterized by Aschoff nodules.
i) Pyrogenic/Erythrogenic/dick/Scarlantinal toxin:
• This toxin, when injected, produced 5. The streptozymase test is a convenient, sensitive and
erythematous toxin (Dick test). This test is specific screening test.
used to identify children susceptible to scarlet
fever, a type of acute pharyngitis with extensive 6. The prophylaxis in prevention of rheumatic fever is
erythemato us rash, caused by Str. pyogenes. achieved by
• Blanching of this erythematous rash on local
injection of convalescent serum is used as 7. Group-B streptococci (Str.agalactiae) is the sing le
a diagnostic test for scarlet fever (Schultz most common cause of neonatal meningitis in west.
Charlton reaction ) Their ability to hydrolyse hippurate acts as important
identification method.
ii) Streptokinase/Fibrinolysin:
• Causes breakdown of the fibrin ba rrier around the 8. Group F streptococci are called "Minute streptococci".
lesions and facilitates the spread of infection.
9. Enterococci or feca l streptococci belong to Group D
iii) Deoxyribonucleases/streptodornase: streptococci.
• Causes depolymerisation of DNA.
STREPTOCOCCUS PNEUMONIAE I PNEMOCOCCUS I
• Helps to liquefy by the thick pus and is DIPLOCOCCUS PNEMONIAE
responsible for thin serous character of
streptococcal exudates (pyrogenic exudates 1. Pnemococci are normal inhabitants of the upper respiratory
contain large amounts of DNA) tract of human beings. It is single most bacterial agent in
pneumonia and in otitis media in children .
iv) Hyaluronidase:
• Causes breakdown of hyaulronidase (universal 2. Pnemococci are slightly elongated cocci, and occurs
intercellular substance) and helps in spread of in pairs.
infection.
3. With one broad end and other pointed end, they have
4. Pathogenicity: a flame shaped or lanceolate appearance.
• Sore throat is most common streptococcal disease.
• Ludwig's angina, otitis media, Quinsy, cellulites. 4. On blood agar, they have draughtsman or carom coin
appearance.
• The two typical streptococca l infections of the
skin are erysipelas and impetigo.
5. They are sensitive to most antibiotics, penicillins being
• Erysipelas is rare and involves the superficial
the drugs of choice. The sensitivity of pnemococci to
lymphatics of older patients.
optochin is useful in differentiation of t hem from
• Impetigo is found mainly in young children . streptococci.
• Impetigo and streptococcal infection of scabies
lesions are the main causes leading to acute 6. Pnemococci possess a specific polysaccharide capsule
glomerulonephritis in chi ldren. that is responsible of "Quellung" reaction.
Dental ;lut.,e

7. Pat hogenicity: CORYNEBACTERIUM DIPHTHERIAE


• Pnemococci are one of the most common
bacteria causing pneumonia, both lobar and 1. Contains volutin I metachromatic I Babes - Ernest
bronchopneumonia. granules, which are composed of polymetaphosphate.
They are often sit uated at poles of the bacilli and are
• Pneumococci and haemophilus influenzae are the
two most common organisms associated with the called "Polar bodies".
acute exacerbations in chronic bronchitis.
2. In smears, the bacilli are arranged at resembling the
• Meningitis is the most serious of pneumococcal
letters V or L. This is due to incomplete separation of
infections.
the daughter cells after binary fission . This arrangement
is called Chinese letter or cuneiform arrangement.
NIESSERIA
3. McLeod classified diphtheria bacilli into three types -
1. This group includes two important pathogens
gravis (most serious), intermedius, and mitis (mildest
N.meningitidis and N. gonorrhoeae that occurs in
manifestations).
pairs ( Di plococci).
4. The strain of corynebacterium that is universally used
2. N. Meningitidis:
fo r toxin production is the Park William 8 strain
• Causes cerebrospinal fever or meningococcal (mitis type).
meningitis.
• The colonies are typically lenticular in shape. 5. Nontoxigenic strains of corynebacterium may be
• Modified Thayer-Martin is a selective mediu m for rendered toxigenic by infecting them with beta phage.
both types of niesseria. This is known as lysogenic or phage conversion.
• The prompt oxidase reaction (Koracs method) helps
to identify both types of niesseria in mixed cultures. Eg.: Pnemococci, Hemophilus
Transformation
• Unlike other niesseria, meningococci are influenzae, Bacillus species.
capsulated, and based on the capsular antigens, Eg.: Staphylo cocci (plasmid
t hey are classified into 13 sero groups, of which Transduction
mediated), Diphtheria bacilli
Groups A, B and C are the most important.
Conjugation E. Coli
Group-A Associated with epidemics
Group-B Causes both epidemics and outbreaks 6. Diphtheria produces powerful exotoxin . It causes local
Group-( necrotic changes and the disintegrating epithelial
Causes outbreaks
cells, leucocytes, erythrocytes, and bacteria constitute
the pseudo membrane, which is characteristic of
• Intravenous penicillin is the treatment of choice.
diphtheria.

3. Niesseria gonorrhoeae: (gonorrhoea)


7. Elek's gel precipitation test is used to identify the
• It is a diplococrns with the adjacent sides concave, toxigenic strains. The toxigenic strains produce lines
being typically kidney shaped. of precipitation.
• Contains pili that help in phagocytosis.
8. Schick test is susceptibility test and is no longer in
• The trilaminar outer membrane of gonococci use.
contains lipopolysaccharide (endotoxin) and many
different proteins. Protein I is the major constituent 9. The Dansyz phenomenon and Ehrlich phenomenon ,
and helps in typing of gonococcal strains. which are seen during preparation of diphtheria
• When the infection involves perurethral tissues, it vaccine, is due to the ability of toxins and antitoxins
causes abscesses and multiple discharging sinuses. to combine in varying proportions.
This is known as "Watercan perineum".
MICROBIOLOGY SYNOPSIS

10. Formol toxoid is prepared by incubating the toxin 9. The disease is common in workers carrying loads of
with fo rmalin unti l the product is devoid of toxicity. skins on their bare backs and hence known as "hide
porter's disease".
Adsorbed toxoid is purified t oxoid adsorbed into
insoluble alumini um compounds such as Alu minium 10. Pulmonary anthrax is called "wool sorter's disease"
phosphate. Adsorbed toxoid is more immunogenic because it is used to be common in workers in wool
than formol or fluid toxoid. factories.

11. Diphtheria toxoid is usually given in children along 11. The lesion of anthrax starts as papule with several
with tetany toxoidJ and pertusis vaccine, as the DTP/ satellite lesions filled wit h serum . This lesion is called
DPT/Triple vaccine. A quadruple vaccine containing "malignant pustule".
inactivated polio vaccine in addition, is also available.
12. Ascolis thermoprecipitation test, which is a ring
BACILLUS precipitation test is used for demonstration of ant hrax
ant igen in tissue extracts.
1. Causative agent of anthrax (zoonosis).
13. Vaccines:
2. Bacillus is • Pasteur's vaccine is the anthrax bacillus
• The first pathogenic bacterium to be observed under attenuated by growth at 42 - 43°C.
the microscope. • The sterne vaccine contains spores of a non
• The first communicable disease shown to be capsulated avirulent mutant strain.
transmitted by inoculation of infected blood. • The mazucchi vaccine contains spores of stable
• The first bacillus to be isolated in pure culture and attenuated carbazoo strain in 2% saponin.
shown to possess spores.
• The first bacterium used for preparation of 14. B. cereus causes food poisoning.
attenuated vaccine.
CLOSTRIDIUM
3. The ends of the bacilli are truncated or often concave
and somewhat swollen so that a chain of bacilli 1. Clostridia are motile with peritrichate flagella.
presents a 'bamboo stick' appearance. Exceptions are Cl. perfringens and Cl. tetani type VI.

4. Sporulation of ant hrax bacilli is inhibited by Calcium 2. Cl. perfringens and Cl. butyricium or capsulated, while
ch lo ride. others are not.

5. M'Fadyean's reaction (due to capsule) is employed for 3.


presumptive diagnosis of anthrax in animals. Shape of the
Organism Disease
spores
6. "Medusa head appearance" and "inverted fir tree
Gras
appearance" are cultural characteristics of Bacillius
Cl. perfringens gangrene/
a nth racis. Club shaped
(Cl. welchii) malignant
edema
7. The "string of pearls reaction" and the susceptibility
of anthracis to gamma phage helps to differentiate B. Drum stiick
anthracis from B. cereus and other aerobic spore bearers. (Bamboo stick
Tetanus or
Cl. tetani appearance is
lock jaw
8. "Duckering" is t he destruction of the spores in animal seen in Anthrax
products imported into non-endemic countries, in bacillus)
which formaldehyde is used for disinfection of wool, Cl. bifermentans Spindle shaped --
animal hair and bristles.
Dental ;lut.,e

iii) Prophylaxis:
Cl. tertium Tennis racket --
• Passive immunization is by injection of 1500
Acute
I.V. tetanus antitoxin given subcutaneously or
pseudo-
Cl. difficile -- intra muscularly.
membranous
colitis • Active immunization in form of DTP triple
vaccine or quadruple vaccine (along with polio
Cl. botuli ni um Botulin um vaccine) at the ages of 6 wks, 10 wks, 14 wks
Type A & C or food followed by booster dose at the age of 16 - 24
--
strains of Cl. poisoning months.
perfringens
• Combined immunization consists of
administering to a non immune person exposed
4. Clostrium perfringens: to risk of tetanus TIG injection at one site, along
i) Grows under microaerophilic conditions (other with the first dose of toxoid at the contra lateral
example which is microaerophilic is Actinomyces). site, followed by the second and third doses of
toxoid at monthly intervals.
ii) Naglers reaction is a useful test for the rapid
detection of Cl. perfringens in clinical specimens and SALMONELLA
the toxin involved in this reaction is lecithinase.
1. Most important member of the genus is salmonella
iii) Gas gangrene caused by Cl. novyi is characterized typhi. Also called as Eberth-Gaffky bacillus or
by high mortality and large amounts of edema. Eberthella Typhi.

iv) Laboratory diagnosis of different species: 2. Motile with peritrichate flagella, except S. gallinarum-
• Presence of large, regularly shaped G+ve bacilli pullorum, which is always non-motile.
without spores strong ly suggests Cl. perfringens
infection. 3. On Wilson and Blair bismuth sulphite medium, jet
• "Citron bodies" and boat or leaf shaped black colonies with a metallic sheen are formed due to
pleomorphic bacilli with irregular staining production of H2 S.
suggest Cl. septicum.
• LARGE bacilli with OVAL, SUBTERMINAL spores S. paratyphi and other species that do not form H2 S
indicate Cl. novyi. produce green colonies.
• SLENDOR bacilli with ROU ND, TERMINAL spores
4. Antigens of salmonella typhi
indicates Cl. tetani or Cl.tetanomorphum
• H antigen
5. Clostridium tetani: • 0 antigen
i) Cl. teta ni produces two disti net toxins. • Vi antigen
• Tetanolysin which is a hemolysin. It is not
relevant in pathogenesis of tetanus. 5. When Salmonella are grown on agar containing phenol,
• Tetanospasmin , which is a powerful exotoxin flagella are inhibited. This change is phenotypic and
responsible for tetanus. temporary.

ii) Tetanospasmin resembles strychnine in its effects. 6. To obtain a population of motile cells rich at H antigen,
But it acts presynaptically, unlike strychnine which from such cultures, selection may be carried out by
acts post synaptically. Craigie's tube.

It specifically blocks synaptic inhibition in the 7. Classification of salmonella genus is on antigenic


spinal cord, presumably at inhibitory t erminals characterization based on the Kaufman-White
that use glycine and GABA as neurotransmitters. scheme. This scheme depends on the identification,
by agglutination, of the structural formu lae of the
MICROBIOLOGY SYNOPSIS

0 and H antigens of the strains. The Kauff Mann-White 11. Prophylaxis:


scheme gives species status to each serotype. i) TAB vaccine that contains inactivated S. typhi
and S. paratyphi
8. Enteric fever (Typhoid + Paratyphoid)
ii) Typh ORAL or live oral vaccine, which is a stable
i) The infection is acquired by ingestion . As bile mutant of S. typhi strain Ty21a, lacking the
is a good culture medium for the bacillus, it enzyme UDP-Galactose-4 epi merase.
multiplies in the gall bladder and is discharged iii) The injectable vaccine or Typhim-Vi.
continuously into the intestine where it involves
the Peyer's patches and lymphoid fo llicles of the 12. Salmonella gastroenteritis or food poisoning may be
ileum. caused by any salmonella except S. typhi. In most
parts of the world, S. typhimurium is most common
ii) Ulceration of bowel leads to two major
species causing food poisoning.
complications of the disease i.e., intestinal
perforation and hemorrhage.
0TH ER ENTEROBACTERIA
iii) The typical features are a step-ladder pyrexia,
soft and palpable spleen, and hepatomegaly. 1. The enterotoxins of E. Coli are:
• Heat Rabile toxin (LT)
iv) "Rose spots" that fade on pressure appeal on the
skin are seen during 2nd or 3rd week. • Heat Stable taxi n (ST)
• Vero toxin or shalga Like toxin (SLT)
v) Depending 0 11 periods of shedding of typhoid
bacilli after clinical cure, patients are divided 2. Clinical syndromes associated with E. Coli
into:
• Urinary tract infection
Convalescent carriers 3 wks to 3 months • Diarrhoea
More than 3 months • Septicemia
Temporary carriers
but less than a year • Pyogenic infections
Over a period of one
Chronic carriers
year 3. The screening techniques for identifying bacteriuria
are:
9. Laboratory diagnosis: • Griess nitrite test
i) Blood culture are positive in approximately 90 • Catalase test
percent of cases in first week of fever. • Triphenyl tet razolium chloride (TTC) test
ii) Salmonellae are shed in feces throughout the • Dip slide culture methods
course of the disease both in patients and carriers.
iii) Bone marrow culture (more confirmative) is 4. The five different types of diarrheagenic E.Coli and
the imoortant MCQ's asked till now are
valuable as it is positive in most cases even when
blood cultures are negative. Enteropathogenic Associated with diarrhea in
E. coli (EPEC) infants and children usually
iv) Bile culture is employed usually for the detection
of carriers. Enterotoxigenic
Causes traveller's diarrhoea
E. coli (ETEC)
v) Widal test is used for measurement H and 0
agglutinins for typhoid and paratyphoid bacilli Named so because they
Entero aggregative
in patient's sera. appear agg regated in a
E. coli
"Stacked brick" appearance
10. The tracing of carriers in cities may be accomplished SERE NY test is used in
by the "Sewer-Swab" technique. Enteninvasive E. coli
laboratory diagnosis
Dental ;lut.,e

VERSINIA. PASTEURELLA. FRANCISELLA


• Produces verocytotoxi n
(VT) or shiga - like toxin
Enterohemorrhagic 1. Yersi nia pestis is causative agent of plaque or Black
(SLT)
E.coli or Death.
• The typical sere type
verotoxigenic E.coli
involved in this is
0157:H7
2. Because of bipolar staining, it gives "Safety pin
appearance" in stained smears.

5. Of the two commorn types of dysentry i.e., AMOEBIC


3. Shows "stalacite growth" in gliee broth culture.
and BACILLARY, shigella is responsible for bacillary
dysentery.
HAEMOPHILUS I PFEIFFER'S BACILLUS

VIBRIO CHOLERAE
1. H. influenza is the first free living organism whose
complete genome has been sequenced.
1. Arranged in parallel rows known as "Fish in stream"
appeara nee.
2. Characteristically depends on growth factors X and V
present in the blood.
2. Vibrio colonies are identified by the "String test".
3. The Vfactor is NAO or NADP
3. The typical "Cholera red reaction" is due to formation
of "nitroso-indole"
4. Shows the phenomenon of "Satellitism", which
demonstrates the dependence of H. influenza on V
4. "Copro antibodies" develop in the intestine appear in
factor.
the feces. They consist of IgG, Ig M, IgA.
5. Pathogenicity includes meningitis, pneumonia and
PSEUDOMONAS
laryngo epiglotitis (previous AIIMS MCQ), of which
meningitis is the most serious disease produced by H.
1. Ps. aeruginosa is the most common and most serious
influenzae. Cefotaxime or ceftazidime is the drug of
cause of infection in burns. It is also one of the agents
choice.
responsible for iatrogenic meningitis following lumbar
puncture.
6. H. parainfluenzae differs from H. influenzae in that it
requires only the factors V but not the factor X.
2. It is non-capsulated with a polar flagellum and mucoid
slime layer.
BORDETELLA

3. P. aeruginosa produces two pigments i.e.,


1. In cultures it forms a 'aluminium paint' or "thumb
• pyocyanin (B lue pigment), which inhibit the print' appearance.
growth of many other bacteria and contribute to Ps.
aeruginosa emerging as the dominant bacterium in 2. Colonies on Bordet-Gengou medium resembles
mixed infections. 'Bisected pearls' or 'mercury drops'.
• Fluorescin , a greenish yellow pigment.
3. "Pertusis toxin" is responsible for pathogenesis of
4. Causes "Blue Pus" or pyocyanea "whooping cough".

5. In the community outside the hospital, the most 4. The disease course consists of "catarrhal", "paroxysmal"
common infection caused by Ps. aeruginosa is and "convalescent".
suppurative otitis.
5. The catarrh al stage is "subclinical" and shows maximum
i nfectivity
MICROBIOLOGY SYNOPSIS

6. During the "paroxysmal" stage, the patient is subjected 4. Restriction endonuclease treatment yields nucleic acid
to violent spasms of contin uous coughing, fo llowed by frgments of varying lengths, and the pattern of lysis
a long inrush of air into the almost empty lungs, with is strain specific. This "restrictions fragment Length
a characteristic whoop, so called as whooping cough. polymorphism" (RFLP) enables strain typing for
epidemiological purposes.
BRUCELLA
5. Pathogenesis:
1. Brucella is primarily an intracellular pathogen affecting i) The source of infection is usually an open case of
the reticuloendothelial system. pulmonary t uberculosis.

2. It is a zoonotic disease and t he most important vehicle ii) The virulence is due to their ability to survive
of infection is raw milk. and multiply in macrophages (but not related to
secretion of toxin).
3. Acute brucellosis (also known as undulant fever) is
mostly due t o Br. melitensis (also called micrococcus iii) The on ly specific immune mechanism effective
melitensis because of their resemblance t o small is the cell mediated type and the key cell is
cocci). activated CD4+ helper T cell.

4. Laboratory diagnosis is by castanada method of iv) Because of its devastating morbidity and massive
blood culture. It is a recommended method and most mortality, tuberculosis is also called as "The
definitive method for diagnosis of brucellosis. white plaque" and "The captain of all men of
death".
5. Different diagnostic tests employed for the detection
of brucellosis in cattle are - 6. Diagnosis includes microscopy and concentration
• Rapid agglutination test methods. In microscopy after staining the smears are
graded depending on the number of bacilli seen
• Rose Bengal Card test
+1 3 - 9 bacilli seen in entire smear
• Milk ring test
+2 > 10 bacilli are seen in smear

6. Immunity in brucellosis is mainly cell mediated. +3 > bacilli are seen in most oil immersion
Activated macrophages can kill the bacteria. This fields.
is the most important mechanism in recovery and
immunity in brucellosis. Petroff's method is simple and most widely used
concentration method.
MYCOBACTERIUM TUBERCULOSIS:
7. Montoux test :
1. Acid fastness of mycobacterium is due to presence i) In this test, 0.1 ml of PPD (Purified protein
of mycolic acid in the cell wall (Techoic acid which is derivative) containing 5TU (tuberculin units)
present in cell wall of streptococci and staphylococci is injected intradermally on the flexor aspect of
facilitates adhesion of cocci to host cell) forearm.
ii) The site is examined 48 - 72 hours later and
2. The catalase-peroxidase tests help in differentiating
induration is measured
t ubercle bacilli from atypical mycobacteria and provide
Induration of 10mm diameter - positive
an indication of sensitivity of the strain to isoniazid .
Induration of 6-9mm - equivocal
3. Mycobacterium is not truly lysogenic. Instead of being Induration of 5mm or less - negative
int egrated wit h the bacterial chronomosome, the
iii) A positive tuberclin test indicates hyper sensitivity
phage genome appears free, like a plasmid. This is
to tuberculoprotein denoting infection with
called pseudo Lysogeny.
tubercle bacillus or BCG immunization.
Dental ;lut.,e

iv) False negative tests are seen in 5. Lepromin test


• Miliary tuberculosis i) Described by mitsuda which includes intradermal
injection of Lepromatous tissue rich in Lepra bacilli.
• Inactive PPD preparation
• Improper injection technique ii) The response is biphasic, consisting of two
• lmmuno suppressive therapy separate events.
• Lymphoreticu lar malignancy iii) The first is early reaction of Fernandez, which
• Sa rcoidosi s, etc. carries little significance.

iv) The second and more significant is late reaction


8. Immunoprophylaxis is by intradermal injection of Live
of mitsuda, starting in 1 - 2 weeks.
attenuated Bacillus Calmette Guerin vaccine or BCG
vaccine (developed by Calmette and Guerin). v) Lepromin test is not used to diagnose Leprosy nor
it indicates prior contact with the Lepra bacillus.
M. LEPRAE The test is employed for -
• To classify leprosy patients as this test is +ve
1. The bacilli appear as agglomerates of globi, formed in tuberculoid and -ve in Lepromatous Leprosy
by Virchow's "Lepra cells" or "foamy cells", which are patients.
large undifferentiated histiocytes.
• Positive reaction indicates the persons are more
resistance toleprosy and the prognosis is good.
2. The parallel rows of bacilli in the globi present a "cigar
bundle" appearance.
SPIROCHETES (Treponema + Borrelia + Leptospria)

3. The disease is classified into Lepromatous (host


1. Characteristically contains endoflagella.
resistance is low), tuberculoid, dimorphous and
indeterminate. The Indian classification of Leprosy
2. Treponema pallidum has about ten regular spirals which
has an additional type, the pure neuritic type,
are sharp and angular at regular intervals of 1 µm.
which is bacteriologically negative and shows neural
involvement without any skin Lesion .
3. T. pallidum is very sensitive to both heat and cold.
Susceptibility of T. pallidum to heat is the basis of the
4. Sometimes acute exacerbation, which are of allergic in
"fever therapy" for syphilis.
nature occurs, though Leprosy is a chronic disease. Two
types of such reactions a re:
It is killed in 1 - 3 days at O - 4 °C, so that transfusion
syphilis can be prevented by storing blood in
i) Type I or reversal reaction or lepra reaction
refrigerator before transfusion.
• Seen mostly in borderline leprosy, occurring
either spontaneously or more often during 4. "Cardiolipin' is standing antigen and is chemically
chemotherapy. "diphosphatidyl glycerol'.
• It is a cell mediated immune reaction and
indicated a shift to tuberculoid morphology. 5. Yaws is caused by T. pertenue.

ii) Type II reaction or erythema nodosum leprosum 6. Borrelia readily undergoes antigenic variations in vivo
• Occurs in LL and BL types, a few months after and is believed to be responsible for invivo and is
institution of chemotherapy. believed to be responsible for relapses in the disease.
Ultimate recovery after a number of relapses may
• Characterised by inflamed sub-cutaneous
be due to the development of immunity to all the
nodules, fever, lymph-adenopathy and arthralgia.
antigenic variants.
• It is an arth us type of response to antigens
released fro m dead lepra cells.
7. Borrelia vincenti is a normal mouth commensal
and along with Fusiform bacilli (Fusosobacterium)
MICROBIOLOGY SYNOPSIS

caused ulcerative gingivostomatitis or oropharyngitis Both viruses and rickettsia require living cells for their
(Vincent's angina). This symbiotic infection is known growth. Only difference is that rickettsia contains both
as fusospirochetosis. RNA and DNA, but virus contains either RNA or DNA.

8. Lyme disease or lyme borreliosis as caused by burdorfei. 8. Ch lamydiae are obligate i ntrac::ellular bacterial parasites
of humans. They lack enzymes of the electron transport
9. Lepto spires possess a large number of closely wound chain and so require ATP and nutrient resources from
spirals and characteristic hooked ends. host cells. So they are called as "energy parasites"

Causes icetero haemorrhagic fever or Weil's disease. It 9. The developing chlamydia[ microcolony within the
is associated with 3 R's i.e. rats, rice fields and rainfall. host cell is called the inclusion body.
(AIPG-05)
10. Demonstration of hypersensitivity by skin testing
MISCELLANEOUS: (Frei's test) was widely used formerly for diagnosis
and lymhogranuloma venerum but has been given up
1. The colony of mycoplasma (smallest bacteria) has a because of frequent false positive results.
fried egg appearance. Unique among prokaryotes is
the requirement of most mycoplasmas for cholesterol
VIROLOGY
and related sterols. Mycoplasmas also lack the ability
t o synthesize purines and pyramidines.
GENERAL PROPERTIES OF VIRUSES
2. Actinomycetes are considered to be transitional
forms between bacteria and fungi. Multiple abscesses 1. The extracellular infectious virus particle is called the
with discharging sinuses, sun ray appearance and virion.
demonstrating of sulphur granules in the pus are
ch a racteristic. 2. Large viruses like pox viruses, can be seen under
the light microscope when suitably stained. The
3. On solid media A. israeli produces spidery colonies. virus particles seen in this manner are known as
"elementary bodies".
4. Mycetomas are usually caused by fungi while
Actinomycetic mycetoma or madura mycosis is caused (Also remember that "elementary bodies" are
by Actinomyces, which is a bacteria. extracellular, infective forms of chlamydiae)

5. Campylobacter is typically comma shaped but may 3. Retroviruses have a unique enzyme, RNA dependent
occur as 'S' or multispiral chains. - DNA polymerase or "transcriptase" which can
transcribe RNA into DNA.
Campylobater jejuni is medically most important, as it
causes attacks of diarrhea world wide (more common 4. Viruses are very heat labile (COMED-06). They are
than salmonelle or shigella). inactivated within seconds at 56°(, minutes at 37°C
and days at 4°C.
6. H. pylori is responsible for peptic ulcer disease and
also considered as a risk factor for gastric malignancies s. Ch lorination of drinking water kills most viruses except
such as adenocarcinoma and mucosa associated some like hepatitis, polio virus.
lymphoid tissue (MALT) syndromes.
6. Haemagg lutination and hemadsorption are shown
7. Rickettsia a re G-ve bacilli that shows obligate only by myxoviruses (influenza and parainfluenza).
intracellular parasitism. They are responsible for two Sometimes reversal of haemagglutination occurs
groups of disease i.e., typhus fever and spotted fever. due to Neuraminidase or RDE (receptor destroying
enzyme). This reversal is known as elution.
Dental ;lut.,e

7. Virus particles may be engulfed by a mechanism 15. "Viriod" is a subviral agent characterized by the
resembling phagocytosis; a process known as apparent absence of an extracellular dorma nt phase
"Viropexis". (virion) and by a genome much smaller than those of
known viruses.
8. This period during which t he virus seems to disappear
is known as the elipse phase. 16. "Prions" are infectious protein particles, without any
detectable nucleic acid, resistance to hat, UV rays.
9. A proportion of daughter virions that are produced
may not be effective due to defective assembly and 17. As most viruses are inhibited at temperatures of 39°(,
formation of incomplete viruses. These incomplete fever acts as natural defense mechanism against viral
viruses are seen in influenza infection, which shows infections.
high hemagglutinin titre but low infectivity. This is
known as Von magnus phenomenon . An exception is herpes simples which will get activated
by fever to produce "fever blisters"
10. Viruses which are genetically deficient and are
incapable of producing daughter virions are known as 18. Corticosteroids are contraindicated in most of viral
"defective viruses". infections due to the depressive effect of the immune
response and inhibitions of interferon synthesis. An
11. When a cell is infected with both active and inactive exception is post herpetic neuralgic (PH N) in which a
viruses, t he progeny possessing one or more genet ic short course of systemic corticosteroids will not only
traits of the inactivated viruses are produced. This decreases the severity of PHN but also decreases acute
phenomenon is called "Cross reactivation or marker pain and accelerates the healing of lesions (Burket's
rescue" 10th ed. 330)

12. When a cell is infected with a sing le type of virus 19. Azidothymidine is widely used drug against HIV
suffering from damage to different genes, it may be infection. It is a thymidine analogue and blocks the
possible to obtain a fu ll complement of undamaged synthesis of proviral DNA by inhibiting viral reverse
genes in form of live active virus. This phenomenon is tra nsc ri ptase.
called multiplicity reactivation.
BACTERIOPHAGES
13. The two different viruses multiply in a cell, some "mix
up" may take up during assembly, so that nucleic acid 1. Bacteriophages are viruses that infect bacteria.
of one virus is surrounded by the entire capsid of the Bacteriophages play an important role in the
other virus. This is known as Phenotypic mixing or t ransmission of genetic information between bacteria
transcapsidation. by process of transduction.

14. Based on their affinity to different systems, human 2. Phages exhibit 2 types of life cycles. If the phage
viruses are classified as: multiplies in the bacterium and causes lysis of host
Produces skin lesions bacterium and release of progeny vision, it is called
Virulent or lytic cycle.
Dermotropic E91.: Small pox, Chicken pox,
Measles
3. The infection of a bacterium by the naked phage
Affects nervous system nucleic acid is known as "transfection".
Neurotropic
E91.: Poliomyelitis, Rabies
4. The fi rst products to be synt hesized by phage DNA
Affects respiratory system
Pneu motropic are the enzymes or early proteins necessary for the
E91.: Influenza, Common cold
building of the complex molecules peculiar to that
Affects visceral organs phage.
Vi see rotro pie
E91.: Hepatitis, Yellow Fever
MICROBIOLOGY SYNOPSIS

5. Some imoortant terms· . 10. As a general rule, HSV produces "above the waist" and
Interval between the entry of phage HSV 2 "below the waist" lesions.
nucleic acid into the bacterial cell
Edi pse 11. In Tzanck test, smears are prepared form the base
and the appearance of the first
phase of lesions, and stained with 1% aqueous solution of
infectious intracellular phage
particle toluidine blue 'O" for 15 seconds.

Interval between the infection of a 12. Cytomegaloviruses are formerly known as salivary
Latent
bacterial cell and the first release of gland viruses and they are the largest viruses in the
period
infectious phage particle herpes virus family.
The period during which the number
Rise period of infectious phages released rises, is 13. Adenovirus has the appearance of a space vehicle.
known as Rise period
14. Entero virus 72 is the virus causing infectious
The average yield of progeny phages
Burst size hepatitis (Hepatitis type A), and is classified now as a
per infected bacterial cell
separate genus Hepato virus.
A lysogenic bacterium is resistant to
Super-
reinfection by the same or related 15. Earlier it was thought influenza is ca used by the
infection
phages. This is known as super bacteria Haemophilus. Later it found that haemophilus
immunity
infection immunity is only a secondary invader and influenza is caused by
When a phage is applied on the lawn virus.
culture of a susceptible bacterium, areas
Plaques An unique feature of influenza virus is its ability to
of clearing develops. These zones of
lysis are called plaques undergo antigenic variation, which may be of two
types.
6. Bacteriocins are proteins that are derived from cell
walls of bacteria producing them. They are similar to 'Antigenic drift' is gradually sequential change in
bacteriophages in that both adsorb on the surface of antigenic structure while "antigenic shift" is an
susceptible bacterial cells and kill susceptible cells of abrupt, drastic and discontinuous variation resulting
other strains without lysing them. in a nove virus.

They are considered as products of phage genomes "Zana mivir", a new drug designed to block
that are able to code only for parts of phage particles. neuraminidase is used for treatment and prevention of
influenza. It is administered as a nasa l spray.
7. Vaccinia virus is unique in that it is an "artificial
virus" and does not occur in nature. 16. Chikungunya is caused by Arboviruses and it belongs
to genus Alpha virus and Family Togaviridae. The
8. "Ceiling temperature" is the temperature above vector is Aedes aegypti (A. albopictus).
which "Pocks" are not produced in chick embryos.
17. The Rabies virus isolated from natural human or animal
9. Sub families of herpes viruses: infection is termed the street virus. The disease in
human beings (not in other animals) is called
Herpes simplex 1, 2 or varicella
Alpha virus hydrophobia because the patient exhibits the fear of
zoster virus
water, being incapable of drinking though the subject
Cytomegalo virus and Human B is intolerable to thirst.
Beta virus
cell lymphotropic virus
Epstein-Barr virus, Human herpes In Rabies, demyelination in the white matter is common.
Gamma virus In spinal cord, the posterior horns are more involved.
virus Type 8
Dental ;lut.,e

18. The three types of Neural rabies vaccines are Two types of infections of Hepatitis D are recognized;
i) Semple vaccine: It is a 5% suspension of sheep co-infection and super-infection.
brain infected with fixed virus, and inactivated
wit h Phenol at 37°C. I n coinfection both Hepatitis B& D are transmitted
together at the same time while in superinfection
ii) Beta propriolactone (BPL) vaccines: Hepatitis D virus infection occurs in a person already
It is a modification of semple vaccine in which infected with HBV.
beta propiolactone is used as the inactivating
agent instead of phenol. The major antirabic Type E Hepatitis is called as enterically transmitted.
vaccine producing laboratories in India produce Non-A and Non-8 or epidemic NANB hepatitis.
BPL vaccine.
20. If rubella occurs in very early pregnancy, the fetus may
iii) Infant brain vaccine, which is developed by using die. Congenital malformations are commonest during
infant mouse, rat or rabbit brain. the first trimester which includes cardiac defects,
cataract and deafness that constitute congenital
19. The hepatitis virus group consists of A, B, C, D, E, F & G. rubella syndrome.
Type-A Infective or infectious hepatitis
Several other features like hepatosplenomegaly,
Type-8 Serum hepatitis/ transfusion hepatitis th rombocytopenic purpura, myocarditis and bone
Non-A & Causes by uncharacterized viruses lesion constitutes "Expanded rubella syndrome".
Non-8 other than type A or type B An obvious method of protection is acquiring the
hepatitis Eg.: C, D, E, F, G viruses infection before puberty. This was achieved by "rubella
parties", practiced earlier in australs where adolescent
Type C Transfusion associated hepatitis
girls voluntarily exposed themselves to known rubella
Type Dor Depends on type B vi rus cases.
Delta virus
Transmitted by feco-oral route, and 21. Opportunistic infections and malignancies associated
Type E with HIV infection.
prevalent in developing nations on ly.
It is a variant of hepatitis B virus • Pnemocystis carinii pneumonia
Type-F and is believed to cause transfusion • Toxoplasmosis
Parasitic
associated hepatitis • Cryptosporidiosis
• Generalised strongyloidosis
Type B hepatitis is the most widespread and the
• Candidiasi s
most important type of viral hepatitis. About 1/3'd of
• Cryptococcosis
HBV carriers develop chronic hepatitis, cirrhosis and Mycotic
primary hepatic cancer. • Aspergillosis
• Histoplasmosis
As we can prevent HBV hepatitis by vaccine,
• CMV
Hepatocellular carcinoma becomes the only human Viral
cancer which is vaccine preventable. • Hepes simplex
• Kaposis sarcoma
Hepatitis 'C' virus is t he commonest cause of post- Malignancies • Hodgkin and non-hodgkin
transfusion hepatitis in developing countries. Lymphoma
• Tuberculosis
Delta virus or Hepatitis D virus is a defective RNA
virus dependent on the helper function of HBV for its • Salmonellosis
Bacteria l
replication and expression. So, it has no independent • Camphylobacter
existence and will survive on ly as long as HBVinfection • Nocardia and legionellosis
persists in the host.
MICROBIOLOGY SYNOPSIS

22. Polymerase chain reaction (PCR) is the most sensitive The predominant normal fura of the floor of nose are
and specific test and is the gold standard for diagnosis corynebacterium, streptococci and staphylocci
in all stages of HIV infection.
Mycobacteriu m smegmatis, a ha rmless co mmensal is
Western blot test is the confirmatory test employed found in the smegma of the genitalia of both men and
for diagnosis of HIV infection. In this test, proteins women.
are separated by polyacrylamide gel electrophoresis
and are blotted onto strips of nitrocellulose paper. 24. The probable number of coliforms per 100 ml. is known
These strips are reacted with t est sera and then with as most probable number of coliforms (MPN).
enzyme conjugated antihuman globulin .
25. Classification of drinking water according to
A positive reaction with proteins representing the bacteriological tests
three genes i.e., P24 (gag gene, core protein), P31 MPN per E.Coli count
(pol gene, reverse t ranscriptase) and gp41, gp120 or Class
100ml per 100ml
gp160 (env gene; surface antigens) Class I Excellent 0 0
Class II Satisfactory 1- 3 0
23. The predominant normal microflora of the conjunctie
Class III Suspicious 4 - 10 0
is diptheroids (Corynebacterium xerosis) moraxella
species and cocci organisms. Class IV > 10 0.1 or more
Unsatisfactory

Causative organisms of various diseases


Organism Disease
STREPTOCOCC US VIRIDANS GROUP • Predominant organism of plaque in healthy mouths.
(a-hemolytic) • Subacute bacterial endocarditis.
a) Streptococcus sanguis • Another group of streptococci to cause caries after S.mutans
b) Streptococcus salivarius • Forms longest chains
• First organism to invade the oral cavity after birth
c) Streptococcus mutans • Initiation of smooth surface caries
• Synthesize glucan by its action on dieta ry carbohydrates
Streptococcus pyogenes • Pharyngitis, Cellulitis, Scarlet fever, Quinsy, Ludwig's angina.
(~ - hemolytic) • Rhematic fever
• Most common ly isolated organism from root canals and dental infections
Streptococcus agalactiae • Neonatal meningitis
• Septicemia
Pnemococci (or) Streptococcus • Bacterial pnemoniae
pnemonia • Meningitis and septicemia
Staphylococci • Skin infections
- furuncle, fo lliculitis
- Stye
- Impetigo
- Carbuncle
• Infection of burns and wounds
• Osteomyelitis and sialadenitis
• Toxic shock syndrome
Dental ;lut.,e

• Cystic fibrosis
• Toxic shock syndrome
Staphylococcus aureus or
Staphylococcus pyogenes
• Acute endocarditis
• Food poisoning
• Sialadenitis
Staphylococcus albus or • Postoperative endocarditis
Staphylococcus epidermis • Stitch abscess
Lactobacillus • Progression of carious lesion
Actinomyces viscosus • Multiple abscesses with draining fistulae
Prevotella intermedia • Pregnancy gingivitis
Porphyromonas gingivitis • Generalised juvenile periodontitis
A. actinomycetam comitans • Localised juvenile periodontitis
Capnocytophaga • Pubertal gingivitis
Fusospirochaetes • ANUG (mostly by intermediate sized spirochetes)
Clostridium tetani • Tetanus, (Lock jaw).
Clostridium perfri nge ns • Gas gangrene and gastroenteritis
Clostridium botulinum • Botulism or food poisoning
Clostridium difficile • Acute pseudomembranous colitis
• Typhoid fever
Salmonella
• Osteomyelitis

Klebsiella rhinoscleromatis
• Rhino scleroma, which includes hebra nose, paralysis of soft palate with
enlargement of uvula.
Haemophilus influenza • Epiglottitis
Vibrio cholera • Cholera .
Anthrax bacilli • Anthrax disease or malignant pustule.
T. Pallidum (spirochaetes) • Syphilis

E. Coli (Gram -ve rod)


• Urinary tract infections
• Traveller's diarrhea
Chla mydea • Most common cause for non-gonococcal urethritis
(obligate intra cellular parasite) • Lymphogranuloma venerum
Actinobaci llus • Botryomycosis (bacterial disease)
Trypanosoma cruzi • South american trypanosomiasis or chaga's disease
• Benign tertian malaria
P. vivax • Fever occurs after every 3 rd day.
• Infected erythrocyte shows Schuffner's dots.
• Malignant tertian malaria or Black water fever
• Hemolytic, normochromic, normocytic anemia .
P. falcifarum • Tropical splenomegaly syndrome
• Cerebral malaria
• Infected erythrocyte shows Maurer's dots .
MICROBIOLOGY SYNOPSIS

• Quartan malaria
P. malaria • Fever occurs on every 4th day
• Infected erythrocyte shows Zeimann's dots
• Ovale tertian malaria
P. ovale
• Infected erythrocyte shows Schuffner's dots
Entamoeba histolytica • Amoebic dysentery, Amoebic abscess, Amoebic hepatitis
Trypanosoma • Trypanosomiasis (sleeping sickness)
Leishmania donovani • Kala-azar or Leishmaniasis or Oum Oum fever or Tropical splenomegaly
Schistosoma haematobium
• Schistosomiasis or Bilharziasis
(Blood fluke)
Ancylostoma duodenale
• Ancylostomiasis
(Hook worm)
Enterobius vermicularis
(Pin worm, Thread or Seat worm)
• Pruritis of perianal and perineal area

Ascaris lumbricoides
• Loeffler's syndrome

(Round worm) • Ascaris pneumonia is characterized by presence of "Charcot-Leyden


crystals" in the sputum .
• Elephentiasis
Wuchereria ba ncrofti (Fila rial worm) • Hydrocoele
• Lymphangitis
Candida albicans • Candidiasis
• Acute herpetic gingivostomatitis
Herpes simplex
• Herpes labialis
• Primary infection is chicken pox.
Varicella - Zoster (Herpes zoster) • Secondary infection may involve trigeminal nerve (post herpetic
neuralgia) or facial nerve (Geniculate neuralgia)
Pox virus (largest virus) • Small pox, Molluscum contagiosum
• 'M UMPS', which includes
- Rubbery or elastic swelling of salivary glands frequently elevating the
ear.
Paramyxo virus - Epidydimitis, Acute pancreatitis
- Orchitis and oophoritis
• MEAS LES (Rubeola)
- Presence of Koplik spots is characteristic.
• German measles (Rubella)
TOGA Virus
• Koplik spots are not seen
Coxsackie group A virus • Herpa ngi na
Coxsackie virus A10 • Acute lymphonodular pharyngitis
Coxsackie virus A16 ( PGI-2 011 ) • Hand, Foot & Mouth disease
PPLO (pleura pneumonia like • Behcet's syndrome
organisms)
Dental ;lut.,e

VARIOUS IDENTIFICATION TESTS

Name of the Test Condition


Dick test Scarlet fever
Schultz-Charlton test Diagnostic test for Scarlet fever.
Tzanck test Herpes and Pemphigus
Montoux test Tuberculosis
ASO test (Anti streptolysin-0) Acute rheumatic fever
Casoni's test Hydatid test
ELISA Screening test for AIDS
Western blot test Diagnostic test for AIDS
Northern blot test Identification of RNA
Southern blot test Identification of DNA
Paul Bunnel test Screening test for infectious mononucleosis
Monospot test Diagnostic test for infectious mononucleosis
Kveim-Siltzbach test Sarcoidosis
VDRL, KAHN Non specific tests for syphilis
Treponema pallidum immobilisation test (TPI test) Specific diagnostic test for syphilis
FTA-ABS (Fluorescent treponemal antibody test) Investigation test for syphilis.
Ito's test Chancroid
Frei's test Lymphogranuloma venerium
• Napier's aldehyde or Formol gel test
Leishmania donovani
• Chopra's antimony test
MISCELLANEOUS
Paget's test For small swellings
Rumpel leed phenomenon or tourniquet test Capillary fragility test
Rosewaller test Rheumatoid arthritis
• Rose Bengal test
• Split Lamp test Eye tests for Sjogren's syndrome
• Sc him mers test
Schillers test Carcinoma of cervix
Schilling test Diagnosis of B12 deficiency
Figlu's excretion test Felic acid absorption test
Gordon's biological test Hodgkin's disease
Coombs test For identification of Rh antigen of fetus
Ame's test Simple bacterial test for mutagenic carcinogens
Test for diagnosis of paroxysmal nocturnal
Ham's test
haemoglobinuria (PNH)
MICROBIOLOGY SYNOPSIS

VARIOUS INCLUSION BODIES

Inclusion bodies Organism


Australian antigen or Dane's particle Hepatitis 'B' virus
Babes-Ernest granules Diphtheria
Boelinger bodies (Intracytoplasmic) Fowl pox
Small pox
Gournier bodies (intra cytoplasmic)
Vaccinia (an artificially prepared virus)
Henderson - Peterson (intra cytoplasmic) Molluscum contagiosum
Prowzek bodies (intra cystoplasmic) Trachoma
Negri bodies (both intra cytoplasmic & intranuclear) Rabies virus
Cowdry type A (intranuclear) Herpes, Yellow fever
Cowdry type B (intranuclear) Poliomyelitis virus, Adeno virus
Councillmann bodies and torres bodies Yellow fever
Donovan bodies Lymphogranuloma venerium
Lipschutz bodies Herpes
Owel eye inclusions Cytomegalo virus and Herpes
Leishman-donovan bodies Kala azar
Lewis bodies Parkinsonism
Asteroid and Schaumann bodies Sarcoidosis
MISCELLANEOUS
Schoff body Rheum atic fever
Reily bodies Hurler syndrome
Civatte bodies Lichen planus
• Periapical cyst
Rushton bodies • Dentigerous cyst
• Neonatal cysts
• Multiple myeloma
Russel bodies • Solitary myeloma
• Periapical granuloma
Howell Jolly bodies Megaloblastic anemia
Psammoma bodies Papillary thyroid carcinoma
Heinz bodies G-6-P-D deficiency
Pap pen heimer bodies Sideroblastic anemia
Dohley bodies Chediak higashi syndrome
Morphologic variation of neutrophils, mostly seen in
Dohle bodies
bacterial infections.
Verocay bodies Schwanomma
Dental ;lut.,e

CLASSIFICATION OF CANDIDIASIS

Type Disease
ACUTE
1. Pseudomembran ous • Thrush
• Most commonly occurs in im munocompromised patients
2. Acute atrophic • Only type of candidiasis which is consistently painful
CHRONIC
1. Chronic hyperplastic • Leukoplakic type of candidiasis
• Considered as premaligna nt condition because of its association with oral epidermoid
carcinoma.
2. Chronic atro1Phic • Denture candidiasis or denture sore mouth
candidiasis • Angular cheilitis
• Median rhomboid glossitis

HERPES VIRUSES:
Virus Disease ~
HSV - I • Acute herpetic gingivo stomatitis
(a - virus) --
• Recurrent herpes labialis

HSV - II or Genital herpes • Genital lesions Carrcinoma of uterine


(a - herpes) cervix

• Chicken pox (primary infection)


HSV - III or Varicella zoster
(a - herpes)
• Secondary infection may involve trigeminal
--
nerve (post-herpetic neuralgia or Herpes
zoster) or facial nerve (Geniculate neuralgia)

HSV - IV or Epstei11-Barr virus • Infectious mononucleosis or Kissings disease African jaw lymphomas
(a - herpes) or Burkitt's lymphoma

HSV - V or Cytomegalo virus • Cytomegalo inclusion disease Kaposi's sarcoma


(13 - herpes)
HSV - VI (13 - herpes) • Human blood lymphotrophic virus --
HSV - VII (13 - herpes) • R. K. virus --

OTHER ONCOGENIC VIRUSES


Papovaviridae Cervical, vulvar, penile cancers,
(Human papilloma virus) squamous cell carcinoma
Hepadna viridae Hepatocellular carcinoma
(Hepatitis B virus)
Retro viridae (HTL virus) Adult T-cell leukemia
MICROBIOLOGY SYNOPSIS

INCUBATION PERIODS: Ps. aeruginosa is the most important cause of infection


Cholera Hours to 5 days in burns.

• Diphtheria
2 to 5 days 3. Delayed wound healing or stitch abscess is caused by
• Gonorrhea staph. epidermidis. Staph. epidermidis bacteraemia is
Scarlet fever 1 to 3 days seen commonly in patients with artificial heart valves.
Measles 7 to 14 days
4. Precipitation reactions are due to soluble antigen,
• Chicken pox while the agglutination reactions are due to
• Mumps 14 to 21 days particulate antigen.
• Rubella
Typhoid 5 to 21 days 5. In precipitation reaction, if the amounts of precipitate
in the different tubes are plotted on a graph, the
Tuberculosis Months to years
resulting curve will have three phases:
Hepatitis A 2 to 6 weeks (4 weeks) • An ascending part (prozone or zone of antibody
Hepatitis B 6 weeks to 6 months (12 weeks) excess).
• A peak zone (zone of equivalence)
IN FE CTI VE STAGES OF SOME PARASITES • Descending part (postzone or zone of antigen
Parasite Infection form excess)

Entamoeba histolytica Mature tetranucleate cyst This is called zone phenomenon.


Plasm odium Sporozoite
6. Histocompatability antigens are cell surface
Taenia Cysticercus and proglattid
antigens that induce an immune response leading to
Echinococcus Hexacanth rejection of allografts. The H- 2 antigen is found to be
granulosus (Embryonated egg) major histocompatability antigen coded for a closely
Schistosoma Cercaria larvae linked multiallelic cluster of genes called the Major
Histocompatability Complex (MHC).
Ascaris lumbricoides Embryonated egg,
Rhabditiform larva All animals possess a similar complex of genes on a
Ankylostoma duodena le Fila rifo rm larva segment of one chromosome pair, coding for three
Filarial worms Micro filarial, 3rd stage different classes of proteins.
i) Class I proteins that determine histo-compatability
Enterobius vermicularis Ova and the acceptance or rejection of allografts.
Leishmania Promastigote ii) Class II proteins that regulate the immune
response.
MISCELLANEOUS iii) Class III proteins that include some components
of the complement system and few others
1. Strep. pyogens is, perhaps, the most important cause
of hospital infection. 7. Human MHC antigens are synonymous with human
leucocyte antigens (H LA). An association has been
2. Ps. aeruginosa and other pseudomonas species observed between HLA types and certain diseases.
carry drug resistant plasmids and are considered as • HLA - B27 - Ankylosing spondylytis
important causes of hospital infection because of their (MAHE previous MCQ)
intrinsic resistance to most antibiotics and ability to • HLA - DR4 - Rheumatoid arthritis
survive and even multiply at low temperatures and in
• HLA - DR3 - Many autoimmune conditions.
disinfectant solutions.
Dental ;lut.,e

8. Thymic Hypoplasia or Digeorge's syndrome is a disease 18. Dermatophytes (Tinea or ring worm) are examples
of cellular immunodeficiency due to a developmenta l of pathogenic moulds that infect only superficial
defect involving y d & 4th endodermal pharyngeal keratinized tissues - the skin, hair and nails. T-rubrum
pouches. It leads to aplasia or hypoplasia of the is the most common species infecting human beings.
thymus and parat hyroid glands.
"Favus" is a chronic type of ring worm in which dense
9. Wiskott - Aldrich syndrome is a x-linked disease crusts develop in hair fo llicus that lead to alopecia
(previous AIPG MCQ) and earring.

10. Adenosine deamine (ADA) deficiency is the first 19. Taenia nigra is a localized infection of the stratum
immunodeficiency disease associated with an enzyme corneum caused by cladosporium Wernickii.
deficiency (Previous AIPG MCQ). ADA catalyses the
conversion of adenosine to inosine, an important step 20. " Ectothrix" is a hair infection in which arthrospores
in purine metabolic pathway. are seen as a sheath surrounding the hair and in
"endothrix", the spores are inside the hair shaft.
11. Terminology of grafts:
Name Donor 21. Mycetoma (also known as maduramycosis or madura
foot) is chronic infection of subcutaneous tissue,
Auto Graft/ Autogenous usually of foot and rarely other parts of body.
Self
graft
Iso / Syngenic / Syngraft Identical twin 22. "Asteroid" bodies are seen in sporotrichosis
Genetically unrelated
Allograft / Homograft 23. The fungal infection associated with uncontrolled
member of same species
diabetes is phycomycosis or mucormycosis.
Xenograft /heterograft Different species

12. The sequence of events resulting in the rejection of


allograft is known as the "first set response".

If another graft from same donor is applied, in the


same animal that has rejected a graft by the first set
response, it will be rejected in an accelerated fashion.
This accelerat ed allograft rejection is known as the
"second set response".

13. The method of transferring immunity by means of


lymphoid cells is known as adoptive immunization.

14. The simplest type of fungus is the unicellular budding


yeast. The on ly pathogenic yeast is Cryptococcus
neoformans.

15. Candide albicans is yeast-like fungus.

16. Dimorphic fungi can occur as filaments or as yeasts.


Most fungi causing systemic infections are dimorphic
fungi.

17. The periodic acid Schiff (PAS) and methamine silver


stains helps for staining of fungal elements.
PERIODONTICS

PERIODONTICS
I. REFERENCE BOOKS TAKEN:
1. Carranza's Clinical Periodontology by Newman -11 th ed Volume - I (Pg 1 - 475)

2. Carranza's Clinical Periodontology by Newman -11 1h ed Volume - II (Pg 476 -1371)

3. Carranza's Clinical Periodontology by Newman - 101h & 11th Editions

4. Essentials of Community Dentistry by Soben Peter - 4th & 5th Editions

1. GINGIVA & TOOTH SUPPORTING STRUCTURES


1. Stippling is seen on d) None of the above
a) Marginal gingiva b) Attached gingiva (AIIMS - 89)
c) Interdental gingiva d) None of the above 7. Stippling is absent in:
(MAN -02, AIPG 95) a) Infants only b) Older men
2. With the increase in age keratinisation of gingiva c) Children d) Both infants and old
a) Increases b) Decreases (AIPG - 97)
c) Remains the same d) Increases and then decreases 8. Junctional epithelium is attached to enamel by:
(MAN - 02) a) Reticular fibers b) Desmosomes
3. The colour of gingiva is due to c) Hemidesmosomes d) Collagen
a) Capillaries b) Thickness of epit helium (AIPG -97)
c) Thickness of keratinisation and pigmentation 9. Which type of cells are most numerous in gingiva
d) ALL the above a) Melanocytes b) Keratinocytes
(MAN - 99, KAR - 01) c) Giant cells
4. Junctional epithelium: d) Polymorphonuclear Leukocyt,es
a) Lines the gingival sulcus (AIPG - 96)
b) Is permeable 10. Gingival Col is:
c) Is derived initially from the outer enamel epithelium a) Orthokeratinized b) Para keratinized
d) Is attached to enamel by desmosomes c) Both para and ortho keratinized
(AIIMS -93, MAN - 94) d) Non-keratinized
5. Dentogingival unit comprises of (KAR - 99)
a) Gingival fibres 11, Gingiva is supplied by
b) Gingival fibres and junctional epithelium a) Supraperiosteal vessels b) Subperiosteal vessels
c) P.D. fibres and Ligament c) Apical vessels d) All of the above
d) None of the above (AIPG -2K)
(AP - 99, AP - 03) 12. The junctional epithelium is attached to the tooth by:
6. Free gingival groove represents a) Basal lamina b) Desmosomes
a) Histologic depth of gingival sulcus c) Enamel cuticle d) Nasmyth's membrane
b) False pocket depth (AIPG -99)
c) Depth of gingival sulcus

1) B 2) B 3) D 4) B 5) B 6) A 7) D 8) C 9) B 10) D 11) A 12) A


Dental ;lut.,e i======
13. Gingiva is attached by: 25. Which periodontal fibres are consistent and are
a) Junctional epithelium reconstructed even after the destruction of the alveolar
b) Epithelial attachment bone?
c) Reduced enamel epithelium a) Apical b) Alveolar crest group
d) None of t he above c) Oblique d) Transseptal
(AIPG -93) (AIIMS -93, MAN -94)
14. The length of junctional epithelium is: 26. In periodontal Ligament there is:
a) 0.25 - 1.35 mm b) 0.5 - .75 mm a) Type I collagen b) Type II collagen
c) 1.0 - 2.0 mm d) 1.0 - 1.5 mm c) Type III collagen d) Type IV collagen
(PGI - 99, COMEDK- 06, MHCET-15) (AIPG - 00)
15. Least width of attached gingiva is found on the facial 27. Periodontal ligament is
aspect of a) Narrower on mesial surface
a) Jst premolar b) 2•d premolar b) Wider on mesial surface
c) Lateral incisor d) Canine c) Equal on both the surfaces
(PGI - 01) d) Wider in the anterior teeth
16. The area, which is most prone to infection (AIPG - 00)
a) Marginal gingiva b) Sulcular epithelium 28. The periodontal ligament
c) Interdental col d) Attached gingiva a) Derives its blood supply primarily from t he branches of
(PGI -02) vessels entering the pulp
17. The "V" shaped space that encircles the tooth and b) Has a slow rate of turnover
present between tooth and gingiva is known as: c) Contains epithelial cells of malassez
a) Inter dental papilla b) Free gingiva d) Comprises primarily of type 2 collagen
c) Attached gingiva d) Gingival sulcus (AIIMS -93, AIPG -90)
(KAR -02) 29. PDL is thinnest at:
18. Normal consistency of the gingiva is: a) CEJ b) Apex
a) Soft b) Hard c) Middle d) None of the above
c) Firm d) Firm and Leathery (PGI - 99)
(KAR -02) 30. What type of fibers are principal fibers of the PDL?
19. After the tooth emerges to the occlusal plane the clinical a) E~stic b) Reticu~r
crown height increases by: c) Collagenous d) Collagenous and elastic
a) Passive eruption (AIPG -03, AIIMS -03)
b) Active eruption 31. Periodontal fibers which are most resistant to forces
c) Post-emergence growth spurt along the long axis are:
d) Juvenile occlusal equilibrium a) Apical b) Oblique
(AIPG -01) c) Transept al d) Alveolar crest
20. Gingiva in children (AIPG - 92, 98)
a) More keratinised more stippled 32. The thixotropic theory claims that:
b) More keratinised Less stippled a) The principle fibres of the periodontal ligament has
c) Less keratinised more stippled the major responsibility in supporting the tooth and
d) Less kerati ni sed Less stippled transmitting forces to the bone
( PGI -99) b) The periodontal ligament has the rheologic behaviour of
21. Normal depth of gingival Sulcus in adult does not exceed a thixotropic gel.
a) 0.5 - 1mm b) 2 -3 mm c) The displacement of the tooth is Largely controlled by
c) 1-2 mm d) 1 - 3 mm fluid movements.
(PGI - 03, AIPG-05) d) None of the above
22. Eulanin fibers are found in (KAR -97)
a) Givigiva b) Cement um 33. Alveolar bone is:
c) Alveolar d) Periodontal ligament a) Compact bone b) Cancellous bone
(COMEDK-07) c) Spongy bone d) None of the above
23. The function of Langerhan's cells is (KAR -98)
a) Melanin synthesis b) Protein synthesis 34. Anatomic form of roots of teeth is determined by
c) Antigen presentation to neutrophils a) Dental Lamina b) Neural crest cells
d) Antigen presentation to lymphocytes c) Hertwig's root sheath d) Hammock Ligament
(KCET-2012) (AIPG - 92)
24. Langerhan's cells are absent in 35. Gingival sulcus is V shaped and it barely permits the
a) Junctional epithelium b) Sulcular epithelium entrance of a
c) Oral epithelium d) ALL of the above a) Periodontal hoe b) Periodontal curette
(COMEDK -04)

13} A 14} A 15} A 16} C 17} D 18} C 19} A 20} D 21) B 22) D 23} D 24} A 25} D
26) A 27) A 28) C 29) C 30) C 31) B 32) B 33} A 34} C 35} C
, PERIODONTICS
~

V
521

c) Periodontal probe d) Periodontal sickle c) Langerhan's Cell d) Grey Cell


(BHU-2012) (COMEDK-05)
36. Which of the following are the predominant connective 48. Cementum found on the cervical third of the root
tissue cells of the periodontal ligament? a) Acellular afibrillar cementum
a) Cementoblasts b) Fibroblasts b) Acellular extrinsic fiber cementum
c) Osteoblasts d) Rests of Malassez c) Cellular mixed stratified cementum
(AIIMS -93) d) Cellular intrinsic fiber cementum
3 7. "Indifferent fibres" are (COMEDK-05)
a) Elastic fibres b) Oxytalan fibres 49. The cell that is present in stratum spinosum and stratum
c) Collagen fibres d) None of the above basale is:
(AIPG-2011) a) Basket cell b) Melanocytes
38. With aging changes in periodontal ligament are c) Keratinocyt es d) Merkel cells
a) Decrease in number of cells and increase in collagen fibers (AIPG-07)
b) Increase in number of cells and decrease in collagen fibers 50. Periodontal ligament is thinnest at
c) Increase in number of elastic fibers a) Apex b) Coronal
d) Hyalinisation changes c) Crestal d) Fulcrum of axis of rotation
(AIIMS -2K) (AP-07)
39. Functions of periodontal ligament 51. The inter radicular bone is:
a) Nutritive b) Sensory a) Compact b) Cancellous
c) Formative d) All c) Exophytic d) Osteophytic
(AP -02) (AIIMS-06)
40. Feature of aging pe riodontium is 52. Biological width means
a) Lacunae in bone and cementum a) Sum of connective tissue and junctional epithelium
b) Increased cell size b) Distance from t he tip of t he papilla to t he base of the sulcus
c) Increased cell number c) Distance from marginal gingiva t o muco-gingivaljunction
d) Scalloping of cementum & alveolar bone surface d) Distance from incisal edge of the tooth to muco-gingival
(AIPG-10) junction
41. Unattached gingiva: (KCET-08)
a) Is interdental gingiva b) Is below mucogingival fold 53. Reticular fibers found in periodontal ligament consist
c) Cannot be separated by probe of-
d) Is marginal gingiva a) Type I b) Type III
(AIIMS-06) c) Type IV d) Type XII
42. The oxygen consumption of normal gingiva is: (KCET-09)
a) 1.6 ± 0.3 b) 0.9 ± 0.22 54. Collagen fibers are absent in:
c) 2.7 ± 0.41 d) 1.9 ± 0.21 a) Acellular extrinsic fibre cementum
(AIPG-05) b) Acellular afibrillar cementum
43. A black line on the gingiva which follows the contour of c) Mixed stratified cementum
the margin is due to: d) Intermediate cement um
a) Lead b) Argyria (MCET-07)
c) Iron d) Mercury 55. Which is true of alveolar bone
(AIPG-05) a) Most stable of the periodont al tissues
44. Basal Lamina consists of b) It is in a const ant stat e of flux
a) Type I collagen fibres b) Type II collagen fibres c) It is not regulated by systemic and local influences
c) Type IV collagen fi bres d) Type III collagen fibres d) All of t he above
(KAR-04) (AP-08)
45. Dendritic cells Located among keratinocytes at all 56. The gingiva is attached to the tooth by:
suprabasal levels and belonging to reticuloen dothelial a) Lamina propria b) Periosteum
system derived from bone marrow are c) Epithelial attachment d) Gingival fi bers
a) Melanocytes b) Langerhans cells (AP-08)
c) Lymphocytes d) Merkel cell 57. Stippling of gingiva indicates all, EXCEPT:
(KAR-04) a) Surface keratinisation b) Healthy gingiva
46. Hastin fiber system present in the lamina propria c) Adaptive specialization for reinforcement of function
consists all of these EXCEPT- d) None of the above
a) Oxytalan b) Elastin (MCET-07)
c) Eluanin d) Collagen 58. Lamina densa of the basal lamina beneath the epithelium
(KCET-09) is composed of
47. Which of the following is Non-keratinocyte? a) Type I Collagen b) Type II Collagen
a) Langan's Cell b) Red Blood Cell

36) B 37) C 38) C 39) D 40) D 41) D 42) A 43) D 44) C 45) B 46) D 47) C 48) A&B
49) B 50) D 51) B 52) A 53) B 54) B 55) B 56) D 57) D 58) D
Dental ;lut.,e i======
c) Type III Collagen d) Type IV Collagen 70. Keratin specific to epidermal type differentiation are
(UPSC-09) a) Kl, K2, KlO, K12 b) Kl, K3, KlO, K12
59. Stratification specific keratins present in gingival c) Kl, K2, KlO, K13 d) Kl, K2, Kll, KB
epithelium are (MCET-14)
a) Kl, K2 b) KlO, K12 71. Which of the following fibre groups are NOT attached to
c) K16, K6 d) K5, K14 alveolar bone
(KCET-10) a) Transeptal b) Oblique
60. The gingival fibre group which inserts in two adjacent c) Horizontal d) Dentoperiosteal
teeth is the? (AP-14)
a) Horizontal group b) Circular group 72. The normal gingival color is
c) Trans-septal group d) Oblique group a) Red b) Yellow
(AIPG-09) c) Coral pink d) Whitish gray
61. An isolated area in which root is denuded of bone is (AP-14)
intact and covered only by periosteum and overlying 73. The occurrence of keratinization on the human gingiva is
gingiva is called a) Dependent upon the functional stimulation the attached
a) Fenestration b) Dehiscence gingival receives
c) Infracrestal pocket d) Hemiseptum defect b) A reflection of inflammation in the underlying gingival
(AIIMS-09) connective tissue
62. The type of bone present in the inter-radicular area is c) A variable factor directly related to age
a) Cortical b) Cancellous d) Lacking in the gingival crevice
c) Osteophytic d) Exophytic (PGI JUNE-2014)
(AIPG-09, AIIMS MAY 2012) 74. The main component of keratins in the stratum corneum
63. One of the following group of fibers in the lamina propria is
of the gingiva, help to bind the free gingiva to the tooth a) Keratin polypeptide (68kD)
a) Alveologingival b) Circular b) K19 Keratin polypeptide (40k0 )
c) Dentoperiosteal d) Dentogingival c) K12 Keratin polypeptide (40k0 )
(COMED-10) d) Kl keratin polypeptide (68kD)
64. The periodontium does not include: (APPG-15)
a) Cementum b) Dentin
c) Periodontal ligament d) Bone lining the socket
(IGNOU-10)
65. Bioactive glass (Perio glass) bone graft material is a
type of:
a) Autograft material b) Allograft material
c) Xenograft material d) Non bone graft material
(IGNOU-10)
66. Gingiva is attached to enamel by
a) Hemidesmosomes b) Desmosomes
c) Tight J unction d) Gap Junction
(PGl-08)
67. The sulcular epithelium acts as a semi permeable
membrane through which
a) Bacterial products pass in to the gingiva
b) Fluids from the gingiva seeps in to the sulcus
c) Both of the above d) None of the above
(AIIMS-08)
68. Pick the correct statement. Collagen is
a) the major protein in gums
b) a protein with a high turnover rate
c) rich in basic amino acids
d) rich in essential amino acids
(AP-09)
69. Osteoblast covering the periodontal surface of the
alveolar bone constitute a
a) Modified periosteum b) modified endosteum
c) periosteum d) endosteum
(MCET-14)

59) D 60) C 61) A 62) B 63) B 64) B 65) D 66) A 67) C 68) B 69) B 70) A 71) A
72) C 73) D 74) D
, PERIODONTICS
~

V
523

1. GINGIVA & TOOTH SUPPORTING STRUCTURES - ANSWERS


1. ' B' [Carranza 11th ed 26 / 10th ed 62) • Dendritic cells located at supra basal
Stippling is seen in attached gingiva and central core of layers. Considered as acrophages with
interdental papilla. It is absent in marginal gingiva. Langerhans possible antigenic property.
cells
Stippling is a form of adaptive specialization or • Absent in the junctional epithelium
reinforcement for function. It is produced by alternate of normal gingiva.
rounded protuberances and depressions in the gingival Located in deep layers of epithelium and
surface. Stippling is a feature of healthy gingiva. Loss of Merkel cells harbour nerve endings. They serve as
stippling is reversible and is a common sign of gingival tactile receptors (PGl-13).
disease.
Amongst the non-keratinocytes of oral epithelium
Stippling is absent in infancy and in old aged people. It the non-dentritic tactile sensory cells present in the
appears in children about the age of 5 years. basal cell layer (MHCET-15)
a) Melanocytes b) Langerhan cells
2. ' B' [Carranza 11th ed 18 / 101h ed 52) c) Lymphocytes d) Merkel cells
With the increase in age:
• Keratinisation of gingiva decreases. 10. ' D' [Carranza 11th ed 14 / 10th ed 47]
• Width of the attached gingiva increases. Interdental col is area of periodontium more susceptible
to tissue breakdown because:
• Number of elastic fibres in the periodontal ligament
increases. • It is non-keratinised. (AIIMS MAY 2012)
• Number of collagen fibres decreases. • It is the site where oral hygiene accessibility is not
possible.
The decreasing order of keratinisation in different areas is • It is the area of food entrapment.
palate (most keratinised), gingiva, tongue, cheek (least
keratinised) Junctional and sulcular epithelium are also non-keratinised.

3. ' D' [Carranza 111h ed 25 / 10th ed 61) 11. 'A' [Carranza 111h ed 23 / 10th ed 58]
The 3 sources of blood supply to the gingiva are:
4. ' B' • Supraperiosteal arterioles.
' B' [Carranza 11th ed 20 / 10th ed 54) • Vessels of periodontal ligament, and
5.
The attachment of the junctional epithelium to the tooth • Arterioles that emerge from the crest of i nterdental septa.
is reinforced by the gingival fibres therefore the junctional
epithelium and the gingival fibres are together considered 12. 'A' [Carranza 111h ed 20 / 10th ed 54]
as the dento gingival unit. The junctional epithelium is attached to the tooth by means
of an internal basal lamina and to the gingival connective
6. 'A' [Carranza 11th ed 13 / 10th ed 46) tissue by means of an external basal lamina.

Histologic depth of the gingival sulcus 1.8 mm The internal basal lamina contai ns lamina densa and lamina
Probing depth of t he normal gingival sulcus 2 - 3mm lucida to which hemidesmosomes are attached.

7. ' D' [Carranza 11th ed 26 / 10th ed 63] 13. 'A' [Carranza 11th ed 20 / 10th ed 54]

8. 'C' [Carranza 111h ed 20 / 10th ed 54] 14. 'A' (Carranza 11•h ed 20 / 10•• ed 53]
Junctional epithelium is attached to the tooth surface by Junctional epithelium is a collar like band of non keratinising
means of hemidesmosomes. Keratinocytes are connected to epithelium. It is 3-4 layers thick in early life and increases
one another by option 'B' i.e, desmosomes. to 10-20 layers with age. Its length ranges from 0.25 - 1.35
mm.
9. ' B' [Carranza 11th ed 15 / 10th ed 49)
Keratinocyte is the principal cell of gingival epithelium. 15. 'A' [Carranza 11 1h ed 13 / tQth ed 47]
Melanocytes, langerhans and merkel cells are the non- i) Width of attached gingiva is greatest in the incisor
kerati nocytes. region.
• Maxilla - 3.5-4.5 mm
Dendritic cells located in the basal and • Mandible - 3.3-3.9 mm
Melanocytes spinous layers of gingival epithelium. They
synthesize melanin in melanosomes.
Dental ;lut.,e i======
ii) Least width in the first premolar region Birbeck granules are seen in (APPG-15)
• Maxilla - 1.9 mm a) Merkels cells b) Langerhans cells
• Mandible - 1.8 mm c) Lymphocytes d) Monocytes

24. 'A' (Carranza 11th ed 17 / tOth ed 52)


iii) The width of attached gingiva increases with age and in
supra erupted teeth.
25. ' D' (Carranza 11th ed 29 / 10th ed 70)
Significance of the transeptal fibres
16. ' C' (Carranza 11th ed 13 / 10th ed 53)
• Considered as both gingival and periodontal ligament fibres
17. ' D' (Carranza 11th ed 13 / 10th ed 46) • They are remarkably constant finding and are reconstructed
even after t he destruction of the alveolar bone.
18. 'C' [Carranza 11th ed 26 / 10th ed 62] • They form a dense firm covering over the bone, which is
The consistency of gingiva is firm and resilient. It is due seen after the removal of granulation tissue (AP-2004)
to its collagenous nature and its contiguity with the during the pocket eradication procedures.
mucoperiosteum of alveolar bone.
26. 'A' (Carranza 11th ed 28 / tOth ed 69)
19. 'A' (Carranza 11th ed 27 / tOth ed 64) The most important elements of the periodontal ligament
Active eruption Movement of teeth in a occlusal direction. are the principal fibres. They are composed mainly of type I
The exposure of teeth by apical collagen.
Passive eruption
migration of the gingiva.
Type III collagen is present in reticular fibres and Type IV
20. 'D' (Carranza 11th ed 18 /10th ed 52, 63) collagen is seen in basal lamina.
Summary of periodontium characteristics in children :
Type one Bone, GINGIVAN, principal fibers
a)Gingiva collagen of periodontal ligament.
• Less stippled Type two
CARtwoLAGE
• redder than adults due to increased vascularity collagen
• Interdental col formation and saddle areas Type III Reticular fibers, Granulation tissue, Circum
• Thinner, less keratinized epithelium collagen maxillary suture (PGI JUNE- 2011)
• Affinity for melanin pigmentation "Floor on the four". Basement membrane or
floor is supplied by type four collagen
• Attached Gingiva is narrower in primary dentition and
(Note:- The syndrome associated with a
narrower in the mandible Type four
defect in type IV collagen in basement
• Interdental papillae is shorter and rounder collagen
membrane of renal glomeruli is Al port
syndrome. The patient exhibit hematuria and
b) PDL & Bone renal diseases)
• Wider periodontal membrane space
• Fewer and less dense periodontal fibers 27. 'A' (Carranza 11th ed 33 / 10th ed 73)
Narrow Periodontal Ligament Is Seen In
• Less dense and differentiated collagen fibers
• The mesial surface of teeth due to physiological mesial
• Increased blood and lymphatic supply
migration.
• Thinner lamina dura
• At the fulcrum of rotation of the tooth.
• Flatter alveolar crests
• Teeth that are in hypo function i.e., teeth in unilateral
• Fewer trabeculae chewing habits, open bite and whose antagonist has
been lost.
21. ' B' (Carranza 11th ed 13 / t0th ed 47]
Wide periodontal ligament is seen in:
22. 'D' (Carranza 11th ed 30 / 10th ed 57)
• Trauma from occlusion.
23. ' D' (Carranza 11th ed 17 / 10th ed 52) • Scleroderma.
Langerhan's cells are modified monocytes derived from bone • Osteosarcoma.
marrow located at su prabasal level. They acts as antigen
presenting cells for lymphocytes. They contain g-specific 28. 'C' (Carranza 11th ed 30 / 10th ed 71)
granules called birbeck's granules. Epithelial rests of malassez are the derivatives of the hertwigs
epithelial root sheath, which may become cementicles or
participate in the formation of periapical cysts and lateral
root cysts.
, PERIODONTICS
,......__,,_

V
525

P.L. derives its blood supply from apical vessels before • Option B is Thixotropic theory.
entering the pulp, anastomosing vessels of the gingiva and • Option C is Viscoelastic theory.
transalveolar branches of the intraseptal vessels. The vessels
in the P.L. form a net like plexus, which is closer to the 33. 'A' [Carranza 11th ed 37 / 1Qth ed 79]
bone than to the cement um. The alveolar process consists of:
• an external plate of compact bone.
P.L. has high turnover rate of collagen. The rapid turnover
rate of cells occurs more towards t he side of bone. • inner socket wall of thin compact bone called 'alveolar
bone proper'.
29. 'C' [Carranza 11th ed 33 / 10th ed 73] • Cancellous or spongy trabecular bone, in between these
The P.L. is narrowest at the axis of rotation. two compact bones. It acts as a supporting bone.

No. of roots Location of axis of rotation So the alveolar bone is predominantly compact bone.
Single rooted Junction of middle and apical third of the root
34. 'C' [Carranza 11th ed 42 / 10'h ed 85]
Multi rooted In the bone between the roots.
35. 'C' [Carranza 11th ed 13 / 10th ed 12]
30. 'C' [Carranza 11th ed 28 / 10th ed 68] Gingival sulcus is a shallow 'V' shaped crevice around the
Periodontal ligament consists of principal fibres and elastic tooth bounded by t he surface of t he tooth on one side and the
fibres. Principal fibres are made up of collagen type I. They epithelium lining the free margin of the gingiva on the other
are purely collagenous. side. The clinical evaluation used to determine the depth of
the gingival sulcus involves the introduction of the periodontal
The elastic fibres are present in two immature forms called probe and t he estimation of t he distance it penetrates.
oxytalan and elaunin (AIIMS-07) . Oxytalan fibres run
parallel to the root surface and are attached t o the blood 36. ' B' [Carranza 11th ed 30 / 10th ed 71]
vessels thus they regulate the vascular flow. Fibroblasts perform the dual role of synthesizing new
collagen and degrading old collagen.
PDL does not have? (AIIMS Nov-14)
a) Mature elastic fibers 37. 'C' [Carranza 11th ed 30 / 10th ed 71]
b) Angioblasts and undifferentiated mesenchymal cells Indifferent fiber plexus are small collagen fibers, which are
c) Myelinated nerve fi bers d) Fibroblasts associated with the larger principal collagen fibers, running
in all directions forming a plexus.
31. ' B' [Carranza 11th ed 28 / 10th ed 70]
The principal fibres of P.L. are arranged in 6 groups. 38. 'C' [Carranza 11th ed 53 / 10th ed 95]
Wit h aging increase in elastic fibres occurs whereas the number
• Runs between adjoining teeth and gives of fibrob lasts and collagen fibres are decreased with aging.
Transseptal interproximal support.
fibres • This fiber group is also called as
The number of fibroblasts in the periodontal ligament
interdental ligament .
_ _ _ _ _ _ _ _ _ _ with age
• These run from cementum in a apical a) Increases b) Decreases
Alveolar crest direction to the alveolar crest. c) Remains same d) May increase or decrease
fibres • They prevent extrusion of the tooth (COMED-14)
and resist lateral tooth movements.
• These run from cementum to alveolar 39. ' D' [Carranza 11th ed 31 / 1Qth ed 72]
bone in coronal direction. They are
Oblique fib res 40. 'D' [Carranza 11th ed 52 / 1Q<h ed 93-96]
highest in number, suspend t he tooth and
resist vertical masticatory stresses. EFFECTS OF AGING ON THE PERIODONTIUM:
Horizontal • Extend at right angles from cementum A) Gingival epithelium:
fibres t o alveolar bone. • Thinning and decreased keratinization
• Increase in epit helial permeability
• They arise from root ends and are
absent in incompletely formed roots. • Flattening of ret epegs
Apical fibres
• Migration of junctional epithelium to a more apical
• They resist rotation of the tooth.
position on the root surface
• They fan out from the cementum to • Width of attached gingiva increases
Interradicular
the tooth in the furcation areas of
fibres
multirooted teeth.
B) Connective tissue:
32. 'B' [Carranza 11•h ed 31] • Coarser and denser gingival connective tissues
• Option A is Tensional theory. • Collagen content increases
• Rate of collagen synthesis decreases
Dental ;lut.,e i======
C) Periodontal ligament: 49. 'B' [Carranza 11th ed 16 / 10th ed 51]
• Number of fibroblasts decreases
50. 'D' [Carranza 101hed 235]
• Decreased organic matrix production and epithelial
cell rests
51. 'B' (Carranza 11th ed 41 / 101hed 80]

D) Cementum: Facial and lingual


Compact Bone
portions of sockets
• Increase in cementa[ width 5-10 times
Alveolar bone proper Thin compact bone
• Increase in width is greater apically and lingually
Cancellous bone enclosed
• Accumulation of resorption bays explains the finding Interdental septum
within a compact border
of increasing surface irregularity
Interradicu lar bone Cancellous bone
E) Alveolar bone:
52. 'A' [Carranza 11th ed 989 / 10th ed 1044]
• More irregular periodontal surface of bone and less Biological width is defined as the physiologic dimension of
regular insertion of collagen fibers the functional epithelium and connective tissue attachment.
This has been found to be approximately 2mm (± 30%).
41. 'D' [Carranza 11th ed 12 / 10th ed 46]
It has been theorized that infringement on the biological
42. 'A' width by the placement of a restoration within its zone may
result in gingival inflammation, pocket formation and loss of
alveolar bone. Consequently, it is recommended there should
43. 'D' [Carranza 8 1h ed 227]
be atleast 3.0 mm between gingival margin and bone crest.
Bismuth, arsenic, and mercury produce a black line in This allows for adequate biologic width when the restoration
the gingiva that follows the contour of the margin. Oral is placed 0.5mm within the gingival sulcus.
manifestation of lead poisoning consisting of a bluish-black
line following the contours of the marginal gingiva. 53. 'B' (Carranza 11th ed 28 / 101h ed 69]

44. 'C' [Carranza 11th ed 17 / 10th ed 52] 54. 'B' [Carranza 11th ed 33 / 10th ed 75]
1
Acellular afibrillar cement um contains neither cells nor
45. 'B' (Carranza 11th ed 16 / 10 hed 51] extrinsic or intrinsic collagen fibers, except for a mineralized
For explanation refer to Q.No. 9 ground substance. AAC is formed by cementoblasts and is
found as coronal cementum in humans with a thickness of 1
46. 'D' [Carranza 11th ed 30 / 101h ed 57] to 15µm.
The elastic fiber system is composed of oxytalan, elaunin
and elastin fibres. 55. 'B'

47. 'C' [Carranza 11th ed 17 / 101hed 51] 56. 'D' [Carranza 11th ed 22 / 10th ed 54]
Langerhan's cell is a non keratinocyte seen in epithelium of
mucous membranes. 57. 'D' [Carranza 11th ed 26 / 10th ed 63]
Stippling is produced by alternate rounded protuberances
Langhans cell is a multinuclear giant cell seen in chronic and depressions in the gingival surface. It is seen in
inflammatory diseases like Tuberculosis. attached gingiva and central core of interdental papilla.
It is a form of adaptive specialization or reinforcement for
48. 'A' & 'B' [Carranza 11th ed 33 / 10th ed 75] function . It is a feature of healthy gingiva, and reduction
Schroeder classification of cementum: or loss of stippling is a common sign of gingival disease.
• Acellular afibrillar cementum (AAC) - Conains neither When the gingiva is restored to health after treatment, the
cells nor extrinsic or intrinsic collagen fibres, except stippled appearance returns.
for a mineralized ground substance. Found in coronal
cementum. 58. 'D' (Carranza 11th ed 20 / 101h ed 52]
• Acellular extrinsic fiber cementum (AEFC) - Contains The epithelium is joined to underlying connective tissue
densely packed bundles of Sharpey's fibres and lacks by basal Lamina. It consists of lamina lucida and lamina
cells. Found in cervical third of roots. densa. Hemidesmosomes of the basal epithelial cells abut
• Cellular mixed stratified cementum (CMS() - Composed the lamina lucida. The lamina densa is composed of type IV
of extrinsic and intrinsic fibers and may contain cells. collagen.
Found primarily in apical third of roots and apices and
in furcation areas. The hemidesmosomes of the basal epithelial cells of
• Cellular intrinsic fiber cementum (CIFC) - Contains the gingival epithelium abut the... (MHCET-15)
cells but not extrinsic fibers. It fi lls the resorption a) Lamina propria b) Lamina lucida
lacunae. c) Lamina densa d) Lamina dura
, PERIODONTICS
,......__,,_

V
527

59. 'D' [Carranza 11th ed 20 / 10'h ed 52) 72. 'C' [Carranza 11th ed 25)
k1,k2,k10-k12 Epidermal type differentiation The coral pink color of gingiva is produced by the vascular
supply, the thickness & degree of keratinization of the
Characteristic of highly proliferative epithelium and the presence of pigment containing cells.
k6, k16
epithelia
k5, k14 Stratification specific cytokeratines 73. ' D' [Carranza 11th ed 17)
Present in parakeratinized only areas The sulcular epithelium lines the gingival sulcus. It is
k19 the thin, non-keratinized stratified squamous epithelium
& absent in orthokeratinized areas
without rate pegs. Keratinization is inversely proportional
60. 'C' [Carranza 11th ed 29 / 101h ed 57) to age and inflammation.

61. 'A' [Carranza 11th ed 42 / 1Q1h ed 84) 74. ' D' [Carranza 11th ed 15)
Fenestration: K1 polypeptide (68 KD) is the main component of the
Isolated areas in which the root is denuded of bone and stratum corneum. While in basal cells, the main keratin is
is covered by periosteum and overlying gingiva only. The K19 (40KD)
marginal bone is intact.

Dehiscence:
Denuded areas extend through the marginal bone. Occur
more often on the facial bone than on the lingual.

62. 'B' [Carranza 11th ed 41 / 101h ed 82)


Facial and lingual portions
Compact Bone
of sockets
Alveolar bone proper Thin compact bone
Interradicular bone Cancellous bone
Cancellous bone enclosed
Interdental septum
within a compact border

63. 'B' [Carranza 11th ed 22 / 101h ed 5 7)

64. 'B' [Carranza 11th ed 12 / 101h ed 68)

65. 'D' [Carranza 11th ed 876 / 101h ed 981, 982)


Check explanation of Q.17 in treatment of furcation and
osseus surgery

66. 'A' [Carranza 11th ed 20 / 101h ed 54)

67. 'C' [Carranza 11th ed 19 / 1Q'h ed 52, 53)


This is because the sulcular epithelium is permeable.

68. 'B' [Carranza 11th ed 21 / 1Q'h ed 74)

69. ' B' [Orbans 13th ed 178)


Osteoblasts are derived from the multipotent mesenchymal
cells. They cover the periodontal surface of the alveolar
bone. These constitute the modified endosteum and not
the periosteum. The surface of the bone lining a socket is
regarded as an interior surface of the bone. This surface of
the bone is covered by osteoblasts in various stages. The
collagen fibres of ligament that penetrate the alveolar bone
are present between the cells.

70. 'A' [Check Q.No.59)

71. 'A' [Check Explanation of Q.No.31]


Dental ;lut.,e

2. PERIODONTAL MICROBIOLOGY
1. Of the following four bacterial species, which is LEAST 12. Most important initiative factor for periodontitis is:
LIKELY to be found in plaque? a) Dental plaque b) Calculus
a) Actinomyces viscosus b) Streptococcus mutans c) Trauma from occlusion d) Food debris layer
c) Streptococcus salivarius (AIIMS - 95}
d) Streptococcus sanguis 13. Dental plaque adheres to the tooth because:
(MAN -95} a) Levans are gummy
2. The common etiology of periodontitis is b) Dextrans are insoluble and sticky
a) Occlusal trauma b) Systemic factors c) Plaque grows into the irregularities
c) Local irritating factors d} Hormonal defects d) Microorganisms produce sticky lipoproteins
(MAN - 02) (AIIMS, APPSC -99)
3. Predominant bacteria found in two days old plaque 14. In which of the following conditions is the role of
a) Streptococci b) Bacteroides microbial plaque most obscure:
c) Spirocheates d) Actinomyces a) Periodontitis b) Juvenile periodontitis
(MAN -98, AIPG -98) c) Desquamative gingivitis
4. In deep older plaque d) Necrotising ulcerative gingivitis
a) Streptococci and actinomyces are replaced by rod like (AIIMS -98, 99, AP- 08, AIIMS MAY 2012}
organism 15. "Corn-Cob" appearance seen in
b) Streptococci are completely replaced by neisseria a) Supragingival calculus b) Subgingival calculus
c) Streptococci actinomyces and veilonella remain c) Supragingival plaque d) Subgingival plaque
prominent (KAR -00}
d) Actinomyces are completely replaced by streptococci 16. Central gram negative core supporting outer coccal cells
(MAN - 98) is called:
5. Actinobacillus actinomycetam comitans is a) Bristle brush arrangement
a) Gram positive aero be b) Gram negative aerobe b) Corncob arrangement
c) Obligate anaerobe d) Facultative anaerobe c) Bottle brush arrangement
(MAN -01} d) Hourglass arrangement
6. Actinobacillus actinomycetamcomitans is commonly (KAR -02}
associated with 17. Breakdown of periodontal fibres in periodontitis is
a) Juvenile periodontitis b) Adult periodontitis due to bacterial enzyme: (OR) Which of the following
c) Refractory periodontitis d) All the above bacterial products have been implicated in initiation of
(MAN -01} inflammatory periodontal disease?
7. Supra gingival plaque causes a) Collagenase b) Hyaluronidase
a) Gingivitis b) Periodontitis c) Coagulase d) None of the above
c) Pericoronitis d) Aphtous ulcers (AIIMS - 97}
(MAN -2K} 18. Pellicle formation involves
8. The organism least Likely to be found in normal gingival a) Adsorption of acidic glycoprot eins from saliva
crevices is b) Focal areas of mineralisation
a) Fusobacteri m sp. b) Actionomyces sp. c) Focal areas of necrosis d) Bacterial colonization
c) Diphtheroids d) Streptococci sps. (AIIMS - 2K)
(MAN - 01) 19. Which of the following is the common factor for the
9. The sticky polysaccharide present in dental plaque is initiation of both dental caries and periodontal disease
a) Dextran b) dextrin a) bacterial plaque b) lactic acid
c) Glycogen d) sucrose c) calculus d) no common factor
(MAN -95} (AIIMS -2K}
10. Which of the following organisms is NOT implicated in 20. Severe alveolar bone loss, as observed in juvenile
the etiology of Periodontal disease periodontitis is associated with:
a) Bacteroides b) Wolinella a) Cyclic neutropenia b) Lysis of neutrophils
c) Neisseria d) Eikenella c) Increased phagocytosis
(KAR-2K, AIIMS MAY 2012) d) Neutrophil chemotactic defects or Impaired neutrophil
11, Bacteria, which are not found in normal healthy chemotaxis
periodontium, are: (AIIMS - 2K, AIIMS MAY 2012}
a) Acti no myces b) Capnocytophaga 21. The inorganic component of plaque is primary
c) Veillonella d) Eubacterium a) Calcium and fluoride b) Calcium and sodium
(KAR-2K} c) Calcium and phosphorus

1) C 2) C 3) A 4) A 5) D 6) A 7) A 8) C 9) A 10) C 11) D 12) A 13) B


14) C 15) C 16) B 17) A 18) A 19) A 20) D 21) C
, PERIODONTICS

d) Sodium and calci um 33. The bacteria of oral flora which plays least role in
(KAR -97, COMEDK-15) periodontitis is:
22. Co-aggregation is mainly predominated by a) Actinomyces b) Actinomycetam comitans
a) interaction between Gm + bacteria c) Spirocheates d) Bacteriodes
b) interaction between Gm - bacteria (AIIMS -97)
c) interaction between Gm + and Gm - bacteria 34. Majority of oral microorganisms are:
d) none of the above a) Strict anaerobes b) Gram-positive bacilli
(AP -99, 03) c) Spirochetes d) Facultative anaerobes
23. Which of the following is a periodontal pathogen (AIIMS - NOV 02)
a) S. sanguis b) S. mutans 35. The bacterial flora associated with Juvenile periodontitis
c) A. viscosus d) P. gingivalis is mainly:
(KAR -02) a) Gram +ve aerobic cocci b) Gram +ve anaerobic cocci
24. Increases in steroid hormones are associated with c) Gram -ve aerobic cocci d) Gram -ve anaerobic cocci
significant increases in (KAR -01)
a) Actinomyces viscosus b) Prevotella intermedia 36. The inorganic component of sub-gingival plaque is
c) Streptococcus sanguis d) Campylobacter rectus derived from :
(COMED-2012) a) Saliva b) Crevicular fluid
25. The carbohydrate not associated with dental plaque is: c) A and B d) Fluids ingested
a) Dextrose b) Levans (PGI -99)
c) Glucose d) Rhamnose 37. Which of the following is the most likely source of
( PGI -99, 2K) collagenase?
26. Specific plaque hypothesis was put forward by: a) Staphylococcus aureus b) Bacteroides gingivalis
a) Jenee b) Listgarten c) Treponema microdentum d) Veillonella alkalescens
c) Loesche d) None of the above (AIPG -91, 92)
(KAR - 98) 38. Plaque differs from materia alba in
27. Which is the most numerous component of plaque? a) Presence of bacteria b) Presence of glycoprotein
a) Minerals b) Food debris c) Presence of saliva d) Absence of glycoprotein
c) Microorganisms d) Leucocytes (APPSC - 99)
(AIIMS -92) 39. Bacteria in plaque form
28. Which surface of tooth has most of plaque? a) Intracellular polysaccharides
a) Gingival third of tooth surface b) Extra cellular polysaccharides
b) Incisal one third of tooth surface c) Both A and B d) Complex polysaccharides
c) Middle third of tooth surface (PGI - 2K)
d) Uniform on all surfaces 40. Which of the following is first formed after tooth brushing
(AIIMS -93) a) Mat eria alba b) Plaque
29. Bacterial " finger printing" illustrate that periodontal c) Pellicle d) Calculus
pathogens are (PGI -97, 99)
a) Contagious b) Non-contagious 41. Pellicle is a
c) Transmissible within member of same colonies a) Salivary protein b) Plaque
d) Non transmissible c) Micro organisms d) Calculus
(GCET-14) (PGI -03)
30. In later phases of plaque development the organism 42. Pre-requisite for plaque formation
pre dominates a) Pellicle b) glycoprotein
a) Staphylococci b) Streptococci c) mucopoly saccharide d) dextran
c) Rods and fi laments d) Vibrio and spirochetes (AP - 04)
(KAR -01) 43. Tanerella forsythus is a periodontal pathogen
31. Which of the following plaque is most harmful and a) Gram positive, non-obligate
causes spread of Inflammation to the connective tissue b) Non-pleomorphic rod
leading to bone destruction: c) Non - obligate anaerobe
a) Subgingival epithelial attached d) Non-motile, spindle shaped organism
b) Subgingival unattached (GCET-14)
c) Subgingival plaq11.1e 44. Electronic instrument used to measure gingival crevicular
d) Supragingival plaque fluid is
(PGI - 2K, 01) a) Pericheck b) Periotemp
32. Dental plaque is formed by reaction of enzymes on: c) Perioscan d) Peiotron
a) Sucrose and lipids b) Glucose and proteins (KAR -02)
c) Glucose and lipids d) Sucrose and saliva 45. Ecologic determinants of plaque depend on all, except:
(AIIMS -95, AIPG -01) a) Sugar content of diet b) Host resistance

22) B 23) D 24) B 25) D 26) C 27) C 28) A 29) C 30) C 31) A 32) D 33) A 34) D
35) D 36) B 37) B 38) B 39) C 40) C 41) A 42) A 43) D 44) D 45) C
Dental ;lut.,e i======
c) Age and sex of the patient 57. Commonly isolated organism from oral cavity is
d) Status of dentition a) Streptococcus sanguis b) Streptococcus pyogens
(AlPG-05) c) Neisseria gonorrheoa d) Streptococcus pneumonia
46. Salivary pellicle is composed of the following except: (AIPG-04)
a) lmmunoglobulin G b) lmmunoglobulin A 58. Primary Colonization of plaque is dominated by
c) Amylase d) Albumin a) Facultative Aerobic Gram + ve rods
(AlPG-05) b) Facultative Anaerobic Gram - ve rods
47. Biofilms in general have c) Facultative Aerobic Gram - ve cocci
a) Crystalline structure b) Stereo line structure d) Facultative Anaerobic Gram + ve cocci
c) Organized structure d) Disorganized structure (COMEDK-07, AIIMS NOV-14)
(BHU-2012) 59. Periodontitis is caused by
48. The concept of calculus formation in which seeding a) Malnutrition b) Supra gingival plaque
agents induce small foci of calcification which enlarge c) Bio-film d) Faulty tooth brushing
and coalesce to from a calcified mass is termed as (KCET-07)
a) Mineral precipitation b) Calculus adapation 60. The Gram negative bacteria most numerous in oral cavity
c) Heterogenous nucleation d) None of the above are
a) Streptococci b) Veillonellae
49. A test that can be used for typing of class I c) Selenomous d) Eikenella
histocompatibility antigens is (KCET-07)
a) Cell mediated lympholysis (CML) 61. Which one of the following species of streptococci is
b) Donor-recipient mixed lymphocyte response usually not found in human dental plaque.
c) Primed lymphocyte typing a) S.mutans b) S.sanguis
d) Antibody and complement mediated cytotoxicity c) S.pyogens d) S.salivarius
(COMEDK-05) (AP-08)
50. Sub-gingival scaling alters the microflora of periodontal 62. Which component of diet effects plaque composition
pocket a) Carbohydrate b) Mineral
a) Never gets altered b) Aerobes only c) Protein d) Fats/lipids
c) Gets altered d) Anaerobes only (BHU-07)
(COMEDK-06) 63. Micro organism which uses steroids as growth factor is
51. The primary etiologic factor in the development of a) P.gingivalis b) P.intermedia
furcation defect is c) S.sanguis d) S.mutans
a) Calculus b) Plaque (KCET-08)
c) Cementa[ caries d) Root infection 64. What is/are the method/methods of transferring
(COMEDK-06) information in a biofilm?
52. A glycoprotein that promotes new attachment and a) Quorum Sensing b) Conjugation
increased cell proliferation is: c) Plasmid transfer d) All of the above
a) Fibronectin b) Fibro pectin (UPSC-09, KCET-11)
c) Glycogen d) Glycoaminologlycan 65. The growth of porphyromonas gingivalis is enhanced by
a metabolic byproduct succinate produced by
53. In periodontal disease, ground substance is dissolved by a) Streptococcus b) Ca.pnocytophaga
a) Hyaluronidase b) Coagulase c) Actinomyces d) Veillonella
c) Phosphorylase d) Acid phosphatase (UPSC-09)
(AP-06) 66. All of the following organisms can be detected by BANA
54. The biofilms found on tooth surfaces are termed as: analysis EXCEPT
a) Enamel b) Dental caries a) Tannerella forythia b) Porphyromonas gingivalis
c) Dental plaque d) Saliva c) Capnocytophaga d) A. actinomycetemcomitans
(AlPG-06, AIIMS-07) (KCET-201)
55. The lactoperoxidase thiocynate system present in saliva 67. Bacteria in plaque are held togethe r by
is against a) Hydrophobic forces b) Covalent bonds
a) A. actinomycetemcomitans c) Vander waal's forces d) All of the above
b) Streptococcus (MCET-10)
c) P. gingivalis d) Actinomyces 68. Which one of the following complexes of periodontal
(KCET-2011) micro-organisms is associated with bleeding on probing?
56. Which is the enzyme that prevents the adhesion of a) Red Complex b) Orange Complex
actinomyces to adhere to the tooth structure? c) Green Complex d) Yellow Complex
a) Eno lase b) Glucosyl transferase (UPSC-09)
c) Xanthine oxidase d) Myeloperoxidase
(PGl-05)

46) D 47) C 48) C 49) D 50) C 51) B 52) A 53) A 54) C 55) B 56) D 57) A 58) D
59) C 60) B 61) C 62) A 63) B 64) D 65) B 66) D 67) D 68) A
, PERIODONTICS
,......__,,_

V
531

69. Bacterial communication with each other in a biofilm is d) Associated with gingivitis and periodontitis
known as (KAR-2013)
a) Corncob formations b) Coaggregation 79. According to Glickman, maxim um accumulation of plaque
c) Quorum sensing d) Translocation takes place in approximately
(COMED-10) a) 7 days b) 15 days
70. Which is characteristic of supragingival plaque and not c) 30 days d) 60 days
of subgingival plaque in humans? (AIIMS MAY 2012)
a) Motile bacteria are predominant 80. After cleaning and pumicing the tooth surface, plaque
b) Spirochetes are evident microscopically formation takes place within
c) Gram negative bacteria are predominant a) A few minutes b) 112 to 1 hour
d) Bacterial composition is altered by dietary sugar c) 2 to 4 hour d) After 1 hour
composition (AIIMS MAY 2012)
(AIPG-09) 81. Which of the following species is found in abundance in
71. The number of bacteria in the oral cavity is greater adult plaque?
a) in the morning b) after meals a) Fusobacterium b) Leptotrichia
c) at night d) after brushing c) Bacteroids d) All of the above
(AP-09) (KAR-2013)
72. Which of the following substances play a major role 82. Structures resembling corncob can be seen in a plaque
in regulating cell-cell and cell-matrix interaction in sample that is:
cementum? a) One week old b) Two week old
a) Sharpey's fibers b) Intrinsic fibers c) Three week old d) Four week old
c) Proteoglycans d) Phosphoproteins (PGI JUNE-2014)
(KCET-09) 83. Which of the following bacteria do not invade host tissue
73. Enterococcus faecalis in post treatment periodontitis cell?
a) Cultured easily and disinfected a) T. denticola b) P. intermedia
b) Disinfected with saline and hydrogen peroxide c) P. gingivalis d) F. nucleatum
c) Tolerate pH upto 11.5 (PGI DEC-2013)
d) Treated with intra canal medicaments 84. Lactic acid is produced in mouth by
(KCET-10) a) Saccharolytic bacteria in supragingival region
74. After enamel has been exposed to bacteria, irreversible b) Saccharolytic bacteria in subgingival region
bacterial colonization takes place in about c) Non-saccharolytic bacteria in supragingival region
a) 24 hrs. b) Few minutes d) Non-saccharolytic bacteria in subgingival region
c) 1-2 hrs. d) 2-4 hrs. (PGI JUNE-2014)
(AIPG-09) 85. A complex interaction among lymphocytes, inflammatory
75. Supragingival plaque undergoes which of the following cells and other cellular elements in connective tissue are
changes with time? mediated by a series of low molecular weight proteins
a) Plaque mass decreases called
b) Plaque microflora becomes more gram positive a) Bradykinin b) Histamine
c) Plaque microflora becomes gram negative c) Cytokines d) Kallikrein
d) Plaque microflora becomes predominantly spirochetal (COMEDK-15)
(AIIMS MAY 13, AIPG-09, 11) 86. Not significant in pathogenicity of periodotic problems?
76. The 'Red complex' associated with bleeding on probing is a) Endotoxins from live bacteria
comprised of b) Endotoxins from dead bacteria
a) E.corrodens, A.actinomycetem comitans, Capnocytophaga c) Microbial interactions
b) A.naeslundii, A.viscosus, A.odontolyticus d} Enzyme by microbes
c) P.gingivalis, T.forsythia, T.denticola (PGI JUNE-13)
d) Streptococcus, Fusobacterium, Campylobacter 87. Specific plaque hypothesis state that
(COMEDK-10) a) All plaque is pathogenic
77. Black pigmented bacteroides release ___ as agent b) On ly specific micro-organisms are responsible for
responsible for unpleasant smell of breath. pathogenicity of plaque
a) Methyl mercaptan b) Hydrogen sulphide c) Only specific microbes cause caries
c) Dimethyl sulphide d) Propionic acid d) Plaque is pathogenic only when signs of associated
(AP-2013) disease are present
78. Which of the following is true about tooth associated (AIIMS MAY-14)
subgingival plaque?
a) Has both gram positive and negative bacteria
b) Extends till the junctional epithelium
c) May penetrate cementum

69) C 70) D 71) A 72) C 73) C 74) D 75) C 76) C 77) B 78) C 79) C 80) B 81) A
82) C 83) B 84) A 85) C 86) A 87) D
Dental ;lut.,e

2. PERIODONTAL MICROBIOLOGY - ANSWERS


1. ' C' [Carranza 11th ed 309 / 10th ed 13 7) 8. ' C' [Carranza 11th ed 312 / 10th ed 138)
Streptococcus sanguis, S.mitis, S.mutans and actinomyces Oiphtheroids are usually not seen in the gingival crevices.
viscosus are present in plaque.
9. 'A' [Carranza 11'h ed 316 / 10th ed 140)
Streptococcus salivari us is the first organism to appear in The interbacterial matrix accounts for 20-30% of
the mouth after birth and is the predominant organism the plaque mass. It is made of dextrans and levons.
in saliva. It is not usually found in plaque. It is a non- Dextran is the glucose moiety of sucrose, which is
pathogenic bacterium and forms the longest chains. insoluble, sticky, and Levan is the fructose moiety of
sucrose synthesized by the plaque bacteria.
2. 'C' (Carranza 11t• ed 316 / 10th ed 134)
Local factors are the common etiological agents of Calcium, phosphorus, fluoride, etc., are the inorganic
periodontitis. Systemic factors cause periodontitis in the components of plaque.
presence of local factors by exaggerating the tissue response
to local factors. 10. ' C' [Carranza 11th ed 343 / 10th ed 153)
• The bacteria associated with periodontal health
Systemic factors are important in the pathogenesis of (protective species) are S.sanguis, S.mitis, A.viscosus,
periodontal disease because they can (COMEDK-15) Capnocytophaga, Neisseria, Veillonella, etc.
a) Be the direct cause of periodontal disease • The bacteria associated witlh periodontal disease are
b) Have direct effect on pocket depth P.gingivalis, P.intermedia, A.actinomycetam comitans,
c) Usually determine the patt ern of bone loss Eikenella, Fusobacterium and Eubacterium species, etc.
d) Intensify the response of the periodontitium to the
etiologic and loca l factors 11. ' D' [Carranza 11th ed 309 / 10th ed 153)

3. 'A' [Carranza 11th ed 322 / 10th ed 141) 12. 'A' [Carranza 11th ed 316)
The initial bacteria colonizing the pellicle coated tooth Plaque is a biofilm with a regular intercellular matrix
surface are predominantly gram+ve facultative anaerobic consisting predominantly of microorganisms responsible
cocci followed by rods, such as streptococcus sanguis and for periodontitis. The most common cause of gingivitis and
Actinomyces viscosus. periodontitis is bacteria laden local plaque.

The secondary colonizers adhere to cells of bacteria Which of following is related to gingivitis? (PGI-2011)
already present in plaque. They are predominantly gram a) Plaque b) Calculus
-ve anaerobic rods and filaments such as p. gingivalis,
c) Tooth brush trauma d) Abrasive powder
p.intermedia, Fusobacterium and Capnocytophaga species.
13. ' B' [Carranza 11th ed 316 / 10th ed 140)
Secondary colonizer of plaque are all except (AIPG-14)
a) Prevotella intermediate 14. ' C' [Carranza 11th ed 152/ 10th ed 411)
b) Fusibacterium nucleatum Plaque does not cause desquamative gingivitis.
c) Streptococcus sanguis d) Porphyromonas gingivalis Oesquamative gingivitis is a non-specific manifestation of
certain autoimmune diseases ( Lichenplanus, Pemphigoid
4. 'A' [Carranza 11th ed 324 / 10th ed 143) and Pemphigus) and hormonal changes (menopause).

5. 'D' [Carranza 11th ed 218 / 10'h ed 160) 15. ' C' [Carranza 11th ed 324 / 10th ed 143)
A.actinomycetem comitans is a gram -ve, capnophilic Corncob structures are highly specific cell-to-cell
facultative anaerobic rod type of organism often implicated interactions. The corncob formations have rod shaped
in the pathogenesis of localized juvenile periodontitis. bacterial cells (Eg . Bacterionema or F.nucleatum) that
It is also associated with chronic adult periodontitis and form the inner core of the structure and coccal cells (Eg.
refractory periodontiti s. Streptococci) that attach along the surface of the rod
shaped cell. Supra gingival plaque typically demonstrates
6. 'A' [Carranza 11th ed 218 / tO'h ed 160) corncob structures.

7. 'A' [Carranza 10th ed 138) The ability of different species and genera of plaque bacteria
• Supragingival marginal plaque causes gingivitis. to adhere one another is known as 'Co-aggregation' (or)
• Supragingival and subgingival tooth attached plaque 'test tube arrays'. Co-aggregation is predominant among
gram -ve organisms.
are critical in formation of calculus and root caries.
• Subgingival tissue attached plaque causes soft tissue l6. ' B' [Carranza 11•h ed 325 I 1o•h ed 143)
lesions characteristic of different forms of periodontitis.
, PERIODONTICS
,......__,,_

V
533

17. 'A' [Carranza 111h ed 268) Korman K.S et al in 1982 concluded that increase in steroid
Collagenase is responsible for the breakdown of periodontal hormones are associated with significant increase in the
fibres in periodontitis. It is released by bacteria (P.gingivalis proportions of P. intermedia in subgingival plaque.
mostly), polymorpho-nuclear leukocytes, and some
populations of fibroblasts. Hyaluronidase causes breakdown 25. ' D'
of ground substance and helps in spread of inflammation
(cellulitis). 26. 'C' [Carranza 111h ed 339]
Both specific and non-specific plaque hypotheses are
18. 'A' [Carranza 111h ed 321 / 101h ed 140] proposed by Loesche .
Formation of plaque occurs in 3 phases:
• Formation of pellicle. Non-specific hypothesis states that periodontal diseases
result from elaboration of noxious products by the entire
• Initial colonization of the tooth surface.
plaque flora. The periodontal disease can be treated by
• Secondary colonisation and plaque maturation. debridement (surgical or non surgical) and oral hygiene
measures. Even though non-specific hypothesis is discarded,
Pellicle formation occurs by selective adsorption of salivary much of clinical treatment is still based on non-specific
glycoproteins by electrostatic, vanderwalls and hydrophobic plaque hypothesis only.
forces.
Alternative or specific hypothesis states that only certain
19. 'A' [Carranza 111h ed 322 / 101• ed 135] plaque is pathogenic and recognizes plaque as pathogenic
i) Early supragingival plaque contains - gm+ve cocci and only only when signs of of associated disease are present.
rods while mature supragingival plaque contains gm -ve
rods and filaments. These organisms cause gingivitis. Option 'B', "Listgarten" described four zones (Bacterial zone,
Bacteria found in gingivitis are localized in gingival neutrophil rich zone, necrotic zone and zone of spirochaetal
sulcus and very few may be present in connective tissue. infiltration) in ANUG.

ii) Subgingival tooth associated plaque contains - gm +ve


cocci and rods which result in root surface caries. Specific plaque hypothesis is [AIPG-2012]
a) Specific microbes can cause caries
Eg.: A. viscosus. b) All microorganisms in plaq11.1e are pathogenic
iii) Subgingival tissue attached plaque contains gm - ve c) Specific microorganisms are responsible for
pathogenicity of plaque
bacteria.
d) All microbes cause caries
Eg.: P.gingivalis, bacteroides, etc., which may cause
destruction of the periodontal tissue. 27. 'C' [Carranza 111h ed 318 / 10th ed 137)
20. 'D' [Carranza 11th ed 221)
28. 'A' [Carranza 111h ed 327]
In juvenile periodontitis there is functional defect of PMN Plaque preferentially forms on non self cleansing areas.
chemotaxis (AIIMS MAY 2012) and phagocytosis of Plaque accumulates on the gingival third of tooth surface
microorganisms. This defect may be induced by the invading
due to lack of movement of food and tissues during
bacteria viz., A. actinomycetam comitans. mastication and also in cracks, pits and fissures.
21. 'C' [Carranza 111h ed 317 / 101• ed 140] 29. 'C' [Carranza 11th ed 332)
Plaque contains about 5% of inorganic components. The Molecular finger printing techni ques clearly illustrated that
principal inorganic components of plaque matrix are calcium periodontal pathogens are trarnsmissible within members
and phosphorus. Other elements are magnesium, potassium,
of a fami ly. This bacterial transmission between subjects
sodium and fluoride. should not be confused with contagious.

Fluoride deters the metabolism of plaque bacteria and aids 30. 'C' [Carranza 11th ed 324 / 10th ed 141)
in the remineralisation of the tooth surface. The source
for the inorganic components of supragingival plaque is 31. 'A' [Carranza 111h ed 328 / 10' h ed 138)
saliva and for subgi ngival plaque is GCF.
Subgingival tissue attached plaque consists of gram -ve
rods and filaments. Due to the physical proximity of these
22. 'B' [Carranza 11th ed 324 / 101h ed 143 J bacteria to the host tissues they invade the connective
tissue leading to the destruction of supporting tissues
23. ' D' [Carranza 111h ed 342 / 101h ed 153] (periodontitis).
Option 'B' Streptococcus mutans causes enamel caries.
32. ' D' [Carranza 111h ed 316]
Option 'C' A.viscosus is considered as etiologic organism of
root surface caries. 33. 'A' [Carranza 11th ed 343 / 10' h ed 156)
Actinomyces are associated with periodontally healthy sites.
24. ' B' [Carranza 11 • ed 346 / 10th ed 150)
1
Dental ;lut.,e i ==================

They have least capacity to invade host tissues. 47. ' C' [Carranza 11th ed 328 / 10th ed 138]
Biofilms, in general, have an organized structure, composed
34. ' D' [Carranza 11th ed 309] of microcolonies, of bacterial cells non randomly distributed
in a shaped matrix.
35. 'D' [Carranza 11th ed 218 / 10'h ed 158]
48. ' C' [Carranza 11th ed 295 / 1Qth ed 175]
36. ' B' (Carranza 11•h ed 317 / 10th ed 140]
Theories of mineralization of calculus
The inorganic component of supragingival plaque is derived
from Option W i.e., saliva. Epitactic concept • Seeding agents induce small foci
(or) Heterogenous of calcification that enlarge and
3 7. ' B' (Carranza 11th ed 268] nucleation coalesce to form a calcified mass.
Bacterial enzyme Species • Occur due to rise in pH of saliva
which causes precipitation of
• P.gingivalis calcium phosphate.
Collagenase
• A.actinomycetemcomitans • Colloidal proteins in saliva bind
• P.gingivalis Mineral calcium and phosphate ions and
Trypsin like enzyme • T.denticola preci pita ti on maintains supersaturated solution
leading to its precipitation.
• A.actinomycetemcomitans
• Phosphatase liberated from plaque
• P. intermedia
Phospholi pidase-A precipitates calcium phosphateby
• P. melaninogenica hydrolyzing organic phosphates

38. 'B' (Carranza 11th ed 318 / 10th ed 17 6] 49. ' D' [Rubins Pathology 6th ed 123]
Materia alba is a yellowish or white soft sticky deposit and Major histocompatability complex (MHC) classes I, II and
is less adherent than dental plaque. III molecules are involved with antigen uptake, processing
and presentation.
It consists of microorganisms, desquamated epithelial cells,
salivary proteins but lacks the regular internal pattern that MHC Class I molecules are used to present intracellular
is observed in plaque. antigens to ma• T cells and NK cells. MHC class III molecules
include complement factors, B, C2 and C,.
39. ' C' [Check Explanation Below]
The bacteria found in plaque form both extracellular 50. ' C'
polysaccharides (dextran and Levan) and intracellular
polysaccharide (amylopectin) from carbohydrates. The 51. ' B' [Carranza 11th ed 897 / 10th ed 463]
external polysaccharides causes the attachment of Primary etiologic factor for the development of furcation
the bacteria t o the tooth as microbial plaque; and the defect is plaque.
intracellular polysaccharides are stored in cells and then
metabolized to lactic acid. The contributing etiologic factor is Trauma from occlusion.

40. ' C' [Carranza 11th ed 321 / 10th ed 147] 52. 'A' [Carranza 10th ed 97 4]

41. 'A' [Carranza 11th ed 321 / 10'h ed 147] 53. 'A' [Carranza 11th ed 268 / 10'hed 597]
Pellicle is a salivary glycoprotein without bacteria. It Collagen degradation is caused by collagens like:
is prerequisite for the formation of plaque. Bacteria
• Aspart ate aminotransferase
progressively accumulate to form dental plaque.
• Alkaline phosphatase
42. 'A' [Carranza 11th ed 321 / 10'h ed 147] • 13-Glucoronidase
• Matrix metalloproteinases
43. ' D' [Periodontics revisited by s halubathla 62]
Tanerella forsyt hus is a gram -ve anaerobic spindle shaped Ground substance is degraded by Hyaluronidase.
highly pleomorphic rod. It aids in the development of
periodontal diseases and belong to the red complex bacteria. 54. ' C' [Carranza 11th ed 316 / 10th ed 138]

44. ' D' [Carranza 11'h ed 95 / 10th ed 345] 55. ' B' [Carranza 11th ed 98 / 10'h e d 349]
The lactoperoxidase-thiocyanate system in saliva has been
45. 'C' [Carranza 11th ed 327] shown to be bactericidal to some strains of Lactobacillus
and Streptococcus by preventing the accumulation of
46. ' D' lysine and glutamic acid, both of which are essential for
bacterial growth. Another antibacterial finding includes
lactoferrin, which is effective against Actinobacillus species.
, PERIODONTICS
,.....__,,_

V
535

Mye/operoxidase, an enzyme similar to salivary peroxidase, is • Have been found in tongue, throat and in
released by leukocytes and is bactericidal for Actinobacillus" S. salivarius saliva but is not fou nd in high numbers
but has the added effect of inhibiting the attachment of in dental plaque
Actinomyces strains to hydroxyapatite. Lysozyme works on
both gram negative and gram-positive organisms"; Veillonella • One of the most commonly isolated
species and Actinobacillus actinomycetemcomitans are some bacteria in the oral cavity
S. mitior
of their targets. It probably repels certain transient bacterial • Along with S. sanguis forms the mass
invaders of the mouth. predominant organisms in dental plaque

56. ' D' (Carranza 11th ed 259] 62. 'A' (Carranza 11th ed 316 / 10th ed 137]

57. 'A' (Carranza 11th ed 343 / 1Qth ed 156) 63. ' B' (Carranza 11th ed 346 / 10th ed 150)
S.sanguis is found i n high numbers in sites that do not
demonstrate attachment loss (inactive sites). These species Organisms Growth factors
probably function in preventing colonization or proliferation Early colonizers Use oxygen and lower redox potential
of pat hogenic microorganisms. The H20 2 producedby S.sanguis like streptococci of the environment, which then favo urs
is known to be lethal to cells of A. actinomycetem comitans. and actinomyces the growth of anaerobic species
Its growth is enhanced by metabolic
Which of the following bacteria is lethal to A. actino- by products produced by other
mycetemcomitans? (PGI June-13) microorganisms such as
a) S. sanguis b) S. mitis Porphyromonas
• Succinate from Capnocytophaga
c) S. salivarius d) S. mutans gingivalis
ochraceous (PGl-13)
58. 'D' (Carranza 11th ed 3 22 / lO'hed 146] • Prot oheme from Campylobact er
rectus
59. 'C' [Carranza 111h ed 344 / 101hed 436) Its growth is enhanced by Hemin iron
P. gingivalis
from the breakdown of host hemoglobin
60. 'B' (Carranza 11th ed 343 / 10th ed 156] Increased in st eroid harmones are
• S.sanguis associated with significant increases
P. intermedia
• Veillonella parvula in Prevotella intermedia in subgingival
Beneficial species plaque.
(facultative gram +ve • C.ochraceus
species) Salmonella typhi Tryptophan
• Capnocytophaga
Gonococci Glut athione
• F.nucleatum
To differentiate human tubercle bacilli
• P.gingivalis Niacin test
from atypical mycobacteria
Species in plaque • P.intermedia
induced periodontal • Eikenella 64. 'D' (Carranza 11th ed 33 0 / lQ•h ed 150]
disease • A.actinomycetem comitans Quorum sensfog:
The capacity of bacteria to communicate with each other in
• Eubacterium
a biofilm involves the regulation of expression of specific
• S.sanguis genes through accumulation of signaling compounds. When
Preominant gram +ve • S.mitis these compounds reach a threshold level (quorum cell
species in plaque • S.oralis density) gene expression is activated. Such quorum seems
to play a role in expressing the growth of beneficial species
• A.visosus
to the biofilm and discouraging growth of competitors.
• F. nucleatum
Predominant gram -ve • P.intermedia Conjugation
species in plaque • Veillonella Exchange of genes through a direct interbacterial connection
formed by a sex pilus.
• Capnocytophaga
Transformation
61. 'C' [Carranza 11th ed 344 / 10th ed 135-37]
Movement of small pieces of DNA from t he environment into
• Pit and fissure caries the bact erial chromosome.
• Smooth surface caries
S. mutans 65. 'B' [Carranza 11th ed 331 / 10•h ed 150]
• Seen in plaque
Other similar interactions
• Often called as obligate peri phyte
• Lactate and formate are the byproducts of the metabolism
Str. mitis • Seen in tooth-associated cervical plaque of streptococci and actinomycetes and are used in the
metabolism of other plaque microorganisms.
Dental ;lut.,e i======
• Bacteria degrade host proteins, release ammonia which 79. 'C' (Carranza 11th ed 269]
may be used by bacteria as a nitrogen source. After tooth surface is cleaned during prophylaxis, a dental
• Hemin iron from the breakdown of host hemoglobin may pellicle from saliva appears on the surface within 1 minute.
be important in the metabolism of p.gingivalis. Supragingival plaque may form within one hour after the
teeth are thoroughly cleaned, with maximum accumulation
66. 'D' (Carranza 10th ed 591] reached in approximately 30 days.
8. forsythus, P. gingivalis, small spirochete Treponema
denticola, and Capnocytophaga species share a common 80. 'B' (Carranza 8th ed 375]
enzymatic profile, since all have in common a trypsin
like enzyme. The activity of this enzyme can be 81. 'A' (Shantipriya Reddy 2nd ed 178]
measured with the hydrolysis of the colourless substrate Microbial species in children and adult plaques
N-benzoyl-dl-arginine-2-naphthylamide (BANA). When
the hydrolysis takes place, it releases the chromophore • Leptotrichia
b-naphthylamide, which turns orange red when a drop of Species in greater numbers in • Capnocytophaga
fast garnet is added to the solution. Diagnostic kits have children's plaque • Selenomonas
been developed using this reaction for the identification • Bacteroides
of this bacteria profile in plaque isolates (Perioscan). One
of the potential difficulties of this test is that it may be Species in greater numbers in • Fusobacterium
positive at clinically healthy sites and remains to be proven adult's plaque • Eubacterium
whether this test can detect sites undergoing periodontal
destruction. 82. 'C' (Clinical Oral Microbiology by T. Wallace Macfarlane
30]
67. 'D' (Carranza 11th ed 316 / 10th ed 143] Thick zone of early colonizers and column
One week
like structures that result from rapid
68. 'A' (Carranza 10th ed 143] old plaque
proliferation of streptococci.
Ref. Synopsis
• Composition changes markedly
69. 'C' (Carranza 11th ed 330 / 10th ed 150] Three week • Filamentous organisms now predominate
old plaque • Corncob arrangement of organisms is
70. 'D' (Carranza 10th ed 13 7, 140] observed on the outer surface of plaque.

71. 'A' (Carranza 10th ed 144-146] 83. 'B' (Carranza 11th ed 200]
The two means of tissue invasion are:
72. 'C' (Carranza 11th ed 33 / 10th ed 75] 1) Bacteria may enter host tissues through ulcerations in
Proteoglycans are most Likely to play a role in regulating the epithelium of the gingival sulcus or pockets.
cell-cell and cell matrix interactions both in normal
development as well as in regeneration of cementum. 2) Direct penetration of bacteria into host epithelial or
connective tissue cells. A. actinomycetemcomitans,
73. 'C' [Ingle 6th ed 302] P. gingivalis, F. nucleatum and Trepanoma denticola
have the ability to directly invade the host tissue cells.
74. 'D' (Carranza 10th ed 144-146]
84. 'A' [Dental Caries a treatable infection by Loesche 28]
75. 'C' (Carranza 11th ed 324 / 10th ed 137] • In supragingival sites, saccharolytic microorganisms split
carbohydrates into lactic acid and create a temporary
76. 'C' (Carranza 11th ed 325 / 10th ed 143] acidic environment.
• In subgingival sites, asaccharolytic microorganisms
7 7. 'B' (Carranza 10th ed 331] metabolize nitrogenous compounds derived from GCF
• Black pigmented anaerobic bacteria Like P. gingivalis and create a neutral pH with anaerobic environment
and T. forsythia produce H25. This is responsible for oral abundant in short chain fatty acids and ammonia.
malodor.
85. 'C' (Carranza 11th ed 201]
• Also H25 reacts with bismuth chloride to form bismuth
sulphide as a black precipitate on subgingival plaque. Cytokines are low molecular weight, soluble proteins that act
as messengers to transmit signals from one cell to another.
78. 'C' (Refer Q. No. 19 Explanation] They are effective in very Low concentrations and primarily
Options B and D are characteristic of subgingival tissue act locally in the tissues in which they are produced. They
attached plaque. The bacteria may extend to cementum and are produced by a large number of cell types such as PMNs,
macrophages, lymphocytes, fibroblasts and epithelial cells.
initiate root caries.
They signal, broadcast and amplify immune responses
and are fundamentally important in regulating immune-
inflammatory responses and in combating infections.
, PERIODONTICS
~
537
V
86. 'A' [Microbiology: A clinical approach by Anthony
Strelkauskas 95]
Exotoxins are proteins secreted by living organism. They are
highly antigenic. Endotoxins (LPS) are contained in or are
the part of the bacterial cell membrane. They are released on
the death of the organism. Endotoxin from live bacteria does
not contribute to pathogenicity. The microbial virulence
factors are:
1. Endotoxin (LPS) released by dead bacteria
2. Bacterial enzymes
3. Noxious agents such as NH3, H2S, fatty acids
4. Microbial interactions and tissue invasion
5. Fimbriae and bacterial DNA.

87. 'D' [Carranza 11th ed 257, 258]


Walt er Loesche proposed t he two hypothesis of plaque.
• Nonspecific plaque hypothesis:
This assumes that all plaque is pathogenic

• Specific or alternative plaque hypothesis:


This recognizes plaque is pathogenic only when signs of
associated disease are present.
Dental ;lut.,e

3. DE FENSE MECHANISMS OF GINGIVA & HOST RESPONSE

1. The predominant cell type in gingival crevicular fluid is 11, The most potent bone resorbing interleukin is:
the: a) IL - 8 b) I L - 18
a) Mast cell b) Plasma cell c) IL - 5 d) I L - 3
c) Macrophage d) Polymorphonuclear leukocyte (KAR - 02}
(AIPG -91, 03) 12. Leukotoxin is released by:
2. Gingival crevicular fluid is measured using: a) P. nucleatum b) A. naeslundii.
a) Whatman's tilter paper b) Ph paper. c) B. forsythna d) A. actinomyctem comitans.
c) Mylar strip d) Litmus paper. (KAR - 02}
(KAR -98} 13. Plaque is considered as infection because
3. The predominant immunoglobulin in sulcular fluid is: a) Anlibiolics prevenls ils forma lion
a) lg A b) lg G b) Its presence is evidence of bacterial growth
c) lg M d) lg E c) It is communicable between experimental animals and
(KAR - 2K} probably humans
4. Orogranulocytes are: d) All of the above
a) Granulocyt es present in the gingival connective tissue (KAR - 01}
b) Mast cells present in the G.C.F 14. The gingival crevicular fluid is increased in all except:
c) P.M .Ns. reaching oral cavity t hrough subepithelium a) Gingivitis b) Smoking
d) All of the above c) Periodontal pocket d) Trauma from occlusion
(KAR - 2K) (PGI-06)
5. Drug which reaches maximum concentration in gingival 15. The inorganic component of subgingival plaque is derived
fluid is: from
a) Tetracycline b) Penicillin a) Saliva b) Gingival crevicular fluid
c) Erythromycin d) Sulphonamide c) Inorganic matter of tooth
(AIPG -94} d) Food debris
6. Which cell type migrates into the gingival sulcus in large (KAR-04}
numbers in response to dental plaque: 16. Gingival fluid is a:
a) Mast cells b) Neutrophils a) Transudate b) Exudate
c) Lymphocytes d) Plasma cells c) Can be either of the two (Trasudate & Exudat e)
(AIPG -99} d) Neit her of t he two (Trasudate & Exudate)
7. Sulcular fluid does not perform one of the following
functions: 17. What are the cells that produce PGE2 in the Periodontium?
a) Contains plasma proteins which may improve adhesion a) Macrophages b) Fibroblasts
b) Possesses antimicrobial properties c) Neutrophils d) a and b
c) Exerts antibody activity (COMEDK-2013}
d) Provides nutrition to j unctional epit helium via diffusion 18. To identify key microorganism in periodontal disease,
(KAR -2K} Koch's postulate have been modified by
8. Glucose levels in gingival crevicular fluid (GCF) are: a) Socransky b) Glickman
a) Equal to glucose level in serum c) Russell d) Vermilion
b) Zero (AIIMS MAY-13)
c) 3-4 times greater t han serum levels 19. The bacterial enzyme detected in gingival crevicular
d) More t han 10 times the serum levels fluid is
(COMED-14, AIPG-05} a) Fibronectin b) Cytokines
9. It is likely that cell mediated immune reactions (delayed c) Phospholipase d) Myeloperoxidase
hypersensitivity) occur in periodontitis because subjects (APPG-15}
with periodontitis have
a) High levels of histamine in involved gingival tissue
b) IgG antibodies reactive with plaque bact erial antigens
c) T-Lymphocytes sensitized t o bacterial plaque antigens
d) High levels of collagenase in gingival fluids
(AIIMS MAY 2013, 2K, AIPG -01}
10. Leukotoxin
a) Kills neutrophils b) Attracts neutrophils
c) Aggregat es neutro phi ls d) Enhances phagocyt e activity
(KAR -98}

1) D 2) A 3) B 4) C 5) A 6) B 7) D 8) C 9) C 10) A 11) B 12) D 13) D


14) D 15) B 16) B 17) D 18) A 19) C
, PERIODONTICS
~

V
539

3. DEFENSE MECHANISMS OF GINGIVA & HOST RESPONSE - ANSWERS

1. ' D' [Carra nza 11th ed 95 / 10th ed 348) 7. ' D' (Carranza 11th ed 96 / 10th ed 57)
The predominant cell type in GCF is the PMN. They account Options A, B, C are the functions of GCF.
for 92% of the total leukocytes while t he mononuclear cells
account for 8%. 8. 'C' (Carranza 11th ed 96 / 10thed 346)
Glucose concentration in GCF is 3-4 times greater than in
Among the mononuclear cells, 58% are B-lymphocytes and serum. This is not only due to the metabolic activity of
24% are T-lymphocytes. The normal T:B lymphocyte ratio in adjacent tissues, but also as a function of the local microbial
peripheral blood is 3:1 is and the value is reversed to about flora.
1:3 in GCF.
The prolein conlenl of GCF is less Lhan lhal of serum.
2. 'A' [Carranza 11th ed 95 / 10th ed 345)
Methods of collecting GCF: 9. 'C' (Carranza 11th ed 281)
• Placing filter paper into the sulcus (intracrevicular) Cell mediated immune reactions or delayed hypersensitivity
does not involve antibodies but is based on the interaction
• Placing paper at the entrance of sulcus (extra crevicular)
of antigens with surface ofT-lymphocyt es.
• Placing preweighed twisted threads or micropipettes and
crevicular washings. 10. 'A' (Carranza11th ed 219 / 10th ed 233)
Leukotoxin is an exotoxin produced by A. actinomycetem
Methods of measuri nq the amount of GCF: comitans which has toxic effect on PM N's. This enables these
• The wetted area of the filter paper is stained with microorganisms to evade the host defense of phagocytosis.
Ninhydrin and is measured planimetrically under
microscope. 11. ' B' (Carranza 11th ed 269 / 10th ed 203)
• Electronic method by using fluid collected on a blotter Interluekin 1 (a and b) includes Osteoclast Activating
(perio paper) and employing electronic transducer called Factor (OAF) which causes bone resorption and Lymphocyte
Periotron. Activating Factor (LAF) which has the ability to stimulate
proliferation of T cells.
Paper points, micropipette and preweighed twisted IL-I and TNF (Tumor necrosis factor) are key cytokines in the
threads are used to collect? (PGI June 2011) pathogenesis of periodontitis.
a) GCF b) Saliva
c) Bacteria d) Serum 12. ' D' [Carranza 11th ed 281 / 10th ed 233)
3. ' B' (Carranza 11th ed 283 / 10th ed 349) 13. ' D' (Carranza 11th ed 341)
• Predominant immunoglobulin in GCF - lgG Robert Koch developed the criteria by which a microorganism
• Predominant immunoglobulin in Saliva - IgA can be judged to be the causative agent in human infections.
These are called as "Koch's postulates".
4. 'C' [Carranza 11th ed 98 / 10th ed 350)
Orogranulocytes are the PMNs that reach the oral cavity by SIGMOND SOCRANSKY proposed criteria by which periodontal
migrating through the lining of the gingival sulcus. microorganisms may be judged to be potential pathogens.
They are
5. 'A' (Carranza 11th ed 97 / 10th ed 347) • Be associated with disease with increase in number of
Tetracycline and Metronidazole are excreted through organisms at diseased sites.
the gingival fluid which can be used advantageously in • Be eliminated or decreased with treatment.
periodontal therapy.
• Demonstrate host response.
Concentration of tetracycline in gingival fluid is 2-10 times • Demonstrate virulence factors.
(average 7 times) that of plasma. • Be capable of causing disease in experimental animals.

Which antibiotic has higher levels in GCF as compared The pathogenic potential of bacteria within the plaque
to plasma? (PGI Dec-2013) varies from individual to individual and from one gingival
a) Penicillin b) Tetracycline site to other gingival site.
c) Amoxicillin d) Metronidazole
14. 'D' (Carranza 11th ed 94 / 10th ed 347)
6. ' B' (Carranza 11th ed 95 / 10th ed 348)
Leucocytes constitute a major protective mechanism against 15. 'B' (Carranza 11th ed 318 I 10th ed 140)
the extension of plaque into sulcus. They are attracted by
plaque bacteria.
Dental ;lut.,e

16. 'B' (Carranza 11'h ed 97)


Some considered GCF as a transudate while others considered
it as an inflammatory exudate.

17. 'D' [Carranza 11th ed 269)


The cells that produce PGE2 (Prostoglandin E2 ) in
periodontium are macrophages and fibroblasts. PGE 2 induces
the secretion of metallomatrix proteins and osteoclasic bone
resporption and contributes to the loss of alveolar bone loss
as seen in periodontitis.

18. 'A' [Refer explanation of Q. 13]

19. 'C' [Carranza 11th ed 67)


The enzymes in GCF can be host derived or produced by the
bacteria in the gingival crevice. The following enzymes are
detected in GCF:
• Beta-glucuronidase (lysosomal enzyme)
• Lactic acid dehydrogenase (Cytoplasmic enzyme)
• Collagenases (from PMN, fibroblasts or bacterial origin)
• Phospholipases (lysosomal enzyme)

The majority of GCF elements detected thus far are the


enzymes.
, PERIODONTICS
~

V
541

4. CALCULUS & IATROGENIC DISEASES


1. The green stains frequently seen on children's teeth are d) Gingival itching.
caused by (AIPG -04)
a) Materia alba b) Enamel defects 11. Lateral food impaction occurs mainly due to
c) Dentinal defects d) Chromogenic bacteria a) Un even alteration of occlusal surface
(MAN -94, AP -98, AIIMS MAY 2012) b) Open proximal contact
2. Mineralization inhibitors of calculus formation include c) Open gingival embrasured) All of the above
all of the following except (AP -04)
a) Triclosan b) Pyrophosphate 12. In AIDS patients, occurrence of severe and acute
c) Zinc salts d) Diphosphates periodontal disease are due to
(KAR - 99) a) Reduced number of T - cells
3. Calculus attaches to tooth structure by the following b) Release of lymphokines
ways except: c) Cytotoxic reaction d) All of the above
a) By means of an organic pellicle {KAR -98)
b) Mechanical locking in surface irregularities 13. Linear gingival erythema is a characteristic feature of
c) Close adaptation to undersurface depressions a) ANUG
d) Tags of calcium phosphate reacting chemically with b) Acute herpetic gingivostomatitis
enamel walls c) HIV - Gingivitis d) Chronic gingivitis
(AIPG -94) (KAR -02, COMEDK -06)
4. In periodontal disease, calculus is considered as 14. The gingiva in a patient of HIV is similar to the gingiva in
a) Contributing factor b) Primary factor a) Lichen planus b) ANUG
c) Etiological factor d) Governing factor c) Acute gingivitis d) Chronic gingivitis
(KAR - 03) (PGI - 01)
5. CALCULUS all are true except: 15. A patient has cheilosis, angular stomatitis, glossitis, red,
a) Supragingival calculus is brown in colour itching eyes. The periodontal surgery that was performed
b) Hard and gritty one week ago has not healed properly. Deficiency of
c) Calcified from plaque which of the following vitamin is most likely:
d) Subgingival calculus is more difficult to remove than a) Thiamin b) Riboflavin
supragingival calculus c) Tocopherol d) Calciferol
(AIPG -02)
6. Hard deposits found on tooth and appliances in an 16. Which of the following soft tissue responses may occur
uncleaned mouth is called as a reaction to orthodontic bands?
a) Plaque b) Calculus a) Marginal gingivitis b) Gingival fibrosis
c) Materia alba d) None c) Ulcerative gingivitis d) Fulminating periodontitis
(AP - 01) (AIPG-06)
7. A white soft sticky deposit on tooth surface 17. Subgingival calculus
a) Materia alba b) Linea alba a) cannot occur without supragingival calculus
c) Plaque d) Calculus b) contains less hydroxyapatite than supragingival calculus
(MAN -2002) c) is uncommon in children
8. The yellowish white soft sticky deposit loosely adherent d) derives its minerals from same source as supragingival
present on tooth surface is calcu lus
a) Materia alba b) Food debris (AP-09)
c) Plaque d) Calculus 18. Calculocementum
(AIIMS - 92) a) Non-calcified structures
9. Materia alba b) Calculus embedded deeply in cementum
a) Lack microorganisms c) Non-attachment mechanisms
b) Has an irritating effect on gingiva d) Non similar to cementum
c) Has a regular internal pattern (GCET-14)
d) Is only salivary protein and food debris 19. The consistency of the subgingival calculus is described
(AP - 98) to be:
10. Which of the following is true in case of AIDS patient? a) Clay like b) Flint like
a) Acute pain c) Brick like d) Soft
b) Diffuse red lesion of the attached gingiva. (AIPG-06)
c) Gingiva covered with pseudomembrane 20. The proteins found in attachment plaque
a) Albumin and globulin

1) D 2) D 3) D 4) A 5) A 6) B 7) A 8) A 9) B 10) B 11) C 12) D 13) C


14) B 15) B 16) A 17) C 18) B 19) B 20) D
Dental ;lut.,e

b) Amelogenin and enamelin


c) Sialoproteins
d) Desmoplakin and plakoglobin
(KCET-08)
21. The difference in the colour between sub and
supragingival calculus is related to?
a) pH of plaque b) Death of leucocytes
c) Hemolysis of erythrocytes
d) All of the above
(AIPG-09)
22. Predominant inorganic component of supragingival
calculus is:
a) Magnesium phosphate b) Calcium carbonate
c) Calcium phosphate d) Carbon dioxide
(IGNOU-10)

21) C 22) C
, PERIODONTICS
~

V
543

4. CALCULUS & IATROGENIC DISEASES - ANSWERS

1. ' D' [Carra nza 8th ed 158] 9. ' B' [Carranza 11th ed 296 / 10th ed 176]
STAIN CAUSE The irritating effect of material alba on the gingiva is caused
by bacteria and their products.
Brown stain Lack of oral hygiene
Tobacco stain tenacious dark brown or black deposit 10. ' B' [Carranza 11th ed 238 / 10th ed 528]
Black stain Chromogenic bacteria. The periodontal lesions seen irn HIV-infected patients are:
Green stain Chromogenic bacteria like aspergillus • Linear gingival erythema / HIV-Gingivitis: The
erythematous, easily bleedi ng gingiva may be limited
and penicillium
to marginal gingiva or may be diffuse extending into
Orange stain Serratia marcescens attached gingiva.
Ch lo rh exidine Yellowish brown discoloration of • Necrotising ulcerative gingivitis.
stain teeth due to prolonged use. • Necrotising ulcerative periodontitis or HIV-periodontitis.
2. ' D' 11. 'C' [Carranza 11th ed 298]
Triclosan, pyrophosphate and zinc salts inhibit Factors causing vertical food impaction are:
mineralisation, so used as anti-tartar agents.
• Uneven occlusal wear.
3. ' D' [Carranza 11th ed 293 / 10th ed 173] • Opening of the contact point.
Methods of attachment of calculus to tooth: • Improperly constructed restorations.
• By means of an organic pellicle.
• Mechanical interlocking in surface irregularities. The cusps that tend to forcibly wedge food into interproximal
embrasures are known as PLUNGER CUSPS. Disto lingual
• Close adaptation to under surface depressions. cusp of upper 2"d molar is commonest plunger cusp.
Calculus embedded deeply into the cementum appearing
Lateral pressure from the lips, cheeks and tongue may
morphologically as cementum is called "calculocementum". force food interproximally. This is called as lateral food
impaction. This is more likely to occur when the gingival
4. 'A' [Carranza 11th ed 295 / 10th ed 175] embrasure is enlarged by tissue destruction in periodontal
Calculus does not directly irritate the gingiva but it provides disease.
a fixed nidus for the continued accumulation of plaque and
holds it against gingiva and thus acts as a contributing factor. 12. ' D' [Carranza 11th ed 225 / 10th ed 513]
CD/ T- Lymphocyte levels less than 200 /mm 3 are definitive
5. 'A' [Carranza 11th ed 293 / 10th ed 170]
for AIDS and indicate severe immunodeficiency.
Supra gingival calculus Sub gingival calculus
White or whitish yellow Dense, dark brown or greenish 13. 'C' [Carranza 11th ed 238 / 10th ed 528]
black
14. ' B' [Carranza 11th ed 239 / 10th ed 527]
Hard clay like consistency Hard flint like consistency
Mineral source is sa,liva so Mineral source is GCF and so 15. ' B' [Carranza 11th ed 433 / 10th ed 303]
called as salivary calculus called as serumal calculus Vitamin
Hydroxyapatite and octa- Hydroxyapatite and Periodontal manifestation
deficiency
calcium phosphate are magnesium whitlockite are Hyperkeratosis, hyperplasia of gi ngiva,
the major crystal forms the major crystal forms Vit A
increased pocket formation .
6. ' B' [Carranza 11th ed 293 / 10th ed 170] Vit D Osteoclastic resorption of alveolar bone.
Niacin Necrosis of gingiva.
7. 'A' [Carranza 11th ed 296 / 10th ed 176] Severe lesions of the gingiva and
Riboflavin
periodontal tissues.
8. 'A' [Carranza 11th ed 296 / 10th ed 176]
Necrosis of gingiva, periodontal ligament
Pellicle Salivary g lycoprotein with no bacteria Folic acid
and alveolar bone without inflammation.
Plaque Biofilm that is tenacious attached with abundant Haemorrhage, collagen destruction
bacteria and shows regular internal pattern. and edema of gingiva (boggy gums)
Vit C
Materia Yellow or white soft sticky deposit consisting interference with bone formation and
alba of bacteria but does not contain the regular retarded healing.
internal pattern
Dental ;lut.,e

16. 'A' [Carranza 11th ed 299]

17. 'C' [Textbook of Periodontology and Oral lmplantology by


Nayak 1st ed 84]

18. 'B' [Check Explanation of Q.3] (Calculo Cementum)]

19. 'B' (Carranza 8'h ed 151]

20. 'D' [Through internet sources]


Desmosomes are adhesion complexes that link various
cells to each ot her. They consist of two principal groups of
proteins i.e., desmosomal cadherins and cytoplasmic plaque
associated proteins. The cytoplasmic plaque associated
proteins include plakoglobin, desmoplakin, plakophilins,
envoplakin and periplakin.

21. 'C' [Carranza 10th ed 171]


Subgingival calculus is brown - black due to hemorrhagic
elements from GCF and black pigments from anaerobic rods.

22. 'C' [Carranza 11th ed 293 / 10th ed 172]


The inorganic content of supragingival calculus which
constitutes about 70-90% includes
• Calcium phosphate - 75.9%
• Calcium carbonate - 3.1%
• Magnesium phosphate - traces
, PERIODONTICS
~

V
545

5. GINGIVITIS
1. A thin bluish line around the gingival margin is due to b) Widening of capillaries and venules
the absorption of c) Increased plasma cells and neutrophils
a) Silver b) Mercury d) Infiltration of neutrophils, lymphocytes and plasma cells
c) Drugs like tetracyclines d) Lead (AIPG - 99)
(MAN -94) 12. Gingivitis
2. McCall's festoons are a) Inevitably progresses to periodontitis
a) Also called Stillman's Clefts b) Affects approximately 40% of adolescents
b) Due to poor oral hygiene c) Is characterized by true pocketing
c) Congenital abnormality d) Is a reversible lesion
d) Due Lo Lrauma rrom occlusion (AIPG - 90, AIIMS - 89)
(MAN -99) 13. Gingivitis is initiated most frequently by:
3. Most objective sign of gingivitis is a) Pregnancy b) Malocclusion
a) Increase in gingival size c) Vitamin deficiency d) Local irritating factors
b) Increase in crevicular fluid (AIPG - 90, 03)
c) Increase in gingival redness 14. The most common sequel of gingivitis:
d) Bleeding on probing. a) Pericoronitis b) Periodontitis
(MAN -2K) c) Periodontosis d) Periapical pathology
4. Life-saver shaped enlargements of the marginal gingival (AIPG -90, APPSC -99)
are called 15. Mc Call Festoons are seen in which teeth
a) Gingival crater b) McCall festoons a) Central Incisors b) Canine and Premolars
c) Stillman's cleft d) Window peaks c) Lateral incisors d) Molars
(MAN - 2K) (PGI - 99, 02)
5. Which of the following cells are predominant in the 16. Clinical signs of gingivitis appear in
stage 1 of gingivitis are a) Initial gingivitis b) Early gingivitis
a) Plasma cells b) Neutrophils c) Late gingivitis d) Advanced gingivitis
c) Mast cells d) Lymphocytes (PGI -02)
(MAN -2K) 17. Which of the following is not influenced by systemic
6. The most common form of gingival disease is: disease:
a) Gingivitis b) Periodontitis a) Periodontitis b) Destructive periodontitis
c) Gingival recession d) None of the above c) Hereditary fibromatosis gingivitis
(KAR -97) d) Chronic gingivitis
7. Earliest clinical sign of gingivitis is: (AIPG -2K)
a) Bleeding on probing b) Change in colour 18. A key feature that differentiates stage III gingivitis from
c) Change in contour d) Change in size the stage II lesion is an increase in the number of
(AIPG -91, PGI -03, AP- 05) a) Lymphocytes b) Plasma cells
8. Which single factor could contribute most of the c) Mast cells d) Polymorphs
development of simple gingivitis in children? (PGI -06, AIPG-07)
a) High fever b) Poor oral hygiene 19. Mc Calls festoon result from
c) Oral habits a) Normal physiological process
d) Physical nature of food ingested b) Tooth brush trauma
(KAR -2K) c) Inflammation d) Food lodgment
9. When P.D. probing in gingivitis done, depth is measured (PGI -03, 11)
from 20. Apostrophe shaped area
a) Base of pocket to CEJ a) Stillmans cleft b) Mc Calls Festoon
b) Base of pocket to free marginal gingiva c) Gingival sulcus d) None of above
c) Marginal gingiva to CEJ (PGI - 03)
d) Junctional epithelium to free gingival margin. 21. Bleeding on probing will tell us the:
(PGI -02) a) Gingival health of a child
10. Bacteria found in gingivitis are localized in: b) Status of gingival health on the day of examination
a) Connective tissue fibres b) Gingival sulcus c) 7 days after prophylaxis d) 14 days after prophylaxis
c) Alveolar bone d) Periodontal ligament (AIPG-96)
(AIPG - 98) 22. Pigmentation of chemicals in mucosa is due to
11. The histopathological picture of chronic gingivitis: a) Increased permeability of blood vessels
a) Leucocytic infiltration with increased neutrophils b) Local deposition of chemicals in mucosa

1) D 2) D 3) D 4) B 5) B 6) A 7) A 8) B 9) B 10) B 11) D 12) D 13) D


14) B 15) B 16) B 17) D 18) B 19) C 20) A 21) B 22) C
Dental ;lut.,e i======
c) Sub epithelial precipitation in connective tissue at the 34. Pregnancy gingivitis is caused by:
site due to increased capillary permeability a) Bacteriodes melaninogenicus
d) None of the above b) Actinobacillus actinomycetemcomitans
(PGI - 95) c) Streptococcus sanguis d) Fusobacterium
23. Features of gingivitis (AIIMS MAY 2012)
a) Inflamed tissue b) Bleeding on probing 35. When the dental plaque is allowed to accumulate in the
c) Increased Gingival fluid d) All of above absence of any plaque control procedures, it results in
(AP -02) gingivitis within
24. Gingival Recession is most common due to a) 1 to 3 days b) 7 to 21 days
a) Tooth brush trauma b) TFO c) 2 months to 3 months d) 24 hours to 48 hours
c) Deflective contacts d) ANUG (COMED-14)
(AP -04, AIIMS -95) 36. Atypical gingivitis is
25. Plasma cells are mostly seen in one of the stages of a) other name of plasma cell gingivitis
gingivitis b) gingivitis seen in HIV patients
a) Initial lesion b) Early lesion c) gingivitis seen in leukaemia patients
c) Established lesion d) Advanced lesion d) gingivitis seen in aggressive periodontitis
(KCET-08) (GCET-14)
26. In gingivitis, the im munoglobulins are consistent with 37. Not responsible for chronic gingival inflammation?
the increase in number of a) Neutrophil b) Macrophages
a) Fibroblasts b) Neutrophils c) T-lymphocytes d) Plasma cells
c) Lymphocytes d) Plasma cells (AIIMS NOV-14)
(AP- 08)
27. The following condition do not create gingival defects
necessitating gingivoplasty except
a) Acute necrotizing ulcerative gingivitis
b) Desquamative gingivitis
c) Erosive lichen planus
d) Acute herpetic gingivostomatitis
(AIIMS-08)
28. One common radiographic finding in clinical gingivitis is
a) Vertical bone loss b) Normal bone pattern
c) Horizontal bone loss d) Increase in bone density
(AIIMS-08)
29. The most important cell which contributes to the severity
of periodontal disease is
a) lymphocyte b) neutrophil
c) mast cell d) plasma cell
(AP-09)
30. The change in color, shape and texture of gingiva are
seen in?
a) stage 1 gingivitis b) stage 2 gingivitis
c) stage 3 gingivitis d) stage 4 gingivitis
(AIPG-14)
31. Collagen fibrils have a transverse striation with a
characteristic periodicity of -
a) 64 cm b) 64 nm
c) 640 µm d) 640 nm
(COMEDK-09)
32. Early & established stage of gingivitis is differentiated
by increase in number of
a) Plasma cells b) Neutrophil
c) B-cell d) Macrophages
(AIPG-10)
33. Which stage of gingivitis is also called subclinical
gingivitis -
a) Initial stage b) Advanced stage
c) Early stage d) Established stage
(KCET-09)

23} D 24) A 25} C 26) D 27) A 28} B 29) D 30} C 31} B 32} A 33} A 34} A 35) B
36) A 37} A
, PERIODONTICS
~

V
547

5. GINGIVITIS - ANSWERS
1. 'D' [Carranza 11th ed 113 / 10th ed 366] (int erleukin-8, C5a) and product s derived from t he
Heavy metals cause gingival pigmentation in areas of biofilm (li popolysaccharide).
inflammation only, where the increased permeability permits • The predominant cell in acute (Stage II) gingivitis is t he
the seepage of metal into the surrounding tissue. T-lymphocyte (75% of the total lymphocytes) .
• Junctional epit helium shows formation of retepegs and
Lead line Bluish red or deep blue linear wide intracellular spaces in stage II gingivitis and t hus
(Burtonian line) pigmentation in marginal gingiva bleeding on probing is evident before any colour changes
Silver (argyria) Violet marginal line (PGI Dec-13) . 70°1o of collagen is destroyed around t he
cellular infiltrate.
Bismuth, arsenic
Black marginal line • Then main fiber groups affected in Stage II appers to be
and mercury
circular and dentogingival assemblies.
Delicate, brown or black pigmented non
Mesentric line cariogenic plaque fou nd on the enamel • In St age III, extravasation of RBCs into the connective
at the cervical margin of the tooth tissue and breakdown of hemoglobin into its component
pigments can also deepen the colour of chronically
inflammed gingiva. Also t he junctional epit helium is no
2. 'D' [Carranza 11th ed 116 / 10th ed 3 70]
longer closely attached to the tooth surface.
• Stillman's clefts - apostrophe shaped indentations from
and into the gingival margin. • In Stage IV, t he junctional epit helium migrat es apically
from CEJ.
• MC calls festoons - Life preserver shaped enlargements
of the marginal gingiva occur most frequently in canine 6. 'A' [Carranza 11th ed 60]
and premolar areas. Gingivitis is the most common form of gingival disease. Only
Both stillman's clefts and Mc call's fest oons are some cases of gingivitis progress to periodontitis. About
considered t o be the result of t rauma from occlusion. But 90°1o of population is affect ed by gingivitis by the time they
this association was never substantiated. They represent cross t heir puberty.
peculiar inflammatory changes of the marginal gingiva.
7. 'A' [Carranza 11th ed 103 / 10th ed 364]
3. ' D' [Carranza 11th ed 110 / 10th ed 364]
The two earliest signs of qinqival inflammation are: 8. ' B' [Carranza 11th ed 60 / 10th ed 407]
Local conditions such as materia alba and poor oral hygiene
• Increased GCF production rate
favour t he accumulation of plaque.
• Bleeding on probing
Calculus formatio n is less common in children . Generalized
Bleeding is a more objective sign that requires less subjective calculus formation occurs in children with cystic fibrosis.
estimation by t he examiner. This is related to the increased concentrations of phosphate,
calcium and prot ein in t he saliva.
4. 'B' [Carranza 11th ed 116 / 10th ed 364]
9. 'B' [Check Explanation Below]
5. ' B' [Carranza 11th ed 103 / 10th ed 356] In gingivitis there is no true pocket formation. Whatever the
Predomi- probing depth measured is the depth of the gingival sulcus
Stage Time Findings
nant cell
10. 'B' [Carranza 11th ed 107 / 10th ed 355]
I (Initial) 2-4 days PMN's i GCF flow
Bacteria fo und in gingivitis are localized in the gingival
Vascular sulcus. There is no connective tissue invasion. In
II T-
4-7 proliferation, periodontitis the microorganisms are localized in connective
(Early or lympho-
days erythema, bleeding tissue and result in periodontal destruction.
acute) cytes
on probing.
III (Chronic Bluish to reddened 11. 'D' [Carranza 11th ed 106 / 10th ed 359]
14 - 21 Plasma
or gingiva, changes in Engorgement of blood vessels, bluish red appearance of
days cells gingiva and changes in the size, texture are the clinical
Established) texture, size etc
IV Connective tissue features in stage III or chronic gingivitis.
(advanced)
- - break down
Histologically there is infi ltration of connective tissue and
junctional epithelium with neutrophils, lymphocytes wit h
Important MCOs
predominance of plasma cells.
• The recruit ment of PMNs in stage I gingivitis is
due t o chemoattract ant actions of the host syst em 12. 'D' [Carranza 11th ed 61]
Dental ;lut.,e i======
13. ' D' [Carranza 11th ed 60] 28. 'B' (Ca rranza 11th ed 109 / 10th ed 362]

14. ' B' (Carranza 11th ed 107 / 10th ed 360] 29. 'D' (Carranza 11th ed 104 / 10th ed 438, 360]
Extension of inflammation into the supporting structures
occurs in stage IV (Advanced) gingivitis which is the phase 30. ' C' (Carranza 11th ed 106]
of periodontal breakdown. In chronic (stage III) gingivitis, the blood vessels become
engorged and congested, venous return is impaired and
Periodontitis is always preceded by gingivitis, but not all blow flow becomes sluggish. This results in reddened
gingivitis progresses to periodontitis. The transition from gingiva with superimposed bluish hue. Also the breakdown
gingivitis to periodo,ntitis occurs with increase in the of haemoglobin into its component pigments can further
number of spirochetes, motile organisms and when the deepen the color.
preponderant T-Cell lesion becomes a 8-lymphocytic lesion.
31. 'B' [Ca rranza 11th ed 28 / 10th ed 69]
15. ' B' (Carranza 11th ed 116 / 10th ed 3 70]
32. 'A' [Carranza 11•h ed 104 / 1o•h ed 359,360]
16. ' B' (Carranza 11th ed 104 / 10th ed 358]
Stage I (or) initial gingivitis is subclinical. Erythema and 33. 'A' (Carranza 11th ed 103 / 10th ed 356]
bleeding on probing occurs in stage II (or) early lesion.
34. 'A' [Carranza 11th ed 346]
17. ' D' • Prevotella intermedia species are increased during
Options A, B and C are influenced by systemic diseases. pregnancy gingivitis whereas capnocytophaga species
are increased in puberty gingivitis.
18. ' B' (Carranza 11th ed 104 / 10th ed 356]
• Refer synopsis point 11 for the new names of bacteroides
Refer question No. 5
species.
19. ' C' (Carranza 11•h ed 116 / 10th ed 370] • Prevotella is a new genus that includes bacteroides
intermedia, bacteroides melaninogeni us and related
20. 'A' [Carranza 11th ed 116 / 10th ed 3 70] species.

21. ' B' (Carranza 11th ed 110 / 10th ed 364] 35. ' B' [Carranza 11th ed 106]
Chronic gingivitis occurs 2-3 weeks after the beginning of
22. ' C' (Carranza 11•h ed 113 / 10th ed 366] plaque accumulation.

23. ' D' [Carra nza 11th ed 110 / 10th ed 355] 36. 'A' [Shafer 6th ed 393]
Other names of plasma cell gingivitis are atypical gingivitis
24. 'A' (Carranza 11th ed 115] or plasma cell gingivostomatitis or plasmacytosis of gingiva.
Etiological factors tor gingival recession:
• Faulty tooth brushing technique 37. 'A' [Carranza 11th ed 72]
• High frenal attachment
• Gingival inflammation
• Tooth malposition
• Gingival ablation

25. 'C' [Carra nza 11•h ed 104 / 10th ed 355]


Stage Time Predominant cells
Initial Lesion 2 - 4 days PMNS
Earlier lesion 4 - 7 days Lymphocytes
Established lesion 14 - 21 days Plasma cells
Plasma cells and
Advanced lesion > 21 days
neutrophils

26. 'D' [Carranza 11th ed 106 / 10th ed 359]


The established lesion of gingivitis is characterized by
predominance of plasma cells and B-lymphocytes. The
8-cells found in established lesion are predominantly IgG1
and IgG3 subclasses.

27. 'A' [Carranza 11th ed 609 / 10th ed 914]


, PERIODONTICS
~

V
549

6. GINGIVAL ENLARGEMENTS

1. "Tuberous sclerosis" is seen in 12. Phenytoin causes gingival:


a) Vascular fibroma b) leprosy a) Inflammation b) Hyperplasia
c) t uberculosis d) bone disorders c) Atrophy d) Aplasia
(MAN - 2K) (AIPG - 99)
2. Gingival enlargement is a side effect of 13. Cyclosporine enlargement of gingiva can be classified as:
a) Phenytoin b) Calcium channel blockers a) Non inflammatory b) Inflammatory
c) Acute myelocytic leukemia c) Developmental d) Malignant
d) All of the above (PG! - 98)
(MAN -02, KAR -00) 14. The gingival enlargements in leukemia mainly results from
3. Diffuse gingivitis indudes a) as an inll.ammalory reaclion Lo plaque
a) Marginal gingiva b) leukemic cellular infiltration of gingiva
b) Marginal and papillary gingiva c) Hormonal disturbances d) Developmental in origin
c) Marginal, papillary and attached gingiva (AP - 98)
d) Marginal, papillary, attached and alveolar gingiva 15. The conditioned gingival enlargement
(AP -2K) a) Requires presence of local irritants
4. Changes in the gingiva during pregnancy are attributed to b) Exacerbates existing inflammation
a) Changes in the hormonal level c) Pregnancy & puberty are the examples
b) Altered microorganisms d) All of above.
c) Altered immunological responses level (AP - 03)
d) All the above 16. An example of acute inflammatory gingival enlargement
(MAN - 01) is most likely to be seen in patient exhibiting:
5. Pyogenic granuloma: a) Periodontal cyst b) Pregnancy gingivitis
a) Bleeds on touch b) Painless c) Dilantin hyperplasia d) Hereditary fibromatosis
c) Soft in consistency d) All of the above (AIIMS - 89)
(AIPG - 93) 17. Plasma cell gingivitis is seen in individuals having
6. Drugs associated with gingival changes are: the habit of chewing which of the following types of
a) Phenytoin, cystine and nifedipine flavoured chewing gum?
b) Nifedipine, ibuprofen and lignocaine a) Peppermint flavoured b) Clove flavoured
c) Cyclosporin, chlorine and iodine c) Cinnamon flavoured d) Banana flavoured
d) Phenytoin, hydrogen, hydrogen peroxide and paracetamol (AIPG -89)
(PGI -00, 01) 18. Pyogenic granuloma is
7. Dilantin hyperplasia is treated with a) Granulation tissue overgrowth/ Non-Specific conditioned
a) Gingivectomy b) Gingivoplasty enlargement
c) Apically repositioned flap d) Curettage b) Capillary hemangioma
(AP -97) c) Sarcoidosis d) Tuberculoid leprosy
8. Factor responsible for pregnancy gingivitis is: (KAR -97, APPG-15)
a) Hormonal imbalance 19. A young adult shows non-fluctuant, tender and red
b) Micro-organism associated with plaque swelling in the marginalgingival lesion. This is mostlikely:
c) Poor oral hygiene d) Drugs a) Periodontal abscess b) Periapical abscess
(PG! - 99) c) Gingival abscess d) Periapical sinus
9. Local irritating factors in gingiva is most likely give rise to: (AIPG -04)
a) Pyogenic granuloma 20. Gingival abscess involves
b) Generalized fibrous hyperplasia of gingiva a) Marginal b) attached
c) Mucosal cobblestoning d) Periapical abscess c) inter dental d) marginal & interdental
(AIIMS - 94, 99) (AP - 02)
10. Gingival enlargement in pregnancy is termed: 21. Which organism is increased in pregnant ladies
a) Periodontal abscess b) Angio granuloma a) Porphyromonas intermedius
c) Gingival abscess d) Wegenere's granulomatosis b) Porphyromonas gingivalis
(KAR - 97) c) Porphyromonas melaninogenicus
11. Which of the following drugs is associated with gingival d) Eichenella corrodens
hyperplasia? (PG! - 02)
a) Dihydropyridines b) Tetracyclines 22. Which type of enlargement is seen in puberty?
c) Cyclosporin d) Metronidazole a) Interdental papillae appear bulbous but facial gingiva is
(KAR -97, AIPG -96) not affected

1) A 2) D 3) C 4) A 5) D 6) A 7) A 8) B 9) A 10) B 11) C 12) B 13) A


14) B 15) D 16) B 17) C 18) A 19) C 20) D 21) A 22) B
Dental ;lut.,e

b) Interdental papillae and facial gingiva both are enlarged


c) Interdental gingiva, marginal gingiva and attached
gi ngiva all are enlarged
d) Both marginal gingiva and attached gingiva are enlarged
(AIPG-06)
23. Folic acid deficiency is precipitated by:
a) Aspirin b) Ranitidine
c) Cyclosporin d) Phenytoin
(AIIMS-06)
24. Conditional gingival enlargements is usually not:
a) Hormonal b) Leukemic
c) Granuloma pyogenicum d) Drug induced
(AIPG-05)
25. The predominant fibre groups affected in the early lesion
of gingivitis are:
a) Circular and dentogingival
b) Circular and horizontal
c) Circular and alveolar crest
d) Circular and transseptal
(KAR-04)
26. Based on Bokenkamp's grading of enlargements, an
enlargement covering 3/ 4th of the crown is considered
as
a) Grade I b) Grade II
c) Grade III d) Grade IV
(AP-2012)
27. Frequent periodontal abscesses appear to be an
important periodontal feature in
a) Nutrition deficiency b) Diabetes Mellitus
c) Hormonal imbalance d) Cardiovascular diseases
(COMED-14)
28. Test tube retepegs are seen in the microscopic section of
a) ANUG
b) Inflammatory gingival hyperplasia
c) Gingival enlargement due to dilantin sodium
d) Plasma cell gingivitis
(COMED-14)
29. Parietal abscess is also known as
a) Gingival abscess b) Periodontal abscess
c) Periapical abscess d) Periodontal cyst
(COMEDK-15)

23) D 24) D 25) A 26) C 27) B 28) C 29) B


, PERIODONTICS
~

V
551

6. GINGIVAL ENLARGEMENTS - ANSWERS

1. 'A' [Carranza 11th ed 123 / 10th ed 380] 6. 'A' [Carranza 11th ed 122 / 10th ed 375]
Idiopathic gingival enlargement has been described
in tuberous sclerosis which is an inherited condition 7. 'A' [Carranza 11th ed 835 / 10th ed 920]
characterized by a triad of epilepsy, mental deficiency and
cutaneous angiofibromas. 8. ' B' [Carranza 111h ed 124 / 10th ed 380]

2. ' D' [Carranza 11th ed 120 / 10th ed 375] 9. 'A' [Carranza 11•h ed 126]
Fibrotic gingival enlargement is a side effect of some
anti convulsants (phenytoin), calcium channel blockers 10. ' B' [Carranza 11th ed 124 / 10th ed 381]
(Nifedipine (AIIMS MAY 2012), Nilrendipine) and Gingival enlargemenl in pregnancy is a marginal or discrele
immunosuppresant drugs (cyclosporine). tumor like lesion and is known as pregnancy tumor or
fibrohaemangioma or angiogranuloma.
Leukemic gingival enlargement occurs only in acute type and
not in chronic leukemia. It occurs mostly in acute monocytic 11, 'C' [Carranza 11th ed 122 / 10th ed 378]
leukemia. The gingival connective tissue is infiltrated with Cyclosporine is a potent immunosuppressive drug used
immature leukocytes. to prevent organ transplant rejection and treat several
autoimmune diseases. Gingival enlargement, nephrotoxicity,
3. 'C' [Carranza 11th ed 118 / 10th ed 373] hypertension etc are its side effects.
Gingival enlargement Location
Another immunosuppressive tacrolimus exhibits less
• Chronic inflammatory severe gingival enlargement and is used as a substitute for
• Gingival abscess Marginal and interdental cyclosporine. (KCET-11)
• Drug induced gingiva
12. ' B' [Carranza 11th ed 122 / 10th ed 377]
• Pregnancy, puberty
Phenytoin (dilantin) is used to treat all forms of epilepsy
• Idiopathic except petitmal.
• Fibrotic Marginal, interdental and
• Leukemic attached gingiva Gingival enlargement occurs in about 50% of patients taking the
drug and often more in younger patients, phenytoin stimulates
• Plasma cell gingivitis
the fibroblasts and reduces the collagen degradation.

Gingival enlargement is associated with (COMED-14) Ethosuximide, valproic acid, mephenytoin are other
a) Anemia b) Hyperpigmentation antiepileptic drugs, which also cause gingival overgrowth.
c) Leukemia e) Hyperthyroidism
13. 'A' [Carranza 11th ed 122 / 10th ed 378]
4. 'A' [Carranza 11th ed 124 / 10th ed 380]
The factor that is responsible for pregnancy gingivitis is the 14. ' B' [Carranza 11th ed 126 J
dental plaq ue. The increased hormonal levels of progesterone Leukemic enlargements generally bluish red and has a shiny
and estrogen induce changes in vascular permeability surface. The enlargement has both features of leukemic
leading to gingival edema and an increased inflammatory infiltrate and chronic inflammation but mainly results from
response to dental plaque. leukemic infiltration of gingiva.

The three types of conditioned gingival enlargements are 15. ' D' [Carranza 11th ed 124 / 10th ed 383]
hormonal (Pregnancy and Puberty), nutritional (Vit. C) and
allergic (plasma cell) (MCET-07). 16. ' B' [Carranza 11th ed 124 / 10th ed 380]

Which of the following isNOT a conditioned 17. 'C' [Drugs, disease on periodontium 88]
enlargement of the gingiva ? (KAR-2013) The gingiva in plasma cell gingivitis appears red, friable,
a) Pregnancy b) Puberty granular and bleeds easily. It may be associated with
c) Vitamin C deficiency d) Granulomatous disease cheilitis and glossitis. It is allergic in origin possibly related
to components of chewing gum, dentifrices or diet.
5. ' D' [Carranza 11th ed 126 / 10th ed 383 J
Pyogenic granuloma is a nonspecific conditioned gingival Mint or scinnamon is the etiologic agent
enlargement (APPG-15). It is a discrete bright red or
purple tumor like lesion and is friable with surface ulceration l8. 'A' [Carranza 11th ed 126 I 10th ed 383]
and purulent exudation. It is considered as an exaggerated
response to minor trauma (irritants).
Dental ;lut.,e

19. 'C' [Carranza 11th ed 119 / 10th ed 375]


Gingival abscess is a localized, painful rapidly expanding
lesion of sudden onset. It is limited to marginal gingiva (or)
interdental papilla. It is due to foreign substance carried
deep into the tissues such as a tooth brush bristle, a piece
of apple core or a lobster shell.

20. 'D' [Carranza 11th ed 119 / 10th ed 375]

21. 'A' [Carranza 11th ed 346 / 10th ed 380]


• The organism that is significantly increased in pregnancy
is P. intermedia.
• The organism that initiates puberty gingivitis is the
Capnocytophaga species.

Pregnancy gingivitis begins in the 2 nd or 3 rd month and


increases upto 81h month and decreases during 9th month.

Maximum gingivitis during pregnancy occurs during


which months [PGI-2011)
a) 1st month b) 4th month
c) 8th month d) 9t h mont h

22. 'B' (Carranza 11th ed 125 / 10thed 381]


Refer to explanation of Q.No. 3

23. 'D' [Carranza 11th ed 122 / 10'hed 378]


Administration of phenytoin may precipitate megaloblastic
anemia and folic acid deficiency.

24. 'D' [Carranza 11•h ed 124/ 10'hed372]

25. 'A' [Carranza 11th ed 104 / 10thed 358]

26. 'C' [Carannza 11th ed 118 / 10th ed 374]


Grade 0 No signs of gingival enlargement.
Grade I Enlargement confined to interdental papilla.
Enlargement involves papilla and marginal
Grade II
gingiva.
Enlargement covers three quarters or more of
Grade III
the crown.

27. 'B' [Carranza 11th ed 417]

28. ' C' [Carranza 11th ed 121]


Drug induced gingival enlargements show elongated test
t ube shaped retepegs extending deep in to the connective
tissue.

29. 'B' [Carranza 11th ed 137]


Periodontal abscess is also known as parietal or lateral
abscess.
, PERIODONTICS
~

V
553

7. ACUTE GINGIVAL INFECTIONS


1. Punched-out, crater-like depressions at the crest of the d) Necrosis perforating skin of cheek
gingiva, covered by gray pseudo- membranous slough, (KCET-2012)
which is demarcated from the remainder of the gingiva 12. A 15-year-old-boy suffers from vesiculating painful
by linear erythema, are characteristic of ulcers, there is also oral temperature of 103 ° F since 3
a) Desquamative gingivitis b) Erythema multiforme days and lymphadenopathy. The most likely diagnosis is:
c) Lichen planus d) Vincent's stomatitis a) Herpes zoster
(MAN -94, PGI -01) b) Acute herpetic gingivostomatitis
2. ANUG is caused by c) Major apthae d) Recurrent aphthae
a) Borrelia vincenti and fu sobacterium (AIPG -90)
b) Aclinomycelam comilans 13. Gingival deformities are seen in:
c) Actinomyces nauseladii a) ANUG b) Internal resorption
d) Streptococcus mutans c) Cementomas d) Periapical cysts
(MAN - 02) (AIIMS -99)
3. ANUG affects the 14. Drug of choice given in ANUG:
a) Attached gingiva b) Marginal gingiva a) Penicillin b) Metronidazole
c) Alveolar mucosa d) Inter dental papilla c) Both of the above d) Tetracycline
(AP -2K, MAN -02) (PGI - 98, KAR - 98, KEALA-15)
4. Mouthwash used in ANUG is 15. Treatment of ANUG utilizes all except:
a) Chlorhexidine b) Normal saline a) Debridement b) Oral hygiene
c) Hydrogen peroxide d) Stannous fluoride c) Penicillin d) Topical steroid
(AP - 2K, MAN - 2K) (AIIMS -95)
5. Bacterial invasion of connective tissue in ANUG was 16. An 18-year-old-female patient with swelling and pain in
demonstrated by OR EM study of four zones of ANUG was the mandibular third molar area will most likely to have:
described by a) juvenile periodontitis b) pericoronitis
a) Glickmann b) Listgarten c) pregnancy epulis d) plasma cell gingivitis
c) Lindhe d) Waerlug (AIPG -92)
(MAN -01, COMEDK- 06) 17. A 59-year-old female presents with gingival condition
6. The predominant organism found in smears of ANUG is punched out crater-like depression covered with
a) Vibro b) Fusobacterium membrane which can be pulled off painlessly?
c) Spirochetes d) Streptococci a) Diphtheritic lesion
(KAR -98, 00) b) Acute herpetic gingivostomatitis
7. Which of the following may create gingival deformities c) ANUG d) Syphilitic lesion
that require gingivoplasty to eliminate the defects: (AIIMS -92)
a) Erosive Lichen planus b) Desquamative gingivitis 18. Tobacco Chewing/ smoking is the most common
c) Acute Herpetic gi ngivostomatitis predisposing cause of which of the following:
d) Necrotising ulcerative gingivitis a) Cyclic neutropenia b) Juvenile periodontitis
(AIPG -01, 02) c) Juvenile periodontitis
8. ANUG is best visualized d) Necrotising ulcerative gingivitis (ANUG)
a) Crater like lesions b) Severe gingivitis (AIIMS MAY 2012, APPG-15)
c) Recession of gingiva d) None 19. Treatment of acute herpetic gingivostomatitis include :
(PG! -02) a) Corticosteroids to reduce inflammation
9. Color changes seen in ANUG may be b) Penicillin
a) Marginal b) Diffuse c) Local antibiotic application
c) Patch-like d) Lace-like d) None of the above
(AP-2012) (AIIMS -90, AIPG -01)
10. Laboratory test of ANUG is: 20. Metronidazole is the drug of choice for
a) Complement fixat ion b) Dark fie ld examination a) All periodontal disease b) ANUG
c) Tissue culture d) Animal inoculation c) Juvenile periodontitis d) Herpes simplex
(PGI - 01) (AP -01)
11. Stage IV of ANUG according to Hornig and Cohen is 21. Extractions can be done in ANUG patients after
characterized by a) 4 weeks b) At 1st appointment only
a) Necrosis of the ti p of interdental papilla c) 4 months d) 1 week
b) Necrosis extending to the attached gingiva (AP -01)
c) Necrosis exposing alveolar bone

1) D 2) A 3) D 4) C 5) B 6) C 7) D 8) A 9) A 10) B 11) B 12) B 13) A


14) C 15) D 16) B 17) C 18) D 19) D 20) B 21) A
Dental ;lut.,e i======
22. Primary herpetic gingivostomatitis usually occurs: c) 40-50% d) 50- 100%
a) Prior to age 10 b) Between ages 13 to 30 (COMEDK-07)
c) At the onset of the menstrual period 29. In the management of patients with Vincent's infection,
d) During menopause the preferred therapeutic agents are
(AIIMS -NOV 2K) a) Hydrogen peroxide mouthwasl, and metronidazole
23. Not a contributing factors for ANUG b) Cephalosporins and hydrogen peroxide mouthwash
a) Pericoronal flap b) Poor oral hygiene c) Chlorhexidine mouthwash and metronidazole
c) Smoking d) Apthous ulcers d) Crystalline penicillin and chlorhexidine mouthwash
( PGI - 97) (UPSC-09)
24. A 30 years old male reported with chief complaint of 30. A probable etiology of gingivosis is?
constant radiating, gnawing pain that is intensified a) High progesterone levels
by eating spicy foods and chewing. He also reported b) Deficiency of estrogen and testosterone
a "metallic" foul taste and an excessive amount of c) Pregnancy d) Aldosterone deficiency
"pasty"saliva. A general examination revealed fever (AIPG-09, 11)
and increased pulse rate. Intra-oral examination showed 31. According to Horning and Cohen staging of oral
punched-out, crater like depressions at the crest of necrotizing diseases stage one represents
the interdental gingival papillae, covered by a gray a) NUG b) NUP
pseudomembranous slough, in the upper anterior teeth. c) Necrotizing stomatitis d) Noma
(COMEDK-2011, 15) (KCET-10)
A) The most likely infection is 32. Which of the following can lead to gnawing radiating pain?
a) Necrotizing ulcerative gingivitis a) Herpetic stomatitis b) ANUG
b) Localized aggressive periodontitis c) Chronic Periodontitis d) None
c) Acute herpetic gingivostomatitis (PGI-08)
d) Desquamative gingivitis 33. Which of the following condition if not treated, can lead
to indifferent course?
B) This infection is reported to have an increased a) Diphtheria b) Herpangina
incidence in c) Herpetic Gingivostomatitis
a) Diabetes mellitus b) HIV d) ANUG
c) Tuberculosis d) Syphilis (PGI-08)
34. Histological examination of the tissues in desquamative
C) Surgical procedur,e for severe tissue destruction in gingivitis would reveal?
such infection is a) Hyperkeratosis
a) Reshaping the gingiva (Gingivoplasty) b) Elongation of rete pegs
b) Resection of the gingiva (Gingivectomy) c) Loss of basement membrane
c) Apically displaced flap surgery d) No inflammatory response
d) Coronally displaced flap surgery (AIIMS MAY- 2012, PGI JUNE- 14)
35. Which of the following is most useful in differentiating
25. Leukemic gingivitis resembles an acute periodontal abscess from periapical abscess?
a) ANUG b) Thrombo cytopenic purpura a) Type of exudat es b) Nature of swelling
c) Herpetic Gingivost omatitis c) Intensity of pain d) Result of periodontal probing
d) Bullolus pemphigus (AIPG-09)
(PGI-05) 36. Which of the following periodontal disease is associated
26. All of the following are correct with regard to acute with haematological disorder?
necrotizing ulcerative gingivitis except a) AIDS b) Hypophosphatemia
a) There is necrosis of the interdental papilla c) Wegeners Granulomatosis
b) Sloughing of the necrotic tissue presents as a d) Histiocytoses - X
pseudomembrane over the tissues (AIPG-10)
c) It is associated with decreased resistance to infection 3 7. An 18yr old male patient reported with chief complaint
d) It causes chronic inflammation of the gingiva of pain and swelling in the right lower jaw and mild
(AIPG-06) difficulty in opening mouth since one week. A general
27. Most common cause of chronic inflammation of the examination revealed fever and increased pulse rate.
gingiva in a preschool child (COMEDK-2013)
a) Acute herpetic gingivostomatitis 37A. Intra-oral examination showed signs of inflammation
b) Acute necrotisi ng gingivitis around the gingiva of partially impacted third molar
c) Apthous stomatitis d) Vitamin B deficiency tender on palpation with opposing tooth impinging. The
(KAR-05) most likely diagnosis is
28. Incidence of Gingivitis in pregnency is a) Necrotising ulcerative gingivitis
a) 10-20% b) 20-30% b) Localised aggressive periodontitis

22) A 23) D 24A) A 248) B 24C) A 25) A 26) D 27) A 28) D 29) A 30) B 31) A 32) B
33) D 34) C 35) D 36) A 37A) D
, PERIODONTICS

c) Acute herpetic giingivostomatitis 41. Metronidazole is NOT effective in the treatment of


d) Pericoronitis a) Amoebiasis
b) Infections due to Bacteroides fragilis
3 78. What is the microbial etiology of this condition is? c) Infections due to Pneumocystis carinii
a) Klebsiella b) E. Coli d) Acute necrotizing ulcerative gingivitis
c) Streptococci and anaerobic bacteria (COMEDK-15)
d) Actinomycosis 42. Necrotizing Ulcerative Gingivitis has been demonstrated
to be
37(. What would be the ideal management of this case? a) Communicable but not transmissible
a) Antibiotics and Analgesics. b) Both communicable and transmissible
b) Removal of partially impacted lower molar immediately. c) Transmissible but not communicable
c) Irrigation, antibiotics and analgesics. d) Neither transmissible nor communicable
d) Hospitalisation, Irrigation, antibiotics and analgesics (MHCET-15)
and immediate removal of opposing tooth.

3 7D. The ideal surgical management of this condition once


the signs and symptoms subside would be
a) Operculectomy.
b) Delaying the removal of teeth to next episode.
c) Removal of tooth under local anaesthesia.
d) No treatment.

38. Trench mouth is caused by


a) Fusiform bacillus b) Borelli Vincenti
c) Both (a) and (b) d) None of the above
(COMEDK-2013)
39. In a patient of trench mouth, interproximal architecture
has been lost, and having history of smoking, the
antibiotic or combination required to treat this is:
a) Penicilline +Metronidazole
b) Cephalosporins
c) Norfloxacin d) Sulfamethoxazole
(NEET-2013, AIIMS- 2012)
40. 48 year old male reported with the chief complaint of
ulcer on the gums and on general examination headache,
fever and malaise was observed. On oral examination,
sharply punched out crater-like erosions of the
interdental papillae of sudden onset were noted. The
patient also complained of bad breath.
(COMED-14)
40A. Diagnosis is
a) ANUG b) Pericoronitis
c) Chediak Higashi diseases
d) Desquamative gingivitis

408. Inthe first visit these patient should be treated with


a) Antifungal drugs
b) Deep scaling and root planning
c) Superficial scaling, debridement, chlorhexidine
mouthwash
d) Mucogingival surgery

40(. This patient's lower anterior marginal gingiva showed


comma-shaped defect called
a) Stillman's cleft b) Dehiscence
c) Fenestration d) Cul-de-sac

378) C 37C) C 37D) C 38) C 39) A 40A) A 408) C 40() A 41) C 42) C
Dental ;lut.,e

7. ACUTE GINGIVAL INFECTIONS - ANSWERS


1. 'D' [Carranza 11th ed 133 / 10th ed 391) ANUG Marginal gingiva
ANUG is also called as vincents infection, vincents stomatitis,
trench mouth, fetid stomatitis, putrid stomatitis. Herpetic gingivitis Diffuse
Chemical irritation Patch like or diffuse
2. 'A' [Carranza 11th ed 134 / 10th ed 393) Red or bluish red colored marginal
Smears from the lesions of ANUG demonstrate spirochetes Chronic gingivitis
gingiva
(Borrelia vincenti) and fusiform bacilli (fusobacterium).
This symbiotic infection is known as fusospirochetosis. Both 10. 'B' [Carranza 11th ed 312 / 10th ed 393)
ANUG and vincents angina are fusospirochaetal infections. Routine gram staining cannot stain the spirochetes. Darkfield
microscopy is used for identification of spirochaetes.
Vincent's angina is caused by (AP- 2013)
a) Streptococci b) Corynebacterium 11. ' B' [Carranza 10th ed 392)
c) Fusospirochaetal complex d) Treponemapallidum Necrosis of the tip of the interdental papilla
Stage 1
(93%)
3. 'D' [Carranza 11th ed 133 / 10th ed 391)
Punched out, crater-like depressions at the crest of Stage 2 Necrosis of the entire papilla (19%)
interdental papillae, covered by pseudomembrane and Stage 3 Necrosis extending to the gingival margin (21 %)
extending into the marginal gingiva are characteristic of Necrosis extending also to the attached gingiva
ANUG. The pseudomembranous slough is demarcated from Stage 4
(1%)
the remaining gingival mucosa by linear erythema. Attached
gingiva and alveolar mucosa are rarely involved. Necrosis extending into buccal or labial mucosa
Stage 5
(6%)
4. 'C' [Carranza 11th ed 608 / 10th ed 707] Stage 6 Necrosis exposing alveolar bone (l°!o)
3°10 H20 2 with equal dilution of water every 2 hours for 3 days Stage 7 Necrosis perforating skin of cheek (0%)
gives best results in patients with ANUG. H20 2 creates an
oxidative environment and prevents the growth of anaerobic 12. 'B' [Carranza 11th ed 139 / 10th ed 399]
organisms. Herpetic gingivostomatitis is the primary infection of
oral cavity caused by HSV Type I virus. High-grade fever,
3°10 H20 1 diluted to one part peroxide and 2 parts water lymphadenopathy, soreness of mouth, vesicles, which
twice a day is recommended in patients with desquamative rupture to form ulcers, are characteristic. The ulcers are
gingivitis. H1 O1 mouthwash is also recommended in . typically present in masticatory mucosa in contrast with
suppurative lesions such as periodontal abscess and rn apthous ulcers, which occur, predominantly in labile mucosa.
diabetic patients. Preschool children are affected mostly by acute herpetic
gingivostomatitis.
5. 'B' (Carranza 11th ed 134)
Listgarten described the 4 zones in ANUG. They are bacterial 13. 'A' [Carranza 11th ed 133 / 10th ed 391)
zone, neutrophil rich zone, necrotic zone and zone of
spirochetal infiltration. 14. 'C' [Carranza 11th ed 608 / 10th ed 707)

6. 'C' [Carranza 10th ed 393] 15. 'D' [Carranza 11th ed 608 / 10th ed 707)
Spirochetes are classified into three morphologic groups - Corticosteroids are contraindicated in ANUG patients.
small, intermediate and large. Intermediate spirochetes are
present in great numbers in ANUG lesions. 16. 'B' [Carranza 11th ed 141 / 10th ed 401]
Red, painful, swollen and tender gingiva, radiating pain to
7. 'D' [Carranza 11th ed 609 / 10th ed 709) ear, throat, foul taste and inability to close the mouth are
Shelf like gingival margins alter healing may result in plaque characteristic features of pericoronitis.
accumulation. The gingiva is reshaped with periodontal
knife or with electro surgery. This process of eliminating the 17. ' C' [Carranza 11th ed 136 / 10th ed 395)
defects by reshaping is called as Gingivoplasty. Pseudomembrane formation is seen in ANUG, diphtheria,
erythema multiformae, leukemic ulcers, syphilis and
8. 'A' [Carranza 11th ed 133 / 10th ed 391) candidiasis. Pseudo membrane can be easily removable in
ANUG, undetachable in syphilis and is difficult to remove in
9. 'A' [Carranza 11th ed 133 / 10th ed 366) diphtheria.
Colour changes in acute gingival infections differ in both
nature and distribution from chronic gingivitis. 18. ' D' [Carranza 11th ed 13 7 / 10th ed 396]
, PERIODONTICS
,.....__,,_

V
557

19. ' D' [Carra nza 11th ed 611 / 10th ed 711] This can be corrected by reshaping the gingiva surgically
Treatment is palliative to make patient comfortable like or with electrosurgery (Fig. 45-5). Effective plaque control
local anaesthetic mouthwash, increased water intake and by the patient is particularly important to establish and
pain control. The Lesion subsides within 7-10 days. To maintain the normal gingival contour in areas of tooth
control the toxic sympt oms antibiotics are used which is irregularity.
only supportive. Corticosteroids are contra indicated.
25. 'A' (Carranza 11th ed 136 / tQ•h ed 395]
Note: According to ll'h edition, early diagnosis and
immediate initiation of antiviral therapy is treatment 26. ' D' [Carranza 11th ed 133 / 101 hed 392]

20. ' B' [Carranza 11th ed 608] 27. 'A' [Carranza 11th ed 138 / 1Q•hed 398]

21. 'A' [Carranza 11th ed 608 / 10th ed 707] 28. ' D' (Carranza tQth ed 288)
Extractions and periodontal surgery should be done 4 weeks
after the signs and symptoms subside. 29. 'A' [Carranza 11th ed 608]

22. 'A' [Carranza 11th ed 138] 30. 'B' [Carranza 11th ed 152]
Primary herpetic gingivostomatitis develops in both children
and young adults. It rarely occurs before the age of six 31. 'A' [Carranza tQth ed 392]
months, because of the presence of circulating antibodies in According to Horning and Cohen
the infant derived from the mother.
Stage 1 NUG
Secondary herpes may manifest as herpes labialis herpes Stage 2 Either NUG or NUP
genitalis and ocular herpes. It is due to stimulation of the Stage 3 & 4 NUP
latent virus in the neuronal ganglia.
Stage 5 & 6 Necrotizing stomatitis
Herpes is common in pre-school children. ANUG is common Stage 7 Noma
in adolescents and juvenile periodontitis is common in
young adolescents. In patients in whom there is severe immunodeficiency or
malnutrition, NUG and NUP may progress to [BHU-2012]
23. 'D' [Carranza 11th ed 13 7 / 10th ed 396] a) neoplasm b) noma
PREDISPOSING FACTORS OF ANUG c) naevus d) none of the above
i) Local Smoking
Pericoronal flaps 32. 'B' [Carranza 11th ed 133 / tQ•h ed 392]
Injury to the gingiva
33. 'D' [Carranza 11th ed 610 / 101 h ed 392]
Preexisting gingivitis

ii) Systemic Nutritional deficiency 34. 'C' [National Board's Endoperio Dec 1977 Q.41]
Debilitating disease 35. 'D' [Carranza 11'h ed 496 / 101 h ed 557)
iii) Psychosomatic
36. 'A' [Carranza 11th ed 225]
FACTORS CAUSING RECURRENCE OF ANUG AIDS is caused by HIV and is characterized by destruction
• Periocoronal flap of lymphocytes, rendering the patients susceptible to
• Inadequate local therapy opportunistic infections, including destructive periodontal
lesions and malignancies.
• Anterior overbite
37A. 'D' [Carranza 11th ed 141]
24A. 'A' [Carranza 11th ed 133 / 10th ed 392]
378. 'C' [J Oral Maxillo facial surgery; 2000; 58(6); 611-6]
248. ' B' [Carranza 11th ed 136 / 10th ed 397]
Viridans group of streptococci and anaerobes predominate
the suppurative samples collected in pericorinitis.
24C. 'A' [Carranza 11th ed 609 / 10th ed 708-709]
Contouring of the gingiva as an adjunctive Procedure.
37C. 'C' [Carranza 11th ed 610]
Even in cases of severe gingival necrosis, healing ordinarily
Treatment of pericorinitis
leads to restoration of the normal gingival contour (Fig.
45-4). However, if the teeth are irregularly aligned, healing • Gentle flushing with warm water to remove debris and
sometimes results in the formation of a shelf-like gingival exudate.
margin, which favors the retention of plaque and the • Swabbing with antiseptic after elevating the flap gently
recurrence of gingival inflammation. from the tooth with a scaler.
• Antibiotics in severe cases
Dental ;lut.,e

• If the flap is swollen and fluctuant, an incision may be


necessary to estab lish drainage.
• After inflammation subsides, the tooth should be
extracted.

370. ' C' (Carranza 11th ed 610)

38. 'C' [Q .No.1, 2, and Synopsis point 19]

39. 'A' [Q.No. 1, 2, and Synopsis point 19)

40A. 'A' (Check QNo. 1]

408. ' C' [Check Sy nopsis]

40C. 'A' (Check Q.No.2 of chapter 5 gingivitis ]

41. ' C' [KOT 7th ed 8 3 7)


Metronidazole is effective against anaerobic protozoa
like entamoeba and Giardia lambella. Also effective e
against many anaerobic and microaerophilic bacteria like
spirochetes, B.fragilis, Fusobacterium, Cl.perfringens,
Cl.difficile, H. pylori and anaerobic streptococci.

The drug of first choice for Pneumocystis carinii pneumonia


(PCP) is cotrimoxazole (trimethoprim-sulphamethoxazole) at
a dose of 120 mg/kg/day in four divided doses. Pentamidine,
Clindamycin, and Oapsone are other drugs used in PCP
patients.

42. ' C' [Carranza 11th ed 101]


Trans missible denotes the capacity for the maintenance
of infectious agent in successive passages through a
susceptible animal host. Communicable signifies capacity
to spread by modes of direct contact, water, food, airborne
route, eating utensils or by arthropod vectors. A disease
that is communicable is described as contagious. NUG is
transmissible but not communicable or contagious.
, PERIODONTICS
~

V
559

8. PERIODONTAL POCKET

1. A periodontal pocket of 6 mm deep lying coronal to the 10. Periodontitis may best be described as
junctional epithelium is: a) A chronic inflammatory disease with periods of remission
a) Infrabony pocket b) Periodontal pocket and exacerbation
c) True pocket d) Pseudopocket b) A chronic inflammatory disease that usually does not
(AP -98, AIPG -03) manifest itself clinically before the age of 40
2. Which of the following is most appropriate for c) A degenerative disease of the periodontium
pseudopocket? d) An acute inflammatory disease of long duration
a) Is a feature of periodontitis (TNPSC -99)
b) Is associated with attachment loss 11. A compound POL pocket is
c) Is associaled wilh Lrans-seplal fibres loss a) Spiral Lype b) Presenl on 2 or more surraces
d) May have a depth of more than 3 mm c) Infrabony in nature d) All of the above
(AIIMS -92) (KAR -98)
3. Periodontal abscess is formed in 12. The first fibers to be destroyed in periodontitis are?
a) Periapical area b) Alveolar bone laterally a) Free gingival b) Oblique
c) Periodontal ligament laterally c) Horizontal d) Trans septal
d) All of the above (AP-10)
(AP -01) 13. Periodontal pockets which offer the least possibility for
4. In periodontitis maximum destruction is present in: bone regeneration are classified as pockets.
a) Lateral wall of pocket b) Root surface a) suprabony b) one-wall infrabony
c) Junctional epithelium d) None of the above c) two-wall infrabony d) three-wall infrabony
(AIPG -2K, 96) (APPSC - 99)
5. The treatment for acute periodontal abscess is 14. What is the difference between gingivitis and
a) The drainage of abscess through incision only periodontitis?
b) Incision & drainage followed by flap surgery after the a) Gingival sulcus. b) Periodontal pocket.
symptoms subside c) Loss of epithelial attachment.
c) Incision should not be given through ultrasonic tip d) Mobility of tooth.
d) Incision should be given only external level (AIPG -04)
(AP - 03) 15. The least effect on the prognosis of a periodontal disease
6. Which of the following cells are most increased inn umber is
in the connective tissue wall of a periodontal pocket? a) Suppuration from the pocket
a) Plasma cells b) Leukocytes b) Furcation involvement
c) Lymphocytes d) Erythrocytes c) Alveolar bone loss
(AIIMS - 92) d) Co-operation and motivation of the patient
7. The distance between Junctional epithelium and alveolar (AIIMS - 99)
bone: 16. Periodontal pocket wall between tooth and bone is
a) increases with age b) decreases with age a) Supra bony pocket b) Intra bony pocket
c) Remains constant d) None of the above c) Gingival pocket d) Pseudo pocket
(KAR -98) (KAR -96)
8. Which of the following is not true: 17. Exotoxin acting on cementum is obtained from
a) Periodontal cyst always develops from Dentigerous cyst a) Bacteria b) Sulcular fluid
b) In periodontal cyst tooth may be vital c) Calculus d) Saliva
c) In periodontal cyst tooth may be non-vital due to (AIPG -91)
involvement of accessory pulp canals 18. Which cells are decreased in chronic periodontitis?
d) Periodontal cyst may develop due to improper treatment a) B-lymphocytes b) T-lymphocytes
of Deep Periodontal pocket. c) Mast cells d) Plasma cells
(AIPG - 02) (AIIMS - 98)
9. The periodontal pocket principally contains 19. The pocket epithelium shows a series of histopathological
a) Living and degenerated leucocytes changes. Which of the following is true in this regard?
b) Microorganisms, their products and debris a) Necrotic changes b) Proliferative changes
c) Gingival fluids and salivary mucin c) Degenerative changes
d) Desquamated epithelial cells d) Proliferative and degenerative changes
(AP - 2K, 03) (AIPG-06)

1) D 2) D 3) C 4) A 5) A 6) A 7) C 8) A 9) B 10) A 11) B 12) A 13) B


14) C 15) A 16) B 17) A 18) B 19) D
Dental ;lut.,e

20. The distance between the bottom of calculus and alveolar


crest in human periodontal pocket is constant having an
approximate length of
a) 0.97 mm b) 1.97 mm
c) 2.97 mm d) 3.97 mm
(KAR-04)
21. Purulent exudation from the gingival sulci is an
indication of:
a) Deep pockets
b) Severe periodontal attachment Loss
c) Nature of t he inflammatory changes in the pocket wall
d) Shallow pockets
(AIPG-06)
22. The depth of clinical gingival sulcus is the distance
between the gingival margin to the
a) Cementoenamel Junction
b) Alveolar crest
c) Apical extension of junctional epithelium
d) Apical penetration of the probe
(AIIMS -08, AIIMS MAY 2012)
23. Pseudo pocket is seen in the
a) Base of the pocket lies on cementum
b) Gingivitis
c) Periodontitis
d) Base of the pocket and Lies on alveolar bone
(AP-10)
24. Periodontal tissue destruction apically and potentially
migrating towards the gingival margin is called
a) Marginal periodontitis
b) Marginal Gingivitis
c) Retrograde Periodontitis
d) chronic dest ructive periodontitis
(COMED-14)
25. Suprabony pockets are also known as
a) Infrabony b) Intra-alveolar
c) Subcrestal d) SupracrestaljSupra-alveolar
(GCET-14)

20} B 21) C 22) D 23) B 24} C 25} D


, PERIODONTICS
~

V
561

8. PERIODONTAL POCKET - ANSWERS

1. ' D' [Carra nza 11th ed 171 / 10th ed 434] 9. ' B' [Carranza 11th ed 177 / 10th ed 442]
In a pseudo pocket or gingival pocket there is no attachment
loss. The pocket depth is due to t he coronal movement of 10. 'A' [Carranza 11th ed 179 / 10th ed 444]
the gingival margin.
11. ' B' [Carranza 11th ed 171 / 10th ed 435]
In a true pocket there is apical movement of the junctional Simple pocket Involves only one surface.
epithelium due to the destruction of the supporting tissues.
Compound pocket Pocket present on 2 or more surfaces.
In suprabony pocket the base of t he pocket is coronal to the Originate on one tooth surface and
underlying alveolar bone. Spiral pockel Lwisling around Lhe Loolh Lo involve
one or more additional surfaces.
A true periodontal pocket is best defined by, which
one of the following statements? 12. 'A' [Carranza 11th ed 173 / 101 h ed 436]
a) Is one formed by a coronal shift of the gingival
margin, without migration of the epithelia attachment 13. ' B' [Carranza 11th ed 851]
b) Is one which id over 5 min in dept h The pattern of bone destruction in infrabony pockets is
c) Is one which is over 7 min in depth vertical (angular) . Three-wall infrabony defect has the best
d) Is one which is associated with apical migration of possibility for bone regeneration while one wall defects
epithelial attachment and loss of bone usually have to be recontoured surgically.
(AP-14)
14. 'C' [Carranza 11th ed 180]
2. ' D' [Carranza 11th ed 171 / 10th ed 434 J
15. 'A' [Carranza 11th ed 177 / 10th ed 442]
3. 'C' [Carranza 11th ed 182 / 10th ed 448] Pus is a common feature of periodontal disease, but it is on ly
Periodontal abscess is usually formed in t he supporting a secondary sign. Pus indicates neither the depth of pocket
periodontal tissues a long t he lateral aspect of root or in the nor the severity of periodontal dest ruction . The presence
soft tissue wall of a deep periodontal pocket. of pus reflects the nature of inflammatory changes in the
pocket wall.
4. 'A' [Carranza 11th ed 174 / 10th ed 438]
The most severe dest ruction occurs in the Lateral surface of 16. ' B' [Carranza 11th ed 182 / 10th ed 446]
pocket. The epitheli um of lateral pocket wall undergoes Suprabony pocket lnfrabony pocket
degeneration, ulceration and eventually suppuration may occur. Base of pocket is apical to
Base of pocket is coronal
alveolar bone i.e., pocket wall
5. 'A' [Carranza 11th ed 614 / 10th ed 718] to alveolar bone.
Lies between bone and tooth .
The purpose of treatment of an acute periodontal abscess
is to alleviate pain, control the spread of infection and Pattern of bone Pattern of bone destruction is
establish drainage. The drainage can be est ablished t hrough destruction is horizontal. vertical.
the pocket or through an incision from the external surface. The transseptal fibres are
arranged horizontally in The transseptal fibres are
Option B) 'incision and drainage followed by flap surgery the space between base of oblique rather than horizontal.
after the symptoms subside' is the treatment of chronic pocket and alveolar bone.
periodontal abscess.
17. 'A' [Carranza 11th ed 179 / 10th ed 442]
6. 'A' [Carranza 11th ed 174 / 10th ed 438]
Connective tissue is densly infiltrated with plasma cells 18. ' B' [Carranza 10th ed 453]
(80%), Lymphocytes and scattered PMNs. Acute gingivitis is a T-lymphocytic Lesion whereas
periodontitis is a B-lymphocytic lesion . The predominant
7. 'C' [Carranza 11th ed 180 / 10th ed 445 J cell in periodontitis is the plasma cell.
Normally the distance between junctional epit helium and
alveolar bone is about 1.07-1.97mm (1-2mm) 19. ' D' [Carranza 11th ed 179 / 1o•hed 438,436]

8. 'A' [Carranza 11th ed 183 / 10th ed 449] 20. ' B' [Carranza 11th ed 180 / tQ•h ed 445]
Periodontal cyst is an uncommon Lesion occurring most often The normal dist ance between the apical extent of calculus
in the mandibular canine-premolar region. It may develop and the alveolar crest is most constant length of 1.97mm.
from dentigerous cyst, odontogenic cyst, primordial cyst of The normal distance between junctional epithelium and
supernumerary tooth germ from stimulation of epithelial alveolar bone is about 1.07 - 1.97mm (1-2mm).
rests of periodontal ligament.
Dental ;lut.,e

21. 'C' [Carranza 11 1h ed 177 / 101hed 442)


Pus is a common feature of periodontal disease, but is only
a secondary sign. It is not an indication of depth of the
pocket or severity of destruction. It reflects the nature of
the inflammatory (suppurative) changes in t he pocket wall.

22. 'D' [Carranza 111h ed 487 / 101h ed 47 )


2 types of pocket depths
• Biologic/histologic depth:
The distance between the gingival margin and t he base
of pocket (the coronal end of junctional epithelium)
Can be measured only in carefully prepared
histologic sections

• Clinical/ Probing depth:


The distance to which probe penetrates into the
pocket.
Probing force is 25 pounds or 0.75N .

23. 'B' (Carranza 111h ed 171 / 101h ed 434]

24. 'C' [Carranza 11th ed 7 53]


Advanced pulpitis may Lead to pulp necrosis, often
accompanied by inflammatory bone resorption at the root
apex as found in case of chronic periradicular periodontitis
or abscess. This is known as retrograde periodontitis because
it represents the breakdown from apical to cervical direction
and is opposite of orthograde periodontitis which results
from sulcular infection

25. 'D' [Carranza 11th ed 171]


Suprabony pockets are also known as supracrestal or supra
alveolar pockets. Infra bony pockets are also known as
infrabony or subcrestal or intra alveolar pockets.
, PERIODONTICS
~

V
563

9. PATIERNS OF BONE LOSS AND TRAUMA FROM OCCLUSION

1. Buttressing bone formation is a c) Actinomyces viscosus d) Improper tooth brushing


a) congenital abnormality b) occurs endosteally (AIPG - 99, KAR - 97)
c) occurs periost eally d) due to trauma form occlusion 12. What histologic criteria is used to distinguish gingivitis
(MAN - 99) from periodontitis
2. Classification of bony defects are most helpful in a) Occlusal trauma b) Howships lacunae
determining the likelihood of repair after periodontal c) Bone Resorption d) Endosteal Proliferation
treatment is based on (AIPG -02)
a) Depth of bone defect 13. Trauma from occlusion does not effect
b) Number of bone walls remaining a) Alveolar bone and enamel
c) Distance between the facial, lingual walls b) Cementum and enamel
d) Distance from the crest of defect to CEJ c) Epithelial attachment and gingiva
(AP -97, AIPG -01, 03) d) Periodontal ligament and Cementum
3. Which of the following is a common osseous lesion in (AIPG -91, MAN -98)
periodontitis is 14. Common sign of occlusal trauma (TFO) is
a) Exostosis b) crater a) Tooth mobility b) Fractures of cusps
c) buttressing bone d) hemiseptum c) Resorption of alveolar ridge
(AIIMS - 00, APPSC -99) d) Widening of P.D ligament
4. Three-wall defects most frequently occur: (AP- 06, AIIMS -06, COMEDK- 07)
a) On the lingual aspect b) On the facial aspect 15. Which of the following represents early effect of primary
c) In the furcation d) In the interdental area trauma from occlusion
(AIPG - 91, 03) a) Vertical pockets b) Generalized alveolar bone loss
5. Coronoplasty should be carried out in patients with c) Undermining resorption
a) Occlusal prematurities d) Haemorrhage and thrombosis of blood vessels in PD
b) Parafunctional habits ligament
c) Signs of trauma from occlusion (AIPG -01, MAN -97)
d) All of the above 16. Trauma from occlusion refers to
(KCET-09) a) Occlusion which causes trauma
6. The pattern of bone destruction in periodontitis is b) Abnormally of the periodontium
determined by: c) Changes in periodontium due to occlusal forces
a) Depth of the pockets b) Suppuration d) Inflammation of periodontium
c) Pathway of inflammation d) Composition of plaque (MAN -2K)
(KAR - 03) 17. When occlusal forces are increased, the cancellous bony
7. Angular bone defects are classified trabeculae?
a) according to the depth of the pockets a) Increase in number and thickness
b) According to the number of walls involved b) Decrease in number and thickness
c) According to type of pockets c) Decrease in number and increase in t hickness
d) According to the number of walls remaining d) Remains the same
(AP -99) (KAR -97)
8. Buttressing bone formation is a 18. Occlusal correction is done after periodontal therapy in
a) Process which is always peripheral which condition:
b) Occurs normally a) Infrabony pockets b) Suprabony pockets
c) Occurs when excessive occlusal forces are present c) Trauma from occlusion d) Pseudopockets
d) Process which is always endosteal (AIPG -99)
(KAR -98) 19. Radiograph of a periodontal ligament of a tooth which
9. Intrabony defects are has lost its antagonist shows:
a) one-walled defects b) two-walled defects a) Widening of the PDL space
c) three walled defects d) combination of above b) Narrowing of PDL space
(MAN -2K) c) Increased density d) Sclerotic change
10. Radius of action of toxins in the bacterial plaque is (AIIMS -JUN 99)
a) Less than 2.5 mm b) Greater than 3.5 mm 20. The type of load on teeth that is most destructive to the
c) Less than 1.5 mm d) 1.5 - 2.5 mm periodonti um is:
(KAR - 99, MHCET-15) a) Vertical b) Oblique
11. Infrabony pocket results from c) Horizontal d) Apical
a) Trauma from occlusion b) Dental plaque (PG! -00)

1) D 2) B 3) B 4) D 5) C 6) C 7) D 8) C 9) C 10) D 11) A 12) B 13) C


14) A 15) D 16) C 17) A 18) C 19) B 20) C
Dental ;lut.,e i======
21. In primary trauma from occlusion: 31. Excessive forces of occlusion within physislogical limit will:
a) The blood vessels present in periodontal ligament rupture a) Increase width of POL b) Decrease width of POL
and release contents in surrounding space c) Width will remain same d) Trauma from occlusion
b) Adaptive capacity of tissues to withstand occlusal forces (AIIMS-06)
is impaired resulting in injury from normal occlusal forces 32. In response to traumatic occlusal forces, alveolar bone has
c) The alveolar bone is not damaged a) Osteoblasts in areas of tension and osteoclasts in areas
d) Bifurcations and trifurcations are less susceptible to of pressure
injury related to this condition b) Osteoclasts in areas of tension and osteoblasts in areas
(AIIMS - 92) of pressure
22. Clinical signs of occlusal trauma include all except: c) Osteoid in areas of tension and osteoblasts in areas of
a) Periodontal pockets b) Migration of teeth pressure
c) Tenderness on percussion d) Afunctional atrophy
d) Increased width of periodontal ligament (AIIMS-09, AIIMS MAY 2012}
(AIPG -92, 96) 33. In "Jiggling type" of trauma, the occlusal force cause
23. All of the following are signs of periodontal trauma except: a) Funnel shaped destruction of socket wall
a) Pain b) Mobility b) Distinct pressure and tension sides within periodontal
c) Hypersensitivity d) Bleeding ligament
(KAR -01) c) No changes in socket wall
24. Occlusal trauma may best be defined as a condition d) Has no effect on periodontal ligament space
caused due to: (COMEDK-15)
a) The pressure on teeth due to lack of proper arch 34. Trauma from occlusion causes vascular changes in the
alignment periodontium with in
b) Failure of occlusal balance bilaterally a) 45 minutes b) 60 minutes
c) The force or the pressure beyond the physiological limits c) 30 minutes d) 10 minutes
of the periodontium (APPG-15)
d) The force or the pressure within the 35. All are true about osseous crater except
physiological limits of the periodontium a) 2/3rd of all mandibular defects
(AIPG -01) b) 2 times more common in anterior region as compared to
25. Bone loss associated with Trauma from occlusion is: posterior
a) Notching b) Vertical c) 85% lingual and facial height is same
c) Horizontal d) Cystic d) 1/3rd of all defects
{KAR -02) (PGI JUNE-2013)
26. Vertical or angular defects are found in
a) suprabony pockets b) infrabony pockets
c) intrabony pockets d) both b and c
(KCET-10)
27. The necrotising pressure areas, undergoing bone
resorption and endosteal bone formation
a) Juvenile periodontitis b) Occlusal trauma
c) Chronic suppurative periodontitis
d) ANUG
(AIIMS- 2K)
28. Pathologic migration of tooth occurs:
a) Towards the bone resorption area
b) Always from the bone resorption area
c) Inciso occlusal direction
d) Axio inclined direction
(PGI-06)
29. Resorption of thick margins resulting in plateau
formation is called as
a) Crater b) Ledge
c) Lipping d) Trauma from oclusion
(COMEDK-06)
30. How many osseous walls are present in one walled
vertical defects?
a) One wall present b) Two walls present
c) There walls present d) Four walls present
(AIPG-06)

21) A 22) A 23} D 24} C 25} B 26} D 27} B 28} A 29} B 30} A 31} A 32} A 33} A
34) C 35} B
, PERIODONTICS
~

V
565

9. PATTERNS OF BONE LOSS & TRAUMA FROM OCCLUSION - ANSWERS

1. ' D' [Carra nza 11th ed 190 / 10th ed 459] Various pathways of inflammation are:
When excessive occlusal forces resorb bone, the bone • Interproximally from gingiva to bone, from the bone to
attempt s to reinforce the thinned bony t rabeculae with new periodontal ligament.
bone. This attempt to compensate for lost bone is called
• Facially and lingually from gi111giva to bone along the outer
"Buttressing bone formation".
bone and from gingiva into the periodontal ligament.
"Buttressing bone formation" may be central or peripheral.
When the inflammation spreads directly from gingiva to
Peripheral buttressing bone formation is also called LIPPING.
periodontal ligament and from periodontal ligament to
bone, it will result in angular type of bone loss.
To reinforce the thin bony trabeculae by occlusal
forces, bone formed is? (PGI Dec-2013) 7. ' D' [Carranza 11th ed 191 / 10th ed 460]
a) Buttressing bone formation Angular defects are classified according to the number of
b) Reinforcing bone formation walls remaining. The less the number of walls remaining, the
c) Ledge d) Condensing poor is the prognosis.

2. 'B' (Carranza 11th ed 191 / 10th ed 460] 8. 'C' (Carranza 11th ed 190 / 10th ed 459]
Angular defects are classified according to the number of
walls remaining. 9. 'C' (Carranza 11th ed 731 / 10th ed 462]

One wall defect is also called as hemiseptum. Prognosis is 10. ' D' (Carranza 11th ed 188 / 10th ed 456]
poor for one-wall defects, as they have to be recontoured To exert their action, the bone resorption factors should be
surgically. Regenerative procedures cannot be carried out in present in close proximity to the bone surface. The range of
one-wall defects. effectiveness of these toxins is about 1.5 to 2.5mm.

Three-wall osseous defect is also called intrabony defect. Beyond 2.5 mm there is no effect. Large defects far exceeding
Prognosis is better for three wall defects as the regenerative 2.5 mm from tooth surface (Egi:- JP & RPP) are caused by
procedures can be easily carried out. presence of bacteria in the tissues

3. 'B' (Carranza 11•h ed 192 / 10th ed 462] 11, 'A' (Carranza 11th ed 180 / 10th ed 473]
Osseous craters are concavities in the crest of interdental
bone confined within facial and lingual walls. Craters make- 12. 'B' (Carranza 11th ed 188]
up about one third of all defects and about two thirds of all
mandibular defects. 13. 'C' (Carranza 11th ed 302 / 10th ed 473]
Trauma from occlusion mainly affects the supporting tissues
Which of the following is the most occurring defect and does not causes pockets or gingivitis because the rich
in mandible? (AP-2013) blood supply of marginal gingiva is sufficient to maintain it,
Ans: Osseous crater even after the vessels of periodontal ligament are obliterated
by excessive occlusal forces.
4. 'D' [Carranza 11th ed 192 / 10th ed 460]
Three wall defects or intra bony defect appear most frequently 14. 'A' (Carranza 11th ed 202 / 10th ed 474]
on the mesial aspects of second and third maxillary and Increased tooth mobility is t he most common clinical
mandibular molars. sign of trauma from occlusion. This is due to destruction
of periodontal fibres in the injury stage and widening of
5. 'C' [Carranza 11'" ed 197 / 10'" ed 853] periodontal Ligament in final adaptation stage of trauma
Coronoplasty is only indicated when the trauma from from occlusion.
occlusion results in periodontal injury. Prophylactic
treatment of occlu sal pre maturities and parafunctional 15. ' D' (Carranza 11th ed 199 / 10th ed 469]
habits before the evidence of periodontal injury may not Compression of fibres, stasis of blood flow, hemorrhage,
advisable as it may upset the existing balance between the thrombosis of blood vessels are early signs of trauma from
teeth and tissues. occlusion.

6. 'C' [Carranza 11th ed 187 / 10th ed 455 Fig 28-4] 16. 'C' [Carranza 11th ed 198 / 10th ed 467]
The pathway of spread of inflammation is critical, because Trauma from occlusion refers to tissue iniu,y that occurs,
it affects the pattern of bone destruction in periodontal when occlusal forces exceed the adaptive capacity of the
disease. tissues.
Dental ;lut.,e i======
Primary trauma from occlusion occurs due to excessive 24. 'C' [Carranza 11th ed 197 / 10th ed 468)
occlusal forces. Changes produced by primary trauma do not
alter the level of connective tissue and do not initiate in 25. ' B' [Carranza 11th ed 202 / 10th ed 471)
pocket formation. This is probably because the supracrestal
gingival fibres are not affected and thus prevent apical 26. 'D' [Carranza 11th ed 191 / 10th ed 435]
migration of the junctional epithelium.
27. 'B' [Carranza 11th ed 199 / 10th ed 469]
Secondary trauma from occlusion occurs when the adaptive
capacity of the tissues to withstand occlusal forces is 28. 'A' [Carranza 11th ed 203]
impaired by bone loss resulting from marginal inflammation. Pathologic migration occurs most frequently in the anterior
region. The tooth moves towards the bone resorption area.
Secondary occlusal trauma is the effect induced by
occlusal forces (normalfabnormal) on teeth (KAR-13)
29. 'B' [Carranza 11th ed 193)
a) Decreased periodontal support Ledges are plateau like bone margins caused by resorption
b) Decreased crown structure of thickened bony plates.
c) Teeth with small root length
d) All of the above 30. 'A' [Carranza 11th ed 191)
Angular defects are classified on the basis of the number of
osseus walls intact.
17. 'A' [Carranza 10th ed 459]

18. ' C' A three walled defect would have (KAR-2013)


Ans: Three walls intact
19. 'B' [Carranza 11th ed 203 / 10th ed 471]
Insufficient stimulation (Ex: Open bite, absence of functional 31. 'A' [Carranza 11th ed 202 / 101hed 469]
antagonists, unilateral chewing habits) causes Even slightly excessive forces stimulates bone resorption
• Thinning of periodontal ligament of the alveolar bone with a resultant widening of the
periodontal ligament space.
• Atrophy of periodontal fibres.
• Osteoporosis of the alveolar bone. The body then attempts to repair the injury when the forces
• Reduction in bone height. are diminished or if the tooth drifts away from them.

20. 'C' [Carranza 11th ed 197 / 10th ed 467) 32. 'A' [Carranza 11th ed 199 / 10th ed 469]
The periodontal fibres are arranged in such a way that they • Slightly excessive pressure stimulates resorption of the
resist vertical forces effectively. Lateral (horizontal) and alveolar bone, with a resultarnt widening of periodontal
torque (rotational) forces injure the periodontium. ligament space.
• Slightly excessive tension causes elongation of the
21. 'A' [Carranza 11th ed 194 / 10th ed 468]
periodontal ligament fibres and apposition of alveolar
Option 'B' is for secondary trauma from occlusion. Furcation
bone.
areas are most susceptible to injury from trauma from
occlusion.
33. 'A' [Carranza 11th ed 154]
22. 'A' [Carranza 11th ed 202 / 10th ed 471] In jiggling trauma, the direction of forces is multi directional
Trauma from occlusion does not cause pockets or gingivitis, and the bony area of breakdown will be funnel shaped. This
as the marginal gingiva is unaffected by trauma from is due to adoptive remodelling of the periodontium. Because
occlusion because of its rich blood supply. of these bony changes, the tooth becomes loose.

Though trauma from occlusion does not initiate gingivitis or 34. 'C' [Carranza 11th ed 153]
periodontal pockets, it may affect the progress and severity Changes produced when greater pressure applied on
of periodontal pockets started by local irritation. periodontal ligament:
• Greater pressure produces compression of periodontal
23. 'D' [Carranza 111h ed 197, 202 / 10th ed 474) ligament fibers, resulting in areas of hyalinization injury
Signs and symptoms of trauma from occlusion: to the fibroblasts and other cells leading to necrosis of
• Tooth pain, sensitivity to percussion. areas of ligament.
• Increased tooth mobility. • Vascular changes produced in 30min.
• Cementum tears. • Stasis of blood flow occurs in 2-3 hrs.
• Widening of periodontal space. • Disintegration of blood vessel walls occur in 1-7 days
• Root resorption. • Also increased resorption of bone and tooth takes place
• Thickening of lamina dura.
• Vertical or angular bone defects.
, PERIODONTICS
,......__,,_

V
567

35. 'B' [Carranza 11th ed 148]


Craters occur twice as often in posterior segments than in
anterior segments.
Dental ;lut.,e

10. SLOWLY PROGRESSIVE & AGGRESSIVE PERIODONTITIS


1. Type of periodontitis which does not respond to 10. " Mirror image" type of bone loss pattern seen in
therapy evena after repeated treatments is: (OR) A a) Localized juvenile Periodontitis
child is having bone loss around molar and incisor b) Rapidly progressive periodontitis
region with pus discharge and calculus which c) Pre-pubertal periodontitis
cannot be treated by medications is suffering from? d) Adult periodontitis
a) Refractory periodontitis ( KAR -2K, PG! -97}
b) Juvenile periodontitis 11, Periodontosis manifests itself mainly in:
c) Rapidly progressive periodontitis a) Molars b) Upper incisors
d) Chronic adult periodontitis c) Molars and incisors d) Premolars
(KAR - 03) (AIIMS - 89, 2K)
2. Periodontitis is usually severe in patients: 12. An 18 years old male reported with chief complaint
a) Taking phenytoin b) With defective neutrophils of sensitivity and deep, dull, radiating pain during
c) With bruxism d) With overcrowding chewing. Intra-oral examination showed sparase plaque
(AIPG - 90) and dental calculus deposits, distolabial migration of
3. H.I.V. periodontitis is the maxillary incisors with diastema formation, mobility
a) diagnostic of H.l.V. infection of maxillary and mandibular incisors and first molars.
b) characterized by remissions and exacerbations Prescribed radiographs showed an arch shaped loss of
c) characterized by necrosis and periodontal destruction alveolar bone extending from the distal surface of the
d) periodontitis is not seen in H.I.V. infection mandibular second premolar to the mesial surface of
(AP -99} the second molar. There was vertical bone loss in the
4. First clinical sign of localized juvenile periodontitis is maxillary incisor region.
a) Tooth mobility b) Gingival bleeding (COMED-2012)
c) Pathologic migration A) The most likely disease is
d} Vertical bone loss along incisors and molars a) Chronic periodontitis b) Refractory periodontitis
(MAN - 02} c) Localized aggressive periodontitis
s. Juvenile periodontitis is a d) Necrotizing ulcerative periodontitis
a) Definite genetic disease b) Immunological defect
c) Neutrophil chemotactic defect B) The antigens consistently associated with this disease
d) Neutrophil adhesion defect are
(MAN -02, AIIMS -03} a) HLAA9andB15 b) HLA AO and 830
6. A condition, which occurs in puberty and in the absence c) HLA AlS and B9 d} HLA A30 and BO
of large amounts of supragingival plague and resulting
in angular bone defects in the first molar and incisor C) The host modulation thera[Py that may be used as
region is called adjunctive therapy for this disease is
a) Generalized juveni le periodontitis a) Subantimicrobial-dose clindamycin
b) Localized juvenile periodontitis b) Subantimicrobial-dose metronidazole
c) Desquamate gingivitis d) Chronic periodontitis c) Subantimicrobial-dose doxycycline
(MAN -99) d) Subantimicrobial-dose ciprofloxacin
7. Localised juvenile periodontitis is _ _ type of disease
a) Atrophic b) Inflammatory 13. Titres of Actinomycetem comi tans are increased in:
c) Degenerative d) None of the above a) Gingivitis b) Trauma from occlusion
(MAN -02} c) Juvenile periodontitis d) Cyclic neutropenia
8. Aggressive periodontitis is characterized by the following (AIPG - 94}
major features EXCEPT 14. All of the following about Down's syndrome are true
a) Non contributory medical history except:
b) Rapid attachment loss a) POL degeneration takes place
c) Pathologic migration d) Familial aggregation of cases b) Toot h never exfoliates though there is advanced POL loss
(KCET-2012} c) Deep periodontal pockets associated with a substantial
9. The cause of bone destruction in juvenile periodontitis is: plaque
a) Phagocytosis are reduced d) Acute necrotizing lesions are a frequent finding
b) Reduced neutrophilic chemotaxis (AIPG - 02}
c) Decreased host resistance 15. Rapid progressive juvenile periodontitis is seen usually
d) Highly virulent microorganisms at which age?
(AIIMS -90, KAR -98} a) 6-10 years b) 10-20 years

1) A 2) B 3) C 4) C 5) C 6) B 7) C 8) C 9) B 10) A 11) C 12A) C 128) A


12C) C 13) C 14) B 15) B
, PERIODONTICS
~

V
569

c) 30-40 years d) At any age b) Decreased severity of gingival inflammation


(AIPG -94) c) No response on gingiva
16. Prepubertal periodontitis occurs d) Epithelial desquamation
a) Before llyrs b) At 12-15 years (KAR-04)
c) At 18 years of age d) After 20 years 27. One of the following systemic diseases involving
(AP -01) periodontal tissue is not a disorder of neutrophil function
17. Arch shaped bone loss around maxillary molars occurs in: a) Agranaulocytosis b) Chediak-Higashi syndrome
a) Gingivitis b) ANUG c) Leukemia d) Cyclic neutropenia
c) Chronic periodontitis d) Juvenile periodontitis (KAR-04)
(PGI - 98,COMEDK-06) 28. Degeneration of periodontal tissue without inflammation
18. Which of the following is a feature of papillon Lefevre is called
syndrome? a) Periodontosis b) Periodontitis
a) destructive periodontitis c) Pericorinitis d) Oesquamative gingivitis
b) palmar-plantar keratosis (PGI-06)
c) calcification of dura d) all the above 29. Which of the following ailment most commonly causes
(AP - 02) periodontal disorder:
19. Which of the following periodontal diseases does not a) Hypophosphatemia b) Wegener's granulomatosis
have calculus? c) Multiple myeloma d) AIDS
a) ANUG b) Chronic adult periodontitis (AIPG-07)
c) Juvenile periodontitis d) Periodontal abscess 30. Which of the following hematological disease is
(KAR - 04) associated with periodontal disease?
20. Which of the following antibiotic prescribed in case of a) AIDS b) Hypophosphatesia
juvenile periodontitis? c) Wegener's granulomatosis
a) Tetracycline b) Erythromyci n d) Histocytosis
c) Sulfadiazine d) Cephalexin (AIIMS-02)
(KAR -2K, 04) 31. Which is not true about Juvenile periodontitis
21. The syndrome characterized by hyperkeratosis of hands and a) Rate of bone loss is faster than normal periodontitis
feet together with severe destruction of periodontal tissues b) There is predilection for female patients
in primary and permanent teeth and calcification of dura c) The inflammatory process starts about simultaneously as
a) Papillon-lefevre syndrome the bone loss
b) Prepubertal periodontitis d) Frequently bilateral symmetrical patterns of bone loss
c) Juvenile periodontitis occur
d) Rapidly progressing periodontitis (AIIMS-09)
(MAN - 93, COMEDK-06) 32. Most common oral manifestation of HIV infection is
22. Oral hairy leukoplakia is a feature of a) Oral hyperpigmentation b) Oral hairy leukoplakia
a) Whit e sponge nevus b) Hairy cell Leukemia c) Oral candidiasis d) Bacillary angiomatosis
c) Speckled leukoplakia d) None of the above (AP-2013)
(KAR-04) 33. Which of the following is NOT a mechanism by which
23. Early exfoliation of deciduous teeth is seen in diabetes influences periodontium?
a) Papillon Lefevre syndrome a) Hyperglycemia b) Altered collagen metabolism
b) Peter-Kelly syndrome c) Altered neutrophil chemotaxis
c) Pierre robin syndrome d) None of the above d) Increased vascularity
(AP-05) (KAR-2013)
24. Which of the following organisms is not frequently found 34. Periodontal diseases associated with neutrophil disorders
in periodontosis? include
a) Actinobacillus actinomycetemcomitans a) Acute necrotizing ulcerative gingivitis
b) Capnocytophaga b) Localized juvenile periodontitis
c) Mycoplasma d) Actinomyces. c) Rapidly progressive periodontitis
(AIPG-2011) d) All the above
25. Examination of mouth of a patient with HIV/AIDS may (KAR-2013)
commonly reveal 35. Which of the following is true in case of AIDS patients?
a) Macroglossia b) Periodontal diseases a) Acute pain
c) Loss of sensation of taste b) Diffuse red lesion of the gingiva
d) Red tongue c) Gingiva covered with pseudomembrane
(KAR-04) d) Gingival itching
26. Gingival fluid from diabetics contain a reduce level of (AIPG-14)
cyclic adenosine monophosphate which may Lead to
a) Increased severity of gingival inflammation

16) A 17) D 18) D 19) C 20) A 21) A 22) D 23) A 24) D 25) B 26) A 27) C 28) A
29) D 30) A 31) C 32) C 33) D 34) D 35) B
Dental ;lut.,e

36. Which of the following drugs is most frequently used


in treating refractory periodontitis, including localized
juvenile periodontiti s?
a) Metronidazole and Amoxicillin
b) Tetracycline
c) Ciprofloxacin d) Penicillin
(AIPG-14)
37. HIV associated gingivitis is best described by the term?
a) Juvenile gingivitis b) Acute necrotising gingivitis
c) Linear gingival erythema
d) Necrotising ulcerative gingivitis
(AIPG-14)
38. A 21/2 year old child with severe horizontal bone
resorption and normal cementum is suffering from?
a) Hypophosphatasia b) Precocious periodontosis
c) Down syndrome d) Generalized periodontitis
(PGI JUNE-2011)
39. Seroconversion of which virus precedes the development
of epidemic Kaposi sarcoma?
a) CMV b) HSV
c) HHV-8 d) VZV

36) A 37) C 38) B 39) C


, PERIODONTICS
~

V
571

10. SLOWLY PROGRESSIVE & AGGRESSIVE PERIODONTITIS - ANSWERS


1. 'A' [Carranza 9th ed 404) 8. 'C' [Carranza 11th ed 219 / 10th ed 506)
Cases if do not respond to therapy or recur soon after Aggressive periodontitis is distinguished from chronic
adequate treatment are referred as 'Refractory periodontitis'. periodontitis by:
It consists of few cases of SPP, many cases of RPP and all • The age of onset, the rapid rate of disease progression
cases of prepubertal periodontitis.
• The nature and com position of the associated subgingival
2. ' B' [Carranza 11th ed 429] microflora
Severe periodontitis occurs in patients with neutrophil • Alterations in the host's immune response and
defects. • Familial aggregation of diseased individuals.
• Racial influence
Defective neutrophil function is seen in:
• Papillon Lefeure syndrome. 9. ' B' [Carranza 11th ed 221)
• Chediak Higashi syndrome.
10. 'A' [Carranza 10th ed 509)
• Juvenile periodontitis.
• Neutropenia 11, 'C' [Carranza 11t• ed 218 / 10th ed 507]
• Agranulocytosis Periodontosis is the old name for JP or aggressive periodontitis.
• Diabetes Gingivosis is the old name for desquamative gingivitis.

3. 'C' [Carranza 11th ed 239 / 10th ed 502) 12A. 'C' [Carranza 11th ed 218 / 10th ed 507]
AIDS associated necrotising ulcerative periodontitis (NUP)
is characterized by severe soft tissue necrosis with rapid 128. 'A' [Carranza 11th ed 221 / 10th ed 510)
destruction of bone leading to its exposure. The destruction Some of the immune defects have been implicated in the
sometimes extend to the vestibular area and palate and may pathogeneses of aggressive periodontitis. The human
result in necrotising stomatitis. leukocyte antigens, HLA A9 and B15, are considered as
markers for aggressive periodontitis.
4. 'C' [Carranza 11th ed 218 / 10th ed 507)
The most common initial features of juvenile periodontitis 12C. 'C' [Carranza 11th ed 598, 710 / 10th ed 700)
(aggressive periodontitis) are mobility and pathologic Subantimicrobial-dose doxycycline (SDD) may help to
migration of the first molars and the incisors. There is prevent the destruction of the periodontal attachment by
distolabial migration of the maxillary incisors, with diastema controlling the activation of matrix metalloproteinases,
formation. primarily collagenase and gelatinase, from both infilt rating
cells and resident cells of the periodontium, primarily the
Tooth mobility and presence of deep periodontal pockets neutrophils.
with lack of inflammation are the other significant features
of LJP. 13. 'C' [Carranza 11th ed 219 / 10th ed 507]

[According to 10th ed, the terms like juvenile or pre pubertal 14. ' B' [Carranza 11th ed 430 / 10th ed 299)
periodontitis have been replaced by aggressive (Localized or Down syndrome (Mongolism, trisomy 21) is a congenital
generalized) periodo11titis] chromosomal anomaly characterized by mental deficiency
and growth retardation .
5. 'C' [Carranza 11t• ed 221 / 10th ed 511)
Periodontal degeneration, recession, acute necrotising
6. ' B' [Carranza 11'" ed 219 / 10th ed 507] lesions are common features of this syndrome. When
LJP is characterized by distribution of lesions in the 1" compared to normal patients the incidence of caries is
molars and incisors with least destruction in the cuspid- less in down syndrome patien11:s.
premolar area whereas in generalized JP there is generalized
involvement of teeth. 15. ' B' [Carranza 10th ed 507)
Prepubertal periodontitis usually occurs below 11 years of age
Vertical bone Loss around incisors and molars in otherwise whereas juvenile periodontitis is commonly seen between 11
healthy teenagers is diagnostic of LJP. Arc shaped bone and 19 years of age.
loss extending from distal surface of second premolar to
mesial surface of second molar creating a mirror image 16. 'A' [Carranza 11th ed 429]
type of bone loss is characteristic of juvenile periodontitis. Prepubertal periodontitis is commonly associated with
systemic diseases such as:
7. 'C' [Carranza 11th ed 219 / 10th ed 506) • Papillon Lefevre syndrome
• Chediak Higashi syndrome
Dental ;lut.,e i======
• Hypophosphatasia 27. 'C' [Carranza 11th ed 429 / 10th ed 292)
• Downs syndrome Leukemias are malignant neoplasias of WBC precursors.
Leukemic cells tend to displace normal components of bone
• Neutropenias
marrow resulting in reduced production (quantity) of RB Cs,
• Blood dyscrasias such as acute and subacute leukemia WBCs and plat elets leading to anemia, leucopenia (reduction
in number of non malignant WBCs) and thrombocytopenia.
17. 'D' [Carranza 11th ed 219 / 10th ed 509)
28. 'A' [Carranza 10th ed 506)
18. 'D' [Carranza 11 1h ed 429 / 10th ed 298)
29. 'D' [Carranza 11th ed 238)
19. 'C' [Carranza 11th ed 219 / 10th ed 507)
30. 'A' [Carranza 11th ed 238)
20. 'A' [Carranza 11th ed 598 / 10th ed 697)
The regimen of tetracycline advised in JP is 250mg. 6 hrly 31. 'C' [Carranza 11th ed 219 / 10th ed 507)
for 14 days and repeating for every 8 weeks upto 18 months.
Augmentin (Amoxcyllin 250mg + Clavulinate potassium 32. 'C' [Carranza 11th ed 228)
125mg) is another drug usually preferable in treating JP. Candidiasis is fo und approx. 90% of AIDS patients. These
patients also present wit h esophageal candidiasis, a
Drugs of choice in the treatment of localized form of diagnostic sign of AIDS.
Juvenile Periodontitis is
a) Metronidazole b) Penicillins 33. 'D' [Carranza 11th ed 417)
c) Tetracycline d) Clindamycin Effects of diabetes on periodontium
(COMED-14) 1) The glucose content of gingival fluid is increased in
diabetes, this alters the environment of microflora.
21. 'A' [Carranza 11th ed 429 / 10th ed 298) Eg: Subgingival flora composed of mainly
capnocytophaga, anaerobic vibrios and actinomyces
22. 'D' [Carranza 11th ed 230 / 101hed 520) species in type 1 diabetes. P. gingivalis, P. intermedia
Oral hairy leukoplakia primarily occurs in persons with HIV are prominent in severe periodontal lesions with type 2
infection. Found chiefly on t he lateral borders of the tongue diabetes.
bilaterally. The lesions do not rub off and resembles other
keratotic oral lesions. 2) Altered neutrophil function, resulting in impaired
chemotaxis, defective phagocytosis or impaired
23. 'A' [Carranza 11th ed 429 / 10th ed 298) adherence.
Papillon-Lefevre syndrome is inherit ed autosomal recessive 3) Altered collagen metabolism and decreased collagen
disorder characterized by hyperkeratotic skin lesions, severe synthesis.
destruction of the periodontium and calcification of the dura.
34. 'D' [Carranza 11th ed 137, 221)
24. 'D' [Carranza 11th ed 346 / 101h ed 509) • For LJP the recent term is Localized aggressive
• Juvenile periodontitis is also known as periodontosis. periodontitis (LAP).
• The invading flora of juvenile periodontitis has been • Generalized aggressive periodontitis (GAP) encompasses
described as morphologically mixed but composed the diseases previously classified as GJP and rapidly
mainly of gram negative bacteria, including cocci, rods, progressive periodontitis.
filaments and spirochetes.
• Using different met hods including immunocytochemistry Periodontal diseases associated with neutrophil disorders
and electron microscopy, several tissue invading microbes • ANUG
that have been identified in juvenile periodontitis are... • LJP
- Actinobacillus. actinomyceterncomitans • Various forms of aggressive periodontitis
- Capnocytophaga
- Mycoplasma & spirochetes 35. 'B' [Carranza 11th ed 238)
Linear gingival ervthema (LGE)
25. 'B' (Carranza 11th ed 238 / tQth ed 527) • It is a most common persistent, linear, easily bleeding
Gingival and periodontal diseases in HIV: erythematous gingivitis described in HIV-positive
• Linear gingival erythema patients. LGE may be localized or generalized in nature.
• Necrotizing ulcerative gingivitis The LGE may be
• Necrotizing ulcerative periodontitis • Limited to marginal tissue
• May extend into attached gingiva in a diffuse erythema
26. 'A' • May extend into alveolar mucosa
, PERIODONTICS
~
573
V
Spontaneous rem1ss1on occurs. But if unresponsive then
antifungal topical and systemic are advised.

Necrotizing ulcerative gingivitis (NUG) is less commonly


seen in HIV patients. If present resembles ANUG in features
(pseudomembrane) and treatment.

36. 'A' [Carranza 11th ed 697]


The metronidazole-amoxicillin and metronidazole-augmentin
combination provides excellent results in refractory
and localized aggressive periodontitis that had been
treated unsuccessfully wit h tetracyclines and mechanical
debridement.

These drugs have an additive effect regarding suppression of


A. actinomycetem comitans.

This combination may provide a therapeutic benefit by


reducing or eliminating microorganisms and a prophylactic
benefit by giving rise to a predominantly streptococcal
microflora.

37. 'C' [Carranza 11th ed 238]

38. 'B' [Shafer 6th ed 402]


Children with precocious periodontosis or papillon lefevre
syndrome present with severe horizontal bone loss,
premature resorption of roots but with normal pulp tissue
and normal cementum

39. 'C' [Carranza 11th ed 176]


Human herpes virus-8 seroconversion precedes the
development of epidemic Kaposi Sarcoma (KS) by 5-10
years. It has been associated with both AIDS related and
non-AIDS related KS. It appears that decreasing immune
competence results in activation of latent HHV-8. The virus
may be transmitted sexually or from infected mothers to
children.
Dental ;lut.,e

11. DIAGNOSIS, PROGNOSIS AND TREATMENT PLAN

1. The tissue response to oral hygiene instruction is best a) crown - root ratio b) faulty restoration
assessed by c) Approximate bone height
a) Probing the base of the socket d) Morphology of bone deformities
b) Changes in plaque scores (KAR - 99)
c) Reduced tendency to bleed on probing the gingival 11. Diagnosis of bone defect is done accurately by:
margin a) Clinical examination b) Radiographs
d) Reduced tooth mobility c) Surgical exposure d) Probing
(MAN -94, AIIMS -93) (PGI -95)
2. Gingivitis appears on the radiograph as 12. Prognosis is good in:
a) Loss of alveolar bone b) Pockel formalion a) One wall pocket b) Two wall pocket.
c) Change in bone trabeculations c) Three wall pocket. d) Hemiseptum.
d) No observed changes (KAR -98)
(MAN -2K, 98} 13. A 25-year old is too young to have an advanced probing
3. Radiographs are of great value in diagnosing periodontal depth but her youth favours a good prognosis:
disease because they reveal a) Both statement and reason are correct
a) Hard - to - soft tissue relationship b) Both statement and reason are incorrect
b) Morphology of bone deformities c) Statement is correct but the reason is false
c) Presence of pockets d) Statement is incorrect but the reason is true
d) Thickening of the lamina dura (AIPG -99}
(MAN -95} 14. Standard force for evaluating periodontal pockets that
4. DNA probes: have been found to be well tolerated is
a) Measure periodontal pocket depth. a) 0.65 N b) 0.75 N
b) Identify a particular microorganism c) 0.85 N d) 0.95 N
c) Calculate the length of gingival epithelial attachment (AIPG -95}
d) None of the above 15. Radiographically the level of the normal inter proximal
(PGI -99, KCET-08) alveolar crest is related to the
5. If the pocket depth is 5 mm and gingival recession a) Position of cementa - enamel junctions of the approximal
is 3 mm then the total loss of attachment is: teeth
a) 8mm b) 3mm b) Epithelial attachment
c) 5 mm d) 2 mm c) Anatomy of the alveolar process
(AIPG 97, PGI 01} d) Location of the Gingival attachment
6. While using a periodontal probe to measure the depth of (APPSC -99)
the periodontal pocket the pocket is measured from: 16. Which of the following can be determined most accurately
a) Free gingival margin to the cementoenamel junction by radiographs alone?
b) Junctional epithelium to the free gingival margin a) Periodontal pocket b) Periodontal abscess
c) Base of pocket to the cementoenamel junction c) Anatomical root length d) Bifurcation involvement
d) Base of pocket to mucogingival junction
(PGI -01, MAN -95) 17. Which of the following pockets offers best chance for
7. The unbalanced insulin-dependent diabetes which leads bone regeneration:
to accumulation of ketones is associated with one of the a) Suprabony pocket b) One - wall infrabony pocket
following specific character of breath odor/smell c) Two - wall infrabony pocket
a) Rotten eggs smell b) Sweet odor d) Three - wall infrabony pocket
c) Smell of rotten apples d) Fish odor (AIPG -89}
(AP-14} 18. The percentage of upper and lower first molars, where
8. Mobility of teeth is measured using: furcation entrance diameter is narrower than the width
a) Periodontometer b) Periotron of conventional periodontal curette is
c) Spechtometer d) Densitometer a) 28% b) 58%
(KAR -98} c) 82% d) 85%
9. PERIOTEST is used for detecting (MAN -95}
a) Enzymes in GCF b) Tooth mobility 19. First radiographic sign of periodontitis
c) Antibodies d) Inflammatory mediators a) Wedge - shaped radiolucencies
(KAR -99, COMEDK-08) b) Loss of crestal bone
10. In the radiographic assessment of periodontal disease, c) No sign d) Loss of lamina dura
information cannot be obtained is (PGI -95, 02}

1) C 2) D 3) D 4) B 5) A 6) B 7) C 8) A 9) B 10) D 11) C 12) C 13) C


14) B 15) A 16) C 17) D 18) B 19) D
, PERIODONTICS
,.....__,,_

V
575

20. Computerized periodontal probe is 31. Oral malador is detected by


a) Florida probe b) Perio temperature probe a) Gas chromatography b) Gas laser
c) Automated probe d) DNA probe c) Perio-tron d) Perio-scan
(KAR - 99, 03) (AP-2011)
21. The probing pressure applied by CPITN probe 32. According to Miller's classification of increased tooth
a) 10 grams b) 25 grams mobility, grade 3 would be
c) 35 grams d) 45 grams a) 0.1 to 0.2 mm in horizontal direction
(KAR -01) b) Upto 1mm in horizontal direction
22. Which radiographic technique gives three-dimensional c) > 1mm in horizontal direction
view of the alveolar bony defects? d) Mobility in horizontal and vertical direction
a) Intra-oral radiograph (KAR-2013)
b) Digital intra-oral radiography 33. Bone sounding done in modern times is performed by
c) Orthopantamograph which method?
d) Spiral computed tomography a) RVG b) Probing
(AIPG-06) c) CBCT d) Radiovisiography
23. The electronic probing system "Foster Miller Probe" detects: (AIIMS NOV-14)
a) Only the pocket depth b) Only the CEJ
c) CEJ and pocket depth d) Furcation involvement
(AIPG-06)
24. An advanced diagnostic technique which has been
suggested as an alternative to culture methods is:
a) Phase contrast microscopy
b) Direct immunofluorescence
c) Latex agglutination
d) Indirect immonofluorescent microscope assays
(AIPG-06)
25. Other measures to record periodontal inflammation
a) Periotemp b) Perioscan
c) Periostat d) None of the above
(GCET-14)
26. Recently advanced diagnosis of perioddontal organism is
a) Dipslide method b) Nucleic acid probes
c) Roentzen method d) None

27. True for effect of smoking on prognosis of periodontal


diseases following scaling and prophylaxis is
a) Increase in clinical attachments
b) Increase in width of attached gingiva
c) Reduction in negative influence of smoking due to
increased level of plaque control
d) Increase in neutrophilic response
(MCET-10)
28. The following affects prognosis of periodontal treatment
a) Short root, long crown b) long root, short crown
c) Apical pearls d) Root concavities
(MCET-10)
29. Each of the following drug regimens is likely to influence
periodontal treatment planning EXCEPT
a) Dexamethasone b) Dicoumarol
c) Diphenhydramine d) Nitroglycerine
(AP-09)
30. Prognosis is
a) Most likely cause of the disease
b) Least likely cause of the disease
c) Estimation of the likely course of the disease
d) None of the above
(KCET-2011)

20) A 21) B 22) D 23) C 24) A 25) A 26) B 27) C 28) D 29) C 30) C 31) A 32) D
33) C
Dental ;lut.,e

11. DIAGNOSIS, PROGNOSIS & TREATMENT PLAN - ANSWERS


1. ' C' [Carranza 11th ed 527 / 10th ed 548) Smell of rotten apples (Due to Unbalanced insulin
accumulation of ketones) dependent diabetes
2. 'D' [Check Explanation Below]
Gingivitis is inflammation of soft tissues only and it does Fish odor (due to accumulation
not involve the supporting tissues so, no radiographic of dimethylamine and Kidney insufficiency
changes are seen in patients with gingivitis. trimethylamine)
• Intraoral origin
3. ' D' [Carranza 11th ed 499 / 10th ed 565) Smell of sulphur
• Liver disease
Radiographs do not reveal the presence of pockets or
sofl Lissue lesions or morphology of bone deslruclion. 8. 'A' [Periodontology Essentials by Mueller 163)
Transgingival probing and visual examination by surgical It is known as Muhlemann's periodontometer
exposure are the definitive ways for knowing bone
morphology. 9. ' B'

Fuzziness and break in the continuity of the lamina dura at 10. ' D' [Carranza 11t h ed 499 / 10th ed 564]
the mesial and distal surfaces of crest of interdental septum Radiographs reveal only about the amount of bone present
is the earliest sign of periodontitis. and does not give any information about morphology of
bone defects and number of walls involved.
4. ' B' (Carranza 11t h ed 497 / 10th ed 592)
DNA probe identifies the species specific sequences of nucleic 11. ' C' [Carranza 11th ed 854 / 10th ed 565)
acids that make up DNA and thus helps in identification of Transgingival probing and visual examination by surgical
organisms. exposure are the definite ways for knowing bone morphology.

The DNA Library includes probes for P.gingivalis, 12. 'C' [Carranza 11th ed 520)
A.actinomycetam comitans, F.nucleatum B.intermedius and In case of suprabony pocket, prognosis depends on height
T.denticola. of existing bone. In case of infrabony pockets, the contour
of existing bone and the number of remaining osseous walls
5. 'A' [Carranza 11th ed 491 / 10th ed 553) influences prognosis.
• If gingival margin is on anatomic crown then level
of attachment = pocket depth - distance from CEJ to 13. ' C' [Carranza 11th ed 519 / 10th ed 615)
gingival margin. For a younger patient with rapid bone destruction, the
• If gingival margin coincides with CEJ then level of prognosis is considered as poor because of shorter period
attachment= pocket depth. in which the bone Loss has occurred in spite of greater bone
reparative capacity.
• If gingival margin is apical to CEJ then, level of
attachment = pocket depth + distance from CEJ to
14. ' B' (Carranza 11th ed 489 / 10th ed 552)
gingival margin.
A standard probing force of 25 gnns or 0.75 N is found to be
well tolerated by patient.
6. ' B' (Carranza 11th ed 491)
Pocket depth is distance between gingival margin to the
15. 'A' [Carranza 11th ed 498 / 10th ed 561)
base of the pocket (or coronal end of junctional epithelium)
16. ' C' [Carranza 8th ed 3 65)
Level of attachment is the distance between CEJ and base
Definitive diagnosis of furcation involvement, pocket depth
of pocket. Pocket depth is Less important than level of
and periodontal abscess is made by clinical examination
attachment because it is not necessarily related to bone
only.
loss. A tooth with deep pockets may have little bone loss
while a tooth with snallow pockets may have severe bone
17. ' D' [Carranza 11th ed 520)
loss.
18. ' B' (Carranza 11th ed 523 / 10th ed 6 21)
Prognosis is considered as poor if the base of the pocket
(Level of attachment) is close to root apex.
19. ' D' [Carranza 11th ed 502 / 10th ed 565]
Fuzziness or break in the continuity of the lamina dura of
7. 'C' [Carranza 11th ed 464]
crest of interdental bone is the first radiographic change in
Sometimes specific character of the odor can provide
periodontitis.
additional information such as
Wedge shaped radiolucent areas and destruction of alveolar
crest are secondary features in periodontitis.
, PERIODONTICS
,......__,,_

V
577

20. 'A' [Carranza 11th ed 492 / 10th ed 583] • Short, taped roots
CLASSIFICATION OF PROBES • Cervical enamel projections
Class 1 Conventional probes Anatomic • Root concavities
Eg: Williams, Nabers, CPITN probes • Root proximity
Class 2 Pressure sensitive probes • Furcation involvement
Eg: Foster miller probe (also detects CEJ)
• Caries
Class 3 Automated probes Prosthetic • Abutment
Eg: Florida probe (computerized) and
Toronto probe Restorative • Non vital teeth
• Root resorption
21. ' B'
29. '( '
22. 'D'
30. 'C' [Carranza 111h ed 518 / 10th ed 158]
23. 'C' [Carranza tO'h ed 584] Prognosis is a prediction of the probable course, duration,
Refer to explanation of Q. No. 20 and outcome of a disease based on a general knowledge of
the pathogenesis of t he disease and the presence of risk
24. 'A' [Carranza 11th ed 312 / 101hed 590] factors for the disease. It is established after the diagnosis
Dark fie ld or phase contrast microscopy has been suggested is made and before the treatment plan is established.
as an alternative to culture methods on the basis of its
ability to assess directly and rapidly the morphology and 31. 'A' [Carranza 11•h ed 470 / 1o•h ed 335]
motility of bacteria i n a plaque sample.
32. ' D' [Carranza 11th ed 485]
25. 'A' [Check synopsis point No.20]
Millers mobility grading
1
26. ' B' [Carranza 10 hed 592] Slight movement of less than 1mm in
Grade 1
horizontal direction.
27. 'C' [Carranza 11th ed 405 / 101h ed 255 Table 14-3] Movement of more than 1mm in horizont al
Grade 2
Effects of smoking on response to periodontal t herapy. direction
Therapy Effects of smoking • More than 1mm movement in horizontal
• .J, Clinical response to scaling and root Grade 3 direction
planing • Also mobility in vertical direction
Non surgical • .J, Reduction in pocket dept h
33. 'C' [Check Explanation Below}
• .J, Negative impact of smoking with t
Probing under local anesthesia to the bone level is called
level of plaque control
transgingival probing or bone sounding. It confirms the
• .J, Reduction in pocket depth extent and configuration of the intra bony component of
•t Deterioration of furcations after the pocket or of furcation defect s. Conventional radiographs
Surgery by surgery cannot accurately determine these defects. CBCT provides
implants better diagnostic and quantit ative information on periodont al
• .J, Gain in attachment levels
bone levels in 3D. Many studies concluded that CBCT provide
• .J, Bone fill, t recession similar horizontal and vertical dimensions of these defects
Maintenance
•t Pocket dept h during maintenance as provided by the direct probing measurements.
t herapy

28. 'D' [Carranza 111h ed 523 / l01h ed 615)


FACTORS DETERMINING PROGNOSIS
• Smoking
Systemic • Genetic
• Systemic disease
• Plaque and calculus
Local
• Subgingival restorations
Dental ;lut.,e

12. PERIODONTAL INSTRUMENTATION


1. All of the following are contraindications for root 10. Sharpening of dental instruments is helpful in increasing
planning EXCEPT efficiency because
a) deficient or overhanging margins of amalgam restorations a) reduce slippage and less tissue damage
b) erosion and /or abrasion b) increase tactile sensation
c) recession d) root caries c) decrease muscle fatigue
(MAN - 95, 99} d) all the above
2. While sharpening curettes and sickles, the internal angle (MAN -99}
of _ must be preserved between the face and lateral 11, Arkansa's stone is lubricated with light mineral oil to
surface a) Prevent heat build up
a) 50-60 b) 60- 70 b) Remove melal parlicles
c) 70-80 d) 80-90 c) Prevent if from drying up
(MAN -95 , KAR -99) d) All the above
3. During activation of a scaling instrument, the facial (MAN - 94}
surface of the blade and the tooth surface should form 12. Furcation is measured by
an angle of a) Naber's probe b) Fluoride probe
a) > 15 but< 45 b) >30but<60 c) Miller's probe d) CPITN probe
c) > 45 but< 90 d) > 90 but< 180 (AIPG -04, MAN 2K}
(MAN -95} 13. When scaling and root planning, the primary objective of
4. In general, hand sharpening of instruments with the procedure is to
unmounted stones is preferable to sharpening by motor a) Cause shrinkage of gingival tissue
- driven mounted stones because unmouted stones b) Create glass - like root surfaces
a) have a finer grain c) Remove all cementum
b) are less likely to alter the bevel d) Restore the gingival tissue to health
c) are easier to sterilize (MAN -95, PGI -2K}
d) wear away less of the instrument surface 14. Following scaling a patient notices hard black deposits.
(MAN -95) This may be due to:
5. Which part of the instrument should remain parallel a) Shrinkage of gingiva
(whenever possible) to the long axis of the tooth? b) Patient is heavy tea drinker
a) Blade b) Cutting edge c) Poor home care
c) Handle d) Shank d) Blood clot formation on tooth surface
(MAN -95} (AIPG -02}
6. The ultrasonic scaler dislodges only calculus that 15. The diameter of the tip of a periodontal probe is:
a) has been formed recently a) 0.25 mm b) 0.75 mm
b) is already loose c) 0.5 mm d) 1 mm
c) is in direct contact with the tip (AIIMS -DEC 98}
d) is located on the interproximal surfaces 16. Root planning is done to remove cementum that has
(MAN -95) been altered in conact with:
7. Which of the following instruments has a two point a) Exotoxins b) Endotoxins
contact with the tooth. c) Granulation tissue d) Junctional epithelial lining
a) Hoe b) Sickle (AIPG - 99, AP-08}
c) Currete d) Cumin scaler 17. The offset angle of Gracey curette between the blade and
(MAN -02} shank is:
8. Which one of the following is not true about ultra sonics a) 50° b) 70 °
a) Oscillates between 1500 to 3000 Hz c) 80° d) 90 °
b) Releases energy known as cavitation (PGI - 99}
c) Acoustic streaming d) Implosion 18. A dental instrument which creates aerosols containing
(MAN -02} large number of microorganisms is the:
9. A tight grasp by the operator on the explorer will a) Evacuator b) Ultrasonic scaler
a) Prevent tactile sensitivity c) Amalgam condenser d) Endodontic reamer
b) Provide greater maneuverability (AIPG -03}
c) Prevent muscle fatigue of the fingers 19. The objectives in polishing teeth include
d) All the above a) Removal of stains, films and dental plaque
(MAN -99) b) Production of smooth surface, which is more resistant to
reaccumulation of stains and deposits

1) C 2) C 3) C 4) D 5) D 6) C 7) A 8) A 9) A 10) D 11) D 12) A 13) D


14) A 15) C 16) B 17) B 18) B 19) A&B
, PERIODONTICS
,......__,,_

V
579

c) Motivation of patient to practice good home care c) DNA probe d) RNA probe
because he appreciates the appearance and feeling of (KAR -98)
clean mouth 30. The gracey curette is identified by
d) All the above a) Double curved blade
(AP-14) b) Blade at 70° angle in the shank
20. Main aim of root planning is to remove: c) Both A & B
a) Plaque b) Calculus d) Blade at 30 degrees to the shank
c) Necrotic cementum d) All of the above (AP -2K)
(PGI -95, AIPG -04) 31. The relationship between the working edge of the
21. Which of the following methods is not an acceptable way instrument and the tooth surface is:
to determine instrument sharpness? a) Angulation b) Access
a) Evaluation of the cutting edge during scaling procedures c) Adaptation d) Activation
b) Evaluation of light reflection from the cutting edge when (PGI -01)
observed under magnification 32. The instrument best suited for root planning is:
c) Evaluation of the cutting edge against plastic testing stick a) Hoe b) Scaler
d) Evaluation of the cutting edge against the clinicians c) File d) Curette
finger nail. (AIPG -91)
(KAR -99) 33. Functioning of a pace maker can be affected by which of
22. Gracey curettes are area specific instruments because: the following?
a) They are used in the specific areas of the US a) Piezoelectric ultrasonic scaler
b) They adapt to specific areas of the tooth or root b) Magnetostrictive ultrasonic scaler
c) They have more than one angle per cutting edge c) Sonic scalers d) All the above
d) They are used for root planning (KCET-2012)
(PG! -01) 34. Electronic device is contraindicated in
23. The pattern of vibration of magnetostrictive type of a) Pacemakers b) AIDS
ultrasonic scaler is c) TB d) Asthmatics
a) Elliptical b) Linear (PGI -95)
c) Back and forth d) None 35. Angulation of a periodontal instrument refers to the
(KAR -99) a) angle between the face of the blade and the shank
24. Instrument used for removal of broken curette tips from b) angle between the shank and the shaft
the periodontal pocket is: c) angle between the face of the blade and the tooth surface
a) Schwartz periotrievers b) Barnhart instrument d) angle between the shank and the tooth surface
c) EVA system d) Prophy - jet (COMEDK -04, 05)
(KAR - 02, TNPSC - 99) 36. Kirkland and orban knives are used for
25. The finger rest established on tooth surfaces on the a) Curettage b) Gingivectomy
opposite side of the same arch is known as: c) Root planning d) Scaling
a) Conventional finger rest b) Opposite arch finger rest (PG! - 97)
c) Cross - arch finger rest d) Finger - on - finger rest 37. Ultrasonic scaler can be used in
(KAR -02) a) AIDS patients b) Asthamatics
26. Instrumentation zone in periodontal therapy is: c) Pacemaker d) Heavy calculus.
a) The portion of the tooth where calculus (or) necrotic (PGI - 2003)
cementum is found 38. The first instrument used on the facial surfaces of gingiva
b) C.E. Junction while gingivectomy is:
c) Exposed root surfaces a) Orban's knife b) Kirkland's knife
d) Deep periodontal pocket c) BP blade with No. 11 blade
(KAR -03) d) BP blade with No. 12 blade
27. Gracey curettes have a characteristic feature that (AIIMS-06)
a) They are used for area of surface of the tooth 39. Scaling and root planning is performed
b) They have double cutting edge a) As initial treatment b) Post surgically
c) They can be used any where in oral cavity c) In only gingival enlargements
d) All. d) after only antibiotics
(PGI - 02) (AP-05)
28. Dental instrument used with a 'Push' motion is 40. Which of the following is not an instrument grasp during
a) Hoe b) Chisel scaling procedure?
c) Curette d) Sickle scaler a) Pen grasp b) Thumb grasp
(KAR -2K, COMEDK-05) c) Modified pen grasp d) Palm and thumbgrasp
29. Probing depth of a pocket is assessed by: (KAR-04)
a) Williams probe b) Nabers probe

20) C 21) D 22) B 23) A 24) A 25) C 26) A 27) A 28) B 29) A 30) C 31) C 32) D
33) B 34) A 35) C 36) B 37) D 38) B 39) A 40) B
Dental ;lut.,e i======
41. Electro-surgery or surgical diathermy uses controlled c) 100 - 110 degrees d) 120 - 130 degrees
high frequency electrical current in the range of: (KCET-10)
a) 1.5-7.5 million cycles/ second 52. Ultrasonic instrument size required to remove bulk of
b) 7.5-10.0 million cycles/ second restoration & dentin islands
c) 15-20 million cycles/ second a) CPR 1 b) CPR 2
d) 25-30 million cycles/ second c) CPR 5, 6 d) CPR 7, 8

42. Gracey curette No. 11-12 are used for: 53. Which of the following instrument is used to curette
a) Anterior teeth b) Posterior teeth mesial furcation area?
c) Posterior teeth distal a) Naber's probe b) Hu-friedy Mini 5 curette
d) Posterior teeth facial and Lingual c) Hu-friedy After 5 curette
(AIPG-05) d) Universal curette
43. Sickle scaler in cross section appears (PG! JUNE-13)
a) Ro und b) Oval 54. With which of the following instrument stability is
c) Triangular d) Half round highest during scaling and root planning?
(AP-05) a) Files b) HOE scaler
44. In the conventional finger rest, the finger rest is c) Chisel d) Sickle scaler
established on the (PG! JUNE-13)
a) Other side of the same arch 55. Piezo electric scaler has a wavelength of:
b) Tooth surface immediately adjacent to the working area a) 6000-9000 Hz b) 9000-12000 Hz
c) Opposing arch c) 2000-30000 Hz d) 20000-45000 Hz
d) Index finger or thumb of the non operating hand (PGI DEC-13, 12)
(COMEDK-07)
45. The optimal flow rate of coolant for sonic and ultrasonic
subgingival scaling is
a) 1-11 ml/min b) 14-23 ml/min
c) 24-32 ml/min d) 34-40 ml/min
(COMEDK-07)
46. The term curettage is derived from the word "curette"
which means
a) To 'penetrate' b) To 'cleanse'
c) To 'deviate' d) To 'enucleate'
(KCET-08)
47. The main disadvantage of Florida probe system is
a) Inconsistent probing force
b) Does not measure extent of plaque
c) Non reproducibility d) Lack of tactile sensitivity
(COMEDK-08)
48. The ideal tooth - blade working angulation of a
Blade of Gracey curette is angled from lower shank
approximately at
a) 10 - 20 degree b) 40 - 50 degree
c) 60 - 70 degree d) 80 - 90 degree
(COMEDK-08)
49. Scaling stroke?
a) Shorty & powerful pull stroke
b) Moderate to light pull stroke
c) Moderate to Light push stroke
d) A long pulling stroke
(AP-2011)
50. The Final Evaluation of the root surface smoothness after
root planning is done by?
a) Probe b) Explorer
c) Scaler d) Curettes
(PGI-08)
51. Ideal angle the sharpening stone must form with the
face of the blade is
a) 70 - 80 degrees b) 90 - 100 degrees

41) A 42) B 43) C 44) B 45) B 46) B 47) D 48) C 49) A 50) B 51) C 52) B 53) B
54) B 55) D
, PERIODONTICS
~

V
581

12. PERIODONTAL INSTRUMENTATION - ANSWERS


1. 'C' [Carranza 8th ed 459] The spray, which is directed at the tip of instrument,
Over hanging margins of restorations, caries areas of dissipates heat and forms tiny vacuum bubbles that collapse
decalcification and root roughness caused by previous quickly and release energy by a process known as CAVITATION.
instrument ation must be recognized and differentiated from The cavitating water spray serves to flush calculus, plaque
sub gingival calculus. and debris dislodged by the vibrating tip from the pocket.

2. 'C' [Carranza 11th ed 675 / 10th ed 788 Fig 51-111] 9. 'A' [Carranza 111h ed 654 / 10th ed 766]
During sharpening of instruments The most effective and stable grasp for all periodontal
• The face of the blade is parallel to the floor and the instruments is modified pen grasp (APPG-15). This due
stone makes 100-110° angle with the face of the blade to tripod effect, created by the thumb, index finger and the
pad of the middle finger, it ensures greatest control over the
• This automatically preserves an angle of 70° to 80°
instrument and also enhances maneuverability and tactile
angle between the face and lateral surface.
sensitivity.
3. 'C' [Carranza 11th ed 658 / 10th ed 769]
The palm and thumb grasp is unsuitable for the precise and
The exact blade angulation depends on the amount and
controlled movements as it inhibits the maneuverability and
nature of calculus. The optimal angulation during scaling
tactile sensitivity. It is useful for stabilizing instruments
and root planning is between 45° - 90°
during sharpening and for manipulating air and water
syringes.
During scaling strokes on heavy tenacious calculus,
angulation should be just less than 90° so that the cutting
10. ' D' [Carranza 11th ed 674 / 10th ed 787]
edge bites into the calculus.
11. ' D' [Carranza 11th ed 6 74]
An angulation of less than 45° and greater than 90° will
India and Arkansas oilstones are examples of natural
cause the instrument to slide over calculus and causes
abrasive stones. Carborundum, ruby and ceramic stones are
burnishing of calculus.
synthetically produced.
An angulation of greater than 90° is indicated for gingival 12. 'A' [Carranza 11th ed 641 / 10th ed 750]
curettage cases.
Marquis colour
Calibrations are in 3mm sections
4. ' D' [Carranza 11th ed 674 / 10th ed 787] coded probe
Mounted rotary stomes are not recommended because: UNC-15mm colour Colour coding at 5th, 10th and
• They are difficult to control precisely and can ruin the coded probe 15th mm
shape of the instrument. Michigan "O" colour Shows marking of Williams probe
• They tend to wear down the instrument quickly. coded probe with colour markings at 3,6,8 mm
• Generates frictional heat, which may affect the temper Naber's probe (blunt
Used to evaluate furcation areas
of the instrument. and curved probe)
• 0.5 mm ball at the tip and
5. 'D' [Carranza 11th ed 660 / 10th ed 750] millimeter markings at 3.5,5.5
WHO or CPTIN 8.5 and 11.5 mm
6. 'C' [Carranza 11th ed 671 / 10th ed 761] probe
Ultrasonics may be used for scaling, curettage and removing • Shows colour coding from 3.5
of stain. They produce vibrations at the tip of instrument to 5.5 mm
which when adapted to the tooth causes dislodgement of Contains markings at 1,2,3,5, 7,8,9,
Williams probe
calculus. and 10mm

7. 'A' [Carranza 11th ed 647 / 10th ed 758] 13. ' D' [Carranza 11th ed 662]
Hoes are used for removing ledges or rings of calculus. It The primary goal of scaling and root planning is to restore
makes two-point contact with the tooth and this stabilizes the gingiva to health by removing the irritants, which
the instrument and prevents nicking of the root. The blade is provoke the inflammation.
bent at 99° and the cutting edge is beveled at 45 degrees.
Hoes, chisels and files are subgingival scalers. 14. 'A' [Check Explanation Below]
After scaling, the gingiva, which is edematous and swollen,
8. 'A' [Carranza 11th ed 671 / 10th ed 760] undergoes shrinkage and is restored to normal health . This
Ultrasonic vibrations at the tip of instrument range from causes exposure of subgingival calculus in form of hard
20,000 to 45,000 hertz. black deposits.
Dental ;lut.,e i======
15. ' C' [Carranza 11th ed 641 / 10th ed 750] 24. 'A' [Carranza 111h ed 647 / 10th ed 757]
Periotrievers are a set of two double ended, highly
16. ' B' (Carranza 11th ed 662 / 10th ed 776] magnetized instruments used for retrieval of broken tips
When cementum is exposed to plaque and the pocket from pockets and furcations.
environment, its surface is contaminated by endotoxins and
these toxic substances are superficially attached to the root 25. ' C' (Carranza 111h ed 655 / 10th ed 767]
surface. These portions of cementum and residual embedded The finger rest is established on the tooth
calculus are removed from root surface by root planning Conventional
surfaces immediately adjacent to the
procedure. finger rest
working area.
17. ' B' (Carranza 111h ed 660 / 10th ed 754] The finger rest established on tooth
Cross-arch surfaces on the other side of t he same
Gracey curette Universal curette arch .
Area and surface specific. Universal. The finger rest is established on tooth
Only one cutting edge i.e. Both outer and inner cutting surfaces on opposite arch
Opposite-arch
outer edge is used. BLade is edges are used.Blade is Eg.: Mandibular arch finger rest for
curved in two planes i.e., curved in one plane i.e., the instrument ation on maxillary arch.
the blade curves up and to blade curves up and not to The fourth finger of operating hand rests
the side. t he side. Finger-on- on the index finger of the non-operating
The blade is not at a go0 The blade of universal finger hand while the Ungual surfaces of maxillary
angle to the shank. The off curette is at a go0 angle to posterior teeth are instrumented.
set blade angle is 60- 70° . the lower shank. The backs of fingers rest on the right
Palm up extra lateral aspect of mandible face while
18. ' B' (Carranza 11•h ed 671 / 10th ed 761] oral fulcrum the maxillary right posterior teeth are
instrumented.
19. 'A & B' [Quintessence international 40 (9) 783-789]
The front surfaces of fingers rest on
Palm-down
20. 'C' [Carra nza 11th ed 662] left lateral aspect of mandible while
extra-oral
the maxillary left posterior teeth are
fulcrum
21. 'D' [Carranza 11th ed 674 / 10th ed 787]
instrumented.
Sharpness of a instrument can be tested by:
• Keeping the cutting edge under light, a dull instrument An example for an extraoral fulcrum is (AP-2012]
would reflect light while a sharp instrument does not a) Conventional b) Palm down
reflect light as it has less surface area. c) Opposite arch d) Finger on finger
• The instrument is lightly drawn across an acrylic rod Extra oral palm up fulcrum is used to instrument which
called test stick. A dull instrument will slide smoothly of the following areas? (KCET-2012]
but a sharp one would raise a light shaving. a) Maxillary left posterior teeth
b) Maxillary right posterior teeth
22. ' B' (Carranza 11th ed 659 / 10th ed 754] c) Mandibular anteriors d) Maxillary anteriors

23. 'A' [Carra nza 11th ed 671 / 10th ed 830] 26. 'A'
Magnetostrictive and piezoelectric are two types of ultrasonic
units. In magnetostrictive, the patt ern of vibration is 27. 'A' [Carranza 11th ed 643 / 10th ed 754]
elliptical which means that all sides of the tip are active.
In piezoelectric units, the pattern of vibration of the tip is 28. ' B' (Carranza 11th ed 650 / 10th ed 759]
linear (back and forth) meaning that only two sides of the The chisel scaler is used in anterior part of mouth for the
tip are active. proximal surfaces of teeth that are too closely spaced to
permit the use of other scalers. The instrument is activated
In sonic units, the tip will have orbital type of vibrating with a push motion while other scalers are activated with
action. Vibrations at sonic tip range from 2K to 6500 cycles pull motion.
per second.
29. 'A' [Carranza 11th ed 428]
Option 'C' DNA probe is used for identification of species-
Piezoelectric motion, which is seen: [AIIMS-2011)
specific sequences of nucleic acids that make up DNA,
a) Oscillating in a line back & forth
thereby permitting identification of organisms.
b) Figure of 8 motion
c) Heat is not generated
3 0. ' C' [Carranza 11th ed 660 / 10th ed 755]
d) Frequency used is greater than 40 khz
, PERIODONTICS
~

V
583

31. 'C' [Carranza 11th ed 656 / 10th ed 768] 39. 'A' [Carranza 11th ed 662 / tQth ed 628]
Adaptation refers to manner in which the working end of a Scaling and root planning are done in the non surgical or
periodontal blade is placed against the surface of a tooth. Phase I therapy. They are done to restore gingival health
The objective of adaptation is to make the working end of by completely removing elements that provoke gingival
the instrument conform to the contour of tooth surface; inflammation.
Precise adaptation minimizes trauma to t he soft tissues
and root surfaces and ensures maximum effectiveness of 40. ' B' [Carranza 11th ed 655 / 101 hed 766]
instrumentation.
41. 'A' [Carranza 11th ed 797, 820 / tQth ed 899]
32. ' D' [Carranza 11th ed 664 / 10th ed 777] Electrosurgery uses high frequency currents in the range of
1.5 - 7.5 million cycles per second.
33. ' B' [Carranza 11th ed 672 / 10th ed 833] • Single-wire electrodes are used for incising or excising.
Miller et al fo und that atrial and ventricular pacing was
• Loop electrodes for planing tissue.
inhibited by magnetostrictive ultra sonic scaler. But, New
model cardiac pacemakers often have bipolar titanium • Heavy bulkier electrodes for coagulation procedures.
insulation that shields the units from the sonic type devices
making magnetostrictive, Piezoelect ric and Sonic instruments 42. ' B' [Carranza 11th ed 660 / 101 hed 754]
generally safe for use on people wit h pace makers. For explanation refer synopsis.

34. 'A' [Carranza 11th ed 672 / 10th ed 915] 43. 'C' [Carranza 11th ed 641 / 10'h ed 751]
Techniques that make use of electronic devices such as Basic characteristics of a sickle scaler are:
electro surgery and ultrasonic scalers are contraindicated • Triangular shape blade
in persons who have a non-compatible or poorly shielded • Double cutting edge
cardiac pacemaker.
• Pointed tip
35. 'C' [Carranza 11th ed 657 / 10th ed 769]
Basic characteristics of curette are:
Angulation or tooth blade relationship refers to angle
between the face of a bladed instrument and the tooth • Spoon shaped blade
surface. Correct angulation is essential for effective calculus • Rounded tip
removal. For subgingival insertion of a bladed inst rument
such as curette, the angulation should be close to 'O' 44. ' B' [Carranza 11th ed 655 / 1Q1 hed 767]
degrees. Refer to explanation of Q.No. 25

36. ' B' [Carranza 11th ed 820 / 10th ed 792] 45. ' B' [Carranza 11th ed 67 2 / 101 hed 829]
Kirkland knives are commonly used for gingivectomy. These Sonics and ultrasonics contain a water knob, which controls
are used on the facial and lingual surfaces of incisors and the volume of water being delivered to the insert tip.
those distal to the terminal tooth in the arch.
Water contributes to 3 physiologic effects that enhance the
Orbans and Merrifield knives are used for supplemental efficacy of power scalers.
incisions.
Acoustic It is the unidirectional fluid flow causing by
37. ' D' [Carranza 11th ed 672] streaming ultrasound waves.
Contraindications for the use of ultrasonic and sonic
scaling devices: This is created when the movement of the tip
Acoustic
causes the coolant to accelerate producing
• Patients with cardiac pace makers turbulence
an intensified swirling effect.
• Patients with communicable diseases t hat can be
is the formation of bubbles in water caused
t ransmitted by aerosols.
Cavitation by high t urbulence. The bubbles implode and
• Patients at risk of respiratory diseases produce shock waves throughout the water.
• Patients, who are immunocompromised or suffering from
chronic pulmonary diseases The combination of these 3 effects disrupts microflora.

Patients having porcelain or bonded restorations because 46. 'B' [Carranza 10th ed 909]
t hey can be fractured or removed.
47. ' D' [Carranza 11th ed 492 / 10th ed 583]
38. ' B' [Carranza 11th ed 820 / 1Qth ed 913] The Florida probe system is a automated probe system and
Kirkland knives are used for incisions on the facial and consists of a probe handpiece, digital readout, foot switch,
lingual surfaces. Orbans knives are used for supplement al computer interface and computer. The end of the probe tip
interdental incisions. is 0.4mm in diameter. The probing method combines the
advantages of constant probing force with precise electronic
BP Knives 11 and 12 and scissors are auxillary instrument s. measurement and computer storage of data, thus eliminating
Dental ;lut.,e i =================

the potential errors associated with visual reading and the trephine around obstructions within the pulp chamber and
need for an assistant t o record the measurements. locating hidden orifices.

Disadvantages of Florida probe system: The CPR-2 can also be used to safely and efficiently remove
• Lack of tactile sensitivity restorative materials and amalgams, and is capable of
eliminating mat erials extending below the orifice.
• Patient discomfort
• Inaccurate measurements
CPR-30, 40, & 50 (Intensity Setting- Low)
• Underestimation of deep probing depths by the Due to their small cross sectional diameters and lengths, 30,
automated probe. 40 & 50 must be used on very low power.
Perioprobe and interprobe are ot her commercially available CPR's 3, 4, and 5 are used in the coronal, middle and apical
electronic probing systems. Other electronic probing one-thirds of roots.
systems reported in the literature have never been released
for general use. One of these systems is an electronic Uses include: Trephine around posts, chasing calcified
(Foster-Miller) probe. It is capable of coupling pocket depth canals, eliminating brick-hard paste-type material, broken
measurement with detection of cementa-enamel junction instrument removal, and other intra-canal obstructions.
(AIPG-06) , from which the clinical attachment level is
detected. CPR-6 TO CPR-8 (Intensity Setting- Low)
The CPR 6, 7, and 8 ultrasonic instruments have a smaller
Toronto probe is an automated probe that uses (as with cross-sectional diamet er compared to the CPR 3, 4, and 5.
Florida probe) the occlusal-incisal surface to measure clinical The instruments are made of titanium alloy, not NiTi. They
attachment levels. Here, the sulcus is measured with 0.5mm are generally used in the mid and apical portion of the root
nickel-titanium wire. It controls angular discrepancies by
to ditch around broken files, aiding in their removal.
means of a mercury tilt sensor that limits angulation within
± 30 degrees but it requires reproducible positioning of the 53. ' B' (Carranza 11th ed 467)
patients head and cannot easily measure second or third Mini bladed curettes, such as Hufriedy mini five curette are
molars. modifications of the after five curettes. Their blades are half
of the length of the after five or standard gracey curettes.
48. 'C' (Carra nza 11th ed 660 / 10th ed 754) The shorter blade allows easier insertion and adaptation in
deep pockets, furcations, grooves, line angles and deep tight
49. 'A' (Carranza 11th ed 658 / 1Qth ed 770) facial or lingual pockets. There will be less tissue trauma by
The scaling stroke is a short, powerful pull stroke that using these cu rettes.
is used with bladed instruments for the removal of both
supragingival and subgingival calculus. The muscles of the 54. ' B' (Carranza 11th ed 470)
fingers and hands are tensed to establish a secure grasp, and HOE scalers are used for removing ledges or rings of calculus.
lateral pressure is firmly applied against the tooth surface.
They are used in the following manner:
50. 'B' (Ca rranza 11th ed 641 / 1Qth ed 750) • Blade inserted to the base of t he pocket so that it makes
Explorers are used: 2-point contact with the tooth. This stabilizes the
instrument and prevents nicking of the root.
• to locate sublingual deposits and carious areas
• The instrument is activated with a firm pull st roke
• to check the smoothness of root surface after root towards the crown.
planning.
55. ' D' (Carranza 11th ed 474)
51. 'C' (Carra nza 11th ed 675 / 10'h ed 788)
Sonic Units • 2000-6500 cycles/sec
52. 'B' (Dr.Martin Trope Dental Traumatology Update) • 18,000-50,000 cycles/sec
CPR instruments are non-surgical endodontic ultrasonic Ultrasonic • Tips move in elliptical or orbital
instruments designed to function on most brands of Piezo- magnetostrictive stroke pattern giving the tip fo ur
electric type detntal ultrasonic scalers. the guidelines for active working surfaces.
using various types of CPR instruments are:
• 18,000-50,000 cycles/ sec.
Ultrasonic
CPR-1 (Intensity Setting- Medium to High). piezoelect ric • Tips move in linear pattern giving
The CPR-1 can be used to safely remove a variety of retained the tip two active surfaces.
posts & cores and also as an aid in crown & bridge removal.
It may cause severe damage to the prosthesis if placed
directly on ceramics.

CPR-20 (Intensity Setting- Medium to High).


The CPR-2 is used primarily within the pulp chamber. The
CPR-2 is used for eliminating pulp stones, removing dentin,
, PERIODONTICS
~

V
585

13. PLAQUE CONTROL


1. Chlorhexidine gluconate is used as a mouth wash in a 12. Dental floss is used to:
concentration of a) Remove interdental plaque
a) 0.2% b) 20% b) Remove interdental food and debris
c) 2.0% d) None of the above c) Massage gums d) Prevent gingival recession
(PGI -01, AIPG -04) (PGI -01)
2. Portion of tooth brush that assists in removal of plaque is 13. Choose the most effective mechanical device for cleaning
a) Tip of brush b) Tip of bristle concave interdental surfaces of teeth? (OR) Method for
c) Sides of bristle d) Whole brush plaque control on concavities of root?
(MAN -98) a) Conventional tooth brushes
3. Water irrigating devices are most useful in b) Powered loolh brushes
a) Removing plaque b) Preventing plaque formation c) Triangular tooth picks d) Interdental brushes
c) Diluting bacterial products (AIPG -03, AIIMS NOV-14)
d) Reducing periodontal pockets 14. Which of the following is an advisable chemical plaque
(MAN - 98, PGI -2K) controlling agent?
4. Chlorhexidine on tooth causes a) Disclosing agent b) Antibiotics
a) No stain b) Brownish yellow stains c) Enzymes
c) Greenish yellow stain d) Blue back stain d) Antiseptic mouth wash, e.g. ch lorhexidine
(MAN -97) (AIPG -03)
5. Sodium lauryl sulphate used in the toothpaste is 15. Minimum amount of chlorhexidine required in mouthwash
a) Humectant b) Abrasive for plaque prevention:
c) Flavouring agent d) Surfactant a) 0.10% b) 0.12%
(MAN - 01) c) 0.2% d) 0.22%
6. Which tooth brushing technique should be used in (PGI -98, 2K, 11)
a patient with severe gingival recession and loss of 16. Use of floss in healthy mouth is recommended:
gingival contour a) Not recommended b) Once daily
a) Bass b) Stillman c) Each time after meals d) Twice daily
c) Charter d) Multit ufted (AIPG -93, 00, PG! -97)
(MAN, KAR - 01) 17. Effective use of dental floss can be expected to:
7. Which of the following is not found in commercially a) Significantly reduce caries activity
available tooth paste? b) Improve gingival health
a) Humectants b) sweetening agents c) Restrict plaque development
c) detergents d) astringent d) Effectively alter the oral environment
(MAN - 95, 2K) (AIPG - 03, 89)
8. Which one of the following tooth brushing techniques is 18. Gingival massage increases blood supply in:
most likely to clean the gingival sulcus? a) Epidermis b) Basal layer
a) Bass method b) Charter's method c) Lamina propria d) All of the above
c) Modified stillman's method (AIIMS -95)
d) Rolling stroke method 19. The brushing method recommended for individuals
(MAN - 95) with open interdental spaces with missing papillae and
9. One of the following adjunctive aids, necessary to check exposed root surfaces is,
on the efficacy of oral prophylaxis for young patient a) Fones method b) Charters method
a) Disclosing solution b) Sandpaper strip c) Bass Method d) Smith met hod
c) Synder test d) Unwaxed floss (AP-2011)
(MAN -97, KAR -01) 20. The agent used to stain plaque on tooth, gingiva & other
10. Humectant in tooth paste serves to soft tissues is called
a) as an abrasive b) retains moisture a) Staining agent b) Disclosing agent
c) flavoring agent d) foaming agent c) Mordent d) All
(MAN -99) (AP -03)
11. For periodontal patients, the most frequently 21. The use of dental floss will prevent:
recommended tooth brushing technique is: a) Caries at interproximal area
a) Scrub technique b) Roll technique b) Calculus formation
c) Sulcular technique d) Fones technique c) Plaque formation d) All of the above
(AIIMS - MAY 2012, AIPG -05) (AIPG -92)

1) A 2) B 3) C 4) B 5) D 6) B 7) D 8) A 9) A 10) B 11) C 12) A 13) D


14) D 15) B 16) B 17) B 18) C 19) B 20) B 21) C
Dental ;lut.,e i======
22. Chlorhexidine mouth rinsing: c) Two tone dye solution d) Mercurochrome
a) Specifically inhibits gram-negative bacteria (PG! -01)
b) Can inhibit subgingival plaque 33. After periodontal surgery, the method of brushing
c) May disturb taste sensation recommended is
d) Can only be obtained on prescription a) Charters method
{AIIMS -91) b) Bass method
23. Tooth Brush has bristles arranged in c) Modified stillmans method
a) 2 -3 rows b) 2-4 rows d) Scrub method
c) 3-4 rows d) 1-2 rows (AP -2K, 04)
(AP - 01) 34. The stiffness of the bristles of a nylon toothbrush is
24. If the maxillary right second molar is treated successfully dependent on the:
the distal furcation involvement can best be kept plaque a) Diameter and length of filament
free by using: b) Amount of polish of filament
a) Dental Floss b) Perie aid c) Colour of filament
c) Stimudent d) Rubber tip stimulator d) Consistency of shape of the filament
(AIPG -01, 02) (AIIMS -01)
25. Periochip is a commercially available Local drug delivery 35. Generalised gingival recession occurs in:
system containing: a) Trauma from occlusion
a) Doxycycline b) Chlorhexidine b) Incorrect method of tooth brushing
c) Minocycline d) Metronidazole c) Erosive lichen planus
{KAR -03) d) ANUG
26. Saguinarine is (AIIMS -94)
a) An anti plaque agent 36. Bass brushing technique:
b) Derived from blood platelets a) Cleans teeth from gingival margins
c) One of the antiseptic agents in chemical plaque control b) Stimulates gingiva
d) All of the above c) Interdental spaces
(KAR -01) d) All of the above
27. Effect of chlorhexidine on long terms use can (AIPG -93, 95)
a) Suppress gram +ve and gram -ve organisms 37. To maintain good oral hygiene, brushing of teeth should
b) Suppress all microbial flora be done:
c) Does not cause tongue discoloration a) Daily b) Twice a day
d) Has systemic effects c) Thrice a day d) Four times a day
(AP -01) (AIPG -95)
28. The most preferred type of Dental floss is: 38. The rate of calculus formation can be retarded by
a) Unwaxed thin floss b) Unwaxed thick floss a) Using disclosing solutions
c) Waxed thin floss b) Minimizing fluoride uptake
d) Waxed thick floss c) Using an enzyme dentifrice
(AP -98, KAR -2K) d) Having plaque removed regularly
29. The active ingredient present in Tartar control tooth (AIPG -03)
paste is: 39. Desensitizing paste has
a) Pyrophosphate b) Metaphosphate a) NaCl b) Kn03 (Potassium Nitrate)
c) Carboxy methyl d) Bi-carbonate c) Hn0 3 d) KCl
(KAR - 03) (AP - 01, COMEDK, MCET- 07)
30. The tooth brushing method least likely to remove plaque 40. Maintenance of oral health is best achieved by
in the gingival margin is the: a) hard bristle tooth brush
a) Sulcular method b) Scrub method b) Mouth wash
c) Modified stillman d) Roll stroke c) A soft bristle brush and tooth paste
(KAR - 03) d) A soft bristle brush and dental floss
31. Which dentifrices should be used for periodontal (AP - 98)
patients? 41. Chlorhexidine is a useful mouth wash as:
a) Tooth powders with high abrasive content a) It prevents plaque accumulation
b) Tooth pastes with high abrasive content b) It prevents bacterial proliferation
c) Tooth pastes with minimum abrasive content c) It breaks down plaque matrix
d) All of the above d) It causes hemostatsis
(AIIMS-08) (AIPG - 97)
32. Which disclosing solution differentiates between mature 42. Most failures in maintaining adequate plaque removal
and newly formed plaque: results from failure in patient
a) Iodine b) Erythrosine a) Recalls b) Training

22) C 23} B 24} B 25} B 26} A 27} B 28} A 29} A 30} D 31} C 32} C 33} A 34} A
35) B 36} D 37} A 38) D 39} B 40) D 41} B 42} D
, PERIODONTICS
~

V
587

c) Education d) Motivation c) Up and down d) Roll


(AP -98, AIPG -92) (AP-05)
43. The first reference to mouth wash used for the treatment 55. Soft tooth brush is recommended in periodontal patients
of disease of the gum is because soft bristles are:
a) saline gargle b) hexidine a) More flexible allowing easy access to all areas
c) Listerine d) Urine of a child b) Less abrasion of tooth structure than hard brush
(KAR -99) c) Freying is less with soft bristles
44. Which of the following is not a disclosing agent d) None of the above
a) Erythrocin b) Cetyl pyridinim chloride (AP-05)
c) Mercurochrome d) Two tone solution 56. In Which year were powered toothbrushes invented?
(PGI - 95) a) 1921 b) 1929
45. Proxa brushes are recommended to be used in c) 1939 d) 1941
a) Type I embrasure b) Type II embrasure (COMEDK-07)
c) Type III embrasure d) Any of the above 57. What is the function of humectants in dentrifrices:
(COMEDK -04) a) Liquefies the dentrifrice
46. Disclosing solution stains b) Prevents water loss and hardening on exposure to air
a) Food debris b) Material alba c) Flavors the tooth paste
c) Micro organism d) Glycoprotein d) Cleans tooth surface effectively
(PGI - 95) (KCET-07)
47. Action of Dentifrice is 58. Which one of the following is not used as an abrasive in
a) Remove calculus b) Remove stains the dentifrice
c) Remove plaque a) Calcium carbonate b) Silica
d) Cleaning of teeth and polishing of teeth c) Sodium Ch loride d) Amylase
(AP -02) (AP-08, AIIMS-MAY 2012)
48. Brush which should be used for normal healthy gingiva 59. Choose the most effective mechanical device for cleaning
a) Hard b) Medium interdental surface of teeth
c) Soft d) Super soft a) Conventional tooth brush
(PGI -97) b) Powered tooth brush
49. Chemical plaque control c) Triangular tooth picks (ITP)
a) is an effective replacement for mechanical plaque control d) Interdental brush
b) is especially recommended after periodontal surgery (AP-08)
c) is best achieved using cetylpyridium chloride mouthwashes 60. Tooth brush abrasions are more common on the
d) is of no use a) Mandibular t eeth, on the right half of the arch
(KAR - 98) b) Mandibular teeth, on the left half of the arch
50. Which of the following wax coating is present on dental floss? c) Maxillary teeth, on the right half of the arch
a) Besswax. b) Spermaceti wax d) Maxillary t eeth, on the left half of the arch
c) Japan Wax. d) Carnauba Wax. (AIIMS-08, AIIMS MAY 2012)
(AIPG -04) 61. According to ADA specification of a tooth-brush, which
51. Which of the following is used as a thickening agent in of the following is false?
dentifrices? a) Head of the brush should be 2 inches long
a) Calcium Carbonate, Calcium Phosphate and Calcium Sulphate b) 2-4 rows of bristles
b) Sodium Bicarbonat e, Aluminium Oxide c) 5-12 t ufts per row d) 80-86 bristles per tuft
c) Sodium Lauryl Sulphate and Sodium Lauryl Succinate (KAR-2013)
d) Carboxymethyl Cellulose, alginate amylase 62. Which of the following is NOT associated with long-term
(AIPG-06) use of Chlorhexidine?
52. The most widely recommended tool for removing plaque a) Brownish discolouration of teeth
from proximal tooth surfaces b) Oral mucosal erosion
a) Tooth brush b) Mouth wash c) Sweet t aste perception affected
c) Dental floss d) Tooth paste d) Occasional bilateral parotid swelling
(COM ED K-06) (KAR-2013)
53. Embrasure characterized by a slight to moderate 63. Most commonly used technique of flossing?
recession of interdental papilla are: a) Spool method b) Spiral method
a) Type I b) Type II c) Winding method d) Knutson method
c) Type III d) Type IV (PGI DEC-13)
(AIIMS-06) 64. Full mouth disinfection therapy aims at
54. Most detrimental methods of brusing causing damage to a) Full mouth scaling within 24hours
tooth soft tissues and hard tissues is: b) Full mouth scaling within 48 hours
a) Side to side b) Bass c) Quadrant wise scaling over 7-10 days

43) C 44) B 45) B 46) D 47) D 48) C 49) B 50) B 51) D 52) C 53) B 54) A 55) A
56) C 57) B 58) D 59) D 60) D 61) A 62) D 63) A 64) A
Dental ;lut.,e

d) Five days of antibiotic therapy


(PGI JUNE-14)
65. The plaque removal effectiveness of powered toothbrush
over manual toothbrush is best explained by
a) Sonic effect b) Acoustic streaming
c) Hawthorne effect d) Ultrasonic effect
(KERALA-2015)

65) C
, PERIODONTICS
~

V
589

13. PLAQUE CONTROL - ANSWERS


1. 'A' [Carranza 11th ed 634 / 10th ed 741] 6. ' B' [Carranza 9th ed 656]
Chlorhexidine chemically is diguanidohexane with Condition Technique
pronounced antiseptic properties. Daily rinses with 10ml.
of 0.2% aqueous solution of chlorhexidine effectively In gingival recession and root
Modified stillman
inhibit the development of plaque, calculus and gingivitis. exposure
Minimum concentration of chlorhexidine needed to inhibit In patients with healing wounds
Charters
plaque is 0.12%. after surgery
Ro utine t echnique in patients with Bass or sulcular
2. ' B' [Carranza 11th ed 625 / 10th ed 731] or without periodontal disease method
Sofl loolh brush wilh rounded brislle lips arranged in 2-4
rows is recommended. The most recommended technique
Sulcular technique
in patients with periodontal disease
Type of brush Bristle diameter
Recommended technique in children Fones
Soft 0.2mm (0.007 inch)
Least effective method Roll
Medium 0.3mm (0.012 inch)
Most common method of brushing Scrub technique
Hard 0.4mm (0.014 inch)

Stillman's method of brushing was originally developed


Diameter of soft bristle brush is (AIPG-14) to (AP-2013)
a) 0.007 inch b) 0.012 inch a) provide gingival stimulation
c) 0.014 inch d) 0.009 inch b) clean interproximal areas
c) remove plaque from gingival sulcus
3. 'C' [Carranza 11th ed 63 2 / 10th ed 739] d) none of the above
Oral irrigating devices disrupts and detoxifies the bacterial
plaque. They effectively clean the non adherent bacteria and 7. ' D' [Carranza 11th ed 627 / 10th ed 732]
debris in periodontal pockets. Water and dilute chlorhexidine
can be used as irrigating agents. 8. 'A' [Carranza 11th ed 628 / 10th ed 733]
In bass technique or sulcular method of tooth brushing,
4. ' B' [Carranza 111h ed 634 / 10th ed 741] the bristles are placed at gingival margin at an angle of
Chlorhexidine causes Local reversible side effects Like brown 45° to the long axis of tooth. This force the bristle ends
staining of teeth, tongue, silicate, resin restorations and into the gingival sulci and interproximal embrasures and
transient impairment of taste perception. also produces blanching of gingiva. Bass technique helps
in cleaning of cervical third of tooth, gingival sulci and
5. ' D' [Soben Peter 1" ed 354] interproximal areas.

Com position of dentifrice 9. 'A' [Carranza 11th ed 634 / 10th ed 741]


Abrasives Silica, calcium carbonate Disclosing agents stain bacterial deposits on teeth, tong ue
and gingiva. They help in education and motivation of the
Humectants Sorbital patients. It is a simple way to instruct the patients in the
Surfactant/detergent Sodium Lauryl sulphate dental office. Some disclosing agents are:
Flavouring agent Peppermint oil, oil of wintergreen • Erythrosin - Common disclosing agent.
Binding agent Alginates, carrageenates • Bismark brown - Plaque component of Ramjford index
uses this solution.
Antitartar agent Pyrophosphate, Triclosan
• Two tone Sol. - Stains mature plaque green and
Potassium nitrate, strontium
Desensitising agent immature plaque red
chloride
• Merchurochrome
Flouride (1100 ppm) Anticaries agent
• Malachit e green

Which of the following detergent used in dentifrice? 10. 'B' [Soben Peter 2"d ed 464]
a) Carrageenan {KCET-2012) Humectant retains moisture in the dentifrice and prevents
b) Disodium pyrophosphate the dentifrice from drying and hardening on exposure to air.
c) Sodium Lauryl sulphate d) Sorbitol
Dental ;lut.,e i======
11. ' C' [Carranza 10th ed 733] Gingival massage increases keratinisation, epithelialisation
Pattern of movement Brushing technique and blood circulation of the gingiva.

• Roll method 19. ' B' [Soben Peter 3rd ed 437)


Roll
• Modified stillman method Indications of charters method
• Bass (sulcular) {AP-2013) • Open dental spaces with missing papilla and exposed
Vibratory • Stillman root surface
• With FPD and orthodontic appliances
• Charter method
• After periodontal surgery
Circular Fones method
Vertical Leonard technique 20. 'B' [Carranza 11th ed 634 / 10th ed 741)
Horizontal Scrub method
21. 'C' [Carranza 11th ed 629]

An example of vertical method of tooth brushing 22. ' C' [Soben Peter 2"d ed 460)
(KCET-2012) Chlorhexidine inhibits gram +ve, gram - ve, organisms and
a) Leonard's method b) Fone's method yeasts, etc. It is bacteriostatic at Low concentration and
c) Smith's method d) Bass method bactericidal at high concentrations.
Which of the following is not a vibratory method?
(PGI-13) 23. 'B' [Carranza 10th ed 731)
a) Bass b) Stillman According to ADA, tooth brush should have brushing surface of.
c) Charter d) Fones Length 1 - 1.25 inches
Width 5/16 - 3/8 inch
12. 'A' [Carranza 11th ed 629 / 10th ed 735]
Dental floss removes plaque from interproximal surfaces Rows 2 - 4 rows
where toothbrush can't reach. Although it removes Tufts 5 - 12 t ufts/row
interproximal food and debris, chronic food impaction can
be treated only by correcting proximal tooth contacts and 24. 'B' [Carranza 11th ed 631)
plunger cusps. Stim-u-dent is a soft triangular tooth pick used to stimulate
the papillary gingiva in the anterior region of the mouth.
13. ' D' [Carranza 11th ed 631 / 10th ed 737]
Embrasure form Cleansing aid Perioaid is toothpick with handle. It can be used to clean
the sulcus, periodontal pockets and furcations.
TYPE-1 embrasures with tight contact Floss
zones and intact papillae {AIIMS-2012) Floss is inefficient in concave and furcation area.
TYPE-2 embrasures with concave Interdental or
interproximal surfaces and moderate proxa brush 25. ' B' [Carranza 11th ed 699 / 10th ed 809)
papillary recession The chemotherapeutic agents used in local drug delivery
TYPE-3 embrasures with complet e Loss Unit ufted system are:
of papillae brush • Actisite - Tetracycline containing fibers.
• Atridox - 10% doxycycline.
14. ' D' [Carranza 11th ed 634 / 10th ed 741] • Periocline - 2% minocycline.
Anti plague mouth washes: • Periochip - Asmallchipcontaining 2.5mg.ofchlorhexidine.
• Chlorhexidine
• Cetyl pyridine 26. 'A' [Carranza 11th ed 634]
• Essential oil
27. ' B' [Carranza 11th ed 634]
• Sanguinarine
• Sodium benzoate (pre brushing mouth rinse)
28. 'A' [Carranza 11th ed 629 / 10th ed 735)
Waxed floss will Leave a coating on interproximal surfaces to
15. 'B' (Carranza 11th ed 634 / 10th ed 741)
which debris may stick. Thick floss may cause difficulty to
floss in tight contact areas.
16. 'B' (Carranza 11th ed 630)
29. 'A' [Carranza 11th ed 627 / 10th ed 733)
17. ' B' (Carranza 11th ed 629 / 10th ed 735]
Because majority of dental and periodontal disease appears
30. ' D' [Carranza 10th ed 733)
to originate in interproximal areas, removal of interdental
The Least efficient method of t oot h brushing is Roll technique
plaque improves the gingival health.
because it generates only intermittent pressure against
teeth and removal of marginal plaque does not occur.
18. ' C' [Carranza 11th ed 631 / 10th ed 738)
, PERIODONTICS
~

V
591

Scrub technique fai Ls to remove plaque from interdental Glycoprotein of pellicle gets stained with diseasing solution.
areas.
47. ' D' (Carranza 11th ed 627 / tQth ed 732]
31. 'C' [Carranza 11th ed 627 / 10th ed 732]
• Tooth pastes have 20 - 40% abrasives whereas tooth 48. 'C' [Carranza 11th ed 626 / tQth ed 731]
powders have 95% abrasives.
• Dentifrices that provide adequate plaque control with 49. ' B' (Soben Peter 1" ed 368 / Carranza 11th ed 634]
minimum abrasion is preferable. Chemical plaque control is an adjunct to mechanical plaque
control.
32. 'C' (Soben Peter 2nd ed 466]
Two tone solution cont ains FDC green and red. Mature plaque It i s used:
is stained green and newly formed plaque is stained red. • After periodontal surgery.
• In poorly motivated patients.
33. 'A' [Carranza 9th ed 659] • Medically compromised patients.
• Prophylactic rinse during scaling.
34. 'A' (Carranza 11th ed 625 / 10th ed 731] • In gingival enlargements.
Nylon bristles are superior to natural (hog) bristles because • In patients with fixed appliances.
of their homogeneity, uniform size, elasticity and fracture • Ment ally and physically handicapped persons.
resistance. Natural bristles are frayed, softened and become
contaminated easily. 50. 'B' [Manappallil 2nd ed 275]
Spermaceti is obtained from the sperm whale. It is mainly
35. 'B' [Carranza 11th ed 484] used as a coating for dental floss.
Abrasion or hard tissue damage is mostly due to abrasive
dentifrice whereas gingival recession is mostly due to 51. ' D' (Sobenpeter 2nd ed 440]
incorrect method of tooth brushing. Abrasions are mostly
common on maxillary teeth, on the LEFT half of the dental 52. 'C' [Carranza 11th ed 629 / 10th ed 735]
arch for RIGHT handed individual.
53. ' B' [Carranza 11th ed 631 / tO' hed 737, Fig. 50-11]
Other causes of recession are high frena l attachments, toot h
malposition, and gingival ablation. 54. 'A'

36. ' D' (Carranza 11th ed 628 / 10th ed 733] 55. 'A' (Carranza 11th ed 626 / 10th ed 731]
Softer bristles are more flexible, clean slightly below
37. 'A' [Carranza 11th ed 635 / 10th ed 742] the gingival margin when used with a sulcular brushing
If complet e plaque removal is done efficiently then brushing technique and reach farther onto the proximal surfaces.
once daily is sufficient rather than frequency of brushing
alone. 56. 'C' [Carranza 11th ed 626 / 10th ed 731]

If plaque control is not adequate, brushing twice per day 57. 'B' (Sobenpeter 2nd ed 440 / Carranza 11th ed 629]
will help.
58. 'D' (Carranza 11th ed 627 / lO'h ed 732]
38. ' D' (Carranza 10th ed 651] The various abrasives used in dentifrices are calcium
carbonates, silicon oxides, aluminium oxides, granular
39. ' B' [Carranza 11th ed 635 / 10th ed 895] polyvinyl ch lorides and dicalcium phosphate dihydrate. They
compose 20 to 40% of dentifrices and enhance the abrasive
40. ' D' [Carranza 11th ed 629] action of tooth brushing as much as 40 times. Tooth
Dentifrices are aids for cleaning and polishing toot h powders are more abrasive than pastes and contain 95% of
surfaces. For complete removal of plaque, tooth brushing abrasive mat erial. The abrasive qualit y of dentifrices affects
should be followed by flossing. enamel slightly and is a much greater concern for patients
with exposed roots. Dentin is abraded 25 times faster and
41. ' B' (Carranza tl'h ed 634] cementum 35 times faster than enamel.

42. ' D' (Carranza 11th ed 635 / tO'h ed 7 42] 59. 'D' (Carranza 11th ed 631 / 10th ed 737]

43. 'C' [Carranza 11th ed 634] 60. 'D' (Carranza 11th ed 484 / lO'h ed 184]

44. ' B' [Carranza 111h ed 634] 61. 'A' (Essentials of clinical periodontology by Shanthi Priya
Reddy 2nd ed 281]
45. ' B' (Carranza ll' h ed 631 / 10th ed 737] • There should be 80-86 bristles per tuft
• Head of the brush should be 1-11/4 inch by length.
46. ' D' [Dental hygiene theory as practise 477]
Dental ;lut.,e

According to A. D.A., the acceptable dimensions of


tooth brush are: (AIIMS MAY 2012)
Ans: 1 - l1/4 inch long, 5/16 - 3/8 inch wide, 2-4
rows, 5-12 t ufts per row

62. 'D' (Carranza 11th ed 634]


Long term use of CHX causes:
• Yellow brown staining of teeth
• Mucosa[ soreness
• Desquamation
• Altered taste sensation

63. 'A' [Hiremath Community Dentistry 2nd ed 419]


Two frequently used flossing methods are spool and circle
(loop) methods.

Spool technique:
It is recommended for teenagers and adults who have
acquired t he level of neuromuscular coordination and mental
maturity to use floss.

Loop/ Circle technique:


This method is particularly suited for children as well as
adults with less nimble hands or handicaps such as poor
muscular coordination or arthritis.

64. 'A' (Carranza 11th ed 432)


Full mouth disinfection concept:
• Full mouth debridement (scaling and root planing)
completed within 24 hour period.
• Tongue is brushed with 1% chlorhexidine gel for 1 min.
• Mouth is rinsed with 0.2% chlorhexidine solution for 2
min.
• Periodontal pockets are irrigated wit h 1% chlorhexidine
solution.

65, 'C' [Periodontics revisited by Shalu Bathla 1st ed 279)


Studies on plaque removal effectiveness demonstrated that
there is no greater efficiency with powered tooth brushes
over manual tooth brushes. The initial improvement seen
with powered tooth brushes is due to Hawthorne effect.

Hawthorne effect: The effect which occurs when research


subjects behave differently because they realize that they
are being observed.
, PERIODONTICS
~

V
593

14. GINGIVECTOMY AND CURETTAGE


1. A conventional gingivectomy will c) Papillary enlargement.
a) Eliminate infra-bony pockets d) Enlargement due to acute systemic disease.
b) Eliminat e false pocket s (AIPG -04)
c) Preserve width of attached gingiva 12. After curettage re-epithelisation occurs in
d) Facilitate healing by primary intention a) 1-2 days b) 2-3 days
(MAN -94, AIIMS -93) c) one week (2-7 days) d) two weeks
2. Indication of gingivectomy is (MAN -2K, KCET -07)
a) edema of gingiva b) infrabony pockets 13. Soft tissue curettage is used for:
c) adequate attached gingiva a) Shallow pockets with gingivitis
d) pockel deplh below mucogingival junclion b) Deep pockets wit h gingivitis
(MAN -99) c) Infrabony pockets d) Oedematous gingiva
3. The reshaping process of gingiva in the absence of (AIIMS -97)
periodontal pocket is 14. Soft tissue curettage is indicated in:
a) curettage b) gingivoplasty a) Periodontal pockets b) Edematous gingival pockets
c) gingivectomy d) flap operation c) Fibrous gingival pockets d) Supragingival calculus deposits
(AP -03) (AIPG - 98)
4. Knife used for gingivectomy is: 15. The procedure preferred for periodontal surgery in
a) Kirkland b) Orbans anterior teeth
c) Merifield d) Burkett's a) Gingivect omy b) Periodontal flap
(PGI -99) c) Subgingival curettage d) All of t he above
5. Gingival deformities due to fixed orthodontic treatment (AIPG -92, 98)
can be corrected by: 16. Epithelialisation of gingivectomy wound is completed by
a) Gingivectomy b) Gingivoplasty a) 4 days b) 1 month
c) Soft tissue curettage d) All of the above c) 4 months d) 2 months
(AIPG - 91)
6. Gingivectomy is contraindicated in:
a) Gingival hyperplasia b) Suprabony defects
c) Infrabony pockets
d) Phenytoin induced gingival hyperplasia
(AIPG -95)
7. Ditantin hyperplasia is treated with:
a) Gingivectomy b) Gingivoplasty
c) Apically repositioned flap
d) Curettage
(AIPG -94)
8. Gingivectomy
a) Removes supra gingival pockets
b) Removes enlargements
c) Removes supragingival abscess
d) All
(AP -01)
9. Electrocogulation is used for
a) Gingivectomy b) Flap surgeries
c) Mucogingival surgeries d) All of the above
(AP - 2K)
10. The most recommended technique of gingivectomy is by
means of
a) chemicals b) lasers
c) electrodes d) Scalpels
(AP-2013)
11. In which of the following conditions gingivectomy is
contraindicated:
a) Gingival sulcus beyond mucogingival j unction
b) Enlargement of the gingiva

1) B 2) C 3) B 4) A 5) B 6) C 7) A 8) D 9) A 10) D 11) A 12) C 13) A


14) B 15) B 16) B
Dental ;lut.,e

14. GINGIVECTOMY AND CURETTAGE - ANSWERS

1. 'B' (Carranza 11th ed 819 / 10th ed 912) 11. 'A' (Carranza 11th ed 819 / lOth ed 913]
Indications of gingivectomy:
• Elimination of suprabony pockets if the pocket wall is 12. 'C' (Carranza 11th ed 818 / lOth ed 912]
fibrous. Epithelialisation occurs within 2-7 days and immature
collagen fibers appear within 21 days.
• Elimination of gingival enlargements.
• Elimination of suprabony periodontal abscess. 13. 'A' [Carranza 11th ed 816 / tOth ed 909]
Gingival curettage is done for the removal of inflamed soft
2. 'C' (Carranza 11th ed 819 / lOth ed 913) tissue lateral pocket wall.

3. 'B' (Carranza 11th ed 820 / 10th ed 914) Subgingival curettage is done apical to junctional epithelium
Gi ngivoplasty is the re-shaping of the gingiva to create in which connective tissue attachment is severed down to
physiologic gingival contours in the absence of pockets. the osseous crest.
Gingival clefts, craters and the shelf like interdental papilla
caused by ANUG are indications of gingivoplasty. 14. ' B' [Carranza 11th ed 816 / tOth ed 910]

4. 'A' (Carranza 11th ed 820 / 10th ed 913) 15. 'B' [Carranza 11th ed 817 /10th ed 910]
Option 'B', orbans knife and Option 'C' Merrifield knife are Curettage causes shrinkage of interdental papilla so not
used for supplemental interdental incisions. indicated for anterior teeth.

5. 'B' 16. ' B' [Carranza 111h ed 820 / 10th ed 915]


Deformities in the gingiva that interfere with food After gingivectomy, complete epithelialisation of the
excursion, collect plaque and aggravate the disease process surgical wound is completed in about 1 month where as
are eliminated by gingivoplasty. connective tissue repair is completed in about 2 months (7
weeks). The maximum flow of gingival fluid occurs 1 week
6. 'C' (Carranza 11th ed 819 / lOth ed 913) after gingivectomy coinciding with maximum inflammation.
Contra indications of gingivectomy:
• The need for bone surgery or for examination of bone
shape and morphology.
• Bottom of pocket is apical to mucogingival junction.
• Aesthetic regions Eg.: Anterior maxilla.

Disadvantages of gingivectomy:
• Long time to heal.
• Loss of attached gingiva.

7. 'A' (Carranza 11th ed 819 / tOth ed 913)

8. 'D' (Carranza 11th ed 819 / tOth ed 913)

9. 'A' [Carranza 11th ed 821 / lOth ed 915)


Electro surgery can be used for gingivectomy, gingivoplasty,
relocation of frena l attachments, incision of periodontal
abscess and pericoronal flaps. It is not used for procedures
that involve proximity to the bone Eg:- flap and mucogingival
surgeries.

Disadvantages of electrosurgery:
• Cannot be used in poorly shielded cardiac pacemaker.
• Causes unpleasant odor.
• Necrosis of bone occurs if contact with bone occurs.

10. 'D' (Carranza 11th ed 821]


The gingivectomy technique may be performed surgically
by means of scalpels, electrodes, laser beams, or chemicals
although the first one is the only one generally recommended.
, PERIODONTICS
~

V
595

15. FLAP & MUCOGINGIVAL SURGERY


1. An apically repositioned flap 10. If periodontal surgery is necessary in case of horizontal
a) Does not preserve the attached gingiva bone loss in upper anterior region. Which of the following
b) Does not increase the length of clinical crown procedure is the most appropriate?
c) Is the procedure of choice for palatal pockets a) Papilla preservation flap
d) Is a pocket elimination procedure b) Widman flap
(MAN -94, AIIMS -93) c) Coronally displaced flap
2. Periodontal fla p surgery is most difficult in: d) Gingivectomy
a) Incisors (facially) b) Incisors (lingually) (AIPG -04, 06, AIIMS -07)
c) 2nd molars (facially) d) 2"d molars (distally) 11. The procedure preferred for periodontal surgery in
(AIPG - 01) mandibul,ir anterior teeth is
3. Which is true about mucogingival flap designs a) Gingivectomy
a) Flap should be wider at the base b) Periodontal flap
b) Flap should be narrower at the base c) Subgingival curettage
c) Flap margins should not rest on bone d) Grafting
d) Mucogingival flaps should be avoided (AIPG - 98, 01, AIIMS - 94)
(AIIMS -91) 12. In a free gingival graft, what happens to epithelium of
4. In periodontal flap most important factor to be the graft? It:
considered is: a) Remains as such b) Proliferates
a) Depth of vestibule c) Degenerates
b) Frenum attachment d) Has to be removed by the surgeon
c) Amount of attached gingiva (AIIMS -92, 00, AIPG - 03)
d) Free gingiva 13. The success of a free gingival graft procedure depends
(AIIMS -95) upon which of the following
5. The common goal of all periodontal flap procedures is to: a) The graft being immobilized at the recipient site
a) Correct mucogingival inadequacies b) The donor tissue being as thick as possible and containing
b) Remove diseased granulation tissue periosteum
c) Provided access for instrumentation c) A thick blood clot remaining between the recipient and
d) Add bone support where it has been lost donor tissue
(AIPG -90) d) Periosteal fenestration being present
6. Which of the following has greatest impact on success of (AIPG -97)
a periodontal flap procedure? 14. Which of the following is a mucogingival surgery
a) Level of postoperative plaque control a) Free gingival graft b) Widman flap
b) Level of flap at closure c) Gingivectomy d) Gingivoplasty
c) Extent to which flap is reflected (KAR - 01)
d) Type of incision 15. In Tarnow's technique, the method described for denuded
(AIPG 89, 92) root surface is
7. It is most difficult to perform apically positioned flap a) Semilunar coronally repositioned flap
procedure in: b) Split thickness coronally repositioned flap
a) Facial surface of incisors c) Free soft tissue autograft
b) Lingual surface of incisors d) Subepithelial connective flap
c) Facial surface of third molars (AP -2K, COMEDK- 2013)
d) Lingual surface of third molar 16. GTR is related to
(AIPG -91) a) Gingivoplasty b) Frenectomy
8. Infra-bony pockets are treated by: c) Mucogingival surgery d) Gingivectomy
a) Gingivectomy. b) Gingivoplasty. (PGI - 03)
c) Flap operation d) Vestibuloplasty. 17. Placement of graft will be failure in which class of
(KAR -98) recession
9. The flap technique for pocket elimination and to increase a) class I b) Class II
in width of attached gingiva is c) Class III d) Class IV
a) Coronally repositioned flap (AIIMS -2K)
b) Apically repositioned flap 18. Method of increasing attached gingiva
c) Lateral pedicel flap a) Apical flap b) Widman flap
d) Modified widman flap c) Undisplaced flap d) Modified widman flap
(AP-14, KAR -98, AIIMS- 07) (AIIMS -04)

1) D 2) D 3) A 4) C 5) C 6) A 7) C 8) C 9) B 10) A 11) B 12) C 13) A


14) A 15) A 16) C 17) D 18) A
,,-,
596
V Jv --'y -'y 1 Dental ;lut.,e i======
19. A lateral sliding flap done on a tooth with roots denuded 30. A split thickness flap is indicated when:
for 6 years will show a) Osseous craters are present
a) PDL fibres attached to cementum b) Fenestrations and Dehiscence are suspected
b) PDL fibres attached to dentin c) Buttressing bone formation is present
c) Epithelium adjacent to previously denuded root surface d) Three walled osseous defects are encountered
d) Co adaptation to the remaining fibres (COMEDK-05, PGI-05)
(AIPG -02, AIIMS -99) 31. The internal bevel incision of a periodontal flap:
20. Best position for apically displaced flap is a) is directed to the base of the pocket
a) At the alveolar crest b) removes the keratinized tissue completely
b) At 2mm apical to alveolar crest c) is a vertical incision
c) At 1mm apical to alveolar crest d) remove the lining of the pocket wall
d) At 1mm coronal to alveolar crest (COMEDK-05)
(AIPG -02) 32. In modified Widman flap:
21. Lateral repositioning flap is done for a) Internal bevel incision is the last incision
a) gingival recession b) high frenal attachment b) Interdental papilla is eliminated
c) increasing the width of attached gingiva c) Posterior areas are difficult to access
d) shallow vestibule d) Crevicular incision is made frorm base of the pocket of the
(KAR -03) bone
22. The tissue preferred for free gingival graft in order of
preference is 33. Internal bevel gingivectomy is:
a) Alveolar mucosa b) Attached gingiva a) Apically displaced flap b) Distal molar surgery
c) Edentulous mucosa d) Unattached gingiva c) Widman flap d) Undisplaced flap

23. Frenectomy is a 34. Following periodontal surgery the epithelial attachment


a) gi ngival surgery b) plastic surgery healing is completed in
c) osseous surgery d) all of the above a) 2 weeks b) 4 weeks
c) 6 weeks d) 8 weeks
24. After reflection of full thickness flap, post surgical
alveolar bone loss is greatest at 35. Edian-majehar's technique is used
a) Thin facial margin b) Thick lingual margin a) to correct the denuded root surface
c) Inter proximal bone d) No bone loss at all b) to eliminate true pocket
c) to increase the depth of vestibule
25. All of the following are definite indications of d) to gain new attachment
mucogingival surgery except (KAR-01)
a) one wall bony defect b) high frenal attachment 36. Periodontal surgery may result in
c) insufficient width of attached gingiva a) Bone regeneration
d) shallow vestibule b) Pocket elimination
c) Stabilization of the tooth
26. Thick manageable true pockets with no recession are d) None of the above
best treated by (AP-10)
a) Lange(stechnique b) apically displaced flap 3 7.Indications for mucogingival surgery include the
c) free gingival graft d) touch tunnel method following except
a) A shallow vestibule b) Insufficient attached gingiva
27. If pretreatment pocket depth is X mm and width of c) Infrabony pockets d) A high frenum attachment
attached gingiva is Y mm what will be the width of (AIPG-10)
attached gingiva after treatment 38. The advantages of the modified widman flap procedure
a) X+ Y b) X /2 + Y include all except
c) X + 2Y d) X + Y /2 a) Adaptation of healthy tissue to the tooth surface
b) Post operative interproximal architecture is normal
28. The flap technique procedure which does not increase c) The procedure is feasible when implantation of bone is
the width of attached gingiva: contemplated
a) Apically positioned flapb) Undisplaced flap d) Conservation of bone & optimal coverage of root surfaces
c) Fenestration operation d) Free gingival graft by the soft tissues
(PGI- 06) (AIPG-10)
29. The free gingival graft is placed on: 39. The reverse bevel incision is made to
a) On the periosteum b) Bone a) Allow atraumatic reflection of the gingival margin
c) Gingiva d) Periodontal pocket b) Remove the infected tissue in the sulcus
(PGI-05, COMEDK- 2013) c) Provide access to the alveolar crest

19) C 20) B 21) A 22) B 23) B 24) A 25) A 26) B 27) B 28) B 29) A 30) B 31) D
32) D 33) D 34) B 35) C 36) B 37) C 38) B 39) B
, PERIODONTICS
~
597
V
d) All of the above
(AIPG-10, AIIMS MAY 2012)
40. Ideal thickness of a free gingival graft obtained from the
palate is
a) 0.5 - 1 mm b) 1-1.5 mm
c) 1.5 - 2.0 mm d) 2.0 - 2.5 mm
(KCET-2011, COMEDK- 2013)
41. A coronally positioned pedicle graft is used when a
single tooth exhibits gingival recession and (BHU-2012)
a) Bleeding b) Pus discharge
c) Irreversible pulpitis d) Sensitivity

42. In a heart patient with replaced valves, a periodontal


surgery is planned only for gingival tissue then:
a) No need of antibiotic b) Should not be planned
c) Antibiotic coverage is required
d) 0.2% chlorhexidi ne mouth wash is sufficient
(NEET-2013)
43. Which of the following is essentially an excisional
procedure of the gingiva?
a) Modified widman flap b) Undisplaced flap
c) Apically displaced flap d) Papilla preservation flap
(AP-2013)
44. Periodontal surgery is best performed.
a) 4 weeks after completion of the hygienic phase
b) 8 weeks after the completion of the restorative phase
c) 4 weeks before completion of occlusal adjustments
d) Immediately after the completion of the hygienic phase
(AIIMS MAY 2012)
45. What is the type of suture shown below?

a) Sling Ligation b) Closed anchor suture


c) Periosteal suture d) Distal wedge suture
(APPG-15)
46. The strip technique which is a variant of free gingival
graft was developed by
a) Rateit-Schak et.al b) Han et.al
c) Edel et.al d) Bjorn et.al
(MHCET-15)
47. The treatment option for a large area of gingival
enlargement (>6 teeth) with osseous defects and
(limited keratinized tissue)
a) Gingivectomy b) Curettage
c) Periodontal flap d) Gingivoplasty

48. Granulation tissue is replaced by connective tissue in?


a) 7 days b) 14 days
c) 21 days d) 1 month
( PGI DEC-2011)

40) B 41) D 42) C 43) B 44) A 45) D 46) B 47) C 48) C


Dental ;lut.,e

15. FLAP & MUCOGINGIVAL SURGERY - ANSWERS

1. ' D' [Carranza 11th ed 841 / 10th ed 927) 10. 'A' [Carranza 11th ed 847 / 10th ed 886)
• Apically displaced flap can be used for pocket elimination
or widening the zone of attached gingiva. 11. ' B' [Carranza 11th ed 847 / 10th ed 885)
• The pocket wall is transformed into attached gingiva. 12. 'C' [Carranza 11th ed 915 / 10th ed 1010]
• It increases the length of clinical crown and is not The free gingival autograft is used to increase the widt h of
indicated for palatal pockets. attached gingiva or to cover the denuded roots. The graft
• The width of the attached gingiva is increased consists of epithelium and a thin layer of connective tissue
approximately half the pretreatment pocket depth. taken from keratinised donor sites. The ideal thickness
of the graft is 1 to 1.5 mm . The epilhelium or lhe grart
2. ' D' [Carranza 11th ed 847 / 10th ed 945) degenerates and is replaced newly from the borders of
Treat ment of periodontal pockets on the distal surface of recipient site.
terminal molars is complicated by the lack of sufficient
attached gingiva and t he presence of bulbous fibrous tissue 13. 'A' [Carranza 11th ed 914 / 10th ed 1010]
over tuberosity and retromolar pad. The free gingival graft should be immobilized at the recipient
site because movement will interfere with healing. Excessive
3. 'A' tension will warp the graft. Thick blood clot interferes with
The 3 basic requirements of a flap: graft vascularity. Respect for tissues is essential for success.
• It should provide an adequate exposure of the surgical area.
14. 'A' [Carranza 11th ed 910 / 101h ed 1008)
• Flap must have a broad base and good vascular supply.
The following are the mucogi ngival (plastic) surgical
• When placed back after surgery, flap should rest on techniques:
healthy bone.
I) Techniques of increasing the width of attached gingiva
4. 'C' Eg.: Free gingival grafts, apically displaced flap.
Adequate width of attached gingiva is must for optimal
II) Techniques for coverage of denuded roots
gingival health as it resists inflammation more efficiently
and is more protective against the accumulation of plaque. Eg.: Laterally displaced pedicle graft, coronally displaced
flap, free gingival graft etc.
5. ' C' [Carranza 11 1h ed 824 / 10th ed 926)
III)Techniques to deepen the vestibule.
Flap technique provides:
Eg.: free aut ogenous grafts.
• Adequate visibility and accessibility to the root deposits.
• Eliminates or reduces pocket depth. IV) Techniques for the removal of frenum .
• Expose the area to perform regenerative procedures.
15. 'A' [Carranza 11th ed 920 / 10th ed 1019]
6. 'A' Tarnow's technique Semilunar coronally displaced flap
Millers technique Free soft tissue autograft
7. 'C' [Carra nza 8th ed 612)
Langers technique Subepithelial connective tissue graft
8. 'C' [Carranza 11th ed 841 / 10th ed 937] Touch and tunnel Variant of Langer's technique
technique
9. ' B' [Carranza 11th ed 841 / 10th ed 937)
Condition Preferred flap All these techniques are used to cover the surface of denuded
roots.
• For accessibility in anterior teeth
segment Papillary
preservation flap 16. ' C' [Carranza 11th ed 923 / 10th ed 1021]
• For reconstructive osseous surgery
• For accessibility in non-aesthetic Modified Widman 17. ' D' [Carranza 11th ed 917 / 101h ed 1015]
zone flap Miller's classification of Gingival recession
• For osseous defects closure by Marginal tissue recession that does not extend
bone recontouring Apically displaced Class I to mucogingival junction. No soft tissue or
• Decreased width of attached flap bone loss in interdental area.
gingiva with thick pocket wall Marginal gingival recession extends to
• Long narrow gingival defect on Laterally Class II mucogingival junction. No soft tissue or bone
single tooth displaced flap loss in interdental area.
, PERIODONTICS
,......__,,_

V
599

Marginal recession extends beyond MGJ. There 28. ' B' (Carranza 11th ed 913 / 101 hed 1014, 941]
Class III is soft tissue and bone loss interdentally or Free gingival graft and Apically displaced flaps are done
malpositioning of the tooth. to increase the width of attached gingiva.
Recession extends beyond MGJ with severe Also Fenestration operation (partial thickness flap) was
Class IV soft tissue and bone loss with severe tooth designed to widen t he zone of attached gingiva with a
malposition. minimum loss of bone height.

The prognosis for classes I & II is excellent where as for Undisplaced flap surgically remove the pocket. It does not
class III partial coverage can be expected. Class IV recession increase the width of attached gingiva, instead it decreases
has very poor prognosis. the width. Undisplaced flap is essentially an excisional
procedure of the gingiva.
Criterion to distinguish between Miller class II and Ill
recession is (COMEDK-2013) 29. 'A' (Carranza 11th ed 913 / 10' hed 1008]
Ans: Interdental bone Periosteum should be left covering the bone. It serves as a
recipient bed.
18. 'A' (Carranza 11th ed 841 / 10th ed 927]
Grafts can also be placed directly on bone fissure. This is a
19. 'C' (Carranza 11th ed 920] variant technique.

20. ' B' (Carranza 11th ed 858/ 1Qth ed 1014] 30. ' B' (Carranza 11th ed 824 / 101 hed 926, 1014]
The edge of the apically displaced flap when placed 2mm
apical to the crest produces the most desirable gingival 31. ' D' (Carranza 11th ed 825 / 1Q•hed 928]
contour. External bevel incision (gingivectomy) is direct ed towards
the base of pocket and removes the pocket wall.
21. 'A' (Carranza 11th ed 919 / 10th ed 1015]
Internal bevel incision (first incision) removes the pocket
CONDITION FLAP INDICATED
lining.
Long narrow gingival defect Laterally displaced flap.
on sing le tooth 32. ' D' (Carranza 11th ed 825 / 10'hed 940]
Absence of attached gingiva Apically displaced flap . a - Internal bevel incision is the first incision.
with thick pocket wall b - Effort is made to adapt the facial and lingual
Absence of attached gingiva Free gingival graft. interproximal tissue.
with friab le pocket wall or c - Mainly indicated to provide access to the posterior
no pocket areas.
Isolated recession on upper Tarnow's semilunar
teeth coronally displaced flap. 33. ' D' (Carranza 11•h ed 843 / 10'h ed 940]

22. ' B' (Carranza 8th ed 655] 34. ' B' (Carranza 11th ed 833 / 10th ed 935]
The order of preference for a free gingival graft is attached After PD surgery, immediate recall 1 week for removal
gingiva, masticatory mucosa from edentulous ridge, and is done after of pack
palatal mucosa.
After curettage re-epithelisation One week or
23. ' B' (Carranza 11th ed 926 / 1Qth ed 1005] occurs in 2-7 days
Frenectomy is the mucogingival (plastic) surgical procedure Epithelialisation of gingivectomy
4 weeks or 1 month
for the removal of high frenal attachments. wound is completed by
Extractions can be done in ANUG
24. 'A' (Carranza 1Qth ed 1016] 4 weeks
patients after
The disadvantages of full thickness flap are the loss of facial
Establishment of epithelial
bone height particularly if t he bone is thin. It is usually not 1 week
attachment after surgery
preferred in cases of fenest rations and dehiscence.
Complete healing of epithelial
4 weeks
25. 'A' (Carranza 11th ed 910 / 10th ed 1005] attachment after surgery

26. ' B' (Carranza 11th ed 841] 35. 'C' (Carranza 8th ed 662 Fig 59-13]

27. ' B' (Carranza 8th ed 660] 36. ' B' (Carranza 11th ed 841 / 10'h ed 926,927]
The width of attached gingiva is increased by approximately
half the pretreatment depth of the pockets.
Dental ;lut.,e i======
37. 'C' [Carranza 11th ed 910 / 10th ed 1005, 1006] 45. 'D' [Check Explanation Below]
The term mucogingival surgery is used to describe surgical Periodontal pockets distal to last tooth in an arch, as well
procedures for the correction of relationships between the as pockets on lone standing te·eth are often difficult to
gingiva and the oral mucous membrane with reference to eliminate surgically.
three specific problem areas: attached gingiva, shallow
vestibules and frenum interfering with the marginal gingiva. Treatment in those areas is often complicated by:
• the presence of bulbous fibrous tissue over the maxillary
38. 'B' [Jan Lindhe 3 n1 ed 520) tuberosity
• prominent retromolar pad in the mandible
39. 'B' [Carranza 11th ed 825 / 10th ed 928, 929]
Internal bevel incision is also known as reverse bevel • inadequately attached gingiva
incision. It is the incision from which the flap is reflected • abruptly ascending tuberosity
to expose the underlying bone and root. • a close ascending ramus of the mandible

Objectives: The gingivectomy is the first approach in treating distal


• It removes pocket lining pockets that have no osseous lesions and adequate amount
• Conserves the uninvolved outer surface of the gingiva of attached gingiva. However, when only limited amounts
or no keratinizated tissue is present or an angular osseous
• Produces a sharp, thin flap margin for adaptation to the
deformity is present, distal wedge procedure is advocated.
bone-tooth junction
Objectives of distal wedge procedure:
40. ' B' [Carranza 11th ed 914 / 10th ed 77 OJ
Proper thickness is important for survival of the graft. It should • to maintain and preserve attached gingiva.
be thin enough to permit ready diffusion of nutritive fluid • to eliminate periodontal pocket
from the recipient site, which is essential in the immediate • to lengthen clinical crown
post-transplant period. A graft that is too thin may necrose
• to create easily cleansable gingiva-alveolar form
and expose the recipient site. If the graft is too thick, its
peripheral layer is jeopardized because of the excessive tissue
46. ' B' [Check Explanation Below]
that separates it from new circulation and nutrients. Thick
Four variations to the classic free gingival graft technique
grafts may also create a deeper wound at donor site, with the
has been employed.
possibility of injuring major palatal arteries.
• Accordian Technique
The ideal thickness of a graft is between 1.0 and 1.5 mm. • Strip Technique
After the graft is separat ed, loose tissue tabs from the • Connective tissue
under-surface should be removed and the edges are thinned
• Combination technique
to avoid bulbous marginal and interdental contours.

41. 'D' [Carranza 11th ed 920 / 10th ed 1006 & 1019) • Accordian technique involves the use of the alternate
incisions on the opposite sides of the graft to attain
42. 'C' [Davidson 20th ed 633] graft expansion (Rateit Schak 1985)
All patients with prost hetic valves should receive antibiotic • Strip technique consists of obtaining two or three
prophylaxis before dental procedures that involve strips of about 1mm wide and long enough to cover
manipulation of gingival tissue or the periapical region of the entire length of the recipient site (Han, Carranza Jr,
the teeth or the oral mucosa. Takei 1993). These stri pes are placed at the base and at
the cent re of the recipient site and secured by sutures.
43. ' B' [Explanation of Q.No. 28] This prevents large donor site and facilitate healing in
Undisplaced flap is essentially an excisional procedure of the the donor site. But greater shrinkage is expected in the
gingiva. graft.

44. 'A' [Carranza 11th ed 622) • Connective tissue technique involves the use of
Phase I (etiotropic phase) therapy includes scaling, root connective tissue only as the graft material, as it carries
planning and oral hygiene instructions (hygiene phase) as well the genetic message for the overlying epithelium to
as caries control, occlusal therapy, replacement of defective become keratinize (Eden 1974).
restorations, orthodontic movement and smoking cessation. • Combination technique uses the connective tissue
• Many patients have their periodontal disease controlled graft at the apical or deeper portion of the recipient site
with phase I therapy and do not require further surgical and the epithelial grail on the superficial portion of the
intervention. recipient site.
• Assessment is generally made 1 to 3 months after
completion of phase I therapy
• For other phases, refer synopsis.
, PERIODONTICS
,......__,,_

V
601

47. 'C' [Shanthipriya Reddy 2nd ed 157]


Treatment for drug induced gingival enlargements includes:
a) First step:
• Oral hygiene reinforcement
• Possible drug substitution
• Recall follow-ups at least 6-12 months

b) Second step: If en largement persists


• For small areas of enlargements, gingivectomy is
employed. For large areas of enlargement with presence
osseus defects and limited keratinized gingiva,
periodontal flap surgery may be indicated.

48. 'C' [Check Explanation of Q.No.12]


Collagen is the primary component of replaced connective
tissue. Immature collagen fibers appear within 21 days.
Dental ;lut.,e

16. TREATMENT OF FURCATION INVOLVEMENT & OSSEOUS SURGERIES

1. For root resection the choice of therapy is 12. Which of the following grafts has the best prognosis?
a) disto buccal root of upper first molar. (OR) Which of the following graft has high success rate?
b) Mesia buccal root of upper first molar a) Xenograft b) Autograft
c) Palatal root of upper first molar c) Allograft d) Homograft
d) Both mesio buccal and disto buccal root (MHCET-15, AIPG -94)
(KAR -99) 13. Citric acid is applied to root surface:
2. If half of the crown along with root is separated or one a) To reduce caries b) To reduce sensitivity
half is removed is called as: c) To facilitate collagen fibre adhesion on to root surface
a) Radisection b) Hemisection d) None of the above
c:) Apexific:alion d) Coronosec:lion (AP - 97, AIPG - 94)
(AP -2K) 14. Which of the following procedures is least successful in
3. After endodontic therapy has been perfomed on a resolving a periodontally related osseous defect
mandibular molar with both periodontal bifurcation a) Osseous coagulum graft b) Resection surgery
involvement and bifurcation caries: c) Reattachment procedure d) Alloplast graft
a) Hemisection b) Fenestration (AIPG - 98)
c) Root amputation d) Apical curettage 15. The best bone graft which can be utilized for
(AIPG -93, 2002) reconstruction is:
4. Odontoplasty is indicated in: a) Costo chondral graft b) Ca.lvarial graft
a) Grade I furcation involvement c) lilac crest graft d) Metatarsal bone graft
b) Grade II furcation involvement (AP -97)
c) Grade III furcation involvement 16. According to Melcher concept, the regeneration of new
d) Grade IV furcation involvement attachment is from
(PGI -99) a) periodontal ligament b) gingival connective tissue
5. Prognosis is poor for furcation involvement of c) alveolar bone d) junctional epithelium
a) Maxillary l't premolar b) Maxillary 2nd molar (AP - 00)
c) Mandibular 1'1 premolar d) Maxillary 2 nd premolar 17. Bioglass is used in:
(MAN -2K) a) Root biomodification
6. Furcation involvement with no radio graphic evidence is b) Resective osseous surgery
a) Grade II b) Advanced grade II c) Regeneration osseous surgery
c) Grade I d) Early grade III d) Mucogingival surgery
(KAR -95) (KAR -03)
7. Root conditioning with citric acid helps in 18. Chemicals used in root preparation are:
a) Removal of endotoxin b) Removal of smear layer a) Citric acid b) Fibronectin
c) Regeneration of bone d) All the above c) Tetracycline d) All of the above
(MAN -01) (KAR -01)
8. The graft with the maximum osteogenic potential is: 19. Anorganic bone is an example of:
a) Allogaft b) xenograft a) Autografts b) Alloplasts
c) Autocortical d) autocancellus c) Xenografts d) Allografts
(AIPG - 03, MAN - 2K) (KAR - 01)
9. Which of the following is a major contraindication to 20. If interdental bone is apical to interradicular bone, it is
resection of crestal bone? a) positive architecture b) negative architecture
a) Gingival recession b) Post surgical infection c) plateau d) none
c) Removal of healthy boned) Weakening (AP - 01)
(AIPG -03) 21. Biobrane is a commercially available:
10. Craters in interseptal bone are best eliminated by a) Root matrix protein b) Biodegradable membrane
a) Gingivectomy b) Supragingival curettage c) Bone graft material d) Non-bone graft material
c) lnterseptal massage with stimulators (KAR -03)
d) Osseous recontouring 22. Ideal requisites of barrier membrane include all of the
(AIIMS - 89) following except
11. Guided tissue regeneration (GTR) is done in: a) Cell occlusivity b) Tissue compatibility
a) Vestibular deepening procedures c) Space making d) Biodegradable
b) Frenectomy (KAR - 99)
c) Flap surgeries d) Gingivectomy
(PGI -97, 98}

1) A 2) B 3) A 4) A 5) A 6) C 7) D 8) D 9) D 10) D 11) C 12) B 13) C


14) C 15) C 16) A 17) C 18) D 19) C 20) B 21) B 22) D
, PERIODONTICS
,.....__,,_

V
603

23. The best way to fill a bony defect will be with which of 35. What do you understand by isograft?
the following? a) Bone taken from same individual
a) Hydroxy apatite b) Donor graft b) Bone taken from generally similar individual
c) Xenograft d) Autograft c) Bone taken from identical twin
(AIIMS -94) d) Bone taken from the same individual
24. Bone graft procedures are most successful in treating: (AIPG-06)
a) In furcation involvements 36. Which of the following procedure is a sequential step of
b) Deep, two-wall craters osseous resection technique?
c) Narrow three-wall defects a) Hemisection b) Root resection
d) Osseous defects with one remaining wall c) Radicular blending d) Tunneling
(AIPG -91) (COMEDK-05)
25. Graft when taken from a different species is called a: 37. Which one of the following is the role of barrier
a) Xeno graft b) Auto graft membrane in GTR?
c) Allo graft d) Homo graft a) To help overall healing
(AIPG - 94) b) Prevention of epithelial migration
26. Which of the following material has osteo conductive c) To stop bleeding
property d) To prevent the underlying tissues from the infection
a) Autogenous graft b) Hydroxyapatite (AIPG-06, AIIMS-07)
c) Plastic materials d) Cartilage 38. Which of the following is used as a bio-degradable
(AP -02) membrane in "guided tissue regeneration" process?
27. Ostectomy procedure is indicated in: a) Polylactic acid b) Poly tetrafluoroethylene
a) Physiological architecture of alveolar bone. c) Millipore d) Core membrane
b) Correction of Tori. (AIIMS-06)
c) Reverse archit ecture bone defects. 39. In the following periodontal surgeries the one that is
d) Correction of bone ledges. performed first:
(AIPG -04) a) Infrabony pocket t herapy b) Gingivectomy
28. Hydroxyapatite (t. 67) CaP0 4 c) Apically displaced flap d) Gingivoplasty
a) osteoinductive b) osteoconductive (AP-06)
c) both A and B d) None of the above 40. Bioactive glass is a:
(PGI - 96) a) Isograft b) Xenograph
29. The three walled defect responds best to inductive or c) Allograft d) Alloplast
a) Degranulation procedures (COMEDK-05)
b) Deroofing procedures 41. Transplantation between genetically different members
c) Deinvagination procedures of the same species is termed as
d) All of the above a) Autograft b) Isograft
(BHU-2012) c) Allograft d) Xenograft
30. Decalcified freeze-dried bone graft is an example of (COMEDK-201 1)
a) Xenograft b) Autograft 42. Regeneration in periodontal therapy aims at:
c) Alloplast d) Allograft a) Bone refill
(KAR - 99) b) Repair of the bony defect
31. Iliac crest graft should ideally be taken from c) Reconstitution of t he lost periodontium
a) Lateral aspect b) Medial aspect d) Removal of the diseased bone
c) Posterosuperior aspect d) Antero inferior aspect (KCET-07)
(KAR -98) 43. Which is an example of non graft associated regenerative
32. Regeneration of lost periodontium involves technique?
a) Healing by new attachment formation a) HTR polymer b) Bioactive glass
b) Healing by long junctional epit helium formation c) Osseous cogulum d) GTR
c) Healing by re-attachment (COMEDK- 07)
d) Healing by scar formation. 44. Which of the following bone defects offers the best
(TNPSC -99) chance for bone fill?
33. Ramping can be done for a a) 3 Walled defect b) 2 Walled defect
a) one wall defect b) two wall defect c) Osseous crater d) Hemisepta
c) three wall defect d) four wall defect (AIIMS-09 (M))
(COMEDK - 04) 45. Sequential steps for resective osseous surgery are
34. Radicular blending is purely a procedure a) radicular blending, vertical grooving, flattening
a) Osteoplasty b) Ostectomy interproximal bone, gradualizing marginal bone
c) A & B d) None of the above b) vertical grooving, flattening interproximal bone,
gradualizing marginal bone, radicular blending

23) D 24) C 25) A 26) B 27) C 28) B 29) A 30) D 31) B 32) A 33) A 34) A 35) C
36) C 37) B 38) A 39) A 40) D 41) C 42) C 43) D 44) A 45) C
Dental ;lut.,e i======
c) vertical grooving, radicular blending, flattening c) Osteoi nduction d) Osteopromotion
interproximal bone, gradualizing marginal bone (GCET-14)
d) vertical grooving, radicular blending, gradualizing 56. Healing of a wound which simply restores the continuity
marginal bone, flattening interproximal bone of the diseased marginal gingiva is known as
(KCET-10) a) Regeneration b) new attachment
46. If an osseous fill procedure is successful, the type of c) Repair d) reattachment
bone formed in the defect is? (MCET-14)
a) Cancellous b) Bundle 57. Which of the following is most likely side effect of fresh
c) Spongy d) None of the above autogenous iliac crest transplant in managing an infra
(AIPG-09) bony pocket?
47. Decalcified freeze dried bone allograft is considered as a) Infection b) Arthus reaction
a) osteogenic b) osteoinductive c) Root resorption d) Auto erythrocyte sensitivity
c) osteoconductive d) osteoplastic (PG! JUNE-2014)
(KCET-10) 58. Latest technique by which bone graft is obtained without
48. The extent of bony defects can be best estimated by - damaging vital structure, vessels/veins is based on?
a) Long cone parallel radiographic technique a) Piezoelectric vibration b) Osteotome
b) Sequential probing c) Ultrasonic/soft lasers d) Electrocautery
c) Use of Florida probe d) Transgingival probing (AIIMS NOV-14)
(COMEDK-09)
49. Fibronectin is used in periodontal therapy as -
a) Growth factor b) Bone graft
c) Biomodification of root surface
d) GTR membrane
(COMEDK-09)
50. Bone blending is a technique employed for
a) Transplanting cortical bone
b) Transplanting cancellous bone
c) Transplanting a mix ofcancellous bone and hydroxyapatite
crystals
d) Transplanting resorbable hydroxyapatite crystals only
(AIPG-10)
51. Definitive information regarding the bone architecture is
obtained by
a) Periodontal probing b) Radiographic examination
c) Combination of probing & radiographs
d) Surgical exposure of the osseous defect
(COMEDK-2011)
52. RGD motifs are
a) Fibroblasts
b) Bone Sialoproteins & Osteopontin
c) Cementoblasts d) Noncementum proteins
(GCET-14)
53. Butressing bone formation is the tissue response to
increase occlusal forces seen in
a) Stage I injury b) Stage II repair
c) Stage III repair d) None of the above
(GCET-14)
54. Allografts are grafts taken from
a) Same species and individuals who are genetically
related
b) Different species
c) Same species but individual are genetically not related
d) Same species and between genetically identical
individuals
(AP-14)
55. In growth of bone from the margins of the defect with
gradual resorption of the graft is an example for
a) Ontogenesis b) Osteoconduction

46) B 47) B 48) D 49) C 50) A 51) D 52) B 53) B 54) C 55) B 56) C 57) C 58) A
, PERIODONTICS
~

V
605

16. TREATMENT OF FURCATION INVOLVEMENT & OSSEOUS SURGERY - ANSWERS

1. 'A' [Carranza 11th ed 903 / 10th ed 997] Grade IV Moderate cases:


Maxillary 1st molar is most favourable for root resection.
Resection of distobuccal root is the choice of t herapy if • Grade III furcation with • Same as grade III wit h
furcation involvement is in between buccal roots. Furcation orifice uncovered by occlusal adjustment
involvement is Least seen in maxillary premolars. gingiva
Advanced cases:
• Extraction
The root which is preferably removed during root
resection procedure in maxillary molar is? (PGI -2014) 5. 'A' [Carranza 11th ed 523 / 10'h ed 621]
a) Dist o-buccal b) Mesio-buccal Maxillary 1" premolars offer the greatest difficulties due to
c) Palatal d) None of the above inaccessibility and presence of mesial concavity. Because of
this, the prognosis is considered as poor, when the lesion
2. ' B' [Carranza 11th ed 903] reaches the furcation area.
• Root Resection / Amputation is removal of root without • Furcation is most commonly involved in mandibular
removal of any crown portion molars.
• In Hemisection one root with its corresponding crown • Least commonly involved in maxillary 1" premolars
portion is cut and removed • The prognosis is poor if the furcation area is involved in
maxillary 1st premolars.
• In bicuspidisation the molar is simply cut without
removal of any part of crown or root.
6. 'C' [Carranza 11th ed 900 / 10'h ed 992]
Hemisection and bicuspidisation are suitable for mandibular
7. ' D' [Carranza 11th ed 871 / 10th ed 973]
molars.
Citric acid is applied ata pH of 1 for 2 to 3 minutes. It produces
3. 'P: surface demineralization that irnduces cementogenesis and
In combined endoperiodontal problems, endodontic facilitate collagen fibre adhesion on to root surface
treatment should be followed by periodontal therapy to
avoid recurrence of the endodontic infection. It removes the smear layer of microcrystalline debris,
exposes the dentinal tubules and also makes the tubules
4. 'A' [Carranza 11th ed 901 / 10th ed 995 J appear wider. It also eliminates endotoxin and bacteria from
the diseased tooth surface.
Type of furcation and its T t t l
features rea men p an
Other root biomodification materials are fibronectin and
Grade I tetracycline.
• Incipient lesion • Scaling, root planing
• Not detected by X- ray • Odontoplasty One of the following is used for root bio-modification
a) Metronidazole b) Ciprofloxacin (COMED-14)
• Suprabony pocket • Gingivectomy for
c) Penicillin d) Tetracycline
• Edematous gingiva • fibrous pockets.
Grade II Moderate cases: 8. ' D' [Carranza 11th ed 872 / 10th ed 977]
• Cul-de-sac furcatiion • Scaling, root planing Autograft / Grafts taken from one place and replaced in
• Partial penetration of • GTR with bone grafts another position in the same person
autogenous
probe • Osteoplasty Eg.: osseous coagulam, bone blend, intra
graft oral cancellous bone marrow, iliac graft etc.
• May / may not be
Advanced cases:
detected by x-ray Graft taken from different or genetically
Root resection or hemisecton not related individuals within the same
Grade III Allograft /
species. (AP- 2013)
Homograft
• Interradiculrar bone Loss • Scaling root planing Eg.: Undecalcified /decalcified freeze dried
• Through and through • bone grafts.
Tunneling (resection of
passage of probe from gingiva for accessibility) Graft taken and transferred from one species
buccal to palatal or Xenograft /
to another species.
• Root resection and Heterograft
distal side (PGI-12} Eg.: Calf bone, kiel bone and anorganic bone.
hemisection.
• Furcation is coveredby
Grafts transferred from genetically similar
gingiva Isograft
individuals (twins)
Dental ;lut.,e i======
Inert synthetic material introduced into 17. 'C' (Carranza 11th ed 876 / 10th ed 982]
body. Bioglass is an alloplastic / non-bone graft material. It
Alloplastic consists of sodium and calcium salts, phosphates and silicon
Eg.: HTR polymer, calcium phosphate dioxide. The other alloplastic graft materials are
grafts
biomaterials, coral derived materials,
• Hydroxy apatite - Ca /P ratio of 1.67
bioactive glass etc.
• Tricalcium phosphate - Ca /P ratio of 1.5
Autogenouscancellousbone grafts have maxim um osteoge nic • HTR polymers
potential. Most of the bone grafts are osteo conductive only. • Coral derived materials.

9. 'D' (Carranza 11th ed 854 / 10th ed 951) 18. 'D' (Carranza 11th ed 871 / 10th ed 974]
Indications for resective osseous surgery:
• Interdental craters 19. 'C' (Carranza 11th ed 875 / 10th ed 980]
• One wall osseous defects Treated by detergent, sterilized and freeze
Calf bone
• Wide shallow 2 wa ll defects dried.
• Exostosis and Ox (or) calf bone denatured with 20% H202,
Kiel bone
• Supra erupted teeth. acetone and sterilized.
Anorganic Ox bone from which the organic material is
10. 'D' (Carranza 11th ed 857 / 10th ed 951) bone removed by ethelene diamine.
11. 'C' (Carranza 11th ed 870] 20. 'B' (Carranza 11th ed 853 / 10th ed 952]
Guided tissue regeneration is based on the assumption that • If interdental bone is apical to interradicular bone -
only the periodontal ligament cells have the potential for reverse (or) negative architecture
regeneration of the attachment apparatus (New attachment)
• If interradicular bone is apical to interdental bone -
of the tooth. The technique consists of placing barriers of
Postitive architecture
different types to cover the bone and periodontal ligament
to prevent the migration of epithelium. • Interradicular and interdental bone at same height - flat
architecture
12. 'B' (Carranza 11th ed 875 / 10th ed 977) • When the bone is more coronal in interdental region
Allografts and xenografts are foreign to the organism and than facial and lingual surfaces - ideal form.
therefore may provoke an immune response.
21. 'B' (Carranza 11th ed 870 / 10th ed 973]
13. ' C' (Carranza 11th ed 871 / 10th ed 973] The membranes that are kept in GTR technique to prevent
migration of epithelium are of two types
14. 'C' [Carranza 11th ed 867] • Eg: Polytetra fluoro ethelene (gore
• Regenerative (additive) osseous surgery brings about Non- tex).
ideal results. It consists of graft and non-graft associated degradable • It has to be removed 4 weeks after
new attachment procedures. surgery
• Resective (substractive) osseous surgery is an • Eg: Polylactic acid Vicryl Synthetic skin
alternative when former is not feasib le. Biodegradable (Biobrane)
• Reattachment refers to simple repair of the areas of • Resorbed after a few weeks
the root not previously exposed to the pocket such
as treatment of periapical lesions. It is not a ideal 22. ' D' (Carranza 11th ed 870 / 10th ed 973]
procedure. The barrier membranes used in GTR should be biocompatible
• New attachment is the embedding of new periodontal and have cell occlusivity and space making. The membranes
ligament fibers into new cementum and attachment of may be of degradable or nondegradable type.
the gingival epithelium to a tooth surface previously
denuded by disease. 23. ' D' [Carranza 8th ed 629)

15. 'C' (Carranza 11th ed 873 / 10th ed 977] 24. 'C' [Carranza 11th ed 851 / 10th ed 950]
TYPE OF DEFECT SURGICAL PROCEDURE
16. 'A' (Carranza 11th ed 774]
The potential for regeneration Lies in the periodontal One wall osseous defect Resective osseous surgery
ligament cells. If epithelium proliferates along root surface Three wall defects Regenerative osseous surgery
healing by Long junetional epithelium occurs, Wide and shallow two wall Resective osseous surgery
defects
If bone cells arrive first, root resorption and ankylosis occur.
If cells of periodontal ligament arrive first, new attachment Narrow and deep two wall Regenerative osseous surgery
occurs. defects
, PERIODONTICS
,......__,,_

V
607

25. 'A' [Carranza 111• ed 875 / 101• ed 976] 35. 'C' [Carranza 9 1• ed 813]

26. 'B' [Carranza 111• ed 872 / 101• ed 976) 36. 'C' [Carranza 111• ed 857 / 10thed 956)
The grafts placed in the bony defect, it may act as
37. 'B' [Carranza 111• ed 870 / 1o•• ed 972]
Osteogenic • Contains viable bone cells and ay
deposit bone in the defect
38. 'A' [Check Explanation of Q.No. 21]
• It is t he ideal property of a bone
graft
39. 'A'
Eg: autogenous cancellous bone
Ost eoind uctive • Actively induce bone fo rmation 40. 'D' [Carranza 11t• ed 876 / 10th ed 982]
Eg: decalcifi ed freeze dried bone.
Ost eoconduc- • Induces bone formation when placed 41. 'C' [Carranza 111• ed 875 / tot• ed 978]
tive next to viable bone only
Eg: Calci um phosphate bio- materials 42. 'C' [Carranza 111• ed 866 / 10'h ed 968]
Like hydroxyapat ite and t ricalcium
phosphat e. 43. 'D' [Carranza 11t• ed 870 / 10th ed 972]
Curettage, chemical agents, surgical techniques and g uided
27. 'C' [Carranza 111• ed 853 / tO' h ed 952] tissue regeneration are non-bone graft associated procedures
for regeneration.
• Osteoplasty is reshaping of bone witho ut removing
t oot h supporting bone.
44. 'A' [Carranza 111• ed 851 / 101 • ed 982]
Eg.: Correction of shallow craters bone ledges and Refer t o explanation of Q.No. 24
exostoses.
• Ostectomy is re moval of toot h support ing bone. 45. 'C' [Carranza 11t• ed 857 / 101 • ed 956]
Eg.: Correction of negative arc hitect ure, One wall
osseous defects, Flattening of inter proxi mal bone and 46. 'B' [Orban's 121h ed 200)
Grad ualising marginal bone. Bund le bone is formed in areas of recent bone apposition.

47. 'B' [Carranza 111• ed 875 / 101 • ed 979]


Ostectomy refers to (GCET-14) Decalcified freeze dried bone a llograft ( DFDBA) is considered
a) Removal of tooth supporting bones as a n osteoinductive graft whereas Freeze dried Bone
b) Remova l of gi ngiva allograft (FDBA) is considered as osteoconductive material
c) Recontouring bone d) Remova l of basal bone
48. 'D' [Carranza 111• ed 854 / 101 • ed 954]
28. 'B' [Carranza 111• ed 876 / 101• ed 982) 49. 'C' [Carranza 111• ed 872 / 10•• ed 974]

29. 'A' [Carranza 10th ed 460, 462, 950, 951) 50. 'A' [Carranza 111• ed 872 / 101 • ed 976, 977]

30. 'D' [Carranza 111• ed 875) 51. 'D' [Carranza 111• ed 854 / 10'h ed 954-955]

31. 'B' 52. 'B' [Journal of lab investigation 1999; 79 (7): 869-77]
RGD mot ifs are tripeptides of arginine-glycine-asperginine.
32. 'A' [Carranza 111• ed 776) RGD tripeptide is commonly present in extracellular
attac hment prot eins (fibronecti n) and has been shown to
33. 'A' mediat e attachment of osteosarcoma cells and osteoclasts.
Bone sialoprotein (BSP) and osteoponti n (OPN) are secreted
34. 'A' (Carranza 11•• ed 857 / 10•• ed 957] glycoproteins wit h a conserved RG D integrin-binding motif
Steps in osseous resection technique are: and a re expressed predominantly in bone and teeth.
• Vertical grooving
• Radicular blending 53. 'B' [Carranza 11th ed 200)

• Flattening interproximal bone Stages of tissue response to increased occlusal forces


• Grad ualising marginal bone Stage 1: Injury
• Show i ncrease in area of resorption and a decrease in
The first two (vertical grooving & radicular blending) are bone fo rmation.
osteoplastic procedures while the next two are (flattening
• Slightly excessive pressure :
i nterproximal bone & gradualising marginal bone) are
ostectomy procedures. Osteoplasty procedure s are done wit h - Resorpt ion of alveolar bone, widening of the PL
rotary inst ruments where as ostectomy procedures are done space
with hand inst ruments. - Blood vessels are numerous a nd reduced in size
Dental ;lut.,e i======
• Slightly excessive tension : 58 'A' (Carranza 11th ed 709]
- Elongation of PL fibers Piezosurgery is a new advanced piezoelectric ultrasonic
surgical technique. It has the advantages of low surgical
- Blood vessels are enlarged trauma, exceptional control during surgery and fast healing
• Greater pressure : response of tissues. It uses the concept of minimally invasive
surgery in osteotomy and osteop~asty procedures.
- Compression of PL fibers.
- Hyalinization of same areas of PL.
- Necrosis of areas of the ligament.
- Increased resorption of alveolar bone and resorption
of the tooth starts.
• Severe tension :
- Widening of PL
- Thrombosis and haemorrhage of blood vessels
- Resorption of alveolar bon
• Pressure enough to force the root against bone.
- Necrosis of PL
- Undermining resorption

Stage II: - Repair


• Buttressing bone formation
• Lipping

Stage III: - Adaptive remodelling of the periodontium


• Funnel shaped crest and widened PL
• Angular defect
• But no pocket formation
• Mobility

54. 'C' [Check Explanation of Q. No.8]

55. 'B' (Check Explanation of Q. No. 26]

56. 'C' (Carranza 11th ed 535]


Repair:
Repair simply restores the continuity of the diseased
marginal gingiva and re-establishes a normal gingival sulcus
at the same level on the root as the base of the pre-existing
periodontal pocket. Also called as healing by scar formation.
It does not result in gain of gingival attachment or bone
height.

Regeneration:
Natural renewal of a structure, produced by growth and
differentiation of new cells and intercellular substances to
form new tissues or parts. It is a part of healing process.

57. 'C' (Carranza 11th ed 581)


Use of fresh or preserved iliac crest grafts have been used
successfully in osseus defects with various number of walls.
Root resorption is the most common side effect of a fresh
autogenous bone graft in managing an infrabony pocket
and often extends into dentin and the pulp chamber. Other
problems like postoperative infection, bone exfoliation,
sequestration, and recurrence of the defect can also occur.
, PERIODONTICS
~

V
609

17. IMPLANTS AND MISCELLANEOUS

1. Which of the following surgical procedure is carried out b) Collagen adhesion


first: c) Scar tissue attachment
a) Mucogingival surgery b) Gingivectomy d) Connective tissue attachment
c) Flap surgery d) Osseous recontouring (AIIMS - 03)
(AIPG -94, AP-06) 13. Polyglycolic acid suture material ("Vicryl") is a/ an
2. Occlusal guards are given to a) Absorbable natural suture
a) Prevent bruxism b) To guard the dentition b) Absorbable synthetic suture
c) To alter the muscle forces d) None c) Non-absorbable natural suture
(PGI -02) d) Non-absorbable synthetic suture
3. Ocdusal adjustment is affected by all except: (MAN - 94)
a) Compensating curve b) Cusp height 14. The purpose of periodontal dressing is
c) Incisal guidance d) Postural position a) Prevent overgrowth of granulation tissue
(AIIMS - 95) b) Protect the wound from local environmental changes
4. Bruxism is mainly due to c) Make the patient comfortable after periodontal surgery
a) Psychological stress b) Diabetes d) All of the above
c) Functional grinding of teeth (MAN - 98)
d) None of above 15. 10 % KNO, is used to
(AP -02) a) treat hypersensitive dentin
5. Lugo ls solution is for b) reduce caries
a) Plaque b) Keratinised gingiva c) as an acid etchant d) as a base below restoration
c) Attached gingiva d) None (PGI -98)
16. Eugenol in a periodontal dressing could cause
6. Physiological mobility depend on a) allergic reaction b) decreased resistance to heat
a) Diurnal variation b) Person to person c) rapid bacterial growth d) a ll of the above
c) Root morphology d) All of above (PGI -00, AP-05)
17. After removal of periodontal dressing what should be
7. The term gingival ablation indicates done to keep sensitivity to the minimum?
a) Gingival recession due to faulty tooth brushing a) Use of mouthwash b) Use of desensitizing paste
b) Gingival recession accentuated by prominent palatal root c) Keep roots, plaque free d) None of the above
c) Gingival recession due to friction from hard food (AIIMS -00, AIPG -98)
d) Gingival recession due to friction from soft tissue. 18. The most effective caustic that may be used locally in
(TNPSC - 99) the treatment of hypersensitive dentin in the molars is:
8. In smokers the incidences of periodontal disease are: a) Alcohol b) Chlorhexidine
a) 2-6 times b) 6-10 times c) Trichloro acetic acid d) Silver nitrate
c) 10-14 times d) No effect (AIIMS - 93)
(PGI-2011) 19. After PD Surgery patient immediate recall is done after
9. After removal of granulation tissue from periodontal a) 1 week b) 2 weeks
pocket the hard structure seen is c) 3 weeks d) 4 weeks
a) Periosteum b) Transseptal fibres (PGI -00)
c) Sharpey's fibres 20. Not an indication for tooth grinding:
d) Organized granulation tissue a) Wear facets b) TMJ pain
(AP -04) c) Crepitus d) Cementa[ wear
10. Occlusal night guards are used to (PGI-05)
a) Prevent bruxism b) Reduce pocket formation 21. Tannie acid acts as:
c) Redistribute forces on teeth a) Astringent b) Keratolytic
d) Permit eruption or elongation of teeth c) Demulscent d) Antiseptic
(AIPG -02) (AP- 05)
11. Experimental gingivitis was first reported by 22. Bacterial adhesion on tooth is prevented by: (OR)
a) Carranza b) Newman Actinomycetemcomitans attachment to tooth surface can
c) Robert koch d) Herald Loe be prevented by?
( PGI-2011) a) Myeloperoxidase b) Collagenase
12. The soft tissue tooth interface that forms after flap c) Lactoperoxidase d) Lysozyme
surgery in a previously denuded area is (PGl-05, JUNE -13)
a) Longjunctional e~theLlum

1) C 2) C 3) D 4) A 5) B 6) D 7) D 8) A 9) B 10) C 11) D 12) A 13) B


14) D 15) A 16) A 17) C 18) D 19) A 20) A 21) A 22) A
Dental ;lut.,e i======
23. Halitosis is measured with: c) 3.5 mm d) 3.75 mm
a) Periotran b) Osmoscope (COMEDK-08)
c) Periodontometer d) None of the above 34. Periodontal infection is characterized by increased
(AP- 05) sulcula r levels of:
24. In leukemic patient gingival bleeding occurs during oral a) Interleukin b) TNF p
prophylaxis because of: c) IL - 2 d) None of above
a) Increased leukocytes count (MCET-07)
b) Increased Ca level in blood 35. Helium-neon (HeNe) lasers are example for
c) Platelet disorder d) Deficiency in clotting factor a) Excimer lasers b) Gas lasers
(AIIMS- 06) c) Diode lasers d) Solid state Lasers
25. Recent classification of periodontal disease organized by (KCET-08)
American Academy of Periodontology was introduced in 36. .................. are fundamentally dual monocular
the year: telescopes with side by side lenses convergent to focus
a) 1989 b) 1997 on the operative field
c) 1998 d) 1999 a) Loupes b) Ca.meras
(COMEDK- 06) c) Spectacles d) Lenses
26. All of the following are seen in a patient who has the (KCET-08)
habit of smoking EXCEPT 37. A membrane immunoassay has been recently marked
a) Increased pocket depth as " Evalusite' it is designated to detect ...................
b) Attachment loss microorganism.
c) Increased gingival bleeding on probing a) A.actinomycetemcomitans, P.gingivalis & P.intermedia
d) All of the above b) A.actinomycetemcomitans, T.denticola, B.forsythus
(KAR-2013) c) Strep. sanguis, Strep. mitis, T. denticola
27. One of the following drugs can slow the loss of alveolar d) P. gi ngivalis & E. corrodens
bone in periodontitis: (KCET-08)
a) Dexamethasone b) Ibuprofen 38. Periodontal endoscopes work on the principle of:
c) Penicillin d) Calcium channel blockers a) Fiberoptics b) Magnetorestrictive
(KAR-04) c) Piezoelectric d) All of the above
28. Oral prophylaxis is contraindicated in: (MCET- 07)
a) Prepubertal gingivitis b) Pregnancy gingivitis 39. The only valid method to assess periodontal regeneration is
c) Ulcerative refractory gingivitis a) Radiographic analysis b) Clinical attachment Level
d) Leukemic gingivitis c) Reentry operation d) Histologic analysis
(AIIMS- 06) (COMEDK-08)
29. Which of the following statement is not correct: 40. Use of Doxycycline hyclate 20 mg. for a period of time forms
a) Periodontitis is always preceded by gingivitis a) Prophylactic antimicrobial therapy
b) All gingivitis do not progress to periodontitis b) Systemic antimicrobial t herapy
c) Bone destruction in periodontal disease is caused by c) Host modulation therapy
local factors d) Local drug delivery
d) Level of periodontal bone reflectthe present inflammatory (COMEDK-08)
condition 41. PGE2 mediated bone resorption in periodontal disease is
(AIPG-07) inhibited by
30. Halitosis is primarily because of: a) Calcitonin b) Estrogens
a) Hydrogen sulfide b) Alkaline phosphates c) NSAIDS d) Vit amin-D & Calcium
c) Trypsine like enzymes d) Collagenase enzymes (COMEDK-08)
(COMEDK- 07) 42. Which of the following antibiotics is NOT indicated for
31. The percentage of fluoride used in iontophoresis is: periodontal therapy?
a) 1°lo b) 2% a) Erythromycin b) Metronidazole
c) 4% d) 8% c) Tetracycline d) Augmentin
(AIIMS-06, AIPG-06) (KCET- 08)
32. All of the following are true about bacteriocin except: 43. Most common age for incidence of gingivitis in children
a) It has no role in adherence of plaque to the tooth surface a) 1 - 3 yrs. b) 11 - 13 yrs.
b) It can be used in therapy of dental caries c) 6 - 7 yrs. d) 2 - 7 yrs.
c) It is a protein (BHU-07)
d) It enhances the growth of other bacteria 44. BANA test measures activity of
(AIIMS-06) a) Calculus b) Cathepsin
33. The distance between implant and adjustment tooth c) Elastase d) Trypsin like enzyme
should be at least (COMED-10)
a) 7 mm b) 1.5 mm

23) B 24) C 25) D 26) C 27) B 28) D 29) D 30) A 31) B 32) D 33) B 34) A 35) B
36) A 37) A 38) A 39) D 40) C 41) C 42) A 43) C 44) D
, PERIODONTICS

45. Which of the following is the gold standard for testing c) Circadian pattern d) No consistent pattern
breath matador (halitosis)?
a) portable volatile sulfide monitor 55. Which of the following salivary factoris not bactericidal?
b) self examination a) ca 2• and Po/ b) Lysozymes
c) gas chromatography d) organoleptic rating by a judge c) Ig A d) Lectoperoxidase
(KCET-10) (PGI-08)
46. Which of the following cytokines cause bone resorption? 56. FDA approved locally delivered minocycline for
a) IL-1 b) IL-8 subgingival placement is marketed under trade name
c) IFN-gamma d) IL-4 a) periochip b) atridox
(KCET-10) c) arestin d) elyzol
47. Recommended dose of prednisolone for the treatment of (KCET-10)
desquamative gingivitis? 57. Which teeth are least involved in periodontitis?
a) Daily dose of 40 to 50 mg and maintenance dose of 5 to a) Lower incisor and lower molar
10 mg b) Lower premolar and upper canine
b) Daily dose of 30 to 40 mg and maintenance dose of 5 to c) Upper molars and upper incisors
10 mg d) Lower incisors and upper molars
c) Daily dose of 30 t o 40 mg and maintenance dose of 10 (AIIMS-09, 11)
t o 20 mg 58. Radiopaque irregularity associated with the roots of
d) Daily dose of 40 to 50 mg and maintenance dose of 10 mandibular central incisors most likely represent?
to 20 mg a) Root Caries b) Subgingival Calculus
(AIPG-14) c) Subgingival Pellicle d) Hyperplastic Cementum
48. Apical migration of the epithelial attachment with (AIPG-09)
corresponding recession of the marginal gingiva results in? 59. Radiographs are of greater value in diagnosing
a) A shallow sulcus b) Gingival pocket formation periodontal disease because they reveal?
c) Infrabony pocket formation a) Hard to soft tissue relationship
d) Peridontal pocket formation b) Morphology of bone deformities
(AIPG-09, AIIMS MAY 2012) c) Presence of pockets d) Thickening of lamina dura
49. Gingival clefts may be caused by (AIPG-09)
a) Occlusal disharmonies b) Faulty toothbrushing 60. Periodontal pack is least successful in the management of?
c) Normal frenum attachment a) Splinting b) Antimicrobial Properties
d) Use of dental floss c) Hemostasis d) Pain elimination
(AIPG-09} (PGI-08)
50. A radiograph of the mesial aspect of the mandibular right 61. A 7 year old child comes to your clinic with ulcers with
first molar show 2-3 mm of bone loss, yet clinical probing indurated margins in the oral cavity and fever with
indicate a pocket depth of 6-8 mm. This discrepancy is crusting of lips. The probable treatment plan is?
probably due to the? a) Prescribe broad spectrum antibiotic and mouthwash
a) Poor angulation used in taking the radiograph b) Symptomatic treatment and observation
b) Presence of mandibular tori in the area c) Clean the mouth wit h gentle mouthwash
c) Presence of mesial bone masking the destruction d) None of the above
d) Presence of facial or lingual bone masking the destruction (AIPG-09, 11)
(AIPG-09,11) 62. CADIA System i s used to check?
51. The minimum mesiodistal space required for placement a) Progression of Periodontal Disease
of two standard diameter implants (4.0 mm diameter) b) Mobility of Teeth
between teeth is c) Furcation Involvement d) Bleeding Sites
a) 8 mm b) 20 mm (PGI-08)
c) 10 mm d) 14 mm 63. For better osseo integration modifications on implant
(UPSC-09) surfaces include -
52. Most common extra-oral cause of Halitosis is? a) Sandbasting with Aluminum oxide
a) Indigestion b) Chronic Sinusitis b) Etching with Phosphoric acid
c) Alcohol Intake d) Diabet es Mellit us c) Plasma sprayed Titamum
(PGl-08) d) All of t he above
53. Inflammatory changes in the soft tissue surrounding an (MCET-10)
implant is diagnosed as 64. Which of the following influence the ability of osteoclast
a) Peri-implant gingivitis b) Peri-implantitis to resorb bone matrix?
c) Peri-implant mucositis d) Peri-implantosis a) cathepsin Kand carbonic anhydrase
(UPSC-09} b) alkaline phosphat ase
54. Periodontal disease in diabetic patients follows c) both the above d) matrix metalloproteinase
a) Consistent pattern b) Distinct pattern (KCET-10)

45) D 46) A 47) B 48) A 49) B 50) D 51) D 52) B 53) B 54) D 55) A 56) C 57) B
58) B 59) D 60) D 61) B 62) A 63) D 64) A
Dental ;lut.,e i======
65. Tooth mobility is NOT reversible when there is - b) To increase t he surface area
a) Inflammation in periodontium c) To provide a barrier
b) Trauma from occlusion d) To increase tension
c) Loss of alveolar bone (AIPG-2012)
d) Faulty tooth brushing technique 76. Factors which contribute to the risk of developing
(KCET-09) periodontal disease in individual teeth include
66. All these factors affect the height and thickness of facial a) Occlusion b) Excessive occlusal stress
and lingual bony plates EXCEPT- c) Pulpal infections d) All of the above
a) Alignment of teeth (KAR-2013)
b) Angulation of root to the bone 77. Which of the following conditions could arise as a
c) Occlusal forces sequelae of periodontal infection?
d) Mesiodistal angulation of crest of interdental septum a) Ischemic heart disease b) Thrombogenesis
(KCET-09) c) At herosclerosis d) All of the above
67. The unalloyed form of titanium used to form dental (KAR-2013)
implants is in the 78. The plasma coating of a Titanium Dental Implant (TPS) is
a) beta form b) alpha form done to
c) gamma form d) delta form a) To increase its acceptance in bone
(COMED-14) b) To make the Implant Biocompatible
68. In children below 5 years of age, most common cause of c) To avoid contamination of the Implant
gingivitis is d) To improve implant anchorage power in bone.
a) local irritating factors b) herpes
c) vitamin deficiency d) malocclusior 79. CHF and Periodontitis shows?
(AIPG-10) a) Positive relation b) Negative relation
69. The technique to produce limited removal of epithelial c) Weakly positive relation
tissue in the sulcus while a chamfer finish line is being d) No relation
created in tooth structure is called (PGI JUNE-2011)
a) Soft tissue modification b) Gingettage 80. Adequate vestibular depth is important for
c) Gingival displacement d) Electrosurgery a) Good aesthetics b) Good occlusion
(COMEDK-2011) c) Good oral hygiene d) Muscular function
70. The sixth complication of diabetes mellitus is (MHCET-15)
a) Retinopathy b) Rental failure 81. Remaining bone associated with implant papillary level
c) Ischemic heart diseases d) Periodontitis of single tooth implant is denoted by?
(KCET-2011, AP- 2013) a) Bone level surrounding adjacent natural bone height.
71. A probable etiology of gingivosis is? b) 5 mm of implant abutment junction
a) High progesterone levels c) 5 mm of implant bone level
b) Deficiency of estrogen and t estosterone d) According to bone height of implant
c) Pregnancy (PGI JUNE-2011)
d) Aldosterone deficiency 82. In hyperglycemic state, non-enzymatic glycosylation of
(AIPG-2011) proteins and matrix molecules result in
72. Smoking a) MMPs b) BMPs
a) Increases signs of inflammation c) AGEs d) TIMPs
b) Decreases signs of inflammation
c) Has no effect on signs of inflammation 83. Osteogenin is the other name for
d) Decreases periodontal destruction a) BMP-7 b) BMP-3
(AP-2012) c) BMP-2 d) BMP-15
73. Subgingival temperature is lower in
a) Smokers t han nonsmokers 84. The angle of separation between two roots (cones) is
b) Acute gingivitis than chronic gingivitis a) Divergence b) De gree of separation
c) Periodontosis than periodontitis c) Coefficient of separation
d) None of the above d) Furcation
(BHU-2012)
74. Dental and periodontal infections can pose a severe risk 85. Pharmacological agent that inhibits the differentiation
to patients under of osteoclasts through its action on RANK
a) Chemotherapy b) Radiotherapy a) NSAIDs b) SERMS
c) Ozone therapy d) Water therapy c) OPG d) CMTS
(BH U-2012)
75. Avulsed tooth dip into the citric acid because
a) To remove periodontal ligament

65} C 66} D 67} B 68} A 69} B 70} D 71} B 72} B 73} A 74} B 75} A 76} D 77} D
78) D 79) A 80) C 81) A 82) C 83) B 84) B 85) C
, PERIODONTICS
,......__,._

V
613

86. Total arch length reduced due to mesial migration of


teeth?
a) 1 cm b) 3 cm
c) 0.5 cm d) 0.15 cm
(PGI DEC-2011)
87. Mos t plausible factor for dental implant fracture
a) Peri-implantitis b) Bending overloads
c) Non-passive fit prosthesis
d) Manufacturing imperfections
(PGI JUNE-2014)
88. First implant system developed?
a) Noble biocare - Branemark
b) Noble biocare - Steri Oss
c) ITI - Strauman d) Astra
(PGI JUNE-2012)

86) C 87) B 88) A


Dental ;lut.,e

17. IMPLANTS AND MISCELLANEOUS - ANSWERS


1. ' C' [Carranza 11 1h ed 773) 8. 'A' [Carranza 11th ed 406 / 10th ed 126]
The primary objectives of surgical therapy are Cigarette smokers are up to five times more likely than
1. To increase accessibility to the root surface, making it nonsmokers to develop severe periodontitis, and the risk of
possible to remove all irritants (Flap surgery). the disease increases with the amount of cigarettes smoked.
2. Reduce or eliminate pocket depth (Flap Surgery &
Gingivectomy). 9. 'B' [Carranza 8 1h ed 299)
3. Gingivoplasty
10. 'C' [Carranza 8 1h ed 556)
Secondary objectives are
1. Plaslic & Eslhelic surgery (mucogingival) and 11, 'D' [Carranza 11•• ed 109 / 10th ed 156)
2. Preprosthetic surgical techniques Development of gingivitis was extensively studied by loe et
al in a model system called experimental gingivitis.
Development of more sophisticated flap methods have made
gingivectomy to a lesser role in the current period. 12. 'A' [Carranza 11th ed 774)
Epithelial adaptation or healing by formation of long
2. 'C' [Carranza 8th ed 556) junctional epithelium is the close opposition of gingival
Occlusal guards does not prevent bruxism. They interfere with epithelium to the tooth surface without complete
the effects of bruxism . They dissipate the musculoskeletal obliteration of the pocket.
forces (masseter, temporalis) and protects the tooth
surfaces. The facets present on teeth due to bruxism are 13. ' B' [Carranza 111h ed 828 Box 57-1)
called bruxofacets.
14. ' D' [Carranza 111h ed 790 / 10'h ed 891)
3. 'D' Periodontal dressing helps in controlling postoperative
Occlusal adjustment is affected by condylar guidance, incisal infection and hemorrhage. It also facilitates healing by
guidance, compensating curves, cusp height and plane of preventing trauma during mastication.
occlusion.Correction of occlusal supracontacts consists of
grooving, spheroiding and pointing. Composition of eugenol pack:
• Zinc oxide
Restoring the depth of developmental • Eugenol - induces allergic reaction
Grooving
grooves
• Zinc acetate (accelerators) - increases working time
Reducing the supra contact while • Asbestos (binder and filler) - causes lung disease should
Spheroiding
restoring the original tooth contour be eliminated
Pointing Restoring cusp point contours • Tannie acid - causes liver damage, should be eliminated.

4. 'A' [Carranza 11th ed 251) Composition of non-eugenol pack (coe-pack):


Bruxism is considered as multifactorial psychosomatic
• 1' 1 tube contains zinc oxide, an oil (for plasticity) a gum
phenomenon. It is the clenching or grinding of the teeth
(for cohesiveness) and bithiomol (fungicide)
when the individual is not chewing or swallowing.
• The second tube contains liquid coconut fatty acids,
5. 'B' [Check Explanation Below] colophony resin and chlorthymol (bacteriostatic agent)
Lugo ls iodine (or) schillers solution is used to stain • This dressing does not contain asbestos or eugenol
keratinised gingiva. and thereby avoids the problems associated with these
substances
The mucogingival junction can be demonstrated by • Coe-pack is contra indicated when the strength is not a
a) Gention violet primary consideration.
b) Schiler's potassium solution
c) Disclosing solution 15. 'A' [Carranza 11th ed 794 / 10' h ed 896)
d) Chlorhexidine
(AP-14) 16. 'A' [Carranza 11th ed 790 / 101h ed 891)

6. ' D' 17. 'C' [Carranza 111h ed 792 / 101h ed 895)

7. 'D' [Carranza 11•h ed 483) 18. ' D' [Carranza 111h ed 794)
Gingival recession due to friction from soft tissues is called
as " Gingival ablation" while the effects on dentition by
action of soft tissues is called as " Frictional ablation".
, PERIODONTICS
,.....__,,_

V
615

19. 'A' [Carranza 11th ed 791) 31. ' B' [Essentials of operative dentistry by Anand Sherwood
As a general rule, pack is kept for one week after surgery. So 473)
the patient should be recalled one week after surgery for the
removal of periodontal pack. 32. ' D'

20. 'A' [Carranza 101hed 853) 33. ' B' [Carranza 10th ed 1094)
Because occlusal adjustment (grinding) is an irreversible A minimum distance of 1.0 to 1.5mm of bone is desired
intervention, it should rarely be considered as a primary around all surfaces of implant after preparation.
treatment. Minimum required distance
Structure between implant and indicated
21. 'A'
structure
22. 'A' [Carranza 11th ed 98 / 101 hed 348] Buccal plate 0.5mm
Myeloperoxidase is released by leukocytes and is bactericidal • Lingual plate
for actinobacillus and also has the added benefit of inhibiting • Maxillary sinus
the attachment of bacteria. 1mm
• Nasal cavity
23. ' B' [Text book of Oral Medicine by Pramod John 2nd ed • Inferior border
84) Adjacent natural
1mm - 1.5mm
tooth
24. 'C' [Carranza 111h ed 426 / 101• ed 294)
Inferior alveolar 2mm from superior aspect of
Bleeding tendencies (Oral & general) is caused by canal bony canal
thrombocytopenia in leukemia.
Inter implant 3mm between outer edge of
25. 'D' [Carranza 11th ed 60 / 10 hed 100)
1 distance implants
Mental nerve 5mm from anterior or bony foramen
26. 'C' [Carranza 11th ed 295)
Incisive canal Avoid midline maxilla
Periodontal Disease Impact of Smoking
Gingivits • l inflammation 34. 'A' [Carranza 111h ed 269 / 10th ed 239)
The three proinflammat ory cytokines that have a central
• l BOP
role in periodontal tissue destruction are Interleukin-1 (Il-
Periodontitis • I Pocket depth, attachment l), IL-6, and tumor necrosis factor (TNF). The properties
loss and bone loss of these cytokines that relate to tissue destruction involve
• I tooth loss stimulation of bone resorption and induction of tissue
• l response to therapy degrading proteinases. IL-1 exists in alpha and beta forms.
Both forms are the main constituents of "Osteoclast-
activating factor".
Cigarette smoking has all of the following effects on
the periodontium except (PGI June-2014) TNF is also found in TNF-a and TNF-~. IL-I is a potent
a) Increased gingival bleeding stimulant of osteoclast proliferation, differentiation, and
b) Decreased response to therapy activation. TNF-a has similar effects on osteoclasts but is
c) Increase bone loss and attachment loss much less potent than IL-I. Both IL-I and TNF-a induce
d) Decreased polym orphonuclear response production of proteinases in rnesenchymal cells, including
Matrix Metallo Proteinases (MMP), which may contribute to
27. 'B' [Carranza 101hed 457) connective tissue destruction. Even the data from invivo
studies support the concept that IL-I and TNF-a are key
28. ' D' molecules in the pathogenesis of periodontitis.

29. ' D' [Carranza 11th ed 493 / 101hed 452)


Soft tissue changes reveal the present inflammatory Interleukin-1 (IL-1) gene cluster is responsible for
condition while hard tissue changes indicate the past a) Dental Caries b) Periodontal diseases
inflammatory condition. c) Supernumerary teeth di) Peg laterals
(COMEDK-13)
30. 'A' [Carranza 111h ed 464 / 101hed 331)
The unpleasant smell of breath mainly originates from
volatile sulfide compounds especially H2S, dimethyl sulfide,
methyl mercaptan.

The unpleasant smell of breath mainly originates from


Ans: Volatile sulphide compounds (BHU-2012)
Dental ;lut.,e

35. 'B' (The Dental Clinics of North America - Oct. 2000) ii) Immunofluorescent assays
Classification of Lasers
iii) Latex agglutination
• Helium - Neon
• Carbondioxide and monoxide iv) ELISA
• Argon
Gas lasers v) Evalusite: Modification of ELISA technique in which a
• Xenon
membrane immunoassay has been adopted for chairside
• Nitrogen clinical diagnostic used. It involves linkage between
• Excimer Laser (molecular laser) the antigen and a membrane-bound antibody to form
• Ruby laser an immunocomplex that is later revealed t hro ugh a
calorimetric reaction. Evalusit e has been designed to
• Neodynium - YAG (Nd: yAG)
detect A. actinomycetem comitans, P. gingivalis and
• Neodynium - Glass P. intermedia and fo und a detection limit of 105 for A.
Solid lasers • Neodynium - Yttrium actinomycetam comitans, and 106 for P. gingivalis.
• Titanium Sapphire
• Ytterbium Which of the following is not defected by Evalusite
a) A. actinomycetem comitans (MCET-14)
• Holmium - YAG
b) P. gingivalis
Liquid laser • Dye laser c) P. intermedia d) T. denticola
• X-ray laser
Other lasers 38. 'A' (Carranza 11th ed 650 / 10th ed 761)
• Free electron laser
Dental endoscopes are introduced recent ly for use
subgingivally in the diagnosis and t reatment of periodontal
• Excimer lasers are used for caries removal, enamel,
disease. The perioscopy system consists of a 0.99mm-
dentin, bone ablation.
diameter, reusable fibe roptic endoscope over which is filled
• Other gas lasers are used for desensitization of
a disposable, sterile sheath. The· sheath delivers water
hypersensitive dentin
irrigation t hat flushes the pocket while the endoscope is
• Pulsed Nd-YAG laser is used for analgesia, toot h being used for clear visualization deeply int o subgingival
whitening and endodontics pockets and furcations. It permits operators to detect
• Nd: YAP is used for caries ablation and endodontics the presence and location of subgingival deposits and
• Ho: YAG is used for bone ablation and cartilage reshaping guides them in the thorough removal of these deposits.
• CO2, Argon, Nd: YAG Lasers are used for rendering enamel Magnification ranges from 24X to 46X.
less susceptible to decay.
39. ' D' [Carranza 11th ed 869 / 10th ed 970)
36. 'A' [Carranza 11th ed 951 / 10th ed 1030) Clear evidence of a new attachment apparatus can be
Dental loupes are t he most common system of optical det ermined only by hist ologic analysis of tissue blocks
magnification used in periodontics. Loupes are obtained from the healed area.
fundamenta lly dual monocular telescopes with side-by-
side lenses convergent to focus on the operative field. A 40. 'C' [Carranza 11th ed 710 / 10th ed 806, 814-816)
convergent lens optical system is called a Keplerian Optical Host Modulat ory Therapy (HMT) is a treatment concept
Syst em. Three types of Keplerian loupes are typically used in that aims to reduce tissue destruction and stabilize or
periodontics. They are: even regenerate the periodontium. Doxycycline hyclat e
• Simple or single-element loupes (magnification of 1.5x) is available as a 20mg capsule for use by patients twice
• Compound loupes (magnification of 3x) and daily. The mechanism of action is by suppression of the
activity of collagenase, particularly that produced by PMNs.
• Prism telescopic loupes (magnification of 4x)
NSAIDs inhibit the formation of prost oglandins, including
prostaglandin (PGE2). PGE2 is fou nd to upregulate bone
Overall, dental loupes provide a limited range of
magnification, 1.5x to 6x. resorption byosteoclasts. Flurbi profen, an NSAID, significantly
inhibits radiographic alveolar bone loss when compared
37. 'A' (Carranza 10th ed 591) with placebo. A recent study suggested that concomitant
IMPORTANT IMMUNODIAGNOSTIC METHODS: administration of doxycyline and flurbiprofen may result in
enhancement of the anticollegenase effect s of doxycycline.
i) Cytofluorography or flow Cytometry for rapid
identification of oral bacteria. It involves labeling 41. C' (Carranza 11th ed 710 / 10th ed 814, 806)
bacterial cells from a patient plaque sample with both
species - specific antibody and a second fluorescein- 42. 'A' (Carranza 11th ed 694 / 10th ed 799 Tab 52-1]
conjugated antibody. The suspension is then introduced The various antibiotics used for periodont al therapy are:
into the flow cytometer, which separates the bacterial
cells into an almost single cell suspension by means of a • Amoxicillin
laminar flow t hrough narrow tube. • Augmentin (Amoxicillin and clavulanate potassium)
, PERIODONTICS
,......__,,_

V
617

• Minocyline and Doxycycline (Subgingivally applied) 47. ' B' (Carranza 11th ed 158]
• Tetracycline The recommended dose of prednisolone in the treatment
• Ciprofloxacin of desquamative gingivitis is daily dose of 30 - 40mg and
gradually reduced to daily maintenance of 5-lOmg.
• Azithromycin
• Clindamycin 48. 'A' (Carranza 11th ed 491 / 101 h ed 434 - 436]
• Metronidazole
49. 'B' (Carranza 10th ed 370]
43. 'C' [Carranza 11th ed 145 / 10th ed 381]
50. 'D' (Carranza 11th ed 506 / t01 h ed 456]
44. 'D' [Mosby dental dictionary]
BANA: A Chairside Test for Periodontal Risk 51. 'D' (Carranza 11th ed 1044 Fig 69-9 / 10th ed 1094]
The BANA Test is a highly sensitive, inexpensive and easy- The minimum M-D space required for placement of implants
to-use chairside test for periodontal risk. In just 5-minutes,
the BANA Test can detect the bacteria associated with Diameter of Implants Space
periodontal disease simply by applying tongue swabbings or
subgingival plaques to a small test st rip. Narrow (3.25mm) 6mm
Standard (4.1mm) 7mm
The BANA Test is a modification of the BANA hydrolysis Wide (5mm) 8mm
test developed by Dr. Walter Loesche and colleagues at
Wide (6mm) 9mm
the Univ. of Michigan School of Dentistry. It exploits
an unusual Trysin Like enzyme fo und in Treponema Two standard 4.1mm implants 14mm
denticola, Porphyromonas gingivalis and Bacteroides
forsythus, three anaerobic bacteria highly associated with 52. 'B' (Carranza 11th ed 466 / 101 h ed 33 2]
adult periodontitis. Of 60 subgingival plaque species, only ENT causes include acute pharyngitis, chronic sinusitis and
these three possess an enzyme capable of hydrolyzing the post nasal drip
the synthetic trypsin substrate, benzoyl-DL-arginine-
naphthylamide (BANA) present on BANA test strips. If any 53. 'B' (Carranza 11th ed 1171 / tOth ed 1187]
of the three species is present , t hey hydrolize the BANA
enzyme producing B-naphthylamide which in turn reacts 54. ' D' (Carranza 11th ed 417]
with imbedded diazo dye to produce a permanent blue color Periodontal disease in diabetic patients follows no consistent
indicating a positive test. or distinct pattern. Severe gingival inflammation, deep
pockets, rapid bone Loss, frequent periodontal abscess often
How it works occur in poorly controlled diabetic patients with poor oral
For periodontal risk assessment, subgingival plaque is hygiene.
obtained with a curette. The samples are placed on the 55. 'A' (Carranza 11th ed 98 / tOth ed 349]
BANA test strip, which is then inserted into a slot on a
small toaster-sized i ncubator. The incubator automatically 56. 'C' [Carranza 11th ed 701 / t0•h ed 808]
heats the sample to 55° for 5 minutes. If P. gingivalis, B. FDA recently approved Aresti n 2% Minocycline for subgingival
forsythus or T. denticola are present, the test strip turns placement as an adjunct to scaling and root planning.
blue. The bluer it turns, the higher the concentration and
5 7. 'B' (Carranza 8th ed 79]
t he greater the number of organisms.

45. 'D' [Carranza 11th ed 468 I tQth ed 334] 58. 'B' [Natonal Board's Endo-Perio Mar 1987]
A trained judge sniffs the expired air and assesses whether 1
or nor this is unpleasant using an intensity rating, normally 59. 'D' (Carranza 11th ed 499 I l0 h ed 452 - 454]
from Oto 5.
60. 'D' (Carranza 11th ed 790]
The list of physical benefits of a periodontal dressing includes
0 no odor protection of the postsurgical wound from postoperative
1 barely noticeable odor trauma, saliva, and food debris and stabilization of the
2 slight but clearly noticeable blood clot. Secondly, it limits the entry of bacteria and
other microorganisms which may cause infection and other
3 moderate complications. Furthermore, it has been suggested that it
4 strong offensive acts as a splint for loose teeth and to immobilize newly
5 extremely foul positioned grafts and flaps.

46. 'A' (Carranza 11th ed 269 / tQth ed 239] 61. 'B' (Carranza 11th ed 611 / 101 h ed 398]

62. 'A' (Carranza 11th ed 867 / 101 h ed 587]


Dental ;lut.,e i======
Computer Assisted Densitometric Image Analysis System diseases are often painful and are included in the category
(CADIA) is a video camera that measures the light of chronic desquamative gingivitis, also called gingivosis.
transmitted through a radiograph. It is an objective method
for following alveolar bone density changes quantitatively Desquamative Gingivitis- While many diseases and
over time. conditions are associated with desquamative gingivitis,
the majority (75%) are dermatologic. The most common
63. 'D' (Check Explanation Below) dermatologic diseases are cicatricial pemphigoid and lichen
Techniques that maximize bone-implant contact there by planus, which constitute over 95% of desquamative gingivitis
improving osseointegration are : with a dermatologic etiology. Other non-dermatologic
diseases associated with desquamative gingival lesions, such
a) Subtractive processes as hormonal imbalances, chronic infections, and idiopathic
• Electro polishing disorders, occurs less frequently.
• Sand blasting
72. 'B' (Carranza 11th ed 406 / 10th ed 252)
• Etching Controlled clinical studies have constituently demonstrated
that smokers present with less gingival inflammation than
b) Additive process nonsmokers because of decrease in the blood vessel density
• Plasma spray than the number of blood vessels.
• Electrophoretic ion deposition
73. 'A' (Carranza 10th ed 582)
64. 'A'
74. ' B' (Carranza 10th ed 663)
65. '('
75. 'A' (Cohen 9th ed 643)
66. ' D' (Carranza 10th ed 460) Check Q. No 35 in traumatology chapter in endodontics.

6 7. 'B' (Contemporary implant dentistry by Misch 3rd ed 76. 'D' (Carranza 11th ed 513 / 11th ed 222)
527) Risk factors of periodontal disease
The titanium alloys used for dental implant components • Plaque
include microstructural phases of alpha and beta. • Systemic disease
• Acute gingival infections
The alpha phase surface regions of the alloy are similar to
unalloyed titanium in atomic arrangement (close packed • Iatrogenic
hexagonal) whereas beta phases demonstrate a different Margins of restorations
atomic structure (body centered cubic). Contours
Design of dentures
68. 'A' [Infancy through adolescence 4th ed 414, 415)
Restorative materials
69. 'B' [Nallaswamy 1" ed 627) • Malocclusion
Gingettage (Rotary curettage): • Orthodontic therapy
It is a troughing technique, wherein a portion of the • Endodontic infections
epithelium within the sulcus is removed to expose the finish
line. It should be done only on healthy gingival tissue. • Chemical irritation
Tobacco
Indications Smoking
• Absence of bleeding up on probing from gingiva. Radiation.
• Sulcus depth less than 3 mm
77. 'D' (Carranza 11th ed 447 / 11th ed 321)
• Adequate keratinized gingiva
Organ systems and conditions possibly influenced by
Disadvantages periodontal infection are:
a) CVS & CNS
• Damage the periodontium
• Technique sensitive • Atherosclerosis
• Coronary heart disease
70. 'D' (Carranza 11th ed 417 / 10th ed 320) • Angina & MI
• Cerebrovascular accident (Stroke)
71. 'B' (Carranza 11th ed 152 / 10th ed 290, 641, 645]
Gingival changes are also seen in skin and mucous membrane b) Endocrine
diseases such as cicatrial pemphigoid and pemphigus.
• Diabetes mellitus
Gingival conditions seen in mucous membrane or skin
, PERIODONTICS
,......__,,_

V
619

c) Reproductive System 83. 'B' [Carranza 11th ed 585]


• Low birth weight infants Bone morphogenic proteins are a group of proteins that are
important for skeletal development. Each of the protein has
• Preeclampsia
relatively specific functions.
• BMP 2 - Strongest bone producing activity
d) Respiratory
• BMP 7 - Also called as osteogenic protein-1 (OP-1)
• COPD
• BMP 3 - Also known as osteogenin. It has shown to
• Acute bacterial pneumonia stimulate bone formation.

78. 'D' [Check Explanation Below] 84. 'B' [Carranza 9th ed 664]
Applying calcium phosphate (CaP) coating on implants • Degree of sepa ration is the angle of separation between
improves the osteoconductive (osseointegration) the roots (Cones)
properties. Different methods have been developed to coat
• Divergence is distance between two roots
metal implants with CaP layer such as plasma spraying,
biomimetic and electrophoretic deposition. By means of • Coefficient of separation is defined as the length of the
plasma polymerization, positively charged, nanometric thin root cones in relation to the root complex.
coating can be applied to implant surfaces.
The ideal tooth for root a mputation is the ones having
Titanium plasma sprayed (TPS) surface results in increased the following:
surface area and it has been proposed that TPS improves • Short root trunk
implant anchorage power in bone (Osseo integration) . • Divergent roots
• Long, round roots
79. 'A' [Carranza 10th e d 316] • Favourable crown-to-root ratio
A positive relationship exists between periodontitis and
congestive heart failure. Many recent studies reported high • Minimal vertical osseus defects.
monthly tomato vegetable consumption still increases the
risk of CH Fin patients with periodontitis. 85. 'C' [Carranza 11th ed 205]
RANKL means Receptor Activator of Nuclear factor Kappa-B
80. 'C' [Check Explanation Below] Ligand. It is a member of the TNF super family. The RAN KL
Adequate vestibular depth is necessary for adequate oral system is an essential mediator of osteoclast formation,
hygiene. A shallow vestibule may lead to food retention function and survival. RANKL binds RANK on osteoclasts
around the teeth and hampers the efficient use of the tooth or osteoclast precursors to stimulate differentiation into
brush. osteoclasts. It is also essential for the survival of osteoclasts
and thereby playing critical role in bone resorption.
81. 'A' [Peterson 2nd ed 200]
The majority of anterior maxillary single tooth sites present Osteoprotegerin (OPG) acts as RAN KL inhibitor and prevents
with inadequate bone and soft tissue, thereby requiring the the recruitment or differentiation of osteoclasts. Thus they
following measures: affect the pool of mature osteoclasts and their activity.
• Both bone and s oft tissue augmentation: The height High RANKL and low OPG levels are reported in sites with
of papilla reflects the underlying crestal bone height on active periodontal breakdown compared to sites with healthy
the adjacent tooth. gingiva.
• Careful assessment of bone level in the adjacent teeth
enables the surgeon and restorative dentist to inform. 86. 'C' [Carranza 10th ed 86]
• Patient of realistic expectation of retaining or creating Tooth movement does not end when active eruption is
for an esthetic single tooth restoration. completed and the tooth is in functional occlusion. With
time and wear, the proximal contact areas of the teeth are
82. 'C' [Carranza 11th ed 308] flattened and the tooth tends to move mesially. This is
Periodontitis has been determined to be a potential risk referred to as physiologic mesial migration. By age 40 it
factor for poor glycemic control in patients with diabetes. results in a reduction of about 0.5 cm in length of the dental
In hyperglycemic s.tate, numerous proteins and matrix arch from the midline to the 3rd molar.
molecules undergo a non-enzymatic glycosylation, resulting
in advanced glycation and products (AGES). Macrophages 87. 'B' [Misch Contemporary Implants 3rd ed 79]
have high affinity receptors for AGE modified proteins. The most common implant related complications are
This binding initiates a cycle of cytokine upregulation biomechanical problems that occur after the implant is loaded.
mostly IL-1 and TN F-alpha. This results in triggering of
degradative cascade and finally resulting in connective Causes of implant fracture:
tissue degradation. a) Overload induced fractures:
• Para functional habits
• Bending moment caused by occlusal loads
Dental ;lut.,e

b) Manufacturing defects
c) Frame work induced fracture

88. 'A' [Carranza 9th ed 893]


The first implant system that ADA had provisionally
accepted in 1986 is the Noble Biocare System developed by
Branemark.
r PERIODONTICS SYNOPSIS
~

V
621

PERIODONTICS - SYNOPSIS
1. of the gracey curette. This allows the advantage of the
Gracey curette Universal curette area specific shank to be combined with the versatility of
universal curette blade
Area and surface specific Universal
Langer 5 - 6 Mesia[ and distals of anterior teeth
One cutting edge is used Both cutting edges are
i.e. work with outer edge used; work with either Mesial and distals of mandibular posterior
Langer 1 - 2
only outer or inner edge teeth
Blade curved in two planes; Curved in one plane; blade Mesial and distals of maxillary posterior
Langer 3 - 4
curves up and to the side curves up and not to side teeth
Offset 60° blade angle 90° blade angle
7.
2. Gracey curettes: double ended gracey curettes are paired as Angles in instrumentation
No 1 - 2, 3 - 4 Anterior teeth Angulation for blade insertion oo
No 5 - 6 Anterior and premolar teeth Angulation for scaling and root planing 45 - 90°
No 7 - 8, 9 - 10 Posterior teeth: facial and lingual Angulation for curettage > goo

No 11 - 12 Posterior teeth : Mesial surfaces Angulation of blade with shank in universal goo
No 13 - 14 Posterior teeth : Distal surfaces curette
No 15 - 16 Blade of 11 - 12 and shank of 13 - 14 In gracey curette, the angulation of blade 60 - 70°
with shank
Blade of 13 - 14 and shank extended
No 17 - 18 Angle between face and lateral surface of 70 - 80°
by 3mm
the blade
3. Extended shank curettes: Angle for sharpening 100 - 110°
Eg:- After five curettes The blade of hoe is bent at 99°

The shank is extended 3mm than the standard gracey which 8 Professional prophylaxis with 'Prophyjet' is used to remove
allows extension into deeper periodontal pockets. They are stubborn stains. This consists o,f air powder abrasive system
available in all standard gracey numbers except 9 - 10 with sodium bicarbonate and warm saline water jet. It
should not be used in patients with respiratory illnesses,
4. Minibladed curettes: infectious diseases and sodium restricted diets.
Eg:- Mini five curettes
9 Treatment plan:
These are modified after five curettes with the blade length
Emergencies like extractions,
half of that of conventional curettes. The shorter blade Preliminary phase
allows easier insertion and adaptation in deep, narrow incision of periodontal abscess
pockets and furcations. Phase-I (or) Plaque control, diet control,
etiotropic (or) scaling, root planning, temporary
They are available in all standard gracey number except 9 -10. preparatory phase restorations etc.
Phase-II or surgical
5. American Gracey curettes: RCT, implants
phase
These are four minibladed curettes. The blade length is,
Phase-III or Final restorations, fixed and
50% shorter than that of conventional curette and the
restorative phase removable dentures.
blade has been curved slightly upward. This allows the
curettes to adapt more closely to tooth surface especially Phase-IV (or) Periodic recall with checking to
on the anterior teeth and on line angles maintenance phase prevent recurrence.

Sub - 0 and 1 - 2 for anteriors and premolars 10.


Suture d" .
11 - 12 for posterior mesial surfaces . I n 1cations
t ech mque
13 - 14 for posterior distal surfaces When bone grafts are used and when there
Direct or
is need for coverage of interdental bone
6. Langer and mini Langer curettes: loop
with interdental papilla.
This set of 3 curettes combines the shank design of the When flaps are not in close apposition
standard gracey 5 - 6, 11 - 12 and 13 - 14 curettes with Figure of
because of apical displacement of flap or
a universal blade angled at 90° rather than offset blade eight
non scalloped incisions.
Dental ;lut.,e i======
Closing flap mesial or distal to a tooth as in • S. mit is
Anchor
mesial or distal wedge procedures. Yellow complex • S. sanguis
Sling When flap involving two interdental spaces
• S. oralis
ligation and opposing flap not reflected.
Used in wide interdental spaces to properly • V. parvula
Horizontal Purple complex
adopt the inter proximal papilla against • A. odontolyticus
mattresses
bone. • P. gingivalis
Used to hold partial thickness apically Red complex • T. denticola
Periosteal
displaced flap.
• B. forsythus
11. • P. intermedia
Old Name New Name Orange complex • C. rectus
Bacteroides Po rphyromonas gingivalis • F. nucleatum
gingivalis (Black pigmented bacteria) • A. actinomycetam comitans
Bacteroides Green complex • E. corrodens
Prevotella intermedi us
intermedius • Capnocytophaga spp
Prevotella melanogenicus
Bacteroides
(Black pigmented bacteria and is 15. The red complex is important because it is associated
melanogenicus
common in Down's syndrome) with bleeding on probing which is an important feature of
Wollinella recta Campylobacter recta periodontal destruction.
Gingivosis Desquamative gingivitis 16. Inorganic cont ent of calculus:
Periodontosis Juvenile periodontitis • Calculus consists of 70 - 90% inorganic components.
• Calcium phosphate - 75.9%
12.
• Calcium carbonate - 3.1%
Disease Immune response
Elevated antibody titres to int ermediate The crystalline forms of calculus are
ANUG • Hydroxyapatite (58%), Magnesium whitlockite (21%),
sized spirochetes and P. intermedia.
octacalcium phosphate (12%), Brushite (9%).
• Elevated antibody titers to P. gingivalis
and other pathogens. Which of the following crystal form is least found in
Adult calculus? (AP-2013)
• Cell mediated immunity to bacteria.
periodontitis A: Brushit e
• Occurrence of immune complexes in
tissues. • Supragingival calculus contains hydroxyapatite and
octacalcium phosphate most commonly.
• Elevated antibody levels to
A. actinomycetam comitans • Brushite is more common in mandibular anterior region
LJP and magnesium whitelockite in the posterior areas.
• PMN chemot actic and phagocytic
defects. Brushlite crystal in calculus are found more commonly in
• Elevated antibody levels to P. gingivalis Ans: Mandibular anteriors (MCET-14, PGI June- 13)
GJP • PMN chemot actic and phagocytic • Subgingival calculus ctontains same hydroxyapatite
defects. content, more magnesium white lockite, and less
• Elevated antibody levels to several gram brushite and octacalcium phosphate.
RPP negative bacteria. • Hydroxyapatite is the major crystallite in both supra
• PMN or monocyte chemotactic defects. and subgingival calculus (AIIMS 04).

Desquamative Auto immune antibodies resulting from 17. Lvmohocvtes·


gingivitis pemphigus, Pemphigoid and lichen planus
T:B lymphocyte ratio in blood 3:1
13. Bacterionema, veilonella, diphtheroids have ability to form T:B ratio in GCF 1:3
intra-cellular apatite crystals.
kTa ratio in gingivitis 2:1
14. The initial colonizers are members of yellow or purple Tda ratio in periodontitis 1:1
complexes. The secondary colonizers fell into green, orange
or red complexes. Tda ratio in HIV infection 0.6:1
r PERIODONTICS SYNOPSIS
~
623
V

Predominant lymphocyte in early gingivitis T-lym phocytes


Predominant lymphocyte in periodontitis B-lymphocytes
Children rarely experience periodontitis Due to presence of untransformed B-lymphocytes

18. INCISIONS IN PERIODONTAL SURGERY:

A) External bevel incision


• Used in gingivectomy.
• Done with Kirkland knife and B. P. blade No. 11
B) Horizontal incisions
• First incision for periodontal flap procedure.
i) Internal bevel incision (or) • Produces sharp thin flap.
Reverse bevel incision • It removes the pocket lining.
...... -- • Done with B.P. blade No. 11, 15 . - ··· - ···

• Second incision
ii) Crevicular incision
• Carried
..... ... ·······- ··
out with 12 No. blade
- ······- ······- ...... ···- ···· ·····- ·······- ······- ····- ········- ............ ·- ·······-
... ······- ··· ·····- ····. ·- ······- ·······- ·· ···- ···· ····- ·······- ······- ······- · ... ·······- ·······- ·······- ·····- ····

• Third incision
iii) Interdental incision
• Done with orban knife
• Also called as releasing incisions
C) Vertical incisions • Can be utilized on one or both ends of horizontal incisions
• Necessary for repositioning (displaced) flaps.
19. Treatment of ANUG·
• Removal of pseudomembrane and non attached debris after the application of a topical
anesthetic
First visit • Patient is advised to rinse with 3% H202 in equal dilution with warm water every 2 hours.
• Patients with systemic symptoms and Local lymphadenopathy are placed on Amoxicillin 250
or 500mg QID and Metronidazole 250 or 500mg TIO may also be used.
Second visit After 1-2 days of 1st visit, scaling is performed and oral hygiene instructions given.
After 1 - 2 days of 2nd visit, scaling and root planning are repeated. The H202 mouth rinses are
Third visit
suspended and chlorhexidine rinse continued for 2 - 3 weeks.
Patients without other gingival disease are dismissed for 1 week. Tooth extraction or periodontal
Subsequent visits
surgery should be postponed for 4 weeks.

In treatment of ANUG which of the following is not indicated on first day? (PGI June-2012)
Ans: Subgingival scaling

20.
Perio test Test for detection of tooth mobility.
Periodontometer Instrument used for detecting tooth mobility. {AIIMS - 2012)
Peridex, periogaurd Prescription solution of 0.12% chlorhexidine.
Periogard (PGI-13) Chairside test to detect the presence of AST (aspartate aminotransferase) in GCF.
Periomonitor-Hawe measures the amount of AST (aspartate aminotransferase) in GCF.
Pocket watch
Detects aspartate aminotransferase through colorimetric detection
(Chairside)
Periotriever (Schwartz Highly magnetized instruments designed for retrieval of broken instrument tips from periodontal
periotrievers) pocket.
Osmoscope Detects halitosis {AP - 2005)
Detects pocket temperature differences of 0.1°C from a referenced subgingival tern perature.
Perio-temp-probe
Demonstrates the periodontal disease activity by measuring temperature changes in sulcus.
Periotron Electronic instrument used to measure gingival crevicular fluid.
Dental ;lut.,e i======
Periopaper Blotter paper used in measuring GCF by electronic method.
Periochip Chlorhexidine chip placed in the pocket for local drug delivery.
Perioaid Tooth pick with handle.
Periocline 2% minocycline used in local drug delivery.
Periocheck (Chairside) Rapid chair side test kit developed to detect neutral proteases i.e, collagenase in GCF
Diagnostic kit developed for the identification of specific bacteria profile using an enzyme reaction
Perioscan (Chairside)
from plaque isolates.
Periowise premier Can be used with natural[ or implant teeth. The probe has multicoloured markings. Green colour
plastic probe coding from 0-3 mm, a red mark at 5mm and red colour coding from 7 to 10 mm
Periostat 20 mg of doxycycline used as adjunct to scaling and root planing in treatment of chronic periodontistis
Periodontain Virulent factor of P. gingivalis. It is an alpha-1 proteinase inhibitor of human neutrophil elastase.
Used in extraction of a tooth for implant placement. It is a narrow, flat instrument directed apically
Periotome into the sulcus to slightly expand the adjacent periodontal tissues. the tooth is elevated and removed
with forceps using a gentle, rotational movement.
Periostar 2000/3000 auomatic sharpening instruments
Periodontal
Tests the presence of I L-1 gene.
suceptibility test
Prognostic (Chairside) Aids in detection of serine proteinases and elastases
Biolise (Chairside) Aids in detection of elastase
TOPAS (Chairside) Detects toxins derived from anaerobic metabolism and measures GCF protein level

21. Flaps:
Flaps can be classified as full thickness or partial thickness and displaced or undisplaced flaps.

22. Full thickness flaps (Mucoperiosteal):


• Necessary if osseous surgery is contemplated.
• As the bone is stripped of its periosteum, marginal bone loss occurs. The bone loss is greater if the bone is thin.
• Contraindicated when fenestrations and dehiscences are suspected.

23. Partial thickness flap (Mucosa[):


• The periosteum is left on the bone.
• Is indicated when the crestal bone is thin and when fenestrations, dehiscences are present.

24. Undisplaced flap:


• It is located in the position it had before surgery.
• It is also called as Internal bevel gingivectomy.
• Undisplaced flap and gingivectomy are the two techniques that surgically remove the pocket wall.

25. Displaced flap:


• Is made possible by totally separating the attached gingiva from the underlying bone.
• Flaps can be displaced apically, coronally or laterally to its original position.
• Apically displaced flap preserves the outer portion of the pocket wall and transforms it into attached gingiva. So it eliminates
the pocket and also increases the width of attached gingiva.
26.
Majority of oral microorganisms are Facultative anaerobes
Organisms that predominates in early plaque are gram +ve facultative • Streptococcus sanguis
cocci and rods. • Actinomyces viscosus
• P.gingivalis
Organisms that predominates in late plaque are gram-ve • P.intermedia
anaerobic rods and filaments • Fusobacterium
• Capnocytophaga
r PERIODONTICS SYNOPSIS
~
625
V

• S. sanguis
• S. mitis
Bacteria associated with periodontal health are • A. viscosus
• Capnocytopha
• Neisseria, veillonella
• P.gingivalis
• P. i ntermedia
• A.actinomycetam comitans
Bacteria associated with periodontal disease are
• Eikenella
• Fusobacterium
• Eubacteri um
Organism responsible for pregnancy gingivitis Prevotella intermedia/ Bacteroides melaninogenicus
Facultative anaerobic micro organism associated with localized JP A.actinomycetem comitans
Microorganisms responsible for rapid progressive periodontitis A. acti nomycetemcomita ns, Porphyromonas
(AIIMS-14) gingivalis
Organism that initiates smooth surface caries Streptococcus mutans
Organism that causes progression of smooth surface caries Lactobacillus
Organism responsible for root surface caries A.viscosus
Organism commonly absent in gingival sulcus Diptheroids
A. actinomycetam comitans is a Facultative anaerobe (GRAM - VE BACILLUS)
1 gm . of plaque contains 2 X 1011 bact eria
• Organism that forms the longest chains
• First organism to invade the oral cavity
• Most commonly present organism in saliva Streptococcus salivarius
• The organism of streptococcus viridans group to cause
• SABE after streptococcus sanguis
Organism which does not obey Koch's postulates Mycobacterium lepri
• Fusiform bacilli
Organisms associated with ANUG are • Spirochetes
• The spirochetes are mostly of intermediate sized
Co-aggregation is mainly seen in Gram -ve organisms

27. Some numerical values

Thickness of junctional epithelium near the base of sulcus 10 -20 cell thick
Length of junctional epithelium 0.25 - 1.35 mm
Histologic depth of gingival sulcus 1.8 mm
Probing depth of t he normal gingival sulcus 2 - 3 mm.
Sulcus depth in primary dentition 2.1 ± 0.2m
Thickness of periodontal Ligament 0.15 - 0.38mm
(Thickness of periodontal ligament is Least on MESIAL surface of tooth) (Average 0.2mm)
Amount of GCF secreted per day 0.5 - 2.4 ml.
Oxygen co-efficient of normal gi ngiva 1.6 ± 0.37
Dental ;lut.,e i======
Width of attached gingiva
• Maxillary incisor region (greatest) 3.5 - 4.5mm
• Mandibular incisor region 3.3 - 3.9mm
• Maxillary 1st premolar region 1.9mm
• Mandibular 1st premolar region 1.8mm
Edge to edge contact of cementum and enamel occurs in 30% cases
Cementum does not overlap enamel in 10% cases
Concentration of tetracyclines in GCF is _ times that of plasma 2 - 10 times
(average 7 times)
Specific gravity of saliva is 1.003
Width of Cementum (acellular) at the cervical third 20 - 50 µm
Width of cementum (Cellular) at the apex 150 - 200 µm
Gingivitis is at peak level in which age 12 - 15 years
Percentage affected by gingivitis by the time they cross their puberty 80 - 90%
Percentage of population taking dilantin affected by hyperplasia 50 - 60%
Percentage of population taking Nifedipine affected by hyperplasia 20%
Percentage of pregnant women who develop pregnancy gingivitis 30 - 40%
Percentage of pregnancy gingivitis turning to pregnancy tumor 2%
Plaque contains _ bacteria per gram wt. 2 X 1011
Minimum levels of cyclosporine in plasma above which gingival hyperplasia occurs is 150 nanograms / ml.
Average daily increment in calculus former is 0.1 - 0.15%
Percentage of plaque that gets mineralized within 2 days 50%
Percentage of plaque that gets mineralized in 12 days 60 - 90%
Effective radius of action of bacteria in periodontal disease to cause alveolar bone 1.5 - 2.5mm
resorption is
Duration of each masticatory cycle 0.6 - 1 sec.
No. of swallows per day in an individual is 600 times
Total occlusal points in dentition 138
Normal calcium and phosphate precipitate concentration is 35 - 40
Dentin is abraded - times that of enamel 25 times
Cemenlum is abraded - limes lhal of enamel 35 limes
Recommended probi11g force is 0.75 N (or) 25g
Diameter of standard probe tip 0.35 - 0.55mm
Distance between base of sulcus and alveolar crest in healthy periodontium 2mm (1.07 mm)
Total No. of films required for full mouth survey excluding third molars is 18 {14 - IOPA's + 4 - Bitewings)
Effective concentration of chlorhexidine used in mouth wash is 0.2%
Minimum concentration of chlorhexidine needed to inhibit plaque 0.12%
r PERIODONTICS SYNOPSIS
~

V
627

Diameter of bristles
• Soft 0.2mm (0.007 ")
• Medium 0.3mm (0.012 ")
• Hard 0.4mm (0.014 ")
Epithelialisation after curettage occurs in 2 - 7 days
Complete re-epithelialisation after gingivectomy occurs in 1 month
Complete repair of connective tissue after gingivectomy occurs in 5 - 7 weeks
After periodontal surgery, the sulcus can be safely probed after 3 months
Maximum crevicular fluid production coinciding with maximum inflammation occurs_ 7 days
days after gingivectomy
Ideal thickness of graft should be 1 - 1.5mm
Functional integration of graft occurs after 2 - 5 weeks (1 month)
Ca/ P ratio in hydroxyapatite alloplastic material is 1.67
Ca/ P ratio in tricalcium phosphate 1.5
The disparity between centric relation and centric occlusion is 0 - 0.5mm
Junctional epithelium loses its cohesiveness and detaches from the tooth surface, when 60%
the volume of PMN s reaches
Frequency of ultrasonic vibrations 20,000 - 45,000 cycles/sec
Frequency of electrical currents used in electrosurgery 1. 5 - 7. 5 million cycles/sec

MISCELLANEOUS: 8. The most important basic rule of electrosurgery is always


keep the Hp moving. Prolonged or repeated application
1. Acatalasia is a genetic or inherited disorder associated with causes heat accumulation and tissue destruction.
early onset periodontitis, due to deficiency of catalase enzyme.
9.
2. Electro-surgery or Radio surgery, is a surgical technique Type of Electrode Use
performed on soft tissue using controlled high frequency
electrical currents in the range of 1.5 to 7.5 million cycles • Removal of gingival
per second or megahertz. enlargement and gingivoplasty.
Needle electrode • To make incision to establish
3. The four types of electro-surgical techniques are electro- drainage in treatment of acute
section, electro-coagulation, electro-fulguration and periodontal abscess.
elctrodessication. Needle electrode • Used for festooning.
supplemented by • During reshaping, the electrode
4. The three classes of active electrodes used in electro the small ovoid loop is activated and moved in a
surgery are or diamond shaped concise "shaving" motion.
Single-wire electrodes For incising and excising electrodes
Loop electrodes For planing tissue Ball electrode • Hemostasis
Heavy bulkier e lectrodes For coagulation procedures • Control of bleeding points
Located interproximally.
Thin, bar shaped
5. Electrosection or electrotomy or Acusection is used for • Electro surgery is mainly
electrode
incision, excision and tissue planning. helpful for the control of
isolated bleeding points.
6. Electrocoagulation provides control of hemorrhage by using
• Relocation of frenum and
electrocoagulation current.
muscle attachments to
Loop electrode facilitate pocket elimination.
7. Electrosection and electrocoagulation are most commonly
used in all areas of dentistry. • Removal of flap after the acute
Electrofulguration and electrodessication are not used in symptoms subside.
dentistry.
Dental ;lut.,e

10. Disadvantages of electrosurgery:


• Cannot be used in patients who have non-compatible or
poorly shielded cardiac pacemakers.
• Causes an unpleasant odor.
• If the electrosurgery point touches the bone, irrepairable
damage can be done.
• Heat generated by injudicious use can cause tissue
damage and loss of periodontal support.
• When the electrode touches the root areas of cementum
burns are produced.
, COMMUNITY DENTISTRY

COMMUNITY DENTISTRY
I. REFERENCE BOOKS TAKEN:
1. ESSENTIALS OF PREVENTIVE AND COMMUNITY DENTISTRY by SOBEN PETER - 3'd, 41h
& 5th EDITIONS

2. TEXT BOOK OF BIO STATISTICS BY MAHAJAN - 6 1h & 7th EDITIONS.

3. TEXT BOOK OF SOCIAL & PREVENTIVE MEDICINE BY PARK - 201h & 22nd EDITIONS.

4. FUNDAMENTALS OF BIOSTATISTICS BY VEER BALA RASTOGI - 2nd EDITION.

5. RESEARCH METHODOLOGY BY C.K. KOTHARI - 2"d EDITION

6. PRINCIPLES AND PRACTICE OF BIOSTATISTICS BY J.V DIXIT - 4th EDITION

1. BIOSTATISTICS
1. Which of the following demonstrates highest correlation 7. Ranking students from the individual which the highest
between variables? grade - point average to the lowest is an example of a (an)
a) r = + 0.25 b) r=+0.5 a) Ordinal scale b) Nominal scale
c) r= - 0.75 d) r = + 2 c) Interval scale d) Ratio scale
(MAN-1998) (MAN-97)
2. The arithmetical average of a number of observations is 8. A random sample suggests that
called a) A person in a control group will not be a member of the
a) Mean b) median experimental group
c) range d) All of the above b) Any member of a group to be studied has an equal
(MAN-98, 95) opportunity to be included in the study
3. Which of the following is NOT a measure of central c) Every nth name on a list is selected
tendency? d) Subjects are volunteers
a) mean b) mode (MAN-97)
c) range d) median 9. A statistical test which indicates the chance or probability
(MAN-99) of an observed difference between two means occurring
4. Census is conducted every by chance is called:
a) 2 years b) 5 years a) Tests of significance b) Means
c) 7 years d) 10 years c) Ratio d) Normalcy
(MAN- 99) (ATTMS-99)
5. The levels of nicotine in 6 cigarettes were 13.0, 11.8, 10. Best measure for central tendency in a highly skewed
15.4, 17 .4, 31.2 and 24.3 mg. The mean of these values is distribution is:
a) 14.9 b) 15.9 a) Median b) Mean
c) 18.6 d) 8.4 c) Mode d) Range
(MAN-2K) (KAR-2002)
6. Father of health statistics is 11. The graphic representation of frequency distribution
a) John Snow b) John Graunt with X and Y axis is
c) Frederick Mc Kay d) Pierre Fauchard a) Frequency polygon b) Histogram
(MAN-2001) c) Pictograph d) Pie diagram
(AP-2001, 2003)

1) C 2) A 3) C 4) D 5) C 6) B 7) A 8) B 9) A 10) A 11) B
Dental ;lut.,e i======
12. A series may have all except: 24. For testing the statistical significance of the difference
a) No mode b) One mode in heights of school children among three socio
c) Many mode d) Negative mode economic groups the most appropriate statistical test is:
(KAR-2003) a) Student's 't' test b) Chi-squared test
13. The correlation coefficient can range from: c) Pai red 't' test
a) Oto+l b) Oto-1 d) One way analysis of variance (one way ANOVA)
c) -1 to +1. d) 0 to 100 (AIIMS-2003)
(KAR-2003) 25. A screening test is used in the same way in two similar
14. As the sample size increases standard deviation populations but the proportion of false positive results
a) Decreases b) Increases in population A is lower than that in population B. what
c) Remains the same d) may increase or decrease is the likely explanation?
(KAR-2003) a) The specificity of the test is lower in population A
15. In a 3x3 table, the number of degrees of freedom is: b) The prevalence of the disease is lower in population A
a) 4 b) 9 c) The prevalence of the disease is higher in population A
c) 3 d) 81 d) The specificity of test is higher in population A
(KAR-2003) (AIIMS-2003)
16. Most of the values in data spread along the 26. A measure of location which divides the distribution in
a) Mean b) Mode the ratio of 3:1 is:
c) Median d) Arithmetic mean a) Median b) First quartile
(PGI-2002) c) Third quartile d) Mode
17. In a study, variation in cholesterol was seen before and after (AIPG-2003, AIIMS-06)
giving a drug. The test which would give its significance is: 27. Standard deviation is a measure of:
a) Unpaired test b) Paired test a) Correlation b) Statistical significance
c) Chi square test d) Fisher's test c) Central tendency d) Variation
(AIPG-2002) (KAR-98, MAN-98)
18. The correlation between variables A and B in a study was 28. An investigator wants to study the association between
found to be 1.1. This indicates: maternal intake of iron supplements (Yes or No) and
a) Very strong correlation birth weights (in gm) of new born babies. He collects
b) Moderately strong correlation relevant data from 100 pregnant women and their
c) Weak correlation newborns. What statistical test of hypothesis would you
d) Computational mistake in calculating correlation advise for the investigator in this situation?
(AIPG-2002) a) Chi-square test
19. A Gaussian distribution is: b) Unpaired or independent t - test
a) Normal distribution b) Log normal distribution c) Analysis of variance d) Student t - test
c) Poisson distribution d) Variant distribution (AIIMS-2003)
(KAR-2003) 29. Denominator while calculating the secondary attack rate
20. The type II error is the acceptance of a null hypothesis includes:
as true when it is: a) All the people living in next fifty houses
a) True b) Nonexistent b) All the close contacts
c) Negative valued d) False c) All susceptibles amongst clos,e contact
(KAR-2003) d) All susceptibles in the whole village
21. The best sampling procedure that can be followed is (AIPG-2003)
a) Simple random b) Stratified random 30. The most appropriate method of testing the statistical
c) convenient d) Systematic significance of the differences between means of two
(KAR-99) groups is
22. The test done for a statistically significant change in a) Chi square test b) Multiple regression analysis
cholesterol values in a group of patients following an c) Correlation coefficient analysis
interventional programme is: d) Student test
a) Chi square test b) Paired "t" test (AIIMS-2001)
c) Unpaired "t" test d) Fisher test 31. The mean median and mode are the same in a
(AIIMS-2001) a) Normal distribution b) Poisson distribution
23. The systolic blood pressure in normal healthy males of c) Binomial distribution d) None of the above
age group of 40 - 6 0 has a normal distribution with a (KAR-99)
mean of 130 in a sample of 100. People with systolic 32. Correlation between the height and weight of children is
blood pressure more than 130 will be: best represented by:
a) 100 b) 75 a) Histogram b) Line diagram
c) 50 d) 25 c) Scatter diagram d) Bar diagram
(AIIMS-2001) (AIIMS 2001)

12) D 13} C 14} A 15} A 16} A 17} B 18) D 19} A 20) D 21) B 22) B 23) C 24} D
25} C 26} C 27} D 28) B 29) C 30} D 31) A 32) C
, COMMUNITY DENTISTRY

33. Statistics to check out mean of values is done by: 42. When frequency is given as mild, moderate, and s evere,
a) Chi-square test b) Student's analysis the data scale used is
c) Probability test d) Horizontal studies a) Ordinal b) Nominal
(AIIMS-95) c) Variance d) Interval
34. Sensitivity of a test is its ability to identify
a) True negative b) False positive 43. When variables are not measurable, which of the
c) False negative d) True positive following is used to be represented?
(COMEDK-2003) a) Variation b) Ordinal
35. Middle value of a series of a given data arranged in a c) Nominal d) Ratio
ascending or descending order is called
a) Median b) Mode 44. In a random sampling chance of being picked up is
c) Mean d) Average a) Same and known b) Not same and not known
(UPSC-2001) c) Not same but known d) Same and not known
36. Chi square test is applied to test the independence of
cross distribution of two variables and each of them 45. An investigator want to know the similarity of the mean
should have peak flow of expiratory rates and non- smokers, light
a) Equal num ber of cases b) Mutually exclusive cases smokers, moderate smokers, & heavy smokers. Which is
c) Normal distribution d) Non-mutually exclusive cases statistical test of significance:
(AIPG-2K) a) One way ANOVA b) Two way ANOVA
3 7. The positive predictive value of a screening test depends on c) Student -t test d) Chi square test
a) Sensitivity of the test b) Specificity of the test (AIIMS MAY 2012)
c) Value of the test d) Efficiency of the test 46. All of the following are random sampling methods except
(AIPG-2K) a) Simple random b) Cluster sampling
38. Which of the followi ng condition must be met for a valid c) Stratified random d) Quota sampling
student-t test between means of two groups:
a) Number of observation must be same - no 47. Standard error of mean indicates
b) Standard deviation must be approximately same - no a) Dispersion b) Distribution
c) Mean should be approximately same - no c) Variation d) Deviation
d) Sample must be small
(AIIMS MAY 2012) 48. Confidence limits are
39. The usefulness of a screening test depends upon it a) Mean± range b) Mean± standard error
a) Sensitivity b) Specificity c) Mean - standard error d) Mean + standard error
c) Reliability d) Predictive value
(AIIMS -2003) 49. In a sample size of 100 children the mean weight is
40. The number of dental caries cases reported during last found to be 35 kg with the standard deviation of 2,
10 years are given below 250, 320, 190, 300, 5000, 100, calculate the standard error of mean.
260, 350, 320 and 160. The epidemiologist wants to find a) 1 b) 0 .2
out average number of caries cases reported in that town c) 0.01 d) 10
for Last 10 yrs. The most appropriate average per this
data will be 50. If the mean cholesterol value of a group of normal
a) Arithmetic mean b) Mode subjects is 230 mg% with standard error of 10. The 95%
c) Median d) Geometric mean confidence Limits for the population
(AIIMS- 2004) a) 220 and 240 b) 210 and 250
41. In assessing the association between maternal nutritional c) 200 and 260 d) 210 and 240
status and the birth weight of newborns two investigators
A and B studied s eparately and found significant results 51. A normal distribution curve depends on
with P values 0.02 and 0.04 respectively. From this a) Mean and sample size b) Range and sample size
information what can you infer about the magnitudes of c) Mean and standard deviati on
association found by the two investigators d) Mean and Median
a) The magnitude of association fo und by B is more than A
b) the magnitude of association found by A is more than B 52. Which statement is true about normal distribution
c) The estimates of association obtained by A and B will be curve?
equal, since both are significant a) Mean -1, SD - zero b) SD-1, Mean - zero
d) Nothing can be concluded as the information given is c) Curved Skewed towards left
i nadequate. d) Curved Skewed toward right
(AIIMS - 2004)
53. Which of the following is correct?
a) Mean ± 1 SD covers about 65% of the area

33) B 34) D 35) A 36) B 37) A 38) D 39) A 40) C 41) A 42) A 43) C 44) D 45) A
46) D 47) D 48) B 49) B 50) B 51) C 52) B 53) D
Dental ;lut.,e i======
b) Mean ± 2 SD Standard deviation covers about 95% of c) Predictive value d) Repeatability
area. Here Z- score is taken as 2
c) Mean ± 3 SD covers about 99% of area here Z-score is 63. When a circle is divided into different sectors
taken as 2.5. corresponding to the frequencies of the variable in the
d) All of the above distribution, the diagram is known as:
a) Pie b) Bar
54. Which of the following is correct c) Histogram d) Pictogram
a) In normal distribut ion curve Mean = Median = Mode (KAR-04)
b) In Positively Skewed (right skewed) distribution curve 64. Sampling that involves randomly selecting some of the
Mean> Median pre existing natural groups is called:
c) In negatively skewed (left skewed) distribution curve a) Cluster b) Quota
Mean< Median c) Systematic d) Simple random
d) All of the above (KAR-04)
65. The branch of biostatistics which deals with the births,
55. In a periodontia quiz, conducted by Or. Kamala Devi , the deaths and marriage is called as :
score, obtained by 7 students are 10, 10, 10,70,40,20 and a) Health b) Medical
90. Which of the following correctly describes the quiz? c) Demography d) Vital
a) Normal dist ribution b) Positively skewed (KAR-04)
c) Negatively Skewed d) None of the above 66. The number of independent members in the sample is:
a) Null b) Degree of freedom
56. Mean weight of 100 children was 12kgs. The standard c) Bias d) Significance
deviation was 3. Calculate the percent coefficient of (COMEDK-06)
variance. 67. Square root of pq/ n indicates:
a) 25% b) 35% a) Standard error of mean
c) 45% d) 55% b) Standard error of difference of means
c) Standard error of proportions
57. Prevalence of diseases is d) Standard error of difference in proportions
a) Rate b) Ratio (AIPG-06, AIIMS-07)
c) Proportion d) Deviation 68. In health education program, a group of 10 people are
(COMEDK-2013) planning to speak on a single topic of common interest.
58. Study that is used to compare serum cholesterol levels in Which one of the following is best educational approach?
obese and non-obese women and to find the relation or a) Panel discussion b) Symposium
significant prevalence is c) Group discussion d) Workshop
a) Chi-square test b) Pai red 't' test (AIPG-06)
c) Student 't' test d) Z- test 69. Which of the following is true about Chi-square test?
a) Measures qualitative data
59. Standard drug when used 40% patients responded and b) Measures both qualitative and quantitative data
when a new drug was used 60% patients responded. c) Measures the qualitative data between two proportion
Which of the following tests is most useful in this study? d) Measure the quantitative data between two proportion
a) Fisher 't' test b) Paired 't' test (AIIMS-06)
c) Chi square test d) None of above 70. Comparison of proportions can be made by:
a) 't' test b) Chi-square test
60. True regarding chi-square test is c) ANOVA d) Correlation & Regression
a) Null hypothesis is equal (COMEDK-07, PGI JUNE- 13)
b) Does not test the significance 71. Chi-square test is used to measure the degree of:
c) Measures the significance of difference between two a) Causal relationships between exposure and effect
proportion b) Association between two variables
d) None of above c) Correlation between two variables
d) Agreement between two observations
61. More false positive on screening in a community signify (AIIMS-06)
that the 72. Yates correction is necessary in chi-square test when
a) Disease has low prevalence expected frequency in any one cell is
b) Disease has high prevalence a) Less than 5 b) More than 5
c) Test has high sensitivity c) Less than 10 d) More than 10
d) Test has low specifi city (COMEDK-08)
73. Accepting the null hypothesis when it is false:
62. Prevalence of the disease affects a) Type-I error b) Type III error
a) Sensitivity b) Specificity

54) D 55) B 56) A 57) C 58) C 59) C 60) C 61) A 62) C 63) A 64) A 65) D 66) B
67) C 68) C 69) C 70) B 71) B 72) A 73) C
, COMMUNITY DENTISTRY
~

V
633

c) Type II error d) Type IV error c) Mode d) Range


(COMEDK-08) (AIPG-10)
74. What is the characteristic of normal distribution? 85. Listing conditions in some order of severity without
a) Mean = Median = mode attempting to define any mathematical relation between
b) Mean is 1, SD-zero categories is -
c) Curve skewed towards the left a) Nominal scale b) Ordinal scale
d) Curve skewed towards the right c) Internal scale d) Ratio scale
(AIPG-09) (KCET-09)
75. A sampling unit is defined as 86. The ratio of rates of occurrence is
a) A group of sample a) Attributable risk b) Relative risk.
b) A subset of sample c) Occurring risk d) Distribution risk.
c) Representing every member of sample (COMEDK-2011)
d) All of the above 87. An examiner wants to study two groups of infants for 50
(AIPG-09) each, CPAP and double tests were performed, in 30 infants
76. Non parametric test used to compare the medians of two CPAP was positive and in 20 bubble test was positive,
independent samples what should be the test used will be used in this case
a) t-test b) Chi square test a) Chi square test b) Student's test
c) Mann Whitney U test d) Z test c) Paired t test d) Coefficient of variance
(COMEDK-10) (AIPG-2011)
77. Student 't' test is a 88. 'P' value is 0.001 then
a) Non parametric test b) Chi square test a) Results are true of 99.9% of population
c) Parametric test d) Test of non significance b) Test is not significant
(COMEDK-10) c) 1% of significance d) There is no co-relation
78. Diagrams used to show percentage breakdowns for (AIIMS-2011)
qualitative data 89. For a survey, some schools were selected randomly from
a) Pie diagram b) Bar diagram which classes were selected randomly and then sections
c) Histogram d) Cartogram were selected randomly and from which students were
(COM ED K-10) selected randomly, which type of sampling method is this
79. Mode is a) Simple random b) Cluster sampling
a) Value that most frequently occurs in a set of observation c) Stratified sampling d) Multistage sampling
b) When observations are arranged in ascending or (AIPG-2012)
descending order the value that comes in the centre 90. In a class the mean weight of 4 students is 90 kg and
c) Calculated by adding up the values of a group of items standard deviation is 30, then 95% confidence interval
and dividing the sum by the number of items. would be accordingly
d) Best measure for central tendency in a highly scented a) 60-80 b) 45-95
distribution c) 60-120 d) 70-110
(AIIMS-08) (AIPG-2012)
80. In a population S.D is small and if a small sample size is 91. 75% negative predictivity indicate
taken, then a) 75% population have the disease
a) Standard error will be high b) 25% population tested do not have the disease
b) It will not be representative of population c) There is 75% probability that the people tested negative
c) It will give precise result for the disease, do not have the disease in reality
d) Standard deviation is increased d) 75% population tested, 75% do not have the disease
(AIIMS-08, NEET-2013) (PGI-2011)
81. Histogram is a 92. Correlation between two variables are best depicted by
a) Pie diagram b) Bar diagram a) bar diagram b) line diagram
c) Frequency polygon d) Line diagram c) histogram d) scatter diagram
(KCET-10) (AP-2013)
82. Specificity of a sere ening test measures? 93. Standard deviation to mean ratio denotes the
a) True positives b) False positives a) Standard error b) Coefficient of variance
c) False negatives d) True negatives c) Proportion d) Coefficient of skewness
(AIPG-09) (AIPG-2012)
83. In a population sample size depends on 94. Scattered diagram represents
a) expected prevalence b) expected incidence a) Trend over time b) Pie chart
c) universe size d) population size c) Association between two variables
(AIPG-10) d) Frequencies telling occurrences
84. All are the measures of dispersion except (PGI-2011)
a) Variance b) Standard deviation

74) A 75) C 76) C 77) C 78) A 79) A 80) A 81) B 82) D 83) A 84) C 85) B 86) B
87) A 88) A 89) D 90) C 91) C 92) D 93) B 94) C
Dental ;lut.,e

95. All of the following are close to median except


a) 2nd quartile b) 50th percentile
c) 2nd tertile d) 5th decile
(PGI-2011)
96. The process by which controls are selected in such a
way that they are similar to cases with regard to certain
clinical variables in a case control study is known as
a) Confounding b) Matching
c) Bias d) Sampling
(KAR-2013)
97. The mean and median dental caries levels of a Taluk
were 4 and 5. What would be the possible mode for this
population?
a) 7 b) 2
c) 1 d) 9
(KAR-2013)
98. A data collected for specific purpose and used for the
same purpose is
a) Original data b) Primary data
c) Secondary data d) Specific data
(COMED-14)
99. The response which is graded by an observer on agree or
disagree continuum is based on?
a) Visual analogue scale b) Guttman scale
c) Likert scale d) Adjective scale
( PGI DEC-2013)

95} C 96} B 97} A 98} B 99} B


, COMMUNITY DENTISTRY
~

V
635

1. BIOSTATISTICS - ANSWERS
1. 'C' [Soben Peter 4th ed 386/ 5th ed 609] • Census is the collection of information from all the
Correlation coefficient (r) indicates the degree of correlation individuals in a population.
between two variables. Its value ranges from - 1 to +1. The • Sampling is the collection of information from the
correlation coefficient is zero when there is no covariation representative units in the sample.
between two variables.
• In census, as the information is collected from
all individuals, the cost of organisation and time
Independent of the sign, the farther the r value from 'O', the
consumption will be more. Also the accuracy and
stronger the relationship i.e., -1 indicates perfect negative
completeness are less in census when compared to
correlation where as +1 indicates a perfect positive
sampling.
co rrelation. Since O. 75 is farlher from O Lhan 0.25 and 0.5,
option c is the correct answer.
5. 'C' [Soben Peter 4th ed 381-82/ 5th ed 603-04]
2. 'A' [Soben Peter 4th ed 381-82/ 5th ed 603-04]
6. 'B' [Soben Peter 4th ed 3 79/ 5th ed 601]
Arithmetic mean, Median, Mode are common measures of
central tendency.
7. 'A' [C.R. Kothari 2"d ed 71]
a) Arithmetic Mean (X) Data is classified into quantitative and qualitative data.
Qualitative data can be nominal or ordinal and quantitative
• It is simplest measure of central tendency.
data can be discrete (often, integer) or continuous.
• It is obtained by summing up all the observations
divided by the number of observations.
• It is not an appropriate measure for very skewed TYPES OF SCALES IN STATISTICS
distributions as it is very sensitive to extreme scores. a) The information is divided into some
definite qualitative basis.
b) Median (Md) Eg:
• It is the simplest division of a set of measurements Nominal • Male/ female
into two halves - the upper and lower half. The point scale data • White/ black
on the scale that divides the group in this way is
• Urban / Sub-urban / Rural
median.
• Age
• Median is insensitive to small numbers of extreme
scores in a distribution; therefore it is a very useful • Socioeconomic status
measure of central tendency for the highly skewed a)
Information is expressed in ordinal or
distributions. rank order relations.
• When the distribution has odd number of elements, Eg:
middle value is median. When the distribution has • Olympic medals
even number of elements, the average of two middle
scores is Median. • Ranking of medical st udents
Ordinal b) For example, Ramu is taller than Ravi
c) Mode scale data and Ravi taller than Vijay, we know
• It is the most frequently occurring value in a set of something. The information gives the
observations. difference in quality but not quantity. No
conclusion can be drawn about whether
• If only two scores occur with greatest frequency, the the height difference between Ramu and
distribution is bimodal. Ravi is same as the difference between
• If more than two scores occur with greatest Ravi and Vijay.
frequency, the distribution is multimodal.
a) Uses a scale graded in equal increments
3. 'C' [Soben Peter 4th ed 381-82/ 5th ed 603-04] Eg: Height, weight, blood pressure.
Interval For example the difference between
b)
• Mean, median and mode are measures of central tendency. or 2nd and 3rd inches is same as the
• Range (KERALA-15) , standard deviation and coefficient numeric difference between 7th and 8th inches.
of variation are common measures of dispersion or
variation. scale data c) If a measurement has a "mean" and a
"standard deviation", it can be treated as
• Range is the difference between the smallest item and
interval scale.
the value of largest item.

4. 'D' [Park 20th ed 743/ 22nd ed 783]


Dental ;lut.,e i======
a) The interval scale data is placed with d) Judgement:
some meaningful ratio, In this method, the sample selection depends exclusively
on the judgement of the investigator.
Eg:
• Weight 2. Probability sampling:
Ratio • Time It is the recommended method of sampling in which
scale data • Blood pressure each individual unit in the population has a known
probability of being selected. It includes:
• Temperature measured in degrees
Kelvin a) Simple random sampling
b) This type of data is biomedically most Lottery method
significant. Table of random numbers
b) Systematic sampling
8. 'B' (Soben Peter 4th ed 368-69/ 5th ed 591] c) Stratified sampling
Different sampling methods depending upon the type
and nature of the population and the objectives of the Stratified random sampling
investigation are: d) Cluster sampling

1, Non probability sampling:- 3. Other sampling methods


Non-probability samples are not truly representative and a) Multiphase sampling
are therefore less desirable than probability samples. This
b) Multistage sampling
method is used in cases where a researcher may not be able
to obtain a random o,r stratified sample, or it may be too
Simple Random Sampling
expensive or when it may not be necessary to generalize to
a larger population. The three types are: i) It is applicable when the population is small, homogenous
and readily available.
a) Quota sampling ii) Used for clinical t rials like testing the efficacy of a
Here the general composition of t he sample is decided in particular drug.
advance and includes a particular segment of population. Eg.: Ward patients, patients coming to hospital.

Eg: If a researcher is interested in attitudes of members


of different states, he could set a quota of 3% quota Systemic Sampling
people of each state. However, the sample may no i) It is applicable when the population is large, non-
longer be representative of actual proportions in the homogenous and scattered.
population. ii) Sample is calculated by taking every k'h variable, where
'k' is
b) Purposive sampling:
Purposive sampling is done to save a very specific need Total population
K=
or a purpose. A researcher may have a specific group in Sample size desired
mind, such as high level business executives.
Stratified Sampling
A subset of purposive sample is a "Snow ball" sample/
chain referred sample" in which the researcher picks i) It is used when population is large and not homogenous
up the sample along the way, analogous to a snowball ii) Initially, the population is divided into homogenous
accumulating snow. A snow ball sample is achieved by subgroups known as strata and a sample is drawn
asking a participant to suggest someone else who might randomly from each stratum.
be willing or appropriate for the study. Snow ball samples
are particularly useful in hard-to-track populations, such Stratified Random Sampling:
as those with illegal behaviour like drug users, homeless i) In this technique the population to be sampled is
people etc. subdivided into groups known as strata, such that each
group is homogenous in its characteristic.
c) Convenience sampling:
A convenience sample is a matter of taking what you ii) Asimple random sample is t hen chosen from each stratum.
This type of sampling is used when the population is
can get. It is not randomly obtained volunteers would
constitute a convenience sample. heterogenous with regard to the characteristic under
study. This method ensures more representativeness,
provides greater accuracy and can concentrate on a
A non-random sampling technique is (KERALA-2015) wider geographical area.
a) Systematic sampling b) Convenience sampling
c) Multistage sampling d) Cluster sampling
, COMMUNITY DENTISTRY
,.....__,,_

V
637

Cluster sampling: Histogram is a bar diagram without gap between the bars.
i) Cluster sampling involves grouping the population and It is a pictorial diagram of frequency distribution. It is
then surveying the entire population used to depict quantitative data of continuous type. On the
X-axis, the size of an observation is marked. On the Y-axis is
ii) It is used when units of population are natural
the frequencies are marked.
groups or clusters such as villages, schools, wards,
blocks, etc.,
Frequency polygon - This is used to represent frequency
distribution of quantitative data and is useful to compare
Multiphase sampling:
two or more frequency distributions. A point is marked
i) Here sampling is done in different phases. For example: over the mid point of the class interval, corresponding to
in tuberculosis survey. frequency and these points are connected by straight lines.
• First phas e - Montoux test is done in all cases of
sample Pie diagram is commonly used to show percentage
• Second phase - X-ray of chest taken in Montoux breakdowns for qualitative data. It is so called because the
positive cases. entire graph looks like a pie and its components represent
slices cut from pie. A circle is divided into different sectors
• Third phase - Sput um examination in X-ray positive
corresponding to the frequencies of the variables in the
patients.
distribution.
ii) Advantages - Less cost, less laborious and more
purposeful. The total angle at the center of the circle is equal to 360°
segments are drawn in the circle according to the data
Multistage Sampling and are shaded with different shades or colours. But, this
i) It is employed in large country surveys. diagram cannot be used to represent two or more data sets.
ii) Sample procedures carried out in several stages using
random sampling techniques. Line diagram is useful to study the changes of values in
the variable over time and is the simplest type of diagram.
Eg:
On the X-axis, the time such as hours, days, weeks, months,
Country wide survey or years are represented and the value of any quantity
,J, 1st Stage pertaining to this is represented along the Y-axis.
District wide survey by selecting some districts
Cartograms or spot map is used to show the geographical
randomly
distribution of frequencies of a characteristic.
,J, 2nd Stage
Village wide survey by selecting some villages randomly Pictogram is simpler and popular method of presenting
,J, 3rd Stage data to "man in street" and those who can't understand
Some units are selected randomly complicated charts. Here small charts are used to present
the data.
9. 'A' [Soben Peter 5th ed 606)
12. 'D' [Soben Peter 4th ed 382/ 5th ed 604)
The two questions that arise about any hypothesized relation
The mode or modal value is that value in a series of
between two variables are;
observations that occurs with the greatest frequency
a) What is the propability that the relationship exists? (NEET-2013).
b) If the relationship really exists, what is the strength of
the relationship? Eg1. Age of eruption of 3rd molar in a sample of 10 individuals
is 18, 19, 20, 18, 21, 18, 19,22,23,20. Since 18 is repeated
The first question can be answered by tests of significance, more number of times than any other number, mode is 18.
which says the propability of relationship between two
variables is really just a chance occurence. Tests of Eg2. Age of eruption of 3rd molar in a sample of 10 individuals
significance deal with techniques to know how far the is 18, 19, 20, 18, 21, 18, 19, 22, 19, 20. Since 18 & 19
differences between estimates of different samples is due to are repeated more number of times than any other number,
sampling variation. modes are 18 & 19. Such distribution is called bimodal
distribution.
10. 'A' [Soben Peter 4th ed 381-82/ 5th ed 604)
Median is insensitive to small numbers of extremes scores Eg3. Age of eruption of 3rd molar in a sample of 10 individuals
in a distribution, therefore, it is a very useful measure of is 18, 19, 20, 18, 20, 18, 19, 22, 19, and 20. Since 18, 19
central tendency for highly skewed distribution. and 20 are repeated equal number of times; modes are 18, 19
and 20. Such distribution is called multimodal distribution.
11. 'B' [Soben Peter 4th ed 373, 377/ 5th ed 600)
Bar diagram is used to represent qualitative data. It Eg4. Age of eruption of 3rd molar in a sample of 5 individuals
represents only one variable. is 18, 19, 20, 17, 21. Since no number is repeated, there is
no mode for this distribution.
Dental ;lut.,e i======
There will never be a negative mode for any distribution of Chi-square test:
data.
i) It is used in testing hypothesis about nominal scale
data. It is basically a test of proportions.
13. ' C' [Soben Peter 4th ed 386/ 5th ed 609)
ii) It is used to examine differences between frequencies in
14. 'A' [Soben Peter 4th ed 382/ 5th ed 605) a sample.
Range, coefficient of variation and standard deviation are
iii) The three most important applications of Chi-square in
common measures of variability or variation or dispersion.
statistics is as a test of -
Standard deviation (root mean square deviation) is the most
important and widely used measure of studying dispersion. • Proportion
It is the square root of the mean of the squared deviations
• Association
from arithmetic mean and is given by formula
• Goodness of fit
s = JI,d2 iv) More than two groups of subjects can be sampled.
n - 1

Where d = deviation of the items from the mean Fisher's exact test:
=Xi - X It is a test for the presence of association between two
categorial variables, used when the numbers involved are
n = total number of observations. too small to permit the use of a Chi-squared test.

Since n is present in the denominator, it is inversely 18. 'D' [Soben Peter 4th ed 386/ 5th ed 609)
proportional to standard deviation. Hence, as n value
increases, SD decreases and vice versa. 19. 'A' [Soben Peter 4th ed 383/ 5th ed 605)
• In normal distribution, the curve of distribution of a
15. 'A' [Soben Peter 4th ed 385/ 5th ed 607-08) given character is more or less symmetrical. It is bell-
Degree of freedom is calculated by the formula shaped.
d.f. (r-1) x (c-1)
= • In normal distribution, Mean = Mode = Median
Where, r = Number of rows • The shape of normal distribution depends on mean,
c = Number of columns standard deviation and the number and nature of
observation. Total area of this bell-shaped curve is one.
A 3x3 table contains 3 rows and 3 columns. Then
d.f. = (3-1) X (3-1) = 2 X 2 = 4. (Note:- Effect of temperature on enzyme velocity also yields
bell-shaped curve)
16. 'A' [Soben Peter 4th e d 383/ 5th ed 604-06)
20. 'D' [Soben Peter 4th ed 396/ 5th ed 623)
17. 'B' [Sobe n Peter 4th ed 386/ 5th ed 608] Type-I error occurs if the null hypothesis is rejected when
"t" test: it is actually true (PGl-11). Type-II error occurs if the null
It was first derived by Grosset in 1908, who published hypothesis is accepted when it is false (COMEDK-08)
statistical papers under the pen name of "student'. Thus the
test is also known as student's 't' test. The t test is used for
testing the significance of
a) The mean of sample
b) The difference between means or to compare two sample
(with the help of two types of tests i.e. paired 't' test
and unpaired 't' test.
c) Correlation co-efficient. The rate of the type I error is called the size of the test and
denoted by the Greek Letter o: (alpha). It usually equals the
Paired 't' test significance Level of a test. The rate of the type II error
It is applied to paired data of independent observations is denoted by the Greek letter p (beta) and related to the
from one sample only when each individual gives a pair power of a test (which equals 1-(3).
of observation i.e., the same people are sampled on two
different occasions. The power of a statistical test (AIPG-14) is the probability
that it correctly rejects the null hypothesis when the
Unpaired 't' test: null hypothesis is false. The power is also known as the
i) It is applied to unpaired data of independent sensitivity.
observation. In this, two groups of subjects are sampled
on one occasion. 21. 'B' [Soben Peter 4th ed 3 69/ 5th ed 593)
ii) Both paired and unpaired tests are used for two groups
22. 'B' [Soben Peter 4th ed 384/ 5th ed 608)
only i.e., compares two means only
, COMMUNITY DENTISTRY
,......__,,_

V
639

Cholesterol level is a quantitative dat a, measured by interval are types of quantiles. Centiles divides the distribution into
scale and can be summarized using mean and standard 100 equal parts. Quartiles divide the distribution into 4 equal
deviation. To compare mean cholesterol values of a group parts. Deciles divide a distribution into ten equal parts.
before and after intervention, paired t test is used.
1st quartile divides the distribution in the ratio of 1:3
To compare mean cholesterol values of two different groups,
unpaired t test is used. Chi square test is used to compare 2nd quartile divides the distribution in the ratio of 2:2
two or more proportions. 3rd quartile divides the distribution in the ratio of 3:1

23. 'C' [Soben Peter 4th ed 383/ 5th ed 605-06] 27. 'D' [Soben Peter 4th ed 382/ 5th ed 605]
Since the mean is 130, 50% of people (50 people) will have
blood pressure below 130 and 50% (50 people) will have a 28. 'B' [Soben Peter 4th ed 384-86/ 5th ed 608]
blood pressure above 130. There are two groups of people:
1. Infants born to those mothers who have taken iron
24. 'D' [Soben Peter 4th ed 386/ 5th ed 609] supplements
Analysis Of Variance or ANOVA Test or F-ratio
2. Infants born to those mothers who have not taken iron
In any set of experimental values after a biostatistical study,
supplements
there will be some variability which can be attributed t o two
main components.
Birth weight is a quantitative data, measured by interval
i) Variations within a group - These can be scale and can be summarized using mean and standard
• Sex deviation. To compare mean birth weights of two different
• Placebo groups, unpaired t test is used.

• Drug Note: If the same question is asked as incidence of low birth


weight (<2500 gms i.e. low birth weight baby or normal bay
ii) Variation between different groups - These can be which is a qualitative data), then the answer would be chi-
• Sampling error square test.
• Individual differences
29. 'C' [Soben Peter 4th ed 48,49/ Park 22nd ed 96]
Attack rate is an incidence rate used only when a group of
ANOVA is preferred for more than one comparison . It
population is exposed to risk for a limited period of time. It
compares the variance between different groups with the
gives the information on the number of cases in population
variation within a group.
at risk.
One way ANOVA test :
Secondary attack rate (SAR) is calculated by the formula:
This test is indicated when various experimental groups
differs in terms of only one factor at a time.
No. of exposed persons developing the disease
In the above question, children of three socio economic SAR within the range of the incubation period
groups differs in terms of only factor i.e., height of children. Total Number of exposed/ susceptible contacts
So one way ANOVA test is used.
SAR is limited in its application to infectious diseases in
Two way ANOVA or Multifactorial ANOVA: which the primary case is infective for on ly a shorter period
It is indicated when various experimental group differs in of ti me ( Eg. In day for measles or chicken pox).
terms of two factors (For example: height & weight) at a
time. 30. 'D' [Soben Peter 4th ed 385-86/ 5th ed 608]

25. 'C' [Park 20th ed 126-28/ 22nd ed 131) 31. 'A' [Soben Peter 4th ed 383/ 5th ed 606)
Same screening test is used in the same way in two similar In normal distribution, the curve of distribution of a character
populations, but the proportion of false-positive (people is symmet ric. It is also known as guassian distribution .
without disease) in population B is higher than that among
those who t est positive in population Ai.e less no of people Properties of normal cuive:
actually have the disease in population B which inturn
i) Bell shaped
means the prevalence of disease in population Bis less than
Population A. ii) Symmetrical
iii) The height of curve is maximum at the mean.
Simply prevalence is inversely proportional to false +ve cases.
iv) Mean = Median = Mode
26. 'C' [Mahajan 6th ed 49]
Quantiles is a family of values each of which divides a
distribution into equal parts. Centiles, Quartiles and Deciles
Dental ;lut.,e i======
v) The area under the curve between any two points can be POSITIVE PREDICTIVE VALUE:
found out in terms of a relation-ship between mean and i) It is probability of a disease who receives a positive test
standard deviation result.
• Mean± 1 S.D. covers 68.3% of the observation. True positives
• Mean± 2 S. D. covers 95.4% of the observations. For Positive prediactive value = ----~------
(True positives+ False positives)
95% confidence, z score is taken as 2
TP
• Mean ± 3 S.D. covers 99.7% of the observations. TP + FP
For 99% confidence, the Z score or the degree of
confidence is taken as 2.5.
• The Limits on the either side of mean are called t NEGATIVE PREDICTIVE VALUE:
"confidence Limits". i) The probability of no disease in a person who receives a
• The total area of the curve is one, its mean is zero negative test resu Lt.
and standard deviation is one. If the curve is not . . . True negatives
normal, then it is skewed distribution. Negative pred1active value = ( . l . )
True negatives+ Fa se negatives
• In positively skewed distribution curve, (right
skewed) mean > median. TN
TN+ FN
• In negatively skewed or Left skewed distribution
curve, the mean < median. 35. 'A' [Soben Peter 4th ed 382/ 5th ed 604]

32. 'B' [Soben Peter 4th ed 378/ 5th ed 601] 36. ' B' [Check Explanation Below]
Scatter digram shows the relationship between two • Chi-square test is used to determine if categorical
quantitative variables, in the same person or group of data shows dependency or not (data must be mutually
people. Eg: height & weight, Chronological age & dental age exclusive and discrete i.e., qualitative).
• Mutually exclusive means, if a person is counted as
33. ' B' [Soben Peter 4th ed 385-86/ 5th ed 608]
belonging to one of the categories, she cannot also be
counted as belonging to another category.
34. 'D' [Park 20th ed 127/ 22nd ed 131]
SPECIFICITY: 37. 'A' [Park 20th ed 127-28/ 22nd ed 131]
i) It is probability of correctly identifying disease free The positive predictive value of a screening test depends
persons. on sensitivity of the test. The negative predictive value of
ii) It indicates the proportion of truly diseased persons who a screening test, depends on specificity of the test. {Also
are identified as non-diseased by the screening test. It refer Q. No. 34)
is also known as the "true negative rate".
38. ' D' [Dixit 4th ed 102]
Spec1'fic1·ty -_ (T True negatives
.
For large sample sizes, 7! test is usually preferred. For small
rue negatives + Fa lse positives
.. )
samples 't' test is preferred.
TN
TN+ FP 39. 'A' [Park 20th ed 125-27/ 22nd. ed 129-30]
The basic purpose of screening is to sort out from a large
iii) Less the number of false positive cases, more will be the group of apparently healthy persons those likely to have
specificity. a disease or at the increased risk of the disease under
study. Any screening test must satisfy the major criteria
SENSITIVITY: of acceptability, repeatability and validity besides other
i) It is the probability of correctly identifying a case of minor criteria. Also the measures that are used to evaluate
disease. a screening test are:
ii) It indicates the proportion of truly non diseased persons • Sensitivity, which is directly related to screening test as
in the screened population who are identified as diseased sensitivity of a test is to identify correctly all those who
by the screening test. have disease, i.e., "true positive".
• Specificity
True positives • Positive predictive value
Sensivitity
(True positives+ False negatives) • Negative predictive value
TP • Percentage of false negatives
TP + FN • Percentage of false positives
Both the parameters of sensitivity and specificity are used
for assessing criterion validity. 40. ' C' (Sob en Peter 4th ed 381-82/ 5th ed 604]
Though arithmetic mean is the most commonly used measure
of central tendency, but it can not be used in this case as
,......__,._
COMMUNITY DENTISTRY 641
V
it is very sensitive to extreme scores (5000). If there are Confidence limits = 230 ± (2 X 10)
extreme values in the series of data median may be used. = 230 ± 20
41. 'A' [Dixit 4th ed 94-100] = 210 and 250
If the computed p-value is less than 0.05, it means that
the test has achieved statistical significance. Greater the 51. 'C' [Soben Peter 4th ed 383/ 5th ed 606]
value of p nearer to zero, more is the level of significance.
P value of 0.05 means that the investigator is 95% sure that 52. ' B' [Mahajan 6th ed 77-85/ 5th ed 606]
the result was not obtained by chance.
53. ' D' [Soben Peter 4th ed 383/ 5th ed 605-06]
In this question the investigator A is 98% (0.02) sure that
the result did not occur by chance compared to investigator 54. ' D' [Veer Bala Rast ogi 2nd ed 2 06]
B, who is 96% (0.04) sure that the result did not occur by
chance. 55. ' B' [Veer Bala Rastogi 2nd ed 206]
Here, Mean = 10 + 10 +10 +70 + 40 + 20 + 90
If the computed p-value is less than 0.05 we can reject null 7
hypothesis. A Type-II error occurs if the null hypothesis is = 250 / 1 = 36
not rejected when it is really false.
Median = 10, 10, 10, 20, 40, 70, 90 = 20
If the computed p-value is > 0.05, it means that the
estimates of association are not significant and the null Since Mean > Median, a positively skewed distribution
hypothesis is not rejected. A Type-I error occurs if the null correctly described the quiz.
hypothesis is rejected when it is really true.
56. 'A' [Soben Peter 3rd ed 3 7/ Veer Bala Rastogi 2nd ed 155]
42. 'A' [C.R. Kothari 2nd ed 71]
Refer to explanation of Q. No. 7 Coefficient of variation = Standard deviation xlOO
Mean
43. 'C' [C.R. Kothari 2nd ed 71] 3
= X 100 = 25%
Refer to explanation of Q. No. 7 12
5 7. 'C' (Park 22nd ed 58 Last Line/ Soben Peter 5th ed 53]
44. ' D' [Soben Peter 4th ed 369-70/ 5th ed 592] Although referred to as a rate, prevalence rate is a ratio or
percentage or proportion i.e. proportion of people having
45. 'A' [Dixit Biostatistics 4th ed 198] the disease among the estimated population at the same
Ref. Q. No 24 point in time.

46. ' D' [Soben Peter 4th ed 369/ 5th ed 592-93 ] 58. 'C' [Mahajan 6th ed 168-175/ Dixit 4th ed 104]
In the given question, the study was conducted to assess
47. ' D' [Soben Peter 4th ed 382-83/ 5th ed 606] the effect of a drug in lowering serum cholesterol levels
Standard error = ~
(quantitative data) in two groups i.e., obese and non-obese
rn (two Limbs in the study). Thus the mean lowering of serum
cholesterol would be obtained in two samples, thereby
Where, S.D. = Standard deviation
making "two-sampled student's 't' test' as the test of choice.
N = Number of variables
59. 'C' [Mahajan 6th ed 168-175/ Dixit 4th ed 109-116]
48. ' B' [Mahajan 6th ed 132/ Dixit 4th ed 100]
Confidence limits= Mean ± Z score x Standard Error • Chisquare test is used to compare the proportions,
percentages in two or more different group of individuals.
= x ± Z(S.D) / -m • Fisher's test is a variant of chi-square test when the
sample size is< 25-30.
49. ' B' [Mahajan 6th ed 134/ Park 22nd ed 793 ] • In the above question, 40% patients responded are
SD SD 2 compare to 60% patients responded. So the test
Standard error = ~ = ·= = - = O 2
,n ~100 10 · recommended to compare results here is chi-square test.
• Also note that, sign test is used to compare the
50. ' B' [Mahajan 6th ed 118, 119/ Dixit 4th ed 100] proportions, percentages in same group of individuals
Confidence limits= Mean ± Z score x Standard Error before and after intervention.

= x ± Z(S.D) / -m 60. 'C' [Mahajan 6th ed 168-175/ 7th ed 154]


In this question,
x = Mean = 230 mgm %
Z score for 95% confidence limit= 2
Standard error = 10
Dental ;lut.,e i======
61. 'A' [Park 20th ed 127/ Simple biostatistics by Indrayan & For example, the DMF score in a patient is 5. We can predict
Indrayan 1st ed 58] the values of two components freely, but the prediction of
Results of a screening test for a disease is third one depends on the fact that the total score of the
three components should be 5.
Disease
Present Absent • In paired t-test df = n-1
Positive TP FP • In unpaired t-test df = {nl + n2) - 1 where nl and n2 are
Results the number of observations in each of the two series.
Negative FN TN
• If we know the number of rows and columns in a table,
the degrees of freedom is calculated by the formula
From the table:
• Total cases = TP + FN df = (r - 1) x (c - 1), where r =No. of rows
• Healthy people - FP + TN c - No. of columns
• Total population = Cases+ Healthy
67. 'C' [Soben Peter 3rd ed 383/ 5th ed 606]
= (TP+FN) + (FP+TN) Standard error of proportion is frequently used to find the
efficacy of a drug, line of treatment or vaccination.
Total cases
Prevalence of disease X 100 Examples: In 1975, the mean DMF score was 5 and in current
Total population
year the mean DMF is find as 3. Whether this reduction of
DMF score is due to better oral hygiene or reduction of sugar
TP + FN intake or administration of caries vaccine or it was only
- - - - - - - - - X 100
(TP + FN ) + (FP + TN) by chance has to be det ermined by calculation of standard
error of proportion.
So prevalence of the disease is inversely proportional to
prevalence of the disease. As the false positive cases are Standard error of proportion is calculated by the formula :
higher in the population, the population will have low SEP = --J p x q / n, where
disease prevalence. p = proportion of occurrence of an event
q = 1- p
62. 'C' (Park 20th ed 128/ Sample biostatistics by Indrayan n = sample size
1st ed 58)
Positive predictive value of a disease depends on 68. 'C' [Park 20th ed 767/ Soben Peter 5th ed 124]
• Sensitivity • A 'group' is an aggregate of people
• Specificity interacting in face to face situation.
• Prevalence of the disease in the population. • Group discussion is considered as very
effective method of health communication
Positive predictive value of a test is directly proportional to as it permits the individuals to learn by
prevalence of disease. freely exchanging their knowledge, ideas
and opinions compared to other methods.
63. 'A' [Soben Peter 4th ed 3 73/ 5th ed 600) Group
Option 'D' pictogram or picture diagram is a popular method • Group discussion is indicated whenever
to impress the frequency of the occurrence of events to discussion long-term compliance is involved.
common man or 'man in the street' (COMEDK-11). Examples Eg: Cessation of smoking, obesity
are attacks, deaths, accidents, number of patients admitted reduction.
or discharged, etc. Pictograms are a popular method of
presenting data to "man in the street" and those who • For effective group discussion the group
cannol undersland complicaled charls. Here small piclures should compromise not less than 6 and
or symbols are used to present the data. not more than 12 members, who are
seated in a circle, so that each is fully
64. 'A' [Mahajan 6th ed 100/ Soben Peter 5th ed 593 ] visible to all the others.
• It included a series of speeches on a
65. 'D' [Park 20th ed 743/ 22nd ed 443) selected topic without any discussion
Vital statistics deals with registration of vital events i.e., among the symposium members.
live births, deaths, foetal deaths, marriages, divorces, etc. Symposium
• The speakers present their view on
66. ' B' [Soben Peter 4th ed 384/ 5th ed 607 last paragraph] the subject briefly and in the end, the
The degree of freedom is defined as the number of audience may rise questions.
independent numbers in a sample.
, COMMUNITY DENTISTRY
,......__,._

V
643

• Consists of a chairman and 4 to 8 speakers, When multiple groups are studies in terms One-way
who, are qualified to talk about a particular of only one factor ANO VA
topic and discuss a given problem. Two-way
When multiple groups are studied in ANOVA or
• The chairman introduces the topic briefly
terms of two fact ors at a time
and invites the panel speakers t o present multifactorial
Panel their views about the topic. After the main
aspects of the subject are explored by The test that is applied to find the t-test or
discussion significance of difference between two students 't'
panel speakers, the audience are invited
to take part. means

• Panel discussion can be an extremely The test used when investigators have
effective method of education, provided it one set of interval date, one set of T-test
is properly planned and guided. nominal data and on ly two groups?
• A workshop includes a series of meeting, The same people are sampled on two
Paired t-test
with emphasis on individual work, within different occasion.
the group, with the help of consultants Studying of two separate groups to test
and resource personnel. if the difference between the two means Unpaired 't'
• The participants are divided into small is real or it can be attributed to sampling
groups and each group choose a chairman variability such as between means of test
Workshop control and experimental groups
and a record. The individuals try to a solve
a given problem and leave a workshop Spearman's
with a plan of action on the problem. The test that compares two ordinals levels
correlation
(ordinal scale data)
• Learning takes place in a friendly, happy coefficient
and democratic atmosphere under the When a test uses nominal data and Fischer exact
guidance of experts in the subject. associated with fewer than 25 subjects? test

69. 'C' [Mahajan 6th ed 168, 169/ Soben Peter 5th ed 607] 72. 'A' [C.R. Kothari Research Methodology 2nd ed 246/
Chi-square t est is a non-parametric test for qualitative data. Mahajan 7th ed 159]
It is a test to fi nd the significance of difference in two or Yates has suggested a correction for continuity in chi-
more than two proportions. The test of association between square value calculated in connection with a (2x2) table,
two events in binomial or multinomial samples is the most particularly when cell freque ncies are small (since no
important applicatio11 of the test in statistical methods. cell frequency should be less than 5 in case through 10
is better as stated earlier) and chi-square (x2 ) is just on
70. 'B' [Mahajan 6th ed 168, 169/ Soben Peter 5th ed 607] the significance level. This correction suggested by Yates
is popularly known as Yate's correction. It involves the
71. 'B' [Mahajan 6th ed 169/ 7th ed 155] reduction of the deviation of observed from expected
Condition Test used frequencies which of course reduces the value of x2 • The rule
for correction is to adjust the observed frequency in each
Test used to examine differences between
cell of a (2x2) table in such a way as to reduce the deviation
frequencies in a sample.
of the observed from the expected frequency for that cell
Chi-Square
Eg.: The percentage of people receiving by 0.5, but this adjustment is made in all the cells without
dental health education versus the disturbing the marginal totals.
percentage of people not receiving dental
health education 73. 'C' [Soben Peter 4th ed 396/ 5th ed 623]

To find the association between two Chi-square 7 4. 'A' [Sob en Peter 4th ed 383/ 5th ed 606]
variables
When a test uses nominal data only and Chi-square 75. 'C' [Soben Peter 4'h ed 368/ 5th ed 590, 619]
has more than 25 subjects associated A sample is a part of a population, called the 'Universe' or
with the study reference (or) parent population. 'Sampling frame' is the
total of the elements of the survey population, redefined
When one study group is sampled on 3 or
according to certain specifications. It consists of sampling
more occasions (or}
ANOVA units, which are individual entitles that form the focus of
When two or more groups are studies in the study. One of the ideal requirement of the sample is
terms of one or two factors representativeness i.e., each and every member of the
The test that compare the variance sample s hould represent the parent population.
between groups with the variation within ANOVA
the group
Dental ;lut.,e i======
A Sampling unit is defined as (NEET-2013} In statistics data represent (AIIMS May-14)
a) A group of sample b) A subset of sample a) Whole sample b) Individual unit in a sample
c) Representing every member of sample c) The unit in which sample is given(kg, mm etc)
d) Entity like kg/litre d) Observations

76. 'C' [Soben Peter 4th ed 394/ 5th ed 618) 77. 'C' [J.V. Dixit 4th ed 100-102)
Data are basic observations or measurements represented
Chi-square A test for possible association between
as static text, numbers, graphs, or maps; by animated or
test two categorical variables.
interactive graphs or maps; or by sound or video. Data is
classified into quantitative and qualitative data. Each is Type of 2x2 Chi-square test which is used
broken down into two sub-types: qualitative data can be McNemar Test
when the data is paired
ordinal or nominal, and numeric data can be discrete (often,
integer) or continuous. Quantitative data is the dat a with A test for the presence of an association
magnitude. Eg: Height and weight, Blood pressure, etc. Fisher's exact between two categorical variables, used
Qualitative data is the data with frequency only but no test when the numbers involved are too small
magnitude. Eg: Religion, sex, blood group, grey or black hair to permit the use of a chi-square test.
Student 't' Used to test the equality of t he means of
Among tests of significance: test two samples
• Parametric tests are used for quantitative data.
• Non-parametric tests are used for qualitative data. Also refer Q.No. 71.
Parametric tests Non-parametric tests 78. 'A' [Soben Peter 3rd ed 29/ 5th ed 600)
• Chi-square test • Chi-square test Ref. Q. No. 11
• Student 't' test • Wil Coxon signed rank test
• Mann-Whitney U-test 79. 'A' [Soben Peter 4th ed 382/ 5th ed 604)
• Z-test
Refer Q. No. 2
• Pearson correlation • Spearman's correlation
coefficient coefficient 80. 'A' [Soben Peter 4th ed 383/ 5th ed 606)
• F-test (ANOVA) • Mc Nemar's test The relation between St andard deviation (S.D.) and Standard
• Fisher's exact probability test error (S. E.) is given by the equation
• Kruskal Wallis test
S.E. = S.D. / ../N where N = Population
• Kolmogrov-Smirnov test
As 'S.E'. and 'N' are inversely proportional, a smaller size will
Option 'C' Mann-Whitney U test is a non-parametric result in high standard error.
alternative to unpaired't' test. 'U' test is used to compare the
medians of independent samples (or to determine whether 81. 'B' [Soben Peter 3rd ed 29/ 5th ed 600/ Ref. Q. No 11)
two independent samples have been drawn from the same
population). 82. 'D' [Park 2Qth ed 127/ Soben Peter 5th ed 131)
Ref. Q. No. 25
Parametric tests Equivalent non-parametric tests
83. 'A' [Soben Peter 4th ed 370/ 5th ed 594 last line]
Paired 't' test Wilcoxon signed rank test A common question while conducting on investigation is
Mann-Whitney test (compare medians about size of the sample. Bigger the sample, higher will be
Unpaired 't' test the precision of the estimates of the sample. For instance,
of two independent samples)
if a field survey is conducted to estimat e the prevalence rate
Kruskal-Wallis test (compare medians of a disease, the sample size is calculated by the formula
ANOVA
of several independent samples) n = 4pq / L2, Where
Pearson correlation n sample size
Spearman's correlation coefficient
co-efficient
p approximate prevalence rate of the disease
Chi-square test for qualitative data is used q 1- p
• To test the association between two events. L Permissible error
Eg.: Cause & effect
• To find the goodness of fit Precision Depends on: (AIIMS May-13)
• To test the difference between two or more proportions. a) sample size b) Alternate hypothesis
c) Study Design d) Null Hypothesis

84. 'C' [Ref. Q.No. 3/ Soben Peter 4th ed 382/ 5th ed 605)
, COMMUNITY DENTISTRY
,......__,,_

V
645

85. 'B' [C.R. Kothari 2"d ed 71] 94. 'C' [Park 19th ed 699/ Soben Peter 5th ed 600 last line]
The ordinal scale places events in order, but there is no
attempt to make intervals of the scale equal in terms of some 95. 'C' [Check Explanation Below]
rule. Rank orders represent ordinal scales and are frequently Tertile divides an ordered distribution into three parts, each
used in research relating to qualitative phenomena. The use containing a third of population. 2nd tertile means 66.66%
of ordinal scale implies a statement of 'great er t han' or 'less and it is not close to median, which is 50%.
than' without mentioning how much greater or less. For Eg:
the difference between ranks 1 and 2 may be more or less 96. ' B' [Soben Peter 4th ed 60/ 5th ed 65]
than the difference between ranks 5 and 6. An important consideration in case-control study is to ensure
comparability between cases and controls. This involves
86. ' B' [Soben Peter 4th ed 62,68/ 5th ed 74] "matching", which is defined as "the process by which we
Relative risk or risk ratio is defined as t he ratio between the select cont rols in such a way t hat they are similar to cases
incidence of disease among exposed persons and incidence with regard to certain pertinent selected variables (eg., Age)
among non-exposed. It is given by the form ula: which are known to influence the outcome of disease and
which, if not adequately matched for comparability could
. . k Incidence among exposed
Rea
l ti ve ns = -~.~----~---~ distort or confound the results".
Incidence among non-exposed
Relative risk is direct measure of the strength of association Types of matching:
between suspected cause and effect. A relative risk of 1 a) Group matching or frequency matching:
indicates no association (PGl-11); relative risk of > 1
In this type, the cases are divided in to sub types or
indicates positive association. A relative risk of 2 indicates
groups based on their characteristics, Such as age,
that the incidence rate of disease is 2 times higher in the occupation, social class etc, and then establishing
exposed group as compared wit h the unexposed. In general,
appropriate controls. The frequency distribution of
the relative risk can be exactly determined only from a the matched variable must be similar in study and
cohort study.
comparison groups.
87. ' f([Soben Peter 4th ed 384]
In the question, as the data is measured in terms of quality (i.e b) Pair matching (Individual matching or one-to-one
presence or absence of disease, or positive test or negative matching)
test), chi square test is used. Also, as we are comparing two In this type, for each case, a control is chosen which
proportions, chi-square is the test advised here. can be matched quite closely. For example, if we have
selected a 50 year old mason with a particular disease,
88. 'A' [Check Explanation Below] we will search for 50 year old mason without the disease
The 'P' value is the estimated probability of rejecting the as a control. Thus one can obtain pairs of patients and
null hypot hesis of a study question when that hypothesis is controls of the same sex, age etc.
true. If the computed P-value is less than 0.05 then we can
reject null hypothesis. If the computed P-value is> 0.05, it The disadvantage of matching is the tendency for
means that the estimates of association are not significant overmatching i.e. matching on numerous variables.
and the null hypothesis is not rejected. P value of < 0.001
simply means there is less than 1 in a 1000 chance of the Which of the following is a step in case-control study?
stated result being incorrect and the results are correct in (COMEDK-14)
remaining 999 or 99.9% of population. a) matching b) selection
c) followup d) analysis
89. ' D' [Soben Peter 4th ed 289/ 5th ed 593]
97. 'A' [Sob en Peter 4th ed 382/ 5th ed 604]
90. 'C' [Mahajan 6th ed 118-119/ Dixit 4th ed 100] If the median and mean are known, mode can be calculated
by the formula:
91. ' C' [Park's 20th ed 127/ 22nd 131-32)
Mode = 3 median - 2 mean
92. ' D' [Soben Peter 4th ed 3 73/ 5th ed 600 last line]
Scatter diagram shows the relationship between two For t his question,
variables (PGl-11}. If the dots cluster around a straight
line, it shows a Linear relationship. Eg:- Relationship Mode = 3 x 5 - 2 x 4
between sugar intake and dental prevalence shows a positive = 15 - 8

relationship. = 7

93. ' B' [Soben Peter 3rd ed 37] 98. ' B' [C.R. Kothari 2nd ed 94]
Primary data is the original data that has been compiled
Coefficient Standard Deviation for a specific purpose and has not been collated or merged
X 100 with other. It is collected afresh and for the first time and
of variation Mean
thus happens to be original in characters.
Dental ;lut.,e

Primary data is collected by


a) Observation method
b) Interview method
c) Through questionnaires
d) Through schedules
e) Other methods
• Warranty cards
• Distributor audits
• Pantry audits
• Consumer pane~
• Using mechanical devices
• Through projective techniques
• Depth interviews
• Content analysis

Secondary data is data reused for another source. It is


the data, which have been collected by secondary source,
such as journals, magazines, reports, etc., and which have
already been passed through statistical process. By way of
caution, the researcher before using secondary data must
see that they possess fo llowing characteristics
a) Reliability of data
b) Suitability of data
c) Adequacy of data

99. 'B' [Soben Peter 5th ed 385]


Based on the type and number of answers t he two types
of scales most commonly used are the Likert and Guttman
scales.

Likert scale (Summative or Thurstone scale):


• Commonly used to quantify attitudes and behaviour.
• Respondents are asked to select a response that best
represents the rank or degree of their answer i.e., whether
the respondent st rongly agrees, agrees, disagrees or
st rongly disagrees with a statement. Each response is
assigned a number.

Guttman Scale:- (Cumulative/scalogram):


• Here the survey questions are designed to have binary
answers i.e., Yes or No.
• The respondent's score is the total number of items with
which he agrees or disagrees.
, COMMUNITY DENTISTRY
~

V
647

2. EPIDEMIOLOGY
1. A local restaurant was the focus of a botulism outbreak. 10. The name of the sample group of individuals when
The state epidemiologist most likely used which format followed for successive examination to evaluate the
to determine the cause of the disease? effect of oral preventive programmes is:
a) Restrospective study b) Prospective study a) Panel group b) Cohort group
c) Cross sectional study d) Case-control study c) High risk group d) Prospective group
(MAN-1997) (PGI- 2001)
2. The term prevalence refer to the 11, Repetition of a study in a given group is
a) Cumulative effect of a disease a) Panel study b) Cohort study
b) Number of new cases of a disease c) Group study d) High risk
c) proporlion of populalion affected by a disease (PGI- 2002)
d) Rate of disease 12. Prevalence of a disease is:
(MAN-97 ,98) a) Rate b) Ratio
3. Epidemiology i s primarily concerned with the c) Proportion d) Deviation
a) Identification of the agent causing disease in the host (AIPG-99)
b) Occurrence and distribution of disease in the population 13. Incidence is:
c) Predisposing factors to disease a) That portion of population affected by a disease or other
d) Relationship between organisms and their environment condition at a given time
(MAN- 95) b) Incidence is greater than prevalence
4. Which of the following is the ultimate goal of dental c) No. of new cases of a specific disease within a defined
epidemiology? population over a period of time
a) Control of dental diseases d) None of the above
b) Description of dental diseases (AIPG - 2K)
c) Development of indices for dental diseases 14. The rate of the yearly occurrence of dental Caries attack
d) Identification of dental diseases is referred to as: (or) No. of new caries cases occurring
(MAN- 98) within one year in a patient is referred to as:
5. Vipeholm' study is used in a) Caries mortality
a) Oral cancer b) AIDS b) Caries experience
c) Caries d) Periodontal disease c) Incidence of caries d) Caries prevalence
(MAN- 99) (AIPG -2003)
6. Difference between descriptive or analytical study and 15. What kind of sampling should be done when there is a
experimental study is small group of patients, easily available?
a) Descriptive study has a small sample a) Random sampling
b) Experimental study requires a strong knowledge of b) Multiphase sampling
statistics by the operator c) Multistage sampling d) None of the above
c) Analytical study requires the use of sophisticated (AIPG-96)
instruments 16. Occurrence of malocclusion in given population is best
d) All variables are under the control and can be modified described as:
by the operator a) Prevalence rate b) Incidence rate
(MAN- 99) c) Morbidity d) Probability
7. In the etiology of dental caries one of the following is (AIIMS MAY 2012, AIPG-14)
not an environmental factor 17. A disease seen in excess of normal in a community
a) Saliva b) Micro organisms results in:
c) Refined carbohydrates d) Structure of the tooth a) Epidemic b) Endemic
(MAN -2002) c) Pandemic d) Endozootic
8. The type of study used in the 10 year Fluoride tablets (PGI- 95, PGI - 05)
study can be also be described as a (an) 18. Severity of a diseases is measured by:
a) Case control study b) Cross section study a) Case fatality rate b) Proportional mortality rate
c) Count study d) Experimental study c) Relative risk d) Attributable risk
(MAN- 97) (KAR- 2002, PGI JUNE-11)
9. True about prevalence & incidence is: 19. Which of the following is not a part of epidemiological
a) Both are rate b) Both are not rate triad:
c) Incidence is rate & prevalence is not a) Agent b) Host
d) Prevalence is rate & incidence is not c) Disease d) Environment
(NEET-2013) (AP- 04, PGI- 03)

1) C 2) C 3) B 4) A 5) C 6) D 7) D 8) D 9) C 10) A 11) A 12) C 13) C


14) C 15) A 16) A 17) A 18) A 19) C
Dental ;lut.,e i======
20. Residents of three villages with three different types 28. The study of disease frequency & distribution is
of water supply were asked to participate in a study to a) Standard deviation b) Variance
identify cholera carriers. Because several cholera deaths c) Epidemiology d) Survey
had occurred in the recent past, virtually everyone (AP- 2003}
present at the time submitted to examination. The 29. All of the following are advantages of case control
proportion of residents in each village who were carriers studies except:
was computed and compared. This study is a: a) Useful in rare diseases
a) Cross-sectional study b) Case-control study b) Relative risk can be calculated
c) Concurrent cohort study d} Non-Concurrent c) Odds ratio can be calculated
(AIIMS- 2003) d) Cost-effective and inexpensive
21. In a case control study, the cases were found to be (AIPG- 2002}
consuming zarda pan more frequently than controls and 30. To achieve caries reduction in adults the diet modification
the difference was statistically significant. It indicates: should be:
a) Zardapan is the cause of the disease a) Rich in calcium and vitamins
b) The incidence of the disease was more in those consuming b) Restrict sugars to 5 times a day
zardapan c) Restrict sugars to 6 times a day
c) The disease can be cured if zardapan is stopped d) Restrict in between meals sugar intake
d) An association exists between zardapan and disease (PGI- 2001}
(AIPG- 03} 31. The cumulative effect of disease past and present upto
22. The case control studies following the distribution of the time of examination is termed
smokers and non-smokers in patients of oral carcinoma a) Incidence b) Prevalence
and control. The Odds ratio {OR) of getting oral cancer in c) Occurrence d) Experience
smokers to that of non smokers is: {KAR-99}
32. Sensitivity is usually expressed in:
a) Units b) Alphabets
c) Ascending order d) Percentage
(KAR - 2003)
33. Best approach to study a rare disease is:
a) Case control study b) Cross-sectional study
a) 0.44 b) 1.0 c) Cohort study d) Clinical trial
c) 1.5 d) 2.25 (KAR -2002)
(AIIMS- 2001} 34. Which of the following used as a reference for a study
23. The tobacco preparation mainly used to clean the tooth is through a period of time
a) Kh~~ b) Mawa a) Base line data b) Prevalence
c) Masheri d) Dhumti c) Incidence d) Rate
(KCET-2011} (PGI- 2003}
24. Cross-sectional studies are carried on 35. The father of epidemiology is
a) Same individuals but of same age a) Hippocrates b) Alfred Fones
b) Different individuals but of same age groups c) Cholera d) John Snow
c) Same individuals at different ages (KAR-2013)
d) Different individuals of different age groups 36. In which of the following infectious disease, the
(AIIMS- 2K} prevalence is most likely to exceed incidence?
25. Berkesonian bias in a case control study is a bias due to: a) Measles b) Influenza
a) Presence of confounding factors c) Leprosy d) Rubella
b) Different admission rates for different diseases
c) Bias introduced by investigator 3 7. A community suffering from dry mouth condition is
d) Patient can not recall or gives false information Likely to have
(KAR- 2002) a) Increased incidence of dental caries
26. For large country surveys the sampling method of choice is: b) Decreased incidence of dental caries
a) Multiple sampling b) Multiphase sampling c) Increased incidence of crack tooth syndrome
c) Cluster sampling d) Multistage random sampling d) Decreased incidence of crack tooth syndrome
(PGI- 2001} (UPSC-2001}
27. Case control studies are also called: 38. Index ages for the survey are {OR} the age groups
a) Retrospective b) Prospective recommended in the national pathfinder survey are
c) Prevalence d} Incidence a) 5, 15, 25, 35, 65 to 70 years
{PGI- 2001} b) 5, 10,15,30,60 t o 70 years
c) 5, 12, 15, 35 to 44 and 65 to 74 years

20) A 21) D 22) D 23) C 24) D 25) B 26) D 27) A 28) C 29) B 30) D 31) B 32) D
33) A 34) B 35) D 36) C 37) A 38) C
, COMMUNITY DENTISTRY

d) 5 to 10, 15 to 35, 40 to 65 years 48. " Galiveedu", a town in Andhra Prades h has a population
(MAN- 2K, 2001, KAR-04, COMEDK-08) of 12000. In 2003, 2000 residents of the town are
39. The occurrence of dental caries in a population at a given diagnosed with the disease. In 2004, 1000 residents
point of time may be termed as: (OR) A researcher is of the town are discovered at the same disease. The
interested in recording the number of individuals in incidence and prevalence of the disease in 2004 are
a particular geographic region who have common cold a) 1000/12000 and 3000/12000
at some point during the month of February. Which of b) 1000/10000 and 3000/10000
the following measures of morbidity would be most c) 2000/12000 and 1000/12000
appropriate in answering thiis question? d) 1000/10000 and 3000/12000
a) Incidence b) Point prevalence
c) Period prevalence d) None of the above 49. A malarial survey is conducted in 50 villages having
(PGI- 2003, COMEDK-15) a population of one lakh. 500 cases turned out to be
40. An epidemiological investigation undertaken to test the malaria positive. The annual parasite incidence is
hypothesis regarding the causation of a disease is called a) 20% b) 5%
a) Case control study b) Cohort study c) 0.5% d) 0.4%
c) Prospective study d) Descriptive study
(UPSC-2001) 50. The major purpose of randomization in a clinical trial is
41. The number of new cases of a specific disease occurring to:
in a defined population during a specified period of time a) Facilitate double blinding
is known as b) Help ensure the study subjects are representative of
a) Incidence b) Point prevalence general population
c) Distribution d) Prevalence c) Ensure the groups are comparable on baseline
(U PSC-2001) characteristics
42. Screening for the oral cancer comes under which level of d) Reduce selection bias in allocation to treatment
prevention (AIPG-06)
a) Primary level b) Secondary level 51. The purpose of double blinding in clinical trials is to:
c) Tertiary level d) None of the above a) Achieve comparability between study and control
(KAR-98) groups
43. Dentist participating in the delta dental planers paid b) Avoid observes bias
90th percentile whereas non-participating dentists are c) Avoid subject bias
paid d) Avoid observer and subject bias
a) 15th percentile b) 30th percentile (AIPG-06)
c) 100th percentile d) 50th percentile 52. Bias is any systematic error in the determination of
(COMEDK-2013) association between:
44. The analytical study where population is the unit of a) Relative risk and attributable risk
study is b) Incidence and prevalence
a) Cross-sectional b) Ecological c) Exposure and disease
c) Case-control d) Cohort study d) Disease and environmental factors
(AIIMS-2004)
45. To test the association between risk factor and disease, 53. The prevention of emergence or development of risk
which of the following is the weakest study design? factors in countries where they have not appeared is
a) Case control study b) Ecological study what type of prevention:
c) Cohort Study d) Cross- sectional study a) Primary b) Secondary
(AIIMS-2004, COMEDK-08) c) Tertiary d) Primordial
46. Vitamin A prophylaxis is an example of: (COMEDK- 06)
a) Primordial prevention b) Health promotion 54. Emporiatrics is the word used to describe the science of:
c) Specific protection d) Disability limitation a) Epidemics b) Feet
(NEET-2013) c) Health of travellers d) Child health
47. Which of the following is correct? (COMEDK-06)
a) Health promotion and specific protection comes under 55. When a disease spreads from one country to another in a
primary prevention short time it is called:
b) Early diagnosis and prompt treatment comes under a) Endemic b) Pandemic
secondary prevention c) Epidemic d) Sporadic
c) Disability limitation and rehabilitation comes under (COMEDK- 06)
tertiary prevention 56. Primordial prevention, a new concept receiving special
d) All of the above attention in the prevention of:
a) Acute disease b) Chronic disease
c) Epidemics d) Pandemics

39) B 40) A 41) A 42) B 43) D 44) B 45) B 46) C 47) D 48) D 49) B 50) D 51) D
52) C 53) D 54) C 55) B 56) B
Dental ;lut.,e i======
5 7. Endemic Disease means that a disease: 68. The study of the distribution and determinants of health
a) Occurs clearly in excess of normal expectancy and related states and event a specified population and
b) Is constantly present in a given population group the application of this study to the control of health
c) Exhibits seasonal pattern problem is called
d) Is prevalent among animals a) Entomology b) Ecology
(AI-05) c) Epidemiology d) Anthropology
58. If incidence = 50 cases/1000 population/year and mean (COMEDK-10)
duration of disease .. 5 years, then prevalence would be: 69. Double Blind Study, true is ?
a) 10 b) 250 a) The research investigator is blind to the dependent &
c) 1/10 d) 1/250 independent variable.
(AIPG-06) b) The subject in the study do not know the examiner doing
59. For the calculation of positive predictive value of a the st udy
screening test, the denominator is comprised: c) The subject & the examiner do not know the details of
a) True +ves + False -ves the study
b) False +ves + True - ves d) Only the examiner know the group to which subject belongs
c) True +ves + False +ves d) True +ves + True - ves (PGI-08)
(AIIMS-06) 70. The basic tools of measurement in epidemiology are -
60. Prevention of disease has •.•••• levels: a) Incidence, prevalence and index
a) One b) Two b) Morbidity, mortality and disability
c) Three d) Four c) Rates, ratios and proportions
d) Endemic, pandemic and epidemic
61. Regarding case control study all the following are correct. (COMEDK-09)
except: 71. In an epidemiological study, the first case which comes
a) Risk factors can be identified to the attention of the investigator is
b) It measures incidence a) Reference case b) Index case
c) Used in the study of rare diseases c) Primary case d) Secondary case
d) Requires few subjects (AIPG-10)
(AIPG-06, AIIMS-07) 72. The heart of randomized control trial (RCT)
62. Blinding can be done to eliminate which type of bias: a) Protocol b) Follow-up
a) Berksonian b) Recall c) Assessment d) Randomization
c) Confounding d) Interviewers (COMEDK-09, 11)
(COMEDK-06) 73. The study which proceeds from cause to effect -
63. What we will define it " ratio of no. of death under 1 year a) Retrospective b) Case control
of age to total live birth per 1000 live births per year" ? c) Cohort d) Descriptive
a) Infant mortality rate b) Child mortality rate (COMEDK-09)
c) Child morbidity rate d) Life expectancy rat e 7 4. If odd's ration is one then
(AIIMS-06) a) High association b) Low association
64. Randomized controlled trial has: c) False statement d) None
a) Same characteristics of experimental and control group (PGI-2011)
b) Different experimental group 75. The common first approach to test causal hypothesis is
c) Does not eliminate bias d) None of the above a) Descriptive study b) Case-control study
c) Cohort study d) Randomized controlled trial
65. Good clinical practice is observed in all phases of clinical (KCET-2011)
trials, EXCEPT? 76. Rural and Urban difference in prevalence is seen in all of
a) Preclinical b) Phase I the following, except:
c) Phase II d) Phase V a) Lung Cancer b) Tuberculosis
(AIIMS-07) c) Mental illness d) Chronic Bronchitis
66. Not true about crude birth rate: (AIPG-2011)
a) It is a measure of fertility 77. Reliability of a screening test refers to:
b) It is actually a ratio not a rate a) Accurat ely measures what it is supposed to measure
c) It is independent of age of population b) Gives same values even on repeated testing
d) Numerator does not include still births c) The extent to which the observer can go in finding the
(NEET-2013) result
67. The measure of strength of association between risk d) Depends on knowledge of the observer
factor and outcome is (AIPG-2011)
a) Odds ratio b) Attributable risk 78. If the prevalence is low as compared to the incidence of
c) Relative risk d) Poisson's ratio disease, it implies
(COMEDK-10) a) Disease is very fatal and/or easily curable

57) B 58) B 59) C 60) D 61) B 62) D 63) A 64) A 65) A 66) B 67) A 68) C 69) C
70) C 71) B 72) D 73) C 74) D 75) B 76) B 77) B 78) A
, COMMUNITY DENTISTRY
~

V
651

b) Disease non fatal 88. Multiple regression analysis is applied to


c) Assumption is wrongly framed a) Compare medians of two independent samples
d) Nothing can be said, as they are independent b) Compare medians of several independent samples
(AIPG-2011) c) Tests association between two categorical variables
79. Mass chemoprophylaxis in endemic area is recommended d) Tests relation between dependent variable and several
for all of the following, except: independent variables
a) Yaws b) Le prosy (MHCET-15)
c) Trachoma d) Filaria 89. According to Hierarchy of Evidence for Scientific
(AIPG-2011) Literature, Randomized control trials are
80. The frequency of a disease or characteristic expressed a) Level I Evidence b) Level II Evidence
per unit size of the population or group in which it is c) Level III Evidence d) Level IV Evidence
observed is ( KERALA-2015)
a) Rate b) Ratio 90. A tableau was held in a village to educate the villagers
c) Proportion d) Average about early detection of oral cancer. This type of health
(KCET-2011) education is called
81. The concept which views health as 'absence from disease' a) Symposium b) Seminar
is called c) Role play d) Learning by doing
a) Biomedical concept b) Ecological concept (KOMEDK-15)
c) Psychosocial concept d) Holistic concept 91. Which of the following is NOT a primary requirement for
(KCET-2011) conducting screening for disease control?
82. The definition of Epidemiology as 'The study of the a) Prevalence of the disease should be high in the
distribution and determinants of disease frequency in population under study
man was given by b) Should be a favo urable prognosis for early treatment of
a) John M. Last b) Winslow the disease.
c) MacMahon d) Frost c) The government should fund the program
(KCET-2012) d) The disease should be serious
83. Socratic method of communication is used to (COMEDK-15)
a) One way communication 92. The floating tip of the iceberg represents what the
b) Two way communication physician sees in the community as
c) Verbal communication d) Formal communication a) Clinical cases b) Latent cases
(KCET-2012) c) Undiagnosed cases d) Carriers
84. Genetic factors, Age, Gender & Socioeconomic status are ( KERALA-2015)
also known as
a) Risk Indicators b) Risk Determinants
c) Risk Markers d) Risk Factors
(COMEDK-14)
85. The pattern of change of disease of mortality and
morbidity where the pandemics of infection are replaced
by degenerative and manmade disorders as the main
cause of morbidity and mortality is known as?
a) Epidemiological transition
b) Demographic t ransition
c) Paradoxical transition d) Reversal of transition
(AIIMS NOV-14)
86. When we try to find out significance of difference in
height of children between 2 classes in a school, which
test is used?
a) Unpaired test b) ANOVA
c) Chi square d) Coefficient of variance
(AIIMS MAY-14)
87. A study was done where the sale of anti-asthma drug was
recorded and the number of deaths because of asthma
was measured over a period of 15 years (1975-1990).
This is which type of study.
a) Ecological b) Case reference
c) Experimental d) Psephology
(AIIMS NOV-14)

79) B 80) A 81) A 82) C 83) B 84) B 85) A 86) A 87) A 88) D 89) B 90) C 91) C
92) A
Dental ;lut.,e

2. EPIDEMIOLOGY - ANSWERS

1. 'C' [Soben Peter 4th ed 58, 59, 64)


Cross-sectional studies are, in fact, surveys that take place over a Limited period. They are normally concerned with detecting
disease, estimating its prevalence in different populations and with investigating the effect of the presence of different
determinants on disease prevalence. Case control study begins with identification of subjects with a specific disease and
studied for risk factors associated with that disease.

In Cohort study, a group of population who has been exposed to a risk factor is identified, followed over an extended period
of time and compared with a group not exposed to a risk factor.

CHARACTERISTIC CROSS-SECTIONAL STUDIES CASE-CONTROL STUDIES COHORT STUDIES


Time 0 ne time point Retrospective Prospective
• Longitudinal study
• Forward looking study
Other names Prevalence study Case reference study
• Follow up study
• Incidence study
Incidence No No Allows the study of incidence
Prevalence Allows the study of prevalence No No
Casuality No Yes Yes
Role of disease Measures disease Begin with disease End with disease
Association of risk factor and Many risk factors for a single Single risk factor affecting many
Assesses
disease disease diseases
Provides direct estimate of relative
Data analysis Chi-sguare to assess association Odds ratio to estimate risk
risk (COM EDK-2013)
• Quick and inexpensive
• Incidence can be calculated
• Used to calculate prevalence • Useful to study rare diseases
Advantages • Provides direct estimate of
• Faster • Require few subjects
relative risk
• Easy to conduct
• Recall bias and selection bias • Expensive
• Unusable for acute diseases are present • Time consuming
Disadvantages
• Over represent • Miss the undiagnosed or • Involves large number of
asymptomatic case subjects

2. 'C' [Soben Peter 4th ed 49/ Park 22nd ed 58) A case series describes the characteristics of a number
of patients with a given disease. Case series report is an
3. 'B' [Soben Peter 4th ed 49/ 5th ed 47) objective report of clinical characteristic or outcome
from a group of clinical subjects. Biased selection or
4. 'A' [Soben Peter 4th ed 44/ 5th ed 48) unrepresentativeness of study subjects and Lack of control
group are basic problems in generalizing the results obtained
5. 'C' [Soben Peter 4th ed 94/ 5th ed 268)
An ecological study is a study in which data of average
6. 'D' [Park 20th ed 59] exposure and outcome for a population are used to compare
One of the important distinctions between descriptive and with similar data for other populations in order to Look for
experimental research is that in descriptive research, no associations between exposure and the outcome.
manipulation of naturally occurring phenomena occurs,
while in experimental research, manipulation and control Classification of epidemiology
become more import measures. A) DESCRIPTIVE
B) ANALYTICAL - 4 types
A case report is most basic type of descriptive study of
• Ecological study - Populatio n is unit of study.
individuals, consisting of a careful, detailed report of the
profile of a single patient by one or more clinicians. • Cross sectional - Individual is unit of study.
• Case-control - Individual is unit of study.
• Cohort study - Individual is unit of study.
, COMMUNITY DENTISTRY
~

V
653

C) EXPERIMENTAL Total number of cases (old & new)


• Randomised controlled trials (RCT) - patient is unit of a disease at a given point of time
Prevalence =
Estimated total population at the X 100
of study
• Field t rials - Healthy people are unit of study same point in time
• Community trials - Healthy people are unit of study Incidence indicates the number of new cases of a specific
disease occurring in a particular period of time. It is a rate.
7. 'D' [Soben Peter 4th ed 87, 88/ 5th ed 260)
Caries is a multifactoria l disease in which there is interaction Number of new cases of a specific
between three principle factors (keys caries triology). Incidence d_is_e=as,...e_a_t_a--=,g1_·v-=e_n_ti_·m_e....,,..p,...e_rio_d__
= __ x 1000
• A susceptible tissue. The population at risk
• Microflora with a cariogenic potential.
Note: for this question COMEDK has accepted even ratio as
• A suitable host tissue. correct answer
Later Newbrun added a fo urth factor, time, to Keys triad 13. 'C' [Soben Peter 4th ed 48/ 5th ed 52)
and this concept is known caries tetrad.
14. 'C' [Soben Peter 4th ed 48/ 5th ed 52)
From the question, even-though we can make out that
• Caries incidence is the number of new teet h affected by
Saliva, microorganisms and refined carbohydrates are part of
environmental factors that play a role in dental caries, truly caries in a defined population during a specific period of
time
speaking, none of the options are environmental factors. All
the four options are host fact ors. • Caries experience includes the total number of carious
lesions occuring within a specified time and it is
The environmental factors of dental caries include: cumulative in nature.
• Geographic variations (latitude, distance from seacoast, • Caries prevalence is all old and new cases at a given
sunshine, temperature, relative humidity, rainfall, point in time or over a period of time.
fluoride, total water hardness, trace elements, soil)
• Urbanization 15. 'A' [Soben Peter 4th ed 369/ 5th ed 592)
• Nut rition
16. 'A' [Soben Peter 4th ed 160/ 5th ed 350)
• Social factors
Cross-sectional or prevalence studies are more useful for
chronic diseases such as malocclusion. The use of incidence
For more details, Refer Hiremath 1st ed Pg 121-24.
is usually rest ricted to acute conditions.
8. 'D' [Soben Peter 4th ed 71-74)
17. 'A' [Soben Peter 4th ed 576/ 5th ed 701 Point 46)
9. 'C' [Soben Peter 4th ed 47, 49) • The occurrence in a community or
region of cases of an illness, specific
10. 'A' [Soben Peter 4th ed 64, 65/ 5th ed 71) health-related behavior, or other
Normally cohort analysis compares results from more than health-related events clearly in excess
one cohort against the response of another cohort group. of normal ex~ectancl£; the word also is
Cohort is defined as a group of people who share a common Epidemic used to describe outbreaks of disease
characteristic or experience within a defined time period in animals or plants
(Eg: Age, occupation, pregnancy etc). Thus a group of • It refers to the number of reported
people had born on the same day or in the same period of cases being more than expected for
time (usually a year) form a birth cohort. In a cohort study that time and for that population.
design, the individuals examined over time may not be Eg: Influenza
the same but they should be representative of a particular Persistent finding of the disease in the
group (or cohort) of individuals who have shared a common Endemic populalion.
experience. Eg: Flourosis, Typhoid
Spreading of disease from one country
Panel studies "involves the collection of data on several to another in short time.
Pandemic
occasions over time from the same sample of respondent s. Eg: Influenza, Cholera, Plague,
Panel study uses the same group of people at multiple points Environmental conjunctivitis.
in time. Type of epidemic disease which spread
by person t o person contact and
11. 'K [Ref. Q.No. 10) Prosodemic
evolve more slowly.
Eg: Cerebrospinal fever
12. 'C' [Soben Peter 4th ed 49) Sporadic Scattered incidence of disease.
Prevalence is the proportion of a given population affected Endemic disease occurring in animals.
Endozootic
by a condition at a given point of time.
Epizootic Epidemic disease occurring in animals.
Dental ;lut.,e

Disease spreads from vertebrate = 30 X 30 = .2_ = 2 25


Anthrozoonoses 20 X 20 4 •
animals to man.
Disease spreads from man to Odds ratio is used for case-control studies. On the
Zoo anthroposes
vertebrate animal. other hand relative risk is used for Cohort studies.
Spreading of disease either from An odds of ratio of 2.25 means that the risk of oral cancer
Amphixenosis man to vertebrate animals or from is 2.25 times higher in people who smoke than who do not
vertebrate animals to man. smoke.
Exotic Disease imported into the country.
23. ' C' (Soben Peter 4'h ed 139/ 5th ed 324)
18. 'A' [Park 20th ed 54/ 5th ed 55) • Bidi
Case fatality rates (type of moratlity rate) are usually
• Cigarettes
indicated in cases of acute infectious diseases (Eg: Food
poisoning, Cholera, measles). • Chutta / cigar / cheroor
Various smoking • Chillum
It represents the killing power of the disease. It is simply habits in India • Dumti
the ratio of deaths to cases.
• Gudakhu
Case fatality rate is calculated by t he formula: • Hookah
• Hookli
Total number of deaths due to a
particular disease • Paan
X 100 • Pan masala
Total number of cases due to to
the same disease • Gutka
• Zarda
19. ' C' [Soben Peter 4th ed 4/ 5th ed 29) Various forms of
• Snuff
The occurrence and manifestations of any disease, whether smokeless tobacco
• Khaini
communicable or non-communicable are determined by
the interactions between the agent, the host and the • Mainpuri tobacco
environment, which together constitute the epidemiological • Mawa
triad. • Mishri / masheri

20. 'A' [Soben Peter 4th ed 56, 57) SMOKING HABITS:


Cross-sectional study measures the disease and assesses the a) Chillum is a long conical clay pipe filled with coarsely
association of risk fact or and disease. In sudden outbreaks of cut tobacco pieces and a glowing charcoal is kept on top
disease, a cross-sectional study involving the measurement
of the tobacco.
of several exposures is the first step to investigate the cause.
In this study, the measurements of exposure and effect are b) Chutta is made up of air cured, fermented tobacco
made at the same time. wrapped in a dried tobacco leaf.
c) Cigarettes:- About lgm of cured tobacco is covered with
21. ' D' [Check Explanation Below] paper. The tobacco is generally treated with a variety
Case control or retrospective studies messures the prevalence of sugars, flavouring and aromatic ingredients. They are
of the disease and also assesses the association of risk factor estimated to contain 1-1.4mg of nicotine and 19-27 mg
and disease. Option (B) incidence of a disease is measured of tar. Only about 510/o are filter tipped and filter length
by cohort studies. averages 12mm. The filters of Indian made cigarettes
comparatively trap less nicotine.
22. ' D' (Soben Peter 4th ed 63/ 5th ed 67-68) d) Dhumti: - Rolled leaf tobacco is used inside a leaf of
"Odds ratio" is the indirect method of estimating relative jack fruit free or leaf of banana plant. Us11d for r,1v11rs11
risk. smoking especially among women.
e) Gudakhu: - Paste of powdered tobacco molasses and
other ingredient primarily used to clean the tooth.
Used predominantly by women in Bihar.
f) Hookah:- also called water pipe or hubble-bubble and is
used in places with a strong mogul cultural influence.
Hookah is purely of Indian origin which corresponds with
. ax d ad the introduction of tobacco in India. The tobacco smoke
Odd s ratio=--= is drawn through the water in the base of the hookah
bX C be
which cools and filters the smoke.
Exposed with disease X Unexposed without disease
g) Hookli: - it is clay pipe of short stem with a mouth piece
Exposed without disease X Unexposed with disease
and bowl. Used in Bhavnagar district of Gujarat.
, COMMUNITY DENTISTRY
,......__,._

V
655

SMOKELESS TOBACCO: 1) Selection bias


a) Pan / beeda / t ambula:- contains are areca nut and • Prevalence - Incidence bias.
Lime mainly and may also include catechu, cardamom, • Berkesonian Bias (Admission rate bias)
cinnamon, coconut, cloves, sugar and tobacco Bias in
(thambakoo) wrapped in betel leaf. case 2) Information bias
b) Zarda:- tobacco leaves are boiled along with lime and control • Memory or recall bias
spices until evaporation. The residual tobacco is dried studies • Telescopic bias
• Interviewer's bias
and coloured with dyes. It is chewed
c) Gutka:- preparation of crushed betel nut, tobacco and 3) Bias due to confounding
sweet or savory flavourings 1) Due to subject variation
d) Pan Masala:- mixture of betel leaf with lime, areca nut, 2) Due to observer bias
Bias in
clove, cardamom, mint, to bacco essence in form of 3) Investigator bias
RCT
granules.
In RCT, these biases can be reduced by binding
e) Snuff:- cont ained air cured and fire-cured tobacco leaves
and is used orally or nasally. Bajjar is a dry snuff used 1) Hawthorne bias
Others
by about 14% of the women in Gujarat. A twig is dipped 2) Pygmalion effect
into snuff and applied over the tooth and gingiva.
1) Selection Bias:
f) Khaini:- it is mixture of tobacco, lime occasionally used It is due selection of cases and controls that may not
with arecanut. The ingredients are vigorously mixed with be representative of cases and controls in the general
t he thumb to make the mixture alkaline and is placed in population. The selection bias can be best controlled by its
the premolar region of the mandibular groove. This fo rm prevention.
of smokeless tobacco is widespread in use in Maharashtra
and several states of North India. A) Prevalence - Incidence studies
g) Mainipuri tobacco:- used in villages of Uttar Pradesh. A • This is due to selection of only survivals among the
high prevalence of oral cancer and leukoplakia is found prevalent cases.
in uses of mainip,uri tobacco. • If the exposure occurred years before, mild cases
i) Mawa:- contains thin shavings of arecanut with the improved, or severe cases that died would have been
missed and not counted.
addition of some tobacco and slake in a cellophane
cover. • This bias is not often a problem in cohort studies and
experiments, but is quite common in case-control
j) Mishri / Masheri :- contains roasted tobacco with or studies.
without catechu. Used primarily to clean teeth.
Note : The form of smoked tobacco used t o clean teeth B) Berkesonian bias:
is Gudakhu. • Berkson's bias occurs when the investigator uses only
hospital records to estimate the population prevalence.
24. ' D' [Park 20th ed 66/ 5th ed 60, 69] • The bias arises because of the different rates of
In cross-sectional study, researchers study individuals of admission to hospitals for people with different diseases
different ages at same time i.e, different age groups at (i.e., hospital cases and cont rols)
same time (AIIMS May-13). In this study, the investigator • Many case-control studies collect cases from hospitals
studies a group of people from atleast two different age and identify controls from among patients in hospitals
groups to determine the possible differences between the admitted for unrelated events.
groups on some measure such as food preferences, amount • These controls may not be representive of general
of physical activity etc or other variable. In longitudinal population.
studies, same individual is studied at different ages. • Different rates of admission will be reflected in biased
estimates of the relative risks.
25. ' B' [Park 20th ed 6 6/ Soben Peter 5th ed 68]
Bias is any systematic error in the determination of Selection bias in cohort study arises because the originally
the association between the exposure and disease. The selected members of cohort may refuse to participate (non-
possibility of bias must be considered when evaluating a consent bias) or missing of study records in non-concurrent
possible cause and effect relationship. cohort study or some individuals are missing or incomplete.
One of t he major problems in cohort st udy is to accomplish
1) Section bias the successful follow up of all members of the cohort and
Bias in may lead to fo llow up or drop-out bias. Follow-up bias can
2) Information bias
cohort
3) Confounding bias be reduced by intensive follow-up of all study participants
studies and by establishing criteria for follow up that will assure
4) Post hoc bias
that all members of the cohort have an equal opportunity
for being diagnosed as having the outcome variable.
Dental ;lut.,e i======
2) Information Bias: satisfaction with their respective physicians by using
questions like "you don't like your doctor, do you?"
A) Memory or recall bias:
When cases and controls are asked questions about their 6) Pygmalion Effect
past history, it may be more likely for the cases to recall
the existence of certain events or factors than t he controls. • It is a type of experimental expectancy.
For example, those who have had a myocardial infarction • Researcher's beliefs affect the outcome, which can be
might be more likely 1to remember and recall certain habits avoided by "double-blind design".
or events than those who have not. Thus cases may have a
different recall of past events than controls. An experimental studies "bias" may arise from three sources.
a) Bias from patient/ subject variation in which the
B) Telescopic bias: participants may subjectively feel better or report
If a question refers to recent past (say last month), episodes improvement if they knew t hey were receiving a new
that occurred Longer ago may also be reported. form of treatment.
b) Observer Bias/ Investigator bias in which the
C) Interviewers bias / exposure suspicion bias: investigator measuring the outcome of a therapeutic
This bias occurs when the interviewer know the hypothesis trial may be influenced if he knows beforehand the
and also knows who the cases are. This prior information particular procedure or therapy to which the patient has
may lead him to question the cases more thoroughly than been subjected.
controls regarding a positive history of the suspected causal
factor. This type of bias can be eliminated by double binding c) Evaluation bias in which the investigator may be
(COMEDK - 06) subconsciously give a favourable report of the outcome
of the trial. In order to reduce these problems, a
3) Confounding Bias: technique known as 'blinding' is adopted, which will
ensure that the outcome is assessed objectively.
• A confounding factor is defined as "One which is
associated, both with exposure and disease, and is
Blinding can be done in 3 ways.
distributed unequally in study and control groups. More
specifically a "confounding factor' is one that, although a) Single blind trial: The trial is so planned that the
associated with "exposure" under investigation, is participants are not aware whether he belongs to 'st udy'
itself, independently of such association, a "risk factor" group or 'control' group. For example, for study group a
for the disease. drug is given and for contro[ group a placebo is given
with same colour and size of the original drug.
• For example, age could be confounding variable. When
we are investigating the relationship between tobacco- b) Double-blind trial: The trial is so planned that neither
chewing and oral cancer since this disease becomes the doctor/investigator nor the participants/subjects
increasingly common with increasing age. know which group of patients are given drugs and which
• This "confounding" effect of age can be neutralized by group are given placebo. These t rials are very useful in
matching so that the groups have an equal proportion of comparing two hypnotics or analgesics where subjective
each age group. information is required.
• In other words, 'matching' protects against an c) Triple-blind trial: In this trial, the participant, the
unexpected strong association between the matching investigator and the person analyzing data are all
factors (Eg: Age) and the disease (Eg: Oral cancer). "blind".

4) Post hoc bias of cohort studies: Double blinding is most frequently used method when a
• Post hoc also manifests itself as a bias towards jumping blind trial is conducted. When a outcome such as death is
to conclusions based upon coincidences. Superstition being measured, blinding is not so essential.
and magical thinking include, for instance, when a sick
person is treated by a witch doctor, or a faith healer, and Study design that is most commonly affected by
becomes better afterward, superstitious people conclude
recall bias (AIIMS Nov-14)
that the spell or prayer was effective.
a) Cohort b) Case control
• The testing of hypothesis that the study was not designed c) Cross sectional d) RCT
to test, but that are suggested by the data, has been
referred to as "data dredging". Finding an association by 26. 'D' [Soben Peter 4th ed 370/ 5th ed 593]
data dredging and then using the same data to test its
significance may lead to unwarranted conclusions; this 27. 'A' [Soben Peter 4th ed 58/ 5th ed 63]
has been termed "Post hoc bias".
28. ' C' [Sob en Peter 4th ed 43/ 5th ed 4 7]
5) Hawthorne bias Various definitions of epidemiology are:
• Type of "measurement bias" • The branch of medical science which treats epidemics
• In this type, the information is gathered in a manner (Parkin)
that distorts the information. Eg. measuring patients
, COMMUNITY DENTISTRY
~

V
657

• The science of the mass phenomena of infectious b) Option 'B' Alfred Fones:-
diseases (Frost) • First used the term "dental hygienist".
• The study of disease, any disease as a mass phenomenon • Father of dental hygiene
(Greenwood)
• The study of the distribution and determinants of disease
c) Option 'C' Cholera is considered as Father of public
frequency in man (MacMohan)
health.
• The study of t he distribution and determinants of
d) Option 'D' John Snow is considered as Father of
health-related states or events in specified populations
epidemiology.
and the application of this study to the control of health
problems (John M. Last)
36. 'C' [Soben Peter 4th ed 54]
Leprosy is a "long-lasting" infectious illness in which the
29. 'B' [Soben Peter 4th ed 63-643/ 5th ed 69]
number of population who have the disease (prevalence)
Relative risk is calculated in cohort study but not in case
is likely to exceed the number of new cases in a year
control study.
(incidence).
30. 'D' [Shobha Tandon 2nd ed 243-44] • Prevalence < Incidence for ultra short diseases
The cariogenic potential sucrose containing foods depends Eg: Food Poisoning.
on the following important variables such as • Prevalence = Incidence for acute diseases
Quantity Eg: Influenza
Frequency of sugar ingestion • Prevalence is zero for suicides which have a zero
Time of sugar ingestion duration.
Type of food i.e retentive or non-retentive • Prevalence = Incidence x du ration
• Prevalence > Incidence for chronic diseases
• WHO has recommended a sugar intake of <10% of total Eg: diabetes, leprosy
energy intake for prevention of caries.
• Intake of free sugars should be limited to 15-20 kg/ 37. 'A' [Soben Peter 4th ed 91/ 5th ed 262]
person/year (40-55 g/day) in the presence of fluoride
and <15 kg/ year (<40 g/day) in the absence of fluoride. 38. 'C' [Soben Peter 4th ed 293/ 5th ed 393]
• Frequency of intake of foods that contain sugar should
be limited to a maximum of four times a day. 39. 'B' [Soben Peter 4th ed 51/ 5th ed 53]
• The time of ingestion is also very important as food
40. 'A' [Soben Peter 4th ed 62-63/ 5th ed 63]
eaten at meals produces Less caries than the same eaten
Option 'D' descriptive study aids in "formulation of
in between meals.
hypothesis". Option 'K case control study is common first
• The solid and retentive sugar containing foods are more approach to test causal hypothesis.
cariogenic than sugar containing foods that are Liquid
and non-retentive. 41. 'A' [Soben Peter 4th ed 48/ 5th ed 52]
• Non-retentive (Liquid) sucrose containing foods, if
consumed during meals are least cariogenic. 42. 'B' [Soben Peter 4th ed 98/ 5th ed 342]
• Health education
31. 'B' [Soben Peter 4th ed 49/ 5th ed 53]
• Plaque contra l programme
32. 'D' [Park 22nd ed 131] • Diet counseling
• All forms of fluoride
33. 'A' [Soben Peter 4th ed 64/ 5th ed 69] Primary • Caries activity test
prevention • Pit and fissure sealant tests and
34. 'B' [Soben Peter 4th ed 50/ 5th ed 54] prophylactic odontomy
• Daily tooth brushing and flossing
35. 'D' [Soben Peter 4th ed 43/ 5th ed 47]
a) Option 'A' Hippocrates:- • Mouth guards in contact sports
• Supervised school brushing programs
• First known epidemiologist and considered as Father of
medicine. • Periodic screening for oral diseases
• The first to apply reason to the study of the phenomena • Preventive resin restorations
of disease and epidemics. Secondary • Prophylaxis by the dentist
• He said disease is not sent by evils or demons, but is the prevention • Simple restorative procedures like
result of natural causes. amalgam filling, pulp capping, etc.
• Scaling and curettage
Dental ;lut.,e

• Pulpotomy 3000
12000
• RCT
• Extraction 49. 'B' [Soben Peter 4th ed 47-48/ 5th ed 52-53]
• RPO and FPO
Tertiary Incidence SOO X 1000 = 5%
• Minor tooth movement 100000
prevention
• Implants 50. 'D' (Soben Peter 4th ed 72/ 5th ed 78]
• Deep curettage, root planing The studies included under experimental epidemiology
• Splinting are:
• Periodontal surgery • Randomised controlled trials (RCT)
• Field trials
43. 'D' (Soben Peter 4th ed 425/ 5th ed 506) • Community trials
Delta dental plans almost exclusively accept the UCR
concept. The way in which a dentist is reimbursed depends In modern usage, experimental epidemiology is often
on whether the dentist is participating or non-participating equated with Randomized controlled trials. Randomization
in the plan. The participating dentists will receive 90th is considered the "heart" of a control trial.
percentile of fees as payment in full. Non-participating
dentists are paid at a considerably lower percentile than the Randomization is a statistical procedure by which the
90th, often at the median or 50th percentile. participants are allocated into two groups i.e., study
group and control group, to receive or not to receive an
44. 'B' [Park 20th ed 59/ 5th ed 60] experimental procedure. Randomization is an attempt
In cross-sectional, cohort and case control studies individual to eliminate "bias" and allow for comparability. It also
is the unit of study. ensures that the investigator has no control over allocation
of participants to either study or control group, thus
45. 'B' [Park 20th ed 66/ Soben Peter 5th ed 62, 63] eliminating what is known as "selection bias". This mean,
by random allocation, every individual gets an equal chance
46. 'C' [Park 22nd ed 41] of being allotted into either group or any of the trial groups.
The currently available interventions aimed at specific
protection are: 51. ' D' [Mahajan 6th ed 210/ Soben Peter 5th ed 79]
• Immunization Blinding can be done in 3 ways.
• Use of specific nutrients a) Single blind trial: The trial is so planned that the
• Chemoprophylaxis participants are not aware whether he belongs to 'study'
group or 'control' group. For example, for study group a
• Protection against occupational hazards drug is given and for contro[ group a placebo is given
• Protection against accidents with same colour and size of the original drug.
• Protection from carcinogens
• Avoidance of allergens b) Double-blind trial: The trial is so planned that neither
• Control of specific hazards in the general environment the doctor nor the participants know which group of
e.g., air pollution, noise control. patients are given drugs and which group are given
placebo. These trials are very useful in comparing two
An example for Specific Protection is (COMEDK-2015) hypnotics or analgesics where subjective information is
a) Immunization b) Health education required.
c) Provision of safe drinking water
d) Restoration of decayed tooth c) Triple-blind trial: In this trial, the participant, the
investigator and the person analyzing data are all
47. 'D' [Soben Peter 4th ed 98/ 5th ed 41-43] "blind".

48. 'D' [Soben Peter 4th ed 48-49/ 5th ed 52-55] Double blinding is most frequently used method when a
Because the 2000 people who got the disease in 2003 are no blind trial is conducted. When a outcome such as death is
longer at risk forgetting the illness in 2004. being measured, blinding is not so essential.
Total number of new cases
Incidence 52. 'C' (Sob en Peter 4th ed 5 74/ 5th ed 68]
Population at risk
1000
10000
Total number of cases
Prevalence
Tota l population at risk
, COMMUNITY DENTISTRY
(""'.,,_

V
659

53. 'D' [Park 20th ed 39/ 22nd ed 39] Emporiatrics is the term coined to describe the science of
In modern days, t he concept of prevention has become the health of these t ravellers. In the age of jet travel, the
broad based and includes 4 Levels of prevention. international travellers are subjected to various forms of
stress (crowding, long hours of waiting etc.) or they may
• It is prevention of the emergence or be exposed to disease that are not covered by international
development of risk factors in countries healt h regulations (e.g. Malaria, Giardiasis, dengue et c.)
or population groups in which they have International travellers have a personal responsibility to
not yet appeared. recognize these risks of travel, which can be minimized
• Primordial prevention is receiving special by immunization, good hygiene, and chemoprophylaxis or
attention in the prevention of chronic chemotherapy.
Primordial diseases.
prevention 55. 'B' (Park 20th ed 90,91/ Soben Peter 5th ed 705]
• In primordial prevention, efforts are
directed towards discouraging children
56. 'B' [Refer Q.No. 53/ Park 20th ed 39/ 22nd ed 39]
from adapting harmful Lifestyles.
• The main intervention in primordial 5 7. 'B' (Soben Peter 4th ed 5 7 4/ 5th ed 700]
prevention is through individual and mass Refer Q.No.17
education
• It can be defined as "action taken prior to 58. 'B' (Soben Peter 4th ed 50-51/ 5th ed 54]
the onset of disease, which removed the Prevalence = Incidence x duration
possibility that a disease will ever occur".
Primary prevention includes t he concept of Prevalence = 50 x 5 = 250
"positive health".
• It simplifies intervention in pre- 59. 'C' [Park 20th ed 128, 129/ 22nd ed 130, 131]
Primary pathogenesis phase of a disease or health Positive True positive
prevention problem, for example Low birth weight. X 100
predictive value (True +ve) + ( False +ve)
• The concept of primary prevention is
now applied in prevention of chronic
Negative True negative
diseases such as coronary heart diseases, X 100
hypertension and cancer based on predictive value (True -ve) + (False -ve)
elimination or modification of "risk
factors" of disease. 60. 'D' [Park 20th ed 39/ 22nd ed 39]
• It is defi ned as a action t hat halts the
61. 'B' [Soben Peter 4th ed 58-64/ 5th ed 63-69/ Ref. Q. No 1]
progress of a disease as its incipient stage
Case control study does not measures incidence. Incidence
and prevents complications.
is measured by cohort studies while prevalence is measured
• The specifi c interventions are early by cross-sectional studies.
diagnosis (screening tests) and adequate
Secondary treatment.
62. 'D' [Soben Peter 4th ed 73/ Park 22nd ed 71]
prevention • Secondary prevention is largely the domain
of clinical medicine. It is more expensive Confounding Bias Eliminat ed by matching
and less effective than primary prevention . Eliminated by appropriate
Selection Bias
• The health programmes initiated by selection of cases & controls
governments are usually at the level of Interviewer's Bias Eliminat ed by double blinding
secondary prevention.
When the disease process has advanced 63. 'A' [Mahajan 6th ed 263/ Soben Peter 5th ed 51]
beyond its early stages, it is still possible Infant moratlityrate includes early, late and post neo-natal
to accomplish prevention by tertiary Level deaths i .e, from birth to 1 yr of life.
Tertiary of prevention i.e., intervention in late
prevention stage of disease. For example, treatment Infant mortality rate does not include? (AIIMS -14)
even if undertaken late in the natural a) Still birth b) Early neonatal death
history of disease may prevent sequelae and c) Lat e neonatal death d) Post neonatal death
"DISABILITY LIMITATION".
64. 'A' [Soben Peter 4th ed 72/ 5th ed 78]
Prevention of onset of risk factor is? (PGI Dec-2011) The reference population in randomized controlled trial is
a) Primary prevention b) Secondary prevention that group to which t he result of the experiment is going
c) Tertiary prevention d) Primordial prevention to be generalized. In short, study or experimental group is
a subject of population with the condition or characteristic.
54. 'C' [Park 20th ed 115/ 22nd ed 118] The selection of the study population will be as similar
to the reference population as possible based on suitable
Dental ;lut.,e i======
inclusion and exclusion criteria. Randomization ensures that • In Phase-IV trials, post marketing studies delineate
the study and comparison groups are similar in all respects additional information includi ng the drug's risk, benefits
except for the intervention. and optimal use.

65. 'A' [www.clinicaltrials.Gov] 66. 'B' [Park 22nd ed 450]


i) In simple language, medical research studies involving A ratio is comparison between two numbers with the same
people are called clinical trials. Ex. Approval of drug or unit. For example, 3 oranges to 2 oranges. On the other hand
new diagnostic technique. a rate is an indication of the measurements of different
ii) The aim of trials is to know safety of the drug by finding units per unit. For example, the statement of 3 oranges/
person shows the relationship between the measurements of
out
oranges per person.
• side effects
• How it works better than currently used treatment. For example, Prevalence proportion is defined as the number
• How the new method helps you to feel better. of all cases of a disease in a given population i.e. the
numerator is part of denominator. The incidence rate is
iii) The approval is done in two stages the number of new cases per population in a given time
period. Here the numerat or is number of new cases while
• Preclinical the denominator is the sum of t he person-time of the at risk
• Clinical which includes 4-phases. population. So prevalence is a ratio and incidence is a rate.
iv) In preclinical trials, before testing the drug on humans,
the drug is experimented invitro or on animals. Depending Birth rate is the simplest indicator of fertility and is
on the results obt ained in the preceding experiments, defined as "the number of live births per 1000 estimated
mid-year population in a given year. It is given by the
the concerned authorities develop different dosages and
pharmaceutical formulations, t aking particular account formula:-
of the physico-chemical and metabolic properties of
the molecule and the biomedical characteristics of the Number of live births during the year
targeted therapeutic application. Finally the drug enters Birth rate = X 1000
Estimated mid-year population
the clinical stage.
v) Clinical trials are conduct ed in phases. The trials at each Here, the numerator is not a part of denominator. So birth
phase have a different purpose and help scientists to rate is actually a rate. Also, the birth rate is an unsatisfactory
answer different questions. measure of fertility because the total population is not
exposed to child bearing (usual reproductive age is 15 - 44
• Phase 'O' is a recent designation for exploratory first- or 49 years). Therefore it does not give a true idea of the
in-human trials, also known as human microdosing fertility of a population.
studies and are designed to speed up t he development of
promising drugs. Distinctive features of phase 'O' trials 6 7. 'A' [Soben Peter 4'h ed 63/ 5th ed 67-68]
include the admi nistration of single subtherapeutic Odds ratio is derived from case control studies. It is a
doses of the study drug to a small number of subjects measure of the strength of the association between risk
{10 to 15) to gather preliminary data on the agents factor and outcome.
pharmacodynamics (what the drug does to the body)
and pharmacokinet ics (what the body does to drugs). Option 'B' attributable risk and option 'C' relative risks are
A phase 'O' study gives no data on safety or efficacy, derived from cohort study or incidence study.
being by definition a dose too low to cause ay therapeutic Relative risk is the ratio between incidence among exposed
effect. and incidence among non-exposed.

• In Phase-I trials, researchers t est an experiment al drug RR = Incidence among exposed!


or treatment in a small group of people (20 - 80) for the Incidence among non-exposed
first time to evaluate its safety, determine safe dose and Relative risk is a direct measure of strength of association.
identify side effects. The larger the relative risk, the greater the strengt h of
• In Phase-II trials, the experimental study drug or association . A relative risk of 10 (for cigarette smoking and
treatment is given to a larger group of people (100 - lung cancer) indicates that the incidence of lung cancer in
300) to see if it is effective and to further evaluate its cigarette smokers is 10 times to that of non-smokers.
safety.
Option 'B' attributable risk is expressed as percent. This is
• In Phase-Ill trials, the experimental study drug or given by the formula
treatment is given to large groups of people (1000 -
3000) to confirm its effectiveness, monitor side effects, Incidence among exposed -
compare it with commonly used treatments and collect incidence among non-exposed x
100
information that will allow the drug or treatment to be Incidence among non-exposed
used safely.
, COMMUNITY DENTISTRY
~

V
661

If incidence among exposed is 10, and among non-exposed 3. Social sciences


is 1, then attributab le risk is 4. Principles of administration

10 - 1 5. Preventive dentistry
= - - x 100 = 90%
10
71. 'B' [Park 20'h ed 91/ 22nd ed 92]
Attributable risk indicates t o what extent t he disease under
a) Primary case: refers to fir:st case of a communicable
study can be attributed to exposed.
disease introduced into the population unit being
An attributable risk of 90% indicates that 90% of lung cancer studied.
is due to smoking and other 10% is due to other reasons b) Index case: refers to first case that comes to attention
(like environmental pollution, occupational exposure to of the investigator. Index case may not be the primary
certain chemicals, etc.) case always.
c) Secondary cases are those developing from contact
Relative risk is a better estimate of strength of association with primary case.
than attributable risk. Attributable risk is a good measure of
extent of public health problem caused by exposure. It is a
Note:
useful tool for assessing priorities for health action.
• The gap in time between onset of primary case and the
68. 'C' [Soben Peter 4th ed 43/ 5th ed 47] secondary case is called the Serial interval.
John M. Last in 1988 has defi ned epidemiology as the study • SAR (secondary attack rate) is defined as "the number
of distribution and determinants of health related states or of exposed persons developing the disease within the
events in specified populations, and the application of this range of the incubation period, following exposure to
st udy to the control of health problems." the primary case"
• Generation time IS "The interval of time between
Which of the following is a component of receipt of infection by a host and maximal infectivity of
Epidemiology? (COMEDK-15) that host
a) Host b) Agent • Communicable period is "the time during which an
c) Determinants d) Time infectious agent may be transfered directly or indirectly
from an infect ed person to another person, from an
69. 'C' [Soben Peter 4th ed 73/ 5th ed 79] infected animal to man, or from an infected person to
Ref. Q.No. 9 animal, including arthropods".

70. 'C' [Soben Peter 4th ed 44/ 5th ed 48] 72. ' D' [Park 201h ed 78/ 22nd ed 79]
Tools of measurement in epidemiology: Ref. Q.No. 50
• Rates
73. 'C' [Soben Peter 4th ed 64, 65/ 5th ed 70]
• Ratios Cohort study proceeds from cause to effect (into future) .
• Proportions So it is always known as prospective study or longitudinal
study. As it yields incidence of the disease, it is also called
Principles of epidemiology:(KAR- 2013) as incidence study.
• Exact observation
Case-control study proceeds backwards from effect to cause.
• Correct interpretation So it is also known as retrospective study.
• Rational explanation
• Scientific construction 7 4. 'D' [Check Explanation Below]
The odd ratio shows the strength of association between
Basic measurements in epidemiology: two binary data values. The odds ratio takes the values
between zero to infinity. An odds ratio of 1 indicates that
• Measurement of mortality the condition or event under study is equally likely to occur
• Measurement of morbidity (incidence & prevalence) in both groups.

Epidemiological methods: 75. 'B' [Soben Peter 4th ed 58/ 5th ed 63]
• Descriptive epidemiology
76. 'B' [Soben Peter 4th ed 54/ 5th ed 59]
• Analytical epidemiology It is well established fact that health and disease are not
• Experimental epidemiology equally distributed in urban and rural populations. Diseases
like chronic bronchitis, lung cancer, cardiovascular disease,
Tools of Dental public health: drug addiction, psychological problems and accidents are
1. Epidemiology usually found to be more in the urban areas; whereas skin
and zoonotic infections and soil transmitted helminthes are
2. Biostatistics
Dental ;lut.,e i ==================

fo und more commonly in rural areas. The variations in the 81. 'A' [Soben Peter 4th ed 3/ 5th ed 29]
disease pattern can be attributed to factors like differences CHANGING CONCEPTS OF HEALTH
in social classes, population density, levels of sanitation,
deficiencies in medical care, levels of education and other a) Biomedical concept: This concept is based on the germ
environmental factors which influence the disease. theory of disease and does not take into consideration
the environmental, social and cultural determinants of
77. 'B' [Soben Peter 4th ed 388/ 5th ed 416, 612] health. In this concept, health is viewed as absence
Reliability is synonymous with repeatability or stability. of disease, i.e if a person is free from disease, he is
A measurement that yields consistent results over time is considered healthy.
said to be reliable. When a measurement is prone to random
error, it lacks reliability. b) Ecological concept: This concept views health as
dynamic equilibrium between man & his environment
Reliability of index means the index should measure and disease is due to maladjustment of human organism
consistently at different times and under a variety of to environment.
conditions. The term "reliability" is virtually synonymous
with reproducibility, which means the ability of the same c) Psychosocial concept: This concept reveals that health is
(intra examiner reproducibility) or different examiners (inter not just a biomedical phenomenon but is also influenced
examiner reproducibility) t o interpret and use the index in by social, psychological, cultural, economic and political
the same way. factors. Thus health is considered as both a biological
and social phenomenon.
78. 'A' [Park 20th ed 57-58/ Soben Peter 5th ed 54-55]
The relation between incidence and prevalence is given by d) Holistic concept: It is a synthesis of all the above
the formu la: concepts and recognizes the strength of social, economic,
political and environmental influences on health. The
Prevalence = Incidence X duration of disease emphasis is on the promotion and protection of health.

The above equation shows that the longer the duration of 82. 'C' [Park 19th ed 48/ 22nd ed 50]
the disease, the greater is its prevalence (MCET-14) . On
the other hand, if the disease is acute and of short duration 83. 'B' [Soben Peter 4th ed 213/ 5th ed 121]
either because of rapid recovery or death, the prevalence Types of communication:
rate will be relatively low compared with the incidence rate.
1. One way and two way communication

79. 'B' [Park 20th ed 41, 236, 287, 297/ 22nd ed 41] a) One way communication (Didactic): Flow of
A mass treatment approach is used in control of diseases information is one way. (PGI DEC- 2013)
like yaws, pinta, bejel, trachoma, filaria and malaria. b) Two way communication (Socratic/dilectic)

2. Verbal and non-verbal communication


Condition Agent for mass chemoprophylaxis
Verbal communication is the traditional way by word
Yaws Penicillin of mout h, non-verbal communication involves a whole
Filariasis DEC or diethyl carhamazine range of bodily movements and facial expressions like
Trachoma Tetracycline/ erythromycin smile, raising eye brows, frowning.

3. Formal and informal communication


80. 'A' [Soben Peter 4th ed 44/ 5th ed 48, last paragraph]
Formal communication follows lines of authority, whereas
• Option 'A' Rate is the frequency of a disease or informal communication is conversing with friends or
characteristic expressed per unit size of the population colleagues.
or group in which it is observed. The time at or during
which the cases are observed is a further specification 84. 'B' [Check Explanatio n Be low]
needed for epidemiologic purposes. Risk is the probability that an individual will develop a
• Option 'B' ratio denotes the relation in size between specific disease in a given period. For many diseases, the
two random quantities. In ratio the numerator is not a disease agent is still unidentified, Eg. Cancer, Coronary heart
part of the denominator. disease, mental illness etc. Where the disease agent is not
• Option 'C' proportion is a ratio which expresses firmly established, the etiology is generally discussed in
the relation in magnitude of a part of the whole. In terms of "Risk factors".
a proportion, the numerator is always a part of the
denominat or. It is usually expressed in percentage. Risk factor:
The term risk factor is used by different authors with atleast
2 meanings.
a) An attribute or exposure that is significantly associated
with the development of a disease (Or)
, COMMUNITY DENTISTRY
~

V
663

b) A determinant t hat can be modified by intervention, c) Finally, in the age of degenerative and man-made
thereby reducing the possibility of occurrence of disease diseases, t he malignancies and cardiovascular diseases
or ot her specified out comes. become more prominent and these disorders are joined
by social pathologies as primary causes of death and
Risk factors can be modifiable (smoking Hypertension, disease. Fertility declines and population growth slows
Elevat ed serum cholesterol, Physical activity, obesity etc.) as incomes increase and medical t echnology becomes
or immutable or un modifiable (Age, sex, Race, family more sophisticated.
history, and genetic factors) . The risk factors that cannot be
modifiable are called as Risk Determinants. Risk factors are In general sense, epidemiologic transition theory recognizes
best confirmed by longitudinal studies. the link between life style and changing experience of
mortality and disease.
Risk indicators:
86. 'A' [Check Explanation Below]
Risk indicators are probable or putative risk factors known As the data given in the question is quantitative (i.e.)
to be associated with higher probability of t he occurrence height of children and t he number of groups are 2 classes
of the disease. Risk indicator is the one which has been the answer is unpaired 't' test.
identified in cross-sectional studies but not confirmed with
longitudinal studies. Question Test Used
• Quantitative data
Eg:
• Before and after tests Paired 't' test
• HIV/AIDS - Indicates the person is at risk of developing
• Single group
TB.
• Osteoporosis - I ndicates the person is at risk of having • Quantitative data
Unpaired 't' test
bone fractures. • Two groups
• Infrequent dental visits - The person is at risk of • Qualitative dat a
Chi-square test
developing dental diseases. • Two groups
• One measurement
Risk markers I Risk Predictors: One way ANOVA
• Three grou ps
It is an attribute or exposure that is associated with
• Two measurements
increased probability of disease, but not necessarily a causal Two way/ Multifa ctorial ANOVA
factor. They can be modified t hrough intervention but are • Three groups
not part of causal chain.
87. 'A' [Park 22nd ed 60]
Eg: Previous history of periodontal disease Here, the study was done in a group of population in which
Bleeding on probing. the sale of anti-asthma drug was recorded and correlated
with the number of deat hs among them. As the unit of
Factors associated with increased risk for disease, but study is population and the study was done to find out
do not cause the disease by themselves are called as the correlation between anti-asthma drug and the
a) Risk predictors b) Risk indicators number deaths, t he type of study used here is ecological or
c) Risk det erminants d) Risk fact ors correlational study.

88. ' D' [Check Explanation Below]


High S.mutans in a child suggests (PGI JUNE-2014)
Multiple regression analysis examines the relationship
a) Low caries incidence b) Risk indicator for caries
between a single dependent variable and two or more
c) Risk promot er for caries d) Risk factor for caries
independent variables. This is done by finding a const ant
called 'Regression co-efficient'. Dependent variable is the
85. 'A' [Check Explanation Below]
one whose value is influenced or is to be predicted while
In 1971, Omran described t he underlying reasons for the
independent variable is t he variable which influences the
demographic transition and used the term epidemiologic
values.
transition to explain t he changing causal factors of the
disease in three stages.
Simple regression analysis is confined t o study only two
a) Age of pestilence and famine characterized by high variables at a time, while multiple regression analysis is
prevalence of endemic disease and under nutrition. At used for st udying more t han two varia bles at a time.
t his st age, famin,es and epidemics of infectious diseases
are common. For example, an investigator can use multiple regression
b) Age of Pandemics is characterized by, increased analysis to determine if the exam anxiety among dent al
population due to decline in mortality rates. Changes students can be predicted based on internal marks, lecture
in mortality rates are linked to improved nutrition, attendance, I.Q. score and revision time. (Here the dependent
sanitation and medical care. variable is exam anxiety and the fo ur independent variables
can be internal marks, attendance, IQ score and revision time).
Dental ;lut.,e i======
Multiple regression analysis allows to det ermine how much • There should be an agreed policy on whom to treat as
variation in exam anxiety can be explained by internal patients
marks, attendance, IQ score and revision time 'as a whole', • An appropriate treatment for those detect ed must be
but also the 'relative contribution' of each independent available.
variable in explaining the variance.
• The cost-benefit and cost-effectiveness must be
favourable.

92. 'A' [Park 22nd ed 37]


The concept of the iceberg phenomenon of the disease is
closely related to the spectrum of diseases. The floating tip
of the iceberg represents what the physician sees in the
A systematic review or meta-analysis of all community i.e., clinical cases. The vast submerged position
relevant randomized controlled trials (RCTs), of t he Iceberg represents the hidden mass of the disease, i.e.,
Level I
or evidence based clinical practice guidelines lat ent, in apparent , presymptomatic and undiagnosed cases
based on systematic reviews. and carriers in t he community. The waterline represents the
Level II Atleast one well designed RCT demarcation between apparent and inapparent disease.
Well-designed controlled trials wit hout
Level III Note:- Iceberg of dent al caries concept was given to Pits
randomization
and Long bottom (PGI June-2014)
Level IV Well-desig ned case control or Cohort study.
Systematic reviews of descriptive and
Level V
qualitative studies
Level VI A single descriptive or qualitative study
Opinion of authorities and /or reports of
Level VII
expert committees.

90. 'C' [Soben Peter 5th ed 125]


Tableau includes a group of models or motionless figures
representing a scene from a story or from hist ory. Tableau
is used in Role playing or social drama among t he options
given.

Role playing I sociodrama:


• One of the group approaches in which the audience
should t ake active part by suggesting alternative
solutions and even by taking part in t he drama.
• The size of group s hould be about 25.
• Puppet show used in villages is a type of socio-drama.
• The drama is fo llowed by a discussion of t he problem and
deriving solutions.
• Useful for children's health education.

91. 'C' [Community Oral health by Cynthia M. Pine 2007 ed


139]
Requirements before launching a screening programme:
• The morbidity and mortality must have public health
importance.
• The disease prevalence in the target population must be
sufficiently high.
• Early intervention must be able to improve the prognosis.
• The screening test must be highly sensitive and specific.
It should be acceptable for population with minimal
adverse effects.
• The natural history of the condition should be adequately
underst ood in order to determine the best time to apply
the screening test.
, COMMUNITY DENTISTRY
~

V
665

3. INDICES
1. The Russell periodontal index is calculated by 10. The disclosing agent used in modified Quigley Hein
a) Adding all the tooth scores and dividing by the number plaque index is:
of teet h examined a) Carbol fuchsin b) Basic fuchsin
b) Adding all the tooth scores and multiplying by the c) Sodium Fluorescein d) Two tone dye
number of teeth examined (KAR-2K, 2Kl )
c) Adding all the tooth scores and dividing by 32 11. For determining periodontal disease an epidemiological
d) None of the above methods survey is done by using:
(MAN-97, 94) a) Gingival index (G.I.) b) Plaque index (P.I.)
2. The PHP index is designed to c) Periodontal index (P.I) d) None of the above
a) Evaluale plaque and calculus on specific Loolh surfaces (AIIMS-97)
b) Score plaque on specific tooth surfaces 12. The indicator teeth used in the Ramjford index are:
c) Scores plaque and gingivitis a) 11,16,26,31,36,46 b) 16,21,24,36,41,44
d) Scores plaque, calculus, gingivitis c) 1, 16,21,26,36,46 d) 11,14,21,24,36,46
(MAN-97) (KAR-2K, 99)
3. Which of the following is a disadvantage of the DMF 13. When the black band of a CPITN probe is completely
index visible while in gingival sulcus, it means
a) It is time consuming a) The individual has advanced periodontal disease.
b) It is a sum of dissimilar items b) The individual has pockets between 4 and 5 mm.
c) It is difficult to determine true caries with the criteria c) The individual needs comprehensive periodontal surgery.
provided in the index d) The individual has calculus and needs scaling and oral
d) It measures cumulative experiences and is not necessarily hygiene instructions.
indicative of current caries activity. (KAR-97)
(MAN- 97) 14. After examining a patient for periodontal problem using
4. In gingival index score 2 means: Russets index you have a meanscore of 3. It indicates:
a) Bleeding on slight provocation a) Established periodontal disease, which is reversible.
b) Spontaneous bleeding b) Established periodontal disease that is irreversible.
c) Colour change in gingiva c) Beginning of periodontal disease.
d) No change in gingiva d) Terminal periodontal disease.
(AIPG-2K,AIIMS -94) (KAR-97)
s. The must age specific index is 15. Cumulative index is:
a) DMFT b) DMF a) Russel's periodontal index
c) def d) All of the above b) Ramjford's periodontal index
(MAN- 97 ) c) PMA (Massler and Schlour)
6. Which of the following is false about gingival index? d) Gingival index (Loe and Silness)
a) It can be used for all teeth or selected teeth (AIIMS- 2001)
b) it uses a scoring system of 0- 3 16. DMF(s) is defined as:
c) it requires the use of a periodontal probe 1. Irreversible index
d) it requires the measurement of the pocket depth. 2. Measures cumulative index
(MAN-95) 3. Morbidity index:
7. The OHI-S index developed by Greene and Vermillion is a) 1 only b) 2 only
used to measure c) 3 only d) All of the above
a) Dental caries b) Gingivitis (PGI-2K, MAN-98)
c) Oral debris and calculusd) Periodontal disease 17. PMA index is used to record the status of:
(MAN-95) a) Gingival disease b) Carious teeth
8. The component of D.M.F. index mostly likely to give false c) Root caries d) Periodontal disease
scores (PGI-2002, AIPG-2003)
a) D- Component b) M- Component 18. CPITN:
c) F- Component d) All usually a) Was observed for the first time by ADA in 1992
(MAN 98) b) Scored by a special periodontal probe of length 3 to 5
9. The scoring given in periodontal index for gingivitis is mm backward for easy visibility
a) 1 b) 2 c) Measure 6 representative teeth in young people
c) 4 d) 6 d) Score of '3' means the treatment involves oral hygiene
(AP- 2003) instructions, scaling and curettage
(PGI- 2001)

1) A 2) B 3) D 4) A 5) C 6) D 7) C 8) B 9) B 10) B 11) C 12) B 13) D


14) B 15) C 16) D 17) A 18) C
Dental ;lut.,e i======
19. DMF index was given by: 31. Russets periodontal index scores range from
a) Klien, Palmar, Knutson and Henry in 1938 a) 0-2 b) 0-4
b) Moller in 1966 c) 0-6 d) 0-8
c) Gruebbel in 1994 d) Katz in 1980 (PGI-2003)
(AIPG- 2003, MAN- 2K) 32. Score 0-0.4 in Dean's index indicates
20. CPITN probe is designed by: a) Negative mottling b) Slight mottling
a) WHO b) Nabers c) Moderate mottling d) Severe mottling
c) Marques d) Williams (PGI- 2002)
(PGl-98, AIPG-2001) 33. DMF teeth
21. "Therapeutic" index is: a) 28 b) 32
a) an approximate assessment of the safety of the drug. c) 30 d) 26
b) The dose expected to kill one half of population. (PGI- 2003)
c) The dose produces desired response. 34. The community periodontal index of Treatment needs
d) The dose produces 100% mortality uses which probe
(KAR-2002) a) Russel's probe b) CPITN-E probe
22. In CPITN index zero score means: c) A and B d) None of the above
a) Gingival bleeding b) Grade 1 mobility of teeth (PGI- 97, AIPG-96)
c) Healthy tooth d) Spontaneous bleeding 35. The index used to record the plaque score is
(AIIMS-2K) a) PMA Index b) Silness and Loe Index
23. Simple and effective index of monitoring periodontal c) Green and vermillion Index
status of individual by general practitioner is d) Russell's index
a) PSR b) PI (COMEDK - 03, 05, AIPG-98)
c) CPITN d) PDI 36. In simplified oral hygiene index, how many surfaces of
(KAR-99) the teeth are examined?
24. Periodontal index is also known as: a) 6 b) 12
a) Loe & Sillness index b) Russel index c) 18 d) 24
c) Ramford index d) Shick & ash index (UPSC 2001)
(KAR- 2K; MAN-2K,2001) 3 7. The CPITN probe has a ball end of which diameter &
25. WHO global programme for oral health targets for 2000 marking at what Level:
include; a) 0.5 mm diameter ball and marking at 3,5,8 and 11 mm.
a) <2 DMFT at 12 years b) <3 DMFT at 12 years b) 1.0 mm diameter ball and marking at 3.5, 5.5, 8.5 and
c) <4 DMFT at 12 years d) <5 DMFT at 12 years 11 mm.
(KAR-2002) c) 0.5 mm diameter ball and marking at 3.5, 5.5, 8.5 and
26. Score of 8 on the Russel periodontal index shows: 11 mm.
a) Irreversible terminal disease d) 1.0 mm diameter ball and marking at 3,5,8 and 11 mm.
b) Est ablished destructive periodontal disease (AIPG - 2004)
c) Beginning destructive periodontal disease 38. The 'd' in 'def' stands for
d) Advanced destructive periodontal disease with loss of a) All primary decayed teeth which require silver fillings
masticatory function b) all decayed primary teeth whk h require extraction
(PGI, KAR-2001, MCET-07) c) All decayed teeth which require extraction in a 10 year
27. The most commonly used index for dental caries is old child
a) DMFS b) DMFS & DMFT d) All decayed primary anterior teeth
c) Russell's index d) Caries index (MAN -2 K,97)
(AP- 2003) 39. Community fluorosis index was given by
28. ALL of the following measure the plaque index except: a) Mc kay b) Jackson
a) Ramjford index b) Moller's index c) Dean d) Black G.V.
c) OHI-S index d) Loe and Silness (MAN -2K)
(AIPG 2001,96) 40 Gingival fodex was given by
29. Sulcus bleeding index measures: a) Sillness and glass b) Loe and sillness
a) Gingivitis b) Dental plaque c) Glass and loe d) Loben and sillness
c) Gingival bleeding d) Gingival recession (MAN-2001)
(AIIMS- 99, AIPG-2001) 41. Ramjford index is used on:
30. In DMF index F indicates a tooth with: a) All permanent teeth
a) Zinc oxide eugenol filling b) Selected six permanent teeth
b) Silver amalgam filling c) Selected twelve permanent teeth
c) Caries below an exiting restoration d) None of the above
d) Permanent filling in a deciduous tooth (PGl-98, 2003)
(KAR- 97)

19) A 20) A 21) A 22) C 23) D 24) B 25) B 26) D 27) B 28) B 29) C 30) B 31) D
32) A 33) B 34) B 35) B 36) A 37) C 38) A 39) C 40) B 41) B
, COMMUNITY DENTISTRY
~

V
667

42. Length of C.P.I.T.N. probe is 53. DMFT & DMFS indices represent:
a) 14.0 mm b) 13.0 mm a) Decayed teeth index
c) 11.5 mm d) 12.5 mm b) Missing/surfaces teeth index
(MAN-2002) c) Filled teeth index d) Caries index
43. C.P.I.T.N is used in
a) Biggest population b) Smallest population 54. According to WHO technical series published, CPITN
c) Diagnostic tool d) Screening purposes probe is also known as:
(MAN-2002) a) 611 b) 617
44. Green Vermillion index measures c) 621 d) 625
a) Periodontal disease b) Oral hygiene (KAR- 04)
c) Bone level d) Gingival health 55. Teeth selected for scoring of PHP-M (Personal Hygiene
(MAN-2002, AIIMS-2001, KAR-2K) Performance Modified) are:
45. In DMF index, according to WHO specification teeth a) Six b) Twelve
missing due to any other reason other than caries is c) Eighteen d) Twenty four
designated as (AIIMS-07)
a) D b) M 56. Select the false principal for DMF Score:
c) F d) None of the above a) Last permanent molars are excluded in scoring
(MAN-2002) b) Naturally exfoliated teeth should not be taken into
46. The DMF index is utilized for consideration
a) All deciduous teeth c) Teeth extracted due to orthodonticstreatmentor periodontal
b) All permanent teeth except the third molars disease or impaction should not be considered
c) Cuspids and molars only d) Endodontically treated tooth should be considered
d) Six permanent teeth
(MAN-95) 57. Revised Deans Fluorosis index has:
47. The community periodontal index for treatment needs is a) Five point scale b) Six point scale
recorded for: c) Seven point scale d) Nine point scale
a) Quadrants b) Sextants
c) One arch d) Whole dentition 58. The index age group for deciduous teeth in year is:
(AIIMS MAY 2012) a) 12 b) 5
48. The W.H.O. periodontal probe (CPITN probe) has a color c) 35 - 44 d) 65 - 74
coded black (COMEDK-06)
a) Oto 0.5 mm b) 3 to 3.5 mm 59. DMF score for any one individual can range from:
c) 8.5 to 11.5 mm d) 3 to 5 mm a) 0-28 b) 0-32
(MAN-2K) c) 0-48 d) 0-64
49. CPITN index is used to assess:
a) The degree of periodontal destruction 60. How many scores are used in Dean's fluorosis index?
b) The amount of calculus present a) 4 b) 6
c) The treatment needs c) 8 d) 5
d) The degree of inflammation of the gingival tissues (AIIMS- 06)
(KAR-97) 61. According to WHO 1986, a molar filled with temporary
50. Extent of severity index, the examination restoration is classified as:
a) Involves all teeth b) Only six teeth a) Sound tooth b) Filled but decay
c) Is done with naber's probe c) Filled with no decay d) Decayed tooth
d) None of the above (AIIMS-06)
(AP-99) 62. DMFT index in 1938 was not developed by:
51. A child has the permanent molar and 3 deciduous a) Dean H.T. b) Henry T. Klein
molars filled with amalgam. 2 other deciduous teeth c) Knutson J.W. d) Carole E. Palmer
were cariously decayed, 2 deciduous upper incisors are (KCET-07)
missing. The decayed and filled score of child is: 63. The PHP index describes:
a) 3 b) 5 a) Performance of hygiene by patient after brushing self
c) 7 d) 8 evaluation
(AIPG-2002) b) Measures oral hygiene and gingival bleeding
52. The mean DMFT values for 12 year old school children is c) Score the plaque surface after treatment
2.5. 68% of the population has DMFT values between 2 d) Measures the oral flora and plaque index
and 3. The standard deviation for the population is: (AIPG-07)
a) 0.5 b) 1 64. Overjet and overbite of malocclusion is assessed using
c) 2 d) 3 WHO survey from 1997 with the help of the instrument:
(AIPG-06) a) Williams probe b) CPI probe

42) C 43) D 44) B 45) D 46) B 47) B 48) C 49) C 50) A 51) B 52) A 53) D 54) C
55) A 56) D 57) B 58) B 59) B 60) B 61) D 62) A 63) A 64) B
Dental ;lut.,e i======
c) Measuring scale d) Straight probe 75. In "Thylstrup Fejereskov Index", 'Opaque white lines are
(COMEDK-08) more pronounced and frequently merge to form small
65. Navy Plaque Index was introduced by: cloudy areas scattered over the whole surface area', the
a) Muhlemann b) Ennever score given is -
c) Grossman & Fedi d) None of the above a) 2 b) 4
(KCET-08) c) 6 d) 7
66. Periodontal Screening and Recorder system (PSR) is (KCET-09)
recorded by using periodontal probe which has _ __ 76. A composite index combining indicators representing
a) Graduation - 1, 2, 3, 4, 5, 6 7 longevity, knowledge and income is -
b) 0.5mm ball tip with color coded 3.5 to 5.5mm a) Physical quality of life index
c) Color coded for every 3mm b) Human development index
d) Graduated - 1, 5, 10 & 15mm c) Sullivan's index d) Human poverty index
(KCET-08) (COMEDK-09)
67. The indicators of the periodontal disease status measures 77. The CPITN probe was designed for the detection of -
all of the following by using CPI index in oral health a) Supragingival calculus b) Debris
surveys EXCEPT: c) Subgigival calculus d) Subgigival stains
a) Bleeding b) Bone loss (KCET-09)
c) Calculus d) Periodontal pocket 78. The index in which the tooth is divided into 9 segments is
(COMEDK-08) a) Patient hygiene performance index
68. Intrabony pockets are given a score of in b) Plaque index of silness and Loe
Russell's periodontal index: c) Modified Navy plaque index
a) 3 b) 4 d) Glass criteria in estimating dental plaque
c) 6 d) 8 (AP-10, AIPG-11)
(MCET-07) 79. In the WHO Oral Health Assessment form (1997) the
69. Which index would you use to assess the severity of following index criteria are used to record the dentofacial
periodontitis in epidemiological studies of a large anomalies
population? a) Dental Aesthetic index
a) PMA index b) Gingival index b) Treatment priority index
c) Periodontal index d) Sulcus bleeding index c) Handicapping malocclusion assessment record
(AIPG-09, 11) d) Index of Orthodontic treatment needs
70. Which of the following plaque indices does not use a (KCET-2012)
disclosing agent? 80. The property of an index to measure what is intended to
a) Plaque component of periodontal disease index measure is known as
b) Debris component of simplified Oral Hygiene Index a) Validity b) Reliability
c) Modified Quigley Hein Plaque Index. c) Quantifiability d) Sensitivity
d) All of the above. ( KCET-2012)
(AIPG-2011) 81. PMA index given by Schour and Massler is a method of
71. Fejereskov and Thylstrup index is used for assessing
a) Caries Index in primary teeth a) Periodontitis b) Oral hygiene
b) Index to measure Fluorosis c) Amount of calculus d) Gingivitis
c) Gingival Bleeding Index (AP-2012)
d) Periodontal Index 82. Which of the following periodontal probes has a ball at
(PGI-08) its tip?
7 2. When white opacities are more extensive but do not a) Marquis b) WI-IO
involve more than 5 0% of the surface dean's fluorosis c) Michigan d) UNC-15
index and grade is: (KAR-2013)
a) 0.5 - Questionable b) 1.0 - Very mild 83. CPITN stands for
c) 2.0 - Mild d) 3.0 Moderate a) Community Periodontal Index with Treatment Needs
(IGNOU-10) b) Community Periodontal Index for Treatment Needs
73. The periodontal Index is c) Community Periodontal Index of Treatment Needs
a) reversible b) irreversible d) Community Periodontal Index and Treatment Needs
c) partially reversible d) none of the above (KAR-2013)
(AP-09) 84. Example for symptom index
74. Who introduced patient hygiene performance index (PHP a) OHi Index b) Plaque Index
Index) - c) Bleeding Index d) DMFT
a) Podshadly & Haley b) Green & Vermilion (KAR-2013)
c) Gruebell d) Quigley & Hein 85. Dean's fluorosis index was modified in the year
(KCET-09) a) 1934 b) 1938

65) C 66) B 67) B 68) D 69) C 70) B 71) B 72) C 73) C 74) A 75) A 76) B 77) C
78) C 79) A 80) A 81) D 82) B 83) C 84) C 85) C
, COMMUNITY DENTISTRY
~
669
V
c) 1942 d) 1945
(KAR-2013)
86. 'Care index' with respect to epidemiology of oral disease
is
a) Ratio of missing teeth to total teeth with caries
experience
b) Ratio of filled teeth to teeth present
c) No. of filled teeth per person
d) Ratio of denture users to non-denture users
(AIIMS NOV-13)
87. DMF index does not show?
a) Number of decay tooth b) Number of restoration
c) Severity of decay d) Missing t ooth due to decay
(PGI OEC-13)
88. DMFT score by 2025 will be?
a) 0 b) 1
c) 1.5 d) 2
(PGI JUNE- 2012)
89. In index of treatment needs (IOTN), treatment grade 4
indicates
a) Extreme/ need treatment
b) Moderate / borderline need
c) Severe / need treatment
d) No need for treatment
(APPG-15)
90. Range of OHi is
a) 0-4 b) 1-3
c) 0-6 d) 0-12
(PGI JUNE-13)
91. Modified gingival index with moderate inflammation,
glazing, redness, edema and/or hypertrophy of marginal
or papillary gingiva. Score will be?
a) 1 b) 2
c) 3 d) 4
(PGI JUNE-2012)
92. In gingival bleeding index given by Ainamo and Bay,
positive score is taken at what time after appearance of
bleeding on probing?
a) 10 Sec. b) 20 Sec.
c) 30 Sec. d) 40 Sec
(PGI JUNE-13)

86) A 87) C 88) B 89) C 90) D 91) C 92) A


Dental ;lut.,e

3. INDICES - ANSWERS
1. 'A' [Soben Peter 4th ed 327/ 5th ed 429) Score Criteria
2. 'B' [Soben Peter 3ed ed 135) 0 Normal
PHP index was developed by Podshadley and Haley to Mild inflammation with slight change in color
assess the extent of plaque and debris over a tooth surface. 1
No Bleeding on Probing (NBP).
i) The teeth examined in PH P index are 16, 26, 36, 46, 11 • Moderate inflammation.
and 31. 2
• BOP is seen.
ii) The surfaces examined are:
• Severe inflammation
• Labial surfaces of incisors 3
• Tendency to spontaneous bleeding.
• Buccal surfaces of maxillary molars
• Lingual surfaces of mandibular molars Gingiva Scores Condition
iii) Patient hygiene performance index is used to assess the 0.1 - 1.0 Mild gingivitis
patient before and after oral hygiene instruction.
1.1 - 2.1 Moderate gingivitis
PHP Index: 2.1 - 3.0 Severe gingivitis
• Used to assess the extent of plaque and debris over a
tooth by using disclosing agent. 5. ' C' [Hiremath 1st ed 196)
• It is used t o both document and assist in motivating The def index measures the dental caries in primary dentition
changes in oral health habits. i.e., in patients who are below 6 years. This index was given
by Grubbel in 1944. Caries indices used for permanent
• It is used to score the patient before and after oral
dentition i.e, deft index and deft index are equivalent to
hygiene instruction and at follow up visits.
OMFT and DMFS indices used for permanent dentition.
• It is used to analyze and evaluate the effectiveness of
home care methods. 0 - decayed primary teet h.
• It can be used for individual patient education i.e., as an C - indicated for extraction / extracted due to caries only
educational aid. F - teeth with permanent restoration due to caries.

3. ' D' [Soben Peter 4th ed 316/ 5th ed 417,448) Modifications of def index:
• Used for the children before the age of exfoliation (5-6
4. 'A' [Soben Peter 4th ed 325/ 5th ed 427)
years)
Gingival index was developed by Loe and Silness in 1963,
solely for the purpose of assessing the severity of gingivitis • After 9 years of age, def index is applied only to primary
and its location in four possible areas by examining only molars.
the qualitative changes (severity of the lesion) of the soft • Only d and f components are calculated after 9 years.
tissue.
In mixed dentition, caries indices for the permanent t eeth
Instruments used are mouth mirror and periodontal probe. (DMFT and DMFS) have to be done separately but the scores
The severity of gingivitis is scored on all teeth or on selected are never added.
index teeth. The six index teeth are, 16, 12, 24, 36, 32 and
44. The tissues surrouning each tooth are divided into four 6. ' D' [Soben Peter 4th ed 3 25/ 5th ed 427, 428)
gingival scoring units i.e. Gingival index is also known as LOE and SILNESS index.
Plaque index is known as SILNESS and LOE index.
• Distofacial papilla
7. ' C' [Soben Peter 4th ed 319/ 5th ed 421)
• Mesiofacial papilla
The indices used for assessing oral hygiene and plaque
• Facial margin are:
• The entire lingual gingival margin • OH! & OHIS
• Plaque index
Unlike the facia l surface, the lingual surface is not subdivided
in an effort to minimize examiner variability in scoring, • Turesky - Gilmore - Glickman modification of the Quigley-
since it will most likely be viewed indirectly with a mouth Hein plaque index.
mirror. • PDiindex

The oral hygiene index (OHI) was developed in 1960 by


Green and Vermillion to classify and assess oral hygiene
status. This index was developed to study variations in
, COMMUNITY DENTISTRY
~

V
671

gingival inflammation in relation to the degree of mental Sum of individual scores


retardation in children. It was depicted as sensitive, simple PI score per person
and rapid method for assessing group or individual oral Number of teeth present
hygiene quantitatively.
Group scores Clinical condition
As the OHi index was time consuming and required more 0 - 0.2 Normal healthy tissues
decision making, green and vermillion in 1964 developed
OHI-S index. 0.3-0.9 Simple gingivitis
1 - 1.9 Beginning of destructive periodontitis
OHI-S (Oral hygiene index simplified) has two components, (reversible)
the simplified debris index and the simplified calculus index. 2-4 Established destructive periodontal
It differs from original OHI index in disease (irreversible)
• The number of teeth examined is only six. They are 16,
4-8 Terminal disease (irreversible)
26, 36, 46, 11, 31 (same as PHP index)
• The selection of surfaces to be scored in OHI-S index One of the classification of indices is Reversible (Eg:
are labial surfaces of incisors, buccal surfaces of upper Gingivitis) and irreversible (Eg: Dental caries, Periodontitis,
molars and lingual surfaces of lower molars. etc.). The periodontal index is the only index which is both
reversible initially and irreversible in later stages. Upto
8. ' B' [Soben Peter 4th ed 343-44/ 5th ed 448-49] score of 1.9 is considered as reversible and for score range
The M-component (Missing tooth) gives false scores: of 2 to 8 it is considered as irreversible.
• When teeth are lost for reasons other than caries.
• When teeth are lost due t o orthodontic & periodontic 10. ' B' (Soben Peter 4th ed 322/ 5th ed 424)
reasons.
11. 'C' [Soben Peter 4th ed 329/ 5th ed 428)
9. ' B' [Soben Peter 4th ed 328/ 5th ed 429]
Indices used for assessing gingival and periodontal diseases 12. 'B' [Soben Peter 4th ed 328/ 5th ed 430)
are - Index Teeth examined
• Papillary Marginal attachment (PMA) Index 16,26 - Buccal surface
• OHi - S
• Gingival Index 36,46 - Lingual surface
• PHP Index 11,31 - Labial surface
• Russel's periodontal index
• Periodontal disease index (PDI) • Plaque Index
16, 36, 12,32, 24, 44
• Community periodontal index of treatment needs (CPITN) • Gingival Index
• Community periodontal index (CPI) • Navy plaque index
16, 36, 21, 41, 24, 44
• PDi index
Russel's periodontal index
i) For adults of age more than 20
Periodontal index is the only index that is used in ten
years - 17, 16, 27, 26, 37, 36,
standard health surveys. Eg.: National health survey, which 47, 46, 11, 31
is the largest health survey in United States. All t eeth are • CPI Index
(all z nd molars are included)
examined. Inst ruments used are mouth mirror and plain • CPITN Index ii) For people upto 19 years - 16,
probe. Russel choose the scoring values of 0, 1, 2, 4, 6, 8.
26, 36, 46, 11, 31 (same as
Russel's rule states that 'when in doubt assign the lesser
OHI-S & PHP)
score'.

In CPITN number of teeth examined in a 15-19 year


Score Criteria to give score
person? (PGI June-13)
0 No inflammation a)6 b)8 c)lO d)1Z
1 Mild gingivitis
13. ' D' [Soben Peter 4th ed 336-38/ 5th ed 438-39)
2 Gingivitis
CPITN probe was first described by WHO (TRS 621-1978). The
Used when radiographs are available. There is weight of CPITN probe is Sgms. (KCET-11). This probe was
4
early notch like resorption of alveolar crest. designed for two purposes namely measurement of pocket
6 Gingivitis with pocket formation. depth, and detection of subgi ngival calculus. A tooth is
probed to determine pocket depth and to detect subgingival
Advanced bone loss with loss of masticatory calculus and bleeding response. The probing force can be
8
function divided into a "Working Component" to determine pocket
depth and a "sensing component" to detect subgingival
Calculation of Index: calculus.
The PI score is obtained by adding all of the individual scores
and dividing by the number of teeth present or examined.
Dental ;lut.,e i======
CPITN probe contains markings at 3. 5mm, 5.5mm, 8.5mm, b) Depending on the extent to which areas of oral cavity
and 11.5mm with a ball tip of 0.5mm diameter. The CPITN-C are measured.
or clinical probe contains black markings between 3.5mm
and 5.5mm and between 8.5mm and 11.5mm. • Full mouth indices: Eg: Russell periodontal index
• Simplified indices: Eg: OHI-S
In young people up t o 19 years, the six index teeth selected
are
16 11 26 c) Based on categories which the index measure:
46 31 36
• Disease index: Eg: D component of DMFT index.
In adults, the 10 specified index teeth examined are • Treatment index: Eg: ' F' Component of DMFT index.
17 16 11 26 27
47 46 31 36 37 • Symptom index: Eg: GingivaljSulcular bleeding index

Code - O Healthy tooth d) Based on special categories:


Code - 1 Bleeding on probing • Simple index: An index that measures the presence or
Plaque retentive factors are either felt or absence of disease
Code - 2
seen Eg: Russel periodontal index
Pathological pocket of 4-5 mm, black band of • Cumulative index: Measures all the evidence of a
Code - 3
the probe is partially visible. condition, past and present.
Pathological pocket of 6mm or more in depth. Eg: DM FT index for dental caries.
Code - 4
Black band of t he probe is not visible.
When only one tooth or no teeth are present Morbidity is defined as "any subjective or objective
Code - X departure, from a st at e of physiological well-being.
in a sextant.

17. 'A' [Soben Peter 4th ed 324/ 5th ed 426]


Classification of Rx needs The basic philosophy used in the development of the PMA
TN - 0 Code O No treatment is needed. index was very similar to the DMF index, i.e., the number
TN - 1 Code 1 Improving personal oral hygiene. of gingival units affected were counted rather than the
severity of the inflammation . The gingival unit is divided
Professional cleaning of teeth and into three component parts.
Code 2 removal of plaque retentive factors
TN - 2 along with oral hygiene instruction.
The gingival portion between
Scaling and root planning along Papillary gingiva
Code 3 the teeth
with oral hygiene instructions.
The marginal collar surrounding
Complex treatment like deep scaling, Marginal gingiva
the teeth
TN - 3 Code 4 root planning and more complex
surgical procedures. The gingival portion overlying
Attached gingiva
the bony alveolar process
14. 'B' [Soben Peter 4th ed 328/ 5th ed 430]
18. ' C' [Soben Peter 4th ed 334/ 5th ed 439]
15. 'C' [Soben Peter 4th ed 324/ 5th ed 426]
Cumulative indices measure all the evidence of past and 19. 'A' [Soben Peter 4th ed 343/ 5th ed 445]
present condition. Both DMF and PMA (Papillary-Marginal- The Decayed - Missing - Filled teeth index was developed
Attachment index) indices are examples of cumulative by Henry T. Klien, Carrole E. Palmer and Knutson T.W. in
indices. The basic philosophy used in development of PMA 1938. This DMFT index by Klein and Palmer is one of the
index was very similar to the DMF index i.e., the number of simplest and most commonly used indices in epidemiologic
gingival units affected was counted rather than severity of surveys of dental caries.
inflammation.
DMFT is Universally accepted (AP-2003) and is based on the
16. 'D' [Soben Peter 4th ed 315-16/ 5th ed 417,445] fact that the dental hard tissues are not self-healing and
Classification of Indices established caries leaves of scar of some sort.
a) Based upon the direction in which their scores can
fluctuate DMFT is applied to permanent teeth and is composed of
three components i.e. 0- decayed, M-missing teeth and
• Irreversible index: Eg: DMFT Index F-filled teeth due to caries.
• Reversible index: Eg: Loe & Silness gingival index
, COMMUNITY DENTISTRY
~

V
673

All the 28 permanent teeth are examined. The teeth not 6. Rate of caries progression cannot be assessed in terms
included are: of how fast caries is progressing or how fast caries has
• Primary teeth are not included. progressed.
7. Does not give the account for treatment needs.
• 3rd Molars
WHO modification of OMF index (1986)
• Unerupted teeth
• All the third molars are included.
• Congenitally missing and supernumerary teeth • Temporary restorations are considered as 'D'
• Teeth removed for orthodontic treatment, impaction, • On ly carious cavities are considered. 'D' the initial lesions
periodontal reasons, and teeth restored for trauma, (chalky spots, stained fissures) are not considered as 'D'.
cosmetic purposes or for use as a bridge abutment are The DMF index can be applied to denote the number
not included. of affected teeth (DMFT) or to measure the surfaces
affected by dental caries (DMFS).
• Primary teeth are not included.

Principles and rules in recording OMFT: WHO modification of dental caries criteria (1997).
1. No tooth must be counted more than once. It is either, The instruments used to record dental caries are a mouth
missing, filled or sound. mirror and the WHO/CPI probe. For individuals 30 years and
older, the M-component should comprise teeth missing due
2. When counting the number of teeth decayed teeth,
to caries or for any other reason. But for subjects under 30
also include those teeth, which have restorations with
years of age, the M-Component should only include teeth
recurrent decay.
missing due to caries.
3. Teeth which are badly delayed and indicated for
extraction are counted as missing. 20. 'A' [Soben Peter 4th ed 334/ 5th ed 438)
4. A Tooth may have several restorations but it is counted
as on tooth. 21. 'A' [Satoskar 21st ed 36)
5. A tooth is considered to be erupted when the occlusal The median lethal dose (LOSO) is the dose (mg/ Fg) which
surface or incisal edge is totally exposed or can be would be expected to kill one-half of an unlimited population
exposed by gently reflecting the overlying gingival of the same species and strain. The median effective dose
tissue with the mirror o explorer. (EOSO) is the dose that produces response in 50% of test
population. Therapeutic index is an approximate assessment
Advantages: of the safety of the drug. It is the ratio of LD50 and ED50.
It is also called as therapeutic window or safety.
1. Caries experience (past and present)d and prevalence of
an individual and community can be found out.
The larger the therapeutic index, the safer is the drug.
2. By using caries experience, oral health status can be Penicillin has a very high therapeutic index, while it is much
estimated indirectly. smaller for digitalis preparation.
3. It gives a broad overview of caries experience in a
population over a period of time. 22. 'C' [Soben Peter 4th ed 336/ 5th ed 439)
4. D-Component gives tooth status affected by dental
caries (caries morbidity). 23. ' O' [Soben Peter 4th ed 328-29/ 5th ed 430]
Periodontal Disease Index (PDI) was given by RamfJord. So
5. M-Component gives tooth lost (caries mortality)
it is also known as RamfJord Index. The PDI is clinician's
6. F-Compound gives the account of filling done among the modification of the Russel Periodontal index. PDI is primarily
population. concerned with an accurate assessment of the periodontal
status of an individual.
Limitations:
1. DM F values are not related to the number of teeth at The PDI is a clinician's modification of PL The most
risk. So, it does not directly give an indication of the important feature of PDI is measurement of the level of the
intensity of attack of caries. periodontal attachment related to CEJ of the teeth.
2. DMF index is invalid in older adults, as teeth can be lost
for reasons other than caries. The three components of POI are:
3. Reaches saturation level at particular point of time a) Plaque component.
when all the teeth are involved and prevents further b) Calculus component
registration of caries attack even when caries activity is c) Gingival & Periodontal component.
continuing.
4. Cannot be used for root caries. The six index teeth are 16, 21, 24, 36, 41, and 44. The
5. Even under extreme condition, the scores are same. surfaces scored are facial, lingual, mesial and distal. Scoring
of plaque is done after staining with Bismarck brown
solution. The scoring is done as followed .
Dental ;lut.,e i======
Scoring criteria of plaque component 24. ' B' [Soben Peter 4th ed 326/ 5th ed 428)
• Option 'N is for gingival index.
Score Criteria
• Option 'C' is for Periodontal Disease Index ( POI)
0 No plaque present
• Option 'D' is for modification of plaque component of POI.
1 Plaque present on some but not on all interproximal,
buccal and lingual surfaces of the tooth. 25. ' B' [www.WHO.int]
2 Plaque present on all interproximal, buccal and In 1981, WHO and the FD! World Dental Federation jointly
lingual surface, but covering less than one half formu lated goals for oral health (health for all) to be
of these surfaces. achieved by the year 2000 as follows:
3 Plaque extending over all interproximal, buccal l. 50% of 5-6 year olds to be free of dental caries.
and lingual surfaces, and covering more than one 2. The global average to be no more than 3 DMFT at 12
half of these surfaces. years of age.
3. 85% of the population should retain all their teeth at
Shick and Ash modification of plaque Criteria:
the age of 18 years.
This modification consists of examining the six selected
teeth by excluding interproximal areas and restricting the 4. A 50% reduction in edentulousness among the 35-44
scoring of plaque to gingival half, of the facia l and lingual year olds, compared with the 1982 level.
surfaces of the index teeth. 5. A 25% reduction in edentulousness at the age of 65
years and over, compared with the 1982 level.
Calculus Component of POI: 6. A database system for monitoring changes in oral health
This component assesses the presence and extent of calculus to be established.
on the facial (buccal / labial) and lingual surfaces of the
6 index teeth. The calculus component of POI also has a 26. ' D' [Soben Peter 4th ed 327/ 5th ed 429)
high degree of examiner reproducibility and also can be
performed quickly. 27. ' B' [Sob en Peter 4th ed 346/ 5th ed 445]

Score Criteria 28. ' B' [Soben Peter 3rd ed 192)


Moller's index is a standardized system for diagnosing,
0 Absence of calculus
recording and analyzing dental caries data.
1 Supragingiva l calculus extending only slightly
below the free gingival margin. 29. ' C' (Soben Peter 3rd ed 151)
2 Moderate amount of supra and / or subgingival Indices used for assessment of gingival bleeding:
calculus. a) Sulcus bleeding index
3 Abundance of supra and subgingival calculus. b) Papillary bleeding index
c) Gingival bleeding index by Carter and Barnes (1974)
Gingival and periodontal component: d) Gingival bleeding index by Ainamo & Bay (1975)
First the gingiva is examined changes in colour, form, e) Modified sulcular bleeding index
consistency and for any evidence of ulceration with
f) Eastman interdental bleeding index
bleeding. Then the crevice depth is recorded in relation to
CEJ. For this component, university of Michigan number 'O'
Indices used to assess gingival inflammation:
probe is used. The probe is graduated at 3, 6, and 8 mm
from the end, making it necessary to estimate intervening a) Papillary Marginal Attachment (PMA) index
measurements: b) Papillary Marginal (PM) index)
c) Gingival index
Score Criteria d) Modified gingival index.
0 No inflammation
Sulcus bleeding index and papillary bleeding index measures
1 Mild inflammation not extending around tooth . the gingival bleeding. Both indices were developed by
2 Mild to moderate inflammation all around the Muhlemann.
tooth.
30. ' B' (Soben Peter 4th ed 345-46/ 5th ed 445-47]
3 Severe gingivitis with t endency to bleed, and
A tooth with caries below the existing restoration is counted
ulcerate.
as a decayed tooth. A tooth with temporary restoration is
4 Gingival crevice not more than 3 mm apical to CEJ. also considered as decayed.
5 Gingival crevice in any of the 7 measured areas
(Buccal, Lingual, mesial, distal) extending apical 31. 'D' [Soben Peter 4th ed 327/ 5th ed 429-30)
to CEJ between 3-6 mm
6 Gingival crevice extending apically to CEJ>6mm
,......__,,_
COMMUNITY DENTISTRY 675
V
32. 'A' [Soben Peter 4th ed 350-51/ 5th ed 455] 0.6 - 1.0 Slight
The original Dean's fluorosis index (1934) is a seven-point
scale which includes normal, questionable, very mild, mild, 1-2 Medium
moderate, moderately severe and severe. However, Dean 2-3 Marked
combined moderately severe and severe categories resulting 3-4 Very marked
in the revised 6-point ordinal scale. This revised index came
into existence in 1942 (KAR- 2013) and is extensively used 40. 'B' (Soben Peter 4th ed 325/ 5th ed 427]
one currently.
41. ' B' (Soben Peter 4th ed 328/ 5th ed 430]
The six index teeth are 16, 21, 24, 36, 41, 44.

42. 'C' (Soben Peter 4th ed 334/ 5th ed 43 7]


Normal (0) Normal Enamel
Questionable The enamel discloses slight aberrations 43. 'D' [Soben Peter 4th ed 333/ 5th ed 436-37]
(0. ) ranging from a few white flecks to Primarily the CPITN is a screening procedure for identifying
5
occasional white spots. actual and potential problems posed by periodontal diseases
both in the community and in the individual.
Very mild (l) Small opaque, paper white areas involving
25% of tooth.
44. 'B' (Soben Peter 4th ed 316/ 5th ed 420]
White opaque areas not involving more
Mild (2)
than 50% of tooth. 45. 'D' (Soben Peter 4th ed 345]
All enamel surfaces are affected. Brown Those teeth are represented as Q or ~-
Moderate (3)
stain is a frequent feature.
All enamel surfaces are affected with 46. ' B' (Soben Peter 4th ed 343/ 5th ed 446]
marked hypoplasia. The teeth show pitting
Severe (4) with widespread brown stains. The general 47. 'B' (Soben Peter 4th ed 333-3 6/ 5th ed 436]
form of tooth is affected with a corroded -
like appearance 48. 'C' [Soben Peter 4th ed 334/ 5th ed 437]

33. ' B' [Soben Peter 4th ed 347/ 5th ed 446] 49. 'C' (Soben Peter 4th ed 333/ 5th ed 436]

34. 'B' [Soben Peter 4th ed 334/ 5th ed 43 7] 50. 'A' [Soben Peter 3rd ed 166]
In Russel's index, a plain probe is used. The extent of severity index (ESI) was developed by
Carlos, Wolfe, and Kingman to assess the extent (number
of sites affected within the mouth) and severity (stage
35. 'B' [Soben Peter 4th ed 321/ 5th ed 422]
of advancement) of loss of periodontal attachment (LPA)
36. 'A' [Soben Peter 4th ed 318/ 5th ed 420] by determining the percentages of sites within the mouth
with LPA >l mm and the mean LPA for affected sites (i.e.
37. 'C' [Soben Peter 4th ed 334/ 5th ed 437] severity). ESI utilizes the Ramfjord method to measure loss
of periodontal attachment. In this method, the mid buccal
38. 'A' [Soben Peter 4th ed 346/ 5th ed 449] and mesiobuccal aspects of 28 teeth (except 3rd molars) are
measured using Ramfjord procedure.
39. 'C' (Soben Peter 4th ed 350/ 5th ed 456]
Community fluorosis index (CFI) Despite its name, some consider ESI not to be a true index
Dean in 1946 devised a method to determine the severity of since it summarizes data and is descriptive rather than
dental fluorosis and a public health problem. It is calculated analytical.
by the following way.
51. 'B' (Soben Peter 4th ed 345]
In case of mixed dentitions DMFT/DMFS and a "deft/defs"
nXw are never added t ogether. Each child is given a seperate
CFI = - -
N index for permanent teeth and another for primary teeth.

n = number of individuals in each category 52. 'A' (Soben Peter 4th ed 382-83]
w = the weighting for each category a) Mean± 1 S.D. covers 68.3% of the observations.
N = Total population
b) Mean± 2 S.D. covers 95.4% of the observations.
c) Mean± 3 S.D. covers 99.7% of the observations.

0 - 0.4 Negative In the question it is given 68% of the population has DMFT
0.4 - 0.6 Borderline values between 2 to 3 i.e. Mean+ 1 SD= 3 and Mean - 1 SD=
2. From the above equation we can make out SD value as 0.5
676 Den ta I !J>ul,e

53. 'D' [Soben Peter 4th ed 343/ 5th ed 445] • Spacing in incisal segments
• Midline diastema
54. 'C' [Soben Peter 4th ed 334/ 5th ed 43 6] • Over jet, over bite, and antero-posterior molar relation,
etc.
55. 'A' (Soben Peter 3rd ed 135)
The amount of irregularities like rotations or displacements
56. 'D' [Soben Peter 4th ed 343-44]
from normal alignment, overjet, overbite and open bite are
A restoration placed on a non-vital or endodontically
measured with CPI probe.
treated tooth, to seal a root canal and not for caries, is not
considered in DMF score.
65. 'C' [Soben Peter 3rd ed 144)
The navy plaque index was developed by GROSSMAN and FEDI
s 7. 'B' (Soben Peter 4th ed 350/ 5th ed 455]
to assess plaque control status among naval pensioners using
d dbclosing )Olution. Thi:: index tt:eth a re 16, 21, 24, 36, 41
58. 'B' [Soben Peter 4th ed 292-94/ 5th ed 393]
and 44. The FACIAL and LINGUAL surfaces are divided into
Index age and age groups for survey are 5 years for primary
Gingival (G), mesial proximal (M) and Distal proximal area
teeth and 12, 15, 35-44 and 65-74 years for permanent teeth.
(D). A score of 3 is given for proximal areas (M,D) and a score
of 2 is given for Gingival area. The highest total for any of the
59. ' B' [Soben Peter 4th ed 346/ 5th ed 445-49]
six teeth scored is the patient's Navy plaque index score.
In original DMF index third molars were not considered and
the score range was Option'/( i.e., 0-28. According to WHO
66. ' B' [www.ada.org]
modification of DMF index {1986), all the third molars are
In many countries, screening surveys have made use of a
considered and so the score range is O - 32.
methodology largely derived from the CPITN. In the United
States, Canada and Brazil, the periodontal screening and
(Note: Physical quality of life index has a maximum score
recording (PSR) has been increasingly advertised and
range of O - 100). used. In England and New Zealand, the Basic Periodontal
examination (BPE) is in use whereas the primary essential
60. ' B' [Soben Peter 4th ed 305-06)
periodontal examination (PEPE) is used in Australia. The
usage of WHO - 621 probe is a common feature in all of
61. ' D' [Soben Peter 4th ed 345/ 5th ed 448)
these methodologies.
62. [Soben Peter 4th ed 343/ 5th ed 445]
' fl{
PSR is a diagnostic screening tool for early detection of
The modified DMFT index was introduced by Joseph Z.
periodontal disease. It uses a plastic PSR probe that ha~ a
Anaise in 1983. The only difference is in the scoring criteria
0.5mm diameter ball tip and a color-coded band extending
for 'D' component of index, which is divided into 4 separate
3.5mm to 5.5mm from the tip. The PSR index divides the
categories. mouth int o 6 sextants and the greatest probe depth in each
sextant of the mouth is determined and recorded. The probe
Category Criteria is gently inserted into the gingival sulcus until resistance
'C' Unfilled that are carious. is met and then explored by "walking" the tooth. Atleast
six areas in each tooth or implant should be examined.
'CF' Filled teet h that are carious. The six areas are mesiofacial, midfacial, distofacial and the
Unfilled or filled carious teeth that are corresponding lingual/ palatal areas.
'IX' indicated for extraction i.e., caries has so
destroyed the crown that it cannot be restored. 67. ' B' [WHO Oral Health survey 4th ed 36]
Unfilled or filled teeth indicated for pulp The three jndicators of periodonta l status used in CPI
'IRC' index are
treatment or RCT.
• Gingival bleeding
The other two categories (T and 'M') are recorded as usual • Calculus
according to WHO criteria.
• Periodontal pockets
63. 'fl{ [Soben Peter 3rd ed 321/ 4th ed 321)
68. ' D' (Soben Peter 4th ed 3 27 / 5th ed 429]
64. ' B' [Soben Peter 5th ed 406)
69. 'C' [Soben Peter 4'h ed 326/ 5th ed 428]
In WHO oral health assessment form {1997) dental aesthetic
Periodontal index was developed by Russel in 1956, over a
index criteria were used to record dentofacial anomalies. It is
trial period of ten years. This index was intended to measure
recommended that this index be used for age groups in which
there are no longer pri mary teeth, usually from 12 years. the presence or absence of
• Gingival inflammation and its severity
The anomalies recorded are: • Pocket formation
• Missing incisor, canine and premolar teeth • Masticatory function
• Crowding in incisal segments
, COMMUNITY DENTISTRY
~

V
677

This index is reported to be useful among large populations, 72. 'C' [Soben Peter 4th ed 352/ 5th ed 455]
(AIIMS MAY 2012) but it is only of limited use for
individuals or small groups. 73. 'C' [Soben Peter 4th ed 327/ 5th ed 428]

70. ' B' [Soben Peter 4th ed 318/ 5th ed 421] 74. 'A' [Soben Peter 4th ed 357/ 5th ed 462)
Mouth mirror and No. 23 (Shepherd's Hook) explorer are
used t o measure debris component of oral hygiene simplified 75. 'A' [Soben Peter 3rd ed 208/ 5th ed 456-58)
index.
76. 'B' [Park 20th ed 16/ Hiremath 1st ed 6)
Option 'K plaque component of periodontal disease index The WHO definition of health is "Health is a state of complete
uses Bismarck brown solution. In Turesky-Gilmore-Glickman physical, mental and social well-being and not merely an
modification of the Quigley-Hein plaque index, t he plaque absence of disease or infirmity.
is assessed on t he labial, buccal, land lingual surfaces of all
teeth using a disclosing agent. The concept of well being is considered as follows:
1. Standard of Living
71. 'B' [Soben Peter 4th ed 359/ 5th ed 456-58]
2. Level of Living
Thylstrup and Fejereskov index was developed with an
aim of developing more sensitive classification system 3. Physical quality of life index
for recording change s found in areas wit h water fluoride 4. Human developmental index
levels that are higher t han studied by Dean. This index is 5. Human poverty index
appealing t o t he clinicians in t hat the classification scale
corresponds closely to the histological changes that occur
1. Standard of living:
in dental flourosis.
Refers to the usual scale of our expenditure, the goods
A 10-point ordinal scale is used to classify changes we consume and the services we enjoy. It includes the
associated with increasing fluoride exposure. level of education, employment status, food, dress,
house, amusements and comforts of modern living.
Score Criteria
2. Level of living:
0 Enamel shows normal t ranslucency
Parallel form for standard living in United Nations
• Thin white opaque lines on all part s of tooth documents. It includes nine components i.e., Housing,
surface. fo od consumption, education, occupation and working
1
• Snow capping of cusps or incisal edges may be conditions, health, social security, clothing, recreation
seen in some cases. and leisure and human right s.
• The white opaque Lines become pronounced
and merge to form small cloudy areas scattered 3. Physical quality of life index (POLI): (MHCET-15}
2 over t he whole surface (KCET-2009) It is composite index with three indicators i.e.,
• Snow-capping is common. • infant mortality
Cloudy areas of opacity may be seen over many • Life expectancy at age one
3
parts of the surface. • Literacy
Entire surface exhibits marked opacity and
4
appears chalky white. The POLI value ranges from Oto 100
Entire surface is opaque with round pits of<
5
2mm in diameter 4. Human development index:
Small pits merge to form bands that are < 2m in It is composite index with three indicators i.e.,
6
vertical heighl • Longevity (life expect ancy at birth)
There is loss of outermost enamel in irregular • Knowledge (Adult literacy rat e and mean years
7 areas and < 1/2 of the surface is involved. The of schooling)
remaining half is opaque.
• Income (Real GDP per capita in purchasing
8 > 11z of out ermost enamel is Lost. power parity in US dollars)
Loss of major part of outer enamel resulting in
9 change of the anat omical shape of t he surface/ The HDI value ranges from Oto 1. The HDI for India is 0.619.
tooth.
5. Human poverty index: (HPI)
Most sensitive index for fluorosis is? (PGI June-13) The HPI is different for developing (HPI-1) and
a) Dean's b) Thylstrup Fejerskov Developed (HPI-2) countries.
c) Surface fluorosis index d) Community fluorosis index
Dental ;lut.,e

The dimensions used fo r HPl-1 for developing countries


are
• Vulnerability to death at a relatively early age, as
measured by the probability at birth of not surviving
II 9
. INCISAL THIRD

••
to age 40.
7 8
• Knowledge - as measured by poor adult literacy rat e. MIDDLE THIRD
• Poor standard of living - lack of access to overall
economic provisioning, as measured by the 4 5 6
unweighted average of two indicators i.e., the GINGIVAL THIRD
1 2 3
percentage of t he population not using an improved
water source and the percentage of children under-
weight for t hat age.
Fig. Labial surface of a anterior toot h

The dimensions used for HPI-2 for developed countries • A modified plaque index per person score is obtained
are by totaling all nine of the sub-division scores per tooth
• Similar t o HPI-1 except t hat probability of non- surface and dividing by t he number of t ooth surfaces
survival is upt o age 60. examined.
• 2nd and 3,d dimensions are similar to HPI-1
79. 'A' (Soben Peter 4th ed 304/ 5th ed 405 1st line]
• Social exclusion - as measured by the rate of long-
term unemployment {12 months or more)
80. 'A' (Soben Peter 4th ed 315/ 5th ed 415 Last line]
77. 'C' (Soben Peter 4'h ed 334-35/ 5th ed 437) Epidemiological indices are attempts to quantitate clinical
conditions on a graduated scale, t hereby facilit ating
78. 'C' (Check Explanation below] comparison among populations examined by the same
Navy plague index: criteria and methods.
• Developed by Grossman and Fedi to assess plaque
control status among naval personnel. Ideally. an index should possess the following properties:
• After application of disclosing agent, the following Clarity, simplicity and objectivity: The index should be
teeth are examined i.e., 16, 21, 24, 36, 41, 44 (Similar easy to apply so that there is no undue time lost during field
to PDI index) examination. the criteria of the index should be clear and
• The surfaces examined are facia l and lingual. The facial unambiguous.
and lingual surfaces are divided into three major areas.
Validity: It must measure what it is intended to measure, so
• Area M - Mesial proximal - Score 3
it should correspond t o clinical stages of the disease under
• Area D - Distal proximal - Score 3 study at each point.
• Area G - Gingival area - Score 2
Reliability: The index should meas.ure consistently at different
times and a variety of conditions. the ability of the same or
• Total of six areas are examined on each index toot h.
different to interpret and use the index in t he same way.
Modified Navy plague index:
Quantifiability: It should be responsive to t he statistical
• Developed by Elliott, Bower, Clemen and Roverstad analysis so that the status can be expressed by a number
• The purpose and i ndex teeth are same as Navy plaque that corresponds to a relative position on a scale of Oto the
index. upper limit.
• This index records presence or absence of plaque, by a
score of 1 or 0, respectively on nine areas of each tooth Sensitivity: Should be able to detect reasonably small shifts
surface of the six index teeth. in either direction in the group or individual conditions.

Acceptability: The use od index should not be painful or


demeaning to the subject.

DMF score in statistics is a (AIIMS Nov-13)


a) Qualitative discrete data
b) Qualitative ordinal data
c) Quantitative data d) Normal data

81. 'D' (Soben Peter 4th ed 324/ 5th ed 426]


, COMMUNITY DENTISTRY
,......__,,_

V
679

82. 'B' [Soben Peter 4th ed 33 7/ 5th ed 43 7)

83. 'C' [Soben Peter 4th ed 333/ 5th ed 436]

84. 'C' [Soben Peter 4th ed 316/ 5th ed 417] 0 Absence of inflammation

85. 'C' [Soben Peter 4th ed 352/ 5th ed 455] Mild inflammation:
Slight change in color, Little change in
1
86. 'A' [Check Explanation Below] texture of any portion of but not the entire
The care index is the proportion of decayed teeth filled by marginal or papillary gingival unit
dentists which reflects the extent of restorative care to teeth. Mild inflammation:
It is calculated as percentages i.e., (FT/DMFT multiplied by 2 Criteria as above but involving the entire
100 for permanent teeth and ft/dmft multiplied by 100 for marginal or papillary gingival unit
primary teeth).
Moderate inflammation:
3 Glazing, redness, edema, and/or hypertrophy
Restorative index is calculated as percentage of filled teeth of the marginal or papillary gingival unit
among decayed and filled teeth i.e, F/DF.
Severe inflammation:
87. 'C' [Soben Peter 5th ed 445] Marked redness, edema, and/or hypertrophy
4 of the marginal or papillary gingival unit;
88. 'B' [Preventive materials, methods and programs by spontaneous bleeding, congestion, or
Axelsson 2004 ed 449) ulceration
The WHO's goals for caries prevalence in 12 year old children
in years 2000, 2010 and 2025 are 3.0, 2.0 and 1.0 DMFT 92. 'A' [Shanti Priya Reddy 2nd ed 45]
respectively. Gingival Bleeding Index (GBI- Carter and Barnes)
• Introduced by Carter and Barnes in 1974, which records
Since 1969, the WHO has compiled a world map of caries the presence or absence of gingival inflammation after
prevalence among 12 year old children, expressed in DMFT. passing unwaxed dental floss into the proximal sulci. It
At the age of 12 year; the five-level scale varied from 0.0 to is readily available, disposable, and can be used by the
more than 6.5 DMFT. instructed patient for self-evaluation.
• The mouth is divided into six segments and flossed in
Colour Level Range of DM FT the following order; upper right, upper anterior, upper
Green Very low caries prevalence 0-1.1 Left, lower left, Lower anterior and lower right.
• Bleeding is generally immediately evident in the area
Blue Low caries prevalence 1.2 to 2.6
or on the floss; however, 30 seconds is allowed for
Yellow Moderate prevalence 2.7to4.4 reinspection of each segment. If copious hemorrhage
Red High preva lence 4.5 to 6.5 occurs the patient may be allowed to rinse in between
segments.
Brown Very high caries prevalence >6.5
• Bleeding is recorded as present or absent. For each
patient a Gingival Bleeding Score is obtained by noting
the total units of bleeding and the total susceptible
areas at risk.

Gingival Bleeding Index (GBI - Ainamo & Bay, 1975)


1 No need of any treatment
• Performed through gentle probing of the orifice of the
2 Mild/ little need gingival crevice. If bleeding occurs within 10 seconds a
3 Moderate / Borderline need positive finding is recorded and the number of positive
sites is recorded and then expressed as a percentage of
4 Severe / Need treatment
the number of sites examined.
5 Extreme / need treatment • Bleeding can also function as a motivating factor in
activating the patient t o better oral home care. It has
90. 'D' [Soben Peter 5th ed 420] been show that the scores obtained with this index
The dental index and calculus index values range from O to correlate significantly to GI (Loe and Silness, 1963) and
6, and OHi value ranges from 0-12. When used OHI-5 index, has been used in profile studies and short-term clinical
the DI-S and CI-S scores range from 0-3 while OHI-S scores trials.
range from O to 6.
Dental ;lut.,e

4. PREVENTIVE DENTISTRY

1. Fluoride safe dose is 12. The term mottled enamel to Colorado stain was given by:
a) 8-16 mg F/kg body weight a) McKay b) Black
b) 16-32 mg F/kg body weight c) Dean d) Jones
c) 30-64 mg F/kg body weight (MAN-2K)
d) 12-30 mg F/kg body weight 13. In dentifrices concentration of fluoride is:
(MAN-2001) a) 800 ppm b) 2000 ppm
2. Active fluoride concentration in the cavity varnish c) 1000 ppm d) 600 ppm
duraphat is (PGl-2K, 01, 02, 03)
a) 22,900 ppm b) 7,600 ppm 14. Safe disposal of mercury is:
c) 22,600 ppm d) 11,600 ppm a) Collecl cardully & recycle
(KAR-2K) b) Controlled combustion
3. Comm unity water fluoridation is important because c) Treatment with chemicals
a) All groups of socio economic status have accessibility to it d) Deep burial
b) does not pose a health hazard (NEET-2013)
c) leads to decrease in the dent al caries incidence 15. Concentration of Fin 1.23% APF gel is (1.23% c)
d) all of the above a) 22600 ppm b) 12300 ppm
(MAN-2K) c) 5620 ppm d) None
4. The recommended concentration of fluoride in a (PGl-95, 2002)
community water supply is 16. 0.2% NaF mouth rinse is re com mended
a) 0.1 ppm b) 0.5 ppm a) Once daily b) By weekly
c) 1.0 ppm d) 2.0 ppm c) Fortnightly d) Twice weekly
(MAN- 95) (PGl-2002, AIPG-96, AP-)
5. The recommendation concentration of acidulated 17. The following is/ are a rich source of fluoride
phosphate (APF) fluoride gel is a) Dried fish b) Tea leaves
a) 2% b) 8% c) Cabbages d) All of the above
c) 1.23% d) 10% (KAR-2K)
(AP-2003) 18. The "Choking off" mechanism takes place after topical
6. ' Colorado stain" was discovered by (OR) application of:
The term colorado satin was given by a) NaF b) SNF2
a) Green b) McKay c) APF
c) Dean d) Knutson d) Sodium mono-fluorophosphate
(MAN-1999, PGI JUNE-14)) (KAR-2K)
7. Fluoride is added to drinking water in the form of 19. The fluoride content of fluor protector varnish is:
a) hydrofluorosilicic acid b) fluoride ions a) 2.26% b) 0.70%
c) APF d) none of the above c) 2.70% d) 0.30%
(MAN-2K) (KAR-2K)
8. The first ADA approved fluoride toothpaste in the year 20. Galagan's formula is to:
a) 1960 b) 1968 a) Measure the amount of fluoride in toothpaste.
c) 1964 d) 1954 b) Measure the amounts of fluoride in drinking water.
(MAN-2001) c) Calculate the fluoride concentration in drinking water
9. Shoe leather survey was carried out by depending on temperature of the area.
a) McKay b) Eager d) Assess t he age of the individual by dentition stat us.
c) Trendley Dean d) Brown (KAR-97)
(MAN-2001) 21. 0.05% of NaF has concentration of F
10. The element fluoride was described by Churchill in: a) 9000 ppm b) 1000 ppm
a) 1934 b) 1942 c) 225 ppm d) 250 ppm
c) 1931 d) 1941 (PGI-2002)
(KAR-2002) 22. During community water fluoridation which of the
11. Fluoride supplement for a 3-year old child living in an following is least important?
area with fluoride content of 0.25 ppm is a) Temperature of the area
a) 0.25 mg b) 0.5 mg b) Geographical location of the area
c) 0.75 mg d) 1 mg c) Physiological response to fluoride
(MAN, KAR-2002) d) Average weight of children
(AIPG-95)

1) A 2) C 3) D 4) C 5) C 6) B 7) A 8) C 9) C 10) C 11) B 12) A 13) C


14) A 15) B 16) B 17) D 18) A 19) B 20) C 21) C 22) D
, COMMUNITY DENTISTRY
,......__,,_

V
681

23. The % reduction in caries to be expected by the 34. The pH of APF gel is-
application of neutral fluoride solution is: a) 2.3 b) 3.2
a) 20% b) 20-50% c) 2.32 d) 1.23
c) variable d) 10% (KAR-99)
(PGl-99) 35. Dental caries is associated with flouride levels in water
24. In topical fluoride application, maximum benefit is below:
derived when the tooth surface is treated for: a) 0.1 mg /L b) 0.5 mg /L
a) 15 seconds b) 30 seconds c) 1.0 mg /L d) 2.0 mg /L
c) 1 minute d) 4 minutes (KAR-98)
(AIPG-95) 36. Determining level of fluoride in community water
25. Knutson's technique utilizes? fluoridation programme depends on:
a) 1.23 % APF b) 8% Snf2 a) Mean annual temperature of the place
c) 2% Na Fluoride d) Any of the above b) Economic factors
(PGI-95) c) Average weight of children of the area
26. 4 yr old child has 0.2 ppm fluoride in drinking water. The d) Altitude of area above sea level
child can be administered: (AIIMS-94)
a) 1 mg tab/day b) Rinses 0.2% NaF 3 7. Duraphat is:
c) Topical application of fluoride a) Lacquer
d) Fluoride containing dentifrices b) Sodium Fluoride in varnish form
(AIIMS-96, 2003) c) Stannous Fluoride d) Sodium Fluoride 2%
27. A child spends 8 years life in a temperate climate where (AIPG-97; KAR-2001)
the drinking water had fluoride of 3ppm. The teeth that 38. How much fluoride is present in a tooth paste with 700
are effected are ppm fluoride?
a) all primary teeth a) 0. 7 mg per one gram of toothpaste
b) all permanent teeth, except third molars b) 7 mg per one gram of tooth paste
c) i ncisors, canines, premolars and third molars c) 7 .2 mg per one gram of toothpaste
d) only molars d) 0.007 mg per one gram of toothpaste
(AIIMS-2K) (AIPG-94)
28. A 3-yr-old child living in area with 0.8 ppm F in water 39. The daily recommended dose of the fluoride for the
should be given F tablets daily: children below 3 years is
a) 0.25 mg b) 1 mg a ) Less or equal to 0.5 mg b) 0.5 to 0.7 mg
c) 0.5 mg d) 0.00 mg c) 1.0 to 1.5 mg d) None of the above
(PGI-95) (AP-98)
29. Dental fluorosis not associated with skeletal flourosis 40. In school water fluoridation programme the level of
are seen at: fluoride as compared to the level in community water
a) 1 ppm fluoride b) 3 ppm fluoride fluoridation is:
c) 8 ppm fluoride d) 10 ppm fluoride a ) Less
(PGI-98, 2001) b) More
30. The best sustaining topical effect of fluoride is obtained c) Same
by: d) Depends on the mean weight of the children
a) NaF b) APF gel (AIPG-95)
c) Duraphat d) SnF 41. Amount of reduction in dental caries achieved by
(PGI- 2001) dentifrices is:
31. The concentration of Naf in a daily mouth rinse is: a) 20-35% b) 35-40%
a) 0.2% b) 0.05% c) 40-50% d) 50-60%
c) 0.03% d) 0.01% (PGI-98,-95)
(PGI- 2001) 42. Absorbed fluoride is eliminated by:
32. The primary action of topical fluoride is a: a ) Kidneys b) Lungs
a) Conversion of hydroxyapatite to fluoroapatite c) Saliva d) Intestines
b) Decrease in the plaque bact eria ( PGl-2001; AP-98N NEET- 2013)
c) Form a reservoir in saliva 43. Stannous fluoride concentration used in topical fluoride is:
d) Improve morphology of teeth a) 8% b) 2%
(AIPG-2002, AIIMS- 2001) c) 1.23% d) 12%
33. NaF application is done: (PGI, MAN-2K)
a) Once a year b) twice a year 44. A 5-year old child is recommended the following fluoride
c) four times a year d) eight times a year dentifrice regimen
(AP-2K) a) Fluoride tooth paste not recommended.
b) Brushing 3 times with fluoride tooth paste

23) B 24) D 25) C 26) A 27) B 28) D 29) B 30) C 31) B 32) A 33) C 34) B 35) C
36) A 37) B 38) A 39) A 40) B 41) A 42) A 43) A 44) C
Dental ;lut.,e i======
c) Once with fluoride paste and two times with normal 54. Who postulated that water fluoridation is associated
paste. with reduced tooth decay?
d) Brushing twice with fluo ride tooth paste and once with a) Dean b) Murray
normal paste c) Horowitz d) Knutson
(KAR-97) (KAR-2K)
45. Brudevold's solution contains: 55. 1 part per million fluorides is:
a) 2 % SnF2 b) 2 % NaF2 a) 1 mg in 10L of water b) 1 mg in 1L of water
c) 1.23 percent sodium fluoride in 0.1 Morthophosphoric acid c) 10 mg in 1L of water d) 100 mg in 10L of water
d) 2 percent sodium fluoride in 0.1 M orthophosphoric acid (PGI-99, 06; AIIMS- 04)
(KAR-97) 56. In school water fluoridation, amount of F used is:
46. When using a supplemental fluoride for the systemic a) 0.7-1.2 ppm b) 1.6-3 ppm
water fluoridation, which is the most important factor to c) 3-4 ppm d) 12 ppm
be taken into consideration: (PGI-95)
a) Mean climatic temperature 5 7. Nalgonda tech is used for
b) Number of topical fluoride treatments to be given a) Deflouridation b) fluoridation
c) Amount of supplemental fluoride given by the physician c) Purification of water d) None
and the dentist (PGI-2002)
d) Mean age of the child 58. Repeated topical applications of a Fluoride agent will
(AIPG-2001) not produce tooth mottling because:
4 7. Fluoride chewable tablets are preferred because: a) The fluoride concentration of the solution is weak
a) More Fis ingested b) The applied fluoride is neutralized by calcium in saliva
b) Easily acceptable c) It is not fluoride that produces the mottling seen in
c) Daily topical application occurs fluorosis
d) Economical d) The tooth is already calcified and cannot be altered in
(AIPG-2002; 93) this fashion
48. Mode of action of systemic fluoride is: (AIPG-2002, PGI JUNE- 14)
a) Formation of protective layer over tooth 59. Fluoride is least effective in which area:
b) Inhibit enzymatic breakdown of enamel a) Cusp b) Pits and fissures
c) As F- content increases, solubility of enamel decreases c) Buccal areas d) Palatal area
d) All of the above (PGI-95)
(AIPG-93) 60. Chronic fluoride toxicity results in:
49. Space maintenance is an example of a) Extreme arthritic changes of the spine resulting in
a) Specific protection b) Disability rigidity
c) Rehabilitation d) Primary prevention b) Calcification of the sheaths of muscles and tendons
(AIPG-2012, 14) c) Osteosclerosis d) All of the above
50. The main site of absorption of ingested fluorides is (PGI-2001)
a) esophagus 61. The newly erupted tooth:
b) stomach and small intestine a) Is protected by dental plaque
c) small and large intestine b) Has an early immunity to caries
d) large intestine c) Has a high affinity for fluoride
(PGI-98,97) d) All of the others
51. Knutson's technique of fluoride application is done the (KAR-2001)
following years:- 62. The best method to prevent initiation of caries in a four-
a) 3, 7, 11, 13 b) 2, 6, 10, 16 year-old child who has no accessibility to community
c) 8, 13, 18, 31 d) 2, 7, 11, 18 water fluoridation is:
(KAR-97; MAN-99) a) Brushing and flossing b) 2.2% NaF tablet daily
52. In acute fluoride toxicity we use: c) 0.05% NaF mouthwash daily
a) Vinegar b) Mill< d) 0.5% NaF toothpaste daily
c) Water d) NaHC03 (PGI-98)
(PGl-2001, 95) 63. Difference between Fluoride Gel and Varnish:
53. A child accidentally swallows 10 cc of a 10% fluoride a) Gel given by tray where as varnish cannot
solution. The most effective immediate action is to: b) Gel application is not for office use
a) Have the child drink copious quantities of water c) More number of applications can be done by varnish
b) Have the child drink a 10% sodium bicarbonate solution d) Gel penetrates the inter proximal surface better, while
c) Have the child drink milk or some other calcium varnish cannot
containing liquid (AIPG-2002)
d) Send the child to the family physician 64. Best way to prevent caries is:
(AIIMS-99, 2K) a) Systematic fluoridation

45} D 46} A 47} C 48} D 49} A 50} B 51} A 52} B 53) C 54} A 55} B 56) C 57} A
58) D 59) B 60) D 61) C 62) B 63) A 64) A
, COMMUNITY DENTISTRY

b) Fluoride mouthwash 76. The minimum amount of fluoride that should be present in
c) Fluoride toothpaste salt for effective anticaries activity in:
d) Fluoride pro phyla xis a) 3mg of F/5gm of salt b) 1.5mg of F/5gm of salt
(PGI-98) c) 10mg of F/5gm of salt d) 100mg of F/5gm of salt
65. Grand rapids & Muskegon study is based on (PGI-98)
a) Field trial 77. A layman before introduction of fluoride in prevention of
b) Observational studies dental caries knew about it as a constituent used in
c) Randomized controlled trials a) Pesticide b) Industrial use
d) Community trials c) Fertilizers d) Irrigant
(AP-2011) (AIIMS-2004)
66. The usual metabolic pathway of ingested fluoride 78. Average daily intake of fluoride from all sources
involves urinary excretion primarily with the remaining combined for adults and children is:
portion found largely in: a) 2.0 to 2.2mg & 1.2mg. b) 1.2 to 2.0mg & 1.2mg.
a) Teeth b) Muscle c) 2.2 to 2.4mg & 1.5mg. d) 2.6 to 2.8mg & 1.4mg.
c) Skeletal tissues d) Liver (AIPG-2004)
(KAR-2K) 79. The WHO index for fluorosis is based on:
67. 8% Snf2 contains how much% offlouride a) Dean's index. b) McKay's index.
a) 1% b) 2% c) Community Fluorosis index.
c) 3% d) 4% d) Public Health fluorosis index.
(PGI -2002) (AIPG -2004)
68. Fluoride action on carbohydrates in reducing the caries 80. Fluoride content is more in which milk product?
is by a) Cheese b) Butter
a) Inhibiting enolase b) Inhibiting fructose c) Toned milk d) Butter milk
c) Inhibiting the sucrase d) Inhibiting glycolase (PGI-2001)
(AP-2K) 81. Among the common food items, fluoride is found in
69. What is the recommended daily dose of fluoride for the higher PPM in:
3 to 12 years old child? a) Pineapple b) Tea
a) 1/ 4 mg. b) 1/z mg. c) Sea Fish d) Coconut Water
c) 1 mg. d) 2 mg. (KAR, AIPG-2004)
(KAR-97) 82. Metallic taste is due to SnF in saliva is
70. Deciduous teeth do not show fluorosis because: a) Stannous triphosphate b) Stannic triphosphate
a) Placenta acts as a barrier c) Tin hydroxyl phosphate d) Stannous hydroxide
b) Fluoride is taken up by the fast growing skeletal tissue (AIPG-2012)
c) The calcification time of deciduous teeth is less 83. Famous Vipeholm study was conducted to:
d) All of the above a) To find out role of sticky and non sticky form of sugars
(AIIMS-99; AIPG-96) on dental caries
71. Mc Kay discovered Colorado stain in: b) To find out the role of water fluoridation on dental caries.
a) 1901 b) 1897 c) To find out role of Xylitol on dental caries
c) 1908 d) 1930 d) All of the above
(PGI-95) (AIPG, AIPG -2004)
72. Which of the following levels of fluorides in drinking 84. Systemic water fluoridation reduces caries
water is associated with enamel fluorosis? a) by 25% b) only on smooth surfaces
a) 0.2 ppm b) 0.7 ppm c) on smooth surfaces and pit and fissures
c) 2.5 ppm d) 1.2 ppm d) is possible only in advanced countries
(KAR-2003) (PGI-97, AP -07)
73. The Fluoride form that has to be freshly prepared before 85. The most common fluoride agent used in dentifrices are
use is: a) Sodium monofluoro phosphate
a) NaF b) Sn F b) Amine fluoride
c) APF solution d) APF gel c) Sodium fluoride in organic base
(AIPG-97) d) Sodium fluoride in orthophosphoric acid
74. Which of the following methods is used fordeflouridation? (COMEDK-05)
a) Boiling b) Nalgonda 86. Nalgonda technique involves addition of following
c) Nalanda d) None of the above chemicals:
(KAR-2002) a) Lime, bleaching powder, filter alum
75. Milk Fluoridation was introduced by b) Magnesia, calcium phosphate, filter alum
a) Knutson b) Bibby c) Alumina, carbon, lime
c) Wespi d) Zeigler d) Sulphonated saw dust, coal, calcium phosphate
(KCET-2011) (UPSC-2001, COMEDK-05)

65) D 66) C 67) B 68) A 69) C 70) D 71) A 72) C 73) B 74) B 75) D 76) B 77) A
78) A 79) A 80) A 81) B 82) C 83) A 84) B 85) A 86) A
Dental ;lut.,e i======
87. Enamel crystallites are stabilized against acid if fluoride b) Accepted even though retention is limited
ions replace c) Limited to buccal pits
a) Phosphate ions b) Calcium ions d) Indicated only before six years of age
c) Strontium d) Hydroxylions (KAR-2001)
(COMEDK-06) 100. Recommended time for acid etching before placing a pit
88. The minimum amount of fluoride required in common and fissure sealant
salt for caries reduction is a) 30 sec. b) 60 sec.
a) 100 mg/kg b) 90 mg/kg c) 90 sec. d) 120 sec.
c) 200 mg/kg d) 350 mg/kg (PGI-98)
(PGI-97) 101. Sealants are effective in
89. Fluoride is carried in the blood via: a) Initial caries b) Secondary caries
a) Plasma b) RBC'S c) Recurrent caries d) Ram pant caries
c) Both plasma&RBC d) WBC'S (AIIMS-89)
(AIPG -2004) 102. Pit & tissue sealants act by
90. Flouride poisoning occurs in a) Killing micro-organism causing caries
a) Village workers b) Tooth paste workers b) Blocking nutrient supply to bacteria
c) Cryolite workers d) Phosphorus workers c) Neutralising acid produced by bacteria
(AP-2001) d) All of these
91. The first country to adopt salt fluoridation was (TNPSC-99)
a) France b) Germany 103. The threshold dose of fluoride that requires immediate
c) Jamaica d) Switzerland emergency treatment and hospitalization in acute
(COMEDK-04) fluoride toxicity is:
92. Most commonly associated ion with remineralisation of a) 4mg b) 5mg
enamel c) 6mg d) 7mg
a) ca•2 b) F- (AI- 05)
e) Po/ d) K· 104. An example for primary prevention of dental diseases is:
(PGI-98) a) Scaling & polishing and filling
93. In ceramic restoration, which can be used b) Extractions, RCT and periodontal surgeries
a) 1.23% APF b) 8% SnF2 c) Wearing gloves and sterilization of the instruments
c) 6% SnF2 d) 2% NaF d) Replacements of lost teeth and orthodontic treatments
(PGI-2003) (AIPG- 06)
94. Which is applied once and left to dry 105. The best fluoride compound recommended for mouth
a) NaF b) SnF2 rinsing is:
c) APF d) Varnish a) Stannous fluoride
(PGI-2002) b) Acidulated phosphate fluoride
95. Which of the following procedures can be replaced by the c) Neutral sodium fluoride
use of occlusal sealants d) Ammonium fluoride
a) Topical fluoride application (COMEDK-05)
b) Prophylactic odontotomy 106. Which of the following techniques would be the best
c) Amalgam fillings d) All the above preventive measure for dental caries?
(AP, MAN-99) a) Elimination of sugars from the diet
96. The process of applying a resin to the fissures of a newly b) Reduce frequency of int ake of cariogenic food stuffs
erupted tooth was introduced by c) Rinse and swish with water after each meal and snacks
a) Black G.V. b) Buonocore d) Substitution of alcohol-based sugar for sucrose
c) Hyatt d) Hororwitz (AIPG-06)
(MAN-2000) 107. Deflouridating material is:
97. Third generation sealants are cured by a) Duraphat b) Fluorprotector
a) Self-cured b) Visible light c) Brudevold's solution d) alumina
c) UV light d) Natural light (PGI- 05)
(MAN-2001) 108. Which of the following is the wrong statement:
98. Using pit and fissure sealant is: a) 2% NaF solution is recommended as caries preventive
a) Specific protection agent
b) Early diagnosis and treatment b) 2% NaF solution can be prepared by dissolving 0.2gm of
c) Secondary prevention d) Limitation of disability NaF powder in 10ml of distilled water in plastic bottle.
(PGI-95) c) If NaF aqueous solution is stored in glass bottle, then
99. The colorless or transparent pit and fissure sealants are the fluoride ions of solution can react with silica of glass
usually forming SiF2 thus reducing the availability of free active
a) Indicated only in case of rampant caries fluorides to anticaries activity

87} D 88} C 89} C 90} C 91} D 92} B 93} D 94} A 95} B 96} B 97} B 98} A 99} B
100) A 101) A 102) B 103) B 104) C 105) C 106} D 107) D 108) D
, COMMUNITY DENTISTRY
(""'.,,_

V
685

d) 2% NaF application should be carried out at the age of 120. Nephrotoxicity of fluoride is:
3,7,12 & 18 years a) 400-500 µg/d l b) 800-900 µg/dl
c) 850-950 µg/dl d) 1000-1200 µg/dl
109. The fluoride concentration in 8% stannous fluoride, (PGI- 06)
topical application, is 121. Opaque white areas covering more than 25% but less than
a) 1000 ppm b) 9200 ppm 50% of the tooth surface is given the Dean's score of:
c) 19500 ppm d) 12300 ppm a) 1 b) 2
(KCET-2011) c) 3 d) 4
110. Which of the following is least likely to be used as a pit (KCET- 07)
and fissure sealant: 122. The following study was conducted to check anticariogenic
a) Cyanocrylates b) Urethane effect of fluoride EXCEPT
c) Gold alloy d) BIS-GMA a) Dutch study b) Newburg-Kingston study
c) Evauston-Oak study d) Shoe-Leather survey
111. Maximum fluoride content is found in: (COMEDK-08)
a) Sea fish b) Tea leaves 123. The mechanism of action of APF is via following reaction
c) Human milk d) Coconut water products:
(AIIMS-06) a) Fluoridated hydroxyapatite
112. The prevalence of dental caries in a community in the b) Stannous trifluorophosphate
year 2000 and 2003 was 18% and 38% respectively. c) Dicalcium phosphate dihydrate (DCPD)
The incidence of caries for the same population in three d) DCPD + Calcium fluoride+ fluorapatite
years would be: (COMEDK-08)
a) 38% b) 20% 124. Fluoride:
c) 10% d) 56% a) Prevent dental caries b) Promote periodontal health
(AIPG-06) c) Both A and B d) None
113. "NALGONDA" technique of defluoridation was developed at: (AP-08)
a) Chennai b) Nagpur 125. The pH of a freshly prepared stannous fluoride solution
c) Nalgonda d) Bangalore is approximately
(KAR-04) a) 2.5 b) 4.5
114. The first community Water Fluoridation was carried out in: b) 6.5 d) 8 .5
a) 1945, Colorado b) 1945, Grand Rapids (KAR-2001)
c) 1945, Oak Park d) 1945, Evanston 126. Fluorides mostly deposited on the enamel surface at:
(AIPG-06, KCET-11) a) Outer surface b) At DEJ
115. Fluoride in any form is contraindicated in: c) Inner surface d) Inter prismatic substance
a) Chronic renal failure b) Osteoporosis (BHU-07)
c) Hypertension d) Thyrotoxicosis 127. School water fluoridation was first initiated in
(KCET-07) a) Seagrove, North Carolina
116. 'Crippling Fluorosis' is characterized by: b) St. Thomas virgin islands
a) Blockage of blood vessels c) Elk lake, Pennsylvania
b) Chipping of teeth d) Pike country, Kentucky
c) Rigid spine (COMED-10)
d) Calcification of blood vessels 128. The supplementation of fluoride to get maximum
(KCET- 07) cariostatic effect should start from?
117. Which is the first study on water fluoridation? a) 3 months after birth b) 6 months after birth
a) Grand rapids (Michigan) & Muskegon c) 9 months after birth d) Birth
b) Newberg & Kingston (PGI-08)
c) Brantferd & Sarnia d) Evanston & Park 129. Thin spiral interdental brushes are used in which type of
(KCET-07, AIPG-06) embrasures?
118, The lethal dose of fluoride for 70kg adult is: a) Type 1 b) Type 2
a) 32-64 mg of fluoride/kg body weight c) Type 5 d) Type 4
b) 50-60 mg of fluoride/kg body weight (KCET-10)
c) 30-70 mg of fluoride/kg body weight 130. The brushing technique indicated for young children is
d) 60-80 mg of fluoride/kg body weight a) Fones method b) Modified Stillman technique
(KCET-07) c) Modified Bass technique d) Charter's technique
119. Lethal dose of fluoride is: (KCET-10)
a) 250 milligrams b) 500 milligrams 131. Which of the following is the main reason for resistance
c) 5 grams d) 10 grams of enamel surface to demineralization?
(AP-07) a) Increased Phosphate content of surface layer
b) Decreased Magnesium content of surface layer

109) C 110) C 111) B 112) B 113) B 114) B 115) A 116) C 117) A 118) A 119) C 120) C 121) B
122) D 123) D 124) C 125) A 126) A 127) B 128) B/D 129) B 130) A 131) C
Dental ;lut.,e i======
c) Increased Fluoride content of surface layer c) Dental professional d) None of the above
d) Decreased Carbonate content of surface layer (KCET-10)
(PGI-08) 143. Simple method to evaluate the plaque after a thorough
132. An example for disclosing solution is brushing can be done by ?
a) Chlorhexidine b) Mercurochrome preparations a) Snyder Test b) Sand Paper Strips
c) Quaternary Ammonium compound c) Disclosing Solution d) Unwaxed Floss
d) Sorbitol (PGI-08)
(UPSC-09) 144. Sodium fluoride inhibits glycolysis by inhibiting the
133. Pit and fissure sealant used in primary prevention of enzyme
caries is a) Enolase b) Aldolase
a) BIS - GMA b) Duraphat c) Catalase d) Dehydrogenase
c) Fluor protector d) APF gel (AP-10)
(KCET-10) 145. Which topical fluoride agent makes use of thixotrophic
134. Temporary water hardness is removed by properties for better penetration into inter proximal
a) Filtration b) Storage areas?
c) Addition of bleaching powder a) Sodium fluoride solution
d) Boiling b) Fluoride foam
(KCET-10) c) Acidulated phosphate fluoride gel
135. Hopewood house study is on d) Fluoride varnish
a) people eating large amounts of sugar (KCET-2011)
b) people eating small amounts of sugar 146. Sorbitol acts in the prevention of dental caries by -
c) people not exposed to sugar a) Microorganisms lack the enzyme to metabolize sorbitol
d) people eating moderate amounts of sugar b) Lactic acid can not be produced
(COMEDK-09) c) Fermentation of sorbitol does not occur
136. The use of pit and fissure sealant is contraindicated in d) sorbitol is not a carbohydrate
a) Arrested caries b) Recalcitrant caries (COMEDK-09)
c) Remedial caries d) Rampant caries 147. Caries status in child is most likely suggested by
(BHU-2012) a) DMFT and past caries experience
13 7. Repeated application of sodium or potassium flouride b) Salivary flow and com position
to teeth of children significantly reduced their caries c) Frequency of sugar intake
prevalence. This was first demonstrated by d) Fluoride concentration in the area
a) Knutson b) Bibby (AIIMS-09)
c) Muhler d) Dean 148. Absorption of fluoride in child takes place mainly in
(KCET-2012) a) Intestine b) Lungs
138. Current recommendation for systemic flouride c) Oral cavity d) Gingiva
supplements for all children residing in areas where the (AIIMS-09)
water flouride deficient until they reach the age 149. Recommended dosage of fluoride mouth rinse for 12-14
a) 12 b) 14 year old is -
c) 16 d) 18 a) 10 ml 0.2% NaF twice daily
(COMED-2012) b) 10 ml 0.2% NaF once daily
139. First flouride varnish discovered in Europe has what c) 10 ml 0.2% NaF twice monthly
percentage of Naf d) 10 ml 0.2% NaF once monthly
a) 2.26% b) 5% (COMEDK-09)
c) 8% d) 10% 150. The procedure where carious tooth tissue is removed
(PGI-2011) by hand instruments, and the cavity is restored with a
140. How much iodine is required in pregnancy? restorative material that 'sticks' to the tooth is
a) 150 mg/day b) 250 mg/day a) Preventive b) ART
c) 350 mg/day d) 450 mg/day c) Type IV restoration d) Piit and fissure sealants
(AIPG-14) (KCET-09)
141. Which of the following vaccine is contraindicated in 151. Pulp capping comes under which level of prevention -
pregnancy? a) Primary b) Secondary
a) MMR b) Rabies c) Tertiary d) Both A and C
c) Hepatitis B d) None (KCET-09)
(AIPG-14) 152. First systemic fluoride electrode is given by?
142. School water fluoridation is considered as the primary a) Shyler and Armstrong b) Frant and Ross 1966
preventive service provided by the c) Estrond 1970 d) Duck and Morgan 1970
a) Community b) Individual (PGI JUNE-14)

132) B 133) A 134) D 135) B 136) D 137) B 138) A 139) B 140) B 141) A 142) A 143) C 144) A
145) C 146) A 147) A 148) A 149) C 150) B 151) B 152) B
, COMMUNITY DENTISTRY
~
687
V
153. In grand rapid survey (1945) of water fluoridation,
which city was used as control?
a) Kingston b) Muskegon
c) Oakpark d) Evanston
(PGI DEC-2013)
154. Primary consideration in any oral health preventive
programme?
a) Fluoride application b) Diet control
c) Plaque control d) Pit & fissure sealant
(PGI JUNE-2011)
155. Fluorides facilitates the precipitation into tooth
structure of fluorapatite from calcium and phosphate
ions present in saliva, replacing the following
a) Soluble salts containing manganese only
b) Soluble salts containing manganese and carbonate
c) Soluble salts containing carbonate only
d) Soluble salts of sodium
(COMEDK-15)

153) B 154) C 155) B


Dental ;lut.,e

4. PREVENTIVE DENTISTRY - ANSWERS

1. 'A' [Soben Peter 4th ed 280/ 5th ed 556)


The Certainly Lethal Dose (CLD) of fluorides is 32-64 mg per kg of body weight i.e, approximately 2.5-5 gms of fluoride in cases
of children and 5-10 gms, one single retained dose, in case of adults.

Management of acute fluoride toxicity


• Give calcium orally (milk)
• Induce vomiting (if dose is >5mg/kg
• Give orally soluble calcium in any form (milk, 5% calcium gluconate or calcium Lactate solution (>5 mg/kg <15mg/kg body
weight)
• In lO°lo calcium gluconate if the dose is >15mg / 1kg.

2. ' C' [Soben Peter 4th ed 255/ 5th ed 532)


Flouride varnish was first developed in Europe by Schimdt. They maintain the fluoride ions in intimate contact with enamel for
longer periods. The two commonly used varnishes are -
• Duraphat: It is the first flouride varnish and contains 22600 ppm fluoride as 5% NaF in a neutral colophonium base (NaF
varnish containing 2.26% fluoride in organic lacquer)
• Fluor Protector (Fluoride concentration is about 7000 ppm or 0. 70%)
• Carex: Contains a Lower fluoride concentration t han Duraphat {1.8% fluoride) and has efficacy equivalent to that of Duraphat.
• Ouraflour - 22.6 mg/ml.
Contains NaF in alcoholic suspension of natural resins with xylitol as sweetening agent.
• Cavity shield: 5% NaF in resinous base.

Rationale for using fluo ride varnishes:


After topical fluoride application, there is substantial leaching of absorbed fluoride from surface enamel. To prevent this
immediate loss, fluoride has been incorporated in varnishes that have ability to adhere to enamel for long periods. The retentive
and possible slow release of fluoride from these products increase the exposure time of the fluoride by several days, without
increasing chair-side time, and allow fluoride to be more permanently bound to the teeth.

3. 'D' [Soben Peter 3rd ed 265/ 5th ed 548-50)


The optimum recommended fluoride levels range from 0.7 t o 1.20 ppm.

4. ' C' [Soben Peter 4th ed 272/ 5th ed 542)


The Concentration fluoride in community water fluoridation is about 0. 7 to 1.2 Ppm and the recommended concentration of
fluoride in school wat er fluoridation is 3.5 to 4.5 PPm. (The currently recommended level is 4.5 to 6.3 PPm F) This upward
adjustment of fluoride is to compensat e for the reduced water in take by the child.

5. ' C' [Soben Peter 4th ed 258/ 5th ed 536)


Characteristic NAF SNF2 APF
PERCENTAGE 2% B°lo 1.23%
PPM FLUORIDE 9200 19,500 12,300
p, OF SOLUTION 7 (Neutral) - APF Solution-3, APF Gel 3-4
METHOD OF Knutson technique, a fo ur visit Paint on technique at 6-12 • Sol is applied by paint-on
APPLICATION procedure at weekly intervals month intervals. It has technique
at ages 3,7, 11 & 13 antimicrobial property, • Gel is .applied by tray technique
both at 6-12 month intervals.
TASTE Non-irritating and acceptable Metallic taste Acidic
PIGMENTATION No Causes pigmentation of tooth Causes etching of porcelain/
and silicate restorations Ceramic (COMED-14) restorations
STABILITY Stable Unstable due t o formation of Unst able in glass container.
Sn(OH) 2 rapidly. Stable in plastic cont ainers.
Caries reduction 30% 32% 28%
, COMMUNITY DENTISTRY
~

V
689

6. ' B' [Soben Peter 4th ed 237/ 5th ed 515 last paragraph] • The first fluoride dentifrice recognized by FDA - SnF2
Historical evaluation of fluorides {PGI JUNE- 2011) in 1955.
• The first fluoride dentifrice (CREST) approved by ADA
• MCKay identified that many of his patients in
was in 1964.
Colorado, USA area had permanent stain on
1901 their teeth; which was, known to the local
9. 'C' (Soben Peter 4th ed 238/ 5th ed 517]
inhabitants as 'Colorado stain'.
• He called the stain 'mottled enamel'. 10. 'C' (Soben Peter 4th ed 239/ 5th ed 517]
• MCKay found an article written by Or.Eager,
reporting the unusual occurrence of brownish 11. ' B' (Soben Peter 4th ed 278/ 5th ed 555]
1912 coloured stains among majority of the residents The supplemental fluoride dosage depends on fluoride
in Naples. Eager had termed these brownish concentration of drinking water and age of the child.
discolourations as "denti di chiaie"
• MCKay confirmed the presence of an unknown
1918 element in the water supply to be definite
causative factor for enamel mottling. Birth to 2 0.25 0 0
• Churchill, a chemist employed an aluminium 2 to 3 0.5 0.25 0
company, Alcoa (Aluminium Corporation of 3 to 14+ 1.0 0.5 0
America) identified the presence of fluoride in
excess in water samples from areas with mottled 12. 'A' (Soben Peter 4th ed 237/ 5th ed 516]
1931 enamel.
• In the same year, two more discoveries of 13. 'C' [Hiremath 1st ed 363]
fluoride in water from areas of endemic fluorosis
was done by Mr & Mrs. Smith in Arizona and by 14. 'A' (Check Explanation Below]
Velu H, Balozet L is France. Mercury produce should never be incinerated. No dangerous
• Trendley H Dean conducted "Shoe leather waste or amalgam is to be placed in the infectious waste
survey" among 21 cities (PGl-14) in ten "red bag". This would include extracted teeth with amalgam.
1931 All the forms of mercury waste should be collected into air
states of USA. This survey established fluoride
as the causative factor for mottling of enamel. tight container and be sent for recycling.
• Dean developed a standard system for
15. ' B' (Soben Peter 4th ed 259/ 5th ed 53 7]
1934 classification of dental fluorosis the mottling
index (Dean's index for fluorosis).
16. ' B' (Soben Peter 4th ed 263/ 5th ed 540]
• Dean et al discovered that lppm of water Naf Mouth rinses are commonly used at
1942
Fluoride reduces caries experience by 60%
• 0.2% Naf which contain 900 PPM of fluoride for weekly
• The world's first artificial fluoridation plant was use. (or)
started at Grand Rapids, USA. • 0.05% NaF which contain 225 PPM of fluoride for daily
• On January 25th, Sodium fluoride was added use.
to the Grand rapids (Michigan) with muskagon
1945
as control. This was historic occasion, because 0.05% Naf mouth rinse is recommended in orthodontics
for the first time a permissible quantity of a patients (PGI June-14) undergoing fixed orthodontic
beneficial dietary nutrient was added to the treatment.
community drinking water.
• WHO advocated that lppm of fluoride in 17. ' D' (Soben Peter 4th ed 240/ 5th ed 519]
1969 community was practical and effective public • Leafy vegetables such as cabbages, Lettuce and brussels
health measure. sprouts contain about 11-26 mg fluoride on a dry weight
basis.
7. 'A' [Soben Peter 4th ed 268-279/ 5th ed 545-46] • In tea plants, several hundred ppm concentration of
Type of flouridation system Component used fluoride have been reported.
Saturater system 4% NaF saturated solution • Dried fish such as Sardines, Salmon, Mackerel and other
fish contain about 20-40 ppm of fluoride on a dry weight
NaF or silicofluoride (most
Dry feeder system basis. The high fluoride level in fish is attributed to the
popular)
fluoride in the skins and in bones.
Solution feeder system Hydrofluosilicic acid
18. 'A' (Soben Peter 4th ed 259/ 5th ed 534]
8. 'C' (Soben Peter 4th ed 261/ 5th ed 539] When NaF is applied to tooth surface, there is initial rapid
• The first clinical trial of fluoride dentifrice: by Bibby in formation of CaF2 followed by drastic reduction in its rate
1942. and this sudden stop of entry of fluoride is termed as
Dental ;lut.,e i======
"Chocking off effect". Fluoride then leaches from calcium 1.5 - 3.0 ppm Dental fluorosis
fluoride. This CaF2 acts as a reservoir for fluoride release and
Severe dental fluorosis and a mild
reacts with the hydroxyapatite crystals to fluorhydroxyapatite. 3.0-8.0 ppm
skeletal fluorosis
19. 'B' (Check Q. No. 2] 8.0 ppm or more Severe form of skeletal fluorosis.

20. 'C' (Soben Peter 4th ed 266/ 5th ed 543 last paragraph] 30. 'C' (Sob en Peter 4th ed 255/ Hi re math 1st ed 359]
The concentration of fluoride in community water based
on climatic conditions and body weight is calculated by 31. 'B' [Soben Peter 4th ed 317/ 5th ed 540]
Galagan and Vermillion formula: Ppm of F = 0.34/E where 0.2% or option 'A is the concentration of NaF in weekly
mouth rinse.
E= - 0.038 + 0.0062 x temperature of area in °F.
32. 'A' (Soben Peter 4th ed 259/ 5th ed 534)
Temperature in ° C Recommended ppm Anticaries effect of fluoride:
<18°( 1.1 to 1.3 1. Increased enamel resistance:
19 - 26.5°( 0.8-1.0 a) By decreasing solubility
>26. 7°( 0.5-0. 7 b) By improving crystallinity
c) Desorbs proteins and bacteria
21. 'C' (Soben Peter 4th ed 263/ 5th ed 540) d) Lowers free surface energy
• 2% NaF contain 9000 ppm of flouride
• 0.05% NaF contains 2/0.05X9000 = 225 ppm of F 2. Remineralization of incipient lesions
3. By antibacterial action
22. 'D' (Soben Peter 4th ed 265-66/ 5th ed 540-50) 4. Altering morphology of teet h (systemic action)

23. 'B' (Soben Peter 4th ed 256/ 5th ed 533] la) By decreasing solubility:
% reduction of
When hydroxyapatite crystals are exposed to low
Method concentration of fluoride, they are converted to flourapatite
dental caries
crystals, which are less soluble in oral acids.
Community water fluoridation and
50-65%
dietary fluoride supplementation lb) By improving crystallinity:
School water and professionally According to void theory, hydroxyapatite crystals contain
35-40% missing hydroxyl ions, fluoride ions will fill up these
applied topical fluoride
occasional void and stabilize the structure by providing
Self applied topical fluoride 20-50%
stronger hydrogen bonds.
Professionally applied topical fluoride 30-40%
Fluoride dentifrices 20-35% le) Desorbs proteins and bacteria:
Higher fluoride concentration results in desorbing proteins
and bacteria. Thus bacterial colonies and plaque formation
Effectiveness of systemic fluoridation in prevention
are minimized.
of caries is? (PGI Dec-2011)
Ans: 60% ld) Lowers free surface energy:
When tooth enamel is treated with metal fluoride solutions
24. 'D' (Soben Peter 4th ed 255-56/ 5th ed 533-3 7) (Stannous, amine), the free surface energy on the enamel
Varnishes increase the time of contact between enamel surface is lowered. This decreases t he wettability of the
surface and topical flo,uride agents favouring the deposition enamel and thus results in reduced plaque formation.
of more permanently bound flourapatite.
2. Remineralisation of incipient carouse lesions:
25. 'C' [Soben Peter 4th ed 265/ 5th ed 533) Fluoride effectively reduces the solubility of enamel by
promoting precipitation of hydroxyapatite and by preventing
26. 'A' (Soben Peter 4th ed 278) the formation of more soluble calcium phosphate materials.

27. 'B' (Check Explanation Below] 3. By antibacterial action:


The permanent 3rd molars are not involved because their At higher concentrations, fluoricle is bactericidal, because
calcification begins after 8 yrs. of its action on two bacterial enzymes enolase and
phosphoglucomutase.
28. 'D' (Soben Peter 4th ed 278)
4. Altering morphology of teeth:
29. 'B' (Soben Peter 4th ed 351/ 5th ed 556] Systemic administration of fluoride, results in shallow
occlusal grooves, lower cuspal height and smaller size teeth.
~ ii+i:Nl@-
o.1-1.2ppm ~
The teeth show more rounded appearance resulting in lesser
chances of plaque formation.
, COMMUNITY DENTISTRY
,......__,,_

V
691

33. ' C' [Soben Peter 4th ed 265/ 5th ed 534] 44. 'C' [Soben Peter 3rd ed 330]
Below 4 yrs Fluoride tooth paste is not recommended
34. ' B' [Soben Peter 4th ed 258/ 5th ed 536]
Brushing once daily with fluoride paste
4-6 yrs
35. ' C' [Soben Peter 4th ed 280/ 5th ed 542] and twice without paste.
lmg/L of fluoride means lPPm of fluoride Twice daily with fluoride paste and once
6-10 yrs
without paste
36. 'A' [Soben Peter 4th ed 265-66/ 5th ed 543] Brushing thrice daily with fluoride
AbovelO yrs
toothpaste.
3 7. ' B' [Soben Peter 4th ed 255]
45. ' D' [Soben Peter 4th ed 258/ 5th ed 536]
38. 'N [Soben Peter 5th ed 543] Brudevold's solution or APF solution is prepared by dissolving
700 ppm = 700mg per 1000 gm or 1kg of paste 20 gms of NaF in 1 Litre (2% NaF) of 0 .1M phosphoric acid
= 0. 7mg/ lgm of tooth paste. and to this 50% hydro fluoric acid is added to adjust the pH
at 3.0 and fluoride ion concentration at 1.23%
1 gm tooth paste (1000 ppm) contain fluoride
a) .25 mg b) .5 mg (PGI- 14) APF gel is prepared by adding gelling agents like
c)lmg d).13mg methylcellulose and hydroxyl ethyl cellulose and the pH is
adjusted between 4-5.
39. 'N [Sob en Peter 3rd ed 334]
46. 'A' [Soben Peter 4th ed 258/ 5th ed 543-44]
AGE REQUIREMENT
For children below 2 yrs 0.25 mg 47. ' C' [Soben Peter 4th ed 278/ 5th ed 555]
2-3yrs 0.5 mg Fluoride tablets are available in 0.55, 1.1, and 2.2 mg tablets
3yrs-13yrs 1.0 mg and they yields 0.25mg, 0.5mg, 1mg of F respectively.

> 13 yrs Fluoride need not be given. The daily recommended dose of Fin children is
• < 2 yrs 0.25 mg
40. ' B' [Soben Peter 4th ed 267/ 5th ed 550]
• 2-3yrs 0.5 mg
41. 'N [Soben Peter 4th ed 261] • 3yrs - 13yrs 1.0 mg.

42. 'N [Soben Peter 4th ed 244-45/ 5th ed 526] Since the child is 4 yrs, a 2.2mg fluoride tablet that yields 1
mg of F- which provides fluoridation both systemically and
43. 'A' [Soben Peter 4th ed 257/ 5th ed 534] topically is given.
Fluoride
The advantage to chewable fluoride tablets is that they
J, provide both topical and systemic benifits.
Absorbs mainly in stomach and upper intestine.
Absorption is passive in nature and reaches Below 18 months of age, fluoride tablets are not given since
maximum plasma concentration within 30 minutes. the child cannot swallow. Fluoride is given in form of syrups
J, J, rather than chewable tablets.

50% excreted in urine. 50% taken by mineralized


48. 'D' [Soben Peter 4th ed 249-50/ 5th ed 528-29]
About 10-25% excreted tissues
in faeces. J,
49. 'A' [Soben Peter 4th ed 98/ Hiremath 2nd ed 543]
Bone: Approximately 99% of
all the fluoride in the body 50. ' B' (Soben Peter 4th ed 243/ 5th ed 521]
About 30% of fluoride is found in calcified tissues.
Maximum amount of fluoride absorption occurs from upper
is excreted in 6 hrs and Accretion is a process GIT followed by small intestine.
60% of the fluoride is where most of the fluoride
excreted in 24 hrs. is buried within the mineral 51. 'A' [Soben Peter 4th ed 265/ 5th ed 534]
crystallites during the period
of crystal growth.
52. ' B' [Soben Peter 4th ed 279/ 5th ed 557]
J, J, In cases of acute fluoride toxicity, orally soluble calcium
• Saliva: 0.05-0.007 ppm • Enamel: 2200-3300 ppm in form of milk or calcium g luconate or calcium lactate
solution is given. The calcium ions compete for fluoride ions
• Sweat: 0.5-0.067 ppm • Dentin: 200-300 ppm
and reduce its absorption.
• Breast milk: 0.4 uM • Cementum: 4400 ppm
• Plaque: 15 - 64 ppm • Pulp: 100-650 ppm 53. 'C' [Soben Peter 4th ed 279/ 5th ed 557]
Dental ;lut.,e i======
54. 'A' [Soben Peter 4th ed 237-39/ 5th ed 518] This happens when the solutions gels or pastes; containing
higher concentration of fluoride to tooth surface and low
55. 'B' [Soben Peter 5th ed 195] pH. Enamel defects such as open carious lesions, incipient
caries (white spots), microcracks, hypomineralized areas;
56. 'C' [Soben Peter 4th ed 267 / 5th ed 550] and the margins of some restorations acquire larger amounts
of fluoride than sound enamel because of their greater
5 7. 'A' [Soben Peter 4th ed 273/ 5th ed 558] porosity and surface. Enamel of e·rupting and newly erupted
Nalgonda technique is useful both for domestic and teeth is most caries susceptible until secondary maturation
community water supp lies. The technique involves the is completed i.e., more than two years of eruption.
addition of aluminate or lime, bleaching power and filter
alum to fluoride water. 62. 'B' [Soben Peter 4th ed 278/ 5th ed 554]
Option '/\ Brushing is a complex skill, so until children
58. 'D' [Soben Peter 5th ed 556] develop adequate dexterity, usually around t he age of 10 to
Enamel fluorosis is a developmental phenomenon and once 12 years, an adult should perform flossing on the child.
the crowns of the teeth are formed no further fluorosis can
be induced by additional intake of fluoride or by posteruptive Option 'C' fluoride mouth rinses should not be used by
topical applications of fluoride. children less than 6 years of age who are not capable
of rinsing adequately because lack of reflex control of
59. 'B' [Hiremath 1st ed 400] swallowing.
Pits and fissures are enamel faults that harbour food and
microorganisms are the most important anatomical features Option 'D' sodium fluoride dentifrices failed to show
leading to the development of occlusal caries. Even though significant caries reduction. The world's most widely used
fluorides are highly effective in reducing the smooth surface fluoride is combination of sodium monofluorophosphate
caries, unfortunately they are not effective in protecting the (MFP) and insoluble metaphosphate. Current ly available
occlusal pits and fissures where 90% of all carious lesions dentifrices contain 900 or 1000 ppm (0. 76% sodium MNF)
occur. for adults whereas for children they are available at about
500 ppm fluoride concent rations.
60. 'D' [Soben Peter 4th ed 278/ 5th ed 55 7-58]
Skeletal fluorosis ocw rs from ingestion of water containing Also the accidental ingestion of dentifrice is greater among
fluoride levels over 8ppm for long periods of time. In India young children . The average amount ingested varies from
the disease was first reported by Vishwanathan in residents about 35% in children aged 2-4 years and only 3% in adults.
of Madras. However, shortt was the first to identify the As a precaution, parent s should monitor the t ooth brushing
disease as "fluorosis" in individuals in Nellore district, A. P. of very young children to prevent excessive dentifrice use
and ingestion of fluoride.
The fluoride dosage necessary to produce pathological
skeletal fluo rosis is estimated to be 20-80 mg/day for a 63. 'A' [Soben Peter 4th ed 264, 256/ 5th ed 533, 542]
period of 10 to 20 years. In India, it is most commonly seen Fluoride gels can be applied even in office by dentist. As
in Punjab. Neurological Manifestations of fluorosis occur if trays are used for applying the gel solutions, it is possible to
the duration of ingestion is more than 20 years. treat two or four quadrants simultaneously and this results
in substantial saving of time on the other hand, the fluoride
Other symptoms of skeletal fluorosis: application is done first on the lower arch and then on
• Severe pain in back bones, joints, stiffness in joints and the upper arch, using a single tufted small brush starting
spine. In its severest form it may severely immobilize with the proximal surfaces. The varnish can be applied in
the patient, which is known as crippling fluorosis . the interproximal areas by using small cotton pellet or
microbrush or floss. Compared t o regular gels, t hixotropic
• Outward bending of legs and hands is seen in advanced
gels are more easily forced into the interproximal spaces
stages. This is known as knock-knee syndrome.
than conventional gels.
• Pregnant, lactating mother and children are most
vulnerable. Calcium fluoride is more toxic to fetus than 64. 'A' [Hiremath 17th ed 354, Tab 35-2]
sodium fluoride. The effects of Systemic fluoride effects are longer and
throughout life (fluoride is incorporated in the tooth
61. 'C' [Soben Peter 4th ed 252/ 5th ed 524] structure during developing stage) whereas the effect of
In newly erupted teeth, enamel is porous and the porous topical fluorides is for shorter duration and does not last
enamel appears to absorb more fluoride. Acquisition of for long (fluoride is lost from the tooth surface after the
fluoride by the enamel surface appears to continue as long application). Hence requires repeated application.
as the tissue remains porous.

Penet ration of fluoride into fully mineralized enamel is slow


because of porosity as low as 0.1%. Because of this, the
fluoride application to fully matured enamel req uires the
creation of porosity or destruction of the apatite lattice.
, COMMUNITY DENTISTRY
~

V
693

65. 'D' [Textbook of Primary Preventive Dentistry by Norman. 71. 'A' [Soben Peter 4th ed 237/ 5th ed 515)
O. Harris 61h ed 185-186)
The history of community water fluoridation is categorised 72. 'C' [Soben Peter 4th ed 351/ 5th ed 556)
into four separate periods or phases.
73. 'B' (Soben Peter 4th ed 257/ 5th ed 535)
Phase Includes SnF2 is not stable due to rapid formation of Sn(OH)2. So
fresh solution of stannous fluoride is prepared for each
• Noticing Colorado stains by MCKay. patient.
Termed Mottled enamel.
• Churchill demonstrated an association 74. 'B' (Soben Peter 4th ed 273/ 5th ed 558)
between high levels of naturally Nalgonda technique was developed by the national
Clinical
occurring fluoride in water and environmental engineering research institute (NEERI) at
discovery phase
mottled teeth. Nagpur in 1974. The process comprises Addition in sequence
• Observation by McKay and Black that of sodium aluminate (filter alum), lime and bleaching powder
people with fluorosis experienced less to fluoride water followed by flocculation, sedimentation,
decay. and filtration . The technique is extremely useful for both
domestic as well as for community water supplies.
• Deans 21-City study, which showed
Epidemiological
that the more fluoride in water, the
phase. 75. 'D' (Soben Peter 4'h ed 276)
fewer dental caries in children.
Milk fluoridation was introduced by Zeigler in Swiss city of
• Characterized by series of community Winterthur in 1953. In 1971, Dr Edgar Borrow established
d i nical trials that compared the the Borrow foundation with the aim of promoting the use of
dental and medical results following milk as a vehicle for fluoride for the benefit of children's oral
the deliberate addition of fluoride health. The first community based milk fluoridation scheme
Demonstration to the drinking water in four cities was introduced in 1988, in Bulgaria.
phase paired with four control cities.
• Grand rapids was the first city in the 76. ' B' [Soben Peter 5th ed 551)
world to fluoridate its drinking wat er Salt fluoridation was introduced by Wespi in Switzerland
with Michigan as control city. in 1948. Switzerland was the first country to adapt salt
fluoridation . The recommended level of fluoride can be kept
Technology • Includes implementation of
transfer phase fluoridation in many large U.S. cities. at 200, 250 and 350 mg of F per kg salt.

Jamaica is the only country where virtually all destined salt


66. 'C' [Soben Peter 4th ed 245/ 5th ed 521)
for human consumpiton on the island has been fluoridated.
The minimum amount is 200 mg/kg.
67. 'B' [Soben Peter 4th ed 2594/ 5th ed 537)
8% SnF2 contains 19500 PPM (KCET-11) of F. = 1.95% of F
77. 'A' (Check Explanation Below]
Fluorine has played a significant role in insect control since
68. 'A' [Soben Peter 4th ed 249-50/ 5th ed 196)
about 1896 when sodium fluoride and various iron fluorides
The enzyme most sensitive to fluoride during the formation
of acid is enolase and in the formation of polysaccharides is were patented in England as insecticides. Sodium fluoride
was used in the United States for cockroach control before
phosphoglucomutase. Fluoride acts on enolase enzyme and
thus prevents the conversion of glucose to lactic acid. 1900 and was introduced in 1915 for the control of poultry
lice. However, the use of fluorine insecticides did not become
69. 'C' [Soben Peter 3rd ed 334] general until the 1930's when the disadvantages of arsenical
residues on food crops became apparent and the inorganic
fluorine compounds were introduced as safer substitutes.
70. 'D' (Soben Peter 4th ed 244-45/ Hire math 1st ed 365]
The fluorosis tends 1to be greater in permanent teeth than
78. 'A' [Soben Peter 4th ed 242/ Hiremath 1st ed 347)
primary teeth because of
• Placenta acts as partial barrier i.e, placenta acts 79. 'A' (Soben Peter 4th ed 350-51/ 5th ed 455)
as regulator of fluoride especially when the
concentration of fluoride reaching fetus is greater 80. 'A' [Ref:- The concentration of fluorine in different
than maternal levels. (AIIMS May-13, 14) diary products made from contaminated milk. SCi total
• Much of mineralization of primary teeth occurs before environ. 1986; 50:183-9 /Textbook of biochemistry for
birth. medical students by Vasudevan 6th ed Pg 430)
• Period of enamel formation for primary teeth is shorter Fluoride rich sources are sea fish, cheese, Tea, Jowar.
than for permanent teeth. Fluorosis is highly prevalent in areas where Jowar is staple
diet.
• Enamel of primary teeth is thinner than that of
permanent teeth.
81. 'B' [Soben Peter 4th ed 241/ 5th ed 520)
Dental ;lut.,e i======
The decreasing order of flouride content in food items is tea • Upon withdrawal of sugar rich foods, the increased
leaves, dried sea fish, turmeric etc. caries activity rapidly increases.
• Caries lesions may continue to appear despite the
82. ' C' [Soben Peter 4th ed 258/ 5th ed 535) avoidance of refined sugar and maximum restrictions
When SnF2 is applied in low concentration, tin-hydroxy- of natural sugar.
phosphate is formed, which gets dissolved in oral fluids
and is responsible for the metallic taste after t opical
2) HOPEWOOD HOUSE STUDY:
application of stannous fluoride.
a) Investigators are Harris and Sullivan (1942-1967).
At very high concentrations, calcium tri-flouro-stannate b) Hopewood house is an institution for children of
gets formed along with tin-tri-flourophosphate. The tin- age 7 to 14 years
tri-flourophosphate is responsible for making the tooth
structure more stable and less susceptible to decay. c) The absence of meat and a rigid restriction of
refined carbohydrate were the two principal featu res
Calcium fluoride is also the end product of both at low of Hopewood house diet. Also the water fluoride
and high concentrations. The calcium fluoride so formed concentration was non-significant and the oral
further reacts with hydroxyapatite and a small fraction of hygiene was poor t hroughout the study.
flourhydroxyapatrite a lso get s formed . d) At the end of study, the 13-year old children had
a mean DMFT per child of 1.6. The corresponding
83. 'A' [Soben Peter 4th ed 94-95) figure for the general child population was 10.7.
Diet studies related to dental caries are
e) 53% of the children at t he Hopewood house were
1. Vipeholm study caries free whereas only 0.4% of the 13-years old
2. Hopewood house study children of the general popu lation are caries free.
3. Turku sugar study
f) Conclusion of the study: in institutionalized
4. Hereditary fructose intolerance children, dental caries can be reduced by
5. Tristan da Cunha (effect of food habits in causing dental carbohydrate restricted diet even in presence of
caries in migrated people) unfavourable oral hygiene.

1. VIPEHOLM STUDY: 3) TURKU SUGAR STUDY


a) Described by Gustaffson et al in 1954, and a) Investigators are Scheinin and Makinen et al in
summarized by Davies in 1955. 1975 for a period of 25 months.

b) It was a five year investigation of 436 adult inmates b) Investigated the effects of sucrose, fructose and
of Vipeholm hospital of Sweden, an institution for xylitol on caries development.
the mentally challenged.
c) Conclusion
c) The inmates are divided into 7 groups based on • Sucrose - highly cariogenic
diet given • Frucose - Cariogenic as sucrose for first 12
• Control group months but become less at the end of 24 months
• Sucrose group • Xylitol - Anticariogenic with 85% reduction in
• Bread group dental caries

• Caramel group
4) TRISTAN DA CUNHA
• Chocolate group
a) It is an island in South Atlantic area
• 8 Toffe and
• 24 Toffe b) lnvesligalor is Fisher

c) Because of volcano, entire community moved to England.


d) The institution diet was nutritious, but contained little
sugar, with no provisions for between meal snacks d) Fisher carried dental examination in 1932, 1937,
1953 in island and again in England in 1962 and
e) Main conclusions: again 1966 in the island when the people of Tristan
• An increase in carbohydrate (mainly sugar) definitely de Cunha returned to their island from England
increases the caries activity.
e) The result showed caries prevalence in first molars
• The risk of caries is greater if the sugar is consumed
of 6 - 19 years old was 0% 1932 & 1937 (before
in a form that will be retained on the surfaces of
migration to England) but increased to 50% in 1962
teeth.
and 80% in 1966 (After migration to England).
• The risk of sugar increases, if the sugar is consumed
between meals..
, COMMUNITY DENTISTRY
,......__,,_

V
695

5) HEREDITARY FRUCTOSE INTOLERANCE (HFI) soluble calcium and phosphate to fluoride containing tooth
a) HFI is due to reduced levels of enzyme Fructose- paste or mouth-rinse might be expected to enhance the
1-phosphate aldolase, an enzyme required for efficiency of fluoride mediated remineralization.
metabolism of sucrose or fructose.
93. ' D' [Soben Peter 4th ed 259/ 5th ed 534]
b) Persons affected with HFT tend to avoid any food
that contains fructose or sucrose, because the 94. 'A' (Soben Peter 4th ed 265/ 5th ed 534)
ingestion of these foods cause symptoms of nausea, NaF is applied by Knutson technique, in which 2% NaF solution
vomiting, excessive sweating and even coma. is applied on air-dried teeth and are allowed for 3 to 4 minutes.

c) Newbrun in 1969 tabulated the caries prevalence of SnF2 is applied by paint on technique in which fresh ly
31 persons with HFI and found that dental caries prepared SnF2 Solution is continuously applied.
prevalence was extremely low.
APF sol is applied by paint on technique and APF gel is
The study on diet and dental caries in which sucrose applied by tray technique. The fluoride is applied repeatedly
was replaced by xylitol was (KERALA-2015) through out the four minute period.
a) Hopewood house study b) Vipeholm study
c) Tristan da Cunha study d) Turku Sugar st udy 95. ' B' [Soben Peter 4th ed 441/ 5th ed 562)
Prophylactic odontotomy, advocated by Hyatt, is more
84. ' B' [Soben Peter 4th ed 249/ 5th ed 543] invasive procedure of fi lling the fissures with silver or copper
Systemic water fluoridation is reduces caries mainly in oxyphosphate cement as sooni as the erupted and then
smooth surface carries and the reduction in caries would be later, when they were fully erupted a small occlusal cavity
50 to 70%. The only countries with 100% water fluoridation is prepared and filling it with amalgam. This procedure is
are Ireland, Hongkong, and Singapore (PGI JUNE- 12). replaced by less invasive / non-invasive techniques of
occlusal sealants.
85. 'A' [Soben Peter 4th ed 262/ 5th ed 539)
Combination of sodium monofluoro phosphate and insoluble Other techniques:
metaphosphate abrasive system is the world's mostly used • Bodecker suggested widening the fissures mechanically
com bi nation in fluoride dentifrice. This is because of high so that they would be less retentive to food particles
compatibility of MFP is on with a wider variety of abrasives and called it as fissure eradication.
and also it does not required acid PH. MFP dentifrices have • Later in 1955, Buonocore introduced a method adhering
a neutral or slightly alkaline PH and do not stain enamel resin to an acid et ched enamel surface of newly erupted
surfaces or the margins of restorations. teeth.
86. 'A' [Soben Peter 4th ed 273/ 5th ed 558) 96. ' B' [Soben Peter 4th ed 441/ 5th ed 562)
87. ' D' [Soben Peter 4th ed 259/ Hiremath 1st ed 351) 97. 'B' [Soben Peter 4th ed 441 last Line]
Visible light is cured of 430-490 nm wave length. They are
88. 'C' [Soben Peter 4th ed 275/ 5th ed 551) either unfilled (usually white) or filled (usually clear).
89. 'C' [Soben Peter 3rd ed 280-81/ 4th ed 244-45/ 5th ed 1st generation sealants Uses ultraviolet light
522) 2nd generation sealants Chemically cured resins
Fluoride exists in both ionic and bound forms in plasma
with bound form being, present in longer quantity. The ionic 3rd generation sealants Uses visible light
form is also called as inorganic form and is asymmetrically 4th generation sealants Fluoride containing sealants
distributed in blood between plasma and the blood cells.
98. 'A' (Soben Peter 4th ed 98, Tab 2/ 5th ed 292)
90. 'C' (Soben Peter 4th ed 23 7/ Fluorides in caries
prevention by Murray 3rd ed 325) 99. ' B ' [Hiremath 1st ed 403)

91. ' D' (Soben Peter 4th ed 275/ 5th ed 551) 100. 'A' (Soben Peter 4th ed 443/ Hire math 1st ed 403]
The etching time varies according to concentration of
92. ' B' [Soben Peter 4th ed 249/ 5th ed 529) orthophosphoric acid and in accordance with manufacturer's
Saliva provides a natural source of calcium and phosphate instructions. Some books say that the etching time is 20-60
ions for remineralisation. However in absence of fluoride, seconds while some authors recommend 35-40% phosphoric
saliva is not a very effective remineralizing medium. It has acid for 20 seconds for permanent teeth and 30 seconds for
been shown that even trace concentrations of fluoride ions primary teeth.
are effective in promoting calcium hydroxyapatite formation
from super saturated solutions of calcium and phosphate. For 101. 'A' (Soben Peter 4th ed 441/ Hiremath 1st ed 404, 407)
this reasons, fluoride is added to toothpastes, mouth-rinses, In some studies, sealants have been purposively placed
and drinking water as an anticaries agent. The addition of over small, overt initial carious lesions. When compared
Dental ;lut.,e i======
with control teeth, many of the sealed teeth have been Compared to total amount of sugar ingested, physical form
diagnosed as sound after 3 to 5 years. This is due to the of carbohydrates and frequent intake of sugars are much
seal that does not permit the movement of fluids or tracer more important in cariogenicity. Frequent intake of sucrose
isotopes between the sealant and the tooth. for atleast 60 - 100 minutes/day is associated with the
formation of Rampant caries.
102. 'B' [Soben Peter 4th ed 441/ Hiremath 1st ed 407)
Sealants mainly act by physical obstruction of pits and grooves. 107. 'D' [Soben Peter 4th ed 273/ 5th ed 558]
This prevents the penetration of fermentable carbohydrates Alumina is an important component in Nalgonda technique
and so the remaining bacteria cannot produce acid. of defluoridation.

103. 'B' [Soben Peter 5th ed 557) 108. 'D' [Soben Peter 4th ed 265/ 5th ed 534]
Management of acute fluoride toxicity In Knutson technique of NaF application, treatments are
given in a series of 4 appointments. Teeth are isolated by
Less than 5 mg/ Giive oral Calcium / milk to relieve GIT quadrant or by half mouth and then 2% NaF solution is
Kg symptoms painted on the air-dried and pumice cleaned teeth so that
• Admit to hospital. Empty the all the surfaces are visibly wet. Then the teeth are allowed
Less than 5 mg/ stomach by inducing vomiting. to dry for 3 to 4 minutes. A second, third and fourth fluoride
Kg to< 15 mg/ • Give oral calcium (milk or 5% application is scheduled at int ervals of approximately one
Kg calcium gluconate or calcium lactate week. This four visit procedure is recommended for ages of
solution). 3, 7, 11 and 13 years, coinciding with the eruption of
different groups of primary and permanent teeth.
• Admit to hospital immediately
• Induce vomiting and begin cardiac 109. ' C' [Soben Peter 4 1h ed 260/ 5th ed 537)
monitoring. Stannous fluoride has been used at 8% and 10%
• Administer IV 10ml of 10% calcium concentrations in solutions equivalent to 2 and 2.5%
> 15 mg/Kg fluoride. Although the 10% solution is usual led used for
gluconate.
body weight adults and 8% for children, there is no evidence of an actual
• Adequate urine output should be
maintained using diuretics clinical difference between two.
• Electrolytes, especially calcium and 110. ' C' (Soben Peter 4th ed 442/ 5th ed 563)
potassium should be monitored. Materials used as sealants:
a) Cyanoacrylates
104. 'C' [Soben Peter 5th ed 292] b) Polyurethanes
Option 'A' is a part of secondary prevention while option 'B' c) Oimethacrylates
& 'D' are examples of tertiary prevention. d) Glass ionomer
For protection of personnel and patients in dental care
111. 'B' [Soben Peter 4th ed 242/ 5th ed 520)
settings, medical gloves always must be worn by DHCWs The dried tea leaves contain 100 - 400 ppm of fluoride. On
(Dental patents and Dental Health Care workers) . Non- an average tea leaves contain 98 ppm of fluoride. Fluoride
sterile gloves are appropriate for examinations and sterile concentration of human milk ranges from 6 - 12 mg/ml. The
gloves are used for surgical procedures. Surgical gloves or sea fish contain fluoride in the range of 10 - 20 ppm.
examination gloves slhould not be washed before use nor
they should be washed for reuse. Washing of gloves may 112. ' B' [Check Explanation Below]
cause "WICKING" i.e., penetration of liquids through The prevalence of dental caries in year 2000 and 2003 was
undetected holes in gloves. 18% and 38%. The incidence or the number of new caries
for the same population in three years would be 20% (38%
105. 'C' [Soben Peter 4th ed 263/ 5th ed 540] - 18%).
Sodium fluoride rinses are intended to be used by forcefully
swishing 10ml of the liquid around the mouth for 60 secs. 113. 'B' [Soben Peter 4th ed 273/ 5th ed 558]
before expectorating it.
114. ' B' [Soben Peter 4th ed 266/ 5th ed 543]
106. 'D' [Soben Peter 4th ed 92-93]
The first compound added in Grand rapids was sodium
American association of pediatric dentistry supports the fluoride.
use of alcohol-based sugar substitutes such as xylitol as a
part of preventive strategy aimed specifically at long term 115. 'A' [Soben Peter 4th ed 247-48/ 5th ed 526)
caries pathogen suppression and caries reduction in higher- The chief organ of excretion of fluoride is kidney (NEET-
risk populations. Alcolnol based sugar substit utes cannot be 2013) by the mechanism of simple passive diffusion.
metabolized by plaque bacteria and thus exhibits anticaries The renal clearance of fluoride is 50% of that of creatinine.
effect. The renal clearance rate of fluoride is about 35mljmin.
, COMMUNITY DENTISTRY
~

V
697

In case of chronic renal fai lure, there will be increased concentration is therefore always relatively high at the
plasma and bone fluoride levels leading to fluoride toxicity. enamel surface compared with the interior. Fluoride
So fluoride is contraindicated in patients with chronic renal concentration also varies systematically from place to place
failure. in the tooth surface and the pattern changes with age.
In newly erupted teeth (anterior teeth) surface fluoride
116. 'C' [Soben Peter 4th ed 281/ 5th ed 558] concentrations were found to be the highest in the first
"Crippling fluorosis" is seen in several form of dental and formed enamel near the incisal edge and decreased steeply
skeletal fluorosis when the water fluoride level is more than towards the more recently formed cervical region. In older
8ppm or more. In "Crippling fluorisis", the spine becomes teeth, this pattern is inverted due to wear.
rigid and the joints stiffen, virtually immobilizing the patient.
127. 'B' [Soben Peter 3rd ed 350 / 4th ed 350]
117. 'fl [Sob en Peter 4th ed 266/ 5th ed 543] School water fluoridation studies
118. 'A' [Soben Peter 4th ed 279-80/ 5th ed 556] 1954 Virgin islands
The safety tolerated dose (STD) of fluoride for 70kg adult is 1958 • Pike County, Kentucky
8-16 mg F/kg body weight. The certainly lethal dose for 70kg • Elk lake, Pennsylvania
adult is 32-64 mg F/ kg body weight, which is approximately
5-6 gms. 1968 • Seagrove, North Carolina

119. 'C' [Soben Peter 4th ed 279/ 5th ed 557] 128. 'B/0' [Soben Peter 4th ed 278/ 5th ed 555)
According ADA council in scientific affairs,
120. 'C' [Soben Peter 4th ed 266/ Fluoride varnish for recommendations, new dosage schedule approved in
community based caries prevention in children by April 1994 are
Peterson and pakho mov Pg 7] Age Flouride Cone. in drinking water (ppm)
In 1973 Cousins and mazze suggested that a plasma level of
850 mg/ml is nephrotoxic. < 0.3 > 0.3 < 0.6 > 0.6
0-6 - - -
The normal plasma fluoride concentration of water fluoride mont hs
level of 1.2ppm is 0.038 ppm or 2µm. The nephrotoxic 6 months - 0.25 mg/day - -
threshold level for fluoride has been found to be 850ng/ml 3 yrs
which is equal to 850-950µg/dl.
3 - 6 yrs 0.5 mg/day 0.25 mg/day -

121. 'B' [Soben Peter 4th ed 350/ 5th ed 455] 6 - 16 yrs lmg/day 0.5 mg/ day -
Ref. Q.No. 79

122. ' O' [Soben Peter 4th ed 23 7/ 5th ed 517] According to recommendation of American Academy
Though shoe-leather survey, fluoride was established as the of Pediatrics
causative factor for mottling of enamel.

123. 'D' [Soben Peter 4th ed 258/ 5th ed 536, 537] < 0.3 > 0.3 < 0.6 > 0.6
When APF is applied on the teeth, it initially leads to Birth to 2 0.25 0 0
dehydration and shrinkage of HA crystals and results in 2 to 3 0.5 0.25 0
formation of Dicalcium phosphate di hydrate (DCPD) and CaF2.
3 to 14+ 1.0 0.5 0
The DCPD is highly reactive with 'T' and leads to formation
of "FA". CaF2 that forms is partly lost by dissolution in the
saliva, but there is evidence that a substantial amount is But according to " Fluoride and dental caries by Ernest
retained, probably by transformation to FA. Newbrun 1•t ed Pg 69", in order to provide maximum benefits
to both deciduous and permanent teeth, it is recommended
124. 'C' [Check Explanation Below] that fluoride supplementation by tablets, drops or lozenges
The relation between fluoride and dent al caries is widely begin as soon after birth as possible and continue at least
established. But the relationship between fluoride and through age 15. According to this explanation, option 'D' is
periodontal health is controversial and its beneficial action more appropriate than option 'B'.
on periodontium could be due to its antibacterial action
against organisms in plaque. 129. 'B' [Text book of Community Dentistry by Satish Chandra
and Shaleen Chandra 1'1 ed 131)
125. 'A' [Q.No. 5/ Hiremath 1st ed 358] The interdental cleaning aid is selected depending on the
relative size of the embrasure. For healthy interdental papilla
126. 'A' [Sob en Peter 4th ed 245/ 5th ed 524] occupying complete embrasure, superfine thin dental floss is
Fluoride distribution is not uniform across the thickness selected. If about 50% of the embrasure is occupied, t hen
of enamel. The accumulation of fluoride by enamel seems thin fine-pointed small spiral interdental brushes should be
largely restricted to the surface region and the fluoride used. If there is complete loss of interdental papilla and
Dental ;lut.,e i======
gingiva from embrasure, bristle ended unstuffed brushes and 135. 'B' [Soben Peter 4 th ed 96/ 5th ed 270)
thick spiral interdenta l brushes should be used.
136. 'D' [Textbook of Pediatric Dentistry by Muthu and
130. 'A' [Soben Peter 4th e d 126/ 5th ed 309] Sivakumar 1st ed 216)
Also refer Q.No. 6 in Chapter "Plaque control" of periodontics Indications of pit and fissure sealants:
1. Presence of deep occlusal pit and fissures of newly
131. 'C' [Soben Peter 4th ed 249/ 5th ed 528] erupted teeth (molars and premolars).
2. Presence of lingual pits or palatal pits in relation to
132. 'B' [Soben Peter 4 th ed 120/ 5th ed 303]
upper lateral incisors and molars.
Disclosing agents contain a dye that imparts its colour to
soft deposits (Bacterial plaque) but can be rinsed easily 3. Presence of incipient lesion in pit and fissure system .
from clean tooth surfaces. 4. Children and young people with medical, physical or
intellectual impairment with high caries risk.
Examples of some disclosing agents are: 5. Children and young people with signs of higher caries
• Erythrosin - Common disclosing agent activity and coming from non-flouride area.
• Bismark brown - Plaque component of Ramjford index
uses this solution Contraindications:
• Two tone solution - Stains mature plaque as green and 1. Presence of shallow pit and fissures of molars and
immature plaque as red premolars.
• Mercurochrome 2. An open occlusal caries lesion with extension into dentin.
• Malachite green 3. Presence of large occlusal rest oration .
4. Presence of interproximal lesions and rampant caries.
Uses of disclosing agents:
5. Partially erupted tooth where in isolation is a problem .
• Personalized patient instruction and motivation
• Self-evaluation by the patient 6. Un-cooperative children, where getting adequate dry
field is a problem.
• To evaluate the effectiveness of oral hygiene
maintenance.
13 7. 'B' [Sob en Peter 4th ed 252/ 5th ed 531)
• In research studies. with regard to effectiveness of plaque Bibby in 1942 was the first to demonstrate that repeated
control devices like toothbrushes and dentifrices etc. application of NaF or KF to teeth of children significantly
reduced their caries prevalence. This finding was the fore
133. 'A' [Soben Peter 4th e d 441/ 5th ed 562] runner of numerous studies designed to test the effectiveness
Options 'B', 'C' and 'D' are examples of topical fluorides. of various topical agents and the best mode of applying
them to teeth.
134. 'D' [Soben Peter 4th ed 463/ 5th ed 96)
Hardness is soap destroying power of water due to various 138. 'A' [Understanding Dental Caries by Nikiforuk Pg 38]
chemicals in water like - According to Nikiforuk for maximum benefit, the ingestion
• Calcium carbonate of fluoride supplements should be commenced shortly after
• Magnesium bicarbonate birth, and continued until the age of 12 years. (It is 16
• Calcium sulphate years according to ADA).
• Magnesium sulphate
139. 'B' [Hiremath 1st ed 360)
Hardness is classified as First fluoride varnish (Duraphat) was introduced in Europe
in 1964. It conains 5% NAF2 or 2.26% Fluoride.
a) Temporary hardness or carbonate hardness is due to
presence of calcium and magnesium bicarbonates
140. 'B' [Park 22nd ed 5 7 8)
b) Permanent hardness or Non-carbonate hardness: is The daily req uirement of iodine for normal adults is 150 mg.
due to calcium and magnesium sulphates, chlorides and For pregnancy women, the daily requirement shall be 250
nitrates.
mg.
The hardness of drinking water should be moderately hard {1-3 141. 'A' [Park 22nd ed 140]
mEq/ itre). Temporary hardness of water can be removed by Contraindications of measles vaccine:
• Boiling - removes temporary hardness
• Pregnant women
• Addition of lime [Ca(OH)J
• To patients allergic to neomycin, gelatin or other
Both temporary and permanent hardness of water is removed components of vaccine
by • Severely immunocompromised patients due to congenital
• Addition of sodium carbonate (Na/o3] disease, severe HIV infection, advanced leukemia or
lymphoma, serious malignant disease, treatment with
• Permutit process ( Base exchange process) high-dose steroids.
, COMMUNITY DENTISTRY
~

V
699

142. 'A' [Soben Peter 4 1h ed 98/ 5th ed 292]

Preventive Early diagnosis and Disability


Health promotion Specific protection Rehabilitation
services prompt treatment limitation
• Diet planning • Appropriate use of fluoride • Self examination • Use of dental • Use of dental
• Demand for • Ingestion of fluoridated and referral services services
Services by preventive services water. • Use of dental
individual • Periodic visits to • Use of fluoride dentifrice services
dental office and other prescriptions
• Oral hygiene practices
• Dental health • Community or school water • Periodic screening • Provision of • Provision of
education programs fluoridation and referral dental services dental services
Services by • Promotion of • School fluoride mouth ri nse • Provision of dental
community research and lobby and tablets services
efforts • School based sealant
programs
• Patient education • Topical application of • Complete • Complex • RPD & FPO
• Plaque control fluorides examination and restora·tive • Implants
program • Fluoride supplements and prompt treatment dentistry
Services by • Minor tooth
rinses of incipient lesions • Pulpotomy
dental • Diet counseling movements
• Pit and fissure sealants • Simple restorative • RCT
professional • Caries activity tests
restorations (PRR) dentistry
• Recall and • Extractions
• Preventive resi n
reinforcement
• Pulp capping

PREVENTION OF PERIODONTAL DISEASE

Levels of
PRIMARY SECONDARY TERTIARY
prevention

Preventive Specific Early diagnosis and Disability


Health promotion Rehabilitation
services protection prompt treatment limitation

• Demand for preventive services • Oral hygiene • Self examination


Services by • Periodic visits to dental office practices and referral • Use of dental • Use of dental
individual • Use of dental services services
services

• Dental health education • Supervised • Periodic screening


programs school-brushing and referral
Services by programs • Provision of • Provision of
• Promotion of research and • Provision of dent al
community dental services dental services
lobby efforts services
• Provision of oral hygiene aids

• Patient education • Oral prophylaxis • Complete • Deep curettage


• Plaque control program • Correction examination • Root planning
• RPO & FPO
Services by • Recall and rei nforcement of tooth • Scaling and • Splinting
malalignment curettage • Implants
dental • Periodontal
professional • Corrective, • Mi nor tooth
surgery
restorative and movements
• Selective
occlusal services
extraction
Dental ;lut.,e

PREVENTION OF ORAL CANCER

Preventive Specific Early dfagnosis and Disability


Health promotion Rehabilitation
services protection prompt treatment limitation

• Demand for • Avoidance of • Self examination and


Services by preventive services known irritants referral • Use of dental • Use of dent al
individual • Periodic visits to • Use of dental services services services
dental office

• Dent al health • Periodic screening and


education programs referral
Services by • Provision of • Provision of
community • Promotion of • Provision of dental dental services dent al services
research a nd lobby services
efforts

• Patient education • Removal of known • Complete examination • Maxillofacial


Services by irrit ant s • Biopsy • Chemot herapy prost hodontics
dental • Oral cytology • Radiation therapy • Plastic surgery
professional • Complete excision • Surgical therapy • Speech therapy
• Counseling

PREVENTION OF OROFACIAL DEFECTS, MALOCCLUSION AND ACCIDENTS

Preventive Health Specific Early diagnosis & Disability


Rehabilitation
services promotion protection prompt treatment limitation
• Use of prot ective • Use of dent al • Use of dental • Use of dent al
Services by devices services services services
individual
• Habit control
• Dental health • Mout h guard • Provision of • Provision of • Provision of
education program dent al services dental services dent al services
programs • Safety of
Services by
• Promotion of children's toys
community
protective garb, • Safety of school
lobby efforts buildings and play
grounds
• Patient education • Genetic counseling • Minor • Major • Maxillofacial
• Prenalal care orthodontics ort hodontics removable & fixed
• Surgical therapy prosthodontics
Services by • Parental
dental counseling • Plastic surgery
professional • Caries control • Speech therapy
• Space • Counseling
maintenance

Fluoridation of water would be an example of (COM EDK-15)


a) A primary prevention strategy b) A secondary prevention strategy
c) A t ertiary prevention strat egy d) It is not a prevention st rategy
Space maintainers are categorised in to which level of health protection? (AIIMS May-14)
a) Specific protection b) Health promotion c) Disease limit ation d) Tertiary protection
, COMMUNITY DENTISTRY
~

V
701

Screening and referral to community level; level of 148. 'A' [Soben Peter 4t• ed 243/ 5th ed 521)
prevention is? (PGI June-13) The absorption of most water-soluble fluoride compounds
a) Primordial b) Primary is rapid and occurs mainly in stomach followed by upper
c) Secondary d) Tertiary intestine. Absorption is passive in nature and no active
transport mechanism is found to be involved. The maximum
143. 'C' [Soben Peter 4th ed 120/ 5th ed 302, 303] plasma concentration of fluoride (0.15 to 0.25 ppm)
following the oral intake of fluoride on a fasting stomach is
144. 'A' [Soben Peter 4th ed 250/ 5th ed 529) reached within 30 minutes.
Studies done on streptococcus mutans indicate that the
following are inhibited by fluoride. Approximately 50% ingested fluoride will be excreted in
the urine and most of the remaining will be taken up by
• Enolase and phospo transferase system. Enolase is highly the mineralized tissues. Blood plasma is considered the
sensitive to fluouride inhibition and may be partially central compartment into which fluoride must pass for the
inhibited by 0.5 - lppm fluoride. subsequent distribution and elimination. About 10 - 25% of
• Bacterial phosphatases which are involved in degradation the daily intake of fluoride is not absorbed and is excreted
of sugar phosphates. in the faeces.
• Cation (potassium) transport which is closely associated
with carbohydrate metabolism. Accretion:
It is a process where most of the fluoride is buried within
These inhibitory meclnanisms are affected by the hydrogen ion the mineral crystallites during the period of crystal growth.
concentration of the plaque fluid. A decrease in pH, results A decrease in carbonate content is generally found in
in a greater inhibitory action on bacterial carbohydrate highly fluoridated bone and tooth mineral, due to direct
metabolism. Saliva can serve as source of fluoride for the substitution of carbonates by fluoride.
plaque and therefore may affect bot h salivary and plaque
organisms. 149. 'C' [Hiremath 1" ed 363-65)
The use of fluoride mouth rinse was first described by Bibby
145. 'C' [Soben Peter 3rd ed 293 / Textbook of Primary et al in 1946. In 1975 ADA accepted neutral sodium fluoride
Preventive Dentistry by Norman. 0. Harris 6'" ed 249) and acidulat ed phosphate fluoride mouth rinses as effective
Thixotropic gels are another form of acidulated phosphate caries preventive agents. Later a stannous fluoride mouth
fluoride with identical active fluoride system as conventional rinse was also accepted by the ADA.
APF solutions. The term thixotropic denotes a solution that
sets in gel-like state but is not a true gel. On the application Naf mouth rinse dosage
of pressure, thixotro pie gels behave like solutions and are
a) Once daily: 0.02% or 0.05% (AP-14)
more easily forced into the interproximal spaces than
conventional gels. b) Once weekly or fortnightly 0.2% Naf or 0.5°/o NaF

146. 'A' [Cariology by Ernest Newbrun 3n1 ed 148) 150. 'B' [Soben Peter 4th ed 448/ 5th ed 570)
Sorbitol is a poly alcohol sugar substitute. Most oral ART (Atruamatic Restorative Treatment) is a procedure based
microorganisms lack the enzymatic makeup to utilize on removing carious tooth tissues using hand instruments
sorbitol. An important exception is S.mutans. Most caries- alone and restoring the cavity with an adhesive restorative
inducing strains of this organism may be distinguished from material.
other oral streptococci by t heir ability to utilize sorbitol.
However, the fermentation or sorbitol even by s.mutans is In ART, he technique used for restoring the cavity with GIC
slow, and hence a very little drop in the pH of dental plaque is "Press finger technique".
is recorded after the ingestion of sorbitol as compared to
rapid drop fo llowing the use of sucrose. 151. 'B' [Soben Peter 4'" ed 98/ 5th ed 292)
Ref. Q.No. 142
147. 'A' [Hiremath l '' ed 329]
The predictors of caries risk are: 152. 'B' [Check Explanation Below]
Potentiometric analysis of fluoride content in solutions by
1. Multiple new lesions (High DM FT Score)
using fluoride ion selective electrode is simple, reliable
2. Multiple restorations and cheap. The most important electroanalytic method for
3. History of premature extraction for caries determination of fluoride ion in water solution is usage of
4. Anterior teeth caries or restorations (Rampant form of caries) ion selective electrode for fluorides.
5. History of frequent replacement of restorations
Fluoride selective electrode is very sensitive and is used with
Points 2, 3, 4 are indicators of past caries experience. homogenic membrane made from fluoride Lantana (LaF3},
which was first suggested by Fran and Ross in 1966.
Dental ;lut.,e

153. 'B' [Soben Peter 5th ed 543]

City fluoridated Control city

Ground Rapids
Muskegon
(Michigan)
Newburgh Kingston
• Sarnia (Control)
Brantford
• Stratford (auxillary control)
Evanston Oak-Park
Tiel Culemborg

154. 'C' [Norman 0. Harris 6th ed 646)


• Plaque control is considered as primary Level of prevention
of both dental caries and periodontal disease.
• Booth dental caries and periodontitis are the result of
prolonged presence of pathogenic plaques affecting the
enamel, cementum and /or contiguous gingiva.
• In most cases, both the diseases are controlled by
mechanical and chemical plaque control regimens.
• Both the plaque diseases must go through a continuum
of two reversible interim stages from histological
normalcy to clinical pathology.

155. 'B' [Sturdevant 6th ed 77)


One of the anti caries effects of fluoride is t hat, the presence
of fluoride ion greatly enhances the precipitation of salivary
calcium and phosphate ions onto tooth structure. This
insoluble precipitate replaces the soluble salts containing
manganese and carbonates that were lost because of
bacteria mediated demineralization. This exchange process
results in enamel becoming more acid resistant.

Note:
In high concentrations of 12,000 ppm, used in topical
fluoride treatments, fluoride ion is directly toxic to some
oral organisms including mutan streptococci.
, COMMUNITY DENTISTRY
~

V
703

5. CARIES ACTIVITY TESTS


1. Caries vaccine works by all the following except c) Caries susceptibility test
a) Monoclonal antibodies are formed which attach to the d) None of the above
t ooth surface by adhering to the salivary pellicle (PGI-2003)
b) Strep.mutans binds to the antibodies 12. Snyder's test measures
c) New flora is established a) Amount of acid produced
d) Strep.fecalis takes the place occupied by Strep.mutans. b) Number of new caries lesion
(MAN-2001) c) Buffering action of saliva
2. The foolproof method to check the diet counseling i s d) Antibiotic activity of saliva
a) Lactobacillus test b) Snyder test (PGl-97)
c) Vinegar Lesl d) None or Lhe above 13. The lactobacillus count test to assess caries a ctivity was
(PGl-99, 06, MCET-07) given by
3. The valuable test for evaluating caries activity in very a) Snyder b) J ay
young children is c) Hadley d) Edelstein
a) Lactobacillus colony count test (COMEDK-2013)
b) Swab test 14. In the lactobacillus count test, which reading will be
c) Alban test d) Calorimetric snyder test inferred as " high risk"
(KCET-10) a) >100/ml b) >1000/ml.
4. Caries activity is increased in c) >10,000/ml d) None of the above
a) Xerostomia b) Sugar diet (KAR-2002)
c) Taking much carbohydrate food 15. Fluoride pit and fissure s ealants belong to which
d) None generation
(PGI-2001, 02) (MAN-94) a) I b) II
5. The dye used in the " Reductase enzyme" (caries activity c) III d) IV
test) is (AIIMS-04)
a) Diazoeresorcinol b) Bromocresol green 16. Yard-stick of detection in snyder's test is
c) 1% acid red in propylene glycol a) acid formation b) pH change
d) Methyl blue c) count of microorganisms
(KAR-99) d) enamel solubility
6. Snyder's test tell us the: (PGI-08)
a) Exact nature of microorganisms
b) Combined acidogenic nature of microorganisms
c) Enamel dissolving capacity of saliva
d) All of the above
(AIPG-96)
7. In Snyder's Test, the saliva is said to be highly cariogenic
if colour change is observed at (hrs)
a) 24 b) 48
c) 72 d) 96
(COMEDK-03)
8. Vaccine fo r caries is based on which immunoglobulin
a) IgG b) lgA
c) IgE d) lgM
(AIIMS-04)
9. Lactobacillus count AFTER CARIES ACTIVITY test is
10,500CFU. What is the rate of caries progression ?
a) Slight b) Mild
c) Moderate d) Severe
(AIIMS-09)
10. Best caries activity test used for field study is
a) Lactobacillus count test b) Streptococcus mutans
c) Snyder's test d) Saliva / tongue blade method
(PGI-2003)
11. Albans test is an example of
a) Caries activity te.st b) Acid producing test

1) C 2) D 3) B 4) A 5) A 6) B 7) A 8) B 9) D 10) D 11) A 12) A 13) C


14) C 15) D 16) B
Dental ;lut.,e

5. CARIES ACTIVITY TESTS - ANSWERS


1. ' C' [Soben Peter 5th ed 281-283) Option 'B' bromocresol green is used as a indicator dye in
snyder test.
2. 'D' [Check Explanation Below]
In this question option 'D' is more appropriate than option The dye used in modified
'K as previous history and diet chart/ diet history is gold Basic Fuschin (PGl-13)
Quigley Hein index
standard to check the diet counseling. When option 'D' None
of the above is not given, then option 'K Lactobacillus test The dye used in "Salivary
Diazoresorcinol
is correct answer. Reductase test
The dye used in Snyder test Bromocresol green
Best method of checking on effectiveness of diet The disclosing agent that
counselling is ( PGI Dec-2011) differentiates between mature Two tone solution
a) Lactobacillus count b) Snyder's test and newly formed plaque
c) Latest dietary history d) Salivary reductase test The caries detection dye
1% acid red solution in
used to stain infected dentin
3. 'B' [Soben Peter 4th ed 434/ 5th ed 285) 0.2% propylene glycol
contains
Swab test was developed by Grainger et al. It has an
advantage over the other tests in that no collection of Dye used in staining of fracture
2% iodine
saliva is necessary. So it is valuable in evaluating caries line in a suspected tooth
activity in very young children. The dye used in carisolv is Erythrosine dye

Procedure: 6. ' B' [Soben Peter 4th ed 433/ 5th ed 284J


The oral flora is sampled by swabbing the buccal surfaces Snyder test measures the ability of the salivary
of the teeth with a cotton applicator, and the sample is microorganisms to form organic acids from a carbohydrate
subsequently incubated in the medium. The change in pH medium. In this test the glucose agar medium contains an
following a 48-hour incubation period is either read on a pH indicator dye " Bromocresol green" which changes color
meter or read by the use of a color indicator. from green to yellow in the range of PH 5.4 to 3.8

Principle: Colour observations in snyder test :-


Same as Snyder test 24 hrs 48hrs 72hrs
INTERPRETATION If yellow If yellow If yellow
pH Caries activity marked caries definite caries Limited caries
=, 4.1 Marked caries activity susceptibility susceptibility susceptibility

4.2 to 4.4 Active If green If green If green


continue to continue to caries inactive
4.5 t o 4.6 Slightly active incubate and incubate and
> 4.6 Caries in active observe at 48 hrs observe at 72 hrs

4. 'A' [Soben Peter 4th ed 91/ 5th ed 262) 7. 'A ' [Soben Peter 4th ed 43 4/ 5th ed 284)
The quantity of saliva secreted normally is 700-800 mljday.
In conditions like "salivary gland aplasia "and "xerostomia", 8. ' B' [Soben Peter 4th ed 101/ 5th ed 281)
the salivary flow is entirely lacking and the result will be
rampant form of dental caries 9. ' D' [Soben Peter 4th ed 433/ 5th ed 284)
No of /c.c. Symbolic Degree of caries
5. 'A' [Soben Peter 4th ed 436/ 5th ed 287) organism designation activity
Salivary reductase test (treatex test) measures the activity
of the reductase enzyme present in salivary bacteria. 0-1000 ± Little or none
1000-5000 + Slight
Colour Time Score Caries Activity
5000-10,000 ++ Moderate
Blue 15 min 1 Non conducive
More than 10,000 +++ Marked
Orchid 15 min 2 Slightly conducive
Red 15 min 3 Moderately conducive 10. ' D' [Soben Peter 4th ed 434-3 5/ 5th ed 289 - 90)
Saliva I Tongue blade method:
Red Immediately 4 Highly conducive
This test estimates the number of S.mutans in mixed
Pink or paraffin stimulated saliva when cultured in Mutans Salivarius
Immediately 5 Extremely conducive
white Bacitracin (MSB) agar. This was developed for use in large
, COMMUNITY DENTISTRY
,.....__,,_

V
705

number of school children. This is simplified and practical


method for field studies.

Procedure:
The subjects chew a piece of paraffin wax and then sterile
young blades are rotated in patients mout h 10 times so that
and is then pressed onto MSB agar and incubated at 37
degrees C and number of colony forming units is counted.

11. 'I([Soben Peter 4th ed 436/ 5th ed 288)


Albans test is a simplified substitute for the snyder test and
can be used for routine dental office use. This test uses the
same formula as a snyder media with the exception that less
agar is used.

At the time of test, the patient is asked to spit unstimulated


saliva directly into tube and is incubated for 4days

12. 'A' [Sob en Peter 4th ed 433/ 5th ed 284]

13. 'C' [Ref. Soben Peter 4th ed 432)


Lactobacillus colony test is the oldest and most widely used
microbiolgical method for assessing the caries activity. This
method, first by Hadley, measures the number of aciduric
bacteria in a patient's saliva.

14. 'C' [Soben Peter 4th ed 433/ 5th ed 284)


Refer Q.No. 9

15. ' D' [Soben Peter 4th ed 441/ 5th ed 563)

16. 'B' [Soben Peter 4th ed 433/ 5th ed 284]


Refer Q.No. 6
Dental ;lut.,e

6. MISCELLANEOUS

1. Examining a patient in dental office is equivalent to c) Polio d) Cancer


community dental health if (MAN-99)
a) Surveying b) Analysing of data 12. When did India become a member of W.H.O.
c) Programme planning d) Evaluation a) 15th August 1947 b) 12th January 1948
(MAN-1998} c) 26th January 1970 d) 15th September 1947
2. Pathfinder study as envisaged by WHO has the following (MAN-2002}
salient features: 13. Indian dentists act was formulated in
a) 0.1 or 1 percent of the population is usually assessed. a) 1942 b) 1947
b) Four specific age groups are included. c) 1948 d) 1949
c:) The age groups included a re 7, 11, 35 and 65 years. (MAN-2001, COMEDK-05)
d) A and B of above only. 14. WHO head - quarters of southeast Asia is at
(KAR-97) a) New Delhi b) Ceylon
3. The personnel involved in the integrated child c) Katmandu d) Islamabad
development program of WHO is (MAN-2002}
a) Dhai b} Anganvadi workers 15. "Loss of Head" is
c) Village health workers a) Death of a head of a family
d) School health teacher b) Increased resistance to filtration in a sand filter (related
(MAN-2002; KCET -07} to filtration capacity)
4. Non-residential foreign dental practitioners having c) Death of a head in village panchayat
dental qualification are eligible to become _ _ __ d) Death of a school teacher who is also a healthy worker
members of IDA. of a school
a) Director b) Associate (MAN-2002, KCET-08}
c) Honorary d) Affiliated 16. In slow sand filters, the percentage of waters that is
(KAR-2013} bacteria free
5. The health promotional phase in public health is between a) 100% b) 99.9%
the years c) 99% d) 90%
a) 1880-1920 b) 1920-1960 (MAN- 2002, COMEDK-08)
c) 1960-1980 d) 1981-ZK 17. World health day is on
(COMEDK-04} a) 14th April b) 14th March
6. The point at which the chlorine demand of water is met c) 7th March d) 7th April
is termed (MAN-2001}
a) Break point b) Back wash 18. First Dental college established in India is
c) Loss of head d) Chlorine demand a) Madras dental college, 1910
(COMEDK-06) b) Nair dental college, Bombay, 1920
7. The Bhore committee on health survey and development c) Sir Ahmedi dental college, Calcutta 1920
submitted its report in d) Maulana azad college, Delhi, 1935
a) 1943 b) 1946 (MAN -99, COM EDK- 06}
c) 1962 d) 1963 19. Main objective of Oral Health Programme by year 2000 is:
(COMEDK-04) a) Caries free mouth b) No caries up to 12 yrs of age
8. Occupational hazards to the dentists result from c) DMF<3, up to 12 yrs of age
a) Posture b) Accidental infection d) None of the above
c) Dermatitis d) All of the above (PGI-95)
(KAR-99) 20. Which of the following is an occupational disease of
9. Ideally Dentist population ration should be dentist
a) 1:8000 b) 1:10000 a) Syphilis b) HIV
c) 1:20000 d) 1:40000 c) HBV d) All the above
(MAN-98} (KAR-2000)
10. Which of the following aren't included in the WHO 21. The world health organization's objective in dental
classification of operating dental auxiliaries? health is that by the year 2000 a person, at age of 12,
a) Dental hygienist b) School dental nurses shall not have decayed, missing and filled teeth (DMF)
c) Therapists d) Denturists more than:
(MAN-99) a) 1 b) 2
11. The WHO theme for 1996-97 is c) 3 d) 4
a) Oral healt h b) Cities (KAR- 98}

1) A 2) D 3) B 4) D 5) B 6) A 7) B 8) D 9) C 10) D 11) B 12) B 13) C


14) A 15) B 16) B 17) D 18) C 19) C 20) D 21) C
, COMMUNITY DENTISTRY
,......__,,_

V
707

22. Waste water from kitchen is: b) End of chlorination process


a) Sullage b) Sewage c) When free residual chlorine starts appearing
c) Garbage d) Kitchen water d) After partial saturation of water with chlorine
(KAR-2003) (AIPG- 06)
23. Residual chlorine of water should be: 34. The term best describes a disease transmitted to man
a) lmg./ltr. after 1 hr. b) lmg./ ltr. after 1'2 hr. through animals:
c) 0.5mg./ltr after 1 hr. d) 0.5 mg./ltr. after 1'2 hr. a) Communicable b) Transmissible
{KAR-2003) c) Zoonotic d) Vertically transmissible
24. Removal of bacteria is: (AIPG-05)
a) Better with slow sand filter 35. Awakening the fundamental desire to learn is known as:
b) Better with rapid sand filter a) Motivation b) Interest
c) Equal with both c) Learning d) Comprehension
d) Depends on hardness of water (KAR-04)
(KAR-2003) 36. Tasks that involve exposure blood, body fluids or tissues
25. The dentist to population ratio according to the Bhore are classified by OSHA regulations under category:
Committee a) I b) II
a) 1 : 6,000 b) 1 : 4,000 c) III d) IV
c) 1 : 2,000 d) 1 : 750 (COMEDK-05)
(KAR-99) 37. The term "Secular Trend" implies
26. Where are the dentists feet during any procedure when a) Being liberal minded in diagnosing
four handed dentistry is practiced b) Cold professional attitude
a) On the stool support rim c) Changes in occurrence over a long period of time
b) On the floor d) Migration from rural areas
c) On the dental chair base (KAR-2013)
d) Where ever is convenient 38. The diagnostic power of a test to correctly exclude the
(KAR-99) disease is reflected by:
27. The insurance concept is essentially one of a) Sensitivity b) Specificity
a) Budgeting payment b) Subsidise financing c) Positive predictivity d) Negative predictivity
c) Capitation d) Sharing risk (AI- 05)
(KAR 99) 39. Colombo plan is a plan of:
28. Temporary hardness of water is due to the presence of a) UNO b) World Bank
following salts of calcium and magnesium: c) Common Wealth countries
a) Bicarbonates b) Sulphates d) Government of Ceylon
c) Chlorides d) Nitrates (COMEDK- 05)
(KAR-98) 40. The method of prepayment wherein the patient pays a
29. Clinical wastes in hospitals are to be disposed in which percent of total cost:
colour bags a) Loan b) Co-insurance
a) Yellow b) Black c) Reasonable fee d) Fee for service
c) Green d) Red ( PGI-05)
(COMDEK-04) 41. ' POTABLE WATER' is one which:
30. The permissible levels of E coli (number/ml) in potable a) Is safe drinking
water is b) Can be carried from one place to another
a) 20 b) 15 c) Found within 5 meters within the surface of earth
c) 10 d) O d) None of the above
(COMEDK-04) (COMEDK-05)
31. New Zealand type of school dental nurse can perform all 42. The National AIDS Control Program has the following
of the following di nical duties except: components, except:
a) Prophylaxis b) Pulp capping a) Sero surveillance
c) Root canal treatment d) Topical fluoride application b) Health, education & information
(KAR-04) c) Screening of blood and blood products
32. Which of the following is not an indicator for nutritional d) Banning of sexual contact with foreigners
values? (AIPG-06)
a) Vital signs b) Weight and height 43. The following tests are used to check the efficiency of
c) Mid arm circumference pasteurization of milk except:
d) Prevalence of low birth weight a) Phosphatase test b) Standard plate count
c) Coliform count
33. Break point chlorination means: d) Methylene blue reduction test
a) Start of chlorination process (AIPG-05)

22) A 23) C 24) A 25) B 26) B 27) D 28) A 29) A 30) D 31) C 32) A 33) C 34) C
35} A 36) A 37) C 38) D 39) C 40} B 41) A 42) D 43} D
Dental ;lut.,e i======
44. Any principal/dean of recognized Dental college can 56. In lake water, the highest fluoride value ever recorded in
become a member of Dental Council of India. The 2800 PPM. In which country this lake is situated:
maximum members in such category can be: a) Japan b) Kenya
a) Two b) Three c) Libya d) South Africa
c) Four d) Six (KAR-04)
5 7. A stipulated flat sum the patient has to pay towards the
45. The method of presenting data of geographic distribution is: cost of service before it goes into:
a) Histogram b) Pie chart a) Ground insurance b) Co-insurance
c) Frequency polygon d) Cartogram c) Co-payment d) Deductible
(COMEDK- 05) (COMEDK- 07, MCET-07)
46. Oral hygiene was first practiced by: 58. Ethical principles that requires health professionals
a) Chinese b) Etruscans to fully inform their patients and protect their
c) Sumerians d) Sushruta confidentiality is the principle of:
(COMEDK-06) a) Justice b) Autonomy
4 7. Dental auxiliaries are not in existence in: c) Veracity d) Beneficence
a) America b) India (COMEDK- 07)
c) Saudi Arabia d) New Zealand 59. First step in Public Health procedure is:
(KAR-04) a) Analysis of Programme b) Planning of programme
48. What is the color coding of bag in hospitals to dispose c) Situation analysis d) Appraisal of programme
off human anatomical wastes such as body parts: (COM ED K-07)
a) Yellow b) Black 60. is anaerobic method of solid waste disposal:
c) Red d) Blue a) Bangalore method b) Mechanical composting
(AIPG-05, COMEDK- 2013, KAR - 2013) c) Controlled tipping d) Incineration
49. Which of the following is not a method of chlorination of (COM ED K-07)
water: 61. Current concepts of "evaluating health care" require a
a) Perchloron b) Ozonation scientific basis for treatment referred as:
c) Chlorine gas d) Chloramine a) Evidence based therapy b) Periodontal maintenance
(AI-05) c) Tooth mortality
50. Of the following minerals the highest amount found in d) Prevention of loss of attachment
human body is: (KCET- 07)
a) Sodium b) Calcium 62. Interruption of transmission of disease:
c) Phosphorous d) Iron a) Disease control b) Disease eradication
(AI- 05) c) Disease elimination d) Disability limitation
51. The increase in caries incidence is seen when the salivary (KCET- 07)
flow is: 63. As per Bio Medical Waste Management & Handling rules,
a) < 5ml/min b) < 6ml/min waste sharps should be contained:
c) < 7mljmin d) < 8mljmin a) White container b) Yellow container
c) Red container d) Black container
52. Variable which can be controlled or manipulated: (COM ED K-07)
a) Independent b) Dependent 64. The model of web of causation of diseases was first
c) Discrete d) Parameter suggested by:
(AP- 05) a) Pettenkofer b) Mac Mohan & Pugh
53. The age of the child used in the determination of child c) John M. Last d) C.E.A. Winslow
mortality rate is: (KCET-07)
a) 0-5 years b) 1-4 years 65. "Disability limitation" is a mode of intervention in:
c) 5-8 years d) 0-1 years a) Primary level of prevention
(AIIMS-06) b) Secondary level of prevention
54. The headquarters of UNICEF is situated at: c) Tertiary level of prevention
a) Geneva b) London d) Primordial prevention
c) New York d) Washington DC (KCET-07)
(KAR- 04) 66. Index used to measure acute and chronic malnutrition
55. Elements of primary health care include all of the can be:
following except: a) Height with weight b) Weight with age
a) Adequate supply of safe water and basic sanitation c) Height with age d) Body mass index
b) Providing essential drugs (AIPG-07)
c) Sound referral system 67. Where was the atraumatic restorative treatment first
d) Health education introduced?
(AIIMS-06) a) Tanzania b) China

44) C 45) D 46) C 47) C 48) A 49) B 50) B 51) A 52) A 53) B 54) C 55) C 56) B
57) D 58) B 59) C 60) A 61) A 62) C 63) A 64) B 65) C 66) B 67) A
, COMMUNITY DENTISTRY

c) U.S.A. d) New Zealand 78. 'Rural Health Scheme' in India was launched in the year:
(KCET-07) a) 1978 b) 1975
68. An attribute or exposure that is significantly associated c) 1977 d) 1976
with the development of a disease is: (KCET-08)
a) Agent factor b) Risk factor 79. Touidine blue is to:
c) Exposure factor d) None of the above a) Differentiate between malignant transformation
(KCET-08) b) Differentiate lichen planus from leukoplakia
69. The process of enabling people to increase control over c) Differentiate between pemphigus and lichen planus
and improve health: d) Differentiate between candidiasis & leukoplakia
a) Health promotion b) Primordial prevention (AIIMS-07)
c) Health education d) Primary prevention 80. Effective evaluation refers to:
a) Whether programme results meet predetermined
70. _ _ _ _ measures the occurrence of some particular objectives
event in a population during a given time period: b) The long term outcome of the programme
a) Incidence b) Prevalence c) Assess the extent to which programme implementation
c) Proportion d) Rate complies with the program plan
d) Whether programme benefits exceeds the cost incurred
71. Incineration is not done for (COM ED K-08)
a) cytotoxic drugs 81. Life cycle of filaria is?
b) cotton contaminated with blood a) Cyclodevelopmental b) Cyclopropagative
c) Waste sharps d) human tissue c) Propagative d) Transovarian
(AIPG-10) (AIPG-09)
72. The Alma-Ata declaration on 'primary health care' was 82. Global age for caries detection and survey is
made in the year: a) 5 b) 15
a) 1980 b) 1978 c) 12 d) 34
c) 1982 d) 1977 (MCET-10)
(COMEDK-2013) 83. Disclosing agents are used to motivate and educate
73. One of them is NOT a principle of health education: patients about plaque and oral hygiene methods. These
a) Credibility b) Feedback agents can make the patient visualise subgingival
c) Setting an example d) Utilization plaque.
(KCET-08) a) Both are true b) Both are false
74. List of the charges established or agreed to, by a dentist c) 1'' is true, 2nd is false d) znd true, 1" is false
for specific dental services: (MCET-10)
a) Table of allowances b) Fee schedule 84. In India, the cause for maximum maternal mortality is?
c) Reasonable fee d) Capitation a) Anemia b) Hemorrhage
(KCET-08) c) Abortion d) Sepsis
75. The commonly used theory to predict individual's (AIPG-09)
behaviour regarding preventive health care is: 85. Specificity is
a) Social cognitive theory a) Ability of a test to identify correctly all those who have
b) Health belief model the disease
c) Transtheoritical model d) Salutogenic model b) Ability of a test to identify correctly those who do not
(KCET-08) have the disease
76. An ongoing learning opportunity that enables all c) Diagnostic power of a test
students (K-12) to be productive learners and to make d) The amount of previously unrecognized disease that is
well considered health decisions throughout their lives: diagnosed through screening
a) Comprehensive school health programme (COM EDK-10)
b) Health promotion 86. A most insanitary method that creates public health
c) Comprehensive school health education hazards, nuisance and severe pollution of environment is
d) Health literacy a) Incineration b) Controlled tipping
(KCET-08) c) Dumping d) Composting
77. In one of the principles of health education 'soil, seed, (COM EDK-10)
sower', soil is: 87. In National Water Supply and Sanitation programme a
a) Medium by which facts can be transmitted problem village is defined as all except?
b) Health facts that are to be given a) Distance of safe water is greater than 1.6 Km
c) The people to whom health education is given b) Water is exposed to the risk of cholera
d) People who give health education c) Water source has excess iron and heavy metals
(KCET-08) d) Water infested with Guniea worm
(AIPG-09, AIIMS NOV- 14)

68) B 69) A 70) D 71) C 72) B 73) D 74) B 75) B 76) A 77) C 78) C 79) A
80) A 81) A 82) C 83) C 84) B 85) B 86) C 87) D
Dental ;lut.,e i======
88. Which one of the following is NOT an approach to the b) Panel discussion and group discussion
health education ? c) Newspapers and television
a) Regulatory approach b) Service approach d) Symposium and seminar
c) Health education approach (KCET-10)
d) Primary health centre facilities 101. BMI to classify obesity should be ?
(COM EDK-10) a) ~20 b) ~30
89. Chronic carrier state is seen in all except c) ~40 d) ~50
a) Measles b) Diptheria (AIPG-09)
c) Typhoid d) Gonorrhea 102. The assessment or measurement of the quality of care and
(AIIMS-09) the implementation of any necessary changes to either
90. Which of the following vaccine is not included in EPI maintain or improve the quality of care rendered is
schedule a) Quality Assessment b) Quality Assurance
a) DPT b) MMR c) Quality Planning d) Guidelines
c) BCG d) OPV (COM EDK-2013)
(AIIMS-09) 103. Hospital wastes should ideally he disposed by
91. Dental hygienist is included under a) Burial b) Composting
a) Non-operating auxiliaries c) Incineration d) Trench method
b) Operating auxiliaries (KCET-10)
c) Dental specialist d) Passive dental manpower 104. India aims to eliminate which of the following diseases
(KCET-10} by 2015:
92. Maintenance of standards in dental education is the a) Malaria b) Tuberculosis
responsibility of c) Kala Azar d) Filariasis
a) IDA b) DCI (AIPG-2011)
c) WHO d) FDI 105. True about meningococcal vaccine is -
(COM EDK-10) a) Capsule polysaccharide act as a virulent factor for the
93. Dentist act was introduced on production of antibody
a) 27th March 1948 b) 29'h March 1948 b) Conjugate vaccine is not given in outbreaks
c) 27th March 1949 d) 29th March 1949 c) Polysaccharide vaccine should be given to children below
(COM EDK-10} 2 years of age
94. Adjuvant used in DPT vaccine is? d) Vaccine with polysaccharide B is immunogenic
a) Zinc b) Aluminium (AIIMS-09)
c) Copper d) Magnesium 106. Caries estimated by all except
(AIPG-09) a) Family member b) Genetic factors
95. Occupational cancer involve following organs except? c) Cephalometric analysis d) Salivary flow
a) Lung b) Breast (AIPG-10)
c) Bladder d) Skin 107. Domestic solid waste does not contain
(AIPG-09) a) Night soil b) Ash
96. Xylitol is a low caloric sweetener inhibits the growth of c) Rubbish d) Garbage
a) S.mutans b) S.pyogens (KAR-2013)
c) Lactobacillus d) Actinomycosis 108. The most difficult source of infection among the
(KCET-10) following to control is
97. SOAP note refers to a) airborne b) human to human
a) Confidentiality of patient record c) vector borne d) vertical
b) Classification of patient (AIPG-10)
c) Informed consent d) Classification of problem 109. Screening procedures help in reducing incidence of all of
(KCET-10) the following diseases except
98. Which of the following is not transmitted by lice a) oral cancer b) breast cancer
a) Q fever b) Trench fever c) lung cancer d) cervix cancer
c) Relapsing fever d) Epidemic typhus (AIPG-10)
(AIIMS-09) 110. Active search for disease in an apparently healthy
99. Which type of dental auxiliary is advisable for Indian individual is
conditions? a) Monitoring b) Case finding
a) School Dental Nurse b) Dental Hygienist c) screening
c) Health Educator d) Sentinel surveillance
d) Expanded duty dental auxiliary (AIPG-10, PGI JUNE- 13)
(AP-09) 111. is defined as an organization of member agents
100. Mass media includes the following having social relation amongst themselves -
a) Audiovisual aids a) Family b) Society

88) D 89) A 90) B 91) B 92) B 93) B 94) B 95) B 96) A 97) D 98) A 99) A 100) C
101) B 102) B 103) C 104) D 105) A 106) C 107) A 108) A 109) C 110) C 111) B
, COMMUNITY DENTISTRY
~

V
711

c) Community d) Culture 123. True about ASHA is


(KCET-09) a) Mental health worker
112. Collection of health information, its use and its b) involved in minimum need program
transmission to other Levels of the system by non- c) deployed 1/1000 population
professional health workers is - d) to replace anganwadi workers in ICDS program
a) Sentinel surveillance b) Lay reporting (AIPG-10)
c) Census d) Survey 124. The following term is used to describe the impact of
(KCET-09) school based health education programmes on parents.
113. The sampling method used in pathfinder survey is - a) Side effect b) Secondary effect
a) Stratified cluster sampling c) Ripple effect d) String effect
b) Simple random sampling (COMEDK-2011)
c) Snowball sampling d) Two stage sampling 125. Examination is carried in individual clinical practice
(COMEDK-09) corresponding terms in public health practice is
114. Practices that have been repeated by a number of a) Diagnosis b) Evaluation
generation, practices that tend to be followed simply c) Assessment of need d) Appraisal and review
because they have been followed in the past. This refers to - (COMEDK-2011)
a) Habits b) Beliefs 126. The Indian Government passed the "Cigarettes and other
c) Customs d) Convention Tobacco Products Act" to prohibit advertisement of and
(KCET-09) regulate the production, supply and distribution of
115. All of the following are village level workers under tobacco products in the year
primary health care EXCEPT - a) 1973 b) 1983
a) Village health guide b) Anganwadi worker c) 1993 d) 2003
c) Clerks d) Trained birth attendants (KCET-2011)
(KCET-09) 127. The ratio between the achievement of program activity
116. Equitable distribution, community participation, and the desired level which, the planners had proposed
intersectional coordination and appropriate technology would result from the program is called
are the principles of - a) Effectiveness b) Efficiency
a) Primary health care b) Social science c) Appropriateness d) Adequacy
c) Primary prevention d) Epidemiology (KCET-2011)
(KCET-09) 128. Which of the following regarding Maternal Mortality Rate
117. The double pot method of adding chlorine to well water (MMR) is not true:
was devised by- a) Numerator includes total number of female deaths within
a) WHO b) UNICEF 42 days of delivery
c) National Environment Engineering and Research Institute b) Denominator includes still births and abortions
d) ICMR c) It is actually expressed as a ratio and not rate
(COMEDK-09) d) It is most common indicator for obstetric care
118. Any loss or abnormality of psychological, or anatomical (AIPG-201 1)
structure or function is 129. The recognized behaviour in a society is called
a) Disease b) Impairment a) Custom b) Tradition
c) Disability d) Handicap c) Culture d) Folkways
(AIPG-10) (COMEDK-2011)
119. Eichosapentaenoic acid is present in - 130. The National Cancer Registry Program (NCRP) was
a) Soyabean oil b) Corn oil initiated by the Indian Council of Medical Research in
c) Sunflower oil d) Fish oil the year
(AIPG-10) a) 1972 b) 1982
120. One sub-centre covers - c) 1992 d) 2002
a) 1000 population b) 2000 population (KCET-2011)
c) 4000 population d) 5000 population 131, The degree to which the health education message is
(KCET-09) perceived as trustworthy by the receiver is called
121. Compared to cow's milk, mother's milk has more a) Setting an example b) Comprehension
a) Lactose b) Fat c) Credibility d) Reinforcement
c) Proteins d) Vitamin D (KCET-2011)
(AIPG-10) 132. Which of the following is least Likely to be seen in
122. The highest percentages of polyunsaturated fatty acids occupational cancer?
are present in a) Skin b) Lungs
a) Groundnut oil b) Soyabean oil c) Liver d) Bladder
c) Margarine d) Palm oil (AIPG-2011)
(AIPG-10)

112) B 113) A 114) C 115) C 116) A 117) C 118) B 119) D 120) D 121) A 122) B 123) C
124) C 125) C 126) D 127) A 128) B 129) D 130) B 131) C 132) C
Dental ;lut.,e i======
133. All are true about OPV except 144. A dental professional may be criminally liable if he/she
a) It is a killed vaccine commits tort
b) Stored at subzero t emperature a) Accidentally b) Contributory
c) Induces intestinal & humoral immunity both c) Intentionally d) Unintentionally
d) Residual neurovirulence is a problem (COMED-14)
(AIPG-2011) 145. True about ESI?
134. NRH M (National Rural Health Mission) done in a) State give 1/8th and ESI gives 7/8th of the sum
a) 2002 b) 2005 b) Employers 8.75°/o of salary and employees give 4.75%
c) 2006 d) 2011 c) Funeral benefits Rs.50,000
(PGI-2011) d) For Person earning Rs. 70/ day has to contribute Rs.300/
135. A progressive increase or decrease of a disease over a month
long period of time, generally several years or decades is (AIPG-14)
known as 146. School based dental health care for whole country is
a) Secular trends b) Cyclic trends adopted by which of the following country?
c) Seasonal trends d) Periodic fluctuations a) USA b) NewZealand
(COMEDK-2011) c) Sweden d) Australia
136. High viscosity of saliva in children increases the (AIIMS NOV-14)
incidence of caries 147. Not true about NRHM
a) Statement is true b) Statement is partially true a) Appoint ASHA
c) Statement is false d) Statement is partially false b) Strengthen institutional delivery under JSY
(AIPG-2011) c) Health and family welfare societies
13 7. The first stated and the largest principle of the Nuremberg d) State and district health mission
Code is (AIIMS NOV-14)
a) Autonomy b) Informed consent 148. Which of the following is not true for incubation period?
c) Justice d) Confidential a) For determining isolation period of a disease
(KCET-2012) b) To prevent infection among contacts
138. Ethics and planning approvals are taken in c) For quarantine of disease
a) Public health practice b) Group practice d) To differentiate among secondary cases and co-primary
c) Individual practice d) Solo practice cases
(COMED-2012) (AIIMS NOV-14)
139. In medical ethics all is involved except 149. Which of the following is a method of choice for
a) Beneficence b) Confidentiality estimating manpower requirements in most countries?
c) Informed consent d) Directive guidance a) Supply and demand model
(AIPG-2012) b) Functional analysis model
140. Which bacteria will not cause food borne infection? c) Target-setting approach model
a) E. Coli b) Salmonella d) The manpower/population ratios
c) Campylobacter d) Pseudomonas
(PGI-2011) 150. Because of the confounding factors, additional criteria
141. The model in which dental care is seen as privilege have been developed by the U.S. public health service
with the professional dominant in practitioner-patient for pit-and-tissue caries diagnosis. Which one of the
relations is following is NOT a factor of these criteria?
a) Commercial model b) Guild model a) Softening at the base of the pit or fissure
c) Interactive model d) None of the above b) Capacity surrounding the pit or fissure
(AP-14) c) Softening enamel that may be flaked away by the explorer
142. "The meeting of accumulate dental needs at the time d) Discoloration of pit and groves, limited to the depth of
of population group is taken into the program and the the fissure or pit.
detection and correction of new increments of dental (COMEDK-15)
disease on a semiannual/ other periods basis" is 151. Which is true of closed dental camp?
a) Incremental Dental Care a) Preselected area like school for camp
b) Comprehensive Dental Care b) Serving large number of people in community
c) Total Dental Care d) Preventive Dental Care c) Resent close dental camp
(COMED-14) d) No such term
143. Texas statewide Preventive Dentistry program is also (PGI JUNE-2014)
known as 152. The school health programme that incorporates
a) Sharp Program b) Tattle tooth program motivation of patients through home visits is
c) Smile Train Program d) Theta Program a) Askov Dental Demonstration
(COMED-14) b) North Carolina State-wide Preventive Dental Health
Programme

133) A 134) B 135) A 136) B 137) B 138) A 139) D 140) D 141) B 142) B 143) B 144) C 145) A
146) B 147) C 148) A 149) D 150) D 151) A 152) D
, COMMUNITY DENTISTRY
~
713
V
c) Tattle Tooth Programme
d) School Health Additional Referral Programme (SHARP)
(KERALA-2015)
153. CARE stands for
a) Co-operative for Assistance and Relief Everywhere
b) Council of American Relief for Everybody
c) Central Association of Relief in Emergency
d) Children Association of Relief in Emergency
(MHCET-15)
154. The Central Government Health Scheme (CGHS) was
established in the year
a) 1948 b) 1953
c) 1954 d) 1955
(MHCET-15)
155. Which one of the following health agencies promotes
GOBI programme?
a) UNDP b) WHO
c) UNESCO d) UNICEF
(APPG-15)
156. A School based dental health education programme that
emphasizes home activities for skill development
a) Health-way arcade b) Head Start
c) Tattle tooth d) Natural Noshers
(COMEDK-15)
15 7. Millennium development goal was adopted in which
year?
a) 1977 b) 2000
c) 1984 d) 2004
(AIIMS MAY-14)
158. Preterm baby is
a) Babies born before the end of 30 weeks
b) Babies born before the end of 37 weeks
c) Babies born before the end of 38 weeks
d) Babies born at completion of 42 weeks
(PGI JUNE-2012)
159. An extracted tooth is discarded in which bag?
a) Yellow b) Red
c) Black d) Blue
(PGI DEC-2013)

153) A 154) C 155) D 156) D 157) B 158) B 159) B


Dental ;lut.,e

6. MISCELLANEOUS - ANSWERS

1. 'A' [Soben Peter 4th ed 32/ 5th ed 248) • Helps to estimate the increase or decrease
Individual Community of a disease
15 years
Examination Survey • Used for assessment of periodontal disease
Diagnosis Analysis indicators in adolescents

Treatment planning Program Planning 35 - 44 • This age is the standard monitor group of
years the adults.
Treatment Program Operation (Mean • The full effect of dental caries, and severity
Payment for service Finance 40 yrs) of periodontal disease can be assessed
Evaluation Appraisal 65 - 74
• Helps in planning appropriate care for the
years
2. ' D' (Old editions of Soben Peter] elderly people and overall effects of oral care
(Mean
services in a population
• Path finder survey was put forth by WHO 70 yrs)
• In this survey 0.1% or 1% of the population is assessed
by 4 specific groups of different ages i.e., 12yrs, 15yrs, The pathfinder method uses a stratified cluster sampling
35 - 44yrs and 65 - 74yrs are examined each group technique, which aims to include the most important
contains 25 - 50 subjects. population based on strata (specific index age groups) and
clusters (naturally formed units like villages and cities).
- according to old editions of Soben Peter
Number of subjects in a National Pathfinder Survey for
But the newer editions of Soben Peter say that each index age group:
Pathfinder surveys are classified in Pilot and National a) From Urban area:
Pathfinder surveys de pending on the number of and type 4 sites in city - 4 X 25 100
of sampling sites and age groups included.
2 sites in 2 towns - 2 X 2 X 25 100
Pilot survey:
• includes only the most important subgroups in the b) From Rural areas:
population and only one or t wo index age groups 1 site in 4 villages
(MCET-14), usually 12 years and one other age group.
in different regions - 4 x 25 100
• provides minimum amount of data needed to commence
palnning. Total = 300 for each index age group

For all index age groups, the total sample is 5 X 300 = 1500
National Pathfinder s urvey:
• includes only the all important subgroups in the 3. ' B' [Soben Peter 4th ed 16/ 5th ed 209)
population and only atleast three index age groups. Village Level Workers:
• provides data for the planning and monitoring of health • Village health guide scheme
services in all countries whatever the level of disease, • Local dias
availability of resources or complexity of services. • Anganwadi workers
• The recommended index ages and age groups are 5, 12, • Accredited social health activist (ASHA)
15, 35-44, 65-74 (KERALA-15).
a) Village Health Guide Scheme:
• The basic philosophy of the scheme is "people's health
AGE SIGNIFICANCE
in people's hands". The main objective of the scheme
• In this age, the children will begin primary was to provide preventive, promotive and curative health
school in some countries. services to people of the village through a volunteer
• This age is of interest in relation to levels of from the village itself.
5 years
caries in the primary dentition which may • Village health guides are chosen by the community
exhibit changes over a shorter time span in which they work and serve as links between the
than the permanent dentition. community and organized health sector.
• Global monitory age of dental care
(KERALA-15) b) Local Dias:
12 years Local dias are traditional birth attendants are trained at PHC
• In this age, the children will go to high
school from the primary school to improve their knowledge in maternal and child health
and sterilization. They are expected to play a vital role in
propagating small family norm.
, COMMUNITY DENTISTRY
,.....__,,_

V
m

c) Anganwadi Worker: 6. 'A' [Soben Peter 4th ed 466/ 5th ed 99)


Anganwadi workers are part of ICDS (Integrated child Principles of Chlorination:
development services) who is trained for 4 months in • Chlorine demand of the water is the difference between
various aspects of health, nutrition and child development. the amount of chlorine added to the water and the
Her functions include health check-up, immunization, amount of residual chlorine remaining at the end of a
supplementary nutrition, health education, non-formal specific period of contact at a given temperature and pH
preschool education and referred services. The work of of water.
anganwadi workers is supervised by Mukhya Sevikas who
• The minimum recommended concentration of free
cover 20-25 anganwadis.
chlorine is 0.5 mg/litre for one hour and it should be
present for a contact period of at least one hour to kill
d) Accredited Social Health Activist (ASHA):
bacteria and viruses.
ASHA activist is one of key components of national rural
health mission and includes trained literate women. ASHA • The point at which the chlorine demand of water i.e.,
will be the first port of call for any health related demands most is called the break point. If further chlorine is
of deprived sections of the population, who find it difficult added beyond the break point, free chlorine begins to
to access health services. appear in the water.
• ASHA will mobilise the community towards local health • The sum of the chlorine demand of the water plus the
planning and increased utilization of the health services. free residual chlorine of 0.5 mg/l constitutes the correct
dose of chlorine to be applied.
• She would provide information to the community on
the determinants of health such as nutrition, basic
7. 'B' [Park 20th ed 792/ 5th ed 816]
sanitation and hygiene practices, healthy living and
Bhore Committee was appointed in 1943 and has submitted
working conditions, information on existing health
its report in 1946. The dentist population ratio according to
services and utilization of t hem.
Bhore committee is 1:4000.
• She will counsel women on birth preparedness,
importance of safe delivery, breast-feeding and Dental Manpower in India (According to Soben Peter 5th ed 484)
complimentary feeding, immunization, contraception
• India has 298 dental institutions producing 25,000 to
and prevention of infections (RITs & STis) .
30,000 BOS graduates every year.
• One ASHA will cat er to a village of with a population of
• In 2004, the dentist population ratio in India was
1000
1:30,000. India (in 2004) had one dentist for 10,000
persons in urban years and 2.5 lakh persons in rural
4. 'D' [Soben Peter 4th ed 541/ 5th ed 20)
areas. This is in great contrast to physician population
Membership of IDA
ratio, which was 1: 2400 in 2000 and 1:1,855 in 2004.
• Honorary members: Persons of high scientific or literary
attainment or persons who have rendered conspicuous • In 1990, t here were 3000 registered hygienists and
5000 Laboratory technicians in India. This implies one
services to association.
hygienist was avilable to 7 dentists, and one laboratory
• Life members: Dental practitioners
technician renders service to 4 dentists, whereas it
• Annual members: By paying annual subscription should ideally be 1:1 ratio.
• Direct members: Persons eligible for membership but
who are not residing or practicing in the area of a Local 8. 'D' [Check Explanation Below]
branch. These members shall be attached to a state The occupational hazards to dentists result from;
branch or to the central head-quarters.
• Student members: a) Biological Hazards:
Accidental exposure to various infectious microorganisms-
• Affiliate members: Non-residential foreign dent al Herpes virus, HIV, BHV, HCV, TB.
practitioners having dental qualification. All affiliated
members will be attached only to the head office. b) Chemical Hazards:
If a person has to be elected as honorary member, at Least • Exposure to methyl methacrylate or various metals or
25 members of the association or 10 members of the central silica.
council should propose his/her name. • Exposure to mercury
• Exposure to latex from gloves or medical equipment.
5. 'B' [Soben Peter 4th ed 8/ 5th ed 33] • Exposure to a variety of disinfecting and cleaning
Disease control phase 1880 - 1920 agents.
Health promotional phase 1920 - 1960
c) Physical Hazards:
Social engineering phase 1960-1980 • Ergonomic hazards, Electrical hazards.
Health for all phase 1981-2000 • Exposure to X-rays, UV-A radiation.
• Exposure to lasers
• Falling hazards associated with slips and falls
Dental ;lut.,e i======
• Cuts from sharp in.strument Antimicrobial resistance; No action today, No cure
• Exposure to cryogenic agents in cryosurgical procedures 2011
tomorrow.
• Burns from handling recently heat-sterilized equipment 2012 Good health adds life to years
• Nuisance or irritating noise
Cut your risk of heart attack and stroke - cont rol
• Poor indoor air quality. 2013
your blood pressure.

d) Psychological Hazards: 12. ' B' [Soben Peter 4th ed 697/ 5th ed 229]
Mental illness due to work stress. WHO was officially born on the 7th April 1948. Since then,
7th April is celebrated every year as "World health day".
9. 'C' [Soben Peter 4th ed 412] India became a member on 12th January 1948.
In 2004, the dentist population raised t o 1:30,000 (1:10,000
in urban areas and 1:2.5 lakh persons in rural areas) . Ideally 13. ' C' [Soben Peter 4th ed 673/ 5th ed 8)
the ratio of dentist to dental hygienist or dental technicians Dentist act of India was introduced on 29th March 1948.
should be 1:1.
10. ' D' (Soben Peter 4th ed 412-13/ 5th ed 476] 14. 'A' [Soben Peter 4th ed 696/ 5th ed 230]
Classification of dental auxillaries
i) Non-operating 15. ' B' [Park 20th ed 5 74/ Soben Peter 5th ed 99]
• Dental assistant As filtration proceeds, the suspended impurities and bacteria
clog to the filters and they soon become dirty and begin to
• Receptionist
lose their efficiency. This is known as Loss Of Head. (KAR
• Dental laboratory technician or denturist -05)
• Dental health educator
When the "loss of head" approaches 7-8 feet , filtration is
ii) Operating
stopped and the filters are subjected to Back Washing, in
• Dental nurse which washing is accomplished by reversing the flow of
• Dental therapist water through the sand bed. It dislodges the impurities and
• Dental hygienist cleans up the sand bed.

The term four-handed dentistry is given to the art of seating 16. ' B' [Soben Peter 4th ed 743/ 5th ed 99]
both the dentist and the dental assistant within easy reach Slow sand filter is earlier one in which 99.9% of biological
of the patient's mouth. The dental nurse scheme was impurities are filtered out. But the increased demand of
established in New Zealand in 1921. water in the cities has made rapid sand fi lters, the ideal
choice. The bacterial quality of water is less satisfactory, so
11. 'B' [Soben Peter 4th ed 560/ 5th ed 235] chlorination is compulsory before supply.
WHO THEMES
1995 Target 2000, a world without polio
1996 Healthy cities for better life
Emerging infectious diseases global alert and
1997 30th January Antileprosy day.
response
7th April World health day
Safe mother hood: pregnancy is precious- lets
1998 31st May (AIIMS MAY- 14) World no tobacco day
make it special
1999 Healthy aging and healthy living 1st July Doctor's day
2000 Blood saves lives 2nd October Anti-drug addiction day.
2001 Mental health: stop exclusion-dare to care 13th October Anti-natural disaster day
2002 More for health 1st December Anti-AIDS day
2003 Healthy environment for children 11th December UNICEF day
2004 Road safety is no accident
18. ' C' (Soben Peter 4th ed 65 6/ 5t h ed 7]
2005 Make every mother and child count
Baltimore college of dentistry
2006 Working toget her for health 1840
(first dental college)
2007 Invest in health, build a safer future Sir Ahmed, dental college, Calcutta
1930
2008 Protecting health from climate change (First dental college in India)
Save hospitals - Make hospitals safe in Harward university (1st University to affi liate
2009 1865
emergencies dental education)
2010 1000 cities, 1000 lives
Dental ;lut.,e i======
Plastic bag/ Autoclaving/ Example:
Blue/White puncture microwaving/ If a scientist conducts an experiment to test the theory that
Cat. 4, 7 a vitamin could extend a person life span, then:
translucent proof chemical
container treatment • The independent variable is the amount of vitamin that
Cat. 5, Disposal in is given to the subjects within the experiment. This is
Black Plastic bag controlled by the experiencing scientist.
9, 10 secured land fill
• The dependent variable, or the variable being affected
by the independent variable, is Life span.
Expired and outdated drugs are managed by
a) Place in secured Landfill (AIPG-14) 53. 'B' [Park 20th ed 25/22nd ed 23]
b) Collect for 10 yrs safely and then bury them
Child mortality rate is defined as the number of deaths at
c) Incineration d) Autoclave ages 1 - 4 years in a given year, per 1000 children in that
age group at the mid-point of the year concerned.
49. 'B' [Soben Peter 4th ed 466/ 5th ed 100]
Fordisi nfecting Large bodies of water, chlorineisappliedas:
54. ' C' (Soben Peter 4th ed 553/ 5th ed 227)
• Chlorine gas The main aim of United Nations Children Fund (UNICEF) is
• Chloramine to meet the emergency needs of children around the world.
• Perchloron The headquarters is situated in New York.
Chlorine gas is the first choice, because it is cheap, quick in 55. ' C' (Soben Peter 4th ed 13/ 5th ed 38-39]
action, efficient and easy to apply. The disinfecting action The alma ata declaration has outlined 8 essential components
of chlorine is due to hypochlorous acid and to a small extent of primary health care:
due to hypochlorite ions.
1) Education about health problems and their preventive
50. 'B' [Park 20th ed 537-40/22nd ed 576) and controlling methods.
2) Adequate supply of safe water and basic sanitation.
The body minerals are classified into
3) Provision of essential drugs.
Major minerals Ca, P, Na, K, Mg
4) Promotion of food supply and proper nutrition.
Trace elements Fe, F, Zn, Cu, Mn, Iodine, Cobalt,
Chromium, Molybdenum, Selenium, 5) Maternal and child health care, including family planning.
Nickel, Tin, Silicon, Vanadium 6) Immunization against infectious diseases.
Trace elements with Lead, mercury, barium, boron, 7) Prevention and control of endemic diseases.
no known function Aluminium 8) Appropriate treatment of common diseases and injuries.

51. 'A' [Soben Peter 3rd ed 241) 56. 'B' [Soben Peter 4th ed 241/ 5th ed 520)
Saliva is also called 'Liquid enamel' as it is a rich source of In Lake water, the highest fluoride value ever recorded is
various minerals. The amount of saliva secreted normally is 2800 ppm in lake Nakuru, in volcanic areas of Kenya.
5 ml/ 5 min i.e. 700-800 ml/ day. The quantity of saliva may
influence caries incidence as is especially evident in cases of 57. 'D' (Soben Peter 4th ed 423/ 5th ed 504)
salivary gland and xerostomia in which salivary flow may be Refer Q.No. 40
entirely lacking, with rampant caries the typical result. The
average unstimulated flow rate of saliva is about 0.3 ml per 58. 'B' [Sob en Peter 4th ed 505/ 5th ed 13 2 - 133]
minute. The major ethical principles are:
a) To do no harm (non malificience)
Normal flow rate of saliva is? (PGI Dec-2011) b) To do good (beneficience)
a) 1.1 ml/5 min b) 5.6 ml/5 min c) Respect for persons (autonomy and informed consent)
c) 3.7 mljS min d) 2.5 mljS min
d) Veracity or truthfulness
52. 'A' [Mahajan 6th ed 195/7th ed 17 8] e) Confidentiality
Variable is any character or quality that varies. A variable
which can be controlled or manipulated in a study is Autonomy is the principle that dictates that health care
IN DEPEND ENT variable. Independent variable influences the professionals respect the patient's right to make decisions
behaviour of another variable which is known as DEPENDENT concerning the treatment plan. Patients should not be
variable. DEPENDENT variable depends on or is influenced by bystanders in their treatment b11Jt are active participants.
another variable. Eg. Age (Independent variable) influences Informed consent, which is "voluntary", "comprehending"
blood pressure (dependent variable). and "confidential" is an essential component of a patient's
right to autonomy.
Independent variables provide the "input" which is modified
by the model to change the output. The values that result
from independent variables are known as dependent variables.
Dental ;lut.,e

surrounding atmosphere. These droplet s contain Eg: Man-mosquito (malaria) Man-snail


millions of bacteria and viruses and are source of (Schistosomiasis)
infection to others. b) Man-vertebrate host - non-vertebrate host
Eg: Respiratory infections, common cold, diphtheria, Eg: Mammal-arU1ropad-man (plaque) Bird-
whooping cough, TB etc. arthropod-man ( encephalitis)
3) Contact with soil: c) Man and 2 intermediate hosts
Eg: Hookworm, tetanus, mycosis, etc. • Man-cyclops-fish-man (Fish tape worm)
• Man-snail-fish-man (Clonorchis sinesis)
4) Inoculation into skin:
Eg: Rabies virus by dog bite, Hepatitis B virus 111) By methods in which vectors are involved in
through contaminated needles and syringes, etc. transmission and propagation of parasites.
5) Transplacental (or vertical transmission) : a) Mechanical transmission:
The disease agent produces malformations of the The infectious agent is mechanically
embryo by dist urbing its development. t ransported by flying arthropod through
• TORCH agents soiling of its feet or proboscis or by passage
of organisms through its GIT and passively
- Toxoplasma gondii
excreted. There is no development or
- Rubella multiplication of the infectious agent on
- Cytomegalo virus or within the vector.
- Herpes virus
• Varicella virus, Hepatitis B, Coxasackie B, AIDS b) Biological transmission:
• Syphilis i) Propagative :
• Some non-living agents like Thalidomide, diethyl The agent merely multiplies in vector
sti lbestro l but there is mo change in form.
Eg: Plague bacilli in rat feeas.
B. INDIRECT TRANSMISSION BY 5 ' F's
ii) Cyclo-Propagativet
- Flies
The agent changes in form and number.
- Fingers
Eg: Malaria parasites in mosquito.
- Fomit es
- Food and iii) Cyclo-developmental:
- Fluid The agent undergoes only development
but no multi plication.
1. Vehicle-borne: Eg: Microfilawia in mosquito.
In this type, t he transmission of the infectious When the infectious agent is
agent occurs through the agency of water, food transmitted vertically from the infected
(including raw vegetables, fruit s, milk), ice, blood, female to her progeny in t he vector, it
other biological products such as tissues and organs. is known as transovarial transmission.
Eg: - S. aureus in flood Transmission of the disease agent from
one stage of the life cycle to another as
Hepatitis A in water
for example, nymph to adult is known
Hepatitis B, malaria, syphilis, infectious as trans-stadial t ransmission.
mono nucleosis and cyt omegalovirus
infection t hrough blood
3) Air-borne:
a) Droplet nuclei:
2. Vector-borne: Epidemiological classification
Eg: TB, influenza, chicken pox, measles, Q-fever
I) By Vector
and many respiratory infections.
a) Invertebrat e type
Eg: Flies & mosquitoes, ticks, mit es, b) Dust:
cockroaches, bugs, flees, et c.
During t he act of sweeping, dusting and bed-
b) Vertebrate type making, t he dust is released into t he air and
Eg: Mice, rodents, bat s becomes once again airborne.
Eg: Strepto and st aphylococcal infections,
II) By transmission chain Pneumonia, TB, 0-fever, Psittacosis
a) Man and a non-vertebrat e host
, COMMUNITY DENTISTRY
~

V
725

4) Fomite borne: Specificity is the ability of a test to identify correctly those


Fomit es include inanimate articles like soiled who do not have the disease i.e., "true negatives" {AIPG-
clothes, towels, linen, handkerchiefs, cups, spoons, 14).
pencils, books, toys, etc.
In addition t o sensitivity and specificity, the performance of
Eg: Diphtheria, typhoid, Hepatitis-a, Bacillary
a screening test is measured by its "predictive value", which
dysentery, eye and skin infections
reflects the diagnostic power of test i.e., Option 'C'. The
"predictive value of a positive test" indicates the probability
5) Unclean hands and fingers: that a patient with a positive test result has, in fact, the
Eg: Staphylococcal and streptococcal infections disease in question. The more prevalent a disease is in a
Typhoid fever given population, the more accurate will be the predictive
value of a positive screening test.
Hepatitis 'K
Int estinal parasites Yield: It is the amount of previously unrecognized disease that
Dysentery is diagnosed as a result of the screening effort. (Option 'D').

82. 'C' [Soben Peter 4 1h ed 293 last line/ 5th ed 393] Note: (COMEDK - 2010)
Ref. Q.No.2 Radiographs in diagnosis of periodontal disease are more
specific (identifies who do not have disease) and less
83. 'C' [Soben Peter 4 1h ed 120/ 5th ed 302 - 304] sensitive (due to the fact that radiographic diagnosis of
Disclosing agents can make the patients visualize bone loss is possible when there is minimum of 30 - 40%
supragingival plaque only. bone loss)

84. 'B' [Park 20'h ed 482/ 22nd ed 518] 86. 'C' [Soben Peter 4th ed 468/ Park 22nd ed 102]
Indicators of maternal and child health (MCH) care are: Solid waste includes:
1. Maternal mortality rate • Garbage (food wastes)
• Rubbish (Paper, Plastics, Wood, metal, glass)
2. Mortality in infancy and childhood
• Demolition products (Bricks, Pipes et c.)
a) Perinatal mortality rate
• Sewage treatment residue (sludge and solids from the
b) Neonatal mortality rate coarse screening of domestic sewage)
c) Post-neonatal mortality rate • Dead animals
d) Infant mortality rate • Nature and other discarded material. Strictly speaking it
e) 1-4 year mortality rate should not contain night soil.
f) Under 5 mortality rate
Methods of disposal of solid waste:
g) Child survival rate
• Dumping
• Controlled tipping or sanitary land-fill
The main causes of maternal mortality in India are:
• Hemorrhage (38%) • Incineration
• Composting
• Sepsis (11%)
• Abortion (8%) • Manure pits
• Obstruct ed labour (5%) • Burial
• Hypertensive disorders (5%)
a) Dumping
• Other conditions (34%)
Refuse is dumped in low lying areas. This is an easy
85. 'B' [Park 201h ed 127/ 22nd ed 131] method of disposal of dry refuse and is also suitable for
Various measures used to evaluate a screening test are- reclamation of land. The drawbacks are :
• The refuse is exposed to flies and rodents
a) Sensitivity
b) Specificity • Source of nuisance because of the small and
unsightly appearance
c) Positive predictive value
• Drainage from dumps contributes to the pollution of
d) Negative predictive value surface and ground water.
The term sensitivity was introduced by Yerushalmy. It is Because of the drawbacks, Dumping is considered as most
defined as the ability of a test to identify correctly all those unsanitary method of waste disposal (KAR- 2013)
who have the disease i.e., "true positives".
Dental ;lut.,e i======
b) Controlled Tipping (Sanitary landfill) f) Burial:
It is most satisfactory method of refuse disposal where A trench is excavated, and at the end of each day the
suitable land is available. In this method, the material trench is filled with earth and compacted. This method
is placed in a trench, adequately compacted and covered is suitable for small camps.
with earth at the end of the working day. In controlled
tipping, three methods are used (RAT) 87. 'D' [Park 2Q1h ed 391/ 22nd ed 428]
According to National Water Supply and Sanitation
i) Ramp method Programme, a "problem village" is defined as the one
ii) Area method • where there is no source of safe water within a distance
of 1. 6km.
iii) Trench method
• where water is available at a depth of more than 15
meters.
i) Ramp method:
• where water is exposed to risk of cholera.
This method is used where terrain is moderately
sloping. • where water source has excess salinity, iron, fluorides
and other toxic elements.
ii) Area method:
88. 'D' [Soben Peter 4th ed 206/ 5th ed 114]
This method is used for filling land depressions. Various approaches to health education are:
This method has the disadvantage of requiring
• Regulatory or legal approach
supplemental earth from outside sources.
• Administrative or service approach
iii)Trench method: • Educational approach (Best & most effective) [PGI - 14]
This method is used where level ground is available. • Primary health care approach
Chemical, bacteriological and physical changes
89. 'A' [Park 2Q1h ed 136/ 22nd ed 92)
occur in the buried refuse. The temperature rises to
A carrier is defined as "an infected person or animal that
over 60°C within 7 days and kills all the pathogens
harbours a specific infectious agent in t he absence of
and hastens the decomposition process. It takes 4
discernible clinical disease and serves as a potential source
to 6 months for complete decomposition of organic
of infection for others" carriers may be classified as below:
matter into an innocuous mass.
A) Type
c) Incineration: • Incubatory
Main disadvantage of incineration is loss of much needed • Convalescent
manure to the community. Therefore, it has a limited
application and is the best met hod for disposal of health • Healthy
care or Biomedical waste.
B) Duration
d) Composting: • Temporary
It is process of nature where matter breaks down under • Chronic
bacterial action resulting in the formation of humus -
like material - called the compost. The compost has C) Portal of exit
considerable manurial value for the soil. The principal
bye-products are Co 2 , H20 and heat. The methods of • Urinary
composting are - • Intestinal
• Respiratory
i) Bangalore method (hot fermentation process) • Others
is an anaerobic method of composting. It was
by Indian Council of Agricultural Research at the i) Incubatory carriers:
Indian Institute of Science, Bangalore. • These carriers shed infectious agent during
ii) Mechanical composting (aerobic method) incubation period of disease.
Eg: Measles, mumps, polio, pertusis, influenza,
e) Manure pits: diphtheria, and hepatitis-B.
Manure pits are dug by individual householders to dump ii) Convalescent carriers:
the garbage, cattle dung, straw and leaves.
• Shed the disease agent during the period of
convalescence (during recovery from illness) .
Eg: Typhoid, cholera, diphtheria, whooping
cough.
, COMMUNITY DENTISTRY
~

V
727

• A convalescent carrier can pose a serious threat c) At 16-24 months - Measles - 2,


to the unprotected household members and - DPT and OPV Boosters,
those in immediate environment.
- Japanese encephalitis
Eg: Typhoid fever patient will excrete the bacilli
for 6-8 weeks. - Vitamin A {Then, one dose every
6 months up to the age of 15 yrs)
iii) Healthy carriers:
• Emerge from subclinical cases (those who d) At 5 - 6 years DPT Booster,
developed carrier state without suffering from - Vitamin A (2 ml or 2 lakh IU)
overt disease)
Eg: Polio, cholera, salmonellosis and diphtheria. e) At 10 and 16 yrs - Tetanus toxoid. The second dose
should be given at an interval of
iv) Temporary carriers: 1 month.
• Shed the infectious agent for short periods of
time. Incubatory, convalescent and healthy The Indian Academy of Pediatrics recommends inclusion of
carriers are included in this category. more vaccines in the immunization schedule.
• Hib conjugate 6, 10 & 14 weeks
v) Chronic carriers: • MMR 15 months
• Excretes the infectious agent for indefinite • Typhoid 2,5,8 & 12 years
periods. Chronic carriers are far more important
sources of infection than cases.
Eg: Typhoid, hepatitis-B, malaria, gonorrhoea, Immunization of 5 yrs old by national immunization
cerebra-spinal meningitis. schedule? (AIPG-14)
a) Pentavalent Vaccine, Vit. A b) Measles
Measles is caused by RNA paramyxovirus. The only source c) DT, OPV, Vit. A d) DPT Booster, Vit. A
of infection is a case of measles. Carriers are not known to Universal immunization programme of government of
occur. India does not include (AIIMS May-14)
a) BCG b) MEASLES
90. 'B' [Park 20'h ed 113/ 22nd ed 114]
c) HPV d) DPT
Expanded Programme on Immunization (EPI) is a global
immunization programme, which is now called Universal
91. 'B' [Soben Peter 4'hed 412/ 5th ed 476-77/ Ref. Q.No. 10]
Immunization Programme (UIP) .
92. 'B' [Soben Peter 4th ed 544/ 5th ed 10, 11]
WHO EPI schedule for the developing countries include:
• BCG 93. 'B' [Soben Peter 4th ed 529/ 5th ed 8]
• Oral polio Dentist act of India was introduced on the 29th March 1948.
• DPT It is an act to regulate the profession of dentistry. The act
• Hepatitis-B contains five chapters and their sub-section.
• Hemophilus infenzae type-B 94. 'B' [Park 20'" ed 147/ 22nd ed 153]
• Yellow fever DPT vaccine:
• Measles With DPT vaccine the infant can be immunized simultaneously
against three diseases i.e., Diphtheria, Pertusis and Tetanus.
INDIAN NATIONAL IMMUNIZATION SCHEDULE: The pertusis component in the vaccine enhances the potency
a) For pregnant women of the diphtheria toxoid.

• TT-1 Early in pregnancy They are two types of DPT vaccine - plain and adsorbed.
• TT-2 4 weeks after TT-1 Adsorption is usually carried out by addition of Aluminium
• TT Booster If received 2 TT doses in a phosphate or Aluminium hydroxide. Adsorption process
pregnancy within the last 3 years increases the immunological effectiveness of the vaccine.
WHO recommends that only adljuvant DPT preparations be
b) For infants
utilized in immunization programmes.

• At birth BCG & OPV - O dose, Hep. B-0 95. 'B' [Park 20'" ed 713/ 22nd ed 753]
• At 6 weeks BCG (if not given at birth) The sites of the body that are commonly affected by
DPT-1, OPV-1 & Hep.B- 1 occupational cancers are
• Skin
• At 10 weeks DPT-2, OPV-2 & Hep.B- 2
• Lungs
• At 14 weeks DPT-3, OPV-3 & Hep.B- 3 • Bladder and
• At 9 months Measles, Vitamin A (1 Lakh IU) • Blood-forming organs
Dental ;lut.,e

96. 'A' [Primary preventive dentistry by Norman.0. Harris 6th • Seminars and conferences
ed 12] • Role playing

97. 'D' [Refer Text below] C) Mass media


The SOAP note (an acronym for Subjective, Objective,
Assessment and Plan) is a met hod of documentation
• TV
employed by health care providers to write out notes in • Radio
a patient's chart. SOAP note classifies the problem into • Newspapers
subjective component (the patient's current condition • Printed material
in narrative form), objective component (vital signs, • Direct mailing
examination, findings, and Laboratory findings, etc.). • Posters, bill boards, signs
Assessment (includes quick summary of the patient problem • Health museums and exhibitions
with main symptoms, diagnosis and differential diagnosis) • Folk media
and fina lly planning the treatment.
• Internet
98. 'A' [Park 201h ed 672, Table 3/ 22nd ed 722]
• Mobile
Louse (lice)-borne diseases: 101. 'B' [Park 201• ed 347/ 22nd ed 3 69]
• Epidemic typhus Body Mass Index (BM!) is a simple index of weight-for-height
• Relapsing fever that is commonly used to classify underweight, overweight
• Trench fever and obesity in adults. It is calculated by using the formula -
• Pediculosis BMI = Weight (in Kgs)
( Height in meters 2
Option 'K Q-fever is spread by soft-tick.

99. 'A' [Hiremath 1st ed 206] Classification of adults according to BMI


The most suitable types of auxiliary for the Indian set-up
will be school going dental nurse and the EFODA. School
dental nurse will be extremely useful for taking care of
oral and dental problems among the school going children
who constitute about 40% of the total population. Besides
this, providing appropriate oral health care to school going
children in younger age, helps in preparing children as a) Pre-obese 25.00 - 29.99
better patients and development of positive attitude to oral b) Obese Class I 30.00 to 34.99
health at a Latter part of Life.
c) Obese Class II 35.00 to 39.99
The auxillary personnel's who exist currently in India are the d) Obese Class III ~ 40.00
dental surgery assistant, Laboratory technician and dental
hygienists. 102. 'B' [Dentistry, Dental practice and Community by Burt
6th ed 73)
As the awareness about oral health is very poor, Dental The term quality assessment i's defined by the ADA as
health educator is more advisable for Indian conditions. "the measure of the quality of care provided in a particular
setting." Quality assurance in turn is defined as "the
100. 'C' [Soben Peter 41h ed 217 / 5th ed 125 - 126) assessment or measurement of the quality of care and the
Health communication can be divided into three approaches implement ation of any necessary changes to either maintain
or improve the quality of care rendered".
A) Individual a1rnroach
a) Personal contact The difference in these definitions is important: Quality
b) Personal letters assessment is Limited to the appraisal of whether or not
c) Home visits standards of quality have been met, whereas quality
assurance includes the additional dimension of action to
B) Group approach take corrective steps if these are needs t o improve the
a) Lecture (Chalk and Talk) situation.
b) Demonstration 103. 'C' [Soben Peter 4 1h ed 471/ 5th ed 103)
c) Discussion methods Options 'a', 'b' and 'd' are used for general wastes.
• Group discussion
• Panel discussion
• Workshop
• Symposium
104. 'D' [Park 20th ed 776/ 22nd ed 816, table 1)
, COMMUNITY DENTISTRY 729

Group of people united for a specific


National health policy 2002 goals, which have to be achieved Association
purpose. Eg: IDA
by 01-08-2011, are
Family is a primary unit in all societies. It
• Eradicate polio and yaws is a group of biologically related individuals
2005 Family living together and eating from a common
• Eliminate leprosy
kitchen.
• Achieve zero level growth of HIV/ AIDS 2007
• Eliminate Kala-Azar It is accumulation of learned behaviours,
• Reduce mortality by 50% on account of TB, beliefs and skills of mankind as whole. It
Culture comprises everything which one generation
malaria and other vector and water borne
2010 can hand down to the next.
diseases.
• Reduce prevalence of blindness to 0.5%
• Reduce IMR to 30/100 and MMR to 100 I Lakh. 112. 'B' [Pork 2Qth ed 743/ 22nd ed 783] .
Because of slow progress in the development of comprehensive
Eliminate lymphatic filariasis 2015
vital registration system, some countries have attempted to
employ first-line health workers (Eg: Village health guides)
105. 'A' (Park 20th ed 151/ 22nd ed 157)
to record births and deaths in the community. This approach
Conjugate vaccines are preferred over polysaccharide
is known as "lay reporting of health information". Lay
vacancies due to their potential for hard protection and
reporting is defined as the collection of information, its use,
their immunogenicity more than polysaccharide in children
and its transmission to other levels of the health system by
< 2 years of age. Meningococcal polysaccharide vaccines are
non-professional health workers.
administered as a single dose to person ~ 2 years.
113. 'A' (Soben Peter 4th ed 293/ 5th e d 392)
The meningococcal vaccine is offered only to travelers
at significant risk of infection. The vaccines are purified,
114, 'C' [Soben Peter 4th ed 182/ 5th ed 173)
heat-stable, lyophilized capsular polysaccharides from
Option '/\ habits are habituated routines of behaviour that
meningococci. are repeated regularly, tend to occur subconsciously, and
t end to occur without directly thinking consciously about
106. 'C' [Shafers 5th ed 569, 593/ 5th ed 273 -75)
those behaviours. A habit is a purely personal affair, not
entailing any obligation .
107. 'A' [Soben Peter 4th ed 468/ 5th ed 101)
Eg: Smoking a cigarette after dinner.
108. 'A' (Park 20th ed 93, 94/ 22nd ed 94-95)
Option 'B' - Beliefs are views held by people and they are
Episodes of infection either by droplet nuclei and dust are
permanent, stable, almost unchanging. They are usually
difficult to control. Reliance is placed on early diagnosis
derived from our parents, grandparents and other people we
and treatment of patients, personal hygiene, and proper
respect. As beliefs are held strongly, they are often difficult
handling of secretions and excretions.
to change.
109. 'C' [Park 20th ed 336)
Option 'D' - Convention is a set of agreed, stipulated or
Screening is defined as "the search for unrecognized disease
generally accepted standards, norms or criteria, often taking
or defect by means of rapidly applied tests, examination or
the form of a custom. A convention may retain the character of
other procedures in apparently healthy individuals."
an "unwritten" law or custom. (Eg: the manner in which people
greet each other, such as by shaking each other's hands) or may
Screening for lung cancer includes t wo techniques i.e., chest
become law and a regulatory legislation may be introduced to
radiograph and sputum cytology. Mass radiography has
formalize or enforce the convention (Eg: laws which determine
been suggested for early diagnosis at six monthly intervals,
which side of the road vehicles must be driven)
but the evidence in support of this is not convincing.
So it is not recommended. It is do ubtful whether the
115. 'C' [Soben Peter 4'" ed 15/ 5th ed 207]
disease satisfies the criteria of suitability for screening.
116. 'A' (Soben Peter 4th ed 13/ 5th ed 39 - 40)
110. 'C' [Park 20th ed 123/ 22nd ed 127)
The five principles of primary health care are
Ref. Q.No. 109
a) Equitable distribution
111. 'B' [Park 2Qth ed 583/ 22nd ed 622) b) Community participation
Organization of member agents that have a c) Inter-sectorial coordination
Society
system of social relationships. d) Appropriate technology
A group of members living together in e) Focus on prevention
Community such a way that the members share basic
conditions of a common life.
Dental ;lut.,e i======
Which of the following is NOT a principle of primary Population Population
health care? (KAR-2013) Centre coverage in coverage in hilly,
a) Inter-sectorial coordination General areas tribal areas
b) Equitable distribution Sub centre 5000 3000
c) Community participation
d) Planning and evaluation Primary health 20,000
30,000
centre (PHC) (AIIMS-14)
117. 'C' [Soben Peter 4th ed 467 / 5th ed 101] Community
1,20,000 80,000
Double pot is most effective and economical method of health centre
disinfecting wells in rural areas. This method was devised by
the National Environmental Engineering Research Institute, 121. 'A' [Park 20'h ed 545 Table 22/ 22nd ed 584 Table 21]
Nagpur, India. This method uses two cylindrical pots, one
placed inside the other. A hole of 1 cm in diameter is made 122. 'B' [Check Explanatfon Below]
in upper portion of inner pot and 4 cm above the bottom in The rich sources of PUFA are sun flower oil, cotton seed
the outer pot. oil, Fish oil, Corn Oil, Soyabean oil etc. Soyabean oil is the
world's leading source of PUFA {61% PUFA)
Mixture of 1 Kg bleaching powder and 2 Kg of coarse sand
is prepared and the inner pot is filled with this mixture. The 123. 'C' [Park 20th ed 380/ Soben Peter 5th ed 211]
inner pot is introduced into the outer one, and the mouth
of the latter closed with polyethylene foil. The double pot 124. ' C' [Jong's Community Dental Health 5th ed 295]
is then lowered at least 1 meter below the water level to "Ripple effect" or "Spread effect" are the terms used
prevent damage by the buckets used for drawing water. to describe the impact of school-based health education
programs on parents. Here the information and skills taught
118. 'B' [Park 20th ed 41/ Soben Peter 5th ed 42] to children are passed to other family members. This ripple
The sequence of events leading to disability and handicap effect is another way of getting a message into the home
are as follows: and community.
Disease (accident)
.j, 125. 'C' [Soben Peter 4'h ed 32-33/ 5th ed 246, 248]
Individual examination in clinical practice is survey in public
Impairment (loss of foot ) health practice. They survey helps in assessment of the socio
.j, economic status, health problems, and the resources of the
community available for eliminating the problem.
Disability (cannot walk)
.j, 126. 'D' [Soben Peter 4th ed 148/ 5th ed 334]
Handicap ( unemployed) The cigarette act 1975 has made it necessary to print
warnings on cigarettes (not on Bidis). In 2003, the Indian
Impairment: parliament passed the 'cigarettes and other tobacco products
Impairment is defined as "any loss or abnormality of act' to prohibit the advertisement of and to provide for the
psychological, physiological or anatomical structure or regulation of trade and commerce in, and production supply
function". and distribution of cigarettes and other tobacco products.

Disability: 127. 'A' [Soben Peter 4th ed 407 / 5th ed 377]


Disability is defined as "any restriction or lack of ability Criteria used in evaluation of dental services (WHO
to perform an activity in the manner or within the 1972)
range considered normal for a human being". Because of a) Effectiveness
impairment, the affected person may be unable to carry out b) Efficiency
certain activities considered normal for his age, sex, etc.
c) Appropriateness
Handicap: d) Adequacy
As a result of disability, the person experiences certain
disadvantages in life and is not able to discharge the a) Effectiveness:
obligations required of him and play the role expected of • It is 'the ratio between the achievement of the program
him on society. This is termed handicap, and is defined activity and the desired level, which, the planners had
as "a disadvantage for a given individual, resulting from proposed would result from the program", i.e., have the
impairment or a disability, that limits or prevents the stated objectives been achieved?
fulfillment of a role that is normal for that individual." • The three variables useful in evaluating effectiveness are
resources, activities and objectives.
119. 'D' [Park 20'h ed 529, Table 3/22nd ed 566, table 3]

120. 'D' [Soben Peter 4th ed 16/ 5th ed 211]


, COMMUNITY DENTISTRY
,......__,,_

V
731

b) Efficiency: c) Taboo:- Taboo is a strong social prohibition or ban against


It is "the result that might be achieved through expenditure words, objects, actions or discussions that are considered
of a specific amount of resources and the result that might undesirable or offensive by a group, culture, society or
be achieved through a minimum of expenditure". community. Eg : Abstinence from beef, pork and smoking in
Hindus, Muslims and Sikhs respectively.
c) Appropriateness:
The appropriateness of the program will be judged by lay 130. ' B' (Soben Peter 4'h ed 135/ 5th ed 319)
decision makers. Tne appropriateness of the strategy of The boost for cancer registration in India was started in
the program should be evaluated because even though a 1982, through initiation of National cancer registry program
progress is effective, it may not be appropriate problem for (NCRP) by Indian counci l of medical research. In India, a
the community. national cancer control program was launched in 1985.

d) Adequacy: 131. 'C' [Soben Peter 4th ed 207/ 5th ed 115)


It is the extent to which the population in need is covered • Option 'A' setting an example means that the health
by the services or the extent to which the services covered educator should follow what he preaches. He should set
the various aspects of the problems. an example for other people to follow.
• Option 'B' comprehension refers to level of understanding
128. ' B' [Park 20'h ed 479/ 22nd ed 516] of the people who receive the health education. The health
Maternal mortality rate measures the risk of women dying educator should first determine the Level of literacy and
from "puerperal causes" and is defined as: understanding of the audience and act accordingly. Words
Total No. of female deaths due to that are strange or new to the people should not be used.
complications of pregnancy, child birth or • Option 'C' credibility is the degree to which the message
within 42 days of delivery from "puerperal is perceived as trust worthy by the receiver. The message
causes" in an area during a given year should be scientifically proven based on facts and should
X 1000 be compatible with Local culture and social goals.
Total No. of live births in the
• Option ' D' reinforcement refers to the repetition needed
same area and year.
in health education. It is not possible for the people to
Ideally the denominator should include all deliveries and learn new things in a short period of time. So, repetition
abortions. As the numerator is not part of the denominator, is a good idea. This principle can be called as "booster
the maternal mortality rate is actually a ratio. dose" in health education.

129. ' D' [Soben Peter 4th ed 182/ 5th ed 173] 132. 'C' [Park 2Qth ed 338, 710, 711/ 22nd ed 753]
Social norms indicate the established and approved ways of Refer Q. No. 95.
doing things, of dress, of speech and of appearance. These
vary and evolve not only through time but also vary from 133. 'A' [Park 20thed 180-182/ 22nd ed 186-89, Table 1)
one age group to another and between social classes and The two types of vaccines used for prevention of poliomyelitis
social groups. Social norms are "the rules that a group uses are
for appropriate and inappropriate values, beliefs, attitudes • Inactivated (Salk) polio vaccine (IPV)
and behaviours". Failure to stick to the rules can result in
severe punishments. The most feared of which is exclusion • Oral (Sabin) polio vaccine (OPV)
from the group. There are three types of norms: folkways,
mores and taboos. Oral polio vaccine (OPV) contains live attenuated virus (type
1, 2 & 3).

a) Folkways:- Folkways are the patterns of conventional


134. ' B' [Park 19th ed 364/ Soben Peter 5th ed 211)
behaviour in a society norms that apply to everyday matters. The Government of India launched "National Rural Health
They are conventions and habits learned from childhood. Mission" on 5th April 2005 for a period of 7 years. The
Breaking or questioning a folkway does not cause severe mission seeks Lo improve rural heallhc.:are delivery syslem.
punishments but may cause the person to be laughed at, The main aim of NRHM is to provide accessible, affordable,
frowned upon or scolded. In Indian culture folkways include accountable, effective and reliable primary healthcare, and
welcoming guests with respect called "Athithi devo bhava". bridging the gap in rural healtticare t hrough creation of a
b) Mores:- Mores are much more strictly enforced cadre of accredited social health activist (ASHA).
than folkways. They are norms or lessons which express
fundamentals values of society. Mores derive from the The scheme proposes new mechanisms like ASHA, Janani
established practices of a society rather than its written Suraksha Yojana (motherhood protection program), and
laws. Mores are distinguished from fo lkway by the severity plans to strenghten the infrastructure of sub-centers, PHCs,
of response they invoke. While breaking a folkway is likely to CH Cs.
t urn heads of passerby, breaking a more will offend observers
and possibly bring punishment. Some important mores are District becomes the core unit of planning, budgeting and
converted into laws in order to ensure implementation. implementation of the programme. All vertical health and
family welfare programmes at district level will merge into
Dental ;lut.,e i======
one common District Health Mission and at state level into empiric foundation rather than a scientific basis, as judged
State Health Mission. There will be provision of mobile by the paucity of pertinent experimental studies reported
medical unit at district level for improved outreach services. in scientific literature. Miller thought that salivary viscosity
was not of great importance in the carious process,
135. 'A' [Soben Peter 4th e d 53/ 5th ed 58] since numerous cases could be found in which saliva was
The six steps involved in descriptive steady are extremely viscous and the patient were free of caries. The
1. Defining the population to be studied reverse also been shown. Occasional workers have reported,
however that a high caries incidence is associated with
2. Describing the diseases under study
thick mucinous salvia. The viscosity of saliva is largely due
3. Describing the diseases in terms of time to the mucin content, but the significance of this substance
in relation to dental caries is not entirely clear.
A) Time 13 7 . 'B' [Sob en Peter 4th ed 506/ 5th ed 133]
a) Short-term fluctuations Informed consent is the first stated and the largest principle
(i) Common source epidemics of Nuremberg Code. The Nuremberg Code identifies four
attributes of consent without which consent cannot be
• Single exposure or point source epidemics considered valid.
• Contin uous or multiple exposure epidemics
Content must be:
(ii) Propagated epidemics
• Person to person. • Voluntary
• Arthropod vector • Legally competent
• Animal reservoir. • Informed
• Comprehending
(iii) slow or modern epidemics
138. 't,; [Textbook of Community Dentistry by Marya 1st ed 94]
b) Periodic fluctuations
Procedural steps in dental public health practice:
i) Seasonal trends • Survey
ii) Cyclic trends • Analysis
c) Long term or secular trends • Program planning
• Ethics (informed consent) and planning approval
B) Place distribution • Program operation
• International variations • Financing
• Nationalvariations • Program appraisal
• Rural-urban variations
139. 'D' [Soben Peter 4th ed 505/ 5th ed 132-33]
• Local distributions
140. 'D' [Park 19th ed 524/ 22nd ed 609]
C) Person distribution
Campylobacter species is most common cause of diarrhoeal
• Age illness worldwide.
• Bimodality
• Gender 141. 'B' [Burt & Eklund 6th ed 3]
Four models of dental professionalism or dentist-patient
• Ethnic group/ ethnicity
relationship:
• Occupation • Commercial model
• Socioeconomic status • Guild model
• Marital status • Agent model
• Behaviour • Interactive model

4) Measurement of the diseases The first is commercial model in which dental care is viewed
5) Comparing wit h known indices as a commodity sold by the practitioner. The services are
thus not based on client's needs, but rather on what the
6) Formulating an etiological hypothesis.
client is able t o willing to buy.
136. 'B' [Soben Peter 4'h ed 91/ 5th ed 262/ Shafer 6th ed
The second is guild model, in which dental care is seen as
427] privilege with the professional dominant in practitioner
The viscosity of saliva has been suggested to be of some
patient relations. In guild model, the professional is the
significance in accounting for differences in caries activity repository of all knowledge and wisdom, the patient is a
between different persons. This idea appears to have an passive recipient, and the practit ioner has an ethical trust
, COMMUNITY DENTISTRY
,......__,,_

V
733

to provide t he best quality care. This model has commonly Corporation share of expenditure on medical care is
seen in the United States. 7/8th of total cost of medical care.

The agent model of dentist-patient relationship reverses Rajiv Gandhi Shramik Kalyan Vojna (Under ESI Scheme)
the dentists and patient's roles from the guild model. • The scheme came into effect from 1st April, 2005.
In this model, the whole decision-making activity in dent al
• It provides an unemployment allowance for the
care is assigned t o t lhe patient.
employees covered under ESI scheme who are rendered
unemployed involuntarily due to retirement/closure of
Interactive model, the dental care is considered as
factory etc..
a partnership of equals. In this model, practitioner
and patient jointly determine care provided through a
CGHS (Central Government Health Scheme)
combination of professional expertise and patient values.
Started by Indian ministry of health and family welfare
in 1954, with the objective of providing comprehensive
142. 'B' [Soben Peter 5th ed 501)
medical care facilities to central governmment employees
Option 'K Incremental dental care is defined as "Periodic
and their family members.
care so spaced that increments of dental disease are treated
at the earlliest time consistent with proper diagnosis and
operating, in such a way that there is no accumulation of Universal Health Insurance Scheme (UHIS)
dental needs beyond the minimum". • It is a govt. sponsored communit based insurance
scheme for below powerty Line (BPL families) .
Option 'B' Comprehensive dental care is "the meeting of
• The scheme provides reimbursement of medical expenses
accumulate dental needs at the time of population group is
upt o Rs. 30,000/- towards hospitalization floated
taken into the program and the detection and correction of
amongst the entire family, death cover due to an
new increments of dental disease on a semiannual / other
accident at 25,000/- to the earning head of the fami ly
periods basis"
and compensation due to loss of earning of the earning
member at Rs 50/- per day upto maximum of 15 days.
143. 'B' [Soben Peter 5th ed 494]
• Option 'K SHARP is - School health additional referral
146. 'B' [Soben Peter 5th ed 479)
program
• Option 'B' Tattle Tooth Program is- Texas Statewide 147. 'C' [Soben Peter 5th ed 211)
Preventive Dentistry Program
• Option 'C' Smile Train is the world's largest cleft lip and 148. 'A' [Park 22nd ed 95, 112)
palate charity • Incubation period is "the time interval between
• Option 'D' THETA Program is - Teenage Health Education invasion by an infectious agent and appearance of the
Teaching Assisstants Program first sign or symptom of the disease in question".
• Colgate's Bright Smiles, Bright futures Program was • The term latent period is used in non-infectious
developed worldwide to teach children positive oral diseases as the equivalent of incubation period in
health habits of basic hygiene, diet and physical activity. infectious diseases. It is defined as the "the period from
This program also encourages the dental professionals to disease inaction to disease detection".
emphasize the importance of oral health as a part of • Median incubation period is defined as "the time
child's overall physical and emotional development. required for 50 percent of cases to occur following
exposure".
144. 'C' • Period of surveillance is usually equal to the maximum
incubation period of the disease.
145. 'A' [Park 22nd ed 760)
ESI and CGHS schemes cover two large groups of wage- The importance of incubation period in epidemiological
earners in country. studies.
a) Tracing the source of infection and contacts and take
ESI (Employees State Insurance)
appropriate control measures.
• Launched in 1948, ESI covers employees drawing
b) Period of surveillance (quarantine) is determined
wages not exceeding Rs. 15,000 permont h. It is run by
contributions by employees and employers and grants c) Helps to prevent clinical illness through immunization.
from central and state governments. d} Identification of point source or propagated epidemics
• The employer contributes 4.75% of total wage bill; based on whether cares will occur within one incubation
the employee contributes 1. 75% of wages. Employees period (point source) or cases occur later than the known
getting daily wages of below Rs. 70/ - are exempted from length of incubation period (propagated epidemics.
payment of contribution. e) To estimate the prognosis of disease.
• The State Govemment's share of expenditure or medical
care is 1/8th of total cost of medical care; the ESI
Dental ;lut.,e i======
The duration of isolation is determined by duration relationship of sugar, starch and caries. The program
of communicability of the disease and the effect of is divided into five levels, each level having its own
chemotherapy on infectivity. defined specific content. The five different levels are

149. 'D' [Check Explanation Below]


Four models for estimating health personnel needs have Pre-school Designed for children too young to read
been described by Hall and Mejia.
Level I Kindergarten through grade 3
a) Health-demands model
b) Health-needs model Level II Grades 4 through 6
c) Service-target model Level III Grade 7 through 9
d) Manpower/ population ratio model.
Level IV Grade 10 through 12
The first three models estimate the number of practitioners
needed on the basis of services required, the fourth model • Evaluation of programme: Dr. Oliver Ezell found that
i.e, manpower/population ratio model has been the method the programme influenced favourably the oral health
of choice in most countries for estimating the demand for behaviour than did the traditional approach.
health personnel. It is easy to use; the cost involved is low
and interpretation is easy.
b) Tattle tooth programme - Texas State wide preventive
dentistry program
150. 'D' [Sturdevant 4th ed Table 3-9]
Caries cavitation is difficult to detect in pits and fissures • Developed in 1974-1976. Tattle tooth II program was
because it is difficult to distinguish from the normal anatomic developed in 1989.
form of these features. Explorer tip binding is not by itself • The basic goal of the program is to reduce dental disease
a sufficient indication to diagnose caries. Injudicious use and develop positive dental habits to last a lifetime.
of the explorer may cause a cavitation. Because of these • The major thrust of tattle tooth is to convince students
confounding factors, the U.S. Public Health Service has that preventing dental disease is important and that
developed additional or criteria for diagnosis of pit and they can do it.
fissure caries.
• Topics covered in the curriculum include correct brushing
and flossing techniques, awareness of importance
These factors are:
of safety and factual information relating to dental
a) Softening at the base of the pit or fissure diseases, its causes and preventive techniques.
b) Opacity surrounding the pit or fissure indicating
undermining or demineralization of the enamel.
c) Askov Dental demonstration:
c) Softened enamel that may be flaked away by the explorer.
• Askov is a small farming community with a population
mostly of Danish extraction.
151. 'A' [Text book of Community dentistry by Satish Chandra
• The demonstration includes:
1st ed 205]
The two types of dental camps are: All methods of preventing dental caries except
communal water fluoridation
a) Open camps: These are for all and any person can utilise
the facilities available in the camp. These types of camps Dental health education
are more useful for the poor rural population. Dental care by nearby dentists
b) Closed camps: These are organised for a particular
preselect ed group Like school children or office or factory d) North Carolina state wide [Preventive dental health
workers. program
• A unique public and private partnership programme with
152. 'D' [Soben Peter 5th ed 498]
preventive and educational activities that modify the
SOME SCHOOL ORAL HEALTH PROGRAMMES:
behaviour patterns of individuals to improve their oral
a) Learning about your oral health- A prevention health habits through dietary changes, tooth brushing
oriented school program and flossing.
• Developed by ADA. • Young children are the primary focus for education
• This is a comprehensive programme covering current because the earlier a child is reached, the greater the
dental concepts. potential for positively affecting the child's attitudes,
• The primary goal is "to develop knowledge, skills and values and behaviour.
attitudes needed for prevention of dental diseases • Fluoride recognized as the most effective public health
among school children". measure for dental caries.
• Consideration is also given to increasing knowledge • The program is unique that, it is designed to reach
regarding diet and dental health, stressing on the several segments of the population. You children,
, COMMUNITY DENTISTRY
~

V
735

parents, teachers, dental professionals and community students, school personnel, families and other members
leaders. of the community through schools.
• Teachers are believe to be key in the educational program • The strategies include:
Research to improve school health programs.
e) Head start-preschool dental health program: Building capacity to advocate for improved school
• Focuses on providing comprehensive education, health, health programs
nutrition and parent involvement services to low-income Strengthening national capacities
children and their families.
Creating networks and alliance for the development
of health promoting schools.
f) Early head start program:
The Goal of WHO's global school health initiative
• Similar to Head start program. is to increase the number of schools that can be
• As a part of this program, dental hygiene students truly be called "Health promoting schools",
provides oral care for children (new born to 3 years age "A health promoting school can be characterized as
children), with a particular emphasis on early childhood a school constantly strengthening its capacity as a
caries. healthy setting for living, learning and working".
• One of the goals of the early head Start program is to
prevent destruction of the dentition and pressure the 153. 'A' [Soben Peter 5th ed 226]
level of oral health found in early infancy. Health agencies around the world:
a) The international red cross and red crescent movement
g) School Health additional referred program (SHARP) b) The Pan American sanitary Bureau (PASB)
• This program was instituted in Philadelphia with the c) OIHP (Office International D'Hygiene publique)
purpose of motivating the parents through home visits. d) The Rock Feller foundation.
• The nurses made daytime visits to families in which e) International Labour Organization (ILO)
mothers were at home. Working parents were contacted
f) The Ford Foundation
by phone.
g) The United Nations Relief and Rehabilitation
Administration (UNRRA)
h) Natural Noshers:
h) Cooperative for assistance and relief everywhere, INC.
• It is a school-based oral health education program that (CARE)
emphasizes home activities for skill development.
i) The Food and Agriculture Organization (FAO)
• Natural Noshers contain two distinct oral health
j) World Health Organization (WHO)
messages, one relating to prevention of gum disease and
the other to dental decay. k) United Nations Children's Fund (UNICEF)
• The results of this study indicate that the parents of l) The Colombo Plan
children who had been taught 'Natural Noshers' had m) United States Agency for International Development
reported new dental information more often than the (USAID)
parents in the control group. n) United Nations Development Program (UNDP)
o) World Bank Groups:
i) Teenage health education teaching assistants The International Bank for Reconstruction and
programme (THETA) Development (IBRD)dl
• The philosophy of the programme is to train high school The International Finance Corporation (IFC)
children to teach preventive dentistry to elementary
The International Development Association (IDA)
school children.
The International Centre for the settlement of
investment disputes (ICSID)
j) Colgate's bright smiles, bright futures:
The Multilateral Investment Guarantee Agency
• Under this program, children in primary schools, receive (MIGA).
instructions in dental care from members of the dental Note:-
profession nominated by the IDA. Currently UNICEF is promoting a campaign known as GOBI
• Education is imparted through audio-visuals and printed campaign, to encourage 4 strategies for a child health
literature. revolution.
• Free dental health care packs are distributed to • G - Growth charts to monitor child development
encourage good oral hygiene. • 0 - Oral rehydration to treat dehydration
• B - Breast Feeding
k) WHO's global school health initiative: • I - Immunization against Measles, Diphtheria, Polio,
• The initiative is designed to improve the health of Pertusis, Tetanus and TB.
Dental ;lut.,e

154. ' C' [Soben Peter 5th ed 215]

155. 'D' [Park 22nd ed 862]

156. 'D' [Jong's Community Dental Health 5th ed 295]

15 7. 'B' [Soben Peter 5th ed 3 5]


• Millennium development goals - 2000 in New York
• Health for all of 21st century - May 1998.

158. 'B' [Park 22nd ed 495]


Low birth weight < 2.5 Kg
Prete rm < 37 Weeks (259 days)
Extreme preterm < 28 Weeks
Very preterm 28 to < 32 Weeks
Moderate to Late
32 to 37 Weeks
pre term
Term babies 37 to 42 Weeks (259-293 days)
Post term > 42 Weeks/294 days of gestation

159. 'B' (Soben Peter 5th ed 110]


• Anatomical wastes such as extracted tooth should be
collected in yellow plastic bag and will be treated by
incinerator or deep burial.
• Extracted tooth with amalgam fillings is considered as
contact amalgam waste. They are placed in separate air
tight containers and will be sent for recycling.
r GENERAL ANATOMY

GENERAL ANATOMY
I. REFERENCE BOOKS TAKEN:
1. B.D. CHAURASIA'S HUMAN ANATOMY- VOLUME III
- HEAD, NECK AND BRAIN - 3rd, 4th, 5th & 6th editions (BDC H & N)

2. HUMAN EMBRYOLOGY BY INDERBIR SINGH - 7th edition (Inderbir singh 7th ed)

3. Text Book of Histology By Inderbir singh - 4th edition

4. B.D. CHAURASIA'S HUMAN ANATOMY- VOLUME II - Lower Limb & Abdomen - 4th & 6th editions

5. B.D. CHAURASIA'S HUMAN ANATOMY- VOLUME I - Upper Limb & Thorax - 4th & 6th editions

6. B.D. CHAURASIA'S GENERAL ANATOMY - 4th edition

1. OSTEOLOGY
1. Bregma is the name given to the junction of the c) Palatine d) Temporal
a) Coronal and sagittal sutures (KAR -97)
b) Frontal bone with the nasal bone 5. Highest point on skull
c) Lambdoid and sagittal sutures a) Pterion b) Porion
d) Two parietal bones c) Lambda d) Vertex
(AIPG -97, MAN -94) (PGI -03)
2. 'Pterion' is: 6. Which of the following structures is not present on
a) Is a point of articulation four skull bones the internal surface of mandible?
b) Is a point where 'bregma' and 'Lambda' meet a) Genia l tubercle b) Mylohyoid ridge
c) It is the region of the antero-lateral fontanelle c) Lingula d) Mental fora men
merge (AIPG -04)
d) Lies deep to the zygomatic arch 7. Among all of the following foramens in the base of
(MAN -2K) skull, which is, the most posteriorly present:
3. Lateral part of middle cranial fossa and posterior a) Foramen spinosum b) Foramen rotundum
cranial fossa are divided by: c) Foramen Lacerum d) Foramen Ovate
a) Petrous temporal bone (AIPG -04)
b) Crista galli 8. Mental foramen is Located
c) Transverse groove a) Between roots of premolars
d) Sphenoid bone b) Between roots of molars
(AIIMS -2K) c) Near canine d) Between Incisions
4. The maxilla articulates with all of the following (AIPG -95)
bones, except one. Identify the exception: 9. The palatine bone furnishes the Link between
a) Frontal b) Zygomatic a) Maxilla and the sphenoid bone

1) A 2) A 3) A 4) D 5) D 6) D 7) C 8) A 9) A
Dental ;lut.,e

b) Sphenoid and the ethmoid bone c) Anterior fontanelle open


c) Sphenoid and the vomer d) Mastoid process is of adult size
d) None of the above (AIPG -99, AIIMS -98)
(APPSC -99) 19. Supra meatal triangle externally represents:
10. Lingula gives attachment to: a) Transverse sinus b) Promontory of middle car
a) Upper medial incisor c) Internal acoustic meatus
b) Sphenomandibular ligament d) Mastoid antrum
c) TemporomandibL1lar ligament (KAR -03)
d) All 20. Anterior limit of infratempo ral fossa is:
(TN PSC -99) a) Lateral pterygoid plate
11. Number of Bones in adult skull are: b) Maxillary posterior wall
a) 18 b) 20 c) Pterygomaxillary fissure
c) 22 d) 40 d) Mastoid process
(AP -03,99) (AIIMS -92)
12. The point where the parieto mastoid, occipito 21. Which of the following muscles originates from the
mastoid, and lambdoid sutures meet is: zygomatic process of maxilla?
a) Pterion b) Obelion a) Middle temporal b) Lateral pterygoid
c) Asterion d) Bregma c) Superficial layer of masseter
d) Posterior portion of the buccinator
13. Which of the following is unpaired bone of facial (TNPSC -99}
skeleton: 22. Muscle, which pulls the disk of TMJ downward:
a) Nasal b) Lacrimal a) Lateral pterygoid b) Medial pterygoid
c) Inferior nasal concha d) Varner c) Digastric d) Mylohyoid
(AIPG -96)
14. Foramen magnum transmits all except: 23. Foramen transversarium transmit
a) Vertebral artery b) Spinal branch 101h nerve a) Inferior jugular vein b) Inferior petrosal sinus
c) Spinal cord d) Vertebral venous plexus c) Sigmoid sinus d) Vertebral artery

15. Structures passing through foramen spinosum is: 24. Structures passing through foramen ovale
a) Accessory meningeal artery a) Emissary vein b) Mandibular nerve
b) Middle meningeal artery c) Trigeminal nerve d) A & B
c) Mandibular nerve d) Maxillary nerve
(KAR -94) 25. Which of the following is present in the posterior
16. Foramen caecum is seen in: cranial fossa in a five year old child:
a) Ethmoid Bone b) Tongue a) Foramen rotundum b) Foramen lacerum
c) Sphenoid d) A and B c) Juglar foramen d) Foramen spinosum
(PGI-2011)
17. Bone better described as "bat with extended wings" 26. Which structures pass through infra orbital fissures
is: a) Superior ophthalmic vein
a) Ethmoid b) Sphenoid b) Ophthalmic artery
c) Nasal d) Mandible c) Trochlear nerve d) Zygomatic nerve
(AIPG -97, 94)
18. ALL the following features of skull of a newborn are 27. Which of the following does not pass through
true except: superior orbital fissure
a) Diploe not formed a) Occulomotor nerve b) Optic
b) Styloid process has not fused with rest of temporal c) Opthalmic division of trigeminal
bone d) Trochlear

10) B 11) C 12) C 13) D 14) B&C 15) B 16) D 17) B 18) D 19) D 20) B 21) C 22) A
23) D 24) D 25) C 26) D 27) B
r GENERAL ANATOMY

28. Which vertebra has the most prominent spine 3 7. The type of suture represented by sagittal suture of
a) C2 b) C1 the cranial vault is
c) \ 0 d) L2 a) Serrate b) Denticulate
c) Squamous d) Plane
29. Accessory meningeal artery enters cranial cavity (COMED-2010)
through: 38. Which of the following is the weakest part of the
a) Foramen lacerum b) Foramen rotundum orbit?
c) Foramen spinosum d) Foramen ovale a) Media l wall b) Lateral wall
(COMEDK- 06) c) Floor of the orbit d) Roof of the orbit
30. Mental spines provide attachment to (KCET-2009)
a) Genioglossus 39. The hyoid bone lies in the midline at the front of
b) Anterior & posterior bellies of digastric the neck at the level of the
c) Mylohyoid a) Third cervical vertebra
d) Superior constrictor of pharynx b) Fourth cervical vertebra
(COM EDK-07) c) Fifth cervical vertebra
31. Following foramina are found in greater wing of d) Seventh cervical vertebra
sphenoid except: (BHU-2012)
a) Foramen Rotundum b) Canaliculus innominatus 40. The orbital opening is somewhat
c) Foramen Spinosum d) Optic canal a) Quadrangular b) Ova l
(KAR-2013) c) Oblong d) Circular
32. The first costochondral joint is a: (BHU-2012)
a) Fibrous joint b) Synovialjoint 41. Mandibular fossa is a part of
c) fmosis d) Synarthrosis a) Mandible bone b) Maxilla bone
c) Sphenoid bone d) Temporal bone
33. The typical cervical differs from thoracic vertebra in (COMED-2012)
that it 42. Which of the following is located medial to the
a) Has a triangular body third molar at the junction of the maxilla and the
b) Has a foramen transversarium horizontal plate of the palatine bone
c) Superior articu lar facet directed backwards and a) Posterior nasal spine
upwards b) Mylohyoid line
d) Has a large vertebral body c) Pterygoid hamulus d) Greater palatine foramen
(AIIMS-2009) (KCET-2012)
34. The joint between the atlas and axis is 43. All of the following canals open on the posterior
a) Synovial wall of the pterygo palatine fossa EXCEPT
b) Closely related to the first cervical nerves a) Greater palatine canal
c) Allow rotation of the head b) Foramen Rotundum
d) Supported by the alar ligaments c) Pterygoid canal d) Palatovaginal canal
(KCET-2010) (COMED-2012)
35. Joint between two bony surfaces linked by cartilage 44. Pneumatic bone is all except?
in the plane of body is called a) Mastoid b) Mandible
a) Syndesmosis b) Symphysis c) Maxilla d) Ethmoid
c) Synchondrosis d) Suture (NEET-2013)
(MCET-2010) 45. What is the number of bones a neonate has in the
36. Which out of the following bones ossify first? skeleton?
a) Mandible b) Nasa l bone a) 270 b) 250
c) Verner d) Occipital c) 230 d) 206
(PGI-2008) (KAR-2013)

28) B 29) D 30) A 31) D 32) D 33) B 34) C 35) C 36) A 37) A 38) A 39) A 40) A
41) D 42) D 43) A 44) B 45) A
Dental ;lut.,e

46. Deepest layer of deep cervical fascia is c) Nasal d) Occipital


a) Prevertebral b) Carotid sheath (PGI JUNE-13)
c) Pretracheal d) Temporal
(AIIMS MAY-13)
4 7. Not a part of ethmoid bone is
a) Inferior turbinate b) Agar nasi cells
c) Uncinate process d) Crista galli
(AIIMS NOV-13)
48. A dome shaped skull is known as
a) Brachy-cephaly b) Oxy-cephaly
c) Scapho-cephaly d) Rhomb-encephaly
(GCET-14)
49. The suture between the two halves of the Frontal
bones is
a) Metopic b) Symphysis
c) Mendosa[ d) Coronal
(GCET-14)
50. The cribriform plate connects
a) Nasal cavity and anterior cranial fossa
b) Nasal cavity and orbit
c) Nasal cavity and oral cavity
d) Anterior cranial fossa and orbit
(AIIMS MAY-14)
51. The pneumatisation process in maxillary sin us
completes in?
a) 6-8 years b) 13-14 years
c) 8-10 years d) 18-20 years
(PGI DEC -13)
52. Inca bone or Goethe's ossicles is a sutural bone
present in?
a) Bregma b) Lambda
c) Pterion d) Asterion
(PGI DEC-13)
53. In the base of the skull which of the following
foramen is most anteriorly placed?
a) Stylomastoid foramen
b) Foramen spinosum
c) Foramen ovale
d) Juglar fora men
(PGI DEC-13)
54. In a five year old child which of the following
fora men is Located in posterior cranial fossa?
a) Jugular Foramen b) Foramen Rotundum
c) Fora men Spinosl.lm d) Foramen Lacerum
(PGI JUNE-11)
55. Which of the following is a pneumatic bone?
a) Maxilla b) Mandible

46) A 47) A 48) B 49) A 50) A 51) D 52) B 53) C 54) A 55) A
r GENERAL ANATOMY
~

V
741

1. OSTEOLOGY - ANSWERS
1. 'A' [BDC H & N 4th ed 4/ 5th ed 5] 11. 'C' [BDC H & N 4th ed 1]
Vertex Highest point on sagittal suture Out of 22 bones (8+14)
a) The calvaria or brain case is composed of 8 bones.
Meeting point between the coronal and
Bregma b) The facia l skeleton is composed of 14 bones.
sagittal sutures
Meeting point between the sagittal and
Lambda According to 6th ed the No of bones are 28 (14 + 14)
lambdoid sutures
It is the point on the sagittal suture
Obelion 12. 'C' [BDC H & N 4th ed 5/ 5th ed 5]
between two parietal fo ramen
Obelion is the point on the sagittal suture between
It is the area in the temporal fossa where the two parietal fora men.
4 bones i.e, fronta l, parietal, temporal
and greater wing of sphenoid join to form 13. 'D' [BDC H & N 4th ed 1/ 5th ed 1]
Pterion a H-shaped suture. The anterior branch
Mandible and vomer are the two unpaired bones of
of middle meningeal artery, middle
facial skeleton.
meningeal vein and lateral sulcus of
cerebral hemisphere lies deep to pterion.
14. 'B & C' [BOC H & N 4th ed 18/ 5th ed 19]
It is the meeting point of parietomastoid, Structures passing through FORAMEN MAGNUM are:
Asterion
occipitomastoid, and lambdoid sutures
i) Through anterior part

2. 'A' [BDC H & N 4th ed 10, 280/ 5th ed 11, 283] • Apical ligament of dens
Pt erion is the area in the temporal fossa where 4 bones • Membrana tectoria
(frontal, parietal, temporal and sphenoid) adjoin each ii) Through subarachnoid space
other forming an H-shaped suture. • Spinal accessory nerve
• Vertebra l arteries
3. 'A' [BDC H & N 4th ed 23/ 5th ed 25]
• Anterior and posterior spinal arteries
4. 'D' [BDC H & N 4th ed 7/ 5th ed 8] iii) Through posterior part
Maxilla articulat es with nasal, lacrimal, frontal, • Lower part of medulla
palatine and zygomatic bones. • Tonsils of cerebellum
• Meninges
5. 'D' [BDC H & N 4th ed 5/ 5th ed 5]

15. 'B' [BDC H & N 4th ed 18/ 5th ed 18]


6. 'D' [BDC H & N 4th ed 32/ 5th ed 8, 32, 34]
The foramen spinosum transmits the middle meningeal
Mental foramen is present on the outer surface of body
artery, meningeal branch of mandibular nerve (nervus
of mandible between the roots of two premolars.
spinosus) and the middle meningeal vein.
7. 'C' [BDC H & N 4th ed 22/ 5th ed 15, 23]
16. 'D' [BDC H & N 4th ed 21, 250/ 5th ed 20, 250]
8. 'A' [BDC H & N 4th ed 32/ 5th ed 8, 32, 34]
17. 'B' [BDC H & N 4th ed 22/ 5th ed 22]
9. 'A' [BDC H & N 4th ed 14]
18. 'D' [BDC 3rd ed 309/ 5th ed 11]
In children, mastoid process first appears during the
10. 'B' [BDC H & N 4th ed 32/ 5th ed 32]
second year.
Lingula is a sharp tongue shaped projection at the
anterior margin of mandibular foramen . It gives
19. 'D' [BDC H & N 4th ed 263/ 5th ed 11, 259]
attachment to sphenomandibular ligament.
Dental ;lut.,e

20. 'B' [BDC H & N 4th ed 11/ 5th ed 147) 32. 'D' [Grays 39th ed 952)

21. 'C' [BDC H & N 4th ed 145/ 5th ed 10, 149) 33. 'B' [BDC Vol. 3, 4th ed 40, 41/ 5th ed 40)
Foramen transversarium is typically exclusive to the
22, 'A' [BDC H & N 4th ed 152/ 5th ed 156) transverse process of cervical vertebrae. It is not
present in thoracic and lumbar vertebra.
23. 'D' [BDC H & N 4th ed 202/ 5th ed 200)
Fora men transversarium transmits:
24. 'D' [BDC H & N 4th ed 18/ 5th ed 18) • The vertebral artery
• The vertebral veins
25. 'C' [BDC H & N 5th ed 25)
• Branch from inferior cervical ganglion.
26. 'D' [BDC H & N 4th ed 28/ 5th ed 28)
34. 'C' [BDC H & N 5th ed 203)
The inferior orbital fissure transmits
'Atlas' is the fi rst cervica l vertebra which is just under
• Maxillary nerve
the head. The 'Axis' is the second cervica l verbra. The
• Zygomatic nerve joint between the atlas and axis is a pivot type of
• Orbital branches of pterygopalatine gang lion joint. It is called atlanto-axial joint. It allows rotating
• Infraorbital vessels the head left and rig ht.

27. 'B' [BDC H & N 4th ed 25/ 5th ed 26) 35. 'C' [Inderbir Singh 8th ed 124/ BDC General
Superior orbital fissures transmits Anatomy 4th ed 63)
Clavicle is the first bone to ossify and mandible is the
• Lacrimal, frontal, trochlear nerves in its lateral part
second bone to ossify in th body.
• Occulomotor, naso celiary and abducent nerves in
its medial part
36. 'A' [BDC H & N 5th ed 34)

28. 'B' [BDC H & N 4th ed 43/ 5th ed 43)


37. 'A' [BOCH & N 5th ed 5 Fig 1.2)
Sagittal, Lambdoid and coronal sutures are serrated
29. 'D' [BDC H & N 4th ed 17/ 5th ed 18, 22 Fig 1.21]
sutures.
Refer Synopsis Point No. 8
Denticulate means very small teeth like projections.
30. 'A' [BDC H & N 4th ed 158/ 5th ed 164, 252)
Genial tubercles are 4 in number, superior and inferior.
Squamous means scaly or plate-like.
Geniog lossus originates from superior genial tubercles
or superior mental spines while geniohyoid originates
38. 'A' [BDC H & N 5th ed 28)
from interior mental spines or inferior genial tubercles.
The medial wall is thin and is formed by the fronta l
process of maxilla, lacrimal bone, orbit al plate of
31. 'D' [BDC H & N 4th ed 21, 22 Fig 1.21/ 5th ed 22 ethmoid and sphenoid. The lateral wa ll is the thickest.
Fig 1.21]
So option 'K is correct.
The foramens that pass through greater wing of
sphenoid are (ROSE) i.e 39. 'A' [BDC H & N 5th ed 39]
• Foramen Rotundum
• Hyoid bone is 'U' shaped bone
• Foramen Ovale
• It develops from 2nd and 3rd branchial arches.
• Fora men Spinosl.lm
• Situated in anterior mid line of the neck between
• Emissary sphenoidal foramen or foramen vesalis. the chin and thyroid cartilage.
• Lies at the level of 3rd cervical vertebra behind and
The lesser wing of sphenoid is connected to body
the base of the mandible in front.
of sphenoid by ant erior and posterior root s, which
enclose optic canal. Optic canals lead to orbit.
r GENERAL ANATOMY

40. 'A' [BOCH & N 5th ed 7) These fascia bounded connective tissue spaces in the
neck are important clinically because they provide
41. ' O' [BOCH & N 5th ed 154) routes for the spread of inflammatory processes. The
Mandibular fossa is a part of temporal bone. It along deepest layer of the deep cervical fascia, is called the
with articular tubercle forms the upper articular surface prevertebral layer.
of TMJ.
47. 'K [Check Explanation Below]
Not e: Lower articular surface is formed by head of The cells of ethmoid are agar nasi cells, terminal
mandible air cells, infundibular air cells and conchae bullosa
(Pneumatized anterior middle turbinate). The parts of
42. ' O' [BOCH & N 5th ed 46) ethmoid bone are cribriform plate (horizontal plate),
perpendicular plate and two lateral labyrinths. The
43. 'A' [BOCH & N 5th ed 148) uncinate process is a thin curved bar of bone from the
• Greater palatine canal opens in posterior surface of lateral side of ethmoida l labyrinth. Inferior turbinate
hard palate. or inferior nasal choncha is part of Nose.
• Foramen rotundum - middle cranial fossa
48. ' B' [Check Explanation Below]
• Foramen lacerum - pterygoid canal Cranio-synostosis is condition in which one or more of
• Palatovaginal canal - pharynx the sutures of infant skull fuses prematurely. It leads
to deformity of the skull.
44. 'B' [BOCH & N 5th ed 4, 230)
Pneumatic bones contain air filled spaces. These are
Type of
confined in close proximity to nasal cavity. Premature closure of
Craniosynostosis
Eg:- Maxilla
Oxycephaly All Sutures
Ethmoid
Sphenoid Brachycephally (Skull
Bilateral Coronal suture
Frontal expands horizontally)
Temporal Scaphocephaly (long
Sagittal suture
narrow skull)
Mandible is not a pneumatic bone. Plagiocephaly
(Asymmetrical, oriented Unilateral coronal suture
45. '/( [Human Anatomy by Saladin, Kenneth 2007 yr to one side)
ed 203) Platycephaly (Tower
Unilateral
At birth a newborn baby has 270 bones. In adult skull with the peak at
occipitoparietal
human the skeleton has 206. This is due to fusion of the occiput)
some bones. Among 206 bones, half of them are in the
hands and feet. Other names of Oxycephally are acrocephaly or tower
skull or steeple skull or turricephaly or dome shaped
46. 'A' [BDC H&N 5th ed 81) skull.
The deep fascia of the neck is condensed to form the
following layers: 49. 'K [Refer Synopsis Point 4]

1. Investing layer
50. 'K [BOC H&N 6th ed 45]
2. Pretracheal layer
The cribriform plate of the ethmoid bone separates
3. Prevertebral layer the anterior cranial fossa from the nasal cavity. It
4. Carotid Sheath is a horizontal perforated bony lamina, occupying
5. Buccopharyngeal fascia ethmoidal notch of frontal bone. The cribriform plate
6. Pharyngobasilar fascia contains foramina for olfactory nerve rootlets.

51. 'D' [Misch dental implant 2nd ed 556]


Dental ;lut.,e

• The primary pneumatisation of the maxillary sinus • Foramen spinosum


occurs at about 3 months of fetal development. • Jugular foramen
• By the chi ld's first year, the maxi llary sinus expands • Stylomastoid foramen
laterally underneath the infraorbital canal, which is
protected by a thin bony ridge. The antrum grows 54. 'A' (BDC H&N 6th ed Pg 17)
apically and replaces the space previously occupied
• Foramen rotundum, ovale, spinosum and foramen
by developing dentition.
lacerum are present in the middle cranial fossa.
• At 12 years of age, pneumatisation extends to the
• Jugularforamen, fora men magnum and stylomastoid
plane of lateral orbital wall, and the sinus floor is
fora men are present in the posterior cranial fossa.
level with the floor of the nose.
• During later years, pneumatisation spreads 55. 'A' (BDC H&N 6th ed 35)
inferiorly as the permanent teeth erupts. The pneumatic bones (the internal cavities are
• Antero-posteriorly, the sinus expansion corresponds filled with air) of the skull are:
to the growth of mid-face and is completed only • Mastoid process of temporal bone
with the eruption of 3rd permanent molars when
• Maxillary sinus
the person is about 16 to 18 years of age.
• Sphenoid sinus
52. 'B' (Grays Anatomy 40th ed 1279) • Eth moid sin us
Wormian bones, also known as intra sutural bones, are
extra bone pieces that occur within a suture in the
cranium. These are inegular isolated bones that appear
in addition to the UJsual centers of ossification of the
cranium. An isolated wormian bone at the lambda is
referred as an Inca bone or Goethe's Ossicle. Inca
bones are also named as interparietal bones as they
lie in between the two limbs of the lambdoid suture of
the parietal bones.

Wormian bones are a marker for some diseases and


important in the primary diagnosis of brittle bone
disease: osteogenesis imperfecta.

Wormian bones may also be seen in:


• Pycnodysostosis
• Osteogenesis i mperfecta
• Rickets
• "Kinky-hair" Menke's syndrome
• Cleidocranial dysostosis
• Hypoparathyroidism and hypophosphatasia
• Otopalatodigital syndrome
• Primary acro-osteolysis
• Down's syndrome

53. 'C' (BDC H&N 6th ed Pg 14 Fig 1.11a)


The foramena situated in the base of the skull from
anterior to posterior are:
• Fora men ova le
r GENERAL ANATOMY
~

V
745

2. SALIVARY & THYROID GLANDS


1. Submandibular gland is situated in : c) Sternohyoid d) Dig astric
a) Digastric triangle b) Carotid triang le (AIIMS -99)
c) Muscular triang le 9. Sub Lingual gland is situated between:
d) Deep to hyoglossus muscle a) Hyoglossus and genioglossus
(AIPG -95) b) Hyog loss us and sty log loss us
2. Secretomotor supply of parotid comes from : c) Genioglossus and mandible
a) Greater Petrosal nerve d) Geniohyoid and geniog lossus
b) Auriculotemporal nerve (KAR -97)
c) Maxillary nerve d) Chorda Tympani 10. Which of the following structures is not found in
(AIPG -02, 94) parotid gland:
3. All the following nerves may be injured while doing a) ECA b) Facial nerve
excision of submandibular salivary glands except: c) Auriculo temporal nerve
a) Lingual nerve b) Spinal accessory nerve d) Buccal branch of mandibular nerve
c) Mandibular branch of facia l nerve (KAR -98)
d) Hypoglossal nerve 11. Isthmus of thyroid gland is across tracheal rings
(KAR -03) a) 2 nd to 4 th b) 3 rd to 5th
4. The orifice of the parotid duct is Located: c) 5 th and 5 t h d) 4 th only
a) At the hamular notch
b) In proximity to the incisive papillae 12. The left subclavian artery is a branch of
c) On the buccal mucosa near the maxi llary second a) Brachio cephalic trunk
molar b) Ascending aorta
d) Slightly posterior to the mandibular central incisors c) Arch of aorta d) Left common carotid
(AIPG -03)
5. Duct of parotid gland crosses over masseter muscle 13. Inferior thyroid artery arises from
and pierces the: a) 1'1 part of subclavian artery
a) Buccinator muscle b) Superior constrict or b) Thyrocervical trunk of 1'1 part of subclavian artery
c) Masseter d) Zygomaticus major c) 3rd part of subclavian artery
(AIIMS -89) d) Internal carotid artery
6. The secretomotor supply of the parotid gland is (COMEDK -06)
through: 14. Superior parathyroid glands are derived from
a) Otic ganglion b) Gasserian gang lion a) 1•1 branchial pouch b) 2 nd branchial pouch
c) Geniculate gang lion d) Submandibular ganglion c) 3rd branchial pouch d) 4 th branchial pouch
(PGI -01)
7. Surgical excision of parotid gland endangers which 15. Middle thyroid vein drain into _ _ _ vein
of the following structures: a) External jugular vein
a) Hypoglossal nerve b) Anterior jugular vein
b) Motor nerve of the muscles of mastication c) Internal jugular vein
c) External carotid artery, auriculotemporal nerve, d) Brachiocephalic
facial nerve
d) Lesser occipital nerve and spinal accessory nerve 16. The level of branching of common carotid artery
(PGI -99) a) Upper border of thyroid cartilage
8. Which of the following muscle divides the sub- b) Lower border of cricoid
mandibular gland into a superficial and deep part? c) Upper border of cricoid
a) Gen iog loss us b) Mylo hyoid d) Hyoid

1) A 2) B 3) B 4) C 5) A 6) A 7) C 8) B 9) C 10) D 11) A 12) C 13) B


14) D 15) C 16) A
Dental ;lut.,e

2. SALIVARY & THYROID GLANDS - ANSWERS


1. 'A' [BDC H & N 4th ed 158/ 5th ed 165)

Gland Situation Duct opening Secretion

Below external acoustic meatus Stensons duct - After piercing the


between the ramus of mandible and buccinator opens into the vestibule of
Parotid Purely serous
the sternocleidomastoid. It is the mouth op posite the crown of maxillary
largest of all the salivary glands z nd molar

Roughly 'J' shaped salivary gland, Wharton's duct opens on t he floor


situated in the anterior part of of mouth, on the summit of the sub- Mixed and
Sub-
di gastric triangle. The gland is divided lingual papilla, at the side of frenulum predominantly
mandibular
into large superficial and small deep of the tongue serous
parts by mylohyoid muscle
Smallest of the t hree salivary glands. BarthoUns duct - 10-15 ducts emerge
It is situated above the mylohyoid, from the gland. Most of them open Mixed and
Sublingual
below the mucosa of the floor of the directly into the floor of mouth on the predominantly
gland
mouth summit of sublingual fold. A few of mucous
t hem join the submandibular duct

2. 'B' [BDC H & N 4th ed 137/ 5th ed 145) 10. 'D' [BDC H & N 4th ed 136/ 5th ed 143, 144)
The preganglionic fibres begin in the inferior salivatory Structures within the Rarotid gland:
nucleus, pass th rough the 9th nerve and relay in the • Arteries - External carotid artery
otic ganglion. The postgang lionic fibres reach the
gland through the auriculotemporal nerve.
• Veins - Retromandibular vein
• Nerves - facial nerve & its branches
3. 'B' [BDC H & N 4th ed 161-62/ 5th ed 159, 166) • Parotid lymph nodes
Option 'I\ lingual nerve supplies sensory fibres to
submandibular gland. Option 'D' Hypoglossal nerve is 11. 'A' [BDC H & N 4th ed 166/ 5th ed 172)
related to medial part of hyog lossus muscle.
12. 'C' [BOC H & N 4th ed 173/ 5th ed 177, 178)
Option 'C' mandibular branch of facial nerve passes Subclavian artery is the principal artery of the upper
posteroinferior to angle of mandible, so the incision limb. It also supplies a considerable part of the neck
must be placed more than one inch below the angle to and brain t hrough its branches.
preserve the nerve.
On the right side, it is a branch of the brachia cephalic
4. 'C' [BDC H & N 4th ed 137/ 5th ed 144) artery. On the left side it is a branch of arch of aorta.

5. 'A' [BDC H & N 4th ed 136/ 5th ed 144) 13. 'B' [BDC H & N 4th ed 169/ 5th ed 179)
The thyroid gland is supplied by the superior and
6. 'A' [BDC H & N 4th ed 137/ 5th ed 145) inferior thyroid arteries. The superior thyroid artery is
first anterior branch of external carotid artery while
7. 'C' [BDC H & N 4th ed 136-37/ 5th ed 146) the inferior thyroid artery arises from thyrocervical
trunk of 1st part of subclavian artery. I n 8% of the
8. 'B' [BDC H & N 4th ed 158/ 5th ed 166) individuals; the thyroid is supplied by "thyroidea ima
artery".
9. 'C' [BDC H & N 4th ed 162/ 5th ed 167)
Sublingual gland is situated medial to the sublingual The superior thyroid artery supplies one third of
fossa of the mandible and lateral to genioglossus thyroid lobe and the upper half of isthmus The inferior
thyroid artery supplies lower 2/3'd' of thyroid lobe and
r GENERAL ANATOMY

lower half of isthmus, sternomastoid muscle trachea, Postganglionic fibres from the ganglion finally enter
parathyroid, thymus, larynx and pharynx. the submandibular gland through branches of ganglion.
The nerve supply of sublingual gland is also similar to
14. 'D' [BOCH & N 4th ed 171/ 5th ed 176] submandibular gland.
The superior parathyroids are also known as parathyroid
IV because they develop from the endoderm of 4th 19. 'C' [BOCH & N 5th ed Pg 166]
pharyngeal pouch. The inferior parathyroids are also Structures on medial surface of submandibular
known as parathyroid III because they develop from 3rd gland
pharyngeal pouch. • Mylohyoid
Pouches Derivatives • Hyoglossus &
• Styloglossus
• Pharyngo tympanic tube
1'1 Pouch • Middle ear cavity 20. 'D' [BOCH & N 4th ed 171/ 5th ed 177]
• Tympanic antrum
• Tonsil 21. 'B' [BOCH & N 4th ed 63/ 5th ed 65]
2"d Pouch
• Tubotympanic recess 22. 'C' [BOCH & N 4th ed 236/ 5th ed 65, 234]
• Inferior parathyroid
Lacrimatory nucleus
3,d Pouch glands,
• Thymus
Pouch Nervus intermedius
4 th Superior parathyroid glands
-,I,
Pouch (ultimo
5 th
Para follicular cells of thyroid Genicu late ganglion
branchial pouch)
-,I,

15. 'C' [BOCH & N 4th ed 170/ 5th ed 174, 175] Greater petrosal nerve
The thyroid is supplied by superior, middle and inferior -,I,
thyroid veins. Nerve of pterygoid canal
-1.,
The superior and middle thyroid veins drains into Pterygopalatine/ Sphenopalatine ganglion
internal jugular vein while the inferior thyroid vein
-,I,
drains into left brachiocephalic vein.
Zygomatic Nerve
Sometimes, "a fourth thyroid vein of Kocher" may -1.,
be present between the middle and inferior veins. It Zygomatico temporal Nerve
drains into internal jugular vein. -,I,
Lacrimal Nerve
16. 'A' [BOCH & N 4th ed 177/ 5th ed 134]
Right common carotid artery is a branch of
Lacrimal gland
brachiocephalic artery. Left common carotid artery is a
branch of arch of aorta.
23. 'C' [BOCH & N 4th ed 212/ 5th ed 211, 212]
Nerve supply of soft palate
17. 'B' [BOCH & N 4th ed 136-39/ 5th ed 144]
Sensory:
• Middle and posterior lesser pa latine nerves
18. 'C' [BOCH & N 4th ed 162/ 5th ed 167]
(maxillary nerve)
Sensory nerve supply of submandibular salivary gland
is from lingual nerve. The secretomotor fibres in the • Glosspharyngeal nerve
superior salivatory nucleus pass through the facial
nerve, chordatympani, and the lingual nerve to reach
the submandibular ganglion.
Dental ;lut.,e

Motor: 27. 'D' [BDC H & N 4th ed 136/ 5th ed 143]


• All muscles except tensor veli palatini supplied by External carotid artery, retromandibular vein,
pharyngeal plexus. (cranial part of accessory nerve superficial temporal vessels, facial nerve and parotid
through vagus) lymphnodes are the structures passing through the
parotid gland.
• Tensor palate is supplied by mandibular nerve.

Special sensory (Gustatory) : Lesser palatine nerves. 28. 'B' [BDC H & N 4th ed 160/ 5th ed 166]

29. 'B' [BOCH & N 4th e d 136/ 5th ed 144)


24. 'A' [BDC H & N 4th ed 156, 162/ 5th e d 145, 167]
Ref Q. No 26
Parasympathetic fibres to parotid gland begin in the
inferior salivary nucleus, pass through IX nerve and
30. 'A' [BOCH & N 4th ed 159, 160/ 5th ed 167)
relay in otic ganglion. The postganglionic fibres reach
Sublingual is smallest of all major salivary glands.
the gland through auriculotemporal nerve.
It lies above Myalohyoid below the mucosa of the
Parasympathetic fibres to submandibular and floor of the mouth. Submandibular gland is roughly 'J'
sublingual salivary glands begins in the superior shaped and is divided by myalohyoid into large part
salivatory nucleus, pass through VII nerve, chorda which is superficial to the muscle and a small part
lying deep to the muscle.
tympanic nerve, and the lingual nerve, to reach the
submandibular ganglion. Postgang lionic fibres emerge
31. 'B' [BDC H & N 4th ed 156/ 5th ed 160]
fro m the ganglion and reach the submandibular gland.
Refer to exp la nation of Q. No 24.
25. 'D' [BDC H & N 4th ed 136/ 5th ed 14 3]
Arteries within parotid gland Preganglionic fibres from inferior salivatory nucleus
• External carotid artery ,!,
• Maxillary artery IX Nerve
• Tern poral vessels ,!,
• Posterior auricular artery Tympanic branch
,!,
Veins within parotid gland Tympanic plexus
• Retromandibular vein ,!,
• Anterior and posterior divisions of retromandibular Lesser petrosal nerve
vein
,!,
Otic gang lion
Nerves within parotid gland
,!,
• Facial nerve and its termina l branches (Improper
inferior alveolar nerve block causes transient facial Post ganglionic fibres
paralysis) ,!,
Auriculotem po ral nerve
26. 'B' [BDC H & N 4th ed 136/ 5th ed 144] ,!,
Parotid duct is a thick walled and 5cm long duct Parotid gland
and it emerges from the middle of anterior border of
the gland. It runs forwards and slightly downwards 32. 'C' [BDC H & N 4th ed 136/ 5th ed 144]
on the masseter. At the anterior border of masseter,
It runs medially and pierces buccal pad of fat, the 33. 'C' [Sembulingam 5th ed 224)
buccopharyngeal fascia and the buccinator (obliquely). Sublingual salivary glands are drained by 8-20
The duct runs fo rwards for a short distance and fina lly excret ory ducts called the ducts of Rivinus. The largest
opens into the vestibule of the mouth opposite the (major) duct of these is called the Bartholins duct.
crown of upper second molar teeth.
r GENERAL ANATOMY
~

V
751

3. MOUTH, PHARYNX & LARYNX


1. The nerve that is related to pyriform recess in 10. Main motor nerve supply to the pharynx is:
pharynx: a) Vagus nerve b) Accessory nerve
a) Recurrent laryngeal b) External laryngeal c) Glossopharyngeal nerve
c) Internal laryngeal d) Glosso pharyngeal d) Facial nerve
(MAN -02, KAR -01) (AIPG -96)
2. Cricothyroid is supplied by: 11. Soft palate is made up of:
a) Vagus b) Recurrent laryngeal nerve a) Palatoglossus and palatopharyngeus
c) Hypoglossal nerve d) Superior laryngeal nerve b) Uvula
(MAR -97) c) Mucous membrane and muscles
3. Muscles which open the glottis are: d) All of the above
a) Cricothyroid b) Thyroa rytenoid (AIIMS -90)
c) Posterior cricoarytenoid 12. The second stage of deglutition is characterized by
d) Lateral cricoarytenoid a) Elevation of larynx
(MAN -2K) b) Momentary apnoea
4. Naso pharynx consists of all except: c) Peristalsis of Pharyngo esophageal sphincter
a) Pyriform recess b) Pharyngeal recess d) Relaxation of pharyngeal constrictors
c) Pharyngeal tonsil d) Salpingopharyngeal fold (MAN-2K)
(AIPG -93) 13. What is maxillary air sinus?
5. Lymph from lower lip-middle part drains directly a) Paranasal air sinus situated close to maxilla
into: b) Venous sinus situated close to maxilla
a) Submandibular nodes c) Paranasal air sinus situated in maxilla
b) Submental nodes d) Venous sinus extending in to the maxilla
c) Sublingual nodes d) Preauricular nodes (KAR-2K)
(AIIMS -94) 14. Epithelium in vocal card is
6. The place where the hard palate is continuous with a) Pseudo stratified columnar
soft palate posteriorly is overlapped by: b) Stratified squamous
a) Alveolar periosteum b) Periosteum c) Simple columnar d) Cuboidal epithelium
c) Mucoperiosteum d) An aponeurosis (AIPG-2002)
(KAR -97) 15. First sin us developed:
7. Abductors of larynx are: a) Maxillary sinus b) Frontal sinus
a) Posterior cricoarytenoids c) Sphenoidal sinus d) Ethmoidal sinus
b) Transverse arytenoids (AIPG-2K)
c) Cricothyroid d) All of the above 16. The frontal paranasal sinus drains into the:
(AIPG -2K, AIIMS -01) a) Superior meatus b) Middle meatus
8. The weakest part of pharynx is: c) Inferior meatus d) Spheno-ethmoidal recess
a) Sinus of morgangini (AIIMS-99)
b) Between thyropharyngeus and crico-pharyngeus 17. Sphenoidal air sinus is supplied by which nerve:
c) Pyriform fossa d) Pharyngeal recess a) Posterior ethmoidal b) Posterior superior
(KAR -03) c) Sphenoidal d) Infratemporal
9. All the following muscles of larynx are supplied by (AIIMS-98)
the recurrent laryngeal nerve except: 18. Nasolacrimal duct opens into
a) Cricohyoid b) Cricothyroid a) Superior meatus b) Middle meatus
c) Arytenoid d) Aryepiglotticus c) Inferior meatus d) none of above
(AIPG , AIIMS -99) (TN PSC-99)

1) C 2) D 3) C 4) A 5) B 6) D 7) A 8) B 9) B 10) B 11) D 12) A 13) C


14) B 15) A 16) B 17) A 18) C
Dental ;lut.,e

19. All of the following statements regarding pharynx b) Developed from 4th branchial cleft
are correct except c) Drained by jugulo digastric lymph nodes
a) The opening of auditory tube is located in the d) Crypts lined by stratified columnar epithelium
lateral wall of nasopharynx
b) The soft palate is at the level of separation of 28. The esophagus commences at the following level
nasopharynx and! the oropharynx a) lower end of cricoid
c) The pharynx is continuous with the esophagus at b) C5 vertebra
the level of sixth cervical vertebrae c) 10cm from incisor teeth
d) The afferent limb of the gag reflex is 101h cranial d) C7
nerve; the efferent limb is IX cranial nerve.
29. Regarding palatine tonsil, which of the following is
20. Which laryngeal cartilage is above glottis an incorrect statement.
a) Arytenoid b) Epiglottis a) Lies on the superior constrictor
c) Cricoid d) Thyroid b) Lymph form it drains into j ugulo di gastric group of
deep the cervical nodes
21. Damage to internal laryngeal nerve results in c) Has sensory innervations from vagus
a) Hoarseness b) Loss of timbre of voice d) is a derivative of the second pharyngeal pouch
c) Anesthesia of larynx d) Breathing difficulty
30. Posterior boundary of carotid triangle is
22. Damage to external laryngeal nerve results in a) Sup. Belly of omohyoid
a) Hoarseness b) Loss of timbre of voice b) Post. Belly of digastric
c) Anesthesia of larynx d) Breathing difficulty c) Sternohyoid d) Sternocleidomastoid

23. Lymph from tonsils drain into the 31. The only pharyngeal muscle innervated by the
a) Jugulo omohyoid node glossopharyngeal nerve is the
b) Jug ulo digastric node a) Stylo pharyngeus b) Palatopharyngeus
c) Submental node d) Upper cervical node c) Superior constrictor d) Middle constrictor

24. Main arterial supply of the tonsil is from 32. Nerve supply of the mucosa of larynx is
a) Facial artery a) External laryngeal and recurrent laryngeal
b) Ascending pharyngeal artery b) Internal laryngeal and recurrent laryngeal
c) Palatine artery d) Greater palatine artery c) External laryngeal d) Superior laryngeal

25. Main nerve supply of palatine tonsils is 33. What is true about pharyngotympanic tube
a) Lesser palatine nerve (Eustachian tube)
b) greater palatine nerve a) It opens into oropharynx
c) glossopharyngea l nerve b) Supplied by IX nerve
d) Facial nerve. c) Lined stratified squamous epithelium
d) It is cartilaginous throughout its length
26. The narrowest part of GIT is
a) 60 cm proximal t o ileoceca l junction 34. Larynx extends from
b) 1'1 part of duodenum a) C2 - C7 b) Cl - C4
c) Esophago gastric junction c) C5 - C6 d) C3 - C6
d) Pharyngo esophageal junction
35. Waldeyer's lymphatic chain is formed by all except
27. Which is true of tonsils a) Palatine tonsils
a) Lies on inferior constrictor b) Pharyngeal tonsils

19) D 20) B 21) C 22) B 23) B 24) A 25) C 26) D 27) C 28) A 29) C 30) D 31) A
32) B 33) B 34) D 35) D
r GENERAL ANATOMY

c) Tubal and lingual tonsils 43. Which of the following statements regarding middle
d) Post auricular nodes constrictor muscle is wrong
(AIIMS MAY- 2014) a) Lies on superior constrictor
36. Tympanic plexus is formed by b) Lymph from it drains in to the deep cervical lymph
a) Tympanic branch of IX nerve nodes
b) Vagus nerve c) Has sensory innervation from vagus
c) Facial nerve d) Mandibular nerve d) Is a derivative of the second pharyngeal arch

37. The arterial supply of trachea is by 44. Middle conchae of nose are a part of?
a) Bronchial artery b) Tracheal artery a) Nasal Bone b) Ethmoid
c) Inferior thyroid artery c) Vomer d) Maxilla
d) Superior thyroid artery (PGI-2008)
(AIPG-2K) 45. Nasal epistaxis occurs due to
38. Trachea bifurcates at the level of a) Anastomosis between two arteries in anterosuperior
a) Sternal ang le part of nasal septum
b) 5 th inter costal space b) Anastomosis between two arteries in anteroinferior
c) 3,d thoracic vertebrae part of nasal septum
d) 61h thoracic vertebrae c) Anastomosis between two veins in anteroinferior
part of nasal septum
39. Little's area constitutes d) Anastomosis between vein and artery at
a) Anastamosis between branch of maxillary artery anteroinferior part of nasal septum
and branch of facial artery
b) Anastamosis between two branches of facial artery
c) Anastamosis between two branches of maxillary
artery
d) None of above
(AIPG-98)
40. Which of the following muscle is attached to
posterior part of pterygomandibular raphe ?
a) Lateral pterygoid b) Buccinator
c) Superior constrictor d) Medial pterygoid
(KCET-2011)
41. The maxillary air sinus opens into middle meatus
at:
a) Infundibulum b) Summit of infundibulum
c) Bulla ethmoidalis d) Hiatus semilunaris
(KCET- 07)
42. Correct about rigllt and left main bronchi is
a ) Right bronchus is more vertical and broader than
the left
b) Right bronchus is more vertical and has a narrow
caliber than the left
c) Left bronchus is more vertical and broader than
right
d) Left bronchus is more vertical and narrower than
right

36) A 37) C 38) A 39) A 40) C 41) D 42) A 43) D 44) B 45) B
Dental ;lut.,e

3. MOUTH, PHARYNX, LARYNX - ANSWERS


1. 'C' [BDC H & N 4th ed 219/ 5th ed 217] 8. 'B' [BDC H & N 4th ed 223/ 5th ed 220]
Pyriform fossa is a depression on lateral wall of The weak area is known as "Killian's dehiscence". In this
Laryngopharynx, present on each side of the in let of area, the thyropharyngeus is a single sheet of muscle
Larynx. Internal laryngeal nerve is present beneath and is not overlapped by internally by the upper and
the mucosa of pyriform fossa or the nerve which is middle const rictors.
related to piriform fos sa is internal laryngeal nerve
(KAR-01). 9. 'B' [BOCH & N 4th ed 244/ 5th ed 242]

2. 'D' [BDC H & N 4th ed 185/ 5th ed 242] 10. 'B' [BOCH & N 4th ed 223/ 5th ed 220]
ALL the muscles of Larynx except cricothyroid are All the muscles of pharynx are supplied by accessory
supplied by recurrent Laryngeal nerve. Cricothyroid is nerve through branches of vagus except for
supplied by external laryngeal nerve, which is a branch stylopharyngeus, which is supp lied by glossopharyngeal
of superior laryngea l nerve. nerve.

3. 'C' [BDC H & N 4th ed 245/ 5th ed 243] 11. 'O' [BDC H & N 4th ed 210/ 5th ed 210]
Action Muscles involved
12. 'A' [BDC H & N 4th ed 224/ 5th ed 221]
Muscles which open
glottis or abductors of
• Posterior
cricoarytenoids 13. 'C' [BDC H & N 4th ed 234/ 5th ed 231]
vocal cord Maxillary sinus is pyramidal in shape, and largest of all
• Lateral para nasal sinuses. It opens into middle meat us of the
cricoarytenoids nose in the lower part of hiatus semilunaris. It is the
Muscles which close the
glottis or adductors of • Transverse arytenoids first paranasal sinus to develop.
vocal cord • Cricothyroids
14. 'B' [BOC H & N 4th ed 242/ 5th ed 242]
• Thyroarytenoids.
Muscles which tense the 15. 'A' [BOCH & N 4th ed 234/ 5th ed 231]
vocal cords
• Cricothyroids
16. ' B' [BDC H & N 4th ed 234/ 5th ed 230]
Muscles which relax the • Thyroarytenoids,
Meatus Openings
vocal cord • Vocalis
Muscles which open the Superior meatus Opening of posterior ethmoidal
inlet of larynx
• Thyroepig lotticus (smallest of all) sinus

Muscles which close the • Aryepiglotticus, • Opening of maxillary sinus


inlet of larynx • Oblique a rytenoids Middle meatus
• Opening of fronta l sinus
• Opening of middle ethmoidal
4. 'A' [BOCH & N 4th ed 216, 219/ 5th ed 217] air sinus
Pyriform recess is seen in laryngopharynx. The Inferior meat us
Nasolacrimal duct
pharyngeal recess (lateral recess or fossa of (largest of all)
Rosenmuller) is present in nasopharynx.
Posterior ethmoidal air sinus opens into
5. 'B' [BOCH & N 4th ed 59/ 5th ed 63]
a) Superior meatus
b) Middle meatus
6. 'O' [BOCH & N 4th ed 211/ 5th ed 210, 211] c) Inferior meatus
d) Spheno-ethmoidal recess
7. 'A' [BOC H & N 4th ed 245/ 5th ed 243] (COMED-2012)
r GENERAL ANATOMY
~

V
757

4. TONGUE
1. Protrusion of tongue is brought out by: 9. The papillae present on margins of the tongue:
a) Genioglossus a) Fungiform papillae b) Filiform papillae
b) Intrinsic muscles of tongue c) Vallate papillae d) Foliate papillae
c) Styloglossus d) Palatoglossus (AIPG -98)
(MAN -02) 10. Anterior 2/3'd of tongue arises from:
2. Hypoglossal nerve supplies to all the following a) Hypobranchial eminence
muscles EXCEPT: b) Two lingual swellings
a) Palatog loss us b) Genioglossus c) Tuberculum impar d) B & C
c) Hyoglossus d) Stylog loss us (AIIMS -90)
(MAN -99; AP -06) 11. The action of styloglossus muscle is
3. The lymphatic drainage from the tip of tongue first a) Protrusion of tongue
passes to: b) Posteriorly retracts the tongue
a) Submental nodes b) Supra clavicular nodes c) Depression of tong ue
c) Sub mandibular nodes d) Elevation of tongue
d) Superior deep cervical nodes (AIPG-2010)
(MAR -98) 12. Tongue develops from which branchial arches?
4. When a patient protrudes his tongue, it deviated a) l,II,IV b) I,11,III
to the right. Which of the following nerves is c) l,III,IV d) I,11,IIl,IV
damaged: (AIPG -93)
a) Left hypog lossal b) Glossopharyngeal 13. Taste sensations from the posterior one-third of
c) Right hypoglossal d) Facial nerve the tongue are carried by_cranial nerve:
(MAN -99) a) VII b) V
5. Impulses generated in the taste buds of the tongue c) X d) IX
reach the cerebral cortex via the: (PGI -2K)
a) Thalamus b) Internal capsule 14. The muscles of the tongue are supplied by:
c) Cervical spinal nerve a) Glossopharyngeal nerve
d) Trigeminal nerve b) Lingual nerve
(KAR -97) c) Chorda tympani d) Hypoglossal
6. In which of the following papillae of the tongue are (AIPG -99, AIIMS -94)
the taste buds predominantly located? 15. Sensory nerve fibres to posterior one third of the
a) Circumvallate b) Filiform tongue is supplied by:
c) Foliate d) Fungi form a) XII cranial nerve b) IX cranial nerve
(AIIMS -93) c) X cranial nerve d) VII cranial nerve
7. Circumvallate papillae are present: (AIIMS -92)
a) Behind sulcus terminalis 16. The extrinsic muscles that aid in depressing the
b) Front of sulcus terminalis tongue are the :
c) Anterior 2/3 of tongue a) Genioglossi and hyoglossi
d) Lateral border of tongue b) Genioglossi and palatoglossi
(AIPG -2K) c) Hyoglossi and stylog lossi
8. All are structures lying deep to the hyoglossus d) Styloglossi and palatoglossi
muscle except: (KAR -97)
a) Hypoglossal nerve b) Lingual artery 17. Main arterial supply to the tongue is:
c) Stylohyoid muscle d) Geniohyoid muscle a) Ascending palatine artery
(AIIMS -98) b) Ascending pharyngeal artery

1) A 2) A 3) A 4) C 5) A 6) A 7) B 8) A 9) A 10) D 11) B 12) C 13) D


14) D 15) B 16) A 17) C
Dental ;lut.,e

c) Lingual artery d) Facial artery 26. The mucosa of the posterior third of the tongue is
(AIPG -96) supplied by
18. Which of the following muscle of tongue runs from a) Facial nerve b) Trigeminal nerve
dorsum of tongue to ventral c) Mandibular nerve d) Glossopharyngeal nerve
a) Verticalis b) Superior longitudinal (KCET-2009)
c) Inferior longitudinal 27. Lymph from tongue not drained by following
d) Transverse vessels
a) Central b) Ventral
19. Safety muscle of tongue is c) Posterior d) Marginal
a) Hyog lossus b) Genioglossus (AIIMS NOV-13)
c) Palatoglossus d) Styloglossus 28. Taste receptors
(KAR -13) a) Primary taste sensations are spatially separated on
20. Deep surface of hyoglossus is related to the surface of the tongue
a) Lingual nerve b) Hypoglossal Nerve b) Are a type of chemoreceptor
c) Submandibular Ganglion c) Are innervated by afferent fibres of V, VII & IX
d) Glossopharyngeal nerve d) All of the above
(COMEDK- 06) (GCET-14)
21. The Tongue: 29. The juguloomohyoid lymph node drains
a) Separated from the epiglottis by glossoepiglottic a) The middle ear b) The anterior scalp
folds c) The tongue d) Upper face
b) Contains foramen caecum which is present on the (KERALA-2015)
dorsum of frenulum
c) Contains 6 - 10 circumvallate papilla located
posterior to sulcus terminals
d) Embryologically derives from 1'1 branchial arch only
(PGI- 06)
22. Palsy of the right genioglossus causes-
a) Deviation of tongue to right
b) Deviation of tongue to left
c) Deviation of soft palate to right
d) Deviation of soft palate to left

23. Base of the tongue is attached to the:


a) Hyoid bone b) Mandible
c) Hyoid bone and mandible
d) None of the above
(PGI- 06)
24. Tongue movement has its primary effect on
a) Palatoglossus arch b) Facial musculature
c) Lips d) Cheeks
(AIIMS-07)
25. Structures related to the medial surface of the
hyoglossus muscle include the following EXCEPT
a) Glossopharyngea l nerve
b) Stylohyoid ligament
c) Lingual artery d) Hypoglossal nerve
(COMEDK-08)

18) A 19) B 20) D 21) A 22) A 23) C 24) A 25) D 26) D 27) B 28) B 29) C
r GENERAL ANATOMY
~

V
763

5. VASCULAR SUPPLY OF HEAD AND NECK


]. The inferior dental artery is a branch of the 8. All the following are branches of external carotid
a) Mandibular artery b) Maxillary artery artery except:
c) Pterygomandibular plexus a) Superior thyroid artery
d) None of the above b) Anterior eth moidal artery
(AIPG -94, MAN -94) c) Posterior auricular artery
2. The middle meningeal artery: d) Occipital artery
a) Enters the skull through the foramen ovale (AIPG-01)
b) Supplies the supe rolateral surface of the ipsilateral 9. Which is the only medial branch of the external
cerebral hemisphere carotid artery?
c) Runs a subdural course within the cranial cavity a) Superior thyroid b) Lingual
d) Gives an interior branch which runs deep to the c) Internal maxillary d) Ascending pharyngeal
pterion (AIPG -03)
(MAN -95) 10. Submandibular gland is supplied by:
3. The cavernous sinus does not communicate with a) Lingual artery b) Facial artery
the c) Submandibular artery
a) Opthalmic vein b) Internal jugular vein d) Inferior alveolar artery
c) External jugular vein d) Pterygoid plexus (AIPG -92)
(MAN -98) 11. The common carotid artery may be palpated at :
4. The artery, which runs along, the lower border of a) Hyoid bone
posterior belly of digastric is: b) Upper border of thyroid cartilage
a) Lingual b) Ascending pharyngeal c) Transverse process of CS
c) Occipital d) Palatine d) Transverse process of C6
(MAN -99) (AIPG -07, NEET-2013)
5. All of the following are true of maxillary artery 12. The arterial supply of trachea is by the:
except: a) Bronchial artery b) Tracheal artery
a) The branches of maxillary artery accompany the c) Inferior thyroid artery
branches of maxillary nerve d) Superior thyroid artery
b) It is one of the terminal branches of ECA (AIPG -01)
c) It passes deep between the two heads of lateral 13. First part of vertebral artery is related to:
pterygoid muscle into the pterygo palatine fossa a) Superior cervical ganglion
d) It is the arterial supply for mandible b) Middle cervical ganglion
(MAN -02) c) Stellate ganglion d) Ciliary gang lion
6. Littel's area constitutes: (KAR -03)
a) Anastamosis between branch of maxillary artery 14. Pulsations felt in the suprasternal space are probably
and the branch of facial artery due to:
b) Anastomoses between two branches of facial artery a) Subclavian artery b) Common carotid artery
c) Anastomoses between two branches of maxillary c) Inferior thyroid artery
artery d) Vertebral artery
d) None of the above (AIPG -99)
(AIPG -98) 15. Ascending palatine artery is a branch of:
7. Ophthalmic artery is a branch of: a) Internal carotid artery
a) Maxillary artery b) Sphenopalatine artery b) External carotid artery
c) ICA d) ECA c) Facial artery
(AIPG -98)

1) B 2) D 3) C 4) C 5) A 6) A 7) C 8) B 9) D 10) B 11) B>D 12) C 13) C


14) C 15) C
Dental ;lut.,e

82. Tributary of the cavernous sinus includes all of the a) For recurrent laryngeal nerve
following, except? b) Superior laryngeal nerve
a) Superior petrosai sinus c) Vagus nerve d) Glossopharyngeal nerve
b) Inferior petrosal sinus (AIIMS MAY-14)
c) Superficial middle cerebral vein 91. The retromandibular vein
d) Deep middle cerebral vein a) Is superficial to the facial nerve within the parotid
(AIPG-2009) gland
83. The transverse ven o us sinus continues as: b) Drains the pterygoid venous plexus through the
a) Straight sinus b) Cavernous sinus maxillary vein
c) Sigmoid sinus d) Ethmoidal sinus c) Forms the externa l jugular vein through its anterior
(IGNOU-2010) branch
84. All of the following supply TMJ except d) Drains into the deep facial vein
a) External carotid artery (KERALA-2015)
b) Internal Maxillary artery
c) Transverse facial artery
d) Branches supplying lateral pterygoid
(AP-2011)
85. Right anterior quadrant of the scalp is supplied by
the following arteries EXCEPT
a) Right supra orbital artery
b) Right supra trochlear artery
c) Right superficial temporal artery
d) Right maxillary artery
(COMED-2012)
86. The following branches originate directly from the
subclavian artery, except
a) Vertebral b) Thyro-cervical trunk
c) Internal Thoracic d) Supra scapular
(GCET-14)
87. Common Facial vein is formed by the union of
a) Anterior facial and posterior facial veins
b) Anterior facial and posterior auricular veins
c) Anterior facial and maxillary veins
d) Anterior facial and anterior branch of the posterior
facial vein
(GCET-14)
88. The external carotid artery divides at the Level of
a) Angle of mandible b) Neck of mandible
c) Oblique line of thyroid cartilage
d) Lower border of cricoid cartilage
(COMEDK-14)
89. Rectus abdominis free flap is supplied by?
a) Intercostals artery b) Iliolumbar artery
c) Epigastric artery d) Thoraco-lumbar artery
(PGI J UNE-13)
90. Inferiorthyroid artery Ligation during thyroidectomy
is done

82) D 83) C 84) C 85) D 86) D 87) D 88) B 89) C 90) A 91) B
Dental ;lut.,e

10. 'B' [BDC H & N 4th ed 161/ 5th ed 167)


Arterial & venous Lymphatic N l
Gland . erve supp y
supply drainage
Parasympathetic:
The preganglionic fibres begin in the inferior
salivatory nucleus, pass through 9th nerve, and the
Lymph drains first lesser petrosal nerve and relay in otic ganglion. The
i) ECA into parotid nodes postganglionic fibres reach the parotid gland through
Parotid and from there auriculo temporal nerve.
ii) EJV to upper deep
cervical nodes Sensory:
Sensory nerve supply is through auriculo temporal
nerve. The parotid fascia is supplied by the sensory
fibres of the greater auricular nerve (C2)
Parasympathetic:

i) Facial artery Preganglionic secretomotor fibres begin in superior


Sub Lymph drains into salivatory nucleus, pass through the facial nerve,
mandibular ii) Veins drain into sub mandibular chorda tympani, and the lingual nerve to reach
gland common facial or lymph nodes submandibular ganglion. Postganglionic fibres
lingual vein
emerges from the ganglion and enter sub mandibular
gland.
i) Sublingual and Lymph drains into
Sublingual
sub- mental submental lymph Nerve supply is similar to sub mandibular gland
gland
arteries nodes

11. 'B>D' [BDC H & N 4th ed 281/ 5th ed 134) that "the bifurcation of the common carotid artery
It is one of the controversial question asked frequently. into the internal and external carotid arteries can be
Different books say different answers. easily palpated just beneath the anterior border of the
sternocleidomastoid muscle at the level of superior
Textbook of Anatomy, 5th ed Volume 3 By Inderbir border of the thyroid cartilage"
Singh Pg 865 says that pulsation of common carotid
artery can be felt at the level of the superior border Quain's Elements of Anatomy, 9th ed Volume 1 by
of the thyroid cartilage, beneath the anterior border Allen Thomson, Sir Edward Albert Sharpey-Schafer,
of the sternocleidomastoid muscle. In the same George Dancer Thane
page he says that "the common caotid artery can be &.
compressed against the carotid tubercle (which is the Manual of Vascular Diseases edited by Sanjay
anterior tubercle of tranverse process of sixth cervical Rajagopalan, Debabrata Mukherjee, Emile R. Mohler
vertebrae). says that "At the level of the upper border of the
thyroid cartilage the common carotid artery ends by
Lippincott's Concise Illustrated Anatomy: Head & dividing into external and internal carotid artery. CCA
Neck By Ben Pansky, Thomas R. Gest Volume 3 Pg can be palpated against the carotid or Chassaignac's
5 says that carotid pulse can be easily palpated just tubercle i.e' tubercle of transverse process of sixth
deep to anterior border of SCM muscle at the level of cervical vertebra".
superior border of thyroid cartilage.
12. 'C' [BOCH & N 4th ed 175,195/ 5th ed 189)
Clinical Anatomy by Regions By Richard S. Snell Inferior thyroid artery is a branch of thyrocervical
8th ed Pg 7 48 in subheading clinical notes says trunk, which arises from the subclavian artery.
r GENERAL ANATOMY

Its terminal part is related to recurrent laryngeal nerve. 18. 'B' [BOCH & N 4th ed 128/ 5th ed 136, 137)
Apart from trachea, it also supplies lower 2/3rds of Facial artery is the anterior branch of external carotid
thyroid lobe and lower half of the isthmus, parathyroid, artery.
thymus, larynx, pharynx, and the surrounding muscles.
19. 'B' [BOC H & N 4th ed 210/ 5th ed 210)
13. 'C' [BOCH & N 4th ed 202/ 5th ed 199, 200) Greater palatine artery is a branch of third or
Vertebral artery arises from the first part of subclavian pterygopalatine part of maxillary artery. It supplies
artery and ends in the cranial cavity by supplying the hard palate and gums. Less palatine artery is a
brain. It also supplies the spinal cord, the meninges branch of greater palatine artery and supplies soft
and the surrounding muscles and bones. In its 151 part pa late and tonsil.
it is related to inferior or Stellate ganglion.
20. 'A' [BOC H & N 4th ed 128, 167 / 5th ed 136)
14. 'C' [BOCH & N 6th ed 189) Superior thyroid artery is the first anterior branch
of external carotid artery. It is intimately related to
15. 'C' [BOCH & N 4th ed 129/ 5th ed 136, 137) external laryngeal nerve. The superior thyroid artery
Facial artery is the anterior branch of external carotid supplies upper one third of thyroid lobe and the upper
artery. It runs upwards first in the neck (cervical part) half of isthmus.
and then on face (facial part)
The inferior thyroid artery is a branch of thyro cervical
Branches Of facial artery trunk, which arises from subclavian artery. Its terminal
i) Cervkal part part is related to recurrent laryngeal nerve. The inferior
• Ascending palatine to root of tongue and the thyroid artery mainly supplies lower 2/3'd' of thyroid
tonsil. lobe and lower half of isthmus, trachea, parathyroid
• Tonsillar to tonsil.
and thymus.

• Submental to submental triangle and sublingual


21. 'B' [BOCH & N 4th ed 127, 281/ 5th ed 134)
salivary gland.
The common carotid artery divides into external and
• Glandular to submandibular salivary gland and internal carotid arteries at the level of upper border of
submandibular lymph nodes thyroid cartilage. The common carotid artery becomes
superficial enough to be palpated here (NEET- 2013).
ii) Facial part
• Superior labfol to upper lip 22. 'fl [BOCH & N 4th ed 128/ 5th ed 137)
• Inferior labial to lower lip Maxillary artery is the large terminal branch of external
carotid artery.
• Lateral nasal to dorsum of nose

23. 'B' [BOCH & N 4th ed 128/ 5th ed 136)


Option 'D' ascending pharyngeal artery is a medial
branch of external carotid artery.
24. 'fl [BOCH & N 4th ed 95/ 5th ed 10 2)

16. 'O' [BOCH & N 4th ed 161/ 5th ed 167) Draining Channels Of Cavernous Sinus
Into Through
17. 'O' [BOCH & N 4th ed 21 6-17/ 5th ed 215)
Transverse sinus Superior pertrosal sinus
Important aggregations of Waldeyer's lymphatic
ring are: Inferior petrosal sinus
Internal jugular
• Right and left palatine tonsils and venous plexus around
vein
• Pharyngeal tonsil internal carotid artery
• Tubal tonsil Facial vein Superior ophthalmic vein
• Lingual tonsil
Dental ;lut.,e

31. 'A' [BDC H & N 4th ed 103, 177/ 5th ed 181)


Through emissary veins
passing through the foramen
32. 'C' [BDC H & N 5th ed 191)
Pterygoid plexus ovate, foramen lacerum and
the emissary sphenoidal • The group of lymph nodes which drain lymph from
fora men the teeth are submandibular nodes. Submental
lymph nodes drain lymph from tong ue and flour
Communication Inter cavernous sinuses and
of mouth which could also be considered as
between the basilar plexus of veins
appropriate option.
left and right
cavernous sinuses • The entire lymph from head and neck drains
ultimately into the deep cervical nodes. So deep
cervical nodes is also correct option.
25. 'C' [BDC H & N 4th ed 147-48/ 5th ed 152)
Infratemporal fossa is supplied mainly by the • Retropharyngeal nodes, drain the pharynx, auditory
accessory meningeal artery, which is a branch of first tube, nose and palate. So these nodes does not
(mandibular) part of maxillary artery. drain teeth.

33. ' B' [BDC H & N 4th ed 128/ 5th ed 136, 137)
26. 'D' [BDC H & N 4th ed 128/ 5th ed 136, 137)

27. 'B' [BDC H & N 4th ed 130/ 5th ed 134 Fig 8.12) 34. 'C' [BDC H & N 4th ed 256/ 5th ed 136, 137)

Ascending pharyngeal artery arises very close to the Superficial temporal and posterior auricular arteries
lower end of the external carotid artery. supply the outer part of external auditory meatus while
the inner part is supplied by deep auricular branch of
28. ' D' [BDC H & N 4th ed 102/ 5th ed 152) the maxillary artery
Middle meningeal artery is a branch of the first part
of maxillary artery. The fronta l (anterior) branch of 35. 'C' [BDC H & N 4th ed 49, 59 Fig 2.22/ 5th ed 62 Fig
middle meningeal artery is the commonest source of 2.21)

extradural hemorrhage.
36. ' B' [BDC H & N 4th ed 59/ 5t h ed 63 Fig 2. 23)

29. 'C' [BDC H & N 4th ed 386/ 5th ed 444, 146 Fig 31) The facial vein communicates with t he cavernous
Circulus arteriosus or "circle of Willis" is a anatomical sinus through superior ophthalmic vein. Infections
peculiarity of cerebral arteries. from upper lip and lower part of nose can spread
in a retrograde direction and cause thrombosis of
It is formed by cavernous sinus. So this area is known as " dangerous
area of face".
• Anteriorly by anterior communicating artery.
• Posteriorly by basilar artery. Cavernous sinus thrombosi s happen s when
• On each side by anterior cerebral, posterior cerebral, infection reaches at (AIIMS Nov-13)
posterior comm1Unicating and interna l carotid a) Pterygoid plexus of vein
arteries. b) Ethmoidal vein
c) Maxillary sinus
Option 'C' posterior inferior cerebellar artery is the d) Submental lymph node
largest branch of the vertebral artery. It supplies
cerebellum, lower part of pons and posterolateral 37. ' B' [BDC H & N 4th ed 14 7/ 5th ed 151)
aspect of the medulla.
38. ' B' [BDC H & N 4th ed 129/ 5th ed 136)
30. 'C' [BDC H & N 4th ed 129/ 5th ed 216) Facial artery arises from the external carotid just above
Tonsil is supplied by ascending palatine and tonsillar the tip of the greater cornua of the hyoid bone.
branches. Both of them arise from the cervical part of
facial artery. 39. ' B' [BDC H & N 4th ed 129/ 5th ed 137)
r GENERAL ANATOMY
~

V
773

40. 'A' [BOCH & N 4th ed 110/ 5th ed 136, 109) Paired venous sinuses
Opthalmic artery is a branch of cerebral part of internal • Cavernous sinus
carotid artery.
• Superior petrosal sinus
• Inferior petrosal sinus
Which of the following is not a branch of
cavernous part of internal carotid artery ? • Transverse sinus
a) Cavernous branch b) Inferior hypophyseal • Sigmoid sinus
c) Meningeal artery d) Opthalmic artery • Sphenoparietal sinus
(COMEDK-14) • Petrosquamous sinus
• Middle meningeal sinus
41. 'C' [BOCH & N 4th ed 179/ 5th ed 183)

Unpaired venous sinuses


42. 'A' [BOCH & N 4th ed 49 - See Fig. 2 .22/ 5th ed 62
Fig 2.21] • Superior sagittal sinus
The supratrochlear and supraorbital veins unite at • Inferior sagittal sinus
medial angle of the eye to form angular vein, which • Straight sinus
continues as facia{ vein. The superficial temporal vein • Occipital sin us
joins the maxillary vein to form retromandibular vein.
• Anterior intercavernous siinus
The anterior division of retromandibular vein joins
with facia l vein to fo rm common facial vein, which • Posterior intercavernous sinus
drains into internal jugular vein. • Basilar plexus of veins

The posterior division of retromandibular vein joins 49. 'A' [BOCH & N 4th ed 99/ 5th ed 106]
with posterior auricular vein to form external jugular
vein, which fina lly drains into subclavian vein. All 50. 'O' [BOC H & N 4th ed 127/ 5th ed 134)
these veins form a "w-shaped" arrangement on each Option 'D' "carotid sinus" is present at the termination
side of face. of CCA or beginning of ICA. It acts as a baroreceptor
and regulates blood pressure.
43. 'C' [BOCH & N 4th ed 56/ 5th ed 61)
Facial artery is the chief artery of face. It is the " Carotid body" is situated behind the bifurcation of
anterior branch of external carotid artery given off common carotid artery. It acts as a chemoreceptor
in the carotid triangle just above the level of greater and responds to changes in 0 2 and CO 2 content of
cornua of hyoid bone. blood.

44. 'A' [BOCH & N 4th ed 58/ 5th ed 61) 51. 'B' [BOCH & N 4th ed 128/ 5th ed 88)
Superficial temporal artery supplies parotid gland,
masseter and the overlying skin. 52. 'O' [BOCH & N 4th ed 151/ 5th ed 156)
Both 'A: and 'B' are terminal branches of external
45. 'A' [BDC H & N 4th ed 59/ 5th ed 61] carotid artery.

46. 'O' [BOCH & N 4th ed 68/ 5th ed 72) 53. 'E' [BOCH & N 4th ed 172-73/ 5th ed 177-179)

47. 'A' [BOCH & N 4th ed 83/ 5th ed 179) 54. 'O' [BOC H & N 4th ed 179-80/ 5th ed 182, 183)
Vertebral artery is the first and largest branch of
subclavian artery. It is one of the two principal arteries 55. 'O' [BOCH & N 4th ed 212-13/ 5th ed 212)
supplying brain. It also supplies the spinal cord, the
meninges and the surrounding muscles and bones. 56. 'K [BOCH & N 4th ed 224/ 5th ed 221)
• Stage I - Voluntary
48. 'O' [BOCH & N 4th ed 93/ 5th ed 101)
• Stage II & Stage III - Involuntary
Dental ;lut.,e

57. 'D' [BDC H & N 4th ed 225/ 5th ed 222] 75. ' B' [BDC H & N 4th ed 218 / 5th ed 216 for 'A'
4th ed 56, 57 / 5th ed 61 for 'B & C'
58. 'A' [BDC H & N 4th ed 149/ 5th ed 152] 4th ed 163/ 5th ed 167 for 'D'
It is a branch of external carot id artery.
59. 'E' [BDC H & N 4th ed 180/ 5th ed 183]
76. ' D' [BDC H & N 4th ed 48/ 5th ed 60]
60. 'D' [BDC H & N 4th ed 128/ 5th ed 136, 137] The subgaleal aponeurotic has loose areolar tissue and
is known as the dangerous area of the scalp because
61. 'A' [BDC H & N 4th ed 178/ 5th ed 135] the emissary veins which open here may transmit
infection from scalp to t he cranial venous sinuses so
62. 'D' [BDC H & N 4th ed 202/ 5th ed 41, 179] subgaleal hematoma should be treated conservatively
because other procedures can lead to contamination.
63. 'D' [BDC H & N 4th ed 95/ 5th ed 102]
77. 'C' [BDC H & N 4th ed 103/ 5th ed 109]
64. 'C' [BDC H & N 4th ed 170/ 5th ed 175]
Dangerous area/zone of scalp is
a) Loose areolar tissue b) Aponeurotic layer
65. 'C' [BDC H & N 4th ed 110/ 5th ed 109]
c) Connective tissue d) Pericranium
(AIIMS MAY-13)
66. 'B' [BDC H & N 4th ed 57 / 5th ed 137]

78. 'D' [BDC H & N 4th ed 180/ 5th ed 183]


67. 'C' [BDC H & N 4th ed 203/ 5th ed 201]
Brachiocephalic vein 1s formed by behind the
sternoclavicular joint by the union of the internal
68. 'B' [BDC H & N 4th ed 148/ 5th ed 153]
jugular vein and the subclavicular vein. The two
Refer synopsis.
brachiocephalic veins of left and right sides unite at
the lower border of the right fi rst costal cartilage to
69. 'B' [BDC H & N 4th ed 59/ 5th ed 63]
form the superior vena cava.

70. 'B' [BDC H & N 4th ed 101/ 5th ed 152]


The tributaries of right brachiocephalic vein are
The middle meningeal artery is the direct branch of the
• Vertebral
first part of Maxillary artery, which again is a terminal
• Internal thoracic
branch of external carotid artery.
• Inferior thyroid
• First posterior intercostal
71. 'C' [BDC H & N 4th ed 103/ 5th ed 109]
Refer Q. No 24
The tributaries of left brachiocephalic vein are

72. 'D' [BDC H & N 4th ed 68/ 5th ed 72]


• Vertebral
The carotid sheath is a condensation of fibroareolar • Internal thoracic
tissue around the main vessels of neck. It encloses • Inferior thyroid
common and internal carotid arteries, internal j ugular • First posterior intercostal
vein and the vag us nerve. • Left superior i ntercostal
• Thymic and pericardia[ veins
73. 'D' [BDC H & N 4th ed 94, 95/ 5th ed 102]
79. 'A' [BDC H & N 5th ed 108]
74. 'D' [BDC H & N 4th ed 95/ 5th ed 102]
Cavernous sinus drains into: 80. 'A' [BDC H & N 4th ed 180]
• Transverse sinus through the superior petrosal sinus
• Internal jug ular vein through the inferior petrosal 81. 'C' [BDC H & N 5th ed 137]
sinus Posterior auricular artery supplies the back of auricle,
• Pterygoid plexus of veins t hrough emissary veins skin over mastoid process and over the back of the
• Facial vein through superior ophthalmic vein. scalp.
r GENERAL ANATOMY

a) Masseter b) Buccinator 24. Sphenoidal air sinus is supplied by _ _ nerve:


c) Medial pterygoid d) Anterior belly of digastric a) Posterior eth moidal nerve
(AIPG -99;PGI -03) b) Posterior superior alveolar nerve
17. TMJ is supplied mainly by: c) Sphenoidal nerve d) Infratemporal nerve
a) Masseteric and auriculotemporal nerve (AIPG -99)
b) Superficial temporal nerve 25. Which of the following is not a branch of ophthalmic
c) Facial nerve peripheral branch nerve?
d) Posterior auricular nerve a) Frontal b) Lacrimal
(AIIMS -90) c) Nasociliary d) Medial ethmoid
18. Which of the following nerves are branches of (AIPG -98)
mandibular nerve? 26. Skin over the prominence of the cheek is supplied
a) Lingual, facial and palatine nerve by:
b) Lingual and inferior alveolar nerves a) Zygomaticofacial b) Zygomaticotemporal
c) Inferior alveolar and cervical nerves c) Auriculotemporal d) Infra trochlear
d) All of the above (AIPG -98)
(AIPG -92) 27. Which of the following is not true regarding
19. The Otic Ganglion: trigeminal nerve?
a) Is in relation to maxillary nerve a) Roots of trigeminal nerve are attached to Pons
b) Receives prega nglionic parasympathetic fibers, b) It has three main divisions
t hrough the greater superficial petrosal nerve c) Trigeminal nerve supplies the dura mater of middle
c) Sends postganglionic parasympathetic fibers to cranial fossa
auriculotemporal nerve d) The trigeminal ganglion contributes to the sensory
d) Is related to the tensor tympani root only
(KAR -97) (AIIMS -91)
20. The maxillary nerve exits the cranium through 28. Which of the following muscles is supplied by
which foramen: mandibular nerve?
a) Ovale b) Rotundum a) Stapedius b) Buccinator
c) Stylomastoid d) Inferior orbital fissure c) Tensor veli palatine d) Posterior belly of digastric
(AIPG -02)
21. Mandibular nerve supplies: 29. Inferior alveolar nerve runs:
a) Anterior belly of digastric, mylohyoid, tensor a) Media l to lateral pterygoid
tympani, tensor palati b) From Infratemporal fossa
b) Posterior belly of digastric, mylohyoid c) Lateral to medial pterygoid
c) Stapes, malleus, incus d) Lateral to lateral pterygoid
d) Anterior belly of digastric, stapes (AIIMS -96)
(AIPG -2K) 30. Structures passing through the sigmoid
22. The nasal septum is supplied by all the following (mandibular) notch are:
except: a) Auriculotemporal nerve
a) Nasopalatine nerve b) Posterior ethmoidal nerve b) Mandibular nerve
c) Pterygopalatine ganglion c) Masseteric nerve and vessels
d) Nasociliary nerve d) Chorda tympani
(AIPG -99) (PGI -99)
23. All are branches of the posterior division of the 31. Mandibular anterior division nerve has:
mandibular nerve except: a) One sensory and all motor branches
a) Lingual nerve b) Inferior alveolar nerve b) One motor and all sensory branches
c) Deep tempora l nerve d) Mylohyoid nerve c) All motor branches d) All sensory branches
(AIPG -99) (AIIMS -2K)

17) A 18) B 19) C 20) B 21) A 22) B 23) C 24) A 25) D 26) A 27) D 28) C 29) C
30) C 31) A
r GENERAL ANATOMY
~

V
779

c) Lingual nerve d) Mylohyoid nerve 58. Branches of facial nerve are:


a) Digastric b) Stylohyoid
50. Anterior belly of digastric is supplied by: c) Posterior auricular d) All of the above
a) Mylohyoid nerve b) Branch of facial nerve (AIPG -95)
c) Branch of cervical plexus 59. All of the following nerves are related to the
d) Hypoglossal nerve mandible except:
(APPSC -99) a) Lingual nerve b) Auriculotemporal nerve
51. All of the following is true of facial nerve except: c) Chorda tympani d) Mental nerve
a) Supplies motor branches to the muscles of facial (AIPG -99)
expression 60. In right facial nerve damage:
b) It has no sensory component a) There is paralysis of facial muscles
c) The muscles of the eyelid will be spared in upper b) There is paralysis of muscles of mastication
motor neuron lesions c) Paralysis of vocal cords
d) It is a nerve of 2nd arch d) A and B
(MAN -2K) (AIPG -2K)
52. All of the following muscles are supplied by the 61. The muscle arising from the outer surface of the
facial nerve except: alveolus at the region of the molars is supplied by
a) Stapedius b) Occipitofrontalis which nerve:
c) Anterior belly of digastric a) Trigeminal b) Facial
d) Bucci nator c) Mandibular d) Inferior alveolar
(AIPG, 99) (PGI -2K)
53. The special visceral afferent fibres of the facial 62. Smiling and frowning are actions produced by the
nerve are located in which nuclei: following nerves:
a) Motor nucleus b) Nucleus ambiguous a) Mastication; trigeminal (V cranial)
c) Nucleus of tractus solitarius b) Mastication; facial (VII cranial)
d) Lacrimatory nucleus c) Facial expression; trigemi nal (V cranial)
(AIIMS -99) d) Facial expression; facial (VII cranial)
54. The nerve supply for motor action of buccinator: (KAR -97)
a) Facial nerve b) Mandibular nerve 63. Muscle spared during injury to cranial root of
c) Maxillary nerve d) Auriculo temporal nerve accessory nerve ?
(KAR -03, AIIMS MAY- 2012) a) Stylopharyngeus b) Palatopharyngeus
55. Secretomotor fibers to the Submandibular salivary c) Salpingopharyngeus d) Cricopharyngeus
gland are carried in all the following except:
a) Facial b) Chorda tympani 64. The nerve supply of stapedius muscle is:
c) Mylohyoid d) Lingual a) III cranial nerve b) V cranial nerve
(PGI -2K) c) VII cranial nerve d) XI cranial nerve
56. Chorda tympani contain: (AIPG -01)
a) Preganglionic sympathetic fibres 65. Recurrent laryngeal nerve has close relation to
b) Preganglionic parasympathetic fibres a) Superior thyroid artery
c) Postganglionic sympathetic fibres b) Middle thyroid vein
d) Postganglionic parasympathetic fibres c) Inferior thyroid artery
(AIPG -93) d) Inferior parathyroid
5 7. Facial nerve has: (KCET-2012)
a) Motor root on ly b) Motor and sensory root 66. Facial nerve:
c) Motor and sympathetic a) Mixed nerve b) Sensory nerve
d) Motor, sympathetic and parasympathetic fibres c) Motor nerve d) Parasympathetic nerve
(AIPG -98) (AP -2K)

50) A 51) B 52) C 53) C 54) A 55) C 56) B 57) B 58) D 59) C 60) A 61) B 62) D
63) A 64) C 65) C 66) A
r GENERAL ANATOMY

b) The posterior belly of digastric c) Sensory innervation to the posterior third of the
c) Mylohyoid tongue
d) The tensor veli pa latine (tensor palatini) d) Sensory innervation to the anterior t wo-thirds of
(AIIMS -02) the tongue
85. Damage to internal Laryngeal nerve results in (AIPG -03)
a) Hoarseness b) Loss of timbre of voice 94. The hypoglossal nerve is the motor nerve to all of
c) Anesthesia of the larynx the muscles of the tongue except, the:
d) Breathing difficulty a) Genioglossus b) Palatoglossus
c) Superior longitudinal muscle
86. Cricothyroid is supplied by d) Inferior longitudinal musde
a) Recurrent laryngeal nerve (AIPG -94)
b) Internal laryngeal nerve 95. In a patient it is noticed that when he protrudes his
c) External laryngea l nerve tongue it deviates to the left. The nerve damaged is:
d) Hypoglossal nerve a) Left hypoglossal b) Left glossopharyngeal
c) Right hypoglossal d) Right glossopharyngeal
87. Nerve supply of mucosa of Larynx is (PG! -2K)
a) External laryngeal b) Internal laryngeal 96. All of the following are features of an injury to
c) Recurrent laryngeal d) B & C the cervical sympathetic trunk EXCEPT:
a) An hydrosis b) Enophthalmos
88. Structure which course through abdomen is c) Mydriasis d) Pt osis
a) Thoracic duct b) Vag us nerve (MAN -95)
c) Aorta d) Facial nerve 97. The optic nerve terminates in the
a) Thalamus b) Pituitary gland
89. All cranial nerves are confined to the head and neck c) Medulla Oblongata d) Lateral Geniculate Body
with exception of (BHU-2012)
a) One b) Two 98. The Largest ganglion in the neck is
c) More than two d) None of the above a) Superior ganglion b) Middle ganglion
(BHU-2012) c) Stellate ganglion d) Second thoracic ganglion
90. Crania Laccessory nerve supplies
a) Sternomastoid b) Trapezius 99. Root value of phrenic nerve
c) Splenius capitis d) Soft palate a) C2 C3 c, b) C1 c2c3
c) C3 c, C5 d) c, C5 C6
91. The muscle of tongue not supplied by the
hypoglossal nerve is: 100. Cetiary ganglion is located
a) Hyoglossus b) Styloglossus a) Between optic nerve and lateral rectus
c) Genioglossus d) Palatoglossus b) Apex of orbit
(MAN -98) c) Apex of orbit and superior rectus
92. Hypoglossal nerve is: d) Apex of orbit between optic nerve and lateral
a) Purely motor b) Purely sensory rectus
c) Mixed nerve d) A peripheral nerve
(AIIMS -90, 91, AIPG -90) 101. Angle of jaw (or) skin over angle of mandible is
9 3. The hypoglossal nerve provides: supplied by
a) Motor innervation to styloglossus and hyoglossus a) Great auricular nerve
muscles on ly b) Lesser occipital nerve
b) Motor innervation to all of the muscles of the c) Greater occipital nerve
tongue both intrinsic and extrinsic d) Posterior auricular nerve
(PGI -05)

85) C 86) C 87) D 88) B 89) A 90) D 91) D 92) A 93) B 94) B 95) A 96) C 97) D
98) A 99) C 100) D 101) A
Dental ;lut.,e

102. The tip of nose is supplied by _ _ _ _ nerve a) Facial Nerve b) Mandibular Nerve
a) Maxillary b) Mandibular c) Glosso pharyngeal Nerve
c) Opthalmic d) Facial d) Maxillary nerve
(COMEDK-06)
103. All of the following are supplied by the ansa 111. When a patient is asked to say "ah", if the uvula is
cervicalis except drawn upwards to the left, the cranial nerve likely
a) Sternohyoid b) Omohyoid to be damaged is
c) Sternothyroid d) Thyrohyoid a) Vagus b) Rt accessory
c) Lt accessory d) Hypoglossal
104. The nerve that emerges from the two superficial (KAR-04)
heads of lateral pterygoid muscle is 112. The following statements concerning chorda
a) Buccal netve b) Massetric nerve tympani nerve are true except that it:
c) Nerve to pterygoid muscle a) Carries secretomotor fibers to submandibular gland
d) Inferior alveolar nerve b) Joints lingual nerve in infratemporal fossa
(PGI-06) c) Is a branch of facial nerve
105. Sensory innervation of larynx as far as the vocal d) Contains postganglionic parasympatheticfibers
folds is by nerve (AIPG-05)
a) External laryngeal b) Recurrent laryngeal 113. Which of the following does not pass through
c) Internal laryngeal d) Glossopharyngeal cavernous sinus?
(COMEDK-06) a) Occulomotor (III) nerve
106. Which of the following is the normal arrangement b) Trochlear nerve
of lingual nerve and vessels in the tongue from c) Medial cerebral artery
medial to lateral d) Trigeminal nerve
a) Nerve, Artery and vein (AIPG-07)
b) Artery, Nerve and vein 114. Which one of the following is a branch of facial
c) Vein, Artery and Nerve nerve
d) Nerve, Artery a) Deep petrosal nerve
(AIPG-05) b) Lesser superficial petrosal nerve
107. Palatine aponeurosis is: c) External petrosal nerve
a) Tendon of levator veli palatine muscle d) Greater superficial petrosal nerve
b) Tendon of Tensor veli palatine muscle
c) A part of musculus uvulae 115. Lacrimation does not occur when facial nerve injury
d) A modification of palatal periosteum is at-
(AIPG-05) a) Geniculate ganglion b) In semicircular canal
108. Motor supply of infrahyoid muscle is: c) At sphenopalatine ganglia
a) Branches of cervical plexus d) At mastoid foramen
b) Vagus nerve
c) Glossopharyngea l nerve 116. Where is cillary ganglion located in the orbit
d) Mandibular nerve a) Between optic nerve and latera l rectus
(AIIMS-06) b) Apex of orbit
109. The cranial nerves which are part of parasympathetic c) Apex of orbit & superior rectus
card via: d) Apex of orbit between optic nerve & lateral rectus
a) III, IV, V, VII b) III, VII, VIII, IX
c) III, VII, IX, X d) VII, IX, X, XI 117. Unilateral supranuclear lesion of facial nerve
involves
110. Pterygopalatine ganglion is functionally connected a) Whole of face
to b) Only ipsilateral upper part of the face

102) C 103) D 104) A 105) C 106) B 107) B 108) A 109) C 110) A 111) B 112) D 113) C 114) D
115) A 116) D 117) C
Dental ;lut.,e

135. Parasympathetic ganglion in head are c) It passes through the pterygomandibular space
a) Three pairs b) Four pairs d) The lingual nerve is the sensory tract to anterior
c) Five pairs d) Two pairs one third of the tongue
(BHU-2012) (MCE-14)
136. Pharyngotympanic tube is supplied by all except: 143. Most common cranial nerve palsy in basilar skull
a) Ascending pharyngeal fracture
b) Ascending palatine a) Optic nerve b) Olfactory nerve
c) Middle meningeal artery c) Facial nerve d) Auditory nerve
d) Artery of pterygoid canal (GCET-14)
(AIIMS MAY 2012) 144. Auriculotemporal nerve encircles which artery
137. If the seventh nerve is damaged on the right side of a) Superficial temporal artery
the face b) Superficia l temporal vein
a) the muscles of mastication would be denervated c) Middle meningeal artery
b) the muscles of facial expression would be d) Deep temporal
denervated (AIIMS NOV-14)
c) taste from the right anterior two thirds of the 145. Which of the following is not present in posterior
tongue would not be affected triangle of neck?
d) sympathetic fibre to salivary gland would be a) Phrenic nerve b) Subclavian nerve
interrupted c) External j ugular vein d) Hypoglossal nerve
(AP-2013) (AIIMS NOV-14)
138. Sensory supply of nasal cavity is from a branch of: 146. Phrenic nerve chiefly supplies?
a) Trigeminal nerve b) Facial nerve a) Stomach b) Diaphragm
c) Occulomotor nerve d) Glossopharyngeal nerve c) Oesophagus d) Ileum
(NEET-2013) (AIIMS MAY-14)
139. Damage to occulomotor nerve causes all except: 14 7. A patient comes with the H/0 swelling in the
a) Ptosis b) Miosis submandibular region. He is diagnosed to have
c) Outward eye deviation submandibular abscess. The incision to drain the
d) Diplopia abscess is put 1 cm below the base of the mandible
(NEET-2013) because
140. Pre-maxillary hard palate is supplied by which a) The submandibular gland appears 1 cm below the
nerve mandible
a) Nasopalatine nerve b) It is difficult to incise along the margin of the
b) Pharyngeal branch of glossopharyngeal nerve mandible
c) Greater palatine nerve c) The marginal mandibular nerve may be injured if
d) Tensor palatine nerve incision is along the base of mandible
(AIIMS NOV-13) d) To avoid injury to the cervical branch of facia l
141. The cell-bodies of sympathetic preganglionic nerve.
neurons are located in the (COMEDK-15)
a) Dorsal root ganglia 148. Submand;bular ganglfon ;s anatomkally related to?
b) Sympathetic chain ganglia a) Chorda tympani nerve
c) Brain stem and spinal cord b) Lingual nerve
d) Thoracic and Lumbar segments of the spinal cord c) Facial nerve d) Hypoglossal nerve
(AP-14)
142. Which of the following statements is correct about
lingual nerve ?
a) It passes th roug In i nfratem para l space
b) It passes through para-pharyngeal space

135) B 136) B 137) B 138) A 139) B 140) A 141) D 142) C 143) C 144) C 145) D 146) B 147) C
148) B
r GENERAL ANATOMY
~

V
785

6. NERVE SUPPLY OF HEAD AND NECK - ANSWERS


1. 'B' [BDC H & N 4th ed 107/ 5th ed 116, 117] 9. 'D' [BDC H & N 4th ed 57 / 5th ed 60 Tab 2. 4]
Levator Palpebrae Superioris elevates the upper eyelid Trigeminal nerve is one of the largest cranial nerve and
and is supplied by III cranial or occulomotor nerve. contains both sensory and motor fibres.

BRANCHES OF TRIGEMINAL NERVE


2. 'B' [BDC H & N 4th ed 113/ 5th ed 370]
Paralysi s of 3m nerve will result in: Sensory rn nature and
a) Opthalmic nerve
• Ptosis (drooping of upper eyelid) smallest of the three division
• Lateral squint b) Maxillary nerve Sensory
• Dilatation of pupil Mixed in nature and largest
c) Mandibular nerve
• Loss of accommodation of the three divisions.
• Slight proptosis (forward projection of eye)
Mental nerve is a branch of inferior alveolar nerve and
• Diplopia (double vision)
it supplies the skin of chin and lower lip.

3. 'A' [BDC H & N 4th ed 113/ 5th ed 370]


10. ' D' [BDC H & N 4th ed 139/ 5th ed 355]
Opthalmic nerve enters the orbit through the superior
4. 'D' [BDC H & N 4th ed 108-09/ 5th ed 116]
orbital fissure (Option 'C'). Maxillary nerve enters
All the extraocular muscles except superior oblique
and leaves the skull through Foramen rotundum
and lateral rectus are supplied by occulomotor nerve.
(Option 'B').
Superior oblique is supplied by trochlear nerve (SOJ;
the lateral rectus is supplied by 6th or abducent nerve
Mandibular nerve leaves the skull through foramen
(LR 6 ).
ovale (Option '/\). Option 'D' stylomastoid foramen
transmits the facial nerve and the stylomastoid branch
5. 'B' [BDC 5th ed 53, 142 Fig.9 .3/ 5th ed 358 Fig
of the posterior auricular artery.
11.9]
One of the terminal branches of facial (7th) nerve is
11. 'K [BDC H & N 4th ed 33 Fig 1.27/ 5th ed 34)
the marginal mandibular nerve. This branch runs below
Option 'B' inferior alveolar nerve is closely related to
the angle of mandible deep to platysma. It crosses the
posterior end of mylohyoid groove. Option 'C' mylohyoid
body of mandible and supplies the muscles of lower lip
nerve and vessels lie in the mylohyoid groove. All the
and chin.
three nerves are branches of mandibular nerve and are
related to medial surface of ramus of mandible.
6. 'C' [BDC H & N 4th ed 94/ 5th ed 101, 102]
St ructures in lateral wall of sinus are:
12. ' D' [BDC H & N 4th ed 151, see 236-37 for options A,
• Occulomotor B & C/ 5th ed 156 see 233-234 for options A, B & C)
• Trochlear, TMJ is supplied by auriculotemporal and masseteric
• Trigeminal ganglion and its three branches nerves, which are branches of mandibular nerve.
BRANCHES OF MANDIBULAR NERVE.
Abducent nerve passes through the centre of sinus. i) Branches from main trunk
• meningeal branch
7. 'A' [BDC H & N 4th ed 94 Fig 6. 6/ 5th ed 101, 102]
• nerve to medial pterygoid
8. ' B' [BDC H & N 4th ed 108 -10/ 5th ed 371]
Superior oblique, which is supplied by trochlear nerve ii) Branches from anterior trunk
is responsible for downward and lateral rotation of the • Buccal nerve (only sensory branch of anterior
eyeball. So, in cases oftrochlear nerve injury, the person division of mandibular nerve and it supplies
will have inability to look downwards and laterally. skin and mucous membrane over bucci nator)
Dental ;lut.,e

31. 'A' [BDC H & N 4th ed 152/ 5th ed 157) 42. ' B' [BDC H & N 4th ed 155/ 5th ed 159)
Lower teeth are supplied by inferior alveolar nerve.
32. ' B' [BDC H & N 4th ed 155/ 5th ed 353, 354)
4 3. ' B' [BOCH & N 4th ed 155/ 5th ed 157)
33. 'C' [BDC H & N 4th ed 140/ 5th ed 34)
Nerves in relation to mandible are 44 . ' B' [BDC H & N 4th ed 263/ 5th ed 211, 356)
• Mental nerve Tensor tympani and the stapedius, both are muscles
of ear ossicles. Both act simultaneously to damp
• Lingual nerve
down the intensity of high pitched sound waves and
• Auriculotemporal nerve thus protect the internal ear. The "tensor tympani" is
• Mylohyoid nerve
• Masseteric nerve supplied by the mandibular nerve while the "stapedius"
is supplied by the facia l nerve.
Chorda tympani is a branch of facial nerve. It carries
preganglionic secretomotor fibres to submandibular 4 5. ' D' [BDC H & N 4th ed 28/ 5th ed 28)
ganglion for supply of the submandibular and Structures passing through inferior orbital fissure
sublingual salivary glands; and gustatory to anterior are:
2;3m, of the tongue. • Maxillary nerve
• Zygomatic nerve
Option 'K lingual nerve is sensory to anterior 2/3'd•
• Orbital branches of pterygo-palatine gang lion
of the tongue and to the floor of mouth. However,
the fibres of the chorda tympani are also distributed • Inferior ophthalmic vein
through the lingual nerve. • Infra orbital vessels

34. 'C' [BDC H & N 4th ed 153/ 5th ed 18) Zygomatic nerve is a branch of maxillary nerve and it
supplies orbicularis oculi
3 5. ' B' (BDC H & N 4th ed 137/ 5th ed 145)
46. ' D' [BDC H & N 4th ed 236/ 5th ed 233, 234)
36. ' B' [BDC H & N 4th ed 145/ 5th ed 149) Pterygopalatine or sphenopalatine ganglion is the
Temporalis is supplied by deep temporal branches of largest parasympathetic ganglion. Topographically,
mandibular nerve. it is related to maxillary nerve but functionally it is
connected to facial nerve
37. 'A' [BDC H & N 4th ed 151/ 5th ed 156)
TMJ is supplied by au riculo temporal & masseteric nerves. Pterygopalatine gang lion supplies parasympathetic
fibres to lacrimal gland and the glands of nose,
38. 'A' [BDC H & N 4th ed 236/ 5th ed 65, 233) nasopharynx, palate and the paranasal sinuses.
Pterygopalatine ganglion or sphenopalatine ganglion
is the LARGEST parasympathetic peripheral ganglion . Option 'B' sublingual gland is supplied by sub mandibular
It contains secretorniotor fibres to the lacrimal gland ganglion.
and to mucous glands of nose, paranasal sinuses, the
palate and the nasopharynx. Option 'C' ciliary ganglion is parasympathetic, and
supplies the sphincter pupillae and the ciliaris muscle.
39. 'B' (BDC H & N 4th ed 155, 161 & fig 11. 2/ 5th ed
159) 4 7. 'A' (BOCH & N 4th ed 155/ 5th ed 259, 260)

40. 'A' (BDC H & N 4th ed 155/ 5th ed 164) 48. ' B' [BDC H & N 4th ed 335/ 5th ed 351)
Nuclei associated with the trigeminal nerve are:
41. ' D' [BDC H & N 4th ed 155/ 5th ed 159) • Mesencephalic
This is due to damage of lingual branch of mandibular • Principal sensory,
nerve.
Dental ;lut.,e

145. 'D' [BDC H& N 6th ed 87]

CONTENTS OF THE POSTERIOR TRIANGLE

• Spinal accessory • Three trunks of branchial plexus


• Four cutaneous branches of cervical (phrenic • Nerve to serratus anterior
nerve) • Nerve to subclavius
Nerves
• Muscular branches to trapezius, levator scapulae, • Suprascapular nerve (C 5 ,C6 )
rhomboids
• C5, C6 roots of the branchial plexus
• Transverse Cervical artery and vein • Third part of subclavian artery and
• Occipital artery subclavian vein
Vessels
• Supra scapular artery and vein
• Lower part of external jugular vein
• Sup raclavicular nodes Supra clavicular chain
Lymph nodes
• Occipital nodes

146. 'B' [BDC H&N 6th ed 169]


Phrenic nerve is mixed nerve carrying motor fibres to
the diaphragm and sensory fibres to the diaphragm,
pleura, pericardium and part of the peritoneum .
The nerve chiefly arises from the C4 but receives
contributions from C3 and C5. The contribution from C5
may come directly from the root or indirectly through
the nerve to the subclavius. In this case, it is known
as accessory phrenic nerve.

147. 'C' [BDC H&N 6th ed 373]


The branch of marginal mandibular nerve runs below
the angle of the mandible deep to the platysma. It
crosses the body of t he mandible and supplies muscles
of the lower lip and chin. To avoid injury, the incision
should be placed 1cm below the base of mandible.

148. 'B' [Refer Synopsis point 39]


r GENERAL ANATOMY
~

V
797

7. MUSCLES OF HEAD AND NECK


1. All of the following muscles take their origin from 10. All are structures lying deep to the hyoglossus
the outer surface of the mandible EXCEPT: muscle except:
a) Buccinator b) Depressor anguli oris a) Hypoglossal nerve b) Lingual artery
c) Mentalis d) Platysma c) Stylohyoid muscle d) Geniohyoid muscle
(AIPG -94, MAN -94) (AIPG -99)
2. Which of the following muscles has dual nerve 11. Which muscle divides the neck into anterior and
supply: posterior triangles?
a) Dig astric b) Lateral pterygoid a) Sternomastoid b) Platysma
c) Masseter d) Temporalis c) Digastric d) Trapezius
(MAN -94) (AIPG -95, 94)
3. All of the following muscles are elevators of the 12. Oral diaphragm is formed by:
mandible EXCEPT: a) Mylohyoid muscle b) Genioglossus muscle
a) Digastric b) Masseter c) Buccinator muscle d) Orbicularis oris muscle
c) Medial pterygoid d) Temporalis (KAR -03)
(MAN -94, AIIMS -93) 13. Which of the following muscles elevates the
4. The infrahyoid muscles are innervated by the: mandible?
a) Ansa cervicalis b) Hypoglossal nerve a) Buccinator b) Temporalis
c) Tenth cranial nerve d) Ansa subclavia c) Caninus d) Lateral pterygoid
(MAN -95) (KAR -01)
5. Superior movement of eye ball is by: 14. Floor of mouth is made by which muscle:
a) SO and IR b) SR and IO a) Genioglossus b) Geniohyoid
c) LR and SR d) MR and SR c) Mylohyoid d) Masseter
(MAN -02) (AIPG -99)
6. Ligamentum denticulatum is: 15. Depression and Protrusion of the mandible:
a) Only a pair of pial extension a) Media l (INTERNAL) pterygoid muscle
b) A dural derivative b) Lateral (EXTERNAL) pterygoid muscle
c) Arachnoid extension only c) Masseter muscle d) Temporalis muscle
d) Extends to sacral segments only (KAR -02; AP -06)
(MAN -99) 16. In relation to the occlusal plane following muscles
7. Retraction of mandible is achieved by: are in descending order:
a) Lateral pterygoid b) Temporalis a) Genioglossus, hyoglossus, cculomoto, cculomot
c) Medial pterygoid d) Masseter b) Genioglossus, cculomoto, cculomot, anterior belly
(AIIMS-13, MAN -02) of digastric
8. Muscle of palate, which works around hamular c) Hyoglossus, genioglossus, cculomoto, anterior
notch and forms a tendon is: belly of digastric
a) Levator palatine b) Palatopharyngeus d) Geniohyoid, genioglossus, cculomot, anterior belly
c) Tensor palatine d) Stylopharyngeus of digastric
(AIPG -97) (KAR -97)
9. In facial palsy the muscle which is paralysed is: 17. Muscles of mastication are supplied by:
a) Levator palpebrae superioris a) Second part of maxillary artery
b) Orbicularis oculi b) Third part of maxillary artery
c) Constrictor pupili c) First part of maxillary artery
d) Dilator pupili d) Facial artery
(AIPG -98) (AIIMS -91)

1) D 2) A 3) A 4) A 5) B 6) A 7) B 8) C 9) B 10) A 11) A 12) A 13) B


14) C 15) B 16) B 17) A
Dental ;lut.,e

18. The action of digastric muscle is: d) Buccinator anteriorly and middle constrictor
a) Depression of mandible (COMEDK -04)
b) Protrusion of mandible 27. Muscle originating from scaphoid fossa is:
c) Side-to-side movement of mandible a) Levator palatine b) Tensor cculo
d) Depressing the floor of the mouth c) Palatoglossus d) Superior constrictor
(AIIMS -90)
19. Medial pterygoid muscle is attached to: 28. The ansa cervicalis innervates which muscle
a) Medial surface of lateral pterygoid plate a) Mylohyoid b) Cricothyroid
b) Lateral surface of medial pterygoid plate c) Stylohyoid d) Sternothyroid
c) Medial surface of medial pterygoid plate
d) Lateral surface of lateral pterygoid plate 29. Stapedius muscle is supplied by _ _ nerve
a) Facial b) Vagus
20. All of the following muscles are grouped together c) Glossopharyngeal d) Trochlear
as "muscles of mastication" except:
a) Buccinator b) Masseter 30. The occulomotor nerve supplies all the muscles of
c) Temporalis d) Pterygoids eye except
(AIIMS -89) a) Lateral rectus b) Superior oblique
21. Temporalis muscle originates from: c) Superior rectus d) Inferior rectus
a) Side of the skull b) Zygomatic process
c) Ramus of the mandible 31. The antagonistic muscle to superior rectus.
d) Pterygopalatine fossa a) Inferior oblique b) Inferior rectus
(AP -2K) c) Superior oblique d) Lateral rectus
22. Mylohyoid muscle:
a) Arises from hyoid bone 32. Anterior and posterior belly of digastric form an
b) Developed from second pharyngeal arch intermediate tendon that attaches to
c) Depresses the hyoid d) Elevates the hyoid a) Mastoid process b) Mandible
(KAR -98) c) Hyoid bone d) Thyroid cartilage
23. Muscle that attaches to zygomatic process of
maxilla: 33. Posterior belly of digastric is attached to
a) Masseter b) Buccinator a) Styloid b) Hyoid
c) Middle temporal d) Medial pterygoid c) Mastoid notch d) Thyroid
(AIIMS -92) (PGI-03)
24. Muscle involved in the rotation and protrusion of 34. All of the following muscles are supplied by
the mandible accessory nerve except
a) Masseter b) Medial pterygoid a) Palatopharyngeus b) Stylopharyngeus
c) Temporalis d) Digastric c) Palatoglossus d) Musculus uvulae
(PGI -01) (PGI-03)
25. Hyperacusis is due to the damage to which of the 35. The occipital bone provides attachment to all
following muscles: except:
a) Orbicularis oris b) Stylog loss us a) Trapezius b) Ligamentum nuchae
c) Stylopharyngeus d) Stapedius c) Sternocleidomastoid
(KAR -98) d) Rectus capitis
26. Which muscles make up the pterygomandibu lar (AIPG-99)
raphae? 36. Which is not anterior triangle of neck:
a) Masseter anteriorly and occulomotor a) Digastrics b) Subclavian
b) Masseter anteriorly and middle pterygoid c) Carotid d) Submental
c) Buccinator anterior and superior constrictor (AIPG-03)

18) A 19) A 20) A 21) A 22) D 23) A 24) B 25) D 26) C 27) B 28) D 29) A 30) A&B
31) B 32) C 33) C 34) B 35) C 36) B
r GENERAL ANATOMY

37. Passavant's muscle is formed by b) Levator palatine forms a delicated tendon which
a) Palatoglossus b) Palatopharyngeus winds round the pterygoid hamulus and flattens
c) Styloglossus d) Superior constrictor out to form the palatine aponeurosis
c) All t he constrictors of pharynx are inserted into
38. Which of the following is correctly matched median raphae on the posterior wall of the pharynx.
a) Doubt - mentalis d) The posterior wall of pharynx, t he upper part of
b) Surprise - Frontalis or epicranius thyropharyngeus is a multiple sheet of muscle and
c) Grief - Depressor anguli oris is overlapped by the upper and middle constrictors.
d) Contempt - Zygomatic minor (PGI-06)
e) All the above 46. The disc of the temporomandibular joint moves
forward principally by:
39. Sternocleidomastoid and trapezius are supplied by a) Condyle b) Stylo mandibular ligament
a) Cranial accessory b) Spinal accessory c) Media l pterygoid muscle
c) Vagus d) Glossopharyngeal d) Lateral pterygoid muscle

40. Abduction of eyeballs is by the action of 47. Which of the following muscles separates the
a) Lateral rectus, superior oblique and the inferior carotid triangle from the digastric triangle?
oblique a) Anterior belly of the diag astric
b) Medial rectus, superior rectus and the inferior b) Posterior belly of the diagoastric
rectus c) Superior belly of omohyoid
c) superior oblique and the superior rectus d) Sternothyroid
d) Inferior oblique and the inferior rectus.
(B HU-07) 48. The lacrimal gland is located in a groove which is
41. Facial muscles are derived from overlap by
a) l't branchial arch b) 2"d branchial arch a) Levator palpebrae superioris muscle
c) 3rd branchial arch d) 4th branchial arch b) Lateral rectus
(COMEDK -06) c) Inferior oblique d) Superior oblique
42. Which of the following is a dilator of palpebral (PGI-06)
fissure? 49. Which of the following is/ are fan shaped:
a) Levator palpebrae superioris a) Middle constrictor b) Temporolis
b) Dilator naris c) Both of the above d) None of the above
c) Orbicularis oculi
d) Occipitofrontalis 50. When the jaw is opened
a) Condyles move upwards
43. Temporalis muscle is inserted into b) Articular disc moves posteriorly
a) Condylar process b) Coronoid process c) Lateral pterygoids contract
c) Lingula d) Ramus of mandible nerve d) Condyles moves around vertical axis
(PGI-05) (KAR-04)
44. Among the muscles of TMJ, the following muscle 51. Genioglossus Muscle is attached fo the posterior
opposing stabilizing and antagonistic muscle force surface of symphysis menti in the
as far as the disc is concerned a) Mental spines b) Superior genial tubercle
a) Medial pterygoid b) Temporalis c) The inferior genial tubercle
c) Lateral pterygoid d) External pterygoid d) Just above the lower border of mandible
(COMEDK-06) (COMEDK-05)
45. Which of the following is correct? 52. Middle constrictor of pharynx has attachment from:
a) Hard palate is formed by maxilla, palatine and a) Body of hyoid bone b) Mandible
vomer bone. c) Pterygomandibular raphae

37) B 38) E 39) B 40) A 41) B 42) A 43) B 44) C 45) C 46) D 47) B 48) A 49) C
50) C 51) B 52) A
Dental ;lut.,e

d) Cricoid cartilage c) Thyrohyoid d) Sternothyroid


(AIPG, AIIMS-07) {AIPG-2011)
53. The following Ligaments are present in 62. When the mouth is opened wide, modiolus becomes
temporomandibular joint except - a) Mobile b) Supple
a) Lateral temporomandibular ligament c) Im mobile d) None of the above
b) Sphenomandibular ligament (B HU-2012)
c) Stylomandibular ligament 63. The posterior bellies of digastric muscle are
d) Alar ligament especially active during
a) Swallowing and speech
54. 'Wry neck' deformity is due to the damage of: b) Swallowing and smiling
a) Platysma b) Sternohyoid c) Swallowing and chewing
c) Sternocleidomastoid d) Omohyoid d) Swallowing and frowning
(BHU-2012)
55. The palatal muscle that ends in a tendon that hooks 64, Which of the following accessory Ligaments of the
around the hamulus and is inserted in the palate is TMJ is Likely to have significance upon mandibular
the movements?
a) Palatoglossus b) Tensor veli palatini a) Sphenomandibular b) Stylomandibular
c) Levator veli palatini d) Palatopharyngeus c) Pterygomandibular d) All of the above
(BHU-07) {KAR-2013)
56. Which one of the following muscles of the soft 65. Which of the following is not an intrinsic muscle of
palate supplied by the mandibular nerve? eye?
a) Levator palati b) Palat glossus a) Dilator papillae b) Sphincter papillae
c) Tensor palati d) Musculus uvulae c) Levator palpebrae superioris
(AP-2009) d) Cilliary muscle
57. All of the following are digastrics, except? (NEET-2013)
a) Muscle fibres in the ligament of Treitz 66. All of the following muscles retract scapula except:
b) Omohyoid a) Trapezius b) Rhomboid major
c) Occipitofrontalis c) Rhomboid minor d) Levator scapulae
d) Sternocleidomastoid (NEET-2013)
(AIPG-2009) 67. Muscles spared by complete transection of cranial
58. Sensory nerve supply of capsule of TMJ is? part of accessory nerve:
a) Auriculotemporal nerve a) Cricopharyngeus b) Palatopharyngeus
b) Facial nerve c) Stylopharyngeus d) Salpingopharyngeus
c) Massetric nerve d) Auricular nerve {AIIMS-2012)
(AIPG-2009, AIIMS MAY 2012) 68. Tensor palati is supplied by:
59. Hypoglossal nerve is a) Facial nerve b) Trigeminal nerve
a) Purely sensory b) Purely motor c) Glossopharyngeal nerve
c) Mixed d) Spinal nerve d) Pharyngeal plexus
(KCET-2010) (AIIMS-2012)
60. Hyoglossus muscle inserts into 69. Tendon of Gracilis, Sartorius and semitendenous
a) Tip of the tongue b) Base of the tongue muscles insert on tibia to form Pesanserinus.
c) Lateral part of t hle tongue Similar structure is seen in
d) Hyoid bone a) TMJ b) Parotid
(AIPG-2010) c) Submandibular gland
61. All of the following muscles are attached to oblique d) Cheek
Line of thyroid cartilage except {AIIMS Nov-13)
a) Superior constrictor b) Inferior constrictor

53) D 54) C 55) B 56) C 57) D 58) A 59) B 60) C 61) A 62) C 63) C 64) B 65) C
66) D 67) C 68) B 69) B
Dental ;lut.,e

7. MUSCLES OF HEAD AND NECK - ANSWERS


1. 'D' [BDC H & N 4th ed 53/ 5th ed 33)
Downward Inferior rectus and superior
Platysma is a muscle of neck.
rotation oblique

2. 'A' [BDC H & N 4th ed 158/ 5th ed 159, 358, 164 Medial rectus, superior rectus
Medial rotation
Tab 11.1) and inferior rectus (no oblique
(adduction)
The digastric muscle is formed by anterior belly and muscle is involved)
posterior belly. Anterior belly of digastric is supplied Lateral rectus
by mylohyoid, which is a branch of mandibular nerve. Lateral rotation
Superior oblique
Posterior belly of digastric is supplied by facial nerve. ( abduction)
Inferior oblique
Option 'B' lateral pterygoid is supplied by nerve to Superior oblique
Intortion
lateral pterygoid, a branch from anterior division of Superior rectus
the mandibular nerve. Medial pterygoid is supplied by
Inferior oblique and
nerve to media l pterygoid, which is branch of main Extortion
trunk of mandibular nerve. Inferior rectus
Elevation of
Levator pa lpebrae superioris
Option 'C' Masseter is supplied by masseteric nerve, a upper eyelid.
branch of anterior division of mandibular nerve.
6. 'A' [BDC H & N 4th ed 86/ 5th ed 92)
Option 'D' Temporalis is supplied by temporal branches
from anterior division of mandible. 7. ' B' [BDC H & N 4th ed 145/ 5th ed 156)

Movement of Jaw Muscles involved


3. 'A' [BOCH & N 4th ed 158/ 5th ed 156)
Digastric muscle depresses the mandible. Depression of • Lateral pterygoid
mandible or • Geniohyoid
4. 'A' [BOCH & N 4th ed 130/ 5th ed 137) opening of mouth
Ansa cervicalis or ansa hypoglossi is formed by a
• Mylohyoid
(MCET-07}
superior and an inferior root. It supplies the infrahyoid • Dig astric
muscles. Th e superior root is the continuation of • Masseter
descending branch of hypoglossal nerve. Its fibres are Elevation of jaw or
derived from 1st cervical nerve. closing of mouth
• Temporalis
• Medial pterygoid
The inferior root is derived from spinal nerves C2
Protrusion
• Lateral pterygoid
and cl. • Medial pterygoid

Option 'D' ansa s ubclavia is a Loop that winds round


• Posterior fibres of
temporalis
the subclavian artery. The middle cervical ganglion is
connected with the inferior cervical ganglion directly Retraction • Masseter
and also through ansa subclavia. • Geniohyoid
• Dig astric,
5. 'B' [BOC H & N 4th ed 109/ 5th ed 118 Tab 7 .1)
Medial and lateral pterygoids
Action Muscles
Lateral movements
acting alternatively
Upward Superior rectus and inferior
rotation oblique 8. 'C' [BDC H & N 4th ed 212/ 5th ed 211, Tab 14.2)
Muscles of the soft palate
• Tensor veli palati
Dental ;lut.,e i======
25. 'D' [BDC H & N 4th ed 139, 140/ 5th ed 271)
Stapedius muscle damps excessive vibration of the
• Downward rotation
Inferior rectus • Medial rotation
stapes caused by high -pitched sounds. In paralysis of
the muscle even normal sounds appear too loud which
• Extortion

is known as "hyperacusis". Medial rectus Medial rotation or adduction


Lateral rectus Lateral rotation or abduction
26. 'C' [BDC H & N 5th ed 33)
• Downward rotation
27. 'B' [BDC H & N 4th ed 213/ 5th ed 211 Tab 14.2)
Su perior oblique • Lateral rotation
• I nt ortion
28. 'D' [BDC H & N 4th ed 130/ 5th ed 137) Inferior oblique
( antagonist to
• Upward rotation
Ansa cervicalis or Ansa hypoglossi supplies the
infrahyoid muscles (Sternohyoid, sternothyroid, superior oblique
• Lateral rotation

thyrohyoid, omohyoid) muscle)


• Extortion

Levator palpebrae
Ansa cervicalis is formed by a superior root and an Elevation of upper eyelid
superioris
inferior root.
32. 'C' [BDC H & N 4th ed 158/ 5th ed 164 Tab 11.1]
Superior root is formed by descending branch of
hypoglossal nerve that carries the fibres of first
33. 'C' [BDC H & N 4th ed 158/ 5th ed 164 Tab 11. 1]
cervical nerve (C 1).The inferior root is formed by C2
and C1 spinal nerves.
34. 'B' [BDC H & N 4th ed 200/ 5th ed 220)
Stylopharyngeus muscle is supplied by glossopharyngeal
Thyrohyoid and superior belly of omohyoid are
nerve.
supplied by C1 through hypoglossal nerve. Ansa
cervicalis supplies the Sternohyoid, sternothyroid and
35. 'C' [BDC H & N 4th ed 73/ 5th ed 6)
the inferior belly of the omohyoid.
36. 'B' [BDC H & N 4th ed 70/ 5th ed 128)
29. 'A' [BDC H & N 4th ed 139/ 5th ed 356)
The side of neck is divided obliquely by
Stapedius is supplied by "nerve to stapedi us" which is
sternocleidomastoid muscle into anterior and posterior
a branch of facial nerve.
triangles. The anterior triangle is subdivided by
digastric muscle and superior belly of omohyoid into
30. 'A & B' [BDC H & N 4th ed 108-09/ 5th ed 348]
All the extraocular muscles of eye except superior • Submental
oblique and Latera l rectus are supplied by occulomotor • Digastric
nerve. • Carotid
• Muscular t riangles
The superior oblique is supplied by 4 t h cranial or
trochlear nerve. The latera l rectus is supplied by The posterior triangle is divided by inferior belly of
abducent nerve. omohyoid into
• Large upper occipital t riangle
31. 'B' [BDC H & N 4th ed 109,110/ 5th ed 118 Tab 7.1)
• Small lower supraclavicular triangle
ACTION OF INDIVIDUAL MUSCLES OF EYE
• Upward rotation 37. 'B' [BDC H & N 4th ed 212/ 5th ed 212)
Superior rectus • Medial rotation The upper fibres of palatopharyngeus form a sphincter
• Intortion internal to superior constrictor. These fibres constitute
passavant's muscle, which on contraction raises
passavant's ridge on the posterior wall of nasopha rynx.
r GENERAL ANATOMY
~

V
305

Passavants ridge (Formed by option " D" superior


• Muscles of face
constrictor and palatopharyngeus) and soft palate (AIIMS-2012)
together close the orpharyngeal isthmus between
• Platysma,
the nasopharynx and oropharynx during 2nd stage of
z nd or Hyoid
• Sta pedius,
swallowing.
arch
• Stylohyoid, Facial nerve
• Auricular muscle
Passavant's ridge (KERALA-2015) • Occipito frontalis
a) Consists of fibres of superior constrictor and • Posterior belly of
pa lat opharyngeus digastric
b) Consists of fi bres of middle constrictor and 3rd arch Stylopharyngeus 9th nerve
pa latopharyngeus Muscles of pharynx, soft Superior
c) Consists of fibres of inferior constrictor and 4 th arch
palate, cricothyroid laryngeal
pa latopharyngeus
Muscles of larynx Recurrent
d) Consists of fi bres of superior constrictor and 6 th arch
except cricothyroid laryngeal
pa lat og loss us

42. 'A' [BDC H & N 4th ed 52]


38. 'E' [BDC H & N 4th ed 51, 52/ 5th ed 58, 59]
The palpebral fissure is surrounded by a sphincter,
Facial Expressions Muscles involved. t he orbicularis oculi, and has a dilator mechanism
Smiling and laughing Zygomaticus major consisting of levator palpebrae superioris (considered
with the orbital muscles) and occipitofrontalis which
Levator labii superioris and
Sadness is part of the scalp.
levator anguli oris
Grief Depressor anguli oris 43. ' B' [BDC H & N 4th ed 145/ 5th ed 149 Tab 10.1]
Anger Dilator nares & Depressor septi Refer synopsis

Corrugator supercelii and 44. 'C' [BDC H & N 4th ed 145/ 5th ed 156]
Frowning
procerus Lateral pterygoid is the on ly muscle, which has it s
Horror, terror, fright Platys ma attachment into the temporo-ma ndibular joint.
Surprise Fronta lis
45. 'C' [BDC H & N 4th ed 221/ 5th ed 218, 219]
Doubt Mentalis All the constrictors of the pharynx are inserted into
Grin ning Risorius median raphae on the posterior wall of the pharynx.

Contempt Zygomaticus minor


Option 'A' (Refer Pg 12) is incorrect because hard
palate is formed from maxilla and palatine bones
39. ' B' [BDC H & N 4th ed 73/ 5th ed 77] only.

40. 'A' [BDC H & N 4th ed 109/ 5th ed 118 Tab 7.1] Option 'B' (Refer Pg 213) is incorrect because tensor
veli palatine is the muscle that forms delicate tendon
41. 'B' [BDC H & N 4th ed 50/ 5th ed 220 Tab 14.4] that winds round the pterygoid hamulus (BHU-07)
Nerve of and flattens out to form the palatine aponeurosis.
Arch Muscles of Arch h h
t e arc
• Muscles of masti- Option ' D' (Refer Pg 223) is incorrect because
cation (NEET- 13) in posterior wall of pharynx, the lower part of
1st arch or • tensor tympani thyropharyngeus is single sheet of muscle and is not
Mandibular
mandibular • t ensor palati overlapped by the upper and middle constrictors. This
nerve
arch • mylohyoid area is known as Killian's dehiscence.
• anterior belly of
digastric 46. ' D' [BDC H & N 4th ed 145/ 5th ed 155]
Dental ;lut.,e

47. 'B' [BDC H & N 4th ed 126 fig 8.7/ 5th ed 128) 52. 'A' [BDC H & N 4th ed 221/ 5th ed 218)
The side of neck is roughly quadrilateral in outline. Hyoid bone
This quadrilateral space is divided obliquely by I
Gives attach ment to
sternoc/eidomastoid muscle in 'anterior' and 'posterior I
Tongue Middle
I
Posterior belly
triangles'. The anterior triangle encloses four supra hyoid
and four infrahyoid muscles. Constrictor of digastric

The anterior triangle is subdivided by digastric and the 53. 'O' [BOCH & N 4th ed 150/ 5th ed 154]
superior belly of omohyoid into
• submental 54. 'C' [BOCH & N 4th ed 74/ 5th ed 79)
• digastric Tortkollis or Wryneck' is a deformity in which the
head is bent to one side and the chin points to the
• carotid
other side. This is due to spasm of the muscles supplied
• muscular triangles.
by spinal accessory nerve i.e, Sternocleidomastoid and
Trapezius.
The carotid triangle is separated from digastric triangle
by posterior belly of digastric.
55. 'B' [BOCH & N 4th ed 212 Tab 14.1/ 5th ed 211 Tab
14.2)
The posterior triangle is subdivided by the "inferior
Tensor veli palatini is a thin triangular muscle of soft
belly of omohyoid" into
palate.
• large, upper occipital triangle
• small lower supraclavicular or subclavian triangle It originates from the latera L side of auditory tube
and base of skull
48. 'A' [BDC H & N 4th ed 62/ 5th ed 65)
Lacrimal gland is situated in the lacrimal fossa. The
t
form a delicate tendon which winds round the
gland is 'J' shaped being indented by the tendon of
pterygoid hamulus
levator palpebrae superioris muscle. It has an orbital
part which is larger and deeper, and a smaller and t
superficial palpebral part that lies within the eyelid. forms palatine aponeurosis
t
The lacrimal sac is situated in lacrimal groove behind finally attached to hard palate.
the medial palpebral ligament. Inflammation of
lacrimal sac is called dacrocystitis. 56. 'C' [BOCH & N 5th ed 211]
Ref. Q.No. 8 for explanation
The lacrimal duct begins at the lower end of lacrimal
sac, run downwards, backwards and laterally, and finally 5 7, 'O' [Gray's Anatomy 39th ed 114, 500, 504]
opens into inferior meatus of nose. A fold of mucous Digastric muscles refer to muscles with two bellies.
membrane called the valve of Hasner forms an imperfect The following are digastric mL1scles.
va lve at the lower end of the duct.
i) Occipitofrontalis:
49. 'C' [BDC H & N 4th ed 145, 2 20, Fig.14.15/ 5th ed
It has two bellies; Occipital bellyofoccipitofrontalis
149 Fig 10.4, 219 Fig 14. 18]
Geniog lossus is also a fan shaped muscle. and frontal belly of occipitofrontalis.

50. 'C' [BOCH & N 4th ed 145/ 5th ed 156] ii) Omohyoid:
It consists of two bellies united at an angle by an
51. 'B' [BDC H & N 4th ed 252/ 5th ed 252 Tab 17. 1] intermediate tendon. The two bellies are inferior
Genioglossus arises from the upper genial t ubercle of belly of omohyoid and superior belly of omohyoid.
the mandible. Genia hyoid arises from the lower genial
tubercle of the mandible.
r GENERAL ANATOMY

iii) Muscle fibres in ligament of Treitz: Modiolus is a chiasma of facial muscles held together
Contain two bellies; belly of skeletal muscle fibres by fibrous tissue located superolateral to each angle of
and belly of smooth muscle fibres. mouth. It is important in moving mouth. It is formed
by
Sternocleidomastoid has two heads of origin but • Orbicularis oris
a single belly and hence cannot be classified as • Buccinator
diagastric muscle. • Levator anguli oris
• Depressor ang uli oris
58. 'A' [BOCH & N 5th ed 354, 156, 158) • Zygomatius major
Branches of auriculotemporal nerve: • Risorius and
• Platysma
i) Slender filaments to the posterior part of the
capsule of TMJ.
63. 'C' [BOCH & N 5th ed 164, 39)
ii) One or two thick branches to the parotid gland
which mingle with the branches of the facial nerve
64. 'B' [BDC 5th ed Pg 154/ 5th ed 154-156)
in its substance.
The 3 functional ligaments of TMJ
iii) Cutaneous branches to the auricle and temple.
• Collateral ligament
• Capsular ligament
59. 'B' [BOCH & N 5th ed 368)
Hypoglossal is the twelfth crania l nerve and it supplies • Temporomandibular ligament
the muscles of the tongue.
The 2 accessory ligaments of TMJ are
60. 'C' [BDC H & N 5th ed 252 Tab 17.1) • Sphenomandibular ligment
• Stylomandibular ligment
61. 'A' [BOCH & N 4th ed 221/ 5th ed 130, 218)

Constrictor Origin & Insertion The sphenomandibular ligament does not have any
significant effect on mandibular movement. The
• Pterygoid ha mulus
• Pterygomandibular raphae stylomandibular ligament becomes tensed in extreme
Superior protrusive movements of mandible and does have
• Side of posterior part of tongue
significant limiting effect on mandibular movements.
• Inserted on to median raphe on
Pterygomandibular raphe or ligament (option 'C') is
posterior wall of pharynx
not a ligament of TMJ.
• 2 parts - Thyropharyngeus &
Cricopharyngeus
65. 'C' [Clinical anatomy by regions by Richard S. Snell
• Thyropharyngeus - from thyroid 9th ed 607)
Inferior cartilage (oblique line of thyroid The intrinsic muscles move structures within the
cartilage) eyeballs whereas extrinsic muscles originate outside
• Cricopharyngeus - from cricoid the eyeballs and insert on their outer surface known
cartilage as sclera.
Originates from posterior surface of
manubrium sterni & adjoining part of Extrinsic muscles of eye:
Sternothyroid
first costal cartilage and inserted on • Levator palpebrae superioris, superior rectus,
to oblique line of thyroid cartilage medial rectus, inferior rectus and inferior oblique.
Originates from oblique line of • Supplied by occulomotor nerve.
thyroid cartilage and inserted on
Thyrohyoid
to lower border of body and greater Intrinsic muscles of eye:
cornua of hyoid bone
• Constrictor papillae of the iris and cilliary muscles.
• Supplied by short ciliary nerves of the occulomotor
62. 'C' [Check Explanation Below]
nerve.
r GENERAL ANATOMY
~

V
309

8. EMBRYOLOGY
1. Upper and Lower Lips are formed from which 8. The posterior part of the tongue develops from:
embryonic processes: a) First Arch b) Third Arch
a) Maxillary and mandibular c) Second Arch d) All of the above
b) Maxillary and median nasal (KAR-2K)
c) Maxillary mandibular lateral nasal and median nasal 9. Meckel's cartilage extends from:
d) Mandibular and median nasal a) Otic capsule b) Styloid bone
(AIPG- 94) c) Hyoid cartilage d) None of the above
2. Palate is formed from: (KAR-01)
a) Median pa latine process 10. Embryologically hard palate develop from:
b) Lateral palatine process a) Maxillary process b) Lateral pa latine process
c) Both (1) and (2) d) Either (1) or (2) c) Frontonasal process d) All of the above
(KAR-97) (AIPG-2K)
3. Cleft Lip occurs due to failure of: 11. The tongue is formed from
a) Fusion of lateral nasal process with maxillary Process a) l 't, 2"d, 3rd branchial arches
b) Fusion of median nasal process with maxillary b) 1•t, 3rd, 5th branchial arches
process c) 1'\ 3rd, 4th branchial arches
c) Fusion of median and lateral nasal process d) 1'\ 4th, 5th branchial arches
d) None of the above (KAR-99)
(AIPG-02) 12. The primary germ Layer endoderm is derived from:
4. Oblique facial cleft results due to: a) Embryonal disc b) Yolk sac
a) Improper fusion between maxillary and Lateral c) Amniotic cavity d) Placenta
nasal process (AIPG-95)
b) Improper fusion between maxillary and Medial 13. The Lateral Lingual swellings and tubercu Lum impar
nasal process give rise to:
c) Improper fusion between frontonasal and a) Anterior 1/3 of tongue
Mandibular process b) Anterior 2/3 of tongue
d) Improper fusion between maxillary and Mandibular c) Posterior 1/3 of tongue
process d) Posterior 2/3 of tongue
(KAR-02) (AIIMS-99)
5. The philtrum of the upper Lip is formed Largely by 14. Number of somatic chromosome is:
the: a) 42 b) 41
a) Lateral nasal process b) Frontonasal process c) 46 d) 44
c) Maxillary process d) Mandibular process (AIPG-97)
(KCET-11) 15. Spheno mandibular Ligament is developed from:
6. Tongue develops from all of the following except a) First arch b) Second arch
a) Tuberculum impar b) Hypobranchial eminence c) Third arch d) Fourth arch
c) Lingual swellings d) Arytenoid swellings (KAR-01)
(AIIMS-98) 16. Muscles of the tongue are derived from:
7. Developmentally stomodeum is separated from the a) 1't pharyngeal swelling
pharynx dorsally by: b) Lateral ling ual swelling
a) Pharyngeal arch c) Occipital myotomes d) Preoptic myotome muscles
b) Buccopharyngeal membrane (COMEDK -06)
c) Rathke's pouch d) Mandibular process 17. Posterior belly of digastric is derived from:
(KAR-2K) a) First brachia[ arch b) First brachia[ pouch

1) C 2) C 3) B 4) A 5) B 6) D 7) B 8) B 9) A 10) D 11) C 12) B 13) B


14) D 15) A 16) C 17) C
r GENERAL ANATOMY

34. The fusion of two bony structures with a ligament 44. A person showing two cell lines derived from one
is known as zygote is
a) Synostosis b) Synchondrosis a) Chimerism b) Mosaicism
c) Dia rth rosi s d) Syndesmosis c) Segregation d) Pseudo-dominance
(AP-08) (AIPG-2010)
35. The period of embryo extends 45. Maximum oral structures are having their origin
a) From the end of first week till the eighth week from
b) From the period of ovum till the tenth week a) Ectoderm b) Mesoderm
c) From oogenesis till the eight week c) Endoderm d) None of the above
d) From fertilization till the eighth week (NEET-2013)
(UPSC-2009) 46. Neural tube formation occurs on
36. Gestation period consists of the following EXCEPT a) 18 to 23 days post fertilization and is the second
a) Germinal period b) Embryonic period stage of craniofacial development
c) Fertilization period d) Foetal period b) 28 to 38 days post fertilization and is the third
(KCET-2010) stage of craniofacial development
37. 1st arch artery is c) 42 to 55 days post fertilization and is the fourth
a) Maxillary artery b) Mandibular artery stage of craniofacial development
c) Carotid artery d) Stapedial artery d) 17th day post fertilization and is the first stage of
(IGNOU-2010) craniofacial development
38. Which part of the body is underdeveloped at birth? (COMEDK-2013)
a) Eyes b) Ears 47. Which of the following is NOT true about para median
c) Face d) Brain pits?
(KCET-2010) a) Found on the lower lip
39. Development of palate beings at the age of b) Found on the upper lip
a) 4weeks b) 6 weeks c) On either side of the midline
c) 13 weeks d) 18 weeks d) Associated with cleft lip and palate
(IGNOU-2010) (KAR-2013)
40. Which of the following is a derivative of the second 48. Which of the following structures develops from all
brachial cleft - the three germ layers?
a) Tympanic cavity b) Meckel's cartilage a) Soft Palate b) Mitral valve
c) Palatine tonsil d) Tongue base c) Tympanic membrane d) Tooth
(KCET-2009) (MCET-14)
41. Umbilical cord contains - 49. Branchial fistula is persistent of
a) 3 arteries & 1 vein b) 1 artery & 3 veins a) First branchial cleft b) First branchial arch
c) 1 artery & 1 vein d) 2 arteries & 1 vein c) Second branchial cleft
(COMED-2009) d) Second branchial arch
42. Which of the following is not a neuroectodermal (MCET-14)
derivative? 50. Hoffbauer's cells are present in
a) Pa neth cells b) Neurons a) Pituitary gland b) Parathyroid
c) Schwann cells d) Odontoblasts c) Placenta d) Pineal gland
(AIPG-2010) (GCET-14)
43. Which is not formed from the cartilaginous part of 51. Choose the correct nerve pharyngeal arch pair
viscerocranium? a) 2nd arch - mandibular nerve
a) Sphenoid b) Vomer b) 4th arch - occulomotar nerve
c) Zygoma d) Lacrimal c) 3rd arch - glossopharyngeal nerve
(AIPG-2010) d) 1st arch - facial nerve
(COMEDK-14)

34) D 35) D 36) C 37) A 38) C 39) A 40) C 41) D 42) A 43) C 44) B 45) A 46) A
47) B 48) C 49) C 50) C 51) C
Dental ;lut.,e

52. What tissue from the implanting embryo directly


interfaces with the endometrial connective tissue?
a) Inner cell mass
b) Extra embryonic mesoderm
c) Epiblast d) Syncytiotrophoblast
(COMEDK-2015)
53. Most common cells of connective tissue are:
a) Fibrobalst b) Plasma cells
c) Basal cells d) Histiocytes
(PGI DEC-2011)
54. Hyaline cartilage is developed from?
a) Mesoderm b) Ectoderm
c) Ectomesoderm d) Endoderm
(PGI JUNE-2012)
55. Period of embryo is?
a) 4 to 16 weeks b) Fertilization to 2 weeks
c) 2 to 8 weeks d) 8 weeks to birth
(PGI JUNE-2012)
56. The number of symphyseal cartilages appearing
during the development of mandible
a) Two b) Three
c) Four d) One
(KERALA-2015)
57. The formation of secondary palate takes place
during
a) 42- 55th day of gestation
b) 10-12th day of gestation
c) 3rd month of gestation
d) 4th month of gestation
(KERALA-2015)

52} D 53} A 54) A 55) C 56) A 57) A


16. 'C' [Inderbir singh 7th ed 162] 20. 'B' [Inderbir singh 7th ed 115]

17. 'C' [Inderbir singh 7th ed 115] 21. 'A'

18. 'A' [Inderbir singh 7th ed 47] 22. 'D' [Inderbir singh 7th ed 115]

19. 'C' [Inderbir singh 7th ed 156] 23. 'A' [Inderbir singh 7th ed 357]
Odontoblasts are the cells of mesodermal origin

24. 'C' [Inderbir singh 7th ed 119]


DERIVATIVES OF ENDODERMAL OR PHARYNGEAL POUCHES
• Pharyngotympanic or auditory t ube,
1st pouch • Middle ear cavity,
• Tympanic antru m

2nd pouch
• Tonsil (Ventral part of pouch)
• Tubotympanic recess (from dorsal part of pouch)

3rd pouch
• Thymus
• Inferior parathyroid
4 th pouch • Superior parathyroid glands

5 th pouch
• The 5 th pouch is incorporated into the 4 th pouch and forms the "caudal pharyngeal
complex.
(Ultimobranchial pouch)
• The caudal pharyngeal complex gives origin to Para follicular cells of thyroid gland .

25. 'A' [Inderbir singh 7th ed 119]

26. 'A' [Inderbir singh 7th ed 115]


Arch Nerve Artery Muscles Skeletal components

1 Mandibular (V3) • Four muscles of mastication • Mandible


• Tensor tympani • Maxilla
• Tensor veli palatini • Malle us
-
• Mylohyoid • I ncus,
• Ant erior belly of digastric. • Sphenomandibular
ligament
• Anterior ligament of
malleus.

2 Facial (VII) • Muscles of facial expression • Lesser cornua and


• Occipito frontalis superior part of hyoid
• Platysma bone
- • Posterior belly of digastric • Styloid process
• Stapedius • Stapes (MHCET-15)
• Stylohyoid • Stylohyoid ligament.
• Auricular muscles.
r GENERAL ANATOMY

3. IX (Glosso - • Right and • Stylopharyngeus • Greater cornua and


pharyngeal) left common inferior part of hyoid
carotid arteries

4. • Superior laryngeal • Right • All the muscles of pharynx • Laryngea l cartilages


• Pharyngeal subclavian except stylopharyngeus
branches artery • All the muscles of palate
from vagus to • Arch of aorta except tensor veli palatini
pharyngeal plexus • Cricothyroid muscle

5. o;sappears soon

6. • Recurrent laryngeal • Right and left • All the intrinsic muscles of • Laryngea l cartilages.
of vagus pulmonary larynx except cricothyroid
arteries • Inferior constrictor
• Ductus
arteriosus

27. 'D' [Inderbir singh 7th ed 7 4]

28. 'B' [Inderbir singh 7th ed 114]


Refer Q. No 15

29. 'C' [Inderbir singh 7th ed 115]

30. 'A' [Inderbir Singh 7th ed 131]


Acrocephaly or Oxycephaly • Premature both the coronal and sagittal sutures resulting in a pointed skull.

Scaphocephaly • Premature union of sagittal suture resulting in a boat-shaped skull.

• Small skull due to fa ilure of brain to grow.


Microcephaly
• Microcephaly is seen in Beckwith - hypoglycemic syndrome and Fanconi syndrome

Plagiocepha ly • Asymmetrical union of sutures resulting in a twisted skull

Anencephaly • Characterized by missing of greater part of the vault of the skull.

• Premature fusion of the coronal sutures forces the skull to grow wide relative to
its length resulting in Short and broad skull.
Brachycephaly • Brachycephalic skull with a flat occipit is seen in Down syndrome.
• Bracepha lic skull with a narrow foramen magnum is seen in Achondroplasia.
• Brachycephalic skull with absence of clavicles is seen in cleidocranial dysplasia.

• Long and thin skull


Dolicocephaly
• Seen in Marfan's syndrome

31. 'B' [Inderbir Singh 7th ed 118]


Pharyngeal arches are rod like thickenings of mesoderm present in the wall of the foregut. Initially, there are six
arches. The fifth arch disappears soon and only five remain. Between pharyngeal arches a series of pouches of
endoderm and ectodermal dips are present. The pouches of endoderm are called endodermal pouches or pharyngeal
pouches. The ectodermal dips are called ectodermal clefts. The endodermal pouches take part in the formation of
Dental ;lut.,e

several important organs (Refer Q. No. 24) . Tonsil is formed from the ventral part of second pouch while tubotympanic
recess is formed from its dorsal part.

32. 'A' [Inderbir Singh 7th ed 134)


Joints:
I. Fibrous/Synarthroses (connects bone without allowing any movement. Eg.: Bones of skull)
• Suture
• Syndesmosis
• Gomphosis

II. Cartilagnious/Amphiarthroses (Bones are attached by cartilage. Eg.: Spine or ribs)


• Primary cartilaginous (Synchondrosis)
• Secondary cartilaginous (Symphyses)

III. SynovialfDiarthroses (Cavities between bones are filled with synovial fluid, greatest movement is seen)
• Plane
• Hinge
• Pivot
• Condyloid
• Saddle
• Ball and socket

• Bones are held by a thin layer of dense fibrous tissue and also by interlocking
Suture projections of the bones
Eg.: Bones of skull (cranial sutures)

• Bones are held by a long cord or sheet of dense fibrous connective tissue or ligamental
Syndesmosis (AP-08) tissue.
Eg.: Tibiofibula; Radio ulna joints

Gomphosis
• Peg in socket fibrous joint
Eg.: Articulations of teeth in their alveolar sockets
• A plate of hyaline cartilage alone connects the bones at the joint and the joints are
Synchondroses immovable.
Eg.: Joint between rib & sternum.

Symphyses
• It involves both hyaline and fibrocartilage. Limited movements are permitted.
• Usually seen in midline.

Synostosis
• Characterized by osseous union between the bones.
• Synostois is an abnormal development of a joint.

33. 'C' [ BDC 4th ed 170)

34. 'D' [Inderbir Singh 7th ed 134)


Refer Q. No. 32
r GENERAL ANATOMY
~

V
321

39. Abduction of shoulder is done by all, EXCEPT c) Cajal d) Pyramidal


a) Teres major b) Teres minor (AIPG-2010)
c) Deltoid d) Supraspinatus 49. All of the following are derivatives of connective
(MCET-07) tissue except?
40. The receptor cells of the olfactory epithelium are a) Endothe~al eel~ b) Clara cells
a) Bipolar neurons b) Unipolar neurons c) Fibroblast d) Mast cells
c) Multipolar neurons d) Stellate cells (AIPG-2010)
(COMEDK-08) 50. Basal lamina of blood vessel in CNS is secreted by
41. All of the following are composite muscles except? a) Endothelial cells b) Oligodendrocytes
a) Pectineus b) Rect us femoris c) Microglia d) Astrocytes
c) Adductor magnus d) Biceps femoris (AIPG-2010)
(AIPG-2009) 51. Infarction of anterior inferior cerebellar artery may
42. What is the type of joint between the ossicles of cause damage to
ear? a) Pyramidal tract
a) Fibrous joints b) Lateral spinothalamic tract
b) Primary cartilaginous c) Vestibular nuclei
c) Secondary cartilaginous d) Spinal nucleus of trigeminal nerve
d) Synovial joint (AIIMS-2009)
(AIPG-2009) 52. The slightly movable articulations in which the
43. All of the following are true about skeletal muscle contiguous bony surfaces are either connected by
except? broad flattened disks of fibrocartilage or united by
a) They are cylindrical in shape interosseous ligaments are known as
b) Have multiple nuclei a) Gomphosis b) Enarthroses
c) Have transverse striations c) Diarthroses d) Amphiarthroses
d) The nuclei are central (AIPG-2010)
(PGI-2008) 53. Which of the following organs has the most
44. The lingual surface of epiglottis is lined by permeable capillaries?
a) Stratified squamous epithelium a) Brain b) Posterior pituitary gland
b) Simple squamous epithelium c) Liver d) Small intestine
c) Stratified columnar epithelium (COMED-2012)
d) Pseudostratified epithelium 54. The cortex of lymph node contains
(COMED-2010) a) Cords of billroth b) Hassal's corpuscles
45. The maxillary sinus drains into c) Lymphatic nodules d) While pulp
a) Middle meatus b) Inferior meatus (KCET-2012)
c) Superior meatus d) Spheno ethmoidal recess 55. Type of collagen present in circumaxillary sutures:
(KCET-2010) a) Type I b) Type II
46. CSF is directly returned to venous system by: c) Type III d) Type IV
a) Choroid plexus b) Cerebral veins (PGI-2011)
c) Arachnoid villi d) Emissary veins 56. Parotid fascia extends anteriorly as-
(COMED-2009) a) Deep cervica l fascia b) Fascia lata
4 7. Only nerve that originates from the dorsal surface c) Masseteric fascia d) Stylomandibular ligament
of the brain stem is (AIPG-2011, 2012)
a) Trochlear b) Occulomotor 5 7. The blood brain barrier is absent in all of the
c) Vagus d) Abducent following except?
(AIPG-2010) a) Area posterna b) Sub fornic region
48. Which cells are not present in cerebral cortex? c) Neurohypophysis d) Habernacular trigone
a) Purkinje b) Stellate

39) A 40) A 41) B 42) D 43) D 44) A 45) A 46) C 47) A 48) A 49) B 50) A 51) C
52) D 53) C 54) C 55) A 56) C 57) D
~
GENERAL ANATOMY 323
V

9. MISCELLANEOUS - ANSWERS
1. 'A' [BDC H & N 4th ed 59/ 5th ed 63, 157)
• Part of scalp just above and
The entire lymph from head and neck drains ultimately
behind the auricle
into deep cervica l nodes, which are grouped into
• The upper half of the medial
jugulo digastric, and jugulo omohyoid node. Post auricular
surface and margin of the
nodes
auricle
The jugulo digastric node is the main node draining the
tonsil while thejugulo omohyoid node is considered as • Posterior wall of external
the main lymph node of tongue. acoustic meatus

• TMJ
The peripheral nodes are classified into two circles
• Parotid gland
superficial and deep. Parotid lymph
• Temple
nodes
• External accoustic meatus
A. Superficial Circle
• Parts of eyelids and the orbit
• Submental
• Submandibular • Part of cheek and Lower Buccal and
eyelid mandibular
• Buccal and mandibular
lymph nodes
• Preauricular (Parotid)
• Post auricular (Mastoid) • Larynx Pre laryngeal
• Trachea, and pretracheal
• Occipital
• Isthmus of thyroid nodes
• Anterior cervical
• Superficial cervical • Oesophagus
• Trachea Para tracheal
B. Deep circle includes • Larynx
• Prelaryngeal and pretracheal • Pharynx
• Paratracheal • Auditory tube,
Ret ropharyngeal
• Retropharyngeal • Soft pa late
nodes.
• Posterior part of hard palate
Part Drains into • Nose
• Chin
• Central part of lower lip 2. 'C' [BDC H & N 4th ed 262/ 5th ed 261)
Sub mental
• Tip of tong ue
3. 'K [BDC H & N 4th ed 150/ 5th ed 155, 257)
• The adjoining gum
• External ear
• Centre of forehead
Elastic • Epiglottis
• Nose and para nasal sinuses cartilage • Cornicula te
• Upper lip
• Cuneiform cartilages .
• Outer part of lower lip with Submandibular
the underlyi11g gum and • TMJ
Fi bro
teeth cartilage
• Intervectebral discs
• Ant. 2/3rds of tongue and • Pubic symphysis.
floor of mouth
Dental ;lut.,e

8. Foramen of skull
Foramen Structures
Transmits supraorbital nerve (which arise from the ophthalmic division of Trigeminal
Supra orbital
nerve) and vessels
Infra orbital Infra orbital nerves and vessels
Mental foramen Mental nerve and vessels

Mastoid foramen
• Emissary vein connecting sigmoid sinus with posterior auricular vein
• Meningeal branch of occipital artery
Incisive foramen • Terminal parts of greater palatine vessels
(Foramen scarpa) • Terminal part of nasopalatine nerve

Greater palatine
• Greater palatine vessels
• Anterior palatine nerve ( or greater palatine nerve)
Lesser palatine • Middle and posterior palatine nerve

Palate vaginal canal


• Pharyngeal branch of pterygo palatine gang lion .
• Pharyngeal branch of maxillary artery
Vomero vaginal canal • Branches of pharyngeal nerves and vessels
• Mandi bu la r nerve
• Accessory meningeal artery
Foramen ovale
• Lesser petrosal nerve
• Emissary vein connecting cavernous sinus wit h pterygoid plexus of veins.
Foramen rotundum • Maxillary nerve (PGI JUNE-11)
Optic canal • Optic nerve and ophthalmic artery
• Middle meningeal artery
Foramen spinosum • Meningeal branch of mandibular nerve (nervus spinosus)
• Posterior trunk of middle meningeal vein
Through anterior part

• Apical ligament of dens


• Membrana tectoria
Through subarachnoid space
• Spinal accessory nerve
Foramen magnum • Vertebral arteries
• Anterior and posterior spinal arteries
Through posterior part
• Lower part of medulla
• Tonsils of cerebellum
• Meninges
Foramen lacerum Meningeal branch of pharyngeal artery
Carotid canal Internal carotid artery along with the venous and sympathetic plexus around the artery
• Hypoglossal nerve
Hypog lossal or • Meningeal branch of hypoglossal nerve
anterior condylar cana l • Meningeal branch of ascending pharyngeal artery
• Emissary vein connecting sigmoid sinus with jugular vein
~
GENERAL ANATOMY SYNOPSIS 331
V

Posterior condyla r • Emissary vein connecting sigmoid sinus with sub occipital venous plexus.
canal
• Present in transverse process of cervical vertebrae
Fora men • Vertebral artery
tra nsversa ri um • Vertebral veins
• Inferior cervical ganglion
Seen in ethmoid bone (Also at the junction of anterior 2/3'd and posterior 1/3rds of
Foramen caecum
tongue.)
This is a inter ventricular fora men through which lateral ventricles opens into 3rd
Foramen monro
ventricle
Foramen of magendie Medial opening in the roof of the 4th ventricle of hindbrain.
Foramen of luschka Opening of lateral recess of 4th ventricle.
Lateral part
• Lacri ma l nerve
• Frontal nerve
• Trochlear nerve
• Lacrimal and middle meningeal artery
Middle part
Superior orbital fissure
• Occulomotor nerve
• Nasociliary nerve
• Abducent nerve
Medial part
• Inferior ophthalmic vein
• Symphathetic plexus around the internal carotid artery
• Maxillary nerve
• Zygomatic nerve
Inferior orbital fissure • Orbital branches of pterygo-palatine ganglion
• Inferior ophthalmic vein
• Infra orbital vessels
Internal acoustic Transmits the 7th and gth cranial nerves and the labyrinthine vessels.
meatus
Anterior part
• Inferior petrosal sinus
• Meningeal branch of ascending pharyngeal artery
• Sigmoid sinus
Jugular fora men Middle part
• 9, 10, 11th nerve (COMEDK-2013)
Posterior part
• IJV
• Meningeal branch of occipital artery
Dental ;lut.,e

9. Structures passing through foramen magnum

• Lower part of medulla




Anterior and posterior spina l arteries
Tonsils of cerebellum
} Descending structures

• Vertebral artery


Apical ligament and membrana tectoria
Spinal accessory nerve
} Ascending structures

10. Mandible - parts, attachments and relations

MENTAL FORAMEN Present below the interval between the premolar teeth.

MYLOHYOID RIDGE / LINE Extends obliquely from below the third molar to median area below the
(Present on inner surface of body) genial tubercles

MYLOHYOID GROOVE Extends on to the body below the posterior end of mylohyoid line
(Present on ramus)

Note:
• The submandibular fossa is present BELOW the mylohyoid line or ridge and it lodges submandibular gland .
• The lingual fossa is present above the mylohyoid ridge and it lodges sublingual gland .
• Infection from mandibular z nd and 3 rd molars spread to submandibular space as their root apices lie below the
mylohyoid line.
• Infection from mandibular premolars and l't molar involve the lingual space as their root apices lie above the
mylohyoid line.
• The mandibular torus is common ly present ABOVE the mylohyoid line.

GENIAL TUBERCLES 4 small elevations near symphysis menti. The upper genial tubercles give
origin to genioglossus and the lower tubercle to the geniohyoid.

DIGASTRIC FOSSA The anterior belly of digastric arises from digastric fossa.
Note: The posterior belly of digastric arises from mastoid notch.

MANDIBULAR or SIGMOID NOTCH Masseteric nerve and vessels pass through the mandibular notch.

LINGULA • Tong ue shaped projection near anterior margin of mandibular foramen


• Gives attachment to sphenomandibular ligament.

INCISIVE FOSSA Gives origin to mentalis.

MENTAL FORAMEN Mental fora men transmits the mental nerve and vessels.

MANDIBULAR FORAMEN Inferior alveolar nerve and vessels enters the mandibular canal t hrough the
mandibular foramen

PTERYGOID FOVEA A depression in the anterior surface of neck of condylar process. It gives
attachment to a latera l pterygoid muscle.
Dental ;lut.,e

18 . Sternocleido mastoid muscle: v. Spinal Cord


i) The sternomastoid and trapezius are large a) The spinal cord is the lower elongated,
superficial muscles of neck. Both of them are cylindrical part of the CNS. It occupies the
supplied by spinal part of accessory nerve. upper two thirds of the vertebral canal.

ii) The trapezius is considered as muscle of upper b) In adults it extends from the level of upper
limb, because of its main action on the shoulder border of the atlas to the lower vertebra L1 or
girdle. upper border of L2 vertebra. In the new born,
the spinal cord ends at L3 •
iii) Actions of Sternocleidomastoid
A) WHEN SINGLE MUSCLE CONTRACTS c) The spinal cord shows 'H' shaped gray matter
• It turns the face to the opposite side. and outer white matter and gives off 31 pairs
of spinal nerves.
• It can also tilt the head towards the
shoulder.
d) The central core of H-shaped gray matter
B) WHEN BOTH MUSCLES CONTRACT TOGETHER divides peripheral white matter into three
• They draw the head forwards as in eating columns.
and in lifting the head from pillow. • Anterior column - Motor in nature.
• With the long us colli, they flex the neck • Posterior column - Sensory.
against resistance. The reverse action • Lateral column - Mixed.
helps in forced inspiration.
e) The cortico spinal tracts or pyramidal tracts
19. Brain and spinal cord are example of descending tracts. These
1. The nervous system may be divided into CNS (made are motor tracts and constitute the upper
up of the BRAIN and SPINALCORD and periph eral motor neuron in the motor pathway from
nervous syste m (consisting of the peripheral the cortex to voluntary muscles.
nerves and ganglia associated with them).
f) The pyramidal tracts are concerned with
ii. The brain consists of voluntary movements of the body.
• Cerebrum comprises two cerebral hemispheres.
• the cerebel{um 20. CRANIAL NERVES

• the mid brain } a) The CRANIAL NERVES are 12 pairs. They are
• thepons Brainstem • Olfactory nerve
• the medulla • Optic
• Occulomotor
The midbrain, the pons and the medulla together • Trochlear
form the brain stem. The medulla is continued
• Trigerni nal
below as the spinal cord.
• Abducent
iii. Peripheral nerves attached to the brain are called • Facia l
cranial nerves and those attached to spinal cord • Vestibule coch lear
are called spinal nerves. • Glossopharyngeal
• Vagus
1v. The nerves supplying the viscera, along with
the parts of brain and spina l cord constitute • Accessory
the autonomic system. The autonomic nervous • Hypoglossal
system is subdivided into two major parts -
symphathetic and parasympathetic.
~
GENERAL ANATOMY SYNOPSIS 341
V

26. INFRAHYOID MUSCLES:

Muscle Origin and insertion Nerve supply Actions


Originates from manubrium sterni and
adjoining parts of the clavicle and the
Sternohyoid Ansa cervicalis Depresses the hyoid bone
posterior sternoclavicular ligament and
inserts into hyoid bone
Originates from manubrium sterni and
Sterno thyroid adjoining part of l't costal cartilage and Ansa cervicalis Depresses the larynx
Inserts into thyroid cartilage
• Depresses the hyoid
bone.
Originates from thyroid cartilage and C1 through • Elevates the larynx
Thyrohyoid
inserts on to the hyoid bone hypoglossal nerve when the hyoid is fixed
by the suprahyoid
muscles.
• Superior belly
Omohyoid (it by the superior
Originates from upper border of scapula root of the ansa
has an inferior
and adjoining part of supra scapular ceNicalis. Depress the hyoid bone.
belly and a
ligament and inserts on the hyoid bone.
superior belly) • Inferior belly by
ansa cervicalis.

27. SUPRAHYOID MUSCLES

MUSCLE ORIGIN AND INSERTION NERVE SIPPLY ACTION


i) ANTERIOR BELLY arises from
Digastric digastric fossa of mandible and the i) Anterior belly
(The other muscles POSTERIOR BELLY arises from the by mylohyoid
mastoid notch of the temporal bone. Depresses the mandible and
with 2 bellies are neNe
elevates the hyoid bone
"occipitofrontalis" ii) Both bellies unite to form a ii) Posterior belly
and "omohyoid") intermediate tendon that is by VII nerve
attached to the hyoid bone.

Originates from styloid process and • Pulls the hyoid bone


before insertion divides into two slips upwards and back-wards.
that pass one on each side of digastric Facial nerve • It fixes the hyoid bone
Stylohyoid muscle. Both slips a re attached to hyoid with other muscles
bone attached to it.
• Elevates the hyoid
• Originates from the mylohyoid line
bone and depresses
to the mandible.
ttie mandible (similar
• Anterior and middle fibres are to dig astric and
Mylohyoid inserted into medial raphae that Mylohyoid nerve geniohyoid).
unites the rig ht and left muscles
• Elevates the floor of
whereas the posterior fibres inserts
mouth during l't stage of
into the hyoid bone
deglutition.
~
GENERAL ANATOMY SYNOPSIS 345
V

33. VASCULAR SUPPLY OF HEAD AND NECK:

ARCH OF AORTA

Brachiocephalic left common carotid Left subclavian artery

Right common carotid artery Right subclavian artery


(Principal artery of upper limb)
Divides into external and internal
carotid arteries at the level of
upper border of thyroid cartilage
External carotid Internal carotid
(Chief artery of supply to (Principal artery of brain
structures in front of the and eye)
neck and in the face)

1
Branches are
1
Branches are Branches are

1. Anterior 1. Cervical part 1. Vertebral artery


• Superior thyroid • no branches in the neck • First and largest branch.
• Lingual
• Facial 2. Petrous part 2. Internal thoracic
• Cardio tympanic
2. Posterior • Pterygoid branch 3. Thyrocervical trunk
• Occipital • inferior thyroid
• Posterior auricular 3. Cavernous part • supra scapular
• Branches to trigeminal • superficial cervical
3. Medial gang lion and hypophysis
• Ascending cerebri 4. Costo cervical trunk
pharyngeal
4. Cerebral part 5. Dorsal scapular
4. Terminal • Opthalmic
• Maxillary • Anterior cerebral
• Superficial tern poral • Middle cerebral
• Posterior cerebral
• Anterior choroidal
~
GENERAL ANATOMY SYNOPSIS 349
V

to supply the submandibular and sublingual • Communications are through thyroid,


glands. tracheal, esophageal and middle cervical
(Note: Symphathetic fibres pass through the cardiac branches.
submandibular ganglion without relay). • It is connected with the inferior cervical
gang lion directly and also through a loop
E. Otic Ganglion: that winds round the subclavian artery. This
• Topographically it is related to MANDIBULAR loop is ansa subclavia.
NERVE, but functionally it is a part of the
GLOS SO PHARYNGEAL NERVE. I. Inferior Cervical Ganglion:
• It is situated in the infratemporal fossa, just • It is often fused with the first thoracic
below the foramen ovale and surrounds the ganglion and is known as cervicothoracic
origin of nerve to medial pterygoid. ganglion.
• The preganglionic parasympathetic fibres • It is also called the stellate ganglion because
from the inferior salivary nucleus are passed it is star-shaped.
through the glossopharyngeal nerve, its • It is formed by fusion of 7th and gth cervical
tympanic branch, the tympanic plexus and the ganglia.
lesser petrosal nerve to reach the ganglion. • Communications are through vertebral,
The post ganglionic fibres reach the parotid subclavian and inferior cervical cardiac
gland through auricu lo temporal nerve. branches.

F. Geniculate Ganglion: 40. Homer's Syndrome:


• The geniculate ganglion is located on the Injury to cervical sympathetic trunk produces Homer's
first bend of the facial nerve. It is a sensory syndrome. It is characterized by
ganglion • Ptosis (i.e., drooping of the upper eyelid)
• The taste fibres present in the nerve are • Miosis (i.e., constriction of pupil)
peripheral processes of pseudounipolar
• Anhydrosis (i.e., loss of sweating on that side of
neurons present in the geniculate ganglion.
the face)
(Note: The three ganglion associated with • Enophthalmos (i.e., retraction of the eyeball)
facial nerve are geniculate, submandibular
• Loss of spinal reflex (i.e., pinching the skin on the
and pterygopalatine)
nape of the neck does not produce dilatation of the
pupil)

SYMPHATHETIC GANGLION 41. SOME KEY FACTS ABOUT CRANIAL NERVES

G. Superior Cervical Ganglion: i) Trigeminal is the largest cranial nerve.


• It is spindle shaped and largest of all the ii) Vagus nerve is with vague or extensive
three ganglia. distribution.
• It is formed by fusion of upper and cervical iii) Olfactory is the smallest cranial nerve.
gang lia.
iv) Trochlear nerve shows largest intracranial course.
• Communications are with 9th, 10th and 12th
cranial nerves and with the external and v) Buccal nerve is the only sensory branch of
recurrent laryngeal nerves. anterior division of mandibular nerve
vi) Smallest branch of trigeminal nerve is
H. Middle Cervical Ganglion: ophthalmic.
• Smallest cervical ganglion. vii) Smallest of three terminal branches of opthalmic
• Formed by fusion of 5th and 6th cervical nerve is lacrimal nerve and frontal nerve is
ganglia. largest terminal branch of opthalmic nerve.
Dental ;lut.,e

viii) The branch of nasociliary that is frequently 42 . BRANCHES OF VAGUS:


absent is posterior eth moidal External and internal laryngeal
Superior
ix) Posterior superior alveolar nerve is a branch of nerves are branches of superior
laryngeal nerve
maxillary nerve in pterygopalatine fossa. laryngeal nerve
x) Infra orbital rnerve (middle & anterior superior • Supplies the cricothyroid
alveolar nerves) is a branch of maxillary nerve in muscles
External
orbital groove. Middle superior alveolar nerve is • Also gives branches to the
laryngeal nerve
present in only in 28% of individuals inferior constrictor and the
xi) Mandibular nerve is largest of three branches of pharyngea l plexus
trigeminal nerve.
Internal • Supplies the mucous
xii) Inferior nerve is larger terminal branch of membrane of the larynx
laryngeal nerve
mandibular nerve. above the level of vocal folds.

xiii) Great er palatine nerve is also known as anterior • Supplies all intrinsic muscles
of the larynx except the
palatine nerve. Lesser palatine nerve includes
cricothyroid.
middle and posterior palatine branches.
xiv) Nervus intermedius is sensory branch of facial • Supplies the mucous
Right recurrent membrane of the larynx
nerve.
laryngeal nerve below the level of vocal folds.
xv) Nervus spinosus is from meningeal branch of
main trunk of mandibular nerve. • Branches to trachea,
oesophagus, inferior
xvi) Greater petrosal nerve is a branch of facia l nerve constrictor and to t he deep
and is parasympathetic to lacrimal glands, glands cardiac plexus.
of nose and pharynx. The ganglion associated
Left recurrent Distribution is similar to right
with greater petrosal nerve is pterygo palatine
laryngeal nerve recurrent laryngeal nerve.
ganglion
xvii) Lesser petrosal nerve is a branch of 43. ACCESSORY NERVE BRANCHES:
glossopharyngeal nerve and it is parasympathetic
to parotid gland through auriculo temporal nerve. i) Accessory nerve has two roots i.e., cranial and
spinal.
The ganglion associated with lesser petrosal nerve
is otic ganglion. ii) The spinal root has independent course and
supplies the sternocleidomastoid and the
xviii) The deep petrosal nerve is a branch of carotid
trapezius muscle.
plexus round t he internal carotid artery. It joins
the greater petrosal nerve to form nerve of iii) The cranial root is accessory to the vagus and is
pterygoid canal. distribut ed through branches of the vagus. It
xix) External petrosal nerve is a branch of the supplies.
sympathetic plexus round the middle meningeal • Muscles of palate
artery • Muscle of larynx
• Muscles of pharynx
• Muscles of heart
, BIOCHEMISTRY
,,...
853
J
~

BIOCHEMISTRY
I. REFERENCE BOOKS TAKEN :
1. TEXTBOOK OF BIOCHEMISTRY By Dr. U. SATYANARAYANA - 3rd & 4th EDITIONS

1. PROTEIN METABOLISM

1. Tertiary structure of proteins is maintained by all c) Purines d) Creatine


except: e) All of the above
a) H2 bond b) Hydrophobic
c) Ionic bond d) Disulphide bond 7. Indole ring is present in:
e) None of the above a) Tryptophan b) Valine
c) Methionine d) Histidine
2. Which of the following is a derived protein:
a) Protamines b) Peptones 8. Which of the fo llowing amino acid is involved in
c) Prolamines d) Lactalbumin gluconeogenesis:
a) Glycine b) Valine
3. Albumins and globulins are _ _ proteins: c) Cysteine d) All
a) Simple b) Derived
c) Conjugated d) Structural 9. Amino acid carrier defect is fo und in:
a) Maple syrup urine disease
4. Quaternary structure of protein is: b) Alkaptonuria
a) The arrangement sequence of amino acids in the c) Phenylketonuria d) Cystinuria
polypeptide chain
b) Inter relation between amino acids in a single 10. The following is false about tryptophan:
polypeptide chain a) Non essential amino acid
c) Inter relation of amino acids in 2 polypeptide b) Involved in serotonin synthesis
chains c) Involved in niacin synthesis
d) The inter relation and arrangement of polypeptides d) Involved in melatonin in synthesis
in a protein with more than 2 polypeptide chains
11. In maple syrup urine disease the amino acids
5. Biuret test is confirmatory test for: excreted in the urine are:
a) Protein b) Fat a} Leucine b) Isoleuci ne
c) Carbohydrate d) None of the above c) Valine d) All of the above
(TNPSC- 99)
6. Glycine is present in: 12. Non essential amino acids are not:
a) Hemoglobin b) Glutathione a) Used by the body

1) E 2) B 3) A 4) D 5) A 6) E 7) A 8) D 9) D 10) A 11) D 12) C


, BIOCHEMISTRY

c) Glucose d) Lipids a) Histidine b) Cystine


(AIPG-01) c) Tyrosine d) Alanine
32. Proteins are absorbed from GIT as: (APPSC-99)
a) Amino acids b) Peptides 42. Ammonia is detoxified in liver to form:
c) Peptones d) All of the above a) Uric acid b) Glutamine
(AIPG-94) c) Creatinine d) Urea
33. Digestion of proteins is initiated by:
a) Amylase b) Sucrase 43. Amino acids excreted in the urine in cystinosis:
c) Chymotrypsin d) Pepsin a) Cystine b) Ornithine
(AIPG-99) c) Arginine d) Lysine
34. Histidine is converted to histamine by: e) All of the above
a) Transamination b) Hydroxylation
c) Decarboxylation d) Reduction 44. Key enzyme in urea synthesis is:
(KAR-02) a) Urease b) Carbamyl synthetase
35. Collagen is rich in: c) Arginase d) Ornithine
a) Glutamate and glycine
b) Alanine and glycine 45. Which is the byproduct of the urea cycle:
c) Praline and glycine d) Glutamate and praline a) Aspartate b) Succinate
(AIPG-2001) c) Ornithine d) Fumarate
36. A mutation that converts an amino acid codon to a
stop codon is a: 46. Major source of ammonia in the kidney is:
a) Nonsense mutation b) Transversion a) Urea b) Aspartate
c) Silent mutation d) Frame shift mutation c) Glutamine d) Glutamate
(COM EDK-04)
37. Which one of the following amino acids is purely 4 7. Urinary protein is detected by:
ketogenic? a) Barfoed test b) Hay's test
a) Praline b) Phenylalanine c) Boiling test d) Ehrlich's test
c) Isoleucine d) Leucine
(COMEDK-04;KAR-2K) 48. Creatine is formed metabolically from:
38. Decarboxylation of which of the following amino a) Tryptophan b) Arginine
acids results in formation of a vasodilator: c) Phenylalanine d) Histidine
a) Valine b) Arginine
c) Histidine d) Glutamic acid 49. Urea is produced by the enzyme:
(APPSC-99;KAR-02) a) Urease b) Uricase
39. All of the following are globular proteins except: c) Arginase d) Glutaminase
a) Pralamines b) Albumin
c) Globulin d) Myosin 50. Which of the following is correct about breakdown
(APPSC-99) of hemoglobin (Hb):
40. A small ca• 2 binding protein that modifies the a) Hb7Heme7bilirubin7Urobilinogen
activity of many enzymes and other proteins in b) Heme7 Hb7 Biliverdin7 Urabilinogen
response to changes of ca+ 2 concentration is known c) Hb7Heme""7Biliverdin""7 Bilirubin""7 Ura bilinogen
as: d) Hb7 Heme""7 Bilirubin""7 Urabilinogen""7 Biliverdin
a) Cycline b) Calmodulin
c) Collagen d) Kinesin 51. Iron is complexed in haemoglobin to:
(KAR- 04) a) Leucine b) Histidine
41. Albinism is a genetic disease that results in c) Isoleucine d) Valine
incomplete metabolism of:

32) A 33) D 34) C 35) C 36) A 37) D 38) C 39) D 40) B 41) C 42) D 43) E 44) B
45) D 46) D 47) C 48) B 49) C 50) C 51) B
Dental ;lut.,e

52. Heme in haemoglobin is: c) Alanine d) Leucine


a) Between Helix C and D
b) Surrounded by non polar environment 62. "Argentaffinoma" is characterized by excess
c) Bonded to E7 histidine excretion of:
d) Protoporphyrin IX a) 5- Hydroxy i ndole acetate
b) 3- Hydroxy phenyl pyruvate
53. At pH 7 the binding of 2,3-DPG to hemoglobin c) Phenyl lactate d) Phenyl acetate
occurs at which site?
a) Sulphydryl group b) Carboxy terminal 63. Which of the following is not a post transcriptional
c) Amino terminal d) Histidine modification of RNA?
a) Splicing b) 5' capping
54. Cytochromes are: c) 3' polyadenylation d) Glycosylation
a) Pyridine nucleotides (AIIMS-06)
b) Riboflavin containing nucleotides 64. The primary role of chaperones is to help in:
c) Metal cont aining flavoproteins a) Protein synthesis b) Protein degradation
d) lron-porphyrin proteins c) Protein denaturation d) Protein folding
(AIIMS- 06)
55. False statement about haemoglobin structure: 65. The protein rich in basic amino acids, which
a) Hb has 2 polypeptide chains functions in the packaging of DNA in chromosomes,
b) Iron is present in ferrous state is:
c) Hb structurally similar to myog lobin a) Histone b) Collagen
d) Ferrous ions are in porphyrin rings c) Hyaluronic acid binding protein
d) Fibri nogen
56. Which of the following is a precursor of (AIIMS- 06)
protoporphyrin: 66. The amino acid residue having an imino side chain
a) Alanine b) Leucine is:
c) Histidine d) Glycine a) Lysine b) Histidine
c) Tyrosine d) Praline
57. Which of the following is not a part of hemoglobin (AIPG-05)
molecule: 6 7. Proteins are linear polymers of amino acids. They
a) Pyrrole rings b) Vinyl groups fold into compact structures. Sometimes, these
c) Histidine d) Ferric ions folded structures associate to form homo-or hetero-
dimers. Which one of the following refers to this
58. The following is not a carrier protein: associated form?
a) Cerruloplasmin b) Transferrin a) Denatured state b) Molecular aggregation
c) Transcobalamine d) Haptoglobulin c) Precipitation d) Quaternary structure
(AIPG- 06)
59. In Hartnup's disease ___is excreted in the urine: 68. The numbers of essential amino acid are:
a) Ornithine b) Glutamine a) 6 b) 8
c) Tryptophan d) Phenylalanine c) 12 d) 16

60. Urea is formed in: 69. Most common non protein nitrogenous fraction of
a) Brain b) Kidney blood:
c) Liver d) Intestine a) Urea b) Uric acid
c) Urobilinogen d) Creatinin
61. Thyroxine and catecholamines are derived from (AP-05)
a) Tyrosine b) Tryptophan

52) D 53) C 54) D 55) A 56) D 57) D 58) D 59) C 60) C 61) A 62) A 63) D 64) D
65) A 66) D 67) D 68) B 69) C
, BIOCHEMISTRY
~

V
m

70. The nitrogen content in 50 gm of a typical dietary c) Mitochondria d) Golgi apparatus


protein is most likely to be (COMED-2012)
a) 5 gm b) 8gm 79. In the body, metabolism of 10 g of protein would
c) 10 gm d) 16 gm produce approximately
(COMEDK-2011) a) 1 Kcal b) 41 Kcal
71. The fastest moving fraction of protein in serum c) 410 Kcal d) 4100 Kcal
when subjected to paper electrophoresis is (COMED- 2012)
a) Albumin b) Alpha 1 Globulin 80. All are genetic amino acid deficiency diseases
c) Beta Globulin d) Gamma Globulin except:
(COMEDK-06) a) Phenylketonuria b) Alkaptonuria
72. The class of amino acids that contains only non c) Homocystinuria d) Galactosemia
essential amino acids is (NEET -2013)
a) Acidic b) Basic 81. Both ketogenic and glucogenic amino acids as
c) Aromatic d) Branched chain a) Isoleucine b) Leucine
(COM EDK-07) c) Argenine d) Glycine
73. The daily requirement of protein for the adults is (AIIMS MAY-13)
a) 6 gms b) 60 gms 82. The amino acid which undergoes most significant
c) 120 gms d) 250 gms hepatic oxidative deamination which is used for
(AP- 08) urea synthesis is
74. All are true about glutathione except? a) Glutamine b) Glutamate
a) It is a tripeptide c) Aspartate d) Arginine
b) It converts hemoglobin to methemoglobin (COMEDK-14)
c) It conjugates xenobiotics 83. Mucin is a?
d) It scavenges free radicals and superoxide ions a) Nucleoprotei n b) Glycoprotein
(AIPG-09, 10) c) Phosphoprotein d) Chromoprotein
75. Colloidal osmotic pressure of plasma is by - (PGI JUNE-2012)
a) Albumin b) Fibrinogen 84. Source of nitrogen in the urea cycle are
c) Globulin d) Prothrombin a) Arginine & Ammonia b) Aspartate & Ammonia
(MCET-10) c) Uric acid & Ammonia d) Glutamate & Ammonia
76. Glutamine replaced by valine in sickle cell anaemia (APPG-2015)
is characterized by
a) Non sense mutation of beta chain
b) Missense mutation of beta chain
c) Degradation of beta chain
d) Deletion of beta chain
(AIPG- 10)
77. Alkaptonuria an inherited metabolic disorder is
due to the deficiency of
a) Homogentisate oxidase
b) Cystathionase
c) Pheylalanine hydroxylase
d) Tyrosine transaminase
(KCET- 2012)
78. Mannose 6 phosphate containing freshly
synthesized proteins are directed to
a) Nucleus b) Lysosomes

70) B 71) A 72) A 73) B 74) B 75) A 76) B 77) A 78) B 79) B 80) D 81) A 82) B
83) B 84) B
Dental ;lut.,e

1. PROTEIN METABOLISM - ANSWERS


1. 'E' [Satyanarayana 3rd & 4th eds 58] In Option 'D' Cystinuria, the carrier system in kidney,
The three dimensional arrangement of protein structure for the reabsorption of amino acids, namely Cysteine,
is referred to as tertiary structure. Hydrogen bonds, Ornithine, Arginine, and Lysine (COAL) becomes
ionic interactions, disulfide bonds and hydrophobic defective leading to the excretion of all these four
interactions contribute to tertiary structure of proteins. amino acids in urine

2, 'B' [Satyanarayana 3rd & 4th eds 65] Option '/:\ Maple syrup urine disease is a disorder of
Classification of proteins branched chain amino acids (valine, leucine and
isoleucine). The disease is due to defect in the enzyme
Simple Eg.: Albumin, Globulin "Branched chain a-ketoacid dehydrogenase". So this
Conjugated Eg.: Nucleoproteins, Glycoproteins, disease is also known as "branched chain ketonuria".
Blood group antigens.
Options 'B & C' Phenylketonun"a and Alkaptonuria are
Derived Eg.: Peptones Coagulated proteins
due to disorders of tyrosine metabolism. Alkaptonuria
is due to defect in the enzyme "homogentisate
3. 'A' [Satyanarayana 3rd & 4th eds 64] oxidase''. The urine of alkaptonuric patients resembles
coke in colour.
4. 'D' [Satyanarayana 3rd & 4th eds 58)
• Primary Structure: Linear sequence of amino acids 10. 'A' [Satyanarayana 3rd & 4th eds 48, 354 -58)
forming the backbone of proteins (polypeptides). Nutritional classification of aminoacids.
• Secondary Structure: The spatial arrangement of i) Essential or indispensable amino acids:-
protein by twisting the polypeptide chain. Argini ne, valine, histidine, isoleucine, leucine
• Tertiary Structure: Most of the proteins are lysine, methionine, phenylalanine, threonine,
composed of single polypeptide chains but some tryptophan.
proteins contain two or more polypeptide chains
and are termed as oligomers. Oligomers possess Arginine and histidine can be partly synthesized
quaternary structure. by adults, hence they a re considered as semi
essential amino acids.
5. 'A' [Satyanarayana .3rd & 4th eds 61]
ii) Non-essential aminoacids: -
6. 'E' [Satyanarayana 3rd & 4th eds 342 Fig 15.13) Glycine, alanine, serine, cysteine, aspartate,
asparagine, glutamate, glutamine, tyrosine and
7. 'A' [Satyanarayana 3rd & 4th eds 354) proline.
Amino acid with indole ring is tryptophan. Amino acid
with imidazole ring is histidine. 11, 'D' [Satyanarayana 3rd & 4th eds 365)

8. 'D' [Satyanarayana 3rd & 4th eds 373) 12. 'C' [Satyanarayana 3rd & 4th eds 48]

. Glucogenic & •
Ketogemc . Glucogemc 13. 'C' [Satyanarayana 3rd & 4th eds 47)
Ketogen1c
Phenylalanine and tyrosine, both are aromatic amino
• Leucine • Tyrosine Remaining acids. The on ly function of phenylalanine is its
• Lysine • Tryptophan amino acids conversion into tyrosine.
• Phenylalanine
14, 'C' [Satyanarayana 3rd & 4th eds 295)
• Isoleucine
Acetone, acetoacetate, and ~-hydroxy butyrate are
known as ketone bodies. They are major fuel source
9. 'D' [Satyanarayana 3rd & 4th eds 361) of brain during prolonged starvation.
, BIOCHEMISTRY

55. 'A' [Satyanarayana 3rd & 4th eds 196] of the two strands of DNA serves as a template and
Hemoglobin contains 4 polypeptide chains. The adult produces working copies of RNA molecules.
hemoglobin is made up of two a -chains and two
~-chains while feta l hemoglobin is made up of two The first formed inactive product in transcription
a-chains and two )'-chains. is referred to as primary transcript. The primary
transcripts undergo certain alterations called post-
56. ' D' [Satyanarayana 3rd & 4th eds 210] transcriptional modifications, to produce functionally
Glycine, and succinyl CoA, both are the starting active RNA molecules. A enzyme called DNA dependent
materials for porphyrin synthesis. RNA polymerase or simply RNA polymerase synthesize
all the RNAs in prokaryotes.
57. ' D' [Satyanarayana 3rd & 4th eds 197]
Ferric (Fe+3 ) ions are seen in methemoglobin. In eukaryotes the nuclei possess three types of RNA
polymerases i.e., RNA polymerase I, NA polymerase
58. 'D' [Satyanarayana 3rd & 4th eds 185 -86] II, and RNA polymerase III. The promoter sites are
Haptoglobin binds with plasma free hemoglobin and Hogness box or TATA box and a second site called
prevents its excretion. CAAT Box. The Hogness box or TATA box is identical to
Pribnow box of prokaryotes.
59. '( ' [Check Explanation Below]
The patients with Hartnup's disease are characterized The primary mRNA t ranscript produced by RNA
by inability to absorb amino acids like tryptophan and polymerase II in eukaryotes referred as hnRNA or
may show typical symptoms of pellagra. heterogenous nuclear RNA. It undergoes post-
transcriptional modifications like terminal base
60. 'C' [Satyanarayana 3rd & 4th eds 337] additions, base modifications, splicing, etc.
Urea is mainly fo rmed in liver and to some extent in
brain. The various changes before the production of functiona l
mRNA are 5' capping, poly-adenylation tail, splicing
61. 'A' [Satyanarayana 3rd & 4th eds 346 Fig 15.17] and removal of introns. These post-translational
Derivates of tyrosine are modifications occur in nucleus. Faulty splicing can
• Melanin cause diseases. Example is ~-Thalassemia.

• Epinephrine 64. ' D' [Satyanarayana 3rd & 4th eds 560, 561]
• Nor epinephrin e } Catecholam;aes Chaperones are heat shock proteins that facilitate
• Dopamine and favo ur the interactions on the polypeptide surfaces
to give the specific conformation of a protein .
• Thyroid hormones Chaperones help the protein to attain compact and
biologically active conformation.
62. 'A' [Satyanarayana 3rd & 4th eds 356]
"Argentaffinomas" or malignant carcinoid syndrome Chaperons are divided into two groups i.e., HSP70
is due to uncontrolled growth of serotonin secreting (Heat Shock Protein 70) and Chaperonin system.
cells of GIT. Normally 1% of 'tryptophan' is utilized for
serotonin synthesis. In case of carcinoid syndrome very The failure of a protein to fold properly leads to its
high amount of tryptophan is diverted for serotonin rapid degradation. Ex. Cystic fibrosis. Sometimes these
production. This results in impaired synthesis of misfolded proteins accumulate in cells and cause
NAO+ and NADP+ and the patient develops the neurological diseases like Alzheimer's disease. Those
symptoms of pellagra due to niacin deficiency. misfolded proteins are called prions.

63. 'D' [Satyanarayana 3rd & 4th eds 546-48] 65. 'K [Satyanarayana 3rd & 4th eds 79]
Transcription is a process in which RNA is synthesized In Eukaryotic cells, the DNA is associated with various
from DNA. Thus, the genetic information stored in proteins t o form chromatin, which is finally organized
DNA is expressed through RNA. In transcription one to chromosomes. The DNA double helix is wrapped
, BIOCHEMISTRY

18. Enzymes concerned with the citric acid cycle are 27. Which of the following is a non-reducing sugar:
found in the: a) Glucose b) Maltose
a) Nucleus b) Ribosomes c) Lactose d) Sucrose
c) Mitochondria d) Nonparticulate cytoplasm (AP-98)
(AIPG-03) 28. Which one of the following is a monosaccharide
19. Kreb' s cycle occurs in _ _ _ _ _ _ conditions: a) Maltose b) Sucrose
a) Aerobic b) Anaerobic c) Fructose d) Starch
c) Microaerophilic d) Aerobic and anaerobic (AIPG- 94)
(AIIMS- 91) 29. The end product of glycolysis under anaerobic
20. In TCA, substrate level phosphorylation takes place conditions is:
in: a) Lactic acid b) Pyruvic acid
a) Alpha Ketoglutarate to succinyl CoA c) Acetoacetic acid d) Oxaloacetic acid
b) Succinyl CoA to Succinate (AIPG-98)
c) Succinate to fumarate 30. In prolonged starvation the main energy source of
d) Oxaloacetate to citrate brain is
(AIPG- 02) a) Glucose b) Ketone bodies
21. Which acid is formed in the citric acid cycle? c) Fructose d) Fatty acids
a) Oxaloacetic acid b) Glutamic acid (COMEDK -2013)
c) Nitric acid d) None of the above 31. The enzyme involved in the first committed step of
(AIIMS-89) glycolysis is:
22. Cane sugar is: a) Phosphofructokinase b) Glucose-6-phosphatase
a) Glucose b) Sucrose c) Hexokinase d) Enolase
c) Fructose d) Maltose (AIPG, 99)
(KAR- 97) 32. Renal threshold for glucose is:
23. The main enzyme responsible for activation of a) 80 mg% b) 100 mg%
xenobiotics is (detoxification) c) 180 mg/dl d) 200 mg%
a) Cytochrome P-450 (TNPSC- 99)
b) Glutathione S-transferase 33. Which one of the following is correctly matched?
c) NADPH cytochrome P-450-reductase a) Isocitrate to oxalo succin ate -1 ATP is formed
d) Glucuronyl transferase b) Succinyl CoA to succinate -1 ATP is formed
(KAR- 99) c) Succinate to fumarate -1 ATP is formed
24. The conversion of glucose-6-P to fructose-6-P is an d) Malate to oxaloacetate -1 ATP is formed
example of which of the following reactions: (TNPSC- 99)
a) Phosphate transfer b) lsomerisation 34. In TCA cycle or tricarboxylic acid cycle, which is
c) Dehydration d) Aldol cleavage first formed:
(AIIMS- 2K) a) lsoci trate b) Citrate
25. What high energy phosphate compound is formed c) Succinate d) Fumarate
in the citric acid cycle through substrate level
phosphorylation: 35. In TCA cycle substrate level phosphorylation occurs
a) ATP b) TTP at:
c) ITP d) GTP a) Succinate dehydrogenase
(AIIMS- 2K) b) Malonate reduction
26. Which of the following is abnormal constituent of c) Thiokinase d) None of the above
urine:
a) Glucose b) Creatine 36. Kreb's cycle does not occur in:
c) Urea d) None of the above a) Muscle b) RBC
(AP- 98) c) Heart d) All of the above

18) C 19) A 20) B 21) A 22) B 23) A 24) B 25) D 26) A 27) D 28) C 29) A 30) B
31) C 32) C 33) B 34) B 35) C 36) B
Dental ;lut.,e

37. In TCA cycle, citrate is converted in to, after losing d) Myophosphorylase


a molecule of H20
a) Isocitrate b) Cisaconitate 46. 1 molecule of glucose forms molecules of
c) Oxaloacetate d) Glutarate pyruvate
a) 1 b) 2
38. Which one of the following is the correct sequential c) 3 d) 5
order in which the given enzymes of kreb's cycle are
formed after a molecule of acetyl CoA: 47. In which type of glycogen storage disease is hyper
a) Citrate, Oxaloacetate, Ketoglutarate uricemia a feature?
b) Ketoglutarate, Oxaloacetate, Citrate a) I b) II
c) Cit rate, Ketoglutarate, Oxaloacet ate c) III d) IV
d) Oxaloacetate, Ketoglutarate, Citrate
48. Step in HMP pathway requiring TPP:
39. Which metabolite of TCA cycle is used in a) G6 PD
detoxification of ammonia in brain: b) 6 Phosphogluconate dehydrogenase
a) Alpha ketog lutarate b) Ornithine c) Transketolase d) Transaldolase
c) Oxaloacetate d) Glycine
49. Galactosemia commonly is due to deficiency of:
40. Inhibition of glycolysis by 0 2 is known as: a) Galactose-1-phosphate uridyl transferase
a) Muni effect b) Pasteur effect b) Galactose-1-phosphatase
c) Hill reaction d) Gluconeogenesis c) Glucose-1-phosphatase
d) Glucose-6-phosphatase
41. Phosphofructokinase is the key enzyme of:
a) Glycolysis b) Gluconeogenesis 50. Sites where HMP shunts can occur include:
c) Beta oxidation d) TCA cycle a) Liver b) WBC
c) Lactating mammary gland
42. In glycolysis ATP is produced by the following d) Testes e) All
enzyme:
a) Hexokinase b) Phosphog lycerate kinase 51. Blood glucose Levels cannot be augmented by
c) Enolase d) Phosphohexose isomerase mobilization of muscle glycogen due to lack of:
a) G-6-P dehydrogenase
43. An enzyme not involved in glycolysis is: b) G-6-phosphatase
a) Enolase b) Phosphoglycero mutase c) Aldolase d) Glucokinase
c) Aldolase
d) Glycerophosphate dehydrogenase 52. Glucose 6 phosphatase deficiency is seen in:
a) Pomper's disease b) Von Gierke's disease
44. The main pathways of metabolism in brain are: c) McArdles syndrome d) Downs syndrome
a) Glycolysis and Citric acid cycle
b) Glycogenolysis and Gluconeogenesis 53. All are true regardfog glucose-6-phosphate
c) Embden-Meyerhof pathway and H.M.P shunt deficiency except:
pathway a) Hyperuricemia b) Hyperglycemia
d) Glycogenolysis and Citric and cycle c) Defective cori cycle
d) Increased mobilization of glycogen from liver
45. McArdles disease is due to the deficiency of:
a) Glucose 1 phosphatase 54. HMP shunt is of great importance in cellular
b) Glucose 1,6 diphosphatase metabolism because it produces:
c) Glucose 6 phosphatase a) ATP b) ADP

37) B 38) C 39) A 40) B 41) A 42) B 43) D 44) A 45) D 46) B 47) A 48) C 49) A
50) E 51) B 52) B 53) B 54) D
, BIOCHEMISTRY
~

V
367

c) Acetyl CoA d) NADPH 64. Dietary fibre is rich in


a) Starch b) Cellulose
55. Which of the following is not a product of HMP c) Collagen d) Inulin
shunt: (KCET-07)
a) NADPH b) D fructose 6 phosphate 65. Increase in pyruvate and lactate is seen in which of
c) D sedoheptulose 5 phosphate the following deficiency?
d) D glyceraldehyde 3 phosphate a) Thiamine b) Pyridoxine
c) Niacin d) Vitamin C
56. NADPH is generated by the action of: (AIPG- 07)
a) Glucose 6 phosphate dehydrogenase 66. Type II glycogen storage disorder is due to
b) Glucose 1 phosphate dehydrogenase deficiency of:
c) Glucose 1, 6 diphosphate dehydrogenase a) alpha-Glucosidase b) alpha-galactosidase
d) All of the above c) Muscle phosphorylase
d) Acid Lipase
57. All these reactions take place inside the (MCET-07)
mitochondria except: 67. Which one of the following enzymes provides a link
a) EMP pathway b) Krebs cycle between glycolysis and the citric acid cycle?
c) Urea cycle d) Electron transfer a) Lactate dehydrogenase
b) Pyruvate Kinase
58. Number of ATP molecules generated in the c) Citrate synthase d) Pyruvate dehydrogenase
conversion of glycogen to lactate is: (COMEDK- 10)
a) 2 b) 36 68. Most lipogenic
c) 38 d) 14 a) Fructose b) Glucose
c) Galactose d) Ribose
59. One molecule of acetyl Co-A gives rise to (AIPG-10)
ATP molecules: 69. The uptake of glucose by the liver increases
a) 2 b) 8 following a carbohydrate meal because
c) 12 d) 32 a) There is increase in phosphorylation of glucose by
glucokinase.
60. The monsaccharide glucose is best described by b) GLUT-2 is stimulated by insulin.
which one of the following statements? c) Glucokinase has a low Km for glucose.
a) It usually exists in the furanose form d) Hexokinase in liver has a high affinity for glucose
b) It is a ketose (COMEDK- 09)
c) It possessesan anomeric C-2 carbon atom 70. Insulin increases the following pathways in liver
d) It formspart of the disaccharide sucrose EXCEPT-
(COMEDK- 05) a) Fatty acid synthesis b) Glycogen synthesis
61. Which is not a oligosaccharide sugar? c) Protein synthesis d) Glucose synthesis
a) Galactose b) Lactose (KCET-09)
c) Maltose d) Sucrose 71. The citric acid cycle is the final pathway for
oxidation of
62. Fructose intolerance is to a) Enzymes b) Vitamins
a) Fructose only b) Fructose and glucose c) Minerals d) None of the above
c) Sucrose on ly d) Fructose and sucrose (BHU- 2012)
72. Acetyl CoA can be converted in to all of the following
63. Glycogen breakdown leads to formation of: except:
a) Glucose b) Lactic acid a) Glucose b) Fatty acid
c) Glucose & Lactic acid d) Glycoprotein

55) C 56) A 57) A 58) A 59) C 60) D 61) A 62) D 63) C 64) B 65) A 66) A 67) D
68) A 69) A 70) D 71) D 72) A
c) Cholesterol d) Ketone bodies
(NEET -2013)
73. A patient with hereditary fructose intolerance is
deficient in which of the following enzymes?
a) Aldo lase b) Fructoki nase
c) Triokinase d) All of above
(COMEDK -2013)
7 4. Which is not a monosaccharide?
a) Glucose b) Galactose
c) Maltose d) Fructose
(AIIMS NOV-13)
7 5. Substance for gluconeogenesis is
a) Glycogen b) Acetyl CoA
c) Glycerol d) Leucine
(MCET-14)
7 6. Xylitol is a
a) Natural sweet amino acid
b) Synthetic sweet amino acid
c) Natural five carbon sugar
d) Synthetic five carbon sugar
(MCET-14)
77. Glucose transporters present in the f3 cells of the
Islets of Langherhans is
a) GLUTl b) GLUT2
c) GLUT3 d) GLUT4
(COMEDK-14)
78. Prolonged carbohydrate deficiency leads to?
a) Metabolic alkalosis b) Ketoacidosis
c) Vitamin C deficiency d) Respiratory acidosis
(PG! JUNE-2012)
79. The glucose transporter which is stimulated by
insulin is located in
a) Brain and retina
b) Liver and beta cells of pancreas
c) Skeletal muscle and adipose tissue
d) RBCs and intestine
(KERALA- 2015)

73) A 74) C 75) C 76) C 77) B 78) B 79) C


Dental ;lut.,e

48. 'C' [Satyanarayana 3rd & 4th eds 273] 61. 'A' [Satyanarayana 3rd & 4th eds 11]
Transketolase is dependent on t hia mine pyrophosphate Glucose, galactose, mannose and fructose are hexose
(TPP), which is a coenzyme of thiamine. Transketolase monosaccharides. Options B, C, D, i.e., sucrose, maltose
levels in blood are useful in monitoring thiamine and lactose are disaccharides or oligosaccharides.
levels in blood.
62. 'D' [Satyanarayana 3rd & 4th eds 280]
49. 'A' [Satyanarayana 3rd & 4th eds 277] Hereditary fructose intolerance is due to absence of
enzyme aldolase B. It causes intracellular accumulation
50. 'E' [Satyanarayana .3rd & 4th eds 271] of fructose 1-phosphate, severe hypoglycemia,
vomiting and jaundice. Fructose-1-phosphate inhibits
51. 'B' [Satyanarayana 3rd & 4th eds 261, 266] liver phosphorylase and blocks glycogenolysis leading
Glucose 6 - phosphatase cleaves glucose 6- phosphates to hypoglycemia.
to glucose. This enzyme is absent in muscle and brain;
hence free glucose cannot be produced from glucose- 63. 'C' [Satyanarayana 3rd & 4th eds 265, 245]
6-phosphate in these tissues. The degradation of stored glycogen in liver and muscle
constitutes glycogenolysis. Degradation of glycogen in
52. 'B' [Satyanarayana 3rd & 4th eds 145] liver, kidney and intestine that contain glucose-6-
phosphatase enzyme produce free glucose. But the
53. 'B' [Satyanarayana 3rd & 4th eds 269] enzyme glucose-6-phosphatase is absent in muscle
and brain . Hence free glucose cannot be produced
54. 'D' [Satyanarayana 3rd & 4th eds 274] from glucose-6-phosphate in these tissues. Here, the
HMP shunt produces two important products i.e., glucose-6-phosphate produced by glycogenolysis will
pentoses and NADPH. Pentoses are useful for synthesis be used fo r glycolysis. And rem ember that in glycolysis,
of nucleic acids and nucleotides. the Glucose-6-phosphate is converted to pyruvate or
lactate.
NADPH is required for biosynthesis of fatty acids
and steroids and is used for synthesis of amino acids 64. 'B' (Satyanarayana 3rd & 4th eds 508]
involving the enzyme glutamate dehydrogenase. It Cellulose occurs exclusively in plants and is the
preserves the integrity of the RBC membrane most abundant organic substance in plant kingdom.
It is a predominant constituent of plant cell wall.
55. 'C' [Satyanarayana 3rd & 4th eds 273] Cellulose is totally absent in animal body. The complex
Sedohe ptulose 7- phosphate is formed in HMP shunt. carbohydrates that are not digested by the human
enzymes are collectively referred to as dietary fiber.
56. 'A' (Satyanarayana 3rd & 4th eds 274] These include cellulose, hemicellulose, pectin, lignin,
gums and mucilage. However, some of these fibers
57. 'A' (Satyanarayana 3rd & 4th eds 245] are digestible by the enzymes of intestinal bacteria
EM pathway or glycolysis takes place in cytosomal (Eg.: Pectins, gums)
fraction of the cell.
65. 'A' [Satyanarayana 3rd & 4th eds 135, 252)
58. 'A' (Satyanarayana 3rd & 4th eds 246] The pyruvate produced in glycolysis is converted to
acetyl CoA by oxidative decarboxylation . This reaction
59. 'C' (Satyanarayana 3rd & 4th eds 256] is catalysed by a multienzyme complex known as
Pyruvate dehydrogenase complex (PDH). The enzyme
60. 'D' [Satyanarayana 3rd & 4th eds 13, 19] PDH requires five cofactors namely TPP (Thiamine Pyro
Glucose is aldose that usually exists in six membered Phosphate), Lipoamide, FAD, Coenzyme a and NAD+.
pyranose form (furanose is five membered form). The a
and 13 cyclic forms of D-glucose differ from each other 66. 'A' (Satyanarayana 3rd & 4th eds 269]
in the configuration only around C1 known as anomeric Pompes disease or Type II glycogen storage disorder is
carbon. Sucrose is made up of cx-D-glucose and 13-D due to defi ciency of lysosomal alpha-1,4 glucosidase
fructose. (acid maltase).
Dental ;lut.,e

of fats and increases the rate of removal of fatty


acids from adipose tissues. Also increased secretion
of glucagon (from pancreas), glucocorticoids (from
adrenal cortex) and decreased secretion of insulin
further enhances the removal of fatty acids. These
fatty acids are used by the peripheral tissues as source
of energy and most of them are converted to ketone
bodies in liver resulting in ketoacidosis.

79. 'C' [Satyanarayana 4th ed 245)


Glucose does not enter the cells for metabolism by
simple diffusion. Two specific transport systems are
recognized for the entry of glucose into the cells.

i) Insulin independent transport system of glucose.


This occurs in hepatocytes, erythrocytes and brain.
ii) Insulin dependent transport system. This occurs in
muscle and adipose tissue.
Dental ;lut.,e

3. LIPID METABOLISM - ANSWERS


1. 'C' [Satyanarayana 3rd & 4th eds 288]
Contribute to 60 - 70% of total
Fatty acids in the body are mostly oxidized by Carbohydrates
caloric requirement of the body
~-oxidation. It occurs in mitochondrial matrix. Fatty
acids are oxidized by most of tissues in body except in Proteins (Body Contribute to 10 - 15% of total
brain, erythrocytes and adrenal medulla. building foods) body energy
• Contribute 15 - 30% of body
2. 'B' [Satyanarayana 3rd & 4th eds 31] energy requirements.
Li noleic and Linolenic acids are essential since the • To guard against the
body cannot synthesize them. Arachidonic acid
development of atherosclerosis
beco mes essential, if its precursor linoleic acid is Lipids
and hyper cholesterolemia,
not provided in the diet. So it is considered as semi the fat supply should not
essential. All essential and semi essential fatty acids be more than 30% of total
are polyunsaturated.
dietary calories.

3. 'A' [Satyanarayana 3rd & 4th eds 31]


7. 'B' [Satyanarayana 3rd & 4th eds 29, 6 50]
Cell membranes are composed of lipids, prot eins and
4. 'C' [Satyanarayana 3rd & 4th eds 317]
carbohydrates. The major lipids that make up the
Classification of Lipoproteins cell membrane are phospholipids. Most important
• Transports exogenous phospholipid component of cell membrane is
triacylglycerol to various cholesterol.
Chylomicrons tissues
8. 'C' [Satyanarayana 3rd & 4th eds 315]
• Contains 99% lipid and 1%
protein
9. 'B' [Satyanarayana 3rd & 4th eds 33]
Very low Transports endogenously "Saponification number" is the number of milligrams
density lipo synthesized triacylglycerols to of KOH required to hydrolyse one gram of fat or oil.
proteins (VLD L) various tissues The value is higher for the fats containing short chain
• Transports cholesterol from fatty acids.
tissues to liver
HDL 10. 'C' [Satyanarayana 3rd & 4th eds 28, 29]
• Plasma HDL is inversely
correlated with CH D. Classification of Lipids:
A} Simple Lipids
• Contains high concentration Eg.: Triacylglycerols
of cholest erol.
• Transport cholesterol from B) Complex Lipids
Low density
liver to other tissues. a) Phospholipids
proteins (LDL)
• Risk of atherosclerosis is • Glycero phospholipids ( contains glycerol):
directly related to LDL Eg.: Lecithin, Cephalin
concentration. • Sphingo phospholipids (contains sphingosine):
Eg.: sphingomyelin
5. 'A' [Satyanarayana 3rd & 4th eds 309, 650]
Cholesterol is most important phospholipid component b) Glycolipids- OR Glycosphingolipids
of cell membrane. Eg.: Cerebrosides, Gang liosides

6. 'B' [Satyanarayana 3rd & 4th eds 510] c) Lipoproteins:


Eg.: HDL, LD L, VLDL, and Chylomicrons.
, BIOCHEMISTRY
~

V
379

11. 'A' [Satyanarayana 3rd & 4th eds 645] 20. 'C' [Satyanarayana 3rd & 4th eds 288]
Arachidonic acid, an unsaturated fatty acid serves as a The inner mitochondrial membrane is impermeable
precursor for synthesis of prostaglandins. to fatty acids. A specialized carnitine carrier system
helps in transport of fatty acids from cytosol to the
12. 'A' [Satyanarayana 3rd & 4th eds 30, 31] mitochondria.
Palm itoleic acid, oleic acid, linoleic acid, linolenic acid
and arachidonic acid are the five unsatured fatty acids. 21. 'N [Satyanarayana 3rd & 4th eds 294]
Acetone, acetoacetate and ~-hydroxybutyrate are
Among them palmitoleic acid and oleic acid are mono known as ketone bodies. Only the first two are the true
unsaturated fatty acids. Linoleic acid, linolenic acid ketones, while ~-hydroxybutyrate does not possess a
and arachidonic acid are essential polyunsaturated keto group.
fatty acids.
The synthesis of ketone bodies occurs in liver in the
13. 'B' [Satyanarayana 3rd & 4th eds 29] mitochondrial matrix. Acetyl CoA, formed by oxidation
of fatty acids, pyruvate and some other amino acids are
14. 'C' [Satyanarayana 3rd & 4th eds 3 7] the precursors for ketone bodies. HMG CoA synthase is
Galactocerebroside and Glucocerebroside are the the regulatory enzyme in synthesis of ketone bodies.
important cerebrosides.
22. 'N [Satyanarayana 3rd & 4th eds 294]
"Gaucher's" disease is characterized by accumulation
of glucocerebroside in liver and spleen due to a 23. ' B' [Satyanarayana 3rd & 4th eds 293]
defect in enzyme f3 - glucosidase.
24. 'C' [Satyanarayana 3rd & 4th eds 294]
15. 'C' [Satyanarayana 3rd & 4th eds 307] Acetyl CoA formed by oxidation of fatty acids is the
Niemann - Pick disease is due to a defect in enzyme precursor for ketone bodies.
sphingomyelinase, (AIPG-11) which results in
accumulation of sphinomyelins in liver, spleen and 25. ' B' [Satyanarayana 3rd & 4th eds 295]
CNS. Thiophorase, an enzyme required for metabolism
of ketone bodies is absent in liver, hence the liver
16. 'A' [Satyanarayana 3rd & 4th eds 322] cannot utilize ketone bodies. Also the tissues like
erythrocytes, which lack the mitochondria, cannot
17. 'C' [Satyanarayana 3rd & 4th eds 298] utilize ketone bodies.
During fatty acid synthesis, acetyl CoA molecule is
carboxylated to malonyl CoA carboxylase. Acetyl 26. 'D' [Satyanarayana 3rd & 4th eds 296]
CoA carboxylase is a regulatory enzyme in fatty acid
synthesis. Glutamate rapidly undergoes oxidative 27. ' D' [Satyanarayana 3rd & 4th eds 309-14]
deamination to liberate ammonia. Derivatives of cholesterol
• Steroid hormones
18. 'C' [Satyanarayana 3rd & 4th eds 292]
• Primary bile acids
(glycocholic acid, taurocholic acid)
19. 'C' [Satyanarayana 3rd & 4th eds 297]
• Secondary bile acids
Functions of NADPH
(deoxycholic acid, lithocholic acid)
• Synthesis of fatty acid, steroids and certain amino
acids. • Vitamin - D

• Protection from H20 2 through anti oxidant reactions


involving NADPH
• Detoxification of drugs
• Preservation of integrity of R.B.C membrane
, BIOCHEMISTRY
~

V
381

47. ' B' [Satyanarayana 3rd & 4th eds 29] 54. 'C' [Check Explanation Below]
Functions of Lipids: Refer Q. No.2 for essential poly unsaturated fatty acids
1. Acts as a fuel reserve of the body (triacylglycerols) (PUFA)
2. Lipids are the constituents of membrane structure
and regulate the membrane permeability The dietary PUFA is converted to arachidonic acid and
Docosahexanoic acid (DHA). DHA is particularly needed
(phospholipids and cholesterol)
for development of brain and retina. It is synthesized
3. Acts as source of fat soluble vitamins (A,D,E and K) endogenously from linolenic acid by brain in small
4. Lipids are important as cellular metabolic amounts. Incorporation of DHA in diet is essential to
regulators (steroid hormones and prostaglandins) infants.
5. Protect internal organs, serve as insulating
materials and give shape and smooth appearance Important source is human milk. In vegetarian
to the body. mothers, the milk DHA is less than 0.1 % and in women
consuming diet rich in fish the DHA is 0.8% in milk.
48. 'C' [Satyanarayana 3rd & 4th eds 297]
55. 'K [Check Q.No.15]
49. 'D' [Satyanarayana 3rd & 4th eds 274]
• The main source of reducing equivalents (NADPH) 56. ' B' [Check Explanation Below]
is the Pentose Phosphate Pathway (PPP) or HMP Triglycerides or triacylglycerol's (TAG) are the most
shunt. predominant storage form of energy. The pancreatic
(main), gastric and lingual lipases help in digesting
• The oxidative reactions of PPP are the chief source
TAG. One TAG produces one MAG (Monoacyl glycerol)
of the hydrogen required for the reductive synthesis
and two fatty acids. Absorption of TAG from gut is
of fatty acids.
mainly in the form of MAG.
50. 'A' [Satyanarayana 3rd & 4th eds 286]
In extreme cases, humans can fast and survive for 60- 5 7. 'K [Satyanarayana 4th ed 2 94]
Refer Q. No.21
90 days.

58. 'B' [Satyanarayana 4th ed 36]


51. ' D' [Satyanarayan 3rd ed 381]
Functions of Phospholipids:
Acetyl CoA is the key and common metabolite produced
1. Essential for the synthesis of different lipoproteins,
from carbohydrate, lipids and amino acids. This enters
and thus participate in the transport of lipids
citric acid cycle and gets oxidized to CO2. Thus, citric
2. Accumulation of fat in liver (fatty liver) can be
acid cycle is the fina l common metabolic pathway for
prevented, hence they are called as lipotropic
the oxidation of all food stuffs.
factors.
3. They are the structural components of membranes
52. 'C' [Satyanarayana 3rd & 4th eds 136]
and regulate membrane permeability.
In thiamine deficiency there is impaired conversion of
4. Phospholipids (lecithin, cephalin and cardiolipin)
pyruvate to acetyl CoA resulting in accumulation of
in the mitochondria are responsible for cellu lar
lactate and pyruvate. Pyruvate may enter brain and its
respiration.
accumulation may be lead to poly neuritis.
5. Act as surfactants and its deficiency in infants
cause respiratory distress syndrome
53. 'D' [Satyanarayana 3rd & 4th eds 320/ Lippincott
6. Participate in absorption of fat
5th ed 234]
7. Arachidonic acid liberated from phospholipids,
Functions of HDL
severs as a precursor for the synthesis of eicosanoids
• Reservoir of apolipo-proteins
(Prostaglandins, Prost acyclins, thromboxanes)
• Transport of unesterified cholesterol from peripheral 8. Reduce cholesterol, as they participate in reverse
tissues to liver as cholesteryl esters. cholesterol transport.
• Esterification of cholesterol
• Key component in cholesterol homeostasis.
Dental ;lut.,e

4. VITAMINS
1. Vitamin also acting as hormone: 10. The following vitamin is important in non-oxidative
a) Vitamin D b) Vitamin A decarboxylation, transamination and trans-
c) Vitamin Bl d) Vitamin C sulfuration reactions:
a) Riboflavin b) Thiamine
2. Two vitamins whose derivatives are involved in c) Pyridoxine d) Pantothenic acid
transformation of serine to glycine are:
a) B6 ad B12 b) B12 and nicotinamide 11. Which vitamin is the most indispensable during
c) Folic acid and B6 d) Folic acid and B12 mitosis:
a) Folic acid b) Pa ntothenic acid
3. Which vitamin is synthesized by intestinal bacteria? c) Ascorbic acid d) Aspartic acid
a) Vit B b) Vit A
c) Vit D d) Vit K 12. Most of vitamin 8 12 in the body is stored as:
a) Methyl B12 b) Hydroxy B12
4. Which vitamin is related to a co-factor in glycine c) Cyano cobalamine d) None of the above
metabolism is:
a) Vit E. b) Folic acid 13. Beri-beri is caused due to the deficiency of:
c) Thiamine d) Cobalamine a) Thiamine b) Pyridoxine
c) Ascorbic acid d) Riboflavin
5. Of prime importance in the structure of flavoproteins (MAN-2K, 94)
is: 14. Which of the following is not true of Vit D?
a) Vit B6 b) Vit B2 a) Its active form is calcitriol
c) Vit B1 d) Vit A b) Increases calcium absorption from the intestines
c) Its deficiency results in rickets
6. Tryptophan Load test helps in the evaluation of d) Its decrease causes phosphate reabsorption from
deficiency of the vitamin: the kidneys
a) Folic acid b) Niacinamide (MAN-2K)
c) Pyridoxine d) Cyano cobolamine 15. A patient with chronic renal failure and bone pains
will have deficiency of which vitamin:
7. Biological activity of tocopherols has been a) Vitamin A b) Vitamin B
attributed, in part to their action as: c) Vitamin C d) Vitamin D
a) Antioxidant (MAN-99)
b) Anticoagulants 16. The action of vitamin K in formation of clotting
c) Provitamin factor is through:
d) Antidotes for selenium poisoning a) Post transcription b) Post translation
c) Golgi complex d) Endoplasmic reticulum
8. Vitamin C is present in largest amount in the body (KAR-99)
in: 17. Specific disease caused by vitamin B1 deficiency:
a) Eye b) Kidneys a) Pellagra b) Angular cheilitis
c) Testes d) Adrenal cortex c) Megaloblastic anemia
d) Peripheral polyneuritis
9. Malonyl aciduria is seen in deficiency of: (PGI-98)
a) Pyridoxine b) B12 18. Tocopherol is associated with:
c) Folic acid d) Riboflavin a) Vitamin A b) Vitamin E

1) A 2) C 3) D 4) B 5) B 6) C 7) A 8) D 9) B 10) C 11) A 12) A 13) A


14) D 15) D 16) B 17) D 18) B
, BIOCHEMISTRY

5 7. Vitamin K dependent clotting fa ctors are: d) G-6-P dehydrogenase


a) II b) VII (AIPG-06)
c) IX d) X 66. All of the following are true about manifestations
e) All of the above of vitamin E deficiency, except:
a) Hemolytic anemia
58. Vitamin B12 is: b) Posterior column abnormalities
a) Extrinsic factor of castle c) Cerebellar ataxia d) Autonomic dysfunction
b) Intrinsic factor of castle (AIPG-05)
c) Cyano cobalamine d) A fat soluble vitamin 6 7. Vitamin A:
a) Is water soluble
59. Coenzyme forms are correctly matched except: b) Deficiency causes impaired vision
a) Biotin - carboxylated biotin c) Maintains normal plasma calcium levels
b) Vitamin B - ATP d) Is required for formation of clotting factors
c) Niacin - NAD+NADP (AIPG-05)
d) Vitamin 82 - FMN+FAC 68. Biotin is required for the activity of -
a) Pyruvate ca rboxylase
60. Which of the following combination is correct? b) Lactate dehydrogenase
a) Thiamine - Acyl CoA c) Succinate thiokinase
b) Biotin-CO2 d) Phosphohexose isomerase
c) ATP-Hydrogen d) All of the above (KCET-09)
69. The maximum content of Vitamin - E is found in:
61. Vit-B 12 is absorbed in the a) Cod liver oil b) Fish liver oil
a) Stomach b) Terminal ileum c) Wheat germ oil d) Liver
c) Lower jejunum d) Proximal ileum
70. Aniacinosis results in
62. Thiamine deficiency can be diagnosed by measuring: a) Perleche b) Beri beri
a) Thiamine levels in blood c) Pellagra d) Nyctalopia
b) Alkaline phosphatase levels in blood (KAR- 01)
c) Transketolase activity in RBC 71. One molecule of B-Carotene gives rise to
d) Plasma pyruvate and lactic acid levels a) 1 unit of Vitamin A b) 2 units of Vitamin A
c) 3 units of Vitamin A d) 4 units of Vitamin A
63. Deficiency of Vitamin C causes the following except (KCET- 08)
a) Painful Swollen Gums 72. Who is known for his work on scurvy?
b) Abnormal Collagen a) Fracastorius b) James Lind
c) Anaemia d) Diarrhoea c) John snow d) Edward Jenner
(COMEDK-06) (KAR- 02)
64. Both Vitamin Kand C are involved in: 73. Thiamine deficiency causes decreased energy
a) The synthesis of clotting factors production because?
b) Post translational modifications a) It is required for the process of transamination
c) Antioxidant mechanisms b) It is co-factor in oxidative reduction
d) The microsomal hydroxylation reactions c) It is co-enzyme for transketolase in pentose
(AIPG-05) phosphate pathway
65. Vitamin B12 acts as a coenzyme to which one of d) It is co-enzyme for pyruvate dehydrogenase.
the following enzymes? (AIPG-09)
a) Isocitrate dehydrogenase 74. The vitamin-K dependent proteins C and S are
b) Homocysteine methyl transferase characterized by their ability to inactivate factor
c) Glycogen synthase a) VIIIa and Va b) VIiia

57) E 58) A 59) B 60) D 61) B 62) C 63) D 64) B 65) B 66) D 67) B 68) A 69) C
70) C 71) B 72) B 73) D 74) A
, BIOCHEMISTRY
~

V
387

4. VITAMINS - ANSWERS
1. 'A' [Satyanarayana 3rd & 4th eds 123, 127] 14. ' D' [Satyanarayana 3rd & 4th eds 12 5)
Calcitriol (1,25- dihydroxy cholecalciferol) increases
2. 'C' [Satyanarayana 3rd & 4th eds 145, 151] intestinal absorption of calcium and phosphate and
Folic acid is considered as co-factor in glycine decreases their excretion through the kidney and thus
metabolism. enhances their absorption.

3. ' D' [Satyanarayana 3rd & 4th eds 130-31] 15. ' D' [Satyanarayana 3rd & 4th eds 127-28]
Biotin, Vitamin K, B12 vitamins are synthesized in the In chronic renal failure cases, there is decreased
gut by intestinal flora. synthesis of calcitriol in kidney which is essential
for bone formation. This condition is known as renal
4. ' B' [Satyanarayana 3rd & 4th eds 151] rickets (renal osteodyst rophy) . It can be treated by
administration of calcitriol.
5. 'B' [Satyanarayana 3rd & 4th eds 137]
Flavin mononucleotide (FMN) and Flavin adenine 16. 'B' [Satyanarayana 3rd & 4th eds 130]
din ucleotide (FAD) are the two co-enzyme forms of
riboflavin . Enzymes that use FMN or FAD are ca lled 17. ' D' [Satyanarayana 3rd & 4th eds 136]
flavoprotei ns. Thiamine pyrophosphate (TPP) which is a coenzyme
of B1 plays an important role in transmission of
6. '(' [Dinesh Puri 3rd ed 378] nerve impulse. It is required for acetylcholine
synthesis and ion translocation of neural tissue.
7. 'A' [Satyanarayana 3rd & 4th eds 128] So B1 deficiency results irn peripheral neuropat hy,
Tocopherols are active forms of vita min E. (An tisteriUty numbness in the legs and irritability etc.
vitamin). It prevents the non-enzymatic oxidations of
various cell components by its antioxidant property. Peripheral neuropathy with demyelination of neurons is
also seen in B6 (pyridoxine) and B12 deficiencies.
8. ' D' [Satyanarayana 3rd & 4th eds 134]
High content of vitamin'(' is found in adrenal cortex 18. 'B' [Satyanarayana 3rd & 4th eds 128]
and gonads.
19. ' B' [Satyanarayana 3rd & 4th eds 117 Table 7.1]
9. ' B' [Satyanarayana 3rd & 4th eds 156] • Water soluble vitamins - B,C
In B12 deficiency cases, the excretion of "methylmalonic
• Fat soluble vitamins - A,D,E,K
acid" in urine is elevated and these levels are used to
assess B12 deficiency.
20. 'A' [Satyanarayana 3rd & 4th eds 132]

10. 'C' [Satyanarayana 3rd & 4th eds 143]


21. ' D' [Satyanarayana 3rd & 4th eds 123]
Deficiency manifestations of vitamin 'A'
11. 'A' [Satyanarayana 3rd & 4th eds 151]
• Night blindness
Folic acid is reqU1ired for synthesis of purines and
pyrimidines, which are involved in synthesis of D.N.A • Xerophthalmia
and R.N .A. • Corneal and conjunctiva[ ulceration
• Bitot spots (triangular plaques in conjunctiva
12. 'A' [Satyanarayana 3rd & 4th eds 154] • Keratomalacia.

13. 'A' [Satyanarayana 3rd & 4th eds 136] 22. ' D' [Satyanarayana 3rd & 4th eds 134]
"Beri - Beri" is mostly seen in people consuming
exclusively polished rice as staple food because 23. 'C' [Satyanarayana 3rd & 4th eds 124]
polishing of rice removes about 80% of thiamine.
Dental ;lut.,e

24. 'A' [Satyanarayana 3rd & 4th eds 123] adaptation time is increased in vitamin A deficient
Vitamins that help in wound healing. individuals.
• Vit. C- Collagen formation
• Vit. A- Apithelial formation 34. 'C' [Satyanarayana 3rd & 4th eds 148]
Coenzyme A is a cent ral molecule involved in
• Vit. D
carbohydrate, Lipid and protein metabolisms.
25. 'B' [Satyanarayana 3rd & 4th eds 133]
35. 'C' [Satyanarayana 3rd & 4th eds 141]
Ascorbic acid, by its reducing property increases iron
Niacin deficiency results in "pellagra", a condition
absorption by keeping it in ferrous form
manifested as dermatitis, diarrhoea, and dementia.
26. 'B' [Satyanarayana 3rd & 4th eds 133]
36. 'C' [Satyanarayana 3rd & 4th eds 132]
27. 'D' [Satyanarayana 3rd & 4th eds 123]
3 7. 'B' [Satyanarayana 3rd & 4th eds 140]
28. 'D' [Satyanarayana 3rd & 4th eds 150]
38. ' B' [Satyanarayana 3rd & 4th eds 138]
Magenta tongue (glossitis), fissures at corners of
29. 'C' [Satyanarayana 3rd & 4th eds 131]
mouth (cheilosis) and enamel hypoplasia in children
Vitamin Antagonist are common manifestations of vitamin A deficiency.
Vitamin K Dicoumarol, Heparin, Salicylates.
39. ' B' [Satyanarayana 3rd & 4th eds 141]
Vitamin B1 Pyrithiamine and Oxythiamine.
Pyridoxine(B 6 ) Iso niazid. 40. 'B' [Satyanarayana 3rd & 4th eds 132]
Biotin Biotin sulphonic acid, Oesthiobiotin
41. 'B' [Satyanarayana 3rd & 4th eds 134]
Felic acid Am inopterin and Methotrexate.
"Scurvy" is characterized by spongy and sore gums,
Loose teeth, anemia, swollen joints, delayed wound
30. 'B' [Satyanarayana 3rd & 4th eds 155] healing, hemorrhage etc
Schilling test is used to detect B12 deficiency.
42. 'A' [Satyanarayana 3rd & 4th eds 13 7, 140]
31. 'A' [Satyanarayana 3rd & 4th eds 130]
Clotting factor II (prothrombin), VII (proconvertin), IX 43. 'D' [Satyanarayana 3rd & 4th eds 122]
(Christmas factor), X (Stuart power factor) are vitamin The recommended daily intake of vitamin 'I< in adults
K dependent factors. They are synthesized in liver. is about 1000 retinol equivalents (3500 IU) for man
and 800 retinol equivalents (2500 IU) for woman.
32. 'A' [Satyanarayana 3rd & 4th eds 134]
Wheat and milk are poor sources of ascorbic acid. 44. 'D' [Satyanarayana 3rd & 4th eds 134]

Milk is not a good source of (KERALA 2015) 45. 'D' [Satyanarayana 3rd & 4th eds 121 Fig 7.3]
a Vitamin C b) Vitamin A
c)Vitamin D d) Vitamin K 46. 'A' [Satyanarayana 3rd & 4th eds 148]
Avidin is a glycoprotein and is known as egg-white
33. 'C' [Satyanarayana 3rd & 4th eds 121] injury factor. Biotin is known as anti-egg white injury
The retina of eye contains 2 types of cells- rods and factor.
cones. Rhodopsin is a conjugated protein present in
rods. It contains 11- cis retinal, which is one of the 47. 'C' [Satyanarayana 3rd & 4th eds 141]
vitamer of vitamin A.
48. 'C' [Satyanarayana 3rd & 4th eds 152]
Rods are involved in dim vision whereas Cones are Folic acid is associated with the metabolism of
involved in Color vision and bright Light vision. Dark histidine. Formiminoglutamate (FIGLU ) formed in
, BIOCHEMISTRY

histidine metabolism accumulates and is excreted in 64. 'B' [Satyanarayana 3rd & 4th eds 130, 132]
urine. Histidine load test utilizing the excretion of The biosynthesis of a protein or polypeptide in a
FIGLU in urine is used to asses folic acid deficiency. living cell is referred to as translation. The proteins
synthesized in translation are not functiona l and many
49. 'A' [Satyanarayana 3rd & 4th eds 130] changes take place in t he polypeptides to become
Vitamin K is taken in the diet or synthesized by functional. These modifications include protein
intestinal bacteria. Its absorption takes place along fo lding, trimming by proteolytic degradation, intein
with fat, transported along with LD L and is stored splicing and covalent changes, which are collectively
main ly in liver. known as post-translational modifications.

50. 'C' [Satyanarayana 3rd & 4th eds 134] Vitamin K brings about post-translational modification
Pigeon chest is seen in deficiency of vitamin D of inactive 2, 7, 9, 10 factors. Vitamin C brings about
(rickets) post-translational modification of proto collagen to
collagen.
51. 'B' [Satyanarayana 3rd & 4th eds 152]
The process of synthesis of RNA from DNA is
52. 'A' [Satyanarayana 3rd & 4th eds 146] transcription. The products formed in transcription
Peripheral neuropathy is seen in deficiency of Vit. B1 , are referred to as primary transcripts, which are
Vit. B6, and Vit.B 12 • usually inactive. They undergo post-transcriptional
modifications like termina l base additions, base
53. 'B' [Satyanarayana 3rd & 4th eds 130] modifications, splicing, etc., to produce functionally
active RNA molecules.
54. 'B' [Satyanarayana 3rd & 4th eds 146]
Biotin is called as anti-egg white injury factor. The prokaryotic mRNA synthesized in transcription
is almost similar to functional mRNA. In contrast,
55. 'A' [Satyanarayana 3rd & 4th eds 135] eukaryotic mRNA undergoes extensive post-
The conversion of pyruvate to acetyl CoA is catalysed by transcriptional changes.
"pyruvate dehydrogenase". This reaction is dependent
on TPP. 65. 'B' [Satyanarayana 3rd & 4th eds 154, 155]

56. 'D' [Satyanarayana 3rd & 4th eds 140] 66. 'D' [Satyanarayana 3rd & 4th eds 129]
Both the coenzymes of niacin (NAD+ and NADP+) are
synthesized by the essential amino acid, tryptophan. 67. 'B' [Satyanarayana 3rd & 4th eds 119-121]

57. 'E' [Satyanarayana 3rd & 4th eds 130] 68. 'K [Satyanarayana 3rd & 4th eds 147]

58. 'A' [Satyanarayana 3rd & 4th eds 153] 69. 'C' [Satyanarayana 3rd & 4th eds 129]

59. 'B' [Satyanarayana 3rd & 4th eds 70. 'C' [Satyanarayana 3rd & 4th eds 141]
146 for option 'A' Aniacinosis is nothing but deficiency of Niacin.
140 for option 'C'
137 for option 'D'] 71. 'B' [Satyanarayana 3rd & 4th eds 119)
60. 'D' !3-carotene or provitamin A is found in plant foods.
It is cleaved in the intestine to produce two moles
61. 'B' [Satyanarayana 3rd & 4th eds 154) of retinal, which is nothing but aldehyde form of
vitamin A.
62. 'C' [Satyanarayana 3rd & 4th eds 136)
72. 'B' [Satyanarayana 3rd & 4th eds 132)
63. 'D' [Satyanarayana 3rd & 4th eds 132, 134) James Lind, a surgeon of ttie English navy in 1753
published "Treatise on Scurvy". Based on the Lind's
Dental ;lut.,e

5. ENZYMES
1. Enzyme + coenzyme constitutes: c) Debranching enzyme
a) Apoenzyme b) Pro enzyme d) Glucose-6-phosphotase
c) Protoenzyme d) Holoenzyme (MAN-2K)
10. Which of the following is correct of the pyruvate
2. Multienzyme complex in humans: dehydrogenase enzyme complex?
a) Fatty acid synthetase a) It is present in cytoplasm
b) Malonyl CoA carboxylase b) Catalyzes the conversion of pyruvate to oxaloacetate
c) Carbamoyl phosphate synthetase c) Requires TPP as cofactor
d) Adenosine phospho ribosyl transferase d) Catalyzes a reversible reaction
(MAN-2K)
3. Hexokinase is a: 11. Elevated levels of serum glutamic oxaloacetic
a) Transferase b) Reductase transaminase is indicative of:
c) Oxidoreductase d) Oxidase a) Deficiency of glutamic acid
b) Deficiency of oxaloacetic acid
4. Key glycolytic enzymes: c) Liver disease d) All of the above
a) Phosphofructokinase b) Hexokinase (MAN-98)
c) Pyruvate kinase d) Glucose 1,6 diphosphatase 12. Competitive enzyme inhibition will cause:
a) Decrease of Km and increase of Vmax
5. Coenzyme responsible for single carbon transfer is: b) Increase of Km and increase of Vmax
a) Acetyl co enzyme A b) Biotin c) Decrease of Km and decrease of Vmax
c) THF4 d) Pyridoxine d) Increase of Km and unchanged Vmax
(MAN-99)
6. Benzidine test is positive with the enzyme: 13. The enzyme that is specifically increased in
a) Cytochrome oxidase b) Dehydrogenase obstructive jaundice is:
c) Hydrolase d) Peroxidase a) Glutamate oxaloacetate transaminase
b) Glutamate pyruvate transaminase
7. Michaelis-Menten hypothesis states that: c) Alkaline phosphatase
a) Rate of enzymatic reaction is independent of d) Acid phosphatase
substrate concentration (KAR-2K)
b) Rate of non enzymatic reaction is proportional to 14. The deficiency of thiamine can be identified by
substrate concentration measuring the red cell:
c) Km is the enzyme - substrate complex association a) Transketolase b) Transaldolase
constant c) Alpha ketoglutarate dehydrogenase
d) Enzyme- substrate complex formation is essential d) Pyruvate dehydrogenase
in enzymatic reaction (KAR-2K)
15. Acetyl Co-A acts as a substrate for all the enzymes
8. Dehydrogenases used as co enzymes all of the except:
following, except: a) HMG-Co A synthase b) Malic enzyme
a) FMN b) FAD c) Malonyl CoA synthetase
c) NADP+ d) Ferro protoporphyrin d) Fatty acid synthetase
(AIIMS-03)
9. Muscle glycogen, which cannot contribute to blood 16. The activity of the following enzyme is affected by
glucose due to the absence of which enzyme? biotin deficiency:
a) Phosphoglutamase b) Branching enzyme a) Transketolase b) Dehydrogenase

1) D 2) A 3) A 4) A 5) C 6) D 7) D 8) D 9) D 10) C 11) C 12) D 13) C


14) A 15) B 16) D
Dental ;lut.,e

b) Fumarase 43. Iron is present in all of the following except


c) Cholinesterase d) Amylase a) Myoglobin b) Cytochrome
(TNPSC- 99) c) Catalase d) Pyruvate Kinase
34. Enzymes concerned with the citric acid cycle are (AIPG-07)
found in: 44. The enzyme phosphofructo kinase-1 is strongly
a) Nucleus b) Mitochondria activated by
c) Ribosomes d) Non -particulate cytoplasm a) Cyclic AMP b) Adenosine tri phosphate
(APPSC- 99) c) Citrate d) Fructose 2,6 bis phosphate
35. Which of the following is a membrane bound enzyme (COMEDK- 08)
that catalyzes the formation of cyclic AMP from ATP? 45. Inactive precursors of enzymes are known as
a) Tyrosine kinase b) Polymerase a) Apoenzymes b) Coenzymes
c) ATP synthase d) Adenylate cyclase c) Proenzymes d) Holoenzymes
(AIPG-04) (KCET-08)
36. The most common enzyme deficiency in man is: 46. One of the earliest symptoms of Vit. A deficiency is
a) Glucose-6-phosphate dehydrogenase a) Nyctalopia b) Xerophthalmia
b) Glucose-6-phosp hatase c) Bitot's spot d) Keratomalacia
c) Hexokinase d) Glucose 1,6-diphosphatase (KCET- 07)
4 7. Cofactor required for lipoprotein lipase activity is
37. The combination of main supporting enzyme is called: a) Apo A-II b) Apo C-II
a) Apoenzyme b) Coenzyme c) Apo C-III d) Apo 8-II
c) Holoenzyme d) Constitutive enzyme (COMEDK- 08)
48. LDH has following number of isoenzymes
38. Coenzyme, in an enzymatic reaction usually a) 5 b) 3
functions to: c) 11 d) 2
a) Activate the substrate (KCET-2011)
b) Increase the active sites of apoenzyme 49. Mutation in the oxidative enzymes (peroxisomes)
c) Enhance the specificity of apoenzyme could lead to
d) Accept one of the cleavage products a) Zellweger's syndrome b) Gaucher's disease
c) Epidermolysis bullosa d) Leber's neuropathy
39. Lactate dehydrogenase is: (COMED-2012)
a) Isozyme b) Coenzyme 50. An example of a preventive antioxidant is
c) Antienzyme d) Zymogen a) Catalase b) Tocopherol
c) Superoxide dismutase
40. When the prosthetic group of an enzyme is an d) Urate
inorganic ion, it is called a: (COMED-2012)
a) Cofactor b) Coenzyme 51. Biotin is required for the activity of
c) Apoenzyme d) Holoenzyme a) Pyruvate carboxylase b) Lactate dehydrogenase
c) Succinate thiokinase d) Phosphohexose iso merase
41. Enzymes stored in muscle is: (KCET-2012)
a) Alkaline phosphatase b) SGOT 52. Which of the following is NOT a characteristic
c) SGPT d) CPK feature of allosteric enzymes?
a) They are multienzyme complexes
42. Magnesium is needed for the activity of which b) Follow Michaelis-Menton kinetics
enzyme? c) Presence of Modulator site
a) Phosphatase b) Aldolase d) Give sigmoid shaped curve
c) Dismutase d) ATPase (COMEDK -2013)
(AIIMS-01)

34) B 35) D 36) A 37) C 38) D 39) A 40) A 41) D 42) D 43) D 44) D 45) C 46) A
47) B 48) A 49) A 50) A 51) A 52) B
Dental ;lut.,e

38. ' D' [Satyanarayana 3rd & 4th eds 96) 44. ' D' [Satyanarayana 3rd & 4th eds 250)
Most of coenzymes are derivatives of B-complex vitamins Phosphofructoki nase (PFK) is the most important
COENZYME VITAMIN regulatory enzyme in glycolysis. PFK is an allosteric
Thiamine pyrophosphate (TPP) enzyme regulated by allosteric effectors. ATP, citrate
Thiamine (Bl) and H+ ions are the most important allosteric
Flavin mononucleotide (FMN)
in hi bi tors, whereas fructose 2, 6 bisphosphonate, ADP,
Flavin dinucleotide (FAD) Riboflavin (82)
AMP and Pi are the allosteric activators.
• Niacin adenine di nucleotide
(NAD) 45. ' C' [Satyanarayana 3rd & 4th eds 102)
Niacin
• Niacin adenine di nucleotide Proenzymes or zymogens are inactive enzymes
phosphate (NADP) which undergo irreversible covalent activation by
Pyridoxal phosphate Pyridoxine (8 6 ) the breakdown of one or more peptide bonds. For
example, Chymotrypsin, pepsinogen and plasminogen
Coenzyme A (CoA) Pantothenic acid are proenzymes that are converted to active enzymes,
Tetrahydrofolate (FHJ Felic acid chymotrypsin, pepsin and plasmin respectively.
Biotin enzyme Biotin (8 7 )
46. 'A' [Satyanarayana 3rd & 4th eds 123)
Methyl cobalamine Cobalamine (8 12) Nyctalopia (Night blindness), fo llowed by xeropthalmia
with Bitott's spots and keratomalacia are deficiency
39. 'A' [Satyanarayana 3rd & 4th eds 104] manifestations of Vit. A.
lsoenzymes catalyse the same reaction but differ
in physical and chemical properties. Lactate 47. ' 8' [Satyanarayana 3rd & 4th eds 318]
dehydrogenase, creatinine phospho kinase, alkaline Lipoprotein lipase is present in the capillary walls of
phosphatase and alcohol dehydrogenase are examples adipose tissue, cardiac and skeletal muscle, besides
of isoenzymes. other tissues. It hydrolyses a portion of triacylglycerols
present in chylomicrons and VLDL to liberate free fatty
40. 'A' [Satyanarayana 3rd & 4th eds 96] acids and glycerol. Lipoprotein Lipase is activated by
apo C II.
Enzyme Inorganic ion
ATPase Mg+ 2 and ca+2 48. 'A' [Harper 25th ed 82)
Eno lase Mg+2 Enzyme and its isoenzyme both catalyze the same
reaction, but exhibit different physical and chemical
Cytochrome oxidase 'Fe' and 'Cu' properties. lsoenzymes are common in sera and tissues
• Alcohol dehydrogenase of all vertebrates, insects, plants and unicellular
organisms. Both the kind and the number of enzymes
• Alkaline phosphatase Zn
involved are equally diverse. Lactate dehydrogenase
• Aldolase
(LDH) has 5 isozymes. They are expressed in heart and
skeletal muscle.
41. 'D' [Satyanarayana 3rd & 4th eds 111)
CPK (creatinine phosphokinase) and aldolase are 49. 'A' [Satyanarayana 3rd & 4th eds 8]
reliable indicators of muscular diseases CPK is the Peroxisome biogenesis disorders are a group of diseases
first enzyme released into circulation after myocardial involving the enzyme activities of peroxisomes. The
infarction. So, CPK estimation is highly useful for early most severe form is Zellweger syndrome characterized
diagnosis of myocardial infarction. by the absence of functiona l peroxisomes. The victims
die within one year after birth.
42. ' D' [Satyanarayana 3rd & 4th eds 91]

43. 'D' [Satyanarayana 3rd & 4th eds 414]


Mg+ 2 serves as a cofacto r for pyruvate kinase.
, BIOCHEMISTRY
(""'.,,_

V
399

50. 'A' [Satyanarayana 3rd & 4th eds 659] • Detoxification of drugs by hydroxylation
The antioxidant enzymes are: • Preserve integrity of RBC membrane.
• Superoxide dismutase: Produces Hydrogen peroxide
from superoxide free radical 55. 'B' [Text book of biochemistry for medical students
• Cata lase: Hydrogen peroxide produced by superoxide by Vasudevan 6th ed 418]
dismutase is metabolized by catalase. In osteopetrosis, also called as marble bone disease,
there is increased bone density. It is due to mutation
• Glutathione Peroxidase: It detoxifies H20 2 to H20,
in gene encoding of carbonic anhydrase type II
while reduced glutathione (G- SH) is converted
enzyme. The deficiency of t his enzyme in ost eoclasts
to oxidized glutathione (GS- SG). Yhe reduced
glutathione can be regenerated by the enzyme leads to inability of bone resorption .
glutathione reductase utilizing NADPH .
56. 'D' [Wintrobes clinical hematology 13th ed Pg 736]
Various enzymes causing non-spherocytic
Antioxidants are scavengers of free radicles which
haemolytic anemia are:
harm the living cells.
• G6PD (most common enzyme deficiency)
51. 'A' [Satyanarayana 3rd & 4th eds 146] • Pyruvate kinase (second most common)
Biotin directly participates as a coenzyme in • Pyrimidine-5-nucleotidase type 1 (third most
carboxylation reactions. common)
• Aldolase
52. B' [Check Explanation Below]
Allost eric enzymes are unique compared to ot her • Hexokinase
enzymes because of its ability to adopt various • 6-phosphofructokinasae
conditions in the environment due to its special
properties. Besides active site, they possess additional Note: Defi ciencies in glyceraldehyde-3 phosphate
sites known as allosteric sites. Allosteric enzymes are dehydrogenase and lactate dehydrogenase are not
an exception to Michaels-Menton model. Because they associated with haemolytic anemia.
have more than two sub units and active sites, they
do not obey the Michaelis-Ment on kinetics but instead
have sigmoidal Kinetics. An example of allosteric
enzyme is haemoglobin .

53. 'D' [Satyanarayana 3rd & 4th eds 256]


Niacin (as NAO+) acts as elect ron acceptor for the
fo llowing enzymes in citric acid cycle
• Isocitrate dehydrogenase
• Alpha Ketoglutarate dehydrogenase
• Malate dehydrogenase

Riboflavin (as FAD) acts as a coenzyme for succinate


dehydrogenase

54. 'D' [Satyanarayana 3rd & 4th eds 274]


Importance of NADPH :
• Biosynthesis of fatty acids and steroids
• Synthesis of certain amino acids involving the
enzyme glutamate dehydrogenase
• Antioxidant action by reducing H20 2 with t he help
of Glutathione
Dental ;lut.,e

6 . NUCLEO PROTEINS & MINERAL METABOLISM


1. Which base is not found in DNA: 11, Unwinding of DNA is done by:
a) Adenine b) Guanine a) DNAse b) Topo isomerase
c) Cytosine d) Uracil c) Ligase d) Reverse transcriptase

2. On complete hydrolysis of DNA we will get all the 12. Codons are present on:
following except: a) m-RNA b) DNA
a) Deoxy pentose sugar b) Phosphoric acid c) t-RNA d) Ribosomal RNA
c) Adenosine d) Purine bases
13. Nonsense codons bring about:
3. DNA double helix is bound by: a) Elongation of polypeptide chain
a) Covalent bond b) Hydrogen bond b) Pre-translational modification of protein
c) Disulfide linkage d) Vander wall forces c) Initiation of protein synthesis
d) Termination of protein synt hesis
4 R-RNA is mainly produced mainly in:
a) Nucleus b) Nucleolus 14. Chargaff rule states that:
c) Ribosome d) Endoplasmic reticulum a) A+G=T+C b) A/f =G/C
c) A=U=T=G=C d) A+T=G+C
5. Genes are:
a) Ribonucleic acid b) Deoxy ribonucleic acid 15. The initiating codon for protein synthesis is:
c) Lipoproteins d) Ch romo proteins a) AUG b) UAA
c) UUU d) UAG
6. All are true of the genetic code except:
a) Degenerate b) Universal 16. Translation occurs at:
c) Punctuation d) Non overlapping a) Mitochondria b) Centrosome
c) Nucleus d) Ribosome
7. Restriction endonuclease cleaves:
a) Double stranded DNA 17. Mutations are due to changes in:
b) Single stranded DNA a) DNA nucleotide sequence
c) Single stranded RNA b) RNA nucleotide sequence
d) Polypeptide c) Amino acid sequence of ribonuclease
d) Cell walls
8. End product of purine metabolism is: (APPSC-99)
a) Creatinine b) Uric acid 18. Peroxidase enzyme contain:
c) Xanthine d) Phosphates a) Chromium b) Selenium
c) Magnesium d) Calcium
9. Which of the following process in involved in
conversion of DNA to RNA: 19. Kinase requires:
a) Conjugation b) Transduction a) Mn++ b) Cu++
c) Translocation d) Transcription c) Mg++ d) Inorganic phosphate

10. The portion of DNA in which RNA polymerase binds 20. Ferritin - an inactive form of iron is stored in:
and starts transcription is called: a) Gut b) Spleen
a) Terminator b) Anti terminator c) Liver d) All of the above
c) Operator d) Promoter region

1) D 2) C 3) B 4) B 5) B 6) C 7) A 8) B 9) D 10) D 11) B 12) A 13) D


14) A 15) A 16) D 17) A 18) B 19) C 20) C
, BIOCHEMISTRY

21. Highest binding of iron in plasma is seen with: 31. The patient suffered from hypogonadism, failure
a) Transferrin b) Ferritin to thrive, loss of taste and unable to maintain
c) Hemoglobin d) Ceruloplasmin stability. This shows the deficiency of:
a) Zinc b) Chromium
22. In human body which of the following trace element c) Copper d) Potassium
is next to iron: (AIIMS-06)
a) Ca++ b) Zn++ 32. Ammonia produced by brain is trapped as
c) Cu++ d) Selenium a) Urea b) Uric Acid
c) Creatinine d) Glutamine
23. The normal value of serum potassium level is
a) 2.8-3.8 meq/L b) 3.8-5 meq/L 33. Denaturation of double stranded DNA involves
c) 5-5.8 meq/L d) 6-7.2 meq/L a) It gets broken down to nucleotides
(KAR-97) b) It becomes single stranded reversibly
24. Daily requirement of iron in man is: c) It becomes single stranded irreversibly
a) lgm b) 10 microgram d) It becomes double stranded irreversibly
c) 10mg d) 20mg (AIPG- 10)
(KAR- 01) 34. Serum creatine kinase-3 (CK-3) is elevated in
25. Normal serum calcium level is: a) Muscular dystrophy b) Myocardial infarction
a) 4-6 mg/dl b) 9-11 mg/dl c) Alcoholic cirrhosis d) Brain tumours
c) 19-21 mg/dl d) 20-30 mg/dl (COMEDK-07)
(TNPSC- 99) 35. Enzymes increase reaction rates by
26. Inside the cell, the substance, which contributes to a) Altering the free energy of the action
most of the osmolality, is: b) In hi biting the backward reaction
a) Protein b) Potassium c) Enhancing the forward reaction
c) Urea d) Phosphate d) Decreasing the energy of activation
(TNPSC- 99) (KCET- 07)
27. Iron absorption is decreased in presence of all 36. Transcription is the synthesis of
except: a) Single stranded complimentary copy of DNA
a) Phytates b) Ascorbic acid b) Double stranded complimentary copy of DNA
c) Tannins d) Phosphates c) Complimentary copy of RNA
(AIPG- 99) d) Complimentary copy of rRNA
28. Normal level of serum phosphorous is: (COMEDK- 08)
a) 2.5 - 4.5 mgm% b) 7-9 mgm% 37. In molecular cloning, Blue-white screening is used
c) 40-50 mgm% d) 1-2 gm% for?
a) To screen for recombinant vectors
29. The sigma subunit of prokaryotic RNA Polymerase: b) To detect gene mutations
a) Binds the antibiotic Rifampicin c) To identify desired chromosomal DNA insert in
b) Is inhibited by a-amanitin plasmid vectors
c) Specifically recognizes the promoter site d ) To detect host DNA in situ
d) Is part of the core enzyme (AIPG-09)
(AIPG-06) 38. Which of the following is used in recombinant DNA
30. In all the following RNA participates directly technology?
except: a) Restriction endonucleases
a) Post translational modification b) PCR
b) Post transcriptional modification c) Reverse transcriptase
c) DNA replication d) Splicing d) FISH
(AIPG-07) (AIPG-10)

21) A 22) B 23) B 24) C 25) B 26) B 27) B 28) A 29) C 30) A 31) A 32) D 33) B
34) A 35) D 36) A 37) C 38) A
, BIOCHEMISTRY
~

V
903

6. NUCLEO PROTEINS & MINERAL METABOLISM - ANSWERS


1. 'D' [Satyanarayana 3rd & 4th eds 73] 5. 'B' [Satyanarayana 3rd & 4th eds 738]
Both DNA and RNA contain same type of purines namely DNA is the reserve bank of genetic information and is
Adenine (A) and Guanine (G). The pyrimidines in DNA organized into genes, which are the fundamental units
are Thymine (T) and Cytosine (C). The pyrimidines in of genetic information.
RNA are Uracil (U) and Cytosine (C).
Genes control protein synthesis through the mediation
2, 'C' [Satyanarayana 3rd & 4th eds 70] of RNA.

3. 'B' [Satyanarayana 3rd & 4th eds 75] 6. 'C' [Satyanarayana 3rd & 4th eds 551]
The two strands of DNA are held by hydrogen bonds The genetic code is considered as Universal, Specific,
formed by complementary base pairs; A-T pair and G-C Non-overlapping and Degenerate.
pair.
The codon degeneracy is explained by 'WOBBLE
HYPOTHESIS', which was put forth by CRICK.
The A-T pair has 2 hydrogen bands while G-C pair has
3 hydrogen bands because of which the G-C pair is 'Wobble hypothesis' is a phenomenon in which a sing le
stronger than A-T pair. tRNA can recognize more than one codon. Wobbling is
attributed to the difference in the spatial arrangement
4. 'B' [Satyanarayana 3rd & 4th eds 6] of the 5'-end of the anticodon.
RNA's (mRNA, tRNA and rRNA) are produced by DNA,
which in turn carry out protein synthesis. Wobbling hypothesis explains the degeneracy of the
THE THREE DISTINCT TYPES OF RNA'S tARE genetic code i.e., existence of multiple codons for
a single amino acids. Although there are 61 codons
• mRNA is synthesized ,n the
for amino acids, the number of tRNAs are around 40,
nucleus as heterogenous nuclear
which is due to wobbling.
RNA.
Messenger • The specific information required 7. 'A' [Satyanarayana 3rd & 4th eds 579 -81]
RNA for the synthesis of a given
(5-10%) protein is present on the mRNA. 8. 'B' [Satyanarayana 3rd & 4th eds 394]
• The DNA passes the genetic Uric acid is the end product of purine metabolism.
information ,n the form of "GOUT" is a metabolic disease associated with over
codons to mRNA to translate production of uric acid.
into a protein sequence.
Ribosomal • rRNAs are factories of protein 9. 'D' [Satyanarayana 3rd & 4th eds 543]
RNA synthesis. Transfer of genetic information
(50-80%) • Synthesized in nucleolus. Transformation through the agency of free DNA
Eg.: Pnemocoocci, Bacillus
• They carry the amino acids
and hand them to the growing Transfer of a portion of DNA from
polypeptide chain. Transduction one bacterium to another by a
• The amino acid is covalently bacteriophage
Transfer RNA bound to tRNA at the 3'end. Substitution of a purine for a
(t-RNA) Each tRNA has a three nucleotide Transversion pyramidine and vice versa in base
base sequence - the anti codon. pairing
The anticodon is responsible to
Transfer of information from
recognize the codon of mRNA for
Translation mRNA to proteins resulting in
protein synthesis.
polypeptide formation.
Dental ;lut.,e

Also further analysis showed that A+G=C+T i.e.,


Replication of messenger RNA
quantity of purines equals to quantity of pyramidines.
Transcription from the DNA. Mediated by "RNA
polymerase II"
15. 'A' [Satyanarayana 3rd & 4th eds 551]
• Formation of DNA from RNA
Reverse and is mediated by "Reverse 16. ' D' [Satyanarayana 3rd & 4th eds 550-51)
Transcription transcriptase".
• Seen in ocogenic viruses 17. 'A' [Satyanarayana 3rd & 4th eds 535]

10. 'D' [Satyanarayana 3rd & 4th eds 544) 18. ' B' [Satyanarayana 3rd & 4th eds 422)
The binding of enzyme RNA polymerase to DNA is the
prerequisite for the beginning of transcription. The 19. 'C' [Satyanarayana 3rd & 4th eds 410]
specific region on the DNA where the enzyme binds is Mg+ 2 serves as cofactor for several enzymes
known as promoter region. requiring ATP. Examples are hexokinase, glucokinase,
Phosphofructoki nase.
11. 'B' [Satyanarayana 3rd & 4th eds 52 7, 544)
20. 'C' [Satyanarayana 3rd & 4th eds 415]
12. 'A' [Satyanarayana 3rd & 4th eds 551] Haemosiderosis and hemochromatosis are the diseased
The codons are composed of the four-nucleotide bases, states associated with iron metabolism.
namely, the purines (adenine and guanine) and the
pyrimidines (cytosine and Uracil). Hemosiderosis is due to excessive iron in the body. It
is commonly observed in patients receiving repeated
The three-nucleotide base sequences in mRNA that act blood transfusions. Bantu-siderosis is due to food
as code words for amino acids in protein constitute cooked in iron pots.
the genetic code or simply codons.
Hemochromatosis is a rare disease in which iron is
The three codons UAA, UAG and UGA do not code directly deposited in the tissues.
for amino acids and they act as stop signals in
protein synthesis. These three codons are known as Bronzed pigmentation of skin, cirrhosis of liver, and
termination codons or non-sense codons. pancreatic fibrosis are the manifestations of this
disorder.
The codons UAA, UAG, UGA are known as Ochre, Amber
and Opal codons. Pancreatic fibrosis may be associated with a condition
known as Bronze Diabetes.
The codons AUG and sometimes, GUG are the chain
initiating codons. AUG serves as initiating codon, 21. 'A' [Satyanarayana 3rd & 4th eds 415)
besides coding for methionine residue in protein Option 'D' ceruloplasmin is involved in the conversion
synthesis. of iron from Fe+2 to fe+3 •

13. 'D' [Satyanarayana 3rd & 4th eds 537, 551] Normal hemoglobin contains iron in Fe+2 form while
methemoglobin contains iron in Fe+3 form.
14. 'A' [Satyanarayana 3rd & 4th eds 73]
According to Chargoff's rule, DNA contains equal 22. 'B' [Satyanarayana 3rd & 4th eds 419)
number of adenine and thymine residues (A=T) and
equal numbers guanine and cytosine residues (G=C). 23. ' B' [Satyanarayana 3rd & 4th eds 412)

RNA does not obey Chargoff's rule. Due to the single 24. 'C' [Satyanarayana 3rd & 4th eds 414]
stranded nature, there is no specific relation between
purine and pyrimidine contents. Thus the guanine
content is not equal to cytosine.
, BIOCHEMISTRY

25. ' B' [Satyanarayana 3rd & 4th eds 406] its depleted levels impair the TCA cycle. The net result
Tetany (seen in hypoparathyroidism) is characterized is decrease in number of ATP produced by the brain.
by reduction of serum calci um to below 7mg / dl. The toxic effects of NH 3 on brain are due to impairment
Symptoms of tet any include neuromuscular irritability, in ATP formation .
spasms and convulsions.
When the levels of ammonia are elevated, intravenous
26. ' B' [Satyanarayana 3rd & 4th eds 412] administration of sodium benzoate and phenyl lactate
Potassium is the principal intracellular cation. is done. These compounds condense with glutamate
and form water soluble products that can be easily
Hypoka lemia (decreased serum potassium) is seen in secreted. By this way, ammonia can be t rapped and
Cushing's syndrome and prolonged cortisone t herapy removed from the body.
while hyperkalemia is seen in renal failure, addison's
disease, etc. 33. 'B' [Wikipedia.org/denaturation]
As DNA strands are complementary and are held by
27. ' B' [Satyanarayana 3rd & 4th eds 414] non-covalent forces, t hey can be separated into
Ascorbic acid promotes iron absorption, while individual strands. This separation or denaturation of
phytates, oxalates, phosphates and tannins reduce DNA is commonly induced by heating the solution and
iron absorption. the dissociation is reversible on cooling.

28. 'A' [Satyanarayana 3rd & 4th eds 410] 34. 'K [Satyanarayana 3rd & 4th eds 107, 111]
Creatinine Kinase (CK) or creatinine phosphokinase
29. 'C' [Satyanarayana 3rd & 4th eds 543-44, Fig 25.2] (CPK) exists as three isoenzymes. Each isoenzyme is a
RNA polymerase enzyme is involved in transcription. dimmer composed of two subunits - M (muscle) or B
It contains five polypeptide subunits - 2 a., 1 13, 1 (brain or both)
131 and one sigma factor. The binding of the enzyme
I so enzyme Subunit Tissue of origin
RNA polymerase to DNA is the prerequisite for the
transcription to start. The specific region on the CPK1 BB Brain
DNA where the enzyme binds is known as promoter CPK 2 MB Heart
region. The sigma factor of RNA polymerase can CPK 1 MM Skeletal muscle
recognize promoter region and helps in initiation of
transcription. The enzyme without sigma factor is CPK 2 is the reliable earliest indication of myocardial
known as core enzyme. infarction. CPK 3 is elevated in muscular dystrophy.

30. 'A' [Satyanarayana 3rd & 4th eds 130, 132, 561] 35. 'D' [Satyanarayana 3rd & 4th eds 98]
RNA have no role in post-translation.
36. 'K [Satyanarayana 3rd & 4th eds 543]
31. 'A' [Satyanarayana 3rd & 4th eds 419] Transcription is a process in which RNA (nothing
but complimentary copy of ONA) is formed from DNA
32. 'D' [Satyanarayana 3rd & 4th eds 336) (nothing but complimentary copy of RNA).
The elimination of NH3 can be shown as
3 7. 'C' [Wikipedia/B luewhite Screen]
Glutamate • Molecular cloning refers to the procedure of isolating
a.-ketoglutarate + NH3 - - - - --- glutamate. a defined DNA sequence and obtaining multiple
dehydrogenase copies of it .
• Cloning of any DNA segment involves 4 steps like
Here the amino group is transferred to a.-ketoglutarat e
fragmentation, Ligation, transfection and screening/
to produce glutamate. Accum ulation of more NH 3 shifts
selection.
the equilibrium to the right with more glutamate
formation , hence more utilization of a.-ketog lutarate. • Blue-white screen is a molecular technique that
a.-ketoglutarate is a key intermediate in TCA cycle and allows the detection of successful ligations in
Dental ;lut.,e

vector based gene cloning. DNA of interest is


ligated into a vector. The vector is then transformed
into competent cell (bacteria) and is grown. If
the ligation is successful, the ba cterial colony
will be white, if not, the colony will be blue. This
technique allows for the quick and easy detection of
successful ligation, without the need to individually
test for each colony.

38. 'A' [Satyanarayana 3rd & 4th eds 579]


Restrict ion endonucleases comprises a group of
enzymes that cleaves double stranded DNA. These
enzymes are sequence specific and each enzyme acts at
a limited number of sites in DNA ca lled 'Recognition' or
'Cutting' sites. These enzymes cut DNA into fragments
at sites determined by the nucleotide sequence of the
DNA.

Basic principles of Recombinant DNA technology:


• Generation of DNA fragments and selection of the
desired piece of DNA.
• Insertion of the selected DNA into a cloning vector
(Eg: a plasmid) to create a 'recombinant' DNA or
'Chimera' DNA.
• Introduction of recombinant vectors into host cells
(Eg: bacteria)
• Multiplication and selection of clones containing
the recombinant molecules.
• Expression of the gene to produce the desired
product.

39. 'C' [Satyanarayana 3rd & 4th eds 72]


Adenosine plays an important role in biochemical
processes, such as ATP, ADP and cAMP. Adenosine is
a purine nucleoside comp rising a molecule of adenine
attached to a ribose sugar molecule moiety via a ~-N9-
glycosidic bond
, BIOCHEMISTRY
(""'.,,_

V
909

38. Ubiquitin is involved in


a) Intracellular proteolysis
b) Electron transport chain
c) Transport of ATP d) Protein folding
(COMEDK-2011)
39. A relatively component of the electron transporting
respiratory chain is
a) Flavoprotein b) Cytochrome Cl
c) Ubiquinone d) Cytochrome A
(COM EDK -2012)
40. The amino acid required for the synthesis of haem
is
a) Glutamine b) Glutamic acid
c) Glycine d) Lysine
(COMEDK -2013)
41. Which of the following is false about starvation
ketoacidosis?
a) Metabolic acidosis b) Smell of acetone in breath
c) Benedict's test +ve d) Rothera's test is +ve
(KAR -2013)

38) A 39) C 40) C 41) C


Dental ;lut.,e

7. GENERAL BIOCHEMISTRY - ANSWERS


1. 'B' [Satyanarayana 3rd & 4th eds 63] excessive production of acetoacetic acid and B-hydroxy
butyric acid.
2. 'C' [Satyanarayana .3rd & 4th eds 639-40]
Cytochrome P450 species are found in the highest The most important cause of metabolic acidosis is
concentration in the microsomes of liver. So, most of the due to excessive production of organic acids, which
detoxification reactions are carried out mainly in liver. combine with NaHco 3 - and deplete the alkali reserve.

3. 'D' [Satyanarayana 3rd & 4th eds 639) Acute metabolic acidosis is usually compen sated by
hyperventilation of lungs, which causes increased
4. 'A' [Satyanarayana 3rd & 4th eds 650] elimination of CO 2 from body. The respiratory
Amiphipathic liquids (contains both hydrophobic and compensation is short lived. Within 3-4 days, the
hydrophilic groups), namely, phospho lipids, glycolipids renal compensation sets and the W ions are excreted
and cholesterol are found in animal membranes. as NH/ ions.

5. 'B' [Satyanarayana 3rd & 4th eds 650] 11. 'C' [Satyanarayana 3rd & 4th eds 6]

6. 'C' [Satyanarayana 3rd & 4th eds 4 7 4] 12. 'A' [Satyanarayana 3rd & 4th eds 6]

7. 'D' [Satyanarayana 3rd & 4th eds 475] 13. 'A' [Satyanarayana 3rd & 4th eds 650]
Sodium bicarbonate and carbonic acid (NaHCo 3 -
H2Co3) is t he most predominant buffer system of the 14. 'C' [Satyanarayana 3rd & 4th eds 7]
extracellular fluid.
15. 'A' [Satyanarayana 3rd & 4th eds 6]
At a normal blood pH of 7.4, the ratio of bicarbonate
to carbonic acid is 20:1. Thus the bicarbonate 16. 'A' [Satyanarayana 3rd & 4th eds 6)
concentration is 20 times higher t han the carbonic The portion of endoplasmic reticulum associated with
acid in the blood and is referred to as alkali reserve ribosomes is known as rough endoplasmic reticulum. It
of the body. is involved in protein synthesis. The smooth endoplasmic
reticulum is without ribosomes and is involved in
8. 'A' [Satyanarayana 3rd & 4th eds 476] synthesis of lipids, metabolism of drugs and supply of
(a+ 2 for cellular functions.
9. 'A' [Satyanarayana .3rd & 4th eds 470]
Na+ is the principal extracellular cation while K+ 17. 'A' [Satyanarayana 3rd & 4th eds 508- 514]
is the intracellular cation. This difference in the
concentration is essential for survival, which is 18. 'A' [Satyanarayana 3rd & 4th eds 407]
maintained by Na+ - K+ pump. Parathyroid hormone causes dimineralisation of bone
and increases the calcium reabsorption by kidney. This
High cellular K+ is required for optimal glycolysis results in elevation of blood calcium levels.
(pyruvate kinase is dependent on K+) and for protein
biosynt hesis. Na+ and K+ gradients across the plasma Calcitonin promotes calcification of bone and increases
membranes are needed for the transmission of nerve excretion of calcium into urine, resulting in low blood
impulse. calcium levels.

10. 'B' [Satyanarayana 3rd & 4th eds 482] 19. 'D' [Satyanarayana 3rd & 4th eds 428, 430]
The primary defect i11 metabolic acidosis is a reduction Group I hormones bind to intracellular receptors to
in bicarbonate concentration. Acidosis is commonly form receptor-hormone complexes through which their
seen in severe uncontrolled diabetes mellitus, due to functions are mediated.
, BIOCHEMISTRY
~

V
913

8. MISCELLANEOUS
1. Heme is converted to bilirubin mainly in : 10. The following are derived from cholesterol, EXCEPT
a) Kidney b) Liver a) Bile acids b) Steroid hormones
c) Spleen d) Bone marrow c) Vitamin D d) Energy
(AIPG-05) (KAR-01)
2. During phagocytosis, the metabolic process called 11. Non polar lipid act as
respiratory burst involves the activation of: a) Electrical insulators b) Ions
a) Oxidase b) Hydrolase c) Non electrical insu lators
c) Peroxidase d) Dehydrogenase d) Electrolytes
(AIPG-06) (COMEDK-08)
3. The fuel value of fat is: 12. Normal serum alkaline phosphatase Level is
a) 4 b) 7 a) 100-400 Boda nsky unit s
c) 9 d) 5 b) 1000-4000 Bodansky unit s
(PGI-06) c) 0.1-0.4 Bodansky units
4. Which of the following is the best site of absorption d) 1-4 Bodansky units
of calcium? (KAR-03)
a) Proximal segment of small intestine 13. True about G protein coupled receptors is?
b) Middle segment of small intestine a) G proteins bind to hormones on the cell surface
c) Ascending colon d) Ileum b) All the three subunits alpha, beta and gamma
(PIG-06) shou ld bind each other for G protein to act
5. The normal range of serum osmolality (in mosm/L) c) G proteins act as inhibitory and excitatory because
is: of difference in alpha subunit
a) 280 to 295 b) 300 to 320 d) G protein is bound to GTP in resting state
c) 350 to 375 d) 200 to 250 (AIPG- 09)
(AIPG-06) 14. The abundant glycoprotein in the basement
6. Transport form of iron is membrane and has binding domain for both ECM
a) Transferrin b) Ferritin and cell-surface receptor is
c) Apoferriti n d) Lactoferri n a) Laminin b) Fibronectin
(COM EDK- 07) c) E - cadherins d) Tenascin
7. Which is the poorest source of calcium among the (KCET-10)
following 15. The transmembrane adhesive molecules present in
a) Rice b) Human milk hemi desmosomes which specifically binds to basal
c) Egg yolk d) Cow's milk Lamina glycoprotein Laminin are
(KAR- 02) a) integrin cx.6-P4
8. Enzyme regulating the conversion of ethanol to b) the catenins desmoplakin
acetaldehyde: c) cadherins desmoglein
a) Alcohol dehydrogenase d) a. and f3 catenin
b) Acetaldehyde dehydrogenase (KCET- 10)
c) Catalase 16. Within the RBC, hypoxia stimulates glycolysis by
d) Enolase which of the following regulating pathways?
(AIIMS-07) a) Hypoxia stimulates pyruvate dehydrogenase by
9. Iron is present in all, EXCEPT increased 2, 3DPG
a) Myoglobin b) Cytochrome b) Hypoxia inhibits hexokinase
c) Catalase d) Pyruvate kinase c) Hypoxia stimulates release of all glycolytic enzymes
(AIIMS-07) from Band 3 on RBC membrane

1) C 2) A 3) C 4) A 5) A 6) A 7) A 8) A 9) D 10) D 11) A 12) D 13) C


14) A 15) A 16) C
, BIOCHEMISTRY
~

V
919

2. Non-enzymatic • Lipid
a) Nutrient • Electrolytes
• ~-carotene • Vitamins
• a-tocopherol • Trace elements
• Ascorbic acid
25. 'A' [Satyanarayana 3rd & 4th eds 428]
b) Metabolic Hormone Second messenger
• Glutathione FSH, LH, TSH cAMP
• Ceruloplasmin Thyrotropin releasing Phosphatidyl inositol/
• Albumin hormone (TRH), Gastrin, calcium
• Bilirubin Cholecystoki ni n
• Transferrin
• Ferritin
26. 'D' [Satyanarayana 3rd & 4th eds 509]
• Uric acid
Beneficial effects of fiber:

22. 'A' [Satyanarayana 3rd & 4th eds 655]


• Prevents constipation
• Eliminates bacterial toxins
23. 'A' [www.davidsor1.edu/molbiology] • Decreases GIT cancers
Sometimes, it is necessary to quantify amounts of RNA • Improves glucose tolerance
or DNA in a clinical sample. For example, in cases of
• Reduces plasma cholesterol level
human immunodeficiency virus infection, knowledge
of the amount of viral RNA in the bloodstream can be
• Satiety value (gives sensation of stomach
fullness)
a valuable guide to prognosis and treatment.

Fibers slow stomach emptying and they delay and


Standard polymerase chain (PCR) reaction:
attenuate the postprandial rise in blood glucose with
It usually examines the DNA produced after 30 or more
consequent with consequent reduction in insulin
cycles, when it has been amplified 10 crore times or
secretion. This effect is beneficial to diabetics.
more. This is a useful qualitative test, but is not good
for quantification.
27. 'B' [Harper 25th ed 294]
The enterohepatic circulation of the bile salts is so
Realtime PCR:
efficient that each day the relatively small pool of bile
These machines have been developed to measure how
acids (3-5g) can be cycles through the intestine 6-10
much product is formed at very early stages in the
times with only a small amount is lost in the fecesi.e,
reaction when it is proceeding at the maximal rate.
1-2% per pass through ent er hepatic circulation.
However, each day, an equivalent amount of bile acid
The formation of product is followed either by using a
is synthesized from cholesterol by the liver, so that a
dye which fluoresces brightly when bound to DNA or by
pool of bile acids of constant size is maintained.
using some form of fluorescently-labeled hybridization
probe.
28. ' D' [Robbins 7th ed 103]
Basement membranes produced by epithelial and
Sensitive optics and powerful electronics are used so
mesenchymal cells are closely associated with the cell
that very small amounts of products can be detected at
surface. They consist of a network of amorphous non-
early cycles and the process can easily be automated.
fibrillar collagen (Type IV), laminin, heparin sulphate,
proteoglycans and other glycoproteins.
24. ' B' [jap.physiology.org/cgi]
Parenteral nutritional so lutions:
Eutactin and nidogenare closely related to laminin.
• Protein Rhodopsin is a cell membrane spanning protein and is
• Carbohydrate not a component of basement membrane.
Dental ;lut.,e

BIOCHEMISTRY - SYNOPSIS
CARBOHYDRATE METABOLISM

1. Classification of Carbohydrates

Monosaccharides Oligosaccharides Polysaccharides

I
Aldoses
I
Ketoses
Eg.: Glucose Eg.: Fructose

Disaccharides Trisaccharides
I Eg.: Raffinose

Reducing Non-reducing
Eg.: Ma ltose (malt sugar) Eg.: Sucrose (cane sugar)
Lact ose ( milk sugar) Trehalose

Homo Polysaccharides Heteropolysaccharides /


mucopo Lysaccharides
Eg.: Starch Eg.: Hyaluronic acid
Dextrin Keratin sulfate
Inulin Chondroitin sulfate
Glycogen Heparin sulfate
Cellulose

2. If 2 monosaccharides differ from each other in their 7. Glucose is called "energy currency" among carbohydrates.
configuration around a sing le specific carbon atom, ATP is called as energy currency of the cell.
they are known as EPIM ERS.
8. Glucose shows STEREO as well as OPTICAL isomerism.
Eg.: Glucose and Galactose (COMEDK-15) It is a dextrorotatory and is an aldose sugar.

3. A method for synthesis of higher monosaccharide from 9. A glycoside cont ains a sugar molecule (glucose) and a
a lower one is called KILIANI SYNTHESIS. non-sugar molecule. A fructoside contains a (fructose)
sugar molecule and a non-sugar molecule.
4. Inversion of sucrose is the process of changing the
optical rotation from dextrorotatory to levorotatory. Digitalis is example of a glycoside.
The hydrolysed mixture of sucrose containing glucose
and fructose is known as INVERT SUGARS. 10. The end products of carbohydrate digestion are
glucose, fructose, galactose (all are monosaccharides).
5. Starch and glycogen are the carbohydrate reserves of
plants and animals respectively. 11. Glucose absorption occurs chiefly by active transport

6. Cellulose is not digested in humans. 12. Blood sugar values


• Fasting - 80 - 120 mg/100 ml.
Dental ;lut.,e

j) The first step in glycolysis is conversion of d) SHes of various metabolic pathways:


glucose to glucose-6-phosphate by the enzyme
• Citric acid cycle
hexokinase or glucokinase. The last step 1s
conversion of pyruvate to oxaloacetate. • Oxidative phosphorylation
• Fatty acid oxidation Mitochondria
k) Enolase enzyme in inhibited by fluoride. For • Electron transfer
blood glucose estimation in laboratory, fluoride • Synthesis of ketone bodies.
is added to t lhe blood to prevent glycolysis by
Protein synthesis Ribosomes
the cells, so that blood glucose is correctly
estimated. Golgi
Glycoprotei n synthesis
complex
l) Substrate level phosphorylation: RNA synthesis Nucleolus
It is the synthesis of ATP without the involvement
• Cholesterol synthesis
of electron transport chain.
• Glycolysis
• Fatty acid synthesis
m) Examples of substrate level phosphorylation in
glycolysis are • Glycogenesis, Cytoplasm
phosphoglycerate • Glycogenolysis
• 1,3-biphospho 3- phospho • HMP shunt
glycerat e kinase glycerate • Gluconeogenesis

pyruvate e) Krebs cycle involves the combination of a two-


• Phosphoeno l pyruvate Pyruvate carbon acetylCoA with a four-carbon oxaloacetate
kinase to produce a six-carbon tricarboxylic acid,
citrate.
• Example of substrate level phosphorylation in
citric acid cycle is f) 12 ATP are formed from one acetyl CoA, thus a
total of 24 are formed in citric acid cycle.
Succinate
Succinyl CoA - - - -~ Succinate. g) Krebs cycle is both cat abolic and anabolic in
thiokinase nature, hence regarded as amphibolic.

14. c;tric acid cycle /Krebs cycle/TricarboxyHc acid 15. Rapaport - leubering cycle:
cycle I TCA cycle. It occurs in erythrocytes and is mainly concerned with
a) Citric acid cycle involves the oxidation of acetyl production of 2,3 - BPG. (2,3 -biphosphoglycerate).
CoA to Co 2 and H20. In presence of 2,3 - BPG, oxyhemoglobin un loads
more oxygen to the tissues.
b) This cycle is the final common oxidative pathway
for carbohydrates, fats and amino acids. This
Increase in erythrocyte 2,3 BPG is seen in hypoxic
cycle utilizes about 2/3rds of total oxygen
conditions, high altitude, fetal tissues, and anemic
consumed by the body.
conditions. In all these cases 2,3 BPG will enhance
the supply of oxygen to tissues.
The name TCA cycle is used, since, at the out set
of cycle, t ricarboxylic acids (citrate, cisaconite
and isocitrate) are participated.

c) The enzymes of TCA cycle are located in


mitochondrial matrix.
BIOCHEMISTRY SYNOPSIS

16. 19. Glucornic acid path way is concerned with


Conversion of glucose to synthesis of glucornic acid, pentoses and vitamin C.
Glycolysis pyruvate or lactate, with L-gulonolactone oxidase converts gulonate to ascorbic
production of ATP acid. As this enzyme is absent in man , Vitamin C
should be supplemented in t he diet.
• Synthesis of glucose
or glycogen from non-
carbohydrate compounds. 20. Glucose is phosphorylated to Glucose-6-phosphate in
Gluconeogenesis presence of Glucokinase or Hexokinase enzyme. This
(regulated by • Lactate, pyruvate, glucogenic reaction is important, because, glucose should be
aminoacids, proprionate and
glucagon of phosphorylated first to enter any met abolic process.
glycerol are the substrates.
pancreas) This reaction is the first step mediated by glucokinase
• Gluconeogenesis fro m lactate
occurs in skeletal muscle and in glucogenesis. The formation of "glucose-6-
is known as CORI cycle. phosphate" is a locking mechanism by which glucose
is kept with in the cell.
• Synthesis of glycogen from
Glycogenesis
glucose is glycogenesis.
21. Glucose-6-phosphatase is absent in muscle and present
• Synthesis of glucose from in liver.
Glycogenolysis glycogen.
• Occurs in Liver and muscle. 22. Starch consists of two polysaccharide components,
a water-soluble amylose and a water insoluble
17. Rapaport - leuberi ng cycle is a supplemental pathway amylopectin.
to glycolysis. It is mainly concerned with the synthesis
of 2,3 -BPG in the RBC. HMP shunt (pentose phosphate
pathway or phosphogluconate pathway) and uronic LIPID METABOLISM - II
acid pathway (glucornic acid pathway) are considered
as alternative path ways to glycolysis. 1. Palmitoleic acid, oleic acid, linoleic acid, linolenic acid
and arachidonic acid are the five unsatured fatty acids.
18. Significance of HMP pathway lies in generation of two
important product s - pentoses and NADPH . Among them palmitoleic acid, oleic acid are mono
unsaturated fatty acids. Linoleic acid, linolenic acid
a) Importance of pentoses. and arachidonic acid are essential unsaturated fatty
• Synthesis of nucleic acids (DNA, RNA) acids.
• Synthesis of nucleotides (ATP, NAO, FAD & COA)
Arachidonic acid becomes essential, if its precursor,
b) Importance of NADPH . Linoleic acid is not present in the diet. It is considered
as the precursor for the synthesis of prostaglandins.
• Synthesis of fatty acids, steroids and amino
acids.
2. The fatty acids in the body are mostly oxidized by
• Formation of reduced glutathione.
B-oxidation. Acetyl CoA is end product of fatty acid
• Maintains the integrity of RBC membrane. oxidation. It enters the citric acid cycle and gets
• Keeps the Fe2 in reduced state so that completely oxidized to CO 2 and H20.
accumulation of met hemoglobin (Fe+3 ) is
prevented. 3. 3 molecules of acetyl CoA are condensed to form
p-hydroxy p-methyl glutaryl CoA (HMG CoA). HMG CoA
Deficiency of Glucose-6-phosphate dehydrogenase, synthase, cat alyses this reaction, thus regulates the
an enzyme involved in HMP shunt will result in synthesis of Ketone bodies.
accumulation of methemog lobin and peroxides in
eryt hrocytes leading to hemolysis.
Dental ;lut.,e

4. Ketone bodies iii) 7 - dehydro cholesterol, an intermediate product


in the synthesis of cholesterol is converted to
i) Acetone, acetoacetate and B-hydroxybutyrate are
cholecalciferol (vitamin D3) by ultraviolet rays in
known as Ketone bodies. But B-hydroxybutyrate
the skin .
does not possess a keto group. When
compared to acetone both acetoacetate and
iv) HMG COA reductase is the rate limiting enzyme
B-hydroxybutyrate are strong acids.
in cholesterol synthesis.
ii) Ketone bodies are water-soluble and energy
v) High density li poproteins (HDL) and the enzyme
yielding. However, acetone is an exception,
lecithin - cholesterol acyl transferase (LCAT) are
since it cannot be metabolized.
responsible for the transport and elimination of
cholesterol from the body.
iii) The synthesis of ketone bodies occur in liver and
the enzymes are located in the mitochondrial
vi) Plasma cholesterol is associated with different
matrix
lipoprotein fractions (LD L,VLDL and HDL).
Significance of Ketone bodies:
Elevation in plasma HDL - cholesterol is
iv) The production of Ketone bodies and their benefi cial to the body, since it protects the
utilization become more significant when body from atherosclerosis and coronary heart
glucose is in short supply to the tissues, as in diseases (CHO). Increase in plasma LDL -
cases of starvation and diabetes mellitus cholesterol is harmful to the body as it may lead
to coronary heart diseases.
v) During prolonged starvation Ketone bodies
are the major fuel source for the brain and vii) The normal plasma cholesterol level is in the range
other parts of CNS. There will be increased of 150 - 250 mg/dl. Increase in plasma cholesterol
degradation of fatty acids (from the fuel reserve (> 250 mg/dl) is known as hypercholesterolemia
triacylglycerol) to meet the energy needs of the and is seen in diabetes mellitus, hypothyroidism
body. This causes an over production of acetyl ( myxoedema), obstructive Jaundice and nephrotic
CoA and consequently ket one bodies. syndrome.

vi) Diabetes Mellitus is associated with insulin 6. Fatty liver


deficiency and this results in impaired The normal concentration of lipid in liver is about
carbohydrat e metabolism and increased lipolysis. 5%. But in certain conditions, lipids especially the
Both of these ultimately results in accumulation triacylglycerols accumulate excessively in liver, and
of acetyl CoA and its conversion to ketone results in fatty liver.
bodies. Diabetic Keto acidosis is dangerous
and may result in coma, and even death, if not Condition where fatty liver occurs
treated. • Diabetes mellitus
• Starvation
5. Cholesterol:
• Alcoholism
i) Cholesterol is found exclusively in animals. It is • Highfat diet
synthesized mostly in liver (50%) and intestine
• Impaired synthesis of lipo proteins
(15%).
• Deficiency of lipotropic factors like choline, betaine,
ii) Derivatives of cholesterol. methionine and inositol.
• Steroid hormones
7. The sudden death of infants in sudden infant death
• Bile acids
syndrome is due to a blockade in B-oxidation
• Synthesis of Vitamin - D caused by a deficiency of medium chain acyl CoA
dehydrogenase.
~
BIOCHEMISTRY SYNOPSIS 927
V

PROTEIN METABOLISM - III • Lysine


Basic amino acids • Arginine
1. CLASSIFICATION OF AMINO ACIDS:
• Histidine

a) Based on ability of the body to synthesize • Asparagine

Essential Non-essential Semi Acidic amino acids


• Aspartic acid
(cannot be amino acids (can essential • Glutamic acid
synthesjzed by be synthesjzed by • Glutamine
the body) the body)
Amino acid with indole ring is • Tryptophan
• Arginine • Alanine • Arginine
Amino acid with imidazole ring is • Histidine.
• Valine • Glycine • Histidine
Amino acid with dica rboxylic • Aspartic acid
• ffjstjdine • Serine
• Isoleucine
group • Glutamic acid
• Leucine Amino acid which is a methyl
group donor • Methionine
• Lysine
• Methionine Optically inactive aminoacid is • Glycine
• Phenyl Amino acid which is present in
alanine peptides and absent in proteins • Ornithine

• Threonine
• Tryptophan 3. Transamination:
The transfer of amino group (-NH 2) from an amino
b) Based on metabolism: acid to a keto acid is known as transamination. This
reaction is mediated by a group of enzymes called
. Glycogenic & .
Ketogenic Kt . Glycogenic transaminases. All the transaminases require pyridoxal
e ogenic
phosphate, a coenzyme derived from Vitamin B6.
• Leucine • Phenyl alanine Rest of 20
• Lysine • Isoleucine amino acids
Transamination reaction is a reversible process and
are only
• Tyrosine is responsible for synthesis of non-essential amino
glycogenic
• Tryptophan acids. Transamination also diverts the excess amino
acids towards energy generation. All the amino acids
c) Based on structure· . except lysine, threonine, praline and hydroxyproline
participate in transamination.
• Glycine
• Alanine
Which pair amino acids, amongst the following do not
• Serine participate in transamination at some point in their
Neutral aliphatic amino acids • Threonine catabolism? (COMEDK-2015)
• Valine a) Leucine, threonine b) Lysine, t hreonine
c) Leucine, tyrosine d) Lysine, tyrosine
• Leucine
• Isoleucine
4. Urea cycle I Krebs-Henseleit cycle
• Phenyl alanine Urea is the end product of protein metabolism. The
Neutral aromatic amino acids • Tyrosine nitrogen of amino acids is converted to ammonia.
• Tryptophan Ammonia is converted to urea and detoxified. The first
two enzymes of urea cycle is present in mitochondria
Neutral heterocyclic Praline
while the remaining three are located in cytosol.
• Cystine
Neutral sulphur containing • Cysteine The first reaction catalysed by carbamoyl phosphate
• Methionine synthase is rate-limiting reaction in urea synthesis.
~
BIOCHEMISTRY SYNOPSIS 929
V

• In HARTNUP'S DISEASE, tryptophan absorption vi) GLUTAMATE AND GLUTAMINE:


by the intestinal and rena l epithelial cells is • Glutamate is present in 2, 7, 9, 10 clotting
most severely affected with a result that typical factors as Y-carboxyg tutamate.
symptoms of pellagra are observed in patients of
Hartnup's disease. This is related to impairment • In t he brain, glutamate is decarboxylated
in the conversion of tryptophan to NAO+ and to GABA. GABA functions as a inhibitory
NADP+, the coenzymes of niacin. neurotransmitter.
7.
iv) SULFUR AMINO ACIDS:
• Cystine, Cysteine and methionine are sulfur Metabolic defect
Amino acid Disorder
(enzyme/other)
containing aminoacids
Phenyl Phenylalanine
• Methionine is responsible for t ransmet hylation ketonuria hydroxylase
Phenyl
reactions during synthesis of compounds t hat alani ne & Homogentisate
Alkaptonuria
contain methyl group. Tyrosine oxidase
Albinism Tyrosinase
• Homocysteine formed from methionine is a
precursor for t he synt hesis of cystei ne. Sulfur amino Cystathionine
Homocystinuria
acids synthase
• HOMOCYSTIN URIA is characterized by Hartnup's Defective intestinal
Tryptophan
accumulation and increased urinary excretion disease absorption
of homocysteine and S- adenosylmethionine Branched Branched chain
due to a defect in the enzyme cystathionine Maple syrup
chain amino a -keto acid
synthase. urine disease
acids dehydrogenase

iv) BRANCHED CHAIN AMINO ACIDS (VALINE,


ISOLEUCINE, LEUCINE) ENZYMES
MAPLE SYRUP URINE DISEASE is a disorder of
branched chain amino acids. The urine of the 1. The functional unit of enzyme is known as holoenzyme,
affected individual smells like maple syrup or which is often made up of apoenzyme (the protein
burnt sugar. Maple syrup urine disease is due to part) and a coenzyme (non-protein part).
defect in the enzyme branched chain a-keto acid
dehydrogenase. 2. The term activator is referred to the inorganic cofactor
necessary to enhance the activity of t he enzyme.
v) HISTIDINE, PROLINE, ARGININE: Cofactor Enzyme
• Histidine is present in heme part of hemoglobin.
• Dehydrogenase
• Histidine is excreted in urine in pregnancy. Zinc • Alkaline phosphatase
• Carbonic anhydrase
• In case of folic acid deficiency, N-formimino-
Copper
• Tyrosinase
glutamate (FIG LU), which is formed in histidine • Cytochrome oxidase
metabolism accumulates and is excreted in • Hexokinase
urine. Histidine load test utilizing the excretion
of FIGLU in urine is used to assess fo lic acid
Magnesium • Glucokinase

deficiency.
• Phosphofructoki nase
Potassium • Pyruvate kinase
• Histidine, proline and arginine are converted to Iron • Peroxidase
glutamate in their metabolism.
Manganese
• Phosphatase
• Glycosyl transferase
Dental ;lut.,e i======

Folic acid • Required for one carbon • Macrocytic anemia, glossitis. • Glossitis:
metabolism. • Aminopterin and methotrexate The filiform papillae
• Required for synthesis of amino are structural analogues of disappear first, but in
acids (glycine, serine), purines folic acid used in treatment advanced cases the
and pyrimidines of many cancers including fungiform papillae are
lost and the tongue
• Tetra hydrofolate (TH F or FH J is leukemia. These drugs block
the formation of THF and becomes smooth and
the active form of fo lic acid
hence DNA synthesis is fiery red in colour.
impaired.
Vit. B1 2 • Also known as erythrocyte • Pernicious anemia • Beefy red tongue with
maturation factor • Neurological manifestations glossopyrosis, glossitis
(anti
• Synthesized by microorganisms due to degeneration of and g lossodynia.
pernicious
vitamin or only. Plant cannot synthesize Vit. posterior and lateral tracts of • Hunter's glossitis or
extrinsic B12 • Animals obtain B12 by eating spinal cord. Moellers glossitis,
factor of foods derived from other animals • Degeneration of myelin sheath which is similar to "bald
castle) or from intestinal bacterial and peripheral nerves also tongue of sand with"
synthesis occurs. seen in pellagra.

MISCELLANEOUS
QUESTION ANSWER
• Water soluble vitamirns are B, C
• Fat soluble vitamins are A, D, E, K
• Vitamins required for tooth development and calcification are
• Enamel hypoplasia is seen in association with deficiency of A, D vitamins
• Hyper vitaminosis is seen in association with which vitamins
• Vitamin involved in collagen synthesis is
• Vitamin used in treatment of methemoglobinemia is
Vit. C.
• Vitamin that acts as a respiratory quotient
• Gingiva is most commonly affected by deficiency of
• Vitamins involved in electron transfer
Vitamin K & B12
• Heat stable and light sensitive vitamins
• Heat labile vitamins are Vit. C, Folic acid, Biotin
• Vitamin that cannot cross placenta
• Vitamin with its action similar t o a hormone
Vit.D
• Vitamin that is synthesized in the skin
• Vitamin stored in fat
• Vitamin stored in liver A, D, K, B12 and foliate
• Vitamin present in cereals Thia mine
• Water soluble vitamirn which is synthesized in the body is Pantothenic acid
• Vitamin that is present in animal food only B12 & D vitamins
• Raw beef tongue and bald tongue of sandwith is seen in deficiency of Niacin
• Magenta colored tongue is seen in deficiency of Riboflavin
~
BIOCHEMISTRY SYNOPSIS 935
V

• Moeller's glossitis or Hunter's glossitis is seen in deficiency of Vit.B 12


• Vitamin that is associated with neonatal jaundice is Vitamin K
• Vitamin necessary for absorption of bile salts Vit. D & K
• Ant i oxidant vitamins are Vitamin A, E, and C
• Vitamins associated with peripheral neuritis are Vitamin B1 , B1 2, B6 , E
• Vitamins required for wound health Vitamins. A & C
• People consuming polished rice as staple food suffer from deficiency of Vitamin B1
• People taking only maize as staple diet suffer from deficiency of Niacin due to low tryptophan
• Toad skin (phrynoderma) is seen in deficiency of Vitamin A and Essential fatty acids
• Vitamin involved in energy releasing reactions
Thiamine (B 1 )
• Vitamin essential for the enzyme pyruvate dehydrogenase.
FMN & FAD of Riboflavin (B 2 ) and NAO+
• Vitamin in volved in redox reactions
& NAOP+ of Niacin

• Vitamin that inhibits lactation


• Vitamin that is used in treatment of homocysteinuria
Vit. B6
• Vitamin necessary for transamination, decarboxylation, deamination,
transulfuration reactions.
• Vitamin that is invo lved in one carbon transfer reaction Tetrahydrofolate (THF) of folic acid
• Vitamin that participates as a coenzyme in carboxylation reactions of Biotin (Vit. 87 or anti egg white injury
gluconeogenesis, fatty acid synthesis factor).
• Vita min, which is related to cofactor in glycine metabolism Folic acid
Vit. B12' It is absorbed from terminal
• Erythrocyte maturation factor
ileum.
Pantothenic acid (B 5)
• The functions of which vitamin are carried through coenzyme A. (It serves as carrier of activated acetyl or
acyl groups)
• Vitamin that causes fncephalomalacia Vitamin E

2. HOL and the enzyme lecithin-cholesterol acyl 6. Respiratory quotient of fat, protein and carbohydrates
transferase are responsible for transport and are 0.7, 0.8 and 1 respectively.
elimination of cholesterol from the body.
7. The substance from which ammonia is produced in
3. Maltose or malt sugar is produced during the course kidney is glutamine.
of digestion of starch by the enzyme amylase. It is
composed of 2 glucose units. 8. Double helix is present in DNA. Triple helix is present
in collagen.
Sucrose (cane sugar) is made of glucose and fructose.
9. Kwashiorkor 1s due to protein deficiency wit h
4. Lactose (milk sugar) is composed of galactose and sufficient calorie intake. It occurs in children between
glucose. 6 months and 3 years of age. Enlarged fatty liver,
atrophy of different tissues and organs but preserved
5. Sucrose is both dextro rotatory and levo rotatory and subcutaneous fat and adipose tissues are features of
this is known as in vert sugars. Kwashiorkor.
~
BIOCHEMISTRY SYNOPSIS 937
V

27. Schilling test is used to detect Vitamin B12 defi ciency.

28. Salkowski test is a test for cholesterol.

29. Blue doper syndrome is associated with t ryptophan


malabsorption.

30. Gusten, a zinc containing protein of the saliva is


important for t aste sensation.

31. Zellweger syndrome is due to absence of peroxisomes


in all most all tissues, as a result of which long
chain fatty acids are not oxidized and accumulated
in live and brain. So this syndrome is also called as
cerebrohepatorenal syndrome.
Dental ;lut.,e

• It accelerates the activity of secretin to produce 2lt... 'B' [Sembulingam 4th ed 210/ 5th ed 224]
more alkaline pancreatic juice. PHASES OF GASTRIC SECRETION

li.... 'C' [Sembulingam 4th ed 201/ 5th ed 215] Cephalic


Purely under nervous (vag us) control
phase
1L. 'D' [Sembulingam 4th ed 211/ 5th ed 224] Gastric This phase is under nervous and
Functions of gastri n are: phase hormonal control.
• Stimulates secretion of HCl and pepsinogen by It lS under cont rol of various
gastric glands. Intestinal
gastrointestina l hormones like gastrin,
phase
• Increases motility of stomach enterogastrone, VIP and GIP, etc.
• It causes secretion of pancreatic juice rich in
enzymes. 2..5.... 'D' [Sembulingam 4th ed 220/ 5th ed 234]

1.8... 'A' [Sembulingam 4th ed 251/ 5th ed 265] 2..6... 'A' [Sembulingam 4th ed 236/ 5th ed 250]
The amylolytic enzymes present in intestinal juice
li.. 'B' [Sembulingam 4th ed 220/ 5th ed 234] convert disaccharides into monosaccharides (glucose
Duodenum and jejunum secretes secretin and and fructose)
cholecystokinin whereas stomach pyloric glands
produce gastrin. Secretin cause secretion of pancreatic 2J...... 'A' [Sembulingam 4th ed 211/ 5th ed 225]
j uice with high water and bicarbonate ion content.
Cholecystokinin ca uses secretion of pancreatic juice 2..8... 'B' [Sembulingam 4th ed 251/ 5th ed 265]
rich in enzymes.
2..9.... '('
Option 'C' gastric motility is increased by gastrin.
3.0.. 'B' [Sembulingam 4th ed 220/ 5th ed 234]
2..0.... 'A' [Sembulingam 4th ed 22~ 5th ed 23~
Bile is secreted in liver by hepatocytes. Cholirectic action 3..1.. 'C' [Sembulingam 4th ed 219/ 5th ed 233]
means sti mulation of bile secretion from liver. From
the liver, bile enters gall bladder where large amounts .12... 'A' [Sembulingam 5th ed 269]
of electrolytes and waters are absorbed rendering the
bile concentrated with bile sa lts, bile pigments and 3..3.... 'A' [Sembulingam 4th ed 231/ 5th ed 246]
cholesterol, etc. Cholagogue action means increased
release of bile from gall bladder into intestine. .3lt... 'C' [Sembulingam 4th ed 210/ 5th ed 224]

2.1.. 'A' [Sembulingam 4th ed 206/ 5th ed 220] l£.. 'C' [Sembulingam 4th ed 226/ 5th ed 239]

2..2.._ 'B' [Sembulingam 4th ed 220/ 5th ed 234] 36. ' B' [Sembulingam 4th ed 215/ 5th ed 229]
SOME PH VALUES
2..3.... 'C' [Sembulingam 5th ed 251, 275]
Glucose is absorbed by sodium co-transport i.e., active Gastric juice il...9. - 1.2

transport. Fructose is absorbed by facilitated diffusion Bile juice in liver 8. - 8..fi_


and pentoses are absorbed by means of simple diffusion.
Bile juice in gall bladder 8. - .8....1
Saliva .6...3..5. - 6...8..5.

3L. 'C' [Sembulingam 4th ed 209/ 5th ed 223]


A part of stomach is incompletely separated from the
main portion and made into a small bag like pouch,
which is called as pavlov's pouch.
Dental ;lut.,e

on the walls of bronchi and bronchioles of lungs. Cyanide poisoning results in hi stotoxic hypoxia. In this
During inspiration, when there is stretching of lung Pa0 2 is normal but the tissue does not utilize oxygen
tissues due to expansion, the stretch receptors are as the poisonous substances destroy the cytochrome
stimulated and produce impulses, which are carried by oxidase system.
vagal afferent fi bers to the respiratory centers.
29. 'C' [Sembulingam 4th ed 622/ 5th ed 660]
Hering-Breuer reflex inhibits the inspiratory center,
stops inspiration and stimulates expiration. This is a 30. 'D' [Sembulingam 4th ed 653/ 5th ed 694]
protective reflex which restricts the inspiration and
limits the over stretching of lung tissues. 31. 'B' [Sembulingam 4th ed 632/ 5th ed 670]
Perfusion means the rate of blood flow to the tissues.
18. 'A' [Sembulingam 4th ed 618/ 5th ed 656] Blood flow to the apex of lung is less, so ventilation
perfusion ratio is maxim um in the apex of lung whereas
19. 'D' [Sembulingam 4th ed 662/ 5th ed 703] blood flow to the lower part of lung is maximum, so
the ventilation perfusion ratio is minimum.
20. 'C' [Sembulingam 4th ed 630/ 5th ed 668]
• Pulmonary ventilation = Tidal volume x 32. 'A' [Sembulingam 4th ed 653/ 5th ed 694]
Respiratory rate
33. 'B' [Sembulingam 4th ed 642/ 5th ed 683]
• Alveolar ventilation = (Tidal volume - dead The presence of Co 2 decreases the affinity of
Space) x respiratory rate hemoglobin for 0 2 and enhances release of oxygen to
the tissues shifting the 0 2 dissociation curve to the
21. 'C' [Sembulingam 4th ed 612/ 5th ed 649] right. This is known as "Bohr effect".

22. 'D' [Sembulingam 4th ed 641/ 5th ed 682] Combination of more amount of 0 2 with hemoglobin
HbF has high affinity for oxygen than adult hemog lobin. displaces Co 2 from hemoglobin. The excess of 0 2
So the curve shifts to left. concentration results in shift of the carbon dioxide
dissociation curve to right. This is known as "Haldane
23. 'B' [Sembulingam 4th ed 641/ 5th ed 682] effect".

24. 'B' [Sembulingam 4th ed 615/ 5th ed 653) 34. 'B' [Sembulingam 4th ed 644/ 5th ed 685)
Primary inspiratory muscles are diaphragm, external
intercostals muscles. Accessory inspiratory muscles are 35. 'C' [Sembulingam 4th ed 617/ 5th ed 655)
sternomastoid scalenic and anterior serrati.
36. 'B' [Sembulingam 4th ed 637 / 5th ed 675)
Primary expiratory muscle is internal intercostals The partial pressure of oxygen in mixed venous blood
muscles. Accessory expiratory muscles are abdominal is 40mm Hg and in arterial blood is 95mm Hg. The
muscles. oxygen content of arterial blood is 19ml% whereas the
oxygen content of venous blood is 14ml%.
25. 'B' [Sembulingam 4th ed 654/ 5th ed 695)
3 7. 'D' [Sembulingam 4th ed 664/ 5th ed 705)
26. 'A' [Sembulingam 4th ed 662/ 5th ed 703) In high altitudes there is increase in red blood cell
count leading to increased 0 2 carrying capacity of
27. 'A' [Sembulingam 4th ed 641/ 5th ed 682) blood. Respiratory rate, heart rate, force of contraction
increases initially, but fina lly reduces.
28. 'C' [Sembulingam 5th ed 695, 700)
Anemia, CO poisoning causes anemic hypoxia in which But due to low barometric pressure, partial pressure of
the 0 2 content of blood is reduced as the amount of oxygen is reduced leading to hypoxic hypoxia.
hemoglobin combining with 0 2 is reduced. Pa0 2 is
normal.
, PHYSIOLOGY
~

V
951

38. 'A' [Sembulingam 4th ed 390/ 5th ed 406) 43. 'B' [Ganong 22nd ed 692)
Decrease in pH of blood or severe acidosis increases Hypercapnia is retention of CO2 in the body. It causes
the rate and depth of respiration called Kussmaul respiratory acidosis and decreases pH of blood as the
breathing plasma HC0 3• may exceed 40meq/L.

39. 'D' [Sembulingam 4th ed 641/ 5th ed 682) Symptoms:


• Confusion
40. 'D'
• Diminished sensory acuity

41. 'A' [Sembulingam 5th ed 659)


• Coma with respiratory depression
Compliance: • Death
The ability of the lungs and thorax to expand or it is
expansibility of lungs and thorax. 44. 'K [Berne's Physiology 5th ed 488)
Blood flow to t he lungs occurs by pulomonary
Work of breathing: and bronc hial circulation. Pulmonary circulation
It is the work done by the respiratory muscles during constitutes the netire output of rig ht ventricle. It
breathing to overcome the resistance in the thorax consists of the mixed venous blood. It is this blood
and respiratory tract. t hat underges geaseous exchange with the alveolar
air.
During work of breathing, energy is utilized to
overcome 3 types of resistance Bronchial circulation forms way minor part of blood
supply to lungs and constitutes a very small part of
i) Airway resistance is the resistance offered to output of t he left vent ricle. This circulation may be
the passage of air through respiratory t ract. It is important in t he airconditioning of t he inspired air.
directly proportional to work of breathing.
45. 'B' [Ganong 22nd ed 690, 684)
ii) Elastic resistance of lungs and thorax • CO poisoning is often referred as a form of anemic
(compliance work) indirectly proportional to hypoxia. Hemoglobin affinity fo r CO is 210 times
work of breathing. than 02. 02 content in blood is reduced.
• When COHb is present, the dissociation curve of
iii) Non-elastic viscous resistance the remaining Hb02 shift to the left, decreasing
Energy is required to overcome the viscosity of the amount of 02 released.
lung tissues and tissues of thoracic cage.
46. 'C' [Guyton 11th ed 540)
42. 'A' [Sembulingam 5th ed 656, Fig. 120-1) • At high altit ude the maximum rate of oxygen
NORMAL VALUES OF INTRAPLEURAL PRESSURE uptake is reduced.
• At the start of inspiration • Work capacity is directly proportional to the
-2mm Hg workload (resistance) and duration of exercise.
• At t he end of expiration
• So work capacity can be increased theoretically
• At the end of inspiration by either:
-6mm Hg
• At the start of expiration - increasing the workload or
At the end of forced inspiration with - increasing the duration of exercise
-70mm Hg
closed glottis (Muller's maneuver)
• But as the rate of oxygen uptake is limited in
At the end of forced expiration -30mm Hg high altit udes, we shall not be able to achieve
At the end of forced expiration with an increase in work capacity by increasing t he
+50mm Hg workload.
closed glottis (Valsalva maneuver)
• So, t o increase the work capacity we shall therefore
have to increase the duration of exercise.
, PHYSIOLOGY
~

V
955

18. GFR is increased when 27. The renal clearance of:


a) Plasma oncotic pressure is increased a) Creatinine indicates glomerular filtration rate
b) Glomerular hydrostatic pressure is decreased b) Inulin is lower than that of urea
c) Tubular hydrostatic pressure is increased c) Chloride increases after an injection of aldosterone
d) Increased renal blood flow d) PAH continues to rise as the plasma concentration
of PAH increases.
19. What is the glomerular filtration rate?
a) 100 ml/min b) 125 ml/min 28. Two substances that can probably be used to
c) 150 mljmin d) 175 mljmin determine filtration fraction are :
a) Inulin and mannitol
20. Major portion of glomerular filtrate is absorbed in: b) Urea and diodrast
a) Loop of Hen le b) Distal convoluted tubule c) PAH and phenol red
c) Collecting duct d) Proximal segment d) Inulin and PAH

21. Renin secretion is stimulated by all except: 29. Which of the following is important in renal
a) Cardiac failure excretion of hydrogen ion?
b) Low Na+ in proximal tubule a) Combining H+ with Cl- to form Hcl
c) Sympathetic stimulation b) Combining H+ and HC0 3 - via enzyme carbonic
d) High Na+ in proximal tubule anhydrase
c) Trapping H+ by acetate
22. Proximal renal tubule is most permeable to: d) Trapping H+ by NH3+
a) Sodium b) Potassium
c) Bicarbonate d) Ammonia 30. Inulin clearance closely resembles:
a) G.F.R b) Renal plasma flow
23. The hyperosmolarity of the renal medulla is due to c) Creatinine clearance
increased content of: d) P.A.H clearance
a) K+ b) Na+
c) Glucose d) NH4+ 31. Macula densa in kidney is located in relation to
a) PCT b) OCT
24. In renal glycos uria, the renal threshold for c) Afferent arteriole d) Efferent arteriole
glucose is:
a) Low b) High 32. Diuresis is caused by:
c) Same d) Greatly increased a) Mannitol b) Glycerol
c) Urea d) All of the above
25. True about nephron is:
a) Na is absorbed actively in descending loop of Henle 33. Ammonia in the kidney tubules is excreted in
b) 60 to 70% of GFR is absorbed in proximal tubule exchange for:
c) Absorption of water occurs in ascending loop of a) Hco 3-
Hen le c) (l·
d) The filtrate reaching distal convoluted tubule is
hypertonic with respect to surroundings 34. The renal blood flow (in ml/mt) is:
a) 250 b) 800
26. Which one of the following is not responsible for c) 1260 d) 1500
concentration of urine in the kidneys :
a) Aldosterone b) Angiotensin II 35. Site of ADH action is:
c) Vasopressin d) Epinephrine a) Proximal tubule b) Loop of Henle
c) Vasa recta d) Collecting tubule

18) D 19) B 20) D 21) D 22) A 23) B 24) A 25) B 26) D 27) D 28) D 29) D 30) A
31) B 32) D 33) B 34) C 35) D
Dental ;lut.,e

36. Most of the sodium is reabsorbed in:


a) PCT b) DCT
c) Loop of henle d) Vasa recta

37. In which segment of the nephron does tubular fluid


has the highest osmolarity?
a) Henle's loop b) Distal tubule
c) Proximal tubule d) Collecting duct
(AIPG -98)
38. Tubuloglomerular feedback occurs in
a) ADH secretion in collecting duct
b) Constant solute foad on distal tubule
c) Macula densa
d) Balance between ascending and descending loop of
hen le
(AIPG-10)
39. The part of nephron "least permeable to water" is
a) Proximal tubule
b) Descending limb of loop of Henle
c) Ascending limb of loop of Henle
d) Collecting tubule
(KAR-04)
40. In clinical practice Glomerular Fittration Rate (GFR)
is estimated using the plasma clearance value of
a) Inulin b) PAH
c) Glucose d) Creatinin
(COMEDK-06)
41. The reabsorption of sodium chloride is increased in
proximal convoluted tubules by hormones secreted
from:
a) Anterior pituitary b) Posterior pituitary
c) Adrenal cortex d) Adrenal medulla
(AIPG-07)
42. Tubular maximum for glucose is
a) 180 mg/dl b) 325 mg/min
c) 375 mg/min d) 375 mg/dl
(COMEDK-10)
43. The clearance of a substance when its concentration
in plasma is 15 mg/dL, its concentration in urine is
150 mg/dl and urine flow is 2 ml/min is
a) 2 ml/min b) 20 ml/min
c) 125 ml/min d) 200 ml/min
(COMEDK-15)

36) A 37) A 38) B 39) C 40) D 41) C 42) C 43) B


, PHYSIOLOGY
~

V
957

3. RENAL SYSTEM EXPLANATORY ANSWERS


1. 'C' [Sembulingam 4th ed 309/ 5th ed 323] (Rennin is an enzyme released from the chief cells of
Normal urinary output 1.15 lts. fundic glands of stomach. It is a milk-curdling enzyme.
It is absent in man).
Increased urinary output Polyuria
Reduced urinary output is Oliguria 6. 'C' [Sembulingam 4th ed 285/ 5th ed 297]
ADH/ Vasopressin is secreted in the supraoptic nucleus
Stoppage of urine formation Anuria
of hypothalamus and is released from the posterior
Proteins in urine Protein uria pituitary.
R.B.C. in urine Haematuria
Anti diuret ic hormone is synthesized in the (AP-15)
Excess accumulation of urea and creatine Uremia
a) Cells of Posterior Hypophysis
Excess glucose in urine Glucosuria b) Hypothalamic nuclei
c) Anterior pituitary d) Pars intermedia
2. ' D' [Sembulingam 4th ed 319/ 5th ed 333]
Some substances i11crease the osmotic pressure in the Note: The initial key given for this question was
tubular fluid. This increased osmotic pressure prevents option D i.e., Pars intermedius. The fact is Pars
the reabsorption of water leading to excretion of intermedia connects adeno and neurohypophysis
excess water through urine. This is called osmotic and secretes melanocyte stimulating hormone in
diuresis. Mannitol, urea, sucrose and glucose are the lower vertebrates.
best osmotic diuretics.
7. 'C' [Sembulingam 4th ed 277/ 5th ed 289]
3. 'C' [Sembulingam 4th ed 307/ 5th ed 321]
A substance that is completely filtered but neither 8. 'C' [Sembulingam 4th ed 307/ 5th ed 321]
absorbed nor secreted should be used to measure GFR.
Inulin is ideal substance used to measure GFR. 9. ' B' [Sembulingam 4th ed 281/ 5th ed 293]
Brush border is seen in proximal convoluted tubule.
A substance which is filtered and secreted but not Hairpin Bend is seen in Loop of Henle.
absorbed should be used to measure Renal plasma /
blood flow. PAH (Para amino hippuric acid) is ideal for 10. ' D' [Sembulingam 4th ed 300/ 5th ed 313]
this.
11. 'A' [Sembulingam 4th ed 278/ 5th ed 290]
4. 'C' [Sembulingam 4th ed 303/ 5th ed 317] Nephron is the functional and structural unit of kidney.
In the renal tubular cells, Co 2 combines with water Each kidney consists of 1-1.3 millions of nephrons.
to form carbonic acid under the influence of carbonic
anhydrase enzyme present in the renal tubular cells. 12. 'A' [Sembulingam 4th ed 295/ 5th ed 308]
The carbonic acid dissociates into bicarbonate and Glucose is completely reabsorbed actively in the
hydrogen ion. Bicarbonate is absorbed into extracellular proximal convoluted tubule if its concentration is
fluid and the hydrogen ion is secreted in exchange for below 180 mg% in blood. This is called the renal
sodium ion absorption. threshold of glucose. Above renal threshold, glucose
appears in urine.
5. 'B' [Sembulingam 4th ed 284/ 5th ed 296]
Renin and erythropoietin are the hormones released 13. 'A' [Sembulingam 4th ed 295/ 5th ed 308]
from the Juxtaglomerular apparatus of kidney. Renin
stimulates the conversion of Angiotensinogen to 14. 'A' [Sembulingam 4th ed 289/ 5th ed 301]
Angiotensin II, wnich further stimulates the adrenal
cortex to secrete a ldosterone. This is called the Renin
-Angiotensin- Aldosterone axis.
, PHYSIOLOGY
~
959
V

Glomerulotubular balance:
An increase in GFR causes an increase in the
reabsorption of solutes, and consequently of water,
primarily in the proximal tubule, so that in general the
percentage of the .solute reabsorbed is held constant.
This process is called glomerulotubular balance and it
is particularly prominent for Na+.

39. 'C' [Sembulingam 4th ed 301/ 5th ed 314]


In the descending loop of Henle, water is reabsorbed
from the tubule into the interstitium by means of
osmosis. This segment is highly permeable to water;
so the fluid inside this segment becomes hypertonic.

The ascending loop of Henle, is impermeable to water.


But there is active reabsorption of sodium which
decreases the osmolarity of tubular fluid to a greater
extent. The fluid inside becomes hypotonic.

40. 'D' [Sembulingam 4th ed 296/ 5th ed 309]

41. 'C' [Sembulingam 4th ed 296/ 5th ed 309]


Mineralocorticoids released from Adrenal cortex
increases the reabsorption of sodium in proximal
tubule, ascending Limb and distal convoluted tubule.

42. 'C' [Ganong 23rd ed 650/Sembulingam 5th ed 308]


Tubular transport maximum (Tm) is the rate at which a
substance is reabsorbed from the renal tubule.

Tm for glucose is 375 mg/min. Renal threshold for


glucose is 180mg/dl.

43. 'B' [Ganong 24th ed 678]


Renal plasma clearance is the volume of plasma from
which a substance is completely removed by the
kidney in a given amount of time. GFR and clearance
are measured in ml/min

Its concentration in urine X


Clearance of Urine flow per unit time
a Substance Arterial plasma level of that
substance

= 150 X 2 / 15
= 20 ml/min
, PHYSIOLOGY
~

V
961

b) A single skeletal muscle and all the motor neurons c) Actin d) Protein-(
that supply it (COMEDK-08)
c) A single motor neuron and all the muscle fibers it 25. Increase in threshold level on applying subthreshold,
innervates slowly rising stimulus is kn own as
d) All the motor neurons in which responses are a) Adaptation b) Accomodation
observed after maximal stimulation of a single c) Refractoriness d) Electrotonus
sensory nerve (AIPG-09)
26. Skeletal muscles
18. The end plate potential is characterized by: a) Contracts when calcium is taken up by sarcoplasmic
a) Propagation b) All or none law reticulum
c) Depolarization d) Hyper polarization b) Contracts when actin and myosin filaments shorten
c) Contraction is initiated by calcium binding to
19. Non true regarding Na/K pump is troponin
a) Pumps Na against a gradient d) Contraction is initiated by calcium binding to
b) 5 Na+ exchanged for 2K+ tropomyosin
c) Increase in intracellular Na (AIIMS-09)
d) Hypocalcemia inhibits the pump 27. The absolute refractory period of an action potential
is due to:
20. Which is caused by acetylcholine through nicotinic a) Insufficient neurotransmitter
receptors: b) Depletion of intracellular Na+
a) Contraction of skeletal muscle c) Inactivation of Na+ K+ ATPase
b) Decrease of heart rate d) Voltage inactivation of the Na+ channel
c) Secretion of saliva d) Contraction of pupils (COMEDK-09)
28. Cardiac muscle contracts due to
21. Succinyl choline acts to block neuromuscular a) Extracellular ca+2 b) Sarcoplasmic ca+2
transmission by: c) Extracellular Na+ d) Intracellular Na+
a) Inhibiting cholinesterase (AIPG-10)
b) Inhibiting the CNS 29. Two major types of muscle fibres are found in humans
c) Depolarizing the motor end plate of skelet al muscle a) White and brown b) White and yellow
d) Blocking the release of acetyl choline at the end c) White and gray d) White and red
plate (BHU-2012)
(AIPG -98) 30. The main types of muscle cells are
22. In smooth muscle there is: a) Skeletal and cardiac b) Smooth and cardiac
a) Absence of t roponin c) Smooth and skeletal d) All of the above
b) Calcium combines with calmodulin leading (BHU-2012)
to initiation of contraction 31. Upstroke of action potential would lead to
c) Depolarization occurs due to influx of calcium ions a) Net current in an outward direction
d) All of the above b) Cell interior becomes more negative
c) Cell interior becomes less negative
23. The number of muscle fibers in a motor unit are d) None of the above
least in (KAR-2013)
a) Laryngeal muscles b) Pharyngeal muscles 32. Small packets of acetylcholine released randomly
c) Muscles of middle ear d) Extraocular muscles from the nerve cell membrane at rest produces
a) Inhibitory post synaptic potential
24. Which one of the following is a regulatory protein b) Miniature end plate potential
of the muscle? c) Action potential d) End plate potential
a) Troponin b) Myosin (COMEDK -2013)

18) C 19) B 20) A 21) C 22) D 23) A 24) A 25) B 26) C 27) D 28) A 29) D 30) D
31) C 32) B
, PHYSIOLOGY
~

V
963

4. MUSCLES AND NEUROMUSCLAR JUNCTION - ANSWERS


1. 'C' [Sembulingam 4th ed 168/ 5th ed 183] 10. 'B' [Sembulingam 4th ed 160/ 5th ed 175]
Muscle contraction is initiated when large number of After death, all the muscles of body undergo severe
calcium bind with troponin C. The calcium ions are contraction and become rigid. This is called the Rigor
the linking or coupling agents between excitation and mortis. It is due to loss of ATP, which are necessary
contraction of muscle. for the separation of cross bridges of myosin filaments
from actin filaments. This results in severe contraction
2. 'C' [Sembulingam 4th ed 148/ 5th ed 163] of muscles without any relaxation.
The structural and functional unit of skeletal muscle
is sarcomere. Sarcomere is the portion of myofibril 11. 'C'
in between two ' Z' lines. Myosin, Actin, Troponin
and Tropomyosin are together called the contractile 12. 'C' [Sembulingam 4th ed 147/ 5th ed 162]
elements or muscle proteins. These contractile Skeletal Striated, voluntary, supplied by somatic
elements are present in sarcomere of myofibril. muscle nerves and contains multiple nuclei beneath
the sarcolemma.
3. 'C' [Sembulingam 4th ed 148/ 5th ed 163] Cardiac Striated, involuntary and supplied by
muscle autonomic nerves.
4. 'C' [Sembulingam 4th ed 168/ 5th ed 183] Smooth Non striated, invo lunta ry and supplied by
Muscle contraction is initiated when large number of muscle autonomic nerves.
calcium ions bind with troponin C. During contraction
the length of actin and myosin filaments does not 13. 'B' (Sembulingam 4th ed 186/ 5th ed 201]
change. H-Zone disappears; distance between 'Z' lines Myasthenia Gravis is an autoimmune disease. The
decreases and the actin filaments glide over myosin body develops antibodies to its own acetylcholine
filaments. receptors and destroys them but the acetylcholine
release is normal. Muscular contraction is very slow
5. 'A' [Sembulingam 4th ed 148/ 5th ed 163] and weak. Fatigue occurs quickly. In severe cases there
is paralysis of muscles.
6. 'A' [Sembulingam 4th ed 184/ 5th ed 199]
14. 'C' (Sembulingam 4th ed 166/ 5th ed 181]
7. 'B' [Sembulingam 4th ed 184/ 5th ed 199] In resting membranes, sodiLlm and chloride are more
outside and potassium ions are more inside. During
8. 'C' [Sembulingam 4th ed 168/ 5th ed 183] depolarization influx of sodium occurs and during
Excitation of muscle generation of action potential repolarization there is efflux of potassium.
rapid spread of action potential via t-tubule system
calcium release from L-tubules contraction of muscle. 15. 'D' [Sembulingam 4th ed 175/ 5th ed 190]
Prolonged plateau phase is seen in cardiac and smooth
muscle due to prolonged depolarization. 16. 'A' (Sembulingam 4th ed 166/ 5th ed 181)

9. 'A' (Sembulingam 4th ed 155/ 5th ed 170] 17. 'C' [Sembulingam 4th ed 176/ 5th ed 191]
In isotonic contraction, the tension remains same and
the muscle is allowed to shorten during contraction. 18. 'C' [Sembulingam 4th ed 166/ 5th ed 181]
In this some external work is done and is technically
called as the 'load is li fted'. 19. 'B' (Sembulingam 4th ed 165/ 5th ed 180)
Active transport of sodium and potassium ions in
In isometric contraction the tension is increased but opposite directions across the cell membrane by means
the muscle is prevented from shortening. External work of an electrogenic pump is called sodium-potassium
done is zero. pump. It moves 3 sodium ions out of the cell and 2
potassium ions inside the cell by using ATP.
, PHYSIOLOGY
~

V
969

d) AV node d) Marks the onset of diasto le


(COMEDK-05) (AIPG-10)
35. In determining blood pressure by auscultatory 43. What is common between systemic and pulmonary
method circulation is
a) The loudest sound is the diastolic pressure a) Volume of the circulation per minute
b) Systolic pressure estimation tends to be lower than b) Peripheral vascular resistance
those make by palpatory method c) Pulse Pressure d) Total Capacity
c) The first sound heard is the systolic pressure (AP-10)
d) The sounds that are heard are generated in heart 44. Stimulation of baroreceptors results in
(COMEDK-07) a) Increase in heart rate
36. Isovolumetric relaxation ends immediately after b) Decreased vagal discharge
a) AV Valve closes c) Increased sympathetic discharge
b) When ventricular pressure falls below aortic pressure d) Decrease in blood pressure
c) When ventricular pressure falls below atrial pressure (KCET-2012)
d) None of the above 45. The function of Carotid body is
(COM EDK- 08) a) Measures the change in Po 2 in arterial blood
3 7. Glomerular capillary pressure differ from other b) Measures Po2 in venous blood
capillaries of body in: c) Measures the change in CO 2 in arterial blood
a) Higher filtration pressure d) Measures the change in CO 2 in venous blood
b) Lower filtration pressure (AIPG-2012)
c) Both of the above d) None 46. The dicrotic notch on the aortic pressure curve is
(MCET-07) caused by
38. The haemoglobin oxygen saturation of blood a) Closure of the pulmonary valve
entering the right ventricle is approximately b) Rapid filling of the left ventricle
a) 97 percent b) 85 percent c) Closure of the aortic valve
c) 75 percent d) 53 percent d) Contraction of the atria
(COMEDK-2012)
39. Which of the following increases turbulence in 47. Peripheral vascular resistance is best given by
blood flow? a) Mean arterial pressure responsible for blood flow to
a) Reynolds number< 2000 organ
b) Decrease in velocity of blood b) Diastolic blood pressure as it decreases till mid-
c) Decrease in dernsity of blood thoracic aorta
d) Increase in diameter of blood vessel c) Pulse pressure as it relates to stroke volume and
(AIPG-09) aortic compliance
40. Heart muscle, true are all except: d) Systolic pressure as it inc rease in descending aorta
a) Act as syncitium b) Has multiple nuclei (AIPG-14)
c) Has gap junctions d) Has branching 48. Carotid body/baroreceptor is located at the origin
(AIIMS-09) of
41. The velocity of conduction fo the Purkinje fibers of a) Common carotid artery
the heart is b) Internal carotoid artery
a) 1 to 4 m/sec b) 5 to 8 m/sec c) Aorta d) External carotid
c) 9 to 12 m/sec d) More than 12 m/sec (AIIMS MAY-14)
(COMEDK-10, KERALA-15)
42. 2nd heart sound is characterized by all except
a) Due to closure of semilunar valves
b) Is occasionally split
c) Has longer duration than 1st heart sound

35) C 36) C 37) A 38) C 39) D 40) B 41) A 42) C 43) A 44) D 45) A 46) C 47) B
48) B
Dental ;lut.,e

5. CARDIOVASCULAR SYSTEM - ANSWERS


1. 'C' [Sembulingam 4th ed 501/ 5th ed 524) Minute volume
iii) Cardiac index
• Due to closure of atrioventricular Body surface area in m2
(Mitral, Tricuspid) valves = 2.8 lt/m 2/min.
simultaneously.
First heart
sound
• Long, soft and low pitched and 6. ' D' [Sembulingam 4th ed 526/ 5th ed 551)
sounds like LUBB.
• Produced during isometric contraction 7. 'C' [Sembulingam 4th ed 528/ 5th ed 553)
and earlier part of ejection period. Factors affecting cardiac output are:
• Venous return
• Due to closure of semi luna r (Pulmonary,
• Force of contraction
Second aortic) valves simultaneously.
• Frequency of heart beat
heart • Short, sharp, high pitched and
• Peripheral resistance
sound sounds like DUBB.
• Produced during the onset of diastole. 8. ' B' [Sembulingam 4th ed 547/ 5th ed 575)
Third heart Produced during rapid filling period and Peripheral resistance is inversely proportional to
sound is ina udible by st et hoscope. radius of blood vessel. The radius of arterioles is the
Fourth least; because of this, the arterioles will show high
Produced during atrial systole and is
heart periphera l resista nee.
inaudible.
sound
9. ' D' [Sembulingam 4th ed 493/ 5th ed 515)
2. 'A' [Sembulingam 4th ed 541/ 5th ed 568) Events of cardiac cycle
Parasympathetic nerve fibres are cardio-inhibitory
Atria contract and small amount of
nerve fi bers. These reach the heart through vagus
Atrial blood enters ventricles. AV valves
nerve. Right vagus supplies SA node. Its stimulation
systole are opened and semi lunar va lves are
causes reduction in the heart rate and force of
closed. Duration is 0.11 secs.
contraction. Left vagus supplies AV node.
Isometric All the valves are closed. Pressure in
3. 'C' [Sembulingam 4th ed 485/ 5th ed 508) contraction ventricles is raised. Duration is 0.05 secs.
Pacemaker is the part of heart from which the impulses Semilunar valves are opened. Ventricles
for heartbeat are produced normally. The pacemaker is Ejection contract and blood is ejected into aorta
Sinoatrial node (SA Node). The impulses are conducted period and pulmonary artery. This period lasts
to different parts of the heart by AV node, bundle of for 0.22 secs.
HIS, purkinje fibres. The velocity of conduction of Proto Semilunar valves are closed. Duration
impulses is maximum in purkinje fibres and is minimum diastole of this period is 0.04 secs.
at AV Node. All the valves are closed and pressure
Isometric
in the ventricles is reduced. Duration
4. 'D' [Sembulingam 4th ed 510/ 5th ed 533) relaxation
is 0.06 secs.
• P wave - Atrial depolarization (KCET- 07) Rapid and AV valves are opened. Ventricles relax
• ORS complex - Ventricular depolarization
slow filling and filling occurs. Duration is 0.3 secs.
• T-wave - Ventricular repolarization
Duration of cardiac cycle is 0.8 secs. Duration of
5. 'C' [Sembulingam 4th ed 526/ 5th ed 551)
systole is 0.3 secs and diastole is 0.5 secs.
i) Stroke volume = 70 mlfbeat
10. ' D' [Sembulingam 4th ed 547/ 5th ed 575)
ii) Minute volume (or)
= Stroke volume x heart rate
Cardiac output
11. 'C' [Sembulingam 5th ed 554)
= 70 x 72 = 5 lt/min.
, PHYSIOLOGY
~

V
971

According to Frank-Starling law, the force of contraction 22. 'A' [Sembulingam 4th ed 577/ 5th ed 607)
depends upon diastolic period and ventricular filling. Rig ht and left coronary arteries supply the heart muscle.
It also depends upon preload and after load. Left coronary artery supplies the anterior and lateral
parts of left ventricle while the right coronary artery
Preload supplies whole right vent ricle and posterior portion of
It is the stretching of cardiac muscle at the end of left ventricle. Heart has the capacity to regulate its
diastole. Preload depends upon venous return and own blood flow. It is known as autoregulation. Heart
ventricular filling. The force of contraction of heart receives its blood supply during diastole while other
and cardiac output are directly proportional to preload. parts of the body receive blood during systole.

After load 23. 'C' [Sembulinga m 4th ed 526/ 5th ed 551)


It is the force against which the ventricles must
contract and eject the blood. The after load for left 24. ' B' [Sembulingam 4th ed 492/ 5th ed 515)
ventricle is determined by aortic pressure and after
load for right ventricular pressure is det ermined by 25. 'C' [Sembulingam 4th ed 501/ 5th ed 524 )
pressure in pulmonary artery. The force of contraction
of heart and cardiac output are inversely proportional 26. ' D' [Sembulingam 4th ed 492/ 5th ed 515)
to after load.
27. ' B' [Sembu lingam 4th ed 542/ 5th ed 569)
12. 'C' [Sembulingam 4th ed 529/ 5th ed 554]
Frank starling's law states that force of contraction is 28. 'A' [Sembulingam 4th ed 528/ 5th ed 553)
directly proportional to initial length of muscle fibres The distribution of cardiac output is according to the
within physiological limits. metabolic activities of various regions of the body. Liver
receives 30% of cardiac outprut followed by kidneys.
13. 'C' [Sembulingam 4th ed 547/ 5th ed 575]
29. ' B' [Sembulingam 4th ed 543/ 5th ed 570)
14. ' B' [Sembulingam 4th ed 556/ 5th ed 585) Baroreceptors are situated in the carotid sinus. The
Whenever there is rapid increase in arterial blood baroreceptors give response to change in blood
pressure, the baroreceptors present in carotid sinus pressure. Peripheral chemo receptors are situated
are stimulated. Tlnis in turn stimulates t he cardio in the carotid and aortic body and give response to
inhibitory centre and suppresses the vasomotor tone. change in chemical constituents of blood.
The heart rate, force of contraction and peripheral
resistance are decreased, ultimately the cardiac output Osmoreceptors are present in Hypothalamus and gives
and arterial blood pressure are reduced. This causes response to changes in the osmolar concentration of
inactivation of baroreceptors. the blood.

15. ' B' [Sembulingam 4th ed 537/ 5th ed 564) 30. ' D' [Sembulinga m 5th ed 503)
Ch ronotropic actionEffect on heart rate
16. ' B' [Sembulingam 4th ed 527/ 5th ed 552) Inotropic action Effect on force of contraction
Effect on conduction of
17. 'E' [Sembulingam 4th ed 488/ 5th ed 511) Dromotropic action
impulse through heart
Effect on excitability of
18. ' B' [Sembulingam 4th ed 493/ 5th ed 516) Bathmotropic action
cardiac muscle.
19. 'A' [Sembulingam 4th ed 487 / 5th ed 510)
31. ' D' [Ganong 21st ed 550]
20. ' D' [Sembulingam 4th ed 487/ 5th ed 510]
32. ' B' [Ganong 21st ed 589]
21. 'C' [S embulingam 4th ed 526/ 5th ed 551]
33. ' B' [Sembulingam 4th ed 529/ 5th ed 554]
Dental ;lut.,e

34. 'A' [Sembulingam 4th ed 487/ 5th ed 510] Tissue Conduction rate
(m/sec.)
35. 'C' [Sembulingam 4th ed 561/ 5th ed 590]
SA node 0.05
Atrial pathway 1
36. 'C' [Sembulingam 4th ed 493/ 5th ed 51 6]
During isometric relaxation, the ventricular pressure AV node 0.02 - 0.05
decreases greatly and when the pressure becomes less Purki njee system 4
than pressure in atria, AV valves open, resulting in Bundle of His 1
rapid filling of the ventricles. Ventricular muscle 1

37. 'A' [Sembulingam 4th ed 291/ 5th ed 303] 42. 'C' [Ganong 22nd ed 569]
The Glomerular capillary pressure is about 60mm Hg.
It is the highest capillary pressure in the body. This 43. 'A' [Sembulingam 5th ed 651]
pressure favors glomerular filtration. Capillary pressure • The lungs receive the whole a mount of blood that
in the lungs is lowest. It is about 7mm Hg. It favors is pumped out from right ventricle. The output of
exchange of gases between blood and alveoli. blood per minute is same in both the right and left
ventricle. It is abut 5 liters.
38. 'C' • The pulmonary vascular resistance is low compared
to systemic vascular resistance.
39. ' D' [Ganong 22nd ed 583]
Reynolds number (Re) = pDV /n, where 44. ' D' [Sembulingam 5th ed 571]
p = Density of fluid The baroreceptors regulate the heart rate through
D = Diameter of tube a reflex called Marey's reflex. Marey's reflex is
V = Velocity of flow cardioinhibitory reflex that decreases heart rate when
n = Viscosity of fluid blood pressure is raised.

Probability of turbulence is directly proportional to Re. 45. 'A' [Ganong 23rd ed 562]
• If Re is <2000 - flow is not turbulent Peripheral arterial chemoreceptors in the carotid
• If Re is >2000 - flow is almost always turbulent and aortic bodies have very high rates of blood flow.
These receptors are primarily activated by a reduction
As Re is proportional to diameter of tube, an increase in partial pressure of oxygen (Pa0 2), but they also
in the D increases the turbulence. respond to an increase in the partial pressure of carbon
dioxide (PaC0 2) and pH. Chemoreceptors exert t heir
40. 'B' [Ganong 22nd ed 78] main effects on respiration; how-ever, their activation
Cardiac muscle also leads to vasoconstriction.
• Striated and Z-lines present
Chemoreceptor discharge may also contribute to the
• Has interca lated discs (maintaining cell-to-
production of Mayer waves. These should not be confused
cell cohesion) and gap junctions for spread of
with Traube-Hering waves, which are fluctuations in
excitation from one fiber to another, thus acting
blood pressure synchronized with respiration.
as a syncitium.
• Has branching but each is a complete unit 46. 'C' [Ganong 23rd ed 512]
surrounded by a cell membrane. The dicrotic notch is a small oscillation on the falling
• Is uninucleated phase of the pulse wave due to vibration setup, when
• Involuntary the aortic valve snaps sh ut. Dicrotic notch is usually
• Has numerous elongated mitochondria visible if the pressure wave is recorded but is not
• RMP = -70mV palpable at the wrist. The plllmonary artery pressure
curve also has a dicnotic notch produced by the closure
41. 'A' [Ganong 22nd ed 547 - 549] of pulmonary valves.
, PHYSIOLOGY
~
973
V

47. ' B' [Guyton 11th ed 174]


Peripheral resistance is defined as resistance to
passage of blood through the small blood vessels,
especially the arterioles.

Diastolic pressure is the minimum pressure that


is obtained at the end of diastole. It is an index to
the peripheral resistance. Hence diastolic BP is more
dependable and important.

Whereas the Systolic BP is subject to wider variation


and so less dependable. Pulse pressure is the index of
cardiac output.

Also Refer explanation of Q.No.8

48. ' B' [Ganong 24th ed 589/ Check Q. No. 29]


Carotid sinus is a small dilation in the internal carotid
artery just above the bifurcation of the common
carotid into external and internal carotid branches.
Baroreceptors are located in this dilation. They are
also found in the wall of the arch of the aorta.
, PHYSIOLOGY
~

V
975

18. Breathing ceases upon destruction of the 28. First change to occur after nerve cut is:
a) Cerebrum b) Medulla oblongata a) Schwan cell mitosis b) Axonal sprouting
c) Hypothalamus d) Cerebellum c) Myelin sheath degeneration
(APPSC -99) d) Nuclear disintegration
19. The sensory receptors serving the stretch reflex are
classified as: 29. First change to occur in the distal segment of cut:
a) Proprioceptors b) Nociceptors a) Myelin degeneration b) Axon al degeneration
c) Exteroceptors d) Cherne receptors c) Mitosis of schwann cell
(APPSC -99) d) Sprouting
20. In CNS, the myelination is carried out by:
a) Schwann cells b) Astrocytes 30. Which sensation is not lost on the side of lesion in
c) Oligodendrocytes d) Microgila brown sequard syndrome?
(TNPSC -99) a) Touch b) Vibration sense
21. Abdominal visceral pain is transmitted by: c) Muscle sense d) Temperature
a) A-fibers b) (-fibers
c) Parasympathetic fibers 31. Wallenberg degeneration is seen in:
d) Sympathetic fibers a) Proximal cut end of nerve with cell body
(TNPSC -99) b) Distal cut end of nerve without cell body
22. Chromatolysis is: c) Both the free ends of the cut nerve
a) Disintegration of Nucleus d) All a re true
b) Disintegration of golgi apparatus
c) Disappearance of nissel granules 32. In cerebellar disease, all the statements are correct
d) Decrease in cell size except:
(TNPSC -99) a) The romberg's sign is positive
23. Aphasia is most likely associated with a lesion in: b) There is adiodokokinesia
a) Broca's area b) Sensory area c) There is pendular knee jerk
c) Primary motor area d) Visual area d) There is involuntary tremor
(TNPSC -99)
24. Body temperature is maintained by: 33. Swallowing center is situated in:
a) Postural position b) Covering of the body a) Midbrain b) Pons
c) Peripheral vasoconstriction c) Medulla d) Cerebellum
d) All of the above
(TNPSC -99) 34. Hyper kinetic syndromes such as chorea and
25. The term 'myopia' refers to: athetosis are usually associated with pathological
a) Near sightedness b) Far sightedness changes in:
c) Constriction of the pupil a) Motor areas of cerebral cortex
d) Dilation of the pupil b) Anterior hypothalamus
(KAR -97) c) Pathways for recurrent collateral inhibition in the
26. Which receptor is responsible for monitoring the spinal cord
rate of muscle stretch? d) Basal ganglia complex
a) Nuclear bag intrafusal fibers
b) Nuclear chain 35. Satiety center in hypothalamus is regulated by:
c) Golgi tendon organ d) Pacinium corpuscles a) Gastric dilatation b) Blood glucose levels
(KAR -97) c) Blood insulin levels d) All of the above
27. Neurotransmitter in nigrostrial pathway is:
a) Dopamine b) GABA 36. Function of GABA on CNS is:
c) Acetyl choline d) Nor epinephrine a) Neuronal inhibition b) Neuronal activation

18) B 19) A 20) C 21) D 22) C 23) A 24) D 25) A 26) A 27) A 28) C 29) B 30) D
31) B 32) A 33) C 34) D 35) B 36) A
, PHYSIOLOGY
~

V
977

c) Axolemma d) Myelin sheath b) Has no Nissl granu les


(AIIMS-06) c) Is at commencement of dendrites
58. The pathway of pain from teeth and temperature is d) Is round in shape
carried by: (COMEDK-09)
a) Corticospinal tract 66. In nerve, the magnitude of the action potential
b) Corticocerebral tract overshoot is normally a function of the
c) Lateral spinothalamic tract a) Magnitude of the stimulus
d) Ventral spinothalamic tract b) Intracellular potassium concentration
(AIIMS-06) c) Extracellular sodium concentration
59. Intention tremor is a feature of d) resting membrane potential
a) Loss of function of Thalamus (COMEDK-2011)
b) Loss of function of Hypothalamus 67. Stimulating mechanoreceptors or nociceptors in
c) Loss of function of Cerebellum mouth triggers the
d) Loss of function of Basal ganglia a) Jaw opening reflex b) Jaw clenching reflex
(KCET-08) c) Jaw gnashing reflex d) None of the above
60. DOPA and 5 - Hydroxytryptophan are clinically (BHU-2012)
important because 68. Following are the changes during accommodation
a) They cross Blood Brain Barriers except
b) They are acidic precursors of Brain amines a) Constriction of pupil b) Dilatation of pupil
c) They act as neuromodulators c) Convergence of eye ball
d) They are metabolites of various neurogenic amines. d) Increase in the anterior curvature of lens
(AIIMS-08) (COMEDK -2013)
61. Epicritic perception of pain occurs at the level of 69. Characteristic feature true about outer membrane
a) Thalamus b) Area 3, 1, 2 of peripheral nerve
c) Areas 5, 7 d) Pulvinar a) Relative permeable to sodium ions
(COMEDK-10) b) Relative permeable to potassium ions
62. Broca's area is concerned with c) Permeable to chloride ions
a) Word formation d) Impermeable to potassium ions
b) Comprehends language not speaking (AIIMS NOV-13)
c) Present in posterior part of temporal lobe 70. Stretch fibers are present in striated muscle,
d) Is the confluence of somatic, auditory and visceral innervation is carried by
areas a) A delta fiber b) C fiber
(AIIMS-09) c) unmyelinated fiber d) Gamma fiber
63. Blood brain barrier is absent in all of the following (AIIMS NOV-13)
areas except 71. Projections from substantia nigra pars compacta to
a) Subfornical region b) Habenuc~artrigone striatum releases
c) Area posterma d) Neurohypophysis a) GABA b) Dopamine
(AIIMS-09) c) Glutamate d) Aspartate
64. Which of the following is not true about (KERALA-2015)
axonotmesis? 72. Cornea is innervated by?
a) Impairment of both sensory and motor function a) A-beta b) A-beta and A-delta
b) Wallerian degerneration c) A-delta and ( -fibre d) C-Fibre only
c) Intact neura l sheath is not present (PGI JUNE-2014)
d) Reversible loss, which will be regenerated in 6-8 wks 73. Tremors on movements is seen in
(AP-2011) a) Spinal cord injury b) Thalamus injury
65. Axon hillock is a part of the soma of neuron which c) Internal capsule injury
a) Has dense Nissl granules

58) C 59) C 60) A 61) B 62) A 63) B 64) C 65) B 66) C 67) A 68) B 69) B 70) D
71) B 72) C 73) D
Dental ;lut.,e

d) Cerebellum injury
(PGI JUNE-2012)
74. The reward or pleasure center is present in
a) Ventral tegmenta l area
b) Ventromedial medulla
c) Magnus raphe nucleus d) Locus coeruleus
(COMEDK-15)
75. The motivational - affect component of pain is
carried by
a) Pa leospi noth ala mic tract
b) Neospi nothalamic tract
c) Spinoreticular tract
d) Dorsal spinocerebellar tract
(KERALA-2015)
76. Neuropraxia is
a) Contusion of a nerve where continuity of both the
epineural sheath and axons are maintained
b) Continuity of axons is maintained but the epineura l
sheath is disrupted
c) Complete loss of nerve continuity
d) Paroxysmal and intermittent intense pain
(KERALA-2015)
77. The regeneration of nerve fibres is possible in the
peripheral nervous system than the central nervous
system because
a) Presence of neurilemmal sheath in peripheral nerve
b) Presence of neurilemmal sheath in central nervous
system
c) The absence of myelin sheath in the central nervous
system
d) Disturbed vascular supply
(COMEDK-15)
78. Krause end bulbs are specialised receptors for
a) Cold b) Pain
c) Touch d) Heat
(APPG-15)

74) A 75) A 76) A 77) A 78) A


Dental ;lut.,e

47. 'A' [Sembulingam 4th ed 201/ 5th ed 215] 52. 'C' [Sembulingam 4th ed 719/ 5th ed 765]
Copious flow of watery saliva is secreted in response to Afferent Efferent
parasympathetic stimulation. Sympathetic stimulation Reflex Response Center
nerve nerve
causes less secretion of saliva, which is thick and rich Corneal Blinking of V cranial Pons VII
in mucin. eye nerve cranial
nerve
48. 'A' [Sembulingam 5 th ed 23, 24]
Conjun- Blinking of V cranial Pons VII
Occluding junctions or Tight junctions:
ctival eye nerve cranial
These prevent the movement of ions and molecules from
nerve
one cell to anot her cell. These junctions are present in
the apical margins of epithelial and endothelial cells Pharyn- Gagging or IX cranial Nuclei X cranial
geal opening of nerve of X nerve
in intestinal mucosa, wall of renal tubule, capillary
mouth crania l
wall and choroid plexus.
nerve
Communicating junction or Gap junctions: Uvular Raising of IX cranial Nuclei X cranial
These junctions permit the movement of ions and uvula nerve of X nerve
molecules from one cell to another cell. Gap junction cranial
is also called nexus. These permit the passage of nerve
glucose, amino acids, ions, etc and also helps in
rapid propagation of action potential from one cell to 53. 'D' [Sembulingam 4 th ed 702/ 5th ed 7 44]
another cell.
54. ' B' [Ganonth 19th ed 605]
49. ' B' [Sembuli ngam 5 th ed 271]
55. 'C'
50. 'B' [Sembulingam 4 th ed 733/ 5th ed 781]
The descending tracts of spinal cord are: 56. 'B' [Sembulingam 4 th ed 707 / 5th ed 749]
Pyramidal tracts: Gamma-aminobutryic acid (GABA) is the major
inhibitory mediator in the brain . It is also present
• Anterior corticospinal tract
in the retina and is the mediator responsible for
• Lateral corticospinal tract presynaptic inhibition.

Extrapyramidal tracts: 51. 'B' [Sembulingam 4 th ed 68 7 / 5th ed 729]


• Reticulospi nal t ract
• Tectospinal t ract 58. 'C' [Sembulingam 4 th ed 728/ 5th ed 775]
• Rubrospinal tract The ascending tracts of spinal cord carry the sensations
to the brain.
• Vestibulospinal tract
Ascending tract Function
• Olivospinal tract
Anterior spino-
Crude touch sensation
51. 'A' [Sembulingam 4 th ed 702/ 5th ed 744] thalamic tract
Receptors Sensation Lateral spino-
Pain and temperature sensations
thalamic tract
Meissner's corpusd e Touch
Subconscious kinesthetic
Merkel's disc Touch Spinothalamic
sensations
Pacinian corpuscle Pressure Spinoreticular Consciousness and awareness
Free nerve ending Pain Spinovestibular Proprioception
Krause's end bulb Cold
Raffi ni's end organ Warmth
, PHYSIOLOGY

59. 'C' [Sembulingam 4th ed 790/ 5th ed 841] • The epicritic relay involves fibers with cell bodies
The functions of cerebellum are in Dorsa l Root Ganglion that enter and ascend
i) Regulation of tone, posture and equilibrium ipsilaterally in fasciculus. The 2nd order neurons
cross in the anterior arcuate fibers and ascend in
ii) Regulation of coordinated movements.
the medial lemniscus of the thalamus. The 3rd
order neurons then travel in the posterior limb of
Cerebellar lesions cause disturbances in posture
the internal capsule on their way to areas 3, 1, 2
equilibrium and the movements. Symptoms appear on
(the postcentral gyrus).
the ipsilateral side of the body.

• Atonia Protopathic:

Disturbances • Hyptonia • Protopathic relays are associated with diffuse


touch and temperature stimulation.
in tone/ • Deviated movement
posture • Protopathic relays involves dorsal root ganglion
• Pendula r knee jerk
fibers that enter the dorsal gray and immediately
• Patellar tendon reflex
descent a couple of levels in Lissauer's tract.
Disturbances • Staggering drunken like gait After their descent they will synapse in the
in • While standing legs are spread nucleus proprius, cross and ascend in the anterior
equilibrium widely spinothalamic tract. Like the epicritic relay it
will synapse in the thalamus on its way to the
• Ataxia (uncoordinated movement)
postcentral gyrus.
• Asthenia (weakness of muscles)
Disturbances • Intentional tremor 62. 'K [GUYTON 11th ed 687]
in • Nystagmus Broca's area (Area 44)
movements Dysarthria (disturbance in speech) Broca's area is located in the inferior frontal gyrus
• immediately anterior to the primary motor cortex and
• Rebound phenomenon
above the sylvian fissure. It is associated with 'word'
• Adiadochokinesis formation.

60. 'A' [KD Tripathi 5th ed 103/ Ganong 22nd ed 94- Repetition is impaired with lesions in Broca's areas,
96] Wernicke's area and the arcuate fascicu lus.
• Both DOPA and 5-Hydroxytryptophan can cross
blood brain barrier and this is clinically important. Arcuate
Broca's area Wernicke's area
• These are not the precursors but are derivatives of fasciculus
brain amines. Located Located in supra Connects
• These are mot neuromodulators. Serotonin in inferior marginal gyrus Wernicke's to
(5-Hydroxytryptamine, 5-HT) & Dopamine are frontal gyrus Brocas area
neuromodulators.
Motor Secondary Important
speech area, speech area role in
61. 'B' [BD Chaurasia 445/ Sembulingam 5th ed 856]
associated associated with 'Repetition'
In sensory systems there are two main forms of sensory
with 'word' 'comprehension'
stimuli
formation of sounds
• Epicritic
Lesions Lesions here Lesions
• Protopathic
produce produce produce
Broca's Wernicke's conductive
Epicritic:
Aphasia Aphasia Aphasia
• Epicritic relays carry sensation associated with
conscious proprioception, vibratory sense and fine
touch.
Dental ;lut.,e i ==================

63. 'B' [Ganong 22nd ed 615) Aecom modation is the adjustment of the eye to see
Four small areas in or near brain stem are said to be either near or distant objects clearly. The following
"outside the blood-brain barrier". They are adjustments are made in the eyeballs during
• Posterior pituitary accommodation reflex when a person looks at a near
object after seeing a far object:
• Area postrema
• Convergence of the eyeballs due to contraction of
• Organum vasculosum of lamina terminalis
the medial recti
• Subfornical organ • Constriction of the pupil due to contraction of
constrictor pupillae of iris
These areas are referred to collectively as • Increase in the anterior curvature of the lens due
circumventricular organs. All have fenestrated to contraction of the ciliary muscle.
capillaries and because of their permeability they are
said to be "outside the blood-brain barrier". 69. 'B' [Ganong 23rd ed 84-85)
In neurons at rest the concentration of K+ is much
64. 'C' [Bailey & Love 24th ed 581) higher inside than outside the cell, whi le it is reverse
Axontmesis is disruption of nerve cell axon, with with Na+. This concentration difference is established
wallerian degeneration occurring below or slightly by the Na+-K+ ATpase
proximal to the site of injury. It is characterized by
damage of axons and myelin sheath and intact schwann The outward K+ concentration gradient results in
cells, epineurium, perineurium, and endoneurium. passive movement of K+ out of the cell when K+
Depending on the severity, axontmesis results in selective channels are open. Similarly the inward Na+
temporary impairment of both sensory and motor concentration gradient results in passive movement
functions. The prognosis is usually good in terms of of Na+ into the cell when Na+ selective channels are
recovery. Rate of recovery depends on the distance from open.
the site of injury, with axonal regeneration occurring
at 1 to 4 mm/day. Peripheral nerves regeneration may As there are more open K+ channels than Na+ at rest,
take several months. the membrane permeability to K+ is greater.

65. 'B' [Sembulingam 5th ed 729) So the intracellular and extracellular K+ concentrations
Axon is the longer process of the nerve cell. Each are the prime determinants of the resting membrane
neuron has on ly one axon. The axon arises from axon potential which is usually about -70mv and which is
hillock of the nerve cell body and it is devoid of Nissl close to the equilibrium potential for K+
granules.
70. 'D' [Check explanation of Q. No.10)
66. 'C' [Guyton 11th ed 183) i) Skeletal muscles contain specialized proprioceptive
During depolarization, there is influx of sodi um ions. sense organs called muscles spindles. They function
The normal resting potential is -90mv. It is neutra lized to detect muscle stretch.
by sodium ions th1.1s rising potential in a positive ii) Each muscle spindle consists of small striated
direction. In large nerve fibres, the membrane potential muscle fibers called intrafusal muscle fibers.
actually overshoots beyond zero and becomes positive. iii) A Gamma (A8) motor neurone innervate the
This magnitude depends on extracellular sodium ion intrafusal muscle fibers.
concentration.
71. 'B' [Ganong 24th ed 147)
67. 'A' [Burket's Oral Medicine 228)
• Dopaminergic neurons are located in several brain
The jaw opening reflex is produced due to nocireceptors. regions.
Stimulation of this receptor causes contraction of
• One is Nigrostriatal system, which projects from
muscle surrounding mandible which inturn causes
the midbrain substantia nigra to the striatum in
opening of the jaw.
the basa l ganglia and is involved in motor control.
68. 'B' [Sembulingam 5th ed 952)
, PHYSIOLOGY
~
985
V

• Another is mesocortical system, which arises


primarily in the ventral tegmenta l area and projects
to the nucleus accumbens and limbic subcortical
areas. It is involved in pleasure, reward behaviour
and addiction and in psychiat ric disorders such as
schizophrenia.

72. 'C' [Ingle 6th ed 388]


Cornea is one of the most densely innervated tissues
in the human bodJy. The endings originate from cell
bodies in the ipsilateral trigemina l gang lion, project
via the ophthalmic nerve, which is composed of
multiple fibers including the high speed, myelinated
A-delta fibers and larger, slower conducting C fibers.
In this way cornea and tooth pulp are unique in that
they are almost exclusively innervated by A delta and
C fibers.

73. 'D' [Check explanation of Q.No.59]

74. 'A' [Check explanation of Q.No.71]

75. 'A' [Guyton 1st South Asian ed 700]


Pain pathway:
A deta and C fibres transmit pain to the spinal cord.
On entering spinal cord from the dorsal spina l roots,
the pain fibers terminate in the dorsal horns. Here,
from second order neurons, the pain signals take two
pathways to the brain through
• Neospinothalamic tract for fast pain (sensory
discriminative)
• Paleospinothalamic tract for slow chronic pain
(affective motivational). This is considered as
phylogenetically older pathway.

These pathways terminate in the lateral aspect of


ventral posterior nucleus (VPN). Third order neurons
from the VPN relay information to the parietal lobe of
somato sensory cortex.

76. 'A' [Check synopsis Point No.3 under CNS]

77. 'K [Check synopsis point No.5 under CNS]

78. K [Check synopsis Point No.7 under CNS]


Dental ;lut.,e

7. HEMATOLOGY
1. Which of the following coagulation factors is 9. Histamine is present on surface of:
deficient in classical hemophilia? a) Lymphocytes b) Mast cells
a) VIII b) IX c) Neutrophils d) Erythrocytes
c) X d) XII (AIIMS -2K, AIPG -94)
(MAN -94) 10. In acute infection which of the following is found
2. Clotting factor common to extrinsic and intrinsic a) Leucopenia b) Leucocytosis
pathway is: c) Neutrophilia d) Neutropenia
a) Factor II b) Factor III (AIPG -93)
c) Factor V d) Factor VII 11. The most common problem with blood transfusion
(MAN -97) is:
3. Which of the following is done for a patient on a) Hemolytic reaction b) Remission of disease
coumarin (warfarin) therapy: c) Hypokalemia d) Metabolic acidosis
a) Partial thromboplastin time (PTT) (AIIMS -01)
b) Prothrombin time (PT) 12. Life cycle of RBC is:
c) Bleeding time (BT) a) 12 days b) 90 days
d) Capillary fragility test (CFT) c) 120 days d) One month
(MAN -99) (AIPG -94, 92)
4. Normal myeloid/erythroid ratio is: 13. The function of vitamin K is:
a) 1:1000 b) 1:5000 a) Thrombin formation b) Prothrombin formation
c) 1:100 d) 1:50 c) Fibrin formation d) Thromboplastin formation
(MAN -99) (AIPG -99)
5. Megaloblasts are precursors of: 14. Hemoglobin has:
a) Megakaryocytes b) Normoblasts a) Four polypeptide chains, 2o., 1~, 1y chain
c) Myelocytes d) Macrocytes b) Four heme molecules and four polypeptide chains
(MAN -01) c) Four heme molecules, 2cx., and 2~ chains
6. The anticoagulant of choice used in the blood bank d) One heme and one globin molecule
is: (PGI -2K)
a) Calcium oxalate 15. Haemoglobin is the major buffer in blood,
b) Heparin solution bicarbonate ions diffuses out of erythrocyte into
c) Acid citrate dextrose solution plasma in exchange of:
d) Sodium fluoride a) Potassium b) Phosphate
(KAR -98) c) Carbonic acid d) Chloride ion
7. To preserve blood for transfusion later: (KAR -97)
a) Dilute with equal volume of 0.9% saline 16. Which of the following agents in not likely to be
b) Add solution of sodium citrate found in plasma?
c) Add solution of calcium chloride a) Thrombin b) Filbri nogen
d) Add fibrinogen c) Prothrombi n d) Calcium ion
(AIIMS -01) (AIPG -98)
8. T-lymphocytes function to: 17. Factor IX is also called as:
a) Antibody production a) Stuart power factor b) Christmas factor
b) Complement activation c) Prothrombi n d) Fibrin factor
c) Lymphokine production and delayed hypersensitivity (AIPG -97)
d) Immediate hypersensitivity 18. Histiocytes are:
(AIIMS -01, 95) a) Scavenger cells b) Allergic cells

1) A 2) C 3) B 4) A 5) B 6) C 7) B 8) C 9) B 10) C 11) A 12) C 13) B


14) C 15) D 16) A 17) B 18) A
Dental ;lut.,e

38. For heparin which of the following is incorrect? 47. The function of plasma cell:
a) Inhibits conversion offibrinogen to insoluble fibrin a) CMI b) Phagocytosis
b) Interferes with action of thrombin c) Opsonisation d) Antibody formation
c) Is released by macrophages (PGI -97, APPSC -99)
d) Is released by mast cells 48. Albumin is
(AIIMS -94) a) Nucleo protein b) Crnromo protein
39. The hemoglobin% of a normal newborn is: c) Phospho protein d) Simple protein
a) 8 b) 12 (AP -04)
c) 10 d) 30 49. Differential white blood cell counts in the
(AIPG -99) laboratory are useful in the diagnosis of:
40. Immunologicaly active cells are: a) Anemia b) Eosinophilia
a) Plasma cells b) MAST cells c) Vitamin deficiency d) Spherocytosis
c) Eosinophils d) R.B.C s (APPSC -99, AP - 2013)
(AIIMS -89) 50. Persons having anti-A and anti-B isoagglutinins in
41. Infants acquire adult hemoglobin level at: their serum belong to which blood group
a) Birth b) End of pt year a) A b) B
c) Puberty d) 4 years c) AB d) 0
(PGI -95) (APPSC -99)
42. Which of the following is not involved in blood 51. The universal blood recipient group is:
clotting? a) 0 -Ve b) 0 +Ve
a) Calcium b) Prothrombin c) AB -Ve d) AB +Ve
c) Fibrinogen d) Fe+2 (APPSC -99)
(AIIMS -89) 52. Serum is blood plasma without:
43. Lysis of RBCs are seen in all of the following except: a) Blood cells b) Fibrinogen
a) Thalassemia b) Methotrexate therapy c) Lymphocytes d) Plasma colloids
c) Obstructive jaundice d) Sickle cell anemia (MAN -96)
(AIPG -90) 53. The principal site for granulocytic hemopoiesis in
44. The only fixed cell of the connective tissue: the adult human is:
a) Histiocyte b) Lymphocyte a) The liver b) The spleen
c) Neutrophils d) Mast cell c) Red bone marrow d) Yellow bone marrow
(KAR -03) (KAR -95)
45. A 55-year-old male accident victim in casualty 54. Survival time of platelet is approximately:
urgently needs blood. The blood bank is unable to a) 10 hours b) 10 days
determine his ABO group, as his red cell group and c) 10 weeks d) 100 days
plasma group do not match. Emergency transfusion (TNPSC -99)
of patient should be with: 55. Plasma thromboplastin antecedent is:
a) RBC corresponding to his red cell group and a) Anti hemophilic - A factor
colloids/crystalloid b) Anti hemophilic - B factor
b) Whole blood corresponding to his plasma group c) Anti hemophilic -C factor
c) 0 -ve RBC and colloids/crystalloid d) Anti hemophilic - F factor
d) AB negative whole blood (APPSC -99)
(AIIMS -03) 56. The largest fraction of the total body fluid is:
46. Fluid content of the blood is: a) Extracellular fluid b) Intracellular fluid
a) Plasma b) Serum c) Plasma d) Whole blood
c) Water d) Tissue fluid (APPSC -99)
(KAR -95) 57. The most common cause of eosinophilia in India is:
a) Allergic disorders b) Worm infestation

38) C 39) D 40) A 41) B 42) D 43) C 44) A 45) C 46) A 47) D 48) D 49) B 50) D
51) D 52) B 53) C 54) B 55) C 56) B 57) B
, PHYSIOLOGY
~

V
989

c) Chronic myelocytic leukemia c) Liberation of histamine


d) Rheumatoid arthritis d) Destruction of old erythrocytes
(KAR -97)
58. Small lymphocyte is: 68. Immune complexes are removed from blood by:
a) Bigger than RBC b) Same as the size of RBC a) B cell b) Basophil
c) Smaller than RBC d) None of these c) Plasma cell d) Kupffer cell
(TNPSC -99)
59. The biological anticoagulant is: 69. Patient with anemia tends to have all, except
a) EDTA b) Sodium citrate a) Compensatory increase in cardiac output
c) Hirudine d) Double oxalate mixture b) Increased incidence of heart murmurs
(TNPSC -99) c) Pallor of mucous membranes
60. Christmas factor is: d) A low P0 2 in arterial blood
a) PTA b) PTC
c) VWA d) Prothrombin 70. CO 2 is transported in plasma as
(KAR -98) a) Bicarbonate b) Carbomino compounds
61. The normal A/G ratio in blood is: c) Dissolved form d) CO
a) 5:1 b) 2:1
c) 1:2 d) 1:1 71. AB blood group antigen are known as_ _factors
a) Duffy b) Landsteiner
62. Ions participating in clotting mechanism are: c) Rhesus d) Lutheran
a) Iron b) Copper
c) Calcium d) Aluminium 72. A reliable screening test for platelet function is:
(AP - 04) a) CT b) PT
63. Acute bacterial pyogenic infection manifests as: c) Thrombin d) Clot retraction time
a) Leukopenia b) Leukocytosis
c) Neutropeni d) Lymphopenia 73. Iron is stored in
(AIPG-2011) a) RBC
64. Increased blood viscosity and slow circulation b) Reticule endothelial system
causes: c) Plasma d) All
a) RBC rouleax formation
b) Increased plasma skimming 7 4. True statement regarding h ematopoiesis includes
c) Increased number of RBC in capillaries a) Spleen plays a major role irn adults (in lymphopoiesis)
d) None b) Committed stem cells differentiates into only one cell
type
65. CO2 is carried in the blood: c) RES plays a role
a) In combination with hemoglobin d) None
b) In physical solution in plasma
c) In combination with plasma proteins 75. Biconcave shape of RBC helps by
d) All of the above a) Increasing flexibility
b) Increasing surface area
66. Thromboxane is produced mainly by: c) Carrying more Haemoglobin
a) Liver b) Platelets d) Passing easily through smaller capillaries
c) Damaged tissue d) Vascular endothelium
76. Decreased MCHC is found in
6 7. The function common to neutrophils, monocytes & a) Microcytic hypochromic anemia
macrophages is: b) Megaloblastic anemia
a) Immune response b) Phagocytosis c) Sideroblastic anemia d) Vit B12 deficiency

58) C 59) C 60) B 61) B 62) C 63) B 64) A 65) D 66) B 67) B 68) D 69) D 70) A
71) B 72) D 73) B 74) C 75) D 76) A
, PHYSIOLOGY
~

V
991

c) Epithelial tissue d) Connective tissue d) Potassium oxalate


(COMEDK-07) (AIIMS-2012)
97. Plasma ceruloplasmin alpha 2 globulin is a 107. Activation of which one of the following coagulation
a) alpha 1 globulin b) alpha 2 globulin factor is not essential for haemostasis
c) beta 1 globulin d) beta 2 globulin a) XII b) XI
(COMEDK-2011) c) IX d) X
98. In human the Hb is: (COMEDK -2013)
a) HbH b) HbA 108. Plasma factor present in both serum & plasma is:
c) HbM d) HbS a) V b) VII
(AIIMS-07) c) II d) Fibrinogen
99. Which of following causes lysis of clot : (NEET-2013)
a) Fibrin b) Plasmin 109. Haemostasis depends up on all the following,
c) Hyaluronidase d) Coagulase EXCEPT
(AIIMS-07) a) Calcium b) Prothrombin
100. Site of RBC formation in 20 year old healthy male is: c) Vitamin B d) Vitamin K
a) Flat bones b) Long bones (AP-2013)
c) Liver d) Yolk sac 110. Which of the following is not a general compartment
(AIPG-10) of body fluid ?
101. Lymphocytes are Located in each of the following a) Peritonea l b) Intracellular
tissues or organs EXCEPT one. The one exception is - c) Interstitial d) Blood plasma
a) Brain b) Spleen (AIIMS NOV-13)
c) Lymph nodes d) Thymus gland 111. Primary sites of Lymphocyte production are
(KCET-09) a) Bone marrow and Thymus
102. The enzyme system which continuously reduces b) Spleen and Lymph nodes
ferric form of iron in the blood to the ferrous form c) Lymph nodes and mucosa associated Lymphoid
is tissue
a) Methemoglobin reductase d) Spleen and mucosa associated Lymphoid tissue
b) Chymotrypsin (AP-14)
c) Amylase d) Dehydrogenase 112. Heparin therapy is best monitored by which of the
(AP-2012) following given test?
103. Embryonic hemoglobin is composed of a) Prothombin time b) Bleeding time
a) Gamma and beta b) Epsilon and gamma c) Activated Partial Thrombo plastin Time
c) Alpha and beta d) Gamma and alpha d) International Normalised Ration
(AIPG-2012) (COMEDK-14)
104. Blood cells arise in bone marrow and are subject to 113. Incorrect statement is
a) Irregular renewal b) Regular renewal a) Half-life of factor 9 is less than factor 8
c) Infrequent renewal d) None of the above b) Half-life of factor 8 is Less than factor 9
(BHU-2012) c) Von willebrand can occur in females
105. The half life of plasma Albumin is approximately d) Hemophilia b does not occur in females
a) ?days b) 20days (PGI JUNE-2014)
c) 60 days d) 90 days
(COMED-2012)
106. In PHC, which anticoagulant is used to send the
blood sample for blood glucose:
a) EDTA b) Heparin
c) Potassium oxalate + sodium fluoride

97) B 98) B 99) B 100) A 101) A 102) A 103) B 104) B 105) B 106)( 107) A 108) B 109) C
110) B 111) A 112) C 113) B
Dental ;lut.,e

43. 'C' [Sembulingam 4th ed 77 / 5th ed 87] 54. 'B' [Sembulingam 4th ed 108/ 5th ed 116]
Hemolysis is seen in thalassemia, sickle cell anemia,
methotrexate therapy and in hemolytic jaundice. 55. 'C' [Sembulingam 4th ed 111/ 5th ed 120]
Factor XI is ca lled as antihemophilic C (or) plasma
44, 'A' [Sembulingam 4th ed 130/ 5th ed 141] thromboplastin antecedent (PTA).

4 5. 'C' [Sembulingam 4th ed 119/ 5th ed 129] Factor IX is called as plasma thromboplastin component
(PTC).
46. 'A' [Sembulingam 4th ed 53/ 5th ed 56]
Blood contains blood cells and liquid portion known 56. 'B' [Sembulingam 4th ed 48/ 5th ed 50]
as plasma. The plasma contains 92% water and 8% Water is the largest fraction of body fluid. Intracellular
organic and inorganic solids. fluid consists of 55% of total body fluid and
extracellular fluid consists of 45% of the total body
47. 'D' [Se mbulingam 4th ed 98/ 5th ed 105] fluid. Extracellular fluid consists of interstitial fluid,
plasma and other fluids.
48. 'D' [Sembulingam 4th ed 54/ 5th ed 57]
The plasma proteins are albumin, globulin and 57. 'B' [Sembulingam 4th ed 90/ 5th ed 96]
fibrinogen. Albumin is the major component among
them. Plasma proteins are essential in coagulation of 58. 'C' [Se mbulingam 4th ed 88/ 5th ed 93]
blood, maintenance of osmotic pressure, transport of A small lymphocyte has a diameter of about 7 microns
enzymes, hormones, gases and as buffers etc. and RBC has a diameter of about 7.5 microns.

49. 'B' [Sembuli ngam 4th ed 89/ 5th ed 94] 59. 'C' [Sembulingam 4th ed 115/ 5th ed 125]

50. 'D' [Sembulingam 4th ed 119/ 5th e d 130] 60. 'B' [Se mbulingam 4th ed 111/ 5th ed 120]
Agglutinogens in Agglutinin in
Group 61. 'B' [Sembulingam 5th ed 43]
RBC plasma

A A Anti B/Beta 62. 'C' [Sembulingam 4th ed 110/ 5th ed 119]


B B Anti A/alpha
63. 'B' [Sembulingam 2nd ed 64]
AB A, B No agglutinin
0 No agglutinogens Anti A and Anti B
• Infections
• Allergy
51. 'D' [Sembulingam 4 th ed 121/ 5th ed 133] Leukocytosis • Common cold
Rh+ve patient can receive blood from both Rh+ve • TB
and Rh-ve persons with out the risk of developing • Infectious mononucleosis
complication but Rh-ve patient can receive blood from
only Rh-ve persons. So AB positive is the universal • Anaphylactic shock
recipient and 'O' negative is the universal donor. • Cirrhosis
Leukopenia • Pernicious anemia
52. 'B' [Sembulingam 4th ed 57 / 5th ed 60) • Typhoid and paratyphoid
In embryonic life, erythropoiesis occurs in yolk sac,
liver and spleen.
• Viral infections

64. 'A' [Sembulingam 4th ed 62/ 5th ed 65]


In postnatal life and adults, erythropoiesis occurs only
in the red bone marrnw.
65. 'D' [Sembulingam 5th ed 50, 61)

53. 'C' [Sembulingam 4th ed 65/ 5th ed 69]


66. 'B' [Se mbulingam 4th ed 108/ 5th ed 116)
, PHYSIOLOGY
~
995
V

67. 'B' [Sembulingam 4th ed 90/ 5th ed 96) 81. 'C' [Sembulingam 4th ed 115/ 5th ed 125]
Partial thromboplastin time is used to assess the
68. 'D' integrity of intrinsic pathway of coagulation of blood.
As hemophilia is caused by factor VIII or IX which are
69. 'D' [Sembulingam 4th ed 77/ 5th ed 87) factors in intri nsic pathway PTT is appropriate.

70. 'A' [Sembulingam 4th ed 38/ 5th ed 42) Prothrombin time is ideal to assess the integrity of
extrinsic pathway of blood coagulation.
71. 'B' [Sembulingam 4th ed 118/ 5th ed 129)
82. 'B' [Guyton 11th ed 427]
72. 'D' [Sembulingam 4th ed 113/ 5th ed 122) Causes of secondary polycythemia
• High altitude
73. 'B' [Sembulingam 4th ed 73/ 5th ed 77)
• Carbon monoxide intoxication
• High affinity hemoglobin
74. 'C' [Sembulingam 4th ed 65/ 5th ed 69)
• Respiratory center dysfunction.
75. 'D' [Sembulingam 4th ed 61/ 5th ed 64) • Cerebellar hemangioblastoma
• Pheoch romocytoma
76. 'K [Sembulingam 4th ed 83/ 5th ed 83)
Mean corpuscular haemog lobin concentration is the The myeloproliferative disorders are characterized
most important absolute va lue in the diagnosis of pathophysiologically by clonal expression of a
Anemia. multipotent hematopoitic progenitor cell with the
Appearance overproduction of one or more of the formed element s
Anemia type MCV MCH MCHC
on the slide of blood. These disorders may transform into acute
Microcytic leukemia naturally or as a consequence of mutagenic
Iron deficiency
Hypochromic + + + treatment.
Megaloblastic
(Pernidous)
Macrocytic t t Normal Myloproliferative disorders classically include:
Hemolytic Normocytic Normal Normal Normal • Polycythemia vera or primary polycythemia
• Idiopathic myelofibrosis
77. 'A' [Sembulingam 4th ed 90/ 5th ed 96) • Essential throm bocytosis
Primary granules of 11eutrophils contain myeloperoxidase • Chronic myeloid leukemia
while secondary granules contains lactoferrin.
83. 'K [Chaudary 4th ed 31]
78. 'A' [Sembulingam 4th ed 58/ 5th ed 61) Iron, copper and cobalt are necessary for hemoglobin
Plasma proteins exert colloidal osmotic pressure. synthesis.
As the concentration of albumin is more than other
plasma proteins, it exerts maximum pressure. 84. 'B'

79. 'D' [Sembulingam 4th ed 115/ 5th ed 125] 85. 'A' [Sembulingam 5th ed 119, 120]
Clotting time is prolonged in the first three options.
Large quantities of aspirin increases bleeding time due 86. 'D' [Sembulingam 4th ed 81/ 5th ed 81)
to its antiplatelet activity. Factors affecting ESR:
• Specific gravity of RBC
Bleeding time is also prolonged in purpura and Von
Willebra nd disease. • Rouleaux formation
• Increase in size of RBC
80. 'D' [Sembulingam 4th ed 112/ 5th ed 121] • Viscosity of blood
• Increased viscosity decreases ESR
, PHYSIOLOGY

102. 'A' [Sembulingam 5th ed 76] NaF is weak anticoagulant on its own, So potassium
Methemoglobin is the abnormal hemoglobin derivative oxa late or lactate is added to supplement its action.
formed when iron molecule of hemoglobin is oxidized
from ferrous state to ferric state. Normal met hemoglobin The fluo ride-oxalate mixture used for this purpose is
level is 0.6%-2.5% of t otal hemog lobin. Under normal composed of 2mg NaF and 6mg potassium oxalate per
circumstances also body faces the threat of continuous ml of blood.
production of methemoglobin. It is prevented by
presence of methemoglobin reductase. It prevents the Note:- >5mg of NaF/ml of blood acts as inhibitor for
oxidation of Fe+2 iron into Fe+3 iron. the enzymatic estimations.

103. 'B' [Ganong 23rd ed 525] 107. 'K [Check Explanation Below]
Hemoglobin Polypeptide chains Coagulation of intrinsic pathway invitro occurs with
• Gower 1: 2 Zeta, 2 epsilon
activation of factor XII. Activation of factor XII is not
essentially required for hemostasis, since patients with
Embryonic • Gower 2: 2 alpha, 2 epsilon deficiency of factor XII do not bleed even though their
• Portland 1: 2 Zeta, 2 gamma APTI (Acquired partial thromboplastin time) values
Fetal (F) 2 o. chains and 2 y chains are prolonged.
Adult (A) 2 o. chains and 2 ~ chains
108. 'B' [Ganong 1st ed 542/ Chandrasoma Taylor 3rd ed
Adult (A2) 2 o. chains and 2 delta chains
426]
Serum has essentially the same composition as
104. 'B' [Ganong 23rd ed 552]
plasma except that its fibrinogen and clotting factors
II, V, and VIII have been removed and it has higher
105. 'B' [Satyanarayana Biochemistry 3rd ed 458]
serotonin content beca use of breakdown of platelet s
during clotting.
106. 'C' [Sembulingam 5th ed 124/ Practical biochemistry
by Shankara 56]
109. 'C' [Sembulingam 5th ed 117]
Anticoagu Lant Uses
• Invivo IV injection . 110. 'B' [Ganong 23rd 2]
Extracellular fluid (ECF) or body fluids or internal sea
• To preserve blood before
Heparin transfusion is divided into two components i.e., interstial fluid
and circulating blood plasma. Other extracellular fluids
• Invitro while collecting blood
that are also considered as interstial fluids are referred
for various investigations.
as trancellular fluids. Eg: CSF, pericardia[ , peritoneal,
Coumarin
GIT secretions and synovia l fluid.
(Dicoumaral • Invivo oral anticoagulants
and warfarin)
111. 'K [Sembulingam 5th ed 97]
• IV injection in lead poisoning
EDTA cases 112. 'C' [Check explanatfon of Q.No.37 in chapter 6 of
• To preserve blood in laboratory General Pathology]
• Only invitro for blood
Oxalate
investigations 113. 'B' [MC Donald 9th ed 488)
• To store blood in banks. The main stay of therapy for optional prophylactic
Citrates • Used for RBC and platelet counts treatment of haemophilia relies upon injections
(formal-citrate) of facto r VIII or factor IX concentrate to prevent
bleeding. As the half-life of factor VIII is only 12
Sodium fluoride stabilizes cell membrane and inhibits hours, injections need to be given on alternate days,
enzymes involved i11 glycolysis. So it prevents red blood whereas factors IX, because of its longer half-life of 18
cells metabolizing any glucose present in the sample. hours, need be given on ly twice weekly.
, PHYSIOLOGY

85. Hypothyroidism should be treated with daily 94. Paneth cells secrete which of the following ?
administration of which of the following thyroid a) Antibacterial substance
hormone preparations? b) Lipase
a) Thyroid extract b) Thyroglobulin c) Maltase d) Secretin
c) Thyroxine (T4) d) Triiodothyronine (T3) (MCET-07)
(COMEDK-05) 95. Glucose tolerance test is usually done to assess:
86. Blood coagulation is impaired in a) Acute Pancreatitis
a) Tetany b) Hyperparathyrodism b) Carcinoma of Head of Pancreas
c) Rickets d) None c) Acinar Function of the Pancreas
d) Endocrine Dysfunction of Pancreas
87. All of the following are adverse effects of long- (AIIMS-07)
term corticosteroids EXCEPT - 96. Tetany is characterised by:
a) Hypoglycaemia b) Psychosis a) Hypotonicity of muscles
c) Peptic ulcers d) Osteoporosis b) Hypertonicity of muscles
(COMEDK-09) c) Increased serum calcium concentration
88. All of the following hormones have cell surface d) None of above
receptors except: (MCET-07)
a) Adrenalin b) Growth Hormone 97. Which is correct?
c) Insulin d) Thyroxine a) U-V rays help in formation of Vit. Din skin
(AIPG-05) b) Renal failure leads t o increase in parathormone
89. Which amongst the following is the gold standard release
for demonstrating hCG c) 25-hydroxycholeca lciferol undergoes hydroxylation
a) Radio immunoassay b) Latex agglutination in kidney
c) Immunoflorescence d) ELISA d) All of the above
(AIPG-10) (PG! DEC-2011)
90. Osteoclasts are inhibited / modified and regulated 98. The effect which is seen due to decrease in serum
by: calcium concentration is
a) Parathyroid hormone a) Relaxation of muscle
b) Calcitonin b) Excitability of the muscle
c) 1,25 - dihydroxycholecalciferol c) Increase the renal absorption
d) Tumour necrosis factor d) Depression of Nervous system
(AIPG-05, COMEDK-08) (AP-08)
91. Which one of the following is a precursor of both 99. Insulin increases the activity of
gonadal and adrenocortical hormones? a) HMG-CoA reductase b) HMG-CoA lyase
a) Progesterone b) Cortisol c) HMG-CoA synthase d) Thiolase
c) Testosterone d) Corticosterone (COMEDK-10)
(COMEDK-05) 100. The most significant immediate result of lowered
92. Following are the features of cretinism, except serum calcium is
a) Pot - belly b) Idiotic look a) weakened heart action
c) Normal intelligence d) Stunted growth b) decalcification of bones
(KAR-04) c) decalcification of teeth
93. Inhibin b hormone has effect on d) hyperirritability of nerves and muscles
a) Sertoli cells (AP-09)
b) FSH 101. Hyperthyroidism can be caused by
c) Seminiferous tubules a) Clonidine b) Amiodarone
d) Spermatogenesis c) Hydralazine d) Penicillamine
(AIPG-10) (COMEDK-10)

85) C 86) A 87) A 88) D 89) A 90) B 91) A 92) C 93) B 94) A 95) D 96) B 97) D
98) B 99) A 100) D 101) B
Dental ;lut.,e

102. Hormone responsible for milk ejection:


a) Oxytoci n b) Estrogen
c) Progesterone d) Prolactin
(IGNOU-10)
103. Hormones FSH & LH of anterior pituitary gland acts
on
a) Testis but not ovaries
b) Ovaries but not testis
c) Both Testis and ovaries
d) Adrenal cortex
(AP-10)
104. Aldosterone secretion is increased when there is a
fall in
a) Plasma K+ b) Plasma Na+
c) pH of the plasma d) Angiotensin II levels
(KCET-2012)
105. Removal of parathyroid gland produces the
following changes EXCEPT
a) Decline plasma calcium level
b) Decrease in plasma phosphate level
c) Neuromuscular hyperexcitability
d) Hypocalcemic tetany
(COMEDK -2013)
106. Fertilized ovum usually gets attached to the uterus
wall in:
a) 14 days b) 28 days
c) 8 weeks d) 1 week
(NEET-2013)
107. The Hormone in hi biting both resorption and
formation of bone
a) Estrogen b) Vitamin D
c) Glucocorticoids d) Pa rathormone
(COMEDK-14)
108. Escape phenomenon in mineralocorticoid excess
occurs due to
a) Renin
b) Mineralocorticoid like action of cortisol
c) ANP (Atrial Natriuretic Peptide)
d) Angiotensin II
(KERALA-2015
109. Which of the following has growth hormone
stimulating activity?
a) IGF-1 b) Somatostati n
c) REM sleep d) Ghrelin
(KERALA-2015)

102) A 103) C 104) B 105) B 106) D 107) C 108) C 109) D


, PHYSIOLOGY
~

V
1007

effect), force of contraction (ionotropic effect) and 45. 'B' [Sembulingam 5th ed 397, 386]
excitability of heart muscle.
46. 'D' [Sembulingam 4th ed 352/ 5th ed 366]
33. 'D' [Sembulingam 4th ed 372/ 5th ed 384]
47. 'B'
34. 'C' [Sembulingam 4th ed 388/ 5th ed 404]
48. 'N [Sembulingam 4th ed 351/ 5th ed 365]
35. 'B' [Sembulingam 4th ed 400/ 5th ed 417]
In Cushing's syndrome there is redistribution of body 49. 'B' [Sembulingam 4th ed 383/ 5th ed 399]
fat in an abnormal manner resulti ng in moon face and
buffalo hump. Hyperglycemia, hypertension, increased 50. 'D' [Sembulingam 4th ed 366/ 5th ed 376]
catabolism of proteins and immunosuppression are
other features of Cushing's syndrome. 51. 'C' [Guyton 10th ed 840]
Most hormones initiate a cellular response by initially
36. 'B' [Sembulingam 4th ed 354/ 5th ed 368] combining with either a specific intracellular or cell
membrane associated receptor protein. For many
37. 'D' [Sembulingam 4th ed 397/ 5th ed 414] hormones, including most protein hormones, the
receptor is membrane-associated and embedded
38. 'B' [Sembulingam 4th ed 387/ 5th ed 403] in the plasma membrane at the surface of the cell.
For hormones such as steroid or thyroid hormones,
39. 'B' [Sembulingam 4th ed 362/ 5th ed 376] their receptors are located intracellularly within the
cytoplasm of their target cell. To bind their receptors,
40. 'C' [Sembulingam 4th ed 363/ 5th ed 377] these hormones must cross the cell membrane. Like all
Thyroxine increases the sensitivity of heart to steroid hormones, estrogens readily diffuse across the
adrenaline. LA with adrenaline is contraindicated in cell membrane. Once inside the cell, they bind to and
patients with thyrotoxicosis. activate estrogen receptors.

41. 'A' [Sembulingam 4th ed 362/ 5th ed 376] 52. 'N [Sembulingam 4th ed 397/ 5th ed 414]

42. 'A' [Orban's 13th ed 212] 53. 'N [Sembulingam 4th ed 354/ 5th ed 368]
There are three types of cells in bone; osteoblasts,
osteocytes and osteoclasts. Osteoclasts do not 54. 'C' [Sembulingam 4th ed 349/ 5th ed 363]
express any PTH receptor. Osteoblasts express Growth hormone does not have any direction action
receptors for various hormones including PTH, Vitamin on bones. It acts through a substance released from
D3, estrogen, and glucocorticoids. PTH hormone acts liver called somatomedin, which acts on the bone and
via osteoblasts to increase the activity and number causes growth changes in them
of osteoclasts. Osteoblasts mediate the signal from
PTH by releasing cytokines, which increase osteoclast 55. 'D' [Sembulingam 4th ed 395/ 5th ed 412]
activity. PTH also recruits more osteoclasts by Steroid hormones are lipid soluble; they are t ransported
accelerating the maturation of osteoclast precursors. by simple diffusion through lipid layers.

43. 'A' [Sembulingam 4th ed 387/ 5th ed 403) 56. 'N [Sembulingam 4th ed 351/ 5th ed 365)
• Alpha cells secrete glucagon ADH is secreted mainly by supraoptic nucleus of
• Beta cells secrete insulin hypothalamus (AP-08) and oxytocin by paraventricular
nucleus. From hypothalamus these hormones are
• Delta cells secrete somatostatin
transported to posterior pituitary through hypothalamo
• PP cells secrete pancreatic polypeptide hypophyseal tract by axonal flow.

44. 'B' [Ganong 21st ed 399]


Dental ;lut.,e

57. 'B' [Sembulingam 4th ed 404/ 5th ed 421]


Hormones are classified based on chemistry as
Thyroxine and catecholamines are synthesized from
amino acid tyrosine. Steroid hormones Eg: Corticosteroids and sex
(derivatives of hormones
58. 'B' [Sembulingam 4th ed 398/ 5th ed 415] cholesterol)
Glucocorticoids exhibit anti-inflammatory effect by Derivatives of Eg:- Thyroid & adrenal
preventing the release of histamine and inhibition of aminoacid tyrosine medullary hormones
migration of leukocytes. • Glycoproteins
Eg:- TSH, LH, FSH
59. 'D' [Sembulingam 4th ed 472/ 5th ed 493]
Protein hormones • Polypeptides -
60. 'B' [Sembulingam 4th ed 372/ 5th ed 384] Eg:- Growth hormone ACTH,
Insulin, Prolactin
61. 'A' [Sembulingam 4th ed 393/ 5th ed 410]
74. 'B' [Sembulingam 4th ed 400/ 5th ed 417]
62. 'B' [Sembulingam 4th ed 394/ 5th ed 411] Zona
Cortisol is called life-protecting hormone as it helps to Mineral corticoids
glomerulosa
withstand the stress and trauma in life. Aldosterone is
called as life saving hormone. Zona fasciculata Gluco corticoids
Sex hormones and glucocorticoids
Zona reticularis
63. 'C' [Sembulingam 4th ed 440/ 5th ed 461] to some extent
In non-pregnant women source of progesterone is
corpus luteum. In pregnancy it is released from corpus 75. 'C' [Sembulingam 4th ed 401/ 5th ed 418]
luteu m and later by placenta.
76. 'B' [Sembulingam 4th ed 400/ 5th ed 417]
64. 'C' [Sembulingam 4th ed 445/ 5th ed 465]
17. 'A' [Sembulingam 4th ed 397/ 5th ed 414]
65. 'C' [Sembulingam 4th ed 363/ 5th ed 377]
78. 'A' [Sembulingam 4th ed 352/ 5th ed 366]
66. 'B' [Sembulingam 4th ed 361/ 5th ed 375]
Thyroxine binds to globulin called Thyroxine Binding 79. 'D' [Sembulingam 4th ed 347/ 5th ed 361]
Globulin.
80. 'A' [Sembulingam 4th ed 412/ 5th ed 429]
67. 'A' [Sembulingam 4th ed 406/ 5th ed 423) Endocrine hormones are secreted in one place and
show their actions on some other places whereas local
68. 'B' [Sembulingam 4th ed 454/ 5th ed 474] hormones exhibit their action in the same area.

69. 'C' [Sembulingam 4th ed 366/ 5th ed 376] Local hormones are prostaglandins, thromboxanes,
acetylcholine, histamine, heparin, gastrointestinal
70. 'C' [Sembulingam 4th ed 360/ 5th ed 374] hormones and serotonin, etc.

71. 'C' [Sembulingam 4th ed 466/ 5th ed 487] 81. 'D' [Sembulingam 4th ed 383/ 5th ed 399]

7 2. 'B' [Sembulingam 4th ed 346/ 5th ed 360) 82. 'C' [Sembulingam 4th ed 407/ 5th ed 424)
Acidophilic cells of anterior pituitary secrete Nor adrenaline is called the "general vasoconstrictor"
somatotropins and Prolactin; Basophilic cells secrete as it causes constriction of blood vessels through
other anterior pituitary hormones. out the body via alpha receptors. Total peripheral
resistance is increased by noradrenaline leading to
73. 'B' [Sembulingam 4th ed 383/ 5th ed 399] increased diastolic blood pressure.
Dental ;lut.,e

96. 'B' [Sembulingam 4 th ed 374/ 5th ed 387] • Stimulates the secretion of estrogen
• Promotes aromatase activity in granulose cells
97. 'D' [Gan ong 24th ed 379, 383] resulting in conversion of androgen into estrogen.
• Vitamin D is synthesized when sun's ultraviolet
B (UVB) rays interact with 7-dehydrocholesterol II) Actions of LH ( Luteinizin g hormone)
present in the skin to form cholecalciferol (Vit. D3).
a) In Males:
• Vit D3 is converted to 25-hydroxycholecalciferol • Known as interstitial cell stimulating hormone
(25-0HD3, Calcidiol) in liver by the action of (ICSH)
enzyme 25-hydroxylase.
• Stimulates Leydig cells in testes
• In kidney, 25- hydroxy cholecalciferol is converted
to 1,25-dihydroxy choleca lciferol (calcitriol or D3). b) In Fema les:
• In chronic renal fai lure, in which plasma calcium
• Maturation of vesicular follicle into graafian
level is low, stimulation of parathyroid gland occurs follicle
resulting in parathyroid hypertrophy and secondary
• Responsible for ovulation
hyperparathyroid ism.
• Necessary for the formation of corups luteum
98. 'B' [Sembulingam 4 th ed 374/ 5th ed 387]
Decreased serum calcium causes hyperexcitability of 104. 'B' [Ganong 23rd ed 356, 357 Table 22-6]
nerves and skeletal muscles. Conditions that increase aldosterone secretion

When • Surgery
99. 'A' [KD Tripathi 5t'1 ed 238]
Insulin inhibits lipolysis i.e., it indirectly promotes g lucocorticoid • Anxiety
cholesterol synthesis. Hence it increases activity of secretion also • Physical trauma
HMG-CoA reductase. increased • Hemorrhage
• High potassium intake
But according to key both A & Dare correct answers.
• Low sodium intake
100. 'D' [Sembulingam 4th ed 374] • Constriction of inferior vena
When the concentration of the ionized calcium is Glucocorticoid cava in thorax
lowered, the more immediate effect is hyperirritability secretion • Standing
of the neuro-muscular system results. unaffected
• Secondary hyperaldosteronism
(in some cases of congestive
101. 'B' [KD Tripathi 5tll ed 482] heart failure, cirrhosis, and
Amiodarone is an antiarryhythmic drug. It causes nephrosis)
drug induced hyperthyroidism. It may be prevented by
monitoring this possible side effect and weighing it
105. 'B' [Se mbulingam 5th ed 384]
against the benefit s of using this heart medication.
Parathyroid decreases blood level of phosphate. It
stimulates resorption of phosphate and increases its
102. 'A' [Sembulingam 5th ed 367]
urinary excretion. So removal of PTH glands increases
blood phosphate level.
103. 'C' [Sembulingam 4th ed 364]

Also refer Q. No. 20


I) Action of FSH (Follicle Stimulating Hormone)
a) In Males: 106. 'D' [Sembulingam 5th ed 478/ Maternity and
FSH along with testosterone accelerates the process
pediatric nursing: 79]
of spermeogenesis.
It takes about 1 week for implantation of the fertilized
ovum to the endometrial lining of uterus.
b) In Females:
• Development of graffian follicle
Dental ;lut.,e

9. MISCELLANEOUS
1. During exercise: 8. is an example for rapidly acting
a) Cerebral bood flow increases if there is increase in neurotransmitter
systolic blood pressure a) Somatostatin b) Calcitonin
b) Body temperature increases c) Substance P d) Glycine
c) Blood flow to muscle increases after 11'2 minute (KCET-10)
d) Lymphatic flow from muscle decreases 9. T-lymphocytes play a primary role in
(AIIMS-07) a) Production of Antibodies
2. When a person lies down : b) Production of Lymphokines and delayed
a) There is immediate increase in venous return hypersensitivity
b) Heart rate becomes stable at slightly more than c) Activation of complement system
normal d) Im mediate Hypersensitivity
c) Cerebral blood flow increases & becomes stable at (AIIMS-08)
more than normal 10. Regarding golgi tendon organ true is?
d) Blood flow to the Apex of lung decreases a) Senses dynamic length of unuscle
(AIIMS-07) b) Involved in reciprocal innervations
3. The percentage of body water is greater in c) a-motor neuron stimulation
a) male than in females d) Sense muscle tension
b) children than in adult (AIPG-09)
c) obese than in lean individua ls 11. To check objective pain response, which is best
d) old than in young objects used method
(KAR-03) a) Facial pain scale b) Knee jerk reflex
4. Open faced nucleus in a cell signifies c) H - Reflex d) R III Reflex
a) th at the cell is resting (AIIMS-08)
b) that the cell is active 12. Valve of Hasner is present at
c) nothing a) Stensons duct b) Common bile duct
d) that the cell is in transition phase c) Naso lacrimal duct d) Maxillary Sinus
(KCET-04) (MCET-10)
5. Meiotic division of male germ cells commence 13. Gut associated Lymphoid tissue (GALT) is primarily
a) during intra-uterine life Located in -
b) just before birth a) Lamina propria b) Sub mucosa
c) by around 6 years after birth c) Muscularis d) Serosa
d) by around 16 years (AIIMS-09)
(COMEDK-10) 14. The processing of short term memory to long term
6. Primary cause of bleeding disorder in Liver damage memory is done in
is due to a) Prefrontal cortex b) Hippocam pus
a) decreased Level of prothrornbin c) Neocortex d) Amygdala
b) Lack of vitamin 1K (AIPG-10)
c) Platelet deficiency d) Lack of vitamin B 15. Which of the following events DO NOT occur in rods
(COM EDK-10) in response to light -
7. Which among the following is also called "peakless" a) Structural changes in rhodopsin
insulin analog? b) Activation of transducin
a) Insulin lispro b) Insulin glargine c) Decreased intracellular cGMP
c) Insulin aspart d) Lente Insulin d) Opening of Na+ channels
(COM EDK-10) (COMEDK-09)

1) B 2) A 3) A 4) B 5) D 6) A 7) B 8) D 9) B 10) D 11) A 12) C 13) A


14) B 15) D
, PHYSIOLOGY
~

V
1013

16. False about blood brain barrier is c) Procollagen marker d) Hydroxy praline
a) There are gaps between tight junctions at the (NEET-2013)
active sites 25. The oral and pharyngeal phases of swallowing take
b) There are no ga ps between active sites place very rapidly and last about
c) Less activity in the membrane a) 3.5 seconds b) 1-1.5 seconds
d) Fibronectin provides a barrier for diffusion c) 6.0 seconds d) 9.5 seconds
(AIPG-10) (AP-2013)
17. The male sex hormone testosterone is produced by 26. In a DNA molecule adenine always forms a linkage
a) Sertoli cells b) Epithelial cells with
c) Interstitial cells of leydig a) Guanine b) Cytosine
d) Primitive term cells c) Thymine d) Uracil
(KCET-2011) (COMEDK-2013)
18. Decreased basal metabolic rate is seen in: 27. Sphygmomanometer reading can be correct only
a) Obesity b) Hyperthyroidism when
c) Feeding d) Exercise a) deflation is 5-10 min Hg per heart beat
(AIPG-2011) b) cuff is slightly full on applying to patient's arm
19. In peripheral tissues which of the following c) the cuff width is 25% of the diameter of the
contains substance P: patient's arm
a) Plasma cell b) Mast cell d) length of the bladder encircle 80% of the patient's
c) Nerve terminal d) Vascular endothelium arm
(AIPG-2011) (MCET - 14)
20. The acrosome reaction occurs, when the sperm 28. What are 'G-proteins' ?
a) Enters the uterine cavity a) Membrane receptors b) Membrane regulators
b) Comes in contact with zona pellucida c) Second messengers d) Membrane channels
c) Penetrates corona radiate (GCET - 14)
d) Penetrates cell membrane of oocyte 29. A man working under conditions of high temperature
(KCET-2011) should increase his intake of
21. Most important function of albumin in body is a) NaCl b) Iron
a) Oncotic pressure b) Drug t ransport c) Vitamin D d) Phosphates
c) Toxic transport d) Coagulation (AP - 14)
(AIPG-2011) 30. The control for mastication is normally exercised by
22. Regulated bone growth through puberty occurs a) Pontine centres b) Sub cortical centres
through c) Cerebellar centres d) Medulla centres
a) Endophyseal plates b) Ectophyeseal plates (COMED - 14)
c) Epiphyseal plates d) Chondrophyseal plates 31. The mode of secretion of sebaceous gland is
(BHU-2012) a) Merocrine b) Holocrine
23. The major functional difference between DNA and c) Apocrine d) Eccrine
RNA is (MHCET-15)
a) RNA contains ribose
b) DNA carries the information in all organisations
c) DNA is localized in nucleus
d) RNA does not contain thymine
(AP-2013)
24. Which of the following is not a marker of bone
formation:
a) Alkaline phosphatase
b) Osteocalcin

16) A 17) C 18) A 19) C 20) B 21) A 22) C 23) B 24) D 25) B 26) C 27) D 28) B
29) A 30) B 31) B
, PHYSIOLOGY

Knee-jerk scale: • Movement of molecules and ions from the blood


• It is a stretch reflex. into brain and spinal cord is restricted by the tight
j unctions between endothelial cells of the CNS.
• Striking the patellar tendon with a tendon hammer
just below the patella stretches the quadriceps • Option 'D' Fibronetin helps in cell adhesion, thus it
tendon. will provide tight junction. On this basis option 'D'
can be excluded.
H-Reflex
• Used to assess fitness of astronauts. 17. 'C' [Chaudary 4th ed 307)
Leydig cells secrete testosterone, which is responsible
• It is reflectory reaction of muscles after electrical
for spermatogenesis.
stimulation of sensory fiber.

18. 'fl [Ganong 23d ed 281)


R-III Reflex:
An obese individual has a Lower basal metabolic rate
• R1, R2 and R3 are usually identified in the than a non-obese individual of the same weight.
electrica lly elicited blink reflex.
• The R3 component of the electrically elicited 19. 'C' [Guyton 11th ed 601)
blink reflex is present in patients with congenital Substance Pis the principal chemical mediator of pain
unsensitivity to pain. impulses from the peripheral nervous system to the
CNS.
12. 'C' [Check Explanation Below]
Valve of Hasner is a fold of mucous membrane at the 20. 'B' [Chaudary 4th ed 309]
lower end of nasolacrimal duct. It prevents air from The acrosome is a cap-Like structure over the anterior
being blown back from the nose into the lacrimal sac. half of the sperm's head. The acrosome reaction is the
reaction that occurs in the acrosome of the sperm as it
13. 'A' [Oxford textbook of Medicine 3rd ed 1836, approaches the egg. As the sperm approaches the zona
2001) pellucida of the egg, which is necessary for initiating
Gut is the largest lymphoid organ in the body. the acrosome reaction, the membrane surrounding the
Gut associated lymphoid tissue (GALT) comprises acrosome fuses with the plasma membrane of the sperm,
cells in the lamina propria, lymphoid nodules and exposing the contents of the acrosome. The contents
i ntraepithelial lymphocytes. include surface antigens and numerous enzymes, which
are responsible for breaking through the egg's tough
14. 'B' [Ganong 22nd ed 269) coating and allowing fertilization to occur.
Hippocampus is tne site in brain where short term
memory is converted into long term memory. 21. 'fl (Guyton 11th ed 188]
Osmotic pressure is determined by the number of
People in whom two hippocampi have been removed molecules dissolved in a fluid rather than by the mass
(for treatment of epilepsy) are unable to establish new of these molecules. 1gm of globulin contains only
long term memory. half molecules as 1gm of albumin. 1gm of fibrinogen
contains only one sixth as many molecules as 1 gram
Hippocampal lesions cause Antegrade amnesia i.e., of albumin.
long term memory prior to accidental event remain
intact. 22. 'C' (Se mbulingam Essentials of Medical Physiology
5th ed 393)
15. 'D' [Sembulingam 5th ed 938, 939) Long bones are formed by a cylindrical tube of bone
tissue which has 3 portions
16. 'A' [Ganong 22nd ed 614, 615) • Diaphysis: The mid portion or mid shaft.
• Blood brain barrier is composed of endothelial • Epiphysis: The wider extremity (or) the head.
cells, smooth muscles, the pial and arachnoid • Metaphysis: The portions between the diaphysis
membranes. It regulates ion flow. and epiphysis.
Dental ;lut.,e

vii) Prenatal erythropoiesis occurs in iii) Iron is present in ferrous (Fe+2) form. The
Mesoderm of yolk sac First 2 months of LU pigment is protoporphyri n formed by fo ur pyrrole
(Mesoblastic stage) life rings (tetrapyrole).

Liver & spleen


3rd to 5th month iv) The globin part contains two a chains and two ~
(Hepatic stage)
chains. But in fetal haemoglobin, there are two
Red bone marrow & liver Last three months of alpha chains and two gamma chains instead of
(myeloid stage) LU. life beta chains.

viii) In adults, after t he age of 20 years, R.B.C. are v) a chains contains 141 amino acids. ~ chains
produced by red bone marrow of all membranous contains 146 amino acids.
bones like skull bones, vertebra, ribs, scapula
and pelvic bones and from the ends of long bones vi) Destruction of hemoglobin
like humerus and femur.
Rupture of R.B.C. releases
ix) Factors influencing erythropoiesis are I
Hemoglobin, which dissociates into
• Erythropoietin
• Vitamin B12 and folic acid Iron Porphyrin Globin
• Iron (Stored as I (utilized for
• BPA (bursa promoting activity) ferritin and Biliverdin resynthesis of
hemosiderin and globin)
x) Erythropoietin is a hormone secreted by is re- ut ilized Bilirubin
juxtaglomerula,r apparatus of kidney. Renal for haemoglobin (major bile
hypoxia is the stimulant for secretion of synthesis) pigment)
erythropoietin. Eryth ropoietin causes formation
and release of new blood cells into circulation.
11. White blood cells
i) Leucocytes are classified into
xi) Both folic acid and Vit. B12 (extrinsic factor) are
a) GRANULOCYTES:
essential for maturation of R. B.C.
• Neutrophils
• Eosinophils
xii) Iron is necessary for the formation of heme part
• Basophils
of hemog lobin.

b) AGRANULOCYTES
xiii) Average life span of red blood cell is about 120
• Monocytes
days. The senile red blood cells are destroyed
• Lymphocytes
mostly in the capillaries of spleen. So the spleen
is known as the GRAVE YARD OF RBC.

xiv) Spleen stores large amount of R.B.C. They are


released into circulation during emergency
conditions like hypoxia and hemorrhage.

10. Hemoglobin
i) Normal hemoglobin value in adult males is 15
gm% and in adult females, it is 14.5 gm%.

ii) Haemog lobin is a conjugated protein consisting


of iron contain ing pigment (heme) and a protein
part (globin).
Dental ;lut.,e i======
• It is increased in polycyathemia and
• Reduction
dehydration. It decreases in anemia, pregnancy, only in the
and cirrhosis of liver. Normocytic RBC number Hemolytic
(Note: for an elective oral surgery the anemia
minimum hematocrit value should be 30)
• Normal MCV,
MCH, MCH

b) Mean corpuscular volume (MCV)


Simple • Decreased Anemias due
microcytic MCV, MCH to infectious
• It is an average volume of single R.BC (normal
values and
is 80-90µ 3 )
inflammatory
• When MCV is in increased, the cell is known
• Normal MCH
cone. diseases
as MACROCYTE (seen in Pernicious and
Hypochromic Decreased Iron
Megaloblastic anemias).
microcytic MCV, MCH, deficiency
• When MCV is decreased, the cell is known as
MCH cone. and anemia
MICROCYTE (seen in Iron deficiency anemia,
colour index
thalassemia and in anemias associated with
infectious and inflammatory diseases)
16. BLOOD GRO UPS
i) Blood groups were classified by LANDSTEINER. He
c) Mean corpuscular Hemoglobin (MCH)
found two antigens or ag gluti nogens in red blood
• It is the amount of hemoglobin in one R.BC. cells called A antigen and B antigen. He noticed
The normal value is 30 picograms. the corresponding antibodies or agglutinins in
• It is normal in pernicious and megaloblastic the serum called anti A antibody and anti B
anemia. It is decreased in hypochromic antibody.
anemia (Eg.: Iron deficiency)
ii) LANDSTEINER's LAW:
d) Mean corpuscular hemoglobin concentration • If a particular antigen is present in RBC, the
(MCHC) corresponding antibody must be absent in the
• It is the concentration of hemoglobin in one serum
RBC. The normal value of MCHC is 30% • If a particular antigen is absent in RBC, the
• It is decreased in Iron deficiency anemia in corresponding antibody must be present in the
which, RBC are microcytic and hypochromic. serum.
• The second part of Landsteiner's law is not
e) Colour index applicable to Rh factor.
• It is the ratio between percentage of
hemoglobin and the RBC cell percentage in iii) Land Steiner discovered two blood group systems
body. called ABO and Rh systems.
• It is raised in pernicious anemia and
megaloblastic anemia. It is reduced in Iron iv) ABO system
deficiency anemia Based on presence of or absence of antigen A
and antigen B, blood is divided into four groups
f) i.e, A, B, AB and O groups.

Anemia Description Example


Group Antigen Antibody in serum
• Increased
A A Anti B (/3)
MCV, MCH, Pernicious
Macrocytic Colour index anemia B B Anti A (a.a.)
AB A and B No antibody
• Normal MCH
cone. 0 No antigen Anti A and anti B
r PHYSIOLOGY SYNOPSIS

v) Rh factor The neutrophils are divided into 5 groups based on


Rh system mainly contains 3 variants - C, D, E number of lobes in nucleus
antigens. The persons having D antigen are called • Group I - One lobe
Rh+ve. The persons without 'D' antigen are called • Group V - 5 lobes
Rh-ve.
Group I cells indicate young neutrophils with one
vi) Incidence of blood groups in normal population lobe. If most of the neutro phi ls belong to group I,
Group 0 45% it indicates pyogenic infection, which stimulate bone
Group A 42% narrow to produce new cells.
Group B 12%
Group AB 4% Group V cells are in senile neutrophils. Group V
neutrophils are seen in bone marrow depression.
Rh+ve 85%
Normally, most of the neutrophils will have 3 lobes
Rh -ve 15%
and belong to group III.

vii) 19. Eosinophils are increased in parasitic infections.


Universal donor 0 -ve
Universal recipient AB +ve 20. Lymphocytes are increased in chronic inflammation.
Blood group with highest incidence 0 +ve
Blood group with lowest incidence AB -ve 21. Monocytes are the cells that are not formed in bone
narrow.
17. Erythroblastosis Fetalis
When a Rh-ve mother carries a Rh+ve fetus, usually
the first child escapes the complications of Rh ENDOCRINOLOGY
incompatibility. When the mother conceives for the
second time and carries again a Rh+ve fetus, the Rh 1. ENDOCRINE GLANDS AND HORMONES:
agglutinins enter the fetus and cause agglutination of
fetal RBC and hemolysis. a) Anterior lobe of pituitary ( Adenohypophysis)
• Acidophilic cells p reduce Growth hormone
The severe hemolysis in fetus causes hydrops fetalis (somatotropin), Prolactin
and kernicterus.
• Basophilic cells produce TSH, ACTH, FSH
(Gametokinetic factor), LH .
In erythroblastosis fetalis, there is deposition of
bilirubin pigment in the enamel and dentin of
b) Middle lobe of pituitary
developing teeth giving them a green, or brown, or
blue hue. • secretes melanocyte stimulating hormone
(M.S.H.)
In some cases enamel hypoplasia involves the incisal
c) Posterior pituitary (Neuro hypophysis)
edges of anterior teeth and middle 3rd of deciduous
cuspid and 1st molar resulting in a characteristic ring • Secretes AD H and Oxytoci n
like defect known as "Rh hump"
d) Thyroid Gland
No treatment for tooth pigmentation is necessary • Triidothyronine (T3) } Secreted by
because it affects the deciduous teeth and presents • Thyroxine (T4) thyroid follicles
only a temporary cosmetic problem.
• Calcitonin (secreted by parafollicular cells)

18. Neutrophils are the 1st cells to appear in acute


e) Parathyroid Glands
inflammation. ARNETH COUNT is used to count the
number of nuclear lobes in hundred neutrophils. • Parathormone
Dental ;lut.,e

f) Pancreas Ami no acid derivates


ex-cells - Glucagon • Catecholamines
or biogenic amines
!3-cells - Insulin • Thyroid hormones
(from tyrosine)
8-cells - Somatostatin
4.
g) Adrenal medulla secretes catecholamines i.e,
Parathormone Calcitonin
• Adrenaline (epinephrine) (Increases serum Ca (Decreases serum Ca level)
• Nor-adrenaline (nor-epinephrine) level)
• Dopamine Secreted by parathyroid Secreted by parafollicular
glands cells of thyroid gland
h) Adrenal cortex: Causes demineralization Causes remineralisation
Zona glomerulosa (Outer layer) Mineral corticoids of bone of bone
Zona fasciculata (Middle layer) Gluco corticoids i i
Zona reticularis Sex hormones and Loss of Ca from bone Increased Ca deposition
(Inner layer) Glucocorticoids in bone

i) Testis
i i
Increased serum ca+2 Decreased serum ca+2
• Testosterone (C19 compound) level level
Decreased Calcium Increased Calcium
j) Ovary excretion from kidney excretion from kidney
• Estrogens (C18 compound) Decreased absorption of Increased absorption of
• Progesterone (from corpus lute um). Ca from GIT Ca from GIT

k) Hypothalamus (5 releasing+ 2 inhibiting) 5.


• Growth hormone releasing hormone • Due to excessive secretion of ACTH
• Growth hormone inhibiting hormone Cushi ng's • Mostly due to basophilic adenoma
disease of Adenohypophysis of pituitary
• Prolactin releasing hormone (PRH)
gland
• Prolactin inhibiting hormone (PIH)
• Gonadotropin releasing hormone(GnRH)
Cushi ng's • It is due to excessive secretion of
syndrome glucocorticoids of adrenal cortex
• Corticotropin releasing hormone (CRH)
• Thyrotrophic releasing hormone (TRH) 6.
Another name for growth hormone
2. Examples of local hormones are
So matotropi n secreted by somatotropes of anterior
Synthesized from arachidonic

}
• Prostaglandins pituitary
• Prostacyclins acid and acts as mediators of
• Thromboxanes inflammation. GH stimulates the liver to secrete
somatomedi n. GH does not have
Somatomedin
3. Classification of hormones based on chemistry: any direct action on bones. It acts
through somatomedin.
Corticosteroids(C21
Steroid hormones
compounds), Sex hormones Somatostatin is an inhibitory
Hormones of placenta, hormone of growth hormone. It is
So matostati n
Protein hormones pancreas, pituitary and secreted by delta cells of islets of
parathyroid glands langerhans in pancreas.
Note:- Insulin, Prolactin, ACTH, GH are polypeptides
while TSH, LH, FSH are glycoproteins
,......__,._

r PHYSIOLOGY SYNOPSIS
V
1025

7. HARMONES AND THEIR EFFECTS


Hormone Biological effect Hypo and Hyper secretion
Growth Stimulates liver to secrete • Gigantism is due to hypersecretion of GH in childhood before
Hormone (GH) somatomedin through which fusion of epiphysis of bone with shaft. Hyperglycemia and pituitary
most of metabolic actions of diabetes are important features.
GH is carried • 8cromegaly is due to hypersecretion of GH in 8dults. Enlargements
of bones (especially mandible), kyphosis (bowing of spine) and
bull dog scalp are important features.
• Dwarfism is due to hypo secretion of GH in infancy or early
childhood. Stunted but proportionated growth with normal menta l
activity and reproductive functions are important feature.
• 8cromicria is due to deficiency of GH in 8dults. It is characterized
by atrophy of extremities of body.
ADH or Kidneyisthetargetorgan. ADH • Diabetes insipidus is due to deficiency of ADH. The disease is
antidiuretic increases water absorption characterized by polyuria (excessive excretion of water through
hormone or from distal convoluted tubule urine), polydipsia (excess thirst) and dehydration.
vasopressi n and collecting duct.
Thyroid • T4 or thyroxine forms a) Hyperthyroidism
hormones about 90% iodothyronins • Due to excess secretion of thyroid hormones.
• T3 forms 8-9% of • Thyroid adenoma, Grave's autoimmune disease are common
iodothyronins and is more causes.
potent than T4
• 2 types - primary and secondary.
• Both T3 and T4 are secreted
• Primary thyrotoxicosis/ Primary hyperthyroidism
by follicular cells of thyroid
- CNS symptoms, intolerance to heat, exophthalmia, tremors
gland
are features.
• Secondary thyrotoxicosis
- CVS symptoms, intolerance to cold, palpitations and
tachycardia are features.

b) Hypothyroidism
• {retinism is due to hypothyroidism in _C_hildren.
• Macroglossia, stunted growth, retarded mental activity and
reproductive system, disproportionate body parts are important
features of cretinism.
• Myxedema is due to hypothyroidism in adults.
• Swelling of face, scaliness of skin, bagginess under eyes,
non pitting type of edema, high BP due to arteriosclerosis,
depressed hair growth and frog like husky voice are important
features of myxedema
Calcitonin • Secreted by parafollicular
cells of thyroid.
• It facilitates the deposition
of Ca in bone and reduces
the blood calcium level.
r PHYSIOLOGY SYNOPSIS

8. a) Goiter in HYPERTHYROIDISM 12. Endocrine gland directly under the control of nervous


system is adrenal medulla
Increased size of gland and increased hormone
secretion 13. Ion required for insulin synthesis is Zn+ 2

So goiter in hyperthyroidism is known as TOXIC 14. Hormones, which cannot be given orally, are Insulin
goiter and Parathormone.

b) Goiter in HYPOTHYROIDISM 15. Adrenaline increases blood glucose level by inhibiting


the release of insulin. So L.A with adrenaline is not


Increased size of gland and decreased hormone
secretion.
indicated in diabetic patients

16. Lorain dwarfism is due to deficiency of somatomedin


while pituitary dwarfism or dwarfism is due to
So known as NON -TOXIC goiter deficiency of growth home in children.

c) Hyperthyroidism is treated by using antithyroid 17. Simmond's disease is due to hypo secretion of all
substances like thiocyanate, propyl thiouracil, anterior pituitary hormones. Loss of hair, loss of teeth
etc. and early senile appearance are the features.

9. 18. Frolich's syndrome (dystrophia adiposgenitalis) is


a) Von Reckling Hausen's disease of skin is seen in due to hypo secretion of both anterior and pituitary
neurofi bra matosis. hormones.

b) Von Reckling Hausen's disease of bone is seen in Features are


hyperpa rathyroidism. i) Dwarfism and obesity in children
ii) Obesity and sexual atrophy in adults.
10.
Hormone Effect 19.

• Glucagon Stimulates neoglucogenesis, Kyphosis • Excess curvature of spine (Potts


glycogenolysis and lipolysis (forward curvature)
• Epinephrine
bending) • Seen in rickets and acromegaly
Insulin
Inhibits neoglucogenesis, and Lordosis Excessive curvature of lumbar spine.
(Opposite to
promote glycogenesis and
glucagon and Scoliosis Lateral curvature of spine
lipogenesis.
epinephrine) • Visible swelling over where the ribs
• Increased Glycogen deposition Rachi tic joins their cartilages
in cells, because of which no rosary • Lordosis, scoliosis, kyphosis and
more glucose enter the cells. So rachitic rosary are a LL features of
Growth ultimately blood glucose level Rickets
hormone increases leading to pituitary
diabetes 20. ~ldosterone - life s~ving hormone
• Increased protein synthesis Cortisol - life protecting hormone
• Lipolysis
Stimulates neoglucogenesis and 21. Chronic adrenal insufficiency or addisons disease is
Thyroxine
glycogenolysis characterized by hyper pigmentation of skin, mucous
Increased gluconeogenesis and membrane, hypotension and muscle weakness.
Cortisol
lipolysis
22. Pheochromocytoma is due to excess secretion
11. Directly acting pituitary hormones are FSH, LH and GH of catecholamines from adrenal medulla. It is
r PHYSIOLOGY SYNOPSIS
~

V
1029

GIT AND LIVER


Secretion Important constituents Function
1. Saliva Salivary amylase, maltose, • Salivary amylase acts on cooked starch and converts it into
(750 - 1000 ml of lysozyme, mucin, lingual maltose.
saliva is secreted / lipase etc. • Lingual lipase secreted from lingual glands is a fat splitting
day) enzyme. It converts triglycerides into fatty acids and 1,2 -
diacylglycerol.
• Mucin lubricates bolus and facilitates swallowing.
• Lysozyme has antibacterial action.
2. Gastric Juice. • Pepsin, Rennin, are • The HCl in the gastric juice destroys many types of bacteria in
• Secreted by chief secreted by chief cells or the food.
cells and parietal pepsinogen cells • IF is necessary for absorption of vitamin B12 or extrinsic factor.
cells of gastric • Hcl and intrinsic factor (IF) Absence of IF can lead to pernicious anemia.
glands are secreted by parietal or • Inactive Pepsinogen is converted into active pepsin in acid
• 1200 - 1500 ml oxyntic cells. medium in presence of HCl. Pepsin acts on proteins and
/day • Gastric lipase ( a weak converts them into peptones and polypeptides. It also causes
• Acidic in nature. lipolytic enzyme than curdling and digestion of milk.
PH is 1.0-2.0 pancreatic lipase), and • Rennin is a milk-curdling enzyme. It is present only in animals
gelatinase are other and is absent in man.
enzymes.
3. Pancreatic Juic Pancreatic juice contains • The inactive trypsinogen converted into active Trypsin by
• 500 - 800 ml/day 99.5% of water and 0.5% of enterokinase of duodenum and by autocatalytic action.
• Alkaline in nature solids. • Trypsin is most powerful protein splitting enzyme. By means of
with a pH of 8.0 • Proteolytic Enzymes hydrolysis, it converts proteins into proteases and polypeptides.
to 8.3. - Trypsin Trypsin also converts chymotrypsinogen into chymotrypsin,
- Chymotrypsin, proelastase into e lastase and procolipase into colipase.
- Ca rboxypeptidase • Chymotrypsin hydrolyses the proteins into polypeptides. Both
- Elastase, trypsin and chymotrypsin causes rapid digestion of milk.
- Nuclease • Carboxy peptidases break the terminal bond of protein
- Collagenase molecules. They act on polypeptides and other proteins and
convert them into aminoacids.
• Lipolytic enzymes • Elastase digests the elastic fibres while the collagenase causes
- Pancreatic lipase digestion of collagen.
- Phospho lipase
• Amylase is the on ly amylolytic enzyme in pancreatic juice. It
converts starch into maltose.
• Amylolytic enzyme
• Pancreatic lipase is a powerful lipolytic enzyme. It converts
- Pancreatic amylase
triglycerides into monoglycerides and fatty acids.
• The activity of pancreatic lipase is accelerated in the presence
of bile salts. The pH required for its activity is 7.0 to 9.0.
4. Bile Juice • Bile salts are the sodium • Em ulsification of fat s: Normally the Lipids are insoluble in
• (800 - 1200 ml/ and potassium salts of water, so the lipolytic enzymes cannot digest the lipids.
day). bile acids, which are The bile salts reduce the surface tension of lipids and make
• Alkaline in nature conjugated with glycine or them water-soluble. This is known as emulsification. Due to
with a pH of 8 to taurine. emulsification, the fat globules broken into minute particles,
8.6 • Bilirubin and biliverdin are so that they can be digested by various enzymes of GIT.
the two bile pigments. • Absorption of fats: When bile salts are combined with lipids,
micelles are formed. The micelles are water soluble and are
easily absorbed. This is known as hydrotropic effect.
r PHYSIOLOGY SYNOPSIS
~

V
1031

• The most important action of secretin is stimulation of


pancreatic bicarbonate juice production.
• The secretion of large amount of watery juice with high
bicarbonate ion. Protects the intestinal mucosa from acid chyme
S cells of duodenum, jejenum
Secretin by neutralizing it and provides an alkaline pH for the activation
and ileum of pancreatic enzymes.
• The other actions of secretin are
- Inhibition of gastric juice secretion and motility of stomach
- Causes contraction of pyloric sphincter.
Enterocrinin ----------- Increases the secretion of succus entericus.

Gastric inhibitory K cells in duodenum and


• Inhibits secretion of gastric juice.
peptide (GIP) jejenum
• Inhibits gastric motility
• Increases insulin secretion by stimulating b-cells of langerhans
Vasoactive
Stomach, small and large
• Inhibition of HCl secretion in gastric juke
intestinal
intestines.
• Secretion of succus entericus with more amounts of water and
polypeptide (VIP} electrolytes.
Hypothalamus, D cells of
Somatostatin islets of langerhans of • Inhibits the secretion of GH, VIP, GIP and Insulin
pancreas

Substance P Small intestine • It increases the mixing and propulsive movements of small
intestine

4. Liver is the largest gland in the body. It weighs about 9. Due to bacterial action in intestine bilirubin is
1.5 kg in man. It is located in the upper right side of converted into urobilinogen, which is excreted through
the abdominal cavity. urine. Some of the Urobilinogen is excreted in feacus
as stercobilinogen.
5. Portal triad consists
• Branch of portal vein 10. The main function of gallbladder is the storage and
concentration of bile
• Branch of hepatic artery
• Tributary of bile duct
11. When a fatty chyme enters the intestine from stomach,
the intestine secretes the hormone cholecystokinin -
6. The bile salts are the sodium and potassium salts pancreozymin. This hormone causes contraction of
of bile acids, which are conjugated with glycine or ga ll bladder.
taurine. The bile acids are derived from cholesterol
12. The glands of stomach are three types.
Cholic acid and chenodeoxy cholic acid are formed in
liver and are known as Primary or Principal bile acids.
a) Fundic glands: These are situated in the body
and fundus of stomach. These glands are also
Deoxy cholic acid and lithocholic acids are formed in
called as main gastric glands or oxyntic glands.
intestine and are known as Secondary bile acids.
Chief cells or pepsinogen cells of fundic glands
secrete pepsinogen and rennin. Parietal cells or
7. Enterohepatic Circulation: The bile salts absorbed
oxyntic cells secrete HCl and IF. Mucus neck cells
from intestine are transported by portal vein back to
secrete mucin.
liver. This is known as enterohepatic circulation of bite
salts.
b) Pyloric glands: These are present in pyloric part
of stomach and secretes mucin and gastrin.
8. Bilirubin and biliverdin are the two bile pigments.
Bilirubin is the major bile pigment. These pigments
are formed during break down of hemoglobin.
,......__,,_

r PHYSIOLOGY SYNOPSIS
V
1035

RESPIRATORY SYSTEM • increases compliance


• plays an important role in the inflation of
1. Respiratory unit is the terminal portion of respiration lungs during birth.
tract. The exchange of gases occurs only in t his part of • plays an important role in host defense within
the respiratory tract . The respiratory unit is composed the lungs against infection and inflammation.
of respiratory bronchiole, alveolar ducts and alveoli.
e) Deficiency leads to -
2. The respiratory muscles are classified into primary and • Acute respiratory distress syndrome (ARDS) or
secondary muscles. hyaline membrane disease.
a) PRIMARY INSPIRATORY MUSCLES: • Increased susceptibility for bacterial and viral
• diaphragm (supplied by phrenic nerve) infections.
• external intercostals muscles (supplied
by intercostal nerve) 5. Compliance:
The ability of lungs and thorax to expand is called
b) ACCESSORY INSPIRATORY MUSCLES: compliance. It is defined as t he change in volume per
• sternomastoid unit change in the pressure.
• scaleni muscles
NORMAL VALUES
• anterior serrate
Lungs alone 0.22 lt/cm of H20 or
• elevators of scapulae and pectorals 220mljcm of H20 pressure
Lungs & thorax 0.13 lt/cm of H20 or
c) PRIMARY EXPIRATORY MUSCLES: 130 mljcm H20 pressure
• Internal intercostal muscles (supplied
by intercostal nerves) FEATURES REDUCING COMPLIANCE:
• Paralysis of respiratory muscles
d) ACCESSORY EXPIRATORY MUSCLES:
• Abdominal muscles • Pleural effusion
• Deformities of thorax like kyphosis and scoliosis.
3. Females have thoracico abdominal breathing. Males • Abdominal thorax (pneumothorax, hydrothorax,
have abdomino thoracic breathing. hemothorax and pyothorax).

4. Surfactant: 6. PULMONARY FUNCTION TESTS:


a) Pulmonary surfactant is a surface-active material
lining the alveolar epithelium. It reduces the I. Lung Volumes
surface tension. a) TIDAL VOLUME (TV):
Volume of air breathed in and out of lungs
b) Surfactant is a lipoprotein complex formed from in a single normal quiet respiration . Normal
phospholipids and other lipids, proteins and ions. value is 500 ml.
It is secreted by Type II pneumocytes and Clara
cells. The characteristic feature of these cells is b) INSPIRATORY RESERVE VOLUME (IRV):
presence of microvilli on their alveolar surface. The additional amount of air that can be
inspired after the end of normal inspiration
c) The major phospholipid present in surfactant is over that of tidal volume. Normal value is
di palmitoyl phosphatidyl choline (DPPC) . The 3300 ml.
ions present in surfactant are calcium ions.
c) EXPIRATORY RESERVE VOLUME (ERV):
d) Functions of Surfactant The additional amount of air that can
• reduces surface tension be expired out forcefully, after normal
• stabilizes alveolar size expiration. Normal value is 1000 ml.
Dental ;lut.,e

d) RESIDUAL VOLUME (RV): TVC is reduced in obstructive diseases like asthma


The amount of air remaining in the lungs after and emphysema. In some restrictive respiratory
a forced expiration. Normal value is 1200ml. diseases like fibrosis also, the FEV is slightly
reduced.
II. Lung capacities
a) INSPIRATORY CAPACITY (IC) IV. Respiratory minute volume (RMV) or
Maximum volume of air that can be inspired pulmonary ventilation:
from the end of expiration. It is the amount of air breathed in and out of
lungs every minute is called respiratory minute
IC= TV+ IRV = 500 + 3300 = 3800 ml. volume. It is the product of tidal volume (TV) and
respiratory rate (RR).
b) VITAL CAPACITY (VC):
It is the maximum amount of air that can RMV = TV X RR= 500 X 12 = 6000 ml.
be expelled out forcefu lly after a maximal
(deep) inspiration . V. Maximum breathing capacity (MBC) or
maximum ventilation volume
VC = IRV + TV + ERV It is maximum amount of air which can be breathed
= 3300 + 500 + 1000 = 4800 ml. in and out of lungs by means of forceful respiration.

Vital capacity is physiologically increased in Normal value in adult male is 150-170 lts/minute,
divers, swimmers, ath letes and people living fema les is 80-100 lts/minute.
at high altitudes. It is decreased in conditions
like old age, obesity (physiological) and VI. Dead space:
diseases of respiratory tract (Eg.: COPD). The part of respiratory tract, where gaseous
exchange does not take place is called the dead
c) FUNCTIONAL RESIDUAL CAPACITY (FRC): space. The parts of respiratory tract, which form the
It is the volume of air remaining in the lungs dead spaces, are nose, pharynx, trachea, bronchi
after normal expiration. Functional residual and bronchioles upto terminal bronchioles.
capacity includes ERV and RV.
Dead space is of two types -
FRC = ERV + RV = 1000 + 1200 = 2200 ml. Anatomical dead space is volume of respiratory
tract from nose to terminal bronchiole.
d) TOTAL LUNG CAPACITY:
It is the amount of air present in the lungs Physiological dead space includes the
after a maximal (deep) inspiration. This anatomical dead space plus the air present in
includes all the volumes. the alveoli which are non-functioning and do not
receive adequate blood flow.
TLC = IRV + TV + ERV + RV
= 3300 + 500 + 1000 + 1200 = 6000 ml. Under normal conditions, the physiological dead
space is equal to anatomical dead space. The
III. Forced expiratory volume (fev) or timed vital volume of normal dead space is 150 ml.
capacity (tvc):
It is the volume of air that can be expired VII. Alveolar ventilation:
forcefully in a given unit time. Alveolar ventilation is amount of air exchanged
for gaseous exchange every minute.
FEVl in 1 second 83% of VC
FEV2 in 2 seconds 94% of VC Alveolar ventilation
FEV3 in 3 seconds 97% of VC = (Tidal volume-Dead space· volume) x Respiratory rate
After 3rd second = 100% of total vital capacity. = (500-150) X 12
= 4.2 lts./min
Dental ;lut.,e

11. IMPORTANT FEATURES OF DIFFERENT TYPES OF HYPOXIA


Hypoxic or arterial Stagnant or ischemic
Features Anemic hypoxia Histotoxic hypoxia
hypoxia hypoxia
Tension of oxygen in Tension of 0 2 is normal. Tension of0 2 is normal. All are normal, but the
arterial blood is lower But the 02 carrying 02 carrying capacity tissues fail to extract
Characteristic
than normal due to low capacity 15 lower than is also normal. But oxygen from blood.
feature
0 2 content of blood normal. the blood flow to the
tissues is deficient.
• Pneumonia • Decreased no. of RBC • Thrombo embolism • Cyanide or sulfide
• Congestive heart • Decreased Hb content • Hemorrhage and poisoning
failure of RBC fluid loss
• T. B. • Formation of altered • Congestive cardiac
Causes • Pnemothorax hemoglobin failure

• Lack of surfactant
• Emphysema
• High altitude
sickness
P0 2 in arterial
Decreased Normal Normal Normal
blood
0 2 carrying
Normal Decreased Normal Normal
capacity of blood
0 2 content of
Decreased Decreased Normal Normal
blood
Blood flow rate Normal Normal Decreased Normal
Utilization of 0 2
Norma l Normal Normal Reduced
by tissues
Oxygen therapy Useful Useful Useful to some extent Not useful

12. OXYGEN DISSOCIATION CURVE: • Increase in H+ concentrate and decrease in


pH.
i) The relationship between the partial pressure • Increase in body temperature.
of oxygen and the percentage saturation of
• Increase in partial pressure of Co 2 (Bohr
hemoglobin with oxygen can be explained
effect).
graphically arnd the graph is called oxygen
hemoglobin dissociation curve.
iv) Factors, which favor dissociation shifts the curve
to LEFT. They are
ii) Under normal condition, the oxygen hemoglobin
• Decrease in 2-3 DPG.
dissociation curve is 'S' or sigmoid shaped.
• Decreased in W concentrate.
iii) Factors, which help in deoxygenation shifts the • In fetal blood, because fetal Hb has got more
curve to RIGHT. They are - affinity for oxygen than adult hemoglobin.
• Excess 2-3 DPG levels (muscular exercise and • Increased in p0 2 and decrease in pC02.
high altitude) • Decreased body temperature.
r PHYSIOLOGY SYNOPSIS
~

V
1041

19. HEART SOUNDS


Heart
Occurs during Cause Characteristics Relation with ECG
sounds
Isometric

First
contraction and Closure of atrio- • Long, soft and low pitched Coincides with 'R' wave of
part of ejection ventricular valves • Resembles LUBB ECG
period

Second
Protodiastole and Closure of • Short, sharp and high
Appears after 0.09 second
part of isometric semilunar valves pitched.
after summit of T wave.
relaxation (COMED-2013) • Resembles DUBB
Rushing of blood Between T wave and P
Third Rapid filling Low pitched
into ventricles wave
Contraction of Between P wave and Q
Fourth Atrial systole Inaudible sound
atrial musculature wave

20. E.C.G
Wave/
From - To Cause
Segment
P wave ------ Atrial depolarization
ORS complex ------ Ventricular depolarization
T wave ------- Ventricular repolarization
It is a rare and insignificant wave of ECG. It is due to
U wave -------
repolarization of papillary muscle
P-R interval Onset of P wave to onset of O wave Atrial depolarization and conduction through AV node
ORS duration Onset of O wave and end of S wave Ventricular depolarization
Q-T interval Onset of O wave and end of T wave Electrical activity in ventricles
ST segment End of S wave and onset of T wave ----------

21. Cardiac murmurs


Cardiac Murmurs

Systolic murmurs
II Diastolic murmurs Continuous murmurs
1. Incompetence of 1. Stenosis of atrioventricular 1. Patent ductus
atrioventricular valves valves arteriosus
2. Stenosis of semilunar valves 2. Incompetence of semilunar
3. Anemia valves
4. Septal defect
5. Coa rctation of aorta
Dental ;lut.,e

ii) Wilson's disease (progressive hepatolenticular vi. Types of dichromatism


degeneration) Cannot appreciate red colour
Protanopia and uses green and blue to
iii) Chorea match colors
This is an abnormal involuntary movement. Cannot appreciate green colour,
Chorea means rapid jerky movements. Deuteranopia uses red and blue to match
colors.
iv) Athetosis:
Can not appreciate blue colour
Characterised by slow rhythmic and twisting
due to defect in third receptor.
movements. Tritanopia
Uses red and green to match
colors
v) Hemiballismu.s:
It is characterized by violent, involuntary
vii. Ishihara's chart is used to test colour blindness.
abnormal movements on one side of the body
involving mostly the arm. While walking the arm
viii. Peripheral end organs for olfaction are olfactory
swings widely and these movements are called
cells.
tingling movements.
1x. Olfactory receptors are phasic receptors and
17. Functions of limbic system:
adapt very rapidly.
• Regulation of endocrine glands
• Olfaction
• Regulation of food intake
• Control of circadian rhythm
• Regulation of autonomic functions

18. Special Senses


1. Rods are the receptors for night vision or dim
light vision whereas cones are the receptors for
daylight or bright light vision.

ii. Myopia or short sightedness is corrected by


using concave lens and hypermetropia or long
sightedness is corrected by using convex lens.

iii. The sense organs for taste are t he taste buds.


• Tip of the tongue - Sweet
• Dorsum of the tongue (anterior) - Salt
• Dorsum of the tongue ( posterior) - Bitter
• Sides of the tongue - Sour

iv. The threshold for bitter taste is very low whereas


sweet taste has a high threshold.

v. The organ of corti present on tectorial membrane


is the sensory organ for hearing.
r PHYSIOLOGY SYNOPSIS
~
1045
V

OUR BOOK SELLERS LIST


ANDHRA PRADESH BI HAR GUJARAT • Jupiter Book World:
Hyderabad Patna Ahmedabad PH.: 09844124538 /
• Paras Medical Books Pvt. Ltd • R. K. Agency • Bharath Medical Books 09342815475
Ph.: 040-24600869, Ph.: 9431024355, Ph.: 9824013920,
32982239 61226633 7926586058 Davangere
• SS Publishers & Distributors • Scientific Book Company • Dipti Medical Book Centre • Vikas Books
Ph.: 040-64595911, Ph.: 9431075236, Ph.: 9825272839, 9886066747
9492029330 6122301744 7922131725
• Universal Books: Gulbarga
CHATTISGARH Vadodara
Ph.: 040-44999999 • Paras Medical books
(212 Ext)
Raipur • Medical Book Company
Ph.: 08472- 250396/
• Shristi Books: Ph.: 0265-2422789
• Osmania Book House, 9986 545620
Ph.: 9229299164,
Ph .: 040-66104253, HARYANA
07714032551,
66831665 Rohtak Mangalore
• Hansa Book Centre:
• Medical Book Company, • Jain Book Depot, • UR Shenoy & Co
Ph.: 7712228043
Ph.: 040-24651038 Ph.: 01262-213355 Ph.: 9844912752/
• Falcon Medical Book House, DELHI 0824424744 /2423944
Ph.: 9290215805 Delhi HIMACHAL PRADESH
• Paras Medical Books, Shim la KERA LA
Karimnagar Ph.: 011-32431153 • Students Stores
Thrissur
• Paras Medical Books, • S.R. Pub lishers & Distributors Ph.: 1772656375 • Cosmo Books
Ph.: 0878-2222046, Ph.: 011-23271632, Ph.: 0487- 2335292
885029367 23276712 JAMMU & KASHMIR
• Universal Book Stall, Jammu Ernakulam/Cochin
Kurnool Ph.: 011-23272595, • Bharatiya Pustakalaya: • New Central Book agency
• Kurnool Medical Books, 09811504952; Ph.: 1912540092 Ph.: 0484-4021054
Ph .: 9885684285 • College Book Stores, • Universal Book Seller: • Paras Medical Books
Ph.: 011-23269426, Ph.: 1912573574 Ph.: 9447022842/
Nellore 23279128 4844012346
JHARKHAND
• Sandhya Book Emporium, • New Central Book Agency, • Pai & Co;
Jamshedpur
Ph.: 9247173544 Ph.: 011-46168051/52/ 53 Ph.: 9388460606,
• Scientific Book Mart
• Peepee Publishers
Ph.: 6572434937
Tirupathi Ph.: 011-65195868, Calicut/Kozhikode
• SLV Books, 09811156083 Ranchi • Ramdas Medical Books
Ph.: 0877-2252894 • Gopal Book Shop • Student Book Depot Ph.: 0495- 2358398,
Ph.: 011-27872538, Ph.: 06512203048/2221907 09895333554
Vijayawada 9811212705
• Paras Medical Books, • All India Book Ho use, KARNATAKA Kottayam
Ph.: 0866-3242002 Ph.: 011-23256036, Bangalore • SGP Medical Books
23267481 • Paras Medical Books, Ph.: 0481-2596108,
Vishakapattanam • Amit Book Depot, Ph.: 080-26708108, 9447196108
• Andhra Medical Book Centre Ph.: 9810016100 32934663
Ph.: 9985716032 • B.Sen Soni & Co., • Prithvi Book Agencies, Trivandrum
• World Medical Book Centre Ph.: 011-23268978, Ph.: 080-26700952, • Liason Book House
Ph.: 9849022192 45643773 26706013 Ph.: 482-382412 / 2361020
• Delhi Book Store • Medical Book Company • Modern Book Centre
Ph.: 01123283333/2350909 Ph.: 080-26707525 Ph.: 4712331826
ASSAM
• JD Book Depot • Kumar Book Ho use, • Prabhu Books
Guwahati
• Elite book Distrib utors Ph.: 09211220709/ Ph.: 0821-2430882, Ph.: 4712478397 / 2473496
09871841875 09844061999
Ph.: 09864503483
Attention MDS Aspirants ...
Revision Batches Starting from
May 31st 2015 for AIPG I APPG-2016
Our centres all over India
85830 76769
Delhi & Rohtak 89015 21591 Kolkatta
91985 75997
99629 46047 88661 15882
Chennai Ahmedabad
94455 52597 99099 21996
Chandigarh 9696673039 Bhavnagar 94086 91092
Bhopal 98930 22115 Indore 99931 36216
92495 52116 Muvattupuzha 94471 60069
Trivandrum
94470 72074 (Ernakulum) 94470 49455
Cuttack 72054 55992 Hyderabad 92462 10072
Coimbatore 98941 73776 Bangalore 95354 44112
98867 25660
Mumbai 99675 55539 99675 55539
80973 12224 Nagpur
95950 52481
Bhimavaram 95025 07299 Nizamabad 99597 65242
Khammam 96660 57533 Rajahmundry 98663 71497
Nellore 90322 55520 Kurnool 97014 81947
Tirupathi & 92473 29080 94943 32737
Vizag & Srikakulam
Kadapa 92473 60900 97030 38666

For merit based offers of upto 100°/o based on your


BDS percentage, previous entrance ranks and for DPA Alumni
Please Contact : 92462 10072

...all the top authors under one roof


Flat No. 101, H.No. 3-2-844, Rajeshwari Gayatri Sadan, Kacheguda, Hyderabad - 500 027.
({) : 92462 10072 + t8I : dentalpulse@yahoo.com

For Academic Contribution and Franchise Queries


Contact: +91 92462 10072
INVITATION FOR AUTHORS

We, at SWAPNA MEDICAL PUBLISHERS are on the constant lookout in bringing out excellent Medical
and Denta l books (PG entrance and UG textbooks) for the benefit of our esteemed readers.

If you are an author w ith a completed manuscript and wish to get it published, then SWAPNA MEDICAL
PUBLISHERS could be the medium for you. We have a good record with excellent publishing quality at
competitive rates, with w idespread d istribution

For further details, e-mail to swapnamedicalpublishers@yahoo.com or smp_hyd@yahoo.co.in

Dr. K. Satheesh Kumar Reddy


For SWAPNA MEDICAL PUBLISHERS
Cell: 09247 188 244, 095338 63720
www.swapnabooks.com

INVITATION FOR FRANCHISES & TEACHERS

DENTAL PULSE ACADEMY (DPA) is an organization that provides the students a common platform for all
MDS entrance examinations like AIPG, AIIMS, PGI. BHU, COMEDK, MAHE and various state entrance
examinations. DPA is dedicated to train and promote the MDS aspirants through our classroom
courses, online exams, live examination centres and postal coaching.

DPA with its goal and objectives, started with a single centre and has grown to 20+ centres across
India with in a short span of three years. As a managing Director of DPA, I cordially invite one and all
with right skills and a ttitude and who wants to become a successful entrepreneur, to be apart of this
ever growing process by becoming a franchisee owner.

Dr. K. Satheesh Kumar Reddy


For Dental Pulse Academy
Cell: 092462 10072
www.dentalpulseacademy.com

SWAPNA MEDICAL PUBLISHERS


101, H.No. 3-2-844, Rajeswari Gayatri Sadan, Kachiguda, Hyderabad-500 027
Cell: 09247 188 244, 0953338 63720, e-mail:smp_hyd@yahoo.co.in,www.swapnabooks.com

Exclusively Distributed By: SRI VENKATESHWARA BOOKS


Hyderabad. Mob: 09247 188 244, 095338 63720, Email: svbookshyd@gmail.com

You might also like